12.03.2019 Views

CarolRivers-combined

Create successful ePaper yourself

Turn your PDF publications into a flip-book with our unique Google optimized e-Paper software.

Dr. Carol Rivers'<br />

PREPARING FOR THE WRITTEN BOARD EXAM<br />

IN EMERGENCY MEDICINE<br />

Eighth Edition<br />

Volume I<br />

Ohio<br />

ACEP<br />

American College of Emergency Physicians<br />

Advocacy<br />

-Education I Leadership


Published by:<br />

Ohio Chapter, American College of Emergency Physicians (Ohio ACEP)<br />

35 7 0 Snouffer Road<br />

Suite 700<br />

Columbus, Ohio 43235<br />

(674) 792-6506<br />

(888) OHACEP4 (642-2374)<br />

ohacep@ohacep.org<br />

www.ohacep.org<br />

Printed in the United States of America<br />

Eighth Edition<br />

ISBN 978-0-9917470-0-7<br />

Date of original release: January 7 992<br />

Most recent update: January 20 7 7<br />

Copyright ©20 7 7 Ohio Chapter, American College of Emergency Physicians. All<br />

rights reserved. No part of this publication may be reproduced or transmitted in<br />

any form, electronic or mechanical, including photocopying, recording, storage<br />

in any information retrieval system, or otherwise, without the prior written<br />

permission of the publisher.<br />

DISCLAIMER<br />

Ohio Chapte,~ American College of Emergency Physicians publishes information<br />

believed to be in agreement with the accepted standards of practice at the date<br />

of publication. Due to the continual state of change in diagnostic procedures,<br />

treatment, and drug therapy, Ohio ACEP and the writers and editors are not<br />

responsible for any errors or omissions. In the practice of medicine, the reader<br />

should confirm the use of any information with other sources.


The object of education is not learning, but discipline<br />

and enlightenment of the mind.<br />

Woodrow Wilson


Dr. Carol Rivers' Preparing for the Written Board Exam in<br />

Emergency Medicine, Eighth Edition<br />

EDITORS<br />

SENIOR ASSOCIATE EDITOR<br />

Ann M. Dietrich, MD, FAAP, FACEP<br />

Associate Professor of Primary Care (Lead), Ohio University Heritage College of Osteopathic Medicine; Pediatric<br />

Associate Medical Director, MedFlight of Ohio, Columbus, Ohio<br />

SECTION EDITORS<br />

Thomas Green, DO, MMM, MPH, CPE, FACOEP, FACEP<br />

Associate Dean, Clinical Affairs; Chief Academic Officer (interim)-HEARTLand Network OPTI and Associate Professor<br />

Family Medicine, Des Moines University College of Osteopathic Medicine; Associate Professor Emergency Medicine/<br />

Family Medicine, Midwestern University-Chicago College of Osteopathic Medicine; Attending Physician, Emergency<br />

Department, Central Iowa Healthcare, Marshalltown, Iowa<br />

Joseph P. Martinez, MD, FACEP, FAAEM<br />

Associate Professor of Emergency Medicine and Medicine, Assistant Dean for Student Affairs, and Assistant Dean for<br />

Clinical Medical Education and Residency Programs Liaison, University of Maryland School of Medicine, Baltimore,<br />

Maryland<br />

Amal Mattu, MD, FACEP<br />

Professor and Vice Chair of Education, Co-Director, Emergency Cardiology Fellowship, University of Maryland School<br />

of Medicine, Baltimore, Maryland<br />

Victor J. Scali, DO, FACOEP-D<br />

Director of EM, Residency Education, and Associate Professor, Department of Emergency Medicine, Rowan University<br />

School of Osteopathic Medicine, Stratford, New Jersey<br />

Howard A. Werman, MD, FACEP<br />

Professor of Clinical Emergency Medicine, The Ohio State University Wexner Medical Center; Medical Director,<br />

MedFlight, Columbus, Ohio<br />

Sandra L. Werner, MD, ROMS, FACEP<br />

Operations Director, Emergency Medicine, Associate Director, Emergency Medicine Residency Program, MetroHealth<br />

Medical Center; Associate Professor, School of Medicine, Case Western Reserve University School of Medicine,<br />

Cleveland, Ohio<br />

CONTRIBUTORS<br />

Michael Abraham, MD, MS<br />

Clinical Assistant Professor, Department of Emergency Medicine, University of Maryland School of Medicine,<br />

Baltimore, Maryland<br />

Nervous System Disorders<br />

Rudd J. Bare, MD, MMEL<br />

Chief of Emergency Medicine; Faculty, Emergency Medicine Residency Program, Western Reserve Hospital, Cuyahoga<br />

Falls, Ohio; Summa Health System EM Residency, Akron City Hospital, Akron, Ohio; Masters in Medical Education and<br />

Administration, University of New England Medical School, Portland, Maine; Assistant Professor of Emergency Medicine,<br />

NEOMED, Rootstown, Ohio; Professor of Emergency Medicine, Ohio University, Athens, Ohio<br />

Environmental Disorders<br />

iv


Michelle Blanda, MD, FACEP<br />

Professor and Chair Emeritus of Emergency Medicine, Northeast Ohio Medical University, Akron City Hospital,<br />

Summa Health System, Akron, Ohio<br />

Renal and Urologic Disorders<br />

Michael C. Bond, MD, FACEP, FAAEM<br />

Associate Professor, Residency Program Director, Department of Emergency Medicine, University of Maryland School<br />

of Medicine, Baltimore, Maryland<br />

Orthopedic Emergencies<br />

Laura J. Bontempo, MD, MEd, FAAEM<br />

Assistant Professor, Emergency Medicine, University of Maryland School of Medicine, Baltimore, Maryland<br />

Cutaneous Disorders<br />

Brian Browne, MD, FACEP<br />

Professor of Emergency Medicine, Chair, Department of Emergency Medicine, University of Maryland School of<br />

Medicine, Baltimore, Maryland<br />

Cutaneous Disorders<br />

Luca R. Delatore, MD<br />

Medical Director of the James Emergency Department, Assistant Professor, Department of Emergency Medicine,<br />

The Ohio State University Wexner Medical Center, Columbus, Ohio<br />

Oncologic Disorders<br />

Paul de Saint Victor, MD, FACEP, MHA, CPE<br />

Associate Director, Emergency Medicine Residency, Mercy St. Vincent Medical Center, Toledo, Ohio; President,<br />

Emergency Department Consultants<br />

Additional Tips for Good Performance, How to Take a Multiple-Choice Exam<br />

Sarah B. Dubbs, MD<br />

Clinical Assistant Professor, Assistant Residency Program Director, Department of Emergency Medicine, University<br />

of Maryland School of Medicine, Baltimore, Maryland<br />

Gynecologic and Obstetric Disorders<br />

Scott Felten, MD, FACEP<br />

US Acute Care Solutions Residency Director, Tulsa, Oklahoma<br />

Immune System Disorders<br />

William R. Fraser, DO, FACOEP-D<br />

Emergency Medicine Residency Director, Doctor's Hospital Ohio Health, Columbus, Ohio<br />

Abdominal and Gastrointestinal Disorders<br />

Hannah L. Hays, MD<br />

Assistant Professor, Department of Emergency Medicine and Pediatrics, The Ohio State University Wexner Medical<br />

Center and Nationwide Children's Hospital and Central Ohio Poison Center, Columbus, Ohio<br />

Toxicologic Disorders<br />

Colin G. Kaide, MD, FACEP, FAAEM<br />

Associate Professor of Emergency Medicine, Attending Physician of Emergency Medicine and Hyperbaric Medicine,<br />

The Ohio State University Wexner Medical Center, Columbus, Ohio<br />

Hematologic Disorders<br />

V


Jonathan Keary, MD, FACEP<br />

Staff Physician, Emergency Services Institute, Cleveland Clinic, Cleveland, Ohio<br />

Endocrine, Metabolic, and Nutritional Disorders<br />

Benjamin J. Lawner, DO, MS, EMT-P, FAAEM<br />

Assistant Professor, Department of Emergency Medicine, University of Maryland School of Medicine, Deputy EMS<br />

Medical Director, Baltimore City Fire Department, Baltimore, Maryland<br />

Emergency Medical Services<br />

Le N. Lu, MD, MS<br />

Clinical Assistant Professor, Director, Pediatric Emergency Medicine Education, Department of Emergency Medicine,<br />

University of Maryland School of Medicine; Director, Pediatric Emergency Department, Upper Chesapeake Medical<br />

Center, Baltimore, Maryland<br />

Pediatric Emergencies<br />

Patrick J. Maloney, MD<br />

Director, Pediatric Emergency Services, Mission Hospital and Mission Children's Hospital, Asheville, North Carolina<br />

Traumatic Disorders<br />

Laura Matrka, MD<br />

Assistant Professor, Otolaryngology-Head & Neck Surgery, The Ohio State University Wexner Medical Center,<br />

Columbus, Ohio<br />

Head, Ear, Eye, Nose, and Throat Disorders<br />

Stacy Mccallion, MD<br />

Summa Health System, Akron City Hospital, Akron, Ohio; Associate Professor of Emergency Medicine, NEOMED,<br />

Rootstown, Ohio; Professor of Emergency Medicine, Ohio University, Athens, Ohio<br />

Environmental Disorders<br />

Michael J. McCrea, MD, FACEP, FAAEM<br />

Assistant Residency Director, Simulation Education Director, Emergency Medicine Residency, Mercy St. Vincent<br />

Medical Center, Toledo, Ohio<br />

Cardiovascular Disorders<br />

Jillian L. McGrath, MD, FACEP<br />

Assistant Professor of Emergency Medicine, Department of Emergency Medicine, The Ohio State University Wexner<br />

Medical Center, Columbus, Ohio<br />

Toxicologic Disorders<br />

Colleen M. McQuown, MD, FACEP<br />

Director of Research Education, Department of Emergency Medicine, Summa Akron City Hospital; Associate Professor,<br />

Northeast Ohio Medical University, Rootstown, Ohio<br />

Other Components of the Practice of Emergency Medicine<br />

Siamak Moayedi, MD<br />

Assistant Professor, Department of Emergency Medicine, University of Maryland School of Medicine, Baltimore, Maryland<br />

Systemic Infectious Disorders<br />

Sreeja M. Natesan, MD<br />

Assistant Professor, Duke University Medical Center, Durham, North Carolina<br />

Thoracic and Respiratory Disorders<br />

vi


Sarah K. Sommerkamp, MD, RDMS<br />

Assistant Professor, Department of Emergency Medicine, University of Maryland School of Medicine, Baltimore, Maryland<br />

Gynecologic and Obstetric Disorders<br />

Matthew Tabbut, MD, FACEP<br />

Attending Physician, Department of Emergency Medicine, MetroHealth Medical Center; Assistant Professor, Case<br />

Western University School of Medicine, Cleveland, Ohio<br />

Procedures and Skills Integral to the Practice of Emergency Medicine<br />

Laura R. Thompson, MD, MS, FACEP<br />

Assistant Professor, Department of Emergency Medicine, The Ohio State University Wexner Medical Center,<br />

Columbus, Ohio<br />

Psychobehavioral Disorders<br />

Mercedes Torres, MD<br />

Clinical Assistant Professor, Department of Emergency Medicine, University of Maryland School of Medicine,<br />

Baltimore, Maryland<br />

Systemic Infectious Disorders<br />

Melissa Tscheiner, MD, FACEP<br />

Adjunct Assistant Professor, University of North Carolina School of Medicine-Asheville; Department of Emergency<br />

Medicine, Mission Hospital, Asheville, North Carolina<br />

Musculoskeletal Disorders (Nontraumatic)<br />

Travis Ulmer, MD, FACEP<br />

Vice President, US Acute Care Solutions, Canton, Ohio; Clinical Assistant Professor of Emergency Medicine,<br />

Ohio University Heritage College of Osteopathic Medicine, Athens, Ohio<br />

Head, Ear, Eye, Nose, and Throat Disorders<br />

PRODUCTION<br />

Laura L. Tiberi, MA, CAE<br />

Ohio ACEP Executive Director<br />

Holly J.F. Dorr, MBA, CAE<br />

Ohio ACEP Deputy Executive Director, Editorial and Production Coordinator<br />

Susan E. Aiello, ELS<br />

Medical Editor<br />

Sheryl Lazenby<br />

Designer<br />

vii


VIII


TABLE OF CONTENTS<br />

VOLUME I<br />

Foreword ........................................................................................................................................................................ x<br />

Introduction .................................................................................................................................................................. xi<br />

Recommended Study Plan ........................................................................................................................................... xiii<br />

Cardiovascular Disorders .............................................................................................................................................. 1<br />

Head, Ear, Eye, Nose, and Throat Disorders ................................................................................................................ 97<br />

Abdominal and Gastrointestinal Disorders ............................................................................................................... 169<br />

Thoracic and Respiratory Disorders .......................................................................................................................... 231<br />

Traumatic Disorders .................................................................................................................................................. 297<br />

Orthopedic Emergencies ........................................................................................................................................... 343<br />

Musculoskeletal Disorders (Nontraumatic) ............................................................................................................... 393<br />

Nervous System Disorders ........................................................................................................................................ 409<br />

Gynecologic and Obstetric Disorders ....................................................................................................................... 443<br />

VOLUME II<br />

Pediatric Emergencies ............................................................................................................................................... 489<br />

Toxicologic Disorders ................................................................................................................................................ 543<br />

Endocrine, Metabolic, and Nutritional Disorders ...................................................................................................... 591<br />

Environmental Disorders ........................................................................................................................................... 633<br />

Psychobehavioral Disorders ...................................................................................................................................... 673<br />

Hematologic Disorders ............................................................................................................................................. 711<br />

Oncologic Disorders ................................................................................................................................................. 743<br />

Systemic Infectious Disorders ................................................................................................................................... 759<br />

Immune System Disorders ........................................................................................................................................ 783<br />

Renal and Urologic Disorders ................................................................................................................................... 803<br />

Cutaneous Disorders ................................................................................................................................................. 829<br />

Emergency Medical Services ..................................................................................................................................... 869<br />

Procedures and Skills Integral to the Practice of Emergency Medicine ..................................................................... 883<br />

Other Components of the Practice of Emergency Medicine ...................................................................................... 941<br />

Mechanics of the Written Board Exam ....................................................................................................................... 975<br />

Additional Tips for Good Performance ....................................................................................................................... 979<br />

ix


FOREWORD<br />

FOREWORD<br />

Welcome to the eighth edition of Dr. Carol Rivers' Preparing for the Written Board Exam in Emergency Medicine. Initially<br />

created by Dr. Carol S. Rivers, a national leader in board review education, this book has long been the gold standard<br />

for comprehensive emergency medicine review and exam preparation. Dr. Rivers was residency trained in emergency<br />

medicine and extensively involved in the educational aspects of emergency medicine as a lecturer, a writer, a featured<br />

speaker, instructor, and tutor ... helping those who wanted and needed a firmer grasp of the practice of emergency<br />

medicine. In 1983, Dr. Rivers began to focus her teaching efforts on preparing physicians for the certifying examinations<br />

in emergency medicine.<br />

We celebrate her passion as a teacher, role model, and mentor. Since her early days as a leader and visionary in the<br />

specialty of emergency medicine, Dr. Rivers devoted her life to the advancement of emergency medicine and to the<br />

improvement of patient care, one physician at a time. Through her educational endeavors, providing publications and<br />

tutorials for emergency physicians and residents, she demonstrated outstanding service, leadership, and commitment to<br />

the specialty of emergency medicine. Ohio ACEP is proud to continue the tradition of the Rivers' board review with this<br />

current revision.<br />

This book was envisioned, written, and compiled to fill a need-the need for a teaching textbook (not another reference<br />

book) aimed directly at the written board examination (not the entire field of emergency medicine) and designed for the<br />

busy physician who has limited time to devote to study and whose study time comes in unequal segments.<br />

In its eighth edition, Dr. Carol Rivers' Preparing for the Written Board Exam continues the tradition as a premier teaching<br />

textbook for preparing for the Written Board exam in emergency medicine. Significant improvements have been made to<br />

this text to enhance the reader's experience and to cover the American Board of Emergency Medicine (ABEM) core content<br />

of the "Model of the Clinical Practice of Emergency Medicine" (The EM Model) and the "Table of Specificity" of the<br />

American Board of Osteopathic Board of Emergency Medicine. We are confident that your use of this two-volume review<br />

text will give you a foundation for exam preparation and for passing an emergency medicine written certification exam.<br />

X


INTRODUCTION<br />

INTRODUCTION<br />

This book has been written primarily for emergency physicians preparing for a written certification or recertification<br />

exam, and for residents preparing for in-service exams in emergency medicine. Its purpose is to provide a concise,<br />

focused review of material that usually appears on these exams. Information that may seem "basic" is included, because<br />

it is essential to your understanding of principles and concepts that are important parts of the exam.<br />

While this book has been used as a reference text, it is primarily a teaching text. The largest division of the book-the<br />

review of medical topics-is essentially a set of notes that contain the knowledge you must have to pass the exam.<br />

Treat them as notes-as your own notes. Read them with a pen, pencil, or highlighter in hand. Underline or highlight<br />

information you especially want to remember. Write notes of your own in the margins and other space provided. As<br />

you read, you will notice that some material is repeated-on purpose. A teaching text requires some redundancy for<br />

emphasis and coherence.<br />

The eighth edition is an expanded and updated revision of the prior editions. New topics have been added that are<br />

pertinent to both the board exam and the residency in-service exams. Outdated information has been removed, and<br />

controversial issues have been clarified wherever possible. To enhance clinical acuity for the resident in training, a few<br />

bedside diagnostic techniques, in addition to descriptions of selected procedures, are included.<br />

You will occasionally be referred to other materials to enhance your study. Depending on how much time you have to<br />

prepare for this exam (see "Recommended Study Plan"), you may find the following references helpful:<br />

1. Tintinalli, et al, Emergency Medicine: A Comprehensive Study Guide, Eighth ed. McGraw-Hill Company, Inc.<br />

(referred to in this text as Tintinalli's text).<br />

2. Marx, et al, Rosen's Emergency Medicine: Concepts and Clinical Practice, Eighth ed., 2 vols. Saunders (referred<br />

to in this text as Rosen's Text).<br />

3. The bookstore at Ohio ACEP (www.ohacep.org)<br />

4. Self-assessment testing products such as the Physician's Evaluation and Educational Review in Emergency<br />

Medicine (PEER) products available through ACEP. A variety of other self-assessment tools are also available<br />

online.<br />

These are supplemental resources. If asked to review a particular subject (eg, LeFort facial fractures), you should go to<br />

Tintinalli's and/ or Rosen's texts and read the appropriate section. Another text that has become popular with practicing<br />

emergency physicians is Hardwood-Nuss' Clinical Practice of Emergency Medicine (Lippincott Williams & Wilkins,<br />

Philadelphia). In general, it is easier to read than Tintinalli or Rosen and uses a numbered, itemized format similar to that<br />

found in this text.<br />

The self-assessment or mock exam products may be valuable in your preparation, because they reacquaint you with the<br />

process of answering multiple-choice questions and help you define areas of weakness that require further study.<br />

Several radiographic images are on the written exam. Whenever possible, those that merit review are noted. As you read<br />

through the medical topics included in this text, it is recommended that you have a pad of note paper and a pen handy.<br />

As you come to a paragraph in which a radiograph or CT review of a particular entity is recommended, jot it down.<br />

When you have completed your I ist, review common radiographs for each diagnosis. This is important for you to do<br />

because the radiographs, CT, and MRI prints on the written board exam are not usually taken from standard textbooks;<br />

they usually come from teaching files. A good text reference is The Radiology of Emergency Medicine by Harris and<br />

Harris. You can also view radiographs, CTs, and MRls online with Ohio ACEP's case pictorial review at www.ohacepelearning.org.<br />

ECGs and other pictorials are also presented on the exam. An ECG text that has been recommended is ECGs for the<br />

Emergency Physician by Mattu and Brady (Blackwell). Board preparation courses traditionally have hundreds of photos<br />

and ECGs that are ideal for many participants. Ohio ACEP's Web-based e-learning site features over 1,400 cases that<br />

include diagnostic photos, radiographs, ultrasounds, and other images integrated with key facts to reinforce key concepts<br />

of emergency medicine. Another excellent pictorial source is Atlas of Emergency Medicine by Knoop, Stack, and Storrow<br />

(McGrawHill). In addition to clinical presentations, each entity includes a differential diagnosis, emergency department<br />

treatment and disposition, and clinical pearls.<br />

A special feature of this text is clinical presentations of specific diagnoses as clinical scenarios in a storytelling format.<br />

This concept is based on the "simple chain technique," the object of which is to "chain" or link one item to the next in<br />

the order you wish to remember them. Basically, this chain is a story that involves all the items you want to remember in<br />

a particular sequence. You will be able to do this by using mental images and tying each item to the next. How does the<br />

chaining technique work? One item acts as a stimulus (or cue) for the next item. It's almost like seeing a slide show or a<br />

movie in your mind's eye, where you can automatically anticipate the next scene. You don't have to strain your mental<br />

faculty searching for it; it's right there.<br />

Chaining has been tested in scientific studies that have verified its effectiveness as a memory-enhancing tool for<br />

simple rote memory tasks. In this respect, imagery is probably the single most important aspect of memory training. In<br />

numerous studies, one group of students would be shown pictures of an array of items to remember, and the other group<br />

XI


INTRODUCTION<br />

would simply be given a list of words to remember. Both groups would be given the same test to determine what they<br />

remembered. The people who had been given images consistently scored higher than those who had been given lists.<br />

The important point here is that vivid images of items will improve your memory of those items.<br />

Why should the seemingly insignificant procedure of linking one item to another dramatically improve recall? The<br />

answer is found to be in the linking itself. Separate pieces of information become unified when connected to each other.<br />

In other words, when 20 pieces of information are presented separately, you have to remember 20 independent segments<br />

of information. But when we link the items together, as we do in chaining, the 20 items actually represent only one<br />

segment of data.<br />

Some areas of this text do contain memory recall facts (ie, lists), and they are unavoidable. However, these "lists" are<br />

easier to remember when they are mixed with imagery. Pure memorizing is a left brain function. Imagery is a right brain<br />

function. When <strong>combined</strong>, the reader remembers more detail.<br />

Before you begin reading this text, briefly scan each of the sections so that you can get an overview of the information<br />

presented. You might want to read the nonclinical/nonacademic sections first, because other activities are discussed<br />

there and you will want to allow time to schedule some or all of them in the time before the exam. You have enough<br />

information in this text to pass the exam, but other activities can help you as well. So pick and choose those that sound<br />

worthwhile, and then plan your time to include them in your final preparation.<br />

xii


RECOMMENDED STUDY PLAN<br />

RECOMMENDED STUDY PLAN<br />

Engage in regular and consistent study, because knowledge is acquired by studying over a reasonable period of time.<br />

What is learned in a hurry is seldom completely learned and soon forgotten. Do not "cram" for this exam. Cramming is<br />

an attempt to learn in a very short period of time what should have been learned through regular and consistent study<br />

over a period of weeks or months. Cramming seldom pays off in terms of effective learning. If you cram, it is likely that<br />

you will be more confused than prepared on the day of the exam.<br />

Conditions and Components<br />

Before you start reading this text, an organized approach will help maximize your time and effort. It is important<br />

to strategize and focus on the topics that generally have a greater degree of emphasis on the exams. The list below<br />

describes the relative weight given to different elements of the EM Model on the ABEM Examination.<br />

Cardiovascular Disorders<br />

Traumatic Disorders<br />

Signs, Symptoms, and Presentations<br />

Abdominal and Gastrointestinal Disorders<br />

Procedures and Skills<br />

Thoracic-Respiratory Disorders<br />

Head, Ear, Eye, Nose, and Throat Disorders<br />

Nervous System Disorders<br />

Systemic Infectious Disorders<br />

Toxicologic Disorders<br />

Obstetrics and Gynecology<br />

Psychobehavioral Disorders<br />

Environmental Disorders<br />

Musculoskeletal Disorders (Nontraumatic)<br />

Other Components<br />

Renal and Urogenital Disorders<br />

Endocrine, Metabolic and Nutritional Disorders<br />

Hematologic Disorders<br />

Immune System Disorders<br />

Cutaneous Disorders<br />

Total<br />

10%<br />

10%<br />

9%<br />

8%<br />

8%<br />

8%<br />

5%<br />

5%<br />

5%<br />

5%<br />

4%<br />

4%<br />

3%<br />

3%<br />

3%<br />

3%<br />

2%<br />

2%<br />

2%<br />

1%<br />

100%<br />

Pediatric: 8% minimum<br />

Geriatric: 6% minimum<br />

The next point you need to consider is how much time you have left before the exam. Check off the time frame that is<br />

most appropriate to your situation.<br />

Less than one month<br />

One to two months<br />

Two to three months<br />

Three months or more<br />

Now that you know which topics are covered most heavily on the written board exam and how much time you have left<br />

to study, a methodical approach is in order. There are four steps in this process:<br />

1. A self-assessment evaluation to determine strong and weak areas.<br />

2. A comparison of your self-assessment score with an analysis of your current level of preparedness for the exam.<br />

3. A study plan that includes the number of hours per day and the number of days per week you need to study.<br />

4. A method of reading and reviewing that promotes high retention and recal I of specific information.<br />

XIII


RECOMMENDED STUDY PLAN<br />

These four points will be described further in the paragraphs that follow. Using the time frame that you previously selected,<br />

choose the category that pertains to you and read the program outlined therein. That is your program, your study-approach<br />

method. Some of you will already have a plan in mind and may decide to incorporate parts of the information from within the<br />

different categories, and that's fine. But for those looking for direction, these categories have been written primarily for you.<br />

Category I: Less than 1 month<br />

1. Preceding each academic chapter is a series of multiple-choice questions. Go through the questions for every<br />

chapter in the academic section, recording your answers in the book or on a separate sheet of paper. When you're<br />

finished, use the answer key at the end of each series of questions and mark your incorrect answers. Use the<br />

pre-chapter multiple-choice question worksheet (on page xvi) to record and determine your percentage of correct<br />

answers for each section.<br />

2. Analyze your percentage score for the questions from each chapter to see where you are in the scheme of things:<br />

a. 85% or higher is a good score. Review the questions missed, and note the correct answers.<br />

b. 75%-85% is reasonably good, but further study is needed. For the questions you missed, review those<br />

particular subjects within the body of that chapter.<br />

c. Anything


RECOMMENDED STUDY PLAN<br />

Category Ill (2-3 months)<br />

1. With additional time to prepare, a self-assessment testing product such as PEER content review or an on line<br />

question assessment tool may be of value to your study plan. These products typically include multiple tests and an<br />

evaluation of your performance on the tests. The tests will give you a good sense of your priority areas.<br />

2. Plan your study schedule. If you have about 2 months, set aside about 3 hours a day, 5 days a week; it is<br />

recommended that you do not study more than 2 hours at one sitting, because your ability to retain information is<br />

likely to diminish after that time. If you have 3 months to study, 2 hours a day, 5 days a week should be sufficient.<br />

3. As you go through this text, use a marker to highlight key points, ie, information you didn't know previously or that<br />

seems important. Be sure to review the radiographs and other pictorials noted in the text, using the recommended<br />

sources. Divide your material into "easy reading" and "hard reading" categories. An example might be:<br />

Easy Reading<br />

Mechanics of the Written Board Exam<br />

"Light" academic chapters (eg, cutaneous, EMS,<br />

emergency department administration)<br />

Additional Tips for Good Performance<br />

Hard Reading<br />

Self-assessment test-taking and review<br />

The first nine academic chapters (especially<br />

Cardiovascular Disorders)<br />

Tintinalli's text reading<br />

Alternate your study time between the two categories. Do "hard reading" during one study session and "easy<br />

reading" the next. This will maximize your learning time as well as allow you to maintain your stamina. When you<br />

have finished, go back and reread this entire review book, looking especially for "pearls" (information that looks<br />

important) and other information that clarifies your understanding of a particular subject. Be sure to go through the<br />

questions preceding each academic chapter and record your scores on page xvi; it will help focus your attention as<br />

you read the academic material. If you have time, reread again but, this time only the material you have highlighted,<br />

and then stop. The material most likely to be tested already appears in bold type.<br />

4. In the final 7-10 days before the exam, go through the flashcards (an additional product available through Ohio<br />

ACEP) and practice clinical scenarios two or three times and stop studying the day before the exam. You're ready.<br />

Category IV (3 months or more)<br />

1. With additional time to prepare, a self-assessment testing product such as PEER content review or an on line<br />

question assessment tool may be of value to your study plan. These products typically include multiple tests and an<br />

evaluation of your performance on the tests. The tests will give you a good sense of your priority areas.<br />

2. Plan your study schedule. If you have 3-4 months before the exam, plan on studying about 2 hours a day, 5 days<br />

a week. If you have more than 4 months, 2 hours a day, 4 days a week is sufficient. You may also want to attend<br />

a written board course. (See the section titled "Additional Tips for Good Performance" for info on written board<br />

courses).<br />

3. As you go through this text, use a marker to highlight key points, ie, info you didn't know previously or that seems<br />

important. The material most likely to be tested already appears in bold type. Be sure to review radiographs and<br />

other pictorials noted in the text, using the recommended sources. Divide your material into "easy reading" and<br />

"hard reading" categories. An example might be as follows:<br />

Easy Reading<br />

Mechanics of the Written Board Exam<br />

"Light" academic chapters (eg, cutaneous, EMS,<br />

emergency department administration)<br />

Additional Tips for Good Performance<br />

Hard Reading<br />

Self-assessment test-taking and review<br />

The first nine academic chapters (especially<br />

Cardiovascular Disorders)<br />

Tintinalli's text reading<br />

Alternate your study time between the two categories. Do "hard reading" during one study session and "easy<br />

reading" the next. This will maximize your learning time as well as allow you to maintain your stamina. When you<br />

have finished, go back and reread the entire book, looking especially for "pearls" (info that looks important) and any<br />

information that clarifies your understanding of a particular subject. Be sure to go through the questions preceding<br />

each academic chapter and record your scores on the next page; it will help focus your attention as you read the<br />

academic material. If you have time, reread again but, this time, only material you have highlighted, and then stop.<br />

4. In the last 2 weeks before the exam, go through the flashcards (an additional product available through Ohio ACEP)<br />

and practice clinical scenarios until you know all the answers. Then stop studying. You are well prepared.<br />

xv


RECOMMENDED STUDY PLAN<br />

Regardless of in which "category" you place yourself, test-taking skills and strategies can play a significant role in your<br />

exam score. Be sure to read "How To Take a Multiple-Choice Exam" in the section titled "Mechanics of the Written<br />

Board Exam." Practice using the techniques described for answering questions for which you don't know the answers.<br />

Start first with the questions in this book. If you have time, do the same with the self-assessment testing products. Use<br />

other sources as well, such as the flashcards (an additional product available through Ohio ACEP) that supplement this<br />

text. Some written board preparation courses also offer practice questions. Under no circumstances is it recommended<br />

that you focus your attention on test-taking techniques at the expense of studying the academics of emergency medicine.<br />

If your knowledge base is lacking, no amount of test-taking skill is going to result in a passing score. In addition, these<br />

techniques don't work all the time. But if you're one of those candidates who scores 73%-74% on exam content,<br />

knowing how to use test-taking techniques can raise your score another percent or two- just enough to help you pass.<br />

Pre-chapter Multiple Choice Self-Assessment Questions Worksheet<br />

Chapter<br />

Cardiovascular Disorders 35<br />

Head, Ear, Eye, Nose, Throat Disorders 56<br />

Abdominal and Gastrointestinal Disorders 39<br />

Thoracic and Respiratory Disorders 27<br />

Traumatic Disorders 30<br />

Orthopedic Emergencies 35<br />

Musculoskeletal Disorders (Nontraumatic) 11<br />

Nervous System Disorders 16<br />

Gynecologic and Obstetric Disorders 19<br />

Pediatric Emergencies 41<br />

Toxicologic Disorders 21<br />

Endocrine, Metabolic, and Nutritional 29<br />

Disorders<br />

Environmental Disorders 34<br />

Psychobehavioral Disorders 15<br />

Hematologic Disorders 15<br />

Oncologic Disorders 11<br />

Systemic Infectious Disorders 14<br />

Immune System Disorders 14<br />

Renal and Urologic Disorders 11<br />

Cutaneous Disorders 17<br />

Emergency Medical Services 9<br />

Procedures and Skills 13<br />

Other Components 22<br />

Total 534<br />

Scoring<br />

• 85% or higher is a good score, Review any questions missed, and note the correct answers.<br />

# answered # of pre-chapter Percentage of Missed questions<br />

correctly questions correct answers to review<br />

• 75%-85% is reasonably good, but further study is needed. For the questions you missed, review those particular subjects within the body of<br />

that chapter.<br />

• Anything


CARDIOVASCULAR DISORDERS<br />

CARDIOVASCULAR DISORDERS<br />

DYSRHYTHMIAS .......................................................................................................................................................... 9<br />

Basic Principles of Cardiac Conduction Disturbances ................................................................................................... 9<br />

Standard ECG and Rhythm Strips ............................................................................................................................ 9<br />

Normal Morphology ............................................................................................................................................... 9<br />

Causes of Abnormal Morphologies ......................................................................................................................... 9<br />

Specific Rhythm Assessments ...................................................................................................................................... 14<br />

Sinus Rhythm ....................................................................................................................................................... 14<br />

Premature Atrial Contractions ............................................................................................................................... 14<br />

Sinus Tachycardia ................................................................................................................................................. 14<br />

Sinus Bradycardia ................................................................................................................................................. 15<br />

Supraventricular Tachycardia ................................................................................................................................ 15<br />

Atrial Fibrillation .................................................................................................................................................. 15<br />

Atrial Flutter ......................................................................................................................................................... 15<br />

Multifocal Atrial Tachycardia ................................................................................................................................ 16<br />

Junctional Premature Contractions ....................................................................................................................... 16<br />

Premature Ventricular Contractions ...................................................................................................................... 16<br />

Ventricular Tachycardia ........................................................................................................................................ 17<br />

Ventricular Fibrillation .......................................................................................................................................... 18<br />

Pulseless Electrical Activity ................................................................................................................................... 18<br />

Bundle-Branch Blocks .......................................................................................................................................... 18<br />

Si noatrial Block .................................................................................................................................................... 1 9<br />

Sick Sinus Syndrome ............................................................................................................................................ 20<br />

Atrioventricular Blocks ......................................................................................................................................... 20<br />

Preexcitation Syndromes ...................................................................................................................................... 21<br />

Etiologies and Management of Dysrhythmias .............................................................................................................. 22<br />

Premature Atrial Contractions ............................................................................................................................... 22<br />

Sinus Tachycardia ................................................................................................................................................. 22<br />

Sinus Bradycardia ................................................................................................................................................. 22<br />

Supraventricular Tachycardia ................................................................................................................................ 23<br />

Atrial Fibrillation .................................................................................................................................................. 25<br />

Atrial Flutter ......................................................................................................................................................... 26<br />

Multifocal Atrial Tachycardia ................................................................................................................................ 26<br />

Junctional Premature Contractions ....................................................................................................................... 27<br />

Premature Ventricular Contractions ...................................................................................................................... 27<br />

Ventricular Tachycardia ........................................................................................................................................ 28<br />

Ventricular Fibrillation .......................................................................................................................................... 29<br />

Pulseless Electrical Activity ................................................................................................................................... 30<br />

Sick Sinus Syndrome ............................................................................................................................................ 30<br />

Atrioventricular Blocks ......................................................................................................................................... 30<br />

Wolff-Parkinson-White Syndrome ......................................................................................................................... 31<br />

Pacemakers ................................................................................................................................................................. 32<br />

Emergency Pacing Techniques .............................................................................................................................. 32<br />

Indications for Temporary Cardiac Pacing ............................................................................................................. 32<br />

Tips for Temporary Transvenous Pacemaker Placement ......................................................................................... 32<br />

r--· Pacemaker Failure ................................................................................................................................................ 33<br />

Automatic Implantable Cardioverter Defibrillators ..................................................................................................... 34<br />

ACUTE CORONARY SYNDROME .............................................................................................................................. 36<br />

Definition .................................................................................................................................................................... 36<br />

Clinical Presentations, Risk Factors, and Predictive Factors ........................................................................................ 36


CARDIOVASCULAR DISORDERS<br />

Diagnosis .................................................................................................................................................................... 37<br />

Management ............................................................................................................................................................... 42<br />

Complications ............................................................................................................................................................. 47<br />

CONGESTIVE HEART FAILURE ................................................................................................................................... 49<br />

Etiology, Precipitating Factors, and Signs and Symptoms ............................................................................................ 49<br />

Diagnostic Evaluation ................................................................................................................................................. 50<br />

Management ............................................................................................................................................................... 50<br />

CARDIOMYOPATHIES AND SPECIFIC HEART MUSCLE DISEASES ............................................................................ 53<br />

Cardiomyopathies ....................................................................................................................................................... 53<br />

Definition ............................................................................................................................................................. 53<br />

Idiopathic Dilated (Congestive) Cardiomyopathy .................................................................................................. 53<br />

Restrictive Cardiomyopathy .................................................................................................................................. 54<br />

Hypertrophic Cardiomyopathy ............................................................................................................................. 54<br />

Specific Heart Muscle Diseases ................................................................................................................................... 56<br />

PERICARDIAL DISORDERS ......................................................................................................................................... 58<br />

Pericarditis .................................................................................................................................................................. 58<br />

Pericardia! Tamponade ................................................................................................................................................ 60<br />

MYOCARDITIS ............................................................................................................................................................ 62<br />

Etiology ....................................................................................................................................................................... 62<br />

Clinical Presentation ................................................................................................................................................... 62<br />

Diagnostic Evaluation ................................................................................................................................................. 63<br />

Management ............................................................................................................................................................... 63<br />

VALVULAR HEART DISEASE ....................................................................................................................................... 65<br />

Mitra! Valve Prolapse ("Click Murmur Syndrome") ..................................................................................................... 65<br />

Mitra! Stenosis ............................................................................................................................................................ 65<br />

Mitra! Regurgitation .................................................................................................................................................... 66<br />

Aortic Stenosis ............................................................................................................................................................ 68<br />

Aortic Regurgitation .................................................................................................................................................... 68<br />

Tricuspid Stenosis ........................................................................................................................................................ 70<br />

Tricuspid Regurgitation ............................................................................................................................................... 70<br />

Prosthetic Valves ......................................................................................................................................................... 71<br />

Conditions Necessitating Antibiotic Prophylaxis for Infective Endocarditis ................................................................ 73<br />

ENDOCARDITIS ......................................................................................................................................................... 74<br />

Definition .................................................................................................................................................................... 74<br />

Pathophysiology .......................................................................................................................................................... 7 4<br />

Clinical Presentation ................................................................................................................................................... 75<br />

Diagnostic Evaluation ................................................................................................................................................. 75<br />

Diagnosis .................................................................................................................................................................... 76<br />

Management ............................................................................................................................................................... 7 6<br />

THORACIC AORTIC DISSECTIONS AND ABDOMINAL AORTIC ANEURYSMS ......................................................... 77<br />

Thoracic Aortic Dissections ........................................................................................................................................ 77<br />

Epidemiology ....................................................................................................................................................... 77<br />

Pathophysiology ................................................................................................................................................... 77<br />

Natural History ..................................................................................................................................................... 77<br />

Classification System ............................................................................................................................................ 77<br />

Clinical Presentation ............................................................................................................................................ 78<br />

Diagnostic Evaluation ........................................................................................................................................... 79<br />

Management ........................................................................................................................................................ 80<br />

Abdominal Aortic Aneurysms ...................................................................................................................................... 81<br />

Definition and Anatomic Location ........................................................................................................................ 81<br />

Pathogenesis ......................................................................................................................................................... 81<br />

Risk Factors .......................................................................................................................................................... 81<br />

Clinical Presentation ............................................................................................................................................ 81<br />

Diagnosis and Management ................................................................................................................................. 82<br />

2


CARDIOVASCULAR DISORDERS<br />

DEEP VENOUS THROMBOSIS .................................................................................................................................... 83<br />

Pathogenesis ................................................................................................................................................................ 83<br />

Clinical Presentation ................................................................................................................................................... 83<br />

Acute Deep Venous Thrombosis ........................................................................................................................... 83<br />

Massive Deep Venous Thrombosis ........................................................................................................................ 83<br />

Risk Factors ................................................................................................................................................................. 83<br />

Predicting Pretest Probability for DVT: The Well's Clinical Criteria ............................................................................. 84<br />

Diagnostic Evaluation ................................................................................................................................................. 84<br />

Pharmacologic Therapy ............................................................................................................................................... 86<br />

Disposition-Admission Criteria ................................................................................................................................. 87<br />

HYPERTENSIVE EMERGENCIES AND URGENCIES ..................................................................................................... 88<br />

Hypertensive Emergencies .......................................................................................................................................... 88<br />

Definition ............................................................................................................................................................. 88<br />

Clinical Presentation ............................................................................................................................................ 88<br />

Management ........................................................................................................................................................ 88<br />

Hypertensive Urgencies .............................................................................................................................................. 90<br />

Definition and Etiology ......................................................................................................................................... 90<br />

Management ........................................................................................................................................................ 90<br />

3


CARDIOVASCULAR DISORDERS: SELF-ASSESSMENT QUESTIONS<br />

CARDIOVASCULAR DISORDERS: SELF-ASSESSMENT QUESTIONS<br />

1. ECG findings of tall T waves, wide QRS complexes, flattened P waves, and a prolonged PR interval are most<br />

consistent with the presence of:<br />

(a)<br />

Hypercalcemia<br />

(b) Hyperkalemia<br />

(c) Hypocalcemia<br />

(d) Hypokalemia<br />

2. The presence on ECG of sagging ST segments, short QT intervals, and flattened or inverted T waves is most<br />

accurately described as:<br />

(a)<br />

Digitalis effect<br />

(b) Digitalis toxicity<br />

(c)<br />

Hypercalcemia<br />

(d) Hypokalemia<br />

3. The treatment of ventricular tachycardia and supraventricular tachycardia is:<br />

(a)<br />

Lidocaine<br />

(b) Adenosine<br />

(c) Cardioversion<br />

(d) Dependent on the hemodynamic stability of the patient<br />

4. All of the following are indications for pacemaker therapy except:<br />

(a) Asymptomatic bifascicular or trifascicular block<br />

(b) Hemodynamically unstable bradycardia unresponsive to drug therapy<br />

(c) Mobitz II second-degree AV block in the presence of an acute Ml<br />

(d) Overdrive of tachydysrhythmias refractory to drug therapy or electrical cardioversion<br />

5. The most common cause of failure to pace is:<br />

(a)<br />

Battery depletion<br />

(b) Oversensing<br />

(c) Undersensing<br />

(d) Wire fracture<br />

6. ST segment elevation in leads V 1<br />

-V 6<br />

with reciprocal ST depression in leads II, Ill, and a VF is characteristic of:<br />

(a) Acute anterior wall Ml<br />

(b) Acute inferior wall Ml<br />

(c) Acute lateral wall Ml<br />

(d) Acute posterior wall Ml<br />

7. All of the following drugs may be given through an endotracheal tube except:<br />

(a) Atropine<br />

(b) Epinephrine<br />

(c) Naloxone<br />

(d) Sodium bicarbonate<br />

8. Appropriate early therapy for acute pulmonary edema may include al I of the following agents except:<br />

(a)<br />

Digitalis<br />

(b) Furosemide<br />

(c) Nitroglycerin<br />

(d) Oxygen<br />

9. What is the earliest radiographic finding of congestive heart failure?<br />

(a)<br />

Alveolar edema<br />

lb) Cardiomegaly<br />

(c) Interstitial edema<br />

(d) Pulmonary vascular redistribution to the upper lung fields<br />

4


CARDIOVASCULAR DISORDERS: SELF-ASSESSMENT QUESTIONS<br />

10. What is the most effective medical therapy for the treatment of atypical chest pain associated with hypertrophic<br />

cardiomyopathy?<br />

(a)<br />

~-blockers<br />

(b) Digitalis<br />

(c) Morphine<br />

(d) Nitroglycerin<br />

11. The diagnostic procedure of choice for detecting a pericardia! effusion occurring in association with pericarditis is:<br />

(a) Chest radiograph<br />

(b) ECG<br />

(c) Echocardiography<br />

(d) Radionuclide scanning<br />

12. The differential diagnosis of neck vein distention associated with hypotension includes al I of the following except:<br />

(a) Acute mitral regurgitation<br />

(b) Infective endocarditis<br />

(c) Pericardia! tamponade<br />

(d) Tension pneumothorax<br />

13. Most pulmonary emboli originate from venous thrombi in the:<br />

(a) Calf<br />

(b) Heart<br />

(c) Lower extremities and pelvis<br />

(d) Upper extremities<br />

14. All of the fol lowing statements regarding the use of aortography in the evaluation of acute aortic dissections are<br />

true except:<br />

(a) It can miss dissection if the false lumen is thrombosed.<br />

(b) It has been the traditional diagnostic gold standard.<br />

(c) It is 1 00% accurate.<br />

(d) Unlike rapid-sequence CT scanning with contrast, aortography also assesses the aortic valves and branches.<br />

15. All of the following statements regarding DeBakey type Ill aortic dissections are accurate except:<br />

(a)<br />

Initial management of these dissections is medical.<br />

(b) They are equivalent to Stanford type B dissections.<br />

(c) They are the most common type of dissection.<br />

(d) Long-term management of these dissections is usually medical.<br />

16. Medical therapy for aortic dissections is aimed at controlling the forces that propagate the dissection. The first-line<br />

agent(s) used to accomplish this goal is/are:<br />

(a)<br />

~-blockers and nitroglycerin<br />

(b) ~-blockers and nitroprusside<br />

(c) Nitroprusside alone<br />

(d) Trimethaphan<br />

17. A 30-year-old woman presents with palpitations of sudden onset and a feeling of nervousness. The patient denies<br />

drug use of any type and has had no other episodes of palpitations or anxiety. The nurse hands you a rhythm strip<br />

demonstrating an irregular tachycardia at 250 beats per minute with QRS complexes that vary in width between<br />

narrow and wide. There is no evidence of torsades. The likely etiology of this rhythm is:<br />

(a) Anxiety<br />

(b) Cocaine use<br />

(c) Mitral valve prolapse<br />

(d) Wolff-Parkinson-White syndrome<br />

5


CARDIOVASCULAR DISORDERS: SELF-ASSESSMENT QUESTIONS<br />

18. Atypical chest pain associated with mitral valve prolapse is treated with:<br />

(a) ~-blockers<br />

(b) Morphine<br />

(c) Nitroglycerin<br />

(d) Oxygen<br />

19. The treatment of choice for hypertension associated with eclampsia is:<br />

(a)<br />

Diuretics<br />

(b) Labetalol<br />

(c) Magnesium sulfate and hydralazine<br />

(d) Nitroprusside<br />

20. The primary difference between a hypertensive urgency and a hypertensive emergency is that in a hypertensive<br />

emergency:<br />

(a) It is usually treated with oral medications.<br />

(b) The diastolic blood pressure is 2:130 mmHg.<br />

(c) There is evidence of acute end-organ dysfunction or damage.<br />

(d) There is no history of hypertension.<br />

21. The ideal location of the catheter tip for temporary transvenous pacing is in the:<br />

(a) Apex of the right ventricle<br />

(b) lntraventricular septum<br />

(c)<br />

Right atrium<br />

(d) Superior vena cava<br />

22. Successful placement of a temporary transvenous pacemaker under ECG guidance is indicated by the observation<br />

of ____ on the cardiac monitor.<br />

(a)<br />

Small positive P waves and near-normal QRS complexes<br />

(b) Prominent inverted P waves and smaller negative QRS complexes<br />

(c) ST elevation<br />

(d) ST depression<br />

23. Placement of a pacemaker magnet over most permanent pacemakers results in:<br />

(a)<br />

Conversion from a demand to a fixed-rate mode<br />

(b) Conversion from a fixed-rate to a demand mode<br />

(c)<br />

Permanent disabling of the pacemaker<br />

(d) Temporary disabling of the pacemaker<br />

24. A 31-year-old man is brought in by ambulance for evaluation of a syncopal episode that occurred while he<br />

was playing a friendly game of competitive basketball. Examination reveals a rapid biphasic carotid pulse and<br />

a prominent systolic ejection murmur along the left sternal border and at the apex. You suspect hypertrophic<br />

cardiomyopathy and ask the patient to perform the Valsalva maneuver while you auscultate his heart. Assuming your<br />

diagnosis is correct, you would expect the intensity of the murmur to ____ with this maneuver.<br />

(a)<br />

Decrease<br />

(b) Disappear<br />

(c)<br />

Increase<br />

(d) Remain unchanged<br />

25. All of the following statements regarding hypertrophic cardiomyopathy are accurate except:<br />

(a)<br />

Amiodarone is the treatment of choice for the ventricular dysrhythmias that occur in patients with this disorder.<br />

(b) Digitalis and vasodilators are typically the most useful agents in the management of this disorder.<br />

(c)<br />

It is inherited in >50% of patients.<br />

(d) It is characterized by septa! hypertrophy without associated ventricular dilatation.<br />

6


CARDIOVASCULAR DISORDERS: SELF-ASSESSMENT QUESTIONS<br />

26. Antibiotic prophylaxis for bacterial endocarditis is definitely recommended in high-risk patients for which of the<br />

following procedures?<br />

(a) Endotracheal intubation<br />

(b) Dental procedures associated with significant gingival bleeding<br />

(c) Placement of a Foley catheter<br />

(d) Suturing of a laceration<br />

27. Antibiotic prophylaxis for endocarditis is indicated for major dental procedures in patients with all of the following<br />

conditions except:<br />

(a) Congenital heart disease<br />

(b) History of bacterial endocarditis<br />

(c) Mitra! valve prolapse<br />

(d) Prosthetic heart valves<br />

28. All of the following statements regarding prosthetic heart valves are accurate except:<br />

(a) Endocarditis should be suspected in any patient with a prosthetic valve who presents with fever, especially if a<br />

new regurgitation heart murmur is heard.<br />

(b) Mechanical valves require lifelong systemic anticoagulation.<br />

(c) Patients with valve dysfunction secondary to thrombus formation usually present with acute onset of CHF,<br />

hypotension, and muting (or loss) of the prosthetic valve sound.<br />

(d) Tissue valves cause greater hemolysis and are more thrombogenic than mechanical valves.<br />

29. In patients with automatic implantable cardioverter defibrillators (AICDs) in place who require CPR:<br />

(a) CPR is performed in the usual manner.<br />

(b) The provider may perceive an AICD shock if the device has not been deactivated.<br />

(c) Perception of an AICD shock by a provider is neither dangerous nor uncomfortable.<br />

(d) Al I of the above are correct.<br />

30. Janeway lesions are most accurately described as:<br />

(a) Nontender, erythematous macular lesions on the fingers, palms, soles<br />

(b) Nontender erythematous nodules on the dorsal aspect of the fingertips<br />

(c) Retinal hemorrhages with central clearing<br />

(d) Tender, erythematous nodules on the volar aspect of the fingertips<br />

31. The most useful test for making the diagnosis of endocarditis is:<br />

(a) Blood culture<br />

(b) CBC<br />

(c) ECG<br />

(d) Erythrocyte sedimentation rate<br />

32. The organism responsible for most cases of right-sided endocarditis is:<br />

(a) Enterococcus spp<br />

(b) Streptococcus epidermidis<br />

(c) Streptococcus viridans<br />

(d) Staphylococcus aureus<br />

33. Which of the following statements regarding mitral stenosis is false?<br />

(a) Typical presenting symptoms include dyspnea on exertion and hemoptysis.<br />

(b) Common ECG findings are left atrial enlargement and atrial fibrillation.<br />

(c) Most cases are the result of rheumatic heart disease.<br />

(d) The most common complication is infective endocarditis.<br />

34. The type of CHF that occurs in association with beriberi is most accurately characterized as:<br />

(a)<br />

High-output left ventricular failure<br />

(b) High-output right ventricular failure<br />

(c) Low-output left ventricular failure<br />

(d) Low-output right ventricular failure<br />

7


CARDIOVASCULAR DISORDERS: SELF-ASSESSMENT QUESTIONS<br />

35. When compared with the ST elevation that occurs in association with acute Ml, the ST elevation that occurs in<br />

association with acute pericarditis is:<br />

(a)<br />

Concave upward in configuration<br />

(b) More diffuse<br />

(c) Associated with concurrent PR depression<br />

(d) All of the above<br />

ANSWERS<br />

1. b 8. a 15. C 22. C 29. d<br />

2. a 9. d 16. b 23. a 30. a<br />

3. d 10. a 17. d 24. C 31. a<br />

4. a 11. C 18. a 25. b 32. d<br />

5. b 12. b 19. C 26. b 33. d<br />

6. a 13. C 20. C 27. C 34. a<br />

7. d 14. c 21. a 28. d 35. d<br />

Use the pre-chapter multiple choice question worksheet (page xvi) to record and determine the percentage of correct<br />

answers for this chapter.<br />

8


CARDIOVASCULAR DISORDERS<br />

DYSRHYTHMIAS<br />

I. BASIC PRINCIPLES OF CARDIAC CONDUCTION DISTURBANCES<br />

A. Standard ECG and rhythm strips<br />

1. Recordings are obtained at a paper speed of 25 mm/sec.<br />

2. The vertical axis measures distance; the smallest divisions are 1 mm long x1 mm.<br />

3. The horizontal axis measures time; each small division is 0.04 sec/mm.<br />

B. Normal morphology<br />

Courtesy of Dr. Michael McCrea<br />

1. P wave= atrial depolarization<br />

a. Upright in leads I, 11, 111, aVL, and aVF; inverted in lead aVR<br />

b. Measures


CARDIOVASCULAR DISORDERS<br />

Courtesy of Dr. Michael McCrea<br />

(2) Conduction delays: PR, QRS, and QT intervals may all be prolonged. QT interval is prolonged<br />

primarily because of selective prolongation of the ST segment.<br />

(3) Dysrhythmias: sinus bradycardia and atrial fibrillation with a slow ventricular response are the most<br />

commonly encountered in this setting; the risk of developing dysrhythmias increases as the core<br />

temperature falls below 86°F (30°C); at core temperatures below 77°F (25°C), spontaneous ventricular<br />

fibrillation and asystole may occur. (These patients must be handled gently, because dysrhythmias are<br />

easily introduced).<br />

b. Management of hypothermia-induced dysrhythmias<br />

(1) Most usually require only supportive therapy, because the dysrhythmias resolve spontaneously with<br />

rewarming.<br />

(2) Cardiac arrest<br />

(a)<br />

In profoundly hypothermic patients who appear to be in cardiac arrest, palpating pulses may be<br />

extremely difficult.<br />

(b) If unsure, began CPR without delay.<br />

(3) Ventricular fibrillation<br />

(a) Often refractory to defibrillation attempts until the patient is rewarmed<br />

(b) Defibrillation should be attempted with up to three shocks but, if unsuccessful, CPR and rapid<br />

rewarming measures should be instituted. Further attempts at defibrillation should be withheld<br />

until the patient's temperature rises above 86°F (30°C).<br />

(c) As the myocardium rewarms, ventricular fibrillation may convert spontaneously or resolve in<br />

response to defibrillation.<br />

(d) Magnesium sulfate produces spontaneous conversion in these patients.<br />

(4) The role of ACLS medications, including vasopressors, in severe hypothermic patients in cardiac arrest<br />

is of uncertain value, and standard algorithms may be used.<br />

(5) If narcotic abuse is suspected, naloxone should be considered, because it may act on central opiate<br />

receptors to decrease the severity of hypothermia seen in overdoses.<br />

(6) In general, a patient is not considered "all dead" until "warm and dead, 11 with warm being 95°F (35°C).<br />

2. Hypokalemia<br />

a. Progressively more prominent U wave (best seen in VJ<br />

b. Flattening ofT wave (earlier) followed by inversion (later)<br />

c. Depression of ST segment<br />

d. Prominent P wave<br />

e. Prolongation of the PR and QT (U) interval<br />

f. Ventricular tachycardia/torsades<br />

g. In the presence of hypokalemia, susceptibility to digitalis toxicity and its associated dysrhythmias is increased.<br />

3. Hyperkalemia<br />

Courtesy of Dr. Michael McCrea<br />

a. Tall hyperacute T wave (earliest ECG finding)<br />

b. Prolonged PR interval<br />

10


CARDIOVASCULAR DISORDERS<br />

c. Flattened or absent P wave<br />

d. Wide QRS complex that eventually blends with the T wave to assume a "sine wave" appearance<br />

e. Heart blocks<br />

f. QT interval normal or shortened<br />

Table 1: Effects on ECC with Increasing Potassium Concentration<br />

Potassium<br />

Concentration (mEq/L)<br />

5.5-6.5<br />

6.5-8.0<br />

>8.0<br />

ECG Changes<br />

Large amplitude T waves, peaked, tented symmetric<br />

PR interval prolongation<br />

P wave flattening/disappearance<br />

QRS widening<br />

Conduction block with escape beats<br />

Sine wave appearance<br />

Ventricular fibrillation<br />

Asystole<br />

4. Hypocalcemia<br />

a. Prolonged QT interval because of prolongation of the ST segment<br />

b. Terminal T wave inversion (less consistent finding)<br />

c. Ventricular dysrhythmias (including torsades de pointes)<br />

5. Hypercalcemia<br />

a. Depressed ST segments, widened T waves, and shortened ST segments and QT intervals.<br />

b. Bradyarrhythmias may occur, with bundle-branch patterns that may progress to second-degree block or<br />

complete heart block.<br />

6. Hypomagnesemia<br />

a. Prolonged PR and QT intervals<br />

b. Widened QRS complex<br />

c. ST segment abnormalities<br />

d. Flattened or inverted T waves (especially in the precordial leads)<br />

e. Ventricular dysrhythmias (premature ventricular contractions, ventricular tachycardia, torsades de pointes,<br />

ventricular fibri I lation)<br />

f. Hypomagnesemia usually occurs in association with other electrolyte abnormalities (particularly<br />

hypokalemia), and many of the ECG findings are similar to those seen with hypokalemia and hypocalcemia<br />

(pictured above).<br />

g. In the presence of hypomagnesemia, susceptibility to digitalis toxicity and its associated dysrhythmias is<br />

increased.<br />

7. Digitalis effects<br />

a. Sagging ST segment with its concavity directed upward (resembles a hockey stick)<br />

b. Short QT interval<br />

c. Flattened or inverted T wave<br />

d. Modestly prolonged PR interval<br />

e. These effects are especially prominent in the lateral leads and occur in most patients who are adequately<br />

digitalized; they are not an indication of digitalis toxicity.<br />

11


CARDIOVASCULAR DISORDERS<br />

8. Digitalis toxicity<br />

a. Pathophysiology: digitalis produces toxicity by<br />

(1) Poisoning the Na+-K+-ATPase pump - increased intracellular entry of Na+ and Ca++ and egress of K+ -<br />

increased excitability - ectopy and tachydysrhythmias<br />

(2) Increasing vagal tone and automaticity - decreased conduction in the AV node - bradydysrhythmias<br />

and AV blocks<br />

b. Factors that increase sensitivity to digitalis and predispose to toxicity<br />

(1) Electrolyte abnormalities (hyperkalemia or hypokalemia, hypomagnesemia, hypercalcemia)<br />

(2) Hypoxia<br />

(3) Metabolic alkalosis<br />

(4) Increasing age<br />

(5) Presence of underlying cardiac disease (ischemia, CHF, congenital heart disease)<br />

(6) Presence of chronic underlying systemic illness (COPD, renal failure, hypothyroidism)<br />

(7) Drug interactions (quinidine, calcium channel blockers, erythromycin, amiodarone, captopril, and<br />

ibuprofen)<br />

c. ECG findings<br />

(1) Premature ventricular contractions (most common arrhythmia), often bigeminal and multiform: the<br />

most common digitalis-induced rhythm disturbance<br />

(2) Junctional tachycardia (common)<br />

(3) Sinus bradycardia<br />

(4) Sinus tachycardia<br />

(5) Sinoatrial and AV nodal blocks<br />

(6) Sinus arrest<br />

(7) Torsades de pointes<br />

(8) Ventricular tachycardia<br />

(9) Ventricular fibrillation<br />

(10) Atrial tachycardias with AV block are very specific but not pathognomonic for digitalis toxicity<br />

(11) Nonparoxysmal junctional tachycardia<br />

(12) Atrial fibrillation with a slow ventricular response, ie, AV dissociation<br />

(13) Bidirectional ventricular tachycardia (highly suggestive of digitalis toxicity but rare)<br />

d. Clinical symptoms<br />

(1) Flu-like syndrome with profound malaise, anorexia, nausea, vomiting, and diarrhea<br />

(2) Visual disturbances (blurred vision, halos around objects, and yellow or green color aberrations)<br />

(3) Mental status changes, including confusion, drowsiness, and psychosis<br />

e. Acute digitalis toxicity is usually seen in young and otherwise healthy patients as a result of either<br />

accidental or intentional overdose; it is commonly associated with hyperkalemia, high digoxin levels,<br />

bradydysrhythmias, and AV blocks. Toxicity in these patients is most closely correlated with the degree<br />

of hyperkalemia (not the serum digoxin level). Chronic digitalis toxicity generally occurs in older cardiac<br />

patients with reduced renal function who are taking diuretics. These patients are usually normo- or<br />

hypokalemic, have digoxin levels that are minimally increased or normal, and most commonly have a<br />

ventricular dysrhythmia.<br />

f. Classic clinical scenario<br />

(1) Acute intoxication: A 3-year-old is brought in by his parents for evaluation after accidental ingestion<br />

of grandpa's "heart pills." Based on information obtained from the parents, the child has ingested 10.7<br />

mg of digoxin sometime within the past 2 hours and has vomited twice. The cardiac monitor shows<br />

a junctional rhythm with sinus block and type I second-degree AV block; laboratory studies reveal a<br />

potassium level of 6.2 mEq/L, along with a markedly increased digoxin level of 61. The child is on no<br />

medications and is otherwise healthy.<br />

(2) Chronic intoxication: A 65-year-old woman with a past medical history of coronary artery disease,<br />

CHF, and renal insufficiency is brought in by ambulance for evaluation. Her medications include<br />

furosemide, digitalis, sublingual nitroglycerin, and baby aspirin. According to family members, she<br />

has become progressively more confused and weak over the past few days and has not been eating<br />

well. The ECG shows a regular wide complex tachycardia with alternating QRS polarity (bidirectional<br />

ventricular tachycardia) and laboratory studies reveal a digoxin level of 3.5 and a potassium<br />

concentration of 3.0 mEq/L.<br />

12


CARDIOVASCULAR DISORDERS<br />

g. Management<br />

(1) IV line, oxygen, pulse oximeter, cardiac monitor<br />

(2) Gastric lavage is contraindicated because of risk of vagal stimulation causing bradycardia or asystole.<br />

(3) Administer multiple doses of activated charcoal to all patients with potentially toxic ingestions;<br />

activated charcoal prevents systemic absorption and, when multiple doses are given, enhances<br />

elimination by interrupting the prominent enterohepatic circulation of digitalis.<br />

(4) Seek and treat factors that may contribute to digitalis toxicity.<br />

(a) Hypokalemia (correct cautiously in the presence of AV blocks; correction can actually exacerbate<br />

AV conduction defects)<br />

(b) Hyperkalemia is best treated with Fab fragments (digoxin-specific antibody fragments); do not<br />

administer calcium; it can potentiate cardiotoxicity.<br />

(c) Hypomagnesemia<br />

(d) Hypoxia<br />

(e)<br />

Dehydration<br />

(5) Control tachydysrhythmias<br />

(a)<br />

Phenytoin and lidocaine are the drugs of choice for tachydysrhythmias.<br />

(b) Magnesium sulfate may also be useful in suppressing ventricular irritability.<br />

(c) Avoid cardioversion (digoxin decreases the fibrillatory threshold); restrict its use to situations of<br />

last resort and use the lowest possible energy level.<br />

(d) Avoid use of bretylium, Class IA antidysrhythmics (eg, procainamide, isoproterenol) and<br />

propranolol; these agents can exacerbate dysrhythmias and AV conduction disturbances.<br />

(6) Manage symptomatic bradycardia or AV block with atropine. If atropine is unsuccessful, cardiac<br />

pacing (external or transvenous) may be used while waiting for Fab fragments to take effect. External<br />

pacing is preferred, because transvenous pacemaker insertion may induce tachydysrhythmias in these<br />

patients.<br />

(7) Fab fragments<br />

(a) Should be administered to patients with:<br />

i. Ventricular dysrhythmias (ventricular fibrillation, ventricular tachycardia)<br />

ii. Symptomatic bradycardias unresponsive to atropine<br />

iii. Hyperkalemia (K+ >5.0 mEq/L) secondary to digitalis intoxication<br />

iv. Coingestions of cardiotoxic drugs (~-blockers, cyclic antidepressants)<br />

v. Large, potentially lethal digitalis intoxications<br />

vi. Ingestions of plants known to contain cardiac glycosides (oleander, lily of the valley) with<br />

severe dysrhythmias<br />

(b) Fab fragments bind free digoxin in the vascular and interstitial spaces and form an inert compound<br />

that is eliminated by the kidneys. Treatment rapidly corrects conduction defects, ventricular<br />

dysrhythmias, and hyperkalemia.<br />

(c) Dosage<br />

i. If the serum digoxin level or total amount of digoxin ingested is known, the formulas found<br />

in the package insert can be used to calculate the number of vials of Fab fragments to be<br />

administered.<br />

11. If the amount of digoxin ingested is unknown, the initial dose of Fab fragments should be<br />

5-10 vials (titrated incrementally).<br />

(d) After administration of Fab fragments, conventional assays for determining digoxin levels (which<br />

measure both bound and unbound digoxin) are unreliable for at least a week.<br />

13


CARDIOVASCULAR DISORDERS<br />

Table 2: ECG Findings in Various Conditions<br />

Condition ST PR QRS QT Pwave Twave Special Features<br />

Hypothermia Long Wide Long "J" wave, Osborne<br />

Hypokalemia Depressed Long Long Flat Progressively more<br />

prominent "U" wave<br />

Hyperkalemia Long Wide Flat Peaked May be associated<br />

with digitalis toxicity<br />

Hypocalcemia Long Inverted<br />

Hypercalcemia<br />

Short<br />

Hypomagnesem i a Abnormal Long Long Flat tSusceptibility to<br />

digitalis toxicity<br />

Digitalis effects Scooped Short Flat<br />

Digitalis toxicity<br />

Premature ventricular<br />

contractions most<br />

common<br />

II. SPECIFIC RHYTHM ASSESSMENTS<br />

A. Sinus rhythm<br />

Courtesy of Or. Michael McCrea<br />

1. Sinus rhythm is 60-100 beats per minute.<br />

2. The rhythm is regular with 1 :1 relationship of the P to QRS.<br />

3. P waves are upright in leads 1 1<br />

11 1<br />

and a VF. (Lead II is the typical lead for a rhythm strip.)<br />

4. There are no extra beats.<br />

B. Premature atrial contractions: extra beats that originate outside the sinus node from ectopic atrial<br />

pacemakers; appear interspersed throughout an underlying rhythm (usually sinus)<br />

1. These ectopic P waves are different in configuration from normal P waves and may or may not be conducted<br />

through the AV node. They usually have a normal PR interval (0.12-0.20 seconds)<br />

2. They are generally followed by a noncompensatory pause; the SA node is reset, and the returning sinus beat<br />

occurs ahead of schedule.<br />

C. Sinus tachycardia: exactly like a sinus rhythm except that the rate is> 100 (and usually


CARDIOVASCULAR DISORDERS<br />

D. Sinus bradycardia: exactly like a sinus rhythm except that the rate is 45) beats per minute<br />

I<br />

Courtesy of Dr. Michael McCrea<br />

E. Supraventricular tachycardia<br />

Courtesy of Dr. Daniel Schwerin<br />

1. P waves are abnormal and may not be visible (often hidden in the preceding T wave), or they may immediately<br />

follow the QRS complex, in which case they are often inverted ("retrograde P waves"); atrial rate is 120-200<br />

beats per minute.<br />

2. Rhythm is regular.<br />

3. QRS complexes are usually narrow but may be wide because of aberrant conduction through a bypass tract or<br />

preexisting bundle-branch block; ventricular rate is 120-220 beats per minute.<br />

F. Atrial fibrillation: an irregularly irregular rhythm due to uncoordinated atrial activation and random<br />

occurrence of ventricular depolarization. The atria are not pumping, but they do discharge electrical<br />

impulses to the ventricles; however, no single impulse depolarizes the atria completely, so only an<br />

occasional impulse gets through to the AV node. It is the most common sustained dysrhythmia; it occurs in<br />

2% of the general population and in 5% of people >60 years old.<br />

Courtesy of Dr. Michael McCrea<br />

1. P waves are absent but small irregular deflections in the baseline ("f" or "fibrillation waves") may be seen.<br />

They are most easily detected in the inferior leads (11, 111, and a VF) and in V 1<br />

-V 3<br />

• The atrial rate is 400-700 beats<br />

per minute.<br />

2. Because P waves are not visible, there is no PR interval.<br />

3. QRS complexes are normal in configuration, unless there is aberrant conduction.<br />

4. The rhythm is irregularly irregular.<br />

5. Ventricular response rate is variable but is generally 160-180 beats per minute in untreated patients; a rate<br />

>200 beats per minute with a wide QRS complex suggests Wolff-Parkinson-White syndrome with conduction<br />

through the accessory pathway; a regular, slow ventricular rate may be digitalis toxicity.<br />

G. Atrial flutter: a very rapid atrial rhythm but, because of AV nodal delay, ventricular responses are slower.<br />

Therefore, atrial flutter always occurs with some sort of AV block (not all impulses are conducted); the<br />

resulting block is either a fixed ratio (2:1, 3:1, 4:1, etc) or variable AV block.<br />

Fixed AV block 3:1 atrial flutter<br />

Courtesy of Dr. Michael McCrea<br />

15


CARDIOVASCULAR DISORDERS<br />

Variable AV block atrial flutter<br />

Courtesy of Dr. Michael McCrea<br />

1. P waves have a characteristic sawtooth pattern and are called "F" or "flutter waves." They are usually best seen<br />

in the inferior leads and leads V 1<br />

and V 2<br />

• The atrial rate is 250-350 beats per minute.<br />

2. The PR interval (when present) is always normal, but not every P wave is followed by a QRS complex.<br />

3. QRS complexes are normal in configuration.<br />

4. The ventricular rate is often 150 + 20 beats per minute but depends on the degree of block present and may be<br />

variable. Suspect atrial flutter with a 2:1 block in patients who present with a fixed regular ventricular rate of<br />

150 beats per minute.<br />

H. Multifocal atrial tachycardia: an irregular rhythm sometimes mistaken for atrial fibrillation; originates from<br />

multiple different atrial sites and is characterized by P waves of varying shape<br />

Courtesy of Dr. Daniel Schwerin<br />

1. There must be at least three distinct types of P waves in one lead; atrial rate is 100-180 beats per minute.<br />

2. The rhythm is irregularly irregular.<br />

3. The PP, PR, and RR intervals vary.<br />

4. Nonconducted (blocked) P waves are frequently present, particularly when the atrial rate is rapid. Classically<br />

seen with COPD and theophylline toxicity.<br />

5. Management is directed at treatment of the underlying condition. Cardioversion is ineffective. The rhythm itself<br />

should not cause hemodynamic instability.<br />

I. Junctional premature contractions: impulses that originate from an ectopic focus within the AV node or the<br />

bundle of His above the bifurcation. They may be isolated, multiple, or multifocal.<br />

Courtesy of Dr. Michael McCrea<br />

1. The ectopic P wave has a different shape and deflection (usually inverted in leads II, Ill, and a VF), and it may<br />

occur before, during, or after the QRS complex.<br />

2. When the P wave precedes the QRS, the PR interval is shorter than normal (often


CARDIOVASCULAR DISORDERS<br />

1. Characteristics<br />

a. Occur earlier than the next expected normal QRS<br />

b. Wider than a normal QRS (usually 2'.0.12 second)<br />

c. The QRS morphology is generally bizarre.<br />

d. A preceding P wave is absent; however, retrograde conduction of a premature ventricular contraction can<br />

occasionally result in an inverted P wave after the QRS complex.<br />

e. The deflection of the ST segment and T wave is opposite that of the QRS.<br />

f. May occur in regular pattern, eg, bigeminy (as above)<br />

g. Generally followed by a compensatory pause; the sinoatrial node is not reset, and the next P wave occurs<br />

at its usual time.<br />

K. Ventricular tachycardia: three or more consecutive premature ventricular contractions occurring at a regular<br />

rate> 120 beats per minute<br />

Courtesy of Dr. Michael McCrea<br />

1. P waves are usually absent; when present, they are either retrogradely conducted or have no relationship to the<br />

QRS (AV dissociation).<br />

2. QRS complexes are wide (2'.0.12 second) and may be bizarre.<br />

3. Fusion beats may be present; these are intermediate in appearance between a bizarre QRS complex and a<br />

normal QRS. When present, the diagnosis of ventricular tachycardia is certain.<br />

4. Capture beats are rarely seen but, when present, confirm the diagnosis of ventricular tachycardia. Capture beats<br />

are the result of an atrial impulse penetrating the AV node from above to stimulate ("capture") the ventricles.<br />

Because ventricular conduction occurs over the normal pathways, the resulting QRS of the captured beat looks<br />

normal (narrow) in appearance.<br />

5. Deflection of the ST segment and T wave is generally opposite that of the QRS complex.<br />

6. Rate is> 120 (usually 150-200) beats per minute.<br />

7. Rhythm is generally regular, although beat-to-beat variation may occur at the onset of tachycardia.<br />

8. QRS axis is generally constant.<br />

9. Ventricular tachycardia is classified as "monomorphic" (QRS complexes look the same) or "polymorphic"<br />

(QRS complexes have varying morphology). Current therapeutic modalities are based on this classification (see<br />

Etiologies and Management of Dysrhythmias, pages 22-32).<br />

10. Differentiation of supraventricular tachycardia with aberrancy from ventricular tachycardia<br />

a. Most published criteria that can be used to diagnose ventricular tachycardia are fairly reliable; however,<br />

there are no reliable criteria to exclude ventricular tachycardia. Because the misdiagnosis of ventricular<br />

tachycardia can be deadly, if there is any doubt about the diagnosis whatsoever, always assume that a wide<br />

complex tachycardia is ventricular tachycardia and treat as such!<br />

b. Fusion and capture beats indicate AV dissociation and are practically diagnostic of ventricular tachycardia.<br />

c. P waves preceding QRS complexes favor aberrancy.<br />

d. QRS concordance (all the QRS complexes from V 1<br />

to V 6<br />

are either positive or negative) strongly favors<br />

ventricular tachycardia. A fully compensatory pause is more likely to occur with ventricular tachycardia.<br />

e. Response to vagal maneuvers (Valsalva maneuver, carotid sinus massage) may occur with aberrant<br />

supraventricular tachycardia, whereas ventricular tachycardia is unaffected. (Carotid sinus massage is<br />

contraindicated in older patients with a history of carotid disease/stroke or the presence of a carotid bruit.)<br />

f. Marked left axis deviation (>30°) suggests ventricular tachycardia; any QRS axis deviation >40° in either<br />

direction (or an upright QRS in aVR) favors ventricular tachycardia.<br />

g. QRS duration >0.14 second favors ventricular tachycardia.<br />

h. QRS morphology in lead V 1<br />

: an RS, R or qR with left "rabbit ear" taller than the right suggests ventricular<br />

tachycardia, whereas an RSR' pattern is more likely supraventricular tachycardia with aberrancy; negative<br />

QRS morphology in this lead with a wide R wave (>0.03 second), RS interval >0.07 second, and a slurred<br />

or notched S wave favors ventricular tachycardia.<br />

1. QRS morphology in lead V 6<br />

: R/S ratio


CARDIOVASCULAR DISORDERS<br />

j. A bundle-branch pattern that varies suggests supraventricular tachycardia with aberrancy.<br />

k. A history of prior heart disease (Ml, CHF, coronary artery bypass graft) strongly favors ventricular<br />

tachycardia (likelihood 85%), as does a prior history of ventricular tachycardia.<br />

I. Age 2:'.50 years old favors ventricular tachycardia, whereas age :S35 years old favors an aberrant<br />

supraventricular tachycardia.<br />

11. Torsades de pointes ("twisting of the points") is a type of polymorphic ventricular tachycardia in which the QRS<br />

axis swings from a positive to a negative direction in a single lead creating a "sine-wave" appearance. It originates<br />

from a single focus and is usually precipitated by diseases or drugs that prolong the QT interval, such as Class<br />

IA antidysrhythmics (procainamide, quinidine), Class IC antidysrhythmics (propafenone, fecainide), tricyclic<br />

antidepressants, droperidol, and the phenothiazines. The <strong>combined</strong> use of certain drugs such as terfenadine plus<br />

ketoconazole or erythromycin also prolong the QT interval and may therefore precipitate torsades. Other causes<br />

include hypomagnesemia and hypokalemia. The rate is typically 200-240 beats per minute.<br />

Courtesy of Dr. Daniel Schwerin<br />

L. Ventricular fibrillation<br />

Courtesy of Dr. Michael McCrea<br />

1. Most commonly recognized as a fine or coarse zigzag pattern without discernible P waves, QRS complexes, or<br />

T waves representing the presence of unorganized ventricular electrical activity<br />

2. By definition, there is no organized perfusion and therefore absence of a pulse.<br />

3. Sometimes the rhythm may look like ventricular tachycardia. The point is moot if the patient has no pulse and<br />

is unresponsive, because the treatment is the same, unsynchronized defibrillation.<br />

M. Pulseless electrical activity (PEA)<br />

1. Refers to a heterogeneous group of rhythms characterized by the presence of some type of electrical activity<br />

other than ventricular tachycardia or ventricular fibrillation in the absence of a perceptible pulse<br />

2. Includes electromechanical dissociation, pseudoelectromechanical dissociation, idioventricular rhythms,<br />

ventricular escape rhythms, bradyasystolic rhythms, and postdefibrillation idioventricular rhythms<br />

3. These dysrhythmias often occur in association with specific clinical conditions that if promptly identified and treated<br />

may result in a return of spontaneous circulation. Remember the "H's and T's," looking for reversible causes.<br />

a. Hypovolemia<br />

b. Hydrogen ion, ie, acidosis<br />

C. Hypothermia<br />

d. Hypo/hyperkalemia<br />

e. Hypoxia<br />

f. Toxins<br />

g. Tamponade, cardiac<br />

h. Tension pneumothorax<br />

i. Thrombosis, coronary<br />

j. Thrombosis, pulmonary<br />

N. Bundle-branch blocks (BBBs)<br />

1. Abnormal conduction abnormalities (not rhythm disturbances) in which the ventricles depolarize in<br />

sequence (rather than simultaneously), thus producing a wide QRS complex (0.09-0.11 - incomplete BBB;<br />

2:'.0.12 second - complete BBB) and an ST segment with a slope opposite that of the terminal half of the<br />

QRS complex<br />

18


CARDIOVASCULAR DISORDERS<br />

2. Right BBB is a unifascicular block in which ventricular activation is by way of the left bundle branch; the<br />

impulse travels down the left bundle, thus activating the septum from the left side (as it normally does in<br />

the absence of right BBB). This is followed by activation of the free wall of the left ventricle and, finally, the<br />

free wall of the right ventricle. Because of the two changes in direction, there is a tendency toward triphasic<br />

complexes in right BBB.<br />

Courtesy of Dr. Michael McCrea<br />

a. Wide QRS complex (::,:0.12 second)<br />

b. Triphasic QRS complex (rSR' variant) in lead VJ<br />

c. Wide S waves in leads I, V 5<br />

, andV 6<br />

d. Normal septa! Q waves in leads I and V 6<br />

because the initial activation of the ventricle occurs in the normal<br />

manner<br />

e. T wave has a deflection opposite that of the terminal half of the QRS complex.<br />

f. Associated axis is variable; a normal axis, left axis deviation, or even right axis deviation may be present.<br />

3. Left BBB is a bifascicular block in which the left ventricle is activated by way of the right bundle branch; the<br />

impulse travels down the right bundle, activating the septum and the free wall of the right ventricle, and then<br />

continues on in the same direction to activate the free wall of the left ventricle. Because the dominant forces<br />

are traveling in the same direction, there is a tendency toward monophasic QRS complexes.<br />

Courtesy of Or. Michael McCrea<br />

a. Wide QRS complex (::,:0.12 sec)<br />

b. Negative wave (QS or rS) in lead VJ<br />

c. Large, wide R waves in leads I, aVL, V 5<br />

, and V 6<br />

d. Absence of normal septa! Q waves in leads I and V 6<br />

e. T wave has a deflection opposite that of the terminal half of the QRS complex.<br />

f. Associated left-axis deviation is most common and implies the presence of additional myocardial disease.<br />

0. Sinoatrial block<br />

1. Occurs when there is abnormal conduction between the sinus node and atrial muscle; recognized by the<br />

unexpected absence of a P wave and its associated QRS complex. Like AV block, sinoatrial block is also<br />

divided into first-, second-, and third-degree varieties.<br />

2. First-degree sinoatrial block<br />

a. Impulse is delayed in its conduction from the sinoatrial node to the atria.<br />

b. Cannot be diagnosed from a surface 12-lead ECG<br />

3. Second-degree sinoatrial block<br />

a. Some of the sinus node discharges are blocked.<br />

b. Recognized on an ECG as the absence of an expected P wave and its associated QRS complex<br />

4. Third-degree sinoatrial block (sinus arrest)<br />

Courtesy of Dr. Michael McCrea<br />

19


CARDIOVASCULAR DISORDERS<br />

a. All of the sinus node discharges are blocked.<br />

b. On ECG, it may appear as a long sinus pause/arrest or a junctional/ventricular escape rhythm.<br />

P. Sick sinus syndrome<br />

1. An abnormality of cardiac impulse formation as well as intra-atrial and AV nodal conduction<br />

2. Manifests as a wide variety of, or combinations of, bradyarrhythmias and tachyarrhythmias<br />

3. Most common in the elderly<br />

4. Presenting symptoms may include dizziness, palpitations, dyspnea, fatigue, lethargy, or syncope.<br />

Documentation of a bradyarrhythmia or tachyarrhythmia in association with these symptoms is the cornerstone<br />

of diagnosis.<br />

Q. Atrioventricular blocks<br />

1. Occur when the conduction between the atria and ventricles is abnormal; the conduction delay can occur in<br />

the atria, the AV node, or the proximal His-Purkinje system.<br />

2. First-degree AV block: normal AV conduction is slightly prolonged.<br />

Note PR 390 msec.<br />

Courtesy of Dr. Michael McCrea<br />

a. P waves and QRS complexes are normal.<br />

b. There is a 1 :1 relationship between the P and QRS.<br />

c. PR interval is prolonged (>0.20 second)<br />

d. The block is most often at the level of the AV node.<br />

3. Second-degree AV block: some atrial impulses are not conducted.<br />

a. Mobitz Type I (Wenckebach)<br />

(1) P waves and QRS complexes are normal, but there are P waves without QRS complexes<br />

(nonconducted P waves).<br />

(2) PR interval progressively lengthens and the RR interval progressively shortens until a beat is dropped.<br />

This cycle repeats itself, producing a pattern referred to as "group beating."<br />

(3) The longest cycles (those of the dropped beats) are less than twice the length of the shortest cycles<br />

(those of the impulses following the dropped beats).<br />

(4) The block is almost always within the AV node.<br />

b. Mobitz Type II<br />

Courtesy of Dr. Michael McCrea<br />

(1) P waves are normal.<br />

(2) QRS complexes are often (but not always) wide because of the common occurrence of a coexisting<br />

bundle-branch block.<br />

(3) PR intervals (when they occur) are always the same duration.<br />

(4) There are dropped beats.<br />

(5) The block is below the level of the AV node, generally in the His-Purkinje system.<br />

20


CARDIOVASCULAR DISORDERS<br />

4. Third-degree AV block: no atrial impulses are conducted; the atria and ventricles beat independently of<br />

one another.<br />

Courtesy of Dr. Michael McCrea<br />

a. P waves appear normal.<br />

b. May occur at the level of the AV node, the bundle of His, or the bundle branches<br />

c. QRS complexes may be narrow or wide, depending on the location of the block: if the block is located<br />

above the His bundle, the QRS complexes will be narrow and reflect a junctional escape rhythm, whereas<br />

if the block is located at or below the bundle of His, the QRS complexes will be wide and reflect a<br />

ventricular escape rhythm.<br />

d. There is no relationship between P waves and QRS complexes.<br />

(1) There is an independent and regular atrial rate (constant PP interval) and a slower independent and<br />

constant ventricular rate (constant RR interval).<br />

(2) The P waves are not related (not conducted) to the QRS complexes but rather "march through them" as<br />

if they were not there.<br />

(3) The PR intervals are variable.<br />

R. Preexcitation syndromes<br />

1. Result from abnormal connections (accessory pathways) between the atria and ventricles. Impulses traveling<br />

down these pathways bypass all or part of the normal conduction system. This results in the ventricles being<br />

activated by atrial impulses sooner than would normally be anticipated (preexcitation) and is reflected<br />

by changes in the surface ECG. The ECG changes seen are determined by the exact pathway the impulse<br />

travels. Wolff-Parkinson-White syndrome and Lown-Ganong-Levine syndrome are the two major variants of<br />

preexcitation. Their characteristic ECG findings are listed below. Patients with preexcitation syndromes are<br />

prone to tachydysrhythmias (especially paroxysmal supraventricular tachycardia and atrial fibrillation) with very<br />

rapid ventricular rates (up to 300 beats per minute).<br />

2. Wolff-Parkinson-White: the accessory pathway (bundle of Kent) connects the atria directly to the ventricles,<br />

completely bypassing the AV node and the infranodal conduction system.<br />

Courtesy of Or. Michael McCrea<br />

a. Short PR interval (


CARDIOVASCULAR DISORDERS<br />

Ill. ETIOLOGIES AND MANAGEMENT OF DYSRHYTHMIAS<br />

A. Premature atrial contractions<br />

1. There are multiple causes (drugs or underlying disease), but they may also occur as a normal variant.<br />

2. Clinical significance<br />

a. Can precipitate supraventricular tachycardia, atrial fibrillation, and atrial flutter<br />

b. Most frequent cause of a pause on the ECG<br />

3. In general, no treatment is indicated. If, however, the premature atrial contractions are frequent or<br />

symptomatic, treatment should be directed toward correcting the underlying cause.<br />

B. Sinus tachycardia<br />

1. There are multiple causes; common ones include:<br />

a. Anxiety (diagnosis of exclusion)<br />

b. Stimulant or sympathomimetic drugs (eg, cocaine)<br />

c. Fever<br />

d. Hypovolemia<br />

e. Hyperthyroidism<br />

f. Pulmonary embolism<br />

g. Anemia<br />

h. Hypoxia<br />

1. Pain<br />

2. Management in most instances should be directed at finding and correcting the underlying cause.<br />

3. In the setting of cocaine or stimulant toxicity, administration of a benzodiazepine is first-line treatment.<br />

C. Sinus bradycardia<br />

1. Common causes<br />

a. Acute inferior wall Ml<br />

b. Vasovagal events (eg, vomiting)<br />

c. Drug effect (eg, ~-blockers, calcium channel blockers)<br />

d. Sick sinus syndrome<br />

e. Hypothermia<br />

f. Hypothyroidism<br />

g. A normal variant, especially in those individuals who exercise aerobically on a regular basis<br />

2. Management<br />

a. Indicated for patients who demonstrate signs of hypoperfusion due to the bradycardia: those with shock,<br />

hypotension, ischemic chest pain, decreased mentation, or acute CHF<br />

b. Intervention sequence<br />

(1) Atropine 0.5 mg every 5 minutes as needed until a response is noted or a total of 0.03-0.04 mg/kg<br />

has been administered (3 mg for most adults).<br />

(a) Should be used cautiously in patients with an acute Ml, because it may worsen ischemia or<br />

precipitate ventricular tachycardia or ventricular fibrillation<br />

(b) Should also be used with caution in patients with Mobitz Type 11 second-degree AV block and<br />

new third-degree AV block with wide complexes - t atrial rate - t AV block - 1 ventricular<br />

rate and t blood pressure<br />

(c) Atropine is ineffective in patients with heart transplants because of lack of vagal innervation to<br />

the transplanted heart. Proceed directly to transcutaneous pacing and/or catecholamine infusion.<br />

(d) Can be parasympathomimetic in doses


CARDIOVASCULAR DISORDERS<br />

(3) Dopamine 5-20 mcg/kg/min or epinephrine 2-10 mcg/min<br />

(a) Should be used when bradycardia is unresponsive to atropine and a transcutaneous pacer is not<br />

readily available<br />

(b) Most useful when associated hypotension is present<br />

(4) Transvenous pacing may be required if symptomatic bradycardia persists despite vasopressors and/or<br />

transcutaneous pacing.<br />

D. Supraventricular tachycardia (SVT)<br />

1. A generic term that refers to all tachydysrhythmias arising above the bifurcation of the bundle of His, including<br />

sinus tachycardia, atrial fibrillation, atrial flutter, multifocal atrial tachycardia, paroxysmal supraventricular<br />

tachycardia, and nonparoxysmal junctional tachycardia. It arises from reentry or an ectopic pacemaker in<br />

the atria. Most clinicians, however, use the term SVT to refer specifically to AV nodal reentry tachycardia<br />

(AVNRT) and other undetermined supraventricular rhythms. In the discussion that follows, SVT refers to AVNRT.<br />

Management is discussed separately for other specific forms of SVT (such as atrial fibrillation).<br />

2. Causes<br />

a. Preexcitation syndromes (Wolff-Parkinson-White and Lown-Ganong-Levine)<br />

b. Mitra! disease (prolapse, stenosis)<br />

c. Digitalis toxicity<br />

d. Drugs (eg, alcohol, tobacco, caffeine)<br />

e. Acute Ml and pericarditis<br />

f. Hyperthyroidism<br />

g. Rheumatic heart disease<br />

3. Management is determined primarily by the patient's hemodynamic stability and secondarily by the width of<br />

the QRS complex.<br />

a. Hemodynamically compromised patients (those with hypotension, ischemic chest pain, a decrease in<br />

mental status, or acute CHF) with a narrow complex SVT should be sedated (if possible) and treated with<br />

synchronized cardioversion. Start with 50 joules.<br />

b. Vagal maneuvers and pharmacologic therapy may be used in the hemodynamically stable patient with<br />

narrow complex SVT.<br />

(1) Vagal maneuvers (such as carotid sinus massage [should not be done if digitalis toxicity has not been<br />

excluded] or Valsalva maneuver) increase vagal tone and may be effective in either terminating the<br />

dysrhythmia or slowing the ventricular rate enough to uncover the actual underlying rhythm. These<br />

maneuvers should be attempted before starting pharmacologic therapy and may also be used to<br />

supplement it. The vagal maneuver of choice, ie, the most effective, is the Valsalva maneuver.<br />

(2) Adenosine, because of its safety profile, is the drug of choice for the hemodynamically stable patient<br />

with narrow complex SVT. It is an ultra-short-acting AV nodal blocker that is very effective in<br />

converting SVT. Its major advantages over verapamil are its short half-life (


CARDIOVASCULAR DISORDERS<br />

(4) ~-blockers such as esmolol, metoprolol, or propranolol are also effective in the treatment of narrow<br />

complex SVT. Esmolol has the advantage of being cardioselective as well as having a very short halflife.<br />

Propranolol is the drug of choice for SVT secondary to thyrotoxicosis, because it partially blocks<br />

the conversion ofT 4<br />

and T 3<br />

• Avoid these drugs in patients with COPD, asthma, or CHF; in those who<br />

have received IV calcium channel blockers; and in those with atrial fibrillation with Wolff-Parkinson­<br />

White, sick sinus syndrome, and advanced AV block.<br />

(5) Digoxin is vagotonic. Compared with the other agents listed above, its effects are mild and<br />

have a much slower onset (may take 2-4 hours or more to work). Digoxin should be avoided if<br />

cardioversion is being considered.<br />

(6) Magnesium sulfate, phenytoin, and lidocaine are the drugs of choice for ectopic SVT caused by<br />

digitalis toxicity. Management should also include correction of hyper/hypokalemia (if present) and<br />

discontinuation of digitalis. In the presence of hemodynamic instability (or potentially lethal digitalis<br />

intoxication), administration of Fab fragments should be considered.<br />

(7) Other antidysrhythmic agents (eg, procainamide, amiodarone, sotalol) may also be effective.<br />

(8) Patients who do not respond to drug therapy may be treated with synchronized cardioversion (as<br />

described above) or overdrive cardiac pacing.<br />

c. Drug dosages and administration<br />

(1) Adenosine 6 mg rapid IV push in a proximal vein followed by a 20-ml bolus of normal saline; if there<br />

is no response after 1-2 minutes, double the dose to 12 mg.<br />

(2) Verapamil 2.5-5 mg IV over 2-3 minutes; a second dose of 5-10 mg may be given in 15-30 minutes<br />

if necessary.<br />

(3) Diltiazem 0.25 mg/kg IV over 2 minutes, followed in 15 minutes by a second bolus of 0.35 mg/kg if<br />

the first bolus was tolerated but ineffective. Smaller dosages should be considered in elderly patients.<br />

(4) Esmolol 300-500 mcg/kg bolus over 1 minute followed by an infusion of 50 mcg/kg/min; the loading<br />

dose may need to be repeated and the infusion rate increased by 50 mcg/kg/min every 5 minutes as<br />

needed to a maximum of 200 mcg/kg/min.<br />

(5) Metoprolol 5 mg IV over 2 minutes; may be repeated twice every 5 minutes for total of three doses.<br />

(6) Propranolol 1 mg IV over 1 minute; this dose may be repeated every 5 minutes up to a total dosage<br />

of 0.1-0.5 mg/kg.<br />

(7) Digoxin 0.5 mg IV push initially, with repeated doses of 0.25 mg every 30-60 minutes as needed;<br />

total dosage should not exceed 0.02 mg/kg.<br />

(8) Magnesium sulfate 1-2 g slow IV push over 1-2 minutes followed by an infusion of 1-2 g/hr<br />

(9) Phenytoin 18 mg/kg IV bolus infusion; dissolve dosage in normal saline and administer at a rate of 50<br />

mg/min or less.<br />

(10) Lidocaine 1-1.5 mg/kg bolus infusion, repeat dosages 0.5-0.75 mg/kg every 5-10 minutes, to<br />

maximum bolus dose of 3 mg/kg. This can be followed by maintenance infusion of 1-4 mg/min.<br />

d. Patients with wide complex tachycardia should be presumed to have ventricular tachycardia.<br />

(1) If unstable _,. synchronized cardioversion<br />

(2) If stable -,. procainamide or amiodarone<br />

(a) Both convert SVT or ventricular tachycardia.<br />

(b) Both should be avoided in patients with tricyclic antidepressant overdose or in the setting of<br />

toxicity from other sodium-channel blockers.<br />

(c) Adenosine may initially slow either rhythm, but dysrhythmia may recur (short therapeutic effect<br />

of the drug).<br />

(d) If drug therapy fails _,. synchronized cardioversion<br />

24


CARDIOVASCULAR DISORDERS<br />

E. Atrial fibrillation (most common SVT)<br />

1. Identify the type of atrial fibrillation the patient has by determining the probable duration of the dysrhythmia.<br />

First-detected episode<br />

of atrial fibri I lation<br />

versus<br />

Recurrent at1·i a I<br />

fibri I lation*<br />

I<br />

I<br />

I<br />

I<br />

I<br />

Paroxysmal<br />

I<br />

I<br />

Persistent<br />

I<br />

I<br />

I<br />

Permanent<br />

1 year<br />

Terminated<br />

spontaneously<br />

Sustained or terminated<br />

therapeutically<br />

Cardioversion failed<br />

or not attempted<br />

*lschemic stroke risk for recurrent atrial fibrillation is 5% per year<br />

or 2-7 times the risk for patients without atrial fibrillation.<br />

2. Search for reversible causes and treat any underlying medical condition; then determine the risk of<br />

subsequent stroke. Conditions with high risk of cardiogenic thromboembolism:<br />

a. Cardiac surgery<br />

b. Acute Ml<br />

c. Hyperthyroidism<br />

d. Myocarditis<br />

e. Acute pulmonary disease<br />

3. Plan the treatment using the following criteria:<br />

a. Cardiovascular stability<br />

b. Duration of the dysrhythmia<br />

c. Underlying cause/condition<br />

d. Presence/absence of an accessory pathway<br />

4. There are fundamentally two ways to manage atrial fibrillation: restore and maintain sinus rhythm, or allow<br />

atrial fibrillation to continue and ensure that the ventricular rate is controlled.<br />

a. Unstable patients _,. immediate electrical synchronized cardioversion<br />

b. Stable patients with significant symptoms _,. pharmacologic therapy<br />

(1) Control the ventricular rate first (choose one)<br />

(a) Calcium channel blocker (eg, verapamil, diltiazem)<br />

(b) ~-blocker (eg, esmolol, atenolol, metoprolol)<br />

(c) Digoxin<br />

(d) Amiodarone<br />

(e) The presence of Wolff-Parkinson-White syndrome is a special circumstance requiring changes in<br />

treatment protocols (see pages 31-32).<br />

(2) Cardiovert the dysrhythmia based on the duration of atrial fibrillation.<br />

(a)


CARDIOVASCULAR DISORDERS<br />

(b) >48 hours duration (higher risk of systemic embolization)<br />

1. Avoid immediate cardioversion if possible.<br />

ii. If early cardioversion (within 24 hours) is anticipated, consider starting heparin and<br />

consulting cardiology for transesophageal echocardiography to exclude an atrial thrombus.<br />

iii. If delayed cardioversion is the best option, anticoagulation for 3 weeks is indicated before<br />

cardioversion.<br />

5. Discussion of selected treatment options<br />

a. Hemodynamically compromised patients should be sedated (if possible) and treated with synchronized<br />

cardioversion; start with 120 joules. In patients who have digitalis toxicity or are hypokalemic,<br />

cardioversion is potentially hazardous; use only as a last resort and start with lower joules. If the patient<br />

has hypertrophic cardiomyopathy (or has been in atrial fibrillation >48 hours), immediate systemic<br />

heparinization is also indicated.<br />

b. Hemodynamically stable patients with atrial fibrillation of acute (2 days duration should, when stable, receive anticoagulation before<br />

pharmacologic or electric cardioversion is attempted to decrease the risk of arterial embolization of an<br />

intra-atrial thrombus.<br />

F. Atrial flutter<br />

1. Causes are essentially the same as for atrial fibrillation, but it is most often associated with postcardiac surgery<br />

and peri-infarction periods.<br />

2. Usually a transitional rhythm between sinus rhythm and atrial fibrillation<br />

3. Management is determined by the patient's cardiovascular stability, duration of the dysrhythmia and whether or<br />

not the patient has an accessory pathway.<br />

a. Hemodynamically unstable patients should be sedated (if time permits) and treated with synchronized<br />

cardioversion; start with 50 joules.<br />

b. Hemodynamically stable patients should be managed with Vagal maneuvers and pharmacologic therapy.<br />

(1) Vagal maneuvers or adenosine may be of diagnostic value; by inducing a transient AV nodal blockade,<br />

they may reveal the characteristic flutter waves of this rhythm and confirm the diagnosis.<br />

(2) Once the diagnosis is confirmed, rate control should be accomplished with a calcium channel blocker<br />

(eg, diltiazem, verapamil) or a ~-blocker (eg, esmolol, metoprolol). Digoxin may also be used but is<br />

considered a second-line drug. Magnesium may be used in combination with one of the above agents<br />

as adjunctive therapy.<br />

(3) Once the ventricular response rate is controlled, pharmacologic conversion to sinus rhythm can be<br />

achieved with procainamide or quinidine.<br />

G. Multifocal atrial tachycardia<br />

1. Causes<br />

a. Decompensated COPD (most common)<br />

b. CHF<br />

c. Sepsis<br />

d. Theophylline toxicity<br />

e. Asthma<br />

26


CARDIOVASCULAR DISORDERS<br />

2. Management<br />

a. Primarily aimed at correcting the underlying disease process<br />

(1) Correct hypoxia with supplemental oxygen (and bronchodilator therapy) in a patient with COPD, and<br />

(2) Evaluate for theophylline toxicity.<br />

b. If the above measures are unsuccessful and the patient is symptomatic, other modalities that may be used:<br />

(1) Calcium channel blockers (eg, diltiazem, verapamil) are usually effective in slowing the ventricular<br />

rate and decreasing atrial ectopy, and may produce conversion to a sinus rhythm in some patients.<br />

(2) Magnesium sulfate decreases atrial ectopy.<br />

(3) ~-blockers are generally not recommended, because they may worsen the underlying pulmonary<br />

disease process.<br />

c. Digoxin and cardioversion are usually ineffective.<br />

H. Junctional premature contractions<br />

1. Primary causes<br />

a. Digitalis toxicity<br />

b. Coronary artery disease<br />

C. CHF<br />

d. Acute Mis (especially inferior wall Mis)<br />

2. Treat the underlying cause; if junctional premature contractions precipitate more lethal dysrhythmias, consider<br />

using IV procainamide.<br />

I. Premature ventricular contractions (PVCs)<br />

1. Causes<br />

a. Hypokalemia<br />

b. Hypomagnesemia<br />

c. Hypoxia<br />

d. Ml<br />

e. Drugs<br />

(1) Alcohol, tobacco, caffeine<br />

(2) Cocaine<br />

(3) Digitalis or quinidine toxicity (PVCs are most common dysrhythmia with digitalis toxicity)<br />

(4) Methylxanthines (commonly used by patients with asthma or COPD)<br />

f. Hyperthyroidism<br />

g. CHF<br />

h. Cardiomyopathy<br />

1. Mechanical: PVCs are not uncommon when a catheter is placed in the right ventricle.<br />

j. Myocardial contusion<br />

2. Management is dictated by the underlying cause and is not indicated in all cases.<br />

a. Patients who are asymptomatic and have PVCs of unknown cause should not be treated (particularly when<br />

the PVCs are an incidental finding); PVCs in this setting may represent a normal variant. Isolated PVCs<br />

occur in as many as 50% of young healthy patients and increase in frequency with age.<br />

b. When an underlying cause is identified (hypokalemia, hypomagnesemia, hypoxia, etc), therapy should be<br />

directed toward correcting the underlying problem (rather than suppressing PVCs). This is usually sufficient.<br />

c. Mechanical causes, if present, should also be corrected.<br />

(1) A central line catheter that is located in the right ventricle may induce PVCs; withdraw the catheter, or<br />

advance it out of the ventricle.<br />

(2) To reduce the likelihood of inducing PVCs during Swan-Ganz placement, inflate the balloon to cover<br />

the catheter tip while advancing into the right ventricle; then deflate the balloon to avoid floating into<br />

the ventricular outflow tract.<br />

(3) If these measures fail to ameliorate the PVCs and the patient is symptomatic, lidocaine may be<br />

administered.<br />

d. "Escape" PVCs (those associated with bradycardia) should be treated with atropine (not lidocaine), because<br />

administration of lidocaine under these circumstances may suppress the existing functioning rhythm.<br />

e. Management of PVCs occurring in association with an acute Ml or ischemia is more controversial. Optimal<br />

treatment of the underlying ischemia/infarction with oxygen, nitroglycerin, aspirin, and percutaneous<br />

27


CARDIOVASCULAR DISORDERS<br />

coronary intervention is clearly the first priority. If these measures fail, most authors currently recommend a<br />

conservative course of watchful waiting.<br />

f. In general, the identification of PVCs should prompt a search for and treatment of the underlying cause,<br />

rather than simply pharmacologic suppression of the PVCs. Empiric use of magnesium is sometimes<br />

recommended. If short bursts of frequent PVCs are occurring, consider empiric administration of a-blockers<br />

(to decrease adrenergic stimulation that often causes ventricular dysrhythmias).<br />

g. Pharmacologic agents<br />

(1) Lidocaine has historically been used as a first-line agent, though it is rarely used empirically now. If the<br />

decision is made to use lidocaine, administer a bolus of 1-1.5 mg/kg followed by a 2-4 mg/min drip.<br />

Repeat boluses of 0.5-0.75 mg/kg may be given every 5-10 minutes as needed to a maximum total<br />

dosage of 3 mg/kg.<br />

(2) Procainamide may be tried if lidocaine is ineffective or contraindicated. It is administered at an<br />

infusion rate of 20-50 mg/min until PVCs are suppressed, the QRS widens by 50% of its original<br />

width, hypotension develops, or a total dose of 17 mg/kg has been administered. This should be<br />

followed by a maintenance drip of 1-4 mg/min.<br />

(3) Magnesium sulfate is often effective in decreasing the frequency of PVCs. The dose is 1-2 g slow IV<br />

push over 1-2 minutes followed by an infusion of 1-2 g/hr.<br />

J. Ventricular tachycardia<br />

1. The causes are basically the same as those for PVCs. The most common causes are ischemic heart disease and<br />

acute Ml.<br />

2. Management depends on the status of the patient and QRS morphology.<br />

a. If there is no pulse for either monomorphic or polymorphic ventricular tachycardia, treat like ventricular<br />

fibrillation: begin CPR and defibrillate as soon as a defibrillator is available. Start with 200 joules.<br />

b. If the patient has a pulse but is hemodynamically unstable (hypotensive, decreased mental status,<br />

ischemic chest pain, acute CHF), synchronized cardioversion is indicated. Sedate first if time allows. Start<br />

with 100 joules, and increase energy if unsuccessful.<br />

c. If the patient is clinically stable (awake, pain free, and has good blood pressure), pharmacologic therapy<br />

is appropriate. Patients who do not convert with these agents should be sedated and cardioverted. If the<br />

patient is stable, the following are recommended:<br />

(1) Monomorphic ventricular tachycardia: procainamide, amiodarone, or sotalol<br />

(a) Procainamide 20-50 mg/min until rhythm is suppressed, QRS widens by more than 50%,<br />

hypotension develops, or maximal bolus dose of 17 mg/kg is reached.<br />

(b) Amiodarone 150 mg IV bolus over 10 minutes, followed by maintenance infusion of 1 mg/min<br />

for 6 hours.<br />

(c) Sotalol 100 mg (1.5 mg/kg) over 5 minutes; avoid if prolonged QT.<br />

(2) Polymorphic ventricular tachycardia<br />

(a) If the baseline QT interval is normal, all of the above agents are considered reasonable.<br />

(b) If the QT interval is prolonged, it should be assumed that the dysrhythmia is torsades de pointes<br />

(atypical ventricular tachycardia). Class 1 A and 1 C antidysrhythmic agents are contraindicated;<br />

they prolong ventricular repolarization, which can worsen torsades.<br />

1. The therapy of choice is magnesium sulfate 1-2 g IV over 1-2 minutes followed by an<br />

infusion of 1-2 g/hr; it is nondysrhythmogenic and rapidly effective.<br />

ii. Overdrive electrical pacing (transcutaneous or transvenous) is also effective by increasing<br />

the heart rate, thereby shortening ventricular repolarization.<br />

iii. Sustained torsades or unstable patients should be treated with direct cardioversion, ie, not<br />

synchronized; start with 200 joules, and begin a magnesium infusion after cardioversion.<br />

(3) Note: If a patient with polymorphic ventricular tachycardia or torsades is unstable but has a pulse,<br />

cardioversion is indicated. Because of concerns in delay of cardioversion due to trouble with<br />

synchronization, even in a patient with a pulse, use direct cardioversion, ie, unsynchronized and<br />

defibrillation doses starting at 200 joules. Once cardioversion is achieved, antidysrhythmic therapy<br />

with, amiodarone or procainamide should be given. If the patient had torsades, magnesium should<br />

be used instead, as noted above.<br />

d. For patients who have been cardioverted back to sinus rhythm or who are experiencing intermittent<br />

ventricular tachydysrhythmias, the use of B-blockers should also be considered. B-blockers decrease the<br />

adrenergic tone that contributes to ventricular tachydysrhythmias.<br />

28


CARDIOVASCULAR DISORDERS<br />

3. It is a common misconception that all patients with ventricular tachycardia will appear clinically unstable;<br />

stable patients with a wide complex tachycardia are frequently assumed to have SVT with aberrancy rather<br />

than ventricular tachycardia. This is an inaccurate assumption.<br />

a. Ventricular tachycardia cannot be differentiated from SVT with aberrancy based on clinical symptoms<br />

and vital signs.<br />

b. Unstable patients with either rhythm should be cardioverted; it is effective in both cases.<br />

c. An ECG should be obtained in all stable patients.<br />

(1) Examine for evidence that favors one dysrhythmia over the other and treat accordingly.<br />

(2) If unable to decide, always assume it is ventricular tachycardia and treat as above based on QRS<br />

morphology and cardiac function.<br />

d. Adenosine will convert some forms of ventricular tachycardia to a sinus rhythm; therefore, use of<br />

adenosine as a diagnostic measure to distinguish ventricular tachycardia from SVT with aberrancy is not<br />

recommended.<br />

K. Ventricular fibrillation (and pulseless ventricular tachycardia)<br />

1. Causes<br />

a. By far the most common cause is acute coronary ischemia or infarction.<br />

b. Other possible causes include electrolyte abnormalities (eg, severe hypokalemia or hypomagnesemia),<br />

overdoses of sympathomimetics, electrical shock, and intracranial events.<br />

2. Management<br />

a. Initial therapy is immediate defibrillation. Time is of the essence. Chest compressions should be<br />

performed while preparing for defibrillation. Begin shocks at maximum joules (360 joules if using a<br />

monophasic defibrillator, 200 joules if using a biphasic defibrillator). Escalating dosages of shocks<br />

and stacked shocks are no longer indicated. After the defibrillation, immediately begin compressions<br />

at 100-120 times per minute for 2 minutes without checking for pulse or checking monitor. For<br />

persistent ventricular fibrillation/pulseless ventricular tachycardia, continue to follow the pattern<br />

of defibrillation - compressions for 2 minutes - check pulse and monitor. A key point is to minimize<br />

interruptions in compressions.<br />

b. If defibrillation is unsuccessful, follow the "CABs" as summarized in the ACLS 2015 Guidelines.<br />

(1) Circulation - high quality chest compressions with minimal interruptions for pulse checks, advanced<br />

airway insertion, or other procedures<br />

(2) Airway -;,. ensure open airway, place an advanced airway if unable to adequately bag-valve-mask;<br />

30 chest compressions to 2 breaths for adults regardless of one or two rescuers. Performance of<br />

endotracheal intubation or bag-valve-mask ventilation should not interrupt chest compressions.<br />

(3) Breathing - if an advanced airway is placed, confirm endotracheal tube placement, secure the tube,<br />

and confirm oxygenation and ventilation using monitored end-tidal CO 2<br />

capnography.<br />

c. Although no reduction in mortality has been shown with use of any IV medications in cardiac arrest,<br />

epinephrine is still recommended every 3-5 minutes as needed in doses of 1 mg for patients who remain<br />

pulseless. At this point, an active search for one of the reversible causes listed above should be sought<br />

and, if found, treated. Vasopressin is no longer recommended to replace either the first or second dose of<br />

epinephrine.<br />

d. If ventricular fibrillation or pulseless ventricular tachycardia persists despite defibrillation, epinephrine,<br />

chest compressions, and treatment of any obvious reversible cause (eg, hyperkalemia), an antidysrhythmic<br />

agent can be administered.<br />

e. No antidysrhythmics have been demonstrated to improve survival. Lidocaine was removed from the<br />

2010 ACLS pulseless arrest algorithm only to be relisted in 2015 as a second-line agent for refractory<br />

ventricular fibrillation/pulseless ventricular tachycardia. Lidocaine may be given as first line if<br />

amiodarone is not available. Exception: If the patient is being treated for torsades de pointes, magnesium<br />

is the drug of choice.<br />

f. Dosages<br />

(1) Amiodarone 300 mg IV bolus; may repeat once at 150 mg IV push followed by a 1 mg/min infusion<br />

for 6 hours, then 0.5 mg/min for 18 hours as needed<br />

(2) Lidocaine 1-1.5 mg/kg IV bolus<br />

(3) Magnesium sulfate 1-2 g IV push for torsades de pointes, known or suspected hypomagnesemia, or<br />

severe refractory ventricular fibrillation<br />

29


CARDIOVASCULAR DISORDERS<br />

L. Pulseless electrical activity<br />

1. Causes<br />

Table 3: The Five H's and the Five T's<br />

!:!.ypovolemia (most common)<br />

!:!.ypoxemia<br />

!:!.ydrogen ion (acidosis)<br />

!:!.yper- and hypokalemia<br />

!:!.ypothermia<br />

Iablets (drug overdose)<br />

Iamponade (cardiac)<br />

Iension pneumothorax<br />

Ihrombosis, coronary (acute coronary syndrome)<br />

Ihrombosis, pulmonary embolism<br />

2. Management<br />

a. Begin resuscitation: perform CPR, intubate, start an IV line.<br />

b. Search for and treat the underlying cause when possible, focusing on the "H's and T's" as above. The<br />

approach should be guided by the clinical findings, medical history, and circumstances that preceded the<br />

development of the rhythm. Appropriate measures include:<br />

(1) Ventilate with 100% oxygen to treat hypoxia.<br />

(2) Listen for breath sounds in both lung fields, and assess the ease of manual ventilation to exclude<br />

tension pneumothorax.<br />

(3) Administer a fluid bolus to correct hypovolemia.<br />

(4) Assess the ECG for evidence of an acute Ml and hyperkalemia.<br />

(5) Check core body temperature to exclude hypothermia.<br />

(6) Look for distended neck veins as signs of tension pneumothorax and cardiac tamponade.<br />

(7) Perform bedside ultrasound if available to exclude pericardia! effusion causing cardiac tamponade and<br />

pneumothorax.<br />

(8) Obtain brief history from EMS or the patient's family or friends regarding the possibility of drug overdose.<br />

c. Administer epinephrine 1 mg every 3-5 minutes. Atropine was removed from the 2010 ACLS pulseless<br />

electrical activity algorithm because of absence of proven reduction in mortality.<br />

M. Sick sinus syndrome<br />

1. Causes<br />

a. lschemia<br />

b. Cardiomyopathy<br />

c. Myocarditis<br />

d. Trauma<br />

e. Atherosclerosis<br />

f. Aging<br />

2. Management depends on the hemodynamic stability of the patient. If symptoms of hypoperfusion are present,<br />

rate stimulation with atropine, a pacemaker (for bradydysrhythmias), or rate control with a calcium channel<br />

blocker, ~-blocker, or digoxin (for tachydysrhythmias) can be cautiously attempted (excessive tachycardia or<br />

bradycardia may result). Hemodynamically stable patients should be referred to a cardiologist for demand<br />

pacemaker insertion and antidysrhythmic therapy.<br />

N. Atrioventricular blocks<br />

1. Causes of impaired conduction<br />

a. Acute Ml (inferior wall or anterior wall)<br />

b. Drugs: digitalis, lidocaine, phenytoin, procainamide, quinidine, ~-blockers, calcium channel blockers,<br />

magnesium infusion (iatrogenic)<br />

c. Inflammation: myocarditis, endocarditis<br />

d. Hyperkalemia<br />

e<br />

Hypermagnesemia<br />

f. Hypothermia<br />

g. Congenital conduction defects<br />

30


CARDIOVASCULAR DISORDERS<br />

2. Management<br />

a. First-degree AV block is often a normal variant. Other causes include increased vagal tone, digitalis<br />

toxicity, and myocarditis; specific therapy is not required.<br />

b. Second-degree AV block<br />

(1) Mobitz Type I (Wenckebach)<br />

(a) Usually transient<br />

(b) Often seen in the setting of an acute inferior wall Ml but can also be caused by digitalis toxicity,<br />

increased vagal tone, and myocarditis<br />

(c) Unless the patient is symptomatic, specific therapy is generally not required.<br />

(d) Symptomatic patients should be treated with atropine; if this fails, transcutaneous pacing is<br />

indicated.<br />

(2) Mobitz Type II<br />

(a) A more severe form of block; implies an organic lesion in the infranodal conduction system and<br />

is often permanent<br />

(b) Usually occurs in association with an acute anteroseptal Ml and can abruptly progress to<br />

complete block<br />

(c) These patients require pacing; transcutaneous pacing may be used as a bridging device, while<br />

preparations are made for insertion of a transvenous pacemaker. Atropine should be avoided (it<br />

can accelerate the atrial rate and produce a relative increase in the AV block - t ventricular<br />

response rate and t blood pressure).<br />

c. Third-degree (complete) AV block<br />

(1) Nodal (narrow complex) third-degree AV block occurs in association with an acute inferior wall Ml<br />

or drug toxicity and is usually transient. It should be treated like second-degree Mobitz Type I AV<br />

block with atropine or transcutaneous pacing.<br />

(2) lnfranodal (wide complex) third-degree AV block often occurs in association with an acute anterior<br />

wall Ml and implies significant structural damage to the infranodal conduction system. It almost<br />

always requires the insertion of a transvenous pacemaker. Transcutaneous pacing or catecholamines<br />

(dopamine or epinephrine infusions) may be used as a bridging device while awaiting pacemaker<br />

insertion. Atropine should be used cautiously (if at all) in this setting and is probably best avoided.<br />

0. Wolff-Parkinson-White (WPW) syndrome<br />

1. Recognition<br />

a. PR interval 200 beats per minute in an adult is very suspicious of an accessory pathway<br />

syndrome.<br />

e. Patients with WPW frequently do not have all of the classic features described above 011 their surface ECG,<br />

particularly if they are in sinus rhythm at the time of evaluation and conduction is occurring through the AV<br />

node in the normal fashion (concealed tract).<br />

2. The primary significance of WPW syndrome is that it predisposes to development of tachydysrhythmias<br />

(particularly atrial fibrillation).<br />

a. Patients with WPW have two parallel conducting pathways with different refractory periods, the AV node<br />

and an accessory pathway called the bundle of Kent. This predisposes them to the development of circusmovement<br />

tachycardias, in which an impulse is conducted anterograde to the ventricles by one pathway<br />

and retrograde back up to the atria by the other.<br />

b. When conduction is occurring anterograde down the AV node and retrograde back up the accessory<br />

pathway, ie, orthodromic tachycardia, the QRS complex appears normal. However, when the impulse is<br />

being conducted anterograde through the accessory pathway and retrograde through the AV node, the QRS<br />

complex is wide and cal led antidromic tachycardia.<br />

c. In the presence of antidromic conduction, the normal restraining effect of the AV node on conduction is<br />

lost, and rapid ventricular response rates (>200 beats per minute) can occur. This is particularly dangerous<br />

in atrial fibrillation, in which ventricular response rates can exceed 300 beats per minute and degeneration<br />

into refractory ventricular fibrillation can occur.<br />

3. Management of supraventricular tachydysrhythmias in patients with WPW is determined by the patient's<br />

cardiovascular stability, specific presenting dysrhythmia, the width of the QRS complex, and (with atrial<br />

fibrillation) how long the patient has been in it.<br />

31


CARDIOVASCULAR DISORDERS<br />

a. WPW with a narrow complex SVT (orthodromic tachycardia; most common presentation) is treated in<br />

the same manner as other re-entrant SVTs.<br />

(1) Treat hemodynamically unstable patients with synchronized cardioversion. Start with 50 joules, and<br />

pretreat with a sedating agent.<br />

(2) In hemodynamically stable patients, vagal maneuvers and drugs that slow conduction through the<br />

AV node (adenosine, calcium channel blockers or ~-blockers, procainamide, or amiodarone) should<br />

be used. Adenosine is the most effective agent given its rapid onset and strong AV nodal-blocking<br />

effects. IV procainamide is also effective but is slower in onset.<br />

b. WPW with a wide complex SVT (antidromic tachycardia)<br />

(1) If the patient is hemodynamically compromised, treat with cardioversion.<br />

(2) In stable patients, IV procainamide or amiodarone can be used effectively. Because the rhythm will<br />

look just like ventricular tachycardia (regular wide complex tachycardia), lidocaine may be chosen as<br />

first-line treatment, and it is unlikely to have any effect on the rhythm or hemodynamic status.<br />

(3) AV nodal-blocking agents (especially calcium channel blockers and ~-blockers, but also digitalis and<br />

adenosine) are contraindicated and should be avoided.<br />

c. WPW with atrial fibrillation or flutter (regardless of QRS duration)<br />

(1) Atrial fibrillation with WPW is characterized by an extremely rapid (rates typically 150-300 beats<br />

per minute), irregularly irregular tachycardia with QRS complexes that randomly vary in width<br />

and amplitudes, distinguishing this rhythm from ventricular tachycardia and from atrial fibrillation<br />

with bundle branch block. Atrial flutter with WPW is characterized by extremely rapid ventricular<br />

rates (often >250 beats per minute) with wide or narrow QRS complexes. The rapidity of the rate<br />

distinguishes this rhythm from typical ventricular tachycardia and from typical atrial flutter.<br />

(2) In hemodynamically compromised patients, cardioversion is the treatment of choice.<br />

(3) In hemodynamically stable patients, IV procainamide is the drug of choice, because it suppresses<br />

conduction through the accessory pathway.<br />

(4) All AV nodal-blocking agents should be avoided, because they can accelerate conduction through<br />

the accessory pathway and increase heart rate. These agents include adenosine, calcium channel<br />

blockers, ~-blockers, digoxin, and amiodarone. Note that amiodarone has both ~-blocking and<br />

calcium channel-blocking effects, and therefore must be avoided.<br />

IV. PACEMAKERS<br />

A. Emergency pacing techniques<br />

1. Emergency cardiac pacing can be accomplished via transcutaneous, transesophageal, or transvenous electrodes.<br />

2. Transcutaneous pacing is the technique of choice in the emergency care setting, particularly in the presence<br />

of an acute Ml in patients who have received (or may receive) thrombolytic therapy. It is the most easily and<br />

rapidly applied technique as well as the least invasive.<br />

B. Indications for temporary cardiac pacing<br />

1. Emergent<br />

a. Hemodynamically unstable bradycardia<br />

b. Bradycardia associated with malignant escape rhythm that does not respond to pharmacologic therapy<br />

c. Overdrive pacing of a refractory tachydysrhythmia<br />

d. The pacing of asystole is no longer recommended.<br />

2. Prophylactic (standby kit at bedside)<br />

a. Stable bradycardias (asymptomatic or those that responded to initial drug therapy)<br />

b. The presence of one of the following in the setting of an acute Ml<br />

(1) Symptomatic sinus node dysfunction<br />

(2) Mobitz Type II second-degree AV block<br />

(3) Complete heart block<br />

(4) Newly acquired or age-indeterminate left bundle-branch block, right bundle-branch block, alternating<br />

bundle-branch block, or bifascicular block<br />

C. Tips for temporary transvenous pacemaker placement<br />

1. The ideal location of the catheter tip is lodged in the trabeculae of the apex of the right ventricle.<br />

32


CARDIOVASCULAR DISORDERS<br />

2. The right internal jugular is the preferred access site, because it provides the most direct route to the right ventricle.<br />

3. Although more time consuming and not always possible, insertion using fluoroscopic or ECG guidance is<br />

preferred to blind placement (which is less reliable in achieving proper catheter positioning).<br />

4. When ECG guidance is used, the V lead is connected to the distal lead of the pacing catheter with a standard<br />

connector or alligator clip. Then, while monitoring the V lead, the pacing catheter is advanced and the size of<br />

the P wave and QRS complex is observed to determine location of the catheter tip; the size of these wave forms<br />

will increase when the corresponding heart chambers are entered. The development of ST elevation ("current of<br />

injury") indicates successful placement of the catheter tip.<br />

5. Pacing results in an abnormal QRS morphology. When the catheter tip is properly placed in the right ventricle,<br />

this ventricle is stimulated first, while stimulation of the left ventricle is delayed. This produces a left bundlebranch<br />

block pattern on the surface ECG.<br />

6. After pacemaker placement, chest radiographs (AP and lateral) should be obtained to confirm appropriate<br />

placement and exclude a procedural pneumothorax.<br />

D. Pacemaker failure<br />

1. Tools for evaluating pacemaker malfunction<br />

a. ECG to confirm that appropriate sensing and capture are occurring and to indirectly assess the position of<br />

the pacing catheter by evaluating QRS morphology<br />

b. Chest radiograph (PA and lateral) to evaluate lead position, exclude cardiac perforation, and look for lead<br />

fracture (not always visible)<br />

c. Pacemaker magnet: application of the magnet over the pacemaker turns off the sensing function (via the<br />

Reed switch) and temporarily converts the pacemaker from the demand (synchronous) mode to the fixedrate<br />

(asynchronous) mode_,. continuous asynchronous pacing at a specified rate (the magnet rate), which is<br />

usually around 70. This allows assessment of whether the pacing function is intact and whether the pacing<br />

stimulus can capture the myocardium. This maneuver is especially helpful in cases in which the baseline<br />

ECG does not reveal any pacemaker spikes. Moreover, it also al lows assessment of the battery status; a<br />

decrease in the magnet rate suggests battery depletion.<br />

2. Signs of pacemaker failure<br />

a. Slowing of the pacing rate is due to battery depletion. Replacement is urgent if the rate is 10% below the<br />

set point.<br />

b. A rapidly paced rhythm resembling ventricular tachycardia ("runaway pacemaker") is usually due to battery<br />

depletion or circuitry malfunction.<br />

3. Specific problems<br />

a. Failure to pace is detected clinically by the absence of pacemaker spikes in a patient whose intrinsic<br />

cardiac rhythm is slower than the programmed pacemaker rate. Causes include:<br />

(1) Wire fracture is accompanied by acute onset of symptoms that may be sustained or intermittent. It<br />

usually occurs at one of three sites: close to the pulse generator, where the lead enters the vein, or<br />

within the heart where the lead makes a sharp bend. The chest radiograph will demonstrate lead<br />

placement and may also reveal wire fracture.<br />

(2) Battery depletion is rare if the patient is being monitored often enough.<br />

(3) Oversensing (sensing electrical events not associated with atrial or ventricular depolarizations)<br />

suppresses impulse generation in pacemakers in the inhibit mode. It is more common in patients with<br />

unipolar leads and the most common cause of failure to pace.<br />

b. Failure to sense or capture is detected clinically by the presence of pacemaker spikes occurring at the<br />

wrong time (failure to sense) or the presence of pacemaker spikes without associated QRS complexes<br />

(failure to capture). Causes include:<br />

(1) Lead malposition<br />

(a) Lead displacement usually occurs in the first month after implantation.<br />

(b) Cardiac perforation usually occurs within 4 days of insertion.<br />

(2) Wire or insulation fracture<br />

(3) Battery depletion<br />

(4) Increased myocardial threshold due to<br />

(a) Fibrosis or inflammation at the electrode tip<br />

(b) Lead displacement<br />

(c) Metabolic and physiologic causes<br />

i. Metabolic acidosis<br />

ii. Hypoxia<br />

33


CARDIOVASCULAR DISORDERS<br />

iii. Hyperkalemia<br />

iv. Antidysrhythmic drugs (particularly ones that prolong the QRS)<br />

v. lschemia<br />

vi. Ml<br />

(5) Undersensing (ie, voltage of the patient's intrinsic QRS complex is too low for the pacemaker to sense)<br />

is more common with bipolar pacemakers. Sensitivity may be increased by converting to a unipolar<br />

pacemaker.<br />

(6) Oversensing (in pacemakers in the triggered mode): pacemaker spikes appear when none is expected<br />

because the pacemaker senses and reacts to events other than true cardiac events.<br />

c. Pacemaker-mediated tachycardia<br />

(1) Occurs only in patients with dual-chamber pacemakers that are programmed for synchronous<br />

atrioventricular pacing<br />

(2) Requires the presence of ventriculoatrial conduction; is a form of reentrant tachycardia<br />

(3) Can be precipitated by a premature atrial contraction or a premature ventricular contraction<br />

(4) Management<br />

(a) May be terminated by using a pacemaker magnet to briefly turn off the sensing function<br />

(b) Definitive therapy requires reprogramming of the atrial refractory period by a pacemaker specialist.<br />

d. Runaway pacemaker<br />

(1) May be triggered by battery depletion<br />

(2) Rarely occurs today because most newer pacemakers have built-in safety circuits.<br />

(3) Heart rate is frequently >200 beats per minute.<br />

(4) Management<br />

(a) Placement of a pacemaker magnet over the pacemaker may convert the pacemaker to the magnet<br />

rate and break the tachycardia.<br />

(b) If this is unsuccessful, and the patient is hemodynamically unstable, the pacemaker must be<br />

disconnected. To do this, exteriorize the pacer and cut the electrode wires. These wires may be<br />

reconnected to a temporary pacer if the patient's underlying rhythm is unstable.<br />

V. AUTOMATIC IMPLANTABLE CARDIOVERTER DEFIBRILLATORS<br />

{AICDs)<br />

A. Indications<br />

1. Placed in patients at high risk of fatal dysrhythmias (ventricular tachycardia, ventricular fibrillation) and sudden<br />

cardiac death<br />

2. Included in this high-risk group are patients who survived an episode of sudden cardiac death and those with a<br />

prior Ml or Brugada syndrome; in these patients, AICDs decrease the risk of sudden cardiac death remarkably<br />

from 30%-45% per year to


CARDIOVASCULAR DISORDERS<br />

(4) Potassium, magnesium, and calcium levels (hypomagnesemia is a common cause of dysrhythmias in<br />

AICD patients)<br />

(5) Telemetry interrogation of the AICD<br />

(6) Consultation with the patient's cardiologist<br />

c. Inactivation: placement of a magnet over the AICD generator will inactivate it and thereby prevent further<br />

shocks (see below)<br />

3. Performance of CPR and defibrillation with an AICD in place<br />

a. CPR is performed in the usual manner; while the provider may perceive an AICD shock, it is neither<br />

uncomfortable nor dangerous.<br />

b. External transthoracic defibrillation may also be performed in the standard manner. The paddles should not<br />

be placed close to the AICD generator; paddle placement is otherwise unchanged. After successful external<br />

cardioversion/defibri llation, the AICD should be tested to confirm that sensing and therapy parameters have<br />

not been altered.<br />

4. AICD inactivation: AICDs are generally inactivated by the presence of a magnet; however, some device-todevice<br />

variability exists.<br />

a. With second-generation AICDs, the placement of a donut-shaped magnet over the upper right quadrant<br />

of the pulse generator for 30 seconds will inactivate the antitachycardic pacing and shock therapy<br />

components of the AICD. Reapplication of the magnet for 30 seconds reactivates it.<br />

b. With the newer third-generation AICDs, placement of the magnet over the pulse generator inactivates<br />

antitachycardic pacing therapy and shocks for as long as the magnet remains in place over the AICD.<br />

Removal of the magnet reactivates the device.<br />

5. Infection: Patients who present with erythema, induration, or drainage at the generator site require<br />

hospitalization for IV antibiotics. Early infections are usually caused by Staphylococcus spp.<br />

35


CARDIOVASCULAR DISORDERS<br />

ACUTE CORONARY SYNDROME<br />

I. DEFINITION<br />

A. A continuum or progression of coronary artery disease from myocardial ischemia to infarction to necrosis<br />

B. Stable angina - unstable angina - acute Ml<br />

II. CLINICAL PRESENTATIONS, RISK FACTORS, AND PREDICTIVE<br />

FACTORS<br />

A. Classic presentations of acute coronary syndrome<br />

1. Stable angina<br />

a. Transient, episodic chest discomfort that is predictable and reproducible, ie, familiar symptoms occur from<br />

a characteristic stimulus that improve with rest or sublingual nitroglycerin within a few minutes<br />

b. These patients are usually sent home or observed briefly in the emergency department.<br />

2. Unstableangina<br />

a. Angina that is new in onset, occurs at rest, or is similar but somewhat "different" than previous episodes,<br />

and is severely limiting or lasts longer than a few minutes. Other signs are an increased frequency of attacks<br />

or resistance to prescribed medications that previously relieved the symptoms (eg, nitroglycerin, aspirin).<br />

b. These patients are admitted for observation or coronary care.<br />

3. Acute Ml<br />

a. Classically, chest discomfort of> 15 minutes duration associated with dyspnea, diaphoresis, light-headedness,<br />

palpitations, nausea, and/or vomiting. Pain radiation to the inner aspect of one or both arms, shoulders, neck,<br />

or jaw is not uncommon and increases the probability that the pain/pressure is ischemic in origin.<br />

b. Classified as non-ST-segment elevation myocardial infarction (NSTEMI) or ST-segment elevation myocardial<br />

infarction (STEM!)<br />

c. These patients are admitted to a cardiac care unit after appropriate treatment, including reperfusion therapy<br />

if needed.<br />

B. Atypical presentations (more common in the elderly, diabetic patients, and women)<br />

1. Chest pain or discomfort (not substernal chest pain/pressure) with or without any of the classic associated<br />

symptoms; a history of angina is often absent.<br />

2. Epigastric discomfort/indigestion or nausea and vomiting (may be the only complaint in women or those with<br />

an inferior wall Ml)<br />

3. Shortness of breath<br />

4. Syncope or confusion<br />

5. Fatigue, dizziness, or generalized weakness<br />

6. Women with prodromal symptoms (unusual fatigue, sleep disturbances, shortness of breath) for a month or more<br />

7. No chest or abdominal pain or discomfort; associated symptoms may or may not be present; patient frequently<br />

has vague complaints ("silent" Ml).<br />

a. Approximately 12.5% of all Mis<br />

b. The fol lowing patients should be suspect:<br />

(1) The elderly<br />

(2) Diabetic patients<br />

(3) Those with spinal cord injuries or disease<br />

(4) Alcoholic patients<br />

(5) Hypertensive patients or those with hypotensive insults<br />

(6) Postoperative patients receiving analgesics and postcoronary artery bypass graft patients; these patients<br />

generally have a worse prognosis than those with a classic presentation.<br />

36


CARDIOVASCULAR DISORDERS<br />

C. Risk factors<br />

1. Major<br />

a. Cigarette smoking<br />

b. Hypertension<br />

c. Diabetes mellitus<br />

d. Hypercholesterolemia<br />

e. Family history of coronary artery disease before age 55 in a first-degree relative<br />

f. Previous history of coronary artery disease, peripheral vascular disease, hypercoagulability, or carotid<br />

arteriosclerosis<br />

2. Other<br />

a. Male sex<br />

b. Advanced age<br />

c. Methamphetamine use<br />

d. Cocaine use (especially within an hour of presentation or when <strong>combined</strong> with ingestion of ethanol--<br />

cocaethylene, a longer-acting and more toxic metabolite of cocaine)<br />

e. Obesity<br />

f. Postmenopausal state<br />

g. Inflammatory disorders (eg, lupus)<br />

h. HIV disease<br />

i. Chronic kidney disease<br />

D. Predictive factors of an evolving acute Ml (greater clinical significance than risk factors)<br />

1. Prior history of ischemic heart disease<br />

2. Chest pain/discomfort that is worse than usual angina<br />

3. Pain that is similar to that of a prior acute Ml, lasts longer than an hour, or radiates to the left shoulder/arm<br />

Ill. DIAGNOSIS<br />

A. History<br />

1. A thorough history is the most important tool for identifying patients with cardiac ischemia and is adequate by<br />

itself to initiate hospital admission.<br />

2. Patients with a suspicious history and normal initial ECG and cardiac enzymes should be admitted to the<br />

hospital or chest pain evaluation/observation unit with a diagnosis of "exclude Ml"; typically,


CARDIOVASCULAR DISORDERS<br />

with ECG evidence of left ventricular hypertrophy (most common) and left bundle-branch block;<br />

however, a new left bundle-branch block in this setting may indicate acute coronary syndrome. An<br />

isolated ST-segment elevation may also occur with a ventricular aneurysm.<br />

(2) Q waves without associated ST and T wave changes may be due to an old (not new) Ml in the<br />

presence of left bundle-branch block; however, Q waves in leads I, aVL, V 5<br />

, and V 6<br />

suggest acute Ml.<br />

(3) Isolated T-wave inversion (no Q waves or ST changes) indicates ischemia (not acute infarction).<br />

c. Reciprocal changes: ST segment depression (horizontal or downsloping) that occurs in leads opposite to<br />

those with ST-segment elevation. The presence of these changes increases the positive predictive value of<br />

the ECG diagnosis of STEMI to >90% and denotes a patient at higher risk of later complications.<br />

d. ECG localization of the site of infarction<br />

(1) Acute anterior wall Ml (left anterior descending): ST-segment elevation in leads I, aVL, and V 1<br />

-V 4<br />

(segment depression in leads II, Ill, and a VF) represents a greater risk of development of conduction<br />

abnormalities (AV blocks, bundle-branch blocks, fascicular, and infranodal blocks) and left<br />

ventricular dysfunction (CHF)<br />

Courtesy of Dr. Michael McCrea<br />

(2) Acute lateral wall Ml (left anterior descending or left circumflex artery): ST segment elevation in<br />

leads I, aVL, and V 5<br />

-V 6<br />

with reciprocal ST-segment depression in leads V 1<br />

, II, Ill, and a VF represents a<br />

risk of development of left ventricular dysfunction.<br />

Courtesy of Dr. Michael McCrea<br />

38


CARDIOVASCULAR DISORDERS<br />

(3) Acute inferior wall Ml (right coronary artery): ST-segment elevation in leads II, Ill, and a VF with<br />

reciprocal ST depression in I, aVL, and V 1<br />

-V 4<br />

represents a greater incidence of increased vagal tonemediated<br />

dysrhythmias such as sinus bradycardia and varying degrees of AV block.<br />

Courtesy of Dr. Michael McCrea<br />

(4) Right ventricular wall Ml (RCA): ST-segment elevation in leads V 3<br />

R and V 4<br />

R represents a greater<br />

risk of developing hypotension and cardiogenic shock. Most right ventricular wall Mis occur in<br />

association with an inferior wall Ml; ST elevation will be mild.<br />

Courtesy of Dr. Michael McCrea<br />

39


CARDIOVASCULAR DISORDERS<br />

(5) Acute posterior wall Ml (circumflex artery or posterior descending artery off right coronary artery):<br />

Because none of the "routine" ECG leads face the posterior surface of the heart, the presence is<br />

usually inferred from the finding of reciprocal changes in the anterior leads, particularly V 1<br />

and V 2<br />

;<br />

ST-segment depression in association with abnormally tall R waves in these leads is the main ECG<br />

feature of this diagnosis. When a posterior wall Ml is suspected, obtain "additional" posterior leads<br />

(V 71<br />

V 8<br />

, and V 9<br />

, placed on the patient's left mid-back area below the tip of the scapula); the presence<br />

of ST-segment elevation in these leads confirms the diagnosis.<br />

Courtesy of Dr. Michael McCrea<br />

e. Additional lead ECGs<br />

(1) Allow more accurate characterization of acute inferior wall Mis by improving identification of<br />

associated right ventricular and posterior wall involvement<br />

(2) Incorporate additional leads to better visualize the right ventricle (V 4 R) and posterior wall of the left<br />

ventricle (V 7<br />

, V 8<br />

, and V 9<br />

), areas that are poorly defined on standard 12-lead ECGs.<br />

(3) These ECGs should be obtained in patients being admitted for suspected acute ischemia who have:<br />

(a) ST-segment depression or suspicious isoelectric ST segments in leads V 1<br />

-V 3<br />

(posterior infarct)<br />

(b) T-segment elevation in leads II, Ill, and a VF (inferior wall infarct) ----,, screen for right ventricular<br />

infarction as well (V 4 R).<br />

(c) Isolated ST-segment elevation in V 1<br />

or ST segment elevation in leads V 1<br />

and V 2<br />

(right ventricular<br />

infarct)<br />

(d) Borderline ST-segment elevation in leads II, Ill, and a VF or in leads V 5<br />

and V 6<br />

(inferior wall infarct)<br />

(e) Symptoms consistent with right ventricular ischemia (eg, epigastric pain, significant hypotension<br />

after administration of nitroglycerin)<br />

(4) Clinical significance<br />

(a) Patients with an acute posteroinferior wall Ml or right ventricular infarction usually have largersized<br />

Mis, lower ejection fractions, and higher morbidity/mortality rates.<br />

(b) Associated hypotension in patients with right ventricular infarction will likely respond to IV<br />

fluids; morphine, nitrates, and diuretics may further compound the situation.<br />

(5) Use of additional lead ECGs should be restricted to the patient population described above, ie, those<br />

patients with suspected posterior to right ventricular infarction. Obtaining additional lead ECGs in<br />

all emergency department patients with chest pain does not appear to have therapeutic or diagnostic<br />

benefits, and results in an unacceptable increase in false-positive diagnoses.<br />

2. The initial ECG is also useful in the following situations:<br />

a. Screening for nonischemic (but potentially serious) causes of chest pain, such as pericarditis and<br />

pulmonary embolism.<br />

b. Stratifying the risk of an adverse outcome in association with in-hospital disposition<br />

(1) Patients at high risk of complications and death should be admitted to a cardiac care unit or ICU.<br />

Included in this group are patients with any of the following ECG findings:<br />

40


CARDIOVASCULAR DISORDERS<br />

(a) lschemic ST or T-wave changes in contiguous leads<br />

(b) Pathologic Q waves (>0.04 seconds long or 25% of R-wave height)<br />

(c) Left ventricular hypertrophy<br />

(d) Left bundle-branch block (new or age uncertain)<br />

(e) Right bundle-branch block and ST changes<br />

(f)<br />

Paced rhythm<br />

(2) Patients at low risk of complications can safely be admitted to a step-down unit. Included in this<br />

group are patients whose initial ECG is either normal or have nonspecific ST-T wave changes.<br />

c. Establishing the criteria that determine which therapeutic interventions will be used (~-blockers,<br />

thrombolytic therapy, etc)<br />

3. Serial ECGs<br />

a. Indicated in patients with nondiagnostic ECGs in whom there is concern of possible ongoing ischemia<br />

b. Capture ischemic ECG changes, demonstrate ECG stability, and detect silent ischemia; ST segment trend<br />

monitoring may improve detection.<br />

c. Patients with normal or nonspecific initial ECGs and subsequent confirmed acute Mis within 72 hours<br />

still have a high rate of mortality and life-threatening complications.<br />

D. Serum markers of acute Ml<br />

1. Myoglobin starts increasing within 2-3 hours of infarction and peaks within 4-24 hours. It is more sensitive<br />

than the total creatine kinase (CK) and CK-MB but is not specific for cardiac muscle and, therefore, has a high<br />

false~positive rate. Therefore, it cannot reliably identify or exclude a Ml at any time junction; it is best used in<br />

conjunction with other markers.<br />

2. CK starts increasing within 4-6 hours of infarction, peaks within 12-24 hours, and returns to baseline in ~48<br />

hours. However, it is not specific for an acute Ml; in addition to heart muscle, it is also found in skeletal muscle<br />

and brain tissue, so increases can be due to disorders of any of these organ systems.<br />

3. CK-MB starts increasing within 4-6 hours of infarction, peaks within 20 hours, and is more specific than CK<br />

(but still not nearly 100% specific for Ml). The newer monoclonal assays that directly measure CK-MB mass<br />

(and subforms) have essentially replaced the old electrophoretic assays, because they are more sensitive and<br />

faster to run.<br />

4. Troponin (TN) T and I: appear in the serum within 3-4 hours of infarction, peak in 12-24 hours, and remain<br />

increased for 1-2 weeks. The newest-generation TN assays detect TN release within 1-2 hours of infarction.<br />

Troponin I is the most specific cardiac marker available (>90%). Troponin Tis not as specific as Troponin I<br />

(but far more than CK-MB) and is valuable in predicting cardiovascular complications in unstable angina and<br />

acute Ml.<br />

5. Appropriate use of serum markers<br />

a. The decision to admit or discharge a patient should be based primarily on the patient's history and clinical<br />

presentation, not on the presence or absence of increased cardiac markers.<br />

(1) Detection of these markers requires that sufficient myocardial eel I damage has occurred and that<br />

enough time has passed for these markers to be released into the serum.<br />

(2) Initial determination of these markers has a low sensitivity for detecting ischemia and cannot be used<br />

to reliably diagnose or exclude the presence of acute coronary syndrome.<br />

(3) No single determination of one serum biomarker of myonecrosis reliably identifies or excludes acute<br />

Ml in


CARDIOVASCULAR DISORDERS<br />

Table 4: Cardiac Serum Markers<br />

Serum Marker Increases (hr) Peaks (hr) Remains High (days)<br />

Myoglobin 2-3 4-24


CARDIOVASCULAR DISORDERS<br />

c. Administer in the emergency department only as a substitute for aspirin in patients with true aspirin allergy.<br />

d. STEMI: loading dose of one of these agents should be given as early as possible or at the time of<br />

percutaneous coronary intervention (PCI). Delaying administration until the coronary anatomy is defined<br />

at catheterization prevents possible delay in coronary artery bypass graft (CABG) if needed because of<br />

prolonged antiplatelet effects of these agents.<br />

e. Unstable angina/NSTEMI: administer a loading dose of one of these as early as possible if there is planned<br />

early intervention, or begin maintenance dose at time of admission if delayed or no intervention is<br />

planned.<br />

f. Prasugrel is contraindicated in patients with any prior history of transient ischemic attack or stroke<br />

because of increased risk of intracranial hemorrhage. Prasugrel should not be administered in the<br />

emergency department for unstable angina/NSTEMI; its use should be deferred to the time of PCI.<br />

3. Glycoprotein lib/Illa receptor antagonists<br />

a. Available agents include abciximab, eptifibatide, and tirofiban.<br />

b. Mechanism of action: in the presence of platelet activation, the glycoprotein lib/Illa receptor inhibitors<br />

block the final common pathway for platelet aggregation.<br />

c. For both STEMI and unstable angina/NSTEMI, lib/Illa inhibitors may be used as adjunctive therapy<br />

and started either before or at the time of catheterization for PCI but do not have to be started in the<br />

emergency department.<br />

d. lib/Illa inhibitors are not indicated for patients who are being medically managed (ie, in patients who are<br />

not undergoing catheterization).<br />

C. Anticoagulant (antithrombotic) therapy<br />

1. Unfractionated heparin (UFH)<br />

a. Indicated for all patients presenting with acute coronary syndrome: STEMI, NSTEMI, and high-risk<br />

unstable angina<br />

b. Acts as an indirect thrombin inhibitor by accelerating the action of antithrombin Ill (a thrombin<br />

inhibitor) and activated factors IX, X, and XI. Its antithrombotic activity complements the antiplatelet<br />

activity of aspirin to prevent progression of ischemia to acute Ml; when used together, heparin and<br />

aspirin are more effective than either used alone.<br />

c. Beneficial effects<br />

(1) In the acute Ml setting, it decreases the incidence of deep-vein thrombosis, reinfarction,<br />

nonhemorrhagic stroke, and the formation (as well as embolization) of left ventricular thrombus.<br />

(2) When administered to patients receiving thrombolytic agents, it prevents/decreases the incidence of<br />

reocclusion.<br />

(3) In patients with unstable angina, it may be useful in decreasing the rate of subsequent transmural<br />

infarction.<br />

(4) It may be preferred by cardiologists who are taking patients to the catheterization laboratory,<br />

because it can be turned off.<br />

d. Recommended as the anticoagulant of choice by the AHA for STEMI patients undergoing PCI. Lowmolecular-weight<br />

heparin (LMWH) is no longer recommended for STEMI in the 2013 AHA STEMI update.<br />

e. Dosage: bolus of 50-70 units/kg (maximum 5,000 units) followed by an initial infusion of 12 units/kg/hr<br />

(maximum 1,000 units/hr) adjusted to maintain a partial thromboplastin time 1.5-2 times control<br />

(50-70 seconds)<br />

2. LMWHs may be considered acceptable alternative to UFH in patients


CARDIOVASCULAR DISORDERS<br />

c. Bivalirudin may also be selected as initial anticoagulant for unstable angina/NSTEMI patients. However,<br />

in those patients, who then require PCI or CABC, bivalirudin should be stopped and the patient switched<br />

to UFH.<br />

4. Direct factor Xa inhibitor<br />

a. Selectively binds to antithrombin Ill, which potentiates factor Xa activation and thus inhibits thrombin<br />

formation<br />

b. Fondaparinux is preferred to LMWH in patients with renal dysfunction.<br />

c. Fondaparinux must be <strong>combined</strong> with UFH to avoid catheter-related thrombosis in patients undergoing PCI.<br />

D. Nitroglycerin<br />

1. Decreases ischemic pain (and consequently catecholamine release); no proven reduction in mortality but<br />

recommended nevertheless if no contraindications<br />

2. Pathophysiology<br />

a. Dilates collateral coronary vessels and increases collateral blood flow to ischemic myocardium<br />

b. Has antiplatelet effects and reduces infarct size and mortality<br />

c. Decreases myocardial oxygen demand by decreasing preload, left ventricular end-diastolic volume, and<br />

afterload (which makes it the drug of choice if coexisting left ventricular failure)<br />

d. May also reduce myocardial susceptibility to ventricular dysrhythmias during ischemia and reperfusion<br />

3. Should be administered to patients with ischemic chest pain whose systolic blood pressure is >90 mmHg. Start<br />

with sublingual nitroglycerin 0.4 mg every 3-5 minutes as needed for pain. Follow with an infusion of 10-20<br />

mcg/min and increase by 5-10 mcg/min every 5-10 minutes until pain is controlled or systolic blood pressure<br />

is decreased by 10%.<br />

4. Adverse reactions<br />

E. ~-Blockers<br />

a. Hypotension usually responds to a fluid bolus and leg elevation; it is particularly common when<br />

nitroglycerin is given to patients with inferior wall Mis with associated right ventricular infarction as well as<br />

to those with hypovolemia or bradycardia; caution should be exercised when administering nitroglycerin to<br />

these patients.<br />

b. Reflex tachycardia may be moderated by the concomitant use of ~-blockers.<br />

c. Administration of nitroglycerin to patients taking sildenafil citrate or other longer-acting medications<br />

for erectile dysfunction is contraindicated. In these patients, nitroglycerin can precipitate a sudden and<br />

profound decrease in blood pressure, which leads to decreased coronary perfusion and progression of<br />

myocardial ischemia to infarction with all of its potential consequences. Nitroglycerin should be avoided<br />

for 12-24 hours after using medications for erectile dysfunction.<br />

1. The routine use of early IV ~-blockers for patients presenting with acute coronary syndrome is associated<br />

with a mild increased risk of cardiogenic shock without a significant decrease in mortality. Consequently,<br />

IV ~-blockers are indicated only in patients with acute coronary syndrome who also demonstrate<br />

tachydysrhythmias (eg, rapid atrial fibrillation) or who have intractable hypertension despite nitrates.<br />

2. ~-blockers should be started orally within the first 24 hours of admission in patients who have none of the<br />

following:<br />

a. Signs of heart failure<br />

b. Evidence of low-output state<br />

c. Risk factors for cardiogenic shock<br />

(1) Age >70 years old<br />

(2) Systolic blood pressure 110 beats per minute or heart rate 240 milliseconds<br />

(2) Second- or third-degree AV block<br />

(3) Active asthma or reactive airway disease<br />

(4) Cocaine toxicity<br />

(5) Acute right ventricular infarction<br />

3. Unless contraindications exist, administer the following:<br />

a. Metoprolol 50-100 mg orally or<br />

b. Atenolol 50 mg orally or<br />

44


CARDIOVASCULAR DISORDERS<br />

c. Labetalol 10-20 mg IV if blood pressure control is needed; may repeat dose or double the dose as needed<br />

in 20-30 minutes or<br />

d. Esmolol 500 mcg/kg IV (over 1 minute) followed by an infusion of 50 mcg/kg/min titrated to a maximum<br />

dosage of 200 mcg/kg/min if rate control of a tachydysrhythmia is needed<br />

4. Pathophysiology<br />

F. Morphine<br />

a. Decrease myocardial oxygen demand by decreasing heart rate and myocardial contractility<br />

b. Increase coronary blood flow by increasing time of diastole and decreasing myocardial wall tension<br />

c. Markedly decrease rate of myocardial rupture (particularly in the elderly)<br />

d. Decrease incidence of ventricular fibrillation and cardiac rupture<br />

e. Decrease platelet aggregation<br />

1. Should be administered only if chest pain persists despite adequate management with antiplatelet,<br />

anticoagulant, and anti-ischemic agents<br />

2. Pathophysiology: decreases pain and anxiety--,. decreases circulating catecholamines--,. decreases tendency<br />

toward dysrhythmias and myocardial oxygen demand<br />

3. Administer 2-4 mg every 5-10 minutes as needed for pain.<br />

4. Adverse effects<br />

a. Hypotension and bradycardia: respond to fluid bolus and atropine<br />

b. Do not administer to patients with a heart rate


CARDIOVASCULAR DISORDERS<br />

d. Agent availability and administration protocols vary from hospital to hospital.<br />

e. American Heart Association/ American College of Cardiology criteria for thrombolysis<br />

(1) Class I (treatment benefit has been established): ST elevation >0.1 mV in two or more contiguous<br />

leads with ischemic symptoms of 110 mm Hg)<br />

(2) Traumatic or prolonged (>10 minutes) CPR<br />

(3) Noncompressible vascular punctures<br />

(4) Major surgery or internal bleeding within 3-4 weeks<br />

(5) Any other CNS disease (structural or functional, eg, dementia) not noted above<br />

(6) Pregnancy<br />

(7) Active peptic ulcer<br />

(8) Current use of anticoagulants (the higher the INR, the greater the risk of bleeding)<br />

(9) Prior exposure/allergic reaction to streptokinase or anistreplase (if using these agents)<br />

h. Complications<br />

(1) Systemic bleeding: incidence 2%-10%; usually occurs in the setting of invasive procedures.<br />

(2) Cerebral hemorrhage: incidence


CARDIOVASCULAR DISORDERS<br />

d. Creatinine >2.5 mg/dL<br />

e. Renal artery stenosis<br />

J. Glucose-insulin-potassium (GIK) therapy<br />

1. The results of a large, controlled, multinational study published in 2005 refuted smaller, earlier studies claiming<br />

a reduced mortality rate with GIK therapy. Therefore, GIK therapy is not recommended in the management of<br />

acute Ml.<br />

V. COMPLICATIONS<br />

A. Myocardial ischemia produces altered electrical depolarization and contractility, which are the major<br />

complications of an acute Ml (ie, dysrhythmias and left ventricular failure).<br />

B. Dysrhythmias<br />

1. The prehospital phase is the period associated with the highest incidence of lethal dysrhythmias; the<br />

occurrence of primary ventricular fibrillation (no evidence of CHF or hypotension) is greatest in the first hour of<br />

infarction.<br />

Courtesy of Or. Michael McCrea<br />

2. Treatment is indicated if the dysrhythmia<br />

a. Exacerbates myocardial ischemia or<br />

b. Could potentially deteriorate into cardiac arrest<br />

c. Treat specific dysrhythmias per current ACLS protocols.<br />

3. Lidocaine is no longer recommended as a prophylactic antidysrhythmic for acute Ml or ischemia; its use in this<br />

setting has been associated with an increased mortality rate.<br />

4. For premature ventricular contractions occurring in the presence of acute Ml, the initial treatment priority<br />

should be to optimally manage the underlying ischemia/infarction with oxygen, aspirin, nitroglycerin,<br />

and reperfusion therapy (ie, thrombolytics or PCI). Any electrolyte abnormalities should also be corrected.<br />

Previously, empiric treatment with ~-blockers was recommended to prevent premature ventricular contractions.<br />

However, new evidence refutes this practice, and there is possibly added harm by increasing risk of<br />

cardiogenic shock in patients with acute Ml.<br />

C. Left ventricular failure (CHF, pulmonary edema, cardiogenic shock)<br />

1. Acute Ml - impaired contractility of the left ventricle<br />

a. Impairment 2:25% of the left ventricle - (HF/pulmonary edema<br />

b. Impairment 2:40% of the left ventricle - cardiogenic shock<br />

2. Pathogenesis: pump failure= CHF; over time, this leads to increased pulmonary vascular congestion -<br />

pulmonary edema. Cardiogenic shock can occur with either acute, severe pump failure or acute, severe<br />

pulmonary edema (each of which further impairs left ventricular function).<br />

3. For management, see CHF/pulmonary edema, pages 50-52.<br />

D. Conduction disturbances<br />

1. AV blocks<br />

a. First-degree AV block and Mobitz Type I second-degree AV block<br />

(1) Generally due to increased vagal tone<br />

(2) Rarely progress to complete AV block<br />

(3) Usually associated with an inferior Ml (think Mobitz Type I and Inferior)<br />

(4) Generally responds to drug therapy (atropine)<br />

b. Mobitz Type II second-degree AV block<br />

(1) Generally due to destruction of infranodal conduction tissue<br />

(2) Sudden progression to complete AV block may occur<br />

(3) Usually associated with an anterior Ml<br />

(4) Prophylactic pacemaker therapy is indicated<br />

47


CARDIOVASCULAR DISORDERS<br />

2. Bundle-branch blocks<br />

a. In general, bundle-branch blocks (old or new) associated with an acute Ml identify patients who are more<br />

likely to develop CHF, AV block, and ventricular fibrillation.<br />

b. In patients with an acute anterior wall Ml and a new right bundle-branch block, there is a high risk of<br />

developing complete AV block and/or cardiogenic shock.<br />

E. Other less common complications of acute Ml<br />

1. Free left ventricular wall rupture<br />

2. Ventricular septa! rupture<br />

3. Papillary muscle dysfunction/rupture<br />

4. Acute mitral regurgitation<br />

5. Left ventricular aneurysm<br />

6. Thromboembolism<br />

7. Acute pericarditis<br />

8. Most of these complications are delayed in onset, occurring 1 or more days after infarction.<br />

48


CARDIOVASCULAR DISORDERS<br />

CONGESTIVE HEART FAILURE<br />

I. ETIOLOGY, PRECIPITATING FACTORS, AND SIGNS<br />

AND SYMPTOMS<br />

A. Left ventricular failure<br />

1. lschemic heart disease (most common cause)<br />

2. Idiopathic dilated cardiomyopathy<br />

3. Hypertension<br />

4. Valvular disease (aortic or mitral)<br />

5. High-output states (anemia, thyrotoxicosis, AV fistula, beriberi, Paget disease)<br />

6. Congenital heart disease<br />

7. Coarctation of the aorta<br />

B. Right ventricular failure<br />

1. Left ventricular failure (most common cause)<br />

2. Pulmonary arterial hypertension (and right ventricular failure= cor pulmonale)<br />

3. Valvular disease (tricuspid or pulmonic)<br />

4. Restrictive or infiltrative cardiomyopathies<br />

5. Myocarditis and some forms of congenital heart disease<br />

6. Right ventricular infarction<br />

7. Pulmonary embolism<br />

8. Chronic pulmonary disease<br />

C. Precipitating factors<br />

1. Myocardial ischemia or infarction<br />

2. Noncompliance with medications<br />

3. Tachydysrhythmias (eg, atrial fibrillation) and severe bradydysrhythmias<br />

4. Dietary indiscretion (sodium overload)<br />

5. Administration of drugs that impair cardiac function (fl-blockers or calcium channel blockers) or result in<br />

sodium retention (glucocorticoids, NSAIDs, nasal decongestants, vasodilators)<br />

6. Increased hemodynamic demand due to infection, trauma, physical overexertion, environmental stress, or<br />

pregnancy<br />

7. Hypoxia (due to pulmonary embolism or pneumonia)<br />

8. Cardiac disease progression (coronary insufficiency and CHF are the most common precipitating causes of<br />

acute cardiogenic pulmonary edema)<br />

9. Severe hypertension<br />

10. COPD (the leading cause of chronic cor pulmonale)<br />

11. Acute myocarditis or endocarditis<br />

12. Acute valvular dysfunction<br />

D. Signs and symptoms<br />

1 . Shortness of breath (most common symptom)<br />

2. Paroxysmal nocturnal dyspnea/orthopnea<br />

3. Nocturnal angina<br />

4. Moist rales and/or wheezing ("cardiac asthma")<br />

5. Cough<br />

6. Fatigue and/or weakness<br />

7. Pleural effusion (usually right-sided)<br />

8. Tachypnea and tachycardia<br />

9. S 3<br />

gallop (due to reduced left ventricular compliance)<br />

10. S 4<br />

gallop (not heard in patients with atrial fibrillation)<br />

11. Hepatojugular reflux--;. jugular venous distention (right ventricular failure)<br />

49


CARDIOVASCULAR DISORDERS<br />

12. Pulsus alternans (alternating strong and weak arterial pulse waveform)<br />

13. Ascites (right ventricular failure)<br />

14. Hepatic enlargement/tenderness (right ventricular failure)<br />

15. Pale, clammy skin or diaphoresis<br />

16. Neck vein distention (right ventricular failure)<br />

17. Anxiety<br />

18. Nocturia<br />

19. Dependent edema (right ventricular failure)<br />

II. DIAGNOSTIC EVALUATION<br />

A. Radiographs<br />

The cardiac silhouette is generally enlarged (although it may be normal size) in all three stages, associated pleural<br />

effusions (usually right-sided) are common, and chest radiograph findings may be delayed up to 12 hours relative to<br />

symptom onset.<br />

1. Stage 1: pulmonary vascular redistribution to upper lung fields ("cephalization")<br />

a. Chest radiograph - fullness or prominence of the pulmonary vessels in the apices<br />

b. Predominant symptom (if any are present) is dyspnea.<br />

2. Stage II: interstitial edema<br />

a. Chest radiograph - pulmonary vessels enlarged with blurred shadows, Kerley B lines (1-cm horizontal<br />

markings at the periphery of lung fields) are also present.<br />

b. Predominant symptom is dry cough.<br />

3. Stage Ill: alveolar edema (frank pulmonary edema)<br />

a. Chest radiograph - bilateral confluent perihilar infiltrates creating a "butterfly" pattern<br />

b. Predominant symptom is a wet cough with production of a frothy, pink sputum.<br />

B. Laboratory studies<br />

1. Brain-type natriuretic peptide (BNP) is used as a serum marker for acute CHF.<br />

2. Pathogenesis: cardiac hypertrophy/wall stress and volume overload - release of endogenous BNP by atrial and<br />

ventricular myocytes - smooth muscle relaxation and vasodilation, diuresis and natriuresis<br />

3. Acute CHF can lead to high serum levels of BNP, which are especially significant in the absence of pulmonary<br />

disease and advancing age (when moderate increases can occur).<br />

4. A serum BNP level 60 mmHg<br />

despite receiving 100% oxygen, displays a progressive increase in pC0 21<br />

or demonstrates increasing<br />

50


CARDIOVASCULAR DISORDERS<br />

acidosis. Do not wait for arterial blood gases if patient's clinical condition is deteriorating. Positive<br />

end-expiratory pressure (PEEP) can then be applied via the endotracheal tube.<br />

(5) Because positive-pressure ventilation (either NIPPV or endotracheal ventilation) increases<br />

intrathroacic pressure, there is a resultant decrease in preload, which can decrease cardiac output<br />

and blood pressure. Therefore, monitor blood pressure carefully.<br />

c. Initial medications<br />

(1) Nitroglycerin<br />

(a) At low doses, nitroglycerin is primarily a venodilator and rapidly acts to decrease preload. Its<br />

effects on arterial dilation are less profound and are usually associated with much higher doses.<br />

Nitroglycerin also acts to increase coronary blood flow by promoting dilation of large epicardial<br />

vessels and is, therefore, the vasodilator of choice in the presence of ischemia or an acute Ml.<br />

(b) Administer sublingual nitroglycerin 0.8-1.2 mg every 5-10 minutes (use higher doses if<br />

blood pressure is moderately to severely increased) as a loading dose, then start high-dose<br />

nitroglycerin IV drip.<br />

(c) Nitroglycerin IV at 10-20 mcg/min; titrate the drip upward in increments of 5-10 mcg/min<br />

every 5 minutes until the desired effect is attained or blood pressure decreases to 100 mm Hg (no signs/symptoms of shock)<br />

(1) Nitroprusside<br />

(a) A potent mixed venous and arteriolar dilator; reduces both preload and afterload, thereby<br />

decreasing pulmonary congestion and reducing cardiac output<br />

(b) Usually reserved for patients with a systolic blood pressure > 100 mm Hg who do not respond to<br />

adequate dosages of the standard preload reducers (nitroglycerin, furosemide) or for patients<br />

with low output who require controlled afterload reduction<br />

(c) Dosage: 0.1-5 mcg/kg/min IV; invasive hemodynamic monitoring is strongly recommended.<br />

b. Systolic blood pressure 70-100 mm Hg<br />

(1) No signs/symptoms of shock -> dobutamine<br />

(a) A direct-acting inotropic agent that is effective in increasing cardiac output and decreasing<br />

pulmonary capillary wedge pressure<br />

(b) Dosage: 2-20 mcg/kg/min IV (patients on ~-blockers may need the higher dosage)<br />

(c) Hemodynamic monitoring is advisable, because dobutamine may worsen hypotension.<br />

(d) If dobutamine successfully improves cardiac output and blood pressure, a nitroglycerin infusion<br />

can be carefully added for preload reduction, as the patient's blood pressure tolerates.<br />

(e) Dobutamine has some vasodilatory effect as well, so some patients will have a reduction in<br />

blood pressure. If this occurs, the addition of a vasopressor may be needed.<br />

(2) Signs/symptoms of shock present -> dopamine (or norepinephrine, below)<br />

(a) Indirect inotropic support as an effective pressor agent; the preferred agent when persistent<br />

oliguria or shock is present (systolic blood pressure


CARDIOVASCULAR DISORDERS<br />

(c) The vasopressor effect of dopamine increases afterload, but it can actually worsen pulmonary<br />

congestion. To counter this adverse effect, the concurrent use of vasodilatory medications<br />

(nitroglycerin or nitroprusside) is recommended, as tolerated by the patient's blood pressure.<br />

(d) Dopamine may also be used in combination with dobutamine; dopamine maintains blood<br />

pressure, while dobutamine prevents further increase in pulmonary congestion.<br />

c. Systolic blood pressure


CARDIOVASCULAR DISORDERS<br />

CARDIOMYOPATHIES AND SPECIFIC HEART<br />

MUSCLE DISEASES<br />

I. CARDIOMYOPATHIES<br />

A. Definition<br />

1. Diseases of the heart muscle of unknown etiology that produce both structural and functional damage to the<br />

myocardium<br />

2. Classified as dilated (most common), restrictive, or hypertrophic based on differences in pathophysiology and<br />

clinical presentation (however, there is some overlap).<br />

B. Idiopathic dilated (congestive) cardiomyopathy<br />

1. Characterized by dilatation of all four chambers (ventricles> atria), increased myocardial mass (hypertrophy),<br />

and systolic pump failure. There is a high association of idiopathic dilated cardiomyopathy with viral<br />

myocarditis; unexplained heart failure is commonly the only manifestation.<br />

2. Clinical presentation<br />

a. History<br />

(1) Signs and symptoms of left- and right-ventricular CHF (dyspnea on exertion and fatigue, which are the<br />

two most common complaints), paroxysmal nocturnal dyspnea, orthopnea, palpitations, dependent<br />

edema, ascites) and/or<br />

(2) Manifestations of systemic or peripheral embolization (acute neurologic deficit, flank pain and<br />

hematuria, pulseless cyanotic extremity)<br />

b. Physical findings<br />

(1) Bibasilar rales<br />

(2) Peripheral edema, jugular venous distention, and hepatomegaly (when associated with a pulsatile liver,<br />

tricuspid regurgitation is also present)<br />

(3) Abnormal heart sounds<br />

3. Diagnostic evaluation<br />

a. ECG<br />

(a) 5 3<br />

and 5 4<br />

gallop (most common auscultatory findings)<br />

(b) High-pitched systolic ejection murmur of mitral regurgitation (best heard at apex)<br />

(c) Holosystolic ejection murmur of tricuspid regurgitation (best heard at lower left sternal border)<br />

(d) Tachycardia (often with a narrow pulse pressure) when CHF is present<br />

(1) Left ventricular or biventricular hypertrophy<br />

(2) Left atrial or biatrial enlargement<br />

(3) Poor R-wave progression<br />

(4) A new bundle-branch block (may occur before any signs of chamber dilation or pump dysfunction)<br />

(5) AV block (usually first- or second-degree)<br />

(6) Atrial fibrillation (most common dysrhythmia)<br />

b. Chest radiograph<br />

(1) Cardiomegaly with enlargement of all four chambers<br />

(2) Pulmonary venous congestion<br />

c. Echocardiography<br />

(1) Enlargement of ventricles and atria<br />

(2) Increased systolic and diastolic volumes<br />

(3) Decreased ejection fraction (


CARDIOVASCULAR DISORDERS<br />

b. Anticoagulants should be administered to patients with:<br />

(1) lntracardiac thrombi<br />

(2) Evidence of pulmonary or systemic thromboembolism<br />

(3) Chronic atrial fibrillation<br />

C. Restrictive cardiomyopathy<br />

1. Characterized by diastolic restriction of ventricular filling; the end-diastolic ventricular volume is low, the enddiastolic<br />

ventricular pressure is high, and cardiac output is decreased-a hemodynamic picture that mimics<br />

that of constrictive pericarditis.<br />

2. Clinical presentation<br />

a. History<br />

(1) Similar to that of patients with dilated cardiomyopathy<br />

(2) Signs and symptoms of systemic and pulmonary venous congestion; however, evidence of right-sided<br />

CHF often predominates.<br />

(3) Exercise intolerance is a common complaint.<br />

b. Physical examination (similar to that of idiopathic dilated cardiomyopathy)<br />

(1) Bibasilar rales<br />

(2) Peripheral edema, jugular venous distention, hepatomegaly<br />

(3) Abnormal heart sounds<br />

(a)<br />

S 3<br />

and S 4<br />

gallop (common)<br />

(b) Systolic ejection murmur of mitral or tricuspid regurgitation<br />

(c) Tachycardia<br />

(4) The apex impulse is usually easily palpable and mitral regurgitation is more common (in contrast to<br />

constrictive pericarditis). Other signs that are not present in constrictive pericarditis are a gallop rhythm<br />

and a positive Kussmaul sign (jugular venous distention on inspiration).<br />

3. Diagnostic evaluation<br />

a. ECG<br />

(1) Chamber enlargement (atria> ventricles)<br />

(2) Nonspecific ST-T-wave changes<br />

(3) Low voltage<br />

(4) Dysrhythmias (especially atrial fibrillation) are common.<br />

b. Chest radiograph<br />

(1) May be normal initially (a small heart suggests constrictive pericarditis)<br />

(2) Cardiomegaly with pulmonary venous congestion is seen with disease progression.<br />

c. Echocardiography<br />

(1) Thickened walls<br />

(2) Normal or slightly enlarged ventricular cavity and moderate to markedly dilated atria<br />

(3) Normal or slightly decreased systolic function<br />

(4) Mitral and tricuspid regurgitation<br />

d. If differentiation from constrictive pericarditis is still uncertain after echocardiography, then CT, MRI, or<br />

<strong>combined</strong> Doppler and two-dimensional echocardiography may be required to make the diagnosis -<br />

thickened pericardium in constrictive pericarditis. Biopsy is the gold standard to exclude constrictive<br />

pericarditis if all else fails.<br />

4. Management<br />

a. Aimed at alleviating symptoms and usually includes a regimen of diuretics and digitalis<br />

b. In the absence of left ventricular enlargement, administration of digitalis may be without beneficial effect.<br />

c. Vasodilators reduce afterload but produce hypotension and have not been found to be beneficial.<br />

D. Hypertrophic cardiomyopathy<br />

1. Characterized by left ventricular hypertrophy without associated ventricular dilatation. The hypertrophy is<br />

generally asymmetric, involving the septum to a greater extent than the free wall; on histologic examination,<br />

the myocardial fibers have a marked and extensive disorganized whorled pattern (particularly in the septum).<br />

Other common features include ventricular cavities that are reduced in size, atrial dilatation, mitral valve<br />

thickening, impaired diastolic relaxation, and restricted left ventricular filling.<br />

54


CARDIOVASCULAR DISORDERS<br />

2. In >50% of cases, this disorder is inherited via an autosomal dominant transmission pattern with variable<br />

penetration. The remaining cases appear to be sporadic.<br />

3. Clinical presentation<br />

a. History<br />

(1) Dyspnea on exertion (most common initial complaint)<br />

(2) Atypical chest pain (poor response to nitroglycerin is common); chest pain that is relieved by assuming<br />

the recumbent position is pathognomonic of hypertrophic cardiomyopathy but rarely encountered.<br />

(3) Palpitations<br />

(a) Ventricular and atrial dysrhythmias are common.<br />

(b) Tachydysrhythmias (particularly atrial fibrillation) are poorly tolerated and may require emergent<br />

intervention.<br />

(4) Syncope and presyncope<br />

(a) Usually related to exertion<br />

(b) Due to dysrhythmias or a sudden decrease in cardiac output<br />

(5) Sudden death (usually due to dysrhythmias that occur with exercise, especially ventricular tachycardia)<br />

(6) Symptom severity correlates with the degree of hypertrophy which, in turn, correlates with the patient's<br />

age, ie, the older the patient, the more severe the symptoms.<br />

b. Physical examination<br />

(1) A prominent "a wave" may be noted on inspection of the neck veins, ie, a pulse wave that reflects the<br />

powerful systolic pressure of the hypertrophied left ventricle; this should not be confused with jugular<br />

venous distention, because jugular venous pressure is not usually increased.<br />

(2) Rapid biphasic carotid pulse (pulsus biferiens)<br />

(3) Abnormal heart sounds<br />

4. Diagnostic evaluation<br />

(a) 5 4<br />

gallop: prominent systolic ejection murmur along the lower left sternal border and at the apex<br />

with radiation to the axil la<br />

(b) This murmur is the result of left ventricular outflow obstruction and mitral regurgitation; it is<br />

increased with maneuvers or therapies that decrease left ventricular end-diastolic volume: the<br />

Valsalva maneuver, sudden standing, exercise, amyl nitrate inhalation, and administration of<br />

isoproterenol. Remember to perform one of these maneuvers when evaluating young patients<br />

presenting with exertional syncope or chest pain.<br />

a. ECG: almost always abnormal<br />

(1) Left ventricular hypertrophy and left atrial enlargement<br />

(2) Changes in the anterior, lateral, or inferior leads<br />

(a) "Septal Q waves" >0.3 mV (represent septal depolarization)<br />

(b) "Dagger-like" deep, narrow (


CARDIOVASCULAR DISORDERS<br />

hypertrophic cardiomyopathy patients have restricted left ventricular filling and, therefore, depend on atrial<br />

contraction to maintain efficient flow; when atrial fibrillation occurs, cardiac output drops suddenly.<br />

b. ~-blockers (eg, propranolol)<br />

(1) Improve most symptoms of this disease (dyspnea, chest pain, dizziness, syncope) and so are the<br />

mainstay of therapy<br />

(2) Decrease myocardial oxygen consumption by decreasing exertion-related outflow obstruction<br />

c. Calcium channel blockers (eg, verapamil, nifedipine)<br />

(1) May be useful in select patients who do not respond to ~-blockers<br />

(2) Decrease outflow obstruction, myocardial oxygen consumption, and incidence of angina; increase<br />

exercise capacity<br />

(3) Avoid in patients with conduction blocks or increased pulmonary venous pressures (CHF).<br />

d. Amiodarone<br />

(1) The treatment of choice for ventricular dysrhythmias; also indicated for patients who do not respond to<br />

~-blockers and calcium channel blockers.<br />

(2) Controls most atrial and ventricular dysrhythmias and appears to prevent sudden cardiac death (the<br />

most common cause of death in these patients)<br />

e. Diuretics are useful in the setting of pulmonary and venous congestion but must be used with caution to<br />

avoid hypotension.<br />

f. Agents that either increase myocardial contractility (digitalis, ~-adrenergic agents) and/or reduce ventricular<br />

volume (eg, nitrates, other vasodilators) should be avoided, because they can increase outflow obstruction.<br />

g. Surgical management (most common procedure is septa! myomectomy)<br />

(1) Reserved for very symptomatic patients with large systolic gradients (>50 mm Hg) who do not respond<br />

to drug therapy<br />

(2) Effectively relieves left ventricular outflow obstruction in 95% of patients but has a mortality rate of<br />

3%-8%<br />

h. Antibiotic prophylaxis is no longer indicated for dental or routine emergency department procedures.<br />

i. Avoiding competitive athletics is advised, because sudden death can follow vigorous exertion.<br />

II. SPECIFIC HEART MUSCLE DISEASES<br />

A. Definition<br />

1. Disorders of the heart muscle associated with a single known cause or systemic disease<br />

2. These diseases were previously referred to as secondary cardiomyopathies, but this terminology is no longer used.<br />

B. Categories<br />

1. Toxins<br />

a. Most commonly associated with dilated cardiomyopathy<br />

(1) Ethanol<br />

(2) Cobalt, cocaine<br />

(3) Lithium<br />

(4) Doxorubicin (adriamycin)<br />

(5) Daunorubicin<br />

(6) Emetine<br />

b. Others<br />

(1) Heavy metals<br />

(2) Amphetamines<br />

2. Nutritional deficiencies<br />

a. Thiamine (B 1<br />

) deficiency (beriberi)<br />

b. Vitamin C deficiency (scurvy)<br />

c. Vitamin B 6<br />

deficiency (pellagra)<br />

d. Selenium deficiency (Keshan disease)<br />

e. Kwashiorkor<br />

56


CARDIOVASCULAR DISORDERS<br />

3. Metabolic<br />

a. Most commonly associated with restrictive cardiomyopathy<br />

(1) Hemochromatosis<br />

(2) Glycogen storage disease Type II<br />

b. Others<br />

(1) Hypothyroidism (myxedema)<br />

(2) Hyperthyroidism (thyrotoxicosis)<br />

(3) Uremia<br />

(4) Pheochromocytoma<br />

(5) Hypophosphatemia<br />

4. Infiltrative (most commonly associated with restrictive cardiomyopathy)<br />

a. Amyloidosis (most common cause in the Western hemisphere)<br />

b. Sarcoidosis<br />

c. Endomyocardial fibrosis (most common cause worldwide)<br />

5. Collagen vascular diseases<br />

6. Neuromuscular<br />

a. Muscular dystrophies<br />

b. Friedreich ataxia<br />

c. Myasthenia gravis<br />

7. Myocarditis<br />

a. Viral<br />

b. Bacterial<br />

c. Parasitic (Chagas' disease due to Trypanosoma cruzi)<br />

d. HIV-associated<br />

8. Peripartum (associated with dilated cardiomyopathy)<br />

9. lschemia<br />

10. Radiation<br />

57


CARDIOVASCULAR DISORDERS<br />

PERICARDIAL DISORDERS<br />

I. PERICARDITIS<br />

A. Etiology<br />

1. Idiopathic (along with viral, most common causes)<br />

2. Infectious agents<br />

a. Viral (along with idiopathic, most common causes)<br />

b.<br />

(1) Coxsackie viruses A and B<br />

(2) Echovirus<br />

(3) Adenovirus<br />

(4) HIV<br />

(5) Epstein-Barr virus<br />

(6) Influenza<br />

(7) Hepatitis B<br />

Bacterial (uncommon cause)<br />

(1) Staphylococcus<br />

(2) Pneumococcus<br />

(3) Streptococcus<br />

(4) Meningococcus<br />

(5) Mycobacterium sp<br />

(6) Rickettsia sp<br />

(7) Borrelia burgdorferi<br />

(8) Mycoplasma<br />

c. Fungal<br />

(1) Histoplasmosis<br />

(2) Blastomycosis<br />

(3) Coccidiomycosis<br />

3. Malignancies<br />

a. Metastatic<br />

(1) Breast<br />

(2) Lung<br />

(3) Melanoma<br />

(4) Leukemia<br />

(5) Lymphoma<br />

b. Primary pericardia! tumors (mesotheliomas): rare cause<br />

4. Systemic illnesses<br />

a. Systemic lupus erythematosus<br />

b. Acute rheumatic fever<br />

C. Rheumatoid arthritis<br />

d. Scleroderma<br />

e. Polyarteritis nodosa<br />

f. Sarcoidosis<br />

g. Myxedema<br />

h. Amyloidosis<br />

5. Medications<br />

a. Anticoagulants<br />

b. Procainamide<br />

c. Hydralazine<br />

d. lsoniazid<br />

58


CARDIOVASCULAR DISORDERS<br />

6. Radiation<br />

7. Cardiac injury<br />

B. Diagnosis<br />

a. Acute Ml<br />

b. Dressler syndrome (late post-Ml pericarditis)<br />

1. Clinical presentation<br />

2. ECG<br />

a. History<br />

(1) Sharp precordial or retrosternal chest pain exacerbated by inspiration, swallowing, or movement<br />

of the upper torso (pain is increased in the supine position but relieved by sitting up and leaning<br />

forward); radiation to the trapezius muscle ridge (especially the left) is common and is a distinctive<br />

characteristic.<br />

(2) Dyspnea<br />

(3) Low-grade, intermittent fever<br />

b. Physical examination: pericardia! friction rub is pathognomonic.<br />

(1) May be positional and intermittent<br />

(2) Best appreciated when the patient is sitting and leaning forward or is in the hands-and-knees position<br />

(3) Scratchy in character; best heard with the diaphragm of the stethoscope positioned along the lower left<br />

sternal border or cardiac apex<br />

(4) Classically triphasic (with presystolic, systolic, and diastolic components) but may be biphasic or<br />

monophasic<br />

a. Normal sinus rhythm or sinus tachycardia; dysrhythmias are uncommon.<br />

b. Initial ST and PR segment changes<br />

(1) Diffuse, nonanatomic ST-segment elevation with upward concavity is seen acutely in multiple leads<br />

except aVR and V,. ST-segment elevation due to acute Ml may also be concave upward but can<br />

also be flat or convex in configuration and has a nondiffuse, anatomic distribution (eg, inferior or<br />

anterior wall pattern); ST elevation in Ill > II - think Ml<br />

(2) ST-segment depression in leads aVR and V, is common but nonspecific; however, ST-segment<br />

depression in any of the other leads strongly suggests Ml and not pericarditis.<br />

(3) PR-segment depression in multiple leads is common with viral pericarditis, most prominent in lead II,<br />

and often the earliest ECG manifestation.<br />

Courtesy of Dr. Daniel Schwerin<br />

c. ST and PR segment changes then normalize<br />

d. Diffuse T-wave inversion a late sign<br />

e. Finally after several weeks, all ECG segments and waves should have returned to baseline.<br />

3. Chest radiograph: usually normal but, in the presence of a large pericardia! effusion (>200 ml), may reveal an<br />

enlarged cardiac si I houette<br />

4. Echocardiography: the most sensitive and most specific procedure for detecting the presence and size<br />

of an associated pericardia! effusion; considered the diagnostic method of choice. Two-dimensional<br />

echocardiography can detect as little as 15 ml of fluid and has the advantage of being able to provide<br />

information about cardiac function. Transesophageal echocardiography and chest CT are superior to<br />

transthoracic echocardiography for demonstration of pericardia! thickening (seen in constrictive pericarditis).<br />

5. CT scan<br />

a. Can define the presence and extent of pericardia! effusion but, unlike echocardiography, cannot assess<br />

cardiac function and may miss hemopericardium because of similar densities of blood and myocardium<br />

b. Usually reserved for times when echocardiography is unavailable or yields equivocal results<br />

59


CARDIOVASCULAR DISORDERS<br />

6. Laboratory studies<br />

a. CBC: increased WBC count with a left shift is common.<br />

b. Erythrocyte sedimentation rate: usually increased<br />

c. BUN/creatinine: useful in excluding uremia<br />

d. Cardiac enzymes: often minimally increased<br />

e. Other specific tests (blood cultures, viral titers, thyroid function tests, etc): should be obtained as<br />

appropriate, depending on suspected underlying etiology<br />

C. Management<br />

1. Because they tend to follow a benign, self-limited course, patients with viral or idiopathic pericarditis may<br />

be treated as outpatients if reliable follow-up can be assured. All others, particularly those with severe<br />

intractable pain or an underlying precipitant that requires specific treatment (Ml, uremia, bacterial infection,<br />

etc) should be hospitalized.<br />

2. Pain relief<br />

a. Anti-inflammatory agents generally provide symptomatic relief within 24 hours of initiation. Use one of<br />

the following:<br />

(1) Aspirin 650 mg orally every 6 hours<br />

(2) Ibuprofen 600-800 mg orally every 6-8 hours<br />

(3) lndomethacin 25-75 mg orally every 6-8 hours<br />

(4) Colchicine decreases the duration of symptoms and recurrences when administered during the first<br />

episode. The recommended dosage is 2 mg/day for 1-2 days, followed by a maintenance dosage of 1<br />

mg/day (0.5 mg twice per day). The major adverse effect is diarrhea; if this develops, a lower dosage<br />

should be used. Additionally, reduced dosages should be used in patients >70 years old, and care<br />

should be exercised in the presence of renal insufficiency or if the patient is taking other drugs that<br />

are metabolized through the cytochrome P450 system.<br />

(5) A randomized trial in 2013 showed significant benefit of the combination of colchicine with<br />

standard NSAID therapy with reduction in duration of symptoms, decreased recurrence of<br />

pericarditis, and decreased need for hospitalization.<br />

(6) Prednisone (60 mg/day orally) should not be administered in the acute setting for patients with acute<br />

pericarditis. Steroids have been found to be an independent risk factor for recurrences.<br />

b. Opiate medications may be used when immediate pain relief is needed.<br />

3. Treat the underlying disorder.<br />

a. Antimicrobial agents for patients with underlying infections<br />

b. Dialysis for uremic patients<br />

c. Cessation of the causative agent for medication-induced pericarditis<br />

4. Pericardiocentesis for tamponade<br />

II. PERICARDIAL TAMPONADE<br />

A. Classic clinical presentation<br />

1. Beck triad (late findings occurring just before cardiac arrest)<br />

a. Hypotension<br />

b. Jugular venous distention<br />

c. Distant or "muffled" heart tones (quiet heart)<br />

2. Narrow pulse pressure<br />

3. Dyspnea<br />

4. Tachycardia (earliest sign)<br />

5. Pulsus paradoxus > 10 mmHg<br />

6. Kussmaul sign: a rise in central venous pressure (by observation of jugular venous pulsation) with<br />

spontaneous inspiration<br />

B. Diagnostic evaluation<br />

1. ECG<br />

a. Low QRS voltage: a nonspecific finding of pericardia( effusion<br />

60


CARDIOVASCULAR DISORDERS<br />

b. Total electrical alternans: a beat-to-beat alternating pattern primarily affecting the QRS complex that<br />

occurs from shifting pericardia! fluid and heart position-highly suggestive of the presence of large<br />

pericardia! effusions, but this finding is poorly sensitive<br />

2. Chest radiograph: in patients with chronic effusions, may reveal an enlarged "water bottle"-shaped cardiac<br />

silhouette without associated pulmonary redistribution; this is not the case in acute traumatic tamponade, in<br />

which the cardiac silhouette is typically normal.<br />

3. Echocardiography<br />

a. The gold standard for diagnosing pericardia! effusion; it can quickly and accurately confirm the presence<br />

of tamponade.<br />

b. If rapidly available and the patient's condition allows, it should be performed whenever pericardia!<br />

tamponade is suspected.<br />

c. Findings consistent with tamponade<br />

(1) A large pericardia! effusion, visualized as an echo-free space behind the left ventricle and in front of<br />

the right ventricle<br />

(2) Diastolic collapse of the right ventricle and the right atrium<br />

(3) Swinging motion of the heart in the effusion (producing electrical alternans)<br />

4. Differential diagnosis in patients with neck vein distention and hypotension<br />

a. Acute pericardia! tamponade<br />

b. Tension pneumothorax<br />

c. Massive pulmonary embolism<br />

d. Acute left ventricular Ml with cardiogenic shock<br />

e. Acute right ventricular Ml<br />

f. Superior vena cava syndrome<br />

g. Acute valvular dysfunction with regurgitation<br />

5. Clinical picture of a patient who needs immediate pericardiocentesis<br />

a. Air-hunger and drowsiness or confusion<br />

b. Thready pulse and pulsus paradoxus that is >50% of the pulse pressure<br />

c. t jugular venous distention and t systolic blood pressure<br />

C. Management<br />

1. Perform stabilizing/temporizing measures while arranging for pericardiocentesis.<br />

a. Establish two large-bore IV lines, provide supplemental oxygen (5-10 L/min), and place cardiac monitor.<br />

b. Provide aggressive volume resuscitation with crystalloid solution or blood.<br />

c. Administer dobutamine or dopamine as needed for inotropic support.<br />

d. Obtain immediate cardiology and cardiothoracic surgery consult, depending on availability. Urgent<br />

pericardiocentesis can usually be performed in the catheterization laboratory by cardiology, but if the<br />

cardiothoracic surgeon is unable to perform the procedure or it is unsuccessful, he or she will likely need<br />

to perform pericardiotomy in the operating room.<br />

2. Definitive therapy is pericardiocentesis. Although pericardiocentesis can be performed blindly or by using<br />

ECG guidance, ultrasound guidance is safer and is the technique of choice if it can be performed in a timely<br />

manner.<br />

61


CARDIOVASCULAR DISORDERS<br />

MYOCARDITIS<br />

I. ETIOLOGY<br />

A. Viruses: most common cause of myocarditis<br />

1. Coxsackie A and B (historically most common viral cause)<br />

2. Adenovirus (increase in incidence in the 1990s)<br />

3. Parvovirus 819 (increase in incidence in 2000s)<br />

4. Epstein-Barr<br />

5. Hepatitis Band C<br />

6. Human herpesvirus-6<br />

7. Echovirus<br />

8. Poliovirus<br />

9. HIV<br />

10. Cytomegalovirus<br />

11. Influenza and parainfluenza<br />

B. Bacteria<br />

1 . f3-hemolytic streptococci (rheumatic fever)<br />

2. Corynebacterium diphtheriae<br />

3. Neisseria meningitidis<br />

4. Borrelia burgdorferi (Lyme disease)<br />

5. Mycoplasma pneumoniae<br />

C. Parasites<br />

1. Chagas' disease<br />

2. Toxoplasmosis<br />

3. Trichinosis<br />

D. Systemic diseases<br />

1 . Kawasaki syndrome<br />

2. Systemic lupus erythematosus<br />

3. Sarcoidosis<br />

4. Giant-cell myocarditis<br />

E. Drug hypersensitivity<br />

1. Sulfonamides<br />

2. Penicillins<br />

3. Methyldopa<br />

F. Toxins<br />

1. Cocaine<br />

2. Inhalants (eg, toluene)<br />

II. CLINICAL PRESENTATION<br />

A. Determined by degree of cardiac involvement; ranges from nonspecific symptoms of fatigue and dyspnea<br />

to florid CHF, significant dysrhythmias, and sudden cardiac death.<br />

B. History of a preceding or concurrent viral illness is common.<br />

C. Often presents in association with acute pericarditis<br />

D. Signs and symptoms<br />

1. Fever and retrosternal or precordial chest pain (frequent complaints)<br />

2. Fatigue, palpitations, dizziness, and/or syncope<br />

62


CARDIOVASCULAR DISORDERS<br />

3. Signs and symptoms of CHF (dyspnea, rales, peripheral edema, jugular venous distention)<br />

4. Sinus tachycardia disproportionate to the degree of fever(> 101 °F [38.5°C]) present and probably secondary to<br />

associated heart failure<br />

5. Cardiac dysrhythmias or conduction disturbances<br />

6. Abnormal heart sounds<br />

a. Pericardia! friction rub (when associated pericarditis is present)<br />

b. Soft S 1<br />

c. S 3<br />

or S 4<br />

gallop<br />

d. Murmurs of mitral or tricuspid regurgitation<br />

E. Suspect giant-cell myocarditis in patients with ventricular tachycardia, heart block, and progressively<br />

worsening clinical course despite therapy; high risk of death and often require cardiac transplant<br />

Ill. DIAGNOSTIC EVALUATION<br />

A. ECG<br />

1. Nonspecific ST-T wave changes (may be localized or diffuse)<br />

2. Dysrhythmias ranging from sinus tachycardia (most common) to atrial or ventricular dysrhythmias<br />

3. Conduction disturbances (AV block, bundle-branch block)<br />

4. Low QRS voltage<br />

5. Pseudoinfarction patterns, nonspecific ST-T wave changes<br />

B. Chest radiograph<br />

1. Generally normal<br />

2. Cardiomegaly, pulmonary venous hypertension, and/or pulmonary edema may be present.<br />

C. Echocardiography<br />

1. Demonstrates dilated chambers with either diffuse hypokinesis or focal wall motion abnormalities<br />

2. Most useful to exclude other causes of heart failure<br />

D. Laboratory studies<br />

1. CBC reveals mild to moderate leukocytosis.<br />

2. Erythrocyte sedimentation rate is increased.<br />

3. Cardiac enzymes<br />

a. Characteristically rise and fall slowly over a period of days (unlike the rapid rise seen in acute Ml)<br />

b. Troponin I levels are more sensitive and specific than creatine kinase-MB levels.<br />

E. Endomyocardial biopsy<br />

1. Can provide definitive diagnosis but suffers from poor sensitivity and specificity<br />

2. Most indicated for evaluation of suspected giant-cell myocarditis or in cases of fulminant myocarditis<br />

IV. MANAGEMENT<br />

A. Primarily supportive<br />

1. Admit to ICU.<br />

2. Bed rest; avoid strenuous activity.<br />

B. Pharmacotherapy<br />

1. Administer antibiotics if an underlying bacterial cause is present.<br />

2. lmmunosuppressive agents (steroids, cyclosporin) and NSAIDs are contraindicated in early myocarditis.<br />

3. High-dose IV gamma globulin beneficial in preliminary studies in children with Kawasaki syndrome.<br />

4. Antiviral agents (eg, interferon) are currently under investigation.<br />

5. Manage CHF with the usual drug protocols; however:<br />

a. Digoxin should be used with caution, because the inflamed myocardium is very sensitive to it.<br />

b. ACE inhibitors (eg, captopril) are particularly beneficial in that they decrease cellular necrosis and<br />

inflammation.<br />

63


CARDIOVASCULAR DISORDERS<br />

6. Manage dysrhythmias with the standard antidysrhythmic agents and electrical therapies.<br />

a. Amiodarone recommended for ventricular dysrhythmias.<br />

b. Patients may require automatic implantable cardioverter defibrillator for persistent ventricular dysrhythmias.<br />

c. Temporary or permanent implanted pacemaker may be needed for persistent heart block.<br />

64


CARDIOVASCULAR DISORDERS<br />

VALVULAR HEART DISEASE<br />

I. MITRALVALVE PROLAPSE ("CLICK MURMUR SYNDROME")<br />

A. Epidemiology and etiology<br />

1. Most common valvular heart disease; present in 5 %-10% of the population, with young women most<br />

frequently affected (female-to-male ratio of 2:1 ).<br />

2. It can occur as an autosomal dominant congenital disorder or as part of a connective tissue disorder (Ehlers­<br />

Danlos syndrome, Marfan syndrome) in association with skeletal abnormalities (such as severe scoliosis,<br />

"straight" back, pectus excavatum), or it may occur sporadically.<br />

B. Clinical presentations<br />

1. Typical patients<br />

a. Young woman with palpitations<br />

b. Young female athlete passed out during a training or practice session. She is anxious and complains<br />

of chest pain and palpitations. A tachydysrhythmia and orthostatic hypotension are found on physical<br />

examination.<br />

c. Older man with a syncopal episode at home<br />

2. Physical examination findings<br />

a. The murmur is a high-pitched, late systolic regurgitant murmur heard best at the apex using the<br />

diaphragm of the stethoscope. One or more mid- or late-systolic clicks may also be heard.<br />

b. Maneuvers or therapies that decrease left ventricular volume (Valsalva maneuver, sudden standing,<br />

inhalation of amyl nitrate, isoproterenol infusion) - earlier and greater prolapse - movement of the<br />

click closer to S 1<br />

and an increase in the duration of the murmur<br />

c. Maneuvers or therapies that increase left ventricular volume (passive leg raising, maximal isometric<br />

handgrip, squatting, phenylephrine infusion) - delay of prolapse until late in systole - movement of the<br />

click and murmur away from S 1<br />

and a decrease in the duration of the murmur<br />

C. Diagnostic evaluation<br />

1. ECG: although most often normal, findings may include flattened or inverted T waves in the inferior leads<br />

(II, Ill, and a VF) and prolongation of the QT interval.<br />

2. Chest radiograph: unless significant mitral regurgitation is present, the lungs and cardiac silhouette are<br />

usually normal.<br />

3. Echocardiography: confirms the diagnosis; two-dimensional echocardiography is more sensitive than M-mode.<br />

D. Complications: although most patients are asymptomatic and without complications, patients may<br />

present with any of the following:<br />

1. Atypical (nonexertional, sharp) chest pain: thought to be due to localized ischemia from tension on the<br />

papillary muscles or to coronary artery spasm<br />

2. Palpitations, light-headedness, or syncope: due to dysrhythmias (premature atrial contractions, premature<br />

ventricular contractions, paroxysmal supraventricular tachycardia, ventricular tachycardia)<br />

3. Sudden death (rare): due to ventricular tachycardia or ventricular fibrillation<br />

4. Transient ischemic attacks or strokes: believed to result from embolization of leaflet thrombi<br />

5. CHF: due to severe mitral valve regurgitation associated with mitral valve prolapse<br />

E. Treatment: reserved for symptomatic patients<br />

1. !3-blockers for patients with chest pain and dysrhythmias<br />

2. Antiplatelet (eg, ASA) or anticoagulant therapy for patients with a history of systemic embolization (transient<br />

ischemic attack or stroke)<br />

II. MITRAL STENOSIS<br />

A. Etiology<br />

1. Rheumatic heart disease is responsible for 2:90% of cases of isolated mitral stenosis.<br />

2. Congenital malformations<br />

65


CARDIOVASCULAR DISORDERS<br />

3. Calcification of the mitral annulus and leaflets<br />

4. Left atrial myxoma<br />

B. Clinical presentation<br />

1. History<br />

a. Exertional dyspnea (most common symptom)<br />

b. Hemoptysis (second most common symptom)<br />

c. Orthopnea and paroxysmal nocturnal dyspnea<br />

d. Palpitations (premature atrial contractions, paroxysmal atrial fibrillation)<br />

e. Fatigue<br />

f. Systemic emboli<br />

2. Physical examination findings<br />

a. Mitral fades (malar rash)<br />

b. Palpable diastolic thrill at the apex<br />

c. loud S 1<br />

with early diastolic opening snap followed by a low-pitched, rumbling diastolic murmur heard<br />

best at the apex<br />

C. Diagnostic evaluation<br />

1. ECG<br />

a. left atrial enlargement (P mitrale)<br />

b. Atrial fibrillation<br />

c. Right ventricular hypertrophy (if marked pulmonary hypertension is present)<br />

2. Chest radiograph<br />

a. Straightening of the left heart border due to left atrial enlargement<br />

b. Calcification of the mitral annulus and leaflets<br />

c. Findings of pulmonary congestion<br />

d. Right ventricular lift (or hypertrophy) if pulmonary hypertension is present<br />

D. Complications<br />

1. Atrial fibrillation (most common)<br />

2. Embolic events, especially with atrial fibrillation<br />

3. Frequent respiratory infections<br />

4. Infective endocarditis (rare)<br />

5. Massive pulmonary hemorrhage from rupture of pulmonary bronchial venous connections<br />

E. Management<br />

1. Rate control with IV diltiazem or digoxin for atrial fibrillation with rapid ventricular response<br />

2. Blood transfusion and, possibly, surgery for massive hemoptysis<br />

3. Diuretics for pulmonary congestion<br />

4. Anticoagulation for patients with systemic embolization and atrial fibrillation<br />

Ill. MITRAL REGURGITATION<br />

A. Can be acute or chronic; these two disease entities differ significantly in terms of their causation,<br />

disease course, presentation, and management.<br />

B. Acute mitral regurgitation is the result of rupture of chordae tendineae, papillary muscle, or valve<br />

leaflet and is abrupt in onset.<br />

1. Etiology<br />

a. Acute Ml<br />

b. Infectious endocarditis<br />

c. Trauma<br />

2. Clinical presentation<br />

a. Symptoms are those of fulminant CHF/pulmonary edema (eg, dyspnea) and those of the disease process<br />

that precipitated the event (eg, chest pain in the acute Ml patient)<br />

66


CARDIOVASCULAR DISORDERS<br />

b. Physical examination findings<br />

(1) Tachycardia, tachypnea, and rales<br />

(2) Hypotension<br />

(3) Jugular venous distention with a prominent cannon "a" wave<br />

(4) Palpable thrill at apex<br />

(5) Soft s 1<br />

(6) Loud apical systolic murmur (radiating to the axilla) that can be holosystolic, late systolic, or<br />

crescendo-decrescendo<br />

(7) S 3<br />

and S 4<br />

(a common finding)<br />

3. Diagnostic evaluation<br />

a. ECG<br />

(1) Absence of left atrial enlargement and left ventricular hypertrophy<br />

(2) Sinus tachycardia<br />

b. Chest radiograph<br />

(1) Normal cardiac silhouette<br />

(2) Severe pulmonary edema<br />

4. Management<br />

a. Airway and hemodynamic support (may include intra-aortic balloon pump to augment cardiac output)<br />

b. Supplemental oxygen<br />

c. Afterload reduction<br />

d. Emergent consult with a cardiothoracic surgeon<br />

e. Treatment of underlying cause (eg, antibiotics for infectious endocarditis)<br />

C. Chronic mitral regurgitation evolves slowly and frequently coexists with mitral stenosis.<br />

1. Etiology<br />

a. Rheumatic heart disease is most common cause.<br />

b. Mitral valve prolapse<br />

c. Connective tissue disorders (eg, Marfan syndrome)<br />

2. Clinical presentation<br />

a. History<br />

(1) Exertional fatigue or dyspnea<br />

(2) Orthopnea and paroxysmal nocturnal dyspnea<br />

(3) Systemic emboli<br />

b. Physical examination findings<br />

(1) Left parasternal heave and thrill<br />

(2) High-pitched holosystolic murmur that radiates to the axilla<br />

(3) Soft S 1<br />

that is often obscured by the murmur<br />

(4) Wide-split S 2<br />

(5) S 3<br />

and S 4<br />

are common with severe regurgitation.<br />

3. Diagnostic evaluation<br />

a. ECG<br />

(1) Left atrial enlargement<br />

(2) Left ventricular hypertrophy with strain<br />

(3) Atrial fibrillation (75% of patients)<br />

b. Chest radiograph<br />

(1) Left atrial enlargement<br />

(2) Left ventricular enlargement<br />

(3) Pulmonary congestion<br />

4. Management<br />

a. Rate control of atrial fibrillation with diltiazem or digoxin<br />

b. Diuretics for symptoms of pulmonary congestion<br />

c. Anticoagulation if atrial fibrillation is present<br />

67


CARDIOVASCULAR DISORDERS<br />

IV. AORTIC STENOSIS<br />

A. Etiology<br />

1. Congenital bicuspid aortic valve is the most common cause in patients 65 years old.<br />

3. Rheumatic heart disease with associated mitral valve disease is also common.<br />

B. Clinical presentation<br />

1. Symptoms appear late in the disease process; patients remain relatively asymptomatic until the valve opening<br />

decreases to


CARDIOVASCULAR DISORDERS<br />

(2) Apprehension<br />

(3) Other signs of heart failure (eg, paroxysmal nocturnal dyspnea, orthopnea)<br />

(4) Chest pain<br />

b. Physical examination findings<br />

(1) Tachypnea, tachycardia, and inspiratory rales<br />

(2) Normal or low systolic and diastolic pressures<br />

(3) Normal pulse pressure (difference of 30-40 mmHg between systolic and diastolic blood pressures)<br />

(4) Signs of decreased cardiac output (cool, pale extremities, peripheral cyanosis, hypotension,<br />

diaphoresis, confusion)<br />

(5) Short diastolic murmur of aortic regurgitation best heard at the left sternal border with the<br />

diaphragm of the stethoscope<br />

(6) Diminished or absent S 1<br />

(7) Loud S 3<br />

(8) Midsystolic flow murmur<br />

3. Diagnostic evaluation<br />

a. ECG<br />

(1) Sinus tachycardia<br />

(2) Nonspecific ST-T wave changes<br />

(3) Left ventricular strain<br />

b. Chest radiograph<br />

(1) Normal cardiac silhouette<br />

(2) Signs of increased pulmonary venous pressure and pulmonary edema<br />

4. Management<br />

a. Determine and treat the underlying cause (eg, administer IV antibiotics to patients with bacterial<br />

endocarditis).<br />

b. Treat CHF with supplemental oxygen, diuretics, digitalis, and vasodilators (eg, nitroglycerin,<br />

nitroprusside).<br />

c. Obtain immediate cardiothoracic surgery consult for emergent valve replacement.<br />

C. Chronic aortic regurgitation<br />

1. Etiology<br />

a. Rheumatic heart disease<br />

b. Congenital (bicuspid) valve<br />

c. Prior infective endocarditis<br />

d. Diseases that dilate the aortic wall (eg, Marfan syndrome, ankylosing spondylitis)<br />

2. Clinical presentation<br />

a. History<br />

(1) Exertional fatigue and dyspnea<br />

(2) Orthopnea, paroxysmal nocturnal dyspnea<br />

(3) Palpitations<br />

(4) Angina<br />

(5) Pulmonary edema<br />

b. Physical examination findings<br />

(1) Wide pulse pressure<br />

(2) Displaced, hyperdynamic point of maximal impulse<br />

(3) Soft S 1<br />

(4) High-pitched, decrescendo diastolic blowing murmur best heard along the left sternal border (sine<br />

qua non of aortic regurgitation)<br />

(5) Rapid and forceful carotid upstroke with dramatic collapse (Corrigan or water-hammer pulse)<br />

(6) Head bobbing with each heartbeat (deMusset sign)<br />

(7) Prominent nail pulsations (Quincke pulse)<br />

(8) Singsong murmur over the femoral artery (Duroziez murmur)<br />

(9) A presystolic or mid-diastolic murmur (Austin-Flint murmur) may be present with severe aortic<br />

regurgitation.<br />

69


CARDIOVASCULAR DISORDERS<br />

3. Diagnostic evaluation<br />

a. ECG<br />

(1) Left ventricular hypertrophy<br />

(2) Conduction abnormalities such as left bundle-branch block<br />

b. Chest radiograph<br />

(1) Cardiomegaly<br />

(2) Aortic root dilatation<br />

(3) Pulmonary vascular congestion<br />

4. Management: treat CHF with supplemental oxygen, diuretics, vasodilators, and digitalis.<br />

VI. TRICUSPID STENOSIS<br />

A. Etiology<br />

1. Rheumatic heart disease<br />

2. Endocarditis secondary to IV drug abuse<br />

B. Clinical presentation<br />

1. History: patients complain of fatigue and symptoms due to increased systemic venous pressure (eg, edema).<br />

2. Physical examination<br />

a. Diastolic murmur best heard along the left sternal border and accentuated with inspiration<br />

b. Peripheral edema<br />

c. Hepatosplenomegaly<br />

d. Ascites<br />

e. Jugular venous distention and giant jugular cannon "a" waves<br />

3. Tricuspid stenosis rarely exists in isolation. Coexisting mitral and aortic valve disease is common and, when<br />

present, typically dominates and determines the clinical course.<br />

C. Diagnostic evaluation<br />

1. ECG<br />

a. Tall, pointed P waves if sinus rhythm is present<br />

b. Atrial fibrillation<br />

2. Chest radiograph: enlarged right atrium<br />

D. Management<br />

1. Rate control with IV diltiazem for atrial fibrillation with rapid ventricular response<br />

2. Anticoagulation for chronic atrial fibrillation<br />

3. Treatment of underlying cause (eg, IV antibiotics for infectious endocarditis)<br />

VII. TRICUSPID REGURGITATION<br />

A. Etiology<br />

1. Right ventricular dilatation secondary to pulmonary hypertension<br />

2. Rheumatic heart disease<br />

3. Infective endocarditis<br />

4. Trauma<br />

B. Clinical presentation<br />

1. History<br />

a. Fatigue and dyspnea on exertion<br />

b. Peripheral edema<br />

c. Throbbing in the neck and abdomen<br />

d. Anorexia<br />

70


CARDIOVASCULAR DISORDERS<br />

2. Physical examination<br />

a. Palpable left ventricular heave<br />

b. Prominent systolic pulsations of the jugular veins known as "v" waves or "CV" waves<br />

c. Holosystolic murmur best heard at the xiphoid area adjacent to the left sternal border<br />

C. Diagnostic evaluation<br />

1. ECG<br />

a. Right atrial enlargement<br />

b. Right ventricular enlargement<br />

c. Atrial fibri I lation (80% of patients)<br />

2. Chest radiograph<br />

a. Right atrial enlargement<br />

b. Right ventricular enlargement<br />

c. Normal pulmonary vasculature<br />

D. Management: patients in atrial fibrillation require rate control with IV diltiazem and long-term<br />

anticoagulation.<br />

VIII. PROSTHETIC VALVES<br />

A. Types and characteristics: there are two types of prosthetic valves (mechanical and bioprosthetic), each<br />

with its own particular characteristics.<br />

1. Mechanical (nontissue) valves<br />

a. Manmade materials<br />

b. Life span >20 years<br />

c. Typically make a loud metallic closure sound and softer opening click and, when located in the aortic<br />

position, an associated systolic ejection murmur is normally present.<br />

d. Require life-long systemic anticoagulation<br />

e. Cause greater hemolysis and are more thrombogenic than tissue prostheses<br />

f. Common designs include the tilting disc variety (eg, Bjork-Shiley, Medtronic-Hall), the caged ball variety<br />

(eg, Starr-Edwards), and the bi leaflet variety (eg, St. Jude).<br />

2. Bioprosthetic (tissue) valves<br />

a. Made with human, porcine, or bovine tissue cups<br />

b. Life span 8-10 years<br />

c. Make opening and closing sounds that are similar to (but louder than) those of native valves<br />

d. Except for the initial postoperative period and patients in atrial fibrillation, anticoagulation is optional; ASA<br />

is sufficient in most patients.<br />

e. Cause less hemolysis and are less thrombogenic than mechanical valves<br />

B. Complications and their clinical presentations<br />

1. Thromboembolic events<br />

a. Most serious complication of prosthetic valves; occur more frequently with mechanical valves than with<br />

bioprosthetic ones<br />

b. Patients with valve dysfunction secondary to thrombus formation typically present with acute onset of<br />

CHF, hypotension, and muting or loss of the prosthetic valve sounds. Signs and symptoms in patients with<br />

embolic events depend on the location of the emboli and may include paralysis, aphasia, abdominal pain,<br />

chest pain, or a cold extremity.<br />

2. Primary valve failure<br />

a. Significant cause of morbidity and mortality<br />

b. May result in regurgitant blood flow, acute valvular occlusion, embolization of a prosthetic fragment, or<br />

severe hemolysis<br />

c. The Bjork-Shiley 60° and 70° convexoconcave valves were taken off the market, because they were<br />

associated with a high incidence of strut fracture with resultant embolization of the disc. Patients with this<br />

complication present with sudden onset of CHF, hypotension, loss of the metallic valve sound, and a new<br />

regurgitant murmur. Although these valves are no longer manufactured, many patients still have them.<br />

71


CARDIOVASCULAR DISORDERS<br />

3. Paravalvular leak<br />

a. A portion of the prosthetic valve becomes unseated from the valve annulus.<br />

b. If immediately after surgery, it is usually due to suture disruption, whereas delayed leaks are generally due<br />

to endocarditis.<br />

c. More common with mechanical valves<br />

d. Patients typically present with sudden onset of pulmonary edema or severe hemolytic anemia. Physical<br />

examination reveals a regurgitant murmur.<br />

4. Endocarditis<br />

a. The causative organism varies with the length of time the valve has been present;


CARDIOVASCULAR DISORDERS<br />

IX. CONDITIONS NECESSITATING ANTIBIOTIC PROPHYLAXIS<br />

FOR INFECTIVE ENDOCARDITIS<br />

A. Patients at high risk of infection<br />

1. Prosthetic cardiac valves or prosthetic material used for cardiac valve replacement<br />

2. A history of infective bacterial endocarditis<br />

3. Congenital heart disease (CHD), unrepaired cyanotic CHD, underlying palliative shunts and conduits<br />

4. Completely repaired CHD with prosthetic material or device, whether placed by a surgery or by catheter<br />

intervention, during the first 3 postoperative months<br />

5. Repaired CHD with residual defects at the site or adjacent to the site of a prosthetic patch or prosthetic device<br />

6. Valve regurgitation caused by a shunt abnormal valve in a cardiac transplant patient<br />

B. Emergency department procedures that merit prophylaxis<br />

1. Antibiotic prophylaxis is recommended for emergency department patients at high risk (as above) undergoing<br />

dental procedures that manipulate the gingiva or periapical region of the tooth or that perforate the oral<br />

mucosa.<br />

2. Antibiotic prophylaxis is not recommended for the vast majority of emergency department procedures such as<br />

laceration repair, intubation, line placement, or Foley catheterization.<br />

3. No specific recommendations are made for or against prophylaxis for emergency department procedures<br />

that involve infected skin (eg, cutaneous abscess drainage) or musculoskeletal tissue, but it is reasonable to<br />

administer antibiotics to only high-risk patients.<br />

73


CARDIOVASCULAR DISORDERS<br />

ENDOCARDITIS<br />

I. DEFINITION<br />

A. A localized infection of the endocardium, the hallmark of which is vegetation<br />

B. Can involve the valve leaflets, the walls of the heart cavities, or the tissue surrounding prosthetic heart valves<br />

II. PATHOPHYSIOLOGY<br />

A. Injury to the endothelium results in formation of a platelet-fibrin complex that is subsequently colonized by<br />

microorganisms during periods of transient bacteremia.<br />

B. Although endocarditis does occur in patients with normal valves, patients with congenital/acquired valvular<br />

disease and prosthetic valves are most commonly affected.<br />

C. Risk factors<br />

1 . Prosthetic valve(s)<br />

2. Congenital valvular heart disease (eg, mitral valve prolapse)<br />

3. Acquired valvular heart disease (eg, rheumatic heart disease)<br />

4. IV drug abuse<br />

5. Calcific valve degeneration<br />

6. Indwelling venous catheters, vascular shunts<br />

7. Hemodialysis<br />

8. Peritoneal dialysis<br />

9. Cardiac surgery<br />

10. HIV infection<br />

11. History of endocarditis<br />

12. Extensive burn injury<br />

D. Causative organisms<br />

1. Although endocarditis is most commonly caused by bacteria, it may also be caused by fungi, rickettsiae, and<br />

viruses.<br />

2. The causative organism varies with the type of valve involved (native versus prosthetic) and in the presence of<br />

IV drug abuse or immunocompromise.<br />

3. Common bacterial pathogens<br />

a. Native valves<br />

(1) Non-viridans streptococci (most common)<br />

(2) Staphylococcus aureus<br />

(3) Streptococcus viridans<br />

(4) Enterococci<br />

b. Prosthetic valves<br />

(1) Coagulase-negative Staphylococcus (early, 60 days after surgery)<br />

(3) S viridans<br />

(4) Enterococci<br />

c. IV drug abusers, immunocompromised patients<br />

(1) S aureus<br />

(2) Streptococci<br />

(3) Gram-negative bacilli<br />

74


CARDIOVASCULAR DISORDERS<br />

E. Left- versus right-sided disease<br />

1. Left-sided endocarditis....,. systemic vascular involvement<br />

a. Most common in patients with acquired valvular disease and congenital heart disease (and is more<br />

common than right-sided disease)<br />

b. Depending on the virulence of the infecting organism, it can present as an acute or subacute illness.<br />

c. The mitral valve is most commonly affected.<br />

d. 5 viridans and 5 aureus are the most frequent pathogens.<br />

2. Right-sided endocarditis....,. pulmonary vascular involvement<br />

a. Primarily a disease of IV drug abusers<br />

b. Usually has an acute presentation<br />

c. Most commonly involves tricuspid valve<br />

d. Caused by 5 aureus in -75% of cases<br />

Ill. CLINICAL PRESENTATION<br />

A. History<br />

1. Protean and nonspecific; may include intermittent fever, chills, sweating, malaise, fatigue, weight loss, chest<br />

pain, cough, and neurologic complaints/focal deficits<br />

2. Most common presentation in children is malaise and weight loss.<br />

B. Physical findings<br />

1. Fever (most common finding)<br />

2. Heart murmur<br />

3. CHF<br />

4. Signs of metastatic infection (eg, meningitis, pneumonia)<br />

5. Ophthalmologic signs<br />

a. Conjunctiva! hemorrhages<br />

b. Roth spots (retinal hemorrhages with central clearing)<br />

6. Cutaneous signs<br />

a. Splinter hemorrhages<br />

b. Osler nodes (tender erythematous nodules found on the volar surface of the fingertips)<br />

c. Janeway lesions (nontender erythematous macular lesions appearing on the fingers, palms, and soles)<br />

d. Petechiae<br />

7. Neurologic findings<br />

a. Focal motor deficits<br />

b. Altered level of consciousness<br />

IV. DIAGNOSTIC EVALUATION<br />

A. Laboratory studies<br />

1. CBC reveals leukocytosis with a left shift and a mild normocytic anemia.<br />

2. Erythrocyte sedimentation rate, C-reactive protein, and serum rheumatoid factor are increased.<br />

3. Urinalysis reveals microscopic hematuria in >50% of patients.<br />

B. Chest radiograph<br />

1. Often unremarkable<br />

2. May reveal septic emboli in patients with right-sided endocarditis<br />

C. ECG<br />

1. Usually normal<br />

2. May demonstrate conductive deficits in the presence of extensive myocardial damage<br />

75


CARDIOVASCULAR DISORDERS<br />

D. Blood cultures<br />

1. The most useful test for making the diagnosis; positive in >90% of patients with bacterial endocarditis.<br />

2. Three blood cultures from three different sites should be obtained, and each should be evaluated for aerobic,<br />

anaerobic, and fungal pathogens.<br />

E. Echocardiography<br />

1. Useful if positive for vegetations, but a negative examination does not exclude the diagnosis.<br />

2. Transesophageal echocardiography<br />

a. More sensitive in revealing vegetations than transthoracic echocardiography, especially in patients with<br />

prosthetic valves.<br />

b. Recommended as the initial study for low-risk patients<br />

c. Recommended as the initial study for high-risk patients and as the follow-up study for low-risk patients with<br />

a negative or technically inadequate transthoracic echocardiography<br />

V. DIAGNOSIS<br />

A. A presumptive diagnosis of endocarditis should be made in the patient presenting with risk factors and<br />

clinical findings (eg, a patient with a prosthetic valve or a recent history of IV drug use and fever).<br />

B. Positive blood cultures and evidence of valvular injury or vegetations usually confirm the diagnosis; if any<br />

doubt, a tissue biopsy is the gold standard.<br />

VI. MANAGEMENT<br />

A. Antibiotic therapy<br />

1. Ideally, empiric antibiotic therapy should be started after appropriate blood cultures have been obtained.<br />

However, in hemodynamically unstable patients, antibiotic therapy should be started immediately and not<br />

delayed while waiting for culture results.<br />

2. The initial antibiotic regimen selected should reflect the susceptibilities of the suspected organism, the<br />

acuteness of the presentation, and local resistance patterns.<br />

3. Acceptable empiric antibiotic regimens<br />

a. In patients with native valves and a subacute presentation:<br />

(1) Penicillin G (or ampicillin) IV plus nafcillin (or oxacillin) IV plus gentamicin IM or IV or<br />

(2) Vancomycin IV plus gentamicin IM or IV in penicillin-allergic patients<br />

b. In patients with a prosthetic valve:<br />

(1) Vancomycin IV plus gentamicin IV plus rifampin orally<br />

(2) Early consult with a cardiothoracic surgeon should be obtained in all patients with a prosthetic valve.<br />

c. Vancomycin-resistant enterococcal infections occur, especially in IV drug abusers, patients with prosthetic<br />

valves, and those recently hospitalized or taking antimicrobial agents. Current suggested therapy for these<br />

patients: quinupristin/dalfopristin combination plus doxycyline and rifampin.<br />

B. Although ambulatory treatment is possible in certain select cardiovascularly stable patients, patients with<br />

endocarditis should generally be admitted to the hospital for treatment, particularly those with prosthetic<br />

valves or a history of IV drug abuse.<br />

76


CARDIOVASCULAR DISORDERS<br />

THORACIC AORTIC DISSECTIONS AND<br />

ABDOMINAL AORTIC ANEURYSMS<br />

I. THORACIC AORTIC DISSECTIONS<br />

A. Epidemiology<br />

1. Thoracic aortic dissections are 2-3 times more common than abdominal aortic aneurysm (AAA) ruptures.<br />

2. Men are affected more commonly than women (ratio 3:1).<br />

3. Most patients are 50-70 years old.<br />

4. Risk factors (> 1 increases overall risk)<br />

a. A history of systemic hypertension: this is the most common predisposing factor, and it is present in<br />

70%-90% of patients.<br />

b. Connective tissue disorders<br />

(1) Ehlers-Danlos syndrome<br />

(2) Marfan syndrome (patients may present when


CARDIOVASCULAR DISORDERS<br />

2. DeBakey classification<br />

a. Type I: ascending aorta and part of distal aorta (most common type)<br />

b. Type II: ascending aorta only<br />

c. Type Ill: descending aorta only<br />

(1) Subtype IIIA (extension limited to above the diaphragm)<br />

(2) Subtype 111B (continuation below the diaphragm)<br />

3. Stanford classification<br />

a. Type A: ascending aorta (DeBakey type I and type II)<br />

b. Type B: limited to the descending aorta (DeBakey type Ill)<br />

E. Clinical presentation<br />

1. Pain is the most common presenting symptom.<br />

a. The pain is excruciating, starts abruptly, is maximal from its onset and migrates as the dissection<br />

propagates. It generally (but not always) has a tearing or ripping quality and is most commonly located in<br />

the chest, upper back, or abdomen.<br />

b. The location of pain provides a clue to the location of the dissection.<br />

(1) Ascending (proximal) dissections typically present with anterior chest pain (± radiation into the neck,<br />

jaw, or arms).<br />

(2) Arch dissections cause pain in the neck and jaw.<br />

(3) lnterscapular back pain (± radiation into the abdomen or lumbar area) is typical of descending (distal)<br />

dissections.<br />

2. Based on the location of the dissection, patients may also present with:<br />

a. Acute stroke, visual changes<br />

b. Acute paraplegia, spinal cord deficits<br />

c. A cool, pulseless extremity<br />

d. CHF and pulmonary edema<br />

e. Abdominal pain (mesenteric ischemia), nausea and vomiting<br />

f. Flank pain, hematuria, or oliguria (impaired renal blood flow)<br />

g. Syncope<br />

h. Acute Ml (if dissection involves a coronary artery or ostia)<br />

i. Aortic regurgitation<br />

3. A significant number of patients present with only ischemic or neurologic complications of a "silent" (painless)<br />

aortic dissection (most frequently encountered in Type A dissection that has extended to involve the descending<br />

aorta [similar to a Debakey Type I dissection]). This is important, because inadvertent administration of a<br />

thrombolytic agent to a patient who seems to be having a "stroke" or an "Ml" would be disastrous.<br />

78


CARDIOVASCULAR DISORDERS<br />

4. Blood pressure findings<br />

a. Normal or decreased - ascending aorta<br />

b. Increased - descending aorta<br />

c. A difference in extremity blood pressure of 20 mm Hg<br />

5. Shock-like appearance (cool, clammy skin) despite increased blood pressure<br />

6. Focal neurologic deficits<br />

7. Unequal or absent pulses (hallmark of aortic dissection) between extremities<br />

8. Diastolic murmur of aortic insufficiency (with Types I, II, and A)<br />

9. Signs of cardiac tamponade (with Types I, II, and A)<br />

F. Diagnostic evaluation<br />

1. Initial studies<br />

a. Chest radiograph<br />

(1) Should be taken immediately<br />

(2) Patient should be in upright position.<br />

(3) Usually abnormal but findings are nonspecific; a normal chest radiograph does not exclude the<br />

diagnosis.<br />

(4) Suggestive findings<br />

b. ECG<br />

(a) Mediastinal widening >8 cm (most common finding)<br />

(b) A change in the configuration or size of the thoracic aorta when compared with older<br />

radiographs; loss of space between the aorta and pulmonary artery<br />

(c) "Eggshell" or "calcium" sign - extension of the aortic shadow by >5 mm beyond its calcified<br />

aortic wall (when present, is pathognomonic for dissection)<br />

(d) A blurred aortic knob or one with a localized hump<br />

(e)<br />

(f)<br />

A "double density" appearance of the aorta, which suggests the presence of true and false<br />

channels; the false lumen is less radiopaque than the true lumen.<br />

On the right side of the radiograph<br />

i. Deviation of the trachea/nasogastric tube<br />

ii. Shift and elevation of the right mainstem bronchus<br />

iii. Deviation of the right paraspinous line<br />

(g) On the left side of the film<br />

i. A pleura-apical cap<br />

ii. Depressed left mainstem bronchus<br />

iii. Pleural effusion<br />

(1) Abnormal in most patients with aortic dissection<br />

(2) Changes consistent with acute ischemia have been reported in up to 40% of cases (the most common<br />

misdiagnosis in these patients). STEM\ ECG patterns may occur in up to 8% of cases and usually<br />

involve the inferior leads, because they involve the right coronary artery. A dissection-induced Ml<br />

should be excluded (with transesophageal echocardiography or CT) in patients who are candidates<br />

for thrombolytic therapy and have clinical findings that suggest the possibility of thoracic aortic<br />

dissections.<br />

(3) Varying degrees of AV block can be produced by propagation of the dissection into the ventricular<br />

septum.<br />

(4) Left ventricular hypertrophy (due to long-standing hypertension) is a frequent finding.<br />

2. Definitive studies: one (or more) of the following tests may be used to confirm the diagnosis. Each has certain<br />

advantages and disadvantages (and their 24-hour availability is variable). The diagnostic approach chosen will<br />

vary from institution to institution and patient to patient depending on the technology available, the speed<br />

of obtaining each diagnostic test, the experience of the radiologist in interpreting each test, the preference of<br />

the surgeon, the probability of the diagnosis, and comorbid disease of the patient. When the probability of<br />

dissection is high, the most readily available study should be the initial study of choice.<br />

a. Transesophageal echocardiography (with color-coded Doppler imaging)<br />

(1) Most expedient technique for confirming the diagnosis of aortic dissection; the diagnostic study of<br />

choice in many hospitals<br />

(2) Sensitivity and specificity nearly 100%<br />

79


CARDIOVASCULAR DISORDERS<br />

(3) All the information required for decision-making regarding emergency surgical intervention can be<br />

obtained with this study; in addition to identifying the dissection, it detects the intimal tear site, flow in<br />

the proximal coronary arteries, aortic regurgitation, and pericardia/ effusion; it also differentiates true<br />

from false lumens.<br />

(4) Advantages: only minimally invasive, requires only 10-15 minutes to complete, does not necessitate<br />

exposure to IV contrast, and can be performed at the bedside in the emergency department while the<br />

patient is being treated and appropriately monitored.<br />

(5) Disadvantages: contraindicated in patients with esophageal disease (eg, strictures, varices), and the<br />

necessary equipment/expertise are not readily available in all hospitals.<br />

b. Aortography<br />

c. MRI<br />

(1) The traditional diagnostic gold standard but now largely replaced by CT aortography<br />

(2) Sensitivity and specificity ~90%<br />

(3) In addition to confirming the diagnosis, it defines the intimal tear site, the extent of the dissection, and<br />

assesses the aortic valves and branches.<br />

(4) Disadvantages: invasive, time-consuming, requires special personnel, cannot be performed in the<br />

emergency department, necessitates exposure to IV contrast, is not 100% accurate (can miss dissection<br />

if the false lumen is thrombosed)<br />

(5) Best reserved for situations in which transesophageal echocardiography is not available or is<br />

nondiagnostic<br />

(1) Sensitivity and specificity 1 00%<br />

(2) Delineates the desired anatomy: type and extent of dissection, site of the intimal tear, presence of aortic<br />

insufficiency, and differential flow velocities in the true and false channels and in aortic branch vessels<br />

(3) Advantages: noninvasive, does not require IV contrast material or ionizing radiation<br />

(4) Disadvantages: time-consuming (requires >60 minutes to perform), cannot be performed in the<br />

emergency department or on patients with metallic implants, allows only limited access to the patient<br />

during the examination, some patients cannot tolerate the scanner, most units cannot accommodate<br />

monitoring or advanced life support equipment<br />

d. CT aortography<br />

G. Management<br />

(1) CT aortography has almost 100% specificity and sensitivity and is now accepted as a diagnostic gold<br />

standard.<br />

(2) Can confirm the diagnosis, define the extent of the dissection, and distinguish between Type A and<br />

Type B dissections but cannot identify aortic insufficiency or extension to the aortic branches<br />

(3) Advantages: less invasive and provides greater contrast resolution than aortography, may reveal other<br />

abnormalities in cases in which dissection is excluded<br />

(4) Disadvantages: time-consuming, cannot be performed in the emergency department, exposes the<br />

patient to IV contrast, and may miss a dissection flap (if it is moving rapidly)<br />

(5) good choice for emergency departments without access to transesophageal echocardiography because<br />

of its high sensitivity<br />

1. All patients require 10-15 units of blood on stand-by and immediate consult with a thoracic surgeon.<br />

2. Initial management for all types of dissections is medical and involves controlling the forces that propagate<br />

the dissection: pulse rate, systolic blood pressure, and rate of elevation of the aortic pulse pressure (dP/dT). If<br />

suspicion of dissection is high, begin treatment immediately; do not delay to await confirmatory study results.<br />

a. IV ~-blockers (eg, propranolol, metoprolol, atenolol, or esmolol) are administered first and titrated to a<br />

heart rate of 60-80 beats per minute.<br />

b. Nitroprusside is then started and titrated to a systolic blood pressure of 100-110 mmHg.<br />

3. Hypotensive patients should be managed with small boluses of a crystalloid solution or blood.<br />

4. Pain should be treated with IV narcotics.<br />

5. Long-term management<br />

a. Dissections involving the ascending aorta (Types A, I, and II) are treated surgically.<br />

b. Dissections involving only the descending aorta (Types B and Ill) are treated medically unless<br />

complications are present.<br />

80


CARDIOVASCULAR DISORDERS<br />

II. ABDOMINAL AORTIC ANEURYSMS (AAA)<br />

A. Definition and anatomic location<br />

1. Aortic aneurysms are true aneurysms (they involve al I three layers of the arterial wal I).<br />

2. The vast majority (97%) are infrarenal in location.<br />

B. Pathogenesis<br />

1. Although AAAs often occur in patients with atherosclerotic disease, atherosclerosis is no longer believed to be<br />

the primary etiologic factor in the development of these aneurysms.<br />

2. They are currently theorized to be due to a combination of genetic, structural, and metabolic factors including:<br />

a. Genetic predisposition<br />

b. Increased levels of elastase and collagenase<br />

c. Failure or loss of blood vessel elastin<br />

d. Copper deficiency<br />

e. Infection (mycotic aneurysms)<br />

f. Inflammatory disorders<br />

g. Local mechanical forces<br />

C. Risk factors<br />

1. Advanced age (75% of AAAs are in patients >60 years old)<br />

2. Male sex (particularly white males)<br />

3. Family history of an AAA in a first-degree (blood) relative<br />

4. Smoking history<br />

5. Hypertensive history<br />

6. History of coronary artery disease or peripheral vascular disease<br />

7. Increased serum cholesterol levels<br />

D. Clinical presentation<br />

1. Frequently a middle-aged or older man (and a heavy smoker) who had a syncopal episode at home with<br />

transient improvement<br />

2. Sudden onset of severe abdominal, back, or flank pain with or without an associated syncopal episode (due<br />

to sudden hemorrhage) is the classic presentation: retroperitoneal bleeding with hematoma formation (usually<br />

on the left) occurs in most patients and accounts for the variety of clinical presentations that are possible and<br />

confusing.<br />

a. Flank pain (usually on the left side) may radiate to the groin and be accompanied by microscopic<br />

hematuria, thus simulating a kidney stone (the most common misdiagnosis).<br />

b. Low back pain may be dull and radiate into the legs, thus mimicking musculoskeletal back pain.<br />

c. Abdominal pain that is localized to the left lower quadrant and accompanied by guaiac-positive feces are<br />

common in patients with diverticulitis, but it is also seen in a ruptured AAA (and may be associated with a<br />

nonpulsatile mass).<br />

d. A scrotal hematoma can be interpreted as a "mass" in the scrotum and thus simulate an incarcerated hernia.<br />

e. Ecchymoses can result from significant bleeding and may be seen on the abdominal wall, flank, scrotum,<br />

penis, inguinal region, perineum, or perianal area.<br />

f. Femoral neuropathy (pain in the hip and thigh, quadricep muscle weakness, diminished sensation over<br />

the anteromedial thigh and a weakened patellar reflex) may be from femoral nerve compression due to a<br />

hematoma.<br />

3. Physical examination findings are variable.<br />

a. A pulsatile (and occasionally tender) mass is palpable in the epigastric area in 77% of patients with a<br />

ruptured AAA; unruptured aneurysms are smaller in size and less frequently detected; in the presence of<br />

obesity or abdominal distention, a pulsatile mass may not be felt.<br />

b. A tender (pulsatile) mass is highly suggestive of a rapidly expanding or recently ruptured AAA; most intact<br />

aneurysms are not tender.<br />

c. Bruits may be heard over the abdominal aorta or femoral arteries.<br />

d. Signs of distal extremity ischemia (not always present): unequal or unsynchronized distal pulses and cool,<br />

pale skin.<br />

81


CARDIOVASCULAR DISORDERS<br />

E. Diagnosis and management<br />

1. Presence of a symptomatic AAA is usually suggested by the clinical presentation and physical examination.<br />

This information alone is sufficient evidence in many patients to take them immediately to the operating<br />

room (particularly the hemodynamically unstable ones).<br />

2. Once this diagnosis is suspected, the following measures should be undertaken immediately:<br />

a. Two large-bore IV lines (normal saline or lactated Ringer's), cardiac monitor, pulse oximeter,<br />

supplemental oxygen<br />

b. Draw blood for preoperative laboratory studies, and type and cross for at least 10 units of blood.<br />

c. Order an ECG.<br />

d. Obtain immediate surgical consult (preferably with a vascular surgeon).<br />

3. Further evaluation is determined by the hemodynamic stability of the patient.<br />

a. Hemodynamically unstable patients with a suspected AAA require prompt surgical repair. Fluid and blood<br />

resuscitation should be initiated in the emergency department, and the patient should be transferred<br />

to the operating room for definitive treatment as soon as the surgeon is available. Any delay for further<br />

diagnostic testing increases the risk of death from exsanguination.<br />

b. Hemodynamically stable patients in whom this diagnosis is suspected may undergo further diagnostic<br />

testing (under close medical supervision) before surgical intervention. The various imaging modalities<br />

available are described below. Of these, ultrasonography or CT is the most appropriate studies in this<br />

setting.<br />

(1) Plain abdominal radiographs (AP and lateral films): although most patients with an AAA (75%) have<br />

suspicious findings on plain radiographic evaluation (aneurysmal calcification, soft-tissue mass, loss<br />

of renal shadow, renal displacement, and/or change in the posterior peritoneal flank stripe) except<br />

for aneurysmal calcification, none of these findings is specific for AAA and their absence does not<br />

reliably exclude the presence of an AAA. Furthermore, plain radiographs are not very accurate in<br />

determining the extent of the aneurysm, rupture, or associated vascular pathology.<br />

(2) Ultrasonography has a sensitivity of nearly 100% in detecting an AAA and has the advantage of<br />

being quick, readily available, noninvasive, and portable. It cannot, however, accurately detect<br />

aneurysmal leaking and complications such as visceral or renal artery involvement. Moreover, the<br />

study may be limited in the presence of obesity or intestinal gas. Thus, ultrasound is most useful in<br />

determining whether or not an aneurysm is present, which may be more than enough information in<br />

the appropriate setting.<br />

(3) CT with contrast also has a sensitivity of ~ 100% in detecting an AAA but is more accurate than<br />

ultrasound in detecting aneurysmal rupture and visceral artery involvement; it is not limited by<br />

obesity or intestinal gas. Visualization of the retroperitoneal structures and diagnosis of other<br />

pathology are also superior with this modality. Disadvantages include exposure to IV contrast<br />

material and longer study time.<br />

(4) MRI: excellent at imaging the aorta and is superior to CT in assessing branch vessel involvement.<br />

However, it is of limited use in this setting because it is time consuming, has limited availability,<br />

cannot be performed in the emergency department or in the patient with metallic implants, and<br />

allows only limited access to the patient during the study. This modality is usually reserved for<br />

evaluation of asymptomatic patients.<br />

(5) Angiography: although useful in determining the anatomy of the aorta and demonstrating occlusive<br />

lesions, angiography is not entirely reliable in detecting the presence (high false negative rate) or<br />

diameter of an AAA. Furthermore, it is time consuming, invasive, requires specialized personnel, and<br />

necessitates exposure to IV contrast. For these reasons, angiography is the least desirable study for<br />

emergent diagnostic evaluation of these patients.<br />

82


CARDIOVASCULAR DISORDERS<br />

DEEP VENOUS THROMBOSIS (DVT)<br />

I. PATHOGENESIS<br />

DVTs have their origin from Virchow's triad of venostasis, hypercoagulability, and vessel wall injury/abnormality.<br />

II. CLINICAL PRESENTATION<br />

A. Acute DVT<br />

1. The physical manifestations of DVT are determined by the degree of thrombosis present (partial versus totally<br />

occluding), its location, and the extent of collaterals at the level of the occlusion. Physical findings may be<br />

minimal or absent and cannot be relied on by themselves to make or exclude the diagnosis.<br />

2. Common signs and symptoms<br />

a. Unilateral pain, swelling, edema (most reliable sign), and tenderness<br />

b. Presence of a palpable cord (most often detected in the popliteal fossa), superficial venous dilatation,<br />

discoloration, and Homans sign (the least reliable finding)<br />

3. Unilateral swelling is the most specific physical finding (especially if the measured difference is >3 cm).<br />

4. Tenderness, erythema, or induration in the groin and popliteal fossa (where the femoral and popliteal veins are<br />

quite superficially located) are also highly suggestive of acute thrombosis of the underlying vessel.<br />

B. Massive DVT: an ischemic form of venous occlusion due to massive iliofemoral thrombosis that also involves<br />

most of the venous collateral system<br />

1. Phlegmasia cerulea dolens (painful blue inflammation)<br />

a. Occurs in 40 years old<br />

4. MI/CHF/stroke<br />

5. Obesity<br />

6. Estrogen therapy<br />

7. Polycythemia vera<br />

8. Pregnancy or postpartum state<br />

9. Immobility or prolonged bed rest<br />

83


CARDIOVASCULAR DISORDERS<br />

10. Recent trauma (including burns) or surgery<br />

11. Inherited abnormalities of coagulation (deficiency of antithrombin Ill, protein C, or protein S) or fibrinolysis<br />

12. Catheter placement (central venous, Swan-Ganz) and IV drug abuse<br />

13. Thrombocytosis<br />

14. Immune (eg, AIDS) and autoimmune (eg, systemic lupus erythematosus)<br />

IV. PREDICTING PRETEST PROBABILITY FOR DVT:<br />

THE WELL'S CLINICAL CRITERIA<br />

Table 5: Wei l's Clinical Prediction Rule for DVT<br />

Active cancer (palliative, within 6 months or ongoing treatment)<br />

Paralysis, paresis, or recent immobilization<br />

Recently bedridden (>3 days) or major surgery (within 4 weeks)<br />

Localized tenderness along deep venous system<br />

Entire leg swollen<br />

Calf swelling >3 cm when compared with opposite leg (10 cm below tibial tuberosity)<br />

Pitting edema (greater in suspected leg)<br />

Collateral superficial veins (nonvaricose)<br />

Alternative diagnosis as likely as (or greater than) DVT<br />

-2<br />

Total score interpretation<br />

c:".3 = high pretest probability (>65% risk)<br />

1-2 = moderate pretest probabi I ity (1 7% risk)<br />

:SO= low pretest probability (


CARDIOVASCULAR DISORDERS<br />

2. Contrast venography<br />

3. MRI<br />

a. Formally the accepted standard against which all other diagnostic tests are measured<br />

(1) Largely been replaced by duplex ultrasonography<br />

(2) Now used primarily in patients with uncertain ultrasound results<br />

b. Advantages<br />

(1) Sensitivity and specificity of nearly 1 00 %<br />

(2) Outlines the entire venous system<br />

c. Disadvantages<br />

(1) Invasive, painful, expensive, limited in availability, and associated with risk of contrast-related<br />

allergic reactions and postvenography phlebitis/DVT<br />

(2) One-quarter of patients with suspected DVT have contraindications to venography (renal failure, dye<br />

allergy, pregnancy) or nondiagnostic studies.<br />

a. Detects calf, thigh, pelvic, renal, and pulmonary thrombi; can also diagnose other anatomic causes of leg<br />

pain and swelling<br />

b. Advantages<br />

(1) Noninvasive<br />

(2) No ionizing radiation or IV contrast<br />

(3) 97% sensitive and 95% specific for DVT<br />

c. Disadvantages<br />

(1) Expensive, not readily available, and requires significant patient cooperation<br />

(2) Should not replace ultrasound as the primary screening tool<br />

d. Most useful during the second and third trimesters of pregnancy when ultrasound is less accurate<br />

4. o-dimer assay<br />

a. o-dimer<br />

B. Diagnosis<br />

(1) Fibrin degradation product, which is increased in patients with DVT and pulmonary embolism<br />

(2) Increased in many conditions (poor specificity) such as acute Ml, stroke, trauma, and pregnancy<br />

b. There are a number of rapid o-dimer assays, including qualitative whole blood agglutination tests<br />

(SimpliRED) and quantitative turbidimetric or ELISA tests, in which levels >500 mg/Lare abnormal.<br />

c. Like duplex ultrasonography and impedance plethysmography, it is more sensitive for proximal clots<br />

(93%) than for distal ones (70%).<br />

1. DVT should be suspected from the patient's signs, symptoms, and history of risk factors. However, a clinical<br />

diagnosis of DVT is insensitive and inaccurate; therefore, the diagnosis must be confirmed (with one of the<br />

diagnostic tests described above) before treatment.<br />

2. The current recommended diagnostic algorithm is to use a combination of a clinical scoring system (eg,<br />

Well's Criteria [see Table 5]) to determine pretest probability in conjunction with a o-dimer assay and/or<br />

venous ultrasound.<br />

3. For patients at low risk of DVT<br />

a. A negative o-dimer excludes DVT.<br />

b. A positive o-dimer warrants venous ultrasound. If this is negative, DVT is excluded. If positive, proceed<br />

with anticoagulation.<br />

4. For patients at moderate or high risk of DVT<br />

a. Initially order venous ultrasound. If positive, anticoagulate.<br />

b. However, a negative ultrasound in a predicted moderate- or high-risk patient does not exclude DVT. It is<br />

recommended to then order a o-dimer to further risk stratify these higher-risk patients.<br />

(1) If the o-dimer is negative, a DVT is excluded, and no further testing is required.<br />

(2) If the o-dimer is positive, the patient requires a repeat ultrasound in 2-7 days.<br />

5. The treatment of isolated calf DVT remains controversial, even though ~25% of these thrombi propagate<br />

proximally and may subsequently embolize. If an isolated calf DVT is detected, the risk and benefit of<br />

anticoagulation should be weighed against patient's pre-test probability.<br />

a. For moderate- and high-risk patients, anticoagulation should be started unless contraindications exist.<br />

b. For low-risk patients, repeat ultrasound is required with either no treatment or simply starting aspirin<br />

therapy.<br />

85


CARDIOVASCULAR DISORDERS<br />

VI. PHARMACOLOGICTHERAPY<br />

A. Goals<br />

1. Prevent pulmonary embolism<br />

2. Reduce morbidity associated with the acute event<br />

3. Prevent (or minimize) postphlebitic syndrome<br />

B. Anticoagulation<br />

1. In 2016, the American College of Chest Physicians released their most current guidelines on "Antithrombotic<br />

Therapy for VTE Disease."<br />

a. Multiple new oral anticoagulants have been released in recent years, collectively called "NOAC" (nonvitamin<br />

Koral anticoagulants).<br />

(1) Dabigatran<br />

(2) Rivaroxaban<br />

(3) Apixaban<br />

(4) Edoxaban<br />

b. Advantages<br />

(1) Oral dosing as opposed to injections<br />

(2) No need for blood level monitoring<br />

c. Disadvantages<br />

(1) Lack of reversibility (some new agents are reported to be coming to market soon for specific NOACs)<br />

(2) Lack of real-time monitoring for therapeutic effect with recurrent presentations suggestive of venous<br />

thromboembolism (VTE)<br />

d. Recommendations<br />

(1) For patients with VTE and no cancer, a NOAC is recommended over vitamin K antagonist (VKA)<br />

therapy, and VKA therapy is recommended over low-molecular-weight heparin (LMWH).<br />

(2) For patients with VTE and cancer, LMWH is recommended overVKA or NOAC therapy.<br />

2. IV heparin (unfractionated) or LMWH are still viable treatment options forVTE. However, LMWH and<br />

unfractionated heparin are contraindicated in patients with a history of heparin-induced thrombocytopenia;<br />

these patients should receive alternative agents. Four LMWHs are approved for therapy, but enoxaparin is most<br />

commonly prescribed.<br />

a. Enoxaparin<br />

(1) 1 mg/kg SC every 12 hours (inpatient or outpatient)<br />

(2) 1.5 mg/kg/day SC (inpatient only)<br />

3. Patients with antithrombin Ill deficiency require pretreatment with antithrombin Ill concentrate or fresh frozen<br />

plasma to replenish this factor before heparinization.<br />

4. Warfarin 5 mg/day should generally be started on day one of treatment; anticoagulation parameters should be<br />

checked on day three.<br />

5. Outpatient treatment may be considered for carefully selected patients, especially with development of<br />

NOACs, which do not require therapeutic blood level monitoring.<br />

a. No serious concomitant disease requiring hospitalization<br />

b. Communication and transportation is adequate<br />

c. If patient is placed on warfarin, INR blood level checked in 2-3 days<br />

C. Thrombolytic therapy<br />

1. Compared with heparin therapy, thrombolytic therapy resolves symptoms more rapidly, preserves valve<br />

integrity, and may decrease the incidence of postphlebitic syndrome. However, indications for thrombolytics in<br />

patients with DVT are unclear.<br />

2. Usually reserved for patients


CARDIOVASCULAR DISORDERS<br />

D. Inferior vena caval interruption with a Greenfield filter<br />

1. Primary indication<br />

a. Proximal DVT (above the level of the knee) in a patient with a contraindication to anticoagulation or<br />

thrombolytic therapy<br />

b. Those who require urgent surgery that precludes anticoagulation or those in whom treatment has not been<br />

effective<br />

2. Other indications<br />

a. Recurrent DVT despite adequate anticoagulation<br />

b. Presence of a large free-floating caval thrombus<br />

c. Chronic recurring embolization in a patient with pulmonary hypertension<br />

3. Patients with malignancies seem to benefit most from this procedure.<br />

VII. DISPOSITION-ADMISSION CRITERIA<br />

A. Extensive ileofemoral DVT with circulatory compromise<br />

B. Increased risk of bleeding requiring close monitoring of therapy<br />

C. Limited cardiorespiratory reserve<br />

D. Risk of poor compliance with home therapy or inadequate assistive support<br />

E. Contraindications to LMWH heparin necessitating IV heparin therapy<br />

87


CARDIOVASCULAR DISORDERS<br />

HYPERTENSIVE EMERGENCIES AND URGENCIES<br />

I. HYPERTENSIVE EMERGENCIES<br />

A. Definition<br />

1. Patients with severely increased diastolic blood pressure (> 140 mm Hg) and evidence of acute end-organ<br />

dysfunction or damage<br />

2. The presence of end-organ damage (not the absolute blood pressure) determines a patient with a hypertensive<br />

emergency.<br />

3. The heart, brain, and kidneys are the organs most often affected.<br />

B. Clinical presentation<br />

1. Malignant hypertension<br />

2. Hypertensive encephalopathy<br />

3. Hypertension with acute intracranial events (hemorrhagic stroke, thrombotic stroke, subarachnoid hemorrhage)<br />

4. Aortic dissection<br />

5. Acute pulmonary edema<br />

6. Acute myocardial ischemia or injury<br />

7. Eclampsia<br />

8 Acute hypertensive renal insufficiency<br />

9. Catecholamine-induced hypertensive crisis: an acute increase in circulating catecholamines that produces an<br />

increased blood pressure with headache, palpitations, sweating, and tachycardia. Causes include:<br />

a. Pheochromocytoma (adrenal tumor)<br />

b. Concomitant use of monoamine oxidase inhibitors and sympathomimetic agents. Commonly implicated<br />

sympathomimetics include:<br />

(1) Tyramine (Chianti wine, aged cheese, beer, pickled herring, chicken liver)<br />

(2) Ephedrine and phenylpropanolamine (found in OTC cold and cough preparations and appetite<br />

suppressants)<br />

c. Acute cocaine intoxication<br />

d. Acute clonidine withdrawal (particularly when withdrawn simultaneously with a 13-blocking agent)<br />

C. Management<br />

1. The goal of therapy is to halt and reverse the progression of endorgan dysfunction while maintaining organ<br />

perfusion and avoiding complications. To do this, the blood pressure must be lowered rapidly and in a<br />

controlled manner. The exact extent of blood pressure reduction is determined by the clinical situation;<br />

however, a reasonable target is to reduce the blood pressure by 20%-30% of pretreatment levels over the first<br />

hour of therapy.<br />

2. Medications are usually given by the IV route, which provides a faster onset of action and is more easily titrated.<br />

3. The antihypertensive agent of choice varies with the specific hypertensive emergency.<br />

a. Malignant hypertension and hypertensive encephalopathy: Historically, nitroprusside was the drug of<br />

choice. However, IV labetalol is now used most commonly, particularly when intra-arterial pressure<br />

monitoring is not available. Nicardipine IV is also effective.<br />

b. Hypertension with an ischemic stroke: treatment of hypertension in the immediate poststroke period can<br />

be detrimental in these patients and should not be initiated in most instances; acute reduction in blood<br />

pressure can reduce perfusion to the surrounding watershed areas and may thereby extend the infarction.<br />

Therefore, blood pressure reduction should be considered only in patients with extremely high blood<br />

pressures (>220/120 mmHg). Short-acting agents with few CNS effects (such as IV nitroprusside or IV<br />

labetalol) are the drugs of choice.<br />

c. Hypertension with hemorrhagic stroke: although somewhat more controversial, most authors agree that<br />

mild to moderate hypertension is generally well-tolerated in these patients, but that severe hypertension<br />

(blood pressu1·e > 220/120 mm Hg) may promote further hemorrhage and should be judiciously controlled<br />

with a titratable agent such as IV labetalol (other acceptable agents: esmolol, enalapril, nicardipine).<br />

d. Hypertension with subarachnoid hemorrhage: nimodipine or nicardipine (calcium channel blockers) is the<br />

initial therapeutic agent of choice. In addition to its antihypertensive effect, nimodipine also decreases the<br />

cerebral vasospasm that occurs after subarachnoid hemorrhage. The goal of treatment is to reduce the mean<br />

88


CARDIOVASCULAR DISORDERS<br />

arterial pressure to prehemorrhagic levels. Blood pressure reduction is associated with a decrease in the<br />

risk of rebleeding in these patients.<br />

e. Aortic dissection: nitroprusside in combination with an IV B-blocker (propranolol or esmolol) is the therapy<br />

of choice; the B-blocking agent should be administered first to prevent the reflex tachycardia that can occur<br />

in association with nitroprusside. IV labetalol as a single drug therapy is the alternative agent of choice.<br />

f. Acute myocardial ischemia or injury: IV nitroglycerin is the primary agent of choice because of its<br />

beneficial effects on coronary perfusion. IV B-blockers can be considered, although they typically affect<br />

the heart rate more than blood pressure, so their effect is frequently inadequate. In addition, their use in<br />

acute Ml is still controversial. If blood pressure remains increased despite aggressive use of nitroglycerin,<br />

nitroprusside is the alternative of choice; however, it can increase myocardial ischemia via the coronary<br />

steal syndrome, so it is prudent to run nitroglycerin concurrently.<br />

g. Acute pulmonary edema: if standard measures for pulmonary edema (eg, noninvasive positive pressure<br />

ventilation, high-dose IV nitroglycerin, IV furosemide) fail to adequately decrease blood pressure, specific<br />

antihypertensive therapy should be initiated. IV nitroglycerin is the drug of choice. If blood pressure<br />

remains increased despite aggressive use of nitroglycerin, nitroprusside is the alternative agent.<br />

h. Eclampsia: IV magnesium sulfate in combination with IV hydralazine are first-line agents. IV labetalol is<br />

also safe and effective. Nitroprusside may be used in postpartum eclampsia but, because of the risk of fetal<br />

cyanide poisoning, is relatively contraindicated antepartum; it should be reserved for those patients who do<br />

not respond to the above-mentioned treatment regimens.<br />

1. Acute hypertensive renal insufficiency: nitroprusside is the drug of choice, but patients must be carefully<br />

monitored for thiocyanate toxicity (which is more common in this setting). Nifedipine or labetalol are good<br />

alternatives.<br />

j. Catecholamine-induced hypertensive crisis due to:<br />

(1) Pheochromocytoma or monoamine oxidase inhibitor interaction: phentolamine followed by an IV<br />

B-blocker are first-line agents; nitroprusside followed by an IV B-blocker is also effective.<br />

(2) Acute clonidine withdrawal: restart clonidine; phentolamine followed by an IV B-blocker can be<br />

effective.<br />

(3) Acute cocaine intoxication: sedation with a benzodiazepine such as diazepam is usually very<br />

effective. If hypertension persists despite adequate sedation, administer phentolamine (followed by an<br />

IV B-blocker if severe tachycardia is present). Nitroglycerin can be used if the patient is experiencing<br />

cardiac ischemia.<br />

4. Medications to avoid in hypertensive emergencies<br />

a. Malignant hypertension/hypertensive encephalopathy<br />

(1) ACE inhibitors<br />

(2) Clonidine<br />

(3) Pure B-blockers<br />

b. Acute myocardial ischemia or injury/aortic dissection<br />

(1) Diazoxide<br />

(2) Hydralazine<br />

(3) Minoxidil<br />

c. Acute pulmonary edema<br />

(1) Diazoxide<br />

(2) Hydralazine<br />

(3) Minoxidil<br />

d. Eclampsia<br />

(1) Diuretics (furosemide, bumetanide, hydrochlorothiazide)<br />

(2) ACE inhibitors<br />

(3) Nitroprusside: relatively contraindicated antepartum because of possible cyanide toxicity<br />

e. Acute hypertensive renal insufficiency<br />

(1) Diazoxide<br />

(2) Pure B-blockers as single agents<br />

f. Catecholamine-induced hypertensive crisis<br />

(1) Minoxidil<br />

(2) Pure B-blockers as single agents<br />

89


CARDIOVASCULAR DISORDERS<br />

II. HYPERTENSIVE URGENCIES<br />

A. Definition and etiology<br />

1. A diastolic blood pressure ?:115 mmHg without evidence of end-organ dysfunction or damage<br />

2. These patients are asymptomatic and usually present to the emergency department for an unrelated problem.<br />

3. Most common in patients with chronic hypertension who are noncompliant with their medications<br />

B. Management<br />

1. Goal of therapy is to lower the blood pressure gradually over a period of 24-48 hours with oral medications.<br />

Catastrophic consequences may occur with IV administration.<br />

2. Commonly used agents<br />

a. Nifedipine<br />

b. Labetalol<br />

c. Clonidine<br />

d. ACE inhibitors<br />

90


CARDIOVASCULAR DISORDERS: PRACTICE CLINICAL SCENARIOS<br />

CARDIOVASCULAR DISORDERS: PRACTICE CLINICAL SCENARIOS<br />

Answers immediately follow the practice clinical scenarios.<br />

Scenario A<br />

Presentation: A middle-aged man who has diabetes and hypertension presents with substernal, left-sided<br />

chest pain and a crushing/heavy pressure sensation, radiating to the left jaw and down the left arm. He<br />

appears diaphoretic and in obvious distress.<br />

Courtesy of Dr. Michael McCrea<br />

What is the diagnosis?<br />

Scenario B<br />

Presentation: A patient with acute decompensated heart failure presents with respiratory distress and gives<br />

a history that includes increasing lower extremity edema, orthopnea, paroxysmal nocturnal dyspnea, and a<br />

nonproductive cough.<br />

Physical examination: The patient is in respiratory distress with hypoxia. Physical examination findings<br />

include jugular venous distention, rales on lung auscultation, and symmetrical, pitting lower extremity<br />

edema.<br />

What is the diagnosis?<br />

Scenario C<br />

Presentation: An elderly patient with a history of uncontrolled hypertension presents with sudden onset of<br />

severe ripping, tearing chest pain radiating to the mid back between the scapulae. Since onset, the pain in<br />

the back has migrated caudally.<br />

What is the diagnosis?<br />

91


CARDIOVASCULAR DISORDERS: PRACTICE CLINICAL SCENARIOS<br />

Scenario D<br />

Presentation: A 45-year-old woman with end-stage renal disease presents to the emergency department<br />

from her dialysis center after a syncopal episode. She has missed the previous three weeks of dialysis.<br />

On examination, she is hypotensive, minimally responsive, and in severe respiratory distress. She is<br />

tachycardic and has barely audible heart sounds with marked jugular venous distension.<br />

What is the diagnosis?<br />

Scenario E<br />

Presentation: A 22-year-old man with a history of Wolff-Parkinson-White (WPW) syndrome presents after<br />

a syncopal episode at work. On arrival at the emergency department, he is awake but has a blood pressure<br />

of 100/50 mm Hg and is feeling lightheaded. His ECG shows an irregularly irregular wide-complex<br />

tachycardia with QRS complexes that vary in width.<br />

What is the diagnosis?<br />

Scenario F<br />

Presentation: A 66-year-old man presents with palpitations. He has been feeling weak and tired today and<br />

had chest pain earlier but that has resolved. On arrival, he appears well and nontoxic. After the patient has<br />

been placed on the monitor, the nurse calls outs and shows you this rhythm strip:<br />

Courtesy of Dr. Michael McCrea<br />

What is the diagnosis?<br />

ScenarioG<br />

Presentation: A 74-year-old woman is transferred from her extended-care facility for a "psych work-up."<br />

She is confused, hallucinating, and states all the nurses are "angels" because "everyone has a halo." Her<br />

past medical history is significant for atrial fibrillation and CHF, and she is currently being treated for<br />

a UTI. On examination, she has an irregularly irregular bradycardic pulse. After initial management is<br />

started, the nurse tells you the patient has a potassium level of 6.9 mEq/L.<br />

What is the diagnosis?<br />

Scenario H<br />

Presentation: A 73-year-old man presents from church after a syncopal episode with severe left-sided back<br />

pain. He states he has a history of kidney stones. His blood pressure is 86/43 mmHg with a heart rate of<br />

127 beats per minute. On examination, he has significant left-sided abdominal tenderness with ecchymosis<br />

along his left flank.<br />

What is the diagnosis?<br />

92


CARDIOVASCULAR DISORDERS: PRACTICE CLINICAL SCENARIOS<br />

ANSWERS TO PRACTICE CLINICAL SCENARIOS<br />

Scenario A<br />

Diagnosis: STEMI<br />

Diagnostic evaluation: The ECG shows ST-segment elevation in contiguous leads in an anatomic<br />

distribution (anterior).<br />

Management: After IV access has been established, the patient should be placed on a monitor, provided<br />

low-flow supplemental oxygen for any dyspnea or hypoxia, and given aspirin 325 mg (chewed) unless he<br />

has a true aspirin allergy. Nitroglycerin can be administered as needed for chest pain. Coronary reperfusion<br />

is the definitive treatment and wi 11 be guided by avai labi I ity of PCI. If the patient presents to a hospital<br />

with PCI capabilities, the cardiac catheterization team should be activated immediately and the patient<br />

prepared for emergent PCI with a first medical contact-to-device time of 90 minutes. If the patient had<br />

presented to a hospital that does not have PCI capabilities, the goal for "door-in, door-out" at the initial<br />

hospital is 30 minutes, with a first medical contact-to-device time of 120 minutes, allowing for transport<br />

time to the PCI hospital. The patient should also be given a heparin bolus followed by infusion.<br />

Scenario B<br />

Diagnosis: decompensated congestive heart failure<br />

Diagnostic evaluation: Possible triggers for acute decompensation include medication noncompliance,<br />

dietary indiscretion (excess sodium intake), dysrhythmias, or acute coronary syndrome. Chest radiograph<br />

may show cardiomegaly depending on the cause of the heart failure. Chest radiograph will also show<br />

pulmonary venous congestion. An ECG should be obtained to evaluate for concurrent dysrhythmia or<br />

acute coronary syndrome. Laboratory studies should include renal function testing, cardiac biomarkers,<br />

and a brain natriuretic peptide.<br />

Management: The patient should be started on Bi PAP with supplemental oxygen if needed for hypoxia.<br />

ACE inhibitors and high-dose nitroglycerin should be given to lower afterload and preload, respectively.<br />

After this, if the patient is volume overloaded and has normal renal function, furosemide should be given<br />

for diuresis.<br />

Scenario C<br />

Diagnosis: thoracic aortic dissection<br />

Diagnostic evaluation: Depending on the location of the dissection, there may be differences in the<br />

upper extremity blood pressures or a pulse difference between the right arm and left arm (Debakey 1/<br />

II or Stanford A). A dissection below the origin of the left subclavian artery (Debakey Ill or Stanford B)<br />

would not necessarily have a difference in upper extremity pulses or blood pressures but would likely<br />

have a difference when comparing upper and lower extremities. An ECG should be done to exclude<br />

acute coronary syndrome as a cause of the chest pain or to evaluate for concomitant STEMI, because a<br />

proximal dissection could involve the aortic root and coronary arteries. An initial chest radiograph should<br />

be obtained, because there are classic findings of aortic dissection on chest radiograph, most notably<br />

a widened mediastinum. However, the lack of classic radiographic findings does not exclude aortic<br />

dissection, especially in a patient with a classic presentation and risk factors as above. In stable patients,<br />

CTA of the chest or an echocardiogram (preferably transesphogeal) could be ordered depending on<br />

availability. Unstable patients should not have any additional imaging in the emergency department and<br />

should proceed to the operating room.<br />

Management: Rapid control of heart rate and blood pressure is the primary treatment concern while<br />

arranging for definitive surgical care, especially in unstable patients. A ~-blocker should be given to<br />

decrease the heart rate, and a vasodilator such as nitroprusside should then be given for rapid control of<br />

blood pressure. The patient should be typed and crossmatched for 10 units of blood while preparing for the<br />

operating room.<br />

93


CARDIOVASCULAR DISORDERS: PRACTICE CLINICAL SCENARIOS<br />

Scenario D<br />

Diagnosis: pericardia! tamponacle<br />

Diagnostic evaluation: This patient exhibits the classic findings of Beck's triad for pericardia! tamponacle:<br />

hypotension, clistenclecl neck veins, and distant heart sounds. She is a noncompliant dialysis patient and<br />

has developed a large pericardia! effusion that needs to be drained emergently. Bedside ultrasound should<br />

be performed to confirm the diagnosis. An ECG may show electrical alternans.<br />

Management: After placing the patient on the monitor and placing two large-bore IV lines, the patient<br />

should be given crystalloicl fluids (2 L bolus) to treat hypotension and help increase preloacl to combat<br />

tamponacle collapse of the ventricles. Emergent pericarcliocentesis should be performed, ideally with<br />

ultrasound guidance, although ECG monitoring with an ECG lead clipped to the needle may be performed<br />

as well.<br />

Scenario E<br />

Diagnosis: atrial fibrillation with WPW<br />

Diagnostic evaluation: WPW can present with a variety of tachyclysrhythmias with important distinctions<br />

in management. The rhythm described, an irregularly irregular wide complex rhythm, is the most<br />

concerning and serious of the WPW presentations. The irregular nature suggests this is not simple AV<br />

reentry tachycardia but rather atrial fibrillation.<br />

Management: In a patient with known WPW, the fact that the QRS is wide means that the anterogracle<br />

direction is clown the accessory pathway and then conducted retrograde back through the AV node.<br />

Therefore, any AV nodal blocking agent, eg, aclenosine, ~-blockers, calcium channel blockers, or cligoxin,<br />

are contraindicated in management of this patient. This would also extend to amioclarone, because it has<br />

AV nodal blocking properties as well. Treating this patient with any AV nodal blocking agent could result in<br />

fatal deterioration to ventricular fibrillation, which may be refractory to therapy. Therefore, the treatment of<br />

choice for this patient is procainamicle. Should the patient become unstable, he would require immediate<br />

carclioversion, as with any unstable tachycarclic patient. However, because the rhythm is a wide complex<br />

irregular tachycardia, per the ACLS guidelines, treatment would be with an unsynchronized carclioversion<br />

at clefibrillation closes, ie, 200 joules.<br />

Scenario F<br />

Diagnosis: ventricular tachycardia<br />

Diagnostic evaluation: The rhythm strip shows a regular wide complex tachycardia, which is concerning<br />

for ventricular tachycardia in a 66-year-olcl man who had chest pain before presentation. A 12-leacl ECG<br />

should be obtained to confirm the diagnosis of ventricular tachycardia. Although multiple clinical decision<br />

tools exist to assist in the differentiation of ventricular tachycardia versus SVT with aberrant conduction, if<br />

there is any doubt, the patient should be treated as if in ventricular tachycardia.<br />

Management: Because this a wide complex tachycardia and the patient is currently stable, ACLS guidelines<br />

state that the patient should be treated with either amioclarone 150 mg over 10 minutes or procainamicle<br />

25-50 mg/min infusion. An external defibrillator should be applied to the patient in case any clinical<br />

deterioration occurs. Should the patient become unstable or his chest pain recur, he should immediately<br />

be treated with synchronized carclioversion, with a recommended starting close of 100 joules and<br />

increasing closes as needed.<br />

94


CARDIOVASCULAR DISORDERS: PRACTICE CLINICAL SCENARIOS<br />

ScenarioG<br />

Diagnosis: digitalis toxicity<br />

Diagnostic evaluation: The patient is exhibiting classic signs of chronic digitalis toxicity induced by a<br />

change in renal function due to the UTI for which she is being treated. The ECG shows atrial fibrillation<br />

with slow ventricular response and ST segment downsloping. The classic ST changes of digoxin indicate<br />

only that the patient is on digoxin, not that the patient has digitalis toxicity. Checking the potassium level is<br />

of utmost importance, because an increased level is itself an indication for digoxin-specific antibody.<br />

Management: Because the patient is bradycardic, initial treatment with atropine should be attempted<br />

and an external pacemaker should be at the bedside. If the bradycardia is not responsive to atropine or<br />

the patient becomes symptomatic, external transcutaneous pacing is indicated. However, this patient<br />

meets multiple criteria for digoxin-specific antibodies: bradycardia and hyperkalemia. Hyperkalemia<br />

may be treated with insulin, glucose, and bicarbonate, but classic teaching is to avoid administration<br />

of calcium. Similarly, classic teaching is to avoid transvenous pacing because of concern of inducing a<br />

tachydysrhythmia while floating the pacemaker wire.<br />

Scenario H<br />

Diagnosis: ruptured abdominal aortic aneurysm (AAA)<br />

Management: This patient has a classic presentation for a ruptured AAA, including the foil of a possible<br />

kidney stone. The combination of syncope, hypotension with a pulsatile abdominal mass, and ecchymosis<br />

on the flank alone are indication to call emergently for vascular surgery. No additional testing is necessary,<br />

although a bedside ultrasound to confirm the AAA may be completed, a CT at this point would be<br />

inappropriate given the instability of the patient. Fluid resuscitation should be started until crossmatched<br />

blood is available while the operating room is prepared.<br />

95


96<br />

NOTES


HEAD, EAR, EYE, NOSE, AND THROAT DISORDERS<br />

HEAD, EAR, EYE, NOSE, AND THROAT DISORDERS<br />

OTOLOGIC EMERGENCIES ...................................................................................................................................... 107<br />

Anatomy ................................................................................................................................................................... 1 07<br />

Sensory Supply to the Ear ................................................................................................................................... 1 07<br />

Auditory Transmission ........................................................................................................................................ 107<br />

Infections .................................................................................................................................................................. 107<br />

Acute Otitis Externa (Swimmer's Ear) .................................................................................................................. 1 07<br />

Malignant (Necrotizing) Otitis Externa ................................................................................................................ 107<br />

Posttraumatic Perichondritis ............................................................................................................................... 108<br />

Acute Otitis Media ............................................................................................................................................. 1 08<br />

Bullous Myringitis .............................................................................................................................................. 109<br />

Mastoiditis ................................................................................................................................................................ 1 09<br />

Sudden Hearing Loss ................................................................................................................................................. 110<br />

Vertigo ...................................................................................................................................................................... 111<br />

Trauma ...................................................................................................................................................................... 112<br />

EYE EMERGENCIES ................................................................................................................................................... 113<br />

Conjunctivitis ............................................................................................................................................................ 113<br />

Viral Conjunctivitis ............................................................................................................................................. 113<br />

Bacterial Conjunctivitis ...................................................................................................................................... 113<br />

Chlamydia! (inclusion) Conjunctivitis ................................................................................................................. 114<br />

Vernal Conjunctivitis .......................................................................................................................................... 115<br />

Inflammatory Disorders of the Eyelids and Cornea ................................................................................................... 115<br />

Anterior Blepharitis ............................................................................................................................................ 115<br />

Hordeolum (Stye) ............................................................................................................................................... 116<br />

Chalazion ........................................................................................................................................................... 116<br />

Dacryocystitis ..................................................................................................................................................... 116<br />

Corneal Ulcers ................................................................................................................................................... 117<br />

Herpes Simplex Keratitis ..................................................................................................................................... 117<br />

Herpes Zoster Ophthalmicus .............................................................................................................................. 118<br />

Ultraviolet Keratitis ............................................................................................................................................. 119<br />

Periorbital and Orbital Cellulitis ............................................................................................................................... 119<br />

Acute Eye Pain ........................................................................................................................................................... 121<br />

Acute lritis .......................................................................................................................................................... 121<br />

Acute Angle Closure Glaucoma .......................................................................................................................... 122<br />

Acute Visual Loss ....................................................................................................................................................... 123<br />

Central Retinal Artery Occlusion ........................................................................................................................ 123<br />

Central Retinal Vein Occlusion ........................................................................................................................... 124<br />

Optic Neuritis .................................................................................................................................................... 124<br />

Eclipse Burn ....................................................................................................................................................... 125<br />

Retinal Detachment. ........................................................................................................................................... 126<br />

Vitreous Hemorrhage ......................................................................................................................................... 126<br />

Functional Blindness .......................................................................................................................................... 127<br />

Temporal (Giant Cell) Arteritis ............................................................................................................................ 127<br />

Nonacute Visual Loss ................................................................................................................................................ 127<br />

Primary Open Angle Glaucoma .......................................................................................................................... 127<br />

Cytomegalovirus Retinitis ................................................................................................................................... 128<br />

97


HEAD, EAR, EYE, NOSE, AND THROAT DISORDERS<br />

Trauma ...................................................................................................................................................................... 128<br />

Chemical Burns .................................................................................................................................................. 128<br />

Lid Lacerations ................................................................................................................................................... 129<br />

Corneal Injuries .................................................................................................................................................. 129<br />

Ruptured Globe .................................................................................................................................................. 130<br />

Hyphema ........................................................................................................................................................... 131<br />

Lens Subluxation and Dislocation ...................................................................................................................... 131<br />

Blowout Fractures of the Orbit... ......................................................................................................................... 132<br />

Retrobulbar Hematoma ...................................................................................................................................... 132<br />

lntraocular Foreign Body .................................................................................................................................... 132<br />

Traumatic lritis/lridocyclitis ................................................................................................................................. 133<br />

Traumatic Miosis and Mydriasis ......................................................................................................................... 133<br />

Anisocoria ................................................................................................................................................................. 133<br />

Ophthalmic Medications ........................................................................................................................................... 134<br />

Packaging ........................................................................................................................................................... 134<br />

Topical Anesthetics ............................................................................................................................................. 134<br />

Mydriatics and Cycloplegics ............................................................................................................................... 135<br />

Miotics ............................................................................................................................................................... 135<br />

Topical Antibiotics .............................................................................................................................................. 135<br />

Antivirals ............................................................................................................................................................ 136<br />

Topical Steroids .................................................................................................................................................. 13 6<br />

Hyperosmotic Agents ......................................................................................................................................... 136<br />

Carbonic Anhydrase Inhibitors ........................................................................................................................... 136<br />

Topical Adrenergic Agents .................................................................................................................................. 136<br />

Tonometry ................................................................................................................................................................. 137<br />

NASAL TRAUMA ....................................................................................................................................................... 138<br />

Nasal Fractures ......................................................................................................................................................... 138<br />

Major Complications ................................................................................................................................................. 138<br />

Septal Hematoma ............................................................................................................................................... 138<br />

CSF Rhinorrhea .................................................................................................................................................. 138<br />

Hemorrhage ....................................................................................................................................................... 139<br />

NASAL FOREIGN BODIES ........................................................................................................................................ 140<br />

Clinical Presentation ................................................................................................................................................. 140<br />

Diagnosis and Treatment ........................................................................................................................................... 140<br />

EPISTAXIS ................................................................................................................................................................. 141<br />

Vascular Supply to the Nose ..................................................................................................................................... 141<br />

Etiology ..................................................................................................................................................................... 141<br />

Clinical Presentation ................................................................................................................................................. 141<br />

Treatment .................................................................................................................................................................. 142<br />

Complications ........................................................................................................................................................... 142<br />

FACIAL INFECTIONS ................................................................................................................................................ 143<br />

Sinusitis ..................................................................................................................................................................... 143<br />

Parotitis ..................................................................................................................................................................... 144<br />

FACIAL FRACTURES .................................................................................................................................................. 146<br />

Mandibular Injuries ................................................................................................................................................... 146<br />

Hal !marks of Mandibular Dysfunction ................................................................................................................ 146<br />

Mandibular Fractures .......................................................................................................................................... 146<br />

Mandibular Dislocation ...................................................................................................................................... 146<br />

Temporomandibular Joint Dysfunction ............................................................................................................... 147<br />

Midfacial Fractures ................................................................................................................................................... 147<br />

Midfacial Area .................................................................................................................................................... 147<br />

Isolated Zygomatic Arch Fractures ...................................................................................................................... 147<br />

Zygomatic-Maxillary Complex ("Tripod") Fractures ............................................................................................ 148<br />

Orbital Floor Fractures ........................................................................................................................................ 148<br />

Multiple Facial Fractures ..................................................................................................................................... 148<br />

98


HEAD, EAR, EYE, NOSE, AND THROAT DISORDERS<br />

ACUTE AIRWAY OBSTRUCTION .............................................................................................................................. 150<br />

Clinical Presentation ................................................................................................................................................. 150<br />

Differential Diagnosis ............................................................................................................................................... 150<br />

ORAL AND PHARYNGEAL INFECTIONS ................................................................................................................. 154<br />

Oral Infections .......................................................................................................................................................... 154<br />

Masticator Space Abscess ................................................................................................................................... 154<br />

Ludwig Angina ................................................................................................................................................... 154<br />

Pharyngeal Space Infections ..................................................................................................................................... 155<br />

Parapharyngeal Abscess ...................................................................................................................................... 155<br />

Peritonsil lar Abscess ........................................................................................................................................... 155<br />

Retropharygeal Abscess ...................................................................................................................................... 156<br />

Prevertebral Infection ......................................................................................................................................... 156<br />

PHARYNGITIS .......................................................................................................................................................... 158<br />

Trauma ...................................................................................................................................................................... 158<br />

Irritant Inhalant ........................................................................................................................................................ 158<br />

Viruses ...................................................................................................................................................................... 158<br />

Infectious Mononucleosis ................................................................................................................................... 158<br />

Herpes Simplex Pharyngitis ................................................................................................................................ 158<br />

Bacteria ..................................................................................................................................................................... 159<br />

Group A j3-Hemolytic Streptococci ..................................................................................................................... 159<br />

Group C and G Streptococci .............................................................................................................................. 159<br />

Diphtheria .......................................................................................................................................................... 160<br />

Gonorrhea .......................................................................................................................................................... 1 60<br />

Fungi ......................................................................................................................................................................... 160<br />

DENTAL EMERGENCIES ............................................................................................................................................ 161<br />

Neuroanatomy of the Face ........................................................................................................................................ 161<br />

Facial and Oral Anesthesia ........................................................................................................................................ 162<br />

Anatomy of a Tooth ................................................................................................................................................... 162<br />

Dental Emergencies .................................................................................................................................................. 163<br />

Trauma ............................................................................................................................................................... 163<br />

Hemorrhage ....................................................................................................................................................... 1 63<br />

Orofacial Pain .................................................................................................................................................... 164<br />

Systemic Diseases with Oral Manifestations ....................................................................................................... 165<br />

Infections .................................................................................................................................................... 165<br />

Automimmune/lnflammatory States ............................................................................................................. 165<br />

Toxic/Metabolic States ................................................................................................................................. 165<br />

Granulomatous Diseases ............................................................................................................................. 165<br />

Benign Tumors and Tumor-Like Lesions ....................................................................................................... 165<br />

Blood Dyscrasias ......................................................................................................................................... 165<br />

HIV/AIDS .................................................................................................................................................... 165<br />

99


HEAD, EAR, EYE, NOSE, AND THROAT DISORDERS: SELF-ASSESSMENT QUESTIONS<br />

HEAD, EAR, EYE, NOSE, AND THROAT DISORDERS:<br />

SELF-ASSESSMENT QUESTIONS<br />

1. All of the following are true of peripheral vertigo except:<br />

(a) Onset is gradual.<br />

(b) Nystagmus may be horizontal or horizontorotary in direction.<br />

(c) It may be associated with nausea, vomiting, hearing loss, and tinnitus.<br />

(d) Episodes generally self-limited.<br />

2. The most reliable sign of acute otitis media is:<br />

(a) A tympanic membrane that is erythematous<br />

(b) The presence of fever<br />

(c) Loss of mobility of the tympanic membrane on pneumatic otoscopy<br />

(d) The presence of a scarred and retracted tympanic membrane<br />

3. Which of the following statements regarding malignant otitis externa is true?<br />

(a) It is a common complication of otitis externa that afflicts otherwise healthy patients.<br />

(b) It is treated on an outpatient basis with oral antibiotics.<br />

(c) It is caused by Pseudomonas.<br />

(d) Patients with this disease process are generally afebrile and experience little pain.<br />

4. Which of the following is least characteristic of croup?<br />

(a) Barking cough<br />

(b) High fever<br />

(c) Preceding upper respiratory infection<br />

(d) Insidious onset<br />

5. All of the following are true of temporal arteritis except:<br />

(a) It is associated with a markedly increased sedimentation rate.<br />

(b) It may present as sudden, painless, unilateral loss of vision.<br />

(c) It is treated with steroids.<br />

(d) It is a nonsegmental vasculitis.<br />

6. Appropriate antibiotic therapy of pharyngitis caused by group A ~-hemolytic streptococci can limit or prevent all of<br />

the following complications except:<br />

(a) Rheumatic fever<br />

(b) Glomerulonephritis<br />

(c) Pharyngeal space infections<br />

(d) Spread of the infection to others<br />

7.<br />

____ is/are the most common cause(s) of pharyngitis.<br />

(a) Group A ~-hemolytic streptococcus<br />

(b) Mycoplasma<br />

(c) Viruses<br />

(d) Gonorrhea<br />

8. Which statement is true regarding group A ~-hemolytic streptococcus pharyngitis?<br />

(a) Accurate diagnosis can be made on the basis of clinical findings alone.<br />

(b) It commonly affects children


HEAD, EAR, EYE, NOSE, AND THROAT DISORDERS: SELF-ASSESSMENT QUESTIONS<br />

10. Al I of the following statements regarding Ellis II fractures are true except:<br />

(a) They involve both the enamel and dentin.<br />

(b) They are associated with hot and cold sensitivity.<br />

(c) Bleeding from the tooth is characteristic.<br />

(d) Dental follow-up within 24 hours is recommended.<br />

11. Which of the following descriptive statements is most consistent with alveolar osteitis?<br />

(a) It occurs within 24 hours of extraction and responds to oral analgesics.<br />

(b) It is seen several days after extraction, after an initial pain-free time interval, and is not relieved by oral analgesics.<br />

(c) It is associated with dental caries and treated by tooth extraction.<br />

(d) It is a periodontal lesion in which bacteria invade non-necrotic tissue.<br />

12. Which is true regarding treatment of avulsed teeth?<br />

(a) The tooth should be wiped clean before replanting.<br />

(b) Primary teeth should be replaced immediately.<br />

(c) Handling the tooth by the root is preferred.<br />

(d) Hank's solution and milk are better transport solutions than tap water.<br />

13. The most common arterial source of posterior epistaxis is:<br />

(a)<br />

Sphenopalatine artery<br />

(b) Anterior ethmoidal artery<br />

(c) Posterior ethmoidal artery<br />

(d) Kiesselbach plexus<br />

14. All the following statements regarding posterior epistaxis are accurate except:<br />

(a) It is less common than anterior epistaxis.<br />

(b) It most commonly occurs in children and young adults.<br />

(c) Placement of a posterior pack can result in hypoxia and hypercarbia.<br />

(d) Patients treated with a posterior pack should be admitted.<br />

15. The treatment of choice for simple group A ~-hemolytic streptococcus is:<br />

(a) Penicillin<br />

(b) Erythromycin<br />

(c) Cephalosporins<br />

(d) TMP-SMX<br />

16. The following statements regarding glomerulonephritis are accurate except:<br />

(a)<br />

It is a suppurative complication of group A ~-hemolytic streptococcus infection.<br />

(b) It can result from either pharyngeal or cutaneous infection with group A ~-hemolytic streptococci.<br />

(c)<br />

Patients usually present after a latent period of 1 .5-3 weeks.<br />

(d) The most useful laboratory test for making the diagnosis is the urinalysis.<br />

17. The following statements regarding rheumatic fever are accurate except:<br />

(a) The Jones Criteria (revised) are used to establish the diagnosis.<br />

(b) Evidence of a preceding group A ~-hemolytic streptococcus infection plus the presence of one major and one<br />

minor criteria make the diagnosis highly probable.<br />

(c) Patients usually present with severe migratory joint pain.<br />

(d) Morbidity is most closely related to the development of carditis and valvular damage.<br />

18. Bacterial tracheitis is most commonly caused by ___ _<br />

(a)<br />

Haemophi/us influenzae<br />

(b) Streptococcus pneumoniae<br />

(c) Staphylococcus aureus<br />

(d) Group A ~-hemolytic streptococci<br />

101


HEAD, EAR, EYE, NOSE, AND THROAT DISORDERS: SELF-ASSESSMENT QUESTIONS<br />

19. A mother brings in her 3-year-old child for evaluation. She states that he was playing on the floor while she was<br />

preparing dinner and he suddenly started coughing. As you evaluate the child, you note that he is playful and in no<br />

acute distress. Which of the following statements is most accurate?<br />

(a) You should reassure the mother and send the child home; further evaluation is unnecessary, because foreign<br />

body aspiration is unlikely in this scenario.<br />

(b) You should obtain further history and perform PA and lateral chest radiographs; normal findings on these<br />

radiographs exclude the possibility of foreign body aspiration.<br />

(c) You should obtain further history and perform PA, lateral, and (possibly) bilateral decubitus chest radiographs;<br />

if these radiographs are negative (and your suspicion for foreign body aspiration remains high), bronchoscopy<br />

should be arranged.<br />

(d) You should immediately arrange for diagnostic bronchoscopy in every child with this presentation.<br />

20. Which of the following is not true regarding peritonsillar abscesses?<br />

(a) It most commonly occurs as a complication of suppurative tonsillitis.<br />

(b) Needle aspiration followed by incision and drainage provides sufficient initial drainage for most abscesses.<br />

(c)<br />

It is most common in infants.<br />

(d) The causative organism(s) is usually polymicrobial.<br />

21. A child presents with complaint of decreased hearing after being slapped on the ear by his brother. Examination<br />

reveals a small perforation of the tympanic membrane and a conductive hearing loss but is otherwise entirely<br />

normal. The child is acting normally, appears to be comfortable, and has normal vital signs. The most appropriate<br />

treatment is:<br />

(a)<br />

Emergent ENT referral for possible surgical repair.<br />

(b) A prescription for an oral antibiotic and referral for follow-up with ENT in 24 hours<br />

(c) A prescription for a topical antibiotic and referral for follow-up with ENT in 1 week.<br />

(d) Recommend the ear be kept dry, and refer to ENT on a nonurgent basis.<br />

22. Which of the following statements regarding the diagnosis of sinusitis is most accurate?<br />

(a)<br />

It requires diagnostic imaging in most instances.<br />

(b) Normal findings on plain radiographs (Water, Caldwell, submental-vertex, and lateral views) excludes the<br />

diagnosis.<br />

(c)<br />

Plain films are the most useful in diagnosing sphenoid and ethmoid sinusitis.<br />

(d) The "gold standard" for diagnosing sinus disease and its complications is CT of the sinuses.<br />

23. Which of the following statements regarding hairy leukoplakia is false?<br />

(a) It is a white plaque-like lesion with hairy projections.<br />

(b) It is usually asymptomatic.<br />

(c)<br />

It is easily removed with a tongue blade.<br />

(d) It is one of the most common virally induced, oral diseases of HIV-infected individuals, with a prevalance as<br />

high as 25%-50%.<br />

24. The initial study of choice for confirming the presence of a retropharyngeal abscess is a soft-tissue lateral radiograph<br />

of the neck. To avoid obtaining a false-positive result, this film must be taken:<br />

(a)<br />

During expiration with the neck in slight extension<br />

(b) During expiration with the neck in slight flexion<br />

(c)<br />

During inspiration with the neck in slight extension<br />

(d) During inspiration with the neck in slight flexion<br />

25. Croup (laryngotracheobronchitis) is most commonly caused by:<br />

(a)<br />

Respiratory syncytial virus<br />

(b) Parainfluenza virus<br />

(c) Adenovirus<br />

(d) Influenza virus<br />

102


HEAD, EAR, EYE, NOSE, AND THROAT DISORDERS: SELF-ASSESSMENT QUESTIONS<br />

26. A 3-year-old is brought in for evaluation of sore throat, fever, and refusal to eat. The child's voice is muffled.<br />

Examination reveals a unilateral bulging of the posterior pharyngeal wall, tender anterior cervical adenopathy, and<br />

temperature of 102°F (39°C). The most likely diagnosis is:<br />

(a) Retropharyngeal abscess<br />

(b) Peritonsillar abscess<br />

(c) Ludwig angina<br />

(d) Masticator space abscess<br />

27. An infant who was delivered at home and received no perinatal care is brought in 12 days after birth for evaluation<br />

of a purulent eye discharge and cough. Examination reveals diffuse conjunctiva! injection and normal pupillary<br />

reactivity. The most likely etiologic agent is:<br />

(a) Staphylococcus aureus<br />

(b) Adenovirus<br />

(c) Chlamydia trachomatis<br />

(d) Neisseria gonorrhoeae<br />

28. A sexually active 20-year-old man presents with redness and irritation of his right eye with an associated discharge.<br />

Findings include diffuse conjunctiva! injection and a copious purulent ocular discharge. Visual acuity and pupillary<br />

reactivity are normal. The most likely etiologic agent is:<br />

(a)<br />

Neisseria gonorrhoeae<br />

(b) Herpes simplex<br />

(c) Adenovirus<br />

(d) Vernal conjunctivitis<br />

29. The most appropriate therapy for a patient with conjunctivitis and corneal ulceration due to Neisseria gonorrhoeae is:<br />

(a) Discharge to home with topical erythromycin or tetracycline ophthalmic ointment<br />

(b) Discharge to home with tobramycin ophthalmic ointment or drops<br />

(c) Hospital admission with administration of IM or IV ceftriaxone<br />

(d) Hospital admission with administration of IV ceftriaxone, topical erythromycin, oral erythromycin (in children)<br />

or doxycycline (in adults), and saline irrigation<br />

30. Which of the following entities is most characteristically associated with painless decrease (or loss) of vision?<br />

(a) Central retinal artery occlusion<br />

(b) Acute iritis<br />

(c) Acute narrow angle closure glaucoma<br />

(d) Optic neuritis<br />

31. Which of the following is associated with pain and loss of central vision only?<br />

(a) Primary open angle closure glaucoma<br />

(b) Acute iritis<br />

(c) Acute anterior uveitis<br />

(d) Optic neuritis<br />

32. A patient presents with eye pain, slight blurring of vision and severe photophobia. Examination reveals a red eye<br />

with a ciliary flush, a constricted pupil, and a clear cornea. Slit lamp examination reveals flare and cells in the<br />

anterior chamber. The most likely diagnosis is:<br />

(a)<br />

Acute angle closure glaucoma<br />

(b) Retrobulbar neuritis<br />

(c) Acute iritis<br />

(d) Primary open angle closure glaucoma<br />

33. Which of the following is most appropriate in the treatment of acute traumatic iritis?<br />

(a)<br />

Oral steroid therapy<br />

(b) Topical steroids (in consultation with an ophthalmologist)<br />

(c) Antibiotic ointment or drops<br />

(d) Epsilon aminocaproic acid<br />

103


HEAD, EAR, EYE, NOSE, AND THROAT DISORDERS: SELF-ASSESSMENT QUESTIONS<br />

34. The mechanism of action by which timolol and acetazolamide work in treatment of acute narrow angle glaucoma is:<br />

(a)<br />

They pull the iris back from its anterior position, thereby opening the angle and allowing escape of aqueous humor.<br />

(b) They decrease the secretion of aqueous humor by the ciliary body.<br />

(c) They cause pupillary block by dilating the iris and allowing the egress of aqueous humor.<br />

(d) None of the above<br />

35. A 77-year-old woman presents with a headache and loss of vision in her left eye. Visual acuity is measured as bare<br />

light perception in this eye. The cornea is clear, pupillary reactivity is normal, and the sedimentation rate is 60. The<br />

most appropriate therapy for this patient is:<br />

(a) Topical pilocarpine solution<br />

(b) A topical steroid agent<br />

(c) Ophthalmology referral for follow-up in 3-4 days<br />

(d) High-dose corticosteroids, preferably IV<br />

36. Which of the following is a common sign of a blowout fracture of the orbit with entrapment?<br />

(a) Ptosis<br />

(b) Exophthalmos<br />

(c) Afferent pupil defect<br />

(d) Pain and diplopia on upward gaze<br />

37. A patient presents with a laceration to his upper eyelid. On evaluation, you note a deep horizontal laceration<br />

toward the middle of the upper lid and associated ptosis. The globe itself does not appear to be injured. The most<br />

likely associated injury is damage to the:<br />

(a)<br />

Lacrimal canaliculi<br />

(b) Orbital septum<br />

(c) Levator muscle/tendon<br />

(d) Canthal tendon<br />

38. The finding most consistent with a ruptured globe:<br />

(a) Teardrop or irregularly shaped pupil<br />

(b) Increased intraocular pressure<br />

(c) Exophthalmos<br />

(d) Diminished intraocular pressure<br />

39. The shortest-acting cycloplegic agent is:<br />

(a)<br />

Cyclopentolate<br />

(b) Tropicamide<br />

(c)<br />

Homatropine<br />

(d) Atropine<br />

40. The topical antibiotic agent associated with the development of skin sensitivity in 10%-15% of patients is:<br />

(a)<br />

Sulfacetamide<br />

(b) Chloramphenicol<br />

(c) Erythromycin<br />

(d) Neomycin<br />

41. Topical ophthalmic drops have color-coded caps to facilitate the identification of the various types of agents. The<br />

color cap associated with miotic agents is:<br />

(a)<br />

Green<br />

(b) Red<br />

(c) Yellow<br />

(d) White<br />

42. The presence of a Marcus-Gunn pupil on examination is most indicative of:<br />

(a) Acute iritis<br />

(b) Optic neuritis<br />

(c) Vitreous hemorrhage<br />

(d) Functional blindness<br />

104


HEAD, EAR, EYE, NOSE, AND THROAT DISORDERS: SELF-ASSESSMENT QUESTIONS<br />

43. A 70-year-old woman presents with obvious signs and symptoms of acute angle closure glaucoma. Her past medical<br />

history is significant for poorly controlled CHF. Which of the following would be inappropriate?<br />

(a)<br />

Pilocarpine 2% solution<br />

(b) Glycerol 50% solution<br />

(c) Timolol 0.5% solution<br />

(d) Acetazolamide<br />

44. Which of the following statements regarding periorbital cellulitis is most accurate?<br />

(a) Children > 10 years old are rarely affected.<br />

(b) Patients present with erythema, warmth, and swelling of one or both eyelids.<br />

(c) Patients complain of pain with ocular movement, and ophthalmoplegia may be present.<br />

(d) Fever is uncommon.<br />

45. Which of the following statements regarding use of a tonometer is most accurate?<br />

(a)<br />

Its use is contraindicated in the presence of a ruptured globe.<br />

(b) lntraocular pressure is always high in acute iritis.<br />

(c) Use of an air-puff tonometer requires direct eye contact.<br />

(d) The Schiotz tonometer gives a direct reading in millimeters of mercury.<br />

46. Which of the following conditions does not classically produce a decrease in visual acuity?<br />

(a) Traumatic lens dislocation<br />

(b) Retrobulbar hematoma<br />

(c) Periorbital cellulitis<br />

(d) Retrobulbar neuritis<br />

47. A 25-year-old patient presents with a foreign body sensation in his left eye, photophobia, and tearing. Evaluation reveals<br />

a visual acuity of 20/30, diffusely reddened eye, decreased corneal sensation, and a dendritic lesion on fluorescein<br />

staining. Which of the following could produce rapid worsening of his condition and should not be prescribed?<br />

(a) A topical antibiotic<br />

(b) A topical steroid<br />

(c) A topical antiviral agent (in consultation with an ophthalmologist)<br />

(d) A mydriatic agent<br />

48. A patient who uses extended-wear soft contact lenses presents with complaints of photophobia, eye pain, blurred<br />

vision, and a purulent discharge. Examination reveals a corneal ulcer. The causative agent in this setting is most likely:<br />

(a) Neisseria gonorrhoeae<br />

(b) Klebsiella<br />

(c) Adenovirus<br />

(d) Pseudomonas<br />

49. Immediate ophthalmology consult, hospital admission and treatment is mandatory for which of the following conditions?<br />

(a) Orbital (postseptal) cellulitis<br />

(b) Herpes zoster ophthalmitis<br />

(c) Corneal ulcers<br />

(d) Acute iritis<br />

50. Regarding chemical burns of the eye, which of the following statements is most accurate?<br />

(a) Alkali burns are usually worse than acid burns.<br />

(b) Alkali burns produce a coagulation necrosis.<br />

(c) A neutralizing agent should be used to irrigate the eye.<br />

(d) Irrigation can be discontinued after 30 minutes, once the pH of the conjunctiva! fornix is neutral at 8.5.<br />

51. The most appropriate management sequence for an acid or alkali burn of the eye is:<br />

(a)<br />

Detailed eye examination, assessment of visual acuity, copious irrigation, pH assessment<br />

(b) Evaluation of visual acuity, pH assessment, copious irrigation, detailed eye examination<br />

(c) pH assessment, copious irrigation, evaluation of visual acuity, detailed eye examination<br />

(d) Copious irrigation, pH assessment, evaluation of visual acuity, detailed eye examination<br />

105


HEAD, EAR, EYE, NOSE, AND THROAT DISORDERS: SELF-ASSESSMENT QUESTIONS<br />

52. A 42-year-old man presents with painful swelling below the inner aspect of his right eye of 1 day duration.<br />

Examination reveals localized erythematous swelling and tearing. His visual acuity is 20/20, the remainder of his<br />

eye examination is unremarkable, and he otherwise appears well. The most appropriate treatment is:<br />

(a)<br />

Immediate incision and drainage<br />

(b) Admission for parenteral antibiotics<br />

(c) A topical antibiotic ointment<br />

(d) A broad-spectrum oral antibiotic and warm compresses<br />

53. A patient with AIDS and a CD4 count


HEAD, EAR, EYE, NOSE, AND THROAT DISORDERS<br />

OTOLOGIC EMERGENCIES<br />

I. ANATOMY<br />

A. Sensory supply to the ear<br />

1. Tympanic membrane: auriculotemporal (V3), cranial nerves IX and X<br />

2. Auricle: auriculotemporal (V3), cranial nerves XII and X, lesser occipital, greater auricular<br />

3. External auditory canal: auricolutemporal (V3), cranial nerves VII, IX, and X<br />

B. Auditory transmission<br />

Sound waves enter ear canal_,. tympanic membrane_,. ossicles _,. perilymph _,. endolymph _,. organ of Corti _,.<br />

spiral ganglion_,. cochlear nulei _,. temporal lobes<br />

II. INFECTIONS<br />

A. Acute otitis externa (swimmer's ear)<br />

1 . Pseudomonas aeruginosa and Staphylococcus aureus are responsible for most cases of otitis externa. Other<br />

pathogens include Proteus, Streptococcus, and fungi (usually Aspergillus).<br />

2. A major predisposing factor is exposure to a warm, moist environment, so most cases occur in the summer<br />

months and in patients exposed to tropical environments; foreign body trauma and water exposure are<br />

obvious risk factors.<br />

3. Clinical presentation<br />

a. Itching: may be intense with severe fungal infections<br />

b. Pain: may be intense, constant, and aggravated by pulling on the ear or pressing on the tragus<br />

c. Sense of fullness in the ear<br />

d. Purulent discharge<br />

e. External auditory canal is often edematous and erythematous with significant debris.<br />

f. If the tympanic membrane can be visualized, it is sometimes red, thick, and covered with flat vesicles or<br />

areas of desquamating epithelium.<br />

4. Differential diagnosis<br />

a. Malignant otitis externa<br />

b. Suppurative otitis media with tympanic membrane perforation<br />

c. Contact dermatitis (eg, neomycin)<br />

d. Eczema<br />

e. Psoriasis<br />

f. Temporal bone/external auditory canal malignancy<br />

5. Management<br />

a. Cleanse the ear canal of debris using suction, irrigation, or gentle curettage; this is the most important<br />

part of therapy.<br />

b. Use topical agent<br />

(1) Mild infections: otic hydrocortisone/acetic acid (rarely used)<br />

(2) Moderate infections: otic quinolones with steroids (eg, otic ciprofloxacin/dexamethasone)<br />

(3) Avoid otic aminoglycosides because of ototoxicity.<br />

(4) If significant edema is present, a wick should be inserted in the external canal, moistened with the<br />

desired medication, and left in place for 24-48 hours. Patient must return for removal.<br />

c. Advise the patient to avoid getting water in the ear for 2-3 weeks.<br />

d. Cellulitis is often present in severe cases; a systemic antibiotic (eg, dicloxacillin or amoxacillin/<br />

clavulanate) is required.<br />

B. Malignant (necrotizing) otitis externa<br />

1. Complication of otitis externa that is associated with a high mortality rate<br />

2. Epidemiology<br />

107


HEAD, EAR, EYE, NOSE, AND THROAT DISORDERS<br />

a. Occurs primarily in adults with diabetes<br />

b. Also seen in debilitated and immunocompromised patients<br />

3. Etiology: a Pseudomonas osteomyelitis of the underlying bone of the external canal<br />

4. Clinical presentation<br />

a. Distinguishing features include fever, excruciating pain, presence of friable granulation tissue in the<br />

external auditory canal, and edema and erythema of the pinna and periauricular tissues.<br />

b. Cranial nerve palsies and trismus may also be seen.<br />

5. Diagnostic evaluation: CT is the most appropriate emergency department imaging study to evaluate for<br />

osteomyelitis and determine the extent of disease.<br />

6. Management<br />

a. Immediate otolaryngology consult<br />

b. Hospitalization for IV antibiotics (fluoroquinolones or antipseudomonal cephalosporins) and possible<br />

surgical debridement<br />

C. Posttraumatic perichondritis<br />

1. An infection of the auricular cartilage, usually due to a complication of auricular trauma<br />

a. Auricular hematoma ("cauliflower ear") is the most common cause.<br />

b. Also caused by ear piercing, or after ear surgery<br />

2. Clinical presentation<br />

a. Ear pain, swelling, and fever are typical.<br />

b. Examination reveals erythema and warmth/swelling of the outer ear (usually excluding the lobule).<br />

3. Management<br />

a. Consult with an otolaryngologist<br />

b. Administration of parenteral antibiotics effective against Pseudomonas, Proteus, and Staphylococcus<br />

D. Acute otitis media<br />

1. Epidemiology<br />

a. Children 6 months to 6 years old are most often affected; incidence peaks at 6-18 months of age.<br />

b. More common in the winter and spring months; frequently occurs in association with a viral upper<br />

respiratory infection<br />

2. Pathophysiology<br />

a. Eustachian tube dysfunction plays a central role in pathogenesis.<br />

b. Eustachian tube dysfunction - negative middle ear pressure and retention of secretions - colonization<br />

(usually bacterial, although antibiotics are not always necessary)<br />

3. Etiology - the most common bacterial pathogens:<br />

a. Streptococcus pneumoniae (most common in all age groups), Haemophilus influenzae, Moraxella<br />

catarrhalis, Streptococcus pyogenes (less common)<br />

b. Staphylococcus aureus, Group B streptococcus, and gram-negative enteric bacilli may be seen as<br />

pathogens in the neonatal period.<br />

4. Clinical presentation<br />

a. Represents a continuum of an illness that begins as an asymptomatic otitis media with effusion to acute<br />

otitis media with effusion plus signs and symptoms of an infection<br />

b. History varies from irritability, poor feeding, and ear-pulling in infants, to otalgia and hearing loss in older<br />

children. Upper respiratory symptoms are often present.<br />

c. Physical examination<br />

(1) Cerumen often obscures visualization and prevents initial diagnosis; if present, it must be removed.<br />

(2) The tympanic membrane is usually red or cloudy and may be full or bulging (bony landmarks are<br />

difficult to discern). Otorrhea may be present if spontaneous perforation has occurred. However,<br />

the most reliable sign of acute otitis media is decreased mobility of the tympanic membrane on<br />

insufflation.<br />

(3) Fever is present in only one-third of patients.<br />

5. Management<br />

a. "Wait and see prescription" for treatment of acute otitis media in children >2 years old, unilateral<br />

involvement, and no otorrhea is becoming the standard of care. Parents are asked not to fill the<br />

antibiotic prescription for 72 hours and only if the child is no better or worsening. This approach has<br />

resulted in reduced antibiotic use in children.<br />

108


HEAD, EAR, EYE, NOSE, AND THROAT DISORDERS<br />

b. First-line antibiotics<br />

(1) Amoxicillin 80-90 mg/kg/day; despite increasing resistance, it is still considered initial agent of choice.<br />

(2) Azithromycin, clarithromycin, and clindamycin are alternative agents for penicillin-allergic patients;<br />

they do not cover H influenzae. Cefdinir 14 mg/kg/day if penicillin allergy is not an immediate type<br />

1 hypersensitivity.<br />

c. Second-line agents are usually reserved for treatment failures or for patients having received antibiotics<br />

within the previous 30 days. A broad-spectrum antibiotic that provides coverage of ~-lactamaseproducing<br />

species should be selected; choose one of the following:<br />

(1) Amoxicillin-clavulanate if recent antibiotic exposure or suspicion of H influenzae as a cause<br />

(coexistent purulent conjunctivitis is a sign of H influenzae as cause)<br />

(2) Ceftriaxone (with possible repeat dosing depending on response)<br />

d. Patients should be advised to return in 48-72 hours if signs and symptoms do not improve and<br />

immediately if temperature increases, irritability worsens, or lethargy develops.<br />

e. The possibility of systemic infection should be considered in infants with acute otitis media (especially<br />

those


HEAD, EAR, EYE, NOSE, AND THROAT DISORDERS<br />

D. Diagnostic evaluation<br />

1. CT scan of the temporal bone confirms diagnosis and identifies presence of complications.<br />

2. MRI may be considered, especially if intracranial complications are suspected.<br />

E. Management<br />

1. Analgesia<br />

2. Parenteral antibiotics: vancomycin (initial choice until cultures available); also consider Pseudomonas coverage<br />

for recurrent infection<br />

3. Immediate ENT consult regarding the possible need for surgical debridement, which may be necessary because<br />

of poor bone penetration of antibiotics<br />

4. Hospitalization<br />

IV. SUDDEN HEARING LOSS<br />

A. Pathophysiology<br />

1. Acute hearing loss may be associated with lesions of the external, middle, or inner ear.<br />

2. Lesions of the external auditory canal, tympanic membrane, middle ear, and ossicles result in conductive<br />

hearing loss, whereas lesions of the cochlea, the auditory nerve, and brainstem auditory pathways result in<br />

sensorineural hearing loss.<br />

3. Sudden sensorineural hearing loss may also occur in the absence of any detectable lesions, presumably from<br />

microvascular insult or occult autoimmune process.<br />

B. Etiology<br />

1. Conductive loss<br />

a. Otitis externa<br />

b. Impacted cerumen or foreign body<br />

c. Tympanic membrane perforation<br />

d. Sclerosis of the tympanic membrane or ossicles<br />

e. Middle ear effusion (otitis media, barotitis media)<br />

2. Sensorineural loss<br />

a. Unilateral<br />

(1) Viral neuritis of the cochlear branch of cranial nerve VI I<br />

(2) Acoustic neuroma<br />

(3) Meniere disease<br />

(4) Temporal bone fracture<br />

(5) Autoimmune hearing loss<br />

(6) Sudden sensorineural hearing loss: idiopathic hearing loss that develops suddenly or over 72 hours<br />

b. Bilateral<br />

(1) Ototoxic drug exposure<br />

(a) Antibiotics (aminoglycosides, erythromycin, vancomycin, antimalarials)<br />

(b) NSAIDs, ASA<br />

(c) Loop diuretics (furosemide, ethacrynic acid)<br />

(d) Chemotherapy<br />

(2) Exposure to loud noise<br />

(3) Autoimmune hearing loss<br />

C. Clinical presentation<br />

1. Physical examination should include a thorough HEENT examination, including the cranial nerves; these<br />

patients may have associated deficits of adjacent cranial nerves, particularly V (corneal reflex) and VI I as well<br />

as tinnitus, vertigo, and dysequilibrium.<br />

2. Otoscopic examination may help distinguish patients with conductive hearing loss (due to cerumen impaction,<br />

tympanic membrane perforation, or the presence of middle ear fluid) from those with sensorineural hearing loss.<br />

3. The Rinne and Weber tests are also useful in distinguishing conductive from sensorineural hearing loss.<br />

110


HEAD, EAR, EYE, NOSE, AND THROAT DISORDERS<br />

a. Rinne test: compares air conduction (AC) versus bone conduction (BC) and is performed by placing the<br />

base of a vibrating 512-Hz tuning fork against the mastoid process. It must be pushed against the mastoid<br />

with some force. When the patient can no longer hear the vibration, the tuning fork tips are placed<br />

adjacent to the external auditory canal of the ear. In patients with normal hearing, the vibrations of the<br />

tuning fork will still be heard (AC>BC).<br />

(1) If there is unilateral conductive loss, vibrations will not be heard (BC>AC).<br />

(2) If there is a unilateral sensorineural loss, test will be normal (AC>BC).<br />

b. Weber test: performed by placing the base of the vibrating tuning fork against the middle of the forehead,<br />

again with some force, and asking the patient in which ear the sound is loudest. Normally, it is equal in<br />

both ears.<br />

(1) If a unilateral conductive hearing loss is present, vibrations will be heard better in the ear with<br />

conductive loss.<br />

(2) If a unilateral sensorineural hearing loss is present, vibration will be heard better in the normal ear.<br />

c. If bilateral sensorineural hearing loss is present, both the Rinne and Weber tests will be normal despite a<br />

bilateral decrease in hearing acuity.<br />

D. Diagnostic evaluation<br />

1. CT of the temporal bone should be obtained if fracture is suspected.<br />

2. MRI with attention to auditory canals should be considered if a tumor is suspected based on the history and<br />

examination.<br />

E. Management<br />

1. Early steroids can save hearing in patients with idiopathic sudden sensorineural hearing loss. Consult with<br />

ENT and initiate steroids as early as possible.<br />

2. Cerumen and foreign body impactions should be removed, and infections (otitis media or externa) treated.<br />

3. Patients with tumors require appropriate ENT or neurosurgical consult.<br />

4. If an ototoxic drug is the presumed cause of hearing loss, it should be discontinued.<br />

V. VERTIGO<br />

A. Definition<br />

1. A sensation of movement of oneself or the environment<br />

2. Most commonly described as a feeling of spinning or of being pulled to one side<br />

B. Epidemiology<br />

1. Vertigo is either peripheral or central in origin.<br />

2. Peripheral vertigo accounts for 85 %-90% of cases and is considered less serious prognostically than central<br />

vertigo, which accounts for 10%-15 % of cases.<br />

3. Peripheral vertigo arises from diseases of the vestibular apparatus or cranial nerve VIII and is usually self-limited.<br />

4. Central vertigo originates from diseases of the CNS (brainstem or cerebellum).<br />

C. Peripheral vertigo<br />

1. Abrupt onset<br />

2. Severe intensity<br />

3. Brief/intermittent duration<br />

4. Worsening with change in position<br />

5. Associated symptoms may include nausea and vomiting, diaphoresis, tinnitus, and hearing loss<br />

6. Nystagmus that is:<br />

a. Fatigable<br />

b. Suppressed by visual fixation<br />

c. Unidirectional: horizontal or horizontorotary (never vertical)<br />

d. No associated neurologic abnormalities<br />

D. Central vertigo<br />

1. Insidious onset<br />

2. Mild intensity<br />

3. Prolonged/continuous duration<br />

111


HEAD, EAR, EYE, NOSE, AND THROAT DISORDERS<br />

4. Little or no effect with change in position<br />

5. Associated symptoms may include headache, diplopia, dysarthria, dysphagia, ataxia, facial numbness, and<br />

hemiparesis.<br />

6. Nystagmus that is:<br />

a. Nonfatigable<br />

b. Not suppressed by visual fixation<br />

c. Multidirectional (changes direction with changes in position): horizontal, rotary, or vertical<br />

7. Not usually associated with hearing loss<br />

VI. TRAUMA<br />

A. Lacerations of the auricle<br />

1. Must be repaired carefully to avoid cosmetic defects<br />

2. Primary goal is to reapproximate the cartilage (with absorbable sutures) and then cover the cartilage with skin.<br />

B. Hematomas of the auricle<br />

1. Should be drained and dressed with a compressive dressing that maintains normal ear contours; this will<br />

prevent formation of "cauliflower ear."<br />

2. Patients should be started on oral antibiotics with cartilage penetration (eg, quinolones) and reassessed in 24<br />

hours for reaccumulation of blood, which (if present) will require repeat drainage and dressing replacement.<br />

C. Frostbite of the auricle<br />

1. Should be treated by rapid rewarming<br />

2. Use warm water (40°-42°C) irrigation.<br />

D. Foreign body in the ear-requires urgent referral to an ENT specialist if:<br />

1. It cannot be safely removed without damage to the tympanic membrane.<br />

2. It is expanding organic material (eg, bean).<br />

3. There is any chance it is a battery.<br />

E. Perforation of the tympanic membrane<br />

1. Pathophysiology: a penetrating object, loud noise, infection, lightning strike, or rapid changes in pressure, eg,<br />

scuba diving accidents<br />

2. Clinical presentation<br />

a. Otoscopic examination usually reveals the tear. Immobility of the tympanic membrane on bulb insufflation<br />

or autoi nsufflation confirms it.<br />

b. Acute perforations have irregular borders, sometimes with blood.<br />

c. Chronic perforations have smooth margins without blood.<br />

d. Patients with associated symptoms of complete hearing loss, nausea, vomiting, vertigo, or facial palsy<br />

need immediate ENT referral. These symptoms suggest the presence of concurrent injury to the ossicles,<br />

labyrinth, or temporal bone.<br />

3. Management<br />

a. Most perforations generally heal spontaneously (especially traumatic tears), and follow-up with ENT can be<br />

on a less-urgent basis. In either case, the ear should be kept dry.<br />

b. If coexisting otitis media or externa is present, a topical otic antibiotic suspension should be prescribed.<br />

F. Severe blunt injury to the ear<br />

1. May be associated with fracture of the temporal bone<br />

2. May cause hearing loss or facial nerve injury<br />

3. Evaluation typically involves temporal bone CT<br />

112


HEAD, EAR, EYE, NOSE, AND THROAT DISORDERS<br />

EYE EMERGENCIES<br />

I. CONJUNCTIVITIS<br />

A. Viral conjunctivitis ("pink eye")<br />

1. Viruses, particularly adenoviruses, are the most frequent cause of conjunctivitis.<br />

2. The infection is often unilateral initially but usually spreads quickly (via autoinoculation) to involve both eyes.<br />

3. Up to 50% of patients have constitutional symptoms or a viral prodromal illness consistent with a viral<br />

syndrome (fever, rhinorrhea, myalgias, and preauricular adenopathy).<br />

4. Clinical presentation<br />

a. Patients classically present with redness, itching, and tearing (watery discharge).<br />

b. Typical examination findings include diffuse conjunctiva! injection, a follicular response on the palpebral<br />

(lid) conjunctiva, a thin watery discharge, and preauricular adenopathy. (Herpes simplex keratitis and<br />

chlamydia! conjunctivitis can also be associated with follicular conjunctivitis and preauricular adenopathy.)<br />

c. Unless an associated keratitis is present, the cornea is clear; visual acuity and pupillary reactivity are normal.<br />

Courtesy of David Effron, MD, FACEP<br />

5. Infection with adenovirus types 8 and 19 (and others)<br />

a. Epidemic keratoconjunctivitis (a highly contagious eye infection)<br />

b. Clinical presentation: itchy irritated eyes - tender preauricular nodes (a few days later) - painful keratitis<br />

with severe photophobia and decreased visual acuity (1 week later)<br />

c. Diagnostic evaluation: fluorescein staining of the cornea reveals multiple tiny pinpoint dots.<br />

6. Management<br />

a. Antibiotic therapy: because the symptoms and clinical appearance of viral conjunctivitis are usually<br />

inadequate to distinguish it from bacterial conjunctivitis, most patients are treated with a topical antibiotic.<br />

b. Meticulous hygiene (frequent hand washing, separate hand linens), especially with epidemic<br />

keratoconjunctivitis, because patients may be contagious for 2 weeks.<br />

c. Cool compresses and artificial tears<br />

d. Consider a topical vasoconstrictor-antihistamine combination for relief of symptoms.<br />

e. Obtain prompt ophthalmologic consult for patients with evidence of keratitis.<br />

f. Steroids should be strictly avoided.<br />

B. Bacterial conjunctivitis<br />

1. Clinical presentation<br />

a. Patients usually present with redness, a gritty sensation (like a foreign body), and discharge.<br />

b. The discharge is mucopurulent and often results in the eyelids being matted together on awakening.<br />

c. A Gram stain and culture should be obtained in all cases of neonatal conjunctivitis or whenever Neisseria<br />

gonorrhoeae conjunctivitis is suspected (see below), or with severe or worsening symptoms despite treatment.<br />

d. Examination findings include diffuse conjunctiva! injection, mucopurulent discharge, and a clear cornea<br />

that does not stain with fluorescein dye; visual acuity and pupillary responses are normal.<br />

113


HEAD, EAR, EYE, NOSE, AND THROAT DISORDERS<br />

e. Complications of any bacterial conjunctivitis can include corneal involvement such as keratitis, corneal<br />

ulcer formation, or even corneal perforation.<br />

f. Contact lens use must be discontinued until infection resolves. Topical quinolones are first-line antibiotics<br />

in contact lens wearers because of the risk of Pseudomonas infection.<br />

Courtesy of David Effron, MO, FACEP<br />

2. Types of bacterial conjunctivitis important for this examination<br />

a. Staphylococcus aureus, Streptococcus pneumonia, and Haemophilus influenzae conjunctivitis<br />

(1) Onset is acute.<br />

(2) The discharge is typically mucopurulent in nature.<br />

(3) White ulcers may be seen at the limbus ("marginal ulcers") as a result of an allergic reaction to the<br />

staphylococcal toxin.<br />

(4) There is no central corneal staining with fluorescein dye and cobalt blue light.<br />

(5) Management<br />

(a) Topical antibiotics: quinolones, aminoglycosides, or trimethoprim-polymyxin for 5-7 days<br />

(b) Good hygiene (frequent hand washing, separate hand linens)<br />

(c) Discontinuation of contact lens wear (Contact lens-related conjunctivitis should be treated with<br />

an aminoglycoside or quinolone to cover Pseudomonas.)<br />

(d) Ophthalmologic follow-up in 2-3 days to exclude subsequent corneal involvement<br />

b. N gonorrhoeae conjunctivitis (an ocular emergency)<br />

(1) An extremely aggressive form of conjunctivitis that can ulcerate and perforate an intact cornea within<br />

hours or days<br />

(2) Most commonly seen in newborns (ophthalmia neonatorum), typically in the first 3 days of life; also<br />

seen occasionally in sexually active adults<br />

(3) Incubation period is 2-5 days; onset is hyperacute.<br />

(4) Discharge is characteristically hyperpurulent and an important diagnostic clue.<br />

(5) Diagnostic evaluation: culture (obtained with calcium alginate swab of lower conjunctiva and plated<br />

on sheep blood agar is more sensitive than Gram stain, and should be obtained in all neonatal cases<br />

and when N gonorrhoeae conjunctivitis is suspected)<br />

(6) Management<br />

(a)<br />

Parenteral antibiotic coverage for gonorrhea: IM or IV ceftriaxone<br />

(b) Oral and topical antibiotic coverage for possible concomitant Chlamydia<br />

i. Oral erythromycin (newborns and children), oral doxycycline or azithromycin (adults)<br />

ii. Topical erythromycin<br />

(c) Frequent ocular irrigation with a saline solution<br />

(d) Immediate ophthalmology consult and hospital admission<br />

(e)<br />

Evaluation and treatment of sexual partners<br />

C. Chlamydia! (inclusion) conjunctivitis<br />

1. Leading cause of preventable blindness worldwide<br />

2. Caused by an obligate intracellular parasite that produces a chronic conjunctivitis with scarring of the cornea<br />

and underside of the lids (Arlt lines)<br />

114


HEAD, EAR, EYE, NOSE, AND THROAT DISORDERS<br />

3. Occurs in newborns (ophthalmia neonatorum), in whom an associated chlamydia! pneumonia may also occur;<br />

also seen in sexually active adults<br />

4. Clinical presentation: signs of infection occur 5-14 days after inoculation/birth (compared with only 2-5 days<br />

with gonorrhea).<br />

5. Diagnostic evaluation: Gram stain may not be helpful (lack of organisms), but an immunofluorescent antibody<br />

screen should be positive.<br />

6. Management<br />

a. Systemic and topical antibiotic therapy (nearly half of infants will have concomitant chlamydia! nasopharyngitis)<br />

(1) Oral erythromycin (children and newborns); oral doxycycline or azithromycin (adults), and<br />

(2) Topical erythromycin<br />

b. Evaluation and treatment of sexual partners<br />

D. Vernal conjunctivitis<br />

1. One of several types of allergic conjunctivitis<br />

2. Most commonly affects male children; a family history of atopy is common.<br />

3. A recurrent seasonal inflammation; occurs in the warmer months of the year<br />

4. Characterized by intense itching, burning photophobia, chemosis (edema of the bulbar conjunctiva), bilateral<br />

lid edema, cobblestone papillae under the upper lid, and a stringy mucoid discharge<br />

5. Management<br />

a. A topical antihistamine preparation (eg, levocabastine) or a topical decongestant/antihistamine<br />

combination (eg, naphazoline/pheniramine)<br />

b. A topical mast cell stabilizer (eg, lodoxamide tromethamine or cromolyn sodium)<br />

c. Cool compresses<br />

d. If bacterial coinfection is suspected or the cause of the conjunctivitis is unclear, a topical antibiotic agent<br />

should also be prescribed.<br />

6. A potential complication is a "shield ulcer" produced from the irritated papillae and discharge; if the ulcer<br />

becomes infected or scarred, loss of vision may occur.<br />

II. INFLAMMATORY DISORDERS OF THE EYELIDS AND CORNEA<br />

A. Anterior blepharitis<br />

1. A chronic and frequently bilateral inflammation of the I id margins due to a staphylococcal infection and/or<br />

seborrheic dermatitis; itching and lid margin irritation are common symptoms.<br />

2. Examination reveals crusty, scaly lid margins that may be erythematous and slightly swollen.<br />

Courtesy of David Effron, MO, FACE?<br />

3. Management<br />

a. Warm compresses and gentle scrubbing of the eyelids and lashes with baby shampoo (to remove debris),<br />

followed by application of a topical antibiotic ointment (erythromycin or bacitracin) to the eyelid margins<br />

daily for 4-6 weeks; warm compresses and artificial tears are also helpful.<br />

b. If seborrheic dermatitis of the scalp and eyebrows is present, it should be treated with selenium sulfide<br />

shampoo.<br />

c. Cases resistant to these measures can be treated with oral tetracycline or doxycycline for 2-3 weeks.<br />

11 5


HEAD, EAR, EYE, NOSE, AND THROAT DISORDERS<br />

B. Hordeolum (stye)<br />

1. An acute infection (usually staphylococcal) of the eyelid margin, involving either the meibomian sebaceous<br />

glands, Zeis sebaceous glands, or Moll apocrine sweat glands<br />

2. Clinical presentation: pain, erythema, focal swelling with a tender nodule or pustule on the eyelid margin<br />

3. Treatment: although styes often point and drain spontaneously, topical antibiotic ointments (bacitracin or<br />

erythromycin) and hot compresses hasten the process.<br />

C. Chalazion<br />

1. A subacute or chronic granulomatous inflammation of a meibomian sebaceous gland in the eyelid<br />

2. Clinical presentation: a nontender bump in the mid portion of the eyelid (as opposed to the lid margin in<br />

hordeolum stye)<br />

Courtesy of David Effron, MD, FACEP<br />

3. Treatment<br />

a. Initial treatment is with a topical antibiotic ointment (bacitracin or erythromycin) and hot compresses.<br />

b. Persistent or chronic recurrent chalazions often respond to the addition of doxycycline for 2-3 weeks.<br />

c. Chalazions that persist for several weeks (or months) despite these measures may eventually require<br />

surgical incision and curettage by an ophthalmologist.<br />

D. Dacryocystitis<br />

1. An acute inflammation of the lacrimal sac due to infection or obstruction of the nasolacrimal duct<br />

2. Most often seen in infants and in adults >40 years old<br />

3. Etiology: most common causative organism is Staphylococcus aureus, followed by Streptococcus epidermidis,<br />

Streptococcus spp, and Haemophilus influenzae.<br />

4. Clinical presentation (be able to identify on pictorial)<br />

a. Patients present with epiphora (overflow of tears onto the face) and an acute, unilateral, painful, red<br />

swelling below the medial canthus (location of the lacrimal sac).<br />

b. Digital pressure applied over this area may result in expression of purulent material from the puncta.<br />

Courtesy of David Effron, MD, FACEP<br />

5. Management<br />

a. Broad-spectrum oral antibiotics (eg, amoxicillin-clavulanate), topical ophthalmic antibiotics, warm compresses,<br />

gentle massage of the lacrimal sac (to empty stagnant tear accumulation), and ophthalmologic referral<br />

b. Signs of systemic illness may require admission.<br />

116


HEAD, EAR, EYE, NOSE, AND THROAT DISORDERS<br />

E. Corneal ulcers (a true ocular emergency)<br />

1. Clinical presentation<br />

a. Patients present with pain, foreign body sensation, photophobia, and tearing.<br />

b. Examination reveals conjunctiva! hyperemia, lid edema, and a localized whitish corneal infiltrate that has<br />

a staining epithelial defect; may also be associated with a hypopyon (accumulation of WBCs in the anterior<br />

chamber).<br />

c. Signs of iritis (eg, miosis, cells, and flare in the anterior chamber on slit-lamp examination) may also be<br />

present.<br />

2. Etiology<br />

Courtesy of David Effron, MD, FACEP<br />

a. Often seen in patients who use soft contact lenses<br />

b. The most common pathogen is Pseudomonas; Staphylococcus and Streptococcus are also common.<br />

c. Ulceration may lead to corneal melting and perforation. (Pseudomonas is particularly virulent, able to<br />

destroy the cornea in :::::24 hours).<br />

3. Management<br />

a. Immediate ophthalmology consult for culture and treatment<br />

b. Hospital admission is rarely needed, unless the patient is a neonate or there is extensive corneal involvement.<br />

c. Topical broad-spectrum antibiotics (quinolones such as ciprofloxacin, gatifloxacin, moxifloxacin, or<br />

ofloxacin every 30-60 minutes) or topical fortified antibiotics (cefazolin plus gentamycin)<br />

d. A topical cycloplegic (cyclopentolate) for associated iritis and ciliary spasm to control pain<br />

e. Discontinuation of contact lens use; no patch<br />

F. Herpes simplex keratitis<br />

1. May be the result of primary or recurrent infection with the herpes simplex virus (usually type I).<br />

2. Skin and mucocutaneous lesions are more common in primary infections, whereas corneal involvement<br />

(keratitis) is more common in recurrent infections.<br />

3. Clinical presentation (be able to identify on pictorial)<br />

a. Patients present with ocular pain, a foreign body sensation, photophobia, and tearing.<br />

b. Visual acuity may be reduced.<br />

c. Examination reveals a diffusely reddened eye, decreased corneal sensation, and preauricular adenopathy.<br />

4. Diagnostic evaluation<br />

a. Slit lamp evaluation of the unstained cornea may reveal localized corneal haziness.<br />

b. Fluorescein staining usually demonstrates the classic dendritic (branching) pattern over the cornea in wellestablished,<br />

recurrent infections; however, in primary infections and early on in recurrent infections, only a<br />

superficial punctate keratitis may be seen.<br />

117


HEAD, EAR, EYE, NOSE, AND THROAT DISORDERS<br />

Courtesy of Dr. Thomas Steinemann<br />

Herpes simplex virus keratitis ghost dendrite<br />

5. Management<br />

a. Prompt ophthalmology consult regarding specific treatment recommendations and follow-up is mandatory;<br />

these infections are aggressive and may lead to corneal destruction resulting in permanent disability.<br />

b. A topical antiviral agent (eg, trifluridine or vidarabine) or oral antiviral agent (eg, acyclovir, valacyclovir,<br />

or famciclovir)<br />

c. A topical cycloplegic (eg, cyclopentolate or homatropine) for pain relief from ciliary muscle spasm if an<br />

associated iritis is present<br />

d. Although topical steroids are occasionally prescribed by ophthalmologists for patients with refractory or<br />

advanced stromal keratitis, they can promote rapid progression of the corneal epithelial infection and are<br />

contraindicated in all other forms of herpetic keratoconjunctivitis; they should not be prescribed by the<br />

emergency physician.<br />

G. Herpes zoster ophthalmicus<br />

1. Results from reactivation of latent varicella zoster virus in the trigeminal ganglion<br />

2. Clinical presentation<br />

a. Patients present with pain, paresthesias, tearing, and a unilateral vesicular eruption in the dermatome<br />

supplied by the ophthalmic branch (V1) of the trigeminal nerve (cranial nerve V).<br />

b. Ocular involvement can range from conjunctivitis to iritis, keratitis, corneal anesthesia, or ocular muscle<br />

palsies; lesions on the tip of the nose (Hutchinson sign) signal nasociliary nerve involvement and a very<br />

high likelihood of ocular lesions.<br />

Courtesy of David Effron, MO, FACEP<br />

3. Management<br />

a. Immediate ophthalmology consult<br />

b. Oral acyclovir, valacyclovir, or famciclovir<br />

c. A topical broad-spectrum antibiotic to prevent secondary infection<br />

d. A topical cycloplegic agent and a topical steroid (in consultation with an ophthalmologist) if iritis is present<br />

e. Oral analgesics as needed<br />

118


HEAD, EAR, EYE, NOSE, AND THROAT DISORDERS<br />

H. Ultraviolet keratitis<br />

1. Prolonged ultraviolet radiation exposure can cause damage to the corneal epithelium.<br />

2. Typical sources of ultraviolet radiation<br />

a. Arc welding (welders' keratitis/flash)<br />

b. Reflected sunlight, especially at high altitude (snow blindness)<br />

c. Artificial sunlight (tanning booths, sun lamps)<br />

3. Clinical presentation<br />

a. Symptoms develop 6-8 hours after exposure and include severe pain, photophobia, a foreign body<br />

sensation, tearing, and blepharospasm.<br />

b. There is decreased visual acuity, conjunctiva! injection, and a diffuse punctate keratopathy (multiple<br />

pinpoint fluorescein-stained dots on the corneal surface seen with cobalt blue light).<br />

Courtesy of Dr. Thomas Steinemann<br />

Severe ultraviolet keratitis<br />

4. Management<br />

a. A topical cycloplegic agent (eg, cyclopentolate or homatropine) for pain control<br />

b. A topical broad-spectrum antibiotic ointment<br />

c. Oral analgesic<br />

d. Topical anesthetics are contraindicated in all cases of keratitis, because they can inhibit corneal healing<br />

and lead to corneal ulceration.<br />

e. Follow-up with ophthalmology in 24 hours<br />

Ill. PERIORBITAL AND ORBITAL CELLULITIS<br />

A. Periorbital (preseptal) cellulitis<br />

1. An infection confined to superficial tissues that are anterior to the orbital septum (the fascia I layer that acts as a<br />

partition separating the eyelids from the orbit)<br />

2. More common and generally less serious than orbital cellulitis<br />

3. Children (particularly those


HEAD, EAR, EYE, NOSE, AND THROAT DISORDERS<br />

Courtesy of David Effron, MD, FACEP<br />

6. Diagnostic evaluation<br />

a. Laboratory studies<br />

(1) Blood cultures<br />

(2) CT of the orbits (thin cuts with coronal reconstruction) if orbital cellulitis cannot be definitively<br />

excluded by clinical evaluation<br />

b. Soft-tissue aspirates for Gram stain and culture are no longer recommended because of risk of septum<br />

perforation with subsequent risk of deeper infection.<br />

7. Management<br />

a. A broad-spectrum antibiotic that covers streptococci and staphylococci; older children and adults with<br />

early/mild periorbital cellulitis can be treated on an outpatient basis with an oral antibiotic (eg, amoxicillinclavulanate)<br />

and follow-up in 24 hours.<br />

b. Adults with more advanced infections and children 20 mm Hg with vision loss, surgical treatment might be necessary in<br />

addition to medications to decrease intraocular pressure.<br />

7. Management<br />

a. Hospital admission<br />

b. Broad-spectrum IV antibiotic covering S aureus, S pneumoniae, H influenzae, and methicillin-resistant 5 aureus<br />

120


HEAD, EAR, EYE, NOSE, AND THROAT DISORDERS<br />

c. Ocular antihypertensives (eg, acetazolamide, timolol) if intraocular pressure >20 mmHg<br />

d. Immediate ophthalmology/ENT consults<br />

e. Canthotomy and cantholysis if strong evidence of orbital compartment syndrome<br />

8. Complications<br />

a. Visual loss<br />

b. Cavernous sinus thrombosis<br />

c. CNS involvement<br />

d. Osteomyelitis<br />

IV. ACUTE EYE PAIN (ASSOCIATED DECREASE IN VISUAL ACUITY)<br />

A. Acute iritis (acute anterior uveitis)<br />

1. Classic clinical scenario<br />

a. The patient presents with a painful red eye, severe photophobia, and blurring of vision. He or she often<br />

has a prior history of iritis or similar undiagnosed pain. Generally, only one eye is affected, and symptoms<br />

develop over time.<br />

b. Physical examination reveals a constricted and sometimes irregular pupil, a ciliary flush<br />

(diffuse reddening of the sclera at the limbus), and a decrease in visual acuity.<br />

Courtesy of David Effron, MD, FACEP<br />

c. The intraocular pressure is variable; it is often decreased (due to paralysis of ciliary body function)<br />

but may be normal or increased (due to inadequate aqueous drainage). Slit-lamp examination is<br />

diagnostic and reveals flare (protein) and cells (leukocytes) in the anterior chamber. Keratitic<br />

precipitates (an accumulation of WBCs on the endothelial surface of the cornea) may also be seen.<br />

Courtesy of Dr. Thomas Steinemann<br />

lritis with keratitic precipitates<br />

121


HEAD, EAR, EYE, NOSE, AND THROAT DISORDERS<br />

2. Diagnostic evaluation<br />

a. Presence of consensual photophobia (the photophobia produced by iritis is both direct and consensual)<br />

b. Presence of pain unrelieved by diagnostic application of a topical anesthetic agent; unlike the superficial<br />

pain associated with a corneal abrasion that is readily relieved by the diagnostic application of a topical<br />

anesthetic agent, the deep-seated ache of iritis is not.<br />

3. Etiology<br />

a. Trauma (occurs 1-4 days after trauma)<br />

b. Seronegative arthritides (idiopathic ankylosing spondylitis, Reiter syndrome, etc)<br />

c. Inflammatory bowel disease<br />

d. Chronic granulomatous conditions (tuberculosis, sarcoidosis, etc)<br />

e. Local infection/ulcers<br />

f. Sexually transmitted diseases (syphilis, gonorrhea)<br />

g. Corneal abrasions and foreign bodies<br />

h. Idiopathic<br />

4. Treatment<br />

a. A long-acting topical cycloplegic (eg, cyclopentolate or homatropine) to provide comfort (by eliminating<br />

ciliary spasm) and prevent formation of posterior synechiae (adhesions between the iris and lens)<br />

b. A topical steroid such as prednisolone acetate 1 % to relieve inflammation; consult with an<br />

ophthalmologist first, and arrange follow-up in 24 hours.<br />

5. Complications: if the pupil is not dilated, iritis may result in formation of posterior synechiae; these adhesions<br />

produce permanent pupillary disfigurement and can lead to secondary glaucoma.<br />

B. Acute angle closure glaucoma (an ocular emergency)<br />

1. Classic clinical scenario<br />

a. Occurs in patients with shallow (narrow) anterior chamber angles. Narrowing of this angle (either<br />

congenital or as a result of aging) results in closer contact between the iris and lens, producing resistance<br />

to flow of aqueous humor from the posterior to the anterior chamber (a relative pupillary block). When<br />

the angle closes completely, flow of the aqueous humor is prevented and the intraocular pressure<br />

increases abruptly.<br />

b. This condition is precipitated by stress or pupillary dilation by medications (parasympatholytics,<br />

sympathomimetics or anticholinergics), inadvertent administration of a topical cycloplegic in a patient<br />

with a shallow anterior chamber, or more commonly by moving from daylight to a darkened room.<br />

c. This is predominantly a condition of the elderly, is more common in farsighted (hyperopic) individuals<br />

and in women, and typically occurs abruptly in patients with no previous history of glaucoma.<br />

2. Clinical presentation<br />

a. History: nausea/vomiting, pain (either localized to the eye/eyebrow or generalized as headache pain),<br />

blurred vision, halos around lights (due to corneal edema), and (occasionally) abdominal pain<br />

b. Physical examination<br />

(1) Decreased visual acuity<br />

(2) A shallow anterior chamber (causing a broad eclipse sign using an oblique flashlight)<br />

(3) A red, congested-looking eye with a fixed, mid-dilated, nonreactive pupil, a hazy cornea and ciliary<br />

(perilimbal) injection<br />

(4) Rock-hard consistency of the globe on gentle palpation<br />

(5) lntraocular pressure >40 mmHg (normal range 0-20 mmHg)<br />

Courtesy of Dr. Thomas Steinemann<br />

122


HEAD, EAR, EYE, NOSE, AND THROAT DISORDERS<br />

3. Management<br />

a. Have the patient lie completely supine (gravity may allow the lens to pull away from the iris).<br />

b. Pharmacologic therapy consists of the sequential administration of several different agents (which act by<br />

various means) to decrease the intraocular pressure.<br />

(1) Topical ~-blockers (eg, timolol), topical a-agonists (eg, apraclonidine), and oral or IV carbonic<br />

anhydrase inhibitors (acetazolamide) decrease the secretion of aqueous humor by the ciliary body.<br />

(2) Topical steroids (eg, prednisolone 1 %) reduce inflammation.<br />

(3) Hyperosmotic agents (eg, mannitol) decrease the volume of fluid in the eye and rapidly lower<br />

intraocular pressure (check hourly). Note that osmotic diuretics have the greatest potential for<br />

serious adverse effects, especially in the setting of cardiovascular or renal disease.<br />

(4) Topical miotics (eg, pilocarpine) pull the iris back from its anterior position, thereby reopening the<br />

angle and allowing egress of aqueous humor; they do not usually have an effect until the intraocular<br />

pressure is 90 minutes<br />

usually results in permanent visual loss.<br />

123


HEAD, EAR, EYE, NOSE, AND THROAT DISORDERS<br />

b. Goal is to restore retinal artery blood flow by dislodging the clot, dilating the artery, and lowering<br />

intraocular pressure.<br />

c. Management<br />

(1) Intermittent digital massage of globe (5 seconds on, 5 seconds off) for 5-15 minutes<br />

(2) Acetazolamide 500 mg IV and a topical ~-blocker (timolol 0.5%) to reduce intraocular pressure<br />

(3) Hyperbaric oxygen therapy may be beneficial if readily available.<br />

(4) Immediate ophthalmology consult for paracentesis of the anterior chamber<br />

(5) Evaluate (and treat) for associated disease processes.<br />

B. Central retinal vein occlusion<br />

1. Classic clinical scenario<br />

a. The patient complains of a sudden, painless, monocular decrease in vision. The visual loss is usually noted<br />

on awakening and is due to thrombosis of the central retinal vein.<br />

b. The pupil reacts sluggishly to light, and the funduscopic examination reveals retinal hemorrhages and<br />

tortuous retinal veins ("blood and thunder fundus"). Edema of the disc may also be present. The degree of<br />

visual loss is variable but is typically less severe than that which occurs in association with central retinal<br />

artery occlusion.<br />

Courtesy of Dr. Thomas Steinemann<br />

2. Risk factors<br />

a. Hypertension<br />

b. Diabetes mellitus<br />

C. Arteriosclerosis<br />

d. Chronic glaucoma<br />

e. Vasculitis<br />

3. Management<br />

a. No immediate treatment is effective.<br />

b. Ophthalmology follow-up is required for confirmation of the diagnosis and ongoing monitoring for<br />

development of complications.<br />

c. Identify and treat contributing factors.<br />

d. Aspirin is often recommended but not proved to be beneficial.<br />

4. Complications<br />

a. Neovascularization of the retina and/or iris<br />

b. Neovascular glaucoma<br />

C. Optic neuritis<br />

1. An inflammatory process and focal demyelination of the optic nerve; typically monocular in adults and<br />

binocular in children; more common in women than men<br />

2. Characterized by reduction of central vision with preservation of peripheral vision; visual loss ranges from<br />

mild to severe (finger counting to light perception).<br />

3. Most cases of optic neuritis are retrobulbar and do not have changes in the optic disc.<br />

4. Classic clinical scenario: The patient is usually 15-45 years old and presents with a rapidly progressive (hours<br />

to days) reduction or blurring of vision in association with ocular pain that is worsened by eye movement.<br />

5. Clinical presentation<br />

124


HEAD, EAR, EYE, NOSE, AND THROAT DISORDERS<br />

a. The presence of a Marcus-Gunn pupil (afferent pupillary defect), red vision desaturation (the color red is<br />

perceived as less intense in the affected eye), and a central visual field defect confirms optic pathology.<br />

b. The disc may be normal (retrobulbar optic neuritis) or swollen and hyperemic (papillitis).<br />

c. Remainder of the eye examination is normal.<br />

Courtesy of Dr. Thomas Steinemann<br />

6. A clear association with multiple sclerosis exists; approximately one-third of these patients go on to develop<br />

multiple sclerosis in the ensuing years. MRI of the orbit and brain is both sensitive and specific for optic<br />

nerve inflammation and may also show demyelination in the brain.<br />

7. Management<br />

a. Emergent ophthalmology consult<br />

b. Admission for IV steroids (eg, methylprednisolone) for 3 days, followed by oral steroids for 11 days. Use<br />

of oral steroids initially is controversial and may increase the risk of recurrence of optic neuritis.<br />

D. Eclipse burn (solar retinopathy)<br />

1. Caused by prolonged or accidental viewing of the sun, which leads to permanent central retinal (macular)<br />

loss of vision.<br />

2. Visual acuity is decreased, and funduscopic examination demonstrates discrete retinal disruption in the<br />

macular area.<br />

Courtesy of Dr. Thomas Steinemann<br />

Solar maculopathy<br />

Courtesy of Dr. Rishi P Singh<br />

Solar retinopathy<br />

3. No treatment is effective; the patient should be referred to an ophthalmologist.<br />

125


HEAD, EAR, EYE, NOSE, AND THROAT DISORDERS<br />

E. Retinal detachment (an ocular emergency}<br />

1. Involves a separation of the inner neuronal retina from the outer retinal pigment epithelial layer, usually as<br />

the result of a tear in the retina from vitreous traction.<br />

2. Painless and usually seen in patients with one or more risk factors<br />

a. Severe myopia<br />

b. Older age<br />

c. Trauma<br />

d. Previous cataract surgery<br />

e. An inherited connective tissue disorder (eg, Marfan syndrome)<br />

f. Diabetes mellitus<br />

g. A family history of retinal detachment<br />

3. Classic clinical scenario<br />

a. Early symptoms are flashing lights (as the retina begins to tear) and the presence of "spider webs" or<br />

"coal dust" across the visual field (small vitreous hemorrhages). As the retina detaches, the patient<br />

experiences a sensation of a curtain that is gradually lowering or raising in front of the affected eye.<br />

Visual acuity may or may not be affected, depending on the size or involvement of the macula.<br />

b. Examination with a direct ophthalmoscope may reveal an undulating, dull gray, detached, or even "out<br />

of focus" retina. Direct funduscopy is not sensitive enough to exclude retinal detachment and should<br />

be suspected based on clinical symptoms. The tear itself often originates in the retinal periphery and<br />

can be seen with the indirect ophthalmoscope. Bedside ultrasound (linear probe) may be useful in the<br />

emergency department.<br />

Courtesy of Dr. Thomas Steinemann<br />

4. As long as the macula remains attached, the probability of preserving central retinal vision is very good.<br />

5. Management<br />

a. Immediate ophthalmology consult for evaluation and retinal attachment surgery is indicated.<br />

b. The patient should avoid vigorous activity and preferably remain at bed rest until evaluated by the<br />

ophthalmologist.<br />

~ VltraoushemoIThage<br />

1. A common cause of sudden, painless, unilateral loss of vision<br />

2. Risk factors<br />

a. Greatest risk factor is proliferative diabetic retinopathy<br />

b. Posterior vitreous detachment and retinal vessel occlusion occur most commonly in older patients.<br />

c. Other risks include sickle cell disease, trauma, tumors, arterial microaneurysms, branch vein occlusion,<br />

and macular degeneration.<br />

3. Classic clinical scenario<br />

a. If hemorrhage is preceded by retinal detachment, the patient may experience flashes of light (photopsias).<br />

Otherwise, unilateral painless vision loss occurs (often described as haze, shadows, cobwebs, or smoke).<br />

b. On examination, the red reflex is diminished or absent (may have appearance of a red haze or a black<br />

reflex), and the fundal view is hazy. Visual acuity may or may not be affected. An afferent pupillary defect<br />

suggests a retinal detachment in addition to hemorrhage.<br />

126


HEAD, EAR, EYE, NOSE, AND THROAT DISORDERS<br />

Courtesy of Dr. Thomas Steinemann<br />

4. Management<br />

a. Immediate ophthalmology consult<br />

b. Bedrest with elevation of the head of the bed<br />

c. Some patients may benefit from surgery (photocoagulation, cryotherapy, vitrectomy).<br />

G. Functional blindness<br />

1. Classic clinical scenario<br />

a. Unilateral or bilateral blindness in the presence of normal pupillary reactions, a normal funduscopic<br />

examination, and lack of an afferent pupillary defect is very suggestive of a functional problem (hysteria/<br />

conversion disorder or malingering).<br />

b. An intact visual pathway can be confirmed with use of an optokinetic drum or strip, which will elicit<br />

optokinetic nystagmus.<br />

c. Despite the serious nature of their complaint, hysterical patients are often remarkably calm. Malingerers,<br />

on the other hand, are usually overly emotional and less cooperative.<br />

2. Management: ophthalmologic and psychiatric referrals<br />

H. Temporal (giant cell) arteritis<br />

1. A vasculitis of medium and large arteries that can cause optic nerve infarction and permanent visual loss<br />

(see also page 123); it is notorious for involving the optic nerve within hours or weeks. For unknown reasons,<br />

incidence appears to be rising.<br />

2. Classic clinical scenario<br />

a. Patients usually present with a unilateral headache. Sudden monocular loss of vision may also be a<br />

presenting symptom. Eye pain is not typical. Preceding symptoms may include jaw pain (claudication),<br />

weight loss, and malaise. Up to 50% of these patients have polymyalgia rheumatica (a syndrome of<br />

aching, pain, and stiffness in the proximal muscles, especially neck and shoulders); some patients may<br />

have associated neurologic symptoms that simulate a transient ischemic attack or stroke.<br />

b. Patients are more commonly women, and most patients are ::0:50 years old.<br />

c. Examination can reveal scalp tenderness, temporal arteries that are tender and tortuous (and sometimes<br />

pulseless), visual loss, and an afferent pupillary defect. Funduscopic examination may be normal or show<br />

dilated retinal veins.<br />

d. Diagnosis is suggested by an increased sedimentation rate (>50 mm/hr). Sedimentation rate increases<br />

with age; a simple rule of thumb for estimating the upper range of "normal" for patients >50 years old is<br />

age/2 for men and (age + 10)/2 for women.<br />

e. Diagnosis is confirmed by temporal artery biopsy (although treatment should not be delayed awaiting<br />

biopsy).<br />

3. Management<br />

a. Immediate ophthalmology consult<br />

b. Administration of high-dose steroids (eg, 60 mg oral prednisone)<br />

VI. NONACUTE VISUAL LOSS<br />

A. Primary open angle glaucoma<br />

1. Most common type of glaucoma and a major cause of slowly progressive blindness in the United States<br />

2. A painless condition of chronically increased intraocular pressure that ultimately damages the optic nerve;<br />

frequently bilateral with strong familial tendency<br />

127


HEAD, EAR, EYE, NOSE, AND THROAT DISORDERS<br />

3. Clinical presentation<br />

a. Normal appearing cornea and pupil<br />

b. Increased intraocular pressure (>21 mmHg)<br />

c. Increased optic nerve cup/disc ratio (>0.4)<br />

d. Arcuate scotomas<br />

e. Gradual loss of peripheral vision with sparing of central vision<br />

4. Management<br />

a. Unlike acute angle closure glaucoma, open angle glaucoma is a chronic, painless, nonemergent condition<br />

that does not generally present to the emergency department.<br />

b. Patients should be referred to an ophthalmologist for evaluation and treatment.<br />

c. Treatment: topical agents that decrease intraocular pressure by reducing the production of aqueous<br />

humor (carbonic anhydrase inhibitors, ~-blockers, and a 2<br />

-agonists) and/or increasing aqueous outflow<br />

(prostaglandin agonists, sympathomimetics)<br />

B. Cytomegalovirus (CMV) retinitis<br />

1. A necrotizing retinitis that, if untreated, results in progressive visual loss<br />

2. Occurs in immunocompromised patients, most notably those with advanced AIDS<br />

3. Classic clinical scenario<br />

a. Although patients can be asymptomatic early on, they typically present with scintillating scotomas, visual<br />

distortions, and visual loss.<br />

b. Funduscopic examination reveals fluffy white retinal lesions that are usually perivascular in location and<br />

associated with hemorrhage (the characteristic "pizza pie" or "cheese and ketchup" fundus).<br />

c. Ophthalmic screening is recommended if the CD4 count is


HEAD, EAR, EYE, NOSE, AND THROAT DISORDERS<br />

b. Irrigation should be followed by:<br />

(1) A complete eye examination (including visual acuity and measurement of intraocular pressure,<br />

which requires treatment if increased)<br />

(2) Instillation of a topical cycloplegic (eg, cyclopentolate or homatropine) and an antibiotic ointment<br />

(eg, bacitracin, erythromycin)<br />

(3) Use of a nonpreserved lubricant may prevent symblepharon (adhesion of lid to conjunctiva).<br />

(4) Administration of oral analgesics as needed<br />

(5) Ophthalmology referral for follow-up in 24 hours<br />

(6) Severe burns, particularly those due to alkalis, require immediate ophthalmology consult, discussion<br />

of additional treatment measures (eg, topical steroids), and hospital admission.<br />

B. Lid lacerations<br />

1. Once injury to the underlying globe has been excluded, superficial lacerations can be repaired with 6-0 or<br />

7-0 nonabsorbable suture.<br />

2. Lacerations in the following anatomically high-risk areas require referral for repair:<br />

a. Lacrimal canaliculi (lacerations of the lid margin that lie medial to the lacrimal puncta)<br />

b. Levator muscle/tendon (deep horizontal lacerations of the upper lid): the presence of ptosis is suggestive.<br />

c. Canthal tendons (penetrating wounds that transect the lateral or medial canthi)<br />

d. Orbital septum (deep horizontal lacerations of the upper lid): protrusion of fat into the laceration<br />

confirms septal perforation.<br />

e. Lid margins and lid lacerations with extensive tissue loss<br />

C. Corneal injuries<br />

1. Corneal abrasions<br />

a. Clinical presentation<br />

(1) Patients complain of pain, foreign body sensation, tearing, and photophobia.<br />

(2) Conjunctiva! injection is frequently present; visual acuity may be decreased if the abrasion is large or<br />

centrally located.<br />

b. Diagnostic evaluation<br />

(1) Instillation of a topical anesthetic agent provides temporary pain relief and permits a better<br />

examination. (Although topical anesthetic agents are useful diagnostically, they should never be<br />

prescribed for pain on an outpatient basis; corneal toxicity and retardation of epithelial regrowth<br />

occur with repetitive use.)<br />

(2) Always evert the lids to look for entrapped conjunctiva! foreign bodies; multiple fine linear abrasions<br />

of the corneal surface ("ice rink sign") are highly suggestive of a foreign body trapped under the<br />

upper lid.<br />

(3) Examination using a cobalt blue light source (slit lamp or Wood's light) with fluorescein staining<br />

reveals the abraded area, which appears bright green.<br />

c. Management<br />

(1) A cycloplegic agent (eg, cyclopentolate or homatropine) to relieve ciliary spasm and reduce pain<br />

(2) A topical antibiotic to prevent secondary infection; use an agent with gram-negative coverage<br />

(a topical quinolone [eg, ciprofloxacin, ofloxacin] or a topical aminoglycoside [eg, tobramycin,<br />

gentamicinl) for abrasions associated with soft contact lens use.<br />

(3) Contact lens users should be instructed to refrain from contact lens use until symptoms resolve.<br />

(4) Eye patching<br />

(a) Does not appear to facilitate healing (or provide comfort) and can have detrimental effects,<br />

especially if the injury was caused by vegetable matter or the patient wears contact lenses.<br />

(b) Should clearly be avoided in corneal abrasions due to prolonged contact lens use. These<br />

abrasions are associated with an increased incidence of gram-negative bacterial infections<br />

(particularly Pseudomonas) and ulceration. The warm, dark, and relatively hypoxic environment<br />

created by patching may further predispose these abrasions to infection.<br />

(5) Oral analgesics are seldom needed if the eye is adequately treated with a cycloplegic agent (eg,<br />

cyclopentolate or homoatropine; however, if the patient cannot be redosed with a cycloplegic, then<br />

oral analgesia will become necessary.<br />

(6) Ophthalmology referral for reevaluation in 24-36 hours<br />

129


HEAD, EAR, EYE, NOSE, AND THROAT DISORDERS<br />

2. Corneal foreign bodies and rust rings<br />

a. Clinical presentation: corneal foreign bodies containing iron produce a rust ring that is irritating and can<br />

be difficult to remove.<br />

b. Management (Note that rust ring removal is never a true eye emergency and can be deferred to a<br />

specialist if the physician does not have access to the correct equipment or is not familiar with its use.)<br />

(1) Apply a topical anesthetic agent, and then remove the rust ring with a commercial burr drill or a<br />

sterile 25-gauge needle under slit-lamp magnification.<br />

(2) The burr drill is preferred, because the removal can be accomplished more quickly and easily with<br />

this instrument.<br />

(3) Once the foreign body or rust ring has been removed, apply a topical broad-spectrum antibiotic<br />

(eg, erythromycin, polymixin-trimethoprim, tobramycin, ciprofloxacin, ofloxacin) and a cycloplegic<br />

agent (eg, cyclopentolate or homatropine); refer to an ophthalmologist for reevaluation in 24-48<br />

hours.<br />

(4) If the rust ring cannot be easily removed, treat as above and refer to an ophthalmologist for removal<br />

the next day.<br />

3. Corneoscleral lacerations and perforations<br />

a. Because of Bell phenomenon (eyeball rolling upward and outward in response to eye closure), they are<br />

commonly located in the inferior aspect of the globe.<br />

b. Clinical presentation<br />

(1) Teardrop or irregularly shaped pupil<br />

(2) Flattening of the anterior chamber<br />

(3) Small fragments of black (iris) pigment at the wound edges (uveal prolapse)<br />

c. Diagnostic evaluation<br />

(1) Examination with the cobalt blue light source of the slit-lamp with fluorescein staining will mark the<br />

area of suspected aqueous leak.<br />

(2) The efflux of aqueous humor will cause the fluorescein to flow from the laceration in a "riverlike"<br />

pattern (positive Seidel test), which is diagnostic if present.<br />

d. CT of the orbits with thin sections (cuts of 1-2 mm) and coronal reconstruction may confirm the<br />

presence of a foreign body. Bedside ocular ultrasound is contraindicated if globe rupture is suspected.<br />

e. Management<br />

(1) Do not put any pressure on the globe (not even an intraocular check with a tonopen or Schiotz<br />

tonometer).<br />

(2) Apply a rigid metal eye shield (not a patch) to protect the eye from any inward pressure.<br />

(3) Keep patient nothing per os (because many will need repair in the operating room).<br />

(4) Provide tetanus prophylaxis as needed.<br />

(5) Administer<br />

(a) Prophylactic IV broad-spectrum antibiotics (to cover Staphylococcus and Streptococcus) to<br />

prevent endophthalmitis<br />

(b) Prophylactic antiemetics<br />

(c) Analgesics as needed<br />

(d) Update tetanus immunization<br />

(6) Obtain immediate ophthalmology consult.<br />

D. Ruptured globe<br />

1. Clinical presentation<br />

a. Teardrop or irregularly shaped pupil<br />

b. Hemorrhagic chemosis<br />

c. Decrease in visual acuity<br />

d. Distorted anterior chamber<br />

e. Uveal prolapse<br />

f. Vitreous hemorrhage<br />

g. Positive Seidel test<br />

2. Management: same as that for corneoscleral lacerations and punctures (see above)<br />

130


HEAD, EAR, EYE, NOSE, AND THROAT DISORDERS<br />

E. Hyphema<br />

1. Blunt ocular injury may cause bleeding into the anterior chamber (hyphema). The bleeding originates from<br />

the blood vessels of the ciliary body or iris.<br />

2. Clinical presentation (be able to identify on pictorial)<br />

a. Although some patients with small hyphemas are asymptomatic, most complain of blurred vision, dull eye<br />

pain, and photophobia.<br />

b. Hyphemas are graded according to the percentage of blood in the anterior chamber.<br />

Grade 0: microscopic<br />

Grade 1: layered blood 30<br />

mmHg in a normal patient or >24 mmHg in a sickle cell patient. Avoid carbonic anhydrase inhibitors in<br />

sickle cell patients; they can cause RBCs to sickle in the anterior chamber, which clogs the trabecular<br />

meshwork and further increases intraocular pressure.<br />

g. Immediate ophthalmology consult<br />

h. Administration of other medications (mydriatics, cycloplegics, steroids, and antifibrinolytics) and the<br />

need for hospitalization should be decided in consultation with the ophthalmologist.<br />

(1) A cycloplegic/mydriatic agent is often used to dilate the pupil and avoid "pupillary play" (pupil size<br />

fluctuations due to changing ambient lighting) so as not to stretch the leaking blood vessel.<br />

(2) Steroids and antifibrinolytics may prevent rebleeding. Surgical evacuation may be necessary with<br />

large (grade IV) hyphemas.<br />

4. Complications<br />

a. Rebleeding-usually occurs 2-5 days after the initial bleed (when clots begin to lyse) and is the most<br />

common complication; it can cause an abrupt and sight-threatening rise in intraocular pressure.<br />

b. Blood staining of the corneal epithelium<br />

c. Secondary glaucoma: intraocular hypertension is seen more commonly in sickling hemoglobinopathies<br />

(sickle cell disease, thalassemia) and should be treated initially with topical ~-blockers, avoiding carbonic<br />

anhydrase inhibitors.<br />

d. Anterior and posterior synechia formation<br />

F. Lens subluxation and dislocation<br />

1. Usually caused by blunt trauma to the globe but can occur spontaneously or with trivial trauma in patients<br />

with Marfan syndrome, homocystinuria, tertiary syphilis, and rheumatoid arthritis<br />

2. Lens subluxation produces visual distortion and monocular diplopia, which persists despite covering the<br />

other eye (as opposed to binocular diplopia, which can be corrected by covering either eye).<br />

3. Lens dislocation causes severe blurring of vision.<br />

4. The lens may dislocate either anteriorly into the anterior chamber or posteriorly into the vitreous cavity.<br />

a. Anterior dislocations can produce acute angle closure glaucoma by mechanically blocking the egress of<br />

aqueous humor.<br />

b. Posterior dislocations produce iridodonesis (a quivering of the iris after rapid eye movement).<br />

5. With subluxations, the edge of the lens will be seen on slit-lamp examination when the pupil is dilated.<br />

131


HEAD, EAR, EYE, NOSE, AND THROAT DISORDERS<br />

6. Management<br />

a. Obtain immediate ophthalmology consult.<br />

b. Anterior dislocations require immediate repositioning or removal of the lens.<br />

c. Posterior dislocations are handled more conservatively, often only with refractive correction.<br />

G. Blowout fractures of the orbit<br />

1. Etiology<br />

a. Blunt trauma to the eye transmits hydraulic forces throughout the globe. The floor and medial wall of the<br />

orbit are most susceptible to blowout fractures.<br />

b. Orbital floor fractures (see below) can entrap the inferior rectus and inferior oblique muscles as well as<br />

fat; contusion or laceration of the infraorbital nerve may also occur. Fractures of the medial wall of the<br />

orbit (less common; see also page 148) can entrap the medial rectus muscle.<br />

2. Clinical presentation of orbital floor fractures<br />

a. Enophthalmos (posterior displacement of the eyeball causing slight ptosis)<br />

b. lnfraorbital nerve distribution anesthesia<br />

c. Subcutaneous orbital emphysema (suggests a medial wall fracture with ethmoid sinus disruption)<br />

d. Be sure to perform a careful eye examination in these patients, because up to one in four will have<br />

concomitant injuries to the globe.<br />

e. Pain and diplopia on upward gaze occur secondary to limitation of upward gaze, which indicates<br />

muscular entrapment and is a true emergency.<br />

3. Diagnostic evaluation: CT scanning is the standard of care.<br />

4. Management<br />

a. Exclude entrapment.<br />

b. Although symptoms of fracture without entrapment usually resolve with conservative management,<br />

ophthalmology consult is still indicated to exclude associated injuries to the globe.<br />

c. Broad-spectrum prophylactic antibiotics and decongestants should be considered for those patients<br />

with sinus fractures, especially with evidence or suspicion of sinusitis. Patients with subcutaneous<br />

emphysema should be given nasal precautions (eg, no nose blowing, sneeze with mouth open, no Valsalva<br />

maneuvers).<br />

H. Retrobulbar hematoma (an ocular emergency)<br />

1. Etiology: extreme blunt or deep penetrating trauma to the eye produces orbital hemorrhage with<br />

accumulation of blood behind the globe.<br />

2. Clinical presentation<br />

a. Exophthalmos (forward displacement of the eyeball)-may be severe<br />

b. Decreased visual acuity<br />

c. Eye pain<br />

d. Limited mobility of the globe<br />

e. Increased intraocular pressure<br />

f. Afferent pupillary defect (or Marcus-Gunn pupil)<br />

3. Diagnostic evaluation: CT of the orbit will demonstrate the hematoma.<br />

4. Management<br />

a. Immediate ophthalmology consult for emergent decompression via a lateral canthotomy, cantholysis, and<br />

anterior chamber paracentesis.<br />

b. lntraocular hypertension can be treated with topical ~-blocker as well as IV carbonic anhydrase inhibitor<br />

and mannitol.<br />

I. lntraocular foreign body<br />

1. Clinical presentation and history<br />

a. Work that involves pounding metal on metal or using a grinding wheel<br />

b. Physical examination clue is an irregular pupil that is usually peaked toward the site of penetration.<br />

c. Onset of symptoms may be delayed (c':24 hours)<br />

(1) Reduced visual acuity<br />

(2) Dull, unlocalized ocular pain<br />

132


HEAD, EAR, EYE, NOSE, AND THROAT DISORDERS<br />

2. Diagnostic evaluation<br />

a. Radiographic confirmation and localization of the penetrating foreign body is indicated, because these<br />

foreign bodies may need to be surgically removed.<br />

b. Plain radiographs of the orbit may demonstrate the foreign body.<br />

c. CT of the orbits (1-2 mm cuts with coronal reconstructions) are very helpful in the precise localization of<br />

intraocular foreign bodies (or associated air).<br />

d. MRI also provides accurate localization but is not as readily available and is contraindicated if potentially<br />

any metallic objects (could dislodge them).<br />

e. Although bedside ocular ultrasound may also identify intraocular foreign bodies, it is contraindicated if<br />

globe rupture is suspected.<br />

3. Management<br />

a. Perform a Seidel test to see if an aqueous leak from the site of penetration is present. Do not attempt to<br />

check intraocular pressure if penetration of the globe is suspected.<br />

b. Place a metal shield over the eye, administer a broad-spectrum antibiotic IV, and provide tetanus<br />

prophylaxis as needed.<br />

c. Obtain an immediate ophthalmology consult for the decision to remove (or not) an intraocular foreign<br />

body, which is based on the composition/reactivity of the object as well as on its location in the eye.<br />

(1) Wood, vegetable matter, and metals (eg, iron, copper, and steel) typically incite an intense<br />

inflammatory reaction when left in the eye. BBs and pellets are also poorly tolerated, because they<br />

are composed of iron as well as lead. Emergent surgical extraction is indicated.<br />

(2) Inert foreign bodies (eg, glass, lead, plaster, rubber, silver, stone) that are minimally symptomatic<br />

may be treated with a nonoperative approach. However, many inert foreign bodies can cause toxicity<br />

by virtue of a coating or chemical additive, which may influence management decisions.<br />

J. Traumatic iritis/iridocyclitis<br />

1. Classic clinical scenario<br />

a. The patient has blunt ocular trauma. Onset of symptoms (aching pain, photophobia, and reduced visual<br />

acuity) usually begin 1-4 days after injury. The pupil is constricted, and a deep ciliary flush is usually<br />

present.<br />

b. Slit-lamp examination reveals cells and flare in the anterior chamber.<br />

2. Management<br />

a. A cycloplegic agent (eg, cyclopentolate or homatropine)<br />

b. A topical steroid (only in consult with an ophthalmologist)<br />

c. Ophthalmology referral<br />

K. Traumatic miosis and mydriasis<br />

1. Blunt ocular trauma can produce<br />

a. Bruising and irritation of the iris sphincter___,. constriction and spasm of the pupil (miosis), while<br />

b. Tears in the iris sphincter muscle fibers ___,. loss of constriction ability ___,. dilation (mydriasis)<br />

2. These defects may resolve spontaneously over days to weeks or, in the case of actual iris sphincter muscle<br />

fiber damage, may become permanent.<br />

3. Management<br />

a. No acute treatment is indicated, unless hyphema is present.<br />

b. Referral to an ophthalmologist for follow-up<br />

VIII. ANISOCORIA<br />

A. Clinical presentation<br />

1. Normal finding: simple physiologic anisocoria (unequal pupil size; usually :Sl mm difference) is seen in up to<br />

20% of the population.<br />

2. lntracranial third nerve palsy secondary to an expanding supratentorial mass/bleed or a posterior<br />

communicating artery aneurysm (absence of an isolated third nerve palsy does not exclude an aneurysm); look<br />

for ptosis, diplopia from extraocular muscle dysfunction with "down and out" position of involved eye.<br />

3. Prior eye surgery<br />

4. Traumatic miosis or mydriasis<br />

133


HEAD, EAR, EYE, NOSE, AND THROAT DISORDERS<br />

5. Uniocular drug exposure: pilocarpine 1 % will constrict a pupil that has been dilated from a third nerve palsy<br />

but not from anticholinergic agent exposure.<br />

6. Adie tonic pupil: benign condition primarily affecting women 20-40 years old who often present with blurred<br />

vision<br />

a. Blurred near vision but normal vision at distance<br />

b. Diminished deep tendon reflexes<br />

c. Sector palsies of iris on slit-lamp examination<br />

d. Slow constriction with accommodation<br />

e. Strong constriction to cholinergic drop (pilocarpine 0.1%)<br />

7. Horner syndrome: due to loss of sympathetic innervation<br />

a. Look for ptosis, miosis, and anhidrosis<br />

b. "Dilatation lag" is classic, in which the involved pupil takes as much as 15 seconds to dilate, with<br />

anisocoria greatest at 5 seconds of darkness.<br />

c. Causes include stroke, neck mass or cancer, carotid dissection, brachia! plexus trauma, and others.<br />

B. Diagnostic evaluation<br />

1. Compare pupil sizes in bright light and under dark conditions.<br />

a. With simple physiologic anisocoria, the pupil size discrepancy is the same in all levels of illumination.<br />

Furthermore, the pupils in these patients are round and respond normally to light and accommodation;<br />

associated findings (eg, ptosis) are absent.<br />

b. Anisocoria that is more pronounced in the dark suggests that the smaller pupil is abnormal (eg, Horner<br />

syndrome).<br />

c. Anisocoria that is greater in bright light suggests the larger pupil is abnormal (eg, third nerve palsy, Adie<br />

tonic pupil, or pharmacologic blockade).<br />

2. Inquire about prior eye surgery/trauma.<br />

3. Consider the setting and associated signs and symptoms.<br />

a. Head trauma and altered level of consciousness - exclude an expanding supratentorial mass/hemorrhage<br />

b. Headache and third nerve palsy - exclude a posterior communicating artery aneurysm<br />

c. Miosis, ptosis, and anhydrosis - exclude Horner syndrome<br />

4. Ask about the use of medications.<br />

a. Topical parasympatholytics (eg, atropine) or sympathomimetics (eg, phenylephrine, cocaine)<br />

b. Nebulized anticholinergics (causing ocular exposure)<br />

c. Scopolamine patches (with inadvertent ocular contact via the hands)<br />

IX. OPHTHALMIC MEDICATIONS<br />

A. Packaging<br />

Table 6: Cap/Bottle Colors ofTopical Ophthalmic Medications<br />

Medication<br />

Anesthetics<br />

Mydriatics and cycloplegics<br />

Miotics<br />

Antibiotics<br />

Steroids<br />

Cap/Bottle Color<br />

White or clear<br />

Red<br />

Green<br />

Tan<br />

Pink<br />

B. Topical anesthetics<br />

1. Used to facilitate examination of patients with painful ocular conditions and to allow performance of ocular<br />

procedures (foreign body removal, irrigation, tonometry)<br />

2. Deprive the cornea of its normal protective reflexes; in repeated doses will retard healing of the cornea<br />

3. Should never be dispensed as pain medications for outpatient use<br />

134


HEAD, EAR, EYE, NOSE, AND THROAT DISORDERS<br />

4. Types (no cross reaction between esters and amides)<br />

a. Esters<br />

(1) Proparacaine 0.5%<br />

(a) Rapid onset<br />

(b) Duration of action ~ 15 minutes<br />

(2) Tetracaine 0.5%<br />

(a) More irritating than proparacaine<br />

(b) Delayed onset<br />

(c) Duration of action ~30 minutes<br />

(3) Fluorocaine: a premixed combination of proparacaine and fluoroscein dye that has thimerosal as a<br />

preservative (which may cause a local allergic conjunctivitis in susceptible patients)<br />

(4) Benoxinate 0.4%<br />

(a) Onset 1-2 minutes<br />

(b) Duration of action 15 minutes<br />

b. Amides: dibucaine 0.1 %<br />

C. Mydriatics and cycloplegics<br />

1. Mydriatics<br />

a. Topical sympathomimetic agents that paralyze the iris sphincter only, thereby producing pupillary dilation<br />

(mydriasis) without affecting accommodation<br />

b. Used diagnostically to facilitate evaluation of internal ocular structures<br />

c. Example: phenylephrine 2.5%: onset 15-60 minutes, duration of action 3-4 hours<br />

2. Cycloplegics<br />

D. Miotics<br />

a. Topical parasympatholytic agents that paralyze both the iris sphincter (mydriasis) and the ciliary muscle<br />

(cycloplegia)<br />

b. Used therapeutically in the treatment of iritis and deep corneal abrasions to relieve pain by eliminating<br />

ciliary spasm<br />

c. Use of cyclopentolate or tropicamide is preferred in the presence of acute inflammation, because their<br />

effective duration of action is shorter than that of other agents, which allows repetitive dosing to keep the<br />

patient comfortable.<br />

(1) Cyclopentolate: onset 30-60 minutes, duration :S24 hours<br />

(2) Tropicamide: onset 15-20 minutes, duration 4-6 hours<br />

d. Homatropine is useful (short term) for subacute inflammation (iritis) caused by trauma or abrasion: onset<br />

30-40 minutes, duration 2-3 days<br />

e. Atropine is more effective for chronic inflammation seen after surgery and in severe uveitis; because of the<br />

long duration of action (1-2 weeks), its use in the emergency department is limited.<br />

f. These agents are contraindicated in patients with a history of glaucoma, evidence of increased intraocular<br />

pressure, presence of a shallow anterior chamber, or suspicion of a ruptured globe.<br />

1. Pilocarpine (2%): used for treatment of acute angle closure glaucoma<br />

2. Mechanism of action: constricts the pupil, pulling the iris back from its anterior position, which opens the angle<br />

and allows egress of the aqueous humor<br />

E. Topical antibiotics<br />

1. Solution (drops) are rapidly absorbed and require frequent instillation. In contrast, ointments have a longer<br />

duration of action and require less frequent administration but transiently blur vision when applied.<br />

2. Types/examples<br />

a. Erythromycin: broad-spectrum, primarily bacterostatic<br />

b. Sulfacetamide: broad-spectrum, bacterostatic<br />

c. Bacitracin: broad-spectrum, primarily bacterostatic<br />

d. Aminoglycosides (gentamicin, tobramycin): gram-negative coverage, including Pseudomonas; bactericidal.<br />

Avoid neomycin-containing antibiotics because of the risk of hypersensitivity reaction in 4% of the<br />

population.<br />

e. Polymyxin B/trimethoprim: broad spectrum, especially gram-positive organisms such as Staphylococcus<br />

and methicillin-resistant 5 aureus; bacteriostatic<br />

135


HEAD, EAR, EYE, NOSE, AND THROAT DISORDERS<br />

F. Antivirals<br />

f. Quinolones (ciprofloxacin, norfloxacin, ofloxacin, gatifloxacin, moxifloxacin): broad spectrum, including<br />

Pseudomonas; bactericidal. Drug of choice for monotherapy in treatment of corneal ulcers; poor<br />

streptococci coverage; expensive<br />

1. Interfere with viral DNA synthesis<br />

2. Topical antivirals (vidarabine, trifluridine)<br />

a. Used in treatment of herpes simplex keratitis<br />

b. Shorten treatment time and improve rate of healing<br />

c. Obtain ophthalmologic consult before prescribing.<br />

3. Systemic antivirals: used in consultation with ophthalmology and internal medicine in the treatment of herpes<br />

simplex keratitis, herpes zoster ophthalmicus, and cytomegalovirus retinitis<br />

a. Acyclovir (oral and IV) is useful for both herpes simplex keratitis and herpes zoster ophthalmicus.<br />

b. Valcyclovir (oral) is useful for both herpes simplex keratitis and herpes zoster ophthalmicus; requires less<br />

frequent dosing than acyclovir.<br />

c. Famciclovir (oral) is used to treat herpes zoster ophthalmicus; requires less frequent dosing than acyclovir.<br />

d. Valganciclovir (oral), gangliclovir (IV), or foscarnet (IV) is used in the treatment of cytomegalovirus retinitis.<br />

G. Topical steroids<br />

1. Used in treatment of iritis to reduce inflammation<br />

2. In general, they should not be prescribed in the emergency department; if they are prescribed, this should<br />

be done only in consult with an ophthalmologist, and prompt (within 24 hours) ophthalmology follow-up is<br />

indicated.<br />

3. A primary contraindication to steroid drops is early herpes simplex keratoconjunctivitis.<br />

4. Examples: prednisolone acetate, fluorometholone, dexamethasone<br />

H. Hyperosmotic agents<br />

1. Used in treatment of acute angle closure glaucoma<br />

2. Decrease intraocular pressure by decreasing the volume of fluid in the eye<br />

3. Examples<br />

a. IV mannitol (20% solution): works rapidly, very effective in the emergency department<br />

b. Oral glycerol (50% solution): avoid in diabetic patients because glycerol is metabolized to glucose and can<br />

produce hyperglycemia and ketosis; may cause nausea<br />

c. Oral isosorbide (45% solution): may cause nausea<br />

I. Carbonic anhydrase inhibitors<br />

1. Used in treatment of acute angle closure glaucoma<br />

2. Act to reduce intraocular pressure by decreasing the secretion of aqueous humor by the ciliary body<br />

3. These agents are sulfa compounds; sickle cell patients who present with hyphema are at risk of red cell sickling<br />

in the anterior chamber, further compromising aqueous outflow and increasing intraocular pressure.<br />

4. Examples: acetazolamide (IV or oral), methazolamide (oral), dorzolamide hydrochloride (topical)<br />

J. Topical adrenergic agents: ~-blockers or a 2<br />

-agonists<br />

1. Used in treatment of acute angle closure glaucoma<br />

2. Reduce intraocular pressure by decreasing secretion of aqueous humor by the ciliary body<br />

3. Preferred for initial management of acute angle closure glaucoma, because they do not produce the adverse<br />

effects associated with miotics<br />

4. Most commonly used ~-agonist is timolol (a nonselective ~-blocker), but betaxolol is cardioselective and,<br />

therefore, preferred by some clinicians; both are packaged in bottles with yellow or blue caps.<br />

5. Most commonly used a 2<br />

-agonists are apraclonidine and brimonidine; they are usually packaged in bottles with<br />

white caps.<br />

6. All topical adrenergic agents can cause serious systemic adverse effects (especially cardiopulmonary). Topical<br />

~-blockers can result in heart failure, heart block, and asthma, whereas a 2<br />

-agonists can cause orthostatic<br />

hypotension and syncope.<br />

136


HEAD, EAR, EYE, NOSE, AND THROAT DISORDERS<br />

X. TONOMETRY<br />

A. Measurement of intraocular pressure (IOP) using a calibrated device called a tonometer<br />

B. Indicated whenever the diagnosis of glaucoma or intraocular hypertension (eg, retrobulbar hemorrhage) is<br />

considered. IOP may be low in cases of iritis.<br />

C. Devices<br />

1. Schiotz impression tonometer: a handheld instrument that is available in most emergency departments and<br />

easy to use; a conversion table is required to translate the reading to mmHg.<br />

2. TonoPen: a handheld device that gives a digital readout of the IOP and uses disposable covers<br />

3. Goldman applanation tonometer: an instrument that is used in conjunction with a slit lamp<br />

4. Air-puff tonometer: an instrument that has the advantage of not requiring direct eye contact<br />

D. Contraindications include the presence or suspicion of a penetrating ocular injury (absolute) or an eye<br />

infection (relative).<br />

137


HEAD, EAR, EYE, NOSE, AND THROAT DISORDERS<br />

NASAL TRAUMA<br />

In evaluating nasal trauma, it is important to elicit any history of previous nasal deformity or surgery.<br />

I. NASAL FRACTURES<br />

A. Uncomplicated nasal fracture<br />

1. Can be determined on clinical grounds alone<br />

2. Clinical presentation: suggestive findings include the presence of nasal bridge tenderness, swelling,<br />

ecchymosis, deformity, or crepitance. Anterior rhinoscopy may reveal septal deviation.<br />

3. Diagnostic evaluation: radiographs add little or nothing to initial or subsequent management and need not be<br />

obtained in most isolated cases.<br />

4. Management<br />

a. If patients present early (usually within 2 hours) and the swelling does not preclude an accurate assessment<br />

of nasal contour, early reduction is frequently possible.<br />

b. If swelling does not allow an accurate assessment, reduction can be deferred.<br />

c. All patients with suspected or confirmed nasal fractures should be referred to ENT for reevaluation within<br />

2-5 days. Delaying longer may allow fractures to begin healing in their displaced position (especially in<br />

children), making intervention more difficult.<br />

B. Complex fractures<br />

1. Nasoethmoid fractures and nasal fractures associated with other facial fractures (eg, orbital floor fractures,<br />

tripod fractures of the zygomatic bone)<br />

2. Diagnostic evaluation requires CT scanning to assess the extent of injury present and establish a treatment plan.<br />

3. Management<br />

a. Nasoethmoid fractures are particularly complex and difficult injuries.<br />

b. These fractures are usually produced by a blow to the bridge of the nose and require CT scanning; possibly<br />

admission; and neurosurgical, ENT, and/or ophthalmology consults.<br />

C. Prophylactic antibiotic therapy (antistaphylococcal)<br />

1. Warranted in patients discharged with nasal packing and may be warranted for nasal fractures associated with<br />

a laceration of the nasal mucosa or skin (potential bacterial contamination)<br />

2. Nasal saline irrigation helps remove blood and cleanse the mucosa; cold compresses will help reduce swelling.<br />

II. MAJOR COMPLICATIONS<br />

A. Septal hematoma<br />

1. Clinical presentation: patients with nasal trauma should always be carefully evaluated to exclude the presence<br />

of a septa[ hematoma (a bluish purple, grapelike swelling of the nasal septum).<br />

2. Management<br />

a. These subperichondrial hematomas should be vertically incised and drained.<br />

b. After this, the anterior nasal cavity is packed to prevent blood from reaccumulating.<br />

c. The patient is then placed on an anti staphylococcal antibiotic and referred to ENT for follow-up in 24-48<br />

hours.<br />

3. Complications: if not drained, these hematomas can result in formation of a septa! abscess or avascular<br />

necrosis of the nasal septum with subsequent development of septa! perforation and a permanent "saddlenose"<br />

deformity.<br />

B. CSF rhinorrhea<br />

1. Due to fracture of the cribriform plate of the ethmoid bone<br />

2. Clinical presentation<br />

a. May not develop for days to weeks<br />

b. Should be suspected whenever a patient develops a clear nasal discharge after facial trauma and may be<br />

associated with hyposmia (or anosmia) and headache<br />

138


HEAD, EAR, EYE, NOSE, AND THROAT DISORDERS<br />

c. Usually unilateral and may be increased by having the patient lean forward or by compressing the<br />

jugular vein.<br />

3. Diagnostic evaluation<br />

a. The diagnosis is most reliably confirmed by high resolution CT.<br />

b. Fluid can be collected and send for beta-2 transferrin to confirm CSF.<br />

4. Management<br />

C. Hemorrhage<br />

a. If CSF rhinorrhea is suspected:<br />

(1) Place the patient in the upright position.<br />

(2) Collect fluid if possible.<br />

(3) Obtain immediate neurosurgical and/or ENT consults (depending on local practice environment).<br />

(4) Avoid nasal packing.<br />

(5) Advise patient on nasal precautions (eg, no nose blowing, sneeze with mouth open, no Valsalva<br />

maneuvers).<br />

b. Administration of prophylactic antibiotics (penicillin, cephalosporins) remains controversial and should<br />

be determined by the neurosurgical and/or ENT consultant.<br />

1. Bleeding may be profuse but is usually of short duration.<br />

2. If bleeding does not subside spontaneously, and CSF rhinorrhea is not suspected, nasal packing may be<br />

required; if so, be sure to prescribe an antistaphylococcal antibiotic.<br />

139


HEAD, EAR, EYE, NOSE, AND THROAT DISORDERS<br />

NASAL FOREIGN BODIES<br />

I. CLINICAL PRESENTATION<br />

A. History<br />

1. Most common in children 2-3 years old<br />

2. History is often lacking<br />

B. Physical examination<br />

1. Unilateral foul-smelling nasal discharge, persistent unilateral epistaxis, or a foul body odor<br />

2. Application of a vasoconstrictor (eg, 1 % phenylephrine or oxymetazoline) and a topical anesthetic (eg, 4%<br />

lidocaine) facilitates examination (as well as subsequent treatment) by providing patient comfort and improving<br />

the visual field.<br />

II. DIAGNOSIS AND TREATMENT<br />

A. Inspecting the nares with a nasal speculum in combination with a head lamp usually confirms the diagnosis. The<br />

first step is always to confirm that there is no chance the foreign body is a battery.<br />

B. Removal techniques<br />

1. Many nasal foreign bodies can be removed in the emergency department by using one of the techniques<br />

described below.<br />

2. Method selected depends on the age and cooperativeness of the child as well as the characteristics of the<br />

foreign body.<br />

3. Patient restraint is frequently necessary.<br />

4. Options<br />

a. Positive-pressure techniques have the advantage of being simple and noninvasive and probably should<br />

be tried first. Have the child forcefully blow his or her nose while occluding the uninvolved nostril. This<br />

method works best in older, cooperative children and may require several attempts.<br />

b. A suction catheter with a flange may also be used to remove the foreign body.<br />

c. Forceps (alligator, bayonet), right angle probes, wire loops, or a small Fogarty catheter are acceptable<br />

alternatives, but particular care must be exercised to avoid injuring the mucosa or pushing the object<br />

posteriorly with these instruments.<br />

5. After the foreign body has been removed, the nasal cavity should be inspected for the presence of additional<br />

foreign objects, infection, and trauma.<br />

6. If the foreign body cannot be safely removed, the patient should be referred to an otolaryngologist.<br />

a. Follow-up on an outpatient basis in 24 hours is appropriate for most of these patients, although surgical<br />

removal is occasionally required. Patients should be sent home nothing by mouth (NPO) when possible.<br />

b. However, immediate ENT consult (and possible admission) are required for patients with a lodged<br />

battery, an unknown object (cannot exclude a battery), associated facial cellulitis, or associated systemic<br />

symptoms. The long-term damage created by even brief contact with a battery cannot be overemphasized.<br />

140


HEAD, EAR, EYE, NOSE, AND THROAT DISORDERS<br />

EPISTAXIS<br />

I. VASCULAR SUPPLY TO THE NOSE<br />

A. Upper half of the nose: anterior and posterior ethmoidal arteries (internal carotid__,. ophthalmic arteryethmoidal<br />

arteries)<br />

B. Lower half of the nose: sphenopalatine and greater palatine arteries (both are distal branches of the<br />

external carotid)<br />

C. Most common sources of nosebleeds<br />

1. Anterior nosebleeds_,, anteroinferior nasal septum (Little area): Kiesselbach plexus<br />

2. Posterior nosebleeds, venous _,, posteromedial turbinate: Woodruff plexus<br />

3. Posterior nosebleeds, arterial _,, sphenopalatine artery<br />

II. ETIOLOGY<br />

A. Anterior bleeds (90% of all nosebleeds): more common in children (have a high level of suspicion for blood<br />

dyscrasias [eg, hemophilia, leukemia]) and young adults<br />

1. "Winter syndrome": dry air and upper respiratory infection<br />

2. Trauma: look for septa I hematoma<br />

3. Foreign body: especially children<br />

4. Nose picking: "epistaxis digitorum"<br />

5. Allergies<br />

6. Nasal irritants (eg, cocaine, nasal sprays)<br />

7. Pregnancy: epistaxis due to increased blood volume and venous engorgement<br />

8. Rapid changes in atmospheric pressure<br />

9. Infection: rhinitis, sinusitis<br />

10. Osler-Weber-Rendu syndrome (hereditary hemorrhagic telengectasia)<br />

11. Nasal malignancy<br />

B. Posterior bleeds (10% of all nosebleeds): more common in the elderly<br />

1. Coagulopathy: pharmaceutical anticoagulants, blood dyscrasias, or liver disease<br />

2. Neoplasm<br />

3. Hypertension<br />

4. Nasal malignancy<br />

Ill. CLINICAL PRESENTATION<br />

A. History<br />

1. Onset (spontaneous or precipitated), duration, severity<br />

2. Isolated event or recurrent problem<br />

3. Home management<br />

4. Environment (eg, dry, dusty)<br />

5. Medications and illicit drug use<br />

6. Underlying medical problems (eg, blood dyscrasias, hepatic or renal failure, hematologic cancers)<br />

7. Presence of unilateral obstruction or neurologic deficit suggests possibility of mass<br />

8. Easy bruising and bleeding at other sites<br />

9. Recent chemotherapy or prior history of neoplasm<br />

10. Hypertension is controversial; most literature suggests no association, especially with anterior bleeds.<br />

141


HEAD, EAR, EYE, NOSE, AND THROAT DISORDERS<br />

B. Physical examination<br />

1. Vital signs (note any orthostatic change)<br />

2. Eva I uate the blood and clots<br />

3. Identify location of bleeding site<br />

4. Signs of underlying bleeding disorder (ecchymoses, purpura, petechiae, spider angiomata)<br />

5. Nasal mass<br />

IV. TREATMENT<br />

A. General preparation<br />

1. Assemble the necessary supplies and equipment at bedside; appropriately gown both the patient and yourself.<br />

2. Then, with the patient seated and his or her head in the sniffing position, evacuate accumulated blood and<br />

clots from the nose, apply a topical anesthetic (eg, 4% lidocaine) and vasoconstrictor (eg, 1 % phenylephrine<br />

or oxymetazoline), then use a nasal clamp or have the patient firmly pinch his or her nose for 10 continuous<br />

minutes.<br />

B. Anterior sites<br />

1. Chemical cautery (silver nitrate sticks)<br />

2. A small piece of a hemostatic material on the bleeding site if the bleeding is mild<br />

3. Anterior nasal packing if tamponade is needed to control the bleeding; packs are left in place for 1-3 days.<br />

Prescribe antistaphylcoccal antibiotics if packing placed.<br />

4. Follow-up with ENT or primary care physician in 2-3 days.<br />

C. Posterior sites<br />

1. Posterior packing is indicated; use a gauze pack, intranasal balloon device, or Foley catheter.<br />

2. Admit patients with posterior bleeds.<br />

3. Closely monitor airway status, especially if history of obstructive sleep apnea.<br />

D. Drug treatment<br />

1. Because packing impairs sinus drainage, all patients who require anterior or posterior packing should be<br />

started on an antibiotic (eg, amoxicillin-clavulanate or a first-generation cephalosporin) as prophylaxis against<br />

sinusitis, otitis media, and toxic shock syndrome.<br />

2. Analgesic agents and decongestants are also appropriate.<br />

3. Aspirin and NSAIDs should be avoided.<br />

V. COMPLICATIONS<br />

A. May be due to the bleeding itself or as a consequence of the packs used to tamponade the bleeding<br />

B. Severe bleeding seen more commonly with posterior bleeds; transfusion may be needed if signs of<br />

cardiovascular compromise.<br />

C. Sinusitis and otitis media: due to obstruction of the sinus ostia and the eustachian tubes by nasal packing<br />

D. Toxic shock syndrome: due to the rapid growth of toxin-producing staphylococcal organisms precipitated<br />

by nasal packing<br />

E. Pressure necrosis of the columella or nasal ala: from improper padding<br />

F. Fatal airway obstruction: precipitated by accidental dislodgment of a posterior pack into the airway<br />

142


HEAD, EAR, EYE, NOSE, AND THROAT DISORDERS<br />

FACIAL INFECTIONS<br />

I. SINUSITIS<br />

A. Definition: an infection of the paranasal sinuses (ethmoid, maxillary, frontal, or sphenoid). Maxillary sinusitis<br />

is the most common. Infection of the sphenoid and frontal sinuses is least common.<br />

1. Acute sinusitis: infection 3 months duration<br />

B. Pathophysiology<br />

1. Results from occlusion of the sinus ostia<br />

2. Usually precipitated by a viral upper respiratory infection or allergic rhinitis<br />

3. Ostial obstruction - ideal culture medium in occluded sinus -a- secondary bacterial infection (increased<br />

likelihood after 10-14 days of symptoms)<br />

C. Etiology<br />

1. Acute sinusitis: viral etiology in 98%-99.5%<br />

2. Antibiotics are rarely indicated in immunocompetent adults with symptoms for 50%) or polymicrobial<br />

D. Clinical presentation<br />

1. Two cardinal symptoms: purulent rhinorrhea and nasal obstruction; other symptoms below are also suggestive.<br />

2. Headache or facial pain/pressure in a distribution defined by the sinuses involved<br />

3. Swollen erythematous nasal mucosa<br />

4. Purulent yellow-green nasal discharge<br />

5. Low-grade fever<br />

E. Diagnostic evaluation<br />

1. In most instances, the diagnosis of acute sinusitis can be made based on the history.<br />

2. Use a topical decongestant (eg, 1 % phenylephrine or oxymetazoline) to better visualize purulent secretions on<br />

rhinoscopy.<br />

3. If the diagnosis is uncertain (or complications are suspected), imaging studies may be indicated.<br />

4. CT of the sinuses is the most sensitive technique: the "gold standard" for diagnosing sinus disease and its<br />

comp/ ications.<br />

5. Plain films are no longer used in diagnosis of sinus disease.<br />

F. Management<br />

1. Topical decongestants (eg, oxymetazoline or phenylephrine) for a maximum of 3-5 days to avoid rebound<br />

effect<br />

2. Oral decongestants (eg, pseudoephedrine) for 2-4 weeks<br />

3. Avoid antihistamines while acutely ill.<br />

4. Reserve antibiotic therapy for patients with moderate to severe sinusitis with symptoms persisting over 10<br />

days and specific findings of acute sinusitis (nasal congestion and purulent nasal discharge). Amoxicillin and<br />

TMP-SMX are favored by many as first-line agents. However, in regions with a high percentage of ~-lactamaseproducing<br />

bacteria, a broad-spectrum agent should be selected: amoxicillin-clavulanate or a second- or<br />

third-generation cephalosporin (eg, cefuroxime axetil, cefprozil, cefaclor, cefpodoxime, proxetil, loracarbef).<br />

Antibiotics should be continued for 10-14 days.<br />

5. Adjunctive therapy<br />

a. Analgesics<br />

b. Mucolytics (eg, guaifenesin) and decongestants (pseudoephedrine preferred over phenylephrine)<br />

c. Saline irrigations<br />

d. Home humidifiers<br />

143


HEAD, EAR, EYE, NOSE, AND THROAT DISORDERS<br />

G. Complications<br />

1. Ethmoid sinusitis can lead to periorbital/orbital cellulitis and abscess formation (especially in children).<br />

2. Frontal sinusitis may extend anteriorly and posteriorly.<br />

a. If it extends to the anterior table of the frontal bone, it may produce osteomyelitis of the surrounding bone<br />

("Pott's puffy tumor"). These patients present with a doughy-feeling, tender mass above their eye.<br />

b. If it extends to the posterior table of the frontal bone, it may cause osteomyelitis, which can lead to:<br />

(1) Meningitis<br />

(2) Epidural abscess<br />

(3) Subdural empyema<br />

(4) Brain abscess<br />

3. Sphenoid or ethmoid sinusitis can extend intracranially via the vascular or lymphatic channels of these sinuses,<br />

thus producing cavernous sinus thrombosis.<br />

4. Clinical presentation<br />

a. Proptosis<br />

b. Eyelid edema<br />

c. Dilatation of the episcleral veins<br />

d. Palsies of cranial nerves Ill-VI<br />

e. Venous engorgement of the fundus<br />

f. Decreased mental status<br />

g. High fever<br />

h. Headache<br />

5. Patients demonstrating these complications require admission, parenteral antibiotics, and appropriate consults.<br />

II. PAROTITIS<br />

A. An infection of the parotid gland that can be bacterial (usually Staphylococcus aureus) or viral (eg,<br />

paramyxovirus)<br />

B. Suppurative parotitis<br />

1. Most common in older, debilitated, or postoperative patients with decreased salivary flow due to drugs,<br />

dehydration, or irradiation<br />

2. Etiology: 5 aureus (most common) but often polymicrobial<br />

3. Clinical presentation<br />

a. Tender parotid swelling (often unilateral)<br />

b. Trismus<br />

c. Purulent discharge from the opening of Stensen (parotid) duct (opens into the mouth adjacent to the<br />

maxillary second molar)<br />

d. Fever<br />

4. Management<br />

a. An antistaphylococcal antibiotic (eg, clarithromycin has good salivary penetration); consider Klebsiella,<br />

Pseudomonas, and anaerobic coverage if ill or diabetic.<br />

b. Hydration<br />

c. Local heat<br />

d. Massage<br />

e. Sialogogues (eg, sour candy, lemon juice)<br />

C. Paramyxovirus ("mumps")<br />

1. Epidemiology<br />

a. Most common in winter and spring in children 5-15 years old<br />

b. Rare because of childhood immunizations<br />

2. Clinical presentation<br />

a. Incubation period of 12-25 days<br />

144


HEAD, EAR, EYE, NOSE, AND THROAT DISORDERS<br />

b. Characteristic clinical findings<br />

(1) Tender parotid swelling at the angle of the jaw (frequently bilateral)<br />

(2) Low-grade fever, headache, malaise<br />

(3) Clear saliva from Stensen (parotid) duct<br />

c. Occasionally, the infection may involve:<br />

(1) Gonads (epididymitis, orchitis)<br />

(2) Meninges (meningoencephalitis)<br />

(3) Pancreas (pancreatitis)<br />

3. Management<br />

a. Hydration<br />

b. Analgesics<br />

c. Antipyretics<br />

d. Sialogogues (eg, lemon drops) to encourage salivary flow<br />

4. Infected individuals are contagious up to 6 days before to 9 days after the onset of parotid swelling.<br />

145


HEAD, EAR, EYE, NOSE, AND THROAT DISORDERS<br />

FACIAL FRACTURES<br />

In facial injuries, consider the possibility of concomitant cervical spine injury.<br />

I. MANDIBULAR INJURIES<br />

A. Hallmarks of mandibular dysfunction: limited opening or deviation on opening of the mouth, malocclusion,<br />

pain, and trismus<br />

B. Mandibular fractures<br />

1. Second most commonly fractured facial bone (nose is the first)<br />

2. Because of its ringlike structure, the mandible fractures in two or more places in most cases (so don't stop<br />

looking after finding the first fracture).<br />

3. The most common sites of fracture are the condyle, the body, and the angle.<br />

4. Clinical presentation<br />

a. Most important question to ask is whether the patient feels that his or her teeth are not coming together as<br />

they used to when biting down; this indicates malocclusion.<br />

b Teeth that are angulated and sometimes avulsed ---'> alveolar fractures<br />

c. Lateral crossbite - unilateral condylar fractures<br />

d. Displacement of the lower incisors, interruption of arch continuity - symphysis fractures<br />

e. Ecchymosis or hematoma of the floor of the mouth---'> highly suspicious for a mandibular fracture<br />

f. Anesthesia of the lower lip - injury of the inferior alveolar or mental nerve secondary to a mandibular<br />

fracture<br />

5. Diagnostic evaluation<br />

a. A dental panoramic view of the maxilla and mandible is the best study for diagnosing mandibular fractures.<br />

b. If a dental panoramic view cannot be done, a maxillofacial CT should be obtained.<br />

6. Management<br />

a. Mandibular fractures require consult with an ENT or a plastic or oral surgeon for reduction and fixation.<br />

The time course involved (immediate or delayed) is determined by the specific injury, the presence of<br />

airway compromise, and associated injuries.<br />

(1) Patients with open fractures require antibiotics, and fractures should be reduced within 24 hours. Any<br />

fracture in the tooth-bearing region should be considered "open" because the periodontal ligament<br />

communicates with the oral cavity. Penicillin is the drug of choice. Clindamycin is an alternative for<br />

penicillin-allergic patients.<br />

(2) Patients with closed fractures that are nondisplaced may be discharged to home if there is no question<br />

of airway compromise. All subcondylar fractures and fractures in edentulous patients are considered<br />

"closed" unless accompanied by a laceration.<br />

b. Tetanus prophylaxis should be updated as needed.<br />

7. Disposition<br />

a. If discharged, the patient should be placed on a liquid diet and given adequate analgesia; close follow-up<br />

is required.<br />

b. Admission is required for patients with:<br />

(1) Airway compromise (early intubation is critical)<br />

(2) Excessive bleeding<br />

(3) Severely displaced fractures (impair swallowing)<br />

(4) Grossly infected fractures<br />

(5) Comorbid disease/disability (uncontrolled diabetes, elderly)<br />

C. Mandibular dislocation<br />

1. Can result from trauma, yawning, or laughing<br />

2. The jaw is locked open (mandibular condyle is locked anterior to the articular eminence), and the patient has<br />

difficulty talking or swallowing.<br />

3. Bilateral dislocation presents with an anterior open bite; if the dislocation is unilateral, the jaw is displaced<br />

toward the unaffected side.<br />

146


HEAD, EAR, EYE, NOSE, AND THROAT DISORDERS<br />

4. Diagnostic evaluation<br />

a. Identify on radiograph.<br />

b. Radiographic evaluation to exclude fracture should be obtained before manipulation if the dislocation is<br />

related to trauma.<br />

Courtesy of David Effron, MO, FACEP<br />

5. Management<br />

a. The treatment is manual reduction with downward pressure applied to the posterior teeth to dislodge the<br />

condyle; the chin is then pressed posteriorly, so that the condyle returns to the fossa.<br />

b. Procedural sedation may be required if significant muscle spasm is present.<br />

c. After reduction, the patient should be placed on a soft diet, advised to avoid opening the mouth widely,<br />

and referred to an oral surgeon for follow-up. A muscle relaxant and an NSAID should also be prescribed.<br />

D. Temporomandibular joint (TMJ) dysfunction<br />

1. Clinical presentation<br />

a. Patients typically complain of dull, aching unilateral pain in the region of the TMJ that worsens as the day<br />

progresses. Referred ear pain is also common.<br />

b. On examination, there is pain on palpation of the TMJ that worsens with mouth opening and engagement<br />

of the muscles of mastication (masseter and internal pterygoid muscles). Palpable spasm of these muscles<br />

and a limited ability to open the mouth widely may also be present.<br />

2. Management<br />

a. NSAIDs, a muscle relaxant, and referral to a dentist (with a specialty in TMJ disorders) for a bite splint<br />

b. Warm moist heat to the area, limited jaw movement, and a soft diet are also helpful.<br />

II. MIDFACIAL FRACTURES<br />

A. The midfacial area includes the maxillary teeth up to the lateral ca nth us of the eye.<br />

B. Isolated zygomatic arch fractures<br />

1. Clinical presentation: if depressed, the cheek may be flattened, and opening of the mouth may be painful or limited.<br />

2. Diagnostic evaluation: these fractures are best visualized with a maxillofacial CT scan.<br />

3. Management<br />

a. Treatment is generally delayed until facial swelling has resolved but should be done within 10 days.<br />

b. Surgical reduction is indicated for compromised mandibular excursion and unacceptable cosmesis.<br />

147


HEAD, EAR, EYE, NOSE, AND THROAT DISORDERS<br />

C. Zygomatic-maxillary complex ("tripod") fractures<br />

1. Generally result from a blow to the cheek and involve fractures at three sites: the zygomatic arch,<br />

zygomaticofrontal suture, and infraorbital rim. Fracture of the lateral wall of the maxillary sinus and orbital<br />

floor is also invariably present. A fourth fracture site at the connection with the sphenoid bone can be present.<br />

2. Clinical presentation<br />

a. A palpable step-off deformity of the inferior orbital rim<br />

b. Anesthesia of the cheek, upper teeth, lip, and gum from infraorbital nerve injury<br />

c. Periorbital swelling and ecchymosis<br />

d. Flattening of the cheek (if presents before significant swelling occurs)<br />

e. Diplopia and possible enophthalmos<br />

3. Diagnostic evaluation: maxillofacial CT scanning is replacing plain radiographs in the evaluation of all facial<br />

fractures.<br />

4. Management<br />

a. Control bleeding and arrange appropriate consult with facial trauma.<br />

b. Prescribe adequate analgesia.<br />

c. In the absence of eye involvement, patients with these fractures can often be discharged to home with<br />

follow-up in 5-7 days.<br />

d. Open reduction is required for open/displaced fractures.<br />

D. Orbital floor fractures<br />

1. May occur as isolated injuries resulting from blunt ocular trauma ("blowout" fracture) or as a component<br />

of a zygomatic-maxillary complex fracture. Orbital fat, bone, and extraocular muscles may protrude into<br />

the maxillary sinus and become entrapped. (See also blowout fractures on page 132 and in the chapter on<br />

Traumatic Disorders on pages 315-31 6.)<br />

2. Clinical presentation: enophthalmos, upward gaze palsy with diplopia, hypoesthesia of the infraorbital nerve<br />

3. Diagnostic evaluation: CT scan<br />

4. Management<br />

a. All of these patients should be referred to ophthalmology for fol low-up.<br />

b. Prophylactic antibiotics should be considered for patients with evidence of sinus involvement<br />

(subcutaneous emphysema, blood in the maxillary sinus).<br />

c. Nasal precautions<br />

d. Persistent enophthalmos, visual changes, and muscle entrapment are indications for urgent surgical repair.<br />

E. Categories and characterization of multiple facial fractures<br />

1. Result from massive, direct facial trauma (eg, high-speed deceleration injuries) and may result in airway<br />

compromise<br />

2. Classification<br />

a. LeFort I (palate-facial dysjunction) is a horizontal fracture of the maxilla at the level of the nasal floor.<br />

Courtesy of David Effron, MD, FACEP<br />

b. LeFort II (pyramidal dysjunction) includes fractures through the maxilla, nasal bones, and infraorbital rim.<br />

c. LeFort Ill (craniofacial dysjunction) involves fractures through the zygomaticofrontal suture or zygoma and<br />

the frontal bone above the nose.<br />

148


HEAD, EAR, EYE, NOSE, AND THROAT DISORDERS<br />

Courtesy of David Effron, MD, FACEP<br />

3. Clinical presentation: massive soft-tissue swelling, midfacial mobility, malocclusion, and CSF rhinorrhea<br />

4. Diagnostic evaluation: maxillofacial CT is generally needed to delineate the extent and number of fractures as<br />

well as the presence of associated injuries.<br />

5. Management<br />

a. Airway management, cervical spine assessment (including associated neurologic deficits), and evaluation of<br />

the globe; vascular injury (carotid) may be associated.<br />

b. Administration of prophylactic antibiotics if the sinuses are involved<br />

c. Consult facial trauma specialist.<br />

d. Surgical reduction if weight-bearing buttresses involved<br />

149


HEAD, EAR, EYE, NOSE, AND THROAT DISORDERS<br />

ACUTE AIRWAY OBSTRUCTION<br />

I. CLINICAL PRESENTATION<br />

A. Labored respirations (tachypnea, chest retractions, nasal flaring)<br />

B. Stridor: a high-pitched crowing sound caused by airflow through a partially obstructed upper airway<br />

(larynx or trachea). Although it is most commonly heard during inspiration, it may be biphasic or<br />

expiratory when the obstruction is more distal (trachea).<br />

1. lnspiratory (supraglottic or glottic): indicates obstruction above or at the larynx, eg, epiglottitis<br />

2. Biphasic (subglottic): indicates obstruction below the larynx, eg, croup<br />

3. Expiratory: indicates lower tracheal obstruction<br />

C. Hoarseness<br />

D. Dysphagia<br />

E. Coughing<br />

F. Cyanosis<br />

G. Degree of airflow in the upper airway depends on the diameter of the airway in any given situation:<br />

t airway diameter - t resistance to airflow (by a factor of ~4).<br />

H. Oxygen saturation is not typically a useful measure in situations of suspected airway obstruction<br />

(which presents an issue of ventilation rather than of oxygenation).<br />

II. DIFFERENTIAL DIAGNOSIS<br />

A. The following conditions should be included in the differential diagnosis. Foreign body aspiration<br />

and infections are covered in more detail below.<br />

1. Foreign body aspiration<br />

2. Infections (croup, epiglottitis, bacterial tracheitis, retropharyngeal abscess)<br />

3. Angioedema (hypersensitivity to irritants, allergens, ACE inhibitors)<br />

4. Trauma (including burns, hematomas)<br />

5. Vascular anomalies<br />

6. Neoplasms<br />

B. Foreign body aspiration<br />

1. Most common in children 1-4 years old; boys are more commonly affected.<br />

2. Although any type of object may be aspirated, peanuts are the most common offending agent and one of the<br />

most inflammatory.<br />

3. The narrowest part of the upper airway is where most airway foreign bodies become lodged.<br />

a. In children__,. cricoid cartilage<br />

b. In adults__,. vocal cords<br />

4. Clinical presentation<br />

a. The presenting signs vary with the location of the foreign body and the degree of obstruction produced.<br />

(1) Aphonia __,. complete upper airway obstruction at the level of the glottis or below<br />

(2) Stridor __,. incomplete upper airway obstruction<br />

(3) Wheezing __,. incomplete lower airway obstruction<br />

(4) Coughing__,. incomplete obstruction at the supraglottic, glottic, or distal airways; cough is a<br />

presenting symptom in at least 80% of patients.<br />

b. Consider a diagnosis of foreign body aspiration in a previously well child of the appropriate age who has<br />

a history of any of the following:<br />

(1) A choking episode<br />

(2) An acute bout of paroxysmal coughing<br />

(3) Sudden onset of wheezing without an associated upper respiratory infection or past history of wheezing<br />

(4) Persistent or recurrent pneumonia<br />

150


HEAD, EAR, EYE, NOSE, AND THROAT DISORDERS<br />

5. Diagnostic evaluation<br />

a. The initial approach to the child who has aspirated a foreign body (eg, a penny) is often accomplished<br />

with plain radiographs; diagnostic laryngoscopy or bronchoscopy is sometimes required, especially when<br />

the object is small and radiolucent.<br />

b. Battery ingestion should be excluded first and foremost and, if suspected, requires emergent referral to<br />

ENT. Further assistance in identification of batteries is provided by the National Battery Ingestion Hotline<br />

(202-625-3333) and www.poison.org/battery/guideline; the "double rim" can be useful, but its absence<br />

does not exclude the presence of a battery.<br />

c. PA and lateral chest radiographs can be used to distinguish a radiopaque foreign body in the esophagus<br />

from one in the trachea. In the esophagus, a flat foreign body (like a coin) lies in the frontal/coronal<br />

plane and appears round in the PA view. A tracheal foreign body is generally oriented in the sagittal plane<br />

(aligning with the vocal cords) and appears round in the lateral view.<br />

d. The PA and lateral chest radiographs may also show indirect signs of the presence of an airway foreign<br />

body; complete bronchial obstruction produces resorption atelectasis distal to the site of obstruction,<br />

and pulmonary infiltrates may be seen in the presence of an inflammatory reaction to a foreign body<br />

(particularly vegetable matter).<br />

e. The presence of a radiolucent foreign body that is partially obstructing a mainstem bronchus can<br />

be demonstrated on an expiratory radiograph or bilateral decubitus chest radiographs (in the young<br />

uncooperative child). These views accentuate the "ball-valve" effect of a partially obstructing foreign<br />

body, thereby producing hyperinflation of the obstructed lung (due to air trapping) and a shift of the<br />

mediastinum away from this side. Review radiographs (expiratory and bilateral decubitus chest films) that<br />

demonstrate air trapping.<br />

6. Management<br />

a. Very close airway monitoring while operating room prepared<br />

b. Foreign body removal in the operating room (under anesthesia) by laryngoscopy or rigid bronchoscopy<br />

C. Infections<br />

Croup and epiglottitis are the most important. You should know these "cold" for the exam. Look up the<br />

radiographs so that you can identify these conditions on soft-tissue radiographs.<br />

1 . Croup (laryngotracheobronchitis)<br />

a. Epidemiology<br />

(1) Age group: usually 6 months to 3 years old, with a male predominance<br />

(2) Organism: virus (most commonly parainfluenza virus)<br />

(3) Season: fall and winter (particularly October through December) when the causative viruses are<br />

most prevalent<br />

(4) Site of inflammation/obstruction: glottic and subglottic area<br />

b. Clinical presentation<br />

(1) Prodrome: preceding viral upper respiratory infection of 2-3 days duration with a gradually<br />

increasing cough<br />

(2) Onset and progression: insidious (over days)<br />

(3) General appearance: nontoxic<br />

(4) Barking cough (worse at night)<br />

(5) Hoarse voice<br />

(6) Respiratory distress (tachypnea, dyspnea, retractions, and stridor)<br />

(7) Nasal discharge<br />

(8) No or low-grade fever<br />

c. Diagnostic evaluation<br />

(1) In children who present with the classic clinical picture and the characteristic barking, seal-like<br />

cough, the diagnosis can be made on clinical grounds alone.<br />

(2) When the presentation is less typical, radiographs can exclude other causes of airway compromise<br />

(eg, epiglottitis, foreign body aspiration, retropharyngeal abscess). These radiographs should be taken<br />

in the emergency department as portable studies.<br />

(3) Characteristic radiograph findings<br />

(a) PA chest radiographs - subglottic narrowing of the tracheal air column ("steeple sign")<br />

(b) Soft-tissue lateral of the neck - distended hypopharynx, normal epiglottis, and normal<br />

retropharyngeal space<br />

151


HEAD, EAR, EYE, NOSE, AND THROAT DISORDERS<br />

d. Management<br />

(1) Cool mist (use is not evidence-based)<br />

(2) Oxygen as needed<br />

(3) Hydration (oral or IV)<br />

(4) Steroids (dexamethasone 0.15-0.6 mg/kg orally or 0.6 mg/kg IM as a one-time dose) are beneficial<br />

for outpatient management.<br />

(5) Racemic epinephrine aerosol (0.5 ml of a 2.25% solution diluted in 3 ml of saline) should be<br />

administered to patients with resting stridor or respiratory distress that is not relieved by the above<br />

measures. Although evidence to support the phenomenon of "rebound" is lacking, children sick<br />

enough to receive racemic epinephrine should be observed in the emergency department for 3-4<br />

hours after therapy to assure that they do not return to their pretreatment stridorous state once the<br />

effect of racemic epinephrine wears off.<br />

(6) Admission criteria<br />

(a) Persistent stridor at rest<br />

(b) Inability to tolerate fluids<br />

(c) Unreliable social situation<br />

(d) Incomplete response to racemic epinephrine<br />

(e) Multiple doses of racemic epinephrine<br />

(f) Severe croup at presentation (even if responsive to therapeutic measures), particularly in<br />

children


HEAD, EAR, EYE, NOSE, AND THROAT DISORDERS<br />

(2) In less severe cases, diagnosis may be confirmed with a lateral soft-tissue radiograph of the neck that<br />

demonstrates the classic finding of an enlarged "thumbprint-like" epiglottis. The radiographs should<br />

be portable; if this is not feasible, someone skilled in airway management must accompany the<br />

patient to the radiography suite with airway equipment.<br />

(3) In cooperative patients with mild to moderate symptoms and a negative lateral neck radiograph (only<br />

40% sensitivity and 75% specificity for epiglottitis), careful direct visualization of the epiglottis may<br />

be performed to resolve the problem of false-negative results on lateral neck radiographs in patients<br />

with early epiglottitis. It is unlikely that this procedure will trigger acute airway obstruction in these<br />

patients. If direct inspection of the airway is not feasible, CT may be of value but should be done<br />

only in patients without airway distress.<br />

d. Management<br />

(1) Avoid agitation of the patient; allow him or her to assume a position of comfort and remain in the<br />

company of family.<br />

(2) Provide supplemental humidified oxygen.<br />

(3) Set up airway equipment at bedside.<br />

(4) Obtain immediate ENT and anesthesia consults.<br />

(5) After airway stabilization by consultants, order appropriate laboratory studies, start an IV line and<br />

fluid therapy, and begin antibiotics. Acceptable antibiotics include a second- or third-generation<br />

cephalosporin (cefuroxime, cefotaxime, ceftriaxone) or ampicillin-sulbactam.<br />

(6) Admit to ICU.<br />

(7) If H influenzae type B is isolated, household contacts require prophylaxis with rifampin, particularly<br />

if there is an unvaccinated child :S:4 years old in the contact group.<br />

3. Bacterial tracheitis (membranous laryngotracheobronchitis): a life-threatening, bacterial infection of the<br />

subglottic region that is characterized by copious tracheal secretions. It is often superimposed on a viral<br />

upper respiratory infection and has features of both croup and epiglottitis, as well as inflammation.<br />

a. Epidemiology<br />

(1) Age group: 3 months to 1 0 years old<br />

(2) Organism: most commonly Staphylococcus aureus; others include:<br />

(a) H influenzae<br />

(b) Streptococcus pyogenes<br />

(c) Moraxella catarrhalis<br />

(3) Nonseasonal<br />

b. Clinical presentation<br />

(1) Prodrome: preceding viral upper respiratory infection or croup symptoms of several days duration<br />

(2) Onset and progress: hours to days; once bacterial superinfection occurs, progression is rapid (over hours).<br />

(3) General toxic appearance<br />

(4) Barking cough (present in 50% of cases)<br />

(5) Respiratory distress (stridor and retractions)<br />

(6) High fever<br />

c. Diagnostic evaluation<br />

(1) Suspect this diagnosis when a child with symptoms of viral croup becomes acutely more toxic,<br />

develops a high fever, and has progressive respiratory distress that is unresponsive to usual croup<br />

management.<br />

(2) Direct laryngotracheobronchoscopy (performed by ENT) reveals pseudomembranes and purulent<br />

secretions and confirms the diagnosis. Removal of this debris via suctioning helps prevent airway<br />

obstruction.<br />

d. Management<br />

(1) Provide supplemental humidified oxygen.<br />

(2) Obtain immediate ENT and anesthesia consults.<br />

(3) After airway suctioning and stabilization by consultants, obtain appropriate laboratory studies<br />

(including Gram stain and culture of tracheal secretions), hydrate with IV fluids, and administer<br />

antibiotics effective against S aureus. Acceptable choices include IV nafcillin, methicillin, or oxacillin<br />

plus ceftriaxone or another third-generation cephalosporin.<br />

(4) Admit to ICU.<br />

153


HEAD, EAR, EYE, NOSE, AND THROAT DISORDERS<br />

ORAL AND PHARYNGEAL INFECTIONS<br />

Infections can occur throughout the mouth and throat, and can range from minor localized infections to deep and<br />

spreading infections that may lead to airway compromise and other complications. The deep neck spaces, including the<br />

peritonsillar space, masticator space, parotid space, submandibular space, parapharyngeal space, retropharyngeal space,<br />

mediastinum, etc, essentially all communicate with one another, and infection in one space always has the potential to<br />

spread to the others. The prevertebral space is the only deep neck space that does not communicate as directly with the<br />

others; its microbiology is therefore different and typically related to adjacent spine infection or instrumentation.<br />

I. ORAL INFECTIONS<br />

A. Masticator space abscess<br />

1. The masticator space is the area bounded by muscles of mastication (the masseter and internal pterygoid<br />

muscles).<br />

2. Infection is secondary, resulting from extension of an anterior space infection (buccal, sublingual, or<br />

submandibular spaces) or from an infection around the third molar.<br />

3. Etiology: streptococci and anaerobes<br />

4. Clinical presentation: lateral facial swelling with pain, fever, and trismus<br />

5. Management<br />

a. IV antibiotics (penicillin or clindamycin are the drugs of choice)<br />

b. Emergent ENT or oral maxillofacial surgery consult<br />

c. Admission<br />

B. Ludwig angina<br />

1. A progressive cellulitis of the floor of the mouth; the submandibular and sublingual spaces are involved<br />

bilaterally, producing induration and massive swelling that can result in airway obstruction (33% of cases).<br />

2. Common precipitants include an abscess of (or trauma to) the posterior mandibular molars (second and third<br />

molars), the roots of which communicate directly with the submandibular space. Sometimes, infection of the<br />

anterior teeth leads to sublingual space infection that spreads to the submandibular space, rather than vice<br />

versa.<br />

3. Etiology: combination of anaerobic (Bacteroides spp) and aerobic (streptococci, staphylococci) oral flora<br />

4. Clinical presentation: dysphagia, odynophagia, dysphonia, trismus, drooling, neck and sublingual pain,<br />

massive swelling/induration of the floor of the mouth and anterior neck that is brawny in character, fever,<br />

and an elevated tongue<br />

5. Management<br />

a. If possible, allow the patient to maintain a sitting position; supine positioning can result in sudden airway<br />

obstruction.<br />

b. Set up airway management equipment, including tracheostomy tray, at bedside; the most frequent cause<br />

of death in these patients is asphyxiation.<br />

c. Obtain immediate anesthesia and ENT or oral maxillofacial surgery consults.<br />

d. Administer parenteral antibiotics (penicillin and metronidazole, cefoxitin, clindamycin, ampicillinsulbactam,<br />

or ticarcillin/clavulanate).<br />

e. Admit to ICU.<br />

6. Complications<br />

a. Airway compromise<br />

b. Extension of infection into the deeper layers of the neck or into the thoracic cavity (mediastinitis,<br />

mediastinal abscess)<br />

154


HEAD, EAR, EYE, NOSE, AND THROAT DISORDERS<br />

II. PHARYNGEAL SPACE INFECTIONS<br />

Review soft-tissue imaging of the neck with a radiologist. Ask him or her to teach you the normal size limits of<br />

the following spaces: parapharyngeal, peritonsillar, retropharyngeal, and prevertebral. Then look at imaging that<br />

demonstrates infection or abscess formation in these spaces.<br />

A. Parapharyngeal abscess<br />

1. Occurs in the space lateral to the pharynx and medial to the masticator space; this space extends from the base<br />

of the skull to the hyoid bone.<br />

2. Common precipitants of abscesses in this region are dental, pharyngeal, and tonsillar infections.<br />

3. Etiology: Streptococcus viridans is the most commonly isolated organism from any deep neck space infection,<br />

but a combination of anaerobes and aerobes is common, and the specific organisms isolated may reflect the<br />

original site of infection.<br />

4. Clinical presentation: neck pain, sore throat, dysphagia, odynophagia, unilateral swelling of the neck and<br />

angle of the mandible, restricted movement of the neck, torticollis, pharyngitis, bulging of the pharyngeal wall,<br />

drooling, cervical adenopathy, and fever<br />

5. Management<br />

a. Emergency airway equipment set up at bedside<br />

b. Parenteral antibiotics (same as for Ludwig angina)<br />

c. Consider steroids in nonimmunocompromised hosts<br />

d. Emergent ENT consult<br />

e. Hospital admission<br />

6. Complications: airway obstruction, spread of infection into the surrounding spaces, cranial nerve neuropathies<br />

(IX-XII), carotid artery erosion, and septic thrombosis of the internal jugular vein<br />

B. Peritonsillar abscess<br />

1. Occurs between the tonsillar capsule and the superior constrictor muscle; it is usually the complication of<br />

untreated or partially treated suppurative tonsillitis and is most frequently seen in teenagers and young adults.<br />

2. Etiology<br />

a. Usually a polymicrobial infection caused by a mixture of aerobes and anaerobes.<br />

b. Group A 13-hemolytic streptococci is the predominant species. Other Streptococcus spp, Haemophilus<br />

influenzae type B, Staphylococcus, Bacteroides, and Fusobacterium spp are less frequent pathogens.<br />

3. Clinical presentation<br />

a. Sore throat (?.2 days and localized to one side)<br />

b. Dysphagia and odynophagia-.. drooling<br />

c. Muffled ("hot potato") voice<br />

d. Inferior and medial displacement of the involved tonsil<br />

e. Soft palate fluctuance<br />

f. Deviation of the uvula to the opposite side<br />

g. lpsilateral ear pain<br />

h. Trismus<br />

1. Fever<br />

j. Tender cervical adenopathy<br />

k. Foul breath odor<br />

4. Diagnostic evaluation: if you are unable to visualize the abscess, ultrasound can help identify it. CT can also be<br />

used but is rarely necessary.<br />

5. Management<br />

a. Incision and drainage<br />

(1) Abscess evacuation by needle aspiration or by incision and drainage. Needle aspiration may be<br />

performed by the emergency physician if trained in the technique; incision and drainage is usually<br />

performed by the ENT consultant.<br />

(2) Gram stain and culture of aspirated fluid should be ordered.<br />

b. IV hydration<br />

c. Parenteral antibiotics (penicillin, ampicillin-sulbactam, clindamycin, cefoxitin, or erythromycin)<br />

d. Dexamethasone<br />

155


HEAD, EAR, EYE, NOSE, AND THROAT DISORDERS<br />

6. Disposition<br />

a. After abscess evacuation, many patients have significant relief of their symptoms and may be discharged<br />

to home on oral antibiotics (amoxicillin-clavulanate, clindamycin, or a secondgeneration cephalosporin).<br />

ENT follow-up should be available in event of relapse. Tonsillectomy is not indicated for history of a single<br />

episode of peritonsillar abscess.<br />

b. If the abscess cannot be drained or the patient remains symptomatic, appears toxic, or cannot tolerate<br />

fluids, he or she should be admitted to ENT and continued on IV antibiotics.<br />

C. Retropharyngeal abscess<br />

1. Occurs in the space posterior to the constrictor muscles of the pharynx. Specifically, it is between two layers of<br />

the deep cervical fascia: the alar fascia (a layer of the prevertebral fascia) posteriorly and the buccopharyngeal<br />

fascia (a portion of the pretracheal fascia) anteriorly. The danger space is immediately posterior to the<br />

retropharyngeal space and is the means by which these infections can spread to the mediastinum if they<br />

penetrate the alar fascia. It most commonly affects children 6 months to 3 years old.<br />

2. Etiology: Staphylococcus aureus, group A ~-hemolytic streptococci, and anaerobes are the most common<br />

pathogens.<br />

3. Clinical presentation<br />

a. Sore throat<br />

b. Dysphagia __.. refusal to eat and drooling<br />

c. Labored respirations and (sometimes) stridor<br />

d. Muffled voice<br />

e. Fever<br />

f. Unilateral bulging of the posterior pharyngeal wall<br />

g. Swelling of the anterolateral neck with limited cervical mobility<br />

h. Tender anterior cervical adenopathy<br />

1. Systemic toxicity<br />

j. A preference for the supine position with the head and neck held in extension<br />

k. The presence of chest pain suggests that mediastinal extension has occurred.<br />

4. Diagnostic evaluation<br />

a. A soft-tissue lateral radiograph of the neck is the initial study of choice. Supportive findings include<br />

widening of the retropharyngeal space with anterior displacement of the larynx and the presence of air or<br />

an air-fluid level in this space.<br />

(1) The normal width of the retropharyngeal space is less than half the width of the adjacent vertebral body.<br />

(2) The radiograph must be taken during inspiration with neck in slight extension: the expiratory phase<br />

of respiration and neck flexion __.. buckling and redundancy of the retropharyngeal tissues - falsepositive<br />

result.<br />

b. The presence of chest pain should raise suspicion of mediastinal extension, and plain chest radiographs or<br />

a chest CT should be obtained.<br />

5. Management<br />

a. Airway stabilization equipment at bedside<br />

b. IV hydration<br />

c. Parenteral antibiotics<br />

(1) A penicillinase-resistant penicillin and metronidazole or<br />

(2) Clindamycin or<br />

(3) Cefoxitin or<br />

(4) Ampicillin/sulbactam<br />

d. Emergent ENT consult for incision and drainage<br />

e. Hospital admission<br />

6. Complications: airway obstruction, aspiration, invasion of contiguous structures (mediastinum, vessels), and sepsis<br />

D. Prevertebral infection<br />

1. Occurs in the space posterior to the prevertebral fascia and anterior to the cervical spine; this space extends<br />

al I the way to the coccyx. In the absence of CT or MRI, it can be hard to differentiate from retropharyngeal<br />

infection except by patient age and any relevant medical history; retropharyngeal space infection is more<br />

common in an otherwise healthy child.<br />

156


HEAD, EAR, EYE, NOSE, AND THROAT DISORDERS<br />

2. Usually results from surgical instrumentation, hematologic seeding, or cervical osteomyelitis, eg,<br />

Staphylococcus or tuberculosis; different microbiology than retropharyngeal or other deep neck space<br />

infections, and no odontogenic causes.<br />

3. Clinical presentation: may include bulging of the pharynx (usually bilateral) and tenderness of the cervical<br />

spine on palpation<br />

4. Diagnostic evaluation<br />

a. Lateral neck radiographs revealing retropharyngeal swelling or osteomyelitis of the cervical spine are<br />

suggestive.<br />

b. CT, MRI, or a cervical myelogram are needed to confirm the diagnosis.<br />

5. Management<br />

a. ICU admission<br />

b. Parenteral antibiotics<br />

c. Neurosurgical consult<br />

157


HEAD, EAR, EYE, NOSE, AND THROAT DISORDERS<br />

PHARYNGITIS<br />

Pharyngitis is an inflammation or infection of the mucous membranes of the oropharynx. Both noninfectious (trauma,<br />

irritant gas) and infectious (viral, bacterial, fungal) agents may be responsible. Patient complaints include dysphagia, sore<br />

throat, fever, and cervical adenopathy. Airway compromise is a potential complication. In the emergency department, the<br />

evaluation of pharyngitis is primarily aimed at the diagnosis of group A 0-hemolytic streptococci; appropriate treatment<br />

of this infection limits its spread, decreases the incidence of suppurative complications, prevents rheumatic fever, and<br />

hastens clinical recovery. Pharyngeal injuries, foreign bodies, and burns may present as pharyngitis.<br />

I. TRAUMA<br />

II. IRRITANT INHALANT<br />

A. Many gases and vapors irritate the throat (chlorine, steam, smoke).<br />

B. Assess respiratory effort and oxygenation; supplemental oxygen or intubation may be needed.<br />

Ill. VIRUSES<br />

Viruses are the most frequent cause of acute pharyngitis. Adenovirus, Epstein-Barr virus, influenza virus, parainfluenza<br />

virus, enteroviruses, and herpes simplex virus are commonly implicated. Cough, coryza, conjunctivitis, hoarseness,<br />

diarrhea, and stomatitis are more suggestive of viral etiology. These infections are usually self-limited and require only<br />

symptomatic treatment.<br />

A. Infectious mononucleosis<br />

1. Caused by the Epstein-Barr virus (human herpes virus 4); most commonly affects patients 10-25 years old<br />

2. Clinical presentation<br />

a. Patients classically present with sore throat, fever, malaise, and fatigue. Abdominal pain and/or left shoulder<br />

pain (Kehr sign), particularly if associated with dizziness, nausea, and left upper quadrant tenderness,<br />

suggests splenic rupture (a serious complication).<br />

b. Examination reveals an exudative pharyngitis and tender cervical adenopathy. Posterior cervical<br />

adenopathy is characteristic and helps distinguish infectious mononucleosis from other causes of<br />

pharyngitis. Hepatosplenomegaly is also common; splenic rupture may occur with minor trauma.<br />

3. Diagnostic evaluation<br />

a. Peripheral blood smear reveals a lymphocytosis (>50%) and an increase in the proportion of atypical<br />

lymphocytes (> 1 0%).<br />

b. A monospot test (heterophile antibody) confirms the diagnosis if positive but takes 2-7 days. False-negatives<br />

may also be seen, particularly in children


HEAD, EAR, EYE, NOSE, AND THROAT DISORDERS<br />

IV. BACTERIA<br />

A. Group A (3-hemolytic streptococci (GABHS)<br />

1. Epidemiology<br />

a. Often seen in late winter<br />

b. Usually in patients


HEAD, EAR, EYE, NOSE, AND THROAT DISORDERS<br />

C. Diphtheria<br />

1. This rare but serious cause of pharyngitis typically occurs secondary to noncompliance with diphtheriapertussis-tetanus<br />

(DPT) immunizations. Spread is primarily by contact with respiratory secretions, and the<br />

period of incubation is~ 1 week.<br />

2. Morbidity is due to both infectious and toxic reactions.<br />

a. Infectious invasion causes enough tissue necrosis to produce a pseudomembrane in the posterior pharynx<br />

that can progressively enlarge and lead to airway obstruction.<br />

b. Corynebacterium diphtheriae elaborates a powerful exotoxin that can cause widespread organ damage.<br />

(1) Myocarditis/AV block/endocarditis<br />

(2) Nephritis<br />

(3) Hepatitis<br />

(4) Neuritis with both bulbar and peripheral paralysis<br />

(a) The most commonly observed paralysis involves the intrinsic and extrinsic muscles of the eyes,<br />

leading to ptosis and strabismus.<br />

(b) Involvement of the palate produces a change in voice quality and difficulty speaking. (The<br />

muscles of the palate are usually the first to become paralyzed).<br />

(c)<br />

3. Clinical presentation<br />

Limb paralysis and loss of deep tendon reflexes may also occur.<br />

a. The patient has an acute onset of sore throat, fever, and general malaise, and appears toxic and is<br />

tachycardic. His or her voice is hoarse, muffled, or even absent.<br />

b. Physical examination reveals an exudative pharyngitis with a white to gray, closely adherent<br />

pseudomembrane, marked cervical adenopathy ("bul I neck"), and fetid breath ("dirty mouse" smel I).<br />

A serosanguineous nasal discharge may also be present.<br />

4. Diagnostic evaluation<br />

a. Gram stain of pharyngeal swab specimen reveals gram-positive rods with clubbing.<br />

b. Culture on Loeffler or tellurite media is positive.<br />

c. CBC may show thrombocytopenia.<br />

5. Management<br />

D. Gonorrhea<br />

a. ABC stabilization<br />

b. Definitive treatment is aimed at both the bacteria and the exotoxin and should be started as soon as the<br />

diagnosis is suspected. Do not wait for a positive culture report.<br />

(1) Parenteral penicillin (or erythromycin in penicillin-allergic patients) and<br />

(2) Diphtheria antitoxin<br />

c. All patients should be hospitalized with respiratory isolation.<br />

d. Recommendations for close contacts<br />

(1) Asymptomatic, immunized contacts should be given a tetanus-diphtheria booster if >5 years have<br />

elapsed since their last dose.<br />

(2) Asymptomatic, partially immunized or unimmunized contacts must receive one dose of IM penicillin (7-10<br />

days of oral erythromycin may be used in the penicillin-allergic patient) and begin the immunization series.<br />

1. Should be considered in adolescents and adults who engage in orogenital sex.<br />

2. When it occurs in young children, sexual abuse should be suspected.<br />

3. Treatment is with ceftriaxone (ciprofloxacin resistance is increasingly common). A regimen that is effective against<br />

Chlamydia (eg, azithromycin, doxycycline) should also be prescribed because of the possibility of co-infection.<br />

V. FUNGI<br />

A. Fungi, such as Candida, Cryptococcus, and Histop/asma, can produce pharyngitis in immunocompromised<br />

patients.<br />

B. Candida a/bicans overgrowth<br />

1. Most common in immunocompromised patients (eg, HIV, chemotherapy, diabetes, chronic steroid use),<br />

patients with history of recent antibiotic and steroid courses, and neonates.<br />

2. Clinical presentation: physical examination reveals white, removable plaques on an erythematous base.<br />

3. Treatment: nystatin swish and swallow, or systemic fluconazole<br />

160


HEAD, EAR, EYE, NOSE, AND THROAT DISORDERS<br />

DENTAL EMERGENCIES<br />

I. NEUROANATOMY OF THE FACE<br />

Sensory innervation of the face and mouth is via the three branches of the trigeminal nerve (cranial nerve V): ophthalmic<br />

branch, maxillary branch, and the mandibular branch. The detailed anatomy of these branches is somewhat complex,<br />

especially that of the maxillary branch. The schematics that follow have been simplified to facilitate the learning process.<br />

A. Ophthalmic branch of the trigeminal nerve<br />

Ophthalmic branch of the trigeminal nerve<br />

Supraorbital nerve*<br />

Nasociliary nerve<br />

Forehead and vertex<br />

of the scalp<br />

Cornea and dorsal nose<br />

*Involved in nerve blocks<br />

B. Maxillary branch of the trigeminal nerve<br />

Maxillary branch of the trigeminal nerve<br />

Nasopalatine and greater<br />

(anterior) palatine nerves<br />

Hard palate and<br />

adjacent gingiva<br />

I<br />

I<br />

Anterior<br />

I / '------,---~<br />

V<br />

Maxi I lary central,<br />

lateral, and<br />

cuspid teeth<br />

Superior alveolar nerves<br />

Middle<br />

First and second<br />

bicuspid teeth (and<br />

part of first molar)<br />

*Posterior<br />

Maxillary molar<br />

teeth (except<br />

a part of the<br />

first molar)<br />

lnfraorbital nerve*<br />

Midface, maxillary incisors and premolar teeth,<br />

upper lip, lower eyelid, side of the nose, and part<br />

of the first molar<br />

*Involved in nerve blocks<br />

161


HEAD, EAR, EYE, NOSE, AND THROAT DISORDERS<br />

C. Mandibular branch of the trigeminal nerve<br />

Mandibular branch of the trigeminal nerve<br />

Buccal nerve Lingual nerve Inferior alveolar nerve*<br />

Buccal mucous<br />

membrane and<br />

mucoperiosteum of the<br />

posterior<br />

mandibular teeth<br />

Anterior 2/3 of the<br />

tongue<br />

Mental nerve*<br />

Chin and<br />

lower lip<br />

Mandibular<br />

teeth, lower lip,<br />

and chin<br />

*Involved in nerve blocks<br />

It. FACIAL AND ORAL ANESTHESIA<br />

A. lntraoral local anesthesia can be achieved with an anesthetic agent (eg, lidocaine or bupivacaine). Unless<br />

contraindicated, a vasoconstrictor such as epinephrine (1:100,000) is generally <strong>combined</strong> with the<br />

anesthetic agent to prolong its duration of action.<br />

B. Nerve blocks<br />

1. Supraorbital nerve block - anesthesia to ipsilateral forehead and scalp<br />

2. lnfraorbital nerve block - anesthesia to the ipsilateral area between the lower eyelid and upper lip, including<br />

the maxillary incisors, premolars, and a portion of the first molar<br />

3. Posterior superior alveolar nerve block - anesthesia to ipsilateral maxillary molars (except a portion of the first<br />

molar)<br />

4. Inferior alveolar nerve block - anesthesia to ipsilateral mandibular teeth, lower lip, and chin<br />

5. Mental nerve block - anesthesia to ipsilateral lower lip and chin; bilateral mental nerve blocks are required for<br />

midline lip lacerations due to cross-innervation.<br />

C. Supraperiosteal infiltrations provide anesthesia to individual teeth.<br />

D. Complications<br />

1. Vascular injection and spasm<br />

2. Mechanical nerve injury<br />

3. Needle misplacement resulting in incomplete anesthesia or inadvertent anesthesia of other facial structures<br />

4. Infections in peripharyngeal spaces (especially with inferior alveolar nerve block)<br />

Ill. ANATOMY OF A TOOTH<br />

A. Structural anatomy<br />

1. Two main portions: crown and root<br />

2. Enamel covers the dentin which, in turn, covers the pulp (the neurovascular supply of the tooth).<br />

3. Embedded in the alveolar bone and held in place by cementum and a periodontal ligament.<br />

B. Sensory innervation<br />

1. Maxillary teeth: via the anterior, middle, and posterior superior alveolar nerves<br />

2. Mandibular teeth: via the inferior alveolar nerve<br />

162


HEAD, EAR, EYE, NOSE, AND THROAT DISORDERS<br />

IV. DENTAL EMERGENCIES<br />

A. Trauma<br />

1. Management of tooth fractures is determined by the extent of the fracture and the patient's age.<br />

a. Ellis I<br />

(1) Only the enamel is fractured; no pain or sensitivity to hot and/or cold.<br />

(2) Treatment is elective (dentist).<br />

b. Ellis II<br />

(1) Enamel is fractured and dentin is exposed; hot and/or cold sensitivity often present.<br />

(2) Management<br />

c. Ellis Ill<br />

(a) Apply a dressing of calcium hydroxide paste over the exposed dentin, and cover it with aluminum<br />

foil or dental dry foil.<br />

(b) All patients should see a dentist for follow-up within 24 hours and be advised to avoid<br />

temperature extremes in food and drink.<br />

(1) Enamel is fractured, and both dentin and pulp are exposed; a pink tinge or drop of blood on the<br />

fracture site is characteristic (indicates pulpal exposure). Severe pain is common but may be absent if<br />

the neurovascular supply of the tooth has been disrupted.<br />

(2) This is a true dental emergency, and immediate dental referral is indicated. If a dentist or endodontist<br />

is not immediately available, place a piece of moist cotton over the exposed pulp, and cover with a<br />

piece of aluminum foil or dental dry foil. Provide tetanus immunization as needed.<br />

2. Alveolar fractures<br />

a. May be evident (exposed bone) or diagnosed per radiograph (dental panoramic view)<br />

b. May be associated with dental fractures/avulsions/subluxations<br />

c. Management<br />

(1) Immediate dental/oral surgery evaluation for reduction and fixation; wire stabilization is accomplished<br />

via the dentition.<br />

(2) Prophylactic antibiotics (penicillin or clindamycin) and tetanus immunization (if indicated)<br />

3. Avulsed tooth<br />

a. Permanent tooth<br />

(1) Holding the tooth by its crown, rinse it gently with saline and immediately replace it in the socket. Do<br />

not "brush" the tooth clean, because this will remove the periodontal ligament.<br />

(2) Viability of the tooth decreases with length of time out of the socket(~ 1 % for every 1 minute out).<br />

(3) Immediate dental referral for stabilization is indicated.<br />

(4) Prophylactic antibiotics (penicillin or clindamycin) should be prescribed, and tetanus immunization<br />

provided (if indicated).<br />

(5) If a tooth cannot be immediately reimplanted, the best storage and transplant medium is Hank's<br />

solution. Viability of the periodontal ligament can be maintained for 4~6 hours or more in this<br />

solution. Milk is a good second choice, as is the inner cheek of the mouth of a reliable patient (caution<br />

to avoid aspirating the tooth).<br />

b. Deciduous/primary tooth (children 6 months to 5 years old) should not be replaced, because alveolar<br />

ankylosis may result. These patients should be referred to a pedodontist because spacemaintaining<br />

procedures and/or appliances are sometimes required.<br />

4. Intra-oral lacerations<br />

a. Close with absorbable sutures.<br />

b. Recheck in 24-48 hours.<br />

c. Although controversial, antibiotic coverage (penicillin or clindamycin) is recommended by many,<br />

particularly if the wound is extensive or involves significant amounts of crushed tissue.<br />

B. Hemorrhage may be due to one of the following:<br />

1. Trauma<br />

2. Recent dental manipulation (cleaning, extraction, etc)<br />

3. Gingivitis<br />

4. Blood dyscrasias or coagulopathies<br />

163


HEAD, EAR, EYE, NOSE, AND THROAT DISORDERS<br />

C. Orofacial pain<br />

1. Dental pain<br />

a. Tooth eruption in adults<br />

(1) Usually involves the third molars ("wisdom teeth")<br />

(2) If the gingivae are inflamed (pericoronitis), instruct the patient to irrigate the area with warm saline or<br />

hydrogen peroxide.<br />

(3) If fluctuance and pus are present (pericoronal abscess), make a superficial incision and drain, and<br />

place the patient on an antibiotic (penicillin VK or clindamycin).<br />

(4) Refer these patients to an oral surgeon for definitive treatment (extraction).<br />

b. Dental caries<br />

(1) Oral analgesia and dental referral are usually all that is needed.<br />

(2) Suspect an associated periapical abscess if the patient complains of sharp, severe pain on tooth<br />

percussion, especially if the tooth is sensitive to hot or cold. A fluctuant swelling requires incision and<br />

drainage, an antibiotic, and warm saline rinses every 2 hours. The patient should be seen by a dentist<br />

within 24 hours.<br />

c. Postextraction pain<br />

(1) Periostitis<br />

(a)<br />

Pain within 24 hours of extraction<br />

(b) Responds well to oral analgesics<br />

(2) Alveolar osteitis ("dry socket")<br />

(a) Due to loss of clot plus localized osteomyelitis<br />

(b) Clinical presentation: history of a pain-free interval of 2-4 days after extraction followed by<br />

sudden onset of excruciating pain unrelieved by analgesics, and a foul breath odor.<br />

(c) Management<br />

i. Anesthetic nerve block<br />

ii. Irrigation of the socket<br />

iii. Packing with iodoform gauze saturated with a medicated dental paste, eugenol, or<br />

camphorated phenol provides almost immediate relief.<br />

iv. Dental follow-up in 12-24 hours<br />

d. Acute necrotizing ulcerative gingivostomatitis ("trench mouth")<br />

(1) Only periodontal lesion in which bacteria actually invade non necrotic tissue<br />

(2) Caused by Fusobacterium and spirochetes<br />

(3) Clinical presentation<br />

(a) The patient complains of pain, a metallic taste, and foul breath, which is frequently accompanied<br />

by fever, malaise, and regional lymphadenopathy.<br />

(b) Gingivae are swollen and fiery red; the interdental papillae (tissue between the teeth) are swollen,<br />

ulcerated (or "punched out"), and covered with a grayish pseudomembrane.<br />

(4) Management<br />

(a) Warm saline irrigation and hexadine rinse<br />

(b) Antibiotic (penicillin or clindamycin) for patients with fever and lymphadenopathy provides<br />

dramatic relief within 24 hours.<br />

(c) Systemic analgesics and topical anesthetic agents can provide the patient sufficient relief to use<br />

warm saline rinses, as well as eat and drink.<br />

(d) Dental follow-up is required; a potential complication is the destruction of underlying alveolar bone.<br />

2. Other causes of facial pain<br />

a. Sinusitis (see page 143)<br />

b. Trigeminal neuralgia ("tic douloureux")<br />

c. Herpes zoster (see page 118)<br />

d. lschemic heart disease (see page 36)<br />

e. TMJ disease (see page 147)<br />

f. Temporal arteritis (see page 438)<br />

g. Cluster headache<br />

164


HEAD, EAR, EYE, NOSE, AND THROAT DISORDERS<br />

HEAD, EAR, EYE, NOSE, AND THROAT DISORDERS<br />

D. Systemic diseases with oral manifestations<br />

1. Infections<br />

a. Herpes<br />

b. Coxsackie virus (hand-foot-mouth disease)<br />

c. Gonococcus<br />

d. Syphilis<br />

e. Measles<br />

2. Autoimmune/inflammatory states<br />

a. Systemic lupus erythematosus: large intraoral ulcerations with necrotic borders<br />

b. Reiter syndrome (urethritis, arthritis, conjunctivitis, oral ulcers)<br />

c. Sjogren syndrome<br />

(1) Autoimmune disease characterized by diminished secretion of the lacrimal and salivary glands<br />

(2) Symptoms include gritty sensation of the eyes, dry mouth (xerostomia), and diminished sense of taste.<br />

d. Behcet disease, lesions of inflammatory bowel disease, aphthous ulcers<br />

3. Toxic/metabolic states<br />

a. Heavy metal poisoning<br />

(1) Gingival "lead line" in lead poisoning<br />

(2) Argyria: a blue to bronze discoloration of the oral mucosa in silver poisoning<br />

b. Gingival hyperplasia<br />

(1) Diabetes mellitus<br />

(2) Phenytoin: not related to toxicity; may be secondary to alteration of calcium metabolism<br />

(3) Nifedipine: not related to toxicity; may be secondary to alteration of calcium metabolism<br />

4. Granulomatous diseases<br />

a. Tuberculosis: granulomatous ulcerations of the oral cavity<br />

b. Wegener granulomatosis: gingival hyperplasia with petechiae<br />

5. Benign tumors and tumor-like lesions<br />

a. Pyogenic granuloma<br />

(1) A proliferation of capillary-rich connective tissue that develops mostly on the lips and gingiva and<br />

commonly occurs secondary to trauma<br />

(2) Particularly common during pregnancy ("pregnancy tumor")<br />

b. Epidermoid cyst<br />

6. Blood dyscrasias<br />

a. Acute leukemia: a marked hyperplastic gingivitis (almost covers the teeth) that has a bluish red<br />

discoloration<br />

b. Thrombocytopenic purpura<br />

7. HIV/AIDS<br />

(1) Petechiae of the oral mucosa<br />

(2) Spontaneous gingival bleeding<br />

a. Oropharyngeal candidiasis<br />

(1) Painless curd-like plaques on an erythematous base that are easily removed with a tongue blade<br />

(2) Frequently one of the earliest manifestations of AIDS<br />

b. Hairy leukoplakia<br />

(1) Asymptomatic white patches with hair-like projections; most commonly located on the lateral aspect<br />

of the tongue; cannot be removed with a tongue blade<br />

(2) Believed to be associated with Epstein-Barr virus<br />

(3) 80% of patients presenting with these lesions go on to develop AIDS within the ensuing 3 years.<br />

c. Oral Kaposi sarcoma<br />

(1) Flat bluish red lesions with irregular borders; most commonly found on the hard palate but can be<br />

located anywhere in the mouth<br />

(2) Most common AIDS-related malignancy<br />

165


HEAD, EAR, EYE, NOSE, AND THROAT DISORDERS: PRACTICE CLINICAL SCENARIOS<br />

HEAD, EAR, EYE, NOSE, AND THROAT DISORDERS:<br />

PRACTICE CLINICAL SCENARIOS<br />

Answers immediately follow the practice clinical scenarios.<br />

Scenario A<br />

Presentation: An elderly patient with diabetes complains of constant and severe otalgia that is out of<br />

proportion to the examination, purulent otorrhea, and redness (inflammatory changes) of both the external<br />

auditory canal and periauricular soft tissue.<br />

What is the diagnosis?<br />

Scenario B<br />

Presentation: An elderly patient reports nearly a week of prodromal influenza-like illness with dermatomal<br />

pain, followed by a unilateral rash of the forehead, upper eyelid, and nose.<br />

What is the diagnosis?<br />

Scenario C<br />

Presentation: A 55-year-old patient with significant myopia (near sightedness) and a history of cataract<br />

removal complains of flashing lights with floaters and vision loss. He also reports a shadow that varies from<br />

light grey to completely black spreads across his visual field like a curtain being closed.<br />

What is the diagnosis?<br />

166


HEAD, EAR, EYE, NOSE, AND THROAT DISORDERS: PRACTICE CLINICAL SCENARIOS<br />

ANSWERS TO PRACTICE CLINICAL SCENARIOS<br />

Scenario A<br />

Diagnosis: malignant otitis externa<br />

Diagnostic evaluation: Laboratory studies include a CBC (leukocytes may be normal or mildly increased),<br />

a serum chemistry panel (to look for glucose intolerance) and erythrocyte sedimentation rate (invariably<br />

increased), and cultures of the otorrhea (Pseudomonas is causative organism in 95% of cases). MRI and<br />

CT are helpful in evaluating the extent of soft-tissue involvement, abscess formation, and intracranial<br />

extension. However, they are not reliable for detection of early osteomyelitis.<br />

Management: Management consists of pain control, systemic and ototopic antibiotics that cover<br />

Pseudomonas (eg, fluoroquinolones), and consult with ENT. Mild to moderate cases may be treated on<br />

outpatient basis. More severe cases typically require admission.<br />

Scenario B<br />

Diagnosis: herpes zoster ophthalmicus<br />

Diagnostic evaluation: V1 rash may present at any stage of progression from erythematous macules to<br />

vesicles to pustules that crust over within 1 week. Hutchinson sign (herpes zoster lesion at the tip, side, or<br />

root of the nose) is prognostic of ocular involvement. Corneal epithelial lesions are best seen with a slitlamp<br />

and fluorescein staining. Herpes zoster ophthalmicus is typically a clinical diagnosis, and laboratory<br />

studies are often not necessary. If unclear, a Tzanck smear and Wright stain may be obtained. Nearly 100%<br />

of the U.S. population have been infected with varicella-zoster virus by the age of 60.<br />

Management: Management consists of pain control and antiviral treatment (eg, acyclovir, valacyclovir) for<br />

7-10 days. Oral corticosteroids may be considered. Consult with ophthalmology.<br />

Scenario C<br />

Diagnosis: retinal detachment<br />

Diagnostic evaluation: Laboratory studies are often not necessary unless part of a preoperative plan. Plain<br />

radiographs, CT, and MRI are also not necessary unless traumatic injury indicates. Ocular ultrasound (of<br />

the closed eye) may be diagnostic with experienced operators.<br />

Management: Avoid any pressure on the globe and limit activity. Consult with ophthalmology.<br />

167


168<br />

NOTES


ABDOMINAL AND GASTROINTESTINAL DISORDERS<br />

ABDOMINAL AND GASTROINTESTINAL DISORDERS<br />

Upper Gastrointestinal Bleeding ............................................................................................................................... 176<br />

Evaluation .......................................................................................................................................................... 1 76<br />

Blatchford Score ................................................................................................................................................. 1 77<br />

Specific Treatment of Ruptured Varices ............................................................................................................... 1 77<br />

Treatment of Bleeding Due to Peptic Ulceration ................................................................................................. 177<br />

Esophageal Disorders ................................................................................................................................................ 1 78<br />

Functional Anatomy ........................................................................................................................................... 1 78<br />

Dysphagia .......................................................................................................................................................... 1 78<br />

Esophageal Trauma ............................................................................................................................................. 180<br />

Esophageal Foreign Body Ingestion ..................................................................................................................... 181<br />

Peptic Ulcer Disease ................................................................................................................................................. 183<br />

Pathophysiology ................................................................................................................................................. 1 83<br />

Predisposing Factors ........................................................................................................................................... 183<br />

Clinical Presentation .......................................................................................................................................... 183<br />

Diagnostic Evaluation ......................................................................................................................................... 183<br />

Management of Uncomplicated Peptic Ulcer Disease ........................................................................................ 184<br />

Comp I ications .................................................................................................................................................... 1 85<br />

Perforated Viscus ....................................................................................................................................................... 185<br />

Gallbladder Perforation ...................................................................................................................................... 185<br />

Small-Bowel Perforation ..................................................................................................................................... 186<br />

Large-Bowel Perforation ..................................................................................................................................... 186<br />

Management of a Known or Suspected Perforated Viscus ................................................................................... 186<br />

Acute Abdomen ........................................................................................................................................................ 187<br />

Appendicitis ....................................................................................................................................................... 187<br />

Bowel Obstruction ............................................................................................................................................. 1 89<br />

Volvulus ............................................................................................................................................................. 191<br />

Mesenteric Vascular lschemia/lnfarction ............................................................................................................. 191<br />

Hernias .............................................................................................................................................................. 193<br />

Ileitis and Colitis ................................................................................................................................................. 194<br />

Irritable Bowel Syndrome ................................................................................................................................... 197<br />

Colonic Diverticular Disease .............................................................................................................................. 198<br />

Anorectal Disorders .................................................................................................................................................. 199<br />

Hemorrhoids ...................................................................................................................................................... 1 99<br />

Anal Fissure ........................................................................................................................................................ 200<br />

Peri rectal (Anorectal) Abscesses .......................................................................................................................... 200<br />

Miscellaneous Anorectal Disorders ..................................................................................................................... 201<br />

Fistula-in-Ano ............................................................................................................................................. 201<br />

Rectal Prolapse ........................................................................................................................................... 201<br />

Pi lonodal Sinus ........................................................................................................................................... 202<br />

Anorectal Tumors ........................................................................................................................................ 202<br />

Rectal Foreign Bodies .................................................................................................................................. 202<br />

Diarrhea and Food Poisoning .................................................................................................................................... 203<br />

General Information ........................................................................................................................................... 203<br />

Viral Diarrheal Diseases ..................................................................................................................................... 203<br />

Bacterial Diarrheal Diseases ............................................................................................................................... 203<br />

Parasitic Gastrointestinal Infections ..................................................................................................................... 212<br />

Diarrhea in the Al DS Patient ............................................................................................................................... 21 5<br />

Biliary Tract Disorders ............................................................................................................................................... 216<br />

General Information ........................................................................................................................................... 216<br />

169


ABDOMINAL AND GASTROINTESTINAL DISORDERS<br />

Cholelithiasis ...................................................................................................................................................... 216<br />

Cholecystitis ....................................................................................................................................................... 21 6<br />

Variations and Comp I ications of Cholecystitis .................................................................................................... 21 7<br />

Clinical Presentation .......................................................................................................................................... 217<br />

Diagnostic Evaluation ......................................................................................................................................... 218<br />

Management ...................................................................................................................................................... 21 8<br />

Hepatitis ................................................................................................................................................................... 218<br />

Viral Hepatitis .................................................................................................................................................... 218<br />

Toxic Hepatitis ................................................................................................................................................... 221<br />

Alcoholic Liver Disease ............................................................................................................................................. 222<br />

Syndromes ......................................................................................................................................................... 222<br />

Comp I ications .................................................................................................................................................... 223<br />

Pancreatitis ............................................................................................................................................................... 224<br />

Etiology .............................................................................................................................................................. 224<br />

Clinical Presentation .......................................................................................................................................... 224<br />

Diagnostic Evaluation ......................................................................................................................................... 224<br />

Management ...................................................................................................................................................... 225<br />

Poor Prognostic Signs ......................................................................................................................................... 225<br />

Comp I ications .................................................................................................................................................... 22 6<br />

170


ABDOMINAL AND GASTROINTESTINAL DISORDERS: SELF-ASSESSMENT QUESTIONS<br />

ABDOMINAL AND GASTROINTESTINAL DISORDERS:<br />

SELF-ASSESSMENT QUESTIONS<br />

1. The most common cause of oropharyngeal (transfer) dysphagia is:<br />

(a)<br />

Obstructive lesions<br />

(b) Spasm<br />

(c)<br />

Neuromuscular disorders and inflammatory lesions<br />

(d) Carcinoma<br />

2. All of the following statements regarding Boerhaave syndrome are accurate except:<br />

(a)<br />

It is usually the result of sudden, violent, repeated vomiting.<br />

(b) It is a partial-thickness tear of the esophageal wall.<br />

(c) Associated physical findings with this entity include subcutaneous emphysema and the presence of a<br />

mediastinal crunch (Hamman sound).<br />

(d) Radiographic findings may include a left pneumothorax, a left pleural effusion, mediastinal emphysema, and a<br />

widened mediastinum.<br />

3. The most common area for an esophageal foreign body to lodge in an adult is the:<br />

(a) Aortic arch (T4)<br />

(b) Cricopharyngeal muscle (C6)<br />

(c) Lower esophageal sphincter/diaphragmatic hiatus (Tl 0-Tl 1)<br />

(d) Tracheal bifurcation (T6)<br />

4. Which of the following is 0the most appropriate therapy for a button battery lodged in the esophagus?<br />

(a) Observation<br />

(b) Removal with a Foley catheter<br />

(cl IV glucagon<br />

(d) Immediate removal via endoscopy<br />

5. What is the most likely diagnosis in patients with abdominal pain that awakens them at night and is relieved by food<br />

intake and ingestion of antacids?<br />

(a)<br />

Duodenal ulcer<br />

(b) Myocardial ischemia<br />

(c) Gastric ulcer<br />

(d) Perforated peptic ulcer<br />

6. Factors that have definitely been demonstrated to predispose to peptic ulcer disease include all of the following except:<br />

(a) Alcohol ingestion<br />

(b) Cigarette smoking<br />

(c) Type O blood<br />

(d) Use of NSAIDs or aspirin<br />

7. Which of the following statements regarding sucralfate is accurate?<br />

(a) It neutralizes gastric acid.<br />

(b) It works most effectively at a gastric pH of 4.5.<br />

(c) It inhibits secretion of mucus.<br />

(d) It inhibits pepsin, adsorbs bile acids, and increases mucosa! prostaglandin production.<br />

8. The most common cause of small-bowel obstruction is:<br />

(a)<br />

Neoplasms<br />

(b) Hernias<br />

(c) Adhesions<br />

(d) Gallstones<br />

171


ABDOMINAL AND GASTROINTESTINAL DISORDERS: SELF-ASSESSMENT QUESTIONS<br />

9. All of the following statements regarding volvulus are accurate except:<br />

(a) Although cecal volvulus can occur at any age, it most commonly affects patients in their twenties and thirties.<br />

(b) Volvulus is an example of a closed-loop obstruction.<br />

(c) Sigmoid volvulus occurs in patients with a history of chronic, severe constipation; it primarily affects older,<br />

bedridden patients with significant comorbid disease and psychiatric patients of any age.<br />

(d) Cecal volvulus is more common than sigmoid volvulus.<br />

10. The following statements regarding ulcerative colitis are accurate except:<br />

(a)<br />

Inflammation involves all layers of bowel as well as the mesenteric lymph nodes.<br />

(b) There is a much higher incidence of colon cancer in patients with ulcerative colitis than in the general<br />

population.<br />

(c)<br />

It is characterized clinically by bloody feces associated with crampy abdominal pain.<br />

(d) Complications include intestinal hemorrhage and toxic megacolon.<br />

11. Unlike small-bowel obstruction due to other causes, an important therapeutic modality in the treatment of smallbowel<br />

obstruction due to Crohn disease is:<br />

(a)<br />

Administration of antibiotics<br />

(b) Nasogastric suction<br />

(c) Administration of steroids<br />

(d) IV hydration and correction of electrolyte imbalance<br />

12. All of the following statements regarding pseudomembranous colitis are accurate except:<br />

(a)<br />

It is due to ingestion of broad-spectrum antibiotics (such as the cephalosporins or ampicillin) that alter the<br />

gut flora.<br />

(b) Although they may be delayed, symptoms usually begin within 7-10 days after antibiotic treatment is started.<br />

(c)<br />

Diagnosis is confirmed by identifying the C!ostridium difficile toxin in the feces.<br />

(d) Vancomycin is the treatment of choice for patients with mild to moderate disease.<br />

13. A patient presents with the complaint of slight hematochezia and intense rectal pain with bowel movements that<br />

linger for several hours and then resolve. The most likely diagnosis is:<br />

(a) Internal hemorrhoids<br />

(b) Anal fissure<br />

(c) Thrombosed external hemorrhoid<br />

(d) Perirectal abscess<br />

14. A patient presents to the emergency department complaining of frothy, foul-smelling diarrhea and abdominal pain.<br />

He says he feels bloated and has a lot of gas. Nine days ago he returned from a hiking trip in Colorado, where he<br />

had been drinking water from streams. The most likely cause of his diarrhea is:<br />

(a)<br />

Ciardia lamblia<br />

(b) Vibrio parahaemolyticus<br />

(c) Aeromonas hydrophila<br />

(d) Vibrio cholerae<br />

15. Except for __ , the primary mechanism by which the following bacteria induce illness is by excretion of a toxin.<br />

(a) Staphylococcus aureus<br />

(b) Bacillus cereus<br />

(c) C!ostridium difficile<br />

(d) Yersinia enterocolitica<br />

16. An adolescent presents with fever, crampy abdominal pain, and watery diarrhea. Physical examination reveals<br />

exquisite tenderness in the right lower quadrant, and a wet mount of the feces reveals WBCs. The most likely<br />

organism responsible for this presentation is:<br />

(a)<br />

Yersinia enterocolitica<br />

(b) Shigella<br />

(c) C/ostridium perfringens<br />

(d) Vibrio parahaemolyticus<br />

172


ABDOMINAL AND GASTROINTESTINAL DISORDERS: SELF-ASSESSMENT QUESTIONS<br />

17. The organism most often responsible for traveler's diarrhea is:<br />

(a) Salmonella<br />

(b) Invasive£ coli<br />

(c) Ciardia lamblia<br />

(d) Enterotoxigenic £ coli<br />

18. Staining of the feces for leukocytes is helpful in uncovering the probable cause of an acute episode of diarrhea as<br />

well as determining appropriate therapy, particularly when this information is <strong>combined</strong> with a detailed history.<br />

Leukocytes are typically present with infections induced by all of the following organisms except:<br />

(a) Clostridium difficile<br />

(b) Viruses<br />

(c) Invasive £ coli<br />

(d) Salmonella<br />

19. All of the following statements regarding scombroid fish poisoning are accurate except:<br />

(a) It is most commonly associated with ingestion of dark-fleshed or red-muscled fish.<br />

(b) It results from ingestion of toxins with histamine-like properties that form in improperly preserved or<br />

refrigerated fish.<br />

(c) Epinephrine IM should be administered.<br />

(d) Symptoms include facial flushing, conjunctiva! hyperemia, palpitations, nausea, vomiting, and diarrhea.<br />

20. The most common cause of post-transfusion hepatitis is:<br />

(a) Hepatitis A<br />

(b) Hepatitis B<br />

(c) Hepatitis C<br />

(d) Hepatitis D<br />

21. During the "window period" between the disappearance of HB 5<br />

Ag and the appearance of anti-HB 5<br />

, the only marker<br />

of hepatitis B infection that may be present in the serum is:<br />

(a) HBeAg<br />

(b) Anti-HB 0<br />

(c) Anti-HBc<br />

(d) HBCAg<br />

22. The presence of anti-HBs in the serum indicates:<br />

(a) Ongoing viral replication and high infectivity<br />

(b) Immunity<br />

(c) The carrier state<br />

(d) Low infectivity<br />

23. The presence of marked eosinophilia is typical in patients infected with:<br />

(a)<br />

Necator americanus<br />

(b) Enterobius vermicularis<br />

(c) Ciardia lamblia<br />

(d) All of the above<br />

24. A SO-year-old man with a history of chronic alcoholism presents with hematemesis. The bleeding followed an<br />

episode of violent and repeated vomiting. It was moderate in quantity and associated with pain on swallowing but<br />

has resolved on its own. The diagnostic study of choice for evaluating this patient is:<br />

(a) A chest radiograph<br />

(b) A barium esophagogram<br />

(c) A CT of the chest<br />

(d) Endoscopy<br />

173


ABDOMINAL AND GASTROINTESTINAL DISORDERS: SELF-ASSESSMENT QUESTIONS<br />

25. An 8-year-old child presents with a foreign body sensation in his throat and relates that he "accidentally swallowed"<br />

a quarter a couple of hours before presentation. The PA view of the chest reveals the flat surface of the coin. Where<br />

is the coin located?<br />

(a) In the esophagus<br />

(b) In the trachea<br />

(c) Unable to tell from the information given<br />

26. All of the following types of viral hepatitis may be associated with development of a chronic infectious state except:<br />

(a)<br />

Hepatitis B<br />

(b) Hepatitis C<br />

(c) Hepatitis delta<br />

(d) Hepatitis E<br />

27. All of the following statements regarding Helicobacter pylori are accurate except:<br />

(a) It is a gram-negative, spiral-shaped organism that colonizes only gastric mucosa.<br />

(b) It has been associated with the development of peptic ulcer disease (particularly duodenal ulcers).<br />

(c) In patients with peptic ulcer disease who are infected with this organism, eradication of the infection effectively<br />

eliminates the ulcer diathesis.<br />

(d) Single-agent therapy with either bismuth or amoxicillin is very effective in eliminating this organism.<br />

28. A 38-year-old woman presents with epigastric pain of several hours duration. She states that it started shortly after<br />

eating a meal of fried shrimp and onion rings. She recalls that she has had similar pain in the past, but states that<br />

it has never lasted this long. She is moderately obese and has a history of hypercholesterolemia for which she is<br />

taking gemfibrozil. Her past medical history is otherwise noncontributory, and there is no prior history of abdominal<br />

surgery. Examination reveals epigastric and right upper quadrant tenderness. She is afebrile, and her rectal<br />

examination is negative. The most useful initial test for evaluating this patient is:<br />

(a) A CT scan of the abdomen<br />

(b) Biliary scintiscanning (HIDA, DISIDA)<br />

(c) Abdominal ultrasonography<br />

(d) Plain radiographs of the abdomen<br />

29. The most accurate study for confirming diagnosis of acute cholecystitis is:<br />

(a) A CT scan of the abdomen<br />

(b) Biliary scintigraphy (HIDA, DISIDA)<br />

(c) Abdominal ultrasonography<br />

(d) Plain radiographs of the abdomen<br />

30. All of the following statements regarding anal canal tumors are accurate except:<br />

(a) They are defined as tumors that occur proximal to the dentate line.<br />

(b) Presenting symptoms may include rectal bleeding, decrease in fecal caliber, constipation, and weight loss.<br />

(c) They represent 80% of al I a no rectal tumors.<br />

(d) They are slow to metastasize and have a low-grade malignant potential.<br />

31. Most acute episodes of diarrhea are caused by:<br />

(a) Viruses<br />

(b) Enterotoxin-producing bacteria<br />

(c) Invasive bacteria<br />

(d) Parasites<br />

32. Which of the following statements regarding diarrhea in patients with AIDS is false?<br />

(a) Cytomegalovirus and Cryptosporidium are the two most common causes of diarrhea in these patients.<br />

(b) Multiple organisms are responsible in up to 25% of cases.<br />

(c) It is not a self-limited disease in these patients.<br />

(d) Empiric antibiotics should be administered immediately.<br />

174


ABDOMINAL AND GASTROINTESTINAL DISORDERS: SELF-ASSESSMENT QUESTIONS<br />

33. Which of the fol lowing statements regarding chest pain due to esophageal spasm is least accurate?<br />

(a) It is frequently triggered by emotional upset.<br />

(b) It can be precipitated by drinking extremely hot or cold liquids.<br />

(c) It can produce a dull discomfort, localized pressure, or a sensation of severe squeezing pressure across the<br />

middle of the chest.<br />

(d) Pain associated with esophageal spasm can be distinguished from that which occurs in association with<br />

myocardial ischemia by evaluating its response to nitroglycerin; pain of esophageal spasm is not relieved<br />

by nitroglycerin.<br />

34. The finding of __ on plain abdominal radiographs is strongly suggestive of mesenteric infarction.<br />

(a)<br />

lieus<br />

(b) Pneumatosis intestinalis<br />

(c) Gasless abdomen<br />

(d) Sentinel loop<br />

35. Evaluation of an elderly patient who presents with severe abdominal pain reveals a diagnosis of nonocclusive<br />

mesenteric ischemia. In the absence of peritonitis/necrotic bowel, definitive therapy for this patient consists of:<br />

(a) Systemic heparinization<br />

(b) Urokinase infusion<br />

(c) Intra-arterial papaveri ne infusion<br />

(d) Exploratory laparotomy<br />

36. In a patient with acute pancreatitis, which of the following laboratory reports is most likely to be associated with the<br />

highest mortality rate according to the Ranson criteria?<br />

(a) Amylase 800 units/L, AST 300 units/L, WBC count 3,000/mm 3<br />

(b) Lipase 1100 units/L, AST 350 units/L, WBC count 17,000/mm 3<br />

(c) LOH 300 units/L, ALT 200 units/L, glucose 50 mg/dL<br />

(d) LOH 400 units/L, glucose 400 mg/dL, WBC count 18,000/mm 3<br />

37. Hepatitis D is caused by:<br />

(a) An RNA virus<br />

(b) A defective RNA virus<br />

(c) A DNA virus<br />

(d) A defective DNA virus<br />

38. Which of the following laboratory findings is not consistent with a diagnosis of alcoholic hepatitis?<br />

(a) AST> ALT<br />

(b) AST and ALT levels in the thousands<br />

(c) Prolonged prothrombin time<br />

(d) Increased bilirubin and alkaline phosphatase levels<br />

39. The toxic metabolite responsible for producing hepatic necrosis in patients who overdose on acetaminophen is:<br />

(a) APAP-mercapturate and cysteine<br />

(b) APAP-sulfate<br />

(c) APAP-glucuronide<br />

(d) N-acetyl-p-benzoquinone imine (NAPQI)<br />

ANSWERS<br />

1 . C 8. C 15. d 22. b 29. b 36. d<br />

2. b 9. d 16. a 23. a 30. d 37. b<br />

3. C 10. a 17. d 24. d 31. a 38. b<br />

4. d 11 . C 18. b 25. a 32. d 39. d<br />

5. a 12. d 19. C 26. d 33. d<br />

6. a 13. b 20. C 27. d 34. b<br />

7. d 14. a 21. C 28. C 35. C<br />

Use the pre-chapter multiple choice question worksheet (page xvi) to record and determine the percentage of correct<br />

answers for this chapter.<br />

175


ABDOMINAL AND GASTROINTESTINAL DISORDERS<br />

I. UPPER GASTROINTESTINAL BLEEDING<br />

A. Evaluation<br />

1. Evaluation of the patient with a presumed upper GI bleed must start with an assessment of hemodynamic<br />

stability.<br />

a. Unstable patients require immediate resuscitation and stabilization.<br />

(1) Two large-bore (at least 18-gauge) IV lines for crystalloid infusion (2 L bolus); continue fluids until<br />

vital signs have stabilized or 40 ml/kg have been given (adults).<br />

(2) Cardiac monitoring.<br />

(3) Oxygen administration as needed.<br />

(4) Blood should be sent immediately for hemoglobin and hematocrit, platelet count, prothrombin time/<br />

INR, and type and crossmatch.<br />

(5) Immediate transfusion of type-specific or crossmatched blood (4 units minimum) if available (type 0<br />

if type-specific blood unavailable).<br />

(6) May need reversal of coagulation abnormalities; the following regimen is a Class 1 C recommendation:<br />

(a) Hold anticoagulation therapy<br />

(b) Vitamin K 10 mg (may be given IV slowly)<br />

(c) Fresh frozen plasma or<br />

(d) Prothrombin complex concentrate or<br />

(e) Activated recombinant factor VII<br />

(7) Novel anticoagulants may require specific reversal agents such as idarucizumab for reversal of<br />

dabigatran. Reversal agents for other novel anticoagulants are in development.<br />

b. Patients who remain unstable require immediate consult with a gastroenterologist for immediate<br />

endoscopy.<br />

c. Nasogastric lavage does not decrease mortality or transfusion requirements. Clear aspirate may miss up<br />

to 15 % of clinically relevant lesions.<br />

d. Administration of proton-pump inhibitors for acute undifferentiated upper GI bleed does not improve<br />

mortality. It may decrease incidence of rebleeding (controversial) and does decrease need for surgery.<br />

e. Somatostatin analog (octreotide) initiation does not decrease mortality in upper GI bleed from<br />

esophageal varices. However, administration <strong>combined</strong> with endoscopy increases initial hemostasis and<br />

decreases rebleeding.<br />

f. Restrictive transfusion strategy (transfuse only if hemoglobin 30 (LR +7.5)<br />

(4) Hematocrit 100 beats/min<br />

(3) Nasogastric lavage showing bright red blood<br />

(4) Hemoglobin 90 mg/dl, or WBC >12,000/mm 3<br />

176


ABDOMINAL AND GASTROINTESTINAL DISORDERS<br />

B. Blatchford Score<br />

1. Accurately predicts the need for urgent or emergent intervention in a patient with acute upper GI bleeding<br />

(may still require validation in U.S. hospitals)<br />

2. Does not require nasogastric lavage<br />

3. Based on hemoglobin, BUN, systolic blood pressure, pulse, melena, syncope, and history of hepatic disease<br />

or cardiac failure<br />

4. Specific scores<br />

a. Patients with upper GI bleed and a score of O have an LR of 0.02 of needing urgent intervention.<br />

b. Those with a score of :S:2 have an LR of 0.08 of needing urgent intervention.<br />

C. Specific treatment of ruptured varices<br />

1. Immediate therapy: octreotide or vasopressin infusion<br />

a. Octreotide is considered more effective than vasopressin and has fewer adverse effects.<br />

b. Vasopressin has no proven mortality benefit but may be of use in the exsanguinating patient.<br />

2. Placement of a balloon tamponade device may be used as a temporary measure while awaiting endoscopy.<br />

3. Endoscopic band ligation or sclerotherapy<br />

4. If endoscopic intervention is unsuccessful, perform angiography with embolization of the gastric vein.<br />

5. Transjugular intrahepatic portosystemic shunt placement for refractory bleeding helps prevent rebleeding.<br />

6. Surgical therapy is rarely used but may be only option in continuously bleeding patient.<br />

7. In patients with cirrhosis and upper GI bleed, prophylactic antibiotic use decreases all-cause mortality,<br />

rebleeding, death from bacterial infection, and length of hospital stay.<br />

D. Treatment of bleeding due to peptic ulceration<br />

1. Peptic ulceration is the most common cause of upper GI bleeding, even in patients with history of varices.<br />

2. Duodenal ulcers (most common): 29% of upper GI bleeds<br />

3. Gastric ulcers: 16% of upper GI bleeds<br />

4. Combined, gastric and duodenal ulcers cause 45%-50% of all upper GI bleeding.<br />

5. Management<br />

a. Volume replacement with normal saline or lactated Ringer's via two large-bore IV lines<br />

b. Oxygen as needed<br />

c. Foley catheter placement to monitor volume resuscitation<br />

d. Lavage<br />

(1) Can be used to monitor blood loss and prepare patients for endoscopy but is not helpful in stopping<br />

the bleeding or preventing its recurrence.<br />

(2) To prevent complications, only room-temperature solutions (normal saline or water) should be used;<br />

pneumoperitoneum must first be excluded.<br />

(3) Many gastroenterologists do not consider performance of gastric lavage necessary to establish the<br />

diagnosis or monitor bleeding.<br />

e. Erythromycin 250 mg IV push over 20 minutes provides satisfactory conditions for endoscopy without<br />

the need for nasogastric lavage.<br />

f. IV proton-pump inhibitor therapy should be given to any patient with active upper GI bleeding.<br />

(1) No proven benefit on mortality, but recent evidence suggests decreased rate of rebleeding and<br />

decreased need for surgical intervention.<br />

(2) IV formulations of pantoprazole and esomeprazole are available.<br />

6. Consult with a gastroenterologist and a general surgeon is prudent in any patient with significant upper GI<br />

bleeding (see above).<br />

7. If the patient does not respond to the above measures and bleeding persists, early or emergent endoscopic<br />

evaluation may be needed for both localization of the bleeding site and treatment. Although arteriography is<br />

also useful in locating the bleeding site (if the rate of bleeding is :0:0.5 ml/min), it has largely been supplanted<br />

by endoscopy, which is usually more accurate; it is currently reserved for situations in which endoscopy is<br />

unavailable or nondiagnostic.<br />

8. Surgical intervention is indicated for patients who do not respond to medical therapy and endoscopic<br />

hemostasis.<br />

9. Older patients are more likely to bleed, re-bleed, require surgery to control bleeding, and require blood<br />

products.<br />

177


ABDOMINAL AND GASTROINTESTINAL DISORDERS<br />

II. ESOPHAGEAL DISORDERS<br />

A. Functional anatomy<br />

1. The esophagus originates in the hypopharynx at the level of the cricoid cartilage (C6) and terminates in the<br />

cardia of the stomach (T11 ). It is -25 cm (1 O") long and is divided into three segments (helpful in locating<br />

foreign bodies and abnormalities of contiguous structures on radiograph).<br />

a. Upper (cervical) = 4-5 cm<br />

b. Middle (thoracic) = 15-20 cm<br />

c. Lower (abdominal)= 2-3 cm<br />

2. Layers of the esophagus<br />

a. Inner mucosa: Esophagitis involves both the inner mucosa and submucosa; if severe and prolonged,<br />

esophagitis may result in scarring and stricture formation. Treatment is dilatation. If the cause is eliminated,<br />

recurrent stricture is not a problem.<br />

b. Submucosa: Because there is no serosa, perforation of the submucosa extends into surrounding mediastinal<br />

structures and leads to a diffuse, malignant, often rapidly progressive and fatal mediastinitis.<br />

c. Muscle layers: If the two muscle layers are split by bougienage or by repeated dilatation, they scar, which<br />

leads to stricture formation. Treatment is dilatation, but strictures recur.<br />

3. Cricopharyngeal muscle (upper esophageal sphincter) is located at the level of C6.<br />

4. Muscle composition of the esophagus<br />

a. Upper third is striated muscle.<br />

b. Middle third is smooth and striated muscle.<br />

c. Distal third (and remaining GI tract) is smooth muscle.<br />

5. Neuromuscular anatomy<br />

B. Dysphagia<br />

a. Extrinsic nervous system<br />

(1) Spinal accessory nerve innervates the cervical esophagus.<br />

(2) Vagus nerve innervates the remainder of the esophagus. Stimulation of this parasympathetic nerve<br />

during esophageal intubation and endoscopy causes bradycardia.<br />

(3) Sympathetic fibers from the cervical and thoracic ganglia also innervate the esophagus. Stimulation<br />

causes dysphagia and referred pain to the chest and epigastrium.<br />

b. Intrinsic nerve supply (Auerbach and Meissner plexuses, which are contained between the muscle layers<br />

of the esophageal wall) is altered in motor disorders (achalasia, diffuse spasm) and destroyed by corrosives<br />

and some collagen vascular disorders (scleroderma, those associated with Raynaud syndrome).<br />

1. Dysphagia is defined as "difficulty swallowing"; it signifies the presence of organic pathology of the<br />

esophagus in nearly all cases. Odynophagia means "pain on swallowing"; it often indicates the presence of an<br />

inflammatory or infectious lesion.<br />

2. Oropharyngeal (transfer) dysphagia (difficulty swallowing)<br />

a. Symptoms occur within the first 2 seconds of swallowing (cough, choking, drooling, repeated swallowing<br />

attempts)<br />

b. Neuromuscular disorders account for most cases. (Difficulty swallowing liquids [particularly cold liquids]<br />

suggests a neuromuscular disorder.)<br />

(1) Neuromuscular causes (80% of cases)<br />

(a) Stroke (most common cause)<br />

(b) Polymyositis and dermatomyositis<br />

(c) Scleroderma (>50% complain of dysphagia)<br />

(d) Myasthenia gravis (reversible with edrophonium)<br />

(e) Multiple sclerosis, amyotrophic lateral sclerosis, Parkinson disease<br />

(f)<br />

Lead poisoning<br />

(2) Inflammatory/infectious causes<br />

(a)<br />

Pharyngitis (Streptococcus, Candida, herpes)<br />

(b) Oropharyngeal abscess<br />

(3) Infectious/neuromuscular causes<br />

(a) Poliomyelitis<br />

(b) Diphtheria<br />

178


ABDOMINAL AND GASTROINTESTINAL DISORDERS<br />

(c) Botulism<br />

(d) Rabies<br />

(e) Tetanus<br />

c. Cancer of the tongue, pharynx, or larynx may also cause oropharyngeal dysphagia.<br />

3. Upper esophageal dysphagia<br />

a. Difficulty swallowing occurs within the first 2-4 seconds and is caused by obstructive lesions in most cases.<br />

(1) Intrinsic causes of luminal narrowing are esophageal webs (which are associated with iron deficiency<br />

anemia, Plummer-Vinson syndrome), and carcinoma.<br />

(2) Extrinsic compression can be produced by numerous conditions, eg, thyroid enlargement, Zenker<br />

diverticulum, left atrial enlargement, aortic aneurysm.<br />

b. Dysphagia due to obstructive lesions is typically progressive (starts with solid foods and eventually<br />

progresses to liquids).<br />

4. Lower esophageal dysphagia (difficulty swallowing)<br />

a. Symptoms occur 4-10 seconds after the bolus is swallowed and are usually worse with solids.<br />

b. Clinical presentation: patients complain of a substernal "sticking" sensation and are usually able to<br />

accurately pinpoint the location.<br />

c. Lower esophageal dysphagia is usually due to luminal narrowing. This narrowing can be either constant<br />

(eg, strictures, carcinoma) or intermittent (eg, spasm).<br />

(1) Carcinoma is the most common cause of lower esophageal dysphagia. Difficulty swallowing solids<br />

occurs initially and progresses to difficulty swallowing semisolids and finally liquids.<br />

(2) Achalasia is characterized by marked increase in the resting pressure of the lower esophageal<br />

sphincter and absent peristalsis in the body of the esophagus. Dysphagia is the most common<br />

presenting complaint.<br />

(a) Patients are usually 20-40 years old.<br />

(b) A diagnostic clue is that dysphagia occurs with both solids and liquids.<br />

(c) The patient may regurgitate undigested material; no acidic taste.<br />

(3) Esophageal strictures result from esophageal reflux. There is a history of heartburn before the onset<br />

of dysphagia, which is worse with solid foods and is constant; reflux-induced spasm without stricture<br />

produces intermittent dysphagia.<br />

(4) "Steakhouse syndrome" is characterized by intense discomfort that develops shortly after swallowing<br />

a large piece of meat. It can occur in patients with a normal esophagus but is more commonly<br />

associated with one of the fol lowing if the problem has occurred repeatedly:<br />

(a) Carcinoma<br />

(b) Stricture<br />

(c) Schatzki ring (a fibrous, diaphragm-like stricture near the gastroesophageal junction); is actually<br />

an esophageal web<br />

d. Lower esophageal dysphagia and chest pain: gastroesophageal reflux disease or ischemic heart disease?<br />

Chest pain arising from the esophagus may mimic chest pain due to myocardial ischemia because of<br />

similar segmental innervation of the heart and esophagus; both may present with ST segment abnormalities<br />

on ECG. The history is by far the most useful information in differentiating these two entities.<br />

(1) A substernal burning sensation (heartburn/pyrosis) is a common complaint in patients with esophageal<br />

reflux. However, a dull discomfort, localized pressure, or severe squeezing pain across the middle of<br />

the chest may be seen when reflux is associated with esophageal spasm.<br />

(2) Exacerbation of symptoms with stooping, lying, or leaning forward suggests reflux. Postural<br />

exacerbation of pain is uncommon with ischemic heart disease.<br />

(3) Water brash (a hypersalivation response) is commonly associated with reflux.<br />

(4) Radiation of pain into the abdomen occurs more often in patients with reflux than in those with<br />

coronary artery disease. Radiation into both arms is rarely seen in reflux.<br />

(5) A feeling of fullness after meals and relief of chest pain with antacids are key points in the history. This<br />

history is rarely present in patients with ischemic heart disease.<br />

(6) Diffuse esophageal spasm without reflux may appear clinically similar to myocardial ischemia.<br />

Differentiating features include the following:<br />

(a) Emotional upset frequently triggers spasm.<br />

(b) Chest pain from esophageal spasm occurs spontaneously and most often at rest.<br />

(c) Diffuse spasm may be precipitated by swallowing liquids of extreme temperatures (hot or cold).<br />

179


ABDOMINAL AND GASTROINTESTINAL DISORDERS<br />

(d) Esophageal spasm and myocardial ischemia are both relieved by nitroglycerin. Although the latter<br />

generally responds more quickly, response to nitroglycerin is not useful in distinguishing these clinical<br />

entities.<br />

(e)<br />

If a barium swallow is done, "corkscrew" appearance.<br />

5. Esophagi tis (inflammation of the esophagus)<br />

a. Clinical presentation<br />

(1) Chest pain, dysphagia, odynophagia, or those of reflux<br />

(2) Chest pain is typically constant, acute in onset, and unresponsive to treatment with antacids.<br />

b. Etiology<br />

(1) Most common cause is gastroesophageal reflux disease (GERO)<br />

(2) Infections: typically present with severe odynophagia; may have dysphagia to solids and liquids<br />

(a)<br />

Fungal (Candida albicans)<br />

(b) Viral (herpes simplex, varicella zoster, cytomegalovirus)<br />

(c) Bacterial (mycobacterium)<br />

(d) lmmunosuppressed patients are more likely to have herpes simplex virus or cytomegalovirus as<br />

etiology because of prophylactic treatment of fungal infections.<br />

(3) Radiation<br />

(4) Corrosive agents (alkalis, acids)<br />

(5) Pill esophagitis: antibiotics (especially doxycycline and tetracycline), bisphosphonates, antiinflammatory<br />

agents, potassium chloride, and iron sulfate<br />

c. Management<br />

(1) Fluconazole is the drug of choice for esophageal candidiasis.<br />

(2) lmmunocompromised patients should be assumed to have herpes simplex and should receive acyclovir.<br />

C. Esophageal trauma<br />

1. Etiology<br />

a. Swallowing (foreign body, caustic agent)<br />

b. Instrumental (surgery, bougienage, rigid endoscopy, intubation, etc); iatrogenic perforation is the most<br />

common cause of esophageal perforation.<br />

c. Chest trauma (open, closed)<br />

d. Sudden, violent, and usually repeated increase in the intra abdominal pressure against a weakened<br />

esophageal wall; most common cause is violent and repeated vomiting/retching.<br />

(1) Mallory-Weiss syndrome<br />

(a) A partial thickness tear (mucosal lesion) of the distal esophagus or gastric cardia associated with<br />

dysphagia, odynophagia, and upper GI bleeding.<br />

(b) Clinical presentation: the prominent symptom is bleeding, which is usually mild to moderate and<br />

usually resolves spontaneously.<br />

( c) Predisposing factors<br />

i. Alcoholism<br />

ii. Hiatal hernia<br />

iii. Gastritis/esophagitis<br />

(d) Diagnostic study of choice is endoscopy.<br />

(2) Boerhaave syndrome<br />

(a)<br />

Specifically refers to spontaneous esophageal rupture, usually due to retching<br />

(b) A full-thickness tear (perforation) typically of the left posterolateral aspect of the distal esophagus<br />

(an intrinsically weak area)<br />

(c) Clinical presentation<br />

1. The prominent symptom is severe, lancinating chest pain.<br />

ii. Physical findings include subcutaneous emphysema, mediastinal "crunch" (Hamman sound),<br />

a left pneumothorax and/or pleural effusion, and possibly epigastric tenderness.<br />

(d) Diagnostic evaluation<br />

1. Chest radiograph: characteristic findings include a left pneumothorax, a left pleural effusion,<br />

mediastinal emphysema, and a widened mediastinum.<br />

ii. An esophagram using water-soluble oral contrast confirms the diagnosis.<br />

180


ABDOMINAL AND GASTROINTESTINAL DISORDERS<br />

(e) Management<br />

(f)<br />

• Water-soluble contrast causes less inflammatory response (mediastinitis) than barium but<br />

has a lower sensitivity. If an initial water-soluble contrast esophagram is negative, it should<br />

be followed by a barium study if suspicion persists.<br />

• Option following water-contrast or barium swallow if nondiagnostic is CT scan in the<br />

noncritical patient whose diagnosis is unclear.<br />

i. Fluid resuscitation<br />

ii. Broad-spectrum IV antibiotics<br />

iii. Boerhaave syndrome is a surgical emergency; do not delay surgical consult for CT or other<br />

advanced imaging.<br />

This lesion produces the most malignant type of mediastinitis (acid burn and bacterial infection).<br />

Morbidity and mortality, due to shock and septicemia, occur very rapidly (


ABDOMINAL AND GASTROINTESTINAL DISORDERS<br />

(3) Button batteries in the esophagus require immediate endoscopic removal.<br />

(4) Radiopaque esophageal foreign bodies will show on chest radiographs.<br />

(5) Some fish bones are radiolucent and not visible on plain radiographs. This is also true of plastic objects<br />

and toothpicks.<br />

(6) Radiographic evaluation of asymptomatic patients with suspected or known ingestions should also<br />

be considered, because many patients are asymptomatic; up to one-third of children with esophageal<br />

foreign bodies are asymptomatic. Newer recommendations include use of CT scan to demonstrate<br />

foreign bodies not visualized on plain radiographs.<br />

(7) If the esophageal foreign body is not detected by neck and chest radiographs, an esophagram or<br />

endoscopy should be performed.<br />

(a)<br />

Endoscopy: the diagnostic modality of choice (if readily available), because it can be used to both<br />

visualize and remove the foreign body.<br />

(b) Esophagram: if a perforation is suspected, a watersoluble contrast agent should be selected.<br />

If aspiration is a concern, barium should be used. If both perforation and aspiration are<br />

considerations, a nonionic contrast media should be used. Before the procedure, a nasogastric<br />

tube should be inserted above the obstruction to prevent aspiration of unswallowed liquids that<br />

collect there.<br />

(c) Barium should not be used if subsequent endoscopy is anticipated because it may limit or obscure<br />

visualization of the esophageal foreign body by the endoscopist.<br />

4. Although objects do occasionally lodge or become impacted in the esophagus (particularly those with sharp<br />

edges), most will pass as a matter of course. Low-risk objects (blunt, small, nonmagnetic, or button battery) can<br />

be watched in the esophagus for up to 24 hours. Once an object has passed the gastroesophageal junction, the<br />

probability of eventual passage is >90%. The foreign body usually arrives at the rectum in 3-5 days. As long as<br />

the patient is asymptomatic, management in these cases is expectant.<br />

5. Management<br />

a. Because most esophageal foreign bodies are in the cervical esophagus, removal is usually accomplished<br />

with a laryngoscope and Magill forceps or with an endoscope.<br />

b. Food impactions<br />

(1) A distal esophageal food impaction can occasionally be dislodged with IV glucagon (frequently causes<br />

nausea and vomiting), sublingual nitroglycerin, or nifedipine. IV glucagon relaxes smooth muscle and<br />

decreases lower esophageal sphincter pressure. After a test dose to exclude hypersensitivity, administer<br />

1 mg IV; an additional 2 mg may be given if there is no relief in 20 minutes. Nitroglycerin and<br />

nifedipine are rarely used because of the high incidence of adverse effects and limited efficacy.<br />

(2) Gas-forming agents (eg, effervescent granule packets, carbonated beverages) may also be used to<br />

dislodge an esophageal food impaction. They work by producing CO 2<br />

, which increases intraluminal<br />

pressure and pushes the food bolus into the stomach. These agents should be avoided in patients with<br />

chest pain (possible perforation) and in those with symptoms >24 hours duration.<br />

c. The practice of removing an impacted, smooth foreign body (such as a coin) with a Foley or Fogarty<br />

catheter requires a cooperative patient, fluoroscopic guidance, and an object that has been present for<br />


ABDOMINAL AND GASTROINTESTINAL DISORDERS<br />

h. Sharp or pointed objects, as well as objects >5 cm long and >2 cm wide, must be removed endoscopically.<br />

They should be removed before they pass the pylorus, because 15%-35% will cause perforation, usually in<br />

the region of the ileocecal valve.<br />

i. After an esophageal foreign body has been removed, esophageal function should be thoroughly evaluated<br />

(including motility studies) to ensure that there is no underlying pathology that led to the obstruction. This<br />

is particularly true of adults with meat impactions; underlying pathology of the esophagus is present up to<br />

97% of the time.<br />

Ill. PEPTIC ULCER DISEASE<br />

A. Pathophysiology<br />

1. Peptic ulcers are mucosa! defects that extend beyond the muscularis mucosae. They are most commonly<br />

located along the lesser curvature of the stomach or in the first portion of the duodenum.<br />

a. Gastric ulcers are thought to be due to damage to the gastric mucosa I barrier itself, which allows diffusion<br />

of hydrogen ions from the lumen to the gastric mucosa and results in ulceration.<br />

b. Duodenal ulcers are usually associated with hypersecretion of gastric acid (by an enlarged parietal cell<br />

mass) and with markedly increased gastric emptying that overwhelms the capacity of the duodenum to<br />

neutralize gastric acid, and results in damage to the duodenal mucosa/ barrier. Helicobacter pylori causes<br />

~90°/c, of al I duodenal ulcers.<br />

2. Uncommon sites of peptic ulceration<br />

a. Distal esophagus<br />

b. Ectopic gastric mucosa with a Meckel diverticulum<br />

c. Margins of surgical anastomoses ("marginal ulcers")<br />

3. A stress ulcer is not the same as a peptic ulcer, ie, the mucosa! lesion of a stress ulcer does not extend through<br />

the muscularis mucosae. Stress ulcers are most commonly found in the body and fundus of the stomach and<br />

are a common cause of gastric bleeding (hemorrhagic gastritis).<br />

B. Predisposing factors<br />

1. Peptic ulcer disease is considered to have two main causes: H pylori infection and NSAID usage<br />

2. Zollinger-Ellison syndrome or other gastrin-secreting tumors cause 1 %<br />

3. Cigarette smoking<br />

4. Bile salts<br />

5. Emotional stress<br />

6. Prolonged use of corticosteroids (combination of NSAIDs and corticosteroids increases risk of peptic ulcer<br />

disease 15 times)<br />

7. Caffeinated beverages (coffee, soda, tea)<br />

C. Clinical presentation<br />

1. The pain associated with gastric and duodenal (peptic) ulcers is visceral in nature and, therefore, is vague (not<br />

well defined). It is generally felt in the midline and is typically described as gnawing, aching, or burning.<br />

2. Gastric pain is generally perceived at the midline or to the left of the epigastrium.<br />

3. Duodenal bulb pain is usually perceived to the right of the midepigastrium. With posterior penetration, the<br />

pain may radiate straight through to the back.<br />

4. The patient presents with epigastric pain that may be easily confused with the substernal chest pain of<br />

myocardial ischemia. A significant diagnostic clue is the character of the pain: a patient with peptic ulcer<br />

disease will describe the pain as a burning sensation; the Ml patient generally will not. This descriptor,<br />

however, cannot reliably exclude cardiac causes of pain.<br />

D. Diagnostic evaluation<br />

1. Definitive diagnosis is generally not made in the emergency department.<br />

2. Peptic ulcers may be diagnosed with an upper GI or endoscopy. Endoscopic evaluation with biopsy is key to<br />

differentiating malignant from benign gastric ulcers and should be suspected in patients >50 years old as well<br />

as in those with markers of malignancy.<br />

3. Infection with H pylori can be diagnosed noninvasively via serology or the urea breath test, or by tests that<br />

require endoscopy (rapid urea/Cl O test, histology). An lgG H pylori serum test is also available.<br />

183


ABDOMINAL AND GASTROINTESTINAL DISORDERS<br />

E. Management of uncomplicated peptic ulcer disease is generally on an outpatient basis and may be started<br />

in the emergency department.<br />

1. Advise patients to avoid substances that exacerbate gastric and duodenal ulcers (smoking, alcohol, NSAIDs, etc).<br />

A bland diet with frequent feedings has not been demonstrated to be effective and is probably unnecessary.<br />

2. Provide pain relief and facilitate ulcer healing with one of the following agents:<br />

a. Antacids<br />

(1) Provide pain relief and accelerate healing by neutralizing gastric acid<br />

(2) Dose: 30 ml at bedtime and at 1 and 3 hours after meals<br />

(3) Their major drawback is how frequently they must be taken, which decreases compliance. They may<br />

also cause constipation (aluminum salts) or diarrhea (magnesium salts). In addition, antacids decrease<br />

the absorption of certain drugs (warfarin, digoxin, several anticonvulsants, and some antibiotics).<br />

b. Histamine (H 2<br />

)-antagonists<br />

(1) Promote healing by inhibiting gastric acid secretion<br />

(2) The major advantage of these drugs is their convenient dosing (once or twice daily).<br />

c. Proton-pump inhibitors<br />

(1) Promote healing by blocking the secretion of gastric acid<br />

(2) Inhibit the H+/K+ ATPase enzyme system (the "proton pump") of parietal cells, thereby preventing the<br />

release of hydrogen ions into the gastric lumen<br />

(3) Indicated for the short-term treatment of duodenal and gastric ulcers, and usually reserved for those in<br />

whom H 2<br />

-blockers have not been effective<br />

(4) Agents<br />

(a)<br />

d. Misoprostol<br />

Omeprazole<br />

(b) Lansoprazole<br />

(c) Pantoprazole (available in IV form; has been shown to i bleeding from peptic ulcers)<br />

(d) Esomeprazole and rabeprazole are newer agents.<br />

(1) A synthetic prostaglandin E 1<br />

analogue that acts in a way similar to that of the naturally occurring<br />

prostaglandins secreted by the gastric mucosa in response to injury.<br />

(2) Indicated only for the prevention of NSAID-induced gastric ulcers in high-risk patients (the elderly,<br />

those with concomitant debilitating disease or a history of ulcers)<br />

(3) Can cause spontaneous abortion and is, therefore, contraindicated in pregnant women and in women<br />

of childbearing age who are not using reliable contraceptive measures<br />

e. Sucralfate: drug of choice for erosive esophagitis secondary to large "stuck" pills (eg, doxycycline, docusate)<br />

(1) Works locally at the ulcer site, where it binds to the base of the ulcer, thereby protecting it from the<br />

adverse effect of gastric acid; also adsorbs bile acids, inhibits pepsin activity, and increases mucosa!<br />

prostaglandin production<br />

(2) Dosage is 1 g orally qid on an empty stomach. It works best in an acid environment (pH


ABDOMINAL AND GASTROINTESTINAL DISORDERS<br />

F. Complications<br />

1. The most common complications are bleeding (20% of ulcer patients), perforation (7%), and gastric outlet<br />

obstruction.<br />

2. Elderly patients are at greatest risk of hemorrhage.<br />

3. Ulceration into an artery can lead to life-threatening hemorrhage.<br />

4. Perforation<br />

a. Exposure of the peritoneal cavity to gastric or duodenal contents produces a chemical peritonitis that, in<br />

the absence of prompt and adequate treatment, rapidly progresses to bacterial peritonitis.<br />

b. Clinical presentation<br />

(1) History of ulcer-like pain in weeks before presentation helpful in making diagnosis.<br />

(2) Sudden onset of abdominal pain with guarding and rebound is characteristic of anterior perforations;<br />

back pain is characteristic of posterior perforation of a duodenal ulcer.<br />

(3) Pain may radiate to shoulder and or chest.<br />

(4) Posterior, perforated duodenal ulcer may appear similar to pancreatitis (but the serum lipase will be<br />

normal or slightly increased initially if the perforation is adjacent to the pancreas), or the perforation<br />

may actually cause pancreatitis.<br />

c. Diagnostic evaluation<br />

(1) Absence of free air on radiographs does not exclude the diagnosis.<br />

(a) Anterior perforations: only 60%-70% demonstrate free air.<br />

(b) Posterior perforations: no free air will be evident because the posterior duodenum is located<br />

retroperitoneal ly.<br />

(2) Diagnosis may require CT scan of abdomen or endoscopy.<br />

d. Management<br />

(1) IV fluids (normal saline or lactated Ringer's) and electrolyte replacement<br />

(2) Nasogastric tube drainage<br />

(3) Broad-spectrum IV antibiotics<br />

(4) Immediate surgical consult<br />

5. Gastric outlet obstruction<br />

a. When an ulcer heals, it can form a scar that blocks the pyloric outlet. The resulting obstruction can result in<br />

gastric dilation, vomiting, dehydration, and a hypokalemic, hypochloremic metabolic alkalosis.<br />

b. Clinical presentation<br />

(1) Upper abdominal pain and vomiting (most common)<br />

(2) Early satiety<br />

(3) Recent weight loss<br />

(4) A succussion splash (a splashing sound elicited by gently rocking the abdomen)<br />

c. Diagnostic evaluation: an upright abdominal radiograph typically reveals a prominently dilated stomach<br />

shadow with a large air-fluid level.<br />

d. Management<br />

(1) Constant nasogastric suctioning<br />

(2) Fluid replacement and correction of electrolyte abnormalities<br />

(3) Admission for further evaluation and definitive management<br />

(4) Anticholinergic agents are contraindicated, because they may aggravate gastric distention by<br />

decreasing gastric motility.<br />

IV. PERFORATED VISCUS<br />

A. Gallbladder perforation (rare)<br />

1. Gallstone obstruction of the cystic or common bile duct - gallbladder distention - vascular compromise -<br />

gangrene - perforation<br />

a. More commonly, gallstones erode through the gallbladder wall, cystic duct, or common duct and produce<br />

fistulas between the gallbladder and another portion of the GI tract, rather than perforate directly into the<br />

peritoneal cavity.<br />

185


ABDOMINAL AND GASTROINTESTINAL DISORDERS<br />

b. Gallstone ileus occurs when a large gallstone enters the small intestine through such a fistula. It usually<br />

lodges in the terminal ileum, producing a small-bowel obstruction. May see pneumobilia on abdominal<br />

radiograph.<br />

c. Gangrene of the gallbladder (with subsequent perforation) can occur in the absence of stone formation<br />

(acalculous cholecystitis), especially in diabetic patients.<br />

2. Patients at increased risk<br />

a. Older patients<br />

b. Those with diabetes<br />

c. Those with atherosclerotic cardiovascular disease<br />

d. Those with a history of gallstones and repeated cholecystitis<br />

e. Those with a hemolytic disorder (sickle cell disease)<br />

3. Classic clinical scenario: An older man with a fever appears ill and has a tender right upper quadrant mass on<br />

physical examination. A careful history reveals one or more risk factors. In addition, a nonalcoholic may give<br />

a past history of jaundice or pancreatitis, which should suggest common duct stones. Laboratory studies reveal<br />

leukocytosis, possibly an increased bilirubin, and slightly increased serum amylase. On abdominal radiograph,<br />

a stone may be seen free in the abdomen. Subhepatic or subphrenic abscesses can form after a gallbladder<br />

perforation and cause restricted movement of the right leaf of the diaphragm.<br />

B. Small-bowel perforation<br />

1. Etiology of jejunal rupture<br />

a. Drugs (enteric-coated potassium tablets)<br />

b. Infection (eg, typhoid, tuberculosis)<br />

c. Tumor<br />

d. Strangulated hernia<br />

e. Regional enteritis (Crohn disease)<br />

2. A jejuna I perforation causes a chemical peritonitis that is more severe than that caused by an ilea! perforation<br />

because the pH of the jejuna! contents is higher (pH 8), and there is also a greater number of enzymes<br />

present in this part of the bowel. lleal perforations are associated with considerable bacterial contamination;<br />

these small-bowel perforations can rapidly wall off (particularly in patients with Crohn disease), so that early<br />

evaluation may reveal only localized findings.<br />

3. Mortality varies directly with the extent of peritoneal soiling and time delay to diagnosis and treatment.<br />

C. Large-bowel perforation<br />

1. Etiology<br />

a. Carcinoma<br />

b. Diverticulitis<br />

C. Colitis<br />

d. Foreign body<br />

e. Diagnostic instrumentation<br />

2. Clinical presentation<br />

a. The signs and symptoms of large-bowel perforations are largely due to sepsis and, therefore, are usually<br />

slower in onset than those produced by small-bowel perforations.<br />

b. The patient appears septic and has a feculent breath odor. The abdomen is distended, and the patient has<br />

been unable to pass feces. When a nasogastric tube is inserted and hooked to suction, fecal material is seen<br />

in the aspirate. Radiograph often demonstrates free air and small-bowel obstruction.<br />

D. Management of a known or suspected perforated viscus<br />

1. IV fluid replacement with normal saline or lactated Ringer's; large volumes (up to 12 Lover 24 hours) may be<br />

required because of third spacing.<br />

2. Bladder catheterization (to monitor urine output/volume status)<br />

3. Nothing by mouth; nasogastric tube drainage<br />

4. Broad-spectrum IV antibiotics<br />

5. Immediate surgical consult (delay in surgical intervention significantly increases mortality)<br />

186


ABDOMINAL AND GASTROINTESTINAL DISORDERS<br />

V. ACUTE ABDOMEN<br />

A. Appendicitis<br />

1. The most common indication for emergency surgery; appendicitis is also the most common surgical<br />

emergency seen in pregnancy, and the most common cause of emergency abdominal surgery in children.<br />

2. Although all age groups are affected, the highest incidence is in patients 10-30 years old (particularly males).<br />

3. Pathophysiology<br />

a. Obstruction of the appendiceal lumen (the primary inciting event) - increased intraluminal pressure and<br />

distention - vascular compromise of the appendiceal wall and bacterial invasion<br />

b. Etiologies of appendiceal obstruction include:<br />

(1) Fecalith (most common)<br />

(2) Enlarged lymphoid follicles<br />

(3) lnspissated barium<br />

(4) Worms<br />

(5) Granulomatous disease<br />

(6) Tumors<br />

(7) Adhesions<br />

(8) Dietary matter (seeds)<br />

4. Clinical presentation (listed in order of decreasing frequency)<br />

a. History<br />

(1) Abdominal pain (with migration to right lower quadrant increases likelihood)<br />

(2) Anorexia<br />

(3) Nausea and vomiting<br />

(4) Fever and chills (minimal statistical association between temperature >99°F [37.2°C] and<br />

appendicitis)<br />

(5) Diarrhea (can be a particularly prominent symptom in very young children and patients with pelvic<br />

appendices; often results in misdiagnosis as acute gastroenteritis)<br />

b. Physical evaluation<br />

(1) Abdominal tenderness<br />

(2) Percussion or rebound tenderness<br />

(3) Rectal tenderness (examination not useful to diagnose or exclude)<br />

(4) Cervical motion tenderness (28%-34%)<br />

(5) Psoas sign (right lower quadrant pain on passive extension of right hip)<br />

(6) Obturator sign (right lower quadrant pain on passive internal rotation of the flexed right hip)<br />

(7) Rovsing sign (right lower quadrant pain on palpation of the left lower quadrant)<br />

(8) Right lower quadrant pain on rectal examination (retrocecal appendicitis)<br />

c. Visceral pain is the first symptom to develop; it is dull and vague in character. In nonpregnant patients,<br />

pain usually begins in the periumbilical area and then, over time, moves to the right lower quadrant.<br />

Where the pain eventually migrates is determined by the actual location of the appendix; radiation of<br />

pain to the flank is suspicious of a retrocecal appendicitis. (Remember that kidney stone pain usually<br />

comes on suddenly; retrocecal appendiceal pain usually has a gradual onset). In pregnant patients, the<br />

appendix moves laterally and superiorly as the uterus enlarges, so be suspicious of flank or right upper<br />

quadrant pain. If the appendix is located near the uterus, ovary, or tubes, there will be pain on cervical<br />

motion. (Patients with pelvic inflammatory disease are generally seen later in the course of their illness,<br />

have fewer GI symptoms, and have a more increased sedimentation rate). Anorexia, nausea, and vomiting<br />

usually begin after the onset of pain. A sudden decrease in pain followed by a dramatic increase suggests<br />

perforation.<br />

d. Unfortunately, one-third to one-half of patients do not present "classically" (migratory abdominal pain<br />

and the typical associated signs and symptoms); the pain can be anywhere.<br />

e. Be particularly suspicious in very young and very old patients; their symptoms are frequently vague, and<br />

clinical presentation is less classic. Delayed diagnosis with resulting perforation and increased morbidity<br />

and mortality is common in these age groups.<br />

187


ABDOMINAL AND GASTROINTESTINAL DISORDERS<br />

5. Diagnostic evaluation<br />

a. May provide additional information even when the history and physical examination are classic; they can<br />

also be helpful in less typical cases when the diagnosis remains in doubt.<br />

b. Routine studies<br />

(1) Pregnancy test: should be done in all women of childbearing age to exclude ectopic pregnancy<br />

(2) CBC: a WBC count > 10,000/mm 3 is statistically associated with appendicitis but has such poor<br />

sensitivity and specificity that it is of almost no clinical utility.<br />

(3) Urinalysis: pyuria and hematuria may be seen if the inflamed appendix lies in close proximity to the<br />

ureter; a few WBCs and RBCs in the urine are characteristic.<br />

(4) Plain abdominal radiographs (an option to be considered only if CT is unavailable): a calcified<br />

fecalith in the right lower quadrant is very suggestive of appendicitis but is present in only 2%-22%<br />

of cases.<br />

c. Additional studies may be indicated when the diagnosis remains uncertain and should be chosen in<br />

consultation with the evaluating surgeon. Availability and usefulness of these studies may be institutiondependent.<br />

(1) Abdominal CT is generally considered the radiographic procedure of choice for diagnosing acute<br />

appendicitis in men and nonpregnant women. Data suggest that abdominal CT is >97% sensitive<br />

and 98%-100% specific for acute appendicitis. (In addition, abdominal CT may unmask other cases<br />

of abdominal pain.) Abdominal CT is often performed with IV contrast and oral or rectal contrast;<br />

presence of periappendiceal fat stranding is the most specific radiographic finding for appendicitis.<br />

Unenhanced (noncontrast) CT is also highly sensitive and specific.<br />

(2) ACEP Level B recommendation for CT scan with or without IV, oral, and or rectal contrast states that<br />

addition of IV contrast may increase sensitivity of CT for diagnosis of appendicitis.<br />

(3) Diagnostic laparoscopy has high sensitivity and specificity. It is clearly indicated in equivocal cases<br />

of surgical abdominal pain, and its use as a diagnostic tool is increasing because laparoscopic<br />

appendectomies are now performed more frequently than open surgical appendectomies.<br />

(4) Graded compression ultrasonography has a sensitivity of ~40% and a specificity of -90%, and it<br />

may be especially valuable for evaluating children and pregnant patients. It is noninvasive, without<br />

radiation exposure, safe in pregnancy, rapid, inexpensive, and rarely requires sedation in children.<br />

Visualization of a noncompressible, immobile appendix >6 mm in diameter is very suggestive of<br />

the diagnosis. Furthermore, an alternative definitive diagnosis can be established in up to 50%<br />

of patients when appendicitis is absent. The disadvantages of ultrasound are that it is not always<br />

available, it is operator-dependent, and it has a very poor sensitivity in the presence of perforation.<br />

(5) Lifetime risk of all cancers from one CT abdomen is calculated at 0.14% in neonates and 0.06%<br />

in adults.<br />

(6) In children, use ultrasound to confirm suspected acute appendicitis but not definitely exclude the<br />

diagnosis. Recommendation to follow ultrasound with CT of abdomen and pelvis if diagnosis is still<br />

highly suspected. (ACEP Clinical Policy: Critical Issues in the Evaluation and Management of Patients<br />

with Suspected Appendicitis)<br />

(7) MRI appears to be a useful modality in pregnant women (to avoid ionizing radiation) in whom<br />

ultrasound is inconclusive.<br />

6. Differential diagnosis<br />

a. Mesenteric adenitis<br />

b. Yersinia gastroenteritis<br />

c. Pelvic inflammatory disease<br />

d. Ectopic pregnancy<br />

e. Ovarian cyst<br />

f. Pyelonephritis<br />

g. Crohn disease<br />

h. Diverticulitis<br />

7. Management<br />

a. Nothing by mouth<br />

b. Establish an IV line<br />

c. Obtain early surgical consult. (Delay in laparoscopy or surgical intervention -- gangrene -- perforation<br />

-- increased morbidity and mortality, particularly in the very young and very old.)<br />

188


ABDOMINAL AND GASTROINTESTINAL DISORDERS<br />

d. If a decision to operate is reached, administer prophylactic parenteral antibiotics; they decrease the<br />

incidence of postoperative wound infection and, in patients who have perforated, they decrease the<br />

incidence of postoperative abscess formation. Acceptable regimens:<br />

(1) Piperacillin/tazobactam<br />

(2) Cefoxitin<br />

(3) Cefotetan<br />

(4) Ampicillin, gentamicin, and metronidazole/clindamycin<br />

e. Judicious amounts of parenteral narcotic may be administered for pain relief and to facilitate diagnostic<br />

evaluation. (ACEP Clinical Policy: Critical issues for the initial evaluation and management of patients<br />

presenting with a chief complaint of nontraumatic acute abdominal pain.)<br />

f. Recent literature confirms that delaying appendectomy in stable patients is a safe alternative to<br />

immediate surgery. Antibiotic therapy as noted above should be initiated.<br />

g. Several studies have suggested that nonsurgical management of acute, uncomplicated appendicitis is<br />

acceptable.<br />

B. Bowel obstruction<br />

1. Etiology<br />

(1) Nonoperative treatment for acute appendicitis (the NOTA study) showed good outcomes with a 2-year<br />

recurrence rate of about 14%. Amoxicillin/clavulanic acid was the antibiotic used in this study.<br />

(2) This method of treatment is now considered acceptable in many centers. It is not well studied in<br />

children and elderly patients and should not be used in patients when there is any suspicion of<br />

complicated appendicitis.<br />

a. Small-bowel obstruction<br />

(1) Adhesions (most common)<br />

(2) Hernia (second most common)<br />

(3) Neoplasms (lymphoma, adenocarcinoma)<br />

(4) lntussusception (common in children


ABDOMINAL AND GASTROINTESTINAL DISORDERS<br />

c. Peristalsis is initially increased early in the course of bowel obstructions (hyperactive bowel sounds).<br />

Passage of feces and flatus often continues in the first few hours after an obstruction occurs. With persistent<br />

complete obstruction, the bowel loses its ability to contract rigorously, bowel sounds become infrequent,<br />

and the passage of feces/flatus stops.<br />

d. Strangulation occurs most often in the small bowel and is due to vascular compromise that leads to<br />

infarction. In the large bowel, strangulation can occur with a volvulus, because it is a closed loop<br />

obstruction and vascular supply is quickly compromised.<br />

3. Clinical presentation<br />

a. Pain<br />

(1) Crampy, intermittent, and poorly localized pain is typical.<br />

(2) A change in the quality of the pain (greater intensity, more constant) may indicate that a complication<br />

has developed.<br />

b. Vomiting<br />

(1) The more proximal the obstruction, the sooner it begins.<br />

(2) If it is bilious, the obstruction is distal to the pylorus.<br />

(3) Feculent emesis is associated with distal ilea! and large-bowel obstruction and, if long-standing, may<br />

be due to infarcted bowel.<br />

c. Abdominal distention: the more distal the obstruction, the more pronounced the distention.<br />

d. Tympany may be present on percussion.<br />

e. Abdominal tenderness<br />

(1) Mi Id, diffuse tenderness is typical.<br />

(2) A tender mass may be palpated with a closed-loop obstruction.<br />

(3) Severe localized tenderness or rebound tenderness suggests the possibility of gangrenous or perforated<br />

bowel.<br />

4. Diagnostic evaluation<br />

a. Radiographs of the abdomen should include supine and upright films and are the first choice for imaging.<br />

b. Sm al I-bowel obstruction<br />

(1) Abdominal radiographs are diagnostic in 30%-70% (specificity ~50%).<br />

(2) Small-bowel diameter >3 cm<br />

(3) Air fluid levels<br />

(4) Stepladder appearance of dilated loops<br />

(5) "String of pearls" appearance is a late finding.<br />

(6) Cause of obstruction rarely identified on plain radiographs.<br />

c. CT of the abdomen and pelvis<br />

(1) The American College of Radiology considers CT an appropriate first choice for imaging; emergency<br />

medicine sources usually suggest abdominal radiographs.<br />

(2) Oral contrast not usually required, because fluid in bowel acts as contrast.<br />

(3) IV contrast helps diagnose strangulation.<br />

(4) Transition zone often identified.<br />

(5) Sensitivity 94%; specificity for small-bowel obstruction is 95% for high-grade obstructions but only<br />

50% for low-grade obstructions.<br />

d. Gas patterns in small- and large-bowel obstructions<br />

(1) Colon gas is distinguished from gas in the small bowel by its peripheral location and the presence of<br />

haustrations that do not involve the entire transverse diameter of the bowel.<br />

(2) Small-bowel gas is more central in location, and one can see valvulae conniventes that involve the<br />

entire transverse diameter of the small bowel. They are spaced closer together than the haustrations of<br />

the large bowel.<br />

e. Large-bowel obstruction<br />

5. Management<br />

(1) Abdominal radiographs unreliable in diagnosis (may be only 84% sensitive and 72% specific)<br />

(2) Dilation of colon >5 cm with> 10 cm at cecum<br />

(3) CT of abdomen and pelvis is considered modality of choice.<br />

a. Sm al I-bowel obstruction<br />

(1) IV fluid resuscitation<br />

190


ABDOMINAL AND GASTROINTESTINAL DISORDERS<br />

C. Volvulus<br />

(2) Correction of electrolyte abnormalities<br />

(3) Nasogastric tube decompression is mainstay of conservative therapy; no benefit for long intestinal tubes.<br />

(4) Broad-spectrum antibiotic coverage may be of benefit.<br />

(5) Early surgical consult: patients with pneumoperitoneum, peritonitis, and sepsis require immediate<br />

surgical intervention.<br />

b. Large-bowel obstruction<br />

(1) Volume resuscitation and correction of electrolyte abnormalities.<br />

(2) Patients with perforated bowel, sepsis, etc, require immediate surgery.<br />

(3) Colonic dilatation> 12 cm or of prolonged duration may require immediate intervention.<br />

(4) Nasogastric decompression if vomiting present.<br />

1. Volvulus is a closed-loop obstruction that results from the twisting of a mobile segment of bowel on its<br />

mesocolon; it is responsible for approximately 10%-13% of colonic obstructions in the United States.<br />

2. Sigmoid volvulus is more common (60%) than cecal volvulus (40%).<br />

3. Epidemiology and pathophysiology<br />

a. Sigmoid volvulus occurs almost entirely in two patient populations: elderly, bedridden patients with<br />

debilitating comorbid diseases and patients of any age with profound neurologic or psychiatric illness.<br />

These patients invariably have a history of chronic severe constipation that leads to an elongated<br />

(redundant) sigmoid colon.<br />

b. Cecal volvulus results from incomplete embryologic fixation of the cecum, ascending colon, and terminal<br />

ileum to the posterior abdominal wall. Although it is seen in patients of all ages, it is most common in<br />

patients in their twenties and thirties.<br />

4. Clinical presentation of sigmoid and cecal volvulus<br />

a. Sudden onset of crampy lower abdominal pain<br />

b. Nausea, vomiting, obstipation<br />

c. Diffuse abdominal tenderness<br />

d. Progressive abdominal distention<br />

e. Tympany<br />

f. Gangrenous bowel in 20% of patients with cecal volvulus<br />

5. Diagnostic evaluation<br />

a. Radiographic findings: a markedly dilated single loop of colon<br />

b. Sigmoid volvulus: bent "inner tube" appearance with dilated bowel segment doubled back; the proximal<br />

large bowel is dilated.<br />

c. Cecal volvulus: the loop is usually seen in the mid or upper abdomen toward the left but may be anywhere;<br />

single dilated segment of cecum with a kidney or "coffee bean" shape. The distal large bowel is collapsed,<br />

and the small bowel is dilated.<br />

6. Management<br />

a. Initial measures are the same as those for bowel obstruction (IV hydration, nasogastric tube decompression,<br />

and broad-spectrum antibiotics).<br />

b. Sigmoid volvulus<br />

(1) Surgical consult and nonoperative reduction using a rectal tube via the sigmoidoscope or barium<br />

enema. Because recurrence is common (90%), this is usually followed by an elective resection if the<br />

patient's underlying medical condition can tolerate it.<br />

(2) If nonoperative management fails or strangulation is suspected, immediate operative reduction is<br />

required.<br />

c. Cecal volvulus: early and immediate surgical reduction, because this is a closed-loop obstruction.<br />

D. Mesenteric vascular ischemia/infarction<br />

1. Most commonly affects patients who are >50 years old and have a history of cardiovascular disease<br />

2. Mortality rate is 50% overall (but rises to ::,:70% once infarction has occurred) and reflects the difficulty in<br />

making an early diagnosis as well as the presence of significant underlying disease.<br />

3. Etiology<br />

a. Most common cause is arterial embolus (50% of cases). Another 25% are caused by arterial thrombosis.<br />

b. Nonocclusive mesenteric ischemia accounts for 20% of cases. Common pathway is mesenteric<br />

vasoconstriction due to low flow states with decreased cardiac output. Associated conditions are CHF,<br />

191


ABDOMINAL AND GASTROINTESTINAL DISORDERS<br />

cardiogenic shock, septic shock, etc, and medications that cause splanchnic vasconstriction such as<br />

digoxin, vasopressors. It is common in hospitalized patients.<br />

c. Mesenteric venous thrombosis is responsible for 5%-15% of cases and most commonly occurs in association<br />

with a hypercoagulable state (eg, polycythemia vera, antithrombin Ill deficiency). Affected patients are<br />

usually younger in age, and up to 60% of them have a history of peripheral deep-venous thrombosis.<br />

4. Predisposing factors for<br />

a. Acute mesenteric arterial embolism<br />

(1) Dysrhythmias (particularly atrial fibrillation)<br />

(2) Atherosclerotic heart disease<br />

(3) Valvular heart disease<br />

(4) Recent Ml with mural thrombosis<br />

b. Venous thrombosis<br />

(1) History of prior thromboembolic events<br />

(2) Hypercoagulable states<br />

c. Nonocclusive ischemia<br />

(1) Use of diuretics or vasoconstrictive medications or digitalis<br />

(2) Hypotension<br />

(3) CHF<br />

(4) Dialysis<br />

5. Clinical presentation<br />

a. The patient is typically middle-aged or elderly and presents with severe abdominal pain, fever, and<br />

tachycardia; sudden onset suggests arterial occlusion, while insidious onset suggests venous thrombosis or<br />

nonocclusive ischemia.<br />

b. A history of similar, spontaneously resolving pain episodes after meals ("intestinal ischemia 11 ) is not uncommon.<br />

c. Early on, the pain is poorly localized and out of proportion to findings on physical examination. Diarrhea<br />

is common, and the feces frequently test positive for guaiac. Grossly bloody feces may also be seen. Other<br />

commonly associated symptoms are anorexia, nausea, and vomiting. Abdominal distention and peritoneal<br />

signs are late findings and signal the presence of bowel infarction.<br />

6. Diagnostic evaluation<br />

a. Helical CT angiography is the primary diagnostic modality for patients with a high clinical index of<br />

suspicion. Catheter angiography is used for equivocal cases.<br />

b. CBC reveals hemoconcentration (increased hematocrit) and leukocytosis (WBC count is often > 15,000/mm 3 ).<br />

c. Serum amylase is usually moderately increased (lipase is normal).<br />

d. Serum phosphate may be increased.<br />

e. Serum lactate is nearly 100% sensitive when bowel infarction is present but lacks specificity and is not<br />

often increased without infarction.<br />

f. Blood gases, arterial or venous, often reveal a metabolic acidosis, particularly late in the disease course.<br />

Presence of metabolic acidosis that cannot otherwise be explained should prompt investigation for<br />

mesenteric ischemia in appropriate patients.<br />

g. Plain abdominal and upright chest radiographs are usually the first radiographic studies to be obtained,<br />

although they are often normal (especially early on). Findings may include:<br />

(1) lieus<br />

(2) Small-bowel obstruction<br />

(3) Gasless abdomen<br />

(4) Irregular thickening of the bowel wall ( 11 thumbprinting 11 -although rarely seen, strongly suggests<br />

intestinal infarction)<br />

(5) Gas in the bowel wall (pneumatosis intestinal is) or portal venous system (although rarely seen, strongly<br />

suggests intestinal infarction)<br />

h. Abdominal ultrasound and MRI are not useful in the initial evaluation.<br />

i. Abdominal CT angiography<br />

(1) Most commonly ordered test in the emergency department for evaluation of suspected mesenteric<br />

ischemia<br />

(2) Sensitivity reported to be as high as 96% with a specificity of 94%<br />

(3) Test of choice for suspected mesenteric venous thrombosis<br />

192


ABDOMINAL AND GASTROINTESTINAL DISORDERS<br />

7. Management<br />

E. Hernias<br />

a. Initial stabilization measures<br />

(1) IV of normal saline or lactated Ringer's to correct volume deficit<br />

(2) Supplemental oxygen<br />

(3) Nasogastric tube to decompress stomach and bowel<br />

(4) Initial laboratory studies, type and cross, cultures, and radiographs<br />

(5) Broad-spectrum IV antibiotics<br />

(6) Heparin or low-molecular-weight heparin may be given, particularly for mesenteric venous thrombosis.<br />

(7) Thrombolytics are an option in consultation with an appropriate specialist such as a vascular surgeon<br />

or interventional radiologist.<br />

(8) Correction of precipitating or predisposing causes of ischemia (eg, CHF, dysrhythmias)<br />

(9) Immediate surgical consult<br />

(a) Vascular surgery for possible revascularization<br />

(b) lnterventional radiology for angioplasty or intra-arterial thrombolytic administration<br />

(10) If pressors are required, positive inotropes are preferred. Avoid a-agonists.<br />

(11) If angiography is performed, papaverine may be infused to dilate mesenteric arteries.<br />

b. Specific treatment measures depend on the underlying cause of the ischemia and if peritoneal signs or<br />

necrotic bowel are present.<br />

1. Definitions<br />

(1) Nonocclusive mesenteric ischemia: unless peritonitis or necrotic bowel is present, treatment is<br />

nonoperative; papaverine alone is considered definitive therapy in these patients.<br />

(2) Mesenteric venous thrombosis: immediate anticoagulation with heparin is the initial treatment of<br />

choice and, in the absence of peritonitis and necrotic bowel, may be the only treatment given.<br />

(3) Acute mesenteric arterial occlusion (embolic and thrombotic): initial treatment may be papaverine,<br />

while plans are made for revascularization. Papaverine should be continued postoperatively to treat<br />

vasospasm of the smaller branches of the superior mesenteric artery.<br />

(4) Immediate surgery is indicated for:<br />

(a) Patients with peritonitis<br />

(b) Resection of necrotic bowel<br />

a. Hernia: the protrusion of a structure from its normal position into another through an opening that is either<br />

congenital or acquired<br />

b. External hernia: protrudes to the outside (eg, umbilical)<br />

c. Internal hernia: protrudes within the body (eg, diaphragmatic)<br />

d. lncisional hernia: protrudes through a previous incision<br />

e. Reducible hernia: protruding contents can be pushed back in<br />

f. Irreducible (incarcerated) hernia<br />

(1) Protruding contents cannot be moved back into place<br />

(2) Seen most often when large contents herniate through a small defect<br />

g. Strangulated hernia: vascular compromise of herniated contents<br />

h. "Sports hernia": refers to groin pain related to participation in athletics but is not an anatomic hernia.<br />

2. Epidemiology<br />

a. Men are 8 times more likely than women to develop a hernia, and 20 times more likely to require surgical<br />

repair.<br />

b. Lifetime risk of developing a groin hernia is 25% in men.<br />

c. Women present at an older age than men.<br />

d. 96% of groin hernias are inguinal; 4% are femoral.<br />

3. Types<br />

a. Inguinal hernia<br />

(1) Direct<br />

(a) Protrudes directly through the floor of Hesselbach triangle<br />

(b) Results from relaxation/weakening of the abdominal musculature<br />

193


ABDOMINAL AND GASTROINTESTINAL DISORDERS<br />

(c) Increased frequency with age<br />

(d) Rarely incarcerates<br />

(2) Indirect<br />

(a) Protrudes through the internal inguinal ring lateral to the inferior epigastric vessels<br />

(b) Represents a congenital defect (incomplete closure of processus vaginal is)<br />

(c) Most common hernia occurring in both sexes<br />

(d) Most common in younger patients<br />

(e) Frequently incarcerate, especially in infancy<br />

b. Femoral hernia<br />

(1) Protrudes below the inguinal ligament and the femoral vessels in the femoral canal<br />

(2) More common in women<br />

(3) Frequently incarcerates (40% present as acute incarceration)<br />

c. Umbilical hernia<br />

(1) Usually presents as a lump<br />

(2) Common in newborns<br />

(3) Most close spontaneously by the age of 2-3 years<br />

d. Obturator hernia<br />

(1) Protrudes through the obturator foramen into the medial thigh<br />

(2) Most occur in older women<br />

(3) Patients present with pain and decreased sensation along the medial aspect of the thigh to the knee.<br />

4. Diagnostic evaluation<br />

a. Inguinal: ultrasonography is considered the best initial modality in patients with high clinical suspicion but<br />

negative findings on examination.<br />

b. Femoral: ultrasonography may help differentiate inguinal from femoral hernias, but CT scan may be required.<br />

F. Ileitis and colitis<br />

1. Crohn disease (terminal ileitis, granulomatous ileocolitis, regional enteritis)<br />

a. Pathology<br />

(1) A chronic inflammatory disease that involves all layers of the bowel as well as the mesenteric lymph nodes<br />

(2) Any segment of the GI tract from the mouth to the anus may be affected.<br />

(3) The disease is discontinuous with normal areas of bowel ("skip areas") between one or more involved<br />

areas; the ileum is involved in most cases.<br />

(4) The thickened bowel wall leads to narrowing of the lumen, which frequently results in obstruction.<br />

(5) Longitudinal deep ulcerations are characteristic; with progression of the disease, a "cobblestone"<br />

appearance of the mucosa results from crisscrossing of these ulcers with intervening normal mucosa.<br />

(6) Noncaseating granulomas are present in 2:50% of specimens and are helpful (but not necessary) in<br />

making the diagnosis.<br />

(7) Perianal complications occur in 90% of cases and may be the initial presenting symptom in many<br />

patients.<br />

b. Epidemiology<br />

(1) Onset of disease common in teens and early twenties but can occur at any age<br />

(2) Four to eight times more common in the Jewish population<br />

(3) Genetic predisposition<br />

(4) Etiology remains undetermined.<br />

c. Clinical presentation<br />

(1) Abdominal pain and diarrhea (inflammation of ileum and/or colon)<br />

(a)<br />

Past history of recurrent abdominal pain, diarrhea, and fever for several years<br />

(b) Nocturnal diarrhea may be a diagnostic clue.<br />

(c) Weight loss is common.<br />

(2) Right lower quadrant pain and tenderness (acute inflammation of the terminal ileum only)<br />

(a)<br />

Looks like appendicitis ("pseudoappendicitis"): anorexia, diarrhea, vomiting, fever, leukocytosis<br />

(b) Diagnostic clue: occult blood and fecal leukocytes favor regional enteritis<br />

194


ABDOMINAL AND GASTROINTESTINAL DISORDERS<br />

(3) Extraintestinal manifestations are seen in one-quarter to one-third of cases and may be the presenting<br />

and/or dominant manifestation of the disease process.<br />

(a) Arthritic (peripheral arthritis, ankylosing spondylitis)<br />

(b) Vascular (vasculitis, arteritis, thromboembolic disease)<br />

(c) Hepatobiliary (gallstones, pericholangitis, chronic active hepatitis)<br />

(d) Dermatologic (erythema nodosum, pyoderma gangrenosum)<br />

(e) Ophthalmic (uveitis, episcleritis, iritis, conjunctivitis)<br />

(f)<br />

d. Comp! ications<br />

Renal calculi occur with increased frequency due to increased intestinal absorption of oxalic acid.<br />

(1) Perianal (abscess, fissure, fistula, rectovaginal fistula, rectal prolapse)<br />

(2) Intestinal (obstruction, abscess, fistula, stricture, perforation, hemorrhage)<br />

(3) Toxic megacolon is an infrequent complication of Crohn disease and is associated with massive GI<br />

bleeding in >50% of cases.<br />

(4) Malignancies of both the small and large bowel are three times more common in patients with Crohn<br />

disease than in the general population.<br />

e. Diagnostic evaluation<br />

(1) Previously undiagnosed patients require radiologic, colonoscopic, and histologic confirmation to<br />

establish the local and systemic extent of the disease,<br />

(2) Stable patients with mild disease may be evaluated as an outpatient with close follow-up.<br />

f. Management<br />

(1) Make a diligent effort to exclude associated small-bowel obstruction, because early institution of<br />

nasogastric suction, IV fluids, and steroids may reduce inflammatory edema, restore bowel patency,<br />

and avoid the need for emergency surgery.<br />

(2) Fluid and electrolyte replacement (especially those patients with a history of anorexia, vomiting, and<br />

diarrhea)<br />

(3) Indications for admission<br />

(a) Dehydration or metabolic/electrolyte disturbances<br />

(b) Severe exacerbation of the primary illness<br />

(c) Acute complications (eg, obstruction, hemorrhage, peritonitis)<br />

(4) Outpatient therapy for mild cases<br />

(a)<br />

Bed rest, analgesics, antidiarrheal agent<br />

(b) Milk products should be avoided; they frequently aggravate diarrhea in these patients, in whom<br />

the incidence of lactose intolerance is high.<br />

(c) Steroids: beginning therapy with (or changing the dosage of) should be done in conjunction<br />

with the patient's primary physician, because these agents have many adverse effects and are<br />

ineffective as maintenance therapy.<br />

(d) Mesalamine, a 5'-ASA derivative is preferred over sulfasalazine, because it has fewer adverse<br />

effects.<br />

(e) Antibiotics<br />

(f)<br />

2. Ulcerative colitis<br />

i. May be of use in Crohn disease<br />

ii. Ciprofloxacin or metronidazole show some benefit.<br />

Azathioprine and its metabolite 6-mercaptopurine (immunosuppressive agents) are useful:<br />

i. As steroid-sparing agents (allow patients to decrease their dose of prednisone)<br />

ii. In patients with refractory disease and contraindications to surgery<br />

iii. In patients with enterocutaneous or enteroenteric fistulas<br />

(g) Anti-TNF therapies (infliximab, adalimumab, certolizumab pegol) may be helpful in severe cases.<br />

These agents are rarely initiated in the emergency department, but clinicians should be aware of<br />

immunocompromise in patients taking them.<br />

a. Definition and pathology<br />

(1) Ulcerative colitis is a chronic inflammatory and ulcerative disease of the colon and rectum, most often<br />

characterized clinically by bbgloody diarrhea and crampy abdominal pain.<br />

(2) Inflammation is generally limited to the mucosa and submucosa; the muscular layer and serosa are<br />

usually spared.<br />

195


ABDOMINAL AND GASTROINTESTINAL DISORDERS<br />

(3) Unlike the "skip lesions" of Crohn disease, mucosal involvement in ulcerative colitis is continuous and<br />

uniform; it always begins in the rectum and can remain limited to this area or spread proximally to<br />

involve the upper segments of the colon.<br />

(4) Chronic inflammation leads to the formation of crypt abscesses, epithelial necrosis, and mucosal<br />

ulceration.<br />

b. Epidemiology closely approximates that of Crohn disease.<br />

c. Clinical presentation<br />

(1) Occurs intermittently with complete remission between attacks<br />

(2) Mild disease in 60% of cases<br />

(a) 6 bowel movements/day<br />

(b) Fever, tachycardia, weight loss, anemia<br />

(c) Extraintestinal manifestations (essentially the same as those seen with Crohn disease)<br />

d. Complications<br />

(1) Hemorrhage<br />

(a) Most common<br />

(b) However, massive hemorrhage is infrequent (occurs in only 2%-3% of patients).<br />

(2) Toxic megacolon (occurs more commonly with ulcerative colitis than with Crohn disease)<br />

(a) Clinical picture: The patient appears severely ill and has abnormal vital signs (hypotension,<br />

tachycardia, and fever). The abdomen is distended, tender, and tympanitic. Laboratory studies<br />

reveal leukocytosis with anemia, electrolyte imbalance, and hypoalbuminemia. Plain radiograph<br />

of the abdomen reveals dilatation of the colon with a colonic diameter >6 cm.<br />

(b) Management<br />

i. Nasogastric suction, IV fluids, and broad-spectrum antibiotics<br />

ii. IV steroids (hydrocortisone or methylprednisolone)<br />

iii. Frequent abdominal examinations and radiographs with surgery (colectomy) if no<br />

improvement in 24-48 hours<br />

(3) Perforation (occurs more often during first episodes of colitis)<br />

(4) Obstruction (due to stricture)<br />

(5) Perianal fistulas and abscesses (occur less commonly with ulcerative colitis than with Crohn disease)<br />

(6) Carcinoma of the colon (incidence is 15 times greater in patients with ulcerative colitis than in the<br />

general population and correlates with the duration and extent of disease)<br />

e. Diagnostic evaluation<br />

(1) The most sensitive method for establishing the diagnosis of ulcerative colitis and determining the<br />

extent of disease is a sigmoidoscopy.<br />

(2) Diagnosis of toxic megacolon is usually obvious on plain radiographs.<br />

f. Management<br />

(1) Mild to moderate disease<br />

(a) 5'-ASA derivative such as mesalamine or olsalazine<br />

i. Mainstay of therapy<br />

ii. Therapeutic properties are because of its 5-aminosalicylic acid (5' -ASA) component.<br />

(b) Corticosteroids may also be needed, either as supplemental therapy or as alternative therapy when<br />

mesalamine is ineffective.<br />

(c) 5'-ASA and corticosteroid enemas are useful for patients with ulcerative proctitis and left-sided<br />

colitis.<br />

(d) Supportive therapy (supplemental iron and a lactose-free diet); antidiarrheal agents should be<br />

avoided because they are usually ineffective and can precipitate toxic megacolon.<br />

(e) Azathioprine, cyclosporine, and 6-mercaptopurine are used in patients who do not respond to<br />

steroid therapy.<br />

(2) Severe ulcerative colitis (inpatient therapy)<br />

(a)<br />

IV fluids and correction of electrolyte abnormalities<br />

196


ABDOMINAL AND GASTROINTESTINAL DISORDERS<br />

(b) Nasogastric tube<br />

(c) IV steroids, cyclosporine, or infliximab<br />

(d) Broad-spectrum antibiotics active against coliforms and anaerobes (ampicillin, clindamycin, or<br />

metron idazole)<br />

(e) Surgical consult if toxic megacolon suspected<br />

3. Pseudomembranous enterocol itis<br />

a. Definition: an inflammatory bowel disorder characterized by the formation of yellowish exudative<br />

pseudomembranous-like plaques that overlie and replace necrotic intestinal mucosa, particularly in the<br />

area of the rectosigmoid<br />

b. Etiology and epidemiology<br />

(1) Ingestion of broad-spectrum antibiotics alters the gut flora - proliferation of Clostridium difficile,<br />

a cytopathic toxin-producing bacterium (emerging strain NAP-1 /027 has high association with<br />

fluoroquinolone usage)<br />

(2) Any antibiotic can cause pseudomembranous colitis, but fluoroquinolones, clindamycin,<br />

cephalosporins, and penicillins are the most common.<br />

(3) Nosocomial transmission among hospitalized patients via hands of healthcare workers (contaminated<br />

with C difficile toxin) who are caring for these patients also occurs, even in the absence of<br />

antimicrobial therapy.<br />

(4) Traditionally considered a disease of frail, elderly, hospitalized patients, it is now recognized in those<br />

who have not been exposed to hospital environment or antibiotics. The CDC warns of transmission by<br />

close contact with infected persons in nontraditional patients (such as the young and otherwise healthy<br />

individuals); suggests person-to-person spread.<br />

c. Clinical presentation<br />

(1) The patient presents with crampy abdominal pain, fever, and watery diarrhea (which may be bloody).<br />

(2) Symptoms generally begin 7-10 days after antibiotics are started but may occasionally be delayed<br />

several weeks after antibiotics have been discontinued.<br />

d. Diagnostic evaluation<br />

(1) The presence of C difficile toxin in the feces confirms the diagnosis.<br />

(2) If this assay is not readily available, colonoscopy or sigmoidoscopy can be used to establish a tentative<br />

diagnosis.<br />

e. Management<br />

(1) Discontinue antibiotics and institute supportive therapy with IV fluids and electrolyte replacement.<br />

(2) Oral metronidazole considered first choice for mild to moderate disease.<br />

(3) Oral vancomycin is first choice for severe infection and refractory infections.<br />

(4) Toxic megacolon a potential complication of C difficile infection.<br />

(5) Relapse occurs in about 25% of patients. In patients who have had one relapse, the rate increases to<br />

45% for a second relapse.<br />

(6) Fecal microbiota transplant is an accepted therapy for recurrent C difficile infection.<br />

G. Irritable bowel syndrome (IBS)<br />

1. Definition: a state of disturbed intestinal motility (without anatomic cause) that is functional in nature and<br />

diagnosed only after more serious causes are excluded<br />

2. Epidemiology<br />

a. Patients are usually 20-40 years old.<br />

b. Women are affected more often than men (ratio is 2:1 ).<br />

c. A familial predisposition is present.<br />

d. Psychiatric disease may coexist. Depression and anxiety disorder are common.<br />

e. Fibromyalgia is a frequent comorbid illness.<br />

f. It is exacerbated by stress.<br />

g. Patients with !BS have demonstrable disturbance of rhythmic electrical activity in the intestine.<br />

3. Classic clinical scenario<br />

a. The patient is usually a young woman who presents with recurrent episodes of altered bowel function<br />

(diarrhea or constipation) with or without associated abdominal pain.<br />

b. The abdominal pain is described as crampy or achy in character and is confined to the lower abdomen. It<br />

generally occurs in association with constipation and is relieved by defecation or the passage of gas.<br />

197


ABDOMINAL AND GASTROINTESTINAL DISORDERS<br />

c. Extracolonic symptoms (eg, bloating, belching, gastroesophageal reflux) are common.<br />

d. Presence of fever, abdominal mass, weight loss, and rectal bleeding are not associated with IBS and suggest<br />

a more serious diagnosis.<br />

e. Examination findings are most often nonspecific but may include vague lower abdominal tenderness and a<br />

palpable feces-filled sigmoid colon.<br />

f. Laboratory evaluation is unremarkable.<br />

4. Diagnostic evaluation<br />

a. IBS is a diagnosis of exclusion.<br />

b. Diagnosis is difficult to make in the emergency department. Patients are often discharged with diagnosis of<br />

"abdominal pain, etiology uncertain." If the diagnosis is suspected, patients should be referred to primary<br />

care provider for additional evaluation.<br />

c. The Rome Criteria were developed to standardize research protocols: diagnosis of JBS requires recurrent<br />

abdominal pain or discomfort for at least 3 days per month in the last 3 months associated with at least two<br />

of the following:<br />

(1) Improvement with defecation<br />

(2) Onset associated with change in frequency of defecation<br />

(3) Onset associated with change in form/appearance of feces<br />

5. Management<br />

a. Directed toward relief of symptoms<br />

(1) Constipation predominant: may benefit from high-fiber diet and bulk-forming agents<br />

(2) Diarrhea predominant: antidiarrheals, antispasmodics<br />

b. Drug therapy should be reserved for patients who do not respond to the above measures. It is usually<br />

initiated by the primary care provider and may include one or more of the following:<br />

(1) Antidiarrheal agents<br />

(2) Antispasmodic (anticholinergic) agents<br />

(3) Anxiolytics<br />

(4) Antidepressants<br />

(5) A number of drugs, including 5-hydroxytryptamine (5-HT) (serotonin) 3 receptor antagonists and 5-HT<br />

4 receptor agonists, have been removed from the market because of severe adverse effects.<br />

H. Colonic diverticular disease<br />

1. Diverticula are saclike herniations of colonic mucosa and submucosa through the muscularis; they may penetrate<br />

through areas of potential weakness in the bowel wall, such as where wall is penetrated by blood vessels.<br />

a. Associated with a diet low in fiber and high in refined carbohydrates; promotes less bulky stools, increased<br />

GI transit time, and increased intracolonic pressure.<br />

b. Although the location of diverticula may vary, most are confined to the sigmoid colon (left lower quadrant)<br />

in Western countries.<br />

2. The frequency of diverticular disease is directly correlated with age, especially if the diet is low in fiber and<br />

roughage.<br />

a. Frequency of diverticulosis increases with age: incidence is 10% in those


ABDOMINAL AND GASTROINTESTINAL DISORDERS<br />

(5) Caution in elderly and immunocompromised, because abdominal examinations and systemic<br />

response may be unreliable.<br />

(6) Patients are likely stable for outpatient therapy.<br />

4. Complicated diverticulitis<br />

a. Process has advanced such that microperforation, abscess, or phlegmon has occurred. Fistula formation<br />

may occur, and 2% have free perforation.<br />

b. Physical examination is likely to show peritoneal signs with signs of systemic toxicity.<br />

c. Leukocytosis and sepsis may also occur.<br />

d. Abdominal CT is required to determine extent of pathology and identify complicating factors such as<br />

fistula, perforation, abscess.<br />

e. Diagnostic evaluation<br />

(1) CT of the abdomen is recommended as initial radiologic examination.<br />

(a) Helps establish the diagnosis when it is unclear<br />

(b) Helps establish extent of disease and complications<br />

(c) Excludes other etiologies<br />

(d) No risk of perforation<br />

(2) Colonoscopy is usually avoided when acute because of risk of perforation.<br />

(3) Barium enema is not done in acute stage.<br />

5. Management<br />

a. Uncomplicated diverticulitis: may be treated on outpatient basis<br />

(1) High-fiber diet prevents recurrence for 5 years in 70% of patients.<br />

(2) Antibiotics to cover anaerobes and gram-negative organisms.<br />

(a) TMP-SMX + metronidazole or<br />

(b) Ciprofloxacin + metronidazole or<br />

(c) Amoxicillin/clavulanate for 10 days<br />

(3) Analgesics<br />

b. Complicated diverticulitis<br />

(1) IV fluids and broad-spectrum antibiotics to provide both aerobic and anaerobic bacterial coverage<br />

(a) Ciprofloxacin + metronidazole<br />

(b) Ticarcillin/clavulanate<br />

(c) Ampicillin/sulbactam<br />

(d) lmipenem for more severe cases<br />

(2) Nothing by mouth; bowel rest<br />

(3) Nasogastric suction if ileus or intestinal obstruction is present<br />

(4) Surgery required in only 10% of patients with acute disease<br />

(a) Small abscesses may be treated with antibiotics only; abscesses >4 cm may require percutaneous<br />

drainage.<br />

(b) Surgical consult for all with peritoneal signs or free air<br />

(5) Analgesics<br />

VI. ANORECTAL DISORDERS<br />

A. Hemorrhoids<br />

1. Definition<br />

a. Dilated venules of the hemorrhoidal plexuses<br />

b. Associated with chronic constipation, straining, increased intra-abdominal pressure, pregnancy, increased<br />

portal pressure, and a low-fiber diet<br />

2. Types<br />

a. External hemorrhoids arise from below the dentate line (which separates the rectum from the anus) and are<br />

covered with well-innervated squamous epithelium. They are visible on external inspection.<br />

b. Internal hemorrhoids originate from above the dentate line and are covered by relatively insensitive rectal<br />

mucosa. Although not normally palpable on examination, they can be visualized with anoscopy at the 2,<br />

5, and 9 o'clock positions and may visibly prolapse with the Valsalva maneuver.<br />

199


ABDOMINAL AND GASTROINTESTINAL DISORDERS<br />

3. Clinical presentation<br />

a. Patients with external hemorrhoids most commonly present with painful thrombosis. Examination reveals a<br />

tender mass at the external anal orifice.<br />

b. Patients with internal hemorrhoids usually present with painless bright-red bleeding that occurs in<br />

association with defecation. (Hemorrhoids are the most common cause of rectal bleeding). The blood<br />

covers the feces or drips into the toilet bowl and is usually small in volume.<br />

4. Management<br />

B. Anal fissure<br />

a. Thrombosed external hemorrhoids<br />

(1) WASH regimen<br />

Warm water soaks<br />

Analgesics (mild oral analgesics)<br />

~tool softeners<br />

.!::!.igh-fiber diet<br />

(2) Topical steroids are of limited value and should be avoided.<br />

(3) Topical anesthetics may offer some relief, but their use is controversial.<br />

(4) If pain is severe and the thrombosis is acute (


ABDOMINAL AND GASTROINTESTINAL DISORDERS<br />

2. Etiology: most anorectal abscesses result from obstruction of mucusproducing glands at the base of the anal<br />

crypts. Other causes include inflammatory bowel disease, cancer, radiation injury, trauma, tuberculosis, and<br />

lymphogranuloma venereum. The resulting infections are frequently polymicrobial and involve the colonic flora.<br />

3. Perianal abscesses are the most common.<br />

a. Located outside the anal verge<br />

b. Patient complains of painful perianal mass.<br />

c. Likely afebrile with normal WBC count<br />

d. May be incised in the emergency department<br />

4. lschiorectal abscess<br />

a. Form outside the sphincter muscles and below levator ani<br />

b. Patient likely complains of buttock pain.<br />

c. Fever and increased WBC count are likely.<br />

d. Almost all require drainage in the operating room, because appearance maybe misleading.<br />

5. lntersphincteric abscess<br />

a. Deep to anal sphincter and below levator ani<br />

b. Patient complaint of constant rectal pressure and increased pain with defecation and sitting.<br />

c. Rectal examination shows painful mass occasionally with purulent drainage.<br />

d. Inguinal adenopathy common<br />

e. Require drainage in the operating room<br />

6. Supralevator abscess<br />

a. Perianal and buttock pain, often associated with fever<br />

b. External evidence of infection is lacking. Rectal examination may show tender mass.<br />

c. Surgical drainage in operating room required.<br />

7. Management: anorectal abscesses require early and extensive drainage.<br />

a. Simple perianal abscesses may be drained in the emergency department under local anesthesia.<br />

b. All other perirectal abscesses need to be drained in the operating room.<br />

c. Routine use of antibiotics is not warranted; they should be reserved for immunocompromised patients,<br />

patients with diabetes or valvular heart disease, and patients with associated cellulitis.<br />

8. Complication: fistula formation is a common sequela.<br />

D. Miscellaneous anorectal disorders<br />

1. Fistula-in-ano<br />

a. An abnormal tract between the anal canal and skin that is lined with granulation tissue<br />

b. Most commonly occurs as a complication of a perianal or ischiorectal abscess; may also be associated with<br />

ulcerative colitis, Crohn disease, tuberculosis, radiation, and cancer of the anus or rectum<br />

c. If the tract remains open, there is a persistent, purulent, blood-stained discharge; if the tract becomes<br />

blocked (most common), an abscess may be the only physical finding.<br />

d. Treatment is surgical excision.<br />

2. Rectal prolapse (procidentia)<br />

a. Types<br />

(1) Mucosa! prolapse (seen primarily in young children and in association with internal hemorrhoids)<br />

(2) Prolapse involving all layers of the rectum<br />

(3) lntussusception of the upper rectum into the lower rectum with the apex protruding through the anus<br />

b. Complete rectal prolapse occurs in the very young and the very old (particularly older women).<br />

c. Rectal prolapse in children should prompt consideration of cystic fibrosis.<br />

d. Clinical presentation: patients typically complain of a painless anal mass noticed after straining, coughing,<br />

or defecation.<br />

e. Management<br />

(1) Reduction can be accomplished in almost all patients, although recurrence (particularly in older<br />

patients) is likely.<br />

(2) In the presence of vascular compromise, emergency reduction is mandatory.<br />

(3) After reduction of the prolapse, patients should be placed on fecal softeners and referred for further<br />

evaluation (to uncover any underlying pathology) and potential surgical correction (commonly<br />

required in the elderly).<br />

201


ABDOMINAL AND GASTROINTESTINAL DISORDERS<br />

3. Pilonidal sinus (abscess)<br />

a. Occurs in the midline (at the superior edge of the buttock crease) and is more common in men than women<br />

b. The sinus is formed when an ingrowing hair penetrates the skin and induces a foreign body granuloma<br />

reaction.<br />

c. A single opening at the base of the spine with hair protruding is the most common physical finding.<br />

d. When the sinus becomes plugged and cannot drain---,, abscess<br />

e. Management<br />

(1) Surgical incision longitudinal and lateral to the sacral midline with drainage of purulent material<br />

(2) Refer for definitive surgical care.<br />

(3) Antibiotics are indicated if there is an associated cellulitis or the patient is immunocompromised.<br />

4. Anorectal tumors<br />

a. Classification: based on their virulence, these tumors have been arranged into two groups with distinct<br />

anatomic locations.<br />

(1) Anal canal neoplasms (found proximal to the dentate line)<br />

(a) Represent 80% of anorectal tumors<br />

(b) Have a high-grade malignant potential and metastasize early; prognosis is poor.<br />

(c) Tumors occurring in this region<br />

i. Adenocarcinoma (common)<br />

ii. Mucoepidermoid carcinoma<br />

iii. Malignant melanoma<br />

1v. Kaposi sarcoma<br />

v. Squamous cell carcinoma<br />

vi. Basaloid carcinoma<br />

vii. Vil lous adenoma<br />

(2) Anal margin neoplasms (found distal to the dentate line)<br />

(a) Represent 20% of anorectal tumors<br />

(b) Have a low-grade malignant potential, slow to metastasize, good prognosis<br />

(c) Tumors occurring in this region include:<br />

i. Basal cell carcinoma<br />

ii. Squamous cell carcinoma<br />

iii. Bowen disease<br />

iv. Extramammary Paget disease<br />

v. Giant solitary trichoepithelioma<br />

b. Clinical presentation: pruritus, rectal bleeding, pain, weight loss, anorexia, constipation, decrease in fecal<br />

caliber, tenesmus, and obstruction<br />

c. Management: prompt surgical referral<br />

5. Rectal foreign bodies<br />

a. Diagnostic evaluation<br />

(1) Abdominal radiographs should be obtained to ascertain the location, size, shape, and number of<br />

objects present as well as to exclude the presence of free air.<br />

(2) Multiple views may be needed.<br />

b. Management<br />

(1) If there is clinical or radiographic evidence of free air, the patient should be prepared for surgery and<br />

an immediate surgical consult obtained.<br />

(2) After removal of the foreign body, the patient should be observed for 12 hours, and radiograph studies<br />

and sigmoidoscopy performed.<br />

c. Complications<br />

(1) Perforation is the most frequent; it results from the foreign body itself or attempts to remove it.<br />

(2) Mucosa! tears can also occur.<br />

202


ABDOMINAL AND GASTROINTESTINAL DISORDERS<br />

VII. DIARRHEA AND FOOD POISONING<br />

A. General information<br />

1. The vast majority of patients in the emergency department suspected of having "gastroenteritis" have a benign,<br />

self-limited disease. Evaluation should distinguish between acute or chronic diarrhea as well as the presence of<br />

invasive or noninvasive organisms.<br />

2. Acute gastroenteritis is defined as symptoms being present for 2 weeks) suggests an alternative cause such as intestinal parasites or<br />

other disorders.<br />

4. Invasive gastroenteritis is associated with fever, abdominal pain, blood in the feces, and tenesmus. Presence of<br />

these symptoms suggests a bacterial cause, and testing for specific organisms may be warranted.<br />

5. Noninvasive disease is suggested by the absence of the above symptoms. The feces are not bloody in most<br />

cases. Suggests a viral cause or pre-formed toxin, and testing is rarely warranted.<br />

6. If tested, the presence of fecal blood and leukocytes helps distinguish invasive from noninvasive.<br />

B. Viral diarrheal diseases (most acute episodes of diarrhea)<br />

1. Although many viruses can cause gastroenteritis, two groups have become most common over the last 10<br />

years: reovirus-like agents such as rotavirus and the noroviruses, which include norovirus.<br />

a. Rotavirus infection<br />

(1) Outbreaks are usually sporadic, usually in winter months.<br />

(2) Primarily in children 6-24 months old<br />

(3) Incubation period of 1-3 days<br />

(4) Clinical presentation<br />

(a) Onset usually abrupt with vomiting, low-grade fever, and watery diarrhea. Vomiting usually lasts<br />

1-1 .5 days and resolves spontaneously. Diarrhea may last 4-7 days.<br />

(b) Abdominal cramping is common.<br />

(5) Diagnostic evaluation: diagnosis is confirmed by detection of antigens in the feces by latex<br />

agglutination, PCR, or ELISA. However, testing is not usually required.<br />

(6) Not uncommonly, children become dehydrated and require IV hydration and hospitalization. The<br />

disease is an uncommon cause of mortality in the United States.<br />

(7) A vaccine, introduced in 2006, is given to babies at 2, 4, and 6 months of age.<br />

b. Norovirus<br />

(1) Up to 80% of diarrheal illness in industrialized countries<br />

(2) Most common cause of epidemic, nonbacterial gastroenteritis in the world<br />

(3) Can be transmitted person to person by fecal-oral route<br />

(4) Water and food-borne outbreaks common; up to 50% of food-borne outbreaks caused by norovirus<br />

(a) Stored water on cruise ships a common source<br />

(b) Shellfish, food handlers; raw oysters implicated<br />

(5) In January 2013, the CDC reported a new strain, Gll.4 Sydney, which appears to cause more severe<br />

disease and results in more hospitalizations and deaths.<br />

(6) Clinical presentation<br />

(a) After an incubation period of 1-2 days, patients typically have an abrupt onset of nausea,<br />

nonbloody watery diarrhea, abdominal cramps, and low-grade fever. Vomiting may be severe,<br />

nonbloody, nonbilious.<br />

(b) There is no upper or lower respiratory tract involvement.<br />

(c) Symptoms last for 1-2 days.<br />

(7) Diagnosis is usually clinical, although blood and fecal tests are available.<br />

(8) Management: disease is self-limited and requires only supportive, symptomatic care.<br />

C. Bacterial diarrheal diseases (20% of acute diarrheal cases)<br />

1. Classification<br />

a. Invasive bacteria act primarily on the large bowel and produce diarrhea by damaging cell membranes and<br />

eliciting an inflammatory response - blood and mucus in the feces<br />

203


ABDOMINAL AND GASTROINTESTINAL DISORDERS<br />

(1) Campylobacter<br />

(2) Salmonella<br />

(3) Shigella<br />

(4) Vibrio parahaemolyticus<br />

(5) Vibrio vulnificus<br />

(6) Yersinia enterocolitica<br />

(7) Enteroinvasive E coli<br />

(8) Enterohemorrhagic E coli serotype 0157:H7<br />

(9) Clostridium difficile<br />

b. Enterohemorrhagic E coli serotype 0157:H7 and C difficile are actually toxin-producing bacteria that<br />

induce diarrhea by elaborating a cytopathic toxin that destroys the intestinal wall. They are grouped with<br />

the invasive bacteria, because the clinical syndrome they produce most closely resembles that produced by<br />

these organisms.<br />

c. Enterotoxin-producing bacteria release a toxin that acts primarily in the small intestine and produces<br />

diarrhea by altering water and electrolyte transport in epithelial cells - profuse watery diarrhea<br />

(1) Staphylococcus aureus<br />

(2) Bacillus cereus<br />

(3) Ciguatera fish poisoning<br />

(4) Scombroid fish poisoning<br />

(5) Enterotoxigenic E coli<br />

(6) Clostridium perfringens<br />

(7) Aeromonas hydrophila<br />

(8) Vibrio cholerae<br />

2. Fecal leukocytes (a wet mount or methylene blue stain of the feces)<br />

a. May be used as a screen for those patients who may benefit from antibiotic therapy<br />

(1) Invasive bacterial pathogens producing fever, systemic illness, or bloody feces: Salmonella, Shigella,<br />

Campylobacter, and f coli 0157:H7<br />

(2) Pseudomembranous enterocolitis<br />

(3) Amebiasis<br />

(4) Inflammatory bowel disease (Crohn disease and ulcerative colitis)<br />

Table 7: Fecal Leukocytes in Diarrheal Syndromes<br />

Fecal Leukocytes Present<br />

Campylobacter<br />

Salmonella<br />

Shigella<br />

Vibrio parahaemolyticus<br />

Vibrio vulnificus<br />

Yersinia<br />

Amebiasis<br />

Enteroinvasive E coli<br />

Enterohemorrhagic E coli<br />

Clostridium difficile<br />

Ulcerative colitis<br />

Crohn disease<br />

Fecal Leukocytes Absent<br />

Rotavirus<br />

Norovirus<br />

Enteric type adenovirus<br />

Staphylococcus aureus<br />

Bacillus cereus<br />

Ciguatera fish poisoning<br />

Scombroid fish poisoning<br />

Enterotoxigenic E coli<br />

Clostridium perfringens<br />

Aeromonas hydrophila<br />

Vibrio cholerae<br />

Giardia lamblia<br />

204


ABDOMINAL AND GASTROINTESTINAL DISORDERS<br />

b. Although treatment based on the presence or absence of fecal leukocytes is not 1 00% accurate, it is a good<br />

place to start, and is one of the few test results for diarrheal illnesses that is immediately available. Presence<br />

of fecal leukocytes (>5 WBCs per high-power field) has an LR of 4.5 for the presence of an invasive<br />

pathogen.<br />

3. Traveler's diarrhea<br />

a. Definition: syndrome acquired by travelers who eat food or water that is fecally contaminated<br />

b. Risk factors<br />

(1) Travel to developing tropical regions (Mexico, Latin America, southern Asia, Africa) where sanitation<br />

is poor<br />

(2) Ingestion of food prepared by street vendors<br />

(3) Ingestion of certain unsafe (often contaminated) items such as tap water, ice cubes, unpasteurized<br />

dairy products, raw vegetables, unpeeled fruit, and raw or undercooked seafood or meat<br />

c. Most cases (70%-75%) are caused by bacteria, of which enterotoxigenic E coli is the most commonly<br />

involved pathogen; it is responsible for up to 50% of cases worldwide.<br />

d. Clinical presentation: symptoms often begin abruptly and consist of watery diarrhea and abdominal<br />

cramps; other associated symptoms may be present and vary with the particular etiologic agent involved.<br />

e. Antibiotic prophylaxis<br />

(1) Not recommended for healthy individuals because of the associated risks (eg, emergence of resistant<br />

organisms, allergic reactions, photosensitivity reactions, etc)<br />

(2) Bismuth subsalicylate 2 tablets 4 times a day and strict adherence to preventive methods (avoiding the<br />

above) recommended by the CDC.<br />

(3) Initiation of treatment at the onset of symptoms is preferred.<br />

f. Management<br />

(1) Oral rehydration with electrolyte replacement such as found in sports drinks, caffeine-free soft drinks,<br />

or diluted fruit juice effectively maintains fluid and electrolyte balance in most patients.<br />

(2) Antibiotics<br />

(a) The CDC recommends a fluoroquinolone such as ciprofloxacin or norfloxacin for 3-5 days.<br />

(b) Rifaximin also approved for treatment at onset of symptoms.<br />

(c) TMP-SMX no longer recommend because of resistance.<br />

(3) Anti motility agents such as loperamide provide rapid symptomatic relief but should not be continued<br />

for >2 days and should not be used in patients who have fevers or bloody diarrheas, because these<br />

may increase severity and prolong symptoms.<br />

(4) Bismuth subsalicylate is an anti secretory agent that is also effective in reducing the number of bowel<br />

movements and the duration of illness. It works by decreasing the outpouring of fluids by the mucosa<br />

of the smal I intestine.<br />

4. Specific bacterial infections<br />

a. f coli: several strains have been shown to be diarrheagenic, two of which are discussed below. These<br />

strains differ in terms of their virulence, epidemiology, and clinical characteristics.<br />

(1) Enterotoxigenic f coli<br />

(a) Most common cause of traveler's diarrhea worldwide<br />

(b) Acquired by ingesting fecally contaminated food and water<br />

(c) Causes diarrhea by producing both heat-labile and heat-stable toxins<br />

(d) Clinical presentation<br />

i. Incubation period 1-4 days<br />

ii. Symptoms usually begin suddenly and consist of watery diarrhea associated with abdominal<br />

cramping.<br />

(e) Management<br />

i. Most cases resolve in 2-3 days with supportive measures alone.<br />

ii. The use of antibiotics and antimotility agents (see traveler's diarrhea, above) can shorten the<br />

duration of this syndrome.<br />

(2) Shiga toxin-producing f coli (STEC), also known as enterohemorrhagic f coli<br />

(a) Serotype 0157:H7 is most common isolate in the United States.<br />

(b) Transmission occurs by ingesting small amounts of human or animal feces. Person-to-person<br />

spread is also possible. Exposure to cattle can also transmit disease.<br />

205


ABDOMINAL AND GASTROINTESTINAL DISORDERS<br />

(c) Outbreaks commonly occur due to improperly cooked ground beef, fruits, or vegetables. Highrisk<br />

foods are raw milk, soft cheeses made from unpasteurized milk, unpasteurized apple cider,<br />

and nondisinfected water.<br />

(d) Outbreaks are associated with ingestion of contaminated, undercooked ground beef, seed<br />

sprouts, or unpasteurized milk; however, person-to-person spread also occurs.<br />

(e) Most common in children, the elderly, and post-gastrectomy patients.<br />

(f) This organism produces Shiga toxins that are cytotoxic to the intestinal vascular endothelium.<br />

(g) Clinical presentation<br />

1. Incubation period 3-4 days on average<br />

ii. Patient presents with diarrhea, crampy abdominal pain, and vomiting. The cramps can be<br />

severe enough to masquerade as an acute abdomen. The diarrhea is watery early on but<br />

eventually becomes bloody. Fever is absent or low grade. WBCs are present on the wet<br />

mount but are few in number.<br />

(h) Diagnostic evaluation: fecal testing for Shiga toxin along with cultures for f coli O157:H7<br />

(i)<br />

(j)<br />

Management<br />

i. Antibiotic therapy is contraindicated; antibiotics have not been shown to decrease the<br />

duration of symptoms, and they may increase the risk of hemolytic uremia syndrome.<br />

ii. Antidiarrheal agents may also increase the risk of hemolytic uremia syndrome.<br />

iii. Most cases resolve in 7-10 days with supportive measures.<br />

Complications<br />

i. Hemolytic uremic syndrome<br />

• Occurs in about 8% of children, of which 5%-10% of cases are fatal<br />

• Occurs in nearly 40% of elderly patients who acquire STEC in a nursing-home outbreak;<br />

up to 80% fatality rate<br />

11. Thrombotic thrombocytopenic purpura develops in ~2% of patients, usually<br />

immunocompromised.<br />

b. Shigella (a common cause of bacterial diarrhea)<br />

(1) Shigellosis (bacillary dysentery) occurs worldwide but is especially common in countries where<br />

adequate sanitary facilities are lacking.<br />

(2) The CDC estimates an annual incidence in the United States of 4 cases per 100,000 persons.<br />

(3) Common in "contained" populations, eg, nursing homes, prisons, schools, day care facilities;<br />

outbreaks have been reported in water parks, swimming pools, etc.<br />

(4) Shige!la sonnei causes ~75% of cases in the United States; Shige!!a flexneri causes most of the rest.<br />

(5) Clinical presentation<br />

(a)<br />

Incubation period is 24-48 hours after exposure to even a very small inoculum.<br />

(b) Mild, watery diarrhea with scant systemic symptoms may be seen in most early disease.<br />

Dysentery can then ensue with fevers, crampy abdominal pain, grossly bloody diarrhea, vomiting,<br />

headaches, and myalgia. Profound dehydration may develop, especially in infants and the elderly.<br />

(c) Convulsions may occur in young children, usually those 800 is suggestive of<br />

shigellosis.<br />

(b) A wet mount of the feces with methylene blue demonstrates numerous fecal leukocytes.<br />

(c)<br />

(7) Management<br />

Fecal cultures are positive in most cases, particularly if obtained within the first few days of illness.<br />

(a) Fluid resuscitation or maintenance of hydration is of paramount importance.<br />

(b) Antibiotics decrease the duration of illness and excretion of Shigella organisms in the feces. They<br />

are recommended in patients with dysentery, in the very young and very old, and in institutional<br />

outbreaks. Management should continue for 3 days in immunocompetent patients and for 10 days<br />

in immunocompromised.<br />

i. Quinolones (ciprofloxacin, norfloxacin, ofloxacin) are the drugs of first choice, but they are<br />

contraindicated in pregnancy and in children


ABDOMINAL AND GASTROINTESTINAL DISORDERS<br />

ii. Ceftriaxone may be useful.<br />

iii. Ampicillin and TMP-SMX are not effective and are no longer considered viable treatment<br />

options unless fecal culture indicates effectiveness.<br />

(c) Antimotility drugs should be avoided, because they can prolong duration of disease.<br />

(8) Complications<br />

(a) Dehydration secondary to profuse diarrhea<br />

(b) Arthralgias<br />

(c) Reiter syndrome (nongonococcal urethritis, polyarthritis, conjunctivitis)<br />

(d) Hemolytic uremic syndrome<br />

(e) Febrile seizures (common in children


ABDOMINAL AND GASTROINTESTINAL DISORDERS<br />

(b) Septicemia or typhoid fever (enteric fever): ceftriaxone is the agent of choice; alternatives<br />

include ciprofloxacin, chloramphenicol, and ampicillin.<br />

(c) Antidiarrheal agents are contraindicated, because they prolong the illness and increase the<br />

incidence of bacteremia and the carrier state.<br />

d. Campylobacter enteritis<br />

(1) Campylobacter infection is the most common cause of bacterial diarrhea.<br />

(2) It is transmitted by ingestion of fecally contaminated water or food (particularly poultry) or by direct<br />

contact with feces of infected animals or people; it produces disease by directly invading the colonic<br />

epithelium.<br />

(3) Clinical presentation<br />

(a)<br />

Incidence of infection is greater in young children.<br />

(b) Incubation period is 2-5 days.<br />

(c) Onset of disease is usually rapid and consists of fever and crampy, abdominal pain. Malaise,<br />

myalgia, and headache are common. Abdominal pain may be severe enough to mimic an acute<br />

abdomen.<br />

(d) Diarrhea usually begins 1-2 days after the fever and pain. It is frequently loose and bile colored,<br />

subsequently turning watery and bloody. Gross blood is seen in 60% of cases.<br />

(4) Diagnostic evaluation<br />

(a) Wet mount shows blood and fecal leukocytes.<br />

(b) Fecal culture confirms the diagnosis.<br />

(5) Management<br />

(a) Antimotility agents should be avoided.<br />

(b) Patients with mild disease do not require treatment other than supportive care. Empiric<br />

antibiotics are not recommended.<br />

(c) Patients who are still symptomatic when culture results are returned may be treated with<br />

erythromycin or azithromycin. Ciprofloxacin in no longer recommended because of widespread<br />

resistance.<br />

(6) Complications<br />

(a) Reiter syndrome<br />

(b) Hemolytic uremic syndrome<br />

(c) Guillain-Barre syndrome (a late sequela)<br />

e. Yersinia enterocolitis<br />

(1) Infection results from ingestion of contaminated food or drink and is most common in childhood.<br />

(2) Clinical presentation<br />

(a)<br />

Patient presents with fever and diarrhea 2-6 days after exposure that is accompanied by severe,<br />

cramping abdominal pain that may last up to 2 weeks.<br />

(b) Dysentery (bloody diarrhea) may be present in up to 25% of patients.<br />

(c) On physical examination, the abdomen is tender and, if the organism has caused mesenteric<br />

adenitis or terminal ileitis (which it sometimes does), you may think the patient has appendicitis.<br />

(d) Recent exposure to domestic, farm, or wild animals can be a significant diagnostic clue.<br />

(e)<br />

Postinfectious manifestations (erythema nodosum, polyarthritis) sometimes occur, particularly in<br />

adults.<br />

(3) Diagnostic evaluation<br />

(a) Wet mount of the feces shows fecal leukocytes and occasionally RBCs.<br />

(b) Fecal culture (special techniques are required) confirms the diagnosis.<br />

(4) Management<br />

(a) Mild to moderate infections: supportive therapy; however, Yersinia takes a long time to isolate<br />

and identify.<br />

(b) If antibiotics are started pending results, TMP-SMX is the antibiotic of choice. Quinolones are<br />

also effective.<br />

(c) Antimotility drugs should be avoided.<br />

f. Clostridium perfringens<br />

(1) A common cause of food poisoning; relatively large outbreaks are typical.<br />

208


ABDOMINAL AND GASTROINTESTINAL DISORDERS<br />

(2) Food poisoning occurs when food contaminated with the live organism containing type A heatresistant<br />

spores are ingested. Typically, the food (meat or poultry, gravies, steam-table meats) was<br />

prepared earlier and allowed to cool. C perfringens multiplies and is then ingested. In the gut, the<br />

organism produces an enterotoxin _,. diarrhea and abdominal cramps<br />

(3) Clinical presentation<br />

(a)<br />

Incubation period is generally 1-5 hours.<br />

(b) Diarrhea and abdominal cramps common. Vomiting is not as common.<br />

(c) Fever, headache, and chills are uncommon.<br />

(d) No WBCs are seen on a wet mount of the feces.<br />

(4) Management<br />

(a) This is a self-limited disease, with symptoms resolving in 10-12 hours.<br />

(b) Only supportive treatment is indicated.<br />

g. Staphylococcus aureus (the most common cause of food poisoning)<br />

(1) Illness results from ingestion of a heat-stable enterotoxin.<br />

(a) Cooking at a temperature> 140°F (60°C) will kill bacteria but not destroy the toxin.<br />

(b) Most protein-rich foods (eg, ham, eggs, mayonnaise, potato salad) support growth of staphylococci.<br />

(c) Contaminated food has no abnormal odor or taste.<br />

(2) Large outbreaks are common; the attack rate is >75%.<br />

(3) Clinical presentation<br />

(a) Incubation period is 1-6 hours, followed by abrupt onset of abdominal cramping and violent<br />

vomiting. Diarrhea may or may not occur.<br />

(b) Usual scenario: patient went to a barbecue or buffet about 6 hours before arrival where he or<br />

she had eaten some potato salad or sliced ham that had been sitting at room temperature for<br />

several hours.<br />

(4) Treatment is supportive; resolves in 6-1 0 hours.<br />

h. Bacillus cereus<br />

(1) Two clinical syndromes<br />

(a) An emetic syndrome caused by a heat-stable enterotoxin<br />

1. Begins 1-6 hours after ingestion of contaminated food. Fried rice is a common source; beef,<br />

poultry and milk have been implicated.<br />

ii. Symptoms are indistinguishable from those of staphylococcal poisoning. Abrupt onset of<br />

vomiting and crampy abdominal pain. Up to 25% have diarrhea.<br />

iii. Treatment is supportive; resolves in :::10 hours.<br />

(b) A diarrheal syndrome caused by a heat-labile enterotoxin<br />

1. Begins 6-24 hours after ingestion of contaminated meal (usually meat or vegetables)<br />

ii. Syndrome is similar in presentation to that of Clostridium poisoning. Symptoms include<br />

diarrhea and abdominal cramps. Vomiting is present in 20%.<br />

(2) Diagnosis can be confirmed by isolation of> 10 5 colonies of B cereus from the food source. However,<br />

because of the extremely short nature of the disease, this is rarely done.<br />

(3) Treatment is supportive; resolves in 10-24 hours.<br />

1. Aeromonas hydrophila<br />

(1) More common in summer. Usually acquired by drinking untreated water from a well or spring, or<br />

eating seafood or vegetables.<br />

(2) Most commonly affects children and immunocompromised patients<br />

(3) Clinical presentation<br />

(a) Watery diarrhea, crampy abdominal pain, vomiting (25%), fever (50%)<br />

(b) History of drinking from wells or fresh springs increases likelihood.<br />

(4) Diagnostic evaluation<br />

(a) Confirmed by fecal culture; however, routine culture does not test for Aeromonas, so it must be<br />

specifically requested.<br />

(b) Fecal leukocytes and occult blood are usually absent. However, some patients present with<br />

symptoms suggesting colitis and have fecal leukocytes and blood; this may mimic Crohn disease.<br />

209


ABDOMINAL AND GASTROINTESTINAL DISORDERS<br />

(5) Management<br />

(a)<br />

Supportive measures will suffice in most patients, because disease is usually self-limited.<br />

(b) Antibiotic therapy with one of the following agents:<br />

i. TMP-SMX has shown good activity.<br />

ii. Fluoroquinolones are also effective.<br />

(c) Without antimicrobial therapy, the diarrhea generally persists for c::2 weeks.<br />

(d) Lactobacillus decreases duration of diarrhea in children.<br />

j. Vibrio cholerae<br />

(1) Transmitted by ingestion of contaminated water or seafood (especially raw oysters), which produces an<br />

enterotoxin-mediated diarrheal illness (cholera).<br />

(2) Large epidemics related to ingestion fecally contaminated water supplies.<br />

(3) Clinical presentation<br />

(a) Copious amounts of watery diarrhea ("rice water" feces) is the hal I mark of clinical cholera and<br />

can lead to significant fluid and electrolyte imbalances.<br />

(b) Severe dehydration (loss of isotonic fluid from bowel)<br />

(c) Hyperchloremic acidosis (loss of bicarbonate in the feces)<br />

(d) Hypokalemia (loss of potassium in the feces)<br />

(e)<br />

(f)<br />

Dehydration and hypoglycemia most common lethal complications of cholera in children.<br />

After an incubation period of 2 days, the patient presents with copious watery diarrhea and<br />

abdominal distention. Vomiting may also be present.<br />

(4) Diagnostic evaluation<br />

(a) Diagnosis is confirmed by fecal culture.<br />

(b) Crystal VC® rapid diagnostic test can be used for rapid identification of two serotypes in epidemic<br />

situations to establish diagnosis.<br />

(5) Management<br />

(a)<br />

Fluids (oral or IV) should be the focus of therapy.<br />

(b) Antibiotic therapy in those with moderate to severe dehydration. Doxycycline is drug of choice in<br />

adults; azithromycin is first choice in children and pregnant women.<br />

(c) Zinc is recommended for all patients to decrease duration and quantity of diarrhea.<br />

k. Vibrio parahaemolyticus<br />

(1) Leading cause of seafood-associated gastroenteritis in the United States<br />

(2) Invasive bacterial infection; other Vibrio species produce an enterotoxin to cause disease.<br />

(3) Acquired by ingestion of raw or improperly prepared seafood (especially oysters, clams, shrimp, and<br />

crabs); exposure to brackish or salty water is potential cause as wel I.<br />

(4) In temperate climates, this infection is usually confined to the summer months.<br />

(5) Clinical presentation<br />

(a) Symptoms begin after an average incubation period of 12 hours and range from mild<br />

gastroenteritis to explosive diarrhea with vomiting, cramps, and dysentery.<br />

(b) Fever may also be present.<br />

(c) Can also cause wound infection if open wound exposed to seawater<br />

(6) Diagnostic evaluation<br />

(a) Wet mount of the feces reveals numerous WBCs.<br />

(b) Fecal culture requires special growth media, so laboratory must be notified.<br />

(7) Management<br />

(a)<br />

Symptomatic therapy is generally all that is needed, because the illness is self-limited and resolves<br />

in 2-3 days.<br />

(b) Tetracycline or a quinolone may be given for severe infections (dysentery), but their efficacy<br />

remains unclear; they neither shorten the clinical course nor decrease the duration of shedding of<br />

the organism.<br />

I. Scombroid fish poisoning<br />

(1) The most commonly implicated fish is mahi mahi (blue dolphin or dolphin fish), but it is also<br />

common with ingestion of tuna, mackerel, and other dark-fleshed or red-muscled fish.<br />

(2) Most cases are in Hawaii and Florida, although it can be seen anywhere "fresh fish" is flown in.<br />

210


ABDOMINAL AND GASTROINTESTINAL DISORDERS<br />

(3) Poisoning results from ingestion of heat-stable toxins with histamine-like properties produced by<br />

bacterial action on dark-meat fish; high levels of histamine correlate with the manifestations of the<br />

illness. Formation of the toxin is related to improper preservation and refrigeration of the fish. (This<br />

is not an allergic reaction.)<br />

(4) Clinical presentation<br />

(a) Patient presents with signs and symptoms of histamine intoxication 20-30 minutes after<br />

ingestion of the fish:<br />

i. Facial flushing (resembles a sunburn and can extend over entire upper body)<br />

ii. Throbbing headache<br />

iii. Abdominal cramps<br />

iv. Nausea, vomiting, and diarrhea<br />

v. Palpitations<br />

vi. Bronchospasm or hypotension (with severe toxicity)<br />

(b) Historical clue: at time of ingestion, the patient noticed a sharp, metallic, bitter, or peppery<br />

taste to the fish.<br />

(5) Diagnostic evaluation<br />

(a) Diagnosis is based on clinical grounds with symptoms beginning shortly after ingestion of the fish.<br />

(b) If necessary, histamine levels can be checked on the suspect fish.<br />

(6) Management<br />

(a) Depending on severity of symptoms, H 1<br />

- and H 2<br />

-blockers should be administered IV or orally.<br />

(b) Activated charcoal may be administered if a large amount of fish was ingested.<br />

(c) Additional therapy may be required.<br />

i. Antiemetics (for nausea and vomiting)<br />

ii. Albuterol and steroids (for bronchospasm)<br />

iii. Epinephrine (for anaphylactoid reaction)<br />

m. Ciguatera fish poisoning<br />

(1) Most common cause of nonbacterial fish-associated food poisoning in the United States<br />

(2) Results from ingestion of ciguatoxin, a tasteless, odorless, heat-stable neurotoxin that accumulates<br />

in the flesh of certain carnivorous tropical and semitropical coral-reef fish when a particular<br />

dinoflagellate is present in their food chain during the late spring and summer months. The most<br />

commonly affected species are grouper, snapper, barracuda, king fish, and jack. Because the toxin<br />

accumulates in the flesh of these fish, the bigger and older the fish, the more likely that it carries<br />

the toxin.<br />

(3) Clinical presentation<br />

(a) Ciguatoxin affects the sodium channels, so effects are most pronounced in nervous, cardiac,<br />

and GI tissues.<br />

(b) Incubation period is usually 2-6 hours but ranges from 15 minutes to 24 hours.<br />

(c) GI symptoms last 1-2 days and include abdominal pain, nausea, vomiting, and diarrhea.<br />

(d) Neurologic symptoms occur within hours up to 3 days and include painful paresthesias,<br />

circumoral and throat paresthesias, weakness, and coma. Paralysis of respiratory muscles has<br />

been reported. Pruritus is a common complaint. Neurosensory symptoms may persist for months<br />

and are exacerbated by ingestion of alcohol.<br />

(e) Pathognomonic "hot-cold reversal" likely represents intense dysesthesia experienced with cold<br />

stimuli rather than true reversal of temperature perception.<br />

(f) Cardiovascular symptoms occur least frequently but may include symptomatic bradycardia.<br />

(4) Management<br />

(a) Supportive measures (IV fluids, antiemetics, analgesics)<br />

(b) H 1<br />

- and H 2<br />

-blockers may help with pruritus. Amitriptyline may be useful for pruritus and<br />

dysesthesia.<br />

(c) IV mannitol is no longer considered an effective therapy and is not recommended.<br />

(d) Absolute abstinence from alcohol, seafood, and nuts until all symptoms have resolved;<br />

continued abstinence is recommended for 3-6 months after exposure.<br />

5. Summary recommendations for gastroenteritis<br />

a. Antibiotics should be limited to patients who are significantly ill or toxic in appearance that are believed<br />

(based on clinical grounds) to have an invasive or infectious diarrhea.<br />

211


ABDOMINAL AND GASTROINTESTINAL DISORDERS<br />

b. Early administration of antibiotics to these patients decreases the intensity and duration of symptoms,<br />

prevents systemic and suppurative sequelae, and reduces spread of infection.<br />

c. Unless contraindications exist (eg, pregnancy, children 7 days should be tested for parasites.<br />

1 . Entamoeba histolytica<br />

a. Protozoa with cystic and trophozoite phases<br />

b. Transmission usually occurs through ingestion of cysts (the infective phase) in water or food contaminated<br />

with feces. Cysts are stable in the environment and passed via fecal-oral transmission. Infection occurs as<br />

the cysts change into a trophozoite phase in the terminal ileum or colon. This is the invasive form, which<br />

causes symptoms.<br />

c. Trophozoites can invade the mucosa I barrier of the colon and produce bloody diarrhea and colitis, which<br />

may resemble inflammatory bowel disease. It is estimated that only 10%-20% of those exposed develop<br />

disease symptoms.<br />

d. Risk factors<br />

(1) People who have traveled to tropical areas with poor sanitation; St. Petersburg, Russia, has had an<br />

ongoing outbreak.<br />

(2) Immigrants from tropical areas with poor sanitation<br />

(3) People who live in institutions with poor sanitation<br />

(4) Native Americans living on reservations<br />

(5) Men who have sex with other men<br />

e. Clinical presentation<br />

(1) Chronic amoebic colitis is the usual manifestation.<br />

(a) Gradual onset with intermittent diarrhea<br />

(b) Foul-smelling feces with bloody mucus<br />

(c) Mild abdominal cramping and flatulence<br />

(2) Acute amoebic dysentery<br />

(a) 2-4 weeks after exposure, although can be longer<br />

(b) Sudden onset of severe abdominal cramping<br />

(c) Profuse bloody diarrhea<br />

(3) Amoebic abscess<br />

(a)<br />

Liver (most common)<br />

(b) Fever, right upper quadrant pain and tenderness, weight loss<br />

(c) Usually diagnosed by ultrasound or CT<br />

f. Diagnostic evaluation<br />

(1) Fecal studies<br />

(a) Trophozoites from single fecal specimen only 33% sensitive; examination of three samples within<br />

1 0 days is 90% sensitive.<br />

(b) Fecal leukocytes may be present in small numbers.<br />

(c) Fecal examination in patients with extraintestinal amebiasis is rarely helpful, because organisms<br />

are found in


ABDOMINAL AND GASTROINTESTINAL DISORDERS<br />

2. Giardiasis<br />

a. Causative organism is Ciardia lamblia (also known as Ciardia intestinalis); most frequently diagnosed<br />

parasitic disease in the United States and travelers with chronic diarrhea.<br />

b. Transmitted by cysts via the fecal-oral route.<br />

(1) Found in soi I, food, and water that have been contaminated with feces containing the parasite.<br />

(2) The parasite is protected by an outer coat that allows it to survive outside the body for long periods of<br />

time. This also makes it resistant to disinfection by chlorine.<br />

(3) Most common method of transmission is by drinking contaminated water, ponds, streams etc; often<br />

called "backpackers diarrhea."<br />

(4) Beavers play a role in the spread of this disease by contaminating streams with cysts.<br />

(5) Venereal transmission among homosexuals (by direct fecal-oral contamination) is also common.<br />

c. Symptoms, if they occur, are produced by trophozoites and begin after an incubation period of 1-3 weeks;<br />

most patients, however, are asymptomatic.<br />

d. Risk factors<br />

(1) Backpackers, others who drink unsafe water<br />

(2) Playing in contaminated water, swallowing water<br />

(3) lmmunocompromised patients (lgA deficiency)<br />

(4) People who live in institutions<br />

(5) Children attending daycare centers<br />

(6) Travelers returning from underdeveloped countries (especially Russia) or a backpacking trip to Colorado<br />

(7) Patients with decreased gastric acidity from any cause<br />

e. Classic clinical scenario: The patient has recently returned from a back-packing trip in Colorado (or<br />

from travel to Russia) and presents with abdominal pain and feeling bloated and gaseous. Most common<br />

symptoms are explosive diarrhea, flatulence, colicky pain, and greasy, floating, foul-smelling feces.<br />

f. Diagnostic evaluation<br />

(1) Presence of trophozoites or cysts in the feces; several samples over separate days may be required.<br />

(2) Fecal immunoassays are available that are more sensitive and specific. The CDC recommends using<br />

these tests to screen high-risk patients (eg, daycare), but ova and parasite examination of the feces is<br />

the primary means of diagnosis.<br />

(3) Duodenal aspiration may need to be performed in asymptomatic patients and those with chronic<br />

symptoms.<br />

(4) Eosinophilia is absent.<br />

g. Management<br />

(1) Asymptomatic patients who shed the organism should be treated only to prevent spread to high-risk<br />

household contacts.<br />

(2) All asymptomatic patients should be treated.<br />

(a) Metronidazole (drug of choice)<br />

(b) Furazolidone (less effective than metronidazole but is available as a suspension and, for this<br />

reason, is often preferred in children)<br />

(c) Tinidazole (a congener of metronidazole or albendazole)<br />

h. Patients with recurrent or refractory giardiasis should be tested for immunoglobulin A deficiency.<br />

3. Cryptosporidium<br />

a. Intestinal protozoan coccidial parasite; causes disease with clinical features similar to those of /sospora.<br />

b. Most common cause of chronic diarrhea in patients with AIDS<br />

c. Transmitted by cysts from human or animal sources via the fecal-oral route; highly infectious and highly<br />

resistant to disinfection<br />

d. Symptoms, if they occur, are produced by trophozoites after an incubation period of 1-2 weeks; some<br />

patients are asymptomatic.<br />

e. Risk factors<br />

(1) lmmunocompromised patients (especially those with AIDS)<br />

(2) Homosexual men<br />

(3) Swallowing or drinking untreated water<br />

(4) Children attending daycare centers<br />

213


ABDOMINAL AND GASTROINTESTINAL DISORDERS<br />

f. Clinical presentation<br />

(1) Profuse watery diarrhea, abdominal cramping, anorexia, nausea, and flatulence<br />

(2) In immunocompetent patients, these symptoms may persist for 1-3 weeks and result in significant<br />

dehydration, but they are generally self-limited.<br />

(3) In immunocompromised patients, malabsorption and significant weight loss occurs as well; symptoms<br />

may persist for months to years, producing significant discomfort and morbidity.<br />

g. Diagnostic evaluation<br />

(1) Fecal studies for oocysts using an acid-fast stain or enzyme immunoassay testing for Cryptosporidium<br />

(> 95% sensitive); must specify to the laboratory that the sample is to be tested for Cryptosporidium<br />

(2) Fecal blood and leukocytes are usually absent.<br />

(3) Eosinophilia is absent.<br />

h. Management<br />

(1) lmmunocompetent patients<br />

(a)<br />

Rehydration and symptomatic therapy<br />

(b) Nitazoxanide may hasten recovery.<br />

(2) Immunodeficient patients are resistant to treatment.<br />

(a) When possible, therapy should be directed toward correcting the underlying immunodeficiency.<br />

(b) Nitazoxanide or paromomycin plus azithromycin may decrease symptoms in these patients, but is<br />

not curative.<br />

(c) In AIDS patients, initiation of highly active antiretroviral therapy (HAART) is probably the best<br />

treatment, because it directly increases host immunity by suppressing HIV replication.<br />

4. Cystoisospora (formerly lsospora bell,)<br />

a. Coccidian parasite<br />

b. Transmitted by cysts via the fecal-oral route<br />

c. Usually an opportunistic infection, it is seen most commonly in patients with AIDS (especially in<br />

developing countries); most common in tropical and subtropical regions.<br />

d. Clinical presentation<br />

(1) Symptoms, when present, begin after an incubation period of 1-2 weeks and last for 2-6 weeks;<br />

some immunocompetent patients are asymptomatic, whereas immunocompromised patients tend to<br />

have a severe protracted course.<br />

(2) The typical patient is a 30-year-old Haitian male with a history of AIDS who presents with profuse<br />

watery diarrhea, abdominal cramps, nausea and vomiting. He also complains of weight loss and has<br />

signs of malabsorption. Fecal examination reveals oocysts.<br />

e. Diagnostic evaluation<br />

(1) Diagnosis is usually made by identifying oocysts in the feces, performing acid-fast staining of a fecal<br />

specimen, or identifying intracellular forms on biopsy of the small bowel.<br />

(2) Leukocytosis with moderate eosinophilia occurs in 50% and is an important clue to diagnosis.<br />

f. Management<br />

5. Cyc!ospora<br />

(1) TMP-SMX is the drug of choice; prolonged therapy is required in AIDS patients.<br />

(2) Pyrimethamine, nitrofurantoin, or furazolidone may be used in patients allergic to sulfa.<br />

a. Coccidial parasite found in tropical and subtropical regions.<br />

b. Affects both immunocompetent and immunocompromised individuals<br />

c. North American outbreaks have been traced to foods grown in tropical environments.<br />

d. Disease occurs after eating or drinking contaminated food or water. Person-to-person transmission is unlikely.<br />

e. After about 1 week, explosive watery diarrhea occurs with associated cramping, bloating, and nausea.<br />

Severe diarrhea lasts 2-3 days then becomes intermittent. Fatigue and weight loss are constant features.<br />

f. Disease is self-limited in healthy patients but may last 6-8 weeks, and up to a year in immunocompromised<br />

patients.<br />

g. Diagnosis is made by examination of feces for the oocysts; can be easily confused with Cryptosporidium.<br />

h. Management<br />

(1) TMP-SMX is the drug of choice.<br />

(2) Ciprofloxacin may be useful in allergic patients.<br />

214


ABDOMINAL AND GASTROINTESTINAL DISORDERS<br />

6. Necator americanus (hookworm)<br />

a. Infection occurs when the hookworm larvae penetrate through intact skin on contact with fecescontaminated<br />

soil.<br />

b. Mainly acquired by walking barefoot on contaminated soil.<br />

c. After penetrating the skin, the larvae enter the bloodstream, ascend the trachea, descend to the esophagus<br />

to differentiate into adult worms, and migrate to the upper intestine where they attach to the mucosa! wall<br />

and feed on host blood.<br />

d. Clinical presentation<br />

(1) Intermittent diarrhea, epigastric discomfort, weakness, low-grade fever, cough, rash, and weight loss<br />

(2) Anemia and protein deficiency most serious effects<br />

e. Diagnostic evaluation<br />

(1) Identify ova in the feces.<br />

(2) The presence of hypochromic, microcytic anemia and marked eosinophilia further support the diagnosis.<br />

f. Management<br />

(1) Mebendazole, albendazole, or pyrantel pamoate<br />

(2) An iron supplement<br />

7. Enterobius vermicularis (pinworm)<br />

a. Transmission is via ingestion of E vermicu/aris eggs.<br />

b. Once ingested, eggs develop into adult worms in the large bowel; then the gravid females migrate to the<br />

anus at night to deposit their eggs.<br />

c. Outbreaks often occur in schools, daycare centers, and other institutional settings.<br />

d. Clinical presentation<br />

(1) Patients most commonly complain of pruritus ani, particularly at night.<br />

(2) Secondary skin changes and bacterial infection may also be present.<br />

(3) Other associated problems may include insomnia, enuresis, urinary tract infections, and vaginitis.<br />

e. Diagnostic evaluation<br />

(1) Identify either adult worms migrating in the perinea! area or eggs on a cellophane tape swab of the anus.<br />

(2) Eosinophilia is absent.<br />

f. Management<br />

(1) A single dose of mebendazole or pyrantel pamoate that is repeated again after 2 weeks<br />

(2) All family members should be treated.<br />

E. Diarrhea in the AIDS patient<br />

1. Patients who are HIV-positive are more susceptible to the typical enteric organisms and to many opportunistic<br />

organisms as well. In addition, it is believed that HIV itself may produce an enteropathy.<br />

a. Cytomegalovirus and Cryptosporidium are the two most common causes of diarrhea in these patients.<br />

Mycobacterium avium is another important cause.<br />

b. Multiple organisms are responsible in some patients (up to 25%).<br />

c. HIV-positive patients on the HAART regimen living in developed countries are more likely to have diarrhea<br />

related to their medications (most commonly protease inhibitors) than to an infectious cause. When<br />

diarrhea is the sole complaint of a patient on the HAART regimen, an infectious cause is found in


ABDOMINAL AND GASTROINTESTINAL DISORDERS<br />

VIII. BILIARY TRACT DISORDERS<br />

A. General information<br />

1. Bile is produced in the canaliculi of biliary tract. While fasting, 50% of bile is stored in the gallbladder and<br />

50% flows into the duodenum.<br />

2. Food in the stomach results in release of cholecystokinin-pancreozymin and vagal impulses, which induce<br />

gallbladder contraction.<br />

3. Symptoms related to biliary disease are a frequent reason for emergency department visits.<br />

B. Cholelithiasis<br />

1. Gallstones<br />

a. 20% of women and 8% of men have gallstones.<br />

b. The vast majority of gal I stones are cholesterol stones.<br />

c. Risk factors for cholesterol stones<br />

(1) Female<br />

(2) Obesity<br />

(3) Increased parity<br />

(4) Advancing age<br />

(5) Rapid weight loss<br />

(6) Cystic fibrosis<br />

(7) Medications: oral contraceptives<br />

(8) Family tendency: very high rate in Pima Indians<br />

d. Pigmented stones<br />

(1) Black: associated with hemolysis such as sickle cell disease<br />

(2) Brown: associated with infection (bacterial as well as with Ascaris, Clonorchis)<br />

2. Clinical features: biliary colic<br />

a. Pain usually not colicky in nature (despite the name).<br />

b. Steady constant right upper quadrant pain (may be felt over epigastrium as well)<br />

c. Associated with nausea and vomiting<br />

d. May radiate to scapula or shoulder<br />

3. Diagnostic evaluation<br />

a. ALT and AST to assess for hepatitis<br />

b. Bilirubin and alkaline phosphatase to assess for duct obstruction<br />

c. Lipase to determine if pancreatitis present<br />

d. Ultrasound is the test of choice.<br />

4. Management<br />

a. Correction of fluid and electrolyte abnormalities if vomiting present<br />

b. Relief of nausea and vomiting<br />

c. Pain control<br />

(1) Anticholinergics to decrease spasm: glycopyrrolate<br />

(2) NSAIDs: ketorolac IV frequently useful<br />

(3) Opiates as needed<br />

d. Definitive treatment is cholecystectomy.<br />

C. Cholecystitis<br />

1. Acute cholecystitis represents acute inflammation of the gallbladder.<br />

2. Most cases result from acute obstruction of the cystic duct; 95% of patients will have gallstones identified.<br />

Duct obstruction may also be related to tumor, lymphadenopathy, fibrosis, parasitic disease.<br />

3. Risk factors are identical to those of cholelithiasis.<br />

4. Clinical presentation<br />

a. Patients usually present with right upper quadrant pain that may be colicky but often becomes constant.<br />

b. Fever may be present.<br />

c. Nausea and vomiting are common.<br />

216


ABDOMINAL AND GASTROINTESTINAL DISORDERS<br />

5. Diagnostic evaluation<br />

a. Ultrasonography showing stones, thickened gallbladder wall, and pericholecystic fluid is virtually<br />

diagnostic.<br />

b. Absence of stones has a very high negative predictive value.<br />

D. Variations and complications of cholecystitis<br />

1. Acalculous cholecystitis (5 % of cases)<br />

a. Occurs in postpartum/postoperative patients and those with burns or other major trauma, vascular<br />

disease, diabetes, sepsis, and CHF<br />

b. More common in the elderly<br />

c. Gangrene and perforation are relatively frequent in contrast to calculous cholecystitis.<br />

2. Calculous cholecystitis is the most frequent cause of acute pancreatitis: a gallstone passes into the common<br />

bile duct and may occlude the pancreatic duct at the sphincter of Oddi.<br />

3. Ascending cholangitis<br />

a. Occurs when stones passing through the common bile duct cause a purulent infection within the bile<br />

ducts that extends into the liver; may occur in the absence of stones with sclerosing cholangitis<br />

b. Clinical presentation<br />

(1) Fever and chills, abdominal pain, and jaundice (Charcot triad)<br />

(2) In severe cases, mental confusion and shock may also be present (Reynold pentad).<br />

c. This is a surgical emergency with a high mortality rate requires emergent decompression through open<br />

surgery, percutaneous drainage, or endoscopic retrograde cholangiopancreatography.<br />

4. Emphysematous cholecystitis (rare)<br />

a. Infection with gas-forming bacteria (f coli, C/ostridium perfringens, Klebsiella, and/or anaerobic<br />

streptococci)<br />

5. Empyema of the gallbladder (an advanced stage of cholecystitis)<br />

a. Bacterial invasion of the gallbladder wall<br />

b. Leads to gross suppuration<br />

6. Gallstone ileus (uncommon)<br />

a. Mechanism: a large gallstone erodes into the duodenum, lodges in the terminal ileum, and produces<br />

intestinal obstruction.<br />

b. Most commonly seen in elderly women (causes 25% or all small-bowel obstructions in this group) with<br />

multiple underlying medical problems and a history of gallstones.<br />

c. Diagnostic evaluation: radiographic findings show pneumobilia (air in the biliary tree), evidence of<br />

mechanical small-bowel obstruction, and a stone in the GI tract.<br />

E. Clinical presentation<br />

1. History<br />

a. Family history of gallbladder disease (common)<br />

b. Fatty food intolerance (common)<br />

c. Nausea (common)<br />

d. Bile-stained vomitus (variable because vomiting may not be present)<br />

e. Abdominal pain: most common presenting complaint, often precipitated by ingestion of a fatty or heavy<br />

meal. Initially, the pain is visceral in nature and felt in the midline. With prolonged obstruction of the cystic<br />

duct, somatic fibers become involved and the pain localizes to the right upper quadrant.<br />

(1) Distention of the gallbladder (due to obstruction of the cystic duct by a gallstone)--,, epigastric pain<br />

(2) Prolonged obstruction of the cystic duct--,, increased pressure within the gallbladder--,, chemical<br />

and/or bacterial inflammation of the gallbladder wall --,, right upper quadrant pain (which may radiate<br />

to the interscapular area, right scapula, or shoulder)<br />

2. Physical examination<br />

a. Tachycardia and fever<br />

b. Tenderness in the right upper quadrant or epigastrium<br />

c. Murphy's sign: t pain and transient arrest of inspiration when direct pressure is applied to the right upper<br />

quadrant while the patient is taking a deep breath<br />

217


ABDOMINAL AND GASTROINTESTINAL DISORDERS<br />

F. Diagnostic evaluation<br />

1. Laboratory abnormalities<br />

a. Mild leukocytosis with a left shift<br />

b. Bilirubin and alkaline phosphatase (may be normal)<br />

(1) Mildly increased__,, cholecystitis alone<br />

(2) Greatly increased__,, common duct stone<br />

c. Significantly increased serum amylase or lipase__,, pancreatitis due to a common duct stone<br />

2. Radiographic abnormalities<br />

a. Plain films of the abdomen are usually negative or demonstrate nonspecific findings. Positive findings,<br />

when present, include:<br />

(1) Radiopaque gal I stones (1 0%-1 5%)<br />

(2) Sentinel loop in the right upper quadrant<br />

(3) Air or an air-fluid level in the gallbladder__,, emphysematous cholecystitis<br />

(4) Air in the biliary tree__,, biliary enteric fistula<br />

b. U ltrasonography<br />

(1) Most useful initial test for imaging the gallbladder in the emergency setting<br />

(2) It is rapid, noninvasive, and can identify the following:<br />

(a) Thickness of the gallbladder wall (2:5 mm is diagnostic of cholecystitis)<br />

(b) Gallstones<br />

(c) Common duct stones<br />

(d) Dilated common duct<br />

(e)<br />

(f)<br />

Pericholecystic fluid<br />

Other sources of right upper quadrant pain (pathology of the liver, pancreas, and kidneys)<br />

c. Nuclear scintigraphy (HIDA scan)<br />

(1) Most accurate test for establishing diagnosis of cholecystitis<br />

(2) Indicated if no stones are seen on ultrasound or there was equivocal thickness of the gallbladder wall<br />

(3) If the isotope does not pass into the gallbladder within 1 hour of ingestion, the cystic duct is presumed<br />

to be obstructed.<br />

d. CT scanning rarely offers any advantage over ultrasound, except that it may visualize common duct<br />

stones better.<br />

G. Management<br />

1. Patients with acute cholecystitis (fever, leukocytosis, significant abdominal tenderness, and vomiting) and/or its<br />

comp I ications (pancreatitis, ascending cholangitis, jaundice) should be admitted.<br />

2. Initial therapy should consist of:<br />

a. IV fluids<br />

b. Antiemetics and nasogastric tube suction (if persistent vomiting is present)<br />

c. Surgical consult<br />

d. Pain relief<br />

e. Broad-spectrum IV antibiotics<br />

(1) Current recommendation is monotherapy with piperacillin/tazobactam, ampicillin/sulbactam, or<br />

meropenem. lmipenem/cilastin may be used in life-threatening cases.<br />

(2) Triple antibiotic therapy (ampicillin, an aminoglycoside, and clindamycin or metronidazole) is<br />

indicated in the presence of sepsis.<br />

3. Definitive therapy is cholecystectomy at the earliest convenient time (usually within 72 hours).<br />

IX. HEPATITIS<br />

A. Viral hepatitis<br />

1. Sequence of events: hepatic enzymes begin to increase before the prodromal phase __,, prodromal symptoms<br />

(usually constitutional) for 1-2 weeks__,, icteric phase (when present) may be preceded by pruritus and<br />

dark urine followed by jaundice and possibly right upper quadrant pain with hepatomegaly or splenomegaly<br />

__,, recovery phase (symptoms resolve and hepatic enzymes normalize). In most cases, there is complete<br />

recovery in 3-4 months. (Note that most patients do not develop jaundice.)<br />

218


ABDOMINAL AND GASTROINTESTINAL DISORDERS<br />

2. Hepatitis A<br />

a. Caused by hepatitis A virus (HAV), an RNA virus spread by the fecal-oral route directly via personto-person<br />

contact or indirectly through ingestion of contaminated water or food (particularly raw or<br />

undercooked shellfish)<br />

b. Illness is generally mild with a 15-50 day incubation period.<br />

c. Fecal shedding of the virus is maximal late in the incubation period and early in the prodromal phase.<br />

d. There is no carrier state, and it does not cause chronic liver disease.<br />

e. Serologic marker is anti-HAV.<br />

(1) Anti-HAV lgM indicates acute infection. Levels rise at same time as first increase in ALT.<br />

(2) Anti-HAV lgG indicates past infection or vaccination rather than acute infection.<br />

3. Hepatitis B<br />

a. Caused by hepatitis B virus (HBV), a DNA virus spread primarily by the percutaneous route but may also<br />

be transmitted by intimate contact via blood, semen, or saliva<br />

b. Illness is usually more severe and protracted than that produced by hepatitis A and has an incubation<br />

period of 45-160 days (mean 120 days); fulminant hepatic failure develops in 1 % of cases (rapidly rising<br />

bilirubin levels, coagulopathy, and encephalopathy), and 80% of comatose patients die.<br />

c. Chronic hepatitis or a chronic carrier state develops in 90% of infected neonates and in 10% of infected<br />

adults. These chronic carriers are all at increased risk of developing hepatocellular cancer; those with<br />

circulating virus continue to be a source of possible transmission.<br />

d. Serologic markers in hepatitis B infection<br />

(1) Hepatitis B surface antigen (HB,Ag) represents the outer protein coat of the virus. In most cases<br />

(>90%), it is detectable in the serum before hepatic enzymes increase and clinical symptoms appear;<br />

it lasts -6 months. Its presence implies active hepatitis or the carrier state but not necessarily<br />

infectivity.<br />

(2) Antibody to HB,Ag (anti-HB,) appears in the serum 2-6 months after HB,Ag disappears; it is a sign<br />

of previous infection or vaccination and indicates immunity. Chronic carriers usually have persistent<br />

HB,Ag in their serum and, therefore, do not develop anti-HB,.<br />

(3) Hepatitis B core antigen (HBcAg) (the core of the virus) is present only in hepatocytes, not in serum.<br />

Its presence, in conjunction with a negative anti-HB, is suggestive of early infection.<br />

(4) Antibody to HBcAg (anti-HBc) is present in the serum -2 weeks after HB,Ag appears. It may be the<br />

only marker of recent infection during the "window period" between the disappearance of HB,Ag<br />

and the appearance of anti-HB,. The presence of lgM anti-HBc in high titer indicates acute infection<br />

and high infectivity, whereas the presence of lgG anti-HBc, in association with anti-HB,, implies<br />

remote infection.<br />

(5) Hepatitis B epsilon antigen (HBeAg) is found in serum containing Hb,Ag. Its presence indicates<br />

ongoing viral replication and high infectivity. The antigen disappears in those patients who recover<br />

from hepatitis B but persists in those who develop chronic hepatitis.<br />

(6) Antibody to HBeAg (anti-HBe) appears during the acute phase of hepatitis Band implies that the<br />

patient is not likely to be infective. Anti-HBe persists several months after HBeAg disappears.<br />

(7) Sound confusing? Well, it is, especially because the presence of active infection does not necessarily<br />

correlate with infectivity, or how likely the patient is to transmit the disease.<br />

Table 8: One Method of Evaluating Serologic Results<br />

No infectivity<br />

+ anti-HB,<br />

Low infectivity<br />

+ anti-HB 0<br />

lnfectivity<br />

+ HBSAg<br />

High infectivity<br />

+ HBeAg<br />

(8) Only two serologic markers (HB,Ag and lgM anti-HBJ are actually needed to establish the diagnosis<br />

of infection with hepatitis B and determine its chronicity; HB,Ag confirms that infection is present,<br />

while IgM anti-HBc implies that it is acute.<br />

e. The CDC believes that successful hepatitis B vaccination (ie, that which is documented by anti-hepatitis B<br />

surface antibodies ?::10 IU at any time during a patient's life) confers lifetime protection. Therefore, do not<br />

test the source of (or recipient of) a blood exposure (eg, needle stick) if the patient has been successfully<br />

immunized. Otherwise, perform routine testing (including antihepatitis B surface antibodies).<br />

219


ABDOMINAL AND GASTROINTESTINAL DISORDERS<br />

4. Hepatitis C<br />

a. Caused by hepatitis C virus (HCV) and transmitted percutaneously, with infrequent sexual transmission<br />

(less than HBV)<br />

b. Most common cause of viral hepatitis in the United States<br />

(1) Prior to 1992, most cases occurred in association with the transfusion of blood/blood products; thus,<br />

hepatitis C was referred to as "post-transfusion hepatitis." Since the introduction of a marker in 1992<br />

to screen blood for hepatitis C (anti-HCV), the number of transfusion-related cases of hepatitis Chas<br />

been declining.<br />

(2) Currently, IV and intranasal drug (using shared straws) abuse accounts for a growing number of new<br />

infections with hepatitis C.<br />

c. Incubation period is 15-160 days (mean = 50 days); the illness is similar to (but milder than) that caused<br />

by hepatitis B.<br />

d. Chronic hepatitis develops in 70% of these patients, 20% of whom go on to develop cirrhosis. The<br />

delayed development of hepatocellular cancer in these cirrhotic patients is not uncommon.<br />

e. Anti-HCV is the serologic marker for hepatitis C; it is detectable in the serum 1-6 months after the onset<br />

of symptoms; anti-HCV test is 97% specific but does not distinguish acute from chronic infection.<br />

f. Health care workers who sustain needle-stick injury from HCV-positive patients should be tested<br />

immediately and retested every 2 months for 6 months.<br />

g. Postexposure prophylaxis is not available.<br />

h. Curative therapy is now available.<br />

5. Hepatitis E<br />

a. Caused by hepatitis E virus, an RNA virus; incubation period is 15-60 days.<br />

b. Resembles hepatitis A in its mode of transmission and the clinical course but is associated with a higher<br />

incidence of fulminant liver failure and mortality, particularly in pregnant patients.<br />

c. Chronic infection does not occur.<br />

d. A serologic marker has not yet been identified.<br />

6. Hepatitis delta virus (HDV)<br />

a. A "defective" RNA-containing virus that can only occur in patients who are currently infected with HBV.<br />

Infection may occur concomitantly with an acute HBV infection (co-infection) or as a superinfection in<br />

patients with chronic hepatitis B.<br />

b. Patients with acute co-infection have a better prognosis than those with superinfection; the illness is<br />

generally self-limited.<br />

c. Superinfection is associated with a high mortality rate (relative to acute HBV infection) and frequent<br />

development of chronic HDV hepatitis with subsequent cirrhosis.<br />

d. The serologic marker is anti-HDV.<br />

7. Diagnostic evaluation<br />

a. Serum aminotransferases are increased; AST is 10-100 times normal and ALT is usually higher than AST.<br />

b. Bilirubin (total and direct) is increased.<br />

c. Prothrombin time is usually normal; prolongation points to a more complicated course and a poorer<br />

prognosis.<br />

8. Prevention and prophylaxis<br />

a. Immune globulin, formerly known as gamma globulin or serum immune globulin, is 80%-90% effective<br />

in preventing infection with HAV if administered within 14 days of exposure.<br />

b. Postexposure prophylaxis with immunoglobulin is required for close personal contacts (home, daycare,<br />

etc). Exposed school, hospital, workplace contacts do not require prophylaxis.<br />

c. Hepatitis A vaccine should be recommended for those traveling to endemic areas. The CDC recommends<br />

the vaccine be administered at least 2 weeks before travel to endemic areas. Immunity develops by 20<br />

days after administration. Travelers who cannot receive the vaccine should receive immune globulin<br />

before travel for passive immunity that will last 3 months.<br />

d. Hepatitis B immune globulin (HBIG) and hepatitis B vaccine have both been demonstrated to prevent<br />

hepatitis B; HBIG confers passive immunity, while the vaccine confers active immunity. For maximal<br />

effectiveness, HBIG must be administered within 7 days of exposure; the sooner the better. High-risk<br />

exposure (percutaneous/mucosal from a known HB,AG+ individual) in patients who have completed the<br />

HBV vaccine series who are known to be HB,AB+ do not require HBIG. If unknown response, can test for<br />

presence of surface antibody. If negative, should receive HBIG. Nonresponders to the vaccine should be<br />

treated as unimmunized and receive HBIG.<br />

220


ABDOMINAL AND GASTROINTESTINAL DISORDERS<br />

e. No vaccine is available for hepatitis C. Postexposure prophylaxis with immune globulin is not<br />

recommended, and results with interferon are inconclusive.<br />

(1) For potential exposures, the CDC recommends determining the serologic status of the source case;<br />

however, the average incidence of anti-HCV seroconversion after unintentional needle stick or other<br />

sharp exposure from an HCV-positive source is only 1.8% (range 0-7%)<br />

(2) The exposed worker should have baseline hepatitis C antibody tests and liver function tests (including<br />

AST and ALT), which should be repeated in 6 months.<br />

f. There is no specific vaccine for hepatitis delta. The best preventive measure is vaccination for hepatitis B,<br />

because infection with hepatitis delta cannot occur in the absence of infection with hepatitis B.<br />

g. A hepatitis E vaccine has been very promising in trials but is not yet commercially available.<br />

9. Indications for admission<br />

a. Refractory vomiting<br />

b. Fluid or electrolyte imbalance<br />

c. Prolonged prothrombin time (>3-5 seconds compared with control)<br />

d. Bilirubin >20-30 mg/dl<br />

e. Hypoglycemia<br />

f. lmmunosuppression<br />

g. Severe underlying disease<br />

h. Early signs of encephalopathy (eg, agitation, altered mental status)<br />

i. Age >45-50 years old<br />

B. Toxic hepatitis<br />

1. Hepatotoxins that can produce morphologic changes in the liver and resemble those of acute viral hepatitis<br />

a. Halothane<br />

(1) Produces toxicity via a toxic metabolite and a hypersensitivity reaction<br />

(2) Most cases (75%) occur in patients who have been exposed to halothane in the past.<br />

(3) More common in adults (particularly women) and obese individuals.<br />

(4) Postoperative fever, rash, and eosinophilia may be present.<br />

(5) Onset is abrupt; usually occurs a couple of days after exposure.<br />

(6) Mortality rate is 20%-40% in severe icteric cases.<br />

b. Methyldopa<br />

(1) Produces injury via a toxic metabolite and a hypersensitivity reaction.<br />

(2) Mild transient increase of transaminase levels is seen in ~5% of patients treated with this drug, and<br />


ABDOMINAL AND GASTROINTESTINAL DISORDERS<br />

3. Hepatotoxins that can produce massive hepatic necrosis<br />

a. Acetaminophen (APAP)<br />

I<br />

t<br />

APAP<br />

A PAP-sulfate<br />

(not hepatotoxic)<br />

~<br />

t<br />

NAPQI<br />

(hepatotoxic metabolite)<br />

Cytochrome oxidase<br />

system<br />

I<br />

t<br />

APAP-glucoronide<br />

(not hepatotoxic)<br />

Glutathione<br />

t<br />

APAP-mercapturate and cysteine<br />

(not hepatotoxic)<br />

Mechanism of Toxicity of Acetaminophen<br />

i dose of APAP - saturation of the sulfate and glucuronide pathways and depletion of glutathione ➔ buildup of toxic<br />

metabolite (NAPQI)<br />

b. Carbon tetrachloride<br />

c. Yellow phosphorus<br />

d. Mushrooms (eg, Amanita phalloides)<br />

X. ALCOHOLIC LIVER DISEASE<br />

A. Syndromes<br />

1. Hepatic steatosis (fatty liver)<br />

a. Consumption of even moderate amounts of alcohol can lead to deposition of fat within hepatocytes.<br />

b. Clinical presentation: nontender hepatomegaly associated with mild increase in serum transaminase<br />

c. Resolves in 4-6 weeks with abstinence from alcohol<br />

2. Alcoholic hepatitis<br />

a. Clinical presentation<br />

(1) Ranges from a mild illness to acute liver failure, depending on the degree of hepatocellular necrosis<br />

and intrahepatic inflammation<br />

(2) Presence of encephalopathy is the strongest predictor of short-term mortality. Other scoring systems<br />

are based on presence of jaundice and coagulopathy. Maddrey discriminant function score 2:32 is<br />

prognostic of severe disease, and these patients may benefit from glucocorticoid therapy.<br />

(3) Patients classically present with generalized weakness, anorexia, nausea, abdominal discomfort, and<br />

weight loss.<br />

(4) Urine may be dark in color.<br />

(5) Examination usually reveals a low-grade temperature, jaundice, and a tender, enlarged liver.<br />

b. Diagnostic evaluation<br />

(1) CBC shows macrocytic anemia, leukocytosis, and thrombocytopenia.<br />

(2) Bilirubin and alkaline phosphatase levels are increased.<br />

(3) AST and ALT levels are 2-10 times normal, with the AST level higher than the ALT level.<br />

(4) INR > 1.5 portends a complicated course.<br />

3. Alcoholic (Laennec) cirrhosis<br />

a. An irreversible disease that produces permanent histologic changes in the liver (hepatocytic nodules<br />

and fibrous tissue) that disrupt normal hepatic blood flow and result in portosystemic shunting with<br />

concomitant portal hypertension<br />

222


ABDOMINAL AND GASTROINTESTINAL DISORDERS<br />

b. Clinical presentation<br />

(1) Patients typically present with chronic fatigue and anorexia; however, some remain relatively<br />

asymptomatic until they develop a complication (eg, hepatic encephalopathy, GI bleed).<br />

(2) Classic clinical findings include jaundice, spider angiomata, palmar erythema, gynecomastia, muscle<br />

wasting, Dupuytren contracture, ascites, and pedal edema.<br />

c. Diagnostic evaluation<br />

(1) CBC: anemia, leukopenia, thrombocytopenia<br />

(2) AST and ALT levels minimally increased<br />

(3) Prolonged prothrombin time (useful predictor of severity of liver disease), INR > 1.5<br />

(4) Increased bilirubin (useful predictor of severity of liver disease) and alkaline phosphatase levels<br />

(5) Hypoalbuminemia<br />

(6) Hyponatremia<br />

(7) Hypokalemia<br />

B. Complications<br />

1. Bleeding esophageal varices<br />

a. 25%-40% of patients with cirrhosis bleed from varices, and each episode of hemorrhage carries a 30%<br />

mortality risk.<br />

b. Clinical presentation: hemorrhagic shock with massive hematemesis<br />

c. Diagnostic evaluation: emergency endoscopy should be performed as soon as possible to confirm the<br />

diagnosis, exclude other sources of bleeding, and provide therapy. (See also Esophageal Disorders, pages<br />

178-183.)<br />

2. Portosystemic encephalopathy<br />

a. Due to accumulation of toxic substances in the blood<br />

b. Develops in patients with cirrhosis with extensive portosystemic shunting when the diseased liver is no<br />

longer able to perform its metabolic function of detoxifying wastes adequately<br />

c. Precipitating factors<br />

(1) Azotemia (GI bleeding, high-protein diet)<br />

(2) Injudicious use of medications (sedatives, analgesics, and tranquilizers)<br />

(3) Hypokalemic metabolic alkalosis<br />

(4) Infection<br />

(5) Hypoxemia<br />

(6) Hypoglycemia<br />

(7) Dehydration<br />

d. Clinical presentation<br />

(1) Altered level of consciousness, fetor hepaticus, and asterixis ("liver flap"-best demonstrated with the<br />

dorsiflexed wrist), along with other signs and symptoms of chronic liver disease<br />

(2) Serum ammonia level is usually increased, although levels correlate poorly with mental status.<br />

e. Management<br />

(1) General supportive measures<br />

(2) Correction/elimination of precipitating factors<br />

(3) Administration of lactulose and/or neomycin (to eliminate ammonia)<br />

(4) Protein-restricted diet<br />

3. Hepatorenal syndrome<br />

a. Syndrome of acquired renal failure of unknown cause that occurs in decompensated cirrhotic patients<br />

b. Pathogenesis: vasoconstriction and shunting of blood away from renal cortex---,. ! glomerular filtration<br />

rate ---,. ! renal output_,. azotemia<br />

c. Prognosis is dismal, with a mortality rate of nearly 100%.<br />

4. Spontaneous bacterial peritonitis<br />

a. Occurs in up to 33% of patients with liver disease and ascites<br />

b. Clinical presentation<br />

(1) Patients with liver disease who present with fever, abdominal tenderness, worsening ascites, and<br />

decreasing hepatic function.<br />

(2) Many have no symptoms; consider paracentesis for any patient with ascites ill enough to require<br />

hospital admission.<br />

223


ABDOMINAL AND GASTROINTESTINAL DISORDERS<br />

c. Diagnostic evaluation<br />

(1) Evaluation of ascitic fluid (obtained via paracentesis) confirms the diagnosis.<br />

(2) A granulocyte count >500/mm 3 is very suggestive, with an absolute PMN count >250 cells/mm 3<br />

diagnostic.<br />

(3) A positive culture is diagnostic.<br />

d. E coli and Streptococcus spp are the most common bacterial isolates.<br />

e. Treatment is with IV antibiotics (eg, a third-generation cephalosporin).<br />

f. Mortality rate ranges from 30% to 100%.<br />

XI. PANCREATITIS<br />

A. Etiology<br />

1. Acute pancreatitis is most often due to ethanol ingestion or biliary tract disease (gallstones). Other<br />

causes include abdominal trauma, surgery, endoscopic retrograde cholangiopancreatography, drugs<br />

(thiazides, furosemide, azathioprine, tetracycline, sulfonamides), penetrating peptic ulcer, hyperlipidemia,<br />

hypercalcemia, infections (mumps, viral hepatitis, infectious mononucleosis, Mycoplasma), and pregnancy.<br />

2. Chronic pancreatitis is the result of repeated episodes of acute pancreatitis and is usually due to chronic<br />

alcoholism of long duration (75 % of cases in United States). Other causes include severe protein-calorie<br />

malnutrition, hyperthyroidism, and pancreas divisum.<br />

B. Clinical presentation<br />

1. Patients classically present after ingestion of a fatty meal or after binge drinking and complain of epigastric<br />

pain, nausea, and vomiting.<br />

2. The epigastric pain is constant in nature with radiation directly into the back, and it is often eased when the<br />

patient leans forward.<br />

3. Examination may reveal epigastric tenderness with guarding, tachycardia (due to fever, hypovolemia, and<br />

pain), tachypnea (due to pulmonary involvement), a low-grade fever, and hypoactive bowel sounds. However,<br />

because of the retroperitoneal location of the pancreas, rebound is generally absent.<br />

4. Grey Turner sign (bluish discoloration of the flanks) and Cullen sign (bluish discoloration around the<br />

umbilicus) indicate the presence of retroperitoneal hemorrhage and point to the diagnosis of hemorrhagic<br />

pancreatitis; unfortunately, these signs are not usually present. They are also not specific for pancreatitis.<br />

C. Diagnostic evaluation<br />

1. Serum amylase and lipase levels are typically increased in acute pancreatitis; however, the degree of increase<br />

does not correlate with the severity of disease.<br />

2. Very high levels are usually seen when biliary tract disease (gallstones) is the underlying cause.<br />

3. However, pancreatic enzymes are not usually increased in patients with chronic pancreatitis, because their<br />

pancreas is "burned out."<br />

4. Serum amylase<br />

a. Increases shortly after the onset of symptoms and returns to baseline levels within 1-4 days<br />

b. An increase 1.5 times the upper limit of normal typically indicates acute pancreatitis.<br />

5. Serum lipase<br />

a. Found primarily in the pancreas (unlike serum amylase, which is released by several organs)<br />

b. As sensitive as serum amylase but has the advantage of being more specific; specificity is almost 100%.<br />

c. Assays are readily available, reliable, and inexpensive.<br />

6. Other laboratory studies<br />

a. Glucose to exclude hyperglycemia<br />

b. Calcium to exclude hypocalcemia (most common laboratory abnormality)<br />

c. WBC count: leukocytosis is common<br />

d. Hematocrit O hematocrit is suggestive of hemorrhagic pancreatitis)<br />

e. BUN and creatinine usually increased due to hypovolemia.<br />

f. AST and lactate dehydrogenase: significant increase suggests severe disease.<br />

224


ABDOMINAL AND GASTROINTESTINAL DISORDERS<br />

7. Plain radiographs are most helpful in excluding other diagnoses. Radiographic findings associated with<br />

pancreatitis include:<br />

a. Abdominal films<br />

(1) Scattered calcifications in the area of the pancreas are suggestive of chronic pancreatitis.<br />

(2) Evidence of ileus and air-trapping in the small bowel adjacent to the pancreas - sentinel loop<br />

(suggests acute pancreatitis)<br />

(3) Distended colon and collapse of the distal colon -,, colon-cutoff sign<br />

b. Chest radiograph<br />

(1) Left pleural effusion or elevated left hemidiaphragm<br />

(2) Findings consistent with ARDS may be seen in patients with severe disease.<br />

c. Ultrasound<br />

(1) Should be performed early if gallstone pancreatitis suspected<br />

(2) May miss common duct stones or dilatation of duct<br />

(3) Endoscopic ultrasound may be required.<br />

d. CT scan: used to identify other causes of abdominal pain, establish severity of disease, and exclude<br />

complications such as pancreatic necrosis, hemorrhage, vascular abnormalities, and pseudocyst or<br />

abscess<br />

e. MRI cholangiopancreatography: noninvasive and may help establish cause<br />

D. Management: primarily supportive<br />

1. Hydration<br />

a. Fluid resuscitation with normal saline as required. Most patients are dehydrated because of vomiting and<br />

fluid sequestration.<br />

b. Close monitoring of vital signs, urine output<br />

c. Electrolyte replacement as necessary<br />

2. Pain control<br />

a. Morphine, hydromorphone are appropriate (despite potential to induce spasm of sphincter of Oddi, no<br />

adverse outcome in people has been demonstrated).<br />

b. Meperidine is the traditional choice but has no proven advantage.<br />

3. Control of vomiting<br />

a. IV antiemetics as required<br />

b. Nasogastric suctioning only for patients with intractable vomiting or ileus<br />

c. No evidence to support fasting in mild to moderate pancreatitis<br />

4. Nutritional support<br />

a. Enteral feeding if tolerated<br />

b. If enteral feeding not tolerated, consider parenteral feeding.<br />

5. Monitoring for complications of acute pancreatitis<br />

E. Poor prognostic signs (Ranson criteria)<br />

1. On admission<br />

a. Age >55 years old<br />

b. Leukocytosis (WBC count > 16,000/mm 3 )<br />

c. Hyperglycemia >200 mg/dL<br />

d. AST > 250 U/L<br />

e. Lactate dehydrogenase >350 IU/L<br />

2. 48 hours later<br />

a. Calcium 5 mg/dL<br />

e. Base deficit >4 mEq/L<br />

f. Sequestration >4 L of fluid<br />

3. Mortality correlates directly with the number of poor prognostic signs present; the greater the number, the<br />

greater the mortality.<br />

225


ABDOMINAL AND GASTROINTESTINAL DISORDERS<br />

F. Complications<br />

1. Pleural effusions (usually left-sided)<br />

2. ARDS is due to deactivation of surfactant (most common cause of death due to acute pancreatitis)<br />

3. Pancreatic phlegmon - pseudocyst<br />

4. Pancreatic abscess - sepsis<br />

5. Hemorrhage (intrapancreatic or intraperitoneal) - shock<br />

6. Pancreatic ascites<br />

7. Third spacing of fluids - hypovolemia - acute tubular necrosis<br />

8. Disseminated intravascular coagulation<br />

226


ABDOMINAL AND GASTROINTESTINAL DISORDERS: PRACTICE CLINICAL SCENARIOS<br />

ABDOMINAL AND GASTROINTESTINAL DISORDERS: PRACTICE<br />

CLINICAL SCENARIOS<br />

Answers immediately follow the practice clinical scenarios.<br />

Scenario A<br />

Presentation: A patient presents complaining of vomiting blood (may or may not be actively vomiting<br />

at time of presentation) and has dried blood on the lips. Heart rate is 175 beats per minute, and blood<br />

pressure is 60/30 mmHg.<br />

What is the diagnosis?<br />

Scenario B<br />

Presentation: A patient presents with a history of ingesting a foreign body and complains of a sensation of<br />

something stuck in the throat. Symptoms include vomiting, gagging, choking, and dysphagia.<br />

What is the diagnosis?<br />

Scenario C<br />

Presentation: A patient has had a gradual onset of periumbilical discomfort that subsequently localized to<br />

the right lower quadrant. Other symptoms include anorexia, nausea, and vomiting. The pain started before<br />

the vomiting.<br />

Physical examination: Findings reveal right lower quadrant tenderness to palpation with positive McBurney<br />

and Rovsing signs.<br />

What is the diagnosis?<br />

Scenario D<br />

Presentation: An elderly patient with new onset atrial fibrillation (controlled or uncontrolled) presents with<br />

sudden onset of abdominal pain. The patient is not anticoagulated.<br />

Physical examination: Abdominal examination findings are generally nonspecific despite significant pain.<br />

What is the diagnosis?<br />

Scenario E<br />

Presentation: A patient with known colonic diverticuli presents complaining of lower quadrant abdominal<br />

pain (left sided). The pain is crampy, vague, and poorly localized.<br />

Physical examination: The physical examination is relatively benign. Pain localizes on palpation to the left<br />

lower quadrant or suprapubic region.<br />

What is the diagnosis?<br />

227


ABDOMINAL AND GASTROINTESTINAL DISORDERS: PRACTICE CLINICAL SCENARIOS<br />

ANSWERS TO PRACTICE CLINICAL SCENARIOS<br />

Scenario A<br />

Diagnosis: upper GI bleeding<br />

Management: Initial efforts are directed at determining hemodynamic stability and need for immediate<br />

resuscitation.<br />

Unstable patients require two large-bore IV lines with fluid boluses of normal saline. Administer blood<br />

product transfusion as necessary. If patient is anticoagulated, may need immediate reversal as appropriate.<br />

Immediate endoscopy may be needed for control of esophageal or ulcer bleeding. Administer IV protonpump<br />

inhibitor. Regardless of history, ulcer bleeding is more common than variceal bleeding, but if<br />

variceal bleeding is suspected, administer octreotide.<br />

For stable patients, determine the source of bleeding. If actively vomiting, the source is obvious. If not<br />

vomiting, presence of melena, or nasogastric aspiration with presence of blood or "coffee grounds," is highly<br />

predictive of upper GI source. However, absence of blood via nasogastric tube does not exclude upper GI<br />

source. Bright red blood or blood clots per rectum are more predictive of lower GI source. Determine need<br />

for immediate intervention or admission. Blatchford score helps predict need for intervention.<br />

Scenario B<br />

Diagnosis: esophageal foreign body<br />

Key facts: Most cases (80%) of ingested esophageal foreign bodies are in children. They may not be<br />

old enough to give history. The cricopharyngeus muscle (C6) is the most common site of obstruction in<br />

children


ABDOMINAL AND GASTROINTESTINAL DISORDERS: PRACTICE CLINICAL SCENARIOS<br />

Scenario D<br />

Diagnosis: mesenteric ischemia, embolic occlusion of the superior mesenteric artery from a thrombus in<br />

the atria<br />

Diagnostic evaluation: Laboratory investigation is frequently not helpful early on. Serum lactate is<br />

usually increased but not until bowel infarction has occurred. Acidosis may be present late. Helical CT<br />

angiography of the abdomen is currently the test of choice, although some centers may still perform<br />

conventional angiography.<br />

Alternative presentation: Another typical presentation is an elderly patient with a history of coronary artery<br />

disease who presents with diffuse abdominal pain. The patient may report having had abdominal pain after<br />

eating on prior occasions. This is "intestinal ischemia" due to vascular disease. Examination early on shows<br />

few specific findings. Pain is out of proportion to findings. Feces may be positive for guaiac; there may be<br />

gross blood per rectum.<br />

Management: Fluid resuscitation as required. If pressors are needed, use positive inotropes rather than<br />

vasoconstrictors. If nonocclusive disease, may use interventional radiology or papaverine injection; if<br />

occlusive disease, revascularization is required.<br />

Scenario E<br />

Diagnosis: diverticular disease<br />

Key facts: Signs of systemic toxicity are absent unless diverticuli have perforated or abscess formation has<br />

begun. Peritoneal signs are unlikely unless the patient has complicated diverticular disease, which involves<br />

microperforation of the bowel wall with formation of phlegmon in the immediate area. Minority of these<br />

will have frank free air in abdomen.<br />

Alternative presentation: Another less likely presentation consists of massive lower GI hemorrhage that is<br />

painless. This is responsible for up to 40% of lower GI hemorrhage in elderly patients.<br />

Diagnostic evaluation: CBC, hematocrit, and blood type and screen should be done in patients with<br />

diverticular hemorrhage. WBC count is likely to be increased, particularly in complicated disease.<br />

If complicated diverticulitis is suspected, CT of the abdomen and pelvis should be done. (Neither<br />

colonoscopy nor barium enema are done in the acute setting.) Patients in whom diverticular disease<br />

is suspected but have no known history may also benefit from CT to establish the diagnosis as well as<br />

exclude other pathology. Patients with history of diverticulosis and mild symptoms may not require<br />

imaging in the emergency department.<br />

Management: Patients with uncomplicated diverticulitis can be treated on an outpatient basis with pain<br />

control and antibiotic therapy. Acceptable regimens include ciprofloxacin + metronidazole, TMP-SMX +<br />

metronidazole, or amoxicillin/clavulanate for 10 days.<br />

Patients with complicated diverticulitis need resuscitation as necessary, IV antibiotic therapy similar to<br />

the above or ticarcillin/clavulanate or ampicil lin/sulbactam. Percutaneous drainage of the abscess may be<br />

required. A surgical consult should be obtained for all with free air.<br />

229


230<br />

NOTES


THORACIC AND RESPIRATORY DISORDERS<br />

THORACIC AND RESPIRATORY DISORDERS<br />

Pneumonia ................................................................................................................................................................ 236<br />

Bacterial Pneumonia .......................................................................................................................................... 236<br />

Atypical Pneumonia ........................................................................................................................................... 246<br />

Viral Pneumonia ................................................................................................................................................. 249<br />

Types of Pneumonia: Classic Clinical Scenario and Etiology ............................................................................... 251<br />

Tuberculosis .............................................................................................................................................................. 252<br />

Pleural Effusion ......................................................................................................................................................... 256<br />

Aspiration Pneumonia ............................................................................................................................................... 259<br />

Lung Abscess ............................................................................................................................................................. 261<br />

Empyema .................................................................................................................................................................. 263<br />

Hemoptysis ............................................................................................................................................................... 264<br />

Pneumothorax ........................................................................................................................................................... 266<br />

Asthma ...................................................................................................................................................................... 269<br />

Chronic Obstructive Pulmonary Disease (COPD) ..................................................................................................... 276<br />

Acute Respiratory Distress Syndrome (ARDS) ........................................................................................................... 280<br />

Pulmonary Embolism ................................................................................................................................................ 282<br />

Cyanosis .................................................................................................................................................................... 290<br />

231


THORACIC AND RESPIRATORY DISORDERS: SELF-ASSESSMENT QUESTIONS<br />

THORACIC AND RESPIRATORY DISORDERS:<br />

SELF-ASSESSMENT QUESTIONS<br />

1. The most common cause of community-acquired bacterial pneumonia is:<br />

(a) Group A ~-hemolytic streptococci<br />

(b) Haemophilus influenzae<br />

(c) Streptococcus pneumoniae<br />

(d) Klebsiella pneumoniae<br />

2. A sputum Gram stain revealing encapsulated gram-positive lancet-shaped diplococci is most consistent with:<br />

(a) Staphylococcus aureus<br />

(b) Haemophilus influenzae<br />

(c) Mycoplasma pneumoniae<br />

(d) Streptococcus pneumoniae<br />

3. Although abscess formation is not common, it can be seen in association with each of the following causes of<br />

pneumonia except:<br />

(a)<br />

Klebsiella pneumoniae<br />

(b) Chlamydia pneumoniae<br />

(c) Pseudomonas aeruginosa<br />

(d) Staphylococcus aureus<br />

4. A 55-year-old male smoker presents in August with signs of an atypical pneumonia, relative bradycardia, and<br />

diarrhea. He is on no medications. Laboratory results reveal increased liver enzymes. The most probable cause of<br />

the pneumonia is:<br />

(a) Mycoplasma pneumoniae<br />

(b) Francisella tularensis<br />

(c)<br />

Legionella pneumophila<br />

(d) Coxiella burnetii<br />

5. Al I of the fol lowing statements regarding Klebsiella pneumonia are true except:<br />

(a) Sputum Gram stain reveals encapsulated gram-positive organisms that are seen in pairs.<br />

(b) The sputum is thick and brown in color, resembling currant jelly.<br />

(c)<br />

Patients with alcoholism, diabetes, or COPD are most commonly affected.<br />

(d) Chest radiograph commonly reveals a necrotizing right upper lobe infiltrate.<br />

6. Which of the following statements regarding the TB skin test is most accurate?<br />

(a)<br />

It is positive in 100% of patients with TB meningitis.<br />

(b) A positive reaction indicates the presence of infection with Mycobacterium tuberculosis but not necessarily the<br />

presence of active disease.<br />

(c) It is always positive in patients infected with M tuberculosis.<br />

(d) Criteria for interpreting the test as positive are the same for all patients regardless of background/concurrent<br />

illnesses.<br />

7. lsoniazid therapy for TB is associated with the multiple complications listed below. Which one of these<br />

complications can be prevented or minimized by the simultaneous administration of pyridoxine?<br />

(a)<br />

Phenytoin toxicity<br />

(b) Hepatitis<br />

(c)<br />

Hypersensitivity reaction<br />

(d) Peripheral neuritis<br />

8. On which of the following views of the chest is a small pleural effusion most likely to be detected?<br />

(a) Supine<br />

(b) PA<br />

(c) Lateral decubitus<br />

(d) Lateral<br />

232


THORACIC AND RESPIRATORY DISORDERS: SELF-ASSESSMENT QUESTIONS<br />

9. All of the following are appropriate in the acute treatment of aspiration pneumonia except:<br />

(a) Administration of steroids and prophylactic antibiotics<br />

(b) Administration of supplemental oxygen<br />

(c) Bronchoscopy to remove large particles<br />

(d) Use of continuous positive-airway pressure (CPAP)<br />

10. A patient has pleural effusion on chest radiograph. A diagnostic thoracentesis is performed. Which of the following<br />

is an indication for a thoracostomy tube placement?<br />

(a) No organisms on Gram stain<br />

(b) LOH two times the upper limit of serum LOH<br />

(c) Glucose >60 mg/dL<br />

(d) pH 12 mmHg<br />

16. All of the following statements regarding the use of ~-adrenergic agonists in the treatment of asthma are accurate<br />

except:<br />

(a) Their primary effect is on the large central airways.<br />

(b) They promote bronchodilation by increasing cyclic AMP.<br />

(c) Their onset of action is


THORACIC AND RESPIRATORY DISORDERS: SELF-ASSESSMENT QUESTIONS<br />

1 8. Al I of the following statements regarding noncardiogenic pu Imo nary edema are true except:<br />

(a) The heart size is smal I or normal.<br />

(b) Pulmonary arterial wedge pressure is increased.<br />

(c) Chest radiograph shows bilateral pulmonary infiltrates.<br />

(d) Lung compliance is reduced.<br />

19. Pneumonia is most commonly acquired via ___ _<br />

(a) Hematogenous spread from another site<br />

(b) Direct introduction of organisms into the pleura or lungs<br />

(c) Aspiration of oropharyngeal secretions<br />

(d) None of the above<br />

20. All of the following statements regarding pneumococcal pneumonia are accurate except:<br />

(a) Patients usually give a history of a single shaking chill followed by the development of a cough productive of<br />

rust-colored sputum.<br />

(b) The WBC count is usually


THORACIC AND RESPIRATORY DISORDERS: SELF-ASSESSMENT QUESTIONS<br />

27. A patient received a liver transplant 1 month ago and presents to the emergency department with complaints of<br />

fever, chills, cough, and shortness of breath. What pathogen is most likely causing his illness?<br />

(a) Pseudomonas<br />

(b) Staphylococcus aureus<br />

(c) Cytomegalovirus<br />

(d) Haemophi/us influenzae<br />

ANSWERS<br />

1. C<br />

2. d<br />

3. b<br />

7.<br />

8.<br />

9.<br />

4. c 10. d<br />

5. a 11. d<br />

d<br />

C<br />

a<br />

13. b 19. C<br />

14. C 20. b<br />

15. a 21. b<br />

16. a 22. a<br />

17. b 23. d<br />

6. b 12. b 18. b 24. a<br />

25. C<br />

26. C<br />

27. C<br />

Use the pre-chapter multiple choice question worksheet (page xvi) to record and determine the percentage of correct<br />

answers for this chapter.<br />

235


THORACIC AND RESPIRATORY DISORDERS<br />

I. PNEUMONIA<br />

A. Bacterial pneumonia (most common cause of a focal infiltrate)<br />

1. Epidemiology<br />

a. Accounts for up to 10% of hospital admissions in the United States<br />

(1) Typical versus atypical<br />

(a) Typical: fever, shaking chills, cough with yellow/green sputum, chest radiograph with lobar<br />

infiltrate; Gram stain will reveal organism.<br />

(b) Atypical: little cough or nonproductive cough, chest radiograph with interstitial pattern; organisms<br />

do not show up on Gram stain.<br />

b. Most pneumonias are the result of a single species of bacteria.<br />

(1) Streptococcus pneumoniae<br />

(2) Haemophilus influenzae<br />

(3) Klebsiella pneumoniae<br />

(4) Staphylococcus aureus<br />

(5) Escherichia coli<br />

(6) Pseudomonas aeruginosa<br />

(7) Group A streptococci<br />

(8) Moraxella catarrhalis<br />

c. Mechanisms of infection<br />

(1) Aspiration of oropharyngeal secretions is the primary mechanism of acquisition.<br />

(a)<br />

50%-70% of normal healthy individuals aspirate in their sleep.<br />

(b) An abnormal swallowing mechanism (eg, from an altered mental status) or GI disease can lead to<br />

increased frequency of aspiration.<br />

(2) Other mechanisms<br />

(a)<br />

Hematogenous spread from another site<br />

(b) Colonization<br />

(c)<br />

d. Predisposing factors<br />

Direct introduction of organisms into the pleura or lungs, both of which are uncommon<br />

(1) Impaired cough and gag reflex<br />

(a) Altered mental status<br />

(b) Seizures<br />

(c) Syncope<br />

(d) Stroke<br />

(2) Impaired mucociliary transport<br />

(a) Smoking<br />

(b) Viral or mycoplasmal infection<br />

(c) COPD<br />

(3) Chronic underlying disease<br />

(a) Hepatic or renal failure<br />

(b) Diabetes mellitus<br />

(c) CHF<br />

(4) Impaired immunity<br />

(a) AIDS<br />

(b) Chemotherapy<br />

(c) Alcoholism<br />

(d) Cystic fibrosis<br />

(e) Malnutrition<br />

(f)<br />

(g)<br />

Sickle cell disease, splenectomy<br />

Congenital immune deficiencies<br />

(h) lmmunocompromised patients (elderly, transplant patients)<br />

236


THORACIC AND RESPIRATORY DISORDERS<br />

(5) Underlying lung pathology<br />

(a) Bronchial obstruction (foreign body, tumor or cancer); if recurrent pneumonias, consider foreign<br />

body as etiology, especially in pediatric patients.<br />

(b) Pulmonary embolus, contusion<br />

(c) Atelectasis<br />

(d) COPD (bronchitis, emphysema)<br />

(6) Chest wall dysfunction<br />

(a) Neuropathies or myopathies<br />

(b) Postoperative pain<br />

(c) Chest trauma with rib fractures<br />

(7) Mechanical bypass of normal defense mechanisms<br />

(a)<br />

Endotracheal tube<br />

(b) Nasogastric tube<br />

(c) Chest tube<br />

(d) Bronchoscopy<br />

(e) Tracheostomy<br />

(8) Altered upper respiratory tract flora<br />

(a) Recent antibiotic therapy<br />

(b) Hospitalization<br />

e. Defense mechanisms<br />

(1) Cough and gag reflex prevents gross aspiration.<br />

(2) Tracheobronchial cilia remove particles >5 µm.<br />

(3) Alveolar macrophages remove particles 15,000/mm 3 ) should prompt<br />

consideration of pneumonia as a diagnosis.<br />

(2) In an elderly, debilitated patient, or immunocompromised patient (transplant or cancer patient, or<br />

patient on chronic steroids), the WBC count may be increased, normal, or decreased (even if there is<br />

an associated sepsis); the only clue may be a left shift.<br />

(3) A very high or very low WBC count is associated with increased mortality.<br />

b. Gram stain: sputum gram stain is rarely helpful. Diagnostic samples must have 25<br />

WBCs/high power field. Knowing the gram stain findings for the examination may help with identifying the<br />

etiology.<br />

Table 9: Gram-Stain Findings with Common Organisms<br />

Organism<br />

Streptococcus pneumoniae<br />

Staphylococcus aureus<br />

Influenza<br />

Klebsie//a sp<br />

Legionella sp<br />

Oral flora (aspiration)<br />

Atypicals, viral<br />

Gram-Stain Findings<br />

Gram-positive, lancet-shaped cocci in pairs,<br />

PMNs<br />

Gram-positive cocci in clusters, PMNs<br />

Gram-negative coccobacillus, PMNs<br />

Gram-negative rods, PMNs<br />

Few weakly gram-negative rods, many PMNs<br />

Mixed gram-positive and -negative cocci and<br />

rods, PMNs<br />

Few bacteria, many PMNs, or monocytes<br />

237


THORACIC AND RESPIRATORY DISORDERS<br />

c. Chest radiographs<br />

(1) Types of findings: lobar vs atypical<br />

(a)<br />

Lobar pneumonia: consolidation<br />

i. White area on radiograph<br />

ii. Air bronchograms<br />

iii. Loss of hemidiaphragm, but no volume loss; no pulling up/down of fissures, no collapse of lung<br />

(b) Atypical pneumonia: interstitial infiltrates on chest radiographs<br />

(2) Certain patterns suggest, but do not prove, a specific organism<br />

(a) Streptococcus pneumoniae (most common cause of lobar pneumonia)<br />

i. Single lobar infiltrate<br />

ii. Small pleural effusion<br />

iii. Abscess formation<br />

(b) Haemophilus influenzae<br />

(c)<br />

i. Patchy (frequently basilar) infiltrates<br />

ii. Occasional pleural effusion<br />

Klebsiella pneumoniae<br />

i. Upper lobe infiltrates<br />

ii. Abscess formation<br />

(d) Staphylococcus aureus<br />

i. Patchy, multicentric infiltrates<br />

ii. Abscess formation (most common cause of abscess)<br />

iii. Empyema<br />

iv. Pneumothorax<br />

(e) £ coli: patchy, bilateral lower lobe infiltrates<br />

(f) Pseudomonas aeruginosa<br />

i. Patchy, mid or lower lobe infiltrates<br />

ii. Abscess formation<br />

(g) Group A streptococci<br />

i. Patchy, multilobar infiltrates (usually lower)<br />

ii. Large pleural effusion<br />

(2) When infiltrates (patchy or lobar) are identified, always scan the radiographs for other associated<br />

findings.<br />

(a)<br />

Pneumothorax<br />

(b) Pleural effusion<br />

(c) Abscess formation<br />

(3) Although CT scan may show pneumonia earlier, the radiation exposure makes it less desirable than a<br />

plain chest radiograph.<br />

(4) Leukopenic or dehydrated patients may have normal-appearing radiographs.<br />

(5) Other diagnoses (pathology) to consider when pneumonitis is present on a chest radiograph<br />

(a) Pulmonary infarction<br />

(b) Pulmonary edema<br />

(c) Metastatic cancer<br />

(d) Pleural thickening<br />

(e)<br />

(f)<br />

Parenchymal scarring<br />

Atelectasis<br />

(6) Cavitary lesion on chest radiograph<br />

(a) MRSA<br />

(b) Tuberculosis<br />

(c) Pseudomonas<br />

(d) Fungal superinfection<br />

(e) Cancer<br />

238


THORACIC AND RESPIRATORY DISORDERS<br />

(7) Abscess formation<br />

(a) 5 aureus<br />

(b) Klebsiella<br />

(c) Pseudomonas<br />

d. Pulse oximetry/arterial blood gases: in patients with pneumonia, the lungs may be adequately perfused but<br />

gas exchange at the alveolus is poor, resulting in hypoxia (and VQ mismatch).<br />

e. Sputum analysis<br />

(1) Expectorated sputum has a limited role in diagnosing the microbiologic organism involved because of<br />

contamination with oropharyngeal flora. Sputum Gram-stain results are helpful in making therapeutic<br />

decisions in only about one-third of patients.<br />

(2) Fiberoptic bronchoscopy is the standard invasive procedure of choice for seriously ill or<br />

immunocompromised patients.<br />

(3) Smears for acid-fast bacilli should be done in patients at risk of tuberculosis (immigrants, patients with<br />

AIDS, IV drug abusers, and others who are immunocompromised).<br />

f. Blood cultures<br />

(1) A CMS core competency requirement to be ordered on all ICU patients for pneumonia<br />

(2) Should be obtained before antibiotic administration<br />

(3) To be ordered only for patients who are seriously ill, as well as for those with presumed bacteremia,<br />

comorbid disease, immunosuppression, or rigors<br />

(4) "Routine" cultures on patients with pneumonia are discouraged because of a low yield of clinically<br />

useful data.<br />

g. Pleural fluid aspiration, although not generally an emergency department procedure, is helpful in excluding<br />

empyema and, in the case of a large pleural effusion, drainage can also reduce respiratory embarrassment.<br />

With smaller effusions, determining the glucose and pH of the fluid is helpful in determining treatment.<br />

(1) pH >7.2 or glucose >60 mgldL - antibiotics<br />

(2) pH


THORACIC AND RESPIRATORY DISORDERS<br />

3. Treatment<br />

I<br />

Treatment of<br />

community-acquired pneumonia<br />

I<br />

I<br />

I<br />

Outpatient therapy (oral)<br />

I<br />

I I I<br />

I<br />

I<br />

Inpatient therapy (IV)<br />

I<br />

I<br />

Comorbidity or risk<br />

I<br />

Not in ICU<br />

No history of factors for higher-risk I<br />

comorbid illness 1<br />

organisms<br />

I<br />

Use one: RTF 3<br />

Azithromycin<br />

Clarithromycin<br />

Doxycycl i ne 2<br />

I<br />

Use one:<br />

Levofloxacin<br />

Sparfloxacin<br />

Gatifloxacin<br />

Moxifloxacin<br />

Gemifloxacin<br />

Suspected<br />

aspiration with<br />

infection<br />

I<br />

Amoxacillinclavulanate<br />

or<br />

an RTF 3 alone<br />

I<br />

Recent<br />

antibiotic<br />

treatment for<br />

infection<br />

I<br />

Macrolide<br />

(advanced) +<br />

a E,-lactam or<br />

an RTF 3 alone<br />

ICU<br />

I<br />

1 Ca rdiopulmonary disease, diabetes, uremia,<br />

ma lignancy<br />

2 Fo r patients who cannot take a macrolide<br />

3<br />

RT F = respiratory tract fluoroquinolone<br />

(an tipneumococcal)<br />

If p atient cannot take an RTF, refer to text<br />

for alternative therapy.<br />

No risk of<br />

Pseudomonas<br />

A G,-lactam (cefotaxime<br />

or ceftriaxone) +<br />

azithromycin<br />

or<br />

an RTF 3 if allergic to a<br />

G,-lactam ± clindamycin<br />

At risk of<br />

Pseudomonas<br />

An antipseudomonal<br />

G,-lactam (cefepime,<br />

imipenem, meropenem,<br />

or piperacillin/tazobactam) +<br />

ciprofloxacin<br />

or<br />

I<br />

An antipseudomonal agent+<br />

aminoglycoside + RTF 3<br />

or macrolide<br />

Aztreonam + levofloxacin or<br />

aztreonam + moxifloxacin or<br />

gatifloxacin (± aminoglycoside)<br />

Treatment of Community-Acquired Pneumonia<br />

a. For inpatients with one of the following conditions requiring general medical admission:<br />

(1) Suspected aspiration with infection-.,. amoxicillinclavulanate or clindamycin<br />

(2) Recent antibiotic therapy for another condition -.,. azithromycin or clarithromycin plus a ~-lactam or a<br />

respiratory fluoroquinolone alone (the regimen selected depends on the nature of the recent antibiotic<br />

therapy)<br />

b. For ICU patients in whom:<br />

(1) Pseudomonas infection is not an issue-.,. a ~-lactam and an advanced macrolide (azithromycin,<br />

clarithromycin) or a respiratory fluoroquinolone<br />

(2) Pseudomonas infection is not an issue, but the patient has a ~-lactam allergy-.,. a respiratory<br />

fluoroquinolone (with or without clindamycin)<br />

240


THORACIC AND RESPIRATORY DISORDERS<br />

(3) Pseudomonas infection is an issue----'><br />

(a) An antipseudomonal agent (piperacillin, imipenem, meropenem, cefepime, or piperacillintazobactam)<br />

and ciprofloxacin or<br />

(b) An antipseudomonal agent and an aminoglycoside and a respiratory fluoroquinolone or a<br />

macrolide<br />

(4) Pseudomonas infection is an issue but the patient has a ~-lactam allergy -<br />

4. Types of pneumonia<br />

(a) Aztreonam and levofloxacin or<br />

(b) Aztreonam and moxifloxacin or gatifloxacin (with or without an aminoglycoside)<br />

a. Pneumococcal pneumonia<br />

(1) Etiology<br />

(a) Caused by 5 pneumoniae, a gram-positive lancet-shaped, encapsulated diplococcus<br />

(b) At least 83 serotypes have been isolated.<br />

(2) Most common cause of community-acquired bacterial pneumonia and the number one cause of<br />

bacterial pneumonia in HIV-infected patients<br />

(a) Seen in 1 in 500 persons annually<br />

(b) Peak incidence in winter/early spring<br />

(c) Mortality rate is 90%<br />

are sensitive to ceftriaxone.<br />

(e) Classic patient: extremes of age and chronically ill, immunocompromised (eg, HIV, splenectomy,<br />

transplant)<br />

(3) Classic clinical scenario: The patient appears acutely ill and can usually report exactly when the illness<br />

began (abrupt onset). The presence of fever, tachypnea, and tachycardia is coupled with sharp pleuritic<br />

chest pain associated with marked splinting on the affected side. There may be a history of a single,<br />

acute shaking chill, followed by a cough productive of a rust-colored sputum. Flank or back pain,<br />

anorexia, and vomiting are additional symptoms. On physical examination, the skin may be cyanotic<br />

or jaundiced, auscultation reveals crackles in the involved region (isolated to a specific lobe of the<br />

lung), and there are signs of pulmonary consolidation (bronchial breath sounds, egophony, increased<br />

tactile and vocal fremitus). A pleural friction rub may be heard.<br />

(4) Diagnostic evaluation<br />

(a) WBC count 12,000-25,000/mm 3 but may be higher<br />

(b) Chest radiograph<br />

i. Single lobar infiltrate (patchy in infants and the elderly)<br />

ii. Occasionally bulging fissures<br />

iii. Pleural effusion (25%)<br />

(c) Sputum<br />

(5) Management<br />

i. Rust-colored sputum<br />

ii. Gram stain reveals a single predominant gram-positive organism in pairs or chains.<br />

iii. Sputum culture is positive in 50%, and blood cultures are positive in 30% of cases.<br />

(a) Despite the prevalence of increasingly resistant strains (up to 40%), penicillin is still the drug of<br />

choice.<br />

(b) A macrolide or doxycycline is preferred for uncomplicated infections in outpatients.<br />

(c) Fluoroquinolones are no longer recommended for empiric outpatient therapy in otherwise healthy<br />

patients; they are reserved for outpatients with comorbidities (COPD, diabetes, renal failure, CHF,<br />

or malignancies) and for those who have recently received antibiotics for another infection.<br />

(d) For those patients requiring IV therapy, one of the following protocols is recommended:<br />

i. Cefotaxime or ceftriaxone and a macrolide (regimen preferred by the CDC) or<br />

ii. Monotherapy with an extended-spectrum fluoroquinolone<br />

(e) Vaccinations indicated for:<br />

i. Alcoholics<br />

ii. Cigarette smokers<br />

241


THORACIC AND RESPIRATORY DISORDERS<br />

iii. COPD patients<br />

iv. CHF patients<br />

v. Diabetic patients<br />

vi. lmmunocompromised patients (eg, HIV, cancer, transplant)<br />

vii. Asplenic (functional or anatomical)<br />

viii. Patients >65 years old<br />

(6) Complications<br />

(a) Sepsis<br />

(b) Meningitis<br />

(c) Endocarditis/pericarditis<br />

(d) CHF<br />

(e) Empyema (


THORACIC AND RESPIRATORY DISORDERS<br />

(b) When patients require IV therapy, a second- or third-generation cephalosporin (cefuroxime,<br />

ceftriaxone) should be used.<br />

(6) Complications<br />

(a) Sepsis<br />

(b) Meningitis<br />

(c) Empyema<br />

(d) Arthritis<br />

c. Klebsiella pneumonia<br />

(1) Etiology<br />

(a)<br />

Caused by K pneumoniae, a short, plump, encapsulated gram-negative bacillus that is seen in pairs<br />

(b) On a poor-quality Gram stain, Klebsiella is easily confused with pneumococcus.<br />

(2) Epidemiology: most common in patients with<br />

(a) Alcoholism<br />

(b) Diabetes<br />

(c) COPD<br />

(d) Renal failure<br />

(e) Nursing-home patients<br />

(f)<br />

Recent use of antibiotics<br />

(3) Classic clinical scenario: sudden onset of cough followed by multiple shaking chills and shortness of<br />

breath. The patient (usually a middle-aged or older man, history of alcoholism) complains of pleuritictype<br />

chest pain and is cyanotic. Sputum is currant jelly or dark brown colored. Signs of pulmonary<br />

consolidation are present on physical examination.<br />

(4) Diagnostic evaluation<br />

(a) Leukocytosis (75% of cases)<br />

(b) Chest radiograph<br />

i. Necrotizing right upper lobe infiltrate or abscess formation with an air-fluid level<br />

ii. Perihilar and patchy infiltrates are occasionally seen.<br />

iii. Bulging minor fissure (35 %)<br />

(c) Gross sputum examination<br />

(5) Management<br />

i. Dark brown, tenacious, and occasionally blood-stained<br />

ii. Resembles currant jelly<br />

(a) Attentive airway management because the sputum is frequently so thick that clearance is difficult<br />

(b) IV cephalosporin (ceftriaxone, cefuroxime, or cefotaxime) plus an aminoglycoside (gentamicin,<br />

tobramycin, or amikacin) are the initial agents of choice. Alternative agents include aztreonam<br />

and imipenem.<br />

(6) Complications<br />

(a) Empyema within 24-48 hours (20%)<br />

(b) Sepsis<br />

(c) Pneumothorax<br />

d. Other gram-negative pneumonias (occur rarely)<br />

(1) Etiology<br />

(a) f coli<br />

(b) Pseudomonas<br />

(c) Enterobacter<br />

(d) Serratia<br />

(2) Clinical presentation: usually immunosuppressed and debilitated patients<br />

(3) Treatment: piperacillin/tazobactam<br />

e. Staphylococcal pneumonia<br />

(1) Etiology: large gram-positive cocci in pairs and clusters<br />

(2) Epidemiology<br />

(a) 1 % of all bacterial pneumonias<br />

243


THORACIC AND RESPIRATORY DISORDERS<br />

(b) Peak incidence is during measles and flu epidemics.<br />

(c) Classic patient: IV drug abusers, nursing-home patients, debilitated patients, patients recovering<br />

from influenza infection, very young patients<br />

(3) Classic clinical scenario: The patient presents with fever, multiple chills, and pleuritic chest pain. A<br />

previous course of flu was followed by the insidious onset of a cough productive of purulent sputum.<br />

Coarse rhonchi and rales are heard on chest auscultation; signs of consolidation are rare.<br />

(4) Diagnostic evaluation<br />

(a) WBC count usually> l 5,000/mm 3<br />

(b) Chest radiograph<br />

i. A patchy infiltrate that is initially multicentric or peripheral (because it results from<br />

hematogenous spread) ultimately progresses to necrotizing pneumonia, including lobar<br />

consolidation and abscess formation.<br />

ii. Empyema is common.<br />

iii. A pleural effusion may also occur.<br />

(c) Blood cultures are usually negative unless the pulmonary involvement is metastatic.<br />

(5) Management<br />

(a) IV oxacillin or nafcillin are the antibiotics of choice.<br />

(b) IV vancomycin is the alternative agent; it is usually reserved for patients who are allergic to (or<br />

resistant to) penicillin.<br />

f. Group A streptococcal pneumonia (rare)<br />

(1) Etiology: gram-positive cocci in pairs or chains<br />

(2) Classic clinical scenario: sudden onset of fever, chills, and productive cough. The sputum is usually<br />

bloody and purulent; chest examination reveals fine rales without signs of consolidation.<br />

(3) Diagnostic evaluation: chest radiograph shows multi lobular bronchial infiltrates with a large pleural<br />

effusion.<br />

(4) Management: Because of the high mortality associated with this rapidly progressive pneumonitis,<br />

patients generally require admission for IV antibiotics<br />

(a)<br />

Aqueous penicillin is the drug of choice.<br />

(b) A cephalosporin (ceftriaxone) or erythromycin is an alternative agent.<br />

5. Patient assessment and risk stratification: The Pneumonia Severity Index (ACEP Clinical Policy: Management and<br />

Risk Stratification of Community-Acquired Pneumonia in Adults in the Emergency Department)<br />

a. There are five severity classes based on mortality rate.<br />

(1) Classes I, II, and Ill:


THORACIC AND RESPIRATORY DISORDERS<br />

(5) Laboratory studies and radiographs<br />

(a) Arterial pH


THORACIC AND RESPIRATORY DISORDERS<br />

Table 12: SMART-COP<br />

Community-Acquired Pneumonia Confirmed on Chest Radiograph<br />

Points<br />

Systolic blood pressure 125 beats per minute<br />

!:. Confusion (new onset)<br />

Q<br />

Oxygen low - age-adjusted cut-offs<br />

Age -S50 years, PaO 2<br />


THORACIC AND RESPIRATORY DISORDERS<br />

(b) Earache (33%)<br />

(c) Anorexia, nausea, vomiting, and diarrhea in the first week (12%-14%)<br />

(2) Most patients have segmental rales and rhonchi on physical examination. Other findings include:<br />

(a) Conjunctivitis<br />

(b) Pharyngitis<br />

(c) Bullous myringitis (3%-10%)<br />

c. Comp I ications<br />

(1) Aseptic meningitis or encephalitis<br />

(2) Hemolytic anemia<br />

(3) Glomerulonephritis<br />

(4) Gui I lain-Barre syndrome<br />

(5) Cardiac abnormalities, eg, pericarditis, myocarditis, AV block, CHF<br />

(6) Splenomegaly<br />

(7) Erythema multiforme<br />

d. Diagnostic evaluation<br />

(1) WBC count is usually normal or moderately increased.<br />

(2) Does not show up on Gram stain<br />

(3) Chest radiograph<br />

(a) One or more segmental infiltrates, dense consolidation, or a generalized interstitial pattern may<br />

be present.<br />

(b) Segmental (or patchy) infiltrates are usually in the lower lobes and appear as streaks radiating from<br />

the hilum.<br />

(c) Interstitial pneumonia characterized by a reticulonodular pattern is often associated with<br />

deterioration of pulmonary function and can progress to respiratory failure.<br />

(d) Small pleural effusions are present in 20% of patients.<br />

(e) The hallmark of this disease is the disparity between the patient's clinically benign appearance<br />

and the extensive radiographic findings.<br />

(4) Cold agglutinin titers are increased in up to 60% of these patients and are supportive of the diagnosis<br />

but are neither sensitive nor specific.<br />

(5) A 4-fold increase in complement-fixing antibody titers is diagnostic; an initial titer >1:64 is very<br />

suggestive.<br />

e. Treatment<br />

(1) Erythromycin, clarithromycin, and azithromycin are the drugs of choice.<br />

(2) Fluoroquinolones and doxycycline are alternative agents.<br />

3. Chlamydia! pneumonia (6% of community-acquired pneumonia)<br />

a. Etiology: Chlamydia pneumoniae, an obligate intracellular, gram-negative organism<br />

b. Epidemiology<br />

(1) Common cause of atypical pneumonia in young adults<br />

(2) Spread is from person to person by droplet transmission.<br />

(3) Outbreaks generally occur as a cluster of cases in enclosed populations (eg, boarding schools, dorms,<br />

army barracks, prisons).<br />

c. Classic clinical scenario: The patient is usually a young adult who complains of a prolonged dry cough and<br />

low-grade fever that was preceded by a sore throat. Other complaints may include laryngitis/hoarseness<br />

(present in one-third of patients), mild headache, myalgias, and diarrhea. Examination reveals rales or<br />

rhonchi (and sometimes wheezing) on auscultation of the lungs and a nonexudative pharyngitis.<br />

d. Diagnostic evaluation<br />

(1) WBC count is usually normal.<br />

(2) Chest radiograph typically reveals a subsegmental pneumonitis.<br />

(3) Nasopharyngeal culture or serology confirms the diagnosis.<br />

e. Treatment<br />

(1) Tetracycline, doxycycline, and erythromycin are the agents of choice; a 3-week course of therapy is<br />

recommended.<br />

(2) Azithromycin or clarithromycin (and quinolones) are also effective.<br />

247


THORACIC AND RESPIRATORY DISORDERS<br />

4. Psittacosis<br />

a. Etiology: Chlamydia psittaci<br />

(1) An obligate intracellular, gram-negative organism harbored in avian species<br />

(2) Transmitted by inhalation of infected dust or droplets<br />

b. Epidemiology: owners of pet birds (particularly parrots), pet-shop employees, poultry workers, and<br />

veterinarians most commonly affected<br />

c. Clinical presentation<br />

(1) Hyperpyrexia (up to 105°F [40.6°C])<br />

(2) Severe headache (often the major complaint)<br />

(3) Cough (occasionally associated with hemoptysis)<br />

(4) Hepatosplenomegaly<br />

(5) A flu-like syndrome consisting of malaise, myalgias, and an upper respiratory infection<br />

(6) Relative bradycardia<br />

d. Diagnostic evaluation<br />

(1) Leukopenia (25%)<br />

(2) Proteinuria<br />

(3) Abnormal liver profile (high enzymes)<br />

(4) Patchy perihilar or lower lung field infiltrates on chest radiograph<br />

(5) Increased complement fixation antibody titer<br />

e. Treatment<br />

(a) A 4-fold rise (1 :32) is diagnostic.<br />

(b) A 1 :16 titer is presumptive evidence.<br />

(c) False-positives are seen in patients with brucellosis and Q fever.<br />

(1) Doxycycline is the drug of choice; a 2-week course is recommended.<br />

(2) Azithromycin and clarithromycin are alternative agents.<br />

f. Complications<br />

(1) Multiple and severe without antibiotic therapy<br />

(2) Include hepatitis, myocarditis, endocarditis, meningitis, renal failure, disseminated intravascular<br />

coagulation<br />

5. Q fever pneumonia<br />

a. Etiology<br />

(1) Coxiella burnetii, an obligate intracellular bacterium<br />

(2) Highly infectious; can survive in dried soil or excrement up to 18 months and in tap water or milk for<br />

up to 42 months<br />

b. Epidemiology<br />

(1) People usually become infected by inhaling dust contaminated with excreta, placenta, or uterine<br />

excretions of infected sheep, goats, cattle, or parturient cats.<br />

(2) Slaughterhouse workers, dairy farmers, and those who work closely with animals (especially farm<br />

livestock) are most commonly affected.<br />

c. Classic clinical scenario: The patient is a farmer who presents ill and is diaphoretic and febrile. The usual<br />

history is sudden onset of shaking chills, high fever, myalgias, severe headache, and a nonproductive<br />

cough. Physical examination shows hepatomegaly. Chest findings are frequently minimal (fine rales) or<br />

absent.<br />

d. Diagnostic evaluation<br />

(1) Abnormal liver function studies (85%)<br />

(2) Proteinuria (62%)<br />

(3) Sterile pyuria (12%)<br />

(4) Rounded segmental densities in the lower lobes or lobar consolidation on chest radiograph<br />

(5) Serologic studies are the diagnostic test of choice.<br />

e. Treatment<br />

(1) Early antibiotic therapy helps promote a quick response and prevents relapse or chronic Q fever.<br />

(2) Tetracycline, doxycycline, or chloramphenicol<br />

(3) If patient is pregnant, consider TMP-SMX.<br />

248


THORACIC AND RESPIRATORY DISORDERS<br />

f. Complications associated with a prolonged illness<br />

6. Tularemia<br />

(1) Relapse despite antibiotic therapy<br />

(2) Endocarditis<br />

(3) Hepatitis<br />

(4) Meningitis<br />

a. Etiology: Francisella tularensis, a gram-negative, nonmotile, pleomorphic coccobacillus bacterium that is<br />

harbored principally in hard ticks and wild rabbits<br />

b. Epidemiology<br />

(1) Transmission is usually via direct contact with tissues or body fluids of infected animals, exposure to<br />

an infected tick, or inhalation of contaminated dust or water aerosol.<br />

(2) Hunters, trappers, butchers, cooks, and campers are most commonly affected.<br />

c. Clinical forms<br />

(1) Ulceroglandular (most common): characterized by an indurated skin lesion at the site of inoculation<br />

and regional lymphadenopathy<br />

(2) Typhoidal: characterized by fever, chills, weight loss, and hepatosplenomegaly<br />

(3) Glandular<br />

(4) Oculoglandular<br />

(5) Oropharyngeal<br />

d. Tularemia pneumonia: usually acquired via inhalation of contaminated aerosol or from bacteremia but may<br />

also arise as a complication of either the ulceroglandular or typhoidal forms<br />

e. Classic clinical scenario of tularemia pneumonia: The patient is a rabbit hunter (often a male) presents with<br />

high fever (104°-106°F [40°-41 °C]), shaking chills, and cough (usually nonproductive). Other symptoms<br />

include chest pain, shortness of breath, and hemoptysis. Examination of the chest is often normal but may<br />

reveal rales, consolidation, or a pleural rub. Hepatosplenomegaly and a maculopapular rash may also be<br />

present. Chest radiograph usually reveals bilateral, patchy, poorly defined or ovoid infiltrates, as well as<br />

hilar lymphadenopathy and pleural effusion.<br />

f. Diagnostic evaluation: serologic studies (ELISA) confirm the diagnosis.<br />

(1) A 4-fold rise between acute and convalescent titers is diagnostic.<br />

(2) A single convalescent titer 2':1 :160 is very suggestive.<br />

g. Management<br />

(1) Streptomycin is the drug of choice.<br />

(2) Gentamicin and kanamycin are alternative agents.<br />

(3) Tetracycline and chloramphenicol are also effective but are associated with a high rate of relapse and<br />

should be reserved for patients who cannot tolerate any of the above agents.<br />

(4) A live, attenuated vaccine is available for high-risk laboratory personnel.<br />

h. Complications<br />

C. Viral pneumonia<br />

(1) Mortality rate is 5%-30% without antibiotic therapy and


THORACIC AND RESPIRATORY DISORDERS<br />

(5) Management<br />

(a)<br />

b. Parainfluenza<br />

Primarily supportive; if admission is required, specific therapy may be indicated.<br />

(b) Infants should receive a trial of ~-adrenergic therapy during the first 24 hours.<br />

(1) Second most common cause of pneumonia in children<br />

(2) Also causes croup and bronchitis<br />

c. Adenovi ruses<br />

(1) Target populations: children and military recruits<br />

(2) Clinical presentation: fever, cough, rhinitis, conjunctivitis, and pharyngitis<br />

(3) Diagnostic evaluation: chest radiographs show lower lobe infiltrates.<br />

d. Varicella zoster virus<br />

(1) Occurs primarily in adults; especially severe in pregnant patients<br />

(2) Clinical presentation<br />

(a)<br />

Illness begins with a rash that is followed within a week by fever and cough associated with<br />

tachypnea and dyspnea.<br />

(b) 20%-40% of patients also develop cyanosis, hemoptysis, and pleuritic chest pain.<br />

(3) Diagnostic evaluation<br />

(a) Sputum analysis may reveal multi nucleated giant cells.<br />

(b) Chest radiograph usually reveals an interstitial pneumonia; however, micronodular and lobar<br />

patterns may also be seen.<br />

(4) Management<br />

(a) This is a serious adult illness and requires admission.<br />

(b) Administration of IV acyclovir is indicated.<br />

e. Influenza viruses (most common cause of viral pneumonia in adults)<br />

(1) Occurs between November and April<br />

(2) Clinical presentation<br />

(a)<br />

Usual syndrome has an incubation period of 1-5 days followed 2 weeks later by fever, headache,<br />

and a nonproductive cough.<br />

(b) Patients may have an associated bacterial pneumonia.<br />

(c) Up to 40% of patients with a normal chest radiograph have rales, rhonchi, and wheezing on<br />

examination.<br />

(d) Pure influenza pneumonia (no associated bacterial infection) is much more deadly. The elderly,<br />

pregnant patients, and those with chronic health conditions (eg, heart disease, COPD, diabetes<br />

mel I itus) are at risk.<br />

i. Sudden weakness is followed by dyspnea, cyanosis, and ARDS.<br />

ii. Chest radiographs show bilateral interstitial infiltrates.<br />

(3) Diagnostic evaluation: rapid antigen detection assay is recommended for epidemiologic reasons.<br />

(4) Management<br />

(a)<br />

Primarily supportive<br />

i. Begin antiviral therapy if onset of symptoms within 48 hours or at any time if a high-risk<br />

population.<br />

ii. Zanamivir or oseltamivir is effective for A and B strains.<br />

(b) Amantadine (or rimantadine) is helpful in the treatment and prophylaxis of patients with influenza A.<br />

f. Cytomegalovirus (CMV)<br />

(1) CMV pneumonia is a complication in transplant recipients and patients with advanced AIDS.<br />

(2) Clinical presentation<br />

(a)<br />

CMV can be either a true pathogen or a coexistent organism.<br />

(b) In transplant recipients, CMV is a true pathogen. It generally produces pneumonia within 1-3<br />

months after transplantation and is the most common cause of death in recipients of bone<br />

marrow transplants.<br />

(3) Classic clinical scenario: In AIDS patients, CMV is often found in association with other pathogens and<br />

may represent a coexistent organism rather than a true pathogen. Thus, a definitive diagnosis of CMV<br />

pneumonitis in these patients requires all of the following:<br />

250


THORACIC AND RESPIRATORY DISORDERS<br />

(a) A compatible clinical picture (fever, hypoxia, and infiltrates on chest radiograph)<br />

(b) Positive cultures for CMV<br />

(c) Absence of other pathogenic organisms<br />

(4) Diagnostic evaluation: chest radiograph most often reveals bilateral interstitial (or reticulonodular)<br />

infiltrates that begin in the periphery of the lower lobes and spread centrally and superiorly.<br />

(5) Management<br />

g. Hantavirus<br />

(a) IV ganciclovir or foscarnet<br />

(b) lmmunoglobulin is <strong>combined</strong> with the above regimen in.bone marrow recipients (who are<br />

particularly susceptible to CMV infection).<br />

(1) Infection results from inhalation of aerosols that contain material contaminated with rodent urine and feces.<br />

(2) Epidemiology: residents of the southwestern United States (New Mexico, Arizona, Colorado, Utah) are<br />

most commonly affected.<br />

(3) Clinical presentation: an initial prodrome of fever, myalgia, and malaise progresses over several days<br />

and evolves into a syndrome of severe respiratory distress and shock.<br />

(4) Diagnostic evaluation: chest radiograph reveals bilateral interstitial infiltrates (most prominent in<br />

dependent lobes).<br />

(5) Management: supportive care and IV ribavirin (experimental)<br />

D. Types of pneumonia: classic clinical scenario and etiology<br />

Table 13: Classic Clinical Scenario and Etiology of Pneumonia<br />

Classic Clinical Scenario<br />

Pneumonia plus hyponatremia, increased liver enzymes, watery diarrhea,<br />

nausea/vomiting, seizure, history of being at a convention<br />

Elderly, post influenza, nursing home, cocci in clusters, lung abscess formation<br />

Young, healthy male, 5-day prodrome (upper respiratory infection), respiratory<br />

failure (ARDS)<br />

Young, bullous myringitis, chest radiograph worse than clinical picture<br />

COPD, gram-negative pleomorphic rods<br />

Alcoholic, bulging fissures, currant jelly sputum<br />

Elderly, recent upper respiratory infection, winter months, sudden prostration<br />

Military recruits, fever, cough, rhinitis, conjunctivitis<br />

Rusty sputum, single shaking chill, pleuritic chest pain<br />

Staccato cough, prolonged dry cough in college student<br />

Transplant patient 1-3 months after transplant with cough, hypoxia<br />

IV drug abuser, fungal infection throat, CD4<br />

Hunter, trapper, butcher, cook, high fever<br />

Bird (parrot) owner, relative bradycardia<br />

Slaughterhouse worker, goat, sheep, parturient cats<br />

From southwest US with cough, fever, weight loss, diffuse pulmonary infiltrates<br />

Abrupt onset of fever, infiltrates, wide mediastinum<br />

Cause/Type<br />

Legionella<br />

Staphylococcus<br />

Hanta virus<br />

Mycoplasma<br />

Haemophilus influenzae<br />

Klebsiel/a<br />

Influenza pneumonia<br />

Adenovirus pneumonia<br />

Streptococcus pneumoniae<br />

Chlamydia<br />

CMV pneumonia<br />

Pneumocystis jiroveci<br />

pneumonia<br />

Tularemia pneumonia<br />

Psittacosis pneumonia<br />

Q fever pneumonia<br />

Coccidiomycosis<br />

Bacillus anthracis<br />

251


THORACIC AND RESPIRATORY DISORDERS<br />

II. TUBERCULOSIS (TB)<br />

A. Epidemiology<br />

1. TB is the second most common infectious cause of death (HIV is most common); one-third of the world's<br />

population is infected with Mycobacterium tuberculosis. There are 9 million new cases per year, and 1.7<br />

million deaths per year (2009).<br />

2. TB is an AIDS-defining opportunistic infection and is the only opportunistic infection in AIDS patients that is<br />

transmitted by the respiratory route to both immunocompromised and immunocompetent hosts.<br />

B. Pathophysiology<br />

1. The causative organism is Mycobacterium tuberculosis, a weakly gram-positive obligate aerobic rod with acidfast<br />

staining properties that multiplies very slowly (once every 12-24 hours).<br />

2. Transmission occurs via aerosolized droplets produced by coughing, sneezing, talking, or breathing. Infection<br />

develops when these contaminated droplets are inhaled and reach the alveoli.<br />

3. Once in the alveoli, tubercle bacilli are phagocytized (but not killed) by alveolar macrophages and proliferate<br />

within these cells to form a primary focus of infection (primary TB), usually in the lower lobes. Organisms may<br />

also spread from this initial site of infection through the lymphatics to regional lymph nodes and to distant<br />

organs via the bloodstream. This bacillemia is usually asymptomatic, but it produces metastatic foci throughout<br />

the body that may become active later in life. These foci are preferentially established in areas of high oxygen<br />

tension such as the apical and posterior segments of the upper lobes of the lung, the kidneys, bones, and brain.<br />

4. Most infected patients mount an effective immune response and have no further infectious sequelae.<br />

T lymphocytes reach sufficient numbers to control the infection 2-1 0 weeks (average 6-8 weeks) after<br />

exposure. The tuberculin skin test becomes positive at this time, indicating that cell-mediated immunity has<br />

developed. lmmunocompromised patients, however, may be unable to mount an adequate immune response.<br />

In these individuals, a rapidly progressive primary infection resulting in early death can evolve.<br />

5. After a period of dormancy, some infected patients go on to develop active disease. This generally occurs when<br />

the patient's immune response is altered in some manner.<br />

a. The lifetime risk of reactivation in the general population is 10% but is much greater in patients with<br />

impaired cellular immunity.<br />

b. Conditions associated with an increased rate of conversion to active disease<br />

(1) AIDS (rate of progression is 7%-10% per year)<br />

(2) lmmunosuppressive therapy, including steroids<br />

(3) Renal failure, hemodialysis<br />

(4) Diabetes mellitus<br />

(5) Malnutrition, alcoholism<br />

(6) Malignant disease, malignant lymphoma<br />

(7) Postgastrectomy and postintestinal bypass states<br />

(8) Transplant recipients<br />

C. Clinical presentation<br />

1. Pulmonary TB (inactive, or dormant, foci)<br />

a. Asymptomatic in 90% of patients; can be identified only by development of a positive TB skin test and<br />

possibly a Ghan complex on chest radiograph<br />

b. A pneumonitis may also be seen (usually in the lower lobes).<br />

2. Reactivation TB (endogenous reactivation of dormant foci)<br />

a. Most common clinical form of TB and is seen most often in the elderly<br />

b. Symptoms include low-grade fever, night sweats, malaise, weight loss, and productive cough (most<br />

common symptom), pleuritic chest pain.<br />

c. Signs of chronic wasting are present in most patients.<br />

d. Sites of involvement include the apical and posterior segments of the upper lungs, kidneys, bones/joints,<br />

and brain.<br />

(1) Pulmonary involvement is present in >80% of patients.<br />

(2) Extrapulmonary involvement is present in 15% of the general population but is greater in patients<br />

with HIV.<br />

3. Pulmonary TB (active foci)<br />

a. Clinical onset<br />

252


THORACIC AND RESPIRATORY DISORDERS<br />

(1) Insidious: patients (usually debilitated) present with a chronic cough (most common symptom) and<br />

constitutional symptoms of reactivation such as malaise, weight loss, night sweats, and fever. As the<br />

cough progresses over time, it becomes productive of mucopurulent sputum and is often associated<br />

with hemoptysis. Patients may also complain of a dull ache or tightness in the chest.<br />

(2) Abrupt: some patients present with acute onset of fever, chills, cough (most common symptom), and<br />

myalgias that mimic an episode of acute bronchitis or pneumonia. Unless TB is considered in the<br />

differential diagnosis and smears of sputum for acid-fast bacilli are obtained, these patients may be<br />

misdiagnosed as having bacterial pneumonia.<br />

b. Chest examination is often unremarkable but may reveal rales or consolidation in the presence of extensive<br />

pulmonary involvement.<br />

4. Extrapulmonary TB can result from primary infection or reactivation and may involve almost any organ in the<br />

body. It may also take a disseminated form (miliary TB). The signs and symptoms produced are determined by<br />

the structures that are affected.<br />

a. TB meningitis (most rapidly progressive form ofTB)<br />

(1) Results from seeding during the primary infection or from rupture of a subependymal lesion (Rich foci)<br />

into the subarachnoid space.<br />

(2) Clinical onsets<br />

(a) Insidious: patients often present with a nonspecific febrile illness of 1-6 weeks duration, followed<br />

by intermittent headache, confusion, personality changes, stiff neck, diplopia, photophobia,<br />

cranial nerve palsies, decreased level of consciousness, and seizures.<br />

(b) Fulminant: some patients, particularly children, may present acutely with fever and delirium in<br />

association with a severe headache and stiff neck.<br />

(3) Diagnostic testing<br />

b. Pleural TB<br />

(a) Positive TB skin test (75%)<br />

(b) Cerebrospinal fluid analysis (acid-fast stain not usually positive)<br />

1. t Pressure and protein<br />

ii . .J, Glucose<br />

iii. t WBC count of 100-1,000/mL (predominantly lymphocytes)<br />

(1) Results from rupture of a parenchymal focus into the pleural space<br />

(2) Associated pleural effusion is exudative in nature with laboratory evaluation revealing:<br />

(a) Increased protein<br />

(b) Low pH<br />

(c) Normal or low glucose<br />

(d) WBC count of 1,000-5,000/mL (mostly monocytes)<br />

(e) An acid-fast smear that is often negative<br />

(3) Pleural biopsy is helpful in making the diagnosis.<br />

c. Genitourinary TB<br />

(1) Patients usually present with urinary symptoms (dysuria, frequency) hematuria, or flank pain.<br />

Constitutional symptoms may also be present.<br />

(2) Urinalysis reveals:<br />

(a) Pyuria without bacteriuria<br />

(b) Low pH<br />

d. Miliary (disseminated) TB<br />

(1) A multisystemic process resulting from a progressive primary infection (immunocompromised patients)<br />

or from secondary bloodstream seeding during recrudescence of previously dormant foci<br />

(2) Clinical presentation<br />

(a) Symptoms are usually nonspecific and may include fever, anorexia, weight loss, and weakness.<br />

Depending on the sites of involvement, more specific symptoms (eg, dyspnea, cough, headache)<br />

may also be present.<br />

(b) Physical findings may include:<br />

i. Fever<br />

ii. Pulmonary findings<br />

iii. Hepatomegaly<br />

253


THORACIC AND RESPIRATORY DISORDERS<br />

5. TB and HIV<br />

iv. Lymphadenopathy<br />

v. Splenomegaly<br />

vi. Tubercles of the retina (circumscribed, spheroid, granulomatous lesions with three distinct<br />

zones, the only specific finding)<br />

(3) Diagnostic evaluation<br />

(a) Anemia and WBC abnormalities (leukopenia, leukemoid reactions, agranulocytosis)<br />

(b) Hyponatremia<br />

(c) Negative TB skin test 25%-50% of the time (especially in the elderly and immunocompromised)<br />

(d) Chest radiograph typically reveals small nodular densities that are uniformly distributed<br />

throughout both lung fields. A pleural effusion may also be present.<br />

a. TB is an AIDS-defining illness and generally produces disease at an earlier stage of HIV infection than other<br />

opportunistic infections.<br />

b. The incidence ofTB in HIV-infected patients approaches 60% and currently represents the greatest health<br />

care risk to the general public from the HIV epidemic.<br />

c. Co-infection with TB and HIV results in:<br />

(1) A greater incidence of extrapulmonary disease<br />

(2) More atypical clinical findings<br />

(3) An increased incidence of tuberculin nonreactivity and negative acid-fast bacilli smears<br />

(4) A greater frequency of unusual and atypical chest radiographs<br />

(5) Decreased cavitary disease<br />

(6) More antibiotic resistance<br />

(7) Higher relapse and mortality rates<br />

(8) A larger number of adverse drug reactions and multidrugresistantTB (especially to rifampin and<br />

isoniazid)<br />

D. Diagnostic evaluation<br />

1. TB must be considered in the differential diagnosis of any patient who presents with respiratory complaints<br />

or extrapulmonary symptoms, particularly if the patient is a member of a high-risk group (patients with AIDS,<br />

immigrants, IV drug abusers, residents/employees of longterm care facilities).<br />

2. Tests used to establish the diagnosis include the TB skin test, chest radiograph, and microbiologic studies for<br />

acid-fast baci 11 i.<br />

3. TB skin test<br />

a. The standard test for detecting infection with M tuberculosis<br />

b. Involves intradermal administration of purified protein derivative and is read 48-72 hours after<br />

administration<br />

c. A positive reaction indicates the presence of infection but not necessarily the presence of active disease<br />

(must be confirmed by culture).<br />

d. Criteria for interpreting the test as true-positive vary with patient background and are noted in Table 14.<br />

Table 14: TB Skin Test Interpretation<br />

>5 mm<br />

>10 mm >15 mm<br />

Known or suspected HIV<br />

infection<br />

Close contact with a person with<br />

active TB<br />

lmmunocompromised<br />

Organ transplant<br />

Nodular or fibrotic changes on<br />

chest radiograph<br />

Residents of or employees in highrisk<br />

settings (eg, health care facilities,<br />

homeless shelters, and prisons)<br />

IV drug abusers<br />

Immigrants from an area with a high<br />

incidence of TB<br />

Certain high-risk minority groups<br />

(Hispanics, black Americans, Native<br />

Americans)<br />

Children


THORACIC AND RESPIRATORY DISORDERS<br />

e. A negative test does not exclude the diagnosis, because some patients (particularly those who are<br />

immunocompromised) are anergic.<br />

f. A false-positive test may be due to infection with M avium or M kansasii (nontuberculous mycobacteria).<br />

Clues to this diagnosis include the following:<br />

(1) No history of risk factors for TB<br />

(2) A negative skin test (or a reaction smaller than a true-positive)<br />

(3) History of COPD<br />

4. Chest radiograph<br />

a. Primary TB<br />

(1) Small parenchymal infiltrates located in any area of the lung and unilateral hilar adenopathy<br />

(2) These lesions may subsequently calcify to form a Ghan complex.<br />

(3) Inflammatory infiltrates of the lower lobes with associated hilar adenopathy are seen in patients with<br />

progressive primary infections and clinically evident disease.<br />

(4) Hilar adenopathy is the radiologic hallmark of primary TB.<br />

(5) Pleural effusions are seen in one-third of patients, typically in first 3-4 months after infection.<br />

b. Reactivation/pulmonary TB<br />

(1) Typically affects the apical-posterior segments of the upper lobe. Nodular densities are most often seen<br />

in the apical (Simon foci) or posterior segments of the upper lobe but may also be found in the upper<br />

segment of the lower lobe.<br />

(2) Associated cavitation may or may not be present.<br />

c. Miliary (disseminated) TB<br />

(1) May initially be normal but classically reveals small nodules (1-3 mm) scattered throughout both lung<br />

fields in a miliary pattern.<br />

(2) A pleural effusion (frequently unilateral) may also be present.<br />

d. HIV-infected patients<br />

(1) The chest radiograph is often atypical and may even be normal.<br />

(2) Upper lobe cavitary lesions are rare, while hilar or mediastinal adenopathy and lower lobe infiltrates<br />

are more common.<br />

(3) A diffuse interstitial pattern that is easily mistaken for Pneumocystis jerovici pneumonia may also be<br />

seen.<br />

5. Microbiologic studies<br />

a. Staining of sputum for acid-fast bacilli with Ziehl-Neelsen or fluorescent (fluorochrome) staining (which is<br />

more sensitive)<br />

(1) Provides a rapid presumptive diagnosis ofTB; number of bacilli seen correlates with the degree of<br />

i nfectivity.<br />

(2) Positive smears have a specificity of 98%.<br />

(3) Smear results should be confirmed by culture.<br />

b. Culture of sputum or tissue for acid-fast bacilli<br />

E. Management<br />

(1) More sensitive than staining and is the "gold standard" for confirming the diagnosis ofTB<br />

(2) Although traditional cultures take 3-6 weeks, new radiometric techniques can confirm the diagnosis<br />

in as few as 5 days; DNA probes, reverse transcription, and polymerase chain reaction tests allow<br />

for identification of TB in a matter of hours but, because of technical problems, have not received<br />

approval for routine clinical use.<br />

1. As soon as the diagnosis is suspected, the patient should wear a mask and be placed in an isolation room.<br />

2. Because of the emergence of multidrug-resistant strains ofTB, it is now recommended that initial therapy for TB<br />

include four drugs until susceptibility tests are available.<br />

3. The initial drug regimen of choice is isoniazid, rifampin, pyrazinamide, and streptomycin or ethambutol; in the<br />

absence of drug resistance, isoniazid and rifampin taken for 9 months is curative. (Preventive drug therapy also<br />

includes isoniazid [ie, for those with a positive intradermal skin test].)<br />

4. Adverse effects associated with these drugs:<br />

a. lsoniazid<br />

(1) Multiple neurologic entities, including peripheral neuritis; pyridoxine (vitamin 8 6 ) is administered<br />

concurrently to prevent isoniazid-induced neuropathy.<br />

255


THORACIC AND RESPIRATORY DISORDERS<br />

(2) Hepatitis<br />

(3) Hypersensitivity reactions<br />

(4) Drug-induced interactions with ketoconazole and fluconazole<br />

(5) Intractable seizures if patient is not on pyridoxine in conjunction with isoniazid: must treat with highdose<br />

vitamin 8 6<br />

(6) Lupus<br />

b. Rifampin<br />

(1) Hepatitis<br />

(2) Thrombocytopenia<br />

(3) Drug interactions with coumadin, oral contraceptives, digitalis derivatives, methadone, dapsone,<br />

cyclosporin, corticosteroids, oral hypoglycemic agents, ketoconazole, and fluconazole-,, decreased<br />

blood levels and effectiveness<br />

(4) Orange-colored tears, saliva, and urine<br />

c. Pyrazinamide<br />

(1) Hyperuricemia<br />

(2) Hepatitis<br />

(3) Arthralgias<br />

(4) Rash<br />

d. Ethambutol<br />

(1) Optic neuritis<br />

(2) Rash<br />

e. Streptomycin<br />

(1) Vestibular nerve damage<br />

(2) Nephrotoxicity<br />

5. Before starting therapy with these agents, baseline studies should be obtained.<br />

a. Liver function tests, BUN/creatinine, and CBC with platelet count for all patients<br />

b. Visual acuity (and red-green color perception) for patients being treated with ethambutol<br />

c. Serum uric acid for patients being treated with pyrazinamide<br />

Ill. PLEURAL EFFUSION<br />

A. Definition<br />

1. An abnormally large collection of fluid within the pleural space, reflecting the presence of an underlying<br />

i ntrathoracic or extrathoracic disease process<br />

2. Types of effusions<br />

a. Transudates: excessive hydrostatic pressure (eg, CHF) or insufficient oncotic pressure (t serum protein)-,,<br />

low protein plasma infiltrates<br />

b. Exudates: lymphatic blockage due to malignancy or pleural capillary damage due to infectious disease-,,<br />

high protein plasma infiltrates<br />

3. Most common malignancies causing pleural effusion<br />

a. Bronchogenic lung carcinoma<br />

b. Breast cancer<br />

c. Lymphoma<br />

d. Leukemia<br />

B. Clinical presentation<br />

1. The patient presents with chest pain that is pleuritic with associated symptoms of fever, cough, and shortness of<br />

breath.<br />

2. Physical examination findings can help to narrow differential.<br />

a. Splinting± a pleural friction rub-,, pleurisy<br />

b. Dullness to percussion+ t breath sounds+ t tactile fremitus-,, pleural fluid<br />

c. Bronchial breath sounds+ egophony-,, atelectasis<br />

d. Normal breath sounds+ distended neck veins+ left parasternal lift+ accentuated P 2<br />

on cardiac<br />

examination-,, massive pulmonary embolism<br />

256


THORACIC AND RESPIRATORY DISORDERS<br />

C. Diagnostic evaluation<br />

1. Chest radiographs<br />

a. Small effusions are most easily detected on a lateral decubitus film with the affected side down;<br />

accumulations of 5-50 ml of fluid can be detected with this view.<br />

b. Small effusions can be missed entirely on supine films and are generally not apparent on PA and lateral<br />

films until 2:200 ml of fluid is present.<br />

c. Early signs of smal I effusion<br />

(1) PA view-.. faint obscuring of the costophrenic angle<br />

(2) Lateral view-.. blunting or loss of the costophrenic angle<br />

(3) Lateral decubitus view - fluid layers out<br />

d. Signs of a moderate effusion<br />

(1) Ground-glass appearance of lung fields<br />

(2) No air bronchograms<br />

e. Sign of a large effusion -.. opacification (partial or complete)<br />

f. Pleural fluid in a fissure-.. a "phantom" or "pseudotumor"<br />

2. Ultrasound<br />

3.<br />

a. Small effusions can be easily detected with point-of-care ultrasound.<br />

b. Anechoic space seen on ultrasound image<br />

(1) Complicated effusions may show septations and hypoechoic material.<br />

(2) Important to identify surrounding structures of diaphragm and atelectic lung tissue<br />

Thoracentesis: the definitive procedure<br />

a.<br />

b.<br />

Contraindications<br />

(1) Uncooperative patient<br />

(2) Coughing or hiccups<br />

(3) Local skin infection<br />

(4) Coagu Iopa th ies<br />

(5) Anticoagulant therapy<br />

Technique<br />

(1) Mark the site with ultrasound.<br />

(2) Anesthetize the area with lidocaine.<br />

(3) The posterior entry is the primary approach. Insert the needle immediately above the rib to avoid the<br />

neurovascular bundle.<br />

(4) Loculated effusions may require CT-guided needle aspiration.<br />

(5) Withdrawing fluid<br />

(a) Reexpansion pulmonary edema may occur if fluid is removed too rapidly.<br />

(b) Limit removal of fluid to ~2 L.<br />

(6) After the procedure, obtain a chest radiograph to exclude an iatrogenic pneumothorax.<br />

c. Diagnostic evaluation<br />

(1) CBC with differential<br />

(2) Serum protein, lactate dehydrogenase (LDH), amylase, and glucose (


THORACIC AND RESPIRATORY DISORDERS<br />

f. Pleural fluid characteristics that indicate further drainage is required<br />

(1) Frank pus<br />

(2) Gram stain positive<br />

(3) pH


THORACIC AND RESPIRATORY DISORDERS<br />

D. Critical facts<br />

1. Most common cause of pleural effusion in developed countries: CHF<br />

2. Most common cause of pleural effusion in developing countries: TB<br />

3. Most common cause of transudate: CHF<br />

4. Most common cause of exudate: infection<br />

5. Normal amount of pleural fluid: 0.1-0.2 ml/kg body weight<br />

6. Transudate pathophysiology: increased hydrostatic pressure or decreased oncotic pressure; very little protein<br />

7. Exudate pathophysiology: increased membrane permeability or defective lymphatic drainage; large amount of<br />

protein<br />

8. Massive effusions are more commonly due to malignancy, but CHF can do it, too.<br />

9. Effusions are best seen in lateral decubitus with the effusion side down.<br />

10. Pulmonary embolism is the most commonly overlooked condition in the evaluation of patients with pleural<br />

effusion.<br />

11. If the difference between serum albumin and pleural albumin is > 1.2 g/dL, the effusion is a transudate.<br />

12. If pH


THORACIC AND RESPIRATORY DISORDERS<br />

5. Foreign-body aspiration<br />

a. The leading cause of accidental home death in children 90%) develop signs and symptoms within 1 hour of the event.<br />

5. Patients who have aspirated a foreign body (and have incomplete obstruction) present with choking, a<br />

spasmodic cough, and wheezing. Examination of the chest may reveal asymmetric chest wall movement,<br />

decreased breath sounds, wheezing, and hyperresonance to percussion on the involved side.<br />

6. Putrid discharge in sputum or pleural fluid is diagnostic of anaerobic infections. Anaerobic infections are<br />

typically indolent with absence of rigors. Predisposing conditions include IV drug abuse, alcoholism,<br />

concurrent periodontal disease, recent anesthesia, or dysphagia.<br />

D. Diagnostic evaluation<br />

1. Arterial blood gases<br />

a. The usual finding is hypoxia with respiratory alkalosis.<br />

b. Severe aspiration respiratory failure with a <strong>combined</strong> respiratory and metabolic acidosis<br />

2. Cultures<br />

a. Difficult to isolate from sputum cultures or bronchoscopy/bronchoalveolar lavage<br />

3. Chest radiograph<br />

a. Radiographic findings are often delayed.<br />

(1) Atelectasis is the initial finding and may be seen as early as 1 hour after aspiration.<br />

(2) Infiltrates develop 6-12 hours later and most frequently involve the right lower lobe (if aspiration<br />

occurred in the upright position).<br />

b. Foreign body aspiration_,. an end-expiratory PA film demonstrates a hyperexpanded lung on the involved<br />

side (usually the right side, because the right mainstem bronchus has less of an acute angle than the left).<br />

E. Management<br />

1. Provide supplemental oxygen.<br />

a. High-flow oxygen by nasal cannula or face mask may be sufficient for some patients.<br />

b. Patients who are hypercarbic (or remain hypoxic) despite these measures should be intubated and<br />

mechanically ventilated.<br />

c. If the patient has an altered level of consciousness and a decreased gag reflex, immediate endotracheal<br />

intubation is needed.<br />

260


THORACIC AND RESPIRATORY DISORDERS<br />

2. Consider noninvasive positive-pressure ventilation (NIPPV); if little or no improvement in respiratory distress or<br />

hypoxemia while on NIPPV after 30-60 minutes, intubation will likely be required.<br />

3. Bronchoscopy is useful in removing large particles and clearing the large airways in large-volume aspirations.<br />

Bronchial irrigation with saline solution, however, should be avoided; it has no beneficial effect and may be<br />

harmful.<br />

4. Antibiotics should be reserved for elderly and chronically ill patients, as well as for those who develop clinical<br />

evidence of infection (fever, purulent sputum, leukocytosis). Bacterial aspiration pneumonia occurs over a<br />

period of several days in >60% of cases of chemical aspiration. Antibiotic therapy is based on the origin of the<br />

infection.<br />

a. Aspiration pneumonitis: occurs immediately after aspiration event: supportive care (airway management,<br />

pulmonary toilet), no role for antibiotics because this is a chemical pneumonitis<br />

b. Community-acquired aspiration pneumonia - empiric treatment with a third- or fourth-generation<br />

cephalosporin or a fluoroquinolone (with antipneumococcal coverage) or a (3-lactam with a (3-lactamase<br />

inhibitor and a macrolide<br />

c. Nosocomial aspiration pneumonia - empiric treatment with piperacillin/clavulanate or a fluoroquinolone<br />

(with antipneumococcal coverage) and clindamycin (If an abscess is present, clindamycin has excellent<br />

penetration.)<br />

5. Supportive care measures<br />

a. Humidified oxygen<br />

b. Bronchodilators<br />

c. Chest physiotherapy<br />

F. Complications<br />

1. Acute respiratory failure<br />

2. Severe sepsis<br />

3. Pneumonia, empyema, and lung abscess<br />

4. Pulmonary fibrosis (for chronic aspiration)<br />

G. Mortality rate<br />

1. Varies with the pH and contamination of the aspirate<br />

2. 40%-70% if the pH is


THORACIC AND RESPIRATORY DISORDERS<br />

B. Etiology: most lung abscesses are polymicrobial and involve either strictly anaerobes or a mixture of<br />

anaerobic organisms.<br />

1 . Anaerobic<br />

a. Fusobacterium<br />

b. Bacteroides<br />

c. Streptococci (microaerophilic and anaerobic)<br />

2. Aerobic<br />

a. Staphylococcus aureus (often follows influenza in a flu epidemic)<br />

b. Streptococcus pneumoniae<br />

c. Alpha streptococci<br />

d. Pseudomonas<br />

e. Klebsiella pneumoniae<br />

f. E coli<br />

g. Proteus<br />

h. Nocardia (particularly in immunocompromised individuals on prolonged course of steroids)<br />

3. Other organisms<br />

a. Mycobacterium<br />

b. Histoplasma<br />

c. Coccidioides<br />

d. Lung flukes and Entamoeba histolytica<br />

C. Clinical presentation<br />

1. Coincident with the development of cavitation (1-2 weeks after aspiration)<br />

2. History<br />

a. Systemic signs and symptoms: weakness, fever, weight loss, night sweats<br />

b. Pulmonary signs and symptoms: dyspnea, chest pain, and a cough productive of a fetid and bloody sputum<br />

3. Physical examination<br />

a. Poor dentition, gingivitis, and foul-smelling breath<br />

b. Signs of localized consolidation or cavitation on auscultation<br />

c. Hemoptysis<br />

D. Diagnostic evaluation<br />

1. CBC: increased WBC count with a left shift and anemia<br />

2. Cavitation with an air-fluid level<br />

a. Chest radiograph or CT scan of the chest<br />

b. Most common sites are the posterior segment of the right upper lobe and superior segment of the left and<br />

right lower lobes.<br />

c. Findings that suggest empyema rather than abscess<br />

(1) An air-fluid level at the site of a previous pleural effusion<br />

(2) A cavity with an air-fluid level that tapers at the pleural border<br />

(3) An air-fluid level that crosses a fissure<br />

(4) An air-fluid level that extends to the lateral chest wall<br />

3. Sputum analysis: because of oropharyngeal contamination, only bronchoscopically obtained sputum,<br />

transtracheal, or transthoracic specimens are reliable for anaerobic culture and sensitivity.<br />

E. Management<br />

1. Antibiotic therapy<br />

a. Clindamycin is currently the antibiotic of choice for an uncomplicated lung abscess. It is given IV until the<br />

patient remains afebrile for 5 days and then is continued orally for 6-8 weeks.<br />

b. Alternative agents include penicillin (when penicillin resistance is not a problem), metronidazole (should<br />

not be used alone because there is a 50% failure rate when used alone), or cefoxitin.<br />

2. Indications for surgery<br />

a. Life-threatening hemoptysis<br />

b. Bronchopleural fistula<br />

262


THORACIC AND RESPIRATORY DISORDERS<br />

c. Tumor or empyema<br />

d. A residual cavity<br />

F. Late complications<br />

1. Chronic lung abscess<br />

2. Empyema<br />

3. Bronchopleural fistula<br />

4. Brain abscess<br />

VI. EMPYEMA<br />

A. Epidemiology<br />

1. A collection of pus in the pleural space or fissures<br />

2. Mechanism of formation<br />

B. Etiology<br />

a. Hematogenous or lymphatic spread from pneumonia<br />

b. Infection from a chest tube, thoracentesis, or thoracotomy<br />

c. Esophageal perforation and mediastinitis<br />

d. Rupture of a mediastinal lymph node<br />

e. Aspiration pneumonia<br />

f. Direct extension from retropharyngeal or subdiaphragmatic abscesses or from vertebral osteomyelitis<br />

1. Mycobacterium tuberculosis<br />

2. Staphylococcus (MRSA)<br />

3. Streptococcus pneumoniae<br />

4. Pseudomonas<br />

5. Klebsiella<br />

6. Gram-negative organisms<br />

7. Anaerobes (difficult to isolate)<br />

C. Clinical presentation<br />

1. Acute illness: fever and chills with pleuritic chest pain, cough, and shortness of breath<br />

2. Subacute illness (more common): weight loss and fatigue<br />

3. Physical examination: decreased breath sounds, egophony, dullness on percussion, and decreased excursion of<br />

the involved hemithorax<br />

D. Diagnostic evaluation<br />

1. Chest radiograph: abnormal pleural-based opacity that does not appear to flow freely on lateral decubitus view<br />

2. Thoracentesis: confirms the diagnosis<br />

E. Clinically useful classifications for empyema<br />

1. American Thoracic Society categorization of empyema by volume<br />

a. Minimal: 10 ml of fluid less than half of hemithorax<br />

c. Large: greater than half of hemithorax<br />

2. Fluid analysis categorization<br />

a. pH


THORACIC AND RESPIRATORY DISORDERS<br />

G. Complications<br />

1. Empyema necessitans (dissection into the subcutaneous tissues or through the chest wall)<br />

2. Bronchopleural fistula<br />

3. Permanent loss of lung tissue<br />

VII. HEMOPTYSIS<br />

A. Definitions<br />

1 . Hemoptysis<br />

a. Coughing up blood originating from the pulmonary parenchyma or tracheobronchial tree<br />

b. Can range from blood streaking of sputum to gross blood with the absence of any accompanying sputum<br />

c. Most often not life threatening<br />

d. However, when massive, hemoptysis can produce airway obstruction as well as hemorrhagic shock and<br />

require urgent intervention.<br />

2. Massive hemoptysis<br />

a. A single expectoration >50 ml or 600 ml blood in 24 hours<br />

b. Necessitates transfusion to maintain stable hematocrit<br />

B. Etiology varies with age of patient<br />

1. Infection/inflammation: most common cause<br />

a. Bronchitis (especially chronic): most common cause<br />

b. Pneumonia (especially Klebsie//a, Staphylococcus, or influenza virus)<br />

c. Parasites (ascariasis, schistosomiasis)<br />

d. Endocarditis<br />

e. Bronchiectasis (most common causes of massive hemoptysis are bronchiectasis, tuberculosis, lung abscess,<br />

and neoplasm)<br />

f. Tuberculosis<br />

g. Lung abscess<br />

h. Aspergilloma<br />

2. Neoplasms (especially bronchogenic cancer)<br />

3. Cardiovascular disorders<br />

a. Mitra! stenosis<br />

b. CHF<br />

c. Pulmonary hypertension (primary) or embolism/infarction<br />

d. AV malformation/fistula<br />

e. Congenital heart disease<br />

f. Thoracic aortic aneurysm<br />

g. Pulmonary embolism<br />

4. Trauma<br />

5. Immunologic disorders<br />

a. Goodpasture syndrome<br />

b. Vasculitis<br />

6. Other<br />

a. Cystic fibrosis<br />

b. Blood dyscrasias<br />

c. Drugs (cocaine-induced pulmonary hemorrhage, ASA, tissue plasminogen activator, ethanol, etc)<br />

d. Coagulopathies<br />

7. Idiopathic (-5%-15% of cases)<br />

8. Iatrogenic<br />

a. Swan-Ganz catheter__,, rupture of pulmonary artery<br />

b. Chronic tracheostomy __,, tracheoinominate fistula<br />

264


THORACIC AND RESPIRATORY DISORDERS<br />

C. Hemoptysis versus hematemesis<br />

1. Need to distinguish from hematemesis or blood swallowed from epistaxis via physical examinations<br />

2. "True" hemoptysis is:<br />

a. Initiated and accompanied by vigorous coughing<br />

b. Generally bright red and foamy and does not contain particles of food<br />

D. Clinical presentation: determined in large part by underlying cause<br />

1. Bacterial pneumonia<br />

2. CHF<br />

a. Sm al I amounts of blood in sputum, productive cough, and fever<br />

b. Examination may reveal rales or rhonchi and signs of consolidation.<br />

a. Small amounts of blood in sputum, shortness of breath<br />

b. Examination may reveal rales, an S3 gallop, jugular venous distention, hepatomegaly, ascites, or peripheral<br />

edema.<br />

3. Bronchogenic carcinoma<br />

a. Generally have chronic cough and weight loss<br />

b. Examination may reveal signs of consolidation and clubbing.<br />

4. Tuberculosis<br />

a. May be massive hemoptysis<br />

b. Patients are often cachectic and have a chronic cough.<br />

c. Examination may reveal post-tussive rales.<br />

E. Diagnostic evaluation<br />

1. The severity and suspected etiology of the hemoptysis determine the extent of the evaluation performed in the<br />

emergency department setting.<br />

2. Chest radiograph<br />

a. Should be obtained in all patients and may reveal signs of underlying pulmonary or cardiovascular disease<br />

b. 50% are normal.<br />

3. CBC: establishes a baseline hematocrit<br />

4. Prothrombin time (INR)/partial thromboplastin time and platelet count: useful in patients who are on<br />

anticoagulants to exclude blood dyscrasias<br />

5. Arterial blood gases: indicated in patients with respiratory distress or massive hemoptysis<br />

6. Type and crossmatch: should be obtained in all patients with massive hemoptysis<br />

7. Sputum: should be sent for Gram and acid-fast stains, culture (for bacteria, fungi, and mycobacteria), and<br />

cytologic examination<br />

8. Bronchoscopy<br />

a. Useful initially because both diagnostic and therapeutic<br />

b. Should be obtained on an emergent basis in patients with massive hemoptysis<br />

9. Selective arteriography<br />

a. Can be used to localize and embolize the site of bleeding in patients with massive hemoptysis<br />

b. Particularly useful when the site is peripheral to the bronchoscope's field of view<br />

10. High-resolution chest CT: usually reserved for patients in whom the chest radiograph is normal. Useful in<br />

detecting bronchiectasis or aspergillomas.<br />

F. Management: determined by the volume of hemoptysis and underlying condition<br />

1. Minimal hemoptysis<br />

a. Treatment is aimed at correcting the underlying problem.<br />

b. For example, patients with pneumonia should receive antibiotics.<br />

c. Specific therapy for the hemoptysis itself is generally unnecessary.<br />

2. Massive hemoptysis<br />

a. Initial therapy is aimed at maintaining the airway, stabilizing the patient, and terminating the bleeding.<br />

b. Intubate the patient: mainstem intubation may be helpful if able to isolate nonbleeding hemithorax.<br />

c. Place the patient in Trendelenburg with the bleeding side down (to protect the uninvolved lung and<br />

maximize gas exchange).<br />

d. Suction as needed.<br />

265


THORACIC AND RESPIRATORY DISORDERS<br />

e. Establish two large-bore IV lines and replace blood loss rapidly.<br />

f. Obtain immediate consult with pulmonologist and thoracic surgeon.<br />

g. Consider interventional radiology early for embolization.<br />

h. Perform bronchoscopy to identify source of bleeding.<br />

i. Once the patient is stabilized and bleeding has been controlled, further evaluation and treatment of the<br />

underlying pathology can be done.<br />

VIII. PNEUMOTHORAX<br />

A. Spontaneous pneumothorax<br />

1. A collection of air in the pleural space (in the absence of trauma)<br />

a. Divided into primary and secondary forms<br />

2. Primary (idiopathic) spontaneous pneumothorax<br />

a. Occurs in healthy individuals who lack evidence of underlying pulmonary pathology, usually without a<br />

precipitating event<br />

b. Results from rupture of a subpleural bleb (often apical) or a weak pleural segment__.. air leaking into the<br />

pleural space__.. pulmonary collapse<br />

c. Causative factors are not always present but may include:<br />

(1) Atmospheric pressure changes (eg, scuba diving, fighter pilots)<br />

(2) Performance of the Valsalva maneuver in association with abuse of marijuana/cocaine or injection into<br />

the central venous system (the "pocket shot") by IV drug users<br />

(3) Marfan syndrome<br />

(4) Cigarette smoking<br />

(5) Family history of pneumothorax<br />

(6) Vigorous exercise<br />

d. Incidence is greatest in young adults 20-40 years old.<br />

e. More common in males than females (male to female ratio is 5:1 ); tall, thin males who smoke are most<br />

commonly affected.<br />

f. Onset is usually during rest or sleep.<br />

g. Recurrence rate is 20%-50% over the following 2-5 years, although most recurrences are seen within the<br />

first year.<br />

3. Secondary spontaneous pneumothorax<br />

a. Occurs in individuals with underlying lung pathology that damages the alveolar/pleural barrier or causes<br />

an increase in intrabronchial pressures<br />

b. Associated medical conditions<br />

(1) Airway disease<br />

(a) COPD (most common cause)<br />

(b) Chronic bronchitis<br />

(c) Asthma<br />

(d) Cystic fibrosis<br />

(2) Infection<br />

(a) Pneumonia, particularly Pneumocystis jerovici pneumonia in AIDS patients, 5 aureus, Klebsie/la,<br />

Pseudomonas, 5 pneumoniae<br />

(b) Tuberculosis (rupture of the TB cavity into the pleural space)<br />

(3) Neoplasms<br />

(4) Interstitial lung disease<br />

(a) Collagen vascular disease<br />

(b) Pneumoconioses<br />

(c) Sarcoidosis<br />

(d) Idiopathic pulmonary fibrosis<br />

(5) Other<br />

(a) Toxic drugs, especially aerosolized pentamidine<br />

266


THORACIC AND RESPIRATORY DISORDERS<br />

(b) Chemical and radiation pneumonitis<br />

(c) Smoking<br />

(d) Endometriosis ("catamenial" pneumothorax)<br />

(e) Trauma (penetrating injury via gunshot wound, stab wound to the chest)<br />

c. Individuals >40 years old are most commonly affected.<br />

4. Iatrogenic pneumothorax: occurs in association with performance of procedures such as:<br />

a. Subclavian vein catheterization and CPR (most common causes in emergency department)<br />

b. Endotracheal intubation (increased frequency with mainstem intubation)<br />

c. lntercostal nerve block<br />

d. Percutaneous lung biopsy<br />

e. Thoracentesis<br />

f. Bronchoscopy (with transbronchial biopsy)<br />

g. CT-guided thoracic procedures<br />

B. Classic clinical scenario<br />

1. The patient presents with sudden onset of pleuritic chest pain and dyspnea. Mild tachycardia and tachypnea<br />

may also be present. Your first thought might be pulmonary embolism, especially if the patient is a middle-aged<br />

woman with risk factors for thromboembolic disease. The chest pain is usually anterior but may radiate into the<br />

neck, back, or ipsilateral shoulder. An associated cough and subcutaneous emphysema of the neck and chest<br />

are occasionally present.<br />

2. On physical examination, the breath sounds will decipher the etiology. Classic physical findings on the<br />

affected side are decreased or absent breath sounds, decreased tactile fremitus, and hyperresonance to<br />

percussion on the affected side. However, these signs may be subtle or absent in patients with COPD or small<br />

pneumothoraces.<br />

3. On ECG, ST segment changes and T-wave inversion may be found, thus mimicking cardiac ischemia.<br />

C. Diagnostic evaluation<br />

1 . Chest radiographs<br />

a. Consider portable AP chest film.<br />

b. Best view to find a small pneumothorax: end-expiratory, upright position<br />

c. Findings on a standard PA chest film (taken in inspiration)<br />

(1) A fine line (the edge of the collapsed lung) running parallel to the chest wall but separated from it by<br />

a space<br />

(2) Absence of lung markings along the lung periphery in the space beyond this line<br />

d. If the standard chest radiograph does not reveal a suspected pneumothorax, a supine film may be helpful<br />

(especially in the hypotensive patient; upright films are not advised in these patients).<br />

(1) In this position, a pneumothorax will collect along the costophrenic sulcus creating a "deep<br />

sulcus sign" (rather than along the apex of the lung in an upright position), thus making a small<br />

pneumothorax more apparent.<br />

(2) Lateral decubitus positioning can reveal as little as 5 ml of pleural gas, which would accumulate in<br />

the nondependent lateral location.<br />

e. Radiologic findings that may be confused with pneumothorax<br />

(1) Skin folds<br />

(2) Tubing outlines<br />

(3) Clothing<br />

(4) Bullae or cysts<br />

2. Ultrasound for diagnosis (curvilinear probe)<br />

a. Loss of pleural sliding<br />

(1) Shallow probe depth<br />

(2) Decrease gain.<br />

(3) Minimize any image processing setting; look for artifacts.<br />

(4) Use zoom setting to view pleural line.<br />

(5) Identify "bat sign" of two ribs with pleural sliding between rib cross-section.<br />

b. Absence of B lines/lung rockets/lung comet tail artifact<br />

267


THORACIC AND RESPIRATORY DISORDERS<br />

c. M-mode demonstrates stratosphere or barcode sign; seashore sign present with normal lung sliding (see<br />

Ultrasound, page 929)<br />

d. Lung point (site at which the visceral and parietal pleura separate) may be visible.<br />

3. If the diagnosis is unclear by conventional chest radiograph, chest CT is the best modality to determine<br />

presence, location, and amount of intrapleural air.<br />

D. Management<br />

1 . Observation and tube thoracostomy<br />

2. Approach selected is determined by the size of the pneumothorax, the degree of symptomatology, the presence<br />

of underlying pulmonary pathology, the rel iabi I ity of the patient, and whether the patient has had prior<br />

pneumothoraces.<br />

3. All patients should be placed on supplemental oxygen, because it hastens resolution of the pneumothorax.<br />

4. Observation (inpatient or outpatient)<br />

a. Acceptable if the pneumothorax is small (25%<br />

(4) Bilateral or tension pneumothorax<br />

(5) Trauma<br />

(6) Significant dyspnea<br />

(7) Detectable pleural fluid<br />

(8) Positive-pressure ventilation (require intubation)<br />

(9) Previous contralateral pneumothorax<br />

c. Chest tube management: presence of a persistent air leak should raise concern for a tracheobronchial<br />

injury, particularly in a trauma patient who presented with a penetrating chest wound.<br />

E. Tension pneumothorax<br />

1. A life-threatening complication<br />

2. Evolution: air enters the pleural space on inspiration but cannot escape on expiration (ball-valve effect) -<br />

progressive accumulation of air within the pleural cavity and total collapse of the affected lung - a shift of<br />

mediastinal structures to the opposite hemithorax - compression of the contralateral lung and impairment of<br />

venous return - decrease in cardiac output and development of signs of shock<br />

3. Classic clinical scenario: The patient is hypotensive, cyanotic, and in severe respiratory distress. Air hunger<br />

develops, and the respiratory rate increases. The patient becomes agitated and restless and displays decreasing<br />

mental activity. The trachea is deviated to the contra lateral side (even significant tracheal deviation may be<br />

difficult to detect clinically), and there is hyperresonance to percussion and absent breath sounds on the<br />

involved side. Jugular venous distention may be present. When the diagnosis is made, or even suspected, the<br />

positive intrapleural pressure must be released immediately; do not wait for radiograph confirmation. Insert<br />

a large-bore needle (14 gauge) anteriorly in the involved hemithorax through the second intercostal space<br />

midclavicular line. This allows time for a tube thoracostomy to be performed.<br />

268


THORACIC AND RESPIRATORY DISORDERS<br />

IX. ASTHMA<br />

A. Definition and epidemiology<br />

1. A chronic lung disorder characterized by:<br />

a. Increased airway responsiveness (an exaggerated bronchoconstriction response) to a variety of stimuli<br />

b. Airway inflammation<br />

c. Reversible airway obstruction<br />

2. Can occur at any age but is more common in children and adolescents; 50% of patients develop asthma before<br />

age 1 0; approximately 80% of children with asthma develop symptoms before age 5.<br />

3. Prevalence is increasing; most common chronic disease of childhood<br />

4. Asthma-related morbidity and mortality rates have been climbing over the last 20 years, especially in children<br />


THORACIC AND RESPIRATORY DISORDERS<br />

C. Pathophysiology<br />

I<br />

Triggers<br />

I<br />

I<br />

Non immunologic<br />

(viral repiratory infections,<br />

oxidant pollutants, chemicals)<br />

Immunologic<br />

(antigen)<br />

I<br />

I<br />

i<br />

Release of cytokines<br />

i<br />

Recruitment of inflammatory cells<br />

(eosinophils, neutrophils, lymphocytes,<br />

macrophages, mast cells) to the airway<br />

i<br />

Release of inflammatory mediators<br />

+<br />

Bronchoconstriction/airway<br />

hyperresponsiveness<br />

+<br />

Mucus hypersecretion<br />

+<br />

Airway edema<br />

+<br />

Increased airway resistance<br />

i<br />

Decrease in maximum expiratory flow rates<br />

(.j, PEFR, t FEV,)<br />

+<br />

Air trapping<br />

i<br />

Increased airway pressure+ possible barotramua<br />

I<br />

I<br />

Asthma Triggers<br />

270


THORACIC AND RESPIRATORY DISORDERS<br />

D. Clinical presentation<br />

1. Symptoms usually awaken the patient at night.<br />

a. Cough<br />

(1) May be dry or productive of sputum<br />

(2) May be the only finding in patients with cough-variant asthma<br />

b. Dyspnea<br />

c. Wheezing<br />

d. Chest tightness<br />

2. Typical examination findings<br />

a. Tachypnea<br />

b. Tachycardia<br />

c. Wheezing: Although the most common manifestation of asthma, wheezing is not an accurate indicator of<br />

the severity of an attack; it may be absent in patients with severe obstruction. Also remember that not all<br />

that wheezes is asthma.<br />

d. Prolonged expiratory phase<br />

e. Hyperresonance to percussion<br />

3 Findings suggestive of severe airway obstruction<br />

a. Dyspnea so severe the patient is able to speak only a few words at a time, and air movement on<br />

auscultation of the lungs is poor.<br />

b. Use of accessory muscles (sternocleidomastoid, scalenus) reflects diaphragmatic fatigue.<br />

c. Heart rate > 120 beats per minute<br />

d. Respiratory rate >30 breaths per minute<br />

e. Pulsus paradoxus 2':12 mm Hg (defined as 2':12 mmHg drop in systolic blood pressure during inspiration)<br />

implies that the FEV 1<br />

is less than half the normal predicted FEV1 for that patient and reflects the degree of<br />

airway obstruction, air-trapping, and ventilatory effort.<br />

f. Silent chest indicates that airflow is dramatically reduced and is no longer adequate to promote wheezing.<br />

g. Diaphoresis<br />

h. Inability to recline on the stretcher<br />

i. Paradoxical respirations herald impending respiratory failure.<br />

j. Cyanosis is a late finding appearing just before respiratory arrest.<br />

k. Altered mental status (confusion, agitation, somnolence) may indicate hypercapnia and often reflects<br />

impending respiratory failure.<br />

E. Differential diagnosis<br />

1. Pulmonary embolus: most of these patients have one or more risk factors for deep venous thrombosis/<br />

pulmonary embolism. Order spiral CT with contrast if suspected (V/Q scans may be indeterminant in patients<br />

with acute asthma); angiography is indicated if the CT is negative, but clinical suspicion is high.<br />

2. CHF (cardiac asthma): distinguishing features include older age, bibasilar rales, an S 3<br />

gallop, pink frothy<br />

sputum, and jugular venous distention; ~-natriuretic hormone assays may be useful in distinguishing CHF from<br />

asthma exacerbation.<br />

3. Upper airway obstruction (cancer, laryngeal edema, foreign body inhalation): remember that these patients<br />

usually have stridor (not wheezing); a patient with wheezing usually has lower airway obstruction. If in doubt,<br />

listen to tracheal breath sounds; any abnormality indicates upper airway obstruction.<br />

4. Carcinoid tumors are typically associated with postprandial flushing and GI upset.<br />

5. Chronic bronchitis or COPD with acute exacerbation: these patients are generally older, have a history of<br />

smoking (along with a chronic productive cough), and no true symptom-free intervals.<br />

6. Eosinophilic pneumonias and invasive worm infestations<br />

7. Endobronchial conditions (cancer, foreign body aspiration, bronchial stenosis): wheezing is unilateral<br />

8. Drugs (~-blockers, ACE inhibitors)<br />

9. Allergic or anaphylactic reactions: urticaria, angioedema, hypotension, and/or GI symptoms are clinical clues.<br />

10. Chemical irritants, insecticides, and anticholinergics: a clue is a history of exposure in an otherwise normal<br />

individual.<br />

11. Noncardiogenic pulmonary edema (ARDS)<br />

271


THORACIC AND RESPIRATORY DISORDERS<br />

F. Diagnostic evaluation<br />

1. Pulmonary function tests (PEFR or FEV 1<br />

): Although recommended to gauge treatment efficacy and severity of<br />

the exacerbation, PEFR and FEV 1<br />

have not proved beneficial or predictive based on multiple studies. However,<br />

they are useful in documenting a trend and serve as a comparison to baseline.<br />

2. CBC<br />

a. Should be measured before and after each treatment with an adrenergic agent<br />

b. The PEFR is the easiest test to perform in the emergency department, because it is done at the bedside with<br />

a portable peak expiratory flow meter.<br />

c. These studies provide an objective measure of the degree of airflow obstruction present; they are useful<br />

in assessing the severity of an attack, the response to treatment, and the need for close follow-up or<br />

admission.<br />

d. Indicators of severe bronchospasm and probable hospital admission<br />

(1) PEFR


THORACIC AND RESPIRATORY DISORDERS<br />

6. Chest radiograph<br />

a. May reveal evidence of hyperinflation (increased AP diameter and/or flattening of the diaphragms),<br />

increased bronchial markings, and/or atelectasis but is generally nondiagnostic<br />

b. Is most useful in excluding the complications of asthma and conditions that mimic it<br />

c. Not obtained routinely but indicated in patients who:<br />

7. ECG<br />

(1) Are febrile<br />

(2) Have focal physical findings suggestive of pneumonia or barotrauma (pneumothorax,<br />

pneumomediastinum)<br />

(3) Do not respond to aggressive treatment<br />

(4) Present with their first episode of wheezing<br />

(5) Persistent localized wheezing<br />

(6) Chronic purulent sputum production<br />

(7) Significant hypoxemia<br />

(8) Presence of hemoptysis<br />

a. Unless ischemia is suspected, routine ECGs are generally not helpful diagnostically.<br />

b. A right ventricular strain pattern is seen in 30%-40% of patients.<br />

8. Theophylline level should be checked in patients taking this medication.<br />

G. Management<br />

1. Pharmacologic agents and their mechanism of action<br />

a. ~i-adrenergic agonists<br />

(1) First-line treatment is typically albuterol.<br />

(2) Promote bronchodilation by increasing cyclic AMP<br />

(3) May also modulate mediator release from mast cells and basophils as well as promote mucociliary<br />

clearance<br />

(4) Primary effect is on the small peripheral airways.<br />

(5) Onset of action is


THORACIC AND RESPIRATORY DISORDERS<br />

(1) Inhaled agents: administer albuterol using a handheld nebulizer.<br />

(a) Albuterol 0.5% solution: the preferred agent because of its B 2<br />

-specificity and fewer adverse effects<br />

(b) Dosage: 2.5 mg in 2-3 ml normal saline every 20-30 minutes up to 3 doses, then hourly<br />

(c) Continuous nebulization at 15 mg/hr for 2 hours is safe and effective in adults with severe<br />

bronchospasm; it should be considered for those patients who present with a PEFR


THORACIC AND RESPIRATORY DISORDERS<br />

(7) Patients who receive steroids in the emergency department and are subsequently discharged should<br />

be continued on oral steroids (methylprednisolone or prednisone 1 mg/kg/day) at home for a<br />

minimum of 3-5 days and a maximum of 14 days. Prolonged therapy with a gradually tapering dose is<br />

recommended for patients taking maintenance doses for> 10 days.<br />

(8) Aerosolized corticosteroids are potentially irritating (can stimulate cough or bronchospasm) and,<br />

therefore, should not be used during an acute attack.<br />

e. Other considerations<br />

(1) Evaluate for evidence of dehydration, and administer replacement fluids (orally or IV) as needed.<br />

(2) Do not administer sedatives, narcotics, or tranquilizers.<br />

(3) Avoid the use of IV isoproterenol; it is highly dysrhythmogenic and carries a risk of myocardial injury<br />

resulting from ischemia or infarction.<br />

(4) Magnesium sulfate should be administered in moderate to severe, nonresponding acute asthma. The<br />

dosage is 2-3 g over 10 minutes and should be given while continuing inhalation therapy.<br />

(5) Administration of helium should be considered for patients with severe asthma. It decreases the work<br />

of breathing and, therefore, allows time for medications that have already been administered to work<br />

(thereby avoiding the need for intubation, which is a problem in some asthmatic patients).<br />

(6) Check serum electrolytes in adult patients who require >3 doses of ~-agonists, because these drugs<br />

can cause hypokalemia, hypomagnesemia, and hypophosphatemia.<br />

(7) Reserve antibiotics for patients with evidence of infection (fever, infiltrate on chest radiograph, evidence<br />

of sinusitis); respiratory tract infections that trigger exacerbations of asthma are usually viral in etiology.<br />

f. Noninvasive positive-pressure ventilation<br />

(1) May be used in patients with respiratory failure (or impending failure) who are not responsive to<br />

therapy but who have normal mentation and facial anatomy<br />

(2) Bi level positive-airway pressure is the modality of choice for patients who are able to cooperate.<br />

(a) Start with an inspiratory pressure of 8 cm H 2<br />

O and an expiratory pressure of 3 cm H 2<br />

O.<br />

(b) Increase settings based on pulse oximetry and arterial blood gas results.<br />

(c) Patients who do not improve over 30-60 minutes require intubation.<br />

g. Intubation with assisted ventilation<br />

(1) Mechanical ventilation in these patients can produce excessively high airway pressures and is<br />

associated with considerable morbidity and mortality.<br />

(2) The initial goal of mechanical ventilation in these patients is not necessarily to restore the pCO 2<br />

to<br />

normal; the respiratory rate and tidal volume required to do this can produce excessive peak inflation<br />

pressures - alveolar gas trapping and overdistention - high risk of barotrauma<br />

(3) These patients should be intentionally hypoventilated (a technique referred to as "controlled<br />

mechanical hypoventilation" or "permissive hypercapnia"). This technique increases the amount of<br />

time available for expiration and thereby minimizes air trapping. Suggested initial ventilator settings<br />

are tidal volume 6-8 ml/kg, respiratory rate 8-10 breaths per minute, and an inspiratory flow rate of<br />

80-1 00 L/min. The moderate hypercapnia that may occur with these ventilator settings is wel I tolerated<br />

and less dangerous than persistently high airway pressures.<br />

(4) Volume-cycled ventilators with higher peak pressures and flow rates should be used initially to deliver<br />

an adequate tidal volume. Plateau pressure is most important in minimizing barotrauma. Aim for<br />

plateau pressure of


THORACIC AND RESPIRATORY DISORDERS<br />

(c) Succinylcholine is the paralytic of choice. Pancuronium is useful when muscle paralysis is<br />

required to facilitate ventilation; it does not cause histamine release.<br />

(d) Benzodiazepines may be used when sedation is needed (it will minimize emergence of<br />

hallucinations if ketamine was used); opiates induce histamine release and should be avoided.<br />

(e)<br />

Heliox should be considered for mechanically ventilated asthmatic patients with significant respiratory<br />

acidosis and markedly increased airway pressures; it lowers airway resistance and decreases the work<br />

of breathing. A helium-oxygen mixture of 80:20 or 70:30 (preferred) should be used.<br />

(7) If an asthmatic patient who is intubated becomes hypotensive, hypoxic with high plateau pressures<br />

alarming on the ventilator, detach the patient from the ventilator and help with exhalation because<br />

the patient may have breath stacking and increased his or her intrathoracic pressure causing<br />

decreased preload and hypotension. When returning the patient to the ventilator, ensure an l:E ratio<br />

of 1 :3 to allow adequate time for exhalation.<br />

h. Asthma in pregnancy<br />

(1) Incidence of asthma can increase in pregnancy: one-third of patients with asthma get worse with<br />

pregnancy, one-third have no change, and one-third may get better.<br />

(2) Respiratory alkalosis (pC0 2<br />

of 30-35) is a normal finding in pregnancy. Do not be concerned about<br />

respiratory compromise unless the pC0 2<br />

is >35.<br />

(3) Management of asthma in pregnant patients is similar to that in nonpregnant patients. The goal of<br />

treatment is to ensure adequate oxygenation for both mother and fetus while avoiding (whenever<br />

possible) any drugs that may pose a risk to the fetus.<br />

(a) All patients should receive supplemental oxygen (maternal hypoxia - impaired fetal oxygenation<br />

-s- fetal complications). Place the pregnant patient on her left side to minimize compression of the<br />

inferior vena cava.<br />

(b) Inhaled ~-agonists are the first-line drugs of choice. If parenteral therapy is required, terbutaline<br />

is preferred over epinephrine; the use of SC epinephrine early in pregnancy is associated with an<br />

increased incidence of congenital malformations, and in late pregnancy with premature labor<br />

(and should be avoided).<br />

(c) Corticosteroids are safe in pregnancy and should be administered in all but the mildest<br />

exacerbations.<br />

(d) Anticholinergic agents and theophylline may also be used. The clearance of theophylline in the<br />

third trimester, however, is reduced; serum levels must be carefully monitored.<br />

(e) Asthma medications that are appropriate for administration during pregnancy are also safe for use<br />

during lactation.<br />

X. CHRONICOBSTRUCTIVE PULMONARY DISEASE (COPD)<br />

A. Definition and pathophysiology<br />

1. Often referred to as a single disease but is actually a triad of three distinct disease processes<br />

a. Chronic bronchitis: defined clinically as a productive cough for 3 months over 2 consecutive years,<br />

characterized by airway hypersecretion and inflammation that causes a chronic productive cough for an<br />

extended period of time<br />

b. Emphysema: defined anatomically and characterized by irreversible enlargement of the air spaces distal to<br />

the bronchioles<br />

c. Asthma: characterized by airway hyperreactivity and inflammation. In most COPD patients, these three<br />

entities coexist, but the contribution each makes to pulmonary dysfunction varies from individual to<br />

individual. However, some COPD patients present with predominately emphysema (pink puffers) or<br />

chronic bronchitis (blue bloaters).<br />

2. Evolution<br />

a. COPD begins in the early twenties and is detected only by pulmonary function testing. Early pathologic<br />

changes are completely silent.<br />

b. As the disease progresses, signs and symptoms appear with a full-blown clinical picture as early as in the<br />

thirties in some patients.<br />

c. Chronic bronchitis ensues with a typical history of chronic, recurrent production of excessive mucus.<br />

d. Repeated inflammation leads to increased resistance or decreased caliber of the small bronchi and<br />

bronchioles. As alveolar hypoventilation subsequently occurs, hypoxemia and hypercarbia result.<br />

276


THORACIC AND RESPIRATORY DISORDERS<br />

Ventilation-perfusion mismatching occurs (thus promoting hypoxemia), while increased physiologic dead<br />

space ventilation leads to alveolar hypoventilation, hypercarbia, and further hypoxemia.<br />

e. In addition to obstruction of the peripheral airways, destruction and coalescence of the alveolar cell<br />

structure (particularly in dominantly emphysematous disease) results in reduction of the total "matched"<br />

alveolar-capillary surface area for diffusion of gases (oxygen cannot get in, CO 2<br />

cannot get out).<br />

f. Later on, chronic and progressive airflow obstruction can lead to right-sided cardiac strain that results in<br />

pulmonary hypertension (loud P2 on auscultation) and cor pulmonale.<br />

3. Fourth most common cause of death<br />

B. Risk factors<br />

1. Smoking (most important factor)<br />

2. Exposure to second-hand smoke<br />

3. Environmental pollution<br />

4. Industrial or occupational exposure<br />

5. a 1<br />

-antitrypsin deficiency<br />

6. Cystic fibrosis<br />

7. Recurrent pulmonary infections<br />

C. Etiology of acute decompensation<br />

1. Superimposed respiratory infection (acute bronchitis, pneumonia): the most common precipitating factor of<br />

acute COPD exacerbation. Influenza and pneumococcal vaccination recommended.<br />

2. Changes in ambient temperature, humidity, or air pollution levels<br />

3. Noxious environmental exposures<br />

4. Spontaneous pneumothorax<br />

5. Pulmonary embolus<br />

6. Noncompliance with (or underdosing of) medications<br />

7. Inappropriate treatment (~-blockers, sedatives)<br />

8. Acute CHF<br />

9. Continued cigarette smoking<br />

D. Clinical presentation in <strong>combined</strong> disease<br />

1. Dyspnea is the most common complaint, followed by cough (occasionally with hemoptysis), chest tightness,<br />

and fatigue. A history of morning headache may be due to a rising PaC0 2<br />

(hypercapnia).<br />

2. Physical findings<br />

a. Tachypnea<br />

b. Prolonged expiratory phase<br />

c. Increased AP chest diameter ("barrel chested")<br />

d. Decreased breath sounds<br />

e. Wheezing, rales, and rhonchi<br />

f. Lip-pursing, accessory muscle use, cyanosis, and diaphoresis<br />

g. Pulsus paradoxus<br />

h. Distant heart sounds, a loud P 2<br />

, and right-sided CHF<br />

E. Clinical presentation when emphysema or bronchitis predominates<br />

1. Emphysema ("pink puffer")<br />

a. Dyspneic<br />

b. Thin, anxious, alert appearance<br />

c. Increased AP chest diameter (overinflation)<br />

d. Tachypneic and hypotensive<br />

e. Uses accessory muscles to breathe<br />

f. Decreased breath sounds, faint end-expiratory rhonchi, and hyperresonance on percussion<br />

2. Chronic bronchitis (blue bloater)<br />

a. Productive "wet" cough<br />

b. Stocky build, polycythemic, and cyanotic<br />

c. Normal chest diameter<br />

d. Little air-hunger<br />

277


THORACIC AND RESPIRATORY DISORDERS<br />

e. Subxiphoid or retrosternal heave _,. right ventricular hypertrophy<br />

f. Signs of CHF (jugular venous distention, S 3<br />

or S 4<br />

gallop, murmur of tricuspid insufficiency, scattered rales and<br />

rhonchi, edema)<br />

F. Diagnostic evaluation<br />

1. Chest radiograph<br />

a. Most useful in excluding other disease processes or complications (pneumonia, pneumothorax, atelectasis,<br />

effusions, cancer, CHF): > 15% of acutely decompensated COPD patients have treatable findings.<br />

b. Signs of hyperinflation are common (60% of patients)<br />

(1) Increased AP diameter<br />

(2) Increased retrosternal airspace<br />

(3) Increased parenchymal I ucency<br />

(4) Low and flattened diaphragm<br />

(5) Long and narrow cardiac silhouette (unless chronic bronchitis predominates the clinical picture, in<br />

which case, an enlarged right ventricle is seen on the lateral film)<br />

c. Bullae may also be seen.<br />

2. Arterial blood gases<br />

a. Provide information regarding the seriousness and acuteness of airway compromise in the COPD patient<br />

b. PaO 2<br />

is generally low.<br />

(1) Hypoxemia is common in these patients and worsens as the disease progresses, as well as with acute<br />

exacerbations.<br />

(2) It is due to ventilation-perfusion mismatching and can usually be corrected by increasing the amount<br />

of oxygen being inspired.<br />

c. PaCO 2<br />

(1) Reflects the adequacy of ventilation<br />

(2) Normocarbia may be present early on, but hypercarbia develops with disease progression and may<br />

worsen with acute exacerbations.<br />

(3) A rapid rise in the PaCO 2<br />

decreases the pH, whereas with a more gradual rise, the kidneys are able to<br />

compensate by retaining bicarbonate, which normalizes the pH. Therefore:<br />

(a) An increased PaCO 2 , a normal pH, and an increased bicarbonate level suggest chronic CO 2<br />

retention.<br />

(b) An increased PaCO 2<br />

, a decreased pH, and an increased bicarbonate level point to acute<br />

respiratory failure superimposed on chronic respiratory insufficiency.<br />

3. Pulmonary function tests<br />

a. More useful in patients with asthmatic bronchitis than with COPD in evaluating the degree of airway<br />

obstruction present and in monitoring the response to treatment.<br />

b. Either the FEV 1<br />

(or the PEFR) may be used; both provide a measurable index of airway obstruction.<br />

c. Because bronchospasm is not the major component of lung dysfunction in COPD patients, the response to<br />

treatment is less than that of asthmatics.<br />

4. ECG<br />

d. Lung volume measurements in COPD patients reveal an increase in the amount of dead space and a<br />

decrease in the vital capacity; clinical signs develop when the vital capacity drops 50%.<br />

a. Detects ischemic disease or dysrhythmias associated with COPD (especially multifocal atrial tachycardia<br />

and atrial fibrillation, both of which are common and should be assumed to be secondary to CHF) and may<br />

also demonstrate evidence of COPD or cor pulmonale<br />

(1) P pulmonale (peaked P waves in II, Ill, and AVF) _,. very suggestive of COPD<br />

(2) Low voltage, right axis deviation and poor R wave progression _,. also very suggestive of COPD<br />

(3) Criteria for right ventricular hypertrophy_,. very suggestive of cor pulmonale<br />

b. Troponin elevation is relatively common in COPD patients with acute exacerbations and is not usually<br />

associated with a codiagnosis of acute coronary syndrome.<br />

G. Management<br />

1. Directed toward improving oxygenation and respiratory function<br />

2. Oxygen is the most important therapy.<br />

278


THORACIC AND RESPIRATORY DISORDERS<br />

a. COPD patients who chronically retain CO 2 depend on their hypoxic respiratory drive to breathe. Raising<br />

the p0 2<br />

too quickly in these patients depresses the respiratory center and shuts off this drive. Concern over<br />

suppressing this drive must be balanced against the need to increase the p0 2<br />

to a satisfactory level.<br />

b. To minimize the risk, begin treatment using low concentrations of oxygen and a controlled delivery system<br />

such as a Venturi mask. Start with an Fi0 2<br />

of 24%-28%, and increase it as needed. The goal is to raise the<br />

p0 2<br />

to >60 and the saturation to >88%.<br />

c. Continuous pulse oximetry (supplemented with arterial blood gases when there is concern that the pC0 2<br />

is rising) should be used to monitor response to treatment and guide adjustments in the Fi0 2<br />

; continuous<br />

capnometry can also be useful.<br />

d. Assurance of adequate oxygenation should take precedence over concern about suppressing the hypoxic<br />

respiratory drive; if a high concentration of oxygen is needed, it should be administered.<br />

e. Long-term 0 2<br />

therapy for patients with PaO 2<br />


THORACIC AND RESPIRATORY DISORDERS<br />

e. Dosage guidelines for IV aminophylline<br />

(1) Loading dosage: 5 mg/kg (ideal body weight) over 10-15 minutes<br />

(2) Maintenance dosage: based on ideal body weight and associated conditions<br />

(a) Adult smokers


THORACIC AND RESPIRATORY DISORDERS<br />

c. Toxic inhalation (smoke, corrosives, paraquat, oxygen)<br />

d. Liquid aspiration (gastric contents, fresh- or saltwater)<br />

e. Pneumonia (bacterial or viral)<br />

f. Emboli (pulmonary, fat, air, amniotic fluid)<br />

g. Major trauma (burns, pulmonary contusion, multiple injuries)<br />

h. Transfusion-related acute lung injury<br />

1. Pancreatitis<br />

j. Acute neurologic crisis (head injury, subarachnoid hemorrhage, stroke, brain tumor)<br />

k. Cancer therapy (tumor lysis, radiation)<br />

B. Pathophysiology<br />

Inflammatory<br />

response<br />

Fluid flux into<br />

the lung+ t lung<br />

compliance<br />

This process is frequently<br />

irreversible if the inciting<br />

event is not controlled.<br />

Microvascular<br />

membrane damage<br />

t permeability<br />

of the microvascular<br />

membrane<br />

Pathophysiology of ARDS<br />

1. The inflammatory response results from activation of enzyme systems (which is caused by the inciting event)<br />

and involves a variety of biochemical mediators that typify an inflammatory response.<br />

2. In many disorders associated with permeability pulmonary edema, the activation of enzyme cascades leads<br />

to microvascular membrane damage and increased permeability of the microvascular membrane. There are<br />

primarily two enzyme cascades that are activated:<br />

a. The complement system is triggered by:<br />

(1) Antigen-antibody complexes (the classic pathway)<br />

(2) Endotoxins<br />

(3) Exposure to cell surfaces by bacteria or fungi<br />

(4) Complex polysaccharides<br />

b. The coagulation pathways are activated when collagen in exposed basement membrane is exposed to<br />

plasma. This causes activation of Hageman factor (factor XII) which, in turn, activates components of the<br />

coagulation system, the fibrinolytic system, and the kinin generation system.<br />

3. In addition, metabolism of arachidonic acid through major biochemical pathways leads to the release of a<br />

variety of mediators (including a host of leukotrienes and prostaglandins) that contribute to further endothelial<br />

and alveolar injury.<br />

4. Neutropenic patients may also develop permeability pulmonary edema. Injury to the pulmonary endothelium<br />

results from one of the following:<br />

a. Direct effects of bacterial endotoxin and complement<br />

b. Oxygen toxicity<br />

c. Release of prostaglandins and leukotrienes from alveolar macrophages<br />

5. Once the microvascular membrane is damaged and its permeability is increased, it is no longer an effective<br />

barrier to protein flux. Any subsequent increase in hydrostatic pressure then accelerates fluid flux into the lung.<br />

6. The mechanical properties of the lung are also adversely affected: surfactant is reduced or inactivated___,.<br />

collapse of alveoli___,. stiff lung (which requires greater pressure to inflate it)___,. t lung compliance<br />

281


THORACIC AND RESPIRATORY DISORDERS<br />

C. Diagnostic evaluation<br />

1. Diagnostic criteria<br />

a. Symptom onset is within 1 week of the clinical insult.<br />

b. Chest radiograph: diffuse bilateral interstitial and alveolar infiltrates with a normal-sized heart consistent<br />

with pulmonary edema (noncardiogenic pulmonary edema)<br />

c. Respiratory failure not fully attributed to heart failure or volume overload.<br />

d. Oxygen impairment as measured by Pa0/Fi0 2<br />

with PEEP setting >5.<br />

(1) Mild: PaO/Fi0 2<br />

>200 but 100 but


THORACIC AND RESPIRATORY DISORDERS<br />

B. Clinical importance<br />

1. Significant morbidity and mortality are associated with pulmonary embolism in the United States, and<br />

progressive right ventricular failure is the most common cause of death in these patients (usually secondary to<br />

massive embolization).<br />

2. The diagnosis is missed antemortem in up to 70% of cases (>400 1<br />

000/year); one-third of these patients die,<br />

while the other two-thirds are at risk of recurrent pulmonary embolism and development of pulmonary<br />

hypertension.<br />

3. A significant percentage of patients with DVT develop pulmonary embolism, and many of these cases go<br />

unrecognized. The so-called 11 clinically silent 11<br />

calf vein thrombus can be deadly. In fact, clinically significant<br />

embolization from calf DVT is not uncommon.<br />

C. Risk factors<br />

1. All known risk factors for pulmonary embolism (and DVT) have their basis in Virchow's triad of venostasis 1<br />

hypercoagulability, and vessel wall injury or abnormality. (Most common risk factors in patients with proven<br />

pulmonary embolism are indicated below with an asterisk.)<br />

a. Current DVT* (high risk of occult cancer)<br />

b. Obesity*<br />

c. Prior DVT or pulmonary embolism*<br />

d. Immobility (including travel) or prolonged bedrest*<br />

e. Recent trauma (including burns) or surgery (especially orthopedic)*<br />

f. Carcinoma* or chemotherapy<br />

g. Autoimmune (eg 1<br />

systemic lupus erythematosus) and immune (eg 1<br />

AIDS) disorders<br />

h. MI/CHF/COPD/stroke<br />

i. Estrogen therapy<br />

j. Hypercoagulable settings (use of oral contraceptives, pregnancy, postpartum period)<br />

k. Inherited abnormalities of coagulation or fibrinolysis<br />

I. Indwelling central venous catheter<br />

m. IV drug abuse<br />

n. Polycythemia vera<br />

2. Most patients (c:90%) with thromboembolic disease have at least one of these risk factors, although some<br />

(those with cancer, inherited abnormalities of coagulation) may not be apparent at the time of presentation.<br />

Furthermore, the risk represented by these factors is additive: the greater the number of risk factors present,<br />

the greater the risk of pulmonary embolism. Thus, knowledge of whether or not a patient has risk factors for<br />

pulmonary embolism can be used to increase or decrease clinical suspicion of the diagnosis. For example: A<br />

patient with at least one common risk factor or two or more other risk factors should be considered to have a<br />

moderate to high risk of having a pulmonary embolism.<br />

D. Clinical presentation<br />

1. Presenting signs and symptoms of acute pulmonary embolism are determined by the extent of pulmonary<br />

vascular occlusion (massive versus submassive) and the patient's baseline cardiopulmonary status.<br />

2. History<br />

a. Dyspnea (the most common symptom by far)<br />

b. Pleuritic chest pain<br />

c. Apprehension<br />

d. Cough<br />

e. Hemoptysis<br />

f. Sweating<br />

g. Nonpleuritic chest pain<br />

h. Syncope (more common with massive than submassive emboli)<br />

3. Physical examination findings<br />

a. Tachypnea<br />

b. Rales<br />

c. Accentuated S 2<br />

d. Tachycardia (heart rate > 100 beats per minute)<br />

e. Diaphoresis<br />

283


THORACIC AND RESPIRATORY DISORDERS<br />

f. 5 3<br />

or 5 4<br />

gallop<br />

g. Clinically evident thrombophlebitis<br />

h. Decreased breath sounds<br />

i. Lower-extremity edema<br />

j. Cardiac murmur<br />

k. Cyanosis<br />

I. Hypotension (more common with massive emboli)<br />

4. Dyspnea, pleuritic chest pain, or tachypnea is present in 95% of patients; if all three of these findings are<br />

absent, pulmonary embolism is unlikely (especially if there are no risk factors). The "classic triad" of dyspnea,<br />

pleuritic chest pain, and hemoptysis is uncommon (seen in 100 beats per minute<br />

Immobilization (or surgery) within 4 weeks<br />

History of previous DVT or pulmonary embolism<br />

History of hemoptysis<br />

History of malignancy (treatment within 6 months or palliative)<br />

3.0<br />

3.0<br />

1.5<br />

1.5<br />

1.5<br />

1.0<br />

1.0<br />

Total score interpretation<br />

>6 = high pretest probability (66.7% risk)<br />

2-6 = moderate pretest probability (20.5% risk)<br />


THORACIC AND RESPIRATORY DISORDERS<br />

(3) The A-a gradient is abnormally increased in 95% of patients with pulmonary embolism.<br />

(4) The finding of an increased A-a gradient increases the probability a pulmonary embolism is present,<br />

but a normal gradient does not exclude the diagnosis; therefore, it should not be used as a "screening"<br />

test for pulmonary embolism.<br />

3. Chest radiograph<br />

4. ECG<br />

a. Useful in excluding other disease processes and also needed to interpret the ventilation/perfusion scan<br />

b. Abnormal in >80% of patients with pulmonary embolism, but the findings are neither sensitive nor specific<br />

c. Suspected disease processes in pulmonary embolism<br />

(1) Atelectasis or pulmonary parenchymal abnormalities (consolidation or patchy infiltrates)<br />

(2) Elevated hemidiaphragm<br />

(3) Pleural effusion<br />

(4) Hampton's hump: a triangular pleural-based density with a rounded apex that points toward the<br />

hilum<br />

(5) Westermark sign: dilatation of pulmonary vessels proximal to the embolus in association with<br />

regional oligemia distally<br />

d. Most common radiographic abnormalities are atelectasis, pulmonary parenchymal abnormalities, an<br />

elevated hemidiaphragm, and small unexplained pleural effusions.<br />

e. Hampton hump and Westermark sign are rare but, when present, are very suggestive of pulmonary<br />

embolism.<br />

f. The presence of a normal chest radiograph (present in


THORACIC AND RESPIRATORY DISORDERS<br />

Therefore, V/Q scans are classified as normal or near-normal; indeterminate (most common reading);<br />

or low, intermediate, or high probability of pulmonary embolism based on the number and size ofV/Q<br />

mismatches present.<br />

d. The V/Q scan result should be interpreted in the context of clinical suspicion of pulmonary embolism,<br />

because it increases the predictive value of this test (especially if there is a level of clinical suspicion of<br />

pulmonary embolism before obtaining the V/Q scan results).<br />

e. In patients with low to moderate pretest probability, a normal V/Q scan excludes clinically significant<br />

pulmonary embolism 98% of the time. A normal perfusion scan also decreases the probability that an<br />

angiographically provable pulmonary embolism is present, at least to the point at which further evaluation<br />

is indicated only in the presence of high clinical suspicion.<br />

f. An indeterminate or nondiagnostic (low or intermediate probability) V/Q scan neither confirms nor<br />

excludes the diagnosis of pulmonary embolism and should not be taken as a diagnostic end point.<br />

g. A high probability scan in a patient with high pretest clinical probability for pulmonary embolism is<br />

considered confirmatory and should result in treatment; no further evaluation is required.<br />

h. Advantages<br />

(1) Relatively noninvasive<br />

(2) Can be performed in critically ill patients<br />

(3) Safer for pregnant patients (in whom, after duplex Doppler, it is considered the diagnostic test of<br />

choice; it is less invasive and has less fetal exposure than angiography)<br />

i. Disadvantages<br />

(1) Nondiagnostic most of the time<br />

(2) Less useful in patients with lung disease (or abnormal chest radiograph)<br />

(3) Takes time to perform<br />

(4) Requires isotope injection<br />

2. CT angiography (CTA)<br />

a. Most extensively studied new confirmatory test with a good overall test performance that is equal to or<br />

slightly better than that of the V/Q scan. However, a CTA scan read as negative for pulmonary embolism<br />

does not have as high a negative predictive value as a V IQ scan read as normal; therefore, a negative CTA<br />

in a patient with high probability for pulmonary embolism requires pulmonary angiography.<br />

b. In settings in which a V/Q scan reading is indeterminate (or likely to be so in patients with pulmonary<br />

pathology (eg, COPD, pneumonia, etc), a CTA with contrast is the procedure of choice.<br />

c. An intraluminal filling defect or vascular occlusion characterizes a positive result; the sensitivity and<br />

specificity of CTA are higher for pulmonary embolism in the central vessels than in the peripheral vessels<br />

(95% sensitivity for segmental or large pulmonary embolism, 75% sensitivity for subsegmental pulmonary<br />

embolism). Newer generation CTA (thin collimation spiral CT) may increase diagnostic accuracy.<br />

d. Advantages overV/Q scanning<br />

(1) Can be performed within minutes<br />

(2) Marked reduction of indeterminate readings; useful for its ability to identify a convincing diagnostic<br />

alternative to pulmonary embolism<br />

e. Disadvantages<br />

(1) Cost<br />

(2) Need for patient transport<br />

(3) Use of iodinated contrast material<br />

(4) Radiation exposure<br />

3. Pulmonary angiography<br />

a. Gold standard for diagnosing pulmonary embolism; although false-positive and false-negative results do<br />

occur occasionally, angiography offers the best available combination of sensitivity and specificity for<br />

diagnosing thromboembolic disease.<br />

(1) Current techniques are unable to detect obstruction in small, subsegmental arterial branches, so many<br />

small peripheral emboli may be missed.<br />

(2) Augmentation techniques (subselective injections, magnification, radiography, balloon occlusion<br />

angiography) are able to demonstrate emboli as small as 1 mm in diameter.<br />

(3) Standard angiography (without augmentation techniques) may completely miss emboli as large as 2.5<br />

mm in diameter.<br />

286


THORACIC AND RESPIRATORY DISORDERS<br />

b. A positive study demonstrates an embolus obstructing a vessel (a dye "cut-off" sign) or an intraluminal<br />

filling defect (in more than one projection).<br />

c. Advantage: currently the most reliable test available for diagnosing pulmonary embolism<br />

d. Disadvantages<br />

(1) Invasive<br />

(2) Not universally available<br />

(3) Mortality of 0.1 %-0.5% and morbidity of 1 %-5%<br />

(4) Relatively contraindicated in patients with pulmonary hypertension<br />

e. Complications<br />

(1) More common in older patients with underlying cardiopulmonary disease<br />

(2) Include anaphylactoid reactions, dysrhythmias, cardiac arrest, endocardial injury, and perforation<br />

4. Color-flow duplex ultrasonography<br />

a. In patients with nondiagnostic V/Q scans, this is the next study of choice in excluding pulmonary<br />

embolism, because a lower-extremity DVT confirmed by duplex ultrasound is considered de facto evidence<br />

that pulmonary embolism also exists.<br />

b. Demonstration of a DVT in the setting of suspected pulmonary embolism is an indication to start<br />

appropriate therapy.<br />

c. Negative lower-extremity vascular studies cannot eliminate the possibility of pulmonary embolism and,<br />

therefore, do not change the clinical management. A single negative lower-extremity ultrasound duplex<br />

study should not be used to exclude pulmonary embolism in patients with moderate to high pretest<br />

probability and a nondiagnostic V/Q scan. (ACEP Clinical Policy).<br />

d. In pregnant patients, duplex Doppler is the initial diagnostic study of choice for the evaluation of<br />

pulmonary embolism, because it is the least invasive. In these patients, duplex Doppler is performed first; if<br />

positive, treatment is started, and if negative, then a V/Q or spiral CTA scan is performed.<br />

5. Echocardiography<br />

a. Should be considered for patients with shock of uncertain cause in whom the diagnosis of pulmonary<br />

embolism is suspected but who is not stable enough for other diagnostic tests<br />

b. Right ventricular dysfunction in the absence of an acute Ml or pericardia! tamponade supports the<br />

diagnosis of right ventricular failure secondary to pulmonary embolism.<br />

c. Bedside techniques<br />

(1) Transthoracic echocardiography rarely provides image resolution adequate enough to visualize the<br />

inferior vena cava or cardiac chambers, but it can detect right ventricular strain.<br />

(2) Transesophageal echocardiography has more consistent cardiac imaging, but it is invasive and requires<br />

specialty consult and equipment.<br />

6. Point-of-care ultrasound<br />

a. Useful in unstable patients when comprehensive echocardiography unavailable or delayed<br />

b. Pseudonormalization of chamber sizes (1 :1 ratio); loss of normal right ventricular to left ventricular ratio of<br />

0.6:1 .0<br />

c. McConnell sign: akinesia of mid free wall with normal movement of cardiac apex<br />

d. Right ventricular thrombus may be seen in 4% of patients with pulmonary embolism.<br />

G. Diagnostic approach<br />

1. The clinical presentation, history, risk factors, and physical examination findings lead to clinical suspicion.<br />

2. Diagnostic evaluation proceeds to include an ECG and chest radiograph, which may indicate another diagnosis<br />

or provide additional evidence in support of the diagnosis of pulmonary embolism.<br />

3. If these tests do not reveal another diagnosis (pneumothorax, Ml) that explains the patient's signs and<br />

symptoms, then the patient's pretest probability for pulmonary embolism should be determined and a screening<br />

test performed.<br />

4. Further management decisions should incorporate assessment of the patient's pretest probability of having a<br />

pulmonary embolism and the test results (screening and diagnostic).<br />

5. If CTA reveals pulmonary embolism, the patient should be treated.<br />

6. If CTA is negative and the study was technically adequate, another diagnosis should be considered.<br />

287


THORACIC AND RESPIRATORY DISORDERS<br />

Clinical suspicion for<br />

pulmonary embolism<br />

t " t<br />

Low Intermediate High<br />

( 2% but 40%)<br />

i<br />

I<br />

No further testing for<br />

pulmonary embolism<br />

Consider alternative<br />

diagnosis<br />

D-dimer<br />

t<br />

I<br />

♦<br />

Negative<br />

•<br />

Consider<br />

alternative diagnosis<br />

Positive<br />

I -<br />

t<br />

Allergy to IV dye*, increased<br />

creatinine, pregnant<br />

•<br />

I<br />

•<br />

Yes<br />

No<br />

+ +<br />

V/Q scan<br />

CT angiography<br />

I<br />

I<br />

♦ ♦ t<br />

•<br />

t<br />

Normal<br />

Low or<br />

intermediate<br />

High Positive Negative<br />

• • • •<br />

t<br />

Consider<br />

alternative<br />

diagnosis<br />

CT angiography<br />

or<br />

Treat<br />

presumptively<br />

and admit<br />

Consider<br />

Treat Treat alternative<br />

diagnosis<br />

*Premedication may be necessary in some clinical scenarios to obtain CT pulmonary embolism.<br />

Evaluation of Pulmonary Embolism<br />

288


THORACIC AND RESPIRATORY DISORDERS<br />

H. Management<br />

1 . Objectives<br />

a. Short term: prevent thromboembolic propagation and additional embolic events and eliminate<br />

thromboemboli from the pulmonary vasculature<br />

b. Long term: prevent or minimize recurrence of pulmonary embolism, chronic venous insufficiency, and<br />

development of chronic pulmonary hypertension<br />

2. Basic supportive measures<br />

a. Assess and stabilize the ABCs: establish an IV of normal saline or lactated Ringer's, provide supplemental<br />

oxygen, place the patient on a cardiac monitor, and check a rhythm strip.<br />

b. If patients are hypotensive, any fluid resuscitation should be performed with extreme care. Volume loading<br />

can worsen right ventricular function and can actually worsen blood pressure. If blood pressure or clinical<br />

status worsens with fluids, immediately switch to vasopressors (eg, norepinephrine, dopamine) for blood<br />

pressure support.<br />

3. Anticoagulation with IV unfractionated heparin or low-molecular-weight heparin (LMWH) given SC<br />

a. The cornerstone of therapy for pulmonary embolism, heparinization should be started as soon as the<br />

diagnosis is strongly suspected (unless contraindications exist); do not wait for confirmatory test results.<br />

b. The decision to use unfractionated heparin or a LMWH is an arbitrary one, because both appear to be<br />

equally effective and safe for the initial treatment of submassive pulmonary embolism. The choice is often<br />

made by the admitting/consulting cardiologist.<br />

(1) Because pulmonary embolism is not (generally) treated on an outpatient basis, some cardiologists still<br />

prefer IV unfractionated heparin, because the anticoagulation effect can be monitored more accurately<br />

and the cost is less than that of an LMWH.<br />

(2) Others prefer using an LMWH given SC until the patient can be anticoagulated on warfarin. Again,<br />

better bioavailability, fewer blood draws for laboratory studies, and no need for a continuous infusion<br />

pump are reasons for this choice.<br />

c. Heparin prevents propagation of the clot and decreases the risk of further embolic events but does not<br />

dissolve clots that are already present. It works by binding to (and enhancing) the activity of antithrombin<br />

Ill, a naturally occurring substance that prevents thrombosis by inhibiting activated coagulation factors of<br />

the intrinsic and common pathways, particularly thrombin and Factor Xa.<br />

d. If unfractionated heparin is the agent selected, administer a bolus of 100 units/kg IV (7,000 units/70-<br />

kg patient) followed by a continuous infusion of 18 units/kg/hr. Check the partial thromboplastin time<br />

(PTT) in 6 hours, and adjust the infusion rate as needed to maintain it at 1.5-2.5 times the control value.<br />

This weight-adjusted heparin regimen minimizes the time needed to achieve a therapeutic PTT without<br />

increasing the likelihood of bleeding complications.<br />

e. Enoxaparin is the LMWH of choice and is rapidly becoming the therapeutic method preferred by many<br />

clinicians; the dosage is 1 mg/kg SC every 12 hours or 1 .5 mg/kg/day SC.<br />

f. Both unfractionated heparin and LMWHs are safe to use in pregnancy, because neither crosses the<br />

placenta. Do not give warfarin in pregnancy!<br />

g. Complications<br />

(1) Hemorrhage can be reversed by stopping the infusion and administering protamine sulfate (1 mg<br />

neutralizes 100 units of heparin but not enoxaparin or other LMWHs).<br />

(2) Thrombocytopenia requires discontinuation of all forms of heparin. It is immune-mediated and<br />

generally develops 7-1 0 days after therapy has started.<br />

h. LMWHs have a number of advantages over unfractionated heparin; when these agents become more<br />

widely used for treatment of pulmonary embolism and DVT, they are likely to replace current anticoagulant<br />

therapy with unfractionated heparin.<br />

4. Thrombolytic (fibrinolytic) therapy<br />

a. Current indications (ACEP Clinical Policy: Critical Issues in the Evaluation and Management of Adult<br />

Patients Presenting with Suspected Pulmonary Embolism)<br />

(1) Hemodynamic instability in patients with confirmed pulmonary embolism<br />

(2) Hemodynamic instability in patients with a high clinical index of suspicion (especially with right<br />

ventricular dysfunction on bedside echo)<br />

(3) Right ventricular dysfunction on echo in hemodynamically stable patients with confirmed pulmonary<br />

embolism<br />

b. Thrombolytic agents work by directly lysing clots.<br />

289


THORACIC AND RESPIRATORY DISORDERS<br />

c. Before administering these agents, the diagnosis of pulmonary embolism should ideally be established with<br />

a high degree of certainty. However, if there is a high suspicion of pulmonary embolism in a patient who is<br />

hemodynamically compromised, thrombolytic therapy may be administered before diagnostic testing.<br />

d. Administration of these thrombolytic agents should be followed immediately by full-dose heparin<br />

anticoagulation (There is no increase in bleeding complications when heparin is administered concurrently<br />

with tissue plasminogen activator.)<br />

e. Advantages: in contrast to heparin, thrombolytic therapy prevents the postphlebitic syndrome, reduces the<br />

risk of recurrent pulmonary embolism/DVT, restores normal myocardial and valvular function, normalizes<br />

pulmonary vascular resistance and pulmonary arterial pressure, improves long-term exercise capacity, and<br />

normalizes pulmonary capillary volume and pulmonary gas diffusion. There is also some evidence that it<br />

does decrease mortality but this has not yet been clearly demonstrated.<br />

f. Contraindications: identical to those for acute Ml (see reperfusion therapy in Ml, pages 45-46)<br />

g. Complications<br />

(1) Thrombolytic therapy and anticoagulant therapy produce a similar incidence of systemic bleeding<br />

complications. However, the risk of hemorrhage increases directly with the duration of infusion of<br />

the thrombolytic agent: urokinase and streptokinase are usually infused over a 12-24 hour period,<br />

whereas the usual protocol for TPA is a 2-hour infusion.<br />

(2) If serious bleeding complications occur, the thrombolytic infusion should be discontinued, and<br />

aminocaproic acid should be administered along with transfusions of fresh frozen plasma and<br />

cryoprecipitate.<br />

h. Thrombolytic agents approved for treatment of pulmonary embolism include streptokinase and TPA; TPA is<br />

preferred because it significantly improves hemodynamic parameters in


THORACIC AND RESPIRATORY DISORDERS<br />

2. Etiology of central cyanosis (decreased arterial oxygen saturation)<br />

a. Decreased atmospheric pressure (high altitude)<br />

b. Impaired pulmonary function<br />

(1) Alveolar hypoventilation<br />

(2) Disparities between pulmonary ventilation and perfusion<br />

(3) Impaired oxygen diffusion<br />

c. Anatomic shunts<br />

(1) Certain types of congenital heart disease<br />

(2) Pulmonary AV fistulas<br />

(3) Multiple small intrapulmonary shunts<br />

d. Hemoglobin with low affinity for oxygen<br />

e. Hemoglobin abnormalities (may be drug-induced)<br />

(1) Methemoglobinemia (acquired and hereditary)<br />

(2) Sulfhemoglobinemia (acquired)<br />

(3) Both may result from the use of phenacetin, acetanilid, aniline, and sulfonamides.<br />

3. Etiology of peripheral cyanosis (normally saturated arterial blood with increased oxygen extraction)<br />

a. Reduced cardiac output (eg, CHF)<br />

b. Cold exposure vasoconstriction (eg, frostbite, severe hypothermia)<br />

c. Redistribution of blood flow from extremities (eg, shock states)<br />

d. Arterial or venous obstruction (eg, peripheral vascular disease)<br />

e. Polycythemia<br />

291


THORACIC AND RESPIRATORY DISORDERS: PRACTICE CLINICAL SCENARIOS<br />

THORACIC AND RESPIRATORY DISORDERS:<br />

PRACTICE CLINICAL SCENARIOS<br />

Answers immediately follow practice clinical scenarios.<br />

Scenario A<br />

Presentation: An elderly male patient presents in respiratory distress with an audible wheeze. He has a<br />

significant smoking history, appears malnourished, and has difficulty speaking in complete sentences. He<br />

complains of shortness of breath that has been present for several days. The patient has noted a change in<br />

the character of his sputum. He does not have a fever.<br />

Physical examination: The patient appears tachypneic, cyanotic, and is showing some use of accessory<br />

muscles of respiration. Lung examination reveals grossly diminished air movement throughout with<br />

expiratory wheezing. There is no peripheral edema.<br />

What is the diagnosis?<br />

Scenario B<br />

Presentation: A patient presents with symptoms of urinary tract infection/pyelonephritis. He has not been<br />

feeling well, with intermittent flank pain and nausea, for 3 days. There is no history of CHF, liver disease,<br />

or malnutrition.<br />

Physical examination: Vital signs are temperature 102°F (39°C), blood pressure 110/64 mmHg, heart rate<br />

130 beats per minute, respiratory rate 22 breaths per minute, and SpO 2<br />

95%. The patient is ill appearing.<br />

Chest examination reveals scattered, bilateral rales. The abdomen is soft. There is mild left costoverterbral<br />

angle (flank) tenderness. There is no lower extremity edema.<br />

What is the diagnosis?<br />

Scenario C<br />

Presentation: An adult patient presents with chest pain and shortness of breath that began after landing<br />

from an 8-hour plane flight. The chest pain is pleuritic in nature.<br />

Physical examination: The patient is no distress. Lung examination is clear. One leg appears swollen<br />

compared with the other. Vital signs show tachycardia and tachypnea.<br />

What is the diagnosis?<br />

Scenario D<br />

Presentation: A patient presents with a gunshot wound to the left side of the chest.<br />

Physical examination: The heart rate is 125 beats per minute, blood pressure is 80/40 mm Hg, and the<br />

respiratory rate is 22 breaths per minute with 0 2<br />

saturations 85% on nonrebreather. The patient appears<br />

uncomfortable but is answering questions. Jugular venous distention is noted on neck examination.<br />

Cardiovascular examination reveals tachycardia, regular rhythm, and no murmurs. Pulmonary examination<br />

reveals decreased breath sounds on the left side.<br />

What is the diagnosis and next step in management?<br />

292


THORACIC AND RESPIRATORY DISORDERS: PRACTICE CLINICAL SCENARIOS<br />

Scenario E<br />

Presentation: A patient presents with fever, pleuritic chest pain, and cough.<br />

Physical examination: The patient is ill appearing with a heart rate of 110 beats per minute, blood pressure<br />

of 110/72 mm Hg, and respiratory rate of 24 breaths per minute with 0 2<br />

saturations 90% on 1 L by nasal<br />

cannula. Pulmonary examination reveals decreased breath sounds on the right side with dullness on<br />

percussion. A chest radiograph shows loss of the costophrenic angle with a moderate pleural effusion. A<br />

sample obtained from thoracentesis has a pH of 7.05 with glucose of 55 mg/dL.<br />

What is the diagnosis and next step in management?<br />

Scenario F<br />

Presentation: A white woman with a history of asthma presents with shortness of breath. She complains of<br />

wheezing and cough.<br />

Physical examination: The patient appears in respiratory distress, speaking in 1 or 2 word sentences, with<br />

a respiratory rate of 26 breaths per minute and minimal air movement on pulmonary examination. You<br />

note intercostal retractions. The patient is intubated and given f:1-agonists through the ventilator. Less than<br />

an hour later, you are called into the room because the patient's heart rate is 130 beats per minute and her<br />

blood pressure is 82/55 mmHg. The ventilator shows decreased tidal volumes with high plateau pressures<br />

alarming.<br />

What is the next step in management?<br />

Scenario G<br />

Presentation: An immigrant from Asia presents with cough, fever, night sweats, and fatigue for 2-3 weeks.<br />

She is visiting family here in the United States.<br />

Physical examination: The patient appears cachetic. Temperature is 37.9°(, heart rate is 95 beats per<br />

minute, and respiratory rate is 20 breaths per minute with 0 2<br />

saturations of 95% on room air. Pulmonary<br />

examination reveals decreased breath sounds in the right upper lobe. A chest radiograph demonstrates a<br />

cavitary lesion in the upper lobe of the lung.<br />

What is the diagnosis?<br />

Scenario H<br />

Presentation: A patient presents with acute onset of dyspnea. He states the symptoms started suddenly,<br />

and he denies any fever, chills, or chest pain although he does admit to a nonproductive cough. It is 5 days<br />

after he underwent surgery for a total hip replacement.<br />

Physical examination: The patient's blood pressure is 80/50 mm Hg, heart rate is 130 beats per minute,<br />

and respiratory rate is 28 breaths per minute with 0 2<br />

saturation on 15 L by nonrebreather of 88%.<br />

Cardiovascular examination demonstrates tachycardia, normal rhythm, and no murmurs. Pulmonary<br />

examination demonstrates diminished breath sounds in the bases with no crackles and no rhonchi.<br />

What is the next step in management?<br />

293


THORACIC AND RESPIRATORY DISORDERS: PRACTICE CLINICAL SCENARIOS<br />

ANSWERS TO PRACTICE CLINICAL SCENARIOS<br />

Scenario A<br />

Diagnosis: COPD exacerbation<br />

Management: Begin nebulized albuterol, steroid therapy, and antibiotic coverage for COPD exacerbation.<br />

Influenza testing is appropriate when in season. Chest radiograph will show evidence of hyperinflation<br />

and flattened diaphragms. Consider venous blood gas testing to evaluate for hypercarbia. Hypoxia will be<br />

manifest by increased oxygen requirement. Monitor the patient for improvement or decline in respiratory<br />

status. Noninvasive positive-pressure ventilation may be necessary to assist with work of breathing for<br />

severe exacerbations. Patients may be discharged if there is immediate clinical improvement in symptoms,<br />

and if they appear clinically well and can ambulate safely without evidence of tachypnea or hypoxia.<br />

Patients with persistent distress, continued wheezing, or who need frequent nebulized treatments,<br />

increased oxygen requirements, or Bi PAP should be admitted to an appropriate level of care.<br />

Scenario B<br />

Diagnosis: severe sepsis with evidence of acute lung injury<br />

Management: The patient has acute lung injury from severe sepsis due to pyelonephritis. Chest<br />

radiograph may be suspicious for volume overload with bilateral infiltrates; however, this is related to<br />

pulmonary capillary leak related to sepsis. The patient is in need of early antibiotic and fluid resuscitation.<br />

Noncardiogenic pulmonary edema should not prevent fluid resuscitation. Arterial blood gases can be<br />

helpful to determine the degree of acute lung injury. Increased oxygen requirement may require intubation.<br />

Intubated patients should be treated with lung protective ventilation of 6 ml/kg ideal body weight based<br />

on ARDSNet data.<br />

Scenario C<br />

Diagnosis: pulmonary embolism<br />

Management: There is high clinical suspicion of pulmonary embolism based on history. Empiric<br />

anticoagulation may be appropriate for clinically unstable patients. The diagnosis of pulmonary embolism<br />

is confirmed by CT study. Hypotensive patients or patients with right ventricular strain may benefit from<br />

thrombolysis. Point-of-care ultrasound in the emergency department or comprehensive echocardiography<br />

may be helpful. The decision for thrombolysis can be made in conjunction with appropriate consultants<br />

(pulmonary, cardiology, radiology, surgery, etc), depending on your institution's practice patterns.<br />

Scenario D<br />

Diagnosis: tension pneumothorax<br />

Management: This patient has symptoms and signs consistent with a tension pneumothorax and requires a<br />

needle decompression.<br />

A patient that presents in extremis (tachycardia, hypotension, jugular venous distention) with penetrating<br />

trauma to the left side of the chest is concerning for either a tension pneumothorax or pericardia!<br />

tamponade. Auscultation of breath sounds can differentiate between these two diagnoses. If breath sounds<br />

are absent or diminished on one side, the patient has a tension pneumothorax, and needle decompression<br />

with a 14-gauge needle in the second intercostal space on the affected side should be performed. If breath<br />

sounds are bilateral and symmetric, the patient has cardiac tamponade, and pericardiocentesis should be<br />

performed.<br />

294


THORACIC AND RESPIRATORY DISORDERS: PRACTICE CLINICAL SCENARIOS<br />

Scenario E<br />

Diagnosis: empyema<br />

Management: A sample obtained from diagnostic thoracentesis with a pH of


296<br />

NOTES


TRAUMATIC DISORDERS<br />

TRAUMATIC DISORDERS<br />

General Information ................................................................................................................................................. 302<br />

General Approach to the Trauma Patient. .................................................................................................................. 303<br />

Hemorrhagic Shock .................................................................................................................................................. 307<br />

Common Injuries by Anatomic Location ................................................................................................................... 307<br />

Traumatic Brain Injury (Head Injury) ................................................................................................................... 307<br />

Overview .................................................................................................................................................... 307<br />

Specific Types of Head Injuries .................................................................................................................... 309<br />

Diffuse Brain Injuries ......................................................................................................................... 309<br />

Focal Brain Lesions ............................................................................................................................ 310<br />

Cerebral Edema ................................................................................................................................. 311<br />

Herniation Syndromes ....................................................................................................................... 312<br />

Skull Fractures ................................................................................................................................... 312<br />

Complications of Head Injuries ................................................................................................................... 313<br />

Management ............................................................................................................................................... 314<br />

Facial lnjuries ..................................................................................................................................................... 315<br />

Adult Spinal Trauma ........................................................................................................................................... 316<br />

Spinal Cord Injury ....................................................................................................................................... 316<br />

Vertebral Spine Trauma ............................................................................................................................... 320<br />

Pediatric Vertebral and Spinal Cord Injuries ........................................................................................................ 322<br />

Thoracic Trauma ................................................................................................................................................. 322<br />

Abdominal Injuries ............................................................................................................................................. 326<br />

Pelvic and Extremity Injuries ............................................................................................................................... 329<br />

Unique Situations ...................................................................................................................................................... 329<br />

Electrical Injury .................................................................................................................................................. 329<br />

Rhabdomyolysis ................................................................................................................................................. 331<br />

Thermal Burns .................................................................................................................................................... 331<br />

Trauma in Pregnancy .......................................................................................................................................... 332<br />

Penetrating Trauma ............................................................................................................................................. 334<br />

Blast Injuries ....................................................................................................................................................... 335<br />

Additional Pearls ................................................................................................................................................ 336<br />

297


TRAUMATIC DISORDERS: SELF-ASSESSMENT QUESTIONS<br />

TRAUMATIC DISORDERS: SELF-ASSESSMENT QUESTIONS<br />

1. The smallest amount of blood loss that consistently produces a decrease in the systolic blood pressure in adults is:<br />

(a)<br />

Loss 40% of blood volume<br />

2. When evaluating a child who has sustained blunt trauma as a result of a motor vehicle collision, which of the<br />

following signs would most likely be the earliest and most reliable indication of hemorrhagic shock?<br />

(a)<br />

Altered mental status<br />

(b) Hypotension<br />

(c) Tachypnea<br />

(d) Tachycardia<br />

(e) Hypoxemia<br />

3. The leading cause of death and disability in trauma victims is:<br />

(a)<br />

Head injury<br />

(b) Spinal cord injury<br />

(c) Abdominal injury<br />

(d) Thoracic injury<br />

4. Which of the following statements regarding cardiac tamponade is inaccurate?<br />

(a) It is most commonly caused by blunt chest trauma.<br />

(b) Clinical findings include hypotension, decreased pulse pressure, jugular venous distention, pulsus paradoxus,<br />

and muffled heart tones.<br />

(c) Initial therapy may be IV fluids and pericardiocentesis.<br />

(d) It can manifest as pulseless electrical activity.<br />

5. A patient with chest trauma is hypotensive on presentation and complains of shortness of breath. Examination<br />

reveals jugular venous distention, tracheal deviation, and decreased breath sounds associated with hyperresonance<br />

to percussion on one side of the chest. In addition to providing oxygen, starting IV lines, and placing the patient on<br />

a cardiac monitor, what is the most appropriate immediate intervention?<br />

(a)<br />

Pericardiocentesis<br />

(b) Tube thoracostomy<br />

(c) Intubation<br />

(d) Needle thoracostomy<br />

6. Of patients with the following acute traumatic injuries, which is most likely to survive at the scene of injury and<br />

present to the emergency department with signs of life?<br />

(a) Complete airway obstruction<br />

(b) Atlanta-axial dissociation<br />

(c) Liver laceration<br />

(d) Aortic rupture<br />

(e)<br />

Cardiac laceration<br />

7. The most common abdominal organ injured in blunt trauma is:<br />

(a) Spleen<br />

(b) Liver<br />

(c)<br />

Pancreas<br />

(d) Kidney<br />

8. Of the following potential therapies, which is the most useful in treatment of myoglobinuria/rhabdomyolysis?<br />

(a) Aggressive IV hydration<br />

(b) IV mannitol<br />

(c) Alkalinization of the urine<br />

(d) Loop diuretics<br />

298


TRAUMATIC DISORDERS: SELF-ASSESSMENT QUESTIONS<br />

9. All of the following chest radiograph findings are consistent with the diagnosis of traumatic rupture of the aorta<br />

except:<br />

(a) Widening of the superior mediastinum<br />

(b) Apical pleural cap<br />

(c) Deviation of the trachea to the right<br />

(d) Elevation of the left mainstem bronchus<br />

10. Which of the following techniques provides the best assessment of retroperitoneal organs (eg, pancreas, duodenum)?<br />

(a)<br />

Physical examination<br />

(b) Diagnostic peritoneal lavage<br />

(c) CT of the abdomen and pelvis<br />

(d) FAST examination<br />

11. Although all of the following techniques should be used when available, the most reliable method of confirming<br />

endotracheal tube position is by:<br />

(a) Listening over the upper lung fields for equal breath sounds<br />

(b) Checking the position of the tube on a postintubation chest radiograph<br />

(c) Noting the presence of condensation in the tube<br />

(d) Seeing the endotracheal tube pass through the vocal cords<br />

12. Which of the following patients with head injuries is least likely to have a clinically important traumatic brain injury<br />

(ie, an injury resulting in death, neurosurgical intervention, endotracheal intubation, or prolonged hospitalization)?<br />

(a) A 12-month-old who ran into the corner of a coffee table, fell to the ground, and cried immediately, but is now<br />

acting normally per his parents; the child has a large frontal hematoma but an otherwise normal examination.<br />

(b) A 10-year-old who fell 15 feet while climbing a tree and initially had brief loss of consciousness but was awake<br />

and alert when EMS arrived; en route to the emergency department, he vomited three times and on arrival he is<br />

poorly responsive to voice.<br />

(c) A 1-month-old who was brought in by her parents after reportedly rolling off the changing table (3-4 feet) and<br />

falling to the floor; the parents report that the child cried immediately and has not vomited. On examination,<br />

the child is crying and has signs of external trauma.<br />

(d) A 75-year-old nursing-home resident who tripped and fell while in the bathroom has a large posterior scalp<br />

laceration and hematoma; she is awake and alert, albeit slightly confused, and her neurologic exam is nonfocal.<br />

(e) A 35-year-old unrestrained driver who was involved in a single car roll-over motor vehicle collision at highway<br />

speeds, ejected from the vehicle, and found by EMS in the roadside ditch. He has a Glasgow Coma Score<br />

of 13, is confused, and has abrasions on his face, chest, and extremities. The remainder of the primary and<br />

secondary surveys is unremarkable.<br />

13. All of the following statements regarding epidural hematomas are accurate except:<br />

(a) Associated parietal or temporal skull fracture is common.<br />

(b) CT reveals a lens-like, biconvex lesion.<br />

(c) Signs and symptoms are due to the mass effect of an arterial bleed.<br />

(d) Pupillary findings are typically contralateral to the side of the lesion.<br />

14. All of the following statements regarding basilar skull fractures are accurate except:<br />

(a) Placement of a nasogastric tube and nasotracheal intubation are contraindicated in the presence of these<br />

fractures.<br />

(b) Standard CT scan of the head can reliably exclude basilar skull fractures in patients with clinical suspicion of<br />

such injury.<br />

(c) Clinical findings include CSF rhinorrhea or otorrhea, hemotympanum, raccoon's eyes, Battle sign, and the "ring<br />

sign" on filter paper.<br />

(d) Patients should have a neurosurgical consult and possibly be admitted for observation.<br />

15. Listed below are several cervical spine injuries and their mechanism of injury. Which is incorrectly matched?<br />

(a) Jefferson fracture: axial loading mechanism<br />

(b) Hangman fracture: flexion injury<br />

(c) Clay-shoveler's fracture: flexion injury or direct trauma<br />

(d) Unilateral facet dislocation: flexion-rotation injury<br />

299


TRAUMATIC DISORDERS: SELF-ASSESSMENT QUESTIONS<br />

16. The best view on plain cervical radiographs for visualizing a Jefferson fracture is:<br />

(a)<br />

Open-mouth odontoid<br />

(b) Cross-table lateral<br />

(c) AP<br />

(d) Oblique<br />

1 7. Al I of the following statements regarding pulmonary contusions are accurate except:<br />

(a) They usually result from blunt chest wall trauma.<br />

(b) Associated rib fractures may be present.<br />

(c) Chest radiograph findings are typically delayed 24-72 hours after injury.<br />

(d) Pneumonia is the most common and most significant complication.<br />

18. Cardiac contusions most commonly involve the:<br />

(a)<br />

Left ventricle<br />

(b) Right ventricle<br />

(c) Left atrium<br />

(d) Right atrium<br />

19. The earliest clinical finding in a patient with a compartment syndrome is:<br />

(a) Pain<br />

(b) Paralysis<br />

(c) Palpable tenseness and tenderness of the involved compartment<br />

(cl) Pulselessness<br />

20. The most common intracranial bleed associated with closed head trauma is:<br />

(a)<br />

Subarachnoid<br />

(b) Subdural<br />

(c) Epidural<br />

(cl)<br />

lntraparenchymal<br />

21. Epidural hematomas frequently occur in association with skull fractures that traverse the groove of the:<br />

(a) Middle meningeal artery<br />

(b) Middle cerebral artery<br />

(c) Anterior cerebral artery<br />

(d) Anterior meningeal artery<br />

22. An 80-year-old patient presents to the emergency department complaining of bilateral arm weakness after she fell to<br />

the ground with walking her dog on a leash. Examination findings reveal 3/5 strength in both arms and 4/5 strength<br />

in both legs. Her sensation is normal. The most likely mechanism to cause this specific injury pattern is ___ _<br />

(a)<br />

Hyperflexion<br />

(b) Axial loading<br />

(c) Rotational forces<br />

(cl)<br />

(e)<br />

Penetrating injury<br />

Hyperextension<br />

23. Which of the following treatments used for increased intracranial pressure is most rapidly effective?<br />

(a) Mannitol<br />

(b) Furosemide<br />

(c) Ventilation to a PaCO 2<br />

of 30-35 mmHg<br />

(cl) Steroids<br />

300<br />

24. Which of the following characteristics of a thoracolumbar compression fracture is not associated with vertebral<br />

column instability?<br />

(a)<br />

Compression fractures at multiple levels<br />

(b) Loss of> 1 0% vertebral body height<br />

(c) Kyphosis >30%<br />

(d) Rotational component of fracture<br />

(e) Posterior vertebral wedging


TRAUMATIC DISORDERS: SELF-ASSESSMENT QUESTIONS<br />

25. Of the following modalities, which is least useful in identifying pneumothoraces?<br />

(a) Ultrasound<br />

(b) Upright, expiratory chest radiograph<br />

(c) CT of the chest<br />

(d) Supine, inspiratory chest radiograph<br />

(e) Physical examination<br />

26. Advantages of FAST exam include all of the following except:<br />

(a) High sensitivity for identifying intra-abdominal hemorrhage<br />

(b) High sensitivity for identifying intra-abdominal injuries<br />

(c) May be performed at the bedside<br />

(d) May be repeated if clinical condition deteriorates<br />

(e)<br />

Noninvasive<br />

27. All of the following findings are suggestive of a cord injury except:<br />

(a) Diaphragmatic breathing<br />

(b) Priapism<br />

(c) Flaccid areflexia<br />

(d) Hypertension and bradycardia<br />

28. The presence of which of the following is the most accurate parameter when evaluating diagnostic peritoneal lavage<br />

fluid for suspected intra-abdominal injury?<br />

(a)<br />

Bile<br />

(b) Red blood cells<br />

(c) White blood cells<br />

(d) Amylase<br />

(e) Lactate dehydrogenase<br />

29. Rh immunoglobulin administration would be indicated in all of the following Rh-negative patients except:<br />

(a) G3P2; 38 weeks gestation; tripped and fell onto abdomen this morning; denies pain, contractions, vaginal<br />

bleeding, or discharge<br />

(b) G6P5; 30 weeks gestation; victim of domestic violence (punched and kicked in face, abdomen, back);<br />

complains of painless vaginal spotting<br />

(c) Gl PO; 25 weeks gestation; restrained passenger in low-speed motor vehicle collision; no complaints<br />

(d) Gl PO; 25 weeks gestation; tripped and fell while trail running yesterday, landing on knees and then abdomen;<br />

complains of decreased fetal movement but not pain, cramping, bleeding, or discharge<br />

(e) All of the women should receive Rh immunoglobulin.<br />

30. Which of the following organs is least likely to be injured as a result of primary blast injury?<br />

(a)<br />

Eye<br />

(b) Ear<br />

(c) Lungs<br />

(d) Liver<br />

(e) Colon<br />

ANSWERS<br />

1. C 6. C 11. d 16. a 21. a 26. b<br />

2. d 7. a 12. a 17. C 22. e 27. d<br />

3. a 8. a 13. d 18. b 23. C 28. b<br />

4. a 9. d 14. b 19. a 24. b 29. e<br />

5. d 10. c 15. b 20. a 25. d 30. d<br />

Use the pre-chapter multiple choice question worksheet (page xvi) to record and determine the percentage of correct<br />

answers for this chapter.<br />

301


TRAUMATIC DISORDERS<br />

I. GENERAL INFORMATION<br />

A. Trauma is the leading cause of death in people 1-44 years old worldwide.<br />

B. The primary role of the emergency physician is to assess, resuscitate, and stabilize the trauma patient<br />

on a priority basis; this includes both suspicion for and recognition of potentially serious injuries and<br />

complications.<br />

1. Trauma management is best provided by a multidisciplinary team, including emergency medicine, general<br />

(trauma) surgery, and various surgical subspecialities.<br />

2. Trauma victims have a significantly lower likelihood of morbidity and mortality when treated at a designated<br />

trauma center.<br />

C. Trauma-related mortality has a trimodal distribution, ie, three peak times for trauma deaths.<br />

1. Immediate death<br />

a. Occurs within seconds to minutes of injury<br />

b. Generally die at the scene<br />

c. Accounts for greatest number of trauma deaths<br />

d. Death is usually due to:<br />

2. Early death<br />

(1) Massive head injury<br />

(2) High cervical spine injury with high spinal cord disruption<br />

(3) Cardiac laceration<br />

(4) Aortic rupture<br />

(5) Laceration of other great vessels<br />

(6) Airway obstruction<br />

a. Occurs within minutes to a few hours of injury, the so-called "golden hour"<br />

b. Generally die within 4 hours of arrival at trauma center<br />

c. Death is generally secondary to:<br />

(1) Subdural and epidural hematomas<br />

(2) Ruptured spleen or lacerated liver<br />

(3) Multiple injuries associated with hypovolemic shock<br />

(4) Fracture of the pelvis or multiple long bones<br />

(5) Hemopneumothorax<br />

(6) Tension pneumothorax<br />

(7) Cardiac tamponade<br />

(8) Massive hemothorax<br />

(9) Aortic dissection/rupture<br />

(1 0) Airway compromise<br />

d. Application of advanced trauma life support (ATLS) guidelines regarding rapid ABC assessment and<br />

stabilization (definitive airway management, fluid resuscitation, and blood replacement) reduces the<br />

mortality and morbidity rate of these patients.<br />

3. Delayed death<br />

a. Occurs days to weeks after the initial injury<br />

b. Usually die in intensive care setting<br />

c. Death is usually due to:<br />

(1) Multisystem organ failure<br />

(2) Systemic inflammatory response syndrome<br />

d. With advances in trauma critical care management, relatively few trauma victims die beyond 24 hours after<br />

injury. Therefore, some experts suggest that distribution of trauma mortality is bimodal rather than trimodal.<br />

302


TRAUMATIC DISORDERS<br />

II. GENERAL APPROACH TO THE TRAUMA PATIENT<br />

A. ATLS guidelines provide a clear, organized, and rapid approach to the evaluation and management of<br />

trauma patients.<br />

B. Primary survey: assessment and management of the ABCs<br />

1. Airway and cervical spine assessment<br />

a. Airway management is the single most important prehospital/arrival priority.<br />

b. Immediately assess the airway for potential compromise (nonpatent or poorly protected airway).<br />

(1) Airway patency<br />

(a)<br />

If the patient can speak, the airway is patent.<br />

(b) If the respirations are noisy (stridulous, sonorous, gurgling, etc), partial obstruction is present.<br />

(2) Airway protection<br />

(a) Assume the airway is poorly protected if:<br />

i. Decreased level of consciousness (Glasgow Coma Score ::;8)<br />

ii. Secretions are pooling<br />

iii. Gag reflex is depressed or absent<br />

(b) The poorly protected airway is at risk of aspiration of gastric/oral secretions, blood, food, foreign<br />

bodies.<br />

c. Beware of the potential for associated cervical spine injuries.<br />

(1) Always maintain the cervical spine in neutral position with in line manual cervical immobilization or<br />

appropriate immobilization devices when evaluating and managing the airway.<br />

(2) Significant cervical spine injury should always be suspected in the patient with:<br />

(a)<br />

Blunt trauma<br />

(b) Multiple injuries<br />

(c) Altered level of consciousness<br />

(d) Evidence of blunt injury above the clavicle<br />

(e)<br />

Dive injuries<br />

d. Airway management in patients with possible cervical spine injury<br />

(1) Open the airway using the chin lift or jaw thrust maneuver. Do not use head tilt maneuver.<br />

(2) Remove dentures and foreign bodies if present.<br />

(3) Consider placement of an oropharyngeal (unconscious patient) or nasopharyngeal (conscious<br />

patient) airway.<br />

(4) Patients who do not have a patent airway or are not protecting their airway require definitive airway<br />

management (orotracheal intubation or a surgical airway).<br />

(5) Orotracheal laryngoscopy and intubation with in-line cervical spine immobilization is the procedure<br />

of choice (even in the presence of suspected cervical spine injury).<br />

(a) Front of the cervical immobilization device should be removed while maintaining in-line<br />

cervical spine immobilization to facilitate intubation.<br />

(b) Direct laryngoscopy may be more difficult in the trauma patient.<br />

(c) Video-assisted laryngoscopy has been shown to have a high success rate of intubation with less<br />

movement of the cervical spine.<br />

(d) Fiberoptic laryngoscopy is an option if the patient's condition permits and the physician is skilled<br />

in this technique.<br />

(e) An endotracheal tube introducer (gum elastic bougie) is a valuable tool in airway management<br />

in trauma patients.<br />

(6) Failed airway management<br />

(a) If orotracheal intubation is unsuccessful or relatively contraindicated (see below), surgical airway<br />

is indicated.<br />

(b) Surgical cricothyrotomy is surgical airway of choice for the emergency physician. (Some authors<br />

consider a surgical airway the procedure of choice for patients with severe midfacial injuries;<br />

however, most clinicians would make a single attempt at orotracheal intubation with in-line<br />

manual cervical immobilization before proceeding to a surgical airway.)<br />

303


TRAUMATIC DISORDERS<br />

(c) In children 0.85.<br />

d. Observe, palpate, and auscultate the thorax to quickly identify and manage life-threatening injuries that<br />

may compromise cardiopulmonary function.<br />

(1) Tension pneumothorax: perform needle thoracostomy, and follow immediately with thoracostomy<br />

tube placement.<br />

(2) Sucking chest wound: cover with a sterile occlusive dressing, taped down on three of its four sides.<br />

(3) Flail chest: intubation with positive-pressure ventilation indicated for large flail segments.<br />

e. Ventilator settings should reflect potential for hemodynamic compromise induced by positive-pressure<br />

ventilation in hypovolemic patients.<br />

(1) Positive-pressure ventilation may decrease venous return to the heart (preload) and worsen<br />

hypotension; low tidal volumes are appropriate (5 ml/kg at a rate of 20-30 breaths per minute).<br />

(2) Close monitoring of pulse and blood pressure and frequent arterial blood gases ultimately determine<br />

the optimal ventilator settings.<br />

3. Circulation and hemorrhage control<br />

a. Control obvious external bleeding.<br />

(1) Direct manual pressure followed by application of a pressure dressing is considered first-line<br />

management; elevation of affected extremity may help.<br />

(2) Application of a tourniquet may be necessary to control life-threatening hemorrhage from a severely<br />

injured extremity (eg, explosion injury).<br />

(3) Bleeding from posterior scalp lacerations is often overlooked initially.<br />

(4) Various topical hemorrhage control agents have been adapted from those used on modern battlefields<br />

and are now commercially available.<br />

b. Assess circulation for signs of shock.<br />

(1) Check central and peripheral pulses for quality, rate, and regularity.<br />

(2) Evaluate skin color, capillary refill time, and level of consciousness.<br />

(3) The FAST exam should be performed in the primary survey in unstable patients to assess for internal<br />

hemorrhage.<br />

c. Take measures to control internal sources of bleeding.<br />

304


TRAUMATIC DISORDERS<br />

(1) Unstable pelvic fractures may be temporarily stabilized with a pelvic binder device (either a<br />

commercially available product or a simple bedsheet) to decrease blood loss.<br />

(2) Reduce and temporarily splint obvious long bone fractures to reduce ongoing hemorrhage.<br />

(3) Unstable patients with massive hemoperitoneum should be transferred to the OR as quickly as<br />

possible.<br />

d. Place the patient on a cardiac monitor, and check a rhythm strip.<br />

e. Establish IV access.<br />

(1) Place two short, large-bore peripheral IV catheters (16-gauge or larger). Draw blood for laboratory<br />

studies at the same time.<br />

(2) If peripheral access is difficult or delayed, place an intraosseous catheter in the seriously injured,<br />

hemodynamically unstable patient.<br />

(3) Consider central IV access, but remember that short, large-bore peripheral catheters can infuse fluid<br />

faster than long central lines.<br />

f. Hemorrhagic shock resuscitation<br />

(1) Administer 1-2 L warmed lactated Ringer's or normal saline over 5 minutes (fluid bolus in children is<br />

20 ml/kg x 2 or 3 boluses).<br />

(2) If the patient remains in shock, packed RBCs should be administered.<br />

(3) For patients needing large volumes of packed RBC transfusions, consideration should be given to<br />

administering other blood products (fresh frozen plasma, platelets) as well. There is no standard<br />

approach to massive transfusions; protocols vary by institution. Increased survival has been<br />

demonstrated with use of a 1 :1 :1 ratio of RBCs:platets:fresh frozen plasma.<br />

g. Cardiac tamponade<br />

(1) Classic signs are muffled heart sounds, jugular venous distention, and hypotension (Beck's triad).<br />

(2) Bedside echocardiography (FAST) is highly sensitive for diagnosing pericardia! effusion.<br />

(3) Perform pericardiocentesis.<br />

h. Indications for emergency thoracotomy in the emergency department: no consensus guidelines<br />

(1) Penetrating thoracic trauma<br />

(a)<br />

Hemodynamically unstable on arrival to the emergency department despite adequate fluid<br />

resuscitation; ideally, the patient loses pulses in front of resuscitation team.<br />

(b) Pulseless for


TRAUMATIC DISORDERS<br />

306<br />

c. Patients with severe head injury plus an altered level of consciousness (or a GCS ::08) usually require<br />

intubation for airway protection. The presence of nonpurposeful motor responses (4 or fewer) also suggests<br />

the need for a definitive airway.<br />

5. Exposure/environmental control: completely undress the patient to facilitate a thorough examination, but<br />

remember to cover with warmed blankets to prevent iatrogenic hypothermia.<br />

C. Initial diagnostic testing and continued management<br />

1. Laboratory studies, type and cross, CBC with platelets, prothrombin time (INR)/partial thromboplastin time,<br />

BUN/creatinine, electrolytes, glucose, amylase, lipase, ethanol, toxicology screen, ~-human chorionic<br />

gonadotropin (pregnancy test), urinalysis, arterial or venous blood gases, lactate, etc, as appropriate. Selective<br />

laboratory testing is appropriate.<br />

2. Bedside radiographs: cross-table lateral cervical spine, AP chest, and AP pelvis; may be ordered in major<br />

trauma, especially when there are signs of hemodynamic instability.<br />

3. FAST examination (if not performed in the primary survey): indicated in all trauma patients with signs of<br />

abdominal, pelvic, or thoracic injury; femur fracture; or hemodynamic instability.<br />

4. Cardiopulmonary monitoring: continuous ECG, pulse oximetry, and blood pressure monitoring<br />

5. Nasogastric (or orogastric) tube: should be placed in intubated patients unless contraindications (eg, cribriform<br />

plate fracture, other unstable midface fractures) are present.<br />

6. Bladder catheterization: place Foley catheter in unconscious or intubated patients unless contraindications (eg,<br />

blood at the urethral meatus, scrotal hematoma, or a high-riding prostate on rectal examination) are present.<br />

7. Trauma center transfer: consider need to transfer the patient to a designated trauma center or a hospital that<br />

can provide a higher level of trauma care.<br />

a. All trauma patients whose injuries exceed the management capabilities of their hospital should be<br />

transferred to an appropriate trauma center as quickly as possible (may be mandated in some states).<br />

b. A complete trauma evaluation, including imaging, beyond primary and secondary surveys (ie, ancillary<br />

tests and imaging) is not necessary and should not delay transfer of seriously injured patients.<br />

D. Secondary survey and final case management<br />

1. Obtain an "AMPLE" medical history and inquire about the mechanism of injury.<br />

2. Perform a complete physical examination. Consider performing a rectal examination, checking for sphincter<br />

tone, blood, high-riding prostate, and bowel-wall integrity.<br />

3. Order additional diagnostic studies as indicated, eg, radiographs of thoracolumbar spine and/or extremities, CT, etc.<br />

4. Assess and reassess laboratory data, response to fluids, vital signs, and overall stability of the patient.<br />

5. Administer tetanus booster and antibiotics when indicated.<br />

E. Additional information for pediatric patients<br />

1. Airway management<br />

a. Atropine may be considered when performing rapid-sequence intubation but is no longer routinely<br />

recommended.<br />

b. Cuffed or uncuffed endotracheal tubes are now acceptable in children.<br />

(1) Size: (16 + age in years)/4<br />

(2) If using cuffed tube, downsize ½ size.<br />

c. Needle cricothyrotomy is preferred in small children (60 mm Hg<br />

(2) 1 month to 1 year: >70 mmHg<br />

(3) > 1 year: 70 + 2(age in years)<br />

c. Children have robust compensatory mechanisms and maintain normal blood pressure for a longer time<br />

despite presence of significant hemorrhage and shock.<br />

3. Younger children (


TRAUMATIC DISORDERS<br />

Ill. HEMORRHAGIC SHOCK<br />

A. Hemorrhage-induced hypovolemia is the most common cause of shock in the trauma patient.<br />

B. The earliest signs of hemorrhagic shock are tachycardia and cutaneous vasoconstriction (prolonged<br />

capillary refill time).<br />

C. Once shock is identified, initial management is volume resuscitation with a warmed crystalloid solution;<br />

continued management is determined by the patient's response to initial therapy.<br />

D. The amount of blood loss in an average adult can be estimated based on the patient's initial clinical<br />

presentation as follows:<br />

Table 17: Classes of Hemorrhagic Shock<br />

Urine Output<br />

Class Pulse (beats/min) Blood Pressure CNS Status (ml/hr) Blood Loss<br />

30 120 Decreased Confused 5-15 30%-40%<br />

1,500-2,000 ml<br />

IV >140 Decreased Lethargic nil >40%<br />

>2,000 ml<br />

IV. COMMON INJURIES BY ANATOMIC LOCATION<br />

A. Traumatic brain injury (head injury)<br />

1. Overview<br />

a. Head injury is the leading cause of death and disability in trauma.<br />

b. The most frequent cause of death in patients sustaining severe head injury is uncontrolled intracranial<br />

hypertension and subsequent herniation.<br />

c. Assume a cervical spine injury exists in head-injured patients; if clinical assessment or the mechanism of<br />

injury suggests the possibility of neck injury, immobilize the cervical spine.<br />

d. Altered level of consciousness is the hallmark of brain injury.<br />

e. Indications for head CT in the injured adult patient and risk factors for abnormal findings on CT include:<br />

(1) Abnormal level of consciousness or sensorium (GCS


TRAUMATIC DISORDERS<br />

f. Considerations for head CT in children: The 2009 PECARN Guidelines identify children at low risk of<br />

clinically significant head injury in whom radiation-related risk of CT imaging may outweigh benefit (high<br />

negative predictive value). No CT indicated for:<br />

(1) 2 times<br />

+<br />

Intoxication (drug/alcohol) +<br />

+<br />

+<br />

Anticoagu I ants/<br />

coagulopathy<br />

+<br />

+<br />

Altered level of<br />

consciousness<br />

+<br />

+ (implied by GCS


TRAUMATIC DISORDERS<br />

Table 18: Comparison of Decision Instruments for Head CT (continued)<br />

Criteria Nexus II Canadian New Orleans<br />

Sensitivity<br />

97%<br />

99%<br />

99%<br />

Specificity<br />

47%<br />

47%<br />

33%<br />

Inclusion criteria<br />

None listed<br />

Age2:16 years, GCS 13-15,<br />

no coagulopathy nor on<br />

anticoagulation, no obvious open<br />

sku 11 fracture<br />

Age> 18 years, GCS 15, blunt head<br />

trauma occurring within previous 24<br />

hours causing loss of consciousness,<br />

amnesia, or disorientation<br />

2. Specific types of head injuries<br />

a. Diffuse brain injuries (concussions, diffuse axonal injuries): usually do not have mass lesions necessitating<br />

emergent neurosurgical intervention<br />

(1) Cerebral concussion<br />

(a) Mechanism: head injury -<br />

brief loss of neurologic function<br />

(b) Clinical presentation: clinically variable; may range from minimal to severe and debilitating;<br />

rarely associated with any focal findings. Common symptoms include:<br />

i. Brief loss of consciousness (not necessary for diagnosis): results from impairment of the<br />

reticular activating system<br />

11. Antegrade amnesia (with perseveration)<br />

iii. Headache<br />

iv. Nausea and/or vomiting<br />

v. Abnormal sensorium (usually brief)<br />

vi. Difficulty with cognition<br />

(c) Imaging: CT of the head and/or MRI of the brain (if performed) are normal.<br />

(d) Management: cognitive rest and avoidance of repeated head injury while brain is recovering<br />

i. May take 7-14 days or longer<br />

ii. Symptoms lasting >2 weeks may be characterized as postconcussive syndrome<br />

(2) Diffuse axonal injury<br />

(a) Clinical presentation<br />

i. Prolonged coma (often lasting days to weeks)± posturing<br />

ii. ± Autonomic dysfunction (high fever, hypertension, sweating)<br />

(b) Results in widely scattered microscopic neuronal damage throughout the brain<br />

(c) Imaging: CT of head -<br />

matter<br />

no mass lesion; may show diffuse punctate hemorrhages throughout white<br />

(d) Management: patients require admission and neurosurgical consult (ICP monitoring often<br />

indicated); overall mortality 33%.<br />

(3) Cerebral contusion<br />

(a) Clinical presentation: similar to that of a concussion except:<br />

i. Neurologic dysfunction is more profound and prolonged; patients often demonstrate mental<br />

confusion, obtundation, or coma.<br />

11. Focal deficits may be present if contusions occur in the sensorimotor strip.<br />

(b) Imaging: these lesions are generally found in the frontal and temporal lobes and are visible on CT<br />

scan. They may be produced at the site of impact (coup contusions) or at the opposite pole of the<br />

brain (contrecoup contusions).<br />

(c) Management<br />

1. Admit these patients for observation and obtain neurosurgical consult. Only supportive<br />

management is indicated unless complications develop.<br />

ii. Patients with temporal lobe contusions are at significant risk of progression of edema; the<br />

resulting mass effect may impinge directly on the brainstem. These patients should be closely<br />

observed in a neurosurgical ICU.<br />

309


TRAUMATIC DISORDERS<br />

(d) Complications: potential delayed complications (hours or days) include development of cerebral<br />

edema or an intracerebral hematoma at the site of the contusion, resulting in mass effect (best<br />

detected by repeating the CT 12-24 hours after the initial scan).<br />

b. Focal brain lesions (contusions, hemorrhages, hematomas): commonly cause mass effect and necessitate<br />

emergency neurosurgery<br />

(1) Epidural hematoma<br />

(a) Mechanism and clinical presentation: head injury causing sudden loss of consciousness<br />

(impairment of reticular activating system) as well as intracranial bleeding (usually arterial; most<br />

commonly middle meningeal artery) --,, brain may recover from initial cerebral concussion/<br />

contusion, resulting in a lucid interval (.'.S30%). As bleeding causes increased ICP and mass<br />

effect, altered level of sensorium (coma), a fixed and dilated pupil on the side of the lesion, and<br />

contralateral hemiparesis develop.<br />

i. Transtentorial herniation compresses cranial nerve Ill and the corticospinal tract.<br />

ii. Classic presentation is not always present.<br />

(b) Imaging: CT scan demonstrates the classic lens-like, biconvex lesion. Associated parietal or<br />

temporal skull fracture is common (80%). Epidural hematomas do not cross suture lines but can<br />

cross midline.<br />

(c) Management<br />

i. Immediate neurosurgical consult for surgical decompression<br />

ii. Manage increased ICP (see below).<br />

(d) Mortality is 0-20% depending on neurologic status: normal neurologic examination --,, 0%; coma<br />

--,, 20% without rapid detection and evacuation<br />

Courtesy of Dr. Colin Kaide<br />

(2) Subdural hematoma<br />

(a) More common than epidural hematomas and more often associated with significant primary<br />

brain damage (usually covering the entire surface of the hemisphere); subdural hematomas cross<br />

suture lines but not midline.<br />

(b) Risk factors: patients with brain atrophy (alcoholic patients, the elderly) are especially susceptible;<br />

brain atrophy --,, bridging veins spanning a greater distance and being more easily torn<br />

310


TRAUMATIC DISORDERS<br />

(c) Mechanism and clinical presentation<br />

i. Acute subdural hematoma: symptoms develop within 24 hours of injury.<br />

ii. Subacute or chronic subdural hematoma: symptoms may be delayed up to 2 weeks or more<br />

after injury.<br />

iii. Caused by "mass effect" of a venous hematoma bleed.<br />

iv. Symptoms range from a mild headache and confusion to lethargy and coma (with or without<br />

localizing signs).<br />

(d) Imaging: CT demonstrates the classic crescent-shaped lesion (be able to recognize on CT).<br />

(e) Management: same as for epidural hematoma; immediate neurosurgical intervention is usually<br />

required (except perhaps for mildly symptomatic chronic subdural hematomas).<br />

(f)<br />

Mortality rate for acute subdural hematomas ranges from 30% with early evacuation to 60% with<br />

later evacuation.<br />

i. Mortality is higher in patients who are unconscious from the time of injury.<br />

ii. Patients with a history of a lucid interval (50%-70%) have a better prognosis.<br />

c. Cerebral edema<br />

(1) Pathophysiology<br />

Courtesy of Dr. Sandra Werner<br />

SDH acute and chronic SL<br />

(a) Head trauma -s. brain swelling -s. increased ICP that can lead to brain herniation<br />

(b) Cerebral perfusion pressure= mean arterial pressure minus ICP (normal ICP


TRAUMATIC DISORDERS<br />

(h) Progression of neurologic symptoms<br />

(i) Development of signs of herniation<br />

i. Decreased level of consciousness<br />

ii. Posturing<br />

iii. lpsilateral pupil fixed and dilated, or both pupils are fixed (in midposition).<br />

iv. Loss of oculovestibular reflexes (doll's eyes/ice water calories)<br />

(3) Acute management of ICP<br />

(a) Raise the head of the bed 30°.<br />

(b) Ensure adequate sedation.<br />

(c) Maintain the pC0 2<br />

at ~32-35 mmHg.<br />

i. In the setting of acute herniation, hyperventilation to a pC0 2<br />

of 26-30 mmHg effectively<br />

lowers ICP by causing vasoconstriction but should be reserved for life-threatening situations<br />

as a bridge to more definitive therapy.<br />

11. Effects of hyperventilation are short-lived (1-24 hours)<br />

(d) Ensure adequate oxygenation (intubate early).<br />

(e) Osmotic therapy: administration of hyperosmolar solutions creates an osmolar gradient, drawing<br />

fluid across the blood-brain barrier out of the brain and into the systemic circulation, thereby<br />

reducing intracranial pressure.<br />

(f)<br />

1. Use caution in patients with decreased intravascular volume and hypernatremia; also<br />

controversial with mass lesions.<br />

11. Specific agents<br />

• Mannitol: 0.25-1 mg/kg IV bolus<br />

• Hypertonic saline (3%-23.4%): various dosing regimens (3-5 ml/kg IV bolus)<br />

Consult neurosurgery for consideration of ICP monitoring, seizure prophylaxis, and surgical<br />

intervention.<br />

d. Herniation syndromes<br />

(1) The addition of a mass to the closed cranial vault results in increased ICP which, if unchecked, can<br />

produce cerebral displacement (herniation).<br />

(2) Uncal herniation: occurs when a unilateral mass pushes the medial aspect of the temporal lobe<br />

(uncus) through the tentorial incisura. As the uncus herniates, the following clinical signs develop:<br />

(a) lpsilateral pupil dilatation and fixation (due to the compression of cranial nerve Ill)<br />

(b) Increased respiratory rate (or Cheynes-Stokes respiratory pattern)<br />

(c) Coma<br />

(d) Posturing (decorticate [not always seen], then decerebrate)<br />

(e) Apnea and death<br />

(3) Central herniation: occurs when a mass effect compresses the brainstem in a more symmetrical way;<br />

this results in a progressive loss of brainstem function: t level of consciousness+ t respiratory rate<br />

or Cheyne-Stokes (thalamic, upper brainstem compression)-> fixed midposition pupils (midbrain<br />

compression)-> posturing, loss of oculovestibular reflexes (pontine compression)-> flaccidity, apnea<br />

(medullary compression)<br />

(4) Cerebellar tonsillar herniation: downward displacement of the cerebellar tonsils through the foramen<br />

magnum, resulting in medullary compression-> pinpoint pupils and flaccid quadriplegia (most<br />

common motor deficit)-> apnea and circulatory collapse<br />

e. Skull fractures<br />

(1) Skull fractures help identify patients at high risk of having or developing an intracranial hematoma.<br />

(2) Imaging: routine skull radiographs are not indicated in the evaluation of patients with traumatic brain<br />

injury, because they do not evaluate the integrity of the underlying intracranial space. All significant<br />

head injuries require CT evaluation.<br />

(3) Management<br />

(a) Most patients with skull fractures require neurosurgical consult and admission for observation.<br />

(b) Management is based on type of fracture and presence or absence of underlying brain injury.<br />

(4) Types<br />

(a) Linear, nondepressed skull fractures-> require no specific treatment.<br />

(b) Depressed skull fractures-> may require operative elevation of the bony fragment if fracture<br />

extends below inner table<br />

312


TRAUMATIC DISORDERS<br />

(c) Open skull fractures -<br />

(d) "Egg shell" skull fractures in children -<br />

(e) Basilar skull fractures<br />

(5) CSF leaks<br />

i. Clinical presentation<br />

require early operative intervention and prophylactic antibiotics.<br />

• Periorbital ecchymosis ("raccoon" eyes)<br />

• Retroauricular ecchymosis (Battle sign)<br />

• Hemotympanum or bloody discharge from the ear<br />

• Rhinorrhea or otorrhea<br />

require evaluation for nonaccidental trauma.<br />

• Cranial nerve deficits (I, II, VII, VIII); these signs may not develop for several hours after injury.<br />

ii. Commonly, the only radiographic sign of a basilar skull fracture is a blood-air interface in<br />

the sphenoid sinus; CT scanning (with thin cuts through the basilar skull) is indicated (but<br />

may not demonstrate the fracture).<br />

(a) May be seen from the ear or nose; associated with intracranial air in 20% of cases<br />

(b) Diagnostic evaluation (testing for CSF rhinorrhea)<br />

i. Place a drop of the fluid on filter paper; a clear halo ring suggests CSF.<br />

ii. Send a sample to the laboratory for glucose determination; if >30 mg/dL, CSF is present. (This<br />

test is accurate only if the fluid is clear; bloody fluid may yield a false-positive result).<br />

(c) The major complication is development of a CNS infection.<br />

(d) Most CSF leaks resolve spontaneously with conservative management. However, as many as 1 0%<br />

of patients develop a recurrent post-traumatic CSF leak that increases the risk of delayed bacterial<br />

meningitis.<br />

(e) Neurosurgical consult is indicated.<br />

3. Complications of head injuries<br />

a. Immediate (within 2 days)<br />

(1) Seizures: caused by traumatic neuronal depolarization; associated with low overall morbidity<br />

(2) "Talk and Deteriorate" syndrome<br />

(a) Most frequent neurologic findings are altered mental status and focal deficits.<br />

(b) Most frequent cause is epidural hematoma.<br />

b. Early (usually within 1 week)<br />

(1) Seizures (may indicate progressive injury)<br />

(2) Disseminated intravascular coagulation<br />

(3) Pulmonary edema (neurogenic)<br />

(4) Diabetes insipidus (loss of normal antidiuretic hormone secretion)<br />

(5) CSF leak<br />

(6) Cardiac dysrhythmias<br />

c. Late (after 1 week)<br />

(1) Delayed post-traumatic epilepsy<br />

(a) Acute intracranial hematomas<br />

(b) Depressed skull fractures<br />

(2) Recurrent CSF leak<br />

(3) Hydrocephalus<br />

(4) Subdural hygroma<br />

d. Postconcussion syndrome: symptoms may be noted on initial examination of the patient (or may appear<br />

later, eg, 1 week after the injury) and may persist for weeks, months, or years.<br />

(1) Headache (which may be severe) is the most common complaint.<br />

(2) Other signs/symptoms: irritability, insomnia, dizziness, anxiety, depression, fatigue, visual complaints,<br />

hearing problems, loss of memory, inability to concentrate, and sensitivity to alcohol<br />

e. Post-traumatic epilepsy<br />

(1) A high incidence is associated with<br />

(a)<br />

lntracranial hematoma<br />

(b) Depressed skull fracture<br />

(c) Seizures occurring within the first week of head injury<br />

313


TRAUMATIC DISORDERS<br />

4. Management<br />

(2) Anticonvulsant therapy<br />

(a) There is evidence that phenytoin reduces the incidence of seizures within the first week of injury,<br />

but not thereafter.<br />

(b) No consensus on best first-line antiepileptic agent for long-term management.<br />

a. ABCs and immobilize the cervical spine; measure pulse oximetry, analyze a rhythm strip, and measure<br />

bedside glucose level; all patients with a GCS :S8 should be intubated for airway protection.<br />

b. Assess the patient's level of consciousness using the GCS. A score of :S8 is associated with severe head injury;<br />

a score of 9-13 is associated with moderate brain injury; a score ?:14 is associated with mild head injury.<br />

Table 19: Glasgow Coma Scale<br />

Eye opening response<br />

Best verbal response<br />

Best motor response<br />

Spontaneous<br />

To voice<br />

To pain<br />

None<br />

Oriented<br />

Confused<br />

Inappropriate words<br />

Incomprehensible sounds<br />

None<br />

Obeys command<br />

Localizes pain<br />

Withdraws (pain)<br />

Flexion (pain)<br />

Extension (pain)<br />

None<br />

4<br />

3<br />

2<br />

5<br />

4<br />

3<br />

2<br />

6<br />

5<br />

4<br />

3<br />

2<br />

c. Assess pupillary function: both equality and response to light. The presence of a sluggishly reactive or fixed<br />

and dilated pupil is suggestive oft ICP and uncal herniation.<br />

d. In patients with a decreased level of consciousness, exclude other potential causes (hypoglycemia,<br />

hypoxia, drug overdose, sepsis, etc); administer oxygen, thiamine, D50W, and naloxone, as clinically<br />

indicated. Avoid administration of glucose to head-injured patients who are not hypoglycemic.<br />

e. Consider cervical spine injury if any of the following is present:<br />

(1) Unconsciousness or intoxication<br />

(2) Cervical pain or tenderness<br />

(3) Neurologic deficits<br />

(4). Facial fractures (especially LeFort II or Ill)<br />

(5) Mechanism of injury that can result in cervical injury<br />

f. Search for associated injuries; the finding of subhyaloid hemorrhages (preretinal bleeding) in children is<br />

suggestive of child abuse (shaking injuries or other trauma) or a bleeding disorder.<br />

g. Obtain CT scan and neurosurgical consult as clinically indicated.<br />

h. If signs oft ICP (eg, dilated pupil) are present:<br />

(1) Elevate head 30°.<br />

(2) Ventilate to a PaCO 2<br />

of 35 mmHg.<br />

(3) Obtain emergent CT and neurosurgical consult.<br />

1. Frequently reassess the patient's neurologic status (especially level of consciousness) looking for signs<br />

of evolution.<br />

314


TRAUMATIC DISORDERS<br />

B. Facial injuries (extensive facial injuries may necessitate a surgical airway)<br />

1. Midface fractures: the LeFort classification<br />

a. LeFort I: transverse through the maxilla above the roots of the upper teeth<br />

b. LeFort II: extends up into the midface and includes the maxilla, nasal bridge, lacrimal bones, orbital floors,<br />

and orbital rims<br />

c. LeFort Ill (craniofacial dissociation): extends from nasal bridge posteriorly along the medial and inferior<br />

orbital walls and laterally through the lateral orbital wall and zygoma; fractures extend back through the<br />

nasal bones to the sphenoid base; often associated with CSF leaks<br />

Le Fort Ill<br />

Le Fort II<br />

Le Fort I<br />

Le Fort I<br />

(Only Maxilla Moves)<br />

Le Fort II<br />

(Maxilla and Nose Move)<br />

Le Fort Ill<br />

(Entire Face Moves)<br />

2. Orbital wall fractures: orbital "blowout" fractures most commonly occur through the floor of the orbit but<br />

may also occur medially in the region of the lamina papyracea of the ethmoid bone.<br />

a. Orbital floor fractures<br />

(1) Clinical presentation<br />

(a) Pain and diplopia on upward gaze<br />

(b) Enophthalmos<br />

(c) Hypesthesia in the distribution of the infraorbital nerve (ipsilateral cheek and lip)<br />

(d) Limitation of upward gaze due to entrapment of the inferior rectus and inferior oblique muscles<br />

(e) Subcutaneous orbital emphysema<br />

(2) Radiographic findings (CT is imaging modality of choice; however, if plain radiographs are used,<br />

fractures are best seen on occipitomental, or Waters' view)<br />

(a) Air-fluid (blood) level in maxillary sinus<br />

(b) Prolapse of orbital tissue into the maxillary antrum ("tear-drop" sign)<br />

(c) Bony disruption of the orbital floor<br />

(d) Clouding of the maxillary sinus<br />

(e) Orbital emphysema<br />

315


TRAUMATIC DISORDERS<br />

b. Medial wall fractures<br />

(1) Clinical presentation<br />

(a) Epistaxis<br />

(b) Emphysema of the lids or conjunctiva<br />

(c) Limitation of lateral gaze due to entrapment of the medial rectus muscle (uncommon)<br />

(2) Radiographic findings (CT imaging)<br />

c. Management<br />

(a) Unilateral clouding of the ethmoid sinus<br />

(b) Orbital emphysema<br />

(1) Exclude associated ocular injuries (eg, rupture of the globe, hyphema), which occur in up to 30% of<br />

patients with these fractures; CT scanning may be helpful.<br />

(2) Prescribe decongestants and consider prophylactic antibiotics.<br />

(3) Advise patients to avoid Valsalva maneuvers such as blowing nose, which can worsen orbital emphysema.<br />

(4) Refer patients to an ophthalmologist and/or ENT surgeon or oral maxillofacial surgeon for follow-up;<br />

injuries usually heal without operative intervention, but surgical management is sometimes needed.<br />

(For further discussion of these fractures, see page 132 in HEENT Disorders.)<br />

3. N asa I fractures<br />

a. Imaging of clinically fractured nasal bones is rarely necessary.<br />

b. If all the following criteria are met, imaging is unlikely to affect urgent management, and patient may<br />

follow-up with an appropriate subspecialist in several days.<br />

(1) Tendernesss and swelling isolated to nasal bridge<br />

(2) Can breathe through both nares<br />

(3) No septal deviation<br />

(4) No septal hematoma<br />

4. Zygoma fractures<br />

a. "Tripod" fracture: involves zygoma, lateral orbital wall, and maxilla; usually caused by direct blow<br />

b. Displaced fractures usually require surgical repair<br />

5. Mandibular fractures<br />

a. Simple plain films poorly sensitive<br />

b. Imaging modality of choice: panoramic radiograph or CT<br />

c. Traditionally taught that mandible fractures occur in pairs, but more recent reviews suggest that single<br />

breaks relatively common<br />

6. Dental fractures: generally require antibiotics, analgesics, and referral<br />

a. Ellis classification: I= enamel only, II = enamel and dentin, Ill= enamel, dentin, and pulp<br />

b. Ellis Ill fractures generally require definitive follow-up and management within 48 hours of injury.<br />

C. Adult spinal trauma<br />

1. Spinal cord injury<br />

Table 20: Classification of Cervical Spine Injuries by Mechanism of Injury and Stability<br />

Injury Mechanism Stability<br />

Anterior subluxation Flexion Potentially unstable<br />

Bilateral facet dislocation Hexion Always unstable<br />

Simple wedge fracture Hexion Stable<br />

Clay-shoveler's fracture (C6, C7, T1) Flexion Stable<br />

Hexion teardrop fracture Flexion Extremely unstable<br />

Atlanto-occipital dislocation Flexion Unstable<br />

Anterior atlantoaxial dislocation Flexion Unstable<br />

Odontoid (C2) fracture Flexion/varied Unstable<br />

316


TRAUMATIC DISORDERS<br />

Table 20: Classification of Cervical Spine Injuries by Mechanism of Injury and Stability (continued)<br />

Injury Mechanism Stability<br />

Unilateral facet dislocation (C1-C2) Flexion-rotation Potentially unstable<br />

Rotatory atlantoaxial dislocation Flexion-rotation Unstable<br />

Pillar fracture Extension-compression Stable<br />

Jefferson fracture (burst of C1) Axial loading Extremely unstable<br />

Posterior atlantoaxial dislocation Extension Unstable<br />

Hyperextension-dislocation Extension Unstable<br />

Extension teardrop fracture Extension Unstable in extension<br />

C1 posterior neural arch Extension Potentially unstable<br />

Hangman fracture (C2) Extension Unstable<br />

Hyperextension-fracture/dislocation Extension Unstable<br />

Table 21: Memory Aid for Unstable Cervical Spine Fractures<br />

Letter<br />

Named Fracture<br />

Details<br />

Iefferson<br />

.B_it<br />

Qff<br />

A<br />

Hangman<br />

Ihumb<br />

Jefferson fracture<br />

Bilateral facet dislocation<br />

Odontoid<br />

Any atlanto-occipital<br />

Hangman fracture<br />

Teardrop fracture<br />

Burst fracture of C1<br />

Flexion injury<br />

C2 fracture: type II, type II unstable; type I (tip of the<br />

odontoid) is stable<br />

Flexion and flexion/rotation<br />

Bilateral C2 pedicle fracture (leads to C2 displacing<br />

anteriorly on C3)<br />

Flexion teardrop fracture is extremely unstable<br />

a. Incomplete cord lesions<br />

(1) Central cord syndrome<br />

(a) Usually occurs in older patients with spondylosis or congenital stenosis of the spinal canal<br />

whose necks are subjected to forced hyperextension injuries<br />

(b) Hyperextension - vascular ischemia of central cord (spinothalamic and corticospinal tracts) -<br />

weakness greater in arms than legs and variable loss of bladder control<br />

(c) Management is generally nonoperative.<br />

(d) Prognosis is excellent.<br />

Corticospi na I tract<br />

Spinothalamic tract<br />

317


TRAUMATIC DISORDERS<br />

(2) Anterior cord syndrome<br />

(a) Results from cervical flexion injuries<br />

(b) Flexion ----;, impingement of bony fragments on the spinal canal, cord contusion, or compression<br />

of the anterior spinal artery - loss of anterior cord function (complete motor paralysis and loss<br />

of pain and temperature sensation distal to the lesion) and preservation of posterior column<br />

function (vibration, light touch, position sense)<br />

(c) Immediate neurosurgical consult and emergent MRI or CT myelography<br />

Corticospinal tract<br />

Spinothalamic tract<br />

(3) Brown-Sequard syndrome<br />

(a) Usually results from a penetrating injury (gun shot or knife wound) causing hemisection of the<br />

spinal cord<br />

(b) Hemisection of the spinal cord----;, ipsilateral motor paralysis, loss of proprioception, and vibratory<br />

sensation with contralateral loss of pain and temperature sensation<br />

Corticospinal tract<br />

Spinothalamic tract<br />

b. Clinical presentation<br />

(1) Suspect spinal cord injury in:<br />

(a) Patients with an altered or depressed mental status (including intoxicated patients with head or<br />

facial injuries)<br />

(b) Patients with any focal neurologic deficits<br />

(c) Significant mechanisms of trauma (especially high-speed motor vehicle collisions, falls, football<br />

injuries, and diving accidents)<br />

(d) Patients with unexplained hypotension and associated paradoxical (relative) bradycardia<br />

(spinal shock)<br />

(e)<br />

Elderly patients with suspected minor traumatic injuries<br />

(2) Examination findings highly suggestive of cervical cord injury<br />

(a) Flaccid areflexia<br />

(b) Loss of rectal sphincter tone<br />

(c) Diaphragmatic abdominal breathing<br />

(d) Priapism<br />

(e)<br />

Hypotension and relative bradycardia (spinal shock)<br />

(3) Physical examination<br />

(a) Neurologic examination should focus on identifying focal deficits and attempting to localize the<br />

injury to a specific spinal cord level.<br />

1. Sensory deficits should be localized to a specific dermatome if possible (Table 22).<br />

• Occipital sensation ----;, C2<br />

318


TRAUMATIC DISORDERS<br />

• The hold-up position (with the hands half closed) or the hands and arms flexed at the<br />

chest - C6<br />

• "Burning hands" syndrome (especially in football players) -<br />

11. Testing specific muscle groups may help localize injury (Table 22).<br />

iii. A strong handshake indicates an int act spinal cord down to Tl.<br />

C6-C7<br />

Table 22: Spinal Root Muscle and Sensory Distribution<br />

Root Motor Sensory<br />

C3 Diaphragm, trapezius Lower neck to clavicle<br />

C4 Diaphragm Clavicular area<br />

cs Biceps, deltoid, diaphragm Lateral arm at or above the elbow<br />

C6 Biceps Forearm and radial side of the hand<br />

C7 Triceps Middle finger<br />

cs Finger flexors Lateral aspect of hand<br />

Tl Hand intrinsics Medial side of forearm<br />

T4<br />

T12<br />

Anterior chest wall at level of nipples<br />

Abdominal wall at level of umbilicus<br />

L2-L3<br />

L3-L<br />

Hip flexion<br />

Knee extension<br />

LS<br />

Lateral lower leg and large toe<br />

S1 Ankle planter flexion Small toe and sole of foot<br />

(b)<br />

Other findings<br />

i. Anterior cord function is assessed by testing motor strength, pain, and temperature sensation.<br />

ii. Posterior cord function is assessed by testing light touch, position, and vibratory sensation.<br />

iii. Anal wink (bulbocavernous reflex): squeezing the penis or clitoris causes contraction of the<br />

anal sphincter; this is the last reflex to disappear in patients with spinal shock or a complete<br />

cord lesion and the first to return with improvement.<br />

Iv. Sacral sparing: preservation of sensation at the sacrum in a patient with a spinal cord injury (a<br />

good prognostic sign)<br />

v. Neurogenic shock (loss of sympathetic tone): hypotension associated with bradycardia, warm<br />

pink skin, and adequate urine output<br />

vi. Spinal shock: loss of motor and/or sensory function along with initial loss of reflexes below<br />

the level of spinal injury (differentiate from neurogenic shock)<br />

vii. Horner syndrome: unilateral ptosis, miosis, anhydrosis; indicates injury to the cervical<br />

sympathetic chain at the level of C7-T2.<br />

c. Diagnostic evaluation<br />

(1) Cervical spine radiographs<br />

(a) Cross-table lateral, open-mouth odontoid, and AP views have a high but imperfect diagnostic<br />

yield for fractures.<br />

(b) Reliance on the cross-table lateral view alone is unacceptable; 10%-15% of fractures, particularly<br />

those involving C7, are not visualized on this view.<br />

(c) "Swimmer's" view (lateral view with axial traction on the arms) or a transaxillary view may be<br />

necessary to visualize C7 and the C7-Tl i nterspace.<br />

(d) Pseudosubluxation of C2 on C3 (


TRAUMATIC DISORDERS<br />

(f)<br />

Flexion-extension radiographs: in the awake, alert patient, these may be helpful in evaluating for<br />

vertebral column ligamentous instability; rarely useful in acute injuries because associated softtissue<br />

injuries limit the patient's ability to adequately flex and extend neck.<br />

(2) CT scan: initial technique of choice in the evaluation of acute cervical injury in most emergency<br />

departments.<br />

(3) MRI: a useful examination when ligamentous or cord injury is suspected<br />

d. Cervical spine "clearance": refers to determination that a trauma patient is at very low risk of cervical spine<br />

injury and, therefore, radiographic imaging is not necessary.<br />

(1) Clinical decision rules to identify patients at very low risk of cervical spine injury in whom<br />

radiographic imaging is not necessary<br />

(a) NEXUS criteria: no midline cervical tenderness, no focal neurologic deficits, no intoxication, no<br />

abnormal sensorium/alertness, no distracting injuries<br />

(b) Canadian cervical spine rules: absence of any high-risk characteristics (age >65 years old,<br />

dangerous mechanism, extremity paresthesias) and presence of any low-risk characteristics<br />

(simple rear-end motor vehicle collision, ambulatory at scene, upright position in emergency<br />

department, no midline cervical tenderness, delayed onset of neck pain)<br />

(2) Patients with negative imaging (plain radiographs or CT scan) but persistent midline tenderness,<br />

abnormal sensorium, or abnormal neurologic examination should be maintained in a rigid cervical<br />

collar until more advanced imaging (MRI or flexion/extension radiographs) may be performed or until<br />

patient can be clinically evaluated by above criteria.<br />

e. Management<br />

(a) For the comatose trauma patient with normal CT imaging of the cervical spine, there is<br />

controversy on how to "clear" the cervical spine; some argue a normal CT is sufficient, while<br />

others advocate for MRI before removing collar.<br />

(1) Patients with either suspected or confirmed cervical spine injuries should be immobilized with a<br />

rigid cervical collar and supine positioning; any body movement that may cause manipulation of the<br />

injured spine should be minimized.<br />

(2) Advanced airway management (if indicated): options include nasotracheal intubation (if the patient is<br />

breathing), careful orotracheal intubation with in-line manual cervical immobilization (video-assisted<br />

laryngoscopy may be helpful), and cricothyrotomy (if intubation attempts are unsuccessful).<br />

(3) Complete primary and secondary trauma surveys to identify and manage other injuries.<br />

(4) Obtain immediate neurosurgical consult.<br />

(5) High-dose methylprednisolone no longer routinely recommended. Data suggest limited improvement<br />

in motor function at cost of significant increased risk of complications (infection).<br />

(6) Indications for early surgery<br />

(a)<br />

Failure to achieve closed reduction in patients with acute cord injury who have incomplete<br />

sensorimotor loss<br />

(b) Failure to maintain reduction<br />

(c) Continued bony encroachment despite reduction<br />

(d) Successful reduction but continued compression demonstrated on myelography<br />

2. Vertebral spine trauma<br />

a. Immobilize the entire vertebral column until unstable injuries are excluded.<br />

(1) Obtain plain radiographs while the patient is immobilized.<br />

(a) When evaluating the cross-table lateral cervical spine radiograph, be sure to visualize all 7<br />

cervical vertebrae (count them!) and the C7-Tl interspace.<br />

(b) Do not rely on the cross-table lateral radiograph alone to exclude cervical fractures; open-mouth<br />

odontoid and AP views must also be obtained or CT performed.<br />

(c) Detection of a cervical spine injury on plain radiographs usually necessitates radiographic<br />

evaluation of the entire cervical spinal column with CT.<br />

(2) CT scan: first-line imaging in many emergency departments; indicated if plain radiographs are not<br />

clearly negative or if the patient has neurologic complaints or findings associated with "normal"<br />

radiographs.<br />

(3) MRI: not routinely necessary<br />

(a) May be indicated if the patient is obtunded (neurologic examination not possible) or has<br />

neurologic complaints or findings with normal plain radiographs or CT<br />

320


TRAUMATIC DISORDERS<br />

(b) Can detect the following injuries<br />

i. Spinal epidural hematoma<br />

ii. Herniated disc<br />

iii. Spinal cord contusion/disruption<br />

iv. Paraspinal ligamentous/soft-tissue injury<br />

(4) CT myelography may exclude the presence of acute spinal cord compression caused by a traumatic<br />

herniated disc or epidural hematoma. It is indicated only if MRI is unavailable or contraindicated.<br />

b. Immediate neurosurgical consult is required for patients with positive radiographs as well as those with<br />

neurologic complaints or findings, eg, sensory or motor deficits<br />

c. Cervical spine injury<br />

(1) Should be suspected in any patient with:<br />

(a) An injury above the clavicle<br />

(b) A high-speed vehicular injury<br />

(c) A fall from >10 feet<br />

(d) An electrical injury<br />

(e) An altered level of consciousness<br />

(f)<br />

A diving accident<br />

(g) A football injury sustained while tackling<br />

(2) Common bony injuries<br />

(a) Jefferson fracture (C1 ring blowout): an axial loading injury, best seen on open-mouth odontoid<br />

view; associated C2 fractures are seen in ~40% of these patients.<br />

(b) Odontoid fracture: look for swelling anterior to C2 on the crosstable lateral view and for<br />

abnormalities on the open-mouth odontoid view; additional views including tomograms and CT<br />

are occasionally needed. There are three types of odontoid fractures:<br />

i. Type I is rare (involves tip of the dens).<br />

ii. Type II traverses the dens at the junction of the body of C2 (worst prognosis).<br />

iii. Type Ill involves the vertebral body of C2.<br />

(c) Hangman fracture (unstable bipeduncular fracture of C2): an extension injury<br />

(d) Facet dislocations: may be either unilateral or bilateral; unilateral dislocations result from flexionrotation<br />

injuries, whereas bilateral dislocations are produced by flexion injuries. The lateral view<br />

reveals ?:25% (unilateral) to ?:50% (bilateral) anterior displacement of the superior vertebral body<br />

relative to the adjoining inferior vertebral body.<br />

(e) Clay-shoveler's fracture: an avulsion fracture of the spinous process of C6-T3 (C7 most common);<br />

results from a flexion injury or a direct blow to the spinous process<br />

(f)<br />

Flexion teardrop fracture: associated with significant disruption of posterior ligaments and anterior<br />

cord syndrome<br />

d. Thoracic and lumbar injuries<br />

(1) Should be suspected in patients sustaining multiple traumatic injuries, particularly if the trauma was to<br />

the trunk and the patient complains of pain at these sites or has neurologic findings or complaints.<br />

(2) In patients undergoing chest and/or abdominal helical CT scanning, screening for thoracic or lumbar<br />

spine fractures can be performed on reformatted CT data, obviating the need for plain radiograph<br />

screening.<br />

(3) Specific injuries<br />

(a) Chance fractures: transverse fractures through the vertebral body caused by flexion about an axis<br />

anterior to the vertebral column (most frequently seen after motor vehicle collisions in which the<br />

patient was restrained only by a lap belt); may be associated with retroperitoneal and abdominal<br />

visceral injuries. These are unstable fractures and require operative stabilization.<br />

(b) Fracture-dislocations: caused by extreme flexion or severe blunt trauma to the spine -<br />

of the posterior elements (pedicles, facets, laminae) of the vertebra<br />

i. Because the thoracic spinal canal is narrow in relation to the spinal cord, fracturesubluxations<br />

of the thoracic spine commonly result in complete neurologic deficits.<br />

disruption<br />

11. Chance fractures and fracture-dislocations are extremely unstable and usually require internal<br />

fixation.<br />

321


TRAUMATIC DISORDERS<br />

(c) Compression fractures<br />

1. Most common thoracolumbar fracture<br />

ii. Characteristics associated with spinal instability: loss of 50% of vertebral height, compression<br />

fractures at multiple levels, rotational component of fracture, and significant kyphosis (>30%)<br />

D. Pediatric vertebral and spinal cord injuries<br />

1. Relatively rare compared with adults<br />

2. Children


TRAUMATIC DISORDERS<br />

(d) Management<br />

i. Immediate needle thoracostomy of the affected hemithorax (insert large-bore IV catheter<br />

perpendicular to the chest wall in second intracostal space along midclavicular line)<br />

followed by tube thoracostomy<br />

ii. Alternatively, the catheter can be placed in the usual location of a chest tube (4th intercostal<br />

space anterior to the midaxillary line); this is easier to do at this level and equally effective.<br />

(2) Simple pneumothorax<br />

(a) Clinical presentation<br />

i. Signs/symptoms range from asymptomatic to severe respiratory compromise depending on<br />

severity.<br />

ii. Decreased breath sounds and hyperresonance to percussion on the affected side<br />

iii. Dyspnea, pleuritic chest pain<br />

(b) Imaging<br />

i. Expiratory, upright chest radiograph is more sensitive than standard inspiratory view.<br />

ii. Ultrasound may be used to detect pneumothorax quickly at bedside.<br />

(c) Management: tube thoracostomy<br />

(3) Open pneumothorax ("sucking chest wound")<br />

(a) If the diameter of the defect in the chest wall is approximately two-thirds or more of the<br />

diameter of the trachea, air peferentially moves through the chest wall defect rather than the<br />

trachea with each breath - ineffective ventilation - hypoxia.<br />

(b) Management<br />

i. Cover the defect with a sterile occlusive dressing (petrolatum gauze, plastic wrap); tape it<br />

down on three of its four edges.<br />

ii. Place a chest tube in an area distant from the wound site.<br />

b. Pulmonary contusion<br />

(1) Mechanism of injury: usually results from blunt chest wall trauma (eg, motor vehicle collision with<br />

rapid deceleration)<br />

(2) Clinical presentation includes dyspnea, tachypnea, tachycardia, and chest wall tenderness/<br />

ecchymosis at the site of injury.<br />

(3) Associated rib fractures may also be present (less commonly in children).<br />

(4) Arterial blood gases reveal hypoxemia and a widening alveolar-arterial oxygen gradient.<br />

(5) Chest radiograph findings<br />

(a) Localized to the site of injury<br />

(b) Range from patchy, irregular alveolar infiltrates to frank consolidation<br />

(c) May be delayed up to 6 hours<br />

(6) Management<br />

(a) The goal is adequate ventilation/oxygenation and perfusion to promote healing and prevent<br />

complications (eg, pneumonia, pneumothorax).<br />

(b) Mild to moderate contusions (28% total lung volume or more than one lobe) are more likely to be<br />

associated with significant hypoxemia (p0 2<br />

200 ml of fluid to be evident on chest radiograph<br />

(3) Management<br />

(a) Most hemothoraces are self-limited and need only tube thoracostomy.<br />

323


TRAUMATIC DISORDERS<br />

(b) Consider autotransfusion in patients with massive hemothoraces.<br />

(c) Thoracotomy is indicated for:<br />

i. Blood loss 2':1 1<br />

500 ml in the initial chest tube drainage<br />

ii. Persistent bleeding requiring serial blood transfusions<br />

iii. Refractory hypotension or cardiopulmonary decompensation<br />

iv. Blood loss >200 ml/hr for 2-4 hours<br />

v. 2':50% hemothorax<br />

3. Blunt cardiac injuries<br />

a. General principles<br />

(1) Usually result from high-speed blunt trauma to chest wall (eg, motor vehicle collisions, high-speed<br />

missile)<br />

(2) Associated with potentially fatal complications, including arrhythmias, wall motion abnormalities,<br />

anatomical ruptures, cardiogenic shock<br />

(3) Diagnosis<br />

(a) Cardiac enzymes have limited role in evaluation or management.<br />

(b) ECG is neither sensitive nor specific; however, presence of abnormalities on the initial ECG<br />

correlates with the risk of developing subsequent cardiac complications. Findings are variable<br />

and may include sinus tachycardia (most sensitive, least specific), premature ventricular<br />

contractions, atrial fibrillation, bundle-branch blocks (right bundle-branch block more<br />

common), ST-T wave changes.<br />

(c) 2-D echocardiographic findings: impaired regional systolic function (particularly right<br />

ventricular free wall dyskinesis), increased echogenicity ("brightness"), increased end-diastolic<br />

wall thickness<br />

(4) General management<br />

(a) Patients with conduction abnormalities (or other significant injuries) should be admitted to a<br />

critical care unit.<br />

(b) Asymptomatic patients with normal vital signs, a low-risk profile, and normal ECG may be<br />

released after a short period of observation.<br />

(c) Symptomatic patients with a low-risk profile and normal serial ECGs can probably be sent home<br />

after 6-12 hours of observation; if there is some concern after this time, consider ordering 2-D<br />

echocardiography.<br />

(d) Symptomatic patients with a moderate-risk profile and normal serial ECGs should have<br />

echocardiography performed; they may be sent home after a 12-hour observation period if the<br />

echocardiogram is clearly normal.<br />

(e) High-risk patients (or those with questionable findings on diagnostic evaluation) should be<br />

admitted for observation and cardiac monitoring, because they are at risk of developing<br />

dysrhythmias and cardiac dysfunction.<br />

b. Injury classification<br />

(1) Cardiac concussion (no permanent cell damage): commotio cordis<br />

(a) Mechanism of injury: a sharp, direct blow to the mid-anterior chest wall "stuns" the myocardium<br />

and results in a dysrhythmia<br />

(b) If the episode resolves (spontaneously or with treatment), there are no histopathologic changes.<br />

However, prolonged cellular dysfunction may lead to a nonperfusing rhythm (ventricular<br />

fibrillation, asystole) and irreversible cardiac arrest.<br />

(2) Cardiac contusion (permanent cell injury)<br />

(a)<br />

Mechanism of injury: high-speed deceleration - heart moves forward and forcibly strikes the sternum<br />

i. Injuries that occur during early systole or late diastole, when the ventricles are blood-filled,<br />

are the most damaging.<br />

ii. Right ventricle more commonly injured<br />

(b) Diagnosis may be difficult<br />

(3) Cardiac infarction (cell death)<br />

(a)<br />

Rare complication of blunt trauma<br />

(b) Mechanism of injury: usually results from coronary artery occlusion by arterial spasm, intimal tear,<br />

laceration, dissection, thrombosis, or compression from adjacent hemorrhage, and edema (which<br />

may occur with capillary bleeding in a contused heart); patients with preexisting coronary artery<br />

disease are at greatest risk.<br />

324


TRAUMATIC DISORDERS<br />

(c) Diagnosis: ischemic changes on ECG; increased cardiac enzymes (may be delayed)<br />

(d) Management: admission for cardiac monitoring and medical management of myocardial<br />

infarction<br />

(4) Cardiac tamponade (bleeding into the pericardium under pressure)<br />

(a) Penetrating trauma is the most frequent cause, but tamponade can also occur with blunt trauma<br />

or iatrogenic trauma (central venous pressure catheter placement, pacemaker insertion)<br />

(b) Mechanism of injury (in blunt injury): rapid deceleration with cardiac compression during early<br />

systole or late diastole when the ventricles are filled with blood and the myocardial wall is in the<br />

midst of contracting - rents/tears/lacerations in myocardial muscle - impending myocardial<br />

rupture - hemorrhage into pericardia! sac<br />

(c) Clinical presentation<br />

i. Beck's triad: hypotension, jugular venous distention, and muffled heart tones<br />

ii. Decreased pulse pressure<br />

iii. Rising central venous pressure (earliest response)<br />

iv. Kussmaul sign (distention of neck veins with inspiration)<br />

v. Pulsus paradoxus<br />

vi. A central venous pressure > 15 cm H 2<br />

0 in association with hypotension and tachycardia<br />

(most reliable sign)<br />

(d) Diagnostic evaluation<br />

i. Bedside echocardiography (FAST): preferred diagnostic test; presence of pericardia! fluid<br />

on ultrasound is highly suggestive of active bleeding in the pericardium in the setting of<br />

thoracic trauma.<br />

ii. ECG: electrical alternans is pathognomonic for tamponade but is rarely present in the acute<br />

scenario.<br />

iii. Pulseless electrical activity also occurs with tension pneumothorax, hypovolemia, etc.<br />

(e) Management<br />

i. IV fluids<br />

ii. Pericardiocentesis<br />

iii. Open thoracotomy by a qualified surgeon (definitive treatment)<br />

(5) Myocardial rupture (exsanguination)<br />

(a) Death usually ensues within seconds to minutes of injury at the scene.<br />

(b) Mechanism of injury: compression of a blood-filled chamber (usually a ventricle); most commonly<br />

results from blunt thoracic injury<br />

(c) The patient is protected from immediate exsanguination if the pericardium is intact (tamponade).<br />

(d) Clinical presentation: usually one of pericardia! tamponade in a patient with other known or<br />

suspected thoracic injuries (eg, hemothorax, pneumothorax, aortic dissection/rupture)<br />

(e) Diagnostic evaluation<br />

(f)<br />

i. Chest radiograph is almost always abnormal (nonspecific).<br />

ii. Bedside FAST reveals pericardia\ effusion/tamponade physiology.<br />

Management<br />

1. Fluid resuscitation<br />

4. Traumatic aortic rupture<br />

ii. Immediate decompression of cardiac tamponade by pericardiocentesis (temporizing measure<br />

only)<br />

iii. If the patient's clinical status deteriorates in the emergency department, emergency thoracotomy<br />

and pericardiotomy may be required before definitive repair in the operating room.<br />

a. Mechanism of injury: sudden deceleration (2':45 mph motor vehicle collision impact, damaged steering<br />

wheel or 20-30 foot fall)<br />

b. Site of injury: most ruptures occur at the ligamentum arteriosum (the point of greatest aortic fixation)<br />

just distal to the left subclavian artery and progress from the intima outward toward the adventitia.<br />

c. 80%-90% of patients die at the scene, and as many as 50% of remaining survivors die within 24 hours if<br />

not diagnosed and treated expeditiously.<br />

325


TRAUMATIC DISORDERS<br />

d. Clinical presentation<br />

(1) Retrosternal or interscapular pain (25%): most common<br />

(2) Dyspnea<br />

(3) Harsh systolic murmur over the precordium or in the interscapular area<br />

(4) Upper extremity hypertension in association with decreased or absent femoral pulses<br />

(pseudocoarctation syndrome)<br />

(5) Pulse deficits<br />

(6) Voice change or hoarseness (in the absence of laryngeal injury)<br />

(7) lschemic pain of the extremities<br />

(8) Paraplegia<br />

e. Diagnostic evaluation<br />

(1) Diagnosis is usually suspected from the history of a deceleration injury plus findings on chest<br />

radiograph (any, all, or none may be present):<br />

(a) Widening of the superior mediastinum (>8 cm) on an upright PA chest radiograph (most<br />

common finding)<br />

(b) Obliterated or indistinct aortic knob (most reliable finding)<br />

(c) Deviation of the trachea and/or esophagus (nasogastric tube) to the right<br />

(d) Depression of the left mainstem bronchus >40° below horizontal<br />

(e) Obliteration of the space between the pulmonary artery and the aorta<br />

(f) Left apical pleural cap (obliteration of the medial aspect of the left upper lobe apex)<br />

(g) Multiple rib fractures<br />

(h) Widening and/or displacement of the paratracheal stripe to the right<br />

(i) Widening of the left or right paraspinous stripe<br />

(j) Left hemothorax<br />

(k) Fractures of the first or second ribs or the scapula<br />

(2) Diagnosis is confirmed by:<br />

(a) Angiography is gold standard but invasive<br />

(b) Contrast-enhanced dynamic spiral CT is extremely accurate for demonstration or exclusion<br />

of direct aortic injury. If CT demonstrates traumatic aortic injury (or is not clearly negative),<br />

aortography should be performed.<br />

(c) Transesophageal echocardiography is a diagnostic alternative if both angiographic and CT<br />

findings are equivocal; however, results are highly operator-dependent and patient cooperation<br />

is required.<br />

f. Management<br />

(1) Prompt surgical repair<br />

(2) Medical management while awaiting surgery: goal is to minimize shear forces on intact adventitia of<br />

the aorta by decreasing both heart rate (


TRAUMATIC DISORDERS<br />

Table 23: Comparison of Diagnostic Peritoneal Lavage, Ultrasound, and CT Scan<br />

Diagnostic<br />

Study<br />

Indication<br />

Advantages<br />

Disadvantages<br />

Diagnostic<br />

peritoneal<br />

lavage<br />

Identify intraabdominal<br />

bleeding<br />

and/or signs of<br />

bowel perforation<br />

in hemodynamically<br />

unstable patients<br />

Early diagnosis<br />

Very sensitive<br />

Detects bowel<br />

injury (98%<br />

accurate)<br />

Invasive<br />

Does not detect injuries to diaphragm<br />

and retroperitoneum<br />

Less specific than ultrasound or CT<br />

May overdiagnose injuries<br />

Ultrasound/<br />

FAST<br />

Identify free intraabdominal,<br />

pelvic,<br />

pericardia! fluid in<br />

hemodynamically<br />

unstable or stable<br />

patients<br />

Early diagnosis<br />

Noninvasive<br />

Easily repeatable<br />

86%-97% accurate<br />

Performed rapidly<br />

Operator dependent<br />

Does not detect diaphragm, bowel,<br />

and retroperitoneal injuries<br />

Bowel gas and subcutaneous air<br />

distortion<br />

CT scan<br />

Identify solid or<br />

viscus organ injury<br />

in hemodynamically<br />

stable patients<br />

Most specific for<br />

injury (92%-98%<br />

accurate)<br />

Cost, time, and radiation exposure<br />

Less sensitive for diaphragm, bowel,<br />

and some pancreatic injuries than for<br />

solid-organ injuries<br />

Usually requires transporting patient<br />

out of emergency department<br />

3. CT<br />

b. Should be used in combination with other diagnostic modalities and clinical assessment<br />

(1) Unstable patient+ positive ultrasound_,, laparotomy<br />

(2) Stable patient + positive ultrasound _,, CT (to exclude the need for operative intervention)<br />

(3) Unstable patient+ negative ultrasound _,, evaluate for other etiologies of hemodynamic instability+<br />

serial repeat FAST examinations; or diagnostic peritoneal lavage<br />

(4) Stable patient+ negative ultrasound _,, observation or CT<br />

(5) A negative or equivocal ultrasound does not exclude significant injury or the need for further testing.<br />

c. Advantages<br />

(1) Noninvasive<br />

(2) Detects the presence of intra-abdominal, pericardia!, or pelvic fluid (extended FAST also detects<br />

pleural fluid)<br />

(3) Rapid, safe, and portable (can be performed in the controlled environment of the trauma suite) and<br />

may be repeated as needed<br />

(4) Can be performed without interfering with resuscitative efforts<br />

(5) Does not require administration of contrast agents<br />

(6) No ionizing radiation exposure<br />

(7) Sensitivity 60%-95%, and specificity nearly 100% (serial examinations increase sensitivity)<br />

d. Disadvantages<br />

(1) Can miss bowel and retroperitoneal injuries<br />

(2) Cannot differentiate fluids, eg, ascites versus blood<br />

(3) Presence of >200 ml fluid in intra-abdominal cavity usually necessary for positive examination.<br />

(4) Imaging is impaired in patients who are markedly obese and in those with subcutaneous air/<br />

emphysema or excessive bowel gas.<br />

(5) Operator-dependent<br />

(6) Less sensitive in evaluation of penetrating injuries<br />

a. The diagnostic study of choice for evaluating hemodynamically stable patients with blunt<br />

thoracoabdominal, genitourinary trauma, and/or suspected retroperitoneal injuries; often used in concert<br />

with the FAST examination (ie, initial evaluation with the FAST examination followed by CT)<br />

(1) Normal vital signs + normal ultrasound _,, CT may be deferred<br />

(2) Normal vital signs + positive ultrasound _,, CT may delineate extent of injuries<br />

327


TRAUMATIC DISORDERS<br />

b. Advantages<br />

(1) Noninvasive<br />

(2) Provides information on specific organ injury; when liver or spleen injury is suspected, CT may<br />

reliably exclude injuries that require emergent operative intervention (ACEP Clinical Policy).<br />

(3) Can diagnose retroperitoneal and pelvic organ injuries<br />

c. Disadvantages<br />

(1) Usually necessitates administration of IV and sometimes oral contrast<br />

(2) Has a rather high false-negative rate (2%-25%); misses certain injuries, eg, diaphragm, pancreas,<br />

bladder, and bowel<br />

4. Diagnostic peritoneal lavage<br />

a. Used to identify intra-abdominal bleeding or bowel injury that requires immediate laparotomy in the<br />

unstable patient if a FAST examination is not available or is inconclusive; it is rapidly performed, readily<br />

available, and has a sensitivity of 98%. No longer recommended as a first-line diagnostic assessment<br />

because of the high rate of comp I ications.<br />

b. Indications<br />

(1) In blunt abdominal trauma, this includes:<br />

(a) Negative or equivocal FAST examination in unstable patient with suspected or known blunt<br />

abdominal trauma<br />

(b) Unstable patient with lap belt sign with suspicion of bowel injury<br />

(c) Unexplained hypo- or hypertension<br />

(d) Unstable patient who is unavailable for serial examinations (ie, requires anesthesia for<br />

management of other injuries)<br />

(2) In penetrating trauma, this includes:<br />

(a) Stab wounds with known or suspected peritoneal violation<br />

(b) Gunshot wounds when peritoneal violation is unclear<br />

c. Contraindications<br />

(1) The only absolute contraindication to diagnostic peritoneal lavage is an indication for laparotomy (eg,<br />

free air under the diaphragm, intraperitoneal bladder rupture).<br />

(2) Relative contraindications<br />

(a) Previous abdominal surgery<br />

(b) Morbid obesity<br />

(c) Advanced cirrhosis (ascites)<br />

(d) Severe coagulopathy<br />

(e) Gravid uterus<br />

d. The major disadvantages of diagnostic peritoneal lavage are that it can miss significant retroperitoneal<br />

bleeding and isolated hollow viscus perforation.<br />

e. Procedure<br />

(1) Closed, semi-open, or open technique; if performed during pregnancy, the open technique (or minilap)<br />

should be done using a supraumbilical approach when the gravid uterus is palpable above the<br />

umbilicus.<br />

(2) Stomach and bladder are decompressed (nasogastric tube and Foley catheter) before diagnostic<br />

peritoneal lavage.<br />

(3) A peritoneal lavage catheter is then placed, and aspiration of free intraperitoneal blood attempted. If<br />

blood is aspirated, diagnostic peritoneal lavage is considered positive and is terminated.<br />

(4) If no blood is obtained, 1 L (or 10 ml/kg in children) warmed normal saline or lactated Ringer's is<br />

instilled into the peritoneal cavity and allowed to drain back out. This effluent is then sent to the<br />

laboratory for analysis.<br />

f. RBC criteria for a positive lavage<br />

(1) 2:100,000 RBCs/mm 3 for blunt trauma and anterior abdominal stab wounds<br />

(2) >5,000 RBCs/mm 3 for stab wounds of the lower chest and abdominal gunshot wounds<br />

(3) The RBC count is the most accurate parameter for evaluating lavage fluid. Other criteria (WBC count,<br />

amylase, bile) are less reliable and increase only after a delay of several hours.<br />

(4) Presence of food particulate is specific for hollow viscus perforation.<br />

328


TRAUMATIC DISORDERS<br />

G. Pelvic and extremity injuries<br />

1. Because pelvic injuries are frequently encountered in major blunt trauma cases, an AP pelvic radiograph is<br />

traditionally included as part of the standard trauma evaluation (along with lateral cervical spine and AP chest<br />

radiographs). In stable patients in whom CT imaging is anticipated, plain radiographs are not necessary.<br />

2. During primary survey, extremity evaluation is limited to identifying (and managing) exsanguinating<br />

hemorrhage and to assessing perfusion. It is only during the secondary survey that a more detailed evaluation<br />

of the extremities is performed.<br />

3. Fractures should be immobilized with splints and/or traction, as appropriate, to control pain and bleeding, as<br />

well as prevent further injury.<br />

4. After appropriate radiographic evaluation, dislocations should be rapidly reduced. Reduction should not be<br />

delayed if there are signs of compromised perfusion distal to the injury.<br />

5. Distal neurovascular function should be assessed promptly and reassessed before and after all extremity<br />

manipulations (eg, joint reduction, application of splints, and/or traction).<br />

6. If open wounds/fractures are identified, the patient's tetanus immune status should be updated as appropriate.<br />

7. Systemic antibiotics should be administered to patients with open fractures and contaminated wounds.<br />

8. Fractured bones bleed extensively. Be prepared to transfuse packed RBCs.<br />

9. Compartment syndromes can occur later, especially in the lower leg and forearm.<br />

a. Classic signs of a compartment syndrome (the "6 P's")<br />

(1) Pain out of proportion to the injury and increased with passive stretching and active contraction of<br />

the involved muscles<br />

(2) Paresthesias<br />

(3) Paralysis<br />

(4) Pallor<br />

(5) Palpable tenseness and tenderness of the compartment<br />

(6) Pulselessness (a very late finding)<br />

b. Pain is the earliest finding and is followed by the development of paresthesias. (For specific bony injuries<br />

and further discussion of compartment syndrome, see Orthopedic Emergencies, pages 343-390.)<br />

V. UNIQUE SITUATIONS<br />

A. Electrical injury<br />

1. Three major insults result from electrical injury.<br />

a. Conduction system changes secondary to damage of the body's electrical system<br />

(1) Cardiac conduction (asystole, ventricular fibrillation, or other cardiac dysrhythmias)<br />

(a) Ventricular fibrillation is the most common cause of death in the acute phase of electrical<br />

injuries.<br />

(b) Asystole is the most common witnessed arrhythmia in lightning strikes.<br />

(2) CNS conduction of electricity occurs because the body's electrical system is the preferential path for<br />

current conduction (least resistance).<br />

(a) Results in respiratory problems, apnea, seizures, etc<br />

(b) Patients may have paralysis of respiratory muscles despite a perfusing cardiac rhythm.<br />

(c) Patients may have dilated, nonreactive pupils; these findings do not necessarily reflect overall<br />

neurologic status or prognosis.<br />

b. Thermal tissue damage<br />

(1) The passage of current generates heat - cutaneous burns and muscle injury<br />

(a) Conversion of electrical energy to thermal energy is proportional to the resistance of the tissue,<br />

ie, higher resistance results in more heat.<br />

(b) Tissue resistance is greatest in bone, followed in order by skin, muscle, and nerves.<br />

(2) Thermal burns are proportional to the time of exposure.<br />

(a) Lightning strikes deliver an incredible amount of energy for a very short time period.<br />

(b) AC electrical injuries deliver much less energy for a longer period of time.<br />

(3) Muscle injury - rhabdomyolysis - acute myoglobinuric renal failure<br />

329


TRAUMATIC DISORDERS<br />

c. Blunt trauma<br />

(1) An electric shock can throw the victim down or into the air _,. fractures (particularly long-bone<br />

fractures and spinal compression fractures) and dislocations<br />

(2) An electric shock delivered at a frequency of 40-110 Hz (AC current) _,. tetanic contractions _,.<br />

scapular fractures and shoulder dislocations<br />

2. Classification<br />

a. Amount of voltage<br />

(1) Low voltage: 1,000 volts (high-tension power lines may be 100,000 volts; lightning may be in excess<br />

of 10 million volts)<br />

b. Type of electricity<br />

(1) AC: may cause repetitive muscle contractions and muscle tetany; may cause hand to grasp electrical<br />

source and prolong exposure<br />

(2) DC: generally causes a single muscle spasm that throws victim away from source; results in shorter<br />

duration of electrical current but more associated trauma from fall<br />

c. Type of injury<br />

(1) Direct injury: electrical current passes through the body, which acts as a circuit; there are usually<br />

entrance and exit wounds; the internal injuries are often much more severe than the skin wounds<br />

would suggest<br />

(2) Flash (arc) injury: current strikes the skin but does not enter the body; with flash lightning strikes, the<br />

classic finding is Lichtenberg figures (branching, "feathering" skin lesions)<br />

3. Management: focuses on ABCs: IV, oxygen, cardiac monitor, cervical spine precautions (if indicated)<br />

a. CPR should be initiated and continued despite prolonged periods of non-perfusing rhythms (usually<br />

ventricular fibrillation or asystole); cardiac arrhythmias should be managed according to standard ACLS/<br />

ATLS protocols<br />

b. Return of spontaneous circulation and good neurologic outcomes are more common in victims of electrical<br />

injuries.<br />

c. Remember: patients may have apnea from paralysis of respiratory muscles despite a perfusing rhythm.<br />

d. Remember: victims of lightning strikes may have dilated, nonreactive pupils; these findings do not<br />

necessary reflect neurologic status.<br />

e. Begin fluid resuscitation with normal saline or lactated Ringer's.<br />

(1) If there are signs of shock (hypotension/tachycardia, diaphoresis, altered level of consciousness), a fluid<br />

bolus (20 ml/kg) is indicated.<br />

(2) The rate should then be adjusted as needed to maintain a urine output of at least 0.5-1 ml/kg/hr.<br />

(3) If there is evidence of myoglobinuria, the rate should be increased until urine output is 1.5-2 ml/kg/hr.<br />

f. Obtain an ECG.<br />

g. Laboratory tests: creatine kinase and urinalysis for presence of myoglobin as well as serum potassium and<br />

calcium levels (to exclude hyperkalemia and hypocalcemia).<br />

h. Check carefully for entrance and exit wounds; most common entrance sites are the hand and skull; most<br />

common exit site is the heel.<br />

i. Evaluate patients for evidence of blunt trauma.<br />

j. Wound care: cleanse cutaneous burns, and apply a topical antibiotic dressing (eg, silver sulfadiazine,<br />

mafenide acetate, bacitracin); do not debride soft-tissue injuries involving the hands, digits, or face.<br />

k. Administer tetanus prophylaxis as needed.<br />

I. Monitor for compartment syndromes.<br />

m. Evaluate patients who sustain electrical injuries of the head and neck for traumatic cataracts, and refer them<br />

to an ophthalmologist for follow-up; cataracts may develop shortly after injury, or they may be delayed<br />

months to years.<br />

n. Lip commissure burns require close observation and referral to a plastic or oral surgeon; delayed<br />

hemorrhage (3-14 days after injury) from the labial artery occurs in 10%-15 % of these patients.<br />

4. Disposition<br />

a. Hospitalization is indicated for all patients with high-voltage burns(> 1,000 volts) and for those patients<br />

with low-voltage burns (


TRAUMATIC DISORDERS<br />

b. Asymptomatic patients with low-voltage injuries may be discharged to home after a period of observation<br />

and cardiac monitoring in the emergency department if their ECG and physical examination are normal<br />

and if they have no evidence of cutaneous involvement or urinary heme pigment.<br />

5. Important differences between electrical injuries and thermal burns<br />

a. Electrical injuries cause extensive muscle damage - myoglobin release (physiologically, they are more like<br />

a crush injury than a thermal burn)<br />

b. In electrical injuries, the extent of cutaneous injury does not necessarily correlate with the amount of<br />

underlying tissue damage (neurovascular and musculoskeletal); it is often just the tip of the iceberg. (For<br />

further discussion of electrical injuries, see the chapter on Environmental Disorders, page 633.)<br />

6. Lightning injuries: a small but important subset of electrical injuries; see the chapter on Environmental<br />

Disorders, page 633.<br />

B. Rhabdomyolysis<br />

1. Pathophysiology: injury to muscle - release of myoglobin - damage to kidneys - acute renal failure (90% of<br />

patients recover)<br />

2. Commonly associated electrolyte abnormalities are hypocalcemia, hyperkalemia, and hypouricemia.<br />

3. Serial measurements of the creatine kinase level are used to monitor progress.<br />

4. Management<br />

a. Aggressive IV fluid (normal saline or lactated Ringer's) hydration; administer fluids at a rate sufficient to<br />

maintain urine output at 1.5-2 ml/kg/hr.<br />

b. Unproven and controversial therapy options<br />

C. Thermal burns<br />

(1) Mannitol: aids diuresis, which may help prevent heme-induced renal damage; no clear evidence of<br />

usefulness<br />

(2) Alkalinization of the urine with sodium bicarbonate<br />

(a) In theory decreases renal toxicity of heme; however, there is no clear evidence to support routine<br />

use.<br />

(b) May be a role for alkalinization in severe rhabdomyolysis<br />

(3) Loop diuretics: may exacerbate electrolyte abnormalities; limited role in fluid-overloaded patients only<br />

1. Start with the ABCs. Airway patency and fluid resuscitation are of prime importance.<br />

2. Inhalation injury should be considered in patients with facial burns, singed facial and nasal hair, evidence of<br />

oropharyngeal inflammation, carbon deposits in the oropharynx, carbonaceous sputum, and/or a history of<br />

fire exposure in a confined space.<br />

a. Treat with humidified oxygen and early intubation.<br />

b. Nasopharyngoscopy can be used to determine the extent of injury and the presence of posterior<br />

pharyngeal or laryngeal edema; if negative, need for intubation may be obviated.<br />

3. Early intubation is indicated in patients with:<br />

a. Circumferential burns of the neck<br />

b. Suspected airway/inhalation injury<br />

4. Early burn resuscitation requires fluid replacement based on the percent of body burned.<br />

a. Estimate the percent of body surface area (BSA) burned (second- and third-degree burns only) using one of<br />

the following:<br />

(1) "Rule of Nines": head 9%, each arm 9%, chest, back, and each leg 18%, perineum 1 %<br />

(2) "Rule of Palms": surface area of victim's palm is -1 % of his or her BSA<br />

(3) Lund and Browder charts: more precise than the other two methods, particularly in children<br />

331


TRAUMATIC DISORDERS<br />

Rule of 9's<br />

Source: Dental Day (Wikispaces: http://dentalday.wikispaces.com/file/detail/ru1e_of_nines_2.jpg)<br />

b. There are many fluid replacement formulas; a general rule is to administer a crystalloid solution (eg,<br />

lactated Ringer's) in an amount equal to 2-4 ml/kg/% BSA burned/24 hours. Daily maintenance fluids<br />

should also be administered.<br />

(1) Parkland formula:% total BSA x weight (in kg) x 4<br />

(a) Give the first half of these fluids (plus maintenance fluid rate) over the initial 8 hours (from time<br />

of burn).<br />

(b) Give the second half of these fluids (plus maintenance fluid rate) over the next 16 hours.<br />

(2) Adjust fluid administration as needed based on urine output (goal 0.5-1 ml/kg/hr), heart rate, and<br />

mentation.<br />

For a more detailed discussion of burns and their management, see the chapter on Environmental<br />

Disorders, pages 633-669.)<br />

5. Indications for transfer to burn center (American Burn Association Criteria)<br />

a. ::::25 % total BSA partial-thickness burns (::::20% in children 40 years old)<br />

b. ::::10% total BSA full-thickness burns<br />

c. Burns involving eyes, ears, face, hands, feet, perineum, and any others that may result in significant<br />

cosmetic or functional disability<br />

d. Presence of associated major trauma or inhalation injuries<br />

e. High-voltage electrical burns<br />

D. Trauma in pregnancy<br />

1. Introduction<br />

a. Pregnant patients are fundamentally two patients.<br />

b. Trauma is the most common cause of nonobstetric maternal death during pregnancy.<br />

c. Because fetal survival depends entirely on maternal integrity, initial management should be directed at<br />

resuscitation and stabilization of the mother; prevention of maternal hypoxia and hypotension ensures the<br />

best outcome for both mother and fetus.<br />

d. Early obstetrical, pediatric, and trauma service consults should be obtained.<br />

2. Airway and breathing<br />

a. Use continuous pulse oximetry.<br />

b. Administer supplemental 100% oxygen by mask; because fetal blood functions in a lower portion of a<br />

left-shifted oxygen-hemoglobin dissociation curve, an increase in oxygen tension produces a significant<br />

increase in fetal blood saturation.<br />

332


TRAUMATIC DISORDERS<br />

c. If intubation is required in the breathing patient, standard rapid-sequence intubation with an induction<br />

agent (propofol, etomidate, ketamine, etc) and a neuromuscular blockage agent (succinylcholine,<br />

rocuronium) is recommended.<br />

d. Thoracostomy tube placement (when indicated) should be at the third or fourth intercostal space to avoid<br />

diaphragmatic injury.<br />

3. Circulatory status: hemodynamic parameters are misleading.<br />

a. An increased heart rate and decreased blood pressure may reflect the normal physiologic changes of<br />

pregnancy; flat, supine positioning (see below); or trauma-related shock.<br />

b. Total blood volume increases by 30%-50% during normal pregnancy.<br />

(1) As a result, a pregnant woman may lose 30%-35% of her blood volume (up to 1.5 L) before displaying<br />

clinical signs of hypovolemia.<br />

(2) Maternal blood loss results in decreased uterine blood flow and fetal hypoxia. Thus, the fetus may be<br />

in shock despite the presence of normal maternal vital signs.<br />

c. Supine hypotension syndrome: in flat, supine position, the gravid uterus compresses the inferior vena<br />

cava - decreased preload and cardiac output.<br />

(1) Prevention: patients >20 weeks gestation should be positioned on the left side. If a spinal injury is<br />

suspected, the backboard should be tilted 15 ° to the left.<br />

d. Lactated Ringer's solution is the preferred crystalloid fluid in late pregnancy; normal saline may cause<br />

hyperchloremic acidosis.<br />

4. Obstetric evaluation is conducted during the secondary survey and should include:<br />

a. Fundal height and tenderness<br />

b. Uterine contractions<br />

c. Fetal movement<br />

d. Fetal heart rate ( 160 beats per minute indicates fetal distress, which may be the first sign of<br />

maternal hemodynamic compromise)<br />

e. Inspection of the vulva and outer vaginal vault for blood and secretions (ferning and blue discoloration of<br />

nitrazine paper may aid in distinguishing alkaline amniotic fluid from urine)<br />

f. Pelvic (and rectal) examinations should be performed.<br />

(1) Perform a sterile speculum examination: inspect the cervix for dilation, effacement, and cloudy white<br />

or green fluid coming from the os (suggests prolapse of the umbilical cord, which is an obstetric<br />

emergency requiring immediate cesarean section).<br />

(2) Perform a bimanual examination (unless premature rupture of membranes is suspected) to determine<br />

fetal station, presence of contractions, or evidence of injury.<br />

5. Cardiotocographic monitoring: indicated in all pregnant trauma patients >20 weeks gestation; the fetus may be<br />

in jeopardy, even with minor maternal injury.<br />

a. Indications for immediate obstetrical consultation: uterine irritability, ruptured membranes, vaginal<br />

bleeding, abnormal fetal heart tones/rate (repetitive decelerations, absence of accelerations, beat-to-beat<br />

variability) or significant maternal injuries<br />

b. Usually 4-6 hours of monitoring is recommended (commonly performed in labor and delivery unit but<br />

should be initiated urgently in emergency department).<br />

6. A gestational age >24 weeks is considered to be viable. Estimate gestational age by reported history from<br />

mother if available, fundal height (uterine fundus >3-4 cm/2-3 fingerbreadths above the umbilicus), or<br />

ultrasonography (femur length or biparietal diameter).<br />

7. Traumatic uterine rupture (rare)<br />

a. Clinical presentation ranges from minimal signs and symptoms to maternal shock.<br />

b. Diagnosis<br />

(1) Ultrasonography is the diagnostic modality of choice; findings may include:<br />

(a) Disruption of the myometrium<br />

(b) Extra-uterine or free intraperitoneal fluid<br />

(c) Extra-uterine fetal anatomy<br />

(d) Anahydramnios<br />

(e) Fetal demise<br />

(2) Uterine rupture that is not identified on ultrasound may be diagnosed on MRI or CT.<br />

8. Placental abruption (abruptio placenta) is the most common cause of fetal death after blunt trauma. Late in<br />

pregnancy, abruption can occur after relatively minor injuries.<br />

333


TRAUMATIC DISORDERS<br />

a. Clinical presentation<br />

(1) External vaginal bleeding (absent in up to 30% of cases)<br />

(2) Abdominal cramps/pain<br />

(3) Uterine tenderness/rigidity<br />

(4) Expanding fundal height<br />

(5) Maternal shock<br />

(6) Fetal distress<br />

b. Diagnostic evaluation<br />

(1) Cardiotocographic monitoring (most sensitive modality)<br />

(2) Uterine ultrasonography is only 25 %-50% sensitive.<br />

c. Because abruption may cause disseminated intravascular coagulation, clotting studies should be performed.<br />

9. Fetomaternal hemorrhage may occur in even minor blunt abdominal trauma.<br />

a. In an Rh-negative woman carrying an Rh-positive fetus, this can result in isoimmunization if not detected<br />

and treated.<br />

b. Rh-negative women who sustain abdominal trauma during pregnancy should be given Rh immune<br />

globulin within 72 hours of possible antigenic exposure.<br />

(1) 12 weeks gestation: administer standard dose (300 mcg).<br />

(3) Kleihauer-Betke test is not indicated in the emergency department.<br />

10. Peri mortem cesarean section<br />

a. Restoration of maternal circulation and thus fetal circulation is the optimal management goal. However,<br />

prolonged maternal resuscitation without improvement in maternal circulation may jeopardize the life<br />

of a potentially viable fetus. In this situation, decision to perform a perimortem cesarean section must be<br />

considered.<br />

b. Goal is to deliver fetus within 5 minutes of loss of maternal vital signs. Beyond 20 minutes, there is virtually<br />

no chance of survival with favorable neurologic outcome for the fetus.<br />

11. Prevention: use of seat belts significantly decreases the risk of serious and fatal maternal injuries; properly<br />

worn, three-point restraints (lap and shoulder) are the safest.<br />

E. Penetrating trauma<br />

1. Most penetrating injuries result from gunshot and stab wounds (rather than impalements); all but obviously<br />

superficial penetrating wounds require surgical evaluation.<br />

2. Gunshot wounds<br />

a. Bullets and other high-speed missiles can ricochet internally and cause much more extensive internal<br />

damage than is visible externally.<br />

b. Gunshot wounds to the lower thorax may involve the abdomen as well. The intraperitoneal cavity extends<br />

from the fourth intercostal space anteriorly to the 6th or 7th intercostal space posteriorly and laterally.<br />

c. Gunshot wounds of the abdomen are associated with a high incidence of peritoneal cavity penetration<br />

and intraperitoneal injuries; most require immediate laparotomy.<br />

3. Stab wounds<br />

a. Stab wounds of the abdomen have a relatively low incidence of intraperitoneal injuries (when compared<br />

with gunshot wounds).<br />

(1) Hollow viscus injuries most common.<br />

(2) Management based on clinical presentation.<br />

(a) Extremis: consider resuscitative thoracotomy and emergent laparotomy.<br />

(b) Hemodynamic instability, evisceration, impaled foreign body (eg, knife blade still present in<br />

wound), obvious peritonitis, etc: emergent laparotomy<br />

(c) Patients with no indication for emergent laparotomy: local wound exploration by a surgeon<br />

<strong>combined</strong> with advanced diagnostics (serial examinations, FAST, plain radiographs, CT, diagnostic<br />

peritoneal lavage, laparoscopy) is appropriate.<br />

1. Extraperitoneal wound only: patient may be discharged home.<br />

ii. Peritoneal violation: immediate operative management vs close observation for signs of<br />

peritonitis; only one-half associated with intraperitoneal injury require operative management.<br />

334


TRAUMATIC DISORDERS<br />

b. Stab wounds of the neck<br />

(1) Surgical zones of the neck<br />

(a) Zone I: clavicle to cricoid<br />

(b) Zone II: cricoid to angle of mandible<br />

(c) Zone Ill: angle of mandible to base of skull<br />

----------- -E7 ---<br />

ZONEIII<br />

F. Blast injuries<br />

(2) Evaluation<br />

(a) All penetrating wounds of the neck require surgical consultation unless determined to be<br />

superficial.<br />

(b) Airway management is first priority.<br />

(c) Physical examination, ancillary studies (chest radiograph, CT, bronchoscopy,<br />

esophagogastroduodenoscopy or barium swallow, CT angiography or color Doppler ultrasound),<br />

and/or surgical exploration are mainstay of emergency department evaluation; institutional<br />

protocols vary, and there is no consensus approach.<br />

1. Types and mechanisms of injury<br />

a. Primary blast injuries<br />

(1) The result of a pressure wave (caused by an explosion) as it passes through the body. Distance from<br />

the blast source determines magnitude of exposure.<br />

(2) The ears, lungs, and Cl tract are the most susceptible to injury (tissues with air-fluid interfaces).<br />

Commonly associated clinical findings include:<br />

(a) Tympanic membrane rupture (most sensitive indicator of a primary blast effect)<br />

(b) Air emboli on funduscopic exam<br />

(c) Petechial hemorrhages in the nasopharynx<br />

(3) Damage mechanisms<br />

(a) Spallation: occurs when a pressure wave passes from a dense to a less dense medium; results in<br />

particles of fluid to be thrown from a dense medium and fragmented into a less dense medium as<br />

the pressure wave (eg, explosion in water causes water on surface to splash into air); this affects<br />

organs containing both liquid and gas, eg, lungs.<br />

(b) Implosion: compression of gas pockets are produced by the pressure wave (such as occurs in the<br />

ear causing tympanic rupture, hearing loss, etc).<br />

(c) Pressure differentials: refer to the difference in pressure that develops between the skin and<br />

internal organs at the moment of impact of the shock wave; in the lung, this pressure differential<br />

can contribute to pulmonary hemorrhage.<br />

b. Secondary blast injuries are the result of rapid acceleration of small fragments (flying debris, glass, etc)<br />

caused by the explosion.<br />

c. Tertiary blast injuries are the result of the victim's collision with a hard surface (frequently a lethal event).<br />

d. Quaternary blast injuries are all explosion-related injuries not classified above; includes the exacerbation of<br />

existing conditions, eg, chest pain in a patient with known coronary artery disease.<br />

e. Qui nary: delayed hyperinflammatory response with fever and tachycardia<br />

335


TRAUMATIC DISORDERS<br />

2. Specific injuries<br />

a. Head and neck are the areas most vulnerable to injury.<br />

(1) Brain injury (primarily subarachnoid hemorrhage)<br />

(2) Skull fracture<br />

(3) Ocular injury (common)<br />

(4) Auditory injury<br />

(a)<br />

Hearing impairment<br />

i. May be conductive, sensorineural, or both<br />

ii. Very common in bomb victims (which may lead to an inaccurate assessment because they<br />

are unable to hear verbal commands and respond appropriately)<br />

iii. If associated with brain injury, skull fracture, vertigo, or dizziness, suspect ossicular damage.<br />

(b) Tympanic membrane perforation<br />

1. Mechanisms of injury<br />

• Inertia and pressure differentials -<br />

• Spalling -<br />

ii. Usual site of injury -<br />

large punched-out perforations<br />

linear or small perforations<br />

the pars tensa of the tympanic membrane<br />

(5) Facial injury - spalling in the ethmoid region - anosmia<br />

b. Lungs<br />

(1) Pulmonary edema and hemorrhage (most common in the intercostal spaces) are the predominant<br />

findings<br />

(2) Earliest signs: cough, tachypnea, hypoxemia ("blast lung syndrome")<br />

(3) ARDS may occur 24-48 hours after the blast.<br />

(4) Airway management<br />

(a)<br />

Unassisted ventilation (with the patient on oxygen) is best.<br />

(b) Assisted ventilation (with the patient intubated) carries a risk of tension pneumothorax; in<br />

addition, therapy with positive end-pressure ventilation should be avoided, because there is<br />

potential risk of air embolism associated with its use in these patients.<br />

(5) General anesthesia should be delayed 1-2 days (if possible\ because it is poorly tolerated in patients<br />

with primary blast injury; however, regional or spinal anesthesia may be used.<br />

c. GI tract<br />

G. Additional pearls<br />

(1) Air-containing organs (stomach, duodenum, colon) as well as solid viscera (liver, spleen, diaphragm)<br />

may rupture.<br />

(2) Delayed perforations may also occur (especially in the ileocecal region).<br />

(3) Intestinal injuries are common in underwater blasts.<br />

1. The spleen is the most common abdominal organ injured in blunt trauma, followed by the liver.<br />

2. The liver is the most common abdominal organ injured in penetrating trauma, followed by the small bowel.<br />

3.<br />

4.<br />

5.<br />

6.<br />

7.<br />

8.<br />

9.<br />

10.<br />

The most common cause of sudden death after a motor vehicle collision or fall from height is traumatic aortic<br />

rupture.<br />

A sternal fracture or chest wall contusion or first/second rib fractures may be associated with a potentially more<br />

serious injury, eg, myocardial contusion, traumatic aortic dissection.<br />

Pelvic fractures are associated with bladder injury; check for hematuria and do a urethrogram/cystogram if<br />

appropriate.<br />

Posterior knee dislocations may be associated with injury to the popliteal artery and/or nerve; arteriography<br />

may be indicated (in conjunction with surgical consult) even if pulses are present. If skilled in joint reduction,<br />

attempt one gentle reduction maneuver.<br />

Solitary lap belts are associated with small-bowel injuries and mesenteric lacerations in addition to Chance<br />

fractures.<br />

Up to 20% of patients with severe renal trauma do not have hematuria; if there is a high index of suspicion,<br />

perform a CT scan.<br />

The most common abdominal injury likely to present >24 hours after injury is a contused bowel (eg, duodenal<br />

hematoma); initial CT may be normal.<br />

Serial abdominal reexaminations of the trauma patient are important.<br />

336


TRAUMATIC DISORDERS: PRACTICE CLINICAL SCENARIOS<br />

TRAUMATIC DISORDERS: PRACTICE CLINICAL SCENARIOS<br />

Answers immediately follow the practice clinical scenarios.<br />

Scenario A<br />

Presentation: A patient has sustained blunt thoracic trauma and presents with tachypnea and retractions.<br />

Physical examination: Pulse oximetry is 90% on 100% oxygen. Primary survey reveals a patent airway<br />

but decreased breath sounds on the affected side. The trachea is deviated to the contralateral side. The<br />

ipsilateral chest is hyperexpanded and hyperresonant on percussion.<br />

What is the diagnosis?<br />

Scenario B<br />

Presentation: A 30-year-old man sustains a closed head injury from a 30-foot fall. He had a brief period<br />

of loss of consciousness immediately after the fall. He is now lucid but deteriorating neurologically. He<br />

progressively develops confusion, decreased level of consciousness, and coma.<br />

What is the diagnosis?<br />

Scenario C<br />

Presentation: A patient arrives at the emergency department after a high-speed motor vehicle collision in<br />

which he was unrestrained and possibly ejected from the vehicle. He is tachycardic and hypotensive.<br />

Physical examination: The airway is secured, and no obvious signs of thoracic injury are found. A FAST<br />

exam reveals fluid in Morison's pouch.<br />

What is the diagnosis?<br />

Scenario D<br />

Presentation: An elderly gentleman who is on supplemental oxygen at home presents after sustaining a<br />

flash burn while trying to light his cigarette. He is awake, alert, and talking, and has partial-thickness burns<br />

to his entire face. His nose hairs are singed, and there is soot in his oropharynx. His oxygen saturation is<br />

95% on a nonrebreathing mask.<br />

What is the diagnosis?<br />

Scenario E<br />

Presentation: A 14-year-old presents with abdominal and back pain after a motor vehicle collision in<br />

which he was wearing only a lap belt, and the car rear-ended another vehicle at high speed.<br />

Physical examination: The patient is hemodynamically stable. On examination, there is a linear<br />

ecchymotic bruise or abrasion across the mid-abdomen, and the abdomen is tender. There is point<br />

thoracolumbar midline tenderness.<br />

What is the diagnosis?<br />

337


TRAUMATIC DISORDERS: PRACTICE CLINICAL SCENARIOS<br />

Scenario F<br />

Presentation: A 35-year-old homeless man presents to the emergency department via EMS shortly after<br />

being struck by a motor vehicle while he was attempting to cross the street.<br />

Physical examination: The patient is agitated, combative, and smells of alcohol. He has gurgling upper<br />

airway sounds and shallow but equal breath sounds. He is moving all of his extremities but is not following<br />

commands. His heart rate is 110 beats/minute, blood pressure is 100/60 mm Hg, respiratory rate is 10<br />

breaths/minute, and SpO 2<br />

is 100% on 100% Fi 0 2<br />

•<br />

What is the most appropriate first step in management of this patient?<br />

Scenario G<br />

Presentation: A 3-year-old girl presents to the emergency department via EMS after a motor vehicle collision.<br />

She was an unrestrained rear-seat passenger in her mother's lap when the vehicle hit a tree at 35 mph.<br />

Physical examination: The child is awake and alert but crying and agitated. Her breath sounds are clear,<br />

and her SpO 2<br />

is 100%. Her heart rate is 190 beats/minute, systolic blood pressure is 80 mm Hg, and<br />

capillary refill time is 4-5 seconds.<br />

What is the diagnosis?<br />

Scenario H<br />

Presentation: A 24-year-old G2P1 woman who reports that she is approximately 10 weeks pregnant<br />

presents to the emergency department reporting that she was the victim of domestic violence. She reports<br />

that her significant other kicked and punched her in the chest, back, and abdomen, and threw her down a<br />

flight of stairs. She complains of scattered painful bruises but has no other specific complaints and denies<br />

sexual assau It.<br />

Physical examination: The patient is tearful but in no distress. She is ambulatory. Primary survey does not<br />

reveal any life-threatening injuries. Her vital signs are normal. Secondary survey reveals scattered contusions<br />

but no other serious injuries. A social worker and a police detective are also present at the bedside.<br />

In addition to ensuring that the patient has a safe social environment, what other pregnancy-related<br />

considerations are appropriate?<br />

Scenario I<br />

Presentation: A 21-year-old man presents to the emergency department after being assaulted and<br />

robbed. He reports that he was stabbed in the abdomen with an unknown sharp object. He complains of<br />

abdominal pain but has no other complaints.<br />

Physical examination: The patient is brought back to an examination room in a wheelchair. He is awake,<br />

alert, and speaking. The primary trauma survey reveals normal airway, breathing, and circulation. His<br />

initial vital signs are normal. However, after the patient completely undresses, a 3-cm stab wound is seen<br />

on the anterior abdominal wall just superior to the umbilicus. There is evisceration of bowel loops through<br />

the wound. Complete secondary trauma survey is unremarkable.<br />

What is the appropriate management of this patient?<br />

ScenarioJ<br />

Presentation: A 30-year-old man presents to the emergency department via EMS after a motorcycle<br />

accident. He lost control on an icy road. He was wearing a helmet and did not lose consciousness. He<br />

complains of left leg pain only.<br />

Physical examination: The patient is fully immobilized with a cervical collar and long back board. He is<br />

awake, alert, and talking. This primary survey is normal. His initial vital signs are normal. Secondary survey<br />

reveals a swollen, tender left lower leg (mid-tibia region) but no obvious deformity. He complains of severe<br />

pain in his lower leg, foot, and toes. He also complains that his toes are tingling.<br />

What is the diagnosis?<br />

338


TRAUMATIC DISORDERS: PRACTICE CLINICAL SCENARIOS<br />

ANSWERS TO PRACTICE CLINICAL SCENARIOS<br />

Scenario A<br />

Diagnosis: tension pneumothorax<br />

Diagnostic evaluation: The diagnosis should be made immediately based on clinical presentation. No<br />

laboratory or radiologic studies should be necessary.<br />

Management: Immediate needle thoracostomy should be performed on the affected side, using<br />

a large bore over-the-needle catheter. The needle is inserted over the third rib (second intercostal<br />

space) perpendicular to the chest. A gush of air may be heard. After successful decompression, a tube<br />

thoracostomy should be performed.<br />

Scenario B<br />

Diagnosis: epidural hematoma<br />

Diagnostic evaluation: The diagnosis should be suspected based on the clinical history. It is confirmed by<br />

noncontrast head CT, which shows a classic lens-shaped convex hematoma, usually along the temporal bone.<br />

Management: Airway should be secured before head CT if the patient shows signs of neurologic<br />

deterioration. Standard management of increased ICP should be implemented. In the setting of acute<br />

herniation, hyperventilation is the most rapid measure to decrease ICP. Ultimately, neurosurgical<br />

decompression is definitive care. Consultation should be made early in course.<br />

Scenario C<br />

Diagnosis: intra-abdominal injury resulting in hemorrhagic shock<br />

Diagnostic evaluation: The key to diagnosis is identifying hemorrhagic shock (tachycardia and<br />

hypotension) and performing a FAST examination. The unstable patient should not leave the resuscitation<br />

bay for a CT.<br />

Management: Management in the emergency department focuses on restoring circulatory volume and<br />

oxygen-carrying capacity. Place two large-bore IV lines, and administer crystalloid fluids, 1-2 Lover 5<br />

minutes each. If the patient remains in shock, administer packed RBCs. Ultimately the unstable patient<br />

with intra-abdominal bleeding needs an exploratory laparotomy in the operating room. Consultation with<br />

trauma or general surgery early in the course is essential.<br />

Scenario D<br />

Diagnosis: burn with inhalational injury<br />

Diagnostic evaluation: The key to diagnosis is recognizing that this patient has an inhalation injury and is<br />

at very high risk of developing airway compromise and respiratory failure.<br />

Management: This is an airway emergency. This patient should be intubated immediately. A difficult<br />

airway should be anticipated, and advanced airway equipment, airway alternatives, and surgical airway<br />

equipment readily available. After intubation, partial-thickness burns should be treated by standard<br />

practice with fluid resuscitation, debridement of nonviable tissue, and wound cleansing. The patient<br />

should be transferred to a burn center.<br />

339


TRAUMATIC DISORDERS: PRACTICE CLINICAL SCENARIOS<br />

Scenario E<br />

Diagnosis: lap belt syndrome<br />

Diagnostic evaluation: The key to diagnosis is recognizing that this mechanism of injury <strong>combined</strong> with<br />

the examination findings is very suspicious of "lap belt syndrome" (abdominal wall hematoma/bruise,<br />

thoracolumbar Chance fracture, and small-bowel injury). FAST examination will likely be negative,<br />

because this is a hollow- rather than solid-organ injury. CT is most sensitive but may still not detect some<br />

hollow viscus injuries. Imaging of the thoracolumbar spine is essential.<br />

Management: Intra-abdominal injuries may be missed on initial evaluation, in which case, observation<br />

and serial abdominal examinations are essential. They are usually managed by exploratory laparotomy.<br />

Thoracolumbar Chance fractures are unstable. Spinal immobilization is essential during emergency<br />

department evaluation. These fractures are usually stabilized neurosurgically in the operating room.<br />

Scenario F<br />

Management: endotracheal intubation with in-line cervical spine immobilization<br />

The first step in the evaluation and management of every trauma patient is airway (followed by breathing<br />

and circulation). This pedestrian who was struck by a vehicle has altered mental status from a head injury,<br />

alcohol intoxication, hemorrhagic shock, or a combination of these factors. His gurgling airway sounds<br />

and altered mental status suggest that he cannot protect his airway and is at risk of aspiration. Therefore,<br />

the most appropriate initial step is endotracheal intubation. At the same time, it is important to consider a<br />

cervical spine injury. In-line cervical spine immobilization should be maintained to limit neck movement<br />

during intubation. Once the patient's airway is secured, the primary assessment should proceed to the<br />

patient's breathing, circulation, and disability, etc.<br />

ScenarioG<br />

Diagnosis: hemorrhagic shock<br />

Diagnostic evaluation: Children have very robust cardiovascular compensatory mechanisms. As a result,<br />

hypotension is a late finding in hypovolemic or hemorrhage shock in children. This child's systolic blood<br />

pressure is within the normal range (lowest 5th percentile normal= 70 + 2 x age in years). However, the<br />

child clearly has other signs of poor perfusion, including tachycardia, altered mental status (agitation), and<br />

delayed capillary refill time.<br />

Management: Because this child has signs of poor perfusion, it must be assumed that she is in<br />

hypovolemic/hemorrhagic shock. Initial resuscitation should begin with rapid IV crystalloid bolus infusions<br />

(20 ml/kg over 5 minutes x 2 if necessary). If the clinical response if inadequate, a packed RBC transfusion<br />

should be given (10 ml/kg). At the same time, an attempt should be made to identify the source of<br />

hemorrhage. This would likely include a chest radiograph, FAST examination, and pelvic radiograph. If the<br />

child responds to the resuscitative measures, a CT of the abdomen/pelvis may be considered. However, if<br />

the child remains in shock, exploratory laparotomy by a trauma surgeon may be the best next step.<br />

340


TRAUMATIC DISORDERS: PRACTICE CLINICAL SCENARIOS<br />

Scenario H<br />

Diagnosis: blunt abdominal trauma in pregnancy<br />

Diagnostic evaluation: Even minor blunt abdominal trauma in pregnancy can cause fetomaternal<br />

hemorrhage. In an Rh-negative woman carrying an Rh-positive fetus, this can result in isoimmunization<br />

if not detected and treated. Therefore, it is important to confirm the patient's Rh type. Rh-negative women<br />

who sustain abdominal trauma during pregnancy should be given Rh immune globulin within 72 hours of<br />

possible antigenic exposure. If gestation is estimated to be 12 weeks, the standard 300 mcg dose is recommended. Kleihauer-Betke test is not indicated in the<br />

emergency department.<br />

Blunt trauma in pregnancy can cause placental abruption, uterine rupture, and fetal demise. These<br />

more commonly occur during the later stages of pregnancy. The initial assessment of the fetus includes<br />

measuring fetal heart tones. An ultrasound should be performed if there are concerns for fetal distress or<br />

uterine/placental abnormalities or if the gestational age is unknown.<br />

In major trauma resuscitation of the pregnancy patient, the first priority is always ensuring adequate<br />

cardiopulmonary function of the mother in an effort to maintain adequate fetal perfusion.<br />

Scenario I<br />

Diagnosis: stab wound to abdomen with evisceration<br />

Management: This patient requires emergent exploratory laparotomy. Indications for laparotomy in<br />

abdominal stab wounds include evisceration, signs of peritonitis, signs of hemodynamic instability/shock,<br />

and impaled foreign body.<br />

In patients without indications for emergency laparotomy, management is more complex. The wounds<br />

may be explored for signs of peritoneal violation. If there is no peritoneal violation, the patient may<br />

be discharged home. However, only about one half of patients whose wounds violate the peritoneum<br />

ultimately require laparotomy. Therefore, an approach that involves close observation, serial abdominal<br />

examinations, and selective laparotomy may be appropriate.<br />

ScenarioJ<br />

Diagnosis: acute compartment syndrome of the lower leg<br />

Diagnostic evaluation: Acute compartment syndrome most commonly develops in association with<br />

fractures. The lower leg is most commonly involved. Compartment syndrome develops when pressures<br />

within the extremity compartment increase as a result of soft-tissue swelling and hematoma formation and<br />

compromise perfusion. The first and earliest symptom is pain, usually out of proportion to the apparent<br />

injury. Other later findings include the "6 Ps": paresthesias, paralysis, pallor, pulselessness, palpable<br />

tenderness, in addition to pain.<br />

The initial evaluation should include radiographs of the affected extremity. Fractures and/or dislocations<br />

should be promptly reduced and splinted. If concerns for compartment syndrome persist, a surgeon skilled<br />

in performing fasciotomies should be consulted, if available. Compartment pressures may be measured,<br />

but it is not always necessary to make the diagnosis. Analgesic medications should be administered.<br />

Ultimately, the definitive and potentially limb-saving therapy is fasciotomy.<br />

341


342<br />

NOTES


ORTHOPEDIC EMERGENCIES<br />

ORTHOPEDIC EMERGENCIES<br />

General Principles of Fracture and Dislocation Management ................................................................................... 349<br />

Classification and Definitions ............................................................................................................................. 349<br />

Salter-Harris Classification of Epiphyseal Fractures ............................................................................................. 350<br />

Complications of Fractures/Dislocations ............................................................................................................. 351<br />

Upper Extremity Trauma ........................................................................................................................................... 352<br />

Shoulder Dislocations and Ligamentous Tears .................................................................................................... 352<br />

Scapular Fractures .............................................................................................................................................. 356<br />

Humeral Fractures .............................................................................................................................................. 357<br />

Elbow Injuries .................................................................................................................................................... 359<br />

Radial and Ulnar Injuries .................................................................................................................................... 362<br />

Hand and Wrist Injuries ..................................................................................................................................... 364<br />

Pelvis and Hip Injuries .............................................................................................................................................. 373<br />

Essential Anatomy of the Pelvis ........................................................................................................................... 373<br />

Pelvic Fractures .................................................................................................................................................. 3 73<br />

Hip Dislocations ................................................................................................................................................ 375<br />

Hip Fractures ...................................................................................................................................................... 377<br />

Pediatric Hip Disorders ...................................................................................................................................... 3 77<br />

Knee Injuries ............................................................................................................................................................. 379<br />

Essential Anatomy .............................................................................................................................................. 379<br />

Mechanisms of Knee Injury ................................................................................................................................ 379<br />

Hemarthrosis ...................................................................................................................................................... 380<br />

Physical Examination of the Knee ....................................................................................................................... 380<br />

Knee Dislocation ................................................................................................................................................ 380<br />

Rupture of the Quadriceps Mechanism .............................................................................................................. 381<br />

Patellar Subluxation/Dislocation ......................................................................................................................... 381<br />

Patellar Fracture .................................................................................................................................................. 382<br />

Tibial Plateau Fracture ........................................................................................................................................ 382<br />

Baker Cyst .......................................................................................................................................................... 382<br />

Osgood-Schlatter Disease ................................................................................................................................... 383<br />

Chondromalacia Patellae .................................................................................................................................... 383<br />

Lower Leg Injuries ..................................................................................................................................................... 383<br />

Soft Tissue .......................................................................................................................................................... 383<br />

Anterior Compartment Syndrome ....................................................................................................................... 384<br />

Stress Fractures of the Fibula ............................................................................................................................... 384<br />

Ankle Injuries ............................................................................................................................................................ 384<br />

Essential Anatomy .............................................................................................................................................. 384<br />

Sprains (Ligamentous Injuries) ............................................................................................................................ 385<br />

Ankle Dislocations ............................................................................................................................................. 386<br />

Tendon Injuries ................................................................................................................................................... 387<br />

Ankle Injuries in Children ................................................................................................................................... 388<br />

Foot Injuries .............................................................................................................................................................. 388<br />

Essential Anatomy .............................................................................................................................................. 388<br />

Fractures ............................................................................................................................................................. 388<br />

343


ORTHOPEDIC EMERGENCIES: SELF-ASSESSMENT QUESTIONS<br />

ORTHOPEDIC EMERGENCIES: SELF-ASSESSMENT QUESTIONS<br />

1. Which of the following statements regarding anterior shoulder dislocations is false?<br />

(a) A Hill-Sachs deformity is seen in up to 50% of anterior dislocations.<br />

(b) Associated neurovascular injury is nonexistent.<br />

(c) Subcoracoid dislocations are the most common type.<br />

(d) Treatment consists of reduction and immobilization in a shoulder immobilizer.<br />

2. Which of the following statements regarding posterior shoulder dislocations is accurate?<br />

(a) They are best visualized with an axillary or scapular "Y" view.<br />

(b) They are often associated with damage to the axillary nerve.<br />

(c) They are often associated with fracture of the anterior glenoid rim.<br />

(d) They are the most frequent type of shoulder dislocation.<br />

3. Which of the following muscles of the rotator cuff is tested with the lift off test<br />

(a) Subscapu laris<br />

(b) Supraspinatus<br />

(c) lnfraspinatus<br />

(d) Teres minor<br />

4. Which of the following statements regarding the rotator cuff is false?<br />

(a) All tears require surgical correction.<br />

(b) It may be torn in association with anterior shoulder dislocations.<br />

(c) It permits abduction and controls internal and external rotation of the shoulder.<br />

(d) The tendinous insertions of the rotator cuff are on the greater and lesser tuberosities of the humerus.<br />

5. Which of the following statements regarding humeral fractures is inaccurate?<br />

(a) Fractures through the anatomical neck of the humerus are associated with avascular necrosis of the humeral head.<br />

(b) Humeral shaft fractures are frequently associated with median nerve injuries.<br />

(c) Management of proximal humeral fractures is determined primarily by the amount of displacement present.<br />

(d) Proximal humeral fractures are classified according to the Neer classification system.<br />

6. Which of the following statements regarding supracondylar extension fractures is false?<br />

(a) They are associated with the development of Volkmann ischemic contracture.<br />

(b) They are commonly associated with median nerve injury.<br />

(c) They are more common in children than in adults.<br />

(d) They generally result from a fall on the outstretched arm.<br />

7. A proximal ulna fracture associated with a radial head dislocation is referred to as a ____<br />

(a) Galeazzi fracture<br />

(b) Maisonneuve fracture<br />

(c) Monteggia fracture<br />

(d) Nightstick fracture<br />

8. Which of the following statements regarding the ulnar nerve is not accurate?<br />

(a) It passes through the Guyon canal.<br />

(b) It innervates the interosseous muscles.<br />

(c) It is frequently injured in association with Smith's fractures.<br />

(d) Loss of function results in a "claw hand" deformity.<br />

9. The second most commonly fractures carpal bone in a fall on the outstretched hand is the _____<br />

(a) Lunate<br />

(b) Trapezoid<br />

(c) Scaphoid<br />

(d) Triquetrum<br />

344


ORTHOPEDIC EMERGENCIES: SELF-ASSESSMENT QUESTIONS<br />

10. The incidence of avascular necrosis in association with scaphoid fractures is highest in fractures of the ___ _<br />

(a) Distal scaphoid<br />

(b) Proximal scaphoid<br />

(c) Tubercle of the scaphoid<br />

(d) Waist (middle third) of the scaphoid<br />

11. Which of the following injuries is most frequently associated with the development of acute carpal tunnel<br />

syndrome?<br />

(a) Lunate dislocation<br />

(b) Peri lunate dislocation<br />

(c) Scapholunate dislocation<br />

(d) None of the above<br />

12. Which one of the injuries listed below does not typically occur by falling on the outstretched arm?<br />

(a) Proximal humeral fracture<br />

(b) Radial head subluxation<br />

(c) Rotator cuff tear<br />

(d) Supracondylar fracture<br />

13. The primary cause of death in patients with pelvic fractures is ___ _<br />

(a) Associated injuries<br />

(b) Blood loss<br />

(c) Ruptured diaphragm<br />

(d) Sepsis<br />

14. Which of the following statements regarding posterior hip dislocations is true?<br />

(a) Avascular necrosis is an early complication of this injury.<br />

(b) The limb appears abducted, externally rotated, and flexed.<br />

(c) The mechanism of injury is a direct force applied to the extended knee.<br />

(d) They represent 15% of all hip dislocations.<br />

15. A rectal hematoma or tenderness that can be seen in patients with pelvic fracture is called the ___ _<br />

(a)<br />

Destot sign<br />

(b) Earle sign<br />

(c) McMurray sign<br />

(d) Roux sign<br />

16. Which of the following are stabilizers of the lateral aspect of the knee?<br />

(a)<br />

Biceps femoris muscle iliotibial band<br />

(b) Cruciate ligaments<br />

(c) Semimembranosus muscle and medial collateral ligament<br />

(d) Vastus lateral is and rectus femoris muscles<br />

17. Which of the following statements regarding knee dislocations is not true?<br />

(a)<br />

Findings of a warm foot and palpable pulses exclude the presence of injury to the popliteal artery.<br />

(b) Findings of paresthesia along the dorsal aspect of the foot and foot drop signal the presence of associated<br />

peroneal nerve injury.<br />

(c) The incidence of associated popliteal artery injury is 30%-40%.<br />

(d) Vascular assessment is indicated for all patients with this injury.<br />

18. Which of the following is the earliest sign of compartment syndrome?<br />

(a)<br />

Absence of peripheral pulses<br />

(b) Pain with passive plantar flexion of the foot<br />

(c) Paralysis<br />

(d) Sensory loss distal to the compartment<br />

345


ORTHOPEDIC EMERGENCIES: SELF-ASSESSMENT QUESTIONS<br />

19. Which of the following findings is consistent with a third-degree sprain of the ankle?<br />

(a) Ability to bear weight<br />

(b) Moderate functional loss<br />

(c) Moderate swelling<br />

(d) Positive stress test<br />

20. Which of the following statements regarding calcaneal fractures is false?<br />

(a) A Bohler angle 75% PMNs), normal glucose, and needle-shaped crystals that are positively birefringent under<br />

polarized light. The most likely diagnosis is:<br />

(a)<br />

Gout<br />

(b) Osteoarthritis<br />

(c) Pseudogout<br />

(d) Rheumatoid arthritis<br />

25. A 40-year-old patient presents with low back pain and is noted to have a temperature of 38.4°C on examination.<br />

The most appropriate management of this case is:<br />

(a)<br />

Antibiotics for treatment of UTI and follow up with primary care provider in 3 days.<br />

(b) Bed rest for 2 days with appropriate analgesia and muscle relaxants plus primary care follow-up in 2 days.<br />

(c) Broad-spectrum antibiotics and admission for pyelonephritis.<br />

(d) Immediate MRI of the spine.<br />

346


ORTHOPEDIC EMERGENCIES: SELF-ASSESSMENT QUESTIONS<br />

26. A 62-year-old woman presents with severe shoulder and proximal arm pain after a fall. Radiograph reveals a<br />

proximal humeral fracture, with the lesser tuberosity displaced from the remainder of the humerus by >I cm, and the<br />

greater tuberosity minimally displaced by 2 mm. This type of proximal humeral fracture would most accurately be<br />

classified as a Neer ____ _<br />

(a) One-part fracture<br />

(b) Two-part fracture<br />

(c) Three-part fracture<br />

(d) The Neer classification system does not pertain to this type of fracture.<br />

27. Which of the injuries listed below commonly occurs in association with the particular type of humeral fracture<br />

described in the question above (#26)?<br />

(a) Clavicular fracture<br />

(b) Posterior shoulder dislocation<br />

(c) Anterior and posterior shoulder dislocations<br />

(d) Anterior shoulder dislocation<br />

28. Which of the following statements regarding compartment syndrome is inaccurate?<br />

(a) A compartment pressure of 30 mm Hg makes a definitive diagnosis of compartment syndrome.<br />

(b) Initial management consists of removal of constricting dressings or casts (if present).<br />

(c) It can be caused by crush injuries, fractures, or constrictive dressings or casts.<br />

(d) The most commonly affected compartments are the anterior compartment of the lower leg and the volar<br />

compartment of the forearm.<br />

29. Which of the following is an absolute contraindication to reimplantation of an amputated digit?<br />

(a) Single-digit amputation other than the thumb<br />

(b) Serious underlying systemic illness<br />

(c) Severely damaged or contaminated part<br />

(d) Unstable patient with other life-threatening injuries<br />

30. While awaiting a decision from the vascular surgeon regarding reimplantation, what is the best method of<br />

preserving an amputated part?<br />

(a)<br />

Place it in a container of 10% povidone-iodine solution, and store this container in ice water.<br />

(b) Irrigate it with normal saline or lactated Ringer's to remove gross contamination, wrap it in sterile gauze<br />

moistened with normal saline, place it in a sterile, water-tight container, and store this container in ice water.<br />

(c)<br />

Irrigate it with normal saline or lactated Ringer's to remove gross contamination, wrap it in sterile gauze<br />

moistened with normal saline, and then place it on ice.<br />

(d) Irrigate it with 10% povidone-iodine solution to remove gross contamination, wrap it in sterile gauze<br />

moistened with normal saline, and place it between two ice packs.<br />

31. Which of the following statements regarding scapular fractures is inaccurate?<br />

(a)<br />

Associated injuries, some of which may be life- or limb-threatening, occur in up to 98% of these patients.<br />

(b) Most scapular fractures require orthopedic referral for open reduction and internal fixation.<br />

(c) The most common associated injuries are ipsilateral lung injuries, rib fractures, and clavicle fractures.<br />

(d) They typically occur in association with high-energy trauma.<br />

32. A segond fracture is associated with injury to which major ligament of the knee?<br />

(a)<br />

Anterior cruciate ligament<br />

(b) Medial collateral ligament<br />

(c)<br />

Lateral collateral ligament<br />

(d) Posterior cruciate<br />

33. A 48-year-old man presents with knee pain after a fall. Examination reveals a significant hemarthrosis, which you<br />

aspirate to make him more comfortable. The finding of fat globules in the aspirate is very suggestive of:<br />

(a) A fracture<br />

(b) A peripheral meniscus tear<br />

(c) A tear in the anterior cruciate ligament<br />

(d) None of the above<br />

347


ORTHOPEDIC EMERGENCIES: SELF-ASSESSMENT QUESTIONS<br />

34. Innervation of the extrinsic flexor muscles of the hand is provided by:<br />

(a) The median nerve<br />

(b) The radial nerve<br />

(c) The ulnar nerve<br />

(d) The median and ulnar nerves<br />

35. The patellar grind tests is helpful in making the diagnosis of:<br />

(a) Chondromalacia patellae<br />

(b) Meniscal tears<br />

(c) A torn anterior cruciate ligament<br />

(d) A torn posterior cruciate ligament<br />

ANSWERS<br />

1. b 7. C 13. b 19. d<br />

2. a 8. C 14. b 20. d<br />

3. a 9. d 15. b 21. C<br />

4. a 10. b 16. a 22. C<br />

5. b 11. a 17. a 23. d<br />

6. b 12. b 18. b 24. a 30. b<br />

25.<br />

26.<br />

27.<br />

28.<br />

29.<br />

d<br />

b<br />

b<br />

a<br />

d<br />

31. b<br />

32. a<br />

33. a<br />

34. d<br />

35. a<br />

Use the pre-chapter multiple choice question worksheet (page xvi) to record and determine the percentage of correct<br />

answers for this chapter.<br />

348


ORTHOPEDIC EMERGENCIES<br />

I. GENERAL PRINCIPLES OF FRACTURE AND DISLOCATION<br />

MANAGEMENT<br />

A. Classification and definitions<br />

1. Direct trauma (direct force over the fracture site)<br />

a. "Tapping" (nightstick) fracture<br />

(1) Linear fracture with two fragments<br />

(2) Little or no soft-tissue injury<br />

b. Crush fracture<br />

(1) Comminuted or transverse fracture<br />

(2) Extensive soft-tissue injury<br />

c. Penetrating fracture (often seen with missile wounds)<br />

(1) High-velocity injuries: fragmentation of bone - bone fragments act as secondary missiles - cavitation<br />

and extensive soft-tissue injury<br />

(2) Low-velocity injuries: mild fragmentation of bone<br />

2. Indirect trauma (forces acting at a distance from the fracture site)<br />

a. Traction (tension) fracture: bone is pulled apart - transverse fracture<br />

b. Angulation fracture: bending along the long axis of the bone - transverse fracture with concave surface<br />

often splintered<br />

c. Compression fracture: compression on the long axis of the bone - axial loading - "T" or "Y" fractures<br />

d. Spiral fracture: results from rotational stress (torque)<br />

(1) A pure rotational injury is rare.<br />

(2) When associated with an axial load - oblique fracture<br />

3. Incomplete fractures (torus and greenstick): only one cortex is interrupted; these fractures occur almost<br />

exclusively in children (because of their greater bone elasticity).<br />

a. Torus (buckle) fracture: characterized by a bulging (buckling) of one cortex; results from compressive forces<br />

and usually involves the metaphyseal region<br />

b. Greenstick fracture: characterized by a break in one cortex (convex side) and a bending (bowing) of the<br />

other cortex (concave side); results from an angular force applied to a long bone<br />

4. Complete fracture: involves both cortices<br />

5. Closed (simple) fracture: no communication with the external environment; overlying skin and soft tissue<br />

are intact.<br />

6. Open (compound) fracture: communication with the external environment through a break in the overlying<br />

skin and soft tissue; high risk of infection (osteomyelitis); considered an orthopedic emergency. Management<br />

includes:<br />

a. Early emergency department administration of IV antibiotics (a first-generation cephalosporin with or<br />

without an aminoglycoside)<br />

b. Tetanus prophylaxis<br />

c. Wound irrigation and debridement (usually accomplished in the operating room)<br />

7. Pathologic fracture: results from bone weakness secondary to an underlying disease process (cysts, tumors,<br />

osteogenesis imperfecta, scurvy, rickets, Paget disease, etc); should be suspected when trivial trauma results<br />

in a fracture<br />

8. Stress fracture ("march or fatigue fracture"): results from bone fatigue secondary to repeated or cyclical stress;<br />

often occurs in association with a sudden increase in level of training; usually located in the lower extremities<br />

(metatarsals, navicular, distal tibia/fibula, femoral neck); plain radiographs may be negative for ~2 weeks, but<br />

the fracture will be seen earlier on MRI.<br />

9. Dislocation: displacement of a bone from its normal position so that there is complete disruption of the<br />

articular surface<br />

10. Subluxation: a partial dislocation when there is incomplete disruption of the articular surface<br />

349


ORTHOPEDIC EMERGENCIES<br />

B. Salter-Harris classification of epiphyseal fractures (aids in determining prognosis: the higher the<br />

number, the poorer the prognosis)<br />

1. Type 1: fracture through the epiphyseal plate (physis), resulting in separation of the epiphysis; good<br />

prognosis.<br />

2. Type II: fracture of the metaphysis with extension through the epiphyseal plate (most common type); most<br />

common in children > 10 years old; a metaphyseal fragment called a "Thurston Holland sign"' is present;<br />

good prognosis.<br />

3. Type 111: fracture of the epiphysis with extension into the epiphyseal plate (an intra-articular fracture); open<br />

reduction is often necessary.<br />

4. Type IV: fracture through the metaphysis, epiphysis, and epiphyseal plate (also an intra-articular fracture);<br />

open reduction is usually necessary, and perfect reduction is essential; growth disturbance is common.<br />

350


ORTHOPEDIC EMERGENCIES<br />

5. Type V: crush fracture of the epiphyseal plate (opposite of Type 1 ): most commonly seen in the knee and<br />

ankle; radiographs may look normal; poor prognosis, because the blood supply to the epiphyseal plate is<br />

interrupted.<br />

C. Complications of fractures/dislocations<br />

1. Immediate complications<br />

a. Hemorrhage (can be extensive with pelvic fractures)<br />

b. Vascular injuries<br />

(1) Anterior shoulder dislocation - exclude axillary artery injury<br />

(2) Extension supracondylar fracture - exclude brachia! artery injury<br />

(3) Posterior elbow dislocation - exclude brachia! artery injury<br />

(4) Knee dislocation - exclude popliteal artery injury<br />

c. Nerve injuries<br />

(1) Anterior shoulder dislocation -a, exclude axillary and musculocutaneous nerve injury<br />

(2) Humeral shaft injury -a, exclude radial nerve injury<br />

(3) Extension supracondylar fracture - exclude median, radial, and ulnar nerve injury<br />

(4) Medial epicondylar fracture - exclude ulnar nerve injury<br />

(5) Posterior elbow dislocation - exclude ulnar and median nerve injury<br />

(6) Olecranon fracture - exclude ulnar nerve injury<br />

(7) Acetabular fracture - exclude sciatic nerve injury<br />

(8) Posterior hip dislocation -a, exclude sciatic nerve injury<br />

(9) Anterior hip dislocation -a, exclude femoral nerve injury<br />

(10) Knee dislocation - exclude peroneal and tibial nerve injury<br />

(11) Lateral tibial plateau fracture - exclude peroneal nerve injury<br />

d. Associated soft-tissue and visceral injuries<br />

2. Intermediate complications<br />

a. Compartment syndrome<br />

b. Fat embolism (usually originates from a long bone)<br />

3. Long-term complications<br />

a. Reflex sympathetic dystrophy (complex regional pain syndrome)<br />

b. Volkmann ischemic contracture<br />

c. Nonunion<br />

d. Avascular necrosis (femoral head, proximal scaphoid, capitate, and talus fractures are particularly<br />

predisposed to this complication)<br />

e. Angulation deformities, overgrowth or shortening<br />

f. Infection<br />

g. Joint stiffness<br />

h. Posttraumatic ossification or arthritis<br />

351


ORTHOPEDIC EMERGENCIES<br />

II. UPPER EXTREMITY TRAUMA<br />

A. Shoulder dislocations and ligamentous tears<br />

1. Sternoclavicular joint injuries<br />

a. Classification<br />

(1) First degree (sprain)_,. partial tear of the sternoclavicular and costoclavicular ligaments without<br />

associated clavicular subluxation<br />

(2) Second degree (subluxation)-,. complete tear of the sternoclavicular ligament plus partial tear of the<br />

costoclavicular ligament with subluxation of the clavicle from the manubrium<br />

(3) Third degree (dislocation)-,. complete tear of both the sternoclavicular and costoclavicular ligaments<br />

with dislocation of the clavicle from the manubrium; clavicle can dislocate either anteriorly (most<br />

common) or posteriorly (uncommon, but a true orthopedidvascular emergency).<br />

b. Mechanism of injury<br />

(1) Direct force over sternoclavicular joint-,. posterior dislocation<br />

(2) Fall onto the shoulder-,. anterior or posterior dislocation (large amount of force involved, often<br />

necessitating trauma evaluation and consideration of possible intrathoracic injuries)<br />

c. Clinical presentation<br />

(1) Tenderness and swelling over the sternoclavicular joint<br />

(2) Pain with movement of the ipsilateral extremity and with lateral compression of the shoulders<br />

(3) With third-degree injuries, the medial clavicle is displaced (anteriorly or posteriorly) relative to<br />

the manubrium.<br />

(4) Shortness of breath, dysphagia, or choking (in patients with posterior dislocations associated with<br />

compression of mediastinal structures and risk of injury to the great vessels)<br />

d. Diagnostic evaluation: radiographs; dislocations best demonstrated with CT (obtain with IV contrast to<br />

exclude vascular injury)<br />

e. Treatment<br />

(1) First degree _,. arm sling for 3-4 days and analgesics<br />

(2) Second degree-,. arm sling, analgesics, and orthopedic follow-up<br />

(3) Third degree-,. immediate orthopedic consult and rapid reduction<br />

(a) Establish an IV line and administer analgesia; posterior dislocations often require general<br />

anesthesia in the operating room.<br />

(b) Place the patient in the supine position with a rolled sheet between the shoulders.<br />

(c) Extend, abduct, and apply traction to the ipsilateral arm while an assistant pushes (anterior<br />

dislocation) or pulls with a towel clip (posterior dislocation) the clavicle into its normal position.<br />

(d) Apply an arm sling, and refer for orthopedic follow-up.<br />

(e) Life-threatening injuries to adjacent structures (pneumothorax and compression or laceration of<br />

the esophagus, trachea, or great vessels) occur in up to 25 % of posterior dislocations and must<br />

be attended to promptly.<br />

2. Acromioclavicular separation<br />

a. Classification<br />

(1) First degreerrype I (sprain)_,. partial tear of acromioclavicular ligament without subluxation of the<br />

clavicle; the coracoclavicular ligament is intact.<br />

(2) Second degreefrype II (subluxation) _,. complete tear of acromioclavicular ligament with subluxation<br />

of the clavicle; coracoclavicular ligament is stretched or incompletely torn.<br />

(3) Third degreefrype Ill (dislocation)-,. complete tears of both the acromioclavicular and<br />

coracoclavicular ligaments with dislocation of the clavicle<br />

(4) Fourth degreefrypes IV-VI (displacement) _,. significant displacement of the distal clavicle posteriorly<br />

(Type IV), superiorly (Type V), or inferiorly (Type VI), as well as associated injury/ interposition of the<br />

deltoid or trapezius muscle<br />

b. Mechanism of injury<br />

(1) Fall on the shoulder with the arm adducted (most common)<br />

(2) Fall on the outstretched arm<br />

c. Clinical presentation<br />

(1) Tenderness and swelling over the acromioclavicular joint<br />

352


ORTHOPEDIC EMERGENCIES<br />

(2) Pain with movement of the affected extremity<br />

(3) With Type Ill injuries, the distal clavicle is displaced upward relative to the acromion when compared<br />

with the opposite shoulder. This is best visualized when the patient is examined in the sitting or<br />

standing position with the affected arm hanging at his or her side.<br />

d. Diagnostic evaluation<br />

(1) Radiographic evaluation<br />

(2) Obtain AP views of both clavicles<br />

(3) Stress views (taken with 10 lbs of weight suspended from each wrist) are no longer recommended<br />

because they:<br />

(a) Are associated with increased cost, radiation exposure, and patient discomfort<br />

(b) Provide little additional information and can be misleading<br />

(c) Do not usually alter the course of treatment, because most acromioclavicular separations are<br />

managed conservatively (nonoperatively), and most third-degree injuries are clinically obvious.<br />

(4) Findings depend on the degree of separation.<br />

e. Treatment<br />

(a) First degree -<br />

acromioclavicular joint is radiographically normal.<br />

(b) Second degree - due to upward displacement of the clavicle, the distance between the acromion<br />

and the inferior aspect of the distal aspect of the clavicle is increased by -S½ the width of the<br />

clavicle (or -S1 cm) on AP view. The distance seen between the clavicle and the coracoid process<br />

is normal.<br />

(c) Third degree - the distance between the acromion and the distal aspect of the clavicle is<br />

increased by> 1 /2 the width of the clavicle (or> 1 cm) on the AP view. The distance between the<br />

distal clavicle and the coracoid process is also increased.<br />

(1) Type I - sling for 1-2 weeks and analgesics, followed by early range-of-motion exercises<br />

(2) Type II - sling until acute pain has subsided; analgesics and orthopedic referral for further evaluation<br />

and rehabilitation<br />

(3) Type Ill - immobilization versus surgical fixation is controversial; immobilize in a sling, provide<br />

analgesics, and arrange orthopedic referral.<br />

(4) Types IV-VI - will likely require surgical fixation as definitive treatment; acute management is the<br />

same as that for Type 111.<br />

3. Shoulder dislocation (most common dislocation seen in the emergency department)<br />

a. Diagnostic evaluation: radiographic series<br />

(1) Standard views should always be ordered (AP shoulder, transcapular lateral or "Y" view, and axillary view)<br />

(a)<br />

In anterior dislocations, the AP view detects the most important associated fracture-the humeral<br />

neck; fracture of the lesser tuberosity suggests a posterior shoulder dislocation.<br />

(b) In posterior dislocations, the axillary view is diagnostic. The "Y" view can miss some posterior<br />

dislocations.<br />

(2) An axillary lateral view is ideal for differentiating anterior from posterior dislocations; it often reveals a<br />

compression fracture of the humeral head, but it may be difficult to obtain.<br />

(3) A modified axillary view (called the West Point view) allows visualization of the anterior glenoid<br />

rim; avulsion (Bankart) fractures in this area (that are associated with anterior dislocations) are not<br />

infrequent.<br />

b. Anterior (95%-97% of all shoulder dislocations)<br />

(1) Mechanism of injury: abduction, extension, and external rotation<br />

(2) Types (where the humeral head is)<br />

(a) Subcoracoid (most common)<br />

(b) Subglenoid (head of the humerus is anterior and inferior to the glenoid fossa)<br />

(c) Subclavicular (very rare)<br />

(d) lntrathoracic (very rare)<br />

(3) Clinical presentation<br />

(a) Examination of the shoulder reveals prominence of the acromion process and flattening of<br />

the normal contour of the shoulder. The affected arm is held in slight abduction and external<br />

rotation; the patient is unable to place his or her palm on the uninjured shoulder.<br />

(b) A clinical clue is resistance to internal rotation and adduction.<br />

353


ORTHOPEDIC EMERGENCIES<br />

(4) Complications/associated injuries<br />

(a) Recurrence is the most common complication and is age related; the younger the patient, the<br />

greater the likelihood of recurrence.<br />

(b) Bony injuries<br />

1. Hill-Sachs deformity<br />

• A compression fracture or "groove" of the posterolateral aspect of the humeral head<br />

Source: Hellerhoff (Wikimedia Commons:<br />

http://en. wi ki ped ia.org/wi ki/Fi le:D i slocated_shou Ider _X-ray _03. pn g)<br />

• Results from impaction of the humeral head on the anterior glenoid rim as it dislocates<br />

(or reduces)<br />

• Occurs in up to 50% of anterior dislocations and is particularly common in patients<br />

with recurrent dislocation; the humeral head is damaged by the sharp anterior rim of the<br />

glenoid, creating a lesion called the "hatchet sign," which is apparent on the reduction<br />

film.<br />

11. Avulsion of the greater tuberosity (more common in patients >45 years old and in those with<br />

subglenoid dislocations)<br />

iii. Fracture of the anterior glenoid lip (Bankart fracture)<br />

(c) Nerve injuries<br />

i. May occur when the shoulder is dislocated or reduced; therefore, it is important to check<br />

and document sensation both before and after reduction.<br />

11. Most injuries are neuropraxias and recover well over time.<br />

iii. Axillary nerve injury (most common): examination reveals sensory loss over the lateral<br />

aspect of the shoulder and weakness in shoulder abduction (deltoid muscle).<br />

iv. Musculocutaneous nerve injury: examination reveals weakness of forearm flexors and<br />

supinators, as well as sensory loss along dorsum of forearm.<br />

v. Brachia! plexus injuries: require atraumatic reduction<br />

(d) Rotator cuff tears (higher incidence in patients >40 years old)<br />

(e) Axillary artery injury is rare but should be suspected in an elderly patient with a weak/absent<br />

radial pulse or an expanding hematoma.<br />

c. Posterior (2%-4% of all shoulder injuries)<br />

(1) Mechanism of injury<br />

(a) Convulsive seizure and electric shock are most common mechanisms<br />

(b) Fall on a forward-flexed, adducted, and internally rotated arm<br />

(c) Significant direct blow to the anterior shoulder<br />

(2) The most commonly missed major dislocation of the body, it is often misdiagnosed as "bursitis" or<br />

"adhesive capsulitis." Radiographic diagnosis can be made with an axillary lateral or transscapular<br />

lateral "Y" view; an axillary lateral view is an excellent technique for visualization of a posterior<br />

dislocation, but may be difficult to obtain without adequate analgesia. (Be able to identify.)<br />

354


ORTHOPEDIC EMERGENCIES<br />

Courtesy oi Michael C. Bond, MD<br />

(3) Types<br />

(a) Subacromial (98%)<br />

(b) Subglenoid<br />

(c) Subspinous<br />

(4) Clinical presentation<br />

(a) Examination of the shoulder reveals anterior flatness, posterior fullness, and prominence of the<br />

coracoid process. The affected arm is internally rotated and is usually held in adduction.<br />

(b) A clinical clue is inability to abduct or externally rotate the arm.<br />

(5) Complications/associated injuries<br />

(a) Associated fractures of the posterior rim of the glenoid fossa, anteromedial aspect of the humeral<br />

head (reversed Hill-Sachs deformity), and lesser tuberosity. In fact, in the presence of an isolated<br />

lesser tuberosity fracture, a posterior shoulder dislocation should always be excluded.<br />

(b) Recurrence rate is 30%.<br />

(c) Neurovascular complications are uncommon.<br />

d. Treatment of shoulder dislocation is with reduction using traction, scapular manipulation, or leverage<br />

and often requires procedural sedation. Closed reduction can usually be accomplished in the emergency<br />

department.<br />

(1) Reduction techniques for anterior dislocations<br />

(a) Stimson or hanging-weight<br />

(b) Scapular manipulation<br />

(c) Traction-countertraction<br />

(d) External rotation (Hennipen technique)<br />

(e) Milch forward elevation<br />

(f)<br />

Spaso maneuver<br />

(g) Cunningham technique<br />

(h) The Kocher maneuver (a leverage technique) and the Hippocratic technique are associated with<br />

many complications and should not be used.<br />

(2) Reduction technique for posterior dislocations<br />

(a) Orthopedic consult should precede reduction attempts, because this injury is rare and the rate of<br />

complications (fractures) is high.<br />

355


ORTHOPEDIC EMERGENCIES<br />

(b) Axial traction is applied in-line with the humerus, and then the humeral head is pushed forward.<br />

(c) If this is unsuccessful, reduction may need to be accomplished under general anesthesia.<br />

(3) Shoulder dislocations with an associated fracture should be referred to an orthopedic surgeon for<br />

reduction (closed or open).<br />

(4) After reduction<br />

(a)<br />

4. Rotator cuff tears<br />

Recheck and document neurovascular examination.<br />

(b) Order radiographs to confirm reduction and exclude any associated injuries; humeral neck<br />

fractures are a known complication of anterior shoulder relocation, the result of which is often<br />

avascular necrosis of the humeral head.<br />

(c) Place patient in a shoulder immobilizer or Velpeau dressing. The length of immobilization<br />

will vary from 1 to 4 weeks, depending on the patient's age; the older the patient, the sooner<br />

mobilization should be started to avoid stiffness.<br />

(d) Refer for orthopedic follow-up.<br />

a. The rotator cuff is made up of tendinous insertions of the following muscles that attach to the greater<br />

and lesser tuberosities of the humerus. These muscles permit abduction and control internal and external<br />

rotation of the shoulder.<br />

(1) Subscapularis<br />

(2) Supraspinatus (most frequently injured/torn)<br />

(3) lnfraspinatus<br />

(4) Teres minor<br />

b. Mechanisms of injury<br />

(1) Acute tears usually occur in association with:<br />

(a) Forceful abduction of the arm against significant resistance (such as a fall on the outstretched arm)<br />

(b) Direct force such as a fall on the shoulder<br />

(c) Heavy lifting<br />

(2) Chronic tears (90% of all lesions) result from subacromial impingement and mechanical<br />

degeneration or a decreased blood supply to the tendons, both of which accompany advancing age;<br />

can be insidious in onset and can occur in the absence of trauma, particularly in the elderly.<br />

c. Clinical presentation<br />

(1) Typical patient is a man 2:40 years old.<br />

(2) Pain over the anterior aspect of the shoulder is abrupt in onset and tearing in quality with acute tears;<br />

in chronic tears, the pain is more gradual in onset and is typically described as being worse at night.<br />

(3) Weak and painful abduction or, if the tear is large or complete, inability to initiate abduction; a<br />

positive "drop-arm test" (inability to hold the arm in 90° abduction) is present with significant tears.<br />

(4) Tenderness on palpation over the insertion site of the supraspinatus on the greater tuberosity<br />

d. Radiographs may be normal or show degenerative changes; in the presence of a complete tear, may<br />

demonstrate superior displacement of the humeral head (best seen on an external rotation view).<br />

e. Treatment<br />

(1) Acute tears<br />

(a) Sling immobilization, NSAIDs, early orthopedic referral<br />

(b) Complete tears require early surgical repair (


ORTHOPEDIC EMERGENCIES<br />

(2) Hemothorax<br />

(3) Pulmonary contusion<br />

c. Injuries to the shoulder girdle complex<br />

(1) Clavicle fractures<br />

(2) Shoulder dislocations with associated rotator cuff tears<br />

d. Neurovascular injuries (rare)<br />

(1) Brachia I plexus<br />

(2) Axillary artery or nerve<br />

(3) Subclavian artery<br />

(4) Suprascapular nerve<br />

e. Vertebral compression fractures<br />

3. Mechanism of injury<br />

a. Direct blow to the scapula<br />

b. Trauma to the shoulder<br />

c. Fall on an outstretched hand (FOOSH)<br />

4. Classification<br />

a. Based on anatomic location<br />

b. Fractures of the body, neck, and glenoid are most common.<br />

5. Clinical presentation: The patient is frequently a man who was involved in a high-speed motor vehicle collision<br />

or had a significant fall. He presents with the affected arm and shoulder adducted against his body and<br />

complains of pain over the back of the shoulder. On examination, shoulder pain is increased with abduction of<br />

the arm.<br />

6. Diagnostic evaluation<br />

a. Radiographs<br />

7. Treatment<br />

(1) Routine shoulder radiographs demonstrate most scapular fractures.<br />

(2) Axillary lateral view is helpful in evaluating fractures involving the glenoid fossa, acromion, and<br />

coracoid process.<br />

(3) Chest radiograph to exclude associated pulmonary injury<br />

a. Once any associated injuries have been excluded, most scapular fractures can be managed with sling<br />

immobilization (for 2:2 weeks) and analgesia, followed by early range-of-motion exercises.<br />

b. Orthopedic referral for open reduction and internal fixation is usually reserved for severely displaced or<br />

angulated fractures.<br />

C. Humeral fractures<br />

1. Proximal humeral fractures (most commonly seen in the elderly)<br />

a. Mechanisms of injury<br />

(1) FOOSH - most common<br />

(2) Direct blow to the lateral aspect of the arm<br />

b. Clinical presentation: the patient is usually an older woman with osteoporosis who presents with severe<br />

upper arm and shoulder pain after a fall; she is likely to be holding the arm in adduction.<br />

c. Neer classification system<br />

(1) An anatomic classification system for proximal humeral fractures according to the amount of<br />

displacement of four segments (consider an additional part if fragment separation > 1 cm or<br />

angulation >45°).<br />

(a) Anatomic neck<br />

(b) Surgical neck<br />

(c) Greater tuberosity<br />

(d) Lesser tuberosity<br />

(2) Major categories<br />

(a) One-part fractures (minimally displaced) demonstrate no displacement (fragment separation > 1<br />

cm or angulation >45°). These fractures account for 80%-85% of all proximal humeral fractures.<br />

(b) Two-part fractures demonstrate displacement of only one fragment.<br />

357


ORTHOPEDIC EMERGENCIES<br />

(c) Three-part fractures demonstrate the displacement of two individual fragments from the remaining<br />

proximal humerus.<br />

(d) Four-part fractures demonstrate displacement of all four segments.<br />

Proximal Humerus Fractures<br />

Greater Tuberosity<br />

Lesser Tuberosity<br />

d. Treatment<br />

(1) In general, the amount of displacement determines how these fractures are managed. AP, lateral, and<br />

axillary (if possible) views are essential to make an accurate diagnosis.<br />

(2) One-part fractures_,. immobilize with shoulder immobilizer, sling and swathe or a Velpeau dressing,<br />

analgesics, and refer for orthopedic follow-up.<br />

(3) Two-, three-, and four-part fractures_,. immobilize as above and obtain emergent orthopedic referral.<br />

Many of these fractures require surgical repair. Four-part fractures may require insertion of a prosthesis.<br />

e. Complications and associated injuries<br />

(1) Adhesive capsulitis ("frozen shoulder") is the most common complication and can be minimized or<br />

prevented by early mobilization exercises.<br />

(2) Associated neurovascular injuries (brachia! plexus, axillary nerve, and axillary artery) should<br />

be excluded in all proximal humeral fractures, particularly surgical neck fractures and fracture<br />

dislocations.<br />

(3) Posterior shoulder dislocations frequently accompany fractures of the lesser tuberosity because of<br />

intense contraction of the subscapularis muscle, which inserts at this location.<br />

(4) Anterior and posterior dislocations occur in association with three- and four-part fractures.<br />

(5) Avascular necrosis of the humeral head frequently complicates anatomic neck fractures, four-part<br />

fractures, and fractures of articular surfaces.<br />

2. Humeral shaft fractures<br />

a. Mechanisms of injury<br />

(1) Direct blow (most common)<br />

(2) FOOSH or fall on elbow<br />

(3) Pathologic fractures (particularly from metastatic breast cancer) are also common.<br />

b. Usually involves middle third of the humeral shaft<br />

c. Associated injuries<br />

(1) Most common is damage to the radial nerve_,. wrist drop (inability to extend the wrist, fingers, and<br />

thumb) and loss of sensation in the first dorsal web space<br />

(a)<br />

Nerve damage occurring at the time of injury is often due to neuropraxia and usually resolves<br />

spontaneously.<br />

(b) Nerve palsy occurring after manipulation or immobilization is generally due to nerve entrapment<br />

and requires immediate surgical exploration.<br />

(2) Ulnar and median nerve injury may also occur but are much less common.<br />

(3) Brachia! artery injury<br />

d. Treatment<br />

(1) Most of these fractures are managed nonoperatively with one of the following:<br />

(a) A coaptation ("sugar-tong") splint plus sling and swathe for nondisplaced fractures<br />

(b) A hanging cast for displaced or angulated fractures<br />

(2) Operative management is usually reserved for patients with neurovascular compromise, soft-tissue<br />

interposition, pathologic fractures, or transverse fractures.<br />

358


ORTHOPEDIC EMERGENCIES<br />

e. Complications<br />

D. Elbow injuries<br />

1. Fractures<br />

(1) Delayed union is common and may necessitate prolonged immobilization.<br />

(2) Adhesive capsulitis (stiff/frozen shoulder) may be prevented by early initiation of circumduction<br />

exercises.<br />

a. Radiographic findings helpful in diagnosing occult elbow fractures that may be detected on the lateral view<br />

(1) Fat pad signs<br />

(a) Posterior fat pad sign: indicates distension of the joint capsule by effusion (hemarthrosis) and<br />

probable fracture; never seen on a normal elbow radiograph.<br />

(b) Anterior fat pad sign: not as useful diagnostically as the posterior fat pad sign, because a small<br />

one is present on many normal radiographs. However, superior and anterior displacement of this<br />

fat pad suggests a probable fracture.<br />

(2) Anterior humeral line: a line drawn along the anterior surface of the humerus and extending through<br />

the elbow that is helpful in detecting subtle supracondylar fractures; this line normally transects the<br />

middle of the capitellum but, with supracondylar extension fractures, transects the anterior third of the<br />

capitel lum or passes completely anterior to it.<br />

b. Supracondylar fractures<br />

(1) More common in children


ORTHOPEDIC EMERGENCIES<br />

c. Olecranon fractures<br />

(1) Mechanism of injury<br />

(a)<br />

Direct blow to the olecranon<br />

(b) FOOSH with the elbow inflexion<br />

(2) Clinical presentation<br />

(a) Swelling and tenderness over the olecranon<br />

(b) Inability to extend the elbow against gravity or resistance due to inadequacy of the triceps<br />

mechanism<br />

(3) Associated injuries<br />

(a) Ulnar nerve injury (paresthesias and numbness in the ulnar nerve distribution or weakness of the<br />

interossei muscles) is common.<br />

(b) Usually secondary to ulnar nerve contusion and resolves spontaneously over time<br />

(4) Treatment<br />

(a)<br />

Nondisplaced fractures_,, elbow immobilization in 30° flexion<br />

(b) Fractures with >2 mm displacement_,, emergent orthopedic referral for surgical repair (open<br />

reduction with internal fixation)<br />

d. Condylar fractures<br />

(1) The distal humerus is composed of the medial and lateral condyles, each of which has an articular and<br />

nonarticular surface.<br />

(a) Articular surfaces<br />

i. Trochlea (on the medial condyle)<br />

ii. Capitellum (on the lateral condyle)<br />

(b) Nonarticular surfaces<br />

i. Medial epicondyle<br />

ii. Lateral epicondyle<br />

(2) Condylar fractures usually involve both the articular surface (trochlea, capitellum) and the nonarticular<br />

surface (epicondyle) of the distal humerus.<br />

(3) Fractures of the lateral condyle are more common than those of the medial condyle.<br />

(4) Treatment<br />

(a) Nondisplaced or minimally displaced fractures_,, immobilization in 90° elbow flexion with<br />

forearm supination (lateral condylar fracture) or pronation (medial condylar fracture) and<br />

orthopedic referral<br />

(b) Fractures with >3 mm displacement_,, surgical fixation<br />

e. Articular surface fractures (trochlea and capitellum)<br />

(1) Mechanism of injury<br />

(a) Trochlea and capitellum fractures usually occur in association with posterior elbow dislocations.<br />

(b) Capitellum fractures can also result from FOOSH and may be associated with radial head<br />

fractures.<br />

(2) Treatment<br />

(a)<br />

Nondisplaced fractures_,, splint immobilization<br />

(b) Displaced fractures (even if minimal)_,, emergent orthopedic consult and surgical repair<br />

f. Epicondylar fractures<br />

(1) Most commonly seen in children<br />

(2) Medial epicondylar fractures<br />

(a) Avulsion fractures most often occur in association with posterior elbow dislocations in children<br />

and adolescents but can also occur as a result of repeated valgus stress of the elbow in<br />

adolescents ("little leaguer's elbow"), as well as from a direct blow.<br />

(b) Must evaluate for ulnar nerve injury (present in 60% of patients)<br />

(c) Treatment<br />

i. Nondisplaced (or minimally displaced) fractures---,, immobilization<br />

11. Displaced fractures_,, orthopedic referral and surgical reduction if fragment displaced >3-5<br />

mm or if they are intra-articular<br />

360


ORTHOPEDIC EMERGENCIES<br />

(3) Lateral epicondylar fractures<br />

(a) Rare; generally result from a direct blow<br />

(b) Generally nondisplaced and treated with immobilization<br />

2. Dislocations<br />

a. Posterior elbow dislocations<br />

(1) Most common type of elbow dislocations<br />

(2) Mechanism of injury: fall on the extended (or hyperextended) and abducted arm (FOOSH)<br />

(3) Clinical presentation<br />

(a) Marked swelling with 45° flex ion of the joint<br />

(b) Posterior prominence of the olecranon<br />

(4) Best demonstrated on the lateral surgical of the elbow<br />

Source: www.lifeinthefastlane.com. Used with permission.<br />

(5) Associated injuries<br />

(a) Fractures about the elbow (30%-60% of cases)<br />

(b) Injuries to the ulnar and median nerves (common)<br />

(c) Brachia! artery injury (occurs in up to 8% of cases; should be suspect when a large opening<br />

between the tip of the olecranon and the distal humerus is palpated on examination or seen on<br />

radiograph)<br />

(6) Treatment is reduction. Orthopedic consultation is usually not necessary, and reduction should be<br />

done immediately if there is neurovascular compromise.<br />

(a) Provide procedural sedation.<br />

(b) Apply traction distally at the wrist, while an assistant immobilizes the humerus.<br />

(c) While maintaining traction at the wrist, flex the elbow and apply posterior pressure to the distal<br />

humerus.<br />

(d) A "clunk" will be heard or felt as the elbow is reduced and the articular surfaces mesh.<br />

(e) Move the elbow through its full range of motion to check stability, and then reassess<br />

neurovascular status.<br />

(f)<br />

Immobilize the elbow in a long-arm posterior splint in 120° (full flexion) or as much flexion as<br />

circulation permits and obtain radiographs (after reduction).<br />

(g) Patients can be discharged with instructions to apply ice, elevate, and watch for signs of vascular<br />

compromise. If the patient is not reliable, he or she should be admitted and observed for possible<br />

compartment syndrome.<br />

b. Anterior elbow dislocations (uncommon)<br />

(1) Mechanism of injury: a blow to the olecranon with the elbow inflexion<br />

(2) Clinical presentation: the forearm is elongated and supinated while the elbow is generally held in full<br />

extension<br />

(3) Associated injuries<br />

(a) Incidence of vascular impairment is much higher than with posterior dislocations.<br />

(b) Associated avulsion of the triceps mechanism is common.<br />

361


ORTHOPEDIC EMERGENCIES<br />

(4) Treatment<br />

(a) Once neurovascular status has been assessed and procedural sedation provided, an assistant<br />

immobilizes the humerus while the physician applies in-line traction to the wrist (with one hand)<br />

and downward and backward pressure to the proximal forearm (with the other hand).<br />

(b) Orthopedic consultation should be obtained when possible before reduction of elbow<br />

dislocations with associated fractures. These are difficult or impossible to reduce and may have<br />

neurovascular sequelae.<br />

c. Brachia! artery injury may be present with a severe disruption of the elbow joint with any type of<br />

dislocation. Clues are a wide opening between the olecranon and distal humerus on palpation of the joint<br />

or seen on a lateral radiograph.<br />

d. Radial head subluxation (nursemaid elbow)<br />

(1) Subluxation of the radial head without associated ulnar fracture occurs with some regularity in<br />

children


ORTHOPEDIC EMERGENCIES<br />

e. Treatment<br />

(1) Nondisplaced fractures: sling immobilization followed by early range of motion exercises as tolerated.<br />

(2) Comminuted and displaced fractures: immobilization in a long-arm posterior splint and early (2-5<br />

days) orthopedic referral for screw fixation or radial head excision (with or without silastic implant) if<br />

one of the fol lowing exists:<br />

2. Galeazzi fracture<br />

(a) Marked comminution of the fracture<br />

(b) Angulation of the articular surface >30°<br />

(c) >2 mm offset in a two-part fracture<br />

(d) Fracture involving more than one-third of the articular surface<br />

a. Fracture of the distal radial shaft associated with a distal radioulnar dislocation (radiographic signs may<br />

be subtle)<br />

b. Mechanisms of injury<br />

(1) Direct blow to the back of the wrist<br />

(2) FOOSH in forced pronation<br />

c. Treatment: open reduction and internal fixation<br />

3. Nightstick fracture<br />

a. Isolated fracture of the shaft of the ulna<br />

b. Mechanism of injury: a direct blow to the subcutaneous border of the ulna; usually occurs when a patient<br />

raises up the forearm to protect his or her face from a blow<br />

c. Treatment<br />

(1) Nondisplaced fractures: immobilization in a long-arm cast<br />

(2) Displaced fractures (those with> 10° angulation or displacement >50% of the diameter of the ulna):<br />

orthopedic referral for open reduction and internal fixation<br />

4. Monteggia fracture<br />

a. Fracture of the proximal third of the ulna <strong>combined</strong> with dislocation of the radial head (usually anterior)<br />

b. Mechanisms of injury<br />

(1) Direct blow to the posterior aspect of the ulna<br />

(2) FOOSH with forearm in forced pronation<br />

c. Diagnostic evaluation<br />

(1) The ulnar fracture is often apparent on AP and lateral views of the forearm.<br />

(2) The radial head dislocation is missed in as many as 25% of cases. Avoid this by assessing the<br />

alignment of the radial head with the capitellum (radiocapitellar line on lateral radiograph); if the radial<br />

head is in its normal anatomic position, a line drawn through the radial shaft and head should intersect<br />

the capitellum in all views.<br />

d. Associated injury to the radial nerve is common; usually resolves spontaneously in 6-8 weeks.<br />

e. Treatment<br />

(1) In adults, open reduction and internal fixation of the ulnar fracture followed by closed reduction of the<br />

radial head dislocation is the preferred treatment and is associated with the best functional outcome.<br />

(2) Children can usually be treated with closed reduction under general anesthesia.<br />

5. Fractures of both the radius and ulna<br />

a. Usually displaced<br />

b. Mechanism of injury: most commonly a direct blow to the forearm<br />

c. Associated injuries<br />

(1) Peripheral nerve deficits (radial, ulnar, and median) occur infrequently with closed injuries but can be<br />

seen with open fractures.<br />

(2) Development of compartment syndrome is a major concern.<br />

d. Treatment<br />

(1) Nondisplaced fractures (rare): immobilization in a sugar tong splint<br />

(2) Displaced fractures: generally open reduction and internal fixation with compression plates is required<br />

(in adults); children can sometimes be treated with closed reduction.<br />

e. Complications: compartment syndromes (both anterior and posterior), malunion, and nonunion<br />

363


ORTHOPEDIC EMERGENCIES<br />

F. Hand and wrist injuries<br />

1. Essential anatomy<br />

a. Eight carpal bones<br />

b. Ulnar nerve<br />

(1) Runs deep to the flexor carpi ulnaris tendon and through Guyon canal<br />

(2) Innervates the following muscles:<br />

(a) Hypothenar eminence<br />

(b) lnterosseous muscles<br />

(c) Lumbrical muscles of the ring (4th) and little (5th) fingers<br />

(d) Adductor pollicis brevis<br />

(e) In the forearm, the flexor carpi ulnaris and ulnar aspect of the flexor digitorum profundus<br />

(3) Provides sensation to pal mar and dorsal aspects of the ulnar side of the hand, the little finger, and the<br />

ulnar half of the ring finger<br />

(4) Testing<br />

(a)<br />

Motor test of normal function __.,. abduction of fingers against resistance (patient can spread<br />

fingers apart)<br />

(b) Sensation__.,. best tested over the volar tip of the little (5th) finger<br />

(5) Loss of ulnar nerve function__.,. inability to hold a piece of paper between fingers (early) or claw<br />

hand (/ate)<br />

c. Median nerve<br />

(1) Runs through the carpal tunnel between the flexor carpi radial is and the palmaris longus<br />

(2) Innervates the following muscles:<br />

(a)<br />

Pronator teres<br />

(b) Flexor carpi radial is<br />

(c) Flexor digitorum superficial is and profundus (radial part)<br />

(d) Flexor pollicis longus<br />

(e) Pronator quadratus<br />

(3) The thenar motor branch supplies the following:<br />

(a) Abductor poll icis brevis<br />

(b) Opponens pollicis<br />

(c) Flexor pollicis brevis<br />

(4) The common digital branches innervate the lumbrical muscles of the index (2nd) and middle (3rd) fingers<br />

364


ORTHOPEDIC EMERGENCIES<br />

(5) Provides sensation to the palm on the radial side of the hand and the pal mar aspect of the radial three<br />

and one-half fingers, as well as the dorsal aspect of the tips of the index (2nd) and middle (3rd) fingers<br />

and the radial half of the ring (4th) finger<br />

(6) Testing<br />

(a) Motor test of normal function -<br />

muscles for contractions<br />

(b) Sensation -<br />

opposition of the thumb to each finger while watching thenar<br />

best tested over the volar tip of the index (2nd) finger<br />

(7) Loss of median nerve function - carpal tunnel syndrome; with thenar atrophy in the late stages -<br />

monkey hand<br />

d. Radial nerve<br />

(1) Innervates the following muscles:<br />

(a) Triceps<br />

(b) Brachioradialis<br />

(c)<br />

Extensor carpi radial is longus<br />

(d) Extensor carpi radialis brevis<br />

(e) Supinator<br />

(f)<br />

(g)<br />

Extensor digitorum communis<br />

Extensor digiti mini mi<br />

(h) Extensor carpi ulnaris<br />

(i)<br />

(j)<br />

(k)<br />

(I)<br />

Abductor pollicis longus<br />

Extensor pollicis brevis<br />

Extensor pollicis longus<br />

Extensor indicus proprium<br />

(2) Does not innervate any of the intrinsic muscles of the hand<br />

(3) Provides sensation to the dorsum of the radial aspect of the hand, the dorsum of the thumb, the dorsal<br />

aspect of the index (2nd) and middle (3rd) fingers, and the radial half of the ring (4th) finger as far<br />

distally as the proximal interphalangeal (PIP) joints<br />

(4) Testing<br />

(a) Motor test of normal function -<br />

(b) Sensation -<br />

extension of wrist and fingers against resistance<br />

best tested over the dorsal web space between the thumb and index finger<br />

(5) Primary function - extension of the wrist and metacarpophalangeal (MCP) joints<br />

(6) Loss of radial nerve function - wrist drop, loss of finger extension<br />

e. Intrinsic muscles of the hand<br />

(1) Thenar<br />

(2) Hypothenar<br />

(3) Abductor pollicis<br />

(4) Lumbricals<br />

(5) lnterossei<br />

f. Six groups of extrinsic extensor muscles<br />

(1) Abductor pollicis longus and extensor pollicis brevis<br />

(2) Extensor carpi radial is longus and brevis tendons<br />

(3) Extensor pollicis longus tendon<br />

(4) Extensor indices proprius and extensor digitorum communis<br />

(5) Extensor digiti minimi<br />

(6) Extensor carpi ulnaris<br />

g. Extrinsic flexor muscles<br />

(1) Flexor pollicis longus<br />

(2) Flexor digitorum superficial is<br />

(3) Flexor digitorum profundus<br />

(4) Flexor carpi ulnaris<br />

(5) Flexor carpi radial is<br />

(6) Palmaris longus<br />

365


ORTHOPEDIC EMERGENCIES<br />

2. Regional nerve blocks can be particularly useful when treating finger and hand injuries.<br />

a. Digital blocks<br />

(1) Digital blocks preferred to local infiltration for most finger injuries; local infiltration into the restricted<br />

space of the finger is very painful and can impair capillary refill.<br />

(2) Sensation to the finger (dorsal and volar surfaces and interphalangeal joints) is provided by the palmar<br />

and dorsal digital nerves, which run along the lateral aspects of each phalanx.<br />

(3) Digital blocks are performed by using one of three approaches (each of which has advantages and<br />

disadvantages).<br />

(a)<br />

Dorsal approach: needle is directed into the dorsum of the hand at the metacarpals.<br />

(b) Pal mar approach: needle is directed into the palm over the metacarpal head (very painful).<br />

(c) Web space approach: needle is directed into the interdigital web space.<br />

b. Nerve blocks (wrist)<br />

(1) Useful for extensive hand injuries (particularly the palm, which is very painful to inject into) and MCP<br />

joint injuries, but they require more time and training to learn and, therefore, are less commonly used.<br />

(2) Sensation to the hand is provided by the median, ulnar, and radial nerves; therefore, if complete<br />

anesthesia of the hand is needed, all three of these nerves must be blocked (requires multiple injections).<br />

(3) These blocks are performed on the volar aspect of the wrist at the proximal skin crease where the<br />

tendons are easily palpated and provide landmarks to guide the injections.<br />

(4) More detailed information on performing these blocks can be found in Roberts and Hedges Clinical<br />

Procedures in Emergency Medicine, 6th ed, page 541.<br />

3. Carpal injuries<br />

a. Scaphoid (navicular) fracture<br />

(1) Most common carpal fracture<br />

(2) Mechanism of injury: FOOSH<br />

(3) Initial AP, lateral, and scaphoid radiographic views of the wrist may not demonstrate a fracture in<br />

?:10% of cases. Repeat radiographs of the wrist in 2 weeks will often demonstrate the fracture.<br />

(4) Clinical presentation<br />

(a) Tenderness in the region of the anatomic snuff box<br />

(b) Pain referred to the anatomic snuff box with longitudinal compression of the thumb (axial<br />

loading) or with supination of the hand against resistance; if these findings are present, treat<br />

as a fracture.<br />

(5) Treatment<br />

(a) Nondisplaced and clinically suspected fracture: immobilization in a thumb spica splint and<br />

referral to an orthopedist/hand surgeon. Some hand surgeons immobilize the forearm in a<br />

coaptation (sugar-tong) splint to prevent supination/pronation, because this is thought to<br />

decrease rate of malunion/nonunion.<br />

(b) Displaced fractures usually require open reduction and fixation.<br />

(6) Complications<br />

(a) Avascular necrosis of the proximal fragment; the more proximal, oblique, or displaced the<br />

fracture, the greater the risk because the vascular supply enters the distal part of the bone.<br />

(b) Delayed union, malunion, and nonunion<br />

b. Triquetrum dorsal chip fracture<br />

(1) Second most common carpal fracture<br />

(2) Mechanisms of injury<br />

(a) FOOSH<br />

(b) Direct blow to the dorsum of the hand<br />

(3) Clinical presentation: tenderness immediately distal to the ulnar styloid on the dorsal aspect of the wrist<br />

(4) Best visualized on the lateral view of the wrist<br />

(5) Treatment: immobilization in a volar splint<br />

c. Lunate fracture<br />

(1) Third most common carpal fracture<br />

(2) Mechanism of injury: FOOSH<br />

(3) Clinical presentation: pain and tenderness over the mid-dorsum of the wrist that is increased by axial<br />

compression of the third metacarpal<br />

366


ORTHOPEDIC EMERGENCIES<br />

(4) Plain radiographs of the wrist are often normal; treatment should be started on clinical grounds alone.<br />

(5) Treatment: immobilization in a thumb spica splint and orthopedic referral<br />

(6) Complications<br />

d. Dislocations<br />

(a) Avascular necrosis of the proximal segment (Kienbock disease) is a serious complication.<br />

(b) Most often in patients with congenital shortening of the lunate, but it also results from inadequate<br />

immobilization.<br />

(1) Mechanism of injury: violent hyperextension<br />

(2) Refer all to a hand surgeon<br />

(3) Lunate dislocation<br />

(a) The lunate may dislocate either volarly (most common) or dorsally.<br />

Source: James Heilman, MD<br />

(Wi ki media: http://en. wi ki pedi a.orglwi ki/Fi le: Lunatedislocation L. j pg)<br />

(b) Clinical presentation<br />

1. Pain, swelling, and marked loss of flexion with the wrist, hand, and arm held in anatomic<br />

position<br />

ii. Occasionally, patient complains of tingling in the three radial digits (acute carpal tunnel<br />

syndrome).<br />

(c) Findings on wrist radiograph<br />

1. Best seen on the lateral view: the lunate is displaced volarly relative to the capitate and<br />

carpus, which remain in their normal alignment ("spilled teacup" sign).<br />

ii. On the AP view, the lunate is triangular ("piece of pie" sign). There is also foreshortening of<br />

the wrist and loss of the normal space between the capitate and the lunate.<br />

(d) Associated scaphoid injuries are common.<br />

(4) Perilunate dislocation (most common wrist dislocation)<br />

(a) May be associated with a fracture or dislocation of the scaphoid<br />

(b) The lunate remains in anatomic position relative to the forearm while the capitate is displaced<br />

dorsally because of disarticulation of the capitolunate joint.<br />

(c) Diagnostic evaluation: as with a lunate dislocation, a perilunate dislocation is best diagnosed by<br />

a true lateral film of the wrist.<br />

367


ORTHOPEDIC EMERGENCIES<br />

Source: Hellerhoff<br />

(Wi ki media: http://en. wi kipedi a .org/wi ki/Fi le: Peri I u naere_Luxation. jpg)<br />

(5) Scapholunate dislocation<br />

(a) Clinical presentation: pain with wrist hyperextension and a snapping sensation when the wrist is<br />

deviated in either the radial or ulnar direction<br />

(b) Diagnostic evaluation: radiologic signs<br />

4. Metacarpal injuries<br />

Source: Wikimanos<br />

(Wik i media: http://en. wi kiped i a.orglwi ki/Fi I e: Dynam i c_S-L-gri p. j pg)<br />

1. AP film: The scaphoid is foreshortened and has a dense ring-shaped image around its distal<br />

edge ("signet ring" sign). There is a widening >3 mm in the space between the lunate and<br />

scaphoid ("David Letterman" sign).<br />

11. Lateral film: the angle between the scaphoid and the lunate is increased (>60°).<br />

(c) Treatment: immobilize in a radial gutter splint or posterior mold for orthopedic referral.<br />

a. Metacarpal neck fractures<br />

(1) Result from a punch with a clenched fist<br />

(2) Almost all are unstable.<br />

(3) The proximal fragment angulates in the dorsal direction, while the distal fragment angulates in the<br />

volar direction.<br />

(4) The amount of angulation that is acceptable varies directly with the normal mobility of the involved<br />

metacarpal; the greater the mobility, the greater the degree of angulation that can be tolerated.<br />

368


ORTHOPEDIC EMERGENCIES<br />

(a) Metacarpal neck fractures of the ring (4th) and little (5th) fingers ("Boxer's fracture"); up to 20°<br />

and 40° of angulation, respectively, is acceptable.<br />

(b) Metacarpal neck fracture of the index (2nd) and middle (3rd) fingers; ::c15 ° of angulation<br />

is acceptable.<br />

(5) Rotational deformities, if present, must be completely corrected; look for malalignment of the plane<br />

of the fingernails when the fingers are viewed in the partially flexed position.<br />

Source:www.fpnotebook.com(http://www.fpnotebook.com/_media/OrthoHandPositionFistToScaphoid.jpg).<br />

Used with permission.<br />

b. MCP joint dislocations<br />

(1) MCP dislocations of fingers are rare, while those of the thumb are relatively common.<br />

(2) Usually dorsal and result from hyperextension forces<br />

(3) Types<br />

(a) Simple= subluxation<br />

(b) Complex= complete dislocation<br />

(4) Radiographs<br />

(a) Should be taken both before and after reduction to exclude associated fractures and confirm<br />

adequate reduction<br />

(b) The lateral view usually demonstrates an obvious dislocation.<br />

(5) Treatment<br />

(a) Simple dislocations can usually be managed with closed reduction with adequate anesthesia<br />

(often requires a wrist block of the ulna, median, and/or radial nerves, depending on which finger<br />

is involved), splinting inflexion, and referral to a hand surgeon.<br />

(b) Complex dislocations often cannot be reduced by closed reduction because of interposition of the<br />

volar plate in the MCP joint or entrapment of the metacarpal head between the lumbrical tendon<br />

and a flexor tendon. Irreducible MCP joint dislocations should be splinted and referred to a hand<br />

surgeon for open reduction and operative repair.<br />

(6) Complications<br />

(a) Volar plate injuries<br />

(b) Bony avulsions<br />

(c) Thickening and stiffness of the joint<br />

5. Bennett fracture/subluxation: an unstable fracture at the base of the first metacarpal, often caused by punching;<br />

it should be suspected when evaluating a "sprained thumb."<br />

6. Gamekeeper's or skier's thumb (torn ulnar collateral ligament)<br />

a. Mechanism of injury<br />

(1) Acute and forceful radial deviation of the thumb<br />

(2) Commonly occurs in skiing accidents when the ski pole forcefully abducts the thumb at the MCP joint<br />

as the hand hits the ground in a fall<br />

369


ORTHOPEDIC EMERGENCIES<br />

b. Clinical presentation<br />

(1) Tenderness along the ulnar aspect of the thumb, which is most exquisite at the MCP joint<br />

(2) Thumb grasp and pinch are weak.<br />

c. Diagnostic evaluation<br />

(1) Obtain radiographs of the thumb to exclude associated fractures (present in up to 30% of cases).<br />

(2) Assess joint stability.<br />

d. Treatment<br />

(a) Apply a lateral stress (radial abduction) to the MCP joint of the injured thumb (after adequate<br />

anesthesia).<br />

(b) Stress the normal thumb in the same manner.<br />

(c) The presence of 10°-20° of laxity in the injured thumb when compared with the normal thumb is<br />

consistent with a complete tear.<br />

(1) Incomplete tear (some stability=


ORTHOPEDIC EMERGENCIES<br />

c. Examination of the flexor tendons<br />

(1) Flexor digitorum profundus tendon: immobilize the PIP and MCP joints in extension and ask the<br />

patient to flex the tip of the finger; inability to flex the distal interphalangeal (DIP) joint indicates a<br />

profundus tear that is usually associated with a volar plate slip at the PIP joint.<br />

(2) Flexor digitorum superficial is tendon: hold the uninjured fingers in extension and ask the patient to<br />

flex the injured finger; this blocks the action of the profundus tendon and allows an isolated test of the<br />

superficial is tendon.<br />

(3) Partial tears are detected by evaluating the strength of the flexor tendons against resistance. Patients<br />

with 90% full-thickness lacerations still have normal (although painful) range of motion.<br />

(4) Treatment of partial tendon lacerations is controversial and should be determined in consultation with<br />

a hand surgeon. Many are treated with protective splinting alone.<br />

d. Extensor tendon injuries are usually closed. (Be able to identify on pictorial.)<br />

(1) Mallet finger<br />

(a) Extensor tendon laceration or disruption at the DIP joint (may or may not be associated with<br />

avulsion chip fracture)<br />

(b) Clinical presentation: The patient is unable to extend the DIP joint.<br />

(c) Mechanism of injury: usually a blow to the tip of the extended finger producing sudden forced<br />

flexion<br />

(d) Treatment<br />

i. If there is no associated fracture-,, splint the DIP in full extension to slight hyperextension for<br />

6-8 weeks.<br />

11. If there is an associated fracture-,, the treatment is either splinting as above or surgical<br />

pinning of the avulsed fragment using Kirschner wire fixation.<br />

(e) Complications<br />

i. A delayed complication of an untreated mallet finger is the "swan neck deformity," in which<br />

there is hyperextension of the PIP joint in addition to the mallet flex ion deformity of the DIP<br />

joint.<br />

11. It is the result of increased extension forces on the PIP joint and is produced by proximal and<br />

dorsal migration of the lateral bands.<br />

Courtesy of Michael C. Bond, MD<br />

(2) Boutonniere deformity<br />

(a) Rupture of the central slip of the extensor tendon at the PIP.<br />

(b) Clinical presentation: characterized by flexion of the PIP joint and hyperextension of the DIP joint<br />

371


ORTHOPEDIC EMERGENCIES<br />

(c) Mechanism of injury: a direct blow to (or laceration of) the PIP joint or forced flexion of the PIP<br />

joint against resistance.<br />

(d) This deformity is not always present immediately after the injury, but rather often develops over<br />

time (1-2 weeks after the injury) because the lateral bands of the extensor tendon slip volar to the<br />

axis of the PIP joint and so become paradoxical flexors of this joint.<br />

(e) Treatment: splint the PIP in extension and then refer to a hand surgeon for possible operative<br />

repair.<br />

e. De Quervain stenosing tenosynovitis<br />

(1) An inflammation of the extensor tendons of the thumb<br />

(a) Abductor pollicis longus<br />

(b) Extensor pollicis brevis<br />

(2) Clinical presentation<br />

(a) Complaint of pain along the radial aspect of the wrist that is exacerbated with use of the thumb<br />

(b) Palpation reveals tenderness over the radial styloid.<br />

(c) A positive Finkelstein test (pain on ulnar deviation of the wrist while the thumb is flexed and held<br />

in the palm by the other fingers) confirms the diagnosis.<br />

(3) Mechanism of injury: overuse of the thumb<br />

(4) Treatment<br />

9. Amputated digits<br />

(a) Administer oral NSAIDs.<br />

(b) Place in thumb spica splint.<br />

(c) Inject the first dorsal compartment with a bupivacaine and triamcinolone combination; this can<br />

be done on the initial evaluation or reserved for those patients in whom conservative therapy has<br />

not been effective. (Repeated injections are associated with a risk of subsequent tendon rupture.)<br />

a. Preservation of the amputated part<br />

(1) After irrigating the amputated part with normal saline to remove gross contamination, wrap it in<br />

sterile gauze moistened with lactated Ringer's or normal saline and place it in a sterile, water-tight<br />

container.<br />

(2) Store the container in ice water.<br />

b. Criteria for reimplantation<br />

(1) Young, stable patient<br />

(2) Sharply incised wound with minimal associated damage<br />

(3) Thumb amputation<br />

(4) Multiple-digit amputation<br />

(5) Hand or forearm amputation<br />

(6) Amputation in a child<br />

c. Contraindications to reimplantation<br />

(1) Absolute<br />

(a) Unstable patient with other life-threatening injuries<br />

(b) Severe crush injury<br />

(2) Relative<br />

(a) Severely damaged or contaminated part<br />

(b) Single-digit amputation (other than the thumb)<br />

(c) Avulsion injury<br />

(d) Serious underlying systemic illness (eg, CHF, diabetes mellitus)<br />

(e) Prolonged warm ischemia (2:12 hours)<br />

(f)<br />

Prior injury/surgery to affected part<br />

(g) Emotionally unstable patient<br />

372


ORTHOPEDIC EMERGENCIES<br />

Ill. PELVIS AND HIP INJURIES<br />

A. Essential anatomy of the pelvis<br />

1. Pelvic bones: innominate (consists of the ilium, ischium, and pubis), sacrum, and coccyx<br />

2. On the AP view of the pelvis, the symphysis pubis is usually 5 mm wide, and the sacroiliac joint is normally<br />

2-4 mm wide.<br />

3. The lumbar and sacral nerves run through the pelvis.<br />

4. The ileopectalline divides the upper (false) pelvis and the lower (true) pelvis. Injuries to the true pelvis may be<br />

associated with injuries to the following structures:<br />

a. Bladder or urethra<br />

b. Descending colon<br />

C. Sigmoid colon<br />

d. Rectum<br />

e. Anus<br />

f. Nerve roots<br />

g. Vasculature<br />

B. Pelvic fractures<br />

1. Mechanism of injury<br />

a. Motor vehicle and motorcycle collisions (most common cause of injury in adults)<br />

b. Falls<br />

c. Crush injuries<br />

d. Pedestrian hit by motor vehicle (leading cause of injury in children)<br />

2. Classification: Several different systems have developed over the years to classify pelvic fractures. The<br />

central focus of most of these systems is the underlying stability of the pelvis. The Young-Burgess system is<br />

most favored by emergency physicians, because it is valuable in predicting hemorrhage related to pelvic<br />

fracture and local vascular injury. This system is based on the mechanism of injury (AP compression or lateral<br />

compression) as well as the degree of pelvic instability.<br />

3. Classification types: see page 374 for illustrations; arrow indicates direction of force.<br />

4. Lateral compression<br />

a. Type I: posteriorly directed force - sacral crushing and horizontal pubic ramus fractures ipsilaterally<br />

b. Type II: anteriorly directed force - horizontal pubic ramus fractures with anterior sacral crushing and<br />

disruption of the posterior sacroiliac joints or fractures through the iliac wing<br />

373


ORTHOPEDIC EMERGENCIES<br />

c. Type Ill: anteriorly directed force continued, causing external rotation of the contralateral side<br />

(1) Sacroiliac joint opened posteriorly<br />

(2) Sacrotuberous and spinous ligaments disrupted<br />

5. Anteroposterior (AP) compression<br />

a. Type I: symphysis disrupted but posterior ligamentous structures intact<br />

b. Type II: continues type I, with disruption of the sacrospinous and possibly the sacrotuberous ligaments and<br />

an anterior sacroiliac joint opening<br />

c. Type Ill: continued force disrupts the sacroiliac ligaments<br />

6. Vertical shear<br />

a. Vertical fractures in the rami; disruption of all posterior ligaments<br />

b. Equivalent to an AP type Ill or completely unstable and rotationally unstable fracture<br />

Ligamentous anatomy of the pelvis<br />

A, lateral compression<br />

II<br />

Ill<br />

8, anteroposterior<br />

(AP) compression<br />

t<br />

II<br />

t<br />

Ill<br />

C, vertical shear t<br />

7. Clinical presentation<br />

a. Destot sign: a superficial hematoma above the inguinal ligament or on the scrotum<br />

b. Earle sign: on rectal examination, a large hematoma or tenderness along the fracture line or palpation of a<br />

bony prominence<br />

c. Roux sign: a diminished distance measured from the greater trochanter to the pubic spine on one side<br />

compared with the other (indicates an overlapping fracture of the anterior pelvic ring)<br />

8. Diagnostic evaluation: CT<br />

a. Superior to standard radiographs in evaluating the acetabulum, posterior (femorosacral) arch, and sacroiliac<br />

joint<br />

b. Often provides information on the degree of pelvic instability and the presence of a retroperitoneal<br />

hematoma<br />

374


ORTHOPEDIC EMERGENCIES<br />

9. Associated injuries: more common in children because the developing pelvis provides a lesser degree of<br />

protection<br />

a. Hemorrhage<br />

(1) Primary cause of death in patients with pelvic fractures; up to 6 L of blood can be accommodated in<br />

the retroperitoneal space.<br />

(2) Most common in patients with Type Ill and posterior fractures<br />

(3) Up to 50% of patients require transfusion.<br />

(4) Initial control obtained by application of a pelvic binder or tying a sheet across the pelvis, followed by<br />

external fixation and angiographic embolization in some patients. Resuscitative endovascular balloon<br />

occlusion of the aorta (REBOA) is a newer procedure to limit blood loss in hemodynamically unstable<br />

patients until hemorrhage can be more definitively controlled.<br />

b. Injuries of the urethra and bladder<br />

(1) Most common associated injury<br />

(2) Urethral injuries (particularly posterior urethral tears in men) are more common than bladder injuries.<br />

c. Vaginal laceration or rupture (relatively uncommon but may be seen with anterior pelvic fractures)<br />

d. Nerve damage (most commonly occurs in association with posterior pelvic fractures)<br />

e. Ruptured diaphragm (not uncommon and is often missed)<br />

f. Rectal injuries<br />

(1) Uncommon<br />

(2) Generally occur in association with ischial and urinary tract injuries<br />

(3) Prophylactic antibiotics should be administered without delay.<br />

g. Thoracic aortic rupture (traumatic aortic disruption): incidence is 8 times greater in patients with pelvic<br />

fractures than in patients with blunt abdominal trauma.<br />

10. Complications of pelvic fractures<br />

a. Sepsis<br />

b. Thromboembolic complications (pulmonary and fat emboli)<br />

c. Malunion or delayed union<br />

d. Chronic pain<br />

11. Specific pelvic fractures<br />

a. Pubic rami<br />

(1) A single pubic ramus fracture is the most common pelvic fracture. Common mechanism is a fall on<br />

the buttocks. Fracture may be missed on plain radiographs; order an MRI if in doubt or if it will change<br />

management.<br />

(2) A "straddle injury" is a fracture of all four pubic rami, such as occurs in a fall on the crotch with the<br />

legs apart.<br />

b. Iliac crest (result from impact collisions)<br />

(1) Duverney fracture (pelvic wing fracture)<br />

(2) Ilium fracture - pelvic ring disruption, visceral injuries<br />

c. Multiple pelvic fractures (Malgaigne fracture)<br />

(1) Involves the pubic rami bilaterally and the ilium or sacrum<br />

(2) Results from vertical shear forces (eg, fall from a height)<br />

C. Hip dislocations (an orthopedic emergency [in the absence of a prosthetic femoral head])<br />

1. Anterior: 10% of hip dislocations (Be able to identify on pictorial.)<br />

a. Classified according to the final resting position of the femoral head<br />

(1) Superior iliac: femoral head is palpable in the area of the superior iliac spine.<br />

(2) Superior pubic: femoral head rests near the pubis.<br />

(3) Inferior (obturator): femoral head lies in the obturator foramen.<br />

b. Mechanism of injury<br />

(1) Extreme abduction causes the femoral head to be pushed out through a tear in the anterior capsule.<br />

(2) Causes include:<br />

(a) Motor vehicle collision<br />

(b) A fall<br />

(c) A blow to the back while squatting<br />

375


ORTHOPEDIC EMERGENCIES<br />

c. Clinical presentation<br />

(1) Limb appearance (varies with the type of dislocation present)<br />

(a) With anterior superior dislocations, the limb is slightly abducted, externally rotated,<br />

and extended.<br />

(b) With anterior inferior dislocations, the limb is abducted, externally rotated, and flexed.<br />

(2) Associated physical findings<br />

(a)<br />

Diminishing femoral or distal pulse (or progressive swelling of the leg) indicates the need for<br />

immediate reduction of the hip.<br />

(b) If the femoral nerve is involved, the following will be diminished:<br />

i. Quadriceps function<br />

ii. Deep tendon reflexes at knee<br />

iii. Sensation on anteromedial thigh<br />

2. Posterior: 80%-90% of hip dislocations (Be able to identify on radiograph.)<br />

a. Mechanism of injury: most cases result from motor vehicle collisions in which a direct force is applied<br />

to the flexed knee (hitting the dashboard) that is transmitted to the femoral head, which is pushed out<br />

through the posterior capsule.<br />

b. Clinical presentation<br />

Posterior Hip Dislocation<br />

Anterior Hip Dislocation<br />

3. Treatment<br />

(1) Limb appearance: shortened, adducted, internally rotated, and flexed<br />

(2) Associated physical findings<br />

(a) Evidence of acetabular/femoral fractures<br />

(b) Knee injury<br />

(c) Sciatic nerve injury results in loss of:<br />

i. Muscle function below the knee<br />

ii. Ability to flex the knee<br />

iii. Sensation on posterolateral leg and sole of the foot<br />

a. Early reduction is required; avascular necrosis of the femoral head increases in direct proportion to delay<br />

in reduction.<br />

b. Immediate closed reduction (using procedural sedation) should be attempted in the emergency<br />

department as soon as appropriate radiographs have been obtained to exclude associated injuries. If this<br />

is unsuccessful, closed reduction under general anesthesia is indicated. Neurovascular assessment should<br />

be performed before and after any reduction attempts.<br />

4. Complications<br />

a. Early complications<br />

(1) Anterior dislocations _,. femoral artery, vein, and nerve injury<br />

(2) Posterior dislocation _,. sciatic nerve injury (10% of patients)<br />

b. Late complications (in addition to osteoarthritis)<br />

376


ORTHOPEDIC EMERGENCIES<br />

D. Hip fractures<br />

(1) Anterior dislocation - femoral artery/vein thrombosis - pulmonary embolism and avascular<br />

necrosis of the femoral head<br />

(2) Posterior dislocation - avascular necrosis of the femoral head (15 %-30% of patients)<br />

1. Classification<br />

a. lntracapsular (femoral head and neck fractures) or<br />

b. Extracapsular (trochanteric, intertrochanteric, and subtrochanteric)<br />

c. This classification system has prognostic value in that intracapsular fractures are far more likely to result<br />

in a disrupted vascular supply and subsequent avascular necrosis than extracapsular fractures.<br />

lntracapsular lntertrochanteric Subtrochanteric<br />

2. Clinical presentations<br />

a. An elderly patient complains of hip pain after a fall. The patient limps when he or she walks or may be<br />

unable to walk. The affected leg is shortened, abducted, and in slight external rotation - femoral neck<br />

fracture<br />

b. A patient complains of hip pain after a fall. The leg is shortened and in marked external rotation -<br />

intertrochanteric fracture (fracture line between the greater and lesser trochanters)<br />

c. The ability to ambulate does not exclude a fracture; some patients with nondisplaced or minimally<br />

displaced fractures may be able to bear weight.<br />

3. Diagnostic evaluation<br />

a. Negative plain radiographs do not exclude a fracture.<br />

b. Patients with negative plain radiographs who are unable to ambulate require further evaluation (CT, MRI,<br />

or bone scan).<br />

4. Complications: most common is avascular necrosis (15 %-35 % of femoral neck fractures in the elderly)<br />

E. Pediatric hip disorders<br />

1. Classic clinical scenarios<br />

a. You are examining a female infant with some other condition. Because you routinely perform the<br />

Ortolani maneuver in all children


ORTHOPEDIC EMERGENCIES<br />

a history of trivial antecedent trauma. The affected leg is externally rotated and adducted (and may be<br />

shortened). Further evaluation reveals tenderness around the hip and decreased range of motion (internal<br />

rotation and abduction [full flexion] are limited). Radiographs (AP and bilateral frog-leg lateral views of the<br />

hips) confirm the diagnosis - slipped capital femoral epiphysis.<br />

2. Developmental dysplasia of the hip<br />

a. More common in females than males<br />

b. More common in first born, breech presentation<br />

c. Palpable click or clunk detected with range of motion of affected hip<br />

3. Septic arthritis<br />

a. Staphylococcus aureus most common organism<br />

b. Ultrasound of hip will often show an effusion<br />

c. Diagnosed by arthrocentesis of hip, usually under fluoroscopic guidance by orthopedics<br />

d. Synovial fluid analysis: WBC count 80,000-200,000/mm 3 with >75% PMNs<br />

e Treatment is antibiotics.<br />

4. Transient (toxic) synovitis<br />

a. Nonspecific inflammation of the synovium of the hip<br />

b. Often due to viral illness<br />

c. Diagnosis of exclusion: requires arthrocentesis to exclude septic arthritis<br />

d. Synovial fluid analysis: WBC count 5,000-15,000/mm 3 with


ORTHOPEDIC EMERGENCIES<br />

IV. KNEE INJURIES<br />

A. Essential anatomy<br />

1. There are five stabilizers (two static and three dynamic) of the lateral aspect of the knee.<br />

a. Static (ligamentous) stabilizers<br />

(1) Lateral collateral ligament (LCL)<br />

(2) Lateral joint capsule<br />

b. Dynamic (muscular) stabilizers<br />

(1) lliotibial tract<br />

(2) Popliteus muscle<br />

(3) Biceps femoris muscle<br />

2. There are four stabilizers of the medial aspect of the knee:<br />

a. Static stabi I izers<br />

(1) Medial collateral ligament (MCL): injury to this ligament is much more common than to the LCL<br />

(2) Medial joint capsule<br />

b. Dynamic stabilizers<br />

(1) Semimembranosus muscle<br />

(2) Pes anserinus<br />

3. The quadriceps tendon permits extension and is the primary dynamic stabilizer of the knee. It is formed by<br />

four muscles:<br />

a. Rectus femoris<br />

b. Vastus lateralis<br />

c. Vastus intermedius<br />

d. Vastus medialis<br />

4. The cruciate ligaments<br />

a. Two internal bands extending from the tibia to the femur (one anteriorly, the other posteriorly)<br />

b. Control anteroposterior and rotatory stability of the knee and prevent hyperextension<br />

c. Anterior cruciate ligament (ACL) is the most frequently injured ligament in the knee. It has a rich blood<br />

supply, which accounts for the high incidence of hemarthrosis when this ligament is injured.<br />

d. Posterior cruciate ligament (PCL) is significantly stronger than the ACL and collateral ligaments; thus, injury<br />

to it is rare and is usually associated with severe knee injuries.<br />

B. Mechanisms of knee injury<br />

1. Va Igus (abduction) stress: most common mechanism of injury<br />

a. Definition: outward stress on lower leg with angulation of the foot away from the midline<br />

b. Clinical presentation<br />

(1) MCL (eg, ski injury): 61%<br />

(2) ACL: patient stumbled, caught himself or herself, and now cannot climb steps.<br />

(3) Medial meniscus: patient complains of knee "locking" in place or clicking (medial meniscus attached<br />

toMCL).<br />

2. Varus (adduction) stress: uncommon mechanism of injury<br />

a. Definition: inward stress on lower leg with angulation of the foot toward the midline<br />

b. Clinical presentation<br />

(1) LCL (eg, skier fails to maintain a snow-plow and the tips of his or her skis cross): significant disruption<br />

__,. peroneal nerve injury<br />

(2) Lateral posterior capsule: knee buckling or "giving way"<br />

(3) Lateral meniscus: knee "locking" or clicking<br />

3. Hyperextension stress<br />

a. Definition: backward stress with the knee extended<br />

b. Usually results in injury to the cruciate ligaments<br />

c. Clinical presentation<br />

(1) ACL: hyperextension of the knee<br />

379


ORTHOPEDIC EMERGENCIES<br />

(2) Rupture of the posterior capsule: knee buckling, collapsing, or "giving way"<br />

(3) Isolated PCL: blow to the anterior aspect of the tibia with the knee in extension or hyperextension<br />

4. Anterior and posterior forces of the tibia on the femur<br />

a. Usually occur with the knee in flexion<br />

b. Clinical presentation<br />

(1) ACL/tibia is forced anteriorly relative to the femur.<br />

(2) PCL/tibia is forced posteriorly relative to the femur.<br />

5. Rotational stress<br />

a. Definition: rotation of the knee (tibia relative to femur) with the foot firmly planted on the ground<br />

(1) Internal rotation - ruptures the ACL<br />

(2) External rotation - disrupts the ACL and/or the medial capsular and MCLs<br />

b. Clinical presentation: a sudden stop and pivot while running; partial ligament ruptures are often more<br />

painful than complete tears, and cruciate ligament injuries are more common in women.<br />

C. Hemarthrosis in association with a negative radiograph indicates the presence of one of the following:<br />

1. Ligamentous tear<br />

a. Hemarthrosis usually develops within 2 hours of injury.<br />

b. Torn ACL is responsible for 75 % of all hemarthroses of the knee.<br />

2. Peripheral meniscus tear<br />

a. Effusion develops gradually (6-24 hours after injury).<br />

b. Usually occurs in association with rotational force to a weight-bearing knee<br />

3. Osteochondral fracture<br />

a. Hemarthrosis develops immediately.<br />

b. Usually occurs in children, particularly adolescent males<br />

4. When aspiration of a hemarthrosis reveals the presence of fat globules (lipohemarthrosis), a fracture is likely.<br />

D. Physical examination of the knee should include the following tests:<br />

1. Stability tests<br />

a. Va Igus stress in 30° of flexion - instability indicates rupture of the MCL.<br />

b. Valgus stress in extension - instability indicates rupture of the MCL as well as potential injury to the<br />

cruciate ligaments and posterior capsule.<br />

c. Varus stress in 30° of flexion - instability indicates rupture of the LCL.<br />

d. Varus stress in extension - instability indicates rupture of the LCL as well as potential injury to the iliotibial<br />

tract, popliteus muscle, and lateral capsule and cruciate ligaments.<br />

e. Anterior and posterior drawer signs: test the ACL and the PCL<br />

f. Lach man test: another test of the ACL; more sensitive and accurate than the anterior drawer sign<br />

g. McMurray and Ege tests: aid in diagnosing meniscal tears<br />

(1) McMurray test - joint line tenderness<br />

(2) Ege test - pain and/or a click on maximum rotation of the knee with the patient in a squatting<br />

position (external rotation - medial meniscus tear; internal rotation - lateral meniscus tear)<br />

2. Patellar compression and apprehension test: pain is elicited with chondromalacia patellae (an overuse<br />

syndrome often seen in young women).<br />

E. Knee dislocation (a true orthopedic emergency)<br />

1. Results from violent trauma, most commonly motor vehicle collisions, vehiclepedestrian collisions, and<br />

contact sports<br />

2. Knee dislocations are classified according to the direction of the tibial displacement relative to the femur<br />

a. 5 types: anterior, posterior, medial, lateral, and rotary<br />

b. Anterior and posterior dislocations are the most common (50%-60%).<br />

3. Clinical presentation<br />

a. Diagnosis is essential because of the high incidence of associated injuries.<br />

b. The knee may relocate spontaneously, and the diagnosis of dislocation may be missed. Any patient who<br />

presents after trauma with a grossly unstable knee (a tear in three of the four major ligaments of the<br />

knee) should be assumed to have a spontaneously reduced dislocation until further evaluation excludes<br />

this injury.<br />

380


ORTHOPEDIC EMERGENCIES<br />

c. Complete disruption of all major ligaments plus meniscal injury<br />

d. Popliteal artery injury (21 %-32%) is particularly common with anterior and posterior dislocations<br />

(up to 40%).<br />

e. Peroneal nerve injury (25%-35%)<br />

(1) Paresthesia of the dorsal aspect of the foot<br />

(2) Diminished dorsiflexion of the foot (foot drop)<br />

(3) Decreased sensitivity between the first and second toes<br />

(4) Seen most often with medial and posterolateral dislocations<br />

f. Tibial nerve injury (plantar foot numbness/paresthesias and weakness with plantar flexion) is less<br />

common than peroneal nerve injury.<br />

g. Proximal tibial fractures<br />

h. The knee with complete disruption of all major ligaments (and the joint capsule) may exhibit less swelling<br />

and pain than the less severely injured knee with a tense hemarthrosis.<br />

i. Popliteal artery injury may be present even if the pulses are intact or the foot is warm.<br />

4. Treatment<br />

a. The dislocated knee should be reduced immediately in the neutral position (extension of the knee<br />

stretches the popliteal artery) by longitudinal traction in a sedated patient. Do not wait for radiographic<br />

confirmation.<br />

b. The popliteal, dorsalis pedis, and tibialis posterior pulses should be checked before and after reduction.<br />

c. After reduction, the knee should be immobilized in a posterior splint in 15° flexion.<br />

d. Doppler flow ultrasound, CT angiography, or standard angiography should be strongly considered in all<br />

cases in consultation with a vascular surgeon, even if pulses are present. At a minimum, ankle-brachia!<br />

indexes (ABls) should be obtained to ascertain if these is a vascular injury.<br />

(1) Distal pulses may still be present but a vascular injury (eg, intimal flap injury, pseudoaneurysm or<br />

partial intimal tear) may be present and lead to vascular complications if diagnosis is delayed.<br />

(2) If immediate surgical intervention is indicated, arteriography should be performed in the operating<br />

room, because performance in the radiology department is associated with an additional delay of<br />

1-2 hours before surgery.<br />

e. The peroneal and tibial nerves should be tested before and after reduction.<br />

f. Immediate orthopedic (and possibly vascular) surgery consult is indicated whenever this injury is<br />

suspected.<br />

g. Immediate surgical intervention is indicated for:<br />

(1) Popliteal artery injuries (arterial repair should be accomplished within 6 hours to avoid the<br />

complications of prolonged ischemia).<br />

(2) Open dislocations<br />

(3) Irreducible dislocations<br />

F. Rupture of the quadriceps mechanism<br />

1. May result from one of the following injuries:<br />

a. Quadriceps tendon rupture<br />

b. Patellar tendon rupture<br />

c. Patellar fracture<br />

d. Avulsion of the tibial tuberosity<br />

2. Clinical presentation<br />

a. Complete disruption: the patient can stand but cannot walk or extend the knee; on physical examination,<br />

there is diffuse swelling around the knee and a palpable deficit.<br />

b. Incomplete disruption: active extension is weak and painful.<br />

3. Treatment<br />

a. Early orthopedic consult<br />

b. Complete ruptures_,. early surgical repair<br />

c. Partial tear_,. immobilization in full extension<br />

G. Patellar subluxation/dislocation<br />

1. Mechanisms of injury<br />

a. Sudden flexion and external rotation of the tibia on the femur with concomitant contraction of the quadriceps<br />

b. Direct blow to the patella with the knee inflexion or extension<br />

381


ORTHOPEDIC EMERGENCIES<br />

2. Lateral subluxation!dis/ocation is the most common.<br />

3. Clinical presentation: adolescent females with chronic patellofemoral anatomic abnormalities such as<br />

genu valgum (knock-knees), patellae alta (high-riding patellae), a large Q (quadriceps) angle, or generalized<br />

joint laxity<br />

4. Diagnostic evaluation: obtain AP and lateral radiographs of the knee before and after reduction to exclude<br />

associated osteochondral fracture of the patellae or lateral femoral condyle.<br />

5. Treatment<br />

a. Patellar subluxations often reduce spontaneously.<br />

b. Reduction is accomplished by flexing the patient's hip and applying gentle, medially directed pressure over<br />

the lateral aspect of the pate! lae while extending the knee.<br />

c. Immobilize in full extension.<br />

d. Crutches: patient should be nonweight-bearing.<br />

e. Orthopedic referral for possible surgical intervention.<br />

f. Isometric quadriceps exercise should be started as soon as the patient can do them without pain.<br />

H. Patellar fracture<br />

1. One of several possible injuries to the extensor mechanism of the knee<br />

2. Mechanisms of injury<br />

a. Direct blow (most common), eg, against the dashboard of a car<br />

b. Forceful contraction of the quadriceps muscles<br />

3. Transverse fractures are the most common (50%-80% of cases).<br />

4. Clinical presentation: patients typically present with tenderness and swelling over the patella and limited,<br />

painful knee extension (if the extensor mechanism is intact) or inability to extend the knee (if the extensor<br />

mechanism is torn).<br />

5. Diagnostic evaluation: radiographs, including AP, lateral, and sunrise (skyline) views<br />

6. Treatment<br />

a. Nondisplaced or minimally displaced and the extensor mechanism is intact<br />

(1) Immobilize in full extension, partial weight-bearing advancing to full weight-bearing as tolerated<br />

(2) Orthopedic referral<br />

b. If the fracture fragments are widely displaced (>3 mm) or there is loss of extensor function<br />

(1) Orthopedic referral for surgical intervention is required.<br />

(2) When fracture fragments are widely separated, associated concomitant knee joint injury is frequently<br />

present.<br />

I. Tibial plateau fracture (most common fracture of the knee)<br />

1. Most common mechanism of injury is a strong valgus stress with axial loading.<br />

2. Most condylar fractures (55%-70%) involve the lateral plateau.<br />

3. Vascular complications are common.<br />

a. Fracture fragments in the subcondylar area - popliteal artery<br />

b. Displaced fracture of the lateral condyle- anterior tibial artery<br />

4. An intra-articular fracture<br />

5. Treatment consists of placing the patient in a straight leg knee immobilizer or posterior leg splint.<br />

J. Baker cyst<br />

1. An inflammation of the semimembranosus or medial gastrocnemius bursa produced by protrusion of the<br />

synovial membrane through the posterior aspect of the knee capsule<br />

2. In adults, these cysts are usually due to an intra-articular abnormality of the knee, such as chronic inflammatory<br />

arthritis or a torn meniscus. In children, however, these cysts appear to be a primary lesion.<br />

3. Clinical presentation<br />

a. History of intermittent swelling behind the knee and may complain of local discomfort<br />

b. Examination reveals a tense and occasionally painful fluid-filled sac in the medial aspect of the popliteal<br />

fossa.<br />

c. Rupture of the cyst with inferior dissection of the synovial fluid causes calf pain and swelling, often<br />

simulating thrombophlebitis or a deep vein thrombosis of the calf.<br />

382


ORTHOPEDIC EMERGENCIES<br />

4. Treatment<br />

a. In adults, management is directed toward correction of the underlying joint pathology.<br />

b. In children, treatment is generally unnecessary, because most of these cysts resolve on their own in 1-2 years.<br />

K. Osgood-Schlatter disease (traumatic tibial apophysitis)<br />

1. Inflammation of the apophysis of the tibial tubercle leads to partial avulsion and separation of the tibial tubercle<br />

2. Mechanism: repetitive microscopic injury to the tibial tubercle<br />

3. Clinical presentation<br />

a. Commonly seen in adolescents (particularly males) who are active in sports<br />

b. Pain and swelling in the region of the tibial tuberosity that is increased with forceful leg extension (climbing<br />

stairs, running, jumping); often bilateral (25% of cases)<br />

4. Treatment: ice, NSAIDs, and decreased participation in sports<br />

L. Chondromalacia patellae (patellofemoral pain syndrome/patellar malalignment syndrome)<br />

1 . Most common cause of knee pain<br />

2. Usually occurs in young adults (particularly females)<br />

3. Often bilateral<br />

4. Causes include patellofemoral malalignment, recurrent patellar subluxation/dislocation, and excessive knee<br />

strain (athletes)<br />

5. Clinical presentation<br />

a. Intermittent peripatellar pain that is exacerbated by prolonged sitting and climbing steps<br />

b. Pain with compression of the patellae into the femoral groove (patellar compression test) and with knee<br />

extended and quadriceps tightened<br />

c. Quadricep muscle contraction in anticipation of pain when the patella is displaced laterally while the<br />

knee is extended and relaxed (patellar apprehension test)<br />

d. Findings of patellar malalignment (Q angle >20°) may also be present. The Q angle is formed by the<br />

intersection of two lines: one drawn from the anterior superior iliac spine to the center of the patellae<br />

and the second drawn from the center of the patellae through the tibial tuberosity (the normal Q angle<br />

is 15°).<br />

6. Treatment: rest and NSAIDs, followed by isometric quadricep-strengthening exercises (quadricep contraction<br />

with adduction)<br />

V. LOWER LEG INJURIES<br />

A. The soft tissue surrounding the tibia and fibula is divided into four compartments.<br />

1 . Anterior compartment<br />

a. Dorsiflexor muscles of the ankle and foot (tibial is anterior, extensor digitorum longus, extensor hallucis<br />

longus, and peroneus tertius)<br />

b. Anterior tibial artery<br />

(1) Most frequently injured structure in this compartment<br />

(2) Injury occurs secondary to fractures of the:<br />

(a)<br />

Latera I tibial plateau<br />

(b) Proximal third of the tibia<br />

c. Deep peroneal nerve<br />

(1) Provides sensation to the first dorsal web space of the foot<br />

(2) May be injured because of fracture of the lateral tibial plateau<br />

2. Lateral (peroneal) compartment<br />

a. Everter and plantar flexor muscles of the foot (peroneus brevis and longus muscles)<br />

b. Superficial peroneal nerve<br />

(1) Provides sensation to the dorsum of the foot<br />

(2) Frequently injured because of fracture of the fibular neck or proximal shaft<br />

3. Deep posterior compartment<br />

a. Tibial is posterior, flexor digitorum longus, and flexor hallucis longus muscles<br />

b. Posterior tibial and peroneal arteries<br />

c. Posterior tibial nerve provides sensation to the sole of the foot.<br />

383


ORTHOPEDIC EMERGENCIES<br />

4. Superficial posterior compartment<br />

a. Gastrocnemius, plantaris, and soleus muscles<br />

b. Sural nerve provides sensation to the lateral aspect of the foot and the distal calf.<br />

B. Anterior compartment syndrome<br />

1. More common than the lateral or posterior compartment syndromes<br />

2. Usually occurs in association with proximal tibial fractures but can result from prolonged strenuous exercise<br />

(particularly in runners) or other causes<br />

3. Clinical presentation<br />

a. Anterior tibial pain that is:<br />

(1) Out of proportion to the injury (one of the earliest findings)<br />

(2) Increased with active dorsiflexion of the foot (one of the earliest findings)<br />

(3) Increased with passive plantar flexion of the foot<br />

b. Decreased two-point sensory discrimination (most consistent finding)<br />

c. Paresthesia or hypesthesia in the distribution of the deep peroneal nerve (early finding)<br />

d. Weak dorsiflexion of the ankle and toes<br />

e. Palpable tenseness and tenderness of the compartment<br />

f. Pallor of the skin (late finding)<br />

g. Pulselessness or reduced distal pulses (a very late and frequently absent finding). The presence of<br />

peripheral pulses does not exclude increased intracompartmental pressure; pulses often remain intact.<br />

4. Treatment<br />

a. Remove constrictive dressing or cast (if present).<br />

b. Obtain immediate orthopedic consult and measure intracompartmental pressure.<br />

c. Surgical decompression via fasciotomy is indicated if the diastolic blood pressure minus the<br />

intracompartmental pressure is >30 mmHg.<br />

C. Stress fractures of the fibula<br />

1. Result from overtraining and usually occur at the distal fibula<br />

2. Patients present with pain exacerbated with ambulation.<br />

3. Initial radiographs are usually negative.<br />

4. Treatment is symptomatic.<br />

VI. ANKLE INJURIES<br />

A. Essential anatomy<br />

1. Three groups of ligaments unify the bony structures of the ankle<br />

a. Medial collateral (deltoid) ligament<br />

(1) Provides medial support to the ankle<br />

(2) Originates from the medial malleolus and inserts on the navicular and talus<br />

b. Anterior talofibular, calcaneofibular, and posterior talofibular ligaments - provide lateral support.<br />

c. Ligaments of syndesmosis - bind the lower portions of the tibia and fibula together.<br />

(1) Anterior and posterior tibiofibular ligaments<br />

(2) Transverse tibiofibular ligament<br />

(3) lnterosseous ligament (provides the strongest bond between the tibia and fibula at the ankle joint)<br />

2. Four groups of muscles traverse the ankle joint<br />

a. Anteriorly - dorsiflexion of the ankle<br />

(1) Tibial is anterior<br />

(2) Extensor digitorum longus<br />

(3) Extensor hallucis longus<br />

b. Medially - inversion of the foot<br />

(1) Tibial is posterior<br />

(2) Flexor digitorum longus<br />

(3) Flexor hallucis longus<br />

384


ORTHOPEDIC EMERGENCIES<br />

c. Posteriorly----,, plantar flexion<br />

(1) Gastrocnemius<br />

(2) Plantaris<br />

(3) Soleus<br />

d. Laterally----,, eversion and plantar flexion<br />

(1) Peroneus longus<br />

(2) Peroneus brevis<br />

B. Sprains (ligamentous injury)<br />

1. Lateral collateral ligament (LCL) sprains<br />

a. Most common (85%-90%)<br />

b. Result from inversion and internal rotation of the plantarflexed foot<br />

c. Ligamentous injury occurs in sequence, starting anteriorly with the anterior talofibular ligament and<br />

progressing posteriorly to the calcaneofibular ligament, followed by the posterior talofibular ligament.<br />

d. The most frequently injured ligament ----,, anterior talofibular<br />

2. Medial collateral ligament sprains are uncommon.<br />

a. Result from eversion and external rotation of the foot<br />

b. Often associated with either a fracture of the fibula or a tear of the tibiofibular syndesmosis<br />

3. Distal tibiofibular syndesmotic ligament sprains (high ankle sprains) are also uncommon.<br />

a. Result from dorsiflexion and eversion forces with an axial load<br />

b. Have a prolonged recovery period<br />

4. Classification is based on the clinical presentation and the degree of instability present (as demonstrated by<br />

stress testing).<br />

a. First degree<br />

(1) Mild localized tenderness<br />

(2) Minimal swelling<br />

(3) No instability or functional loss<br />

(4) Able to beai- weight<br />

(5) No abnormal motion<br />

b. Second degree<br />

(1) Moderate swelling and tenderness<br />

(2) Moderate functional loss<br />

(3) Increased pain with stress testing<br />

c. Third degree<br />

(1) Marked tenderness<br />

(2) Egg-shaped swelling over the affected ligament(s) within 2 hours of injury<br />

(3) Significant functional loss<br />

(4) Inability to bear weight<br />

(5) Resistance to motion of the foot<br />

(6) Positive stress test<br />

5. Stress tests for ankle stability are useful in confirming specific ligamentous injury and in distinguishing second<br />

from third-degree sprains.<br />

a. Anterior drawer test: abnormal movement indicates a rupture of at least the anterior talofibular ligament.<br />

b. Inversion stress (or talar tilt) test<br />

(1) Needs to be done only if the anterior drawer test is positive<br />

(2) Abnormal movement indicates rupture of both the anterior talofibular and the calcaneofibular<br />

ligaments.<br />

c. External rotation test<br />

(1) Pain at the syndesmosis or a sensation of lateral talar movement indicates injury to the distal<br />

tibiofibular syndesmotic ligament.<br />

(2) Accurate results require the presence of good muscle relaxation obtained with application of ice or<br />

infiltration of a local anesthetic.<br />

385


ORTHOPEDIC EMERGENCIES<br />

6. Associated injuries<br />

a. Maisonneuve fracture<br />

(1) Proximal fibular fracture that occurs in association with rupture of the deltoid ligament or an<br />

avulsion fracture of the medial malleolus at the insertion site of the distal talofibular ligament<br />

(2) Should be suspected in patients presenting with a history of eversion injury and significant medial<br />

malleolar tenderness and swelling<br />

b. Osteochondral fracture of the talar dome<br />

c. Avulsion fracture of the fifth metatarsal (pseudo-Jones or dancer's fracture) occurs with flexioninversion<br />

injuries of the foot.<br />

d. Peroneal tendon dislocation or subluxation<br />

Fibula<br />

Tibia<br />

Anterior tibiofibular<br />

ligament<br />

Talus<br />

7. Treatment<br />

a. Determined by the degree of injury<br />

b. First- and most second-degree sprains<br />

(1) Ice, elevation, immobilization in a protective device (air cast, brace, splint, Unna boot), and crutches<br />

as needed.<br />

(2) As pain and swelling decrease, continue with graded exercises to increase dorsiflexion and peroneal<br />

strength, weight-bearing as tolerated, and reevaluation by the primary care provider.<br />

c. Severe second-degree sprains<br />

(1) Ice, elevation, splint immobilization (stir-up splint, air cast, or posterior mold), prolonged nonweightbearing,<br />

and crutches<br />

(2) Early orthopedic reevaluation for possible casting<br />

d. Third-degree sprains<br />

C. Ankle dislocations<br />

(1) Ice, elevation, splint immobilization, (stir-up splint, air cast, or posterior mold), prolonged nonweightbearing,<br />

and crutches<br />

(2) Referral to orthopedics<br />

(3) Some of these patients, depending on their level of athletic involvement and other associated injuries,<br />

may be considered for primary surgical repair.<br />

1. Described according to the direction of talar displacement relative to the tibia; five types:<br />

a. Anterior<br />

b. Posterior (common)<br />

c. Medial (common)<br />

d. Lateral<br />

e. Superior (diastasis) (uncommon)<br />

2. Pure ankle dislocations are uncommon; most occur in association with malleolar fractures.<br />

3. Treatment<br />

a. All ankle dislocations require immediate neurovascular assessment and emergent orthopedic consult.<br />

386


ORTHOPEDIC EMERGENCIES<br />

b. If there is any evidence of neurovascular compromise or skin tenting, reduction should be accomplished<br />

immediately (before radiographic evaluation). If these findings are absent, radiographs should be obtained<br />

before reduction to exclude the presence of associated fractures.<br />

c. After administration of adequate analgesia or procedural sedation, promptly reduce the dislocation with<br />

gentle in-line traction.<br />

d. Once reduction has been accomplished, reassess the patient's neurovascular status, order radiographs after<br />

reduction, and place the patient in a combination stir-up/posterior splint.<br />

D. Tendon injuries<br />

1. Achilles tendon rupture<br />

a. Common in sedentary, middle-aged men (mean age 35 years) engaging in weekend athletic activities<br />

b. Up to 25 % of cases are initially misdiagnosed as ankle sprains.<br />

c. Mechanism of injury<br />

(1) Forceful dorsiflexion of the foot with the ankle in a relaxed state<br />

(2) Direct trauma to a taut tendon<br />

(3) An extra stretch applied to a taut tendon<br />

d. Clinical presentation<br />

(1) The patient gives a history of sudden excruciating pain at the back of the ankle that lessens instantly,<br />

and often reports having heard or felt a "pop" or "snap."<br />

(2) Physical examination<br />

(a) Swelling of the distal calf<br />

(b) A palpable defect in the tendon 2-6 cm proximal to its calcaneal insertion site<br />

(c) Weak plantar flexion when compared with the uninjured leg<br />

e. Diagnostic evaluation<br />

(1) Diagnosis is made with the Thompson test (also called Simmonds test), which is performed with the<br />

patient in the prone position.<br />

f. Treatment<br />

(a) Normal function: squeezing the calf produces plantar flexion of the foot.<br />

(b) With a complete tear, plantar flexion will not occur.<br />

(c) Bedside dynamic ultrasound is a quick easy way to confirm the diagnosis.<br />

(d) MRI can be used to confirm the diagnosis if doubt remains, but this is usually unnecessary.<br />

(1) Initial management<br />

(a) Ice, elevation, analgesia, immobilization in a posterior splint in passive equinus (plantar flexion),<br />

and crutches<br />

(b) Orthopedic consult within 48-72 hours<br />

(2) Definitive care (casting versus surgical repair) remains controversial; it is usually determined by<br />

the patient's age, activity level, and underlying medical conditions. Surgical repair appears to be<br />

associated with a better outcome (increased strength and mobility, and decreased incidence of rerupture)<br />

but is typically reserved for younger patients.<br />

2. Peron ea I tendon sub I uxation/dislocation<br />

a. Results from a tear of the superior peroneal retinaculum from its attachment on the fibula. This allows the<br />

peroneus brevis and longus tendons to sublux anteriorly over the lateral malleolus.<br />

b. Often misdiagnosed as an ankle sprain<br />

c. Mechanism of injury: forced dorsiflexion of the ankle with reflex contraction of the peroneal muscles<br />

d. Clinical presentation<br />

(1) The patient typically complains of pain and a clicking or slipping sensation at the back of the ankle.<br />

(2) Diagnosis is made by physical examination.<br />

(a) Tenderness, swelling, and ecchymosis at the posterior aspect of the lateral malleolus<br />

(b) Tensing of the peroneal muscles intensifies the pain.<br />

(c) Eversion is weak.<br />

(d) Anterior subluxation of the tendons with the patient's foot dorsiflexed and everted against<br />

resistance confirms the diagnosis.<br />

e. Plain radiographs may reveal a small avulsion fracture of the lateral ridge of the distal fibula<br />

(pathognomonic for this injury) in 50% of cases.<br />

f. Treatment: splint in mid plantar flexion, and refer to orthopedist for possible surgical repair.<br />

387


ORTHOPEDIC EMERGENCIES<br />

E. Ankle injuries in children<br />

1. Because ligaments are stronger than bone in children, fractures at the epiphyseal plate are more common than<br />

ligamentous injuries.<br />

2. These fractures usually result from indirect forces and are classified according to the Salter-Harris classification<br />

system (see pages 350-351).<br />

3. Tillaux fracture<br />

a. A Salter Ill fracture of the lateral distal tibia commonly seen in adolescents participating in sporting events<br />

b. Consider this diagnosis if the history is suggestive and there is swelling over the anterior ankle.<br />

VII. FOOT INJURIES<br />

A. Essential anatomy<br />

1. The foot is divided into three parts:<br />

a. Hindpart (calcaneus and talus)<br />

b. Midpart (navicular, cuboid, cuneiforms)<br />

c. Forepart (metatarsals and phalanges)<br />

2. The hindpart is separated from the midpart by Chopart joint; the mid part is separated from the forepart by<br />

Lisfranc joint.<br />

3. First metatarsal head bears twice the weight of the other metatarsals.<br />

4. During the push-off phase, the second and third metatarsals bear the most weight; hence, they are prone to<br />

stress fractures.<br />

B. Fractures<br />

1 . Calcaneus<br />

a. The calcaneus is the most frequently fractured tarsal bone. (The talus is the second most commonly fractured.)<br />

b. A calcaneal fracture is usually due to a compression injury (eg, fall from a height with the patient landing<br />

on his or her feet).<br />

c. Clinical presentation<br />

(1) Examination reveals swelling, tenderness, and ecchymosis of the hindfoot and inability to bear<br />

weight on the fracture.<br />

(2) 10% are bilateral, and another 10% are associated with compression fractures of the dorsolumbar<br />

spine.<br />

(3) 26% are associated with other injuries to the lower extremities.<br />

(4) Whenever a calcaneal body fracture is diagnosed, the Bohler angle should be measured to determine<br />

whether or not the fracture is depressed. This is done by measuring the intersection of two lines<br />

on the lateral radiograph. One is drawn from the superior margin of the posterior tuberosity of the<br />

calcaneus and extended through the superior tip of the posterior facet. The other line is drawn from<br />

the superior tip of the anterior process and extended through the superior tip of the posterior facet.<br />

This angle normally measures 20°-40°. If the angle is


ORTHOPEDIC EMERGENCIES<br />

d. Treatment<br />

(1) Intra-articular or displaced calcaneal fractures: management remains controversial (nonoperative<br />

versus immediate surgical reduction); obtain emergent orthopedic consult in the emergency<br />

department.<br />

(2) Nondisplaced or minor extra-articular fractures: ice, elevation, immobilization in a posterior splint,<br />

crutches; obtain orthopedic consult and arrange for early follow-up on an outpatient basis.<br />

2. Lisfranc (tarsometatarsal) fracture/dislocation<br />

a. Mechanisms of injury<br />

(1) Axial load: a fall on the plantar-flexed foot<br />

(2) Compressive forces: crush injury<br />

(3) Rotational forces: twisting of the body around a fixed foot<br />

b. Clinical presentation<br />

(1) Examination reveals significant midfoot swelling and pain, decreased range of motion, and inability<br />

to bear weight.<br />

(2) Paresthesias of the midfoot may also be present.<br />

c. Diagnostic evaluation: radiographs<br />

(1) The key to making this diagnosis is to look for the normal alignment along the medial aspect of the<br />

middle cuneiform with the medial aspect of the base of the second metatarsal; any disruption of this<br />

alignment is indicative of a dislocation.<br />

(2) The second metatarsal functions as the primary stabilizing force of the joint; thus, a fracture through<br />

the base of the second metatarsal (Fleck sign) is indicative of a disrupted Lisfranc joint.<br />

(3) Separation between the base of the first and second metatarsals is highly suggestive of subluxation<br />

and is the most reliable finding.<br />

(4) Weight-bearing radiographs of the Lisfranc joint can help make the diagnosis if the initial plain<br />

radiographs are normal.<br />

d. Treatment<br />

(1) Involves either closed reduction under general anesthesia, or open reduction and surgical fixation<br />

(2) Early orthopedic consult in the emergency department is indicated.<br />

Lisfranc Injury<br />

Second metatarsal<br />

First metatarsal<br />

Lisfranc joint<br />

Second metatarsal<br />

Lisfranc joint<br />

389


ORTHOPEDIC EMERGENCIES<br />

3. Jones fracture (diaphyseal fracture of the fifth metatarsal)<br />

a. A transverse fracture of the proximal diaphysis of the fifth metatarsal<br />

b. In children, the proximal physis of the fifth metatarsal may be mistaken for a fracture.<br />

c. Mechanism of injury: a forceful load applied to the ball of the foot laterally (such as a pivot)<br />

d. Unlike avulsion fractures of the base of the fifth metatarsal (pseudo-Jones or dancer's fracture), these<br />

fractures are slow to heal and are associated with a high incidence of delayed union or nonunion.<br />

e. Treatment<br />

(1) Nondisplaced fractures are usually managed with immobilization in a nonweight-bearing<br />

short-leg cast.<br />

(2) Displaced fractures (and fractures in athletes) are typically managed surgically.<br />

(3) Orthopedic follow-up is required.<br />

390


ORTHOPEDIC EMERGENCIES: PRACTICE CLINICAL SCENARIOS<br />

ORTHOPEDIC EMERGENCIES: PRACTICE CLINICAL SCENARIOS<br />

Answers immediately follow the practice clinical scenarios.<br />

Scenario A<br />

Presentation: An elderly man with a history of prosthetic hip replacement presents complaining of inability<br />

to walk or move the hip. The pain started with minimal trauma.<br />

Physical examination: On physical examination, the leg is shortened, abducted, and internally rotated.<br />

What is the diagnosis?<br />

Scenario B<br />

Presentation: A middle-aged "weekend warrior" felt a snap or pop in his knee after a jump in a weekend<br />

game of basketball.<br />

Physical examination: On physical examination, the patient can stand but is unable to walk or extend the<br />

knee. The patella is high riding.<br />

What is the diagnosis?<br />

Scenario C<br />

Presentation: The patient presents after a fall onto the foot or a twisting injury. Pain on the medial side of<br />

the ankle increases with weight-bearing.<br />

Physical examination: A careful examination of the proximal fibula elicits pain with compression of the<br />

fibula or palpation of the fibula head.<br />

What is the diagnosis?<br />

Scenario D<br />

Presentation: A young football player is brought in from the field complaining of knee pain after taking a<br />

hit during the game. Bystanders and the patient report that the knee appeared deformed.<br />

Physical examination: An effusion is noted in the knee, along with multidirectional laxity (laxity with<br />

valgus, varus, anterior and posterior stress of the ligaments).<br />

What is the diagnosis?<br />

Scenario E<br />

Presentation: Parents bring in a 3-year-old child, reporting arm pain. The child refuses to use the<br />

affected arm.<br />

Physical examination: The elbow is tender over the radial head, but there is no bruising, swelling,<br />

or deformity.<br />

What is the diagnosis?<br />

391


ORTHOPEDIC EMERGENCIES: PRACTICE CLINICAL SCENARIOS<br />

ANSWERS TO PRACTICE CLINICAL SCENARIOS<br />

Scenario A<br />

Diagnosis: posterior prosthetic hip dislocation<br />

Diagnostic evaluation: Pelvis and hip radiographs show the hip posteriorly dislocated. Care should be taken to<br />

ensure that there is no acetabular or pelvic fracture.<br />

Scenario B<br />

Diagnosis: quadriceps rupture<br />

Diagnostic evaluation: Radiographs of the knee may show a high-riding patella (can also be normal). A joint<br />

effusion can often be seen.<br />

Scenario C<br />

Diagnosis: Maisonneuve fracture<br />

Diagnostic evaluation: Ankle radiographs show a fracture of the medial malleolus, possibly with increased space<br />

between the malleolus and talar dome. Tibia/fibula radiographs demonstrate a proximal fibula fracture.<br />

Scenario D<br />

Diagnosis: knee dislocation<br />

Diagnostic evaluation: It is not unusual for the leg to look completely normal except for an effusion. Evaluation<br />

should consist of serial examinations of the leg with measurement of an ankle-brachia! index (ABI). Normally, AB ls<br />

do not definitively exclude arterial injury, because an intimal tear can initially be normal until a thrombosis forms<br />

and occludes the artery. Knee radiographs (AP and lateral) should be obtained to exclude fractures. These patients<br />

are at risk of compartment syndrome and injury to the popliteal artery. Consider obtaining Doppler ultrasound or a<br />

CT arteriogram (or angiogram) of the popliteal artery.<br />

Scenario E<br />

Diagnosis: nursemaid elbow (radial head subluxation)<br />

Diagnostic evaluation: There may be a history of the arm being pulled or yanked. Radiographs are generally<br />

not required; however, PA and lateral views of the elbow should be obtained if there is a concern for fracture or<br />

dislocation or if there is swelling, bruising, or crepitus on examination.<br />

Management: Treatment consists of reduction of the dislocated radial head by hyperpronating the forearm and, if<br />

necessary, flexing the elbow while applying some gentle pressure to the radial head. If pronation is not effective,<br />

reduction may be attempted by supinating the forearm and flexing the elbow.<br />

392


MUSCULOSKELETAL DISORDERS (NONTRAUMATIC)<br />

MUSCULOSKELETAL DISORDERS (NONTRAUMATIC)<br />

Arthritis ..................................................................................................................................................................... 396<br />

Classification ...................................................................................................................................................... 396<br />

Conditions Associated with Symmetric Polyarticular Joint Pain ........................................................................... 396<br />

Conditions Associated with Asymmetric Polyarticular Joint Pain ......................................................................... 396<br />

Conditions Associated with Monoarticular Joint Pain .......................................................................................... 397<br />

Conditions Associated with Migratory Arthritis ................................................................................................... 397<br />

Synovial Fluid Analysis ....................................................................................................................................... 398<br />

Tendinitis and Tenosynovitis ...................................................................................................................................... 398<br />

General Facts ..................................................................................................................................................... 398<br />

Stenosi ng Tenosynovitis or "Trigger Finger" ......................................................................................................... 3 98<br />

De Quervain Stenosing Tenosynovitis ................................................................................................................. 398<br />

Carpal Tunnel Syndrome .................................................................................................................................... 398<br />

Dupuytren Contracture ............................................................................................................................................. 399<br />

Ganglion Cysts .......................................................................................................................................................... 399<br />

Charcot Joint ............................................................................................................................................................. 399<br />

Back Pain ................................................................................................................................................................... 399<br />

Differential Diagnosis ......................................................................................................................................... 399<br />

General Facts ..................................................................................................................................................... 399<br />

Lumbar Disc Syndromes ..................................................................................................................................... 400<br />

Spondylol isthesis ................................................................................................................................................ 400<br />

Spinal Infections ................................................................................................................................................. 401<br />

Spinal Malignancy .............................................................................................................................................. 401<br />

Hand and Wrist Infections ........................................................................................................................................ 401<br />

Paronychia ......................................................................................................................................................... 401<br />

Felon .................................................................................................................................................................. 401<br />

Herpetic Whitlow ............................................................................................................................................... 402<br />

Human Bite Infections Involving the Metacarpophalangeal Joint ("Fight Bites") .................................................. 402<br />

Pyogenic Flexor Tenosynovitis ............................................................................................................................ 402<br />

Deep Pal mar Space Infection .............................................................................................................................. 403<br />

High-Pressure Injection Injuries .......................................................................................................................... 403<br />

Compartment Syndrome (Volkmann lschemia) of the Upper Extremity .................................................................... 403<br />

Rhabdomyolysis ........................................................................................................................................................ 404<br />

393


MUSCULOSKELETAL DISORDERS (NONTRAUMATIC): SELF-ASSESSMENT QUESTIONS<br />

MUSCULOSKELETAL DISORDERS (NONTRAUMATIC):<br />

SELF-ASSESSMENT QUESTIONS<br />

1. Empiric antibiotic coverage from human bite cellulitis associated with involvement of the metacarpophalangeal<br />

joint (clenched fist injuries) should be selected to cover all the following pathogens except:<br />

(a) Anaerobic streptococci<br />

(b) Staphylococcus aureus<br />

(c)<br />

Eikenella corrodens<br />

(d) Pasteurella multocida<br />

2. Synovial fluid analysis with a WBC count of 3,500/mm 3 and PMNs of 60% is most likely related to which disease<br />

process?<br />

(a) Septic arthritis<br />

(b) Osteoarthritis<br />

(c) Pseudogout<br />

(d) Traumatic arthritis<br />

3. "Red flags" in a patient presenting with back pain that require further evaluation in the emergency department<br />

include all the following except:<br />

(a) History of breast cancer<br />

(b) Fever<br />

(c) Acute onset in a patient aged 30<br />

(d) Bladder retention<br />

4. A patient presents with finger pain and a red, swollen index finger with vesicles and itching for 3 days. This<br />

presentation is consistent with a diagnosis of:<br />

(a)<br />

Herpetic whitlow<br />

(b) Felon<br />

(c) Paronychia<br />

(d) Spider bite<br />

5. All of the following are Kanavel cardinal signs of flexor tenosynovitis except:<br />

(a)<br />

Pain with passive extension of the finger<br />

(b) Lack of capillary refill<br />

(c) Finger held in slight flexion<br />

(d) Fusiform swelling of the digit ("sausage digit")<br />

6. Al I of the fol lowing can lead to Volkmann ischemic contracture of the forearm except:<br />

(a)<br />

Arterial injection of drugs<br />

(b) Burns<br />

(c)<br />

Boxer's fracture<br />

(d) Extravasation of blood into the forearm<br />

7. Rhabdomyolysis may occur when ___ _<br />

(a) ATP stores become depleted.<br />

(b) There is damage to the sarcolemma of the cell.<br />

(c) The calcium channels, pumps, and exchangers fail to function.<br />

(d) All of the above<br />

(e)<br />

None of the above<br />

8. Treatment for rhabdomyolysis should be directed at:<br />

(a) Aggressive fluid repletion<br />

(b) Urine alkalinzation<br />

(c) Correcting electrolyte derangements<br />

(d) All of the above<br />

394


MUSCULOSKELETAL DISORDERS (NONTRAUMATIC): SELF-ASSESSMENT QUESTIONS<br />

9. The most common pathogen responsible for epidural abscess is ___ _<br />

(a) Pseudomonas spp<br />

(b) Escherichia coli<br />

(c) Staphylococcus aureus<br />

(d) Mycobacterium tuberculosis<br />

10. A patient presents with acute onset of back pain and complains of pain extending down the leg from the thigh to the<br />

medial aspect of the foot. A decreased knee jerk is present on examination. This correlates to nerve root involvement<br />

at which level of the spine?<br />

(a)<br />

L2<br />

(b) L3<br />

(c) L4<br />

(d) LS<br />

11. Which of the following statements about rheumatoid arthritis is true?<br />

(a) It causes symmetric monoarticular joint pain.<br />

(b) It affects al I joints of the hands, wrist and elbow in an additive manner.<br />

(c) It is a migratory polyarthritis.<br />

(d) It has a genetic predisposition to the HLA-DR4 haplotype.<br />

ANSWERS<br />

1. d<br />

4.<br />

a<br />

7.<br />

d<br />

10. C<br />

2. C<br />

5.<br />

b<br />

8.<br />

d<br />

11. d<br />

3. C 6. C 9. C<br />

Use the pre-chapter multiple choice question worksheet (page xvi) to record and determine the percentage of correct<br />

answers for th is chapter.<br />

395


MUSCULOSKELETAL DISORDERS (NONTRAUMATIC)<br />

I. ARTHRITIS<br />

A. Classification<br />

1. Arthritis can be produced by many conditions and may be classified in several ways.<br />

2. Classification of arthritides by pattern of joint involvement (polyarticular, monarticular, migratory) is useful, but<br />

there is some overlap.<br />

B. Conditions associated with symmetric polyarticular joint pain<br />

1. Rheumatoid arthritis<br />

a. Most commonly affects women; there is a genetic predisposition related to HLA-DR 4 haplotype.<br />

b. Most commonly affects the hand (metacarpophalangeal and proximal interphalangeal joints, spares distal<br />

interphalangeal joints), wrist, and elbow in an additive manner<br />

c. Extra-articular complications may occur, including tenosynovitis, pericarditis, subcutaneous nodules,<br />

vasculitis of the skin, pulmonary fibrosis, mononeuritis multiplex, Sjogren syndrome, and Felty syndrome.<br />

Long-standing rheumatoid arthritis may lead to degeneration of the transverse ligament of the C1-C2<br />

junction, which requires precautions during endotracheal intubation.<br />

d. Treatment is with salicylates/NSAIDs, brief courses of corticosteroids or long-term with antimalarial<br />

agents, gold, and methotrexate.<br />

2. Systemic lupus erythematosus<br />

a. Women 15-40 years of age are most commonly affected.<br />

b. May be genetic (associated with HLA genotypes), environmental (exposure to UV light), or drug-induced<br />

c. Arthralgias and arthritis are the most common presenting complaints; both small and large joints are<br />

affected.<br />

d. An erythematous malar ("butterfly") rash is a typical feature and often accompanies joint involvement.<br />

3. Rheumatic fever<br />

a. A migratory polyarthritis<br />

b. Primarily affects the large joints of the extremities (knees, ankles, elbows, and wrists)<br />

c. Triggered by hyperimmune response after a group A streptococcal (GAS) infection<br />

d. Diagnosis requires evidence of prior GAS infection plus Jones criteria (two major or one major and two<br />

minor criteria).<br />

e. Treatment includes IM or PO penicillin to eradicate GAS and high-dose aspirin or IM or PO hydrocortisone<br />

for 2-4 weeks.<br />

4. Hepatitis B (migratory)<br />

5. Rubella<br />

a. Occurs with both natural and vaccine-induced rubella<br />

b. Most commonly affects young women<br />

c. Finger, wrist, and knee involvement is typical.<br />

C. Conditions associated with asymmetric polyarticular joint pain<br />

1. Reactive arthritis (previously called Reiter syndrome)<br />

a. Usually affects young males 15-35 years old; the HLA-B27 antigen is found in up to 75% of these patients.<br />

b. Clinical presentation<br />

(1) Classic triad: urethritis, conjunctivitis, and arthritis<br />

(2) This syndrome has also been documented to occur after dysentery and cervicitis.<br />

(3) Oligoarthritis usually develops 1-6 weeks after an episode of urethritis (due to Chlamydia) or dysentery<br />

(due to Salmonella, Shigella, Yersinia, or Campylobacter).<br />

(4) The weight-bearing joints of the lower extremities are most commonly affected (usually four or five).<br />

c. Must be differentiated from gonococcal arthritis<br />

2. Gonococcal arthritis<br />

a. A migratory arthritis that primarily affects young adults, particularly women<br />

b. Clinical presentation<br />

(1) The wrists, fingers, knees, and ankles are most affected.<br />

(2) An associated rash (erythematous macules or pustules with a necrotic or purpuric center), which is<br />

distributed primarily on the extremities, accompanies the arthritis in two-thirds of patients (arthritis/<br />

dermatitis syndrome).<br />

396


MUSCULOSKELETAL DISORDERS (NONTRAUMATIC)<br />

c. Cervical, urethral, rectal, and pharyngeal cultures should be obtained.<br />

d. Initial treatment is with parental third-generation cephalosporins.<br />

3. Henoch-Schonlein purpura<br />

a. Most commonly occurs in children 4-11 years old<br />

b. Clinical presentation<br />

4. Lyme disease<br />

(1) Classic triad: migratory arthritis, palpable purpuric rash on the lower extremities/buttocks, and colicky<br />

abdominal pain<br />

(2) Ankles and knees are most commonly affected.<br />

a. An oligoarthritis that typically affects the large joints (particularly the knees); it can be migratory and<br />

usually occurs in those patients not treated with appropriate antibiotics.<br />

b. Arthritic symptoms develop in Stage Ill of the disease (usually months after the initial infection).<br />

D. Conditions associated with monoarticular joint pain<br />

1. Septic arthritis (a medical emergency)<br />

a. Staphylococcus aureus is the most common cause. There is increasing incidence of methicillin-resistant<br />

S aureus (MRSA).<br />

b. In most adults, the knee is the most commonly affected joint. In IV drug abusers, however, the most<br />

common sites are the sacroiliac, sternoclavicular, and intervertebral joints. In children, the knee and hip<br />

are most commonly affected.<br />

c. High-risk populations: IV drug users, immunocompromised patients, diabetics, alcoholics, patients with<br />

chronic arthritis or skin infections, or after intra-articular injections or prosthetic implants.<br />

d. Treatment aimed at likely causative agents and adjusted according to Cram stain and culture results.<br />

Initial treatment in adults is a first-generation cephalosporin plus vancomycin for suspected gram-positive<br />

organisms and a third-generation cephalosporin for gram-negative bacilli.<br />

2. Gout<br />

a. A disease of middle-aged men and elderly adults<br />

b. Caused by deposition of uric acid crystals<br />

c. Usually affects the lower extremities, particularly the great toe ("podagra")<br />

d. Evaluation of synovial fluid reveals needle-shaped crystals that exhibit negative birefringence under<br />

polarized light.<br />

e. Acute treatment initially includes NSAIDs and colchicine orally, or prednisone for resistant cases.<br />

Corticotropin IM in a single dose may also be used, particularly in elderly and hospitalized patients in<br />

whom NSAIDs or colchicine may be problematic.<br />

3. Pseudogout<br />

a. Primarily affects men and women :::60 years old<br />

b. Caused by deposit of calcium pyrophosphate crystals<br />

c. Most commonly affected joint is the knee, followed by the wrist, ankle, and elbow.<br />

d. Treatment mirrors that of gout.<br />

e. Diagnostic evaluation<br />

(1) Evaluation of synovial fluid reveals rhomboid-shaped crystals that exhibit weakly positive<br />

birefringence under polarized light (appears yellow when light is perpendicular to the crystal).<br />

(2) Radiographs of the affected joint may reveal calcification of the cartilage (chondrocalcinosis).<br />

4. Osteoarthritis (degenerative joint disease)<br />

5. Trauma<br />

6. Aseptic arthritis<br />

7. Hemarthrosis<br />

8. Patients with monarticular joint pain should always be considered to have an infectious arthritis until proved<br />

otherwise.<br />

E. Conditions associated with migratory arthritis<br />

1. Rheumatic fever<br />

2. Bacterial endocarditis<br />

3. Hepatitis B<br />

4. Septicemia (gonococcal, meningococcal, streptococci, staphylococci)<br />

5. Henoch-Schonlein pupura<br />

397


MUSCULOSKELETAL DISORDERS (NONTRAUMATIC)<br />

6. Lyme disease<br />

7. Serum sickness<br />

8. Pulmonary infections (mycoplasmosis, histoplasmosis, coccidiomycosis)<br />

F. Synovial fluid analysis<br />

1. The most important diagnostic study in evaluation of acute arthritis<br />

2. Mnemonic for laboratory analysis: "CAPS"<br />

~ulture, cell count, crystals (If there is only enough fluid for one test, it should be culture and sensitivity.)<br />

Appearance<br />

frotein<br />

~ugar, stain (Gram stain)<br />

3. Crystals<br />

a. Gout: negatively birefringent (urate) and needle-shaped<br />

b. Pseudogout: positively birefringent (calcium pyrophosphate) and rhomboid-shaped<br />

4. WBC count<br />

a. 75% PMNs is septic.<br />

5. Glucose is significantly decreased (relative to serum glucose) in both septic and rheumatoid arthritis.<br />

6. Gram stain is usually positive in septic arthritis, except for gonococcal infection, in which it is positive in only<br />

25 % of cases.<br />

II. TENDINITIS AND TENOSYNOVITIS<br />

A. General facts<br />

1. Inflammation of flexor or extensor tendons of the hand from repetitive motion<br />

2. Indicated by tenderness on palpation or pain with active or passive range of motion<br />

3. Management is splinting in position of function, elevation, and NSA!Ds.<br />

B. Stenosing tenosynovitis or "trigger finger"<br />

1. Scarring or inflammation for prolonged periods of time causes nodules to form on the flexor tendon sheaths.<br />

2. Causes friction and "catching" of the finger at the A 1 pulley at the volar crease at the base of each digit<br />

3. Finger "snaps" into extension but may be locked inflexion.<br />

4. Treatment by depot steroid injection or surgical division of the A 1 pulley<br />

C. De Quervain stenosing tenosynovitis<br />

1. Due to excessive use of the thumb, the extensor pollicis brevis and abductor pollicis tendons become inflamed.<br />

2. Causes pain along radial aspect of the wrist that extends to the forearm<br />

3. Diagnosis is confirmed by a positive Finkelstein test (thumb is grasped in a closed fist and the wrist is ulnarly<br />

deviated, producing a sharp pain along the tendon).<br />

4. Management is immobilization of the thumb and wrist, range of motion exercises, and NSAIDs.<br />

D. Carpal tunnel syndrome<br />

1. Peripheral neuropathy involving the median nerve in the carpal tunnel<br />

2. Swelling of tendons in the tunnel causes compression of the median nerve, which triggers paresthesias in the<br />

pal mar aspect of the thumb, the index and long fingers, and the radial aspect of the ring finger.<br />

3. Symptoms include night pain, burning, and tingling in the above-mentioned distribution, and numbness when<br />

the wrist is in prolonged flexion.<br />

4. Typically is triggered by overuse syndromes but may occur with direct trauma, diabetes, rheumatoid arthritis,<br />

and edematous states such as pregnancy or heart failure.<br />

5. Tinel sign (paresthesias triggered by tapping over the flexor retinaculum) supports the diagnosis, but a positive<br />

Phalen sign (flexing the wrist maximally and holding it in this position for 1 minute) is more sensitive.<br />

6. Initial treatment is splinting in a volar splint with the wrist in neutral position and NSAIDs.<br />

7. Surgery may be necessary; emergent hand specialty consult is required for median nerve motor deficits.<br />

398


~<br />

MUSCULOSKELETAL DISORDERS (NONTRAUMATIC)<br />

Ill. DUPUYTREN CONTRACTURE<br />

A. Fibroblastic changes of the subcutaneous tissues of the palm and volar aspects of the fingers<br />

B. Appears to have a genetic component<br />

C. May lead to joint contractures with thickened, nodular pal mar tissues<br />

IV. GANGLION CYSTS<br />

A. Cystic collection of synovial fluid within a joint or tendon sheath<br />

B. Presents as a tender, cystic swelling over or near a tendon sheath<br />

C. Management: pain control and NSAIDs<br />

D. One-third of ganglion cysts resolve spontaneously, but persistent cysts may be referred to a hand specialist<br />

for cyst aspiration, corticosteroid injection, or surgical excision.<br />

V. CHARCOT JOINT<br />

A. Most common presentation is a painful, swollen ankle<br />

B. Usual cause is diabetic peripheral neuropathy (not syphilis)<br />

C. Radiograph of the ankle reveals classic "bag of bones" appearance.<br />

VI. BACK PAIN<br />

A. Differential diagnosis<br />

1. Aortic aneurysm<br />

2. Peptic ulcer<br />

3. Pyelonephritis<br />

4. Pancreatitis<br />

5. Vertebral disc compression or fracture<br />

6. Osteoporosis<br />

7. Carcinoma (primary or metastatic)<br />

8. Ankylosing spondylitis (associated with inflammatory bowel disease)<br />

9. Infection<br />

10. Osteomyelitis<br />

11. Ectopic pregnancy<br />

12. Strain or sprain<br />

13. Pelvic inflammatory disease<br />

B. General facts<br />

1. Back pain is the most frequent (and the most costly) cause of workman's compensation claims/payments.<br />

2.<br />

3.<br />

4.<br />

5.<br />

6.<br />

Most back pain (including pain secondary to disc disease) is self-limited and resolves in :S6-7 weeks, regardless<br />

of treatment.<br />

Most back pain is benign (particularly in healthy, young- and middle-aged adults).<br />

A serious cause of back pain is more likely in patients 55<br />

years old (incidence of 20%).<br />

80% or more of low back pain occurs in the L4-S1 region; L5-S1 is the most frequent location.<br />

When radiographs are indicated (which they are not in most cases), AP and lateral views alone are usually<br />

sufficient.<br />

a. Most healthy patients 18-55 years old with atraumatic, nonspecific, mechanical low back pain can be<br />

managed initially without radiographic evaluation.<br />

399


MUSCULOSKELETAL DISORDERS (NONTRAUMATIC)<br />

b. Lumbosacral films should be obtained as part of the initial evaluation in the following circumstances:<br />

(1) Age 55 years (higher incidence of serious causes)<br />

(2) Trauma<br />

(3) Persistent, unremitting back pain >4-6 weeks duration<br />

(4) Prior referral<br />

(5) Suspicion of serious underlying pathology based on the patient's:<br />

(a)<br />

Past medical history (cancer, IV drug abuse)<br />

(b) Vital signs (fever)<br />

C. Lumbar disc syndromes<br />

(c) Atypical symptoms (night pain)<br />

1. Impingement of a herniated disc on the anterior nerve root produces pain in a dermatomal pattern.<br />

L 1: groin<br />

L2: upper thigh<br />

L3: mid thigh and medial knee<br />

L4: lower thigh, anterior knee, and medial foot<br />

LS: lateral lower leg, first interdigital web space, and dorsal foot<br />

Sl: posterolateral calf and lateral foot<br />

2. Clinical presentation: physical findings of nerve root involvement<br />

a. t Knee jerk= L3 and L4<br />

b. t Dorsiflexion of foot = LS<br />

c. t Ankle jerk (t plantar flexion) and numbness of lateral foot= Sl<br />

3. Cauda equina syndrome (neurosurgical emergency)<br />

a. A serious complication of lumbar disc disease<br />

b. Results from a massive midline lumbar disc herniation that compresses several nerve roots of the cauda<br />

equina<br />

c. Clinical presentation<br />

(1) Patients complain of lower back/leg pain and incontinence (or retention) of bowel and bladder. With<br />

progression, the patient also develops numbness of the feet and trouble walking.<br />

(2) Examination reveals loss of sensation in the "saddle" distribution and loss of anal sphincter tone<br />

(S3, S4, S5).<br />

d. Diagnostic evaluation: an MRI or emergency myelogram may be used to confirm the diagnosis and locate<br />

the disc level but should not delay treatment.<br />

e. Management: immediate neurosurgical consult for operative disc decompression; decompression must be<br />

performed quickly (within several hours) if permanent disability is to be avoided.<br />

D. Spondylolisthesis<br />

1. Definition: displacement of a vertebral body on the one below (most commonly L4 on LS); causes include<br />

degenerative changes and trauma.<br />

2. Clinical presentation<br />

a. Asymptomatic (only 25% of patients develop back or leg pain)<br />

b. Radiographic findings may be incidental and not necessarily the cause of the back/leg pain.<br />

c. Retrolisthesis, however, is almost always associated with back pain.<br />

3. Diagnostic evaluation<br />

a. Best seen on the lateral view of the I umbar spine series<br />

b. The amount of forward displacement is graded; the higher the grade, the worse the slippage.<br />

(1) Grade I: up to 25%<br />

(2) Grade II: 25%-50%<br />

(3) Grade Ill: 50%-75%<br />

4. Management<br />

(4) Grade IV: 75%-100%<br />

a. Outpatient protocol for patients with low back pain without neurologic findings ("lumbar strain")<br />

(1) Strict bed rest is no longer recommended for treatment of lumbar strain. Patients should be encouraged<br />

to do light activities, range of motion exercises, and stretches but should not be expected to do any<br />

prolonged walking, standing, twisting, or lifting of> 15 pounds.<br />

400


MUSCULOSKELETAL DISORDERS (NONTRAUMATIC)<br />

(2) Analgesics (including NSAIDs)<br />

(3) Muscle relaxants (offer some symptomatic benefit)<br />

(4) Follow-up in 7 days<br />

(5) Instructions to return immediately if problems develop (worsening pain, loss of motor strength, bowel<br />

or bladder incontinence)<br />

b. Inpatient management is indicated for patients with:<br />

E. Spinal infections<br />

(1) Cauda equina syndrome<br />

(2) Severe or progressive neurologic deficit<br />

(3) Multiple nerve root involvement<br />

(4) Unmanageable pain<br />

(5) Inadequate support system at home<br />

1. High-risk patients: IV drug users, immunocompromised patients, alcoholics, elderly, patients with prior blunt<br />

trauma or recent bacterial infection<br />

2. Epidural abscess<br />

a. True neurosurgical emergency<br />

b. 20% of patients may have no comorbid illness.<br />

c. Most common bacterial cause is S aureus.<br />

d. Classic presentation is fever with neurologic symptoms, although 50% of patients may have neither finding.<br />

e. Risk of misdiagnosis is high on initial presentation.<br />

f. Mortality is as high as 23%.<br />

g. Treatment should include vancomycin in addition to broad-spectrum IV antibiotics. Consider<br />

Pseudomonas coverage for IV drug users.<br />

3. Spondylitis (osteomyelitis of the vertebral bone)<br />

a. Typically starts as subtle hematogenous seeding of the disc space, causing discitis. Contiguous spread then<br />

leads to vertebral endplate erosion.<br />

b. Most common pathogen is 5 aureus.<br />

c. May see enteric gram-negative bacilli or Mycobacterium tuberculosis (Pott disease)<br />

d. Typically presents with more indolent course and subjective fevers.<br />

F. Spinal malignancy<br />

1. Typically >50 years old<br />

2. Presents with subacute or chronic back pain<br />

3. Pain is worse at night.<br />

4. Risk factors include a history of cancer, unexplained weight loss, persistent pain, and pain lasting> 1 month.<br />

5. Cancers that most commonly metastasize to bone include breast, lung, and prostate.<br />

VII. HAND AND WRIST INFECTIONS<br />

A. Paronychia<br />

1. Infection of the lateral nail fold that sometimes extends to involve the proximal nail fold (eponychium)<br />

2. Causative agents are usually 5 aureus or Streptococcus spp.<br />

3. Patients presenting early with cellulitis may respond to oral antibiotics (a penicillinase-resistant oral penicillin<br />

or a first-generation cephalosporin) and warm soaks alone.<br />

4. If purulent material is present, the nail fold should be incised and drained, followed by daily warm soaks; use<br />

antibiotics only if a surrounding cellulitis is present.<br />

5. If subungual pus is present, the adjacent portion of the nail should be removed.<br />

6. Chronic paronychia may develop in immunocompromised patients. Consider treatment directed at atypical<br />

bacteria or fungal infections (Candida a/bicans).<br />

B. Felon<br />

1. Infection of the pulp space of the fingertip<br />

2. Causative agent is usually S aureus.<br />

401


MUSCULOSKELETAL DISORDERS (NONTRAUMATIC)<br />

3. Management: incision and drainage at the point of maximum tenderness and fluctuance<br />

a. Make a central longitudinal incision starting 0.5 cm distal to the distal flexor crease (to avoid the flexor<br />

tendon sheath) or a unilateral longitudinal incision, and extend this incision into the pulp space. This<br />

approach is preferred because of fewer long-term complications.<br />

b. Pack the wound, splint the finger, and start the patient on an oral first-generation cephalosporin or<br />

antistaphylococcal antibiotic.<br />

c. Remove the packing in 48-72 hours, and start warm soaks.<br />

4. Complications<br />

a. Flexor tenosynovitis and osteomyelitis<br />

b. Skin instability, loss of sensation, and a disrupted pad also may occur.<br />

C. Herpetic whitlow<br />

1. Viral infection of the distal finger<br />

2. Caused by herpes simplex virus (type I typically in children, or type II typically in adults)<br />

3. Typically occurs in healthcare providers with exposure to oral secretions and in patients with coexistent<br />

herpes infections.<br />

4. Clinical presentation<br />

a. Localized burning, itching, and pain precede the development of the classic clear herpetic vesicles.<br />

b. Typically, only one finger is involved.<br />

5. Diagnostic evaluation: Diagnosis can usually be made clinically, but if doubt remains or the presentation is<br />

atypical, it can be confirmed with Tzanck smear (reveals multinucleated giant cells) or viral culture.<br />

6. Management<br />

a. Splinting, elevation, and analgesics for pain relief<br />

b. An oral antiviral agent effective against herpes simplex (eg, acyclovir, famciclovir) if patient is seen early<br />

in the course of the infection (or is immunocompromised)<br />

c. A dry dressing and instructions to prevent autoinoculation or transmission to others<br />

d. Surgical drainage is contraindicated; it can result in secondary infection and delayed healing.<br />

D. Human bite infections involving the metacarpophalangeal joint ("fight bites")<br />

1. Can result in severe complications, including deep palmar space infections, functional loss, and amputation;<br />

these wounds must be aggressively treated and associated tendon injuries must be excluded.<br />

2. Commonly result from punching someone in the mouth (initially denied by many patients)<br />

3. A common mistake is to suture a human bite laceration over the metacarpophalangeal joint; these lacerations<br />

should never be closed primarily but allowed to heal by secondary intention.<br />

4. Frequently found pathogens include anaerobes (especially Eikenella corrodens and anaerobic Streptococcus),<br />

S aureus, and Neisseria spp. Prophylaxis with amoxicillin-clavulanate may prevent this complication.<br />

5. Management<br />

a. Consult with a hand surgeon.<br />

b. Obtain radiographs to exclude fractures and retained foreign bodies.<br />

c. Obtain aerobic and anaerobic wound cultures.<br />

d. Irrigate the wound with normal saline.<br />

e. Splint and elevate the hand.<br />

f. Admit the patient for IV antibiotics (ampicillin/sulbactam).<br />

E. Pyogenic flexor tenosynovitis<br />

1. Typically results from a puncture wound<br />

2. Causative agents are usually S aureus or Streptococcus<br />

3. Diagnosis is based on the presence of Kanavel four cardinal signs of flexor tenosynovitis<br />

a. Finger held in slight flexion<br />

b. Symmetric swelling of the finger<br />

c. Tenderness along the flexor tendon sheath<br />

d. Pain with passive extension of the finger<br />

4. Management<br />

a. Hospitalization and emergent hand consult for surgical drainage<br />

b. IV antibiotics<br />

402


MUSCULOSKELETAL DISORDERS (NONTRAUMATIC)<br />

(1) If the tenosynovitis was caused by penetrating trauma - penicillinase-resistant antistaphylococcal<br />

penicillin or a first-generation cephalosporin<br />

(2) If there is no history (or evidence) of trauma in a sexually active adult, consider disseminated<br />

gonococcal infection and treat empirically with ceftriaxone until culture results are available.<br />

c. Elevation and splinting<br />

d. Tetanus prophylaxis as needed<br />

F. Deep pal mar space infection<br />

1. Characterized by swelling and tenderness localized to the pal mar space<br />

2. Management is incision and drainage in the operating room and IV antibiotics.<br />

G. High-pressure injection injuries (true surgical emergencies)<br />

1. Mechanisms<br />

a. A 1-3 mm wound caused by a grease or paint gun is the hallmark of a high-pressure injection injury.<br />

b. Fusiform swelling and mild pain on initial presentation evolves to a swollen, pale, and severely painful<br />

digit hours later.<br />

c. The fluid frequently travels down the tendon sheath and damages the flexor tendon, with tenderness<br />

typically present along the course of injection.<br />

2. Prognosis<br />

a. Poor, despite the deceptively normal appearance of the hand on initial presentation; amputation rates can<br />

range from 60% to 80%, but recent data demonstrate a 30% rate because of higher vigilance and early<br />

recognition of tissue injury.<br />

b. Factors affecting prognosis<br />

(1) Location of entry wound and the underlying anatomy affected<br />

(2) Physical and chemical qualities of the substance injected (viscosity, corrosiveness); substances that<br />

are low in viscosity and corrosive in nature (eg, paint solvents) produce the most damage.<br />

(3) Velocity of injection: the higher the velocity, the greater the penetration<br />

(4) Duration of exposure to the injected substance: the greater the duration of exposure, the worse the<br />

prognosis.<br />

3. Management<br />

a. Obtain a radiograph to determine the degree of spread of material injected; it may be radiopaque.<br />

b. Splint and elevate the extremity.<br />

c. Administer a parenteral broad-spectrum antibiotic.<br />

d. Update tetanus prophylaxis as indicated.<br />

e. Provide oral or parenteral analgesia.<br />

f. Digital blocks are contraindicated - increased tissue pressure ----;, vascular compromise<br />

g. Obtain immediate hand specialty consult for operative debridement.<br />

VIII. COMPARTMENT SYNDROME (VOLKMANN ISCHEMIA)<br />

OF THE UPPER EXTREMITY<br />

A. Usually involves the flexor (volar) compartment of the forearm<br />

B. Arises from increased pressure in the muscle compartment of the limb (forearm) that compromises the<br />

circulation to the muscles and nerves of that compartment<br />

C. Can be precipitated by an increase in compartment contents, a decrease in compartment size, or externally<br />

applied pressure; causes include:<br />

1. Supracondylar fracture of the elbow<br />

2. Fracture of the radius and ulna<br />

3. Forearm crush injury<br />

4. Extravasation of blood into the forearm<br />

5. Arterial injection of drugs<br />

6. Constrictive dressing or cast<br />

7. Burns<br />

403


MUSCULOSKELETAL DISORDERS (NONTRAUMATIC)<br />

D. Clinical presentation<br />

1. Earliest finding is pain that is:<br />

a. Out of proportion to the injury<br />

b. Increased with passive stretching of the involved muscles<br />

c. Increased with active contraction of the involved muscles<br />

2. Paresthesia or hypesthesia of the nerves traversing the compartment (early finding)<br />

3. Paralysis or paresis of the involved muscles<br />

4. Palpable tenseness and tenderness of the compartment<br />

5. Pallor, cyanosis, or mottling of the skin (late finding)<br />

6. Pulselessness or reduced distal pulses: rarely present (late finding)<br />

E. Management<br />

1. Remove constrictive dressings or casts (if present).<br />

2. Obtain immediate orthopedic consult, and measure intracompartmental pressure.<br />

3. Surgical decompression via fasciotomy is indicated if the pressure is >30 mm Hg (normal pressure is


MUSCULOSKELETAL DISORDERS (NONTRAUMATIC)<br />

.. ~. F. Management<br />

1. Saline<br />

a. High volumes of normal saline<br />

b. Titrate to urine output of 200-300 ml/hour<br />

2. Mannitol<br />

a. Controversial<br />

b. May provide a better prognosis in early acute renal failure by increasing urine flow, which may help<br />

prevent obstruction by myoglobin casts<br />

3. Urine alkalinization<br />

G. Prognosis<br />

a. Increases solubility of myoglobin<br />

b. Goal is urine pH of 6.5<br />

c. May use 2 ampules of sodium bicarbonate in 1 L of half-normal saline or 5% dextrose in water<br />

1. Incidence of acute renal failure is unknown; may range from 13% to 50%.<br />

2. Outcome is good provided there is no renal failure.<br />

3. Incidence of renal failure is higher in patients who used illicit drugs or alcohol or have undergone trauma.<br />

Incidence is particularly higher in patients with more than one recognized cause.<br />

405


MUSCULOSKELETAL DISORDERS (NONTRAUMATIC): PRACTICE CLINICAL SCENARIOS<br />

MUSCULOSKELETAL DISORDERS (NONTRAUMATIC):<br />

PRACTICE CLINICAL SCENARIOS<br />

Answers immediately follow the practice clinical scenarios.<br />

Scenario A<br />

Presentation: A 58-year-old woman with a long history of rheumatoid arthritis presents with fever and left<br />

knee pain for the past 24 hours.<br />

Physical examination: Her knee is swollen, warm, and tender.<br />

Diagnostic evaluation: Laboratory evaluation reveals a leukocytosis and a markedly increased erythrocyte<br />

sedimentation rate. Aside from some soft-tissue swelling, radiographs are unremarkable. Arthrocentesis<br />

reveals purulent fluid(> 100,000 WBCs with a predominance of PMNs), a low glucose, and gram-positive<br />

cocci in clusters on Gram stain.<br />

What is the diagnosis?<br />

Scenario B<br />

Presentation: An obese 48-year-old man presents with left knee pain. He states that the pain developed<br />

over a period of a few hours and is so bad that even the weight of a bedsheet on his knee is too much to<br />

bear. He denies recent trauma and prior injury to the knee but admits to overindulging in rich food and<br />

alcohol the past couple of days. His past medical history is remarkable only for hypertension, for which he<br />

is being treated with a loop diuretic.<br />

Physical examination: The patient's knee is erythematous, swollen, hot, and tender.<br />

Diagnostic evaluation: Radiograph evaluation reveals only soft-tissue swelling. Synovial fluid analysis<br />

demonstrates 20,000 WBCs with a predominance of PMNs, a negative Gram stain, and needle-shaped<br />

crystals that are negatively birefringent.<br />

What is the diagnosis?<br />

Scenario C<br />

Presentation: A 68-year-old woman presents with left knee pain for the past 2 days. She denies acute<br />

trauma and prior injury to the knee.<br />

Physical examination: The patient's knee is erythematous, swollen, and tender.<br />

Diagnostic evaluation: Radiographs reveal presence of calcium deposits. Arthrocentesis demonstrates<br />

20,000 WBCs with a predominance of PMNs, a negative Gram stain, and rhomboid-shaped crystals that<br />

are positively birefringent.<br />

What is the diagnosis?<br />

Scenario D<br />

Presentation: A 23-year-old IV heroin user presents after 2 days of back pain and fevers. She complains<br />

that she has tingling of the legs and cannot get comfortable regardless of how she moves or lays.<br />

Physical examination: The patient is tachycardic and febrile on arrival and has diffuse lumbar back pain on<br />

percussion. Rectal tone is normal.<br />

What is the diagnosis?<br />

406


MUSCULOSKELETAL DISORDERS (NONTRAUMATIC): PRACTICE CLINICAL SCENARIOS<br />

Scenario E<br />

Presentation: A 67-year-old woman with a history of breast cancer presents with 2 months of constant<br />

back pain that is worse at night. There is no history of trauma or fevers, but she also reports a 12-pound<br />

weight loss.<br />

Physical examination: There is midline tenderness over the upper lumbar spine without neurologic<br />

findings. Radiographs demonstrate a pathologic facture of the L 1 vertebral body with loss of height.<br />

What is the diagnosis?<br />

Scenario F<br />

Presentation: A 77-year-old woman with a history of dementia and alcohol abuse was found lying in the<br />

bathroom by her daughter after a fal I the previous night. She complains of diffuse muscle aches, fatigue,<br />

generalized weakness, and dizziness.<br />

Physical examination: The patient has diffuse muscle tenderness and appears dehydrated.<br />

Diagnostic evaluation: A urine sample is dark brown. Laboratory studies show blood in the urine without<br />

RBCs present, and increased levels of potassium, BUN, creatinine, myoglobin, and total creatine kinase.<br />

What is the diagnosis?<br />

407


MUSCULOSKELETAL DISORDERS (NONTRAUMATIC): PRACTICE CLINICAL SCENARIOS<br />

ANSWERS TO PRACTICE CLINICAL SCENARIOS<br />

Scenario A<br />

Diagnosis: septic arthritis<br />

Scenario B<br />

Diagnosis: gout<br />

Scenario C<br />

Diagnosis: pseudogout<br />

Scenario D<br />

Diagnosis: epidural abscess<br />

Diagnostic evaluation: An MRI demonstrates a large epidural fluid collection in the L2-L5 space.<br />

C-reactive protein and WBC levels are increased.<br />

Scenario E<br />

Diagnosis: spinal malignancy<br />

Scenario F<br />

Diagnosis: rhabdomyolysis<br />

408


NERVOUS SYSTEM DISORDERS<br />

NERVOUS SYSTEM DISORDERS<br />

Stroke ........................................................................................................................................................................ 412<br />

Seizures ..................................................................................................................................................................... 418<br />

Coma ........................................................................................................................................................................ 422<br />

The Altered Patient .................................................................................................................................................... 425<br />

Neuropathies ............................................................................................................................................................ 427<br />

Disorders of the Neuromuscular Junction ................................................................................................................. 429<br />

Myasthenia Gravis .............................................................................................................................................. 429<br />

Lambert-Eaton Syndrome ................................................................................................................................... 430<br />

Botulism (Food-Borne) ........................................................................................................................................ 430<br />

Myopathies ............................................................................................................................................................... 431<br />

Peripheral Neuropathies ........................................................................................................................................... 431<br />

Trigeminal Neuralgia (Tic Douloureux) ............................................................................................................... 431<br />

Bell's Palsy .......................................................................................................................................................... 432<br />

Mononeuropathies ............................................................................................................................................. 432<br />

Diabetic Neuropathies ....................................................................................................................................... 433<br />

Myelopathies (Disorders of the Spinal Cord) ............................................................................................................. 433<br />

Volkmann lschemic Paralysis ..................................................................................................................................... 433<br />

Vertigo ...................................................................................................................................................................... 434<br />

Headache .................................................................................................................................................................. 435<br />

Meningitis ................................................................................................................................................................. 438<br />

Hydrocephalus .......................................................................................................................................................... 440<br />

409


NERVOUS SYSTEM DISORDERS: SELF-ASSESSEMENT QUESTIONS<br />

NEUROLOGIC DISORDERS: SELF-ASSESSMENT QUESTIONS<br />

1. Symptoms of a vertebrobasilar artery stroke might include all of the following except:<br />

(a) Tinnitus<br />

(b) Vertigo<br />

(c) Monocular vision loss<br />

(d) "Locked-in syndrome"<br />

2. A 65-year-old patient presents with dizziness (vertigo), nausea and vomiting. On examination, he is noted to have<br />

lateralizing dysmetria and is unable to stand erect or ambulate without assistance. No other deficits are noted. The<br />

most likely diagnosis is:<br />

(a) Lacunar infarct<br />

(b) Vertebrobasilar artery occlusion<br />

(c) Middle cerebral artery occlusion<br />

(d) Cerebellar infarct<br />

3. Transient global amnesia is characterized by all of the following except:<br />

(a)<br />

It generally occurs in patients >60 years old.<br />

(b) It is characterized by loss of recent memory and inability to record new memory.<br />

(c) Motor, sensory, and speech functions are interrupted during the event.<br />

(d) In general, patients have no recollection of events that occur during the episode.<br />

4. The most appropriate therapy for refractory seizures secondary to isoniazid overdose is:<br />

(a) Thiamine<br />

(b) Pyridoxine<br />

(c) Magnesium sulfate<br />

(d) Lorazepam<br />

5. A patient presents with an intracranial hemorrhage and a blood pressure of 220/135 mm Hg. Which of the following<br />

is not a good choice for initial management?<br />

(a) Labetalol IV<br />

(b) Nicardipine IV<br />

(c) Nitroglycerin IV<br />

(d) ~-blocker<br />

6. Acute periodic paralysis generally occurs in males 7-21 years old and is associated with:<br />

(a)<br />

Hypokalemia<br />

(b) Normokalemia<br />

(c) Hyperkalemia<br />

(d) Hypocalcemia<br />

7. Which of the following syndromes generally occurs in association with malignancy?<br />

(a)<br />

Guillain-Barre<br />

(b) Eaton-Lambert<br />

(c) Myasthenia gravis<br />

(d) Polymyositis<br />

8. The most frequent initial symptoms of myasthenia gravis are:<br />

(a)<br />

Dysarthria and dysphagia<br />

(b) Muscle atrophy<br />

(c) Respiratory failure<br />

(d) Ptosis, diplopia, and blurred vision<br />

410


NERVOUS SYSTEM DISORDERS: SELF-ASSESSEMENT QUESTIONS<br />

9. Which of the following is a contraindication to IV tPA in an acute ischemic stroke patient?<br />

(a) Current warfarin therapy with an INR 100,000/mm 3<br />

(d) Cerebral infarction


NERVOUS SYSTEM DISORDERS<br />

I. STROKE<br />

A. Anatomy of cerebral blood flow<br />

1. Anterior circulation: 80% of cerebral blood flow originates from the carotid arteries, which supply the:<br />

a. Frontoparietal lobes<br />

b. Anterior aspect of temporal lobes<br />

c. Optic nerve and retina<br />

d. Many deep gray matter structures<br />

2. Posterior circulation: 20% of cerebral blood flow originates from the vertebrobasilar arteries, which supply the:<br />

a. Medial aspect of temporal lobe<br />

b. Visual occipital cortex<br />

c. Thalamus<br />

d. Brainstem<br />

e. Upper spinal cord<br />

f. Cerebellum<br />

g. Auditory and vestibular functions of the ear<br />

3. Circle of Willis: provides a connection between the anterior and posterior circulations; its integrity determines<br />

the clinical outcome of thromboembolic strokes because of the col lateral flow provided.<br />

B. Pathophysiology of cerebral ischemia and infarction<br />

1. lschemic strokes (may be caused by cerebrovascular thrombosis, emboli, or structural occlusion of a vessel)<br />

a. Cerebral in situ vascular occlusion (thrombosis) results from one of the following:<br />

(1) Atherosclerosis (most common cause in patients >50 years old)<br />

(a) Atherothrombotic infarcts are associated with plaques in the carotid and vertebrobasilar arteries or<br />

the middle cerebral artery.<br />

(b) Cerebral vessel branch points are common areas for thrombotic occlusion, because clot formation<br />

is prone to develop in areas of turbulent blood flow.<br />

(c) Patients of African-American descent and those with diabetes have a particularly high risk of<br />

intracranial thrombosis.<br />

(2) Hypercoagulable states (common cause in patients


NERVOUS SYSTEM DISORDERS<br />

2. Hemorrhagic strokes result from one of the following:<br />

Courtesy of Michael Abraham, MO<br />

CT scan showing right-sided temporoparietal intraparenchymal hemorrhage<br />

a. Spontaneous rupture of a congenital or berry aneurysm (subarachnoid hemorrhage)<br />

b. Spontaneous rupture of an AV malformation (less common cause of a subarachnoid hemorrhage)<br />

c. Spontaneous bleeding from arterioles (accounts for most cerebral hemorrhages) and are secondary to one<br />

of the following:<br />

(1) Chronic hypertension (arteriolar injury)<br />

(2) Ruptured mycotic aneurysm<br />

(3) Neoplasm (especially breast cancer, melanoma)<br />

(4) Thrombocytopenia<br />

d. Amyloid angiopathy<br />

(1) Common cause of intracerebral hemorrhage in the elderly<br />

(2) CT_,. distinctive location of the hemorrhage at the junction of gray and white matter<br />

e. Hemorrhagic transformation of an embolic (ischemic) stroke is:<br />

(1) Characterized by abrupt progression of neurologic dysfunction after a stroke and<br />

(2) Associated with fragmentation of an occluding embolus or administration of thrombolytics (there does<br />

not appear to be an increased risk of hemorrhage in patients on chronic ASA therapy).<br />

C. Incidence of stroke<br />

(3) Not always evident on CT; recent studies suggest that gradient recall echo MRI may be a better choice.<br />

1. Patients with a stroke frequently have a history of one or more of the following:<br />

a. Transient ischemic attack (TIA)-30%~35% of patients will have a stroke within 5 years, the highest<br />

incidence occurring within the first month and the earliest incidence within 2 days<br />

b. Hypertension (most common)<br />

c. Atherosclerosis<br />

d. Cardiac disease<br />

e. Migraine headaches<br />

f. Claudication<br />

g. Diabetes<br />

h. Hyperlipidemia<br />

1. Smoking<br />

j. Thrombocytosis<br />

k. Oral contraceptive use (high-estrogen formulas)<br />

I. E ryth rocytos is<br />

m. Sickle cell disease<br />

413


NERVOUS SYSTEM DISORDERS<br />

2. A carotid bruit<br />

a. Absence of a bruit does not exclude the presence of stenosis.<br />

b. Presence/absence of a bruit has no predictive value regarding the likelihood of stroke related to carotid<br />

artery disease; however, if >70% carotid stenosis is confirmed radiographically, there is a 26% risk of stroke<br />

within 2 years in patients who have had a TIA.<br />

D. Differential diagnosis of stroke (aka mimics)<br />

1. Seizure<br />

a. Although embolic strokes may present with seizures, a transient postictal neurologic deficit (such as Todd<br />

paralysis) is far more common.<br />

b. Patients with other intracranial pathology (post-traumatic hematoma formation, tumor, abscess) may also<br />

present with seizures and neurologic deficits. Patients with new focal deficits in the setting of seizure<br />

should undergo emergent CT scanning to look for structural CNS lesions. (ACE P Clinical Policy: Initial<br />

Approach to Patients Presenting with a Chief Complaint of Seizure Who Are Not in Status Epilepticus)<br />

2. Migraine<br />

a. The aura preceding migraine headache may include focal neurologic deficits (eg, weakness, numbness,<br />

aphasia, visual loss); a previous history of focal symptoms associated with migraine is required to make<br />

this diagnosis. Transient ischemia associated with migraine may also result in a stroke.<br />

b. MRI with diffusion-weighted imaging may help distinguish migraine from cerebral infarction.<br />

3. Trauma to the middle meningeal artery (epidural or extradural hematomas)<br />

4. Traumatic or spontaneous venous bleeding (subdural hematoma)<br />

5. Toxic/metabolic disorder (those listed below may cause focal neurologic deficits and, if present, may exclude<br />

a stroke)<br />

a. Toxic exposure (some drugs, poisons)<br />

b. Hypo/hyperglycemia<br />

c. Hypo/hypernatremia<br />

d. Hepatic encephalopathy<br />

E. lschemic stroke syndromes<br />

1. A TIA should be considered a warning sign of an impending thrombotic or embolic syndrome. It is a "red flag,"<br />

heralding stroke in evolution.<br />

2. Differentiating thrombotic and embolic strokes<br />

a. Thrombotic syndromes are often characterized by a stuttering onset of symptoms. Some patients wake up<br />

with neurologic disability.<br />

b. Embolic syndromes are characterized by abrupt onset of symptoms with maximal deficit that improves<br />

slowly as the embolus breaks up and moves peripherally. Embolic strokes are more likely to occur when<br />

the patient is active.<br />

3. Occlusive stroke syndromes may be embolic or thrombotic.<br />

a. Middle cerebral artery<br />

(1) Most common site of intracranial cerebral artery thrombosis<br />

(2) Clinical presentation (frequently incomplete)<br />

(a) Contralateral hemiplegia, hemisensory loss, and homonymous hemianopsia<br />

(b) Upper extremity deficit that is more severe than lower extremity deficit<br />

(c) Aphasia (if the dominant hemisphere is involved)<br />

(d) Constructional apraxia and agnosia (if the nondominant hemisphere is involved)<br />

(e) Gaze preference in the direction of the lesion<br />

b. Anterior cerebral artery<br />

(1) Clinical presentation<br />

(a) Contralateral leg, foot, and arm paralysis<br />

(b) Sensorimotor deficit greater in the lower (rather than the upper) extremity<br />

(c) Loss of frontal lobe control<br />

i. Abulia (difficulty in responding to questions because of slowed mentation)<br />

ii. Gait apraxia (clumsiness)<br />

iii. Incontinence<br />

iv. Primitive grasp and suck reflexes<br />

414


NERVOUS SYSTEM DISORDERS<br />

c. Posterior cerebral artery<br />

(1) Affects blood supply to the occipital cortex and branches that supply upper midbrain structures<br />

(2) Clinical presentation<br />

(a) Contralateral homonymous hemianopsia (the patient may be unaware of this before clinical<br />

evaluation), hemiparesis, and hemisensory loss<br />

(b) Memory loss<br />

(c) lpsilateral cranial Ill nerve palsy (usually pupil-sparing)<br />

d. Vertebrobasilar artery (not all findings are present)<br />

(1) The vertebrobasilar arteries are affected by both atherosclerotic narrowing and embol ic events that<br />

may result in:<br />

(a) Cerebellar and cranial nerve deficits<br />

(b) Simultaneous deficits on both sides of the body<br />

(2) Clinical presentation<br />

(a) Pain and temperature deficits on one side of the face and on opposite side of the body<br />

(b) lpsilateral cranial nerve palsies<br />

(c) Cerebellar signs<br />

i. Those that suggest labyrinthitis<br />

• Vertigo<br />

• Severe ataxia<br />

• Nausea/vomiting<br />

11. Those that, along with cranial nerve palsies, are not present in patients with labyrinthitis<br />

• Vertical or bidirectional nystagmus<br />

• lpsilateral dysmetria (finger-nose-finger, heel-shin, past-pointing)<br />

• Dysdiakokinesis (rapid alternating movements)<br />

(d) Contralateral hemiplegia<br />

(e) Contralateral sensory deficits<br />

(3) Wallenberg syndrome (lateral medullary plate syndrome)<br />

e. Basilar artery<br />

(a) Occurs when occlusion of the proximal vertebral artery causes infarction of the lateral medulla<br />

(b) Clinical presentation<br />

i. Vertigo<br />

ii. Dysphagia<br />

iii. Horner syndrome<br />

iv. Facial pain/numbness<br />

v. lpsilateral clumsiness<br />

vi. Contralateral cool extremity (distally)<br />

(1) Characterized by severe bilateral signs<br />

(2) Clinical presentation<br />

(a) Quadriplegia<br />

(b) "Locked-in syndrome" (loss of all motor function except upward gaze)<br />

(c) Coma<br />

f. Cerebellar infarcts<br />

(1) Clinical presentation<br />

(a) Ataxia<br />

(b) Dizziness (vertigo)<br />

(c) Nausea/vomiting<br />

(d) Nystagmus (vertical or bidirectional)<br />

(e) Lateralizing dysmetria<br />

(f)<br />

g. Lacunar infarcts<br />

Dysdiadokokinesis<br />

(1) These small cystic infarcts are usually caused by a hypertensive vasculopathy (lipohyalinosis) but may<br />

occur in diabetic patients and can affect both the anterior and posterior cerebral vessels. Lacunar<br />

415


NERVOUS SYSTEM DISORDERS<br />

strokes involve penetrating cerebral arterial vessels lying deep in the gray matter (internal capsule)<br />

or brainstem.<br />

(2) There are four specific lacunar infarct states<br />

(a) The 11 dysarthria-clumsy hand syndrome" (midpons)<br />

(b) Pure sensory syndrome (thalamus)<br />

(c) Leg paresis and ataxia (pons or internal capsule)<br />

(d) Pure motor hemiplegia (pons or internal capsule): most common<br />

h. Transient global amnesia<br />

(1) Associated with ischemia to the temporal lobes or thalamus<br />

(2) Generally occurs in patients >60 years old, lasts from 30 minutes to 36 hours, and then resolves<br />

completely; has also been reported in association with therapeutic use of benzodiazepines (especially<br />

triazolam).<br />

(3) Clinical presentation<br />

(a) Sudden loss of recent memory and inability to record new memories; the patient usually has no<br />

recollection of events that occurred during the episode.<br />

(b) Preservation of long-term past memory and self-identity<br />

(c)<br />

F. Hemorrhagic syndromes<br />

Preservation of motor, sensory, and speech functions<br />

(d) The ability to carry out complex tasks (drive a car, give a speech) during the episode<br />

1. Characterized by abrupt onset of symptoms (usually headache, vomiting) and rapid progression of neurologic<br />

deficits. The patient may be lethargic or exhibit decreased mentation by the time he or she arrives in the<br />

emergency department. However, a more subtle presentation (not marked by a rapid clinical deterioration) is<br />

also common. Initial blood pressure is usually high in patients with significant hemorrhage.<br />

2. Subarachnoid hemorrhage (SAH): nontraumatic<br />

Courtesy of Michael Abraham, MO<br />

Classic CT scan of SAH<br />

a. Highest incidence in women 35-65 years old and in men


NERVOUS SYSTEM DISORDERS<br />

e. Diagnostic evaluation<br />

(1) CT of the brain is most sensitive in the first 6 hours after the bleed and is useful in differentiating SAH<br />

from other causes of severe headache.<br />

(2) ACEP/AHA guidelines suggest that if the scan is negative, a lumbar puncture should be done, because<br />

CT is ~96% sensitive.<br />

(3) MRI may be as accurate as CT in patients with focal signs or symptoms.<br />

(4) Lumbar puncture findings suggestive of SAH<br />

(a) Grossly bloody fluid<br />

(b) CSF does not progressively clear (unlike in a traumatic tap, in which the first tube of CSF taken<br />

immediately after the lumbar puncture shows higher RBC counts than the last tube)<br />

(c) Xanthochromia (takes up to 12 hours to develop and lasts 2 weeks): due to the degradation of<br />

RBC in the CSF<br />

(d) Normal (or ambiguous) CSF findings and a nondiagnostic CT in a patient with a high pretest risk<br />

of SAH do not exclude an unruptured (yet symptomatic) aneurysm. Angiography is indicated in<br />

these patients (CTA/MRA/DSA).<br />

(5) Angiography is not routinely indicated to exclude SAH for patients in whom the CT and lumbar<br />

puncture are both normal. (ACEP Clinical Policy: Critical Issues in the Evaluation and Management of<br />

Patients Presenting to the Emergency Department with Acute Headache)<br />

3. Hypertensive intracerebral hemorrhage<br />

a. Putamen hemorrhage (most common)<br />

(1) Similar in presentation to middle cerebral artery syndrome<br />

(a) Contralateral hemiplegia, hemianesthesia, and homonymous hemianopsia; gaze palsy may be<br />

present.<br />

(b) Aphasia (if the dominant hemisphere is involved) or hemineglect (if the nondominant hemisphere<br />

is involved)<br />

(2) Distinguishing feature: greater depression of consciousness with putamen hemorrhage than with<br />

middle cerebral artery occlusion<br />

b. Cerebellar hemorrhage<br />

(1) Sudden onset of ataxia, dizziness, occipital headache, repeated vomiting, and loss of ability to stand or<br />

walk is typical.<br />

(2) If diagnosed early, this condition is treatable with surgical decompression and hematoma evacuation.<br />

c. Thalamic hemorrhage<br />

(1) Contralateral hemiparesis and hemianesthesia<br />

(2) Sensory loss > motor loss<br />

(3) Restricted upward gaze or abnormal deviation of the eyes without a visual field defect (occasionally)<br />

(4) Coma (may resolve if associated with focal depression of reticular activating system)<br />

d. Pontine hemorrhage<br />

(1) Severe occipital headache with rapid progression to coma (associated with poor prognosis)<br />

(2) Decerebrate posturing<br />

(3) Hyperventilation<br />

(4) Pinpoint pontine pupils ("3 P's"); other causes of pinpoint pupils:<br />

(a) Narcotic overdose (associated with hypoventilation)<br />

(b) Cholinesterase inhibitor poisoning (pesticides)<br />

(c) Pilocarpine eye drops or systemic atropine<br />

(d) Clonidine overdose<br />

(5) Absence of oculovestibular reflexes on caloric testing and doll's head maneuver<br />

G. Treatment of stroke<br />

1. Intubation should be considered in any stroke patient with a depressed level of consciousness.<br />

2. Hypertension is commonly associated with acute stroke. Should it be treated? The general consensus in the<br />

neurology literature is not to treat hypertension in patients with acute thrombotic stroke unless they are<br />

candidates for thrombolysis.<br />

a. Is the patient going to receive thrombolysis?<br />

(1) If yes: lower the blood pressure to 220 mmHg.<br />

417


NERVOUS SYSTEM DISORDERS<br />

(3) Patients with chronic hypertension may suffer greater ischemia when the pressure is lowered.<br />

(4) Perfusion of the ischemic but still viable (ischemic penumbra) tissue around the stroke area may<br />

require a higher blood pressure.<br />

b. Treatment of patients with hemorrhagic stroke is less clear, but the neurosurgery literature suggests that<br />

significant increases in blood pressure be treated. The goal is to reduce the systolic blood pressure to<br />


NERVOUS SYSTEM DISORDERS<br />

B. Types<br />

(2) Extracranial pathology<br />

1. Partial seizures<br />

(a) Toxic (eg, alcohol withdrawal, cocaine, tricyclic antidepressant overdose)<br />

(b) Uremic or hypertensive encephalopathy<br />

(c) Eclampsia<br />

(d) Endocrine/electrolyte abnormalities, hypoglycemia<br />

a. Simple partial (focal) seizures-,, motor, sensory, autonomic symptoms depending on where the electrical<br />

discharge is coming from; no loss of consciousness<br />

b. Complex partial seizures (temporal lobe or psychomotor)-,, motor, sensory, autonomic, or psychic onset<br />

(may mimic functional psychosis) associated with impaired consciousness, automatisms (lip smacking,<br />

picking at clothes), and postictal confusion<br />

c. Secondary generalized partial seizures (tonic-clonic or grand mal)-,, motor, sensory, autonomic or<br />

psychic onset followed by a generalized seizure, coma, and slow return to consciousness; tongue biting or<br />

incontinence may occur.<br />

2. Generalized seizures<br />

a. Nonconvulsive: absence (petit mal) seizures-,, brief (-10 seconds) alteration in level of consciousness (eg,<br />

cessation of activity associated with staring in a school-aged child); motor tone may increase or decrease<br />

with mild clonic movements or automatisms, which may be precipitated by hyperventilation.<br />

b. Convulsive: tonic-clonic, grand mal<br />

3. Todd paralysis: a transient focal paralysis after a seizure that usually lasts 1-2 hours but can last 1-2 days<br />

C. Differential diagnosis<br />

1. Myoclonus (may be associated with a multitude of disorders that include hypotension, Creutzfeldt-Jakob<br />

disease, uremia, etomidate, or lithium toxicity)<br />

2. Narcolepsy/cataplexy<br />

3. Tetanus<br />

4. Syncope<br />

5. Psychogenic seizures (pseudoseizures)<br />

6. Strychnine poisoning (patients remain alert)<br />

Table 24: Historical Clues that Distinguish a Generalized Seizure from Syncope<br />

Generalized Seizure<br />

Occurs without warning<br />

Tonic-clonic movements<br />

Prolonged postictal period<br />

Syncope<br />

Prodrome of darkening vision, nausea<br />

May have brief clonic movements<br />

No postictal period after return of normal cerebral blood flow<br />

D. Diagnostic evaluation for a first-time seizure, questionable seizure, unusual seizure, or seizure that is<br />

complicated by injury, illness, or substance abuse<br />

1. The only routine tests that have a reasonable yield in the evaluation of a first-time seizure are a serum glucose<br />

and sodium; so-called baseline studies (CBC, urinalysis, electrolytes other than sodium, BUN/creatinine, etc)<br />

are only indicated by clinical assessment, so a good history and physical examination are essential.<br />

2. Other tests that may be useful (again, based on history and physical examination)<br />

a. Lumbar puncture<br />

b. Blood alcohol, toxicology screen<br />

c. Anticonvulsant levels (in patients taking them)<br />

d. Chest radiograph (if tumor or aspiration is suspected)<br />

3. Indications for neuroimaging<br />

a. Emergent CT or MRI<br />

(1) Postictal patients with:<br />

(a) Status epilepticus<br />

(b) Recent head injury<br />

419


NERVOUS SYSTEM DISORDERS<br />

(c) Prolonged postictal state<br />

(d) Severe headache<br />

(e) New focal deficit<br />

(2) Patients with new-onset seizures whose fol low-up for routine CT cannot be assured. (ACEP Clinical<br />

Policy: Initial Approach to Patients Presenting with a Chief Complaint of Seizure Who Are Not in Status<br />

Epi lepticus)<br />

b. Urgent CT (performed during emergency department evaluation)<br />

E. Status epilepticus<br />

(1) Suspected CNS lesion<br />

(2) New seizure (in patients at risk of neurocysticercosis or HIV, and those with a history of malignancy)<br />

1. Definition: seizures that persist for 2:5 minutes or repetitive seizures without a lucid interval; permanent<br />

neuronal damage may result within 1 hour in patients with uncontrolled seizures; mortality rate is as high<br />

as 30%.<br />

2. Treatment (adults)<br />

420


NERVOUS SYSTEM DISORDERS<br />

I<br />

Glucose 25-50 g (50% sol)<br />

(if bedside test positive)<br />

I<br />

Naloxone 2-4 mg<br />

(if drug overdose suspected)<br />

I<br />

I<br />

Thiamine 100 mg (if alcoholism or Magnesium sulfate 20 ml (10% sol) IV<br />

malnutrition is suspected)<br />

if patient is eclamptic (pregnant or<br />

I<br />

I<br />

postpartum), alcoholic, or malnourished<br />

Lorazepam 2 mg/min (0.1 mg/kg) IV,<br />

up to 10 mg as needed<br />

OR<br />

Diazepam 5 mg/min IV every<br />

5 min as needed, up to 20 mg<br />

(pediatric: 0.15 mg/kg, up to 1 0 mg)<br />

OR<br />

Midazolam 0.2 mg/kg IN or 0.5 mg/kg buccally<br />

I<br />

I<br />

I<br />

Pyridoxine 5 g (5 or 10% sol) IV<br />

(if isoniazid overdose suspected)<br />

Phenytoin* 18-20 mg/kg IV at a rate of<br />

25-50 mg/min; dissolve calculated dose Phenobarbital 8-20 mg/kg at a rate<br />

in normal saline and administer piggybacked<br />

of 60 mg/min (indicated in patients on<br />

via infusion pump into the main IV line while<br />

barbiturates and in those for whom phenytoin<br />

the benzodiazepine<br />

is contraindicated because of allergy or the<br />

is still being given; it is the long-acting agent of<br />

presence of AV block)<br />

choice for the treatment of<br />

status epilepticus.<br />

I<br />

I I I I<br />

I<br />

I<br />

Phenobarbital Phenytoin - a second Valproic acid<br />

10 mg/kg at a rate of infusion - 9 mg/kg 20-40 mg/kg<br />

100 mg/min (indicated at a rate of 50 mg/min at a rate not to exceed<br />

if seizures persist and up to 20 mg/min<br />

may be repeated up to 30 mg/kg (indicated in<br />

a maximum dosage of patients with a partial<br />

30 mg/kg for refractory response to the initial<br />

status)<br />

infusion)<br />

Levetiracetam<br />

2-4 gm infused over<br />

15 min<br />

*Fosphenytoin may be used instead. A prodrug that is metabolized to phenytoin, fosphenytoin has two<br />

advantages: a more rapid rate of administration IV and it can be given IM. Fosphenytoin is administered in<br />

phenytoin equivalents (loading dose is 15-20 mg PE/kg at 100-150 mg PE/min).<br />

Medications Administered in Initial Stabilization of Status Epilepticus<br />

421


NERVOUS SYSTEM DISORDERS<br />

3. Treatment (children)<br />

a. Airway management (intubation and oxygen)<br />

b. IV line followed by:<br />

(1) Rapid glucose determination - glucose 2-4 ml/kg 25% solution as needed (D10 in neonates)<br />

(2) Pyridoxine 50-100 mg slowly IV (for neonates)<br />

(3) Calcium gluconate 4 ml/kg IV (for neonates)<br />

c. See also "Medications Administered in Initial Stabilization of Status Epilepticus" (figure, above).<br />

Ill. COMA (PATIENT IS UNRESPONSIVE TO VERBAL AND<br />

PAINFUL STIMULI)<br />

A. Etiology: mnemonic "TIPS AEIOU"<br />

Irauma, Iemperature<br />

Infection<br />

forphyria, fsychiatric disorder<br />

.S_pace-occupying lesion, Subarachnoid hemorrhage, Stroke, Shock<br />

Alcohol (drugs and toxins)<br />

_Endocrine, _Exocrine and _Electrolyte disorders<br />

Insulin<br />

Qxygen, Qpiates<br />

!J.remia<br />

1. Trauma (primarily head injury)<br />

a. Subdural hematoma<br />

Courtesy of Michael Abraham, MD<br />

CT scan showing bilateral subdural hemorrhage<br />

b. Epidural hematoma<br />

c. Traumatic subarachnoid hemorrhage<br />

422


NERVOUS SYSTEM DISORDERS<br />

Courtesy of Michael Abraham, MD<br />

CT scan showing subdural, subarachnoid, and intraparenchymal hemorrhage from trauma<br />

2. Temperature<br />

a. Hypothermia: causes coma when temperature is 107.6°F (42°C)<br />

3. Infection<br />

a. Meningitis<br />

b. Encephalitis<br />

c. Sepsis<br />

4. Endocrine, exocrine, and electrolyte disorders<br />

a. Hypo/hyperglycemia<br />

b. Hypo/hypernatremia<br />

c. Hypo/hypercalcemia<br />

d. Hypo/hyperthyroidism<br />

e. Hepatic encephalopathy (t NH)<br />

B. Physical examination<br />

1. Always check and stabilize the ABCs before any detailed evaluation.<br />

2. Because there is frequently little or no known history in these patients, perform a thorough physical<br />

examination, including completely exposing the patient.<br />

3. Smell the breath.<br />

a. Fruity (diabetic ketoacidosis)<br />

b. Feculent (bowel obstruction)<br />

c. Putrid (anaerobic infection)<br />

d. Garlic (organophosphate insecticide)<br />

e. Bitter almonds (cyanide): only 60%-80% of the population can detect this.<br />

f. Alcohols (ethanol, isopropyl alcohol); methanol and ethylene glycol are odorless.<br />

g. Ammonia (uremia)<br />

h. Wintergreen (methyl salicylate)<br />

i. Fetor hepaticus: a musty smell (liver failure)<br />

4. Observe the respiratory pattern; it may have diagnostic value.<br />

a. Cheyne-Stokes: periodic, regularly increasing breaths in a crescendo pattern, followed by decrescendo<br />

breathing and ending with short periods of apnea that are due to dysfunction (structural or metabolic) of<br />

both cerebral hemispheres or upper brainstem<br />

b. Hyperventilation: causes include:<br />

(1) Hypoxia<br />

423


NERVOUS SYSTEM DISORDERS<br />

(2) Metabolic acidosis<br />

(3) Salicylate overdose<br />

(4) Lesions of the midbrain<br />

(5) Hepatic coma<br />

(6) t lntracranial pressure (ICP)<br />

c. Apneustic breathing: an end-inspiratory pause that occurs in association with lesions of the pons in the area<br />

of cranial nerve V<br />

d. Cluster breathing: short bursts of breathing that are associated with lower pontine lesions<br />

e. Ataxic breathing: an irregular breathing pattern that precedes death and is associated with lesions of<br />

the medulla<br />

5. Look for any abnormal posturing. Both decorticate and decerebrate posturing may be seen in patients with<br />

hypoglycemia or severe hepatic encephalopathy.<br />

a. Decorticate posturing<br />

(1) Flexion of the upper extremities and extension of the lower extremities<br />

(2) Suggests a lesion in the upper midbrain or internal capsule<br />

b. Decerebrate posturing: extension of al I extremities; suggests a deeper, more severe dysfunction<br />

(1) Severe midbrain lesions<br />

(2) Posterior fossa lesions that compress or partially damage the midbrain and pons<br />

(3) Severe metabolic disorders (hepatic coma, drug overdose) that depress the function of the deep<br />

diencephalon and forebrain<br />

(4) Bilateral diffuse hemispheric abnormalities (postanoxic cerebral demyelination)<br />

6. Look at the skin.<br />

a. Needle tracks (IV drug abuse)<br />

b. Cyanosis (hypoxia, hemoglobinopathy)<br />

c. Pallor (anemia)<br />

d. Cherry red (carbon monoxide - late finding)<br />

e. Petechial or purpuric rash (meningococcemia, Rocky Mountain spotted fever)<br />

f. Diaphoresis (hypoglycemia, cholinergic/salicylate poisoning, sympathomimetic)<br />

g. Pressure bullae (barbiturates, carbon monoxide, meprobamate)<br />

h. Ecchymoses at common sites of insulin injection (hypoglycemia)<br />

7. Look for ocular signs.<br />

a. Pupillary reactivity (the most important physical sign in distinguishing metabolic from structural causes<br />

of coma)<br />

(1) Reactive (but sluggish) pupils are characteristic of a metabolic or toxic disorder.<br />

(2) Nonreactive pupils are associated with a structural lesion with the following exceptions:<br />

(a) Anoxia (fixed and dilated)<br />

(b) Atropine and scopolamine (fixed and dilated)<br />

(c) Hypothermia (may be fixed)<br />

b. Anisocoria (unequal pupils ;:,1 mm difference)<br />

(1) Found in 10%-20% of the normal population<br />

(2) Other causes<br />

(a)<br />

Local eye trauma (traumatic mydriasis)<br />

(b) Instillation of a cycloplegic agent in one eye<br />

(c) Prior eye surgery<br />

(d) Compression of cranial nerve Ill by an enlarging aneurysm<br />

(e) Uncal herniation<br />

c. Other pupillary abnormalities<br />

(1) Pupil initially constricts to direct light during the swinging light test but dilates paradoxically as the<br />

light swings back to it, because it perceives a less intense stimulus than the normal eye perceived -<br />

Marcus-Gunn pupil (retina/optic nerve pathology [unilateral])<br />

(2) Large, irregular pupil constricts very slowly to direct light - Adie's pupil (diabetic neuropathy, alcoholism)<br />

(3) Dilated pupil unreactive to light (most commonly cranial nerve Ill [or cycloplegic])<br />

424


NERVOUS SYSTEM DISORDERS<br />

(4) Small, unilateral pupil reacts to light__,. Horner syndrome (interruption of the sympathetic nerve<br />

supply, usually in the neck)<br />

(5) Small, bilateral pupils unreactive to light but reactive to accommodation__,. Argyll-Robertson pupil<br />

(suggests neurosyph i I is)<br />

d. Fundi<br />

(1) Spontaneous venous pulsations (normal ICP)<br />

(2) Papilledema ( ICP, chronic)<br />

(3) Subhyaloid hemorrhages<br />

e. Ocular reflexes<br />

(1) Corneal reflex tests cranial nerves V (sensory) and VII (motor); absent with lesions in the posterior fossa<br />

or brainstem.<br />

(2) Oculocephalic reflex (doll's eyes): if the cervical spine has been cleared, turn the head in either the<br />

horizontal or vertical plane; a normal response (intact brainstem in a comatose patient) is conjugate<br />

deviation of the eyes in a direction opposite to the head rotation.<br />

(3) Oculovestibular reflex (cold calorics): Elevate the head 30° by raising the head of the bed, and irrigate<br />

the ear canal with 10 ml cold water.<br />

(a)<br />

Horizontal nystagmus with the fast component away from the irrigated ear is a normal response,<br />

indicating that the brainstem and the cerebral cortex are intact and the patient is not comatose.<br />

(b) Slow conjugate ipsilateral eye deviation toward the irrigated ear occurs in the comatose patient<br />

whose brainstem is functioning.<br />

(c) No response indicates brainstem dysfunction and most commonly results from structural causes.<br />

C. Essential stabilization and assessment measures<br />

1. Airway management with cervical spine control<br />

2. IV line followed by administration of thiamine 100 mg, glucose 50 g of 50% solution (if hypoglycemic), and<br />

naloxone 2-4 mg<br />

3. Oxygen supplementation and pulse oximetry<br />

4. Cardiac monitor<br />

5. Brief history ("what happened"; go through personal effects, EMS history)<br />

6. Physical and neurologic examinations, including Glasgow Coma Score<br />

7. Complete history (including known allergies)<br />

8. Radiographic clearance of cervical spine (if trauma suspected)<br />

9. Laboratory studies and further radiographic evaluation as indicated<br />

IV. THE ALTERED PATIENT<br />

A. Classic clinical scenario<br />

1. A patient is brought to the emergency department by a concerned relative, friend, or neighbor who has noticed<br />

a behavioral change. There is a history of disorientation, forgetfulness, and hallucination.<br />

2. Your task is to determine whether there is organic disease (an underlying medical problem) or a functional<br />

(psychiatric) disorder.<br />

a. Recent onset of a behavioral change with rapid progression suggests an organic cause and can occur in any<br />

age group.<br />

b. The dementias tend to occur in the elderly, and there is a history of gradual onset with slow progression of<br />

a behavioral change; however, they can be exacerbated by organic causes.<br />

B. Etiology: organic causes of behavioral change<br />

1. Drug intoxication or withdrawal<br />

2. Metabolic disorders<br />

3. Toxic exposure<br />

4. Cerebrovascular disease<br />

5. Tumors<br />

6. CNS infections (meningitis, encephalitis, cerebral abscess)<br />

7. Anoxia<br />

425


NERVOUS SYSTEM DISORDERS<br />

8. Hypercapnia<br />

9. Uremia<br />

10. Hepatic encephalopathy<br />

11. Vitamin deficiency (B 121<br />

thiamine, niacin)<br />

12. Sepsis<br />

13. Neurosyphilis<br />

14. Infections in the elderly<br />

C. Psychiatric disorders<br />

1 . Causes of acute mental confusion<br />

a. Schizophrenia<br />

b. Mania<br />

c. Agitated psychosis<br />

d. Psychotic depression<br />

e. Severe adjustment reaction<br />

f. Posttraumatic/dissociative episodes<br />

2. Dementia: a syndrome of global and progressive deterioration of intellectual function as well as impairment of<br />

memory, language, and personality; some of the causes are medically correctable.<br />

a. Reversible causes (20%)<br />

(1) Drug toxicity (al most any agent should be suspect)<br />

(2) Emotional disorders (especially depression)<br />

(3) Metabolic disorders<br />

(4) Nutritional deficiencies (especially B 12<br />

)<br />

(5) Normal-pressure hydrocephalus<br />

(6) Tumors<br />

(7) Infection (including non-CNS, eg, pneumonia, urinary tract infection, diverticulitis)<br />

(8) Complications of arteriosclerosis<br />

(9) Wilson disease<br />

b. Irreversible causes (80%)<br />

(1) Alzheimer disease (50%)<br />

(2) Multi-infarct dementia (20%)<br />

(3) Mixed Alzheimer/multi-infarct dementia (10%)<br />

(4) Rare causes<br />

(a)<br />

Parkinson disease<br />

(b) Huntington chorea<br />

(c) Creutzfeldt-Jakob disease<br />

3. Delirium versus dementia<br />

a. Dementia<br />

(1) Global cognitive impairment<br />

(2) Insidious<br />

(3) Vital signs normal<br />

(4) Typically elderly<br />

(5) Progressive<br />

(6) Focused attention<br />

b. Delirium<br />

(1) Global cognitive impairment<br />

(2) Acute onset<br />

(3) Abnormal vital signs<br />

(4) Younger individual<br />

(5) Fluctuating<br />

(6) Impaired attention<br />

426


NERVOUS SYSTEM DISORDERS<br />

D. Clinical presentation: Evaluation requires screening evaluation consisting of history and physical and<br />

neurologic examinations, including the Mini-Mental State Evaluation; never"OMIT" it:<br />

1. Qrientation (time and place)<br />

2. Memory (recent and past)<br />

3. Intelligence (thought pattern)<br />

4. Ialk (speech pattern)<br />

E. Diagnostic evaluation (directed by physical examination and history)<br />

1. Arterial blood gases<br />

2. CBC and urinalysis<br />

3. Chemistries (glucose, electrolytes, BUN/creatinine)<br />

4. Liver and thyroid function tests<br />

5. Cultures (blood and urine)<br />

6. Toxicology screen (including drug and heavy metals)<br />

7. Chest radiograph<br />

8. CT scan (head)<br />

9. Serum osmolality<br />

10. Serum ammonia<br />

11. Lumbar puncture and evaluation of CSF (if CNS infection suspected)<br />

12. B 12<br />

levels<br />

13. VDRL or FTA<br />

V. NEUROPATHIES (DISORDERS THAT AFFECT PERIPHERAL<br />

NERVES)<br />

A. Etiology (multiple, include toxic, metabolic, and hereditary)<br />

1. Toxins<br />

a. Heavy metals (lead, arsenic, thallium)<br />

b. Drugs (eg, amiodarone, dapsone, vincristine)<br />

c. Industrial solvents<br />

d. Organophosphates<br />

e. Diphtheria, tetanus<br />

f. Ethanol<br />

2. Medical disorders<br />

a. Guillain-Barre syndrome<br />

(1) Miller-Fischer variant (more cranial nerve involvement)<br />

b. Connective tissue disorders<br />

c. Thiamine or vitamin B 12<br />

deficiency<br />

d. Hypothyroidism<br />

e. Lyme disease or tick paralysis<br />

f. Amyloidosis<br />

g. Malignancy<br />

h. Diabetes<br />

1. Leprosy<br />

j. Uremia<br />

B. Clinical presentation<br />

1. Most common<br />

a. Both motor and sensory signs are present.<br />

b. Weakness and sensory impairment are symmetrical but greater distally than proximally - stocking-glove<br />

distribution of sensory symptoms (pain, paresthesia, numbness)<br />

c. Reflexes are usually absent (Achilles reflex is first affected)<br />

427


NERVOUS SYSTEM DISORDERS<br />

2. Other<br />

a. A pure motor or sensory loss predominates or<br />

b. There is proximal (rather than distal) weakness, bu I bar symptoms, or asymmetry.<br />

C. Specific toxic neuropathies<br />

1. Diphtheria<br />

a. Clinical presentation<br />

2. Tetanus<br />

(1) An acutely i 11 patient with fever, tachycardia, and vague history.<br />

(2) Membranous pharyngitis that bleeds when the membrane is pulled back<br />

(3) Mononeuritis of the eyes (ptosis, strabismus, accommodation problems)<br />

(4) Motor involvement in other areas (limbs, palate [usually the first to become paralyzed], urinary, and<br />

anal sphincters) may be present; sensory involvement is rare.<br />

a. Clinical presentation<br />

(1) Symptoms ("4 T's"): trismus (most common), tetany, twitching, and tightness (tight back, arms, and<br />

face; a tight facial expression is called "risus sardonicus"); the patient is awake (consciousness is not<br />

impaired).<br />

(2) Signs of sympathetic nervous system hyperactivity (tachycardia, hypertension, hyperpyrexia, sweating)<br />

are generally present.<br />

b. Differential diagnosis<br />

(1) Dystonic reaction due to phenothiazine therapy<br />

(2) Poisoning with strychnine<br />

(3) Local trismus due to odontogenic infection<br />

c. Treatment<br />

(1) Human tetanus immune globulin, wound debridement, and antibiotics effective against anaerobes<br />

(eg, penicillin G, metronidazole, or a third-generation cephalosporin); clinical disease does not confer<br />

immunity, so give the first dose of tetanus toxoid vaccine series in the emergency department.<br />

(2) Sympathetic hyperactivity may be treated with labetalol.<br />

3. Thallium neuropathy _,, painful paresthesias, diffuse motor weakness, alopecia (1-2 weeks later)<br />

4. Arsenic neuropathy _,, polyneuritis (a late finding) with occupational exposure<br />

5. Lead neuropathy _,, peripheral motor neuropathy (chronic ingestion)<br />

6. Alcohol neuropathy (ethanol, methanol, ethylene glycol) _,, slowly progressive peripheral neuropathy that is<br />

predominantly sensory in a stocking-glove pattern<br />

D. Specific metabolic neuropathies<br />

1. Guillain-Barre syndrome<br />

a. The most common acute polyneuropathy, this syndrome is an ascending neuropathy preceded by a viral<br />

syndrome in 50% of cases.<br />

b. Clinical presentation: sensory abnormalities (especially paresthesias)....,. rapid, progressive, symmetrical<br />

weakness_,, paralysis with loss of deep tendon reflexes_,, respiratory failure<br />

c. Diagnostic evaluation: CSF reveals cytochemical dissociation (normal cells, t protein)<br />

d. Miller-Fisher variant: descending symmetrical neuropathy associated with cranial nerve abnormalities<br />

2. Diabetic neuropathy _,, paresthesias in stocking-glove pattern, poor sensation, muscle weakness<br />

3. Gout neuropathy _,, extremity neuropathy (particularly of the lumbar plexus)<br />

4. Porphyria neuropathy- polyneuropathy (similar to Gillain-Barre but associated with psychosis and abdominal pain)<br />

5. Tick paralysis....,. looks like Guillain-Barre syndrome; search for the tick, especially in hairy areas, or in children.<br />

428


NERVOUS SYSTEM DISORDERS<br />

VI. DISORDERS OF THE NEUROMUSCULAR JUNCTION<br />

A. Myasthenia gravis (most common disorder of neuromuscular transmission)<br />

1. Definition: an autoimmune disease that destroys acetylcholine receptors that leads to poor neurotransmission<br />

with subsequent proximal muscle weakness<br />

2. Classification<br />

a. Group 1: localized, nonprogressive disease with perhaps only ptosis or diplopia when fatigued; responds<br />

well to anticholinesterase therapy (neostigmine, pyridostigmine); excellent prognosis<br />

b. Group 2: generalized disease that involves more than one group of striated muscles, both skeletal and<br />

cranial in origin, but without sensory involvement; usually amenable to drug therapy; good prognosis<br />

c. Group 3: acute fulminant disease with severe bulbar manifestations (dysarthria, dysphagia, respiratory<br />

failure); poor response to drug therapy and poor prognosis<br />

d. Group 4: late severe disease that develops at least 2 years after the onset of symptoms in Groups 1 or 2;<br />

poor prognosis<br />

e. Group 5: muscle atrophy that begins 2:6 months after the onset of generalized disease (Group 2); variable<br />

prognosis<br />

3. Clinical presentation<br />

a. Muscle weakness that is exacerbated by activity, sleeplessness, or alcohol intake and is relieved by rest<br />

b. Ptosis, diplopia, and blurred vision are the most frequent initial symptoms and are the only manifestations<br />

of the disease in 20% of cases; the pupil is not involved.<br />

4. Diagnostic evaluation<br />

a. A double-blind edrophonium test produces transient subjective and objective improvement of symptoms by<br />

preventing rapid breakdown of acetylcholine at the myoneural junction.<br />

b. The electromyogram is diagnostic.<br />

c. Serologic testing for antibodies to acetylcholine receptors is useful when positive; however, a negative test<br />

does not exclude the disorder.<br />

5. Two forms of acute crisis in myasthenia gravis: myasthenic and cholinergic. Distinction between the two with<br />

an edrophonium test may be difficult and hazardous in the extremely weak patient with progressive respiratory<br />

insufficiency.<br />

a. Myasthenic crisis<br />

(1) Occurs in undiagnosed/untreated patients and those with acute exacerbation of their disease process<br />

(2) Due to a functional deficiency of acetylcholine<br />

(3) Can produce severe muscle weakness and respiratory compromise<br />

(4) Administration of edrophonium 1-2 mg IV (a test dose) followed by 5-8 mg IV - clinical improvement<br />

b. Cholinergic crisis<br />

(1) Occurs in patients being treated for myasthenia gravis who receive too much acetylcholinesterase<br />

inhibitor<br />

(2) Due to a functional excess of acetylcholine<br />

(3) Can produce severe muscle weakness and respiratory compromise.<br />

(4) Administration of edrophonium - increased muscle weakness. Note whether cardiopulmonary<br />

symptoms occur (hypotension, bradycardia, wheezing); if so, atropine 1 mg IV (repeated as needed)<br />

should be given but will not improve the signs of neuromuscular blockade.<br />

(5) Another clue is the presence of muscarinic effects: "SLUDGE" (also suggestive of cholinergic crisis)<br />

.S.alivation<br />

lacrimation<br />

l!rination<br />

Qiaphoresis<br />

_Gastrointestinal cramps<br />

J;mesis<br />

429


NERVOUS SYSTEM DISORDERS<br />

c. Schematic for differentiating the two "crises"<br />

Myasthenic crisis<br />

I<br />

Result of undertratment or no<br />

treatment<br />

I<br />

Cholinergic crisis<br />

I<br />

Result of overtreatment<br />

I<br />

Due to a functional<br />

Due to a functional excess of<br />

deficiency of acetylcholine<br />

acetylcholine<br />

I<br />

I<br />

Severe muscle weakness and respiratory compromise<br />

I<br />

Edrophonium 1-2 mg IV followed by 5-8 mg IV<br />

I<br />

I<br />

I<br />

I<br />

I<br />

I<br />

Clinical improvement<br />

Increased muscle<br />

weakness<br />

6. Differential diagnosis<br />

a. Lambert-Eaton syndrome<br />

b. Botulism<br />

c. Tick paralysis<br />

d. Familial periodic paralysis<br />

e. Sleep paralysis<br />

f. Amyotrophic lateral sclerosis<br />

g. Gui I lain-Barre syndrome<br />

h. Organophosphate poisoning<br />

B. Lambert-Eaton syndrome<br />

1. A disorder of neuromuscular transmission most often associated with small-cell cancer of the lung -- aching,<br />

weakness, and fatigability of pelvic girdle and thigh muscles<br />

2. Unlike in myasthenia gravis (which it closely resembles), cranial nerves are generally spared and grip strength<br />

increases with repeated stimulation.<br />

C. Botulism (food-borne)<br />

1. Etiology: ingestion of a preformed toxin from Clostridium botulinum growing in poorly prepared canned food<br />

2. Pathophysiology: the toxin prevents release of acetylcholine from nerve endings -- neurologic symptoms<br />

3. Clinical presentation<br />

a. Symptoms occur within 24-48 hours: the more severe the poisoning, the earlier the symptoms appear.<br />

b. The earliest and most common neurologic symptoms (90%) are blurred vision, diplopia (cranial nerves Ill<br />

and VI), and photophobia.<br />

c. Other symptoms that occur within the first 24 hours are dry mouth, dysphagia, and dysarthria.<br />

d. Some symptoms may occur at any time but usually appear early in severe cases -- ophthalmoplegia,<br />

ptosis, symmetric impairment of cranial nerves and a descending pattern of trunk and extremity weakness;<br />

mentation and sensation are normal.<br />

e. Respiratory failure is the usual cause of death and can occur within 6-8 hours of onset of symptoms.<br />

4. Infant botulism ("floppy baby") can occur with the ingestion of raw honey in children


NERVOUS SYSTEM DISORDERS<br />

VII. MYOPATHIES<br />

A. Differentiating myopathies from neuropathies<br />

Table 25: Characteristics of Myopathies and Neuropathies<br />

Myopathies<br />

Proximal weakness (getting up from a chair, climbing<br />

steps, reaching overhead)<br />

Sensory symptoms absent<br />

Deep tendon reflexes intact<br />

Abnormal laboratory tests:<br />

f WBC count<br />

f erythrocyte sedimentation rate<br />

f muscle enzymes<br />

Neuropathies<br />

Distal weakness with proximal progression<br />

Paresthesias, t sensation (stocking-glove distribution)<br />

t Deep tendon reflexes<br />

Normal laboratory tests<br />

B. Myopathic syndromes<br />

1. Polymyositis____.. acute inflammation__,, muscle pain and weakness__,. f CPI


NERVOUS SYSTEM DISORDERS<br />

2. Treatment<br />

B. Bell's palsy<br />

a. Long-term medical therapy is with carbamazepine.<br />

b. Dosage adjustments are required to achieve therapeutic effect, and the patient also needs to be monitored<br />

for signs of toxicity; therefore, referral for follow-up is necessary.<br />

1. Definition<br />

a. Unilateral facial nerve paralysis (cranial nerve VII) that involves the muscles of the forehead, cheek, and lips<br />

b. Onset of facial weakness is rapid (within 1-2 days) and is frequently preceded by an upper respiratory<br />

infection and 1-2 days of retroauricular pain.<br />

2. It is important to determine if the lesion is located near the origin of cranial nerve VII (brainstem) or somewhere<br />

in its peripheral distribution (face only) to determine if a structural lesion or Bell's palsy is present; significant<br />

clues of an upper motor neuron lesion are sparing of the muscles of the forehead and slower onset of facial<br />

weakness (usually weeks).<br />

3. Differential diagnosis<br />

a. Other palsies may mimic Bell's palsy.<br />

b. Ramsay-Hunt syndrome<br />

(1) Caused by herpes zoster (Bell's palsy may be caused by herpes simplex)<br />

(2) Characterized by typical zoster eruptions on the affected side of the face and neck; if you don't see<br />

them on the skin, look for them in the external auditory canal and/or the tympanic membrane.<br />

(3) Cranial nerve VII palsy due to Lyme disease may occur; if it does, it is bilateral (Bell's is unilateral).<br />

4. Bell's palsy is differentiated from a stroke by involvement of the forehead and lower face, the absence of<br />

aphasia, hemiparesis, and other focal neurologic deficits.<br />

5. Treatment<br />

a. High-dose steroids in a short burst followed by gradual tapering with lower doses are effective if given<br />

within 48 hours (some say may even be effective if


NERVOUS SYSTEM DISORDERS<br />

3. Examination reveals wasting and tenderness of the involved muscles; associated sensory loss is variable.<br />

4. Oculomotor palsy (cranial nerve Ill) is the most common cranial neuropathy; the pupil is usually spared (which<br />

differentiates this neuropathy from one caused by a tumor or aneurysm).<br />

D. Diabetic neuropathies<br />

1. Usually present in lower extremities more often than upper extremities<br />

2. Classically described as a decreased sensation in the "stocking glove" distribution<br />

3. Can also present as painful neuropathies in the lower extremity<br />

IX. MYELOPATHIES (DISORDERS OF THE SPINAL CORD)<br />

A. Syringomyelia<br />

1. Characterized by development of fluid-filled cavity within the spinal cord<br />

2. Dissociated sensory loss; loss of pain and temperature sensibility with preservation of light touch, vibration,<br />

and position; may be impaired in a 11 capelike 11<br />

distribution<br />

B. Multiple sclerosis<br />

1. A demyelinating disorder with myriad potential CNS symptoms, women affected more often than men<br />

2. Spinal cord involvement results in:<br />

a. Upper motor neuron signs (weakness, hyperreflexia)<br />

b. Sensory abnormalities<br />

c. Bladder/bowel dysfunction<br />

d. lntranuclear ophthalmoplegia is pathognomonic<br />

C. Transverse myelitis<br />

1. A postviral or toxic inflammation of the spinal cord (includes radiation effects that may occur during cancer<br />

therapy)<br />

2. Sensation, pain, and temperature sensation are diminished below the level of cord involvement. Vibration and<br />

position sense may also be diminished. Patients may have weakness or paralysis of lower extremities.<br />

D. Epidural mass lesion<br />

1. Any patient with back pain and a neurologic deficit should be evaluated for an epidural mass lesion. Look for<br />

any "red flags" in the history: fever, cancer, extremes of age, incontinence, and IV drug abuse, which should<br />

prompt a more detailed evaluation, including imaging.<br />

2. Clinical presentation<br />

a. Severe pain (may be the only presenting complaint initially)<br />

b. Radiating electrical sensations down the spine (Lhermitte sign)<br />

c. Progressive signs of cord compression, eg, cauda equina syndrome<br />

3. Nontraumatic acute presentations occur in patients with:<br />

a. Epidural hemorrhage (consider anticoagulants)<br />

b. Acute cervical disc syndromes<br />

c. Abscess formation (previous IV drug abuse should raise suspicion)<br />

d. Metastatic tumor<br />

E. Dorsal column disorders<br />

1. Selective injury to the dorsal columns (classically associated with syphilis, alcoholism, and vitamin B 12 deficiency).<br />

2. Results in loss of position sense, vibration, and light touch<br />

X. VOLKMANN ISCHEMIC PARALYSIS<br />

A. Pathophysiology: predisposing condition such as edema, circumferential burn, or tight cast (that results in a<br />

compartment syndrome) - ischemia of the nerves and muscles - paralysis with eventual contracture<br />

B. Clinical features of a compartment syndrome (all of which occur late)<br />

1. Pain out of proportion to the injury<br />

2. Increased pain with passive stretching of the ischemic muscle compartment<br />

3. Pallor, paresthesias, and pulselessness (associated findings)<br />

433


NERVOUS SYSTEM DISORDERS<br />

C. Diagnostic evaluation<br />

1. Diagnosis is confirmed by direct measurement of compartment pressures (which are increased).<br />

2. Normal compartment pressure is 0-8 mmHg.<br />

D. Treatment: fasciotomy of compartment will usually treat this complication.<br />

XI. VERTIGO<br />

A. Definition: the sensation of motion<br />

1. Subjective vertigo: The patient feels that he or she is moving.<br />

2. Objective vertigo: The patient feels that the environment is in motion.<br />

B. Types<br />

1. Central vertigo<br />

a. Due to lesions of the brainstem or cerebellum<br />

b. Accounts for ~10%-15% of cases and has a more ominous prognosis than peripheral vertigo<br />

c. Clinical presentation<br />

(1) Sudden onset of mild, continuous dysequilibrium that lasts > 1 minute<br />

(2) Nausea and vomiting<br />

(a) Mild -,, pontine lacunar infarct or vertebrobasilar TIA<br />

(b) Severe -,, acute cerebellar hemorrhage<br />

(3) Hearing loss and tinnitus are rare.<br />

(4) Spontaneous nystagmus is increased by visual fixation and may be bidirectional or vertical.<br />

(5) Positional nystagmus: no latency, does not fatigue or adapt, is multidirectional<br />

(6) Associated neurologic abnormalities may be present.<br />

(a) Severe ataxia, lateralizing dysmetria, or dysdiadochokinesis -,, cerebellar injury<br />

(b) Dysarthria, dysphagia, diplopia, Horner syndrome, motor weakness, scotomata, or blindness -,,<br />

brainstem injury<br />

d. Etiology<br />

(1) Brainstem ischemia or infarction<br />

(2) Cerebellar hemorrhage or infarction<br />

(3) Vertebrobasilar insufficiency<br />

(4) Basilar artery migraine<br />

(5) Cerebellar or brainstem tumors<br />

(6) Multiple sclerosis<br />

(7) Temporal lobe epilepsy (cortical vertigo)<br />

(8) Posttraumatic or postconcussive syndromes<br />

e. These patients are admitted for further evaluation and treatment.<br />

2. Peripheral vertigo<br />

a. Due to disorders of the external/middle/inner ear or cranial nerve VIII<br />

b. Accounts for ~85% of cases and is generally self-limited<br />

c. Clinical presentation<br />

(1) Sudden onset of intense, intermittent episodes of dysequilibrium associated with a spinning, whirling<br />

sensation (Benign positional vertigo lasts


NERVOUS SYSTEM DISORDERS<br />

(4) Vestibular neuronitis<br />

(5) Benign positional vertigo<br />

(6) labyrinthitis or labyrinthine concussion<br />

(7) Acoustic neuroma<br />

(8) Meniere disease (endolymphatic hydrops or cochlear hydrops)<br />

(a) Clinical presentation: Patient is usually 40-60 years old and presents with a history of vertiginous<br />

episodes accompanied by tinnitus, sensorineural deafness, and aural pressure. Associated nausea<br />

and vomiting are common. Episodes are abrupt in onset and last for minutes to hours.<br />

(b) Differential diagnosis<br />

i. Cerebellopontine angle tumor<br />

11. Acoustic neuroma<br />

• Vertigo may not be present because of CNS accommodation for impaired vestibular<br />

function.<br />

• Additional findings: headache, diplopia, decreased corneal sensitivity, facial weakness,<br />

positive radiologic findings, increased CSF protein<br />

iii. Tertiary syphilis<br />

e. Treatment: focus on cause<br />

(1) Remove inciting medication or treat infection.<br />

(2) Treat BPPV with otolith repositioning maneuvers (Epley). Medication will have limited benefits.<br />

(3) Outpatient with antihistamines or anticholinergics and antiemetics.<br />

(4) Benzodiazepines are effective for symptomatic relief in severe cases.<br />

XII. HEADACHE<br />

A. Screen for the diseases with highest morbidity and mortality.<br />

1. Infection (meningitis, encephalitis)<br />

2. Bleeding (ICH/SAH)<br />

3. Mass (tumor, abscess)<br />

4. Temporal arteritis<br />

5. Central venous thrombosis<br />

6. Glaucoma<br />

7. Carbon monoxide poisoning<br />

B. Pathophysiology<br />

1. Pain-sensitive areas of the cranium are skin, fat, muscles, nerves, the large arteries at the base of the brain, the<br />

dural arteries, the falx cerebri, and the great venous sinuses.<br />

2. The major intracranial structures (brain parenchyma, most of the dura, arachnoid, and pia) have no pain fibers.<br />

3. Headache results from tension, traction, inflammation, distention, and dilation of pain-sensitive structures.<br />

C. Types of headache<br />

1. Migraine<br />

a. Pathophysiology<br />

(1) Previously thought to be due to vasoconstriction followed by vasodilatation; recent technological<br />

advances in biochemistry and pharmacology suggest that this theory is no longer viable.<br />

(2) Current theories include neurogenic inflammation and abnormalities of the following:<br />

(a) Serotonergic transmission (5-hydroxytryptamine)<br />

(b) Trigeminovascular neuronal transmission<br />

(c) Vascular structures<br />

(3) Types of auras (represent manifestations of brain dysfunction)<br />

(a) Visual phenomena (most common), scintillating scotoma, or visual field defects<br />

(b) Motor disturbances, hemiparesis, ophthalmoplegia, aphasia<br />

(c) Sensory abnormalities, dysesthesia<br />

(d) Brainstem disturbances, vertigo, ataxia<br />

435


NERVOUS SYSTEM DISORDERS<br />

b. Factors that provoke or intensify an attack<br />

(1) Menstruation (most common bodily change)<br />

(2) Changes in body rhythm (sleep or food deprivation)<br />

(3) Physical activity<br />

(4) Certain foods (chocolate, hard cheese), beverages (red wine or other alcoholic beverages), and<br />

chemicals (caffeine, MSG, nitrites)<br />

(5) Contraceptive estrogens<br />

c. Clinical presentation<br />

(1) Migraine without aura (80% of cases)<br />

(a) The headache is unilateral (initially), pulsating, and usually severe enough to interfere with<br />

normal daily activities.<br />

(b) Associated findings: nausea and vomiting, photophobia, and phonophobia<br />

(c) A previous history of these attacks (at least five) is required to make the diagnosis.<br />

(2) Migraine with aura (20% of cases)<br />

(a) The headache usually follows the aura after a symptom-free period 4 minutes) or two symptoms that occur in<br />

succession<br />

iii. A single symptom that lasts


NERVOUS SYSTEM DISORDERS<br />

(2) Discharge therapy for patients who are having an exacerbation of cluster headache (if there are no<br />

contraindications)<br />

(a) Trial of prophylactic verapamil 80 mg three times per day, to be increased by follow-up<br />

physician, and<br />

(b) Tapering course of steroids over 1-2 weeks, which is the typical duration of a cluster headache<br />

3. Subarachnoid hemorrhage (SAH)<br />

a. Clinical presentation<br />

(1) An unusual headache of sudden onset that may have been present for several days; physical<br />

examination may be normal, or there may be subtle signs of meningeal inflammation (eg, low-grade<br />

fever or mild nuchal rigidity); there may also be slightly abnormal vital signs (t blood pressure +<br />

t pulse + t respiration = a cerebral event unless proven otherwise)<br />

(2) Sudden onset of a severe "exploding" headache frequently described by the patient as "the worst<br />

headache I've ever had" is the classic presentation. Associated nausea and vomiting are common,<br />

which may lure the unwary clinician into the "migraine mentality;" a clinical clue is that this<br />

headache is unprecedented, ie, unlike any headache the patient has had before, and it is focal.<br />

(3) Neurologic deficits are frequently mild or absent, but nonfocal signs may be present (drowsiness,<br />

confusion, personality changes, dizziness, diplopia, or blurred vision).<br />

(4) Sudden, severe headache followed by loss of consciousness----,, patient presents with hypertension,<br />

hyperventilation, and neurologic devastation; subhyaloid hemorrhage may be found on funduscopic<br />

examination.<br />

b. An SAH is most commonly due to bleeding from a saccular (berry) aneurysm (~75%) or AV malformation<br />

(~10%) but may occur with trauma, vasculitis, and other causes. History often differentiates the two, but<br />

occasionally, the hemorrhage precedes a traumatic incident (eg, the patient loses consciousness and then<br />

falls, hitting his or her head, or the patient loses consciousness while driving, loses control of the car, and<br />

injures himself or herself).<br />

c. Diagnostic evaluation<br />

(1) Brain CT without contrast is the procedure of choice for diagnosing SAH and should be done in any<br />

patient with a new onset of a severe or persistent headache. It has a sensitivity of 95% for detecting<br />

SAH.<br />

(a) If the CT is negative, a lumbar puncture should be performed because some patients with SAH<br />

have a normal CT scan. A yellow supernatant liquid (xanthochromia) that may not be detected<br />

for 12 hours, obtained by centrifuging a bloody CSF sample, can help distinguish SAH from a<br />

traumatic tap (no xanthochromia). (ACEP Clinical Policy: Headaches)<br />

(b) If the diagnosis is still in question, angiography (or repeat lumbar puncture) may be required.<br />

(2) ECG changes (large, broad, or symmetrically inverted T waves, U waves, prolongation of the QRS,<br />

prolonged QT interval) may occur in association with SAH and mislead you into pursuing a cardiac<br />

diagnosis; dysrhythmias may also result from SAH.<br />

(3) Blood glucose should be determined very early in the evaluation process to exclude hypoglycemia as<br />

a cause of an altered mental status.<br />

d. Treatment<br />

(1) Directed toward stabilizing the patient and preventing vasospasm and rebleeding<br />

(2) Obtain immediate neurosurgical consult.<br />

(3) Keep the patient in a quiet room with the lights dimmed.<br />

( 4) Elevate the head of the bed 30°.<br />

(5) Start an IV with normal saline or lactated Ringer's (to maintain normovolemia), administer oxygen,<br />

place patient on pulse oximeter, and monitor.<br />

(6) If systolic blood pressure is > 140 mm Hg, consider labetalol or nicardine.<br />

(7) Provide analgesics and antiemetics (eg, ondansetron) as needed for control of pain, nausea, and<br />

vomiting.<br />

(8) Prophylactic treatment with an anticonvulsant (eg, fosphenytoin) to prevent seizures, which can<br />

t ICP, is controversial.<br />

(9) Other therapeutic modalities should be decided in consultation with the neurosurgeon.<br />

(a) Nicardipine may reduce the risk and severity of vasospasm.<br />

(b) Antifibrinolytics (aminocaproic acid) may prevent rebleeding; their use is decreasing because of<br />

the associated ischemic complications and the trend toward earlier surgical intervention.<br />

(c) Mannitol or hypertonic saline should be started if patient is showing signs of t ICP.<br />

437


NERVOUS SYSTEM DISORDERS<br />

4. Temporal arteritis (cranial arteritis)<br />

a. Definition: a granulomatous inflammation of one or more branches of the external carotid artery<br />

b. Classic clinical scenario: The patient is usually a woman, often >50 years old, who presents with a severe<br />

throbbing, burning headache on one side. Symptoms can include jaw claudication. There is often a past<br />

history of similar headaches that are especially excruciating at night. Associated complaints may include<br />

malaise, night sweats, weight loss, blurred vision, and polymyalgia rheumatica. Physical examination<br />

often reveals one or more of the following:<br />

(1) A tender, warm, and frequently pulseless temporal artery that can sometimes be rolled between the<br />

fingers and skull<br />

(2) Increased burning and decreased throbbing with vessel compression<br />

(3) Decreased visual acuity on the affected side<br />

(4) If some of these findings are absent, be careful not to tag this as "migraine," because failure to<br />

diagnose and treat cranial arteritis can result in permanent neurologic loss, especially blindness.<br />

Strokes may also occur. The sedimentation rate, by the Westergren method, is characteristically<br />

increased (at least >50 mm/hr, often >100 mm/hr); ~25% of patients have only mild increases of the<br />

sedimentation rate; temporal artery biopsy (when positive) confirms the diagnosis.<br />

c. Management<br />

(1) Diagnosis is made by temporal artery biopsy.<br />

(2) Therapy should be started when the diagnosis is suspected; blindness is the most common sequela<br />

and may develop rapidly secondary to ischemic papillitis.<br />

(3) Emergent consult with a rheumatologist is necessary to determine disposition and steroid therapy<br />

regimen (usually with prednisone in large doses_,. 60-80 mg/day)<br />

(4) Urgent consult with an ophthalmologist/neurologist or rheumatologist is recommended; if unilateral<br />

blindness occurs, loss of vision in the other eye will occur in 1-20 days in 75% of cases.<br />

(5) NSAIDs may be administered for pain relief.<br />

5. Muscle tension headache: pain is constant and feels like a tight band around the head.<br />

6. Space-occupying lesions (brain abscess, tumor): headache is nonthrobbing, prolonged, and constant; onset is<br />

gradual, worse in the morning.<br />

7. Ocular headache (orbital and/or periorbital pain): may be due to glaucoma, iritis, or corneal abrasion.<br />

8. Idiopathic intracranial hypertension (pseudotumor cerebri): affects women more often than men; the patient<br />

is usually obese, and headache is usually accompanied by blurred vision and papilledema. CT is negative,<br />

MRI excludes dural sinus thrombosis, and lumbar puncture showing t ICP is both diagnostic and therapeutic.<br />

XIII. MENINGITIS<br />

A. Clinical presentation<br />

1. Acute (pyogenic) meningitis<br />

a. Rapid onset of symptoms (within 24 hours): fever, headache, photophobia, vomiting, confusion, and stiff<br />

neck<br />

b. Etiology: most commonly bacterial; incidence declining due to Haemophilus influenzae type Band<br />

7-valent pneumococcal conjugate vaccine<br />

c. Begin antibiotic therapy within 30 minutes of presentation, ie, do not wait for lumbar puncture to confirm<br />

diagnosis.<br />

2. Subacute (lymphocytic) meningitis<br />

a. Gradual onset of similar symptoms (1-7 days)<br />

b. Etiology: usually viral<br />

c. Begin appropriate therapy (usually supportive) within 2 hours of presentation.<br />

3. Chronic meningitis<br />

a. Insidious onset of symptoms (> 1 week)<br />

b. Etiology: TB, coccidiomycosis, and cryptococcus (most common in HIV patients)<br />

c. Begin appropriate therapy as soon as the diagnosis is made.<br />

4. Physical examination<br />

a. Infants (especially neonates) and the elderly typically lack the usual signs and symptoms; in the elderly, the<br />

only clue may be an altered level of consciousness.<br />

438


NERVOUS SYSTEM DISORDERS<br />

b. Look for papilledema, focal neurologic signs (especially ophthalmoplegia), rashes (including purpura and<br />

petechiae), and sources of infection.<br />

c. Brudzinski sign: passive flexion of the neck results in flexion of the hips and knees.<br />

d. Kernig sign: with the hips flexed, passive extension of the knee (> 120°) produces pain.<br />

B. Diagnostic evaluation<br />

1 . CT scan of the head<br />

2.<br />

a. Done first to exclude a mass lesion in patients who have a depressed mental status, seizure, head injury,<br />

focal neurologic signs, or papilledema, regardless of whether fever is present.<br />

b. Antibiotic therapy should not be delayed in patients with suspected bacterial meningitis; antibiotics should<br />

be administered before the CT scan.<br />

CSF<br />

a. Characteristics<br />

Table 26: Characteristics of CSF in Different Types of Infections<br />

Test Normal Bacterial Viral Fungal TB*<br />

Pressure 300 200<br />

Glucose >40 40


NERVOUS SYSTEM DISORDERS<br />

C. Etiology and antibiotic therapy<br />

Table 27: Etiology and Therapy of Meningitis in Different Age Groups<br />

Age<br />

0-4 weeks old<br />

4-12 weeks old<br />

3 months<br />

to 18 years old<br />

18-50 years old<br />

>50 years old and<br />

alcoholic patients<br />

Etiology<br />

E coli, Group B streptococci, Listeria<br />

monocytogenes, gram-negative bacilli<br />

The neonatal pathogens, Streptococcus<br />

pneumoniae, Neisseria meningitides, and<br />

Haemophilus influenzae<br />

S pneumoniae, N meningitides, and<br />

H influenzae (decreasing incidence since<br />

H influenzae B vaccine)<br />

S pneumoniae, N meningitidis (decreasing<br />

incidence since H influenzae B vaccine)<br />

S pneumoniae, L monocytogenes,<br />

N meningitidis, gram-negative bacilli<br />

Antibiotic Therapy*<br />

Cefotaxime and ampicillin, or<br />

gentamicin and ampicillin**<br />

Cefotaxime and ampicillin, or<br />

ceftriaxone and ampicillin**<br />

Ceftriaxone or cefotaxime<br />

Ceftriaxone<br />

Ceftriaxone and ampicillin**<br />

*If instituted early, reduces morbidity and sequelae; add vancomycin in any patient >30 days old when S pneumoniae resistance is<br />

possible.<br />

**If penicillin-allergic, use vancomycin ± rifampin.<br />

D. Steroid therapy<br />

1. The recommendations for steroids are controversial. When given simultaneously with (or before) the first dose<br />

of antibiotic, dexamethasone may reduce the incidence of sensorineural hearing loss associated with bacterial<br />

meningitis in children; it also decreases the risk of an unfavorable outcome in adults with bacterial meningitis.<br />

2. Indications<br />

a. Consider in children >6 weeks old with H inf/uenzae or 5 pneumoniae meningitis<br />

b. Adults with proven or suspected pneumococcal meningitis<br />

3. Dosage: 0.15 mg/kg IV<br />

XIV. HYDROCEPHALUS<br />

A. Definition: abnormality in formation, flow, or absorption of CSF that leads to excess accumulation of CSF in<br />

the ventricles of the brain<br />

B. Etiology<br />

1. Congenital malformations<br />

2. Mass effect<br />

3. Trauma<br />

4. SAH<br />

C. Clinical presentation: usually a bimodal distribution<br />

1. Children: due to congenital abnormalities; present with vomiting and developmental delay<br />

2. Adults: due to normal-pressure hydrocephalus<br />

D. Treatment<br />

a. Normal-pressure hydrocephalus is defined as a hydrocephalus that has normal opening pressure on lumbar<br />

puncture and normal funduscopic examination.<br />

b. Usually present with ataxia and urinary incontinence<br />

1. Children<br />

a. Immediate neurosurgical consultation for ventriculoperitoneal shunt placement<br />

b. ICP management (elevate head of bed), consider mannitol for temporizing measure<br />

2. Adults<br />

a. Decrease CSF production: acetazolamide or furosemide<br />

b. Repeated lumbar punctures: usually for SAH-induced hydrocephalus, because this may resolve spontaneously<br />

after the initial injury heals<br />

c. Surgical management: ventriculoperitoneal or ventriculo-atrial shunt<br />

440


NERVOUS SYSTEM DISORDERS: PRACTICE CLINICAL SCENARIOS<br />

NERVOUS DISORDERS: PRACTICE CLINICAL SCENARIOS<br />

Answers immediately follow practice clinical scenarios.<br />

Scenario A<br />

Presentation: A 22-year-old man presents with an acute onset of unilateral headache that awoke him from<br />

sleep and is occurring at night.<br />

What is the diagnosis?<br />

Scenario B<br />

Presentation: An 78-year-old woman complains of right-sided headache and pain in her jaw when eating.<br />

What is the diagnosis?<br />

Scenario C<br />

Presentation: A 2-week-old female infant presents with irritability, high fever, and vomiting. The<br />

fontanelle is full.<br />

What is the diagnosis?<br />

441


NERVOUS SYSTEM DISORDERS: PRACTICE CLINICAL SCENARIOS<br />

ANSWERS TO PRACTICE CLINICAL SCENARIOS<br />

Scenario A<br />

Diagnosis: cluster headache<br />

Diagnostic evaluation: If there are no associated neurologic deficits after a thorough neurologic examination, then<br />

no specific testing is necessary.<br />

Management: Oxygen, 100% via face mask, should help with symptoms. Oral carbamazepine should alleviate<br />

symptoms but not acutely. Facial nerve blocks may be considered for outpatients.<br />

Scenario B<br />

Diagnosis: temporal arteritis<br />

Diagnostic evaluation: Diagnosis relies heavily on the clinical scenario. In women with unilateral headaches<br />

and jaw claudication, suspicion for temporal arteritis should be high. The evaluation should include a thorough<br />

ophthalmologic examination and erythrocyte sedimentation rate.<br />

Management: Steroid therapy should be initiated in patients who have a diagnosis of temporal arteritis. Prednisone<br />

at 40-60 mg/kg per day should be sufficient and is customary. Good follow up with ophthalmology and<br />

rheumatology should be arranged. Admission is not necessary unless there are mitigating factors to discharge.<br />

Scenario C<br />

Diagnosis: bacterial meningitis<br />

Diagnostic evaluation: Neonates may present with fever and be inconsolable. In older patients, altered mental<br />

status may be the only symptom. A Gram stain of CSF obtained by lumbar puncture is the "gold standard."<br />

Performing a CT scan before lumbar puncture is not necessary if the patient has normal neurologic and<br />

funduscopic examinations. CT scan may identify alternative reasons for this presentation, such as intracranial<br />

abscess or SAH.<br />

Management: The patient should be treated with empiric antibiotics based on the most likely causative organism.<br />

Double coverage with vancomycin is warranted if there is suspicion of methicillin-resistant Staphylococcus aureus<br />

(MRSA). If clinical suspicion is high, treatment should begin before verification of diagnosis. Pretreating the patient<br />

with steroids is controversial, but it may help alleviate some of the long-term morbidity associated with the disease.<br />

Table 28: Etiology and Therapy of Meningitis in Different Age Groups<br />

Age<br />

0-4 weeks old<br />

4-12 weeks old<br />

3 months<br />

to 1 8 years old<br />

18-50 years old<br />

>50 years old and<br />

alcoholic patients<br />

Etiology<br />

E coli, Group B streptococci, Listeria<br />

monocytogenes, gram-negative bacilli<br />

The neonatal pathogens, Streptococcus<br />

pneumoniae, Neisseria meningitides, and<br />

Haemophilus influenzae<br />

S pneumoniae, N meningitides, and<br />

H influenzae (decreasing incidence since<br />

H influenzae B vaccine)<br />

S pneumoniae, N meningitidis (decreasing<br />

incidence since H influenzae B vaccine)<br />

S pneumoniae, L monocytogenes,<br />

N meningitidis, gram-negative bacilli<br />

Antibiotic Therapy*<br />

Cefotaxime and ampicillin, or<br />

gentamicin and ampicillin**<br />

Cefotaxime and ampicillin, or<br />

ceftriaxone and ampicillin**<br />

Ceftriaxone or cefotaxime<br />

Ceftriaxone<br />

Ceftriaxone and ampicillin**<br />

*If instituted early, reduces morbidity and sequelae; add vancomycin in any patient >30 days old when 5 pneumoniae resistance is<br />

possible.<br />

**If penicillin-allergic, use vancomycin ± rifampin.<br />

442


GYNECOLOGIC AND OBSTETRIC DISORDERS<br />

GYNECOLOGIC AND OBSTETRIC DISORDERS<br />

Bartholin Gland Duct Abscess .................................................................................................................................. 448<br />

Genital Ulcer Infections ............................................................................................................................................ 448<br />

Herpes Simplex Virus ......................................................................................................................................... 448<br />

Syphilis .............................................................................................................................................................. 449<br />

Chancroid .......................................................................................................................................................... 450<br />

Lymphogranuloma Venereum ............................................................................................................................. 451<br />

Genital Mass Lesions ................................................................................................................................................. 451<br />

Human Papillomavirus ....................................................................................................................................... 451<br />

Noninfectious Vaginitis ............................................................................................................................................. 452<br />

Bacterial Vaginosis ............................................................................................................................................. 452<br />

Foreign Body ...................................................................................................................................................... 452<br />

Infectious Vaginitis .................................................................................................................................................... 452<br />

Fungal Infections ................................................................................................................................................ 452<br />

Trichomoniasis ................................................................................................................................................... 453<br />

Cervicitis/Endocervicitis ........................................................................................................................................... 453<br />

Neisseria gonorrhoeae ........................................................................................................................................ 453<br />

Chlamydia trachomatis ....................................................................................................................................... 454<br />

Pelvic Inflammatory Disease ..................................................................................................................................... 455<br />

Ovary ........................................................................................................................................................................ 456<br />

Torsion ............................................................................................................................................................... 456<br />

Ovarian Cystic Masses ....................................................................................................................................... 457<br />

Uterus ....................................................................................................................................................................... 457<br />

Vaginal Bleeding ................................................................................................................................................ 457<br />

Cervical Bleeding ............................................................................................................................................... 458<br />

Endometriosis ..................................................................................................................................................... 458<br />

Pelvic Organ Prolapse ........................................................................................................................................ 458<br />

Tumors ............................................................................................................................................................... 459<br />

Emergency Contraception ......................................................................................................................................... 459<br />

Sexual Assault ........................................................................................................................................................... 459<br />

Normal Pregnancy .................................................................................................................................................... 460<br />

Drugs and Radiation Exposure in Pregnancy ............................................................................................................. 462<br />

Complications of Pregnancy ...................................................................................................................................... 462<br />

First Trimester Bleeding ....................................................................................................................................... 462<br />

Abortion ............................................................................................................................................................. 463<br />

Ectopic Pregnancy .............................................................................................................................................. 464<br />

Rh lsoimmunization ........................................................................................................................................... 466<br />

Infections in Pregnancy ...................................................................................................................................... 466<br />

Hyperemesis Gravidarum/Nausea and Vomiting of Pregnancy ............................................................................ 467<br />

Gestational Diabetes .......................................................................................................................................... 468<br />

Chronic and Gestational Hypertension ............................................................................................................... 468<br />

HELLP Syndrome ................................................................................................................................................ 468<br />

Preeclampsia ...................................................................................................................................................... 468<br />

Eclampsia ........................................................................................................................................................... 469<br />

Hemorrhage, Antepartum ................................................................................................................................... 469<br />

Abruptio Placentae ...................................................................................................................................... 469<br />

Placenta Previa ........................................................................................................................................... 470<br />

Thromboembolism ............................................................................................................................................. 470<br />

Aortic Dissection ................................................................................................................................................ 470<br />

Maternal Cardiac Disease ................................................................................................................................... 471<br />

443


GYNECOLOGIC AND OBSTETRIC DISORDERS<br />

Cardiac Arrest .................................................................................................................................................... 471<br />

Peri mortem Cesarean Section ............................................................................................................................. 471<br />

High-Risk Pregnancy ................................................................................................................................................. 471<br />

Assisted Reproductive Therapies ......................................................................................................................... 471<br />

Normal Labor and Delivery ...................................................................................................................................... 472<br />

Complications of Labor ............................................................................................................................................. 472<br />

Fetal Distress ...................................................................................................................................................... 472<br />

Premature Labor ................................................................................................................................................. 472<br />

Premature Rupture of Membranes ...................................................................................................................... 472<br />

Rupture of Uterus ............................................................................................................................................... 473<br />

Complications of Delivery ......................................................................................................................................... 473<br />

Mal position of Fetus ........................................................................................................................................... 473<br />

Nuchal Cord ...................................................................................................................................................... 473<br />

Prolapse of Cord ................................................................................................................................................. 47 4<br />

Postpartum Complications ........................................................................................................................................ 474<br />

Postpartum Fever ................................................................................................................................................ 47 4<br />

Postpartum Hemorrhage ..................................................................................................................................... 475<br />

Uterine Inversion ................................................................................................................................................ 475<br />

Pituitary Infarction .............................................................................................................................................. 475<br />

444


GYNECOLOGIC AND OBSTETRIC DISORDERS: SELF-ASSESSMENT QUESTIONS<br />

GYNECOLOGIC AND OBSTETRIC DISORDERS:<br />

SELF-ASSESSMENT QUESTIONS<br />

1. Which of the fol lowing statements 1·egarding syphilis, chancroid, lymphogranuloma venereum, and genital herpes<br />

simplex is most accurate?<br />

(a) Treatment of these sexually transmitted infections should be withheld until laboratory confirmation has<br />

been obtained.<br />

(b) Doxycycline is the recommended drug of choice for each of these infections.<br />

(c) Patients with these infections commonly complain of a vaginal or penile discharge.<br />

(d) These infections are characterized by genital ulcers and/or inguinal lymphadenopathy.<br />

2. A female patient presents with the complaint of an itchy, malodorous vaginal discharge that is yellow-gray in color.<br />

On examination, her vaginal mucosa has a stippled appearance, and the vaginal pH is 6.0. Wet mount reveals<br />

motile pear-shaped organisms with flagella at one end. The patient tells you she is 5 weeks pregnant. What is the<br />

most appropriate treatment for this condition?<br />

(a)<br />

Doxycline 100 mg orally bid x 14 days<br />

(b) Fluconazole 150 mg orally once<br />

(c) Metronidazole 500 mg orally bid x 7 days<br />

(d) Metronidazole gel 0.75% vaginally x 5 days<br />

3. A 26-year-old woman presents with painful vesicles on an erythematous base on her vulva, which are also seen on<br />

the cervix during a speculum examination. Which of the following is an indication for intravenous treatment and<br />

hospitalization?<br />

(a)<br />

lmmunocompromised patient with severe infection<br />

(b) Initial episode<br />

(c) Pregnancy<br />

(d) Recurrent episode<br />

4. All of the following are long-term complications associated with pelvic inflammatory disease except:<br />

(a) Chronic pelvic pain<br />

(b) Ectopic pregnancy<br />

(c) Infertility<br />

(d) Pelvic organ prolapse<br />

5. Which of the fol lowing signs is most consistent with a diagnosis of tuba-ovarian torsion?<br />

(a) Bilateral adnexal tenderness<br />

(b) Coiled vessels, or a "whirlpool" sign on ultrasound<br />

(c) Free fluid within the pelvis on CT imaging<br />

(d) Vaginal bleeding<br />

6. Which of the following is correct regarding labor and delivery?<br />

(a)<br />

Braxton Hicks contractions turn into true labor.<br />

(b) The stages of labor are initial, true, and final.<br />

(c) The third stage of labor includes delivery of the placenta.<br />

(d) Cervical change occurs during the second stage of labor.<br />

7. All of the following statements regarding vulvovaginal candidiasis are accurate except:<br />

(a) Candida albicans is part of the normal vaginal flora in up to 20% of women.<br />

(b) Causes of colonization or overgrowth include diabetes mellitus, pregnancy, menstruation, and birth control pills.<br />

(c) Diagnosis is with a potassium hydroxide prep that reveals "clue cells."<br />

(d) Treatment of partners is generally unnecessary, because this is not a sexually transmitted infection.<br />

445


GYNECOLOGIC AND OBSTETRIC DISORDERS: SELF-ASSESSMENT QUESTIONS<br />

8. A young, sexually active woman presents with a history of right lower quadrant pain for the past several hours. Her<br />

last menstrual period was 6 weeks ago, but she says that is not unusual for her. Her vital signs are stable, but she<br />

looks visibly uncomfortable. On abdominal examination, you note tenderness confined to the right lower quadrant,<br />

and pelvic examination reveals right adnexal tenderness and fullness. Which of the following possible diagnoses is<br />

least Ii kely?<br />

(a)<br />

Ectopic pregnancy<br />

(b) Pelvic inflammatory disease<br />

(c) Hemorrhagic corpus luteum cyst<br />

(d) Adnexal torsion<br />

9. In the patient discussed in question #8, which of the following laboratory studies would be least likely to help you<br />

in narrowing the differential and establishing a definitive diagnosis?<br />

(a)<br />

~-human chorionic gonadotropin<br />

(b) CBC<br />

(c) Pelvic ultrasound<br />

(d) Wet prep and nucleic acid amplification tests<br />

10. The incidence of transmission of rubella from an infected mother to her fetus is greatest when maternal infection<br />

occurs:<br />

(a)<br />

In the first month of pregnancy<br />

(b) In the second month of pregnancy<br />

(c) In the third month of pregnancy<br />

(d) Is the same in the first 3 months of pregnancy<br />

11. The drugs considered to be safest for administration during pregnancy are those classified by the FDA as:<br />

(a) Category A drugs<br />

(b) Category C drugs<br />

(c) Category D drugs<br />

(d) Category X drugs<br />

12. A visibly pregnant woman presents to the emergency department in cardiac arrest. Correct management of this<br />

patient includes:<br />

(a) Continue CPR on the backboard, intubate, use ACLS drugs and protocols, place an internal jugular line,<br />

perimortem cesarean section after 5 minutes.<br />

(b) Continue CPR with manual displacement of the uterus, intubate, use ACLS drugs and protocols, place a<br />

subclavian or internal jugular line, perimortem cesarean section before 5 minutes.<br />

(c) Continue CPR with manual displacement of the uterus on the backboard, intubate, avoid ACLS drugs, place a<br />

femoral line, perimortem cesarean section after 5 minutes.<br />

(d) Continue CPR with the backboard at a 30° tilt, intubate, use ACLS drugs and protocols, place a femoral line,<br />

perimortem cesarean section after 5 minutes.<br />

13. Which of the following is not considered to be a risk factor for abruptio placenta?<br />

(a) Hypertension<br />

(b) Abdominal trauma<br />

(c) Primiparity<br />

(d) Smoking<br />

14. A patient is brought in by ambulance for evaluation. The paramedics state that she had been out shopping with a<br />

friend and had a grand mal seizure. A fingerstick done in the field showed a glucose level of 120 mg/dL. There is no<br />

known history of a prior seizure disorder. On evaluation, she looks to be about 17 years old and at least 8 months<br />

pregnant. She is lethargic and hyperreflexic (+¾)and has a blood pressure of 170/100 mm Hg. The most appropriate<br />

initial medication is:<br />

(a)<br />

Hydralazine<br />

(b) Diazepam<br />

(c) Furosemide<br />

(d) Magnesium sulfate<br />

446


GYNECOLOGIC AND OBSTETRIC DISORDERS: SELF-ASSESSMENT QUESTIONS<br />

15. Signs of magnesium toxicity include all of the following except:<br />

(a) Loss of deep tendon reflexes<br />

(b) Respiratory depression<br />

(c) Flushing<br />

(d) Bradydysrhythmias<br />

16. Appropriate initial evaluation of a woman with vaginal bleeding in the third trimester includes all of the<br />

following except:<br />

(a)<br />

Evaluation of the fetal heart tones<br />

(b) Immediate pelvic examination<br />

(c) CBC and type and Rh<br />

(d) Ultrasound<br />

17. A woman who is 34 weeks pregnant presents with right upper quadrant pain of several hours duration. On<br />

examination, she has a temperature of 100.4°F (38°C), a heart rate of 102 beats per minute, blood pressure of<br />

120/68 mm Hg, and a respiratory rate of 18 breaths per minute. She has diffuse right upper quadrant tenderness.<br />

CBC reveals a WBC count of 17,000/mm 3 • Alkaline phosphatase, platelets, and liver function tests are normal, and<br />

urinalysis reveals pyuria without bacteria. The most likely diagnosis is:<br />

(a) Ascending cholangitis<br />

(b) Salpingitis<br />

(c) Appendicitis<br />

(d) Preeclampsia<br />

18. A woman who is 4 days postpartum presents with vaginal bleeding. Examination reveals a temperature of 101 °F<br />

(38.3°C), a purulent discharge, and uterine tenderness. The most likely cause of the postpartum bleeding is:<br />

(a) Retained placental tissue<br />

(b) Endometritis<br />

(c) Uterine atony<br />

(d) Genital tract trauma sustained during delivery<br />

1 9. Al I of the fol lowing statements regarding the Klei hauer-Betke test are accurate except:<br />

(a) It can be used to detect and quantify fetomaternal hemorrhage.<br />

(b) It is useful in determining the amount of Rh immune globulin administered to an Rh-negative woman who has<br />

sustained fetomaternal hemorrhage.<br />

(c) Its use is generally limited to women who are at least 12 weeks pregnant.<br />

(d) A negative test means that no fetomaternal hemorrhage has occurred.<br />

ANSWERS<br />

1. d<br />

2. C<br />

6.<br />

7.<br />

C<br />

C<br />

11 .<br />

12.<br />

a<br />

b<br />

16. b<br />

17. C<br />

3. a 8. b 13. c 18. b<br />

4. d 9. b 14. d 19. d<br />

5. b 10. a 15. c<br />

Use the pre-chapter multiple choice question worksheet (page xvi) to record and determine the percentage of correct<br />

answers for this chapter.<br />

447


GYNECOLOGIC AND OBSTETRIC DISORDERS<br />

I. BARTHOLIN GLAND DUCT ABSCESS<br />

A. Etiology: Barthol in gland or duct cyst can become infected with various organisms (Escherichia coli,<br />

Staphylococcus aureus, mixed anaerobes, and occasionally Neisseria gonorrhoeae or Chlamydia)<br />

B. Clinical presentation<br />

1. Painful unilateral swelling of the posterior-lateral aspect of the vaginal opening<br />

2. Pain is increased with sitting and walking; associated swelling and erythema of the labia majora is common.<br />

C. Management<br />

1. Incision and drainage (on the mucosa! aspect of the vestibule) followed by sitz baths<br />

2. Antibiotics if cellulitis is also present<br />

3. Word catheter placement for 6-8 weeks (to facilitate healing and reepithelialization)<br />

4. Elective marsupialization<br />

II. GENITAL ULCER INFECTIONS<br />

A. Herpes simplex virus<br />

1. Etiology<br />

a. Most prevalent genital ulcer disease<br />

b. Two types: HSV-1 usually causes oral lesions, HSV-2 typically causes genital lesions.<br />

c. Transmission is generally by sexual contact with an infected individual; it may occur in the absence of<br />

visible lesions if viral shedding is present.<br />

d. Incubation period is 8-16 days.<br />

2. Clinical presentation<br />

a. Recurring, painful genital lesions appear as clusters of vesicles on an erythematous base, which<br />

subsequently denude to form ulcers.<br />

b. The lesions may be found at any site, but the most commonly affected areas are:<br />

(1) The cervix and vulva in women (swelling of the vulva may be so severe that the patient may not be<br />

able to void urine)<br />

(2) The glans and prepuce in men<br />

c. Attacks are often heralded by a prodrome of pain, hyperesthesia, burning, or paraesthesia at the skin site.<br />

d. Headache, fever, arthralgias, and regional adenopathy may also be present, particularly with the first<br />

clinical episode.<br />

e. Newborns who acquire the virus at birth may develop devastating complications.<br />

3. Diagnostic evaluation<br />

a. The clinical findings are sufficient to initiate treatment. Do not delay treatment for testing.<br />

b. Laboratory diagnosis<br />

(1) The gold standard is tissue viral culture. Specificity is high, although sensitivity declines as lesions heal.<br />

(2) PCR testing and serologic type-specific antibody testing are also available and more sensitive than<br />

culture.<br />

c. The Tzanck smear used in the past for diagnosis of herpes simplex virus is no longer recommended because<br />

of its lack of sensitivity.<br />

4. Management<br />

a. Acyclovir (oral or IV) remains the antiviral agent of choice.<br />

(1) Accelerates healing<br />

(2) Shortens the duration of viral shedding<br />

(3) Provides partial control of symptoms<br />

(4) Topical acyclovir is not recommended.<br />

(5) Famciclovir and valacyclovir are acceptable alternatives.<br />

b. Indications for treatment with oral acyclovir<br />

(1) Patients with primary lesions<br />

(2) Patients with associated herpes cervicitis, urethritis, proctitis, stomatitis, or pharyngitis<br />

448


GYNECOLOGIC AND OBSTETRIC DISORDERS<br />

(3) Patients with recurrent genital herpes<br />

(4) Episodic infections in HIV-positive patients<br />

c. Indications for hospital admission and treatment with IV acyclovir<br />

(1) Patients with severe genital infections<br />

(2) lmmunocompromised patients with severe infections/disseminated disease<br />

(3) Patients who cannot tolerate or do not respond to oral medication<br />

(4) Neonatal herpes<br />

d. Prescribe analgesics as needed.<br />

e. If an associated cellulitis is present, prescribe antistaphylococcal antibiotics.<br />

f. Advise patients to avoid sexual contact when lesions are present.<br />

5. Complications<br />

B. Syphilis<br />

a. Herpes simplex virus has been epidemiologically associated with both cervical cancer and cancer of the vulva.<br />

b. Genital herpes has been associated with an increased risk of acquiring HIV and other sexually transmitted<br />

infections.<br />

c. Cesarean section is indicated in term-pregnant patients with active genital herpes because of the high<br />

neonatal mortality rate (50%-80%) and morbidity associated with contraction of neonatal herpes during<br />

vaginal delivery.<br />

1. Etiology<br />

a. Caused by a spirochete (Treponema pallidum) that inhabits skin lesions, mucous membranes, saliva,<br />

semen, and blood.<br />

b. Transmission occurs primarily by direct contact with an infected lesion (usually genital). Syphilis may also<br />

be acquired congenitally or by a blood transfusion (if the donor was in a very early stage of the disease).<br />

2. Clinical phases and diagnostic evaluation<br />

a. Primary syphilis<br />

(1) A smal I papule develops at the site of inoculation and becomes a painless, indurated ulcerative<br />

chancre. Chancres develop after an incubation period of 10-90 days, are present for 3-6 weeks, and<br />

resolve spontaneously.<br />

(a) Scrapings obtained from the chancre and examined under darkfield microscopy reveal moving<br />

corkscrew-like treponemes.<br />

(b) Serologic tests (VDRL or rapid plasma reagin) are often negative early in the primary stage.<br />

(2) Inguinal lymphadenopathy (buboes) may be present. Nodes are enlarged, nontender, firm, and<br />

rubbery; they develop ~4 weeks after initial exposure.<br />

b. Secondary syphilis<br />

(1) Characterized by constitutional signs/symptoms and a rash<br />

(2) Rash and lymphadenopathy are the most common symptoms.<br />

(3) The rash of secondary syphilis emerges 4-8 weeks after the initial appearance of the chancre and<br />

lasts for several months. It may be difficult to distinguish from the rash of Rocky Mountain spotted<br />

fever, because both are seen on the palms and soles. Definitive differentiation is with serologic testing<br />

(screen with VDRL or rapid plasma reagin, followed by FT A-ABS if positive).<br />

(a) Erythematous (red) macules, papules, or plaques (condylomata lata) suggest syphilis. The classic<br />

rash is maculopapular, nonpruritic, and symmetric, and the palms and soles are commonly involved.<br />

(b) A fine pink (not red) rash interspersed with macules suggests Rocky Mountain spotted fever.<br />

c. Tertiary syphilis<br />

(1) Onset occurs after a latent (dormant) period of several years.<br />

(2) Characterized by development of cardiovascular and/or neurologic disorders such as thoracic<br />

aneurysm, dementia, and tabes dorsalis (syphilitic myelopathy)<br />

(3) Serology tests are positive.<br />

d. Neurosyphilis and ocular syphilis can occur at any stage and should be considered in the differential<br />

diagnosis of HIV-positive patients with neurologic or eye complaints.<br />

3. Management<br />

a. Exposed patients should be evaluated clinically with treatment initiated on a presumptive basis while<br />

serologic testing is pending.<br />

b. All patients who have syphilis should be tested for HIV infection.<br />

449


GYNECOLOGIC AND OBSTETRIC DISORDERS<br />

C. Chancroid<br />

c. Sexual partners should be notified and treated, which occurs at the state level.<br />

d. Primary, secondary, and early latent (


GYNECOLOGIC AND OBSTETRIC DISORDERS<br />

D. Lymphogranuloma venereum<br />

1. Etiology and epidemiology<br />

a. Uncommon sexually transmitted infection caused by Chlamydia trachomatis<br />

b. Usually seen in men 20-40 years old but can be seen in women as well<br />

2. Clinical presentation<br />

a. A small, shallow, painless vesicle or ulcer appears 3-21 days after exposure. It usually goes unnoticed by<br />

the patient.<br />

b. Localized, tender inguinal (and/or femoral) lymphadenopathy that is unilateral develops 2-24 weeks after<br />

exposure.<br />

c. When the nodes enlarge, coalesce, and ulcerate, constitutional symptoms may develop.<br />

d. The "groove sign" may appear and is characterized by proliferation of inguinal lymphadenopathy above<br />

and below the inguinal ligament.<br />

3. Diagnostic evaluation<br />

a. The lymphogranuloma venereum complement fixation test (which has replaced the Frei skin test) is<br />

diagnostic but not available in many hospital laboratories.<br />

b. Culture is also diagnostic, but many laboratories do not have this capability.<br />

c. The diagnosis is usually made by identification of WBCs with intracellular inclusion bodies from aspirates<br />

of infected tissue.<br />

4. Management<br />

a. Treatment should be started as soon as the diagnosis is suspected, without waiting for laboratory<br />

confirmation.<br />

b. Doxycycline 100 mg orally bid x 3 weeks is the treatment of choice but should be avoided in pregnancy,<br />

while nursing, and in children 40 serotypes.<br />

b. Linked to development of genital warts as well as cervical cancer<br />

2. Clinical presentation<br />

a. Frequently an asymptomatic infection, although clinical presentation depends on the serotype and location<br />

of infection.<br />

b. The virus may infect the oropharynx, genital, or anal tissue.<br />

c. Genital warts are usually flat papules or pedunculated and skin colored.<br />

3. Diagnostic evaluation<br />

a. Diagnostic evaluation should include genital examination in the emergency department for lesions.<br />

Emergency concerns are those of large, obstructing lesions.<br />

b. Serotyping is not required for routine diagnosis.<br />

c. Biopsy is not required unless coexisting neoplasia suspected.<br />

4. Management and prevention<br />

a. External lesions<br />

(1) lmiquimod 3.75% or 5% cream topical or<br />

(2) Podofilox 0.5% solution or gel or<br />

(3) Sinecatechin 15% ointment<br />

(4) Alternatively, the patient may be referred to a specialist for cryotherapy, trichloracidic acid, or surgical<br />

removal.<br />

b. Appropriate cancer screening and follow-up are recommended.<br />

c. Vaccination is recommended for children 11-12 years old to prevent development of cancer. (Vaccination<br />

is not typically done in the emergency department.)<br />

451


GYNECOLOGIC AND OBSTETRIC DISORDERS<br />

IV. NONINFECTIOUS VAGINITIS<br />

A. Bacterial vaginosis<br />

1. Pathophysiology: results from overgrowth of anaerobic vaginal flora (Cardnerella vagina/is, Ureaplasma<br />

urealyticum, etc)<br />

2. Clinical presentation: patients present with complaints of a fishy smelling discharge and mild itching.<br />

3. Diagnostic criteria<br />

a. Homogeneous, grey-white, noninflammatory discharge that coats the vaginal wall rather than pooling on<br />

the vaginal floor<br />

b. "Clue cells" (vaginal epithelial cells covered with bacteria) on microscopic evaluation of vaginal secretion<br />

saline preparation (aka wet mount)<br />

c. Positive amine odor "whiff" test (fishy odor when potassium hydroxide is added to a sample of the vaginal<br />

secretions)<br />

d. pH of vaginal secretions >4.5<br />

e. DNA probe-based test available for point-of-care testing<br />

4. Management<br />

a. Metronidazole 500 mg orally bid x 7 days (Note: Metronidazole can produce a disulfiram ["Antabuse"]<br />

reaction if taken in conjunction with alcohol. Therefore, patients must be advised to avoid ingesting alcohol<br />

while taking metronidazole and for at least 24 hours after completion of therapy.)<br />

b. Metronidazole gel 0.75% intravaginally 5 glday x 5 days or<br />

c. Clindamycin cream 2% intravaginally 5 g at bedtime x 7 days or<br />

d. Other alternatives: oral clindamycin, clindamycin ovules, oral tinidazole<br />

e. For symptomatic pregnant women, oral regimens are preferred.<br />

B. Foreign body<br />

1. Children may insert any object; adults may forget tampons, pessaries, diaphragms, or condoms.<br />

2. Clinical presentation: a foul-smelling and sometimes bloody discharge<br />

3. Management<br />

a. Remove the object.<br />

b. Treat for any vaginitis and arrange for follow-up.<br />

c. Consider toxic shock syndrome if accompanied by fever, rash, etc.<br />

4. Consider the possibility of sexual abuse when vaginal foreign bodies are found in prepubertal girls.<br />

V. INFECTIOUS VAGINITIS<br />

A. Fungal infections<br />

1 . Pathophysiology<br />

a. Candida species, most commonly C albicans<br />

b. Normal vaginal flora in up to 20% of healthy women<br />

c. Symptoms due to overgrowth<br />

2. Clinical presentation<br />

a. Extreme itching<br />

b. A thin watery to thick white discharge sometimes associated with dysuria or dyspareunia<br />

c. Physical examination: thick, white "cottage cheese" discharge, vulvovaginal erythema, and edema; may<br />

have satellite lesions on perineum<br />

3. Diagnostic evaluation<br />

a. Pseudohyphae and spores are seen on 10% potassium hydroxide preparation.<br />

b. pH of vaginal secretions is normal (::C:4.5).<br />

c. Latex agglutination testing is available for point-of-care testing.<br />

4. Classification of vulvovaginal candidiasis<br />

a. Uncomplicated: 90% of cases<br />

(1) Sporadic/infrequent<br />

452


GYNECOLOGIC AND OBSTETRIC DISORDERS<br />

(2) Mild to moderate<br />

(3) Likely to be C albicans<br />

(4) Nonimmunocompromised patients<br />

b. Complicated<br />

(1) Recurrent or severe (2:4 episodes per year)<br />

(2) Candidiasis not caused by C albicans<br />

(3) Pregnancy<br />

(4) Debilitated, diabetic, or immunocompromised patients<br />

5. Management<br />

a. Uncomplicated vulvovaginal candidiasis<br />

(1) Fluconazole 150 mg orally in one dose or<br />

(2) One of the following vaginal preparations every night x 3 days:<br />

(a) Butoconazole 2% cream or<br />

(b) Clotrimazole two (100 mg) vaginal tablets or<br />

(c) Miconazole 200 mg vaginal suppository or<br />

(d) Terconazole 80 mg suppository or 0.8% cream<br />

b. Complicated vulvovaginal candidiasis: longer course of treatment with oral fluconazole or intravaginal<br />

preparations<br />

B. Trichomoniasis<br />

1. Etiology: Trichomonas vagina/is, a flagellated protozoan<br />

2. Clinical presentation<br />

a. Patients present with vulvovaginal itching and a foul-smelling discharge.<br />

b. Dysuria and lower abdominal pain may also be present.<br />

c. Symptoms usually develop after an incubation period of 4-28 days.<br />

d. Profuse, occasionally foamy, yellow-green discharge associated with diffuse vaginal erythema and a<br />

"strawberry cervix and vagina" (punctate hemorrhages)<br />

3. Diagnostic evaluation<br />

a. Wet mount demonstrates a flagellated, motile, tear-drop-shaped organism and many PMNs; sensitivity is<br />

60%-70%.<br />

b. Polymerase chain reaction testing is available for point-of-care testing and has higher sensitivity and<br />

specificity than wet mount.<br />

c. Vaginal pH 2:5.5<br />

4. Management<br />

a. Partner must also be treated.<br />

b. Recommended regimen (including for pregnant patients): metronidazole 2 g orally in one dose or<br />

tinidazole 2 g orally in one dose<br />

c. Alternative regimen: metronidazole 500 mg orally bid x 7 days (this regimen preferred for pregnant<br />

women)<br />

d. 90% of men with this infection are asymptomatic; therefore, sexual partners must be treated if reinfection is<br />

to be prevented.<br />

VI. CERVICITIS/ENDOCERVICITIS<br />

A. Neisseria gonorrhoeae<br />

1. Clinical presentation<br />

a. The presenting complaint is frequently dysuria, a vaginal discharge, or both. The presence of abdominal<br />

pain should raise suspicion that the infection has spread to the uterus and fallopian tubes (pelvic<br />

inflammatory disease).<br />

b. Pharyngeal or anal infection may occur. Although generally asymptomatic, patients with anal infection<br />

may complain of discharge, rectal pain, tenesmus, and/or constipation. Patients with pharyngeal infection<br />

are rarely symptomatic.<br />

c. Gonococcal conjunctivitis is usually the result of inoculation of the eye by a contaminated finger. It<br />

presents with a unilateral copious purulent discharge and marked conjunctiva! injection.<br />

453


GYNECOLOGIC AND OBSTETRIC DISORDERS<br />

d. Hematogenous dissemination may produce any of the following complications and may present with<br />

a characteristic rash (erythematous macules or pustules with a necrotic or purpuric center) and/or<br />

petechiae.<br />

(1) Joint disease<br />

(a) Most common cause of septic arthritis in patients


GYNECOLOGIC AND OBSTETRIC DISORDERS<br />

b. Nucleic acid amplification test is more sensitive than culture.<br />

c. Chlamydia! cervicitis should be suspected if Gram stain reveals PMNs but no organisms.<br />

4. Management<br />

a. Start as soon as the diagnosis is suspected clinically or there is a history of exposure. Do not wait<br />

for culture results, because this may result in development of more advanced disease and continued<br />

transmission of infection.<br />

b. Antibiotic regimens<br />

(1) Azithromycin 1 g orally once or<br />

(2) Doxycycline 100 mg orally bid x 7 days (contraindicated in pregnancy)<br />

(3) Alternatives include erythromycin, levofloxacin, and amoxicillin.<br />

c. If untreated, may lead to infertility and ectopic pregnancy.<br />

d. Sexual partners need to be treated.<br />

VII. PELVIC INFLAMMATORY DISEASE (PID)<br />

A. Etiology<br />

1. Caused by infection of the upper reproductive tract organs that has spread from the cervix/lower tract<br />

2. Pathogenic organisms<br />

a. N gonorrhoeae and C trachomatis are the major pathogens; together they account for approximately<br />

80% of cases.<br />

b. Other pathogens include Trichomonas vagina/is and mycoplasmas.<br />

B. Pathophysiology<br />

1. Normal host barriers (uterotubal cervical mucus, lysozymes, and local immunoglobulin lgA as well as the<br />

cervix itself) usually prevent infection.<br />

2. Menstruation is the most common cause of breakdown of these barriers.<br />

3. During pregnancy, fusion of the chorion and decidua forms another natural barrier, making PIO less common<br />

but higher risk if occurring during pregnancy.<br />

C. Diagnostic evaluation<br />

1. Criteria for diagnosing acute pelvic inflammatory disease<br />

a. CDC recommends empirical treatment for pelvic inflammatory disease in sexually active women with<br />

pelvic pain plus one of the following:<br />

(1) Cervical motion tenderness<br />

(2) Uterine tenderness<br />

(3) Adnexal tenderness (usually bilateral)<br />

b. Additional criteria helpful in increasing specificity but not required for diagnosis<br />

(1) Oral temperature > 101 °F (38.3°C)<br />

(2) Abnormal vaginal or cervical discharge<br />

(3) Abundant WBCs on wet prep<br />

(4) Increased erythrocyte sedimentation rate or C-reactive protein<br />

(5) Laboratory evidence of cervical infection with N gonorrhoeae or Chlamydia<br />

2. Ultrasonography may show tubo-ovarian abscess or pyosalpinx.<br />

D Management<br />

1. All regimens should cover N gonorrhoeae, Chlamydia, anaerobes, gram-negative rods, and streptococci.<br />

2. Outpatient<br />

a. Ceftriaxone 250 mg IM plus doxycycline 100 mg orally bid x 14 days ± metronidazole 500 mg orally bid<br />

x 14 days, or<br />

b. Cefoxitin 2 g IM and probenecid 1 g orally plus doxycycline 100 mg orally bid x 14 days is the traditional<br />

therapy, ± metronidazole 500 mg orally bid x 14 days.<br />

3. Inpatient<br />

a. Recommended admission indications for parenteral treatment<br />

(1) Temperature >38.3°C<br />

455


GYNECOLOGIC AND OBSTETRIC DISORDERS<br />

(2) Nausea and vomiting (unable to take antibiotics)<br />

(3) No response to outpatient therapy within 72 hours<br />

(4) Suspicion of a tubo-ovarian or pelvic abscess (which may require surgery)<br />

(5) Uncertain diagnosis (another surgical emergency cannot be excluded)<br />

(6) Young age (potential for future fertility issues)<br />

(7) WBC >15,000/mm 3<br />

(8) Timely clinical follow-up (within 72 hours of starting antibiotics) cannot be arranged.<br />

b. Drug therapy<br />

(1) Cefoxitin 2 g IV every 6 hours or cefotetan 2 g IV bid plus doxycycline 100 mg IV or orally bid,<br />

± metronidazole 500 mg orally, or<br />

(2) Ampicillin/sulbactam 3 g IV qid plus doxycycline 100 mg orally or IV bid<br />

c. Sexual partners should be evaluated and treated empirically to prevent reinfection.<br />

E. Complications<br />

1. Tubo-ovarian abscess<br />

a. Patients present with constitutional symptoms, unilateral pelvic pain, and vaginal discharge. The laterality<br />

is what differentiates tubo-ovarian abscess from PID.<br />

b. The infected fallopian tube adheres to the ovary, forming an abscess that can go on to rupture and cause<br />

peritonitis.<br />

c. Physical examination: unilateral adnexal tenderness and mass<br />

d. Ultrasound is preferred but can be seen on CT as well.<br />

e. Treatment: parenteral antibiotics with/without operative drainage if unruptured; ruptured tubo-ovarian<br />

abscess requires immediate surgery.<br />

2. Fitz-Hugh-Curtis syndrome (perihepatitis)<br />

a. Caused by inflammation of the liver capsule that leads to adhesions<br />

b. Sharp, pleuritic right upper quadrant pain that accompanies the pelvic pain of PID<br />

c. Pain may refer to the shoulder or upper arm.<br />

3. Chronic inflammation and scarring can lead ectopic pregnancy, infertility, and chronic pelvic pain.<br />

4. Obstetrical complications<br />

a. Intrauterine growth retardation<br />

b. Septic abortion<br />

c. Premature rupture of membranes/preterm delivery<br />

VIII. OVARY<br />

A. Torsion<br />

1. Pathophysiology<br />

a. An abnormal ovary or fallopian tube twists around its vascular pedicle _,. blood supply is compromised _,.<br />

painful degeneration of the ovary or fallopian tube with eventual gangrenous necrosis<br />

b. Ovarian torsion usually occurs in association with an ovary that is enlarged because of a cyst or tumor or<br />

that is overstimulated (ovarian hyperstimulation syndrome) by use of fertility drugs; fallopian tube torsion<br />

usually occurs in association with hydrosalpinx, neoplasm, adhesions, trauma, or previous ligation.<br />

c. May occur at any age (although most common in the mid-twenties) and at any time during the menstrual<br />

cycle<br />

2. Clinical presentation<br />

a. Vital signs: absence of fever, or tachycardia out of proportion to the fever (different from PID, tuboovarian<br />

abscess)<br />

b. History<br />

(1) Onset of pain is usually sudden (frequently during or immediately after sexual intercourse); the<br />

patient may even remember the exact moment. Pelvic infections have a more gradual onset.<br />

(2) The pain is usually unilateral (more commonly on the right), becomes increasingly severe but may<br />

subside and, if so, the patient may not appear to be acutely ill on presentation (not so with pelvic<br />

infections).<br />

(3) Often, there is a history of similar episodes in the past that resolved spontaneously.<br />

456


GYNECOLOGIC AND OBSTETRIC DISORDERS<br />

c. Pelvic examination<br />

(1) Unilateral adnexal tenderness ± mass<br />

(2) During early stages, significant tenderness or discomfort may not be elicited during examination.<br />

3. Diagnostic evaluation<br />

a. Routine laboratory studies are usually normal. Pregnancy testing should be done to exclude ectopic or<br />

concomitant intrauterine pregnancy.<br />

b. Ultrasound may reveal the adnexal mass when the physical examination does not. Color Doppler flow<br />

aids diagnosis. Coiled vessels or "whirlpool sign" is 88% accurate in diagnosing torsion.<br />

c. CT/MRI: thickened fallopian tube, enlarged ovary, ovarian mass, free pelvic fluid, edematous ovary,<br />

deviation of the uterus to the affected side, associated hemorrhage<br />

d. Laparoscopy: the definitive diagnostic procedure<br />

e. Negative imaging findings should be taken with caution when clinical suspicion of torsion is high.<br />

4. Management<br />

a. Surgical correction or resection is the definitive therapy of choice; however, laparoscopy alone is being<br />

used with increasing frequency, because "untwisting" may be possible if torsion is diagnosed early or is<br />

caused by the ovarian hyperstimulation syndrome.<br />

b. Although return of ovarian function is possible even with late presentation, pediatric patients taken to<br />

surgery within 8 hours have the best chance of recovery.<br />

5. Complications<br />

a. Scarred tube - t fertility and t risk of tubal pregnancy<br />

b. Ovarian necrosis - peritonitis and shock, infertility<br />

B. Ovarian cystic masses<br />

1. Two major groups: functional ovarian cysts and ovarian cystic neoplasms<br />

2. Functional ovarian cysts<br />

a. Very common; subcategorized into follicular cysts or corpus luteum cysts<br />

b Originate from ovarian follicles, created by hormonal dysfunction during ovulation<br />

c. May become hemorrhagic<br />

d. Treatment: supportive/observational<br />

3. Ovarian cystic neoplasms<br />

a. Ovarian teratoma<br />

(1) Arises from a germ cell, containing any of the germ layers-ectoderm, mesoderm, endoderm-forming<br />

tissues that are disorganized in structure<br />

(2) Immature teratoma: malignant<br />

(3) Mature cystic teratoma (aka dermoid tumor): benign<br />

(4) 15% undergo torsion<br />

IX.UTERUS<br />

A. Vaginal bleeding<br />

1. Etiology<br />

a. Alterations in the endocrine system secondary to ovarian tumors, corpus luteum cyst, hypothyroidism,<br />

menarche or menopause, excessive exercise, poor diet, medications<br />

b. The most common medications implicated in "breakthrough" bleeding (ie, bleeding that occurs between<br />

periods)<br />

(1) Anticonvulsants<br />

(2) Some antibiotics<br />

(3) Anticoagulants<br />

(4) Exogenous estrogen or progesterone<br />

(5) Oral contraceptives (breakthrough and withdrawal bleeding)<br />

c. Coagu Iopa th ies<br />

d. Pelvic infections<br />

e. Neoplasms (Cervical polyps or carcinoma, leiomyomas, endometrial polyps or carcinoma, ovarian tumors)<br />

457


GYNECOLOGIC AND OBSTETRIC DISORDERS<br />

f. Postoperative (generally occurs when absorbable sutures dissolve)<br />

g. Trauma<br />

h. Foreign bodies<br />

1. Dysfunctional uterine bleeding<br />

2. Management<br />

a. Exclude life-threatening hemorrhage by evaluating hemodynamic stability.<br />

b. Stabilize patients with life-threatening bleeding (the minority) with IV fluids and blood products as needed;<br />

consider IV estrogens.<br />

c. Identify treatable causes through a careful history and physical; important points include:<br />

B. Cervical bleeding<br />

(1) Assume that any woman of childbearing age with abnormal vaginal bleeding is pregnant (intrauterine<br />

or ectopic) until proven otherwise; perform a pregnancy test.<br />

(2) Estimate the blood loss by determining the duration and amount of bleeding as well as the presence of<br />

clots.<br />

(3) In addition to performing a pelvic examination, check other potential sites of bleeding that might be<br />

confused with vaginal bleeding (perineum and rectum).<br />

(4) Order other appropriate laboratory studies as indicated.<br />

1. Mild to moderate - Mansel solution or silver nitrate<br />

2. Severe (postoperative or trauma-induced) - appropriate suturing<br />

3. Refer to a gynecologist for a complete investigation.<br />

C. Endometriosis<br />

1. Definition and clinical presentation<br />

a. Common gynecologic disorder defined as presence of endometrial glands and stroma outside the normal<br />

location; most commonly in pelvic peritoneum, ovaries, rectovaginal septum, and ureter; rarely found in<br />

bladder, pericardium, and pleura.<br />

b. Affects females of reproductive age; patients can be asymptomatic, subfertile, and/or have varying degrees<br />

of pelvic pain.<br />

2. Physical examination: the vagina and cervix usually appear normal. Occasionally, blue or red lesions can be<br />

seen on the cervix posterior vagina; these lesions can be tender or bleed with contact. Bimanual examination<br />

may reveal an adnexal mass representing ovarian endometrioma but is generally nonspecific.<br />

3. Diagnosis: diagnostic tests done in the emergency department include those to exclude other emergent causes<br />

of pelvic pain. Nonemergent diagnostic laparoscopy is the primary method used by gynecologists to diagnose<br />

endometriosis.<br />

4 Management<br />

a. Emergency department management: NSAIDs<br />

b. Gynecology: hormonal therapies (combination oral contraceptives, progestins, etc), surgical treatment<br />

D. Pelvic organ prolapse<br />

1. Definition and clinical presentation<br />

a. Descent of one or more of the following: anterior vaginal wall, posterior vaginal wall, uterus and cervix,<br />

the apex of the vagina after hysterectomy, or the perineum. Small bowel may be included in the presenting<br />

bulge, termed an enterocele.<br />

b. Patients present with complaints of pelvic pressure and vaginal bulging, and can also report urinary stress<br />

incontinence, frequency or urgency, and other voiding dysfunction.<br />

2. Physical examination: If no vaginal bulge is obviously present on visual inspection (before insertion of<br />

speculum), the patient should be directed to attempt a Valsalva maneuver. The prolapse may still not be seen<br />

with this maneuver, because it is a dynamic condition and frequently worsens at the end of the day or with<br />

physical activity. The gynecologist can perform a more extensive examination with a split speculum and<br />

measurements.<br />

3. Laboratory tests/imaging: diagnostic tests done in the emergency department include those to exclude other<br />

emergent causes of pelvic pain and/or urinary symptoms.<br />

4. Management: patients with minimal symptoms can be managed expectantly. For more significant symptoms,<br />

several nonsurgical (pessary, pelvic floor muscle exercise) and surgical (obliterative or reconstructive<br />

procedures) approaches can be managed by a gynecologist.<br />

458


GYNECOLOGIC AND OBSTETRIC DISORDERS<br />

E. Tumors<br />

1 . Leiomyoma<br />

a. Fibroid uterus is a common benign muscular tumor of the uterus.<br />

b. Incidence increases with age; affects >40% of women by age 35.<br />

c. Black American women are more likely to have fibroids.<br />

d. May cause anemia secondary to significant bleeding or become painful when large enough to become necrotic<br />

e. Diagnostic evaluation: ultrasound examination; however, the enlarged uterus may be palpable on bimanual<br />

examination.<br />

f. Management<br />

(1) NSAIDs to control pain<br />

(2) Iron supplementation and potentially blood transfusion for anemia<br />

(3) Hysterectomy, myomectomy, or embolization if indicated<br />

2. Gestational trophoblastic disease (molar pregnancy): see page 462.<br />

X. EMERGENCY CONTRACEPTION<br />

A. Definition: contraceptive care for women after consensual but unprotected sexual intercourse or after<br />

sexual assault. Substantially decreases the likelihood of an undesired pregnancy.<br />

B. Methods available for use as emergency contraception<br />

1. Progestin only<br />

a. Levonorgestrel 0.75 mg within 72 hours of intercourse, then 0.75 mg 12 hours later, or<br />

b. Levonorgestrel 1.5 mg once within 72 hours or up to 120 hours after intercourse<br />

2. Selective progestin-receptor modulator: ulipristal acetate 30 mg once up to 120 hours after intercourse<br />

3. Combination oral contraceptive pill: multiple regimens exist<br />

4. Insertion of copper-containing intrauterine device (although not a practical option in the emergency department)<br />

C. Patients should be counseled to use a barrier contraceptive technique until the next menses (because oral<br />

contraceptive methods will not prevent pregnancy resulting from subsequent episodes of intercourse), as<br />

well as obtain pregnancy testing if menstruation is delayed past its expected onset.<br />

XI. SEXUAL ASSAULT<br />

A. Purpose of evaluation is to diagnose and treat the victim's physical and emotional injuries, and to collect<br />

legal evidence. State and local protocols should be followed. Specialized, trained nurses commonly conduct<br />

this examination.<br />

B. History of the event<br />

1. Where and when did the assault occur?<br />

2. What happened during the assault? Document using the patient's own words with quotations.<br />

3. What happened after the assault?<br />

a. Did the patient bathe, void, defecate, brush teeth, or change clothes?<br />

b. Does she complain of pain or have any other symptoms?<br />

4. Has the patient had sexual intercourse in the last 72 hours? (If so, it may confuse the laboratory analysis of<br />

sperm and acid phosphatase.)<br />

C. Physical examination<br />

1. Perform a general physical examination, noting especially any signs of trauma.<br />

2. Perform a pelvic examination (unless protocol calls for trained sexual assault nurse), looking for evidence of<br />

trauma or infection.<br />

D. Laboratory and evidence collection<br />

1. Standard laboratory studies and radiographs when indicated for trauma, ~-HCG (to exclude preexisting pregnancy)<br />

459


GYNECOLOGIC AND OBSTETRIC DISORDERS<br />

2. Evidence<br />

a. Know what the police want before you examine the patient, or use a standard kit that contains supplies<br />

for the collection. Photographs are becoming increasingly important. Develop protocol for forensic<br />

admissibility and protect the victim's privacy.<br />

b. Maintain an unbroken chain of evidence. Specimens must always be in view or under lock and key. Sign<br />

the outside of each sealed container when transferring custody.<br />

E. Management<br />

1. Standard care for traumatic injuries<br />

2. Prevent infection<br />

a. Offer prophylactic antimicrobial therapy for syphilis, gonorrhea, Chlamydia, and Trichomonas.<br />

b. Offer prophylaxis for hepatitis B with vaccination.<br />

c. If assailant's HIV status is unknown, evaluate risks and benefits of postexposure prophylaxis on a case-bycase<br />

basis.<br />

d. Schedule follow-up visits for repeat serology and cultures.<br />

3. Prevent pregnancy: offer postcoital contraception as described above; refer for repeat pregnancy test as needed.<br />

4. Counseling: initial contact should be made while the patient is in the emergency department by an individual<br />

experienced in rape counseling; if this is not possible, early referral is essential.<br />

XII. NORMAL PREGNANCY<br />

ANY WOMAN OF CHILDBEARING AGE WHO PRESENTS TO THE EMERGENCY DEPARTMENT IS CONSIDERED<br />

PREGNANT UNTIL PROVEN OTHERWISE.<br />

A. Clinical presentation<br />

1. Menstrual period: missed, light, or late<br />

2. Breast swel Ii ng and tenderness at 4 weeks<br />

3. Fatigue, nausea, and urinary frequency at 5-6 weeks<br />

4. Cervical softening (tip at 4 weeks, isthmus at 6-8 weeks)<br />

5. Chadwick sign (bluish discoloration of cervix at 6-8 weeks)<br />

6. Uterus enlarged and soft at 6-8 weeks<br />

B. Pregnancy tests<br />

1. All available tests detect ~-HCG with varying sensitivities and specificities. Production of ~-HCG begins at the<br />

time of implantation (8-9 days after conception). Levels of ~-HCG double every 2 days during the initial weeks<br />

of a normal pregnancy, peak at 8 weeks, and remain detectable up to 60 days after birth or abortion. With the<br />

newer assays, pregnancy can be detected as early as 23-25 days after the last menstrual period. Pregnancy tests<br />

that measure the ~-HCG subunit have the greatest specificity.<br />

2. Causes of false negatives<br />

a. Too early in pregnancy<br />

b. Dilute or old urine: if the specific gravity is below the recommended level set by your laboratory, get a<br />

serum qualitative.<br />

c. Ectopic pregnancy: although rare, ruptured ectopic pregnancies have been reported with very low or absent<br />

~-HCG.<br />

d. Incomplete abortion<br />

3. Causes of false positives (rare)<br />

a. Interference from non-~-HCG substances<br />

b. Exogenous HCG<br />

c. Trophoblastic neoplasm<br />

4. Ultrasound is frequently used in the emergency department to confirm that the pregnancy is intrauterine and to<br />

verify fetal heart rate.<br />

a. Fetal heart can be determined by using M-mode on ultrasound.<br />

b. Fetal age can be determined by measuring the gestational sac, the crown-rump length, the head<br />

circumference, or the femur length. The best measurement depends on the age and position of the fetus.<br />

c. Intrauterine pregnancy should be confirmed by visualization of an intrauterine yolk sac.<br />

460


GYNECOLOGIC AND OBSTETRIC DISORDERS<br />

d. Ultrasound should be performed only by trained personnel.<br />

Courtesy of Sarah Sommerkamp, MD<br />

M-mode ultrasound showing measurement of fetal heart rate<br />

Courtesy of Sarah Sommerkamp, MO<br />

Ultrasound showing crown-rump length measurement for gestational age<br />

C. Physiologic changes of pregnancy<br />

1. Cardiovascular changes<br />

a. Heart rate increases by 15-20 beats per minute above baseline (to an average of 80-90 beats per minute)<br />

by the third trimester.<br />

b. Blood pressure<br />

(1) Systolic and diastolic blood pressure decrease by 10-15 mm Hg in the second trimester (to an average<br />

of 102/55 mm Hg) and then gradually return to prepregnant levels in the third trimester.<br />

(2) After the 20th week of gestation, uterine compression of the inferior vena cava in the supine position<br />

can decrease venous return by as much as 30% and produce hypotension. This can be relieved by<br />

placing the patient in the left lateral decubitus position, inserting a wedge under the right hip, or<br />

manually displacing the uterus.<br />

c. Cardiac output increases by ~30% during the first trimester, increases further (~50%) in the second<br />

trimester and is maintained at this level for the remainder of the pregnancy.<br />

2. Respiratory changes<br />

a. Respiratory rate increases slightly (or not at all).<br />

b. Tidal volume increases by 40%.<br />

c. Functional residual volume decreases by 25% (due to elevation of the diaphragm).<br />

d. pCO 2<br />

decreases to 30-33 mm Hg.<br />

461


GYNECOLOGIC AND OBSTETRIC DISORDERS<br />

3. Hematologic changes<br />

a. Blood volume increases up to 50% at term<br />

b. Hematocrit decreases to the low 30% range by the 30th week.<br />

c. WBC count increases (due to an increase in PMNs); counts up to 18,000/mm 3 are normal in the second<br />

and third trimesters, while counts up to 25,000/mm 3 may be seen during labor.<br />

d. Fibrinogen level is increased (average level is 400-450 mg!dl).<br />

e. Prothrombin time (INR)/partial thromboplastin time are normal despite increases in factors VII, VIII, IX, X, and XII.<br />

XIII. DRUGS AND RADIATION EXPOSURE IN PREGNANCY<br />

A. Drugs<br />

1. The teratogenic risk associated with drugs is greatest during embryogenesis (the first trimester) and is usually<br />

dose-related.<br />

2. The FDA classifies drugs used in pregnancy into five categories (A, B, C, D, X) according to their safety.<br />

a. Drugs in category A have been studied in people and are considered safe for use in pregnancy.<br />

b. Drugs in category B have been studied in animals and are generally considered safe.<br />

c. Drugs in category C should be used with caution.<br />

d. Drugs in categories D and X should be avoided (if possible); studies have demonstrated use in pregnancy<br />

associated with risk to the fetus.<br />

B. Radiation exposure<br />

1. The risk of birth defects from a 1-rad exposure (1,000 mrads) is thousands of times smaller than the<br />

spontaneous risks of congenital malformations or genetic disease. If exposure to the fetus is 10 cm away from the fetus are<br />

not harmfu I.<br />

a. Plain radiographs of cervical spine, chest, and pelvis all deliver


GYNECOLOGIC AND OBSTETRIC DISORDERS<br />

(3) Vaginal passage of characteristic grapelike clusters of vesicles<br />

(4) Uterus large for dates<br />

(5) Anemia<br />

b. Diagnostic evaluation: ultrasound findings typically described as a "snow-storm" appearance<br />

B. Abortion<br />

Courtesy of Sam Hsu, MO<br />

Ultrasound showing a molar pregnancy<br />

c. Management<br />

(1) Uterine evacuation plus careful follow-up of ~-HCG titers to exclude chorionic carcinoma (2%-3%<br />

transform to carcinoma)<br />

(2) Rh immune globulin for Rh-negative women<br />

1. Threatened abortion is the most common cause of bleeding in a primigravida.<br />

a. Definition: any amount of bleeding without passage of products of conception or cervical dilatation<br />

b. Incidence: 30%-40% of clinically pregnant women have first trimester bleeding; 50% of these women<br />

ultimately miscarry.<br />

c. Management<br />

(1) Expectant management: pelvic rest (no douching, tampons, or intercourse until bleeding resolves)<br />

(2) Rh immune globulin for Rh-negative women<br />

(3) Follow-up with obstetrician in 2-3 days<br />

2. Inevitable abortion<br />

a. Definition: the cervical os is open and vaginal bleeding is present.<br />

b. Management<br />

(1) Expectant management: dilation and evacuation, or dilatation and curettage<br />

(2) Rh immune globulin for Rh-negative women<br />

(3) Consult obstetrics for brisk bleeding.<br />

3. Incomplete abortion<br />

a. Definition: the cervical os is open, vaginal bleeding is present, and products of conception are present at<br />

the cervical os or in the vaginal canal.<br />

b. Management<br />

(1) Visible products of conception should be removed with a ring forceps to control bleeding.<br />

(2) Expectant management: dilation and evacuation, or dilatation and curettage<br />

(3) Rh immune globulin for Rh-negative women.<br />

(4) Consult obstetrics for brisk bleeding.<br />

4. Complete abortion<br />

a. Definition: all products of conception have been passed, the cervix is closed, the uterus is firm and<br />

nontender, and the bleeding has almost stopped.<br />

463


GYNECOLOGIC AND OBSTETRIC DISORDERS<br />

b. Management<br />

(1) Up to 80% of first trimester miscarriages complete without intervention<br />

(2) Expectant management: curettage, or dilatation and curettage<br />

(3) Rh immune globulin for Rh-negative women.<br />

5. Missed abortion (more commonly called first or second trimester fetal demise)<br />

a. Definition: failure to pass products of conception beyond 2 months after fetal death; pregnancy test<br />

converts from positive to negative.<br />

b. Management: dilatation and curettage, Rh immune globulin for Rh-negative women<br />

C. Ectopic pregnancy<br />

1. Leading cause of maternal death in the first trimester<br />

2. Risk factors (~50% of ectopic pregnancies have no associated risk factors)<br />

a. Previous ectopic pregnancy, tubal surgery, or sterilization procedure<br />

b. Documented tubal scarring/pathology<br />

c. Diethylstilbestrol exposure in utero<br />

d. Presence of intrauterine device (prevents only intrauterine but not extrauterine pregnancies)<br />

e. History of pelvic inflammatory disease<br />

f. Infertility (in vitro)<br />

g. Multiple sex partners<br />

3. Clinical presentation<br />

a. Patients classically present with a history of amenorrhea followed by abdominal pain and abnormal<br />

vaginal bleeding. Classic triad is present in only 15 % of patients and is neither sensitive nor specific.<br />

b. Ectopic pregnancy with hemorrhage may present with shoulder pain referred from an irritated diaphragm<br />

(Kehr sign). Only 20% of ectopic pregnancies present in extremis. May present with relative bradycardia.<br />

c. Pelvic examination findings<br />

(1) Vaginal bleeding (may be absent)<br />

(2) Unilateral adnexal tenderness± a palpable mass<br />

(3) Uterus normal in size or slightly enlarged<br />

(4) Fullness of cul-de-sac<br />

Table 29: Symptoms and Signs of an Ectopic Pregnancy<br />

Symptom/Sign<br />

Abdominal pain<br />

Amenorrhea<br />

Vaginal bleeding<br />

Dizziness<br />

Pregnancy symptoms<br />

Urge to defecate<br />

Passing tissue<br />

Adnexal tenderness<br />

Abdominal tenderness<br />

Adnexal mass<br />

Uterine enlargement<br />

Orthostatic changes<br />

Fever<br />

% of Women with Symptoms/Signs<br />

80-100<br />

75-95<br />

50-80<br />

20-35<br />

10-25<br />

5-15<br />

5-10<br />

75-95<br />

80-95<br />

50<br />

20-30<br />

10-15<br />

5-10<br />

464


GYNECOLOGIC AND OBSTETRIC DISORDERS<br />

4. Diagnostic evaluation<br />

a. Pregnancy tests: a benign course cannot be assumed with low f3-HCG levels; ruptured ectopic<br />

pregnancies that require surgery have been reported with very low or absent levels of f3-HCG.<br />

b. Pelvic ultrasound<br />

(1) Transabdominal ultrasound can be useful if it shows a clear intrauterine pregnancy in a patient who<br />

is not at risk of a heterotopic (concomitant intrauterine and extrauterine) pregnancy. Discriminatory<br />

zone is user dependent but around 5,000 for the transabdominal approach.<br />

(2) Transvaginal ultrasonography is a sensitive technique; it can identify landmarks consistent with a<br />

normal intrauterine pregnancy at 5 weeks of gestation and may occasionally identify the actual<br />

ectopic pregnancy.<br />

(3) Indeterminate ultrasounds (positive f3-HCG but no definite intrauterine pregnancy or extrauterine<br />

findings of ectopic pregnancy) occur in up to 20% of emergency department patients undergoing<br />

evaluation for first-trimester emergencies; 1,500: ultrasound, serial f3-HCG measurements to establish ectopic pregnancy or<br />

miscarriage.<br />

(2) f3-HCG 3,000 for transvaginal ultrasound.<br />

d. For patients in whom a ruptured ectopic pregnancy is a concern, check the right upper quadrant view/<br />

Morison's pouch for free fluid.<br />

Courtesy of Sandra Werner, MD, ROMS, FACEP<br />

Left: Empty uterus with a yolk sac in the left adnexa (ectopic pregnancy)<br />

Right: Normal intrauterine pregnancy with a fetus surrounded by the uterus<br />

Courtesy of Sarah Sommerkamp, MD, ROMS<br />

Free fluid separating liver, kidney, and bowel in a patient with a ruptured ectopic pregnancy<br />

465


GYNECOLOGIC AND OBSTETRIC DISORDERS<br />

e. Ultrasound consistent with a known or expected ectopic pregnancy requires immediate obstetrics<br />

consultation.<br />

(1) Ruptured: emergent<br />

(2) Unruptured: consider methotrexate therapy versus surgery<br />

(3) Intrauterine pregnancy<br />

5. Management<br />

(a) Follow up with obstetrics.<br />

(b) Follow quantitative B-HCG for doubling every 2 days.<br />

(c) Repeat ultrasound.<br />

(d) Threatened miscarriage precautions<br />

a. Establish an IV line and provide fluid resuscitation as indicated.<br />

b. Obtain CBC, Rh typing as well as type and screen/crossmatch early in suspected cases, comprehensive<br />

metabolic panel, B-HCG, ultrasound.<br />

c. Administer Rh immune globulin to women who are Rh-negative.<br />

d. Definitive therapy<br />

(1) Stable patients with unruptured ectopic pregnancies measuring


GYNECOLOGIC AND OBSTETRIC DISORDERS<br />

2. Appendicitis<br />

(3) Admit pregnant patients with pyelonephritis to the hospital for IV antibiotics (a cephalosporin with or<br />

without an aminoglycoside) and hydration. These patients are at increased risk of developing preterm<br />

labor, bacteremia, and septic shock. Urine cultures should be sent and an obstetric consult obtained.<br />

a. The most frequent surgical emergency of pregnancy<br />

b. The incidence of appendicitis is not higher in pregnancy, but the development of complications (especially<br />

rupture) is more frequent because of delay in diagnosis.<br />

c. Clinical presentation: fever and right-sided pain. Diagnosis is difficult because the pregnant patient has<br />

baseline leukocytosis, and the appendix moves up the right side with uterine displacement. Can be<br />

confused with pyelonephritis.<br />

d. Management: obtain surgical and obstetric consultation. Ultrasonography, CT, and MRI have been used to<br />

assist with diagnosis. Additionally, laparoscopy or laparotomy is the definitive treatment.<br />

3. TORCH infections: toxoplasmosis, other (syphilis), rubella syndrome, cytomegalovirus, herpes simplex<br />

infection<br />

a. Toxoplasmosis (fetal: eye, ear, neurologic dysfunction, rash, prematurity)<br />

(1) Parasitic disease caused by Toxoplasma gondii<br />

(2) Linked with exposure to cats<br />

(3) Treatment: spiramycin in pregnancy<br />

b. Syphilis (fetal: fever, failure to thrive, saddle nose, rash)<br />

(1) Treatment: penicillin<br />

(2) If treated inappropriately, ~50% fatal.<br />

c. Rubel la syndrome (fetal: cataracts, deafness, patent ductus arteriosus)<br />

(1) Viral illness<br />

(2) Order a pregnancy test in all women of childbearing age with rubella.<br />

(3) If pregnancy test is positive, refer patient for genetic counseling.<br />

Time of Maternal Infection<br />

First month<br />

Second month<br />

Third month<br />

Fetal Infection<br />

50%<br />

25%<br />

10%<br />

d. Cytomegalovirus (fetal: neurologic dysfunction, chorioretinitis, hepatosplenomegaly, rash, microcephaly)<br />

(1) Viral ii lness; minimal maternal symptoms<br />

(2) No specific treatment; prevention<br />

e. Herpes simplex infection (fetal: eye, brain, and skin disease)<br />

(1) Women with primary mucous membrane infections in the third trimester have an increased risk of<br />

dissemination, which may be life threatening.<br />

(2) Most (if not all) of the complications associated with lesions during pregnancy can be avoided by taking<br />

antiviral medications in the third trimester. Famciclovir and valacyclovir are pregnancy category B.<br />

(3) Neonatal infection can be contracted via passage through an infected birth canal and is associated<br />

with significant mortality (50%) and morbidity (particularly neurologic sequelae).<br />

(4) Active infection at the time of delivery carries a 30%-40% risk of infecting the infant.<br />

(5) The presence of active lesions in a patient who is in labor is an indication for cesarean section.<br />

(6) Treatment with IV acyclovir for serious infections.<br />

F. Hyperemesis gravidarum/nausea and vomiting of pregnancy<br />

1. 50%-70% of women are affected, usually in the first trimester (probably due to increased ~-HCG levels). Some<br />

studies suggest a vitamin 8 6<br />

deficiency as the cause and recommend supplemental vitamin B 6 .) Persistence of<br />

symptoms past 14 weeks or presence of abdominal pain suggests the possibility of another cause.<br />

2. Management: frequent small meals (dry crackers before rising, toast and cereal when symptomatic) and<br />

supplemental vitamin B 6<br />

may be helpful with or without doxylamine.<br />

3. Hyperemesis gravidarum: persistent nausea and vomiting not caused by other medical conditions, ketonuria<br />

(as a marker of starvation), and at least 5% weight loss from pre-pregnancy weight. IV hydration (D5 lactated<br />

467


GYNECOLOGIC AND OBSTETRIC DISORDERS<br />

Ringer's or D5 normal saline) and antiemetics (vitamin B 6<br />

plus doxylamine, promethazine, metoclopramide,<br />

or ondansetron) are indicated.<br />

a. Hospitalization is indicated if emesis cannot be controlled with above measures.<br />

G. Gestational diabetes<br />

1. Diabetes in a pregnant woman who was not previously diabetic<br />

2. Places woman and fetus at risk of complications: both macrosomy and intrauterine growth restriction, difficult<br />

delivery, fetal hypoglycemia postpartum<br />

3. Initial steps in management are diet and exercise.<br />

4. Oral medications (metformin) or insulin may be required to maintain normoglycemia.<br />

H. Chronic and gestational hypertension<br />

1. Chronic hypertension is systolic blood pressure ?:140 or a disastolic blood pressure 90 mmHg identified before<br />

the 20th week of gestation and that persists for >6 weeks postpartum.<br />

2. Gestational hypertension is a blood pressure ?:140/90 mm Hg on two occasions in a patient who was<br />

normotensive before 20 weeks of gestation. There are no symptoms of preeclampsia or proteinuria.<br />

3. Management depends on severity of disease.<br />

a. Anti hypertensive medications are not routinely started for blood pressure


GYNECOLOGIC AND OBSTETRIC DISORDERS<br />

3. Management<br />

a. History, physical, and laboratory studies to confirm presence of preeclampsia and to exclude other<br />

pathology<br />

b. Consultation with obstetrics for further management (timing of delivery and steroids) and monitoring<br />

c. Antihypertensive therapy is not required unless systolic blood pressure> 160 mm Hg or diastolic blood<br />

pressure> 110 mm Hg.<br />

(1) Hydralazine is used most commonly, but other drugs also work well (eg, labetalol, nicardipine, sodium<br />

nitroprusside).<br />

(2) ACE inhibitors are contraindicated because of the possibility of fetal renal damage.<br />

(3) Prophylactic treatment with magnesium is also recommended to prevent seizures in the preeclamptic<br />

patient (loading dose of 4-6 g over 30 minutes followed by a 2 g/hour drip to maintain a serum level<br />

of 4-7 mEq/L).<br />

K. Eclampsia<br />

1. Diagnosis: preclampsia plus seizure<br />

a. Warning signs of impending seizure<br />

(1) Headache<br />

(2) Visual disturbances<br />

(3) Hyperreflexia<br />

(4) Abdominal pain (epigastric or right upper quadrant)<br />

2. Management<br />

a. Magnesium sulfate<br />

(1) Has both anticonvulsant and antihypertensive properties.<br />

(2) loading dose is 4-6 g IV over 5-15 minutes; then continuous IV infusion at 1-3 g/hour; patient<br />

should be placed on a cardiac monitor and have a Foley catheter in place.<br />

(3) Excretion is 100% renal, so urine output should be maintained at a rate of ~25 ml/hour.<br />

(4) Stop maintenance infusion if deep tendon reflexes disappear.<br />

(5) Watch for signs of magnesium toxicity:<br />

(a) Respiratory depression<br />

(b) Bradydysrhythmias<br />

(c) loss of deep tendon reflexes<br />

(6) Antidote: calcium gluconate (1 g slow IV push)<br />

b. If seizure continues, use typical anticonvulsant drugs (eg, benzodiazepines). Dilantin may be used in<br />

pregnancy.<br />

c. Additional medications may be needed for adequate control of blood pressure. IV labetalol or<br />

hydralazine are typically used.<br />

d. Delivery<br />

(1) Definitive treatment is delivery. Obtain obstetric consult.<br />

(2) Depending on patient specifics, this may be a vaginal or cesarean section delivery.<br />

e. Postpartum eclampsia can occur up to around 4 weeks postpartum. Consider in the differential for<br />

seizure patients.<br />

L. Hemorrhage, antepartum<br />

1. Abruptio placenta: 30% of cases of third-trimester bleeding<br />

a. Definition<br />

(1) Premature separation of a normally implanted placenta from the uterine wall causing visible or<br />

hidden bleeding<br />

(2) Significant unseen blood loss can occur before or without vaginal bleeding.<br />

b. Clinical presentation<br />

(1) Painful uterine bleeding<br />

(a) Bleeding is typically light (but may be heavy) and is usually dark or clotted.<br />

(b) At times, only pain is present, and this can vary from mild to severe.<br />

(2) Uterine findings vary from soft and nontender to slightly tender to rock hard.<br />

c. Risk factors include hypertension, smoking, cocaine, abdominal trauma, multiparity, advanced maternal<br />

age, and prior abruption.<br />

469


GYNECOLOGIC AND OBSTETRIC DISORDERS<br />

d. Management<br />

(1) Establish IV access; draw blood for type and crossmatch, preoperative laboratory studies, and<br />

disseminated intravascular coagulation profile; establish cardiac and fetal monitoring.<br />

(2) Fluid resuscitation and blood replacement as needed<br />

(3) Obtain immediate obstetrics consult, and perform ultrasound to exclude placenta previa (ultrasound<br />

is not sensitive enough to exclude abruptio placenta).<br />

(4) Stable patients may be observed in the hospital under close surveillance if a surgical team is<br />

available.<br />

(5) If the patient is unstable - immediate delivery by cesarean section (unless patient is in advanced<br />

labor); place patient in left lateral decubitus position to relieve inferior vena caval compression (can<br />

increase cardiac output by 30%-40%).<br />

(6) Administer Rh immune globulin to Rh-negative women.<br />

2. Placenta previa: responsible for 20% of cases of third trimester bleeding<br />

a. Definition: implantation of the placenta in the lower uterine segment such that it covers all or part of the<br />

cervical os<br />

b. Clinical presentation<br />

(1) Painless, bright red vaginal bleeding that is sometimes intermittent and progressively severe over a<br />

period of 1-2 weeks<br />

(2) On abdominal examination, the uterus is usually soft and nontender.<br />

c. Risk factors include prior cesarean section, prior previa, multiparity, multiple gestations, and advanced<br />

maternal age.<br />

d. Management<br />

(1) Establish IV access, draw blood for type and crossmatch and preoperative laboratory studies, and<br />

establish cardiac and fetal monitoring.<br />

(2) Obtain immediate obstetrics consult if patient is unstable and order ultrasound for placental location.<br />

(3) Pelvic examination (digital and speculum)<br />

(a) Should not be done unless ultrasound excludes placenta previa '--<br />

(b) Should be done in the operating room by the obstetrician under "double set-up" conditions in<br />

patients who are bleeding rapidly, in labor, or are hemodynamically unstable. In this manner, an<br />

immediate cesarean section can be rapidly performed if uncontrolled bleeding results.<br />

(4) Administer Rh immune globulin to Rh-negative women.<br />

M. Thromboembolism<br />

1. Pregnancy is a hypercoagulable state.<br />

a. The synthesis of coagulation factors VII, VIII, IX, X, and XII is increased.<br />

b. Compression of the inferior vena cava by the enlarging uterus promotes venous stasis.<br />

2. The risk of thromboembolism is 5-6 times greater in pregnant women than in non pregnant women and is<br />

highest in the immediate postpartum period as well as those delivered by cesarean section.<br />

3. Initial diagnostic studies of choice<br />

a. o-dimer is often increased in pregnancy; however, a low o-dimer can reliably exclude clot in a low pretest<br />

probabi I ity patient.<br />

b. Doppler ultrasonography: a positive finding can potentially circumvent the need for CT angiography or a<br />

VQ scan. Does not reliably detect pelvic or calf deep venous thrombosis.<br />

c. Ventilation/perfusion scan or multidetector CT angiography can both be used to diagnose pulmonary<br />

embolism. CT angiography has more radiation directed at the woman (breast cancer concern) than the VQ.<br />

The VQ scan has a higher fetal dose of radiation.<br />

4. Treatment is with low-molecular-weight heparin or heparin, which does not cross the placenta.<br />

5. Warfarin is contraindicated in pregnancy.<br />

N. Aortic dissection<br />

1. Second most common cause of maternal death<br />

2. Imaging is required for diagnosis.<br />

a. Choice (CT, MRI, transesophageal echocardiography) depends on stability of the patient and resources<br />

available at the hospital.<br />

b. A transthoracic or transabdominal ultrasound cannot reliably exclude dissection but can potentially<br />

diagnose it.<br />

470


GYNECOLOGIC AND OBSTETRIC DISORDERS<br />

3. Treatment in pregnant women is similar to that in nonpregnant patients to reduce blood pressure and pulse:<br />

~-blockers as well as hydralazine and nitrates are commonly used.<br />

0. Maternal cardiac disease<br />

1. Frequency has increased because improvements in the care of congenital cardiac disorders has led to adults<br />

with significant cardiac history in the USA.<br />

2. Childhood and adult obesity, hypertension, hypercholesterolemia, and diabetes are increasing the rates of<br />

coronary artery disease.<br />

3. Chest pain and dyspnea are common symptoms in pregnancy. Cardiac disease can present as dyspnea on<br />

exertion, fatigue, and edema.<br />

a. Cardiomyopathy: possible etiologies are pregnancy, viral, alcohol, cocaine, and ischemia.<br />

b. Congestive heart failure: laboratory tests, chest radiograph, and B-type natriuretic peptide (BNP) may help<br />

discern acute heart failure.<br />

(1) Avoid ACE inhibitors in pregnancy.<br />

c. Acute myocardial infarction: ECG<br />

(1) Acute myocardial infarction is more commonly seen in pregnancy<br />

(2) Percutaneous coronary intervention is first-line intervention in pregnancy.<br />

(3) Aspirin, nitroglycerin, and heparin are safe. Clopidogrel is also considered safe, but keep in mind<br />

delivery plan when starting an anticoagulant.<br />

d. Dysrhythmias<br />

P. Cardiac arrest<br />

(1) Common and frequently benign; must exclude life-threatening arrhythmias.<br />

(2) ECG usually sufficient, may require further monitoring.<br />

(3) Supraventricular tachycardia: adenosine can be safely used.<br />

(4) Atrial fibrillation can be managed with rhythm control, rate control, or cardioversion.<br />

(5) Use low-molecular-weight heparin if onset is unclear and patient is stable before cardioversion.<br />

1. CPR: high-quality compressions<br />

a. Continue with chest compressions/airway/breathing with the understanding that the pregnant woman<br />

has lower reserve and increased airway edema and is at increased risk of aspiration due to the physiologic<br />

changes in pregnancy.<br />

2. Manual displacement of the uterus<br />

3. IV access above the diaphragm<br />

4. Defibrillation and drugs should not be withheld or delayed because of the gravid state.<br />

5. Consider the underlying cause of the cardiac arrest.<br />

a. Review H's and T's<br />

b. Ultrasound<br />

6. Prepare for perimortem cesarean section<br />

Q. Perimortem cesarean section<br />

1. Anxiety-provoking procedure: should be started within 4 minutes of cardiac arrest for all women with<br />

estimated gestational age of 20 weeks and beyond; may start earlier if no hope of maternal survival.<br />

2. Can be lifesaving for both mother and fetus<br />

3. Fetal and maternal survival have been documented up to 30 minutes after arrest, but improved neurologic<br />

outcome are linked with early intervention.<br />

4. Hospitals should prepare in advance a team/protocol for a perimortem cesarean section.<br />

XV. HIGH-RISK PREGNANCY<br />

A. Assisted reproductive therapies<br />

1. Increasing incidence<br />

2. Complications<br />

a. Multiple gestations: larger than anticipated fundal height<br />

b. Heterotopic pregnancy: existence of an intrauterine and an ectopic pregnancy at the same time<br />

471


GYNECOLOGIC AND OBSTETRIC DISORDERS<br />

XVI. NORMAL LABOR AND DELIVERY<br />

A. Stages of labor<br />

1. Stage one<br />

a. From the onset of true labor (regular contractions with cervical change) to full dilation<br />

b. Consists of early, active, and transition<br />

2. Stage two<br />

a. From dilation of 10 cm until delivery of infant<br />

b. Support perineum, deliver head, check for nuchal cord, slight downward traction followed by upward<br />

traction to assist shoulder delivery support infant. May then place infant on mother's abdomen for<br />

clamping and cutting of cord. Routine suction of nose and mouth is no longer recommended. Deep suction<br />

for meconium only.<br />

3. Stage three<br />

a. From delivery of infant to delivery of placenta<br />

b. Avoid excessive tension on cord, because it may precipitate uterine inversion.<br />

XVII. COMPLICATIONS OF LABOR<br />

A. Fetal distress<br />

1. Ill defined and multifactorial<br />

2. Can be indicated by increased or decreased fetal heart rate, decreased fetal movement, repeated variable or<br />

late decelerations on fetal monitor<br />

3. Use and interpretation of electronic fetal monitors and tocodynamomoter should be done by appropriately<br />

trained staff.<br />

B. Premature labor<br />

1. Definition<br />

a. Onset of labor after 20 but before 37 weeks gestational age<br />

b. Contractions >30 seconds, regular contractions, 4 times every 20 minutes with changes in the cervix<br />

c. Braxton Hicks contractions: mild intensity and no cervical change<br />

d. Look for infection as cause (urinary tract infection, bacterial vaginosis, sexually transmitted infection,<br />

systemic infection<br />

e. Fetal fibronectin can be helpful to obstetrics to determine likelihood of delivery.<br />

2. Management<br />

a. Less than 34 weeks gestational age: corticosteroids for lung maturity<br />

b. Tocolytics: use with obstetrics consultation<br />

(1) Nifedipine<br />

(2) lndomethacin<br />

(3) Terbutaline<br />

(4) Magnesium<br />

C. Premature rupture of membranes<br />

1. Definition: rupture of fetal membranes before onset of labor<br />

2. Clinical presentation: history of watery vaginal discharge<br />

3. Diagnostic evaluation: after ultrasound to confirm placental placement if bleeding<br />

a. Perform a careful sterile speculum examination without using a lubricant (which can give a false-positive),<br />

and obtain sterile swabs of the posterior vaginal vault for:<br />

(1) A nitrazine test: amniotic fluid is alkaline (blood and semen give false-positives) and turns the yellow<br />

paper to blue-green.<br />

(2) Microscopic examination of a dried slide: amniotic fluid should show "ferning" (blood and<br />

inflammation give false-negatives).<br />

472


GYNECOLOGIC AND OBSTETRIC DISORDERS<br />

b. Obtain cervical cultures for N gonorrhoeae, Chlamydia, group B streptococci, aerobes, and anaerobes for<br />

patients with preterm (occurring before 3 7 weeks gestation) rupture of membranes. A digital examination<br />

should not be done in suspected or confirmed cases.<br />

4. Management is hospitalization.<br />

a. If gestational age is >37 weeks - delivery within 12-24 hours<br />

b. If gestational age


GYNECOLOGIC AND OBSTETRIC DISORDERS<br />

Courtesy of Wesley Fox<br />

C. Prolapse of cord<br />

1. Umbilical cord prolapse: typically occurs after rupture of membranes.<br />

a. Overt: cord lies below the presenting part and can be visualized or palpated.<br />

b. Occult: cord lies adjacent to the presenting part and can be detected by fetal monitoring (fetal heart rate<br />

changes with intermittent cord compression).<br />

2. Management<br />

a. Administer oxygen.<br />

b. Reposition the mother.<br />

c. Overt cord prolapse - knee to chest position (apply continuous upward pressure on the presenting part to<br />

relieve pressure on the cord until the fetus is delivered by cesarean section)<br />

d. Occult cord prolapse - lateral (or Trendelenburg) position<br />

(1) If fetal heart rate returns to normal with cord compression - vaginal delivery<br />

(2) Persistence of fetal heart rate change with cord compression - cesarean section<br />

XIX. POSTPARTUM COMPLICATIONS<br />

A. Postpartum fever<br />

1. Etiology: look for all usual causes of fever<br />

a. Wound infection (cesarean or episiotomy)<br />

b. Urinary tract infection (especially if history of catheterization or forceps delivery)<br />

c. Pneumonia<br />

d. Phlebitis<br />

2. Endometritis<br />

a. Clinical presentation: swollen, tender uterus with foul lochia 1-3 days after birth<br />

b. Management: hospitalization for broad-spectrum antibiotics IV<br />

3. Pelvic thrombophlebitis<br />

a. May mimic endometritis; suspect this diagnosis if patient does not respond quickly to antibiotics.<br />

b. Can cause pulmonary embolus<br />

4. Mastitis<br />

a. Etiology<br />

(1) Usually caused by Staphylococcus aureus<br />

(2) Usually occurs as a complication of breast-feeding (but can occur antepartum and in postpartum<br />

women who are not breastfeeding)<br />

(3) Usual mode of transmission is inoculation of breast milk from the infant's nasopharyngeal secretions;<br />

cracked nipples can also be a route of entry for organisms.<br />

b. Clinical presentation: painful swollen breast consistent with cellulitis, high fever, may result in abscess<br />

formation. Key is differentiating from blocked duct and abscess.<br />

474


GYNECOLOGIC AND OBSTETRIC DISORDERS<br />

c. Management<br />

(1) Simple mastitis<br />

(a) Warm compresses and analgesics for symptomatic relief<br />

(b) Breastfeeding should be continued to assure continued drainage; the infant will not be harmed<br />

because he or she is already colonized.<br />

(c) Oral antibiotics (penicillinase-resistant, eg, dicloxacillin or a cephalosporin)<br />

(2) Mastitis with abscess formation<br />

(a)<br />

B. Postpartum hemorrhage<br />

Refer to a surgeon for incision and drainage (in the operating room).<br />

(b) Continued drainage with a breast pump is indicated, because higher concentrations of bacteria<br />

may be harmful to the infant; some authors recommend discontinuation of breastfeeding until the<br />

infection has cleared.<br />

1. >500 ml blood loss in the first 24 hours<br />

2. Look for the 4 T's<br />

a. Tone: uterine atony<br />

b. Trauma: cervical laceration<br />

c. Tissue: retained placenta<br />

d. Thrombin: disseminated intravascular coagulation<br />

3. Management<br />

a. Resuscitation with crystalloid and blood as needed. Vital sign abnormalities occur late because of<br />

autotransfusion from placenta.<br />

b. Uterine massage<br />

c. Oxytocin<br />

d. Misoprostol<br />

C. Uterine inversion<br />

1. Traction on the umbilical cord during delivery can result in partial or complete uterine inversion.<br />

2. Clinical presentation<br />

a. Partial inversion -a- crater-like depression over the suprapubic area and descended uterine fundus and<br />

cervix on vaginal examination<br />

b. Complete inversion -a- uterus is outside the body and associated with immediate life-threatening hemorrhage<br />

3. Management<br />

a. Immediate anesthesiology and obstetrics consult<br />

b. Begin resuscitation: two large-bore IV lines (crystalloid) and blood crossmatch<br />

c. Tocolytic drugs (terbutaline, ritodrine, MgS0 4<br />

) to relax the uterus so that manual reimplantation can be<br />

achieved<br />

d. Try to reimplant the uterus without removing the placenta (t risk of hemorrhage); must place back through<br />

cervix.<br />

e. Transport patient to OR for reimplantation and possible exploratory laparotomy.<br />

f. Broad-spectrum antibiotics<br />

D. Pituitary infarction (Sheehan's syndrome)<br />

1. Necrosis occurs because of decreased perfusion of the pituitary during delivery.<br />

2. Symptoms caused by failure of function of the hormones.<br />

a. Prolactin: agalactorrhea, amenorrhea<br />

b. Hypothyroid symptoms: cold, constipated, hair loss, bradycardia<br />

c. Adrenal insufficiency: hypoglycemia, hyponatremia<br />

3. Management: replace deficient hormones<br />

a. Patient may require acute steroid, electrolyte management, glucose, thyroid hormone<br />

b. Endocrine consultation<br />

475


GYNECOLOGIC AND OBSTETRIC DISORDERS: PRACTICE CLINICAL SCENARIOS<br />

GYNECOLOGIC AND OBSTETRIC DISORDERS:<br />

PRACTICE CLINICAL SCENARIOS<br />

Answers immediately follow the practice clinical scenarios.<br />

Scenario A<br />

Presentation: A 25-year-old woman presents with a painless rash that has been in the vaginal area for over<br />

a week. On examination, you see heaped-up edges surrounding a painless ulcer.<br />

What is the diagnosis?<br />

Scenario B<br />

Presentation: A 23-year-old woman presents with copious amounts of gray vaginal discharge and a fishy<br />

odor. On physical examination, you see red spots on her cervix but no cervical motion tenderness.<br />

What is the diagnosis?<br />

Scenario C<br />

Presentation: A sexually active young woman complains of bilateral lower abdominal pain for several days<br />

(often following menses) associated with an abnormal vaginal discharge. She has fever> 100.4°F (38°C),<br />

chills, and general malaise.<br />

Physical examination: On physical examination, there is lower abdominal tenderness, cervical motion<br />

tenderness, and bilateral adnexal tenderness.<br />

What is the diagnosis?<br />

Scenario D<br />

Presentation: A 20-year-old woman presents with abrupt onset pain lateralized in the pelvis. The pain is<br />

10/1 0 and started after intercourse. She has had no other symptoms, discharge, or fever.<br />

What is the diagnosis?<br />

Scenario E<br />

Presentation: A 28-year-old pregnant woman at 32 weeks gestational age presents complaining of<br />

right-sided chest pain and shortness of breath. She is tachycardic and tachypneic and has a low oxygen<br />

saturation. Her legs are swollen, and she has just taken a long car trip.<br />

What is the diagnosis?<br />

Scenario F<br />

Presentation: EMS arrives with a visibly gravid woman who is actively seizing.<br />

What is the diagnosis?<br />

476


GYNECOLOGIC AND OBSTETRIC DISORDERS: PRACTICE CLINICAL SCENARIOS<br />

ScenarioG<br />

Presentation: A woman at 35 weeks gestational age comes in feeling general malaise, mild headaches,<br />

nausea, and right upper quadrant pain. She is swollen. Laboratory studies show anemia and<br />

thrombocytopenia and an increased LOH, liver function tests, and uric acid.<br />

What is the diagnosis?<br />

Scenario H<br />

Presentation: A 19-year-old Gl woman presents at 8 weeks with nausea and vomiting for a week. She<br />

states that she has not been able to keep anything down despite following all of her mother-in-law's<br />

recommendations. She tried the vitamin B 6<br />

but "it just came right back up." Her urinalysis shows large<br />

ketones. You give IV fluids containing glucose and antiemetics, but she continues to vomit and the ketones<br />

do not clear.<br />

What is the diagnosis?<br />

Scenario I<br />

Presentation: A 35-year-old woman at 30 weeks gestational age presents for an upper respiratory infection.<br />

She is noted at triage to have a blood pressure of 145/86 mm Hg. She noted that her obstetrician said her<br />

blood pressure was a little high for the first time ever at her last OB visit. She has no headache, shortness of<br />

breath, chest pain, right upper quadrant pain, blurry vision, or change in mental status. She has not been<br />

taking over-the-counter medications other than acetaminophen. Laboratory tests were not believed to be<br />

indicated in this patient.<br />

What is the diagnosis?<br />

ScenarioJ<br />

Presentation: A 21-year-old woman presents with right lower quadrant pain, fever, and vaginal discharge.<br />

She is febrile and slightly tachycardic with normal blood pressure, respirations, and oxygenation. Pelvic<br />

examination reveals right adnexal tenderness and mass. HCC is negative.<br />

What is the diagnosis?<br />

Scenario K<br />

Presentation: A 74-year-old woman presents with intermittent pelvic pressure, stress incontinence, and<br />

multiple urinary tract infections over the past several months. Now she complains of a vaginal mass.<br />

What is the diagnosis?<br />

477


GYNECOLOGIC AND OBSTETRIC DISORDERS: PRACTICE CLINICAL SCENARIOS<br />

ANSWERS TO PRACTICE CLINICAL SCENARIOS<br />

Scenario A<br />

Diagnosis: syphi [is<br />

Scenario B<br />

Diagnosis: trichomoniasis<br />

Scenario C<br />

Diagnosis: pelvic inflammatory disease<br />

Key facts:<br />

This classic clinical scenario is seen in only one-third of cases.<br />

Constitutional symptoms (fever> 100.4 °F [38°C], chills, and general malaise) are frequently present.<br />

GI complaints, such as nausea and vomiting, may also be present but are less common.<br />

Scenario D<br />

Diagnosis: ovarian torsion<br />

Scenario E<br />

Diagnosis: pulmonary embolism<br />

Scenario F<br />

Diagnosis: eclampsia (preeclampsia and grand ma/ seizures or coma)<br />

Scenario G<br />

Diagnosis: HELLP<br />

Scenario H<br />

Diagnosis: hypermemesis gravidarum<br />

Scenario I<br />

Diagnosis: gestational hypertension<br />

ScenarioJ<br />

Diagnosis: tuba-ovarian abscess<br />

Scenario K<br />

Diagnosis: pelvic organ prolapse<br />

478


;,:~t,,:·, ..<br />

.<br />

Volume II<br />

Pediatric Emergencies<br />

Toxicologic Disorders<br />

Endocrine, Metabolic, and Nutritional<br />

Disorders<br />

Environmental Disorders<br />

Psychobehavioral Disorders<br />

Hematologic Disorders<br />

Oncologic Disorders<br />

Systemic Infectious Disorders<br />

Immune System Disorders<br />

Renal and Urologic Disorders<br />

Cutaneous Disorders<br />

Emergency Medical Services<br />

Procedures and Skills Integral to the<br />

Practice of Emergency Medicine<br />

Other Components of the Practice of<br />

Emergency Medicine<br />

Ohio 1·<br />

·~/- · ACEP<br />

·"?:,:: '."\. American College of Emergency Physicians<br />

.. :/,,-· .•. , .. c..'


Dr. Carol Rivers'<br />

PREPARING FOR THE WRITTEN BOARD EXAM<br />

~, IN EMERGENCY MEDICINE<br />

Eighth Edition<br />

Volume 2<br />

./--"'<br />

Ohio<br />

ACEP<br />

American College of Emergency Physicians<br />

Advocacy I Education I Leadership


Published by:<br />

Ohio Chapter, American College of Emergency Physicians (Ohio ACEP)<br />

3510 Snouffer Road<br />

Suite 100<br />

Columbus, Ohio 43235<br />

(614) 792-6506<br />

(888) OHACEP4 (642-2374)<br />

ohacep@ohacep.org<br />

www.ohacep.org<br />

Printed in the United States of America<br />

Eighth Edition<br />

ISBN 978-0-9977410-0-l<br />

Date of original release: January 1992<br />

Most recent update: January 2017<br />

Copyright ©2017 Ohio Chapter, American College of Emergency Physicians. All<br />

rights reserved. No part of this publication may be reproduced or transmitted in<br />

any form, electronic or mechanical, including photocopying, recording, storage<br />

in any information retrieval system, or otherwise, without the prior written<br />

permission of the publisher.<br />

DISCLAIMER<br />

Ohio Chapter, American College of Emergency Physicians publishes information<br />

believed to be in agreement with the accepted standards of practice at the date<br />

of publication. Due to the continual state of change in diagnostic procedures,<br />

treatment, and drug therapy, Ohio ACEP and the writers and editors are not<br />

responsible for any errors or omissions. In the practice of medicine, the reader<br />

should confirm the use of any information with other sources.


The object of education is not learning, but discipline<br />

and enlightenment of the mind.<br />

Woodrow Wilson


Dr. Carol Rivers' Preparing for the Written Board Exam in<br />

Emergency Medicine, Eighth Edition<br />

EDITORS<br />

SENIOR ASSOCIATE EDITOR<br />

Ann M. Dietrich, MD, FAAP, FACEP<br />

Associate Professor of Primary Care (Lead), Ohio University Heritage College of Osteopathic Medicine; Pediatric<br />

Associate Medical Director, MedFlight of Ohio, Columbus, Ohio<br />

SECTION EDITORS<br />

Thomas Green, DO, MMM, MPH, CPE, FACOEP, FACEP<br />

Associate Dean, Clinical Affairs; Chief Academic Officer (interim)-HEARTLand Network OPTI and Associate Professor<br />

Family Medicine, Des Moines University College of Osteopathic Medicine; Associate Professor Emergency Medicine/<br />

Family Medicine, Midwestern University-Chicago College of Osteopathic Medicine; Attending Physician, Emergency<br />

Department, Central Iowa Healthcare, Marshalltown, Iowa<br />

Joseph P. Martinez, MD, FACEP, FAAEM<br />

Associate Professor of Emergency Medicine and Medicine, Assistant Dean for Student Affairs, and Assistant Dean for<br />

Clinical Medical Education and Residency Programs Liaison, University of Maryland School of Medicine, Baltimore,<br />

Maryland<br />

Amal Mattu, MD, FACEP<br />

Professor and Vice Chair of Education, Co-Director, Emergency Cardiology Fellowship, University of Maryland School<br />

of Medicine, Baltimore, Maryland<br />

Victor J. Scali, DO, FACOEP-D<br />

Director of EM, Residency Education, and Associate Professor, Department of Emergency Medicine, Rowan University<br />

School of Osteopathic Medicine, Stratford, New Jersey<br />

Howard A. Werman, MD, FACEP<br />

Professor of Clinical Emergency Medicine, The Ohio State University Wexner Medical Center; Medical Director,<br />

MedFlight, Columbus, Ohio<br />

Sandra L. Werner, MD, RDMS, FACEP<br />

Operations Director, Emergency Medicine, Associate Director, Emergency Medicine Residency Program, MetroHealth<br />

Medical Center; Associate Professor, School of Medicine, Case Western Reserve University School of Medicine,<br />

Cleveland, Ohio<br />

CONTRIBUTORS<br />

Michael Abraham, MD, MS<br />

Clinical Assistant Professor, Department of Emergency Medicine, University of Maryland School of Medicine,<br />

Baltimore, Maryland<br />

Nervous System Disorders<br />

Rudd J. Bare, MD, MMEL<br />

Chief of Emergency Medicine; Faculty, Emergency Medicine Residency Program, Western Reserve Hospital, Cuyahoga<br />

Falls, Ohio; Summa Health System EM Residency, Akron City Hospital, Akron, Ohio; Masters in Medical Education and<br />

Administration, University of New England Medical School, Portland, Maine; Assistant Professor of Emergency Medicine,<br />

NEOMED, Rootstown, Ohio; Professor of Emergency Medicine, Ohio University, Athens, Ohio<br />

Environmental Disorders<br />

IV


Michelle Blanda, MD, FACEP<br />

Professor and Chair Emeritus of Emergency Medicine, Northeast Ohio Medical University, Akron City Hospital,<br />

Summa Health System, Akron, Ohio<br />

Renal and Urologic Disorders<br />

Michael C. Bond, MD, FACEP, FAAEM<br />

Associate Professor, Residency Program Director, Department of Emergency Medicine, University of Maryland School<br />

of Medicine, Baltimore, Maryland<br />

Orthopedic Emergencies<br />

Laura J. Bontempo, MD, MEd, FAAEM<br />

Assistant Professor, Emergency Medicine, University of Maryland School of Medicine, Baltimore, Maryland<br />

Cutaneous Disorders<br />

Brian Browne, MD, FACEP<br />

Professor of Emergency Medicine, Chair, Department of Emergency Medicine, University of Maryland School of<br />

Medicine, Baltimore, Maryland<br />

Cutaneous Disorders<br />

Luca R. Delatore, MD<br />

Medical Director of the James Emergency Department, Assistant Professor, Department of Emergency Medicine,<br />

The Ohio State University Wexner Medical Center, Columbus, Ohio<br />

Oncologic Disorders<br />

Paul de Saint Victor, MD, FACEP, MHA, CPE<br />

Associate Director, Emergency Medicine Residency, Mercy St. Vincent Medical Center, Toledo, Ohio; President,<br />

Emergency Department Consultants<br />

Additional Tips for Good Performance, How to Take a Multiple-Choice Exam<br />

Sarah B. Dubbs, MD<br />

Clinical Assistant Professor, Assistant Residency Program Director, Department of Emergency Medicine, University<br />

of Maryland School of Medicine, Baltimore, Maryland<br />

Gynecologic and Obstetric Disorders<br />

Scott Felten, MD, FACEP<br />

US Acute Care Solutions Residency Director, Tulsa, Oklahoma<br />

Immune System Disorders<br />

William R. Fraser, DO, FACOEP-D<br />

Emergency Medicine Residency Director, Doctor's Hospital Ohio Health, Columbus, Ohio<br />

Abdominal and Gastrointestinal Disorders<br />

Hannah L. Hays, MD<br />

Assistant Professor, Department of Emergency Medicine and Pediatrics, The Ohio State University Wexner Medical<br />

Center and Nationwide Children's Hospital and Central Ohio Poison Center, Columbus, Ohio<br />

Toxicologic Disorders<br />

Colin G. Jaide, MD, FACEP, FAAEM<br />

Associate Pr fessor of Emergency Medicine, Attending Physician of Emergency Medicine and Hyperbaric Medicine,<br />

The Ohio St te University Wexner Medical Center, Columbus, Ohio<br />

Hematologi Disorders<br />

V


Jonathan Keary, MD, FACEP<br />

Staff Physician, Emergency Services Institute, Cleveland Clinic, Cleveland, Ohio<br />

Endocrine, Metabolic, and Nutritional Disorders<br />

Benjamin J. Lawner, DO, MS, EMT-P, FAAEM<br />

Assistant Professor, Department of Emergency Medicine, University of Maryland School of Medicine, Deputy EMS<br />

Medical Director, Baltimore City Fire Department, Baltimore, Maryland<br />

Emergency Medical Services<br />

Le N. Lu, MD, MS<br />

Clinical Assistant Professor, Director, Pediatric Emergency Medicine Education, Department of Emergency Medicine,<br />

University of Maryland School of Medicine; Director, Pediatric Emergency Department, Upper Chesapeake Medical<br />

Center, Baltimore, Maryland<br />

Pediatric Emergencies<br />

Patrick J. Maloney, MD<br />

Director, Pediatric Emergency Services, Mission Hospital and Mission Children's Hospital, Asheville, North Carolina<br />

Traumatic Disorders<br />

Laura Matrka, MD<br />

Assistant Professor, Otolaryngology-Head & Neck Surgery, The Ohio State University Wexner Medical Center,<br />

Columbus, Ohio<br />

Head, Ear, Eye, Nose, and Throat Disorders<br />

Stacy McCallion, MD<br />

Summa Health System, Akron City Hospital, Akron, Ohio; Associate Professor of Emergency Medicine, NEOMED,<br />

Rootstown, Ohio; Professor of Emergency Medicine, Ohio University, Athens, Ohio<br />

Environmental Disorders<br />

Michael J. McCrea, MD, FACEP, FAAEM<br />

Assistant Residency Director, Simulation Education Director, Emergency Medicine Residency, Mercy St. Vincent<br />

Medical Center, Toledo, Ohio<br />

Cardiovascular Disorders<br />

Jillian L. McGrath, MD, FACEP<br />

Assistant Professor of Emergency Medicine, Department of Emergency Medicine, The Ohio State University Wexner<br />

Medical Center, Columbus, Ohio<br />

Toxicologic Disorders<br />

Colleen M. McQuown, MD, FACEP<br />

Director of Research Education, Department of Emergency Medicine, Summa Akron City Hospital; Associate Professor,<br />

Northeast Ohio Medical University, Rootstown, Ohio<br />

Other Components of the Practice of Emergency Medicine<br />

Siamak Moayedi, MD<br />

Assistant Professor, Department of Emergency Medicine, University of Maryland School of Medicine, Baltimore, Maryland<br />

Systemic Infectious Disorders<br />

Sreeja M. Natesan, MD<br />

Assistant Professor, Duke University Medical Center, Durham, North Carolina<br />

Thoracic and Respiratory Disorders<br />

VI


Sarah K. Sommerkamp, MD, ROMS<br />

Assistant Professor, Department of Emergency Medicine, University of Maryland School of Medicine, Baltimore, Maryland<br />

Gynecologic and Obstetric Disorders<br />

Matthew Tabbut, MD, FACEP<br />

Attending Physician, Department of Emergency Medicine, MetroHealth Medical Center; Assistant Professor, Case<br />

Western University School of Medicine, Cleveland, Ohio<br />

Procedures and Skills Integral to the Practice of Emergency Medicine<br />

Laura R. Thompson, MD, MS, FACEP<br />

Assistant Professor, Department of Emergency Medicine, The Ohio State University Wexner Medical Center,<br />

Columbus, Ohio<br />

Psychobehavioral Disorders<br />

Mercedes Torres, MD<br />

Clinical Assistant Professor, Department of Emergency Medicine, University of Maryland School of Medicine,<br />

Baltimore, Maryland<br />

Systemic Infectious Disorders<br />

Melissa Tscheiner, MD, FACEP<br />

Adjunct Assistant Professor, University of North Carolina School of Medicine-Asheville; Department of Emergency<br />

Medicine, Mission Hospital, Asheville, North Carolina<br />

Musculoskeletal Disorders (Nontraumatic)<br />

Travis Ulmer, MD, FACEP<br />

Vice President, US Acute Care Solutions, Canton, Ohio; Clinical Assistant Professor of Emergency Medicine,<br />

Ohio University Heritage College of Osteopathic Medicine, Athens, Ohio<br />

Head, Ear, Eye, Nose, and Throat Disorders<br />

PRODUCTION<br />

Laura L. Tiberi, MA, CAE<br />

Ohio ACEP Executive Director<br />

Holly J.F. Dorr, MBA, CAE<br />

Ohio ACEP Deputy Executive Director, Editorial and Production Coordinator<br />

Susan E. Aiello, ELS<br />

Medical Editor<br />

Sheryl Lazenby<br />

Designer<br />

vii


,~'


INTRODUCTION<br />

VOLUME I<br />

Foreword ........................................................................................................................................................................ x<br />

Introduction .................................................................................................................................................................. xi<br />

Recommended Study Plan ........................................................................................................................................... xiii<br />

Cardiovascular Disorders .............................................................................................................................................. 1<br />

Head, Ear, Eye, Nose, and Throat Disorders ................................................................................................................ 97<br />

Abdominal and Gastrointestinal Disorders ............................................................................................................... 169<br />

Thoracic and Respiratory Disorders .......................................................................................................................... 231<br />

Traumatic Disorders .................................................................................................................................................. 297<br />

Orthopedic Emergencies ........................................................................................................................................... 343<br />

Musculoskeletal Disorders (Nontraumatic) ............................................................................................................... 393<br />

Nervous System Disorders ........................................................................................................................................ 409<br />

Gynecologic and Obstetric Disorders ....................................................................................................................... 443<br />

VOLUME II<br />

Pediatric Emergencies ............................................................................................................................................... 489<br />

Toxicologic Disorders ................................................................................................................................................ 543<br />

Endocrine, Metabolic, and Nutritional Disorders ...................................................................................................... 591<br />

Environmental Disorders ........................................................................................................................................... 633<br />

Psychobehavioral Disorders ...................................................................................................................................... 673<br />

Hematologic Disorders ............................................................................................................................................. 711<br />

Oncologic Disorders ................................................................................................................................................. 743<br />

Systemic Infectious Disorders ................................................................................................................................... 759<br />

Immune System Disorders ........................................................................................................................................ 783<br />

Renal and Urologic Disorders ................................................................................................................................... 803<br />

Cutaneous Disorders ................................................................................................................................................. 829<br />

Emergency Medical Services ..................................................................................................................................... 869<br />

Procedures and Skills Integral to the Practice of Emergency Medicine ..................................................................... 883<br />

Other Components of the Practice of Emergency Medicine ...................................................................................... 941<br />

Mechanics of the Written Board Exam ....................................................................................................................... 975<br />

Additional Tips for Good Performance ....................................................................................................................... 979<br />

ix


PEDIATRIC EMERGENCIES<br />

Life-Threatening Cardiac Dysrhythmias .................................................................................................................... 495<br />

Cardiopulmonary Resuscitation ................................................................................................................................ 495<br />

Airway ................................................................................................................................................................ 495<br />

Breathing ............................................................................................................................................................ 497<br />

Circulation ......................................................................................................................................................... 498<br />

Chest Compressions ........................................................................................................................................... 498<br />

Defibrillation and Cardioversion ........................................................................................................................ 499<br />

Vascular Access .................................................................................................................................................. 499<br />

Life-Threatening Cardiac Arrhythmias ................................................................................................................. 500<br />

Unique Issues in Neonates and Infants ..................................................................................................................... 501<br />

Risk Factors Associated with Neonatal Cardiopulmonary Arrest.. ........................................................................ 501<br />

APGAR Scoring .................................................................................................................................................. 503<br />

Meconi um Staining ............................................................................................................................................ 503<br />

Neonatal Seizures .............................................................................................................................................. 503<br />

Congenital Diaphragmatic Hernia ...................................................................................................................... 504<br />

Tracheoesophageal Fistula .................................................................................................................................. 504<br />

Omphalocele and Gastroschisis ......................................................................................................................... 504<br />

Necrotizing Enterocolitis .................................................................................................................................... 505<br />

Cyanosis ............................................................................................................................................................. 505<br />

Congestive Heart Failure .................................................................................................................................... 506<br />

Jaundice in the Newborn .................................................................................................................................... 506<br />

Apparent Life-Threatening Event ......................................................................................................................... 507<br />

Sudden Infant Death Syndrome .......................................................................................................................... 508<br />

Congenital Heart Disease .......................................................................................................................................... 508<br />

Cyanotic Heart Lesions ....................................................................................................................................... 508<br />

Acyanotic Heart Lesions ..................................................................................................................................... 509<br />

Ductal Dependent Lesions ................................................................................................................................. 510<br />

Congestive Heart Failure .................................................................................................................................... 510<br />

Airway Emergencies .................................................................................................................................................. 511<br />

Upper Airway ..................................................................................................................................................... 511<br />

Lower Airway ..................................................................................................................................................... 513<br />

Pediatric Infectious Diseases ..................................................................................................................................... 518<br />

The Febrile Child ................................................................................................................................................ 518<br />

Bacteremia and Sepsis ........................................................................................................................................ 519<br />

Meningitis .......................................................................................................................................................... 520<br />

Pneumonia ......................................................................................................................................................... 521<br />

Pertussis (Whooping Cough) ............................................................................................................................... 524<br />

Otitis Media ....................................................................................................................................................... 524<br />

Urinary Tract Infection ........................................................................................................................................ 526<br />

Kawasaki Disease (Mucocutaneous Lymph Node Syndrome) .............................................................................. 527<br />

Pediatric Seizures ...................................................................................................................................................... 529<br />

Cerebrospinal Fluid Shunts ....................................................................................................................................... 531<br />

Pediatric Gastrointestinal/Genitourinary Emergencies .............................................................................................. 533<br />

Child Abuse ............................................................................................................................................................... 537<br />

Child Neglect ..................................................................................................................................................... 537<br />

Munchausen Syndrome by Proxy (Polle Syndrome) ............................................................................................ 537<br />

Sexual Abuse ...................................................................................................................................................... 538<br />

Physical Abuse ................................................................................................................................................... 538<br />

489


PEDIATRIC EMERGENCIES: SELF-ASSESSMENT QUESTIONS<br />

PEDIATRIC EMERGENCIES: SELF-ASSESSMENT QUESTIONS<br />

1. The appropriate endotracheal tube for a 6-year-old is:<br />

(a) 4.5 uncuffed<br />

(b) 4.5 cuffed<br />

(c) 5.5 uncuffed<br />

(d) 5.5 cuffed<br />

2. Relative to the location of the adult airway in the neck, the child's airway is:<br />

(a) More anterior and higher<br />

(b) More posterior and higher<br />

(c) More posterior and lower<br />

(d) More anterior and lower<br />

3. Which of the following drugs should not be administered via the endotracheal route?<br />

(a) Atropine<br />

(b) Calcium chloride<br />

(c) Epinephrine<br />

(d) Naloxone<br />

4. Which of the following is not considered a risk factor for sudden infant death syndrome?<br />

(a) Maternal age >35 years old<br />

(b) Low birth weight<br />

(c) Smoking and drug abuse by mother<br />

(d) None of the above<br />

5. All of the following statements regarding congenital diaphragmatic hernias are accurate except:<br />

(a) Left-sided hernias are more common than right-sided hernias.<br />

(b) Associated polyhydramnios is common.<br />

(c) Treatment is intubation, orogastric tube placement, IV hydration, and surgical repair.<br />

(d) Right-sided hernias are called Bochdalek hernias.<br />

6. Which of the following is not a structural component of the tetralogy of Fallot?<br />

(a) Pulmonic stenosis<br />

(b) Ventricular septa! defect<br />

(c) Left ventricular hypertrophy<br />

(d) Dextroposition and overriding of the aorta<br />

7. Initial management of a hypercyanotic ("Tet") spell in a child with tetralogy of Fallot should include all of the<br />

following except:<br />

(a) Administration of supplemental oxygen<br />

(b) Placement of the child in the prone knee-chest position<br />

(c) Morphine 0.1 mg/kg IV, IM, or SC<br />

(d) Propranolol 0.05-0.1 mg/kg IV<br />

8. Which of the lesions listed below is an acyanotic heart lesion?<br />

(a) Transposition of the great vessels<br />

(b) Aortic stenosis<br />

(c) Total anomalous pulmonary venous return<br />

(d) Tricuspid atresia<br />

9. Which of the following statements regarding necrotizing enterocolitis is least accurate?<br />

(a) Premature infants with very low birth weight are most commonly affected.<br />

(b) It typically develops during the first 2 weeks of life.<br />

(c) The first and most frequent clinical finding is abdominal wall erythema.<br />

(d) Pneumatosis intestinal is is the radiographic hallmark.<br />

490


PEDIATRIC EMERGENCIES: SELF-ASSESSMENT QUESTIONS<br />

10. The presence of biphasic stridor on examination localizes the airway obstruction __ _<br />

(a) Above the larynx<br />

(b) At the larynx<br />

(c) Below the larynx<br />

(d) Below the carina<br />

11. The usual causative organism of bronchiolitis is:<br />

(a) Parainfluenza virus<br />

(b) Respiratory syncytial virus<br />

(c) Adenovirus<br />

(d) Influenza B virus<br />

12. The usual causative organism of croup is:<br />

(a) Respiratory syncytial virus<br />

(b) Parainfluenza virus<br />

(c) Adenovirus<br />

(d) Echovirus<br />

13. Anticholinergic agents (atropine, ipratropium bromide) produce their beneficial effects in asthma by:<br />

(a) Increasing cyclic GMP<br />

(b) Decreasing cyclic GMP<br />

(c) Increasing cyclic AMP<br />

(d) Decreasing cyclic AMP<br />

14. All of the following poisonings may be associated with hyperpyrexia except:<br />

(a) Atropine<br />

(b) Salicylates<br />

(c) Amphetamines<br />

(d) Acetaminophen<br />

15. Under normal land conditions, the greatest amount of heat loss from the body occurs via:<br />

(a) Radiation<br />

(b) Evaporation<br />

(c) Convection<br />

(d) Conduction<br />

16. The most common bacterial causative organism of otitis media is:<br />

(a) Streptococcus pneumoniae<br />

(b) Haemophilus influenzae nontypable<br />

(c) Moraxella catarrha/is<br />

(d) Staphyloccoccus aureus<br />

17. The pathogens most often responsible for the production of pneumonia in children :C:5years old are:<br />

(a) Haemophilus influenzae and Streptococcus pneumoniae<br />

(b) Haemophi/us influenzae and Mycoplasma pneumoniae<br />

(c) Mycop/asma pneumoniae and Streptococcus pneumoniae<br />

(d) Mycop/asma pneumoniae and viruses<br />

18. A 3-week-old infant is brought in for evaluation of a staccato cough. His mother states that he was born by normal<br />

spontaneous vaginal delivery and that, aside from some nasal congestion and eye inflammation over the past<br />

few days, he had been well until yesterday. On examination, the child is afebrile but quite tachypneic and has<br />

bilateral conjunctivitis. Pulse oximetry reveals a saturation of 94%, and chest radiograph reveals bilateral interstitial<br />

/~ infiltrates. The organism most likely to produce this constellation of signs and symptoms in an infant this age is:<br />

(a) Group B streptococci<br />

(b) Chlamydia trachomatis<br />

(c) Staphyloccoccus aureus<br />

(d) Mycoplasma pneumoniae<br />

491


PEDIATRIC EMERGENCIES: SELF-ASSESSMENT QUESTIONS<br />

19. The most common bacterial pathogen in all age groups beyond the newborn period is:<br />

(a) Streptococcus pneumoniae<br />

(b) Haemphilus influenzae type B<br />

(c) Staphyloccoccus aureus<br />

(d) Group B streptococci<br />

20. When evaluating a child who presents with pneumonia, findings on the CBC (although certainly not diagnostic) can<br />

suggest a particular etiologic agent. The classic WBC findings that occur in association with particular pathogens are<br />

listed below. Which of the following findings is inappropriately matched?<br />

(a) Normal WBC count and differential/Mycop/asma pneumoniae<br />

(b) High WBC count with a left shift/Streptococcus pneumoniae<br />

(c) Marked lymphocytosis/viruses<br />

(d) Eosinophilia/Ch/amydia trachomatis<br />

21. In children, cardiac arrest is most commonly secondary to:<br />

(a) A primary cardiac event<br />

(b) Respiratory arrest<br />

(c) Shock<br />

(d) None of the above<br />

22. The easiest and most accurate method used in the selection of an endotracheal tube for pediatric intubation is:<br />

(a) A formula that estimates the tube size based on the child's age<br />

(b) A tube size that equals the diameter of the tip of the child's little finger<br />

(c) A tube size that approximates the size of the child's nostril<br />

(d) A length-based resuscitation tape<br />

23. You are attempting to defibrillate a child weighing 20 kg. What dose should be used for the initial attempt?<br />

(a) 20 joules<br />

(b) 40 joules<br />

(c) 60 joules<br />

(d) 80 joules<br />

24. When using the endotracheal route to administer epinephrine during pediatric resuscitation, the concentration of<br />

epinephrine should be ___ that administered when the IV route is used.<br />

(a) The same as<br />

(b) Half<br />

(c) Double<br />

(d) Ten times<br />

25. You have just delivered a baby in a rural emergency department. At 1 minute, the infant is limp, has a slow and<br />

irregular respiratory effort, a heart rate of 80 beats per minute, acrocyanosis, and some reflex irritability. The<br />

appropriate 1-minute APGAR score for this infant is:<br />

(a) 2<br />

(b) 3<br />

(c) 4<br />

(d) 5<br />

26. You are evaluating a 2-year-old child for possible retropharyngeal abscess and decide to obtain a soft-tissue lateral<br />

radiograph of the neck. For an accurate interpretation, the radiograph must be taken:<br />

(a)<br />

During inspiration with the neck in slight extension<br />

(b) During inspiration with the neck in slight flexion<br />

(c) During expiration with the neck in slight extension<br />

(d) During expiration with the neck in slight flexion<br />

492


PEDIATRIC EMERGENCIES: SELF-ASSESSMENT QUESTIONS<br />

27. Which of the following statements regarding bronchiolitis is inaccurate?<br />

(a) It usually affects infants 2 months to 2 years old.<br />

(b) The causative organism is generally respiratory syncytial virus.<br />

(c) Apnea is a serious complication, and premature infants


PEDIATRIC EMERGENCIES: SELF-ASSESSMENT QUESTIONS<br />

36. The drug of choice for the treatment of supraventricular tachycardia in children is:<br />

(a) Adenosine<br />

(b) Propranolol<br />

(c) Verapamil<br />

(d) Procainamide<br />

37. Which of the following statements regarding prostaglandin E, is inaccurate:<br />

(a) It is a potent dilator of the ductus arteriosus.<br />

(b) It can be used to maintain the patency of (or reopen) the ductus arteriosus in infants with ductal-dependent<br />

congenital heart lesions.<br />

(c) Significant adverse effects associated with its use include apnea and hypotension.<br />

(d) The initial infusion rate is 1 mcg/kg/min.<br />

38. The organism(s) most commonly responsible for causing CSF shunt infections are:<br />

(a) Staphylococcus spp<br />

(b) Klebsiel/a<br />

(c) Gram-negative organisms<br />

(d) Group A streptococci<br />

39. The sign/symptom most commonly present in association with CSF shunt infection is:<br />

(a)<br />

Headache<br />

(b) Lethargy<br />

(c) Fever<br />

(d) Meningismus<br />

40. A 10-day-old infant is brought in for evaluation of respiratory distress. His mother states that he was born by normal<br />

spontaneous vaginal delivery and that, aside from some nasal congestion and coughing over the past few days, he<br />

had been well until yesterday. On examination, the child is afebrile but quite tachypneic with moderate retractions.<br />

His pulse is 80 beats per minute, and pulse oximetry is 89% on 100% oxygen. What is the next step in management ,~<br />

of this patient?<br />

(a) Chest radiograph<br />

(b) Positive-pressure ventilation<br />

(c) Begin chest compressions<br />

(d) Epinephrine 0.01 mg/kg IV<br />

41. A 14-day-old infant is brought in for evaluation of respiratory distress. His mother states that he was born by normal<br />

spontaneous vaginal delivery but has been having difficulty feeding over the past 24 hours. On examination, the child<br />

is afebrile but tachypneic with moderate retractions and cyanotic with pulse oximetry of 80%. After placing him on<br />

100% oxygen, an arterial blood gas is obtained and the Pa0 2<br />

is 160 mm Hg. Which of the following is least likely?<br />

(a) Sepsis<br />

(b) Congenital heart disease with right to left shunt<br />

(c) Pneumonia<br />

(d) Primary seizure disorder<br />

ANSWERS<br />

1. C 8. b<br />

2. a 9. C<br />

3. b 10. C<br />

4. a 11. b<br />

5. d 12. b<br />

6. C 13. b<br />

15.<br />

16.<br />

17.<br />

18.<br />

19.<br />

20.<br />

a<br />

a<br />

d<br />

b<br />

a<br />

C<br />

22.<br />

23.<br />

24.<br />

25.<br />

26.<br />

27.<br />

d 29. C<br />

b 30. b<br />

d 31. a<br />

C 32. C<br />

a 33. C<br />

d 34. b<br />

7. d 14. d 21. b 28. d 35. a<br />

36. a<br />

37. d<br />

38. a<br />

39. C<br />

40. b<br />

41. b<br />

Use the pre-chapter multiple choice question worksheet (page xvi) to record and determine the percentage of correct<br />

answers for this chapter.<br />

494


PEDIATRIC EMERGENCIES<br />

A. Unlike in adults, in whom rhythm disturbances are primary cardiac disturbances, in children rhythm<br />

disturbances are frequently secondary problems.<br />

1. Children at risk of dysrhythmias may have congenital or acquired heart disease, systemic disease, intoxication,<br />

or acute hemodynamic alteration.<br />

2. Cardiac arrest is usually secondary to respiratory arrest. Circulatory failure (shock) is the second most common<br />

precipitating event.<br />

3. Symptomatic sinus bradycardia is generally the result of prolonged hypoxemia or increased vagal tone.<br />

4. Primary cardiac events (eg, ventricular fibrillation) are rare.<br />

B. Dysrhythmias by age<br />

Table 31: Most Common Clinically Significant Dysrhythmias by Age<br />

Age (years)<br />


PEDIATRIC EMERGENCIES<br />

(3) The mass of adenoidal tissue is greater and the nasopharyngeal angle is more acute in young<br />

children, making nasopharyngeal intubation more difficult and more likely to result in damage to the<br />

nasopharyngeal soft tissues; therefore, orotracheal intubation under direct visualization is the preferred<br />

method of establishing initial airway control in a child.<br />

(4) The trachea is shorter (5 cm at birth); this increases the potential for accidental intubation of the right<br />

mainstem bronchus.<br />

b. General approach<br />

(1) Determine the proper tube size.<br />

(a)<br />

Use a length-based resuscitation tape (eg, Broselow® tape). This will predict the tracheal tube<br />

size and is useful in determining drug dosages. Tracheal tubes 0.5 mm smaller or larger than the<br />

estimated size should be readily available.<br />

(b) If a tape is unavailable, use the following formulas in children >2 years old:<br />

i. Uncuffed endotracheal tube: tracheal tube size= (age [in years]/4) + 4<br />

ii. Cuffed endotracheal tube: tracheal tube size = (age [in years]/4) + 3.5<br />

(c) The appropriate size tracheal tube for premature infants is 2.5-3.0 mm; for newborns 3.0-3.5<br />

mm; for 1-year-olds 4.0-4.5 mm, and for 4-year-olds 5.0 mm.<br />

(2) Prepare to intubate.<br />

(a)<br />

Place the head in the sniffing position.<br />

(b) Use the chin-lift or jaw-thrust to move the hypotonic mandibular tissue out of the way.<br />

(c) If the respiratory effort is inadequate, preoxygenate with 100% oxygen for 5 minutes (if possible).<br />

(d) There is insufficient evidence to recommend routine cricoid pressure to prevent aspiration during<br />

endotracheal intubation in children. Cricoid pressure should be stopped if it interferes with<br />

ventilation or the ease of ventilation.<br />

(3) Rapid-sequence intubation<br />

(a) Definition: simultaneous administration of potent sedative agent and rapid-acting paralytic<br />

drug as well as medications to ameliorate potential adverse effects of laryngoscopy to facilitate<br />

endotracheal intubation; safe and effective in children when used properly<br />

(b) Medications<br />

1. Pretreatment (controversial)<br />

• Atropine 0.02 mg/kg IV push: no longer recommended according to 2015 AHA<br />

guidelines (if used, the minimum 0.1 mg dose is no longer necessary according to 2015<br />

AHA guidelines, maximum 0.5 mg); theoretically blunts vagal stimulation and prevents<br />

bradycardia associated with laryngoscopy<br />

• Lidocaine 1-1.5 mg/kg IV push: theoretically blunts increase in intracranial pressure<br />

associated with intubation<br />

ii. Induction<br />

• Midazolam 0.1 mg/kg IV push: variable dosing<br />

• Ketamine 1-2 mg/kg IV push: although previously believed to increase intracranial pressure,<br />

newer data suggest this may not be the case.<br />

• Thiopental 3-5 mg/kg IV push: may cause hypotension, laryngospasm<br />

• Etomidate 0.3 mg/kg IV push: hemodynamically stable; caution in septic shock secondary to<br />

adrenal suppression<br />

• Propofol 1-2 mg/kg IV push: may cause hypotension, may be used in some institutions<br />

iii. Paralytic agents<br />

• Rocuronium 0.6-1.2 mg/kg IV push<br />

• Succinylcholine 1-1.5 mg/kg IV push, infants may require 1.5-2 mg/kg; Contraindications<br />

include children with underlying muscular disorders or extensive burns.<br />

(4) Confirm tracheal tube placement after intubation by:<br />

(a) Observing the chest for symmetric movement<br />

(b) Listening over the axillae to confirm equal breath sounds<br />

(c) Listening over the stomach to assure absent breath sounds<br />

(d) Obtaining a chest radiograph; the tip of the tracheal tube should be located above the carina at<br />

the level ofT2.<br />

496


PEDIATRIC EMERGENCIES<br />

(e) End-tidal CO 2<br />

detection<br />

i. In patients with a pulse, this is the best adjuvant method (most sensitive/specific) for<br />

verification of tube placement; however, the most accurate means is observation of the tube<br />

passing through the vocal cords.<br />

(f)<br />

ii. A false-negative reading can occur with severe circulatory collapse; therefore, if the CO 2<br />

is not<br />

detected during CPR, take another look with a laryngoscope.<br />

Clinical deterioration in a previously stable intubated patient receiving positive-pressure<br />

ventilation can be remembered by the mnemonic "DOPE":<br />

,Qislodged tube<br />

Qbstructed tube<br />

Tension fneumothorax<br />

fquipment failure<br />

(5) Rescue airway devices: supraglottic devices, eg, the laryngeal mask airway, may quickly be inserted<br />

blindly in skilled hands.<br />

Table 26: Laryngeal-Mask Airway Sizes<br />

Age (weight)<br />

Size<br />

Newborn (0-5 kg)<br />

3 months to 1 year old (5-1 0 kg)<br />

1-4 years old (10-20 kg)<br />

4-8 years old (20-30 kg)<br />

>8 years old (30-40 kg)<br />

Adolescent and adult<br />

1.5<br />

2<br />

2.5<br />

3<br />

4<br />

3. Surgical airways in children<br />

B. Breathing<br />

a. Needle cricothyrotomy with jet ventilation<br />

(1) Indications: inability to ventilate and inability to intubate in children 8 years old as an alternative to needle cricothyrotomy.<br />

1. Each breath should be delivered slowly (over 1-1.5 seconds).<br />

,--.., 2. The appropriate volume of each breath is the volume that causes the chest to rise; this corresponds to a tidal<br />

volume of 6-8 mUkg with spontaneous breathing, 10-15 ml/kg with volume ventilation.<br />

3. Rate (breaths per minute)<br />

a. Neonates: 40-60<br />

b. Infants


PEDIATRIC EMERGENCIES<br />

c. Children 1-3 years old: 22-30<br />

d. Children >8 years old: 12<br />

4. Mechanical ventilation<br />

a. For children, use a volume-limited ventilator. Initial settings:<br />

(1) Tidal volume 10-15 ml/kg<br />

(2) FiO 2<br />

100%<br />

(3) lnspiratory time 0.5-1.5 seconds<br />

(4) Age-appropriate respiratory rate (adjusted to physiologic needs)<br />

(a) 20-30 breaths per minute in infants<br />

(b) 16-20 breaths per minute in children<br />

(5) Positive end-expiratory pressure 3-5 cm H 2<br />

O<br />

b. For infants 8 years old: 60-100<br />

2. Normal systolic blood pressure based on age (lowest 5th percentile)<br />

a. 60 mmHg<br />

b. 1 month to 1 year old: 70 mmHg<br />

c. > 1 year old: 70 + (2 x age in years) mm Hg (estimate)<br />

D. Chest compressions<br />

1. Should be started in:<br />

a. Children who do not have a pulse or have a heart rate


PEDIATRIC EMERGENCIES<br />

d. Children >8 years old<br />

(1) Rate= 100-120 per minute<br />

(2) Technique: heel of one hand with other hand on top located over lower half of sternum<br />

(3) Compression/ventilation ratio 15:2 (two-rescuer CPR)<br />

(4) Depth = at least 1 /3 the total depth of the chest or 2-2.5 inches (5-6 cm) in adolescents<br />

e. When only one provider is available, the universal 30:2 compression/ventilation ratio should be used.<br />

f. CPR with an advanced airway in place in infants, children, and adolescents: 10 breaths per minute and<br />

compressions at 100-120 per minute; in neonates, should continue at 3:1 compression:ventilation ratio<br />

E. Defibrillation and cardioversion<br />

1. Paddle size<br />

a. Infants (1 year old or >10 kg: adult paddles 8-13 cm<br />

2. Conducting medium<br />

a. Use electrode cream (or paste) or self-adhesive defibrillation pads.<br />

b. Do not use alcohol pads; they can produce significant burns.<br />

c. Do not allow the conducting medium under one paddle to come into contact with the conducting medium<br />

under the other paddle; this can result in "bridging" and delivery of an insufficient amount of current to<br />

the heart.<br />

3. Positioning<br />

a. Right upper-left lower anterior chest or anterior-posterior orientation: pads should not touch and should be<br />

positioned so that there is at least 3 cm between them.<br />

b. Infants may need anteriorposterior positioning.<br />

4. Defibrillation<br />

a. Initial dosage is 2-4 joules/kg; if unsuccessful, use 4 joules/kg for subsequent attempts, increasing up to 10<br />

joules/kg (maximum adult dosage).<br />

b. If unresponsive with shockable rhythm (ventricular fibrillation or pulseless ventricular tachycardia),<br />

initiate CPR before first shock and resume CPR immediately for 2 minutes before checking pulse.<br />

Administer epinephrine before proceeding with additional defibrillation attempts.<br />

5. Cardioversion: initial dosage is 0.5-1 joule/kg.; if unsuccessful, increase the dosage, up to 2 joules/kg.<br />

F. Vascular access<br />

1. Although the tracheal route may be used to administer a limited number of medications, the IV and<br />

intraosseous routes are preferable for drug delivery and are mandatory for fluid therapy.<br />

2. The preferred site of vascular access is the largest vein that can be rapidly cannulated without disrupting the<br />

resuscitation.<br />

a. Peripheral venous access (arm, hand, leg, foot) is usually attempted first.<br />

b. Central venous access has some advantages and should be used if already in place, but it is more difficult<br />

to accomplish in the arrest setting and may be associated with significant complications.<br />

c. Administration of drugs via the peripheral venous route should be followed by a saline flush of c::5 ml to<br />

promote movement of the drug into the central circulation.<br />

3. If peripheral venous access cannot be rapidly obtained (3 attempts or 90 seconds) in a child with a lifethreatening<br />

condition, intraosseous access should be considered. This route is usually easier (more successful<br />

and faster) than other types of access in volume-depleted children. In addition, intraosseous access is no<br />

longer restricted to young children; it is recommended for the entire spectrum of pediatric care.<br />

a. The anteromedial aspect of the proximal tibia (1-3 cm below the tibial tuberosity on the medial flat surface<br />

of the tibia) is the preferred site.<br />

b. Drugs, fluids, and blood products may be given by this route. The initial aspirate can also be used for blood<br />

typing and chemistries.<br />

c. Drug administration should be followed by a saline flush of 5 ml to promote movement of the drug into the<br />

central circulation.<br />

d. Alternative sites for intraosseous placement include distal femur, medial malleolus, proximal humerus,<br />

sternum, and anterior superior iliac spine.<br />

4. The tracheal route is less reliable and therefore less desirable; the kinetics of tracheal drug absorption do not<br />

favor this route.<br />

499


PEDIATRIC EMERGENCIES<br />

a. Medications that are safe and effective when given through the tracheal tube can be remembered by the<br />

mnemonic "LEAN":<br />

.!.idocaine<br />

_Epinephrine<br />

Atropine<br />

Naloxone<br />

b. When this route is used, the dosage administered should be greater than that given by the IV route.<br />

However, a specific tracheal dosage has been established only for epinephrine (0.1 mg/kg; 10 times venous<br />

dosage), and most authors recommend that the dosage be 2-3 times the IV dosage for other medications.<br />

c. Drugs administered by this route should be followed by a 2-5 ml saline flush (or diluted in 2-5 ml of<br />

saline) and followed by several positive-pressure ventilations.<br />

d. Drug administration should be switched to the IV route as soon as it becomes available.<br />

e. Sodium bicarbonate and calcium chloride should not be administered via the tracheal route.<br />

5. In the neonatal period, the umbilical vein is an excellent site of vascular access; it can be used up to 7 days<br />

after delivery.<br />

G. Life-threatening cardiac arrhythmias<br />

1. Unlike in adults, in whom rhythm disturbances are primary cardiac diseases, in children rhythm disturbances<br />

are frequently secondary problems.<br />

a. Most commonly secondary to respiratory (hypoxemia), metabolic/electrolyte, renal disturbances, or drug/<br />

toxin effects<br />

b. Exceptions: children with congenital heart disease, structural heart disease, or acquired heart disease<br />

2. When evaluating and managing life-threatening dysrhythmias, always consider and treat causes and<br />

contributing factors ("6 H's and 5 T's")<br />

a. Hypovolemia<br />

b. Hypoxia<br />

c. Acidosis (hydrogen ion): sodium bicarbonate (1 mEq/kg IV or intraosseous)<br />

(1) No longer first-line agent for acidosis<br />

(2) Indicated for severe refractory acidosis (after adequate ventilation/oxygenation established and<br />

epinephrine ineffective)<br />

(3) In infants and children, use 8.4% solution (1 mEq/mL).<br />

(4) In neonates, dilute 8.4% solution to make 4.2%.<br />

d. Hypokalemia/hyperkalemia<br />

(1) Calcium chloride (for life-threatening hyperkalemia with dysrhythmia)<br />

(a) 20 mg/kg (0.2 ml/kg) slow IV or intraosseous<br />

(b) Preferably should be administered via a central line<br />

(c) Stabilizes cardiac membrane but does not decrease potassium<br />

(2) NaHCO 3<br />

, glucose and insulin, and albuterol are usually a part of the management of hyperkalemia.<br />

e. Hypoglycemia: glucose (0.5-1 g/kg IV or intraosseous)<br />

(1) 2 years old: 1-2 mUkg of D50<br />

f. Hypothermia<br />

g. Toxins, poisons, or drugs: naloxone<br />

(1) Dosage: 0.1 mg/kg IV push or intraosseous (2 mg IV push or intraosseous if >20 kg) for opioid intoxication<br />

(2) Use with caution in neonates, because it can precipitate life-threatening withdrawal in this age group.<br />

h. Cardiac tamponade<br />

i. Tension pneumothorax<br />

j. Thrombosis: acute coronary syndrome, pulmonary embolism<br />

k. Trauma<br />

3. Bradyarrhythmias (most common dysrhythmia seen in pediatric arrest)<br />

a. General principle: outside of the immediate neonatal period, only an emergency requiring intervention if<br />

causing cardiopulmonary compromise<br />

b. Evaluate and support ABCs, including supplemental oxygen.<br />

500


PEDIATRIC EMERGENCIES<br />

c. If pulse 180 beats per minute<br />

iii. Adenosine 0.1 mg/kg IV push (maximum 6 mg); if unsuccessful, 0.2 mg/kg IV push<br />

(maximum 12 mg)<br />

(2) Wide QRS (>90 milliseconds): ventricular tachycardia<br />

(a) Synchronized cardioversion: 0.5-1 joules/kg; if unsuccessful, increase dosage to 2 joules/kg.<br />

(b) Amiodarone 5 mg/kg IV over 20-60 minutes or<br />

(c) Procainamide 15 mg/kg IV over 30-60 minutes<br />

5. Pulseless arrest (ventricular tachycardia/ventricular fibrillation, pulseless electrical activity, asystole)<br />

a. Start CPR, including rescue breathing, cardiac monitor, IV access.<br />

b. Evaluate rhythm morphology.<br />

(1) Ventricular tachycardia/ventricular fibrillation<br />

(a) Defibrillation<br />

i. 2-4 joules/kg; if unsuccessful, then 4 joules/kg up to 10 joules/kg (maximum adult dosage)<br />

for all subsequent shocks.<br />

11. Always perform CPR for 2-3 minutes immediately after any defibrillation attempts before<br />

rechecking rhythm.<br />

(b) Epinephrine<br />

(c) Consider antiarrhythmic<br />

i. Amiodarone (5 mg/kg IV or intraosseous)<br />

ii. Lidocaine (1 mg/kg IV or intraosseous; may give second dose of 1.5 mg/kg if unsuccessful)<br />

iii. Magnesium (25-50 mg/kg IV or intraosseous) for torsades de pointes<br />

A. Risk factors associated with neonatal cardiopulmonary arrest<br />

1. Maternal factors<br />

a. Poor prenatal care<br />

b. Age (35 years old)<br />

c. Preeclampsia/eclampsia<br />

501


PEDIATRIC EMERGENCIES<br />

2.<br />

3.<br />

4.<br />

d. Hypertension<br />

e. Diabetes<br />

f. Drug abuse<br />

g. Medications (lithium, ~-blockers, magnesium)<br />

h. Anemia<br />

i. Renal disease<br />

j. Rh factor incompatibility<br />

k. History of prior perinatal morbidity/mortality<br />

I. Oligohydramnios<br />

m. HIV infection<br />

lntrapartum factors<br />

a. Premature or prolonged labor<br />

b. Abnormal presentation<br />

C. Precipitous or forceps delivery<br />

d. Cephalopelvic disproportion<br />

e. Cesarean section<br />

f. Cord prolapse or compression<br />

g. Analgesia and/or sedatives administered ::,2 hours before delivery<br />

h. Signs of fetal distress on fetal monitoring (late decelerations)<br />

i. Maternal shock<br />

j. Placenta previa/abruptio<br />

k. Premature rupture of membranes<br />

Fetal factors<br />

a. Prematurity and postmaturity<br />

b. Thick meconium<br />

C. Congenital infection<br />

d. Malformations identified by ultrasound<br />

e. Multiple gestations<br />

f. Bradycardia<br />

g. Acidosis<br />

h. Small fetus for maternal dates<br />

Precipitous/imminent delivery of newborn<br />

a. History: 3 questions currently recommended<br />

(1) Term gestation?<br />

(2) Prenatal care?<br />

(3) Single or multiple gestations?<br />

b. Equipment<br />

(1) Resuscitation bag<br />

(2) Neonatal resuscitation masks<br />

(3) Dry towels<br />

(4) Ambient warmer (if available)<br />

(5) Suction equipment (including meconium aspiration device)<br />

(6) Uncuffed entotracheal tubes (sizes 2.5, 3.0, and 3.5)<br />

(7) Laryngoscope (Miller size O and 1)<br />

"'·---·'/<br />

502


~<br />

PEDIATRIC EMERGENCIES<br />

B. APGAR scoring<br />

Table 33: APGAR Score<br />

Score<br />

0 2<br />

Activity (muscle tone) Limp Decreased flexion Good flexion<br />

Pulse (heart rate) Absent 100 beats per minute<br />

Grimace (reflex irritability, None Some motion Cry<br />

stimulus respiration)<br />

Appearance (color) Blue, pale Body pink, extremities blue Pink<br />

Respirations Absent Slow, irregular Good, crying<br />

1. The Apgar score is assigned to newborns 1-5 minutes after delivery. If the 5-minute score is


PEDIATRIC EMERGENCIES<br />

3. Management<br />

a. Assess ABCs: maintain airway, provide supplemental oxygen, establish an IV line (normal saline), and start<br />

appropriate monitoring.<br />

b. Correct underlying metabolic problems if present (hypoglycemia, hypocalcemia, electrolyte imbalance).<br />

c. Identify and treat associated problems (sepsis, acidosis, etc).<br />

d. Begin anticonvulsant therapy.<br />

(1) Phenobarbital is the initial anticonvulsant agent of choice in neonates. Dosage is 15-20 mg/kg IV over<br />

10 minutes; if this is ineffective, additional dosages of 5 mg/kg may be given every 5 minutes up to a<br />

total maximal dosage of 40 mg/kg.<br />

(2) If seizures persist or recur, follow with phenytoin.<br />

(a) Dosage is 20 mg/kg IV; dilute in normal saline and administer slowly at 0.5 mg/kg/min.<br />

(b) Fosphenytoin is another alternative.<br />

(3) If there is still no response, a benzodiazepine (lorazepam or diazepam) may be given. However,<br />

benzodiazepines are not first-line agents in this age group; they produce prolonged and profound<br />

respiratory depression (particularly if used in combination with phenobarbital), and they also displace<br />

bilirubin from albumin.<br />

(a) Lorazepam 0.05-0.1 mg/kg at a rate of 2 mg/min<br />

(b) Diazepam 0.2-0.3 mg/kg at a rate of 1 mg/min<br />

e. Administer pyridoxine 50-100 mg IV or IM for refractory seizures.<br />

E. Congenital diaphragmatic hernia<br />

1. Hernia results from developmental failure of the posterolateral or retrosternal portions of the diaphragm.<br />

2. Left-sided hernias (Bochdalek) are far more common (70%-85%) than right-sided hernias (Morgagni).<br />

3. Associated anomalies include congenital heart disease, GI and genitourinary anomalies, hydronephrosis, and<br />

cystic kidneys.<br />

4. Clinical presentation<br />

a. History: respiratory distress and vomiting, which is caused by herniation of the abdominal viscera into the<br />

chest cavity<br />

b. Physical examination: scaphoid abdomen, absence of breath sounds on affected side of chest, auscultation<br />

of bowel sounds over affected side of chest<br />

5. Chest radiograph: air-filled loops of bowel in the chest, absence of diaphragmatic margin, displacement of<br />

heart and mediastinum, hypoplastic lungs<br />

6. Management<br />

a. Immediate intubation (avoid bag-mask ventilation)<br />

b. Placement of an orogastric tube, IV hydration, and surgery<br />

7. Mortality rate is 50% in severely affected infants; morbidity is even higher.<br />

F. Tracheoesophageal fistula<br />

1. One-third of affected infants are born prematurely.<br />

2. A proximal esophageal pouch and fistula between the trachea and the distal esophagus (tracheoesophageal<br />

fistula) is most common (84%).<br />

3. Associated anomalies and polyhydramnios are common.<br />

4. Infants may present with increased oral secretions, choking, or coughing with attempts at feeding or recurrent<br />

aspiration pneumonia.<br />

5. Inability to pass a catheter into the stomach confirms the diagnosis.<br />

6. Management: reverse Trendelenburg or semi-Fowler position, placement of suction catheter into esophageal<br />

pouch, IV fluids, and surgical correction<br />

G. Omphalocele and gastroschisis<br />

1. Abdominal wall defects resulting from developmental anomalies of the intestines<br />

a. Omphalocele<br />

(1) Defect in the umbilical ring with protrusion of the intestines (covered by the peritoneal sac) outside of<br />

the abdominal wall<br />

(2) Associated anomalies (particularly chromosomal ones) occur in one-third to one-half of these infants.<br />

b. Gastroschisis<br />

(1) Defect in the abdominal wall with antenatal evisceration of abdominal contents without a peritoneal sac<br />

(2) Occurs in association with intestinal atresia<br />

504


PEDIATRIC EMERGENCIES<br />

2. Management<br />

a. Keep the child warm.<br />

b. Place an orogastric tube to decompress the gut.<br />

c. Cover the eviscerated intestines with saline-soaked sterile gauze and place them in a plastic bag.<br />

d. Administer IV fluids and prophylactic antibiotics.<br />

e. Obtain immediate pediatric surgery consultation for operative repair.<br />

H. Necrotizing enterocolitis<br />

1. Primarily affects premature infants with very low birth weights, although may also be seen in term infants<br />

2. Characterized by variable degrees of mucosa! or transmucosal necrosis of the intestines<br />

3. Risk factors<br />

a. Hypertonic feeding solutions or medicines<br />

b. Patent ductus arteriosus and apneic spells<br />

c. Infection<br />

d. lschemia after exchange transfusions<br />

4. Onset usually in first 2 weeks of life<br />

5. Clinical presentation<br />

a. Feeding intolerance<br />

b. Abdominal distention with gastric retention (first and most frequent finding)<br />

c. Bilious emesis<br />

d. Guaiac positive or grossly bloody feces<br />

e. Abdominal wall tenderness or erythema<br />

f. Manifestations of associated sepsis<br />

(1) Metabolic acidosis<br />

(2) Apneic episodes<br />

(3) Temperature instability<br />

(4) Lethargy<br />

6. Radiographic findings on plain abdominal radiographs<br />

a. Pneumatosis intestinalis (radiographic hallmark)<br />

b. Separation of bowel loops (suggests mural edema)<br />

c. A fixed, dilated loop of bowel that fails to move on serial radiographs<br />

d. Air-fluid levels<br />

e. Portal vein gas (sign of severe disease)<br />

f. Pneumoperitoneum (indicative of perforation)<br />

7. Management<br />

I. Cyanosis<br />

a. Discontinue oral feedings.<br />

b. Place a nasogastric tube to decompress the bowel.<br />

c. Obtain cultures (blood, feces, urine, CSF).<br />

d. Administer IV fluids, parenteral feedings, and broad-spectrum antibiotics.<br />

e. Obtain surgical consult.<br />

1. Central<br />

a. Bluish discoloration of tongue, mucous membrane, and peripheral skin; unsaturated hemoglobin usually 5 g<br />

b. Pathologic if persists for >20 minutes after delivery<br />

c. Etiology<br />

(1) Cyanotic heart disease with a right-to-left shunt (the 5 "T's")<br />

(a) Truncus arteriosus<br />

(b) Transposition of the great vessels<br />

(c) Tricuspid atresia<br />

(d) Tetralogy of Fallot<br />

(e) Total anomalous pulmonary venous return<br />

505


PEDIATRIC EMERGENCIES<br />

(2) Primary lung disease<br />

(3) Hypoventilation due to a CNS lesion (eg, severe intracerebral hemorrhage)<br />

(4) Alveolar hypoventilation secondary to shock or sepsis<br />

(5) Methemoglobinemia<br />

d. The "hyperoxia test" (response of the PaO 2<br />

to the administration of 100% oxygen) can provide a<br />

diagnostic clue as to the underlying cause of the central cyanosis.<br />

2. Peripheral<br />

(1) Failure of the PaO 2<br />

to rise above 1 00 mm Hg suggests either cyanotic heart disease with a fixed<br />

right-to-left shunt or methemoglobinemia.<br />

(2) Improvement in the PaO 2<br />

suggests one of the other causes (eg, lung disease, sepsis, CNS disorder).<br />

a. Bluish discoloration of extremities only; oxygen saturation >94%<br />

b. Common in newborns in the first few days of life<br />

c. Generally due to vasomotor instability secondary to a cold environment<br />

J. Congestive heart failure<br />

1. Clinical presentation<br />

a. Feeding difficulty (slow feeder, diaphoresis, and dyspnea on feeding)<br />

b. Tachypnea and tachycardia<br />

c. Rales and rhonchi<br />

d. Hepatomegaly and cardiomegaly<br />

e. Failure to thrive<br />

f. Peripheral edema rare<br />

2. Etiology<br />

a. Usually due to congenital heart disease, most commonly left-to-right shunts that present at 6-8 weeks<br />

b. Other causes include severe anemia, dysrhythmias (supraventricular tachycardia, ventricular tachycardia),<br />

sepsis, and arteriovenous malformation (hepatic, cerebral).<br />

3. Management<br />

a. Supplemental oxygen<br />

b. Semireclining position<br />

c. Restriction of fluid intake<br />

d. Digoxin<br />

(1) The oral digitalizing dosage is 10-20 mcglkg for preterm infants and 30 mcglkg for term infants; the IV<br />

dosage is three-fourths of the oral dosage.<br />

(2) Administer one-half of this dose initially, one-fourth in 8 hours, and the last one-fourth 8 hours later.<br />

(3) Contraindications: tetralogy of Fallot, idiopathic hypertrophic subaortic stenosis and myocarditis (a<br />

relative contraindication), and a heart rate


PEDIATRIC EMERGENCIES<br />

c. Clinical presentation<br />

(1) Lethargy or irritability<br />

(2) Poor feeding<br />

(3) Dehydration<br />

(4) Hypotonia<br />

(5) Seizures<br />

d. Management<br />

(1) IV hydration<br />

(2) Treat any underlying pathology.<br />

(3) Phototherapy and exchange transfusion for severe hyperbilirubinemia: based on total serum bilirubin<br />

gestational age and hours of life (published guidelines exist) but concerning if >20 mgldL<br />

e. Complication of acute bilirubin encephalopathy (kernicterus): neurologically devastating sequelae of<br />

bilirubin crossing the blood-brain barrier, resulting in choreoathetoid cerebral palsy, sensorineural hearing<br />

loss, and gaze palsies<br />

L. Apparent life-threatening event (ALTE)<br />

1. Brief resolved unexplained events (BRUE)<br />

a. AAP Clinical Practice Guideline 2016<br />

b. Definition: an event occurring in an infant 60 days<br />

(2) Born >32 weeks gestation and corrected gestational age >45 weeks<br />

(3) No CPR by trained medical provider<br />

(4) Event lasted


PEDIATRIC EMERGENCIES<br />

b. Certain subgroups have higher rates of subsequent sudden infant death syndrome<br />

(1) Recurrent ALTEs requiring CPR<br />

(2) Premature infants<br />

5. Periodic breathing of infancy<br />

a. Normal breathing pattern of newborns (especially premature neonates) that is often mistaken for ALTEs<br />

b. Characterized by periods of rapid breathing followed by apnea of 3-20 seconds in duration<br />

c. Never associated with cyanosis or bradycardia<br />

M. Sudden infant death syndrome (SIDS)<br />

1. Although the incidence in the United States has been reduced by >50% (down to 3,700 deaths per year) as<br />

a result of the "back to sleep" initiative, which encouraged parents to place sleeping infants in the supine<br />

position, SIDS remains the leading cause of death in infants 1 month to 1 year old.<br />

2. Peak incidence is during the winter months during the hours of sleep (midnight to 9 AM) in infants 2-4 months<br />

old; boys are more commonly affected than girls.<br />

3. Although the exact cause(s) remains unclear, current pathologic data indicate that hypoxia (and possibly<br />

autonomic dysfunction) plays a role in the pathophysiology.<br />

4. Risk factors<br />

a. Maternal<br />

(1) Age


PEDIATRIC EMERGENCIES<br />

(5) Loud, single second heart sound<br />

(6) Patients develop "hypercyanotic" ("Tet") spells with moderate to severe pulmonic stenosis. They are<br />

most common in the first 3 years of life and are brought on by exertion (feeding, straining, crying).<br />

These episodes are treated by placing the child in the prone knee-chest position and administering<br />

supplemental oxygen and morphine 0.1 mg/kg IV, IM, or SC. The morphine may be repeated if the<br />

child does not respond to the initial dose. If there is still no response, propranolol (0.05-0.1 mg/<br />

kg IV) or phenylephrine (1 0 mcg/kg IV followed by 0.5-2 mcg/kg/min) may be given under the<br />

direction of appropriate pediatric consultation.<br />

e. Diagnostic evaluation<br />

(1) Chest radiograph findings<br />

(a) Boot-shaped heart<br />

(b) Diminished pulmonary vascular markings (due to decreased pulmonary blood flow)<br />

(2) ECG findings<br />

(a) Right axis deviation<br />

(b) Right ventricular hypertrophy<br />

3. Other causes of cyanotic heart disease<br />

a. Transposition of the great vessels (most common cause in newborns)<br />

b. Truncus arteriosus<br />

c. Tricuspid atresia<br />

d. Total anomalous pulmonary venous return<br />

e. Pulmonary atresia<br />

f. Ebstein anomaly of the tricuspid valve<br />

B. Acyanotic heart lesions<br />

1. Aortic stenosis (most common)<br />

a. A bicuspid aortic valve is the most frequent cause of significant valvular aortic stenosis in infancy and<br />

childhood; it is one of the most common congenital heart lesions identified in adults.<br />

b. Although generally seen in older children, severe lesions may present in infancy as left heart failure. Critical<br />

stenosis results in heart failure and cardiogenic shock when the ductus arteriosus closes.<br />

c. Clinical presentation<br />

(1) Exercise intolerance<br />

(2) CHF<br />

(3) Chest pain<br />

(4) Syncope<br />

(5) Diamond-shaped systolic ejection murmur radiating to the neck<br />

(6) Systolic ejection click<br />

(7) An S 3<br />

is common; an S 4 indicates severe stenosis.<br />

(8) Palpable thrill over the base of the heart<br />

d. ECG findings: signs of left ventricular hypertrophy or strain may be present<br />

e. Complications<br />

(1) Infectious endocarditis<br />

(2) Sudden death due to ventricular dysrhythmias<br />

2. Ventricular septa! defect<br />

a. Most common congenital heart anomaly<br />

b. Clinical presentation is determined by the size of the lesion.<br />

(1) Small defects: may be asymptomatic<br />

(2) Large defects: signs of CHF (tachypnea, grunting respirations, fatigue with feeding) typically develop<br />

after the first few weeks or months of life as the pulmonary vascular resistance falls and flow across the<br />

defect increases.<br />

(3) Physical findings<br />

(a) Holosystolic murmur at the lower left sternal border<br />

(b) Normal peripheral pulses<br />

509


PEDIATRIC EMERGENCIES<br />

3. Other causes of acyanotic heart disease<br />

a. Atrial septa! defect<br />

b. Patent ductus arteriosus (common in premature infants): characterized by bounding pulses, a continuous<br />

"machinery" murmur, and a suprasternal notch thrill<br />

c. Coarctation of the aorta: cardinal findings are hypertension in the upper extremities and decreased or<br />

absent pulses in the lower extremities.<br />

C. Ductal dependent lesions<br />

1. Cardiac lesions that manifest as outflow tract obstruction (severe coarctation of the aorta, critical aortic<br />

stenosis, hypoplastic left heart syndrome, and tricuspid atresia)<br />

2. Infants with these lesions usually present in profound shock with a classic skin color described as the<br />

"gray baby" within hours to days of delivery.<br />

3. Affected infants become symptomatic as the ductus arteriosus closes, because the systemic circulation<br />

(in left-sided lesions) or pulmonic circulation (in right-sided lesions) depends on shunting.<br />

4. Management<br />

a. ABCs, IV line, monitor<br />

b. Prostaglandin E 1<br />

, a potent dilator of the ductus arteriosus, can be life-saving in these infants. The infusion<br />

(0.05-0.1 mcg/kg/min) should be started, and the child monitored for apnea and hypotension (occurs<br />

with prostaglandin infusion).<br />

c. Obtain urgent cardiology consult.<br />

D. Congestive heart failure<br />

1. Primary cause in infants and young children is congenital heart disease.<br />

2. Etiology based on age of presentation:<br />

a. Newborn (noncardiac causes)<br />

(1) Anemia<br />

(2) Hypoxia<br />

(3) Acidosis<br />

(4) Hypoglycemia<br />

(5) Hypocalcemia<br />

(6) Sepsis<br />

b. Day 1 _.. patent ductus arteriosus<br />

c. First week_.. hypoplastic left heart syndrome<br />

d. 2 weeks _.. coarctation of the aorta<br />

e. 1 month __,. ventricular septa! defect<br />

f. 3 months _.. supraventricular tachycardia (the most common symptomatic dysrhythmia in infants and<br />

children)<br />

g. 1-2 years<br />

(1) Myocarditis<br />

(2) Cardiomyopathy<br />

(3) Severe anemia<br />

(4) Kawasaki disease<br />

h. 1 O years __,. rheumatic fever<br />

3. Clinical presentation<br />

a. Right-sided CHF __,. hepatomegaly<br />

b. Left-sided CHF _.. tachypnea, dyspnea, diaphoresis, poor feeding<br />

c. Both_.. cardiomegaly, tachycardia, cyanosis, failure to thrive<br />

4. Diagnostic evaluation: chest radiograph typically shows cardiomegaly and increased pulmonary marking.<br />

5. Treatment: see management of newborn (see page 506)<br />

510


PEDIATRIC EMERGENCIES<br />

A. Upper airway (see also upper airway emergencies, page 150)<br />

1. Clinical presentation<br />

a. Stridor<br />

(1) lnspiratory (obstruction at or above larynx)<br />

(2) Biphasic (obstruction below the larynx)<br />

(3) Expiratory (bronchial or lower tracheal obstruction)<br />

b. Expiratory wheezing (obstruction below the carina at the level of the bronchi)<br />

c. Tachypnea<br />

d. Nasal flaring<br />

e. lntercostal and substernal retractions<br />

f. Cyanosis<br />

g. Hoarseness<br />

h. Coughing<br />

i. Grunting and expiratory wheezing are signs of lower airway distress.<br />

2. Foreign body aspiration<br />

a. Most common cause of accidental home death in children


PEDIATRIC EMERGENCIES<br />

(b) Evaluation of the epiglottis by using a tongue blade (or a fiberoptic bronchoscope) can be safely<br />

done by experienced physicians, but it remains controversial and should be avoided in children<br />

with any respiratory distress.<br />

(c) Definitive diagnosis is ideally made in the operating room under controlled conditions by direct<br />

visualization of the "cherry-red" epiglottis. This is particularly true when severe respiratory distress<br />

is present.<br />

(3) Provide supplemental humidified oxygen, set up airway stabilization equipment at bedside (the patient<br />

can usually be adequately ventilated with bag and mask if total airway obstruction occurs), and obtain<br />

immediate ENT and anesthesia consultation.<br />

(4) Ideally, the child should be intubated under general anesthesia in the presence of an ENT surgeon. If<br />

unable to intubate, the surgeon will need to perform a tracheostomy.<br />

(5) Once the airway has been stabilized, draw blood for laboratory studies (including cultures); blood<br />

cultures are positive in 80% of patients.<br />

(6) Administer parenteral antibiotics. Acceptable regimens include a second- or third-generation<br />

cephalosporin (cefuroxime, cefotaxime, or ceftriaxone) or ampicillin-sulbactam.<br />

(7) Admitto ICU.<br />

d. Haemophilus influenzae type B cases caused by this organism have markedly decreased since the advent<br />

of the H influenzae B vaccine. Group A f3-hemolytic streptococci (followed by Streptococcus pneumoniae<br />

and Staphylococcus aureus) is responsible for an increasing number of cases.<br />

4. Croup (usual age group 6 months to 6 years old, peak incidence at 2 years old)<br />

a. Clinical presentation: The child has a barky, "seal-like" cough that is usually worse at night and may cause<br />

severe respiratory distress. The illness tends to come on gradually (usually in the late fall or early winter) as<br />

an upper respiratory infection. The cough, hoarseness, and stridor typically appear 1-2 days later. Croup<br />

symptoms peak on the third or fourth day of the illness (which is the first and second day of the croup<br />

symptoms) and generally resolve over a week. Fever is absent or low-grade, and the child is nontoxic in<br />

appearance.<br />

b. Diagnostic evaluation<br />

(1) Radiographs are not usually needed (diagnosis can often be made on clinical grounds alone) but are<br />

useful in excluding epiglottitis.<br />

(2) The PA neck radiograph reveals symmetric subglottic narrowing of the tracheal air column ("steeple<br />

sign"), and the lateral neck radiograph shows a distended hypopharynx and subglottic narrowing. The<br />

epiglottis and retropharyngeal space are normal.<br />

c. Management: supportive<br />

(1) Antibiotics are not indicated because the infecting organism is viral (usually parainfluenza virus).<br />

(2) Cool mist<br />

(3) Oxygen as needed<br />

(4) Hydration (orally or IV)<br />

(5) Racemic epinephrine aerosol (0.5 ml of a 2.25% solution diluted in 2.5 ml of saline)<br />

(a) Reserved for children with resting stridor and respiratory distress<br />

(b) Children who receive it should be observed in the emergency department for 2-4 hours after<br />

administration in case of return to their pretreatment stridorous state once the effect wears off.<br />

(c) If racemic epinephrine is unavailable, aerosolized L-epinephrine 1 :1,000 (5 ml) is equally<br />

effective and may be used.<br />

(6) Steroids (dexamethasone 0.3-0.6 mg/kg IM, IV, or orally as a one-time dose)<br />

(a) Should be administered to all children with croup (mild, moderate, or severe) and especially to<br />

those who receive racemic epinephrine<br />

(b) Hastens symptomatic improvement<br />

(7) Heliox (a mixture of helium and oxygen)<br />

(a) Decreases the work of breathing by improving laminar gas flow through the obstructed airway<br />

(b) Usually reserved for children with severe croup or refractory respiratory distress despite steroids<br />

and racemic epinephrine<br />

(8) Although most children can be discharged, admission should be considered for those who:<br />

(a) Have persistent stridor at rest<br />

(b) Are unable to tolerate oral fluids<br />

(c) Have an incomplete response to racemic epinephrine or require multiple doses<br />

(d) Present with severe croup (particularly those


PEDIATRIC EMERGENCIES<br />

5. Bacterial tracheitis (usual age


PEDIATRIC EMERGENCIES<br />

f. Differential diagnosis includes asthma ("reactive airway disease"), cystic fibrosis, recurrent aspiration,<br />

congenital heart disease, and foreign body aspiration. A careful history and chest radiographs are helpful ~/<br />

in eliminating some of these diagnoses. However, differentiating between asthma and bronchiolitis can be<br />

difficult because of the similarity in presenting symptoms.<br />

g. Prevention: Prophylactic regimens using immune globulin (palivizumab) are available for high-risk infants<br />

(premature infants and those with lung or heart disease) to lessen the risk of subsequent severe respiratory<br />

syncytial virus infection.<br />

h. Risk factors for severe disease<br />

(1) Premature infants<br />

(2) Congenital heart disease<br />

(3) Respiratory syncytial virus subtype A<br />

(4) History of cyanosis/apnea<br />

(5) Age


PEDIATRIC EMERGENCIES<br />

(2) Serum lgE levels and the prevalence of asthma are closely related, lending support to the premise<br />

that asthma nearly always has an allergic basis. Further support comes from the fact that atopic<br />

individuals (those with a genetic predisposition for producing an lgE-mediated response to common<br />

environmental allergens) are at greatest risk of developing asthma.<br />

(3) "Triggers" that can precipitate an asthma attack<br />

(a) Viral respiratory infections (most frequent trigger in children


PEDIATRIC EMERGENCIES<br />

516<br />

1. Speak in complete sentences<br />

ii. Have only end-expiratory wheezing<br />

iii. Are generally not using accessory muscles<br />

iv. Do not have pulsus paradoxus<br />

v. Have a PEFR >80% of predicted/personal best and an SaO 2<br />

>95% on room air (at sea level)<br />

(b) Patients with moderate exacerbations<br />

i. Speak in phrases<br />

ii. Have wheezing throughout exhalation<br />

iii. Commonly use accessory muscles<br />

iv. May have a pulsus paradoxus of 10-25 mm Hg<br />

v. Have a PEFR 50%-80% of predicted/personal best and an SaO 2<br />

of 91 %-95% on room air<br />

(at sea level)<br />

(c) Patients with severe exacerbation<br />

i. Speak one or two words at a time<br />

ii. Commonly have loud wheezing throughout inhalation and exhalation<br />

iii. Are usually using accessory muscles<br />

iv. Frequently have a pulsus paradoxus of 20-40 mmHg<br />

v. Have a PEFR 95% in children and infants.<br />

(2) The initial treatment of choice is aerosolized therapy with a ~-adrenergic agonist; it is associated with<br />

fewer systemic adverse effects and better dilation than parenteral therapy, and it obviates the need for<br />

painful injections.<br />

(a) Albuterol 0.5% solution (5 mg/ml) is the preferred agent because of its ~ 2<br />

-agonist specificity.<br />

1. Administer 0.15 mg/kg (maximum dose is 5 mg; minimum dose is 2.5 mg) in 2-3 ml normal<br />

saline every 20 minutes for 3 doses, then 0.15-0.3 mg/kg up to 10 mg every 1-4 hours as<br />

needed.<br />

ii. Continuous nebulization using 0.5 mg/kg/hr (maximum 15 mg/hr) should be considered for<br />

children who do not respond to this regimen or who present with severe respiratory distress.<br />

iii. Albuterol administered by a metered-dose inhaler (90 mcg/puff) with a spacer is as effective<br />

as nebulizer therapy and can be used in children who are able to coordinate the inhalation<br />

maneuver (usually children >6 years old). The dosage is 4-8 puffs every 20 minutes up to 4<br />

hours, then every 1-4 hours as needed.<br />

(b) Alternative agents include terbutaline (parenteral formulation), bitolterol, and pirbuterol.<br />

These agents do not appear to provide any clear advantage over albuterol, although pediatric ,~<br />

experience with bitolterol and pirbuterol is limited. Because of its lack of ~ 2<br />

-agonist specificity,<br />

metaproterenol is no longer recommended as a first-line agent. Albuterol is also available as an<br />

isomer (levalbuterol); compared with albuterol, less frequent dosing may be possible, and it may<br />

produce less direct effect on ~ 1<br />

-adrenergic receptors and/or fewer cardiac adverse effects, but<br />

these claims have not been consistently demonstrated in clinical trials.


PEDIATRIC EMERGENCIES<br />

(3) Parenteral adrenergic therapy is a useful adjuvant to ongoing aerosolized therapy for the very sick<br />

(those with a decreased level of consciousness) and the very young who may not be able to provide<br />

the necessary inspiratory effort or cooperate. Alternatives include:<br />

(a) Epinephrine (1 :1,000 solution)<br />

i. 0.01 mg/kg (up to 0.5 mg) SC every 20 minutes for 3 doses<br />

ii. IM route has more consistent and rapid absorption than SC route.<br />

(b) Terbutaline (1 mg/ml)<br />

i. Preferred agent because it is ~ 2<br />

-selective<br />

ii. 0.005-0.01 mg/kg (up to 0.4 mg) SC every 20 minutes for 3 doses, then every 2-6 hours<br />

as needed<br />

(4) Corticosteroids are useful in treating the inflammatory aspect of asthma. When administered early,<br />

they prevent progression of the illness and decrease the rate of relapse.<br />

(a) Administer these agents as soon as it is clear that they are indicated:<br />

i. All patients with moderate to severe exacerbations (administer them along with initial<br />

~ 2 -agonist therapy)<br />

ii. Those patients with mild exacerbations who:<br />

• Do not clear completely with initial ~ 2<br />

-agonist therapy<br />

(b) Dosage<br />

• Have recently discontinued an oral steroid preparation<br />

i. Methylprednisolone 1-2 mg/kg IV or orally or<br />

ii. Prednisolone 1-2 mg/kg orally or<br />

iii. Prednisone 1-2 mg/kg orally or<br />

iv. Dexamethasone 0.6 mg/kg IV, IM, or orally<br />

(c) Liquid prednisone/prednisolone/dexamethasone is absorbed rapidly; therefore, unless the<br />

patient is vomiting or in severe distress, there is no advantage to IV or IM administration of<br />

corticosteroids.<br />

(d) Patients who receive prednisone or prednisolone in the emergency department and are<br />

subsequently discharged should be continued on oral steroids at a dosage of 1-2 mg/kg/day x 5<br />

days. Patients who receive dexamethasone should receive one additional dose in 1-2 days<br />

(longer half-life).<br />

(5) lpratropium bromide is an inhaled anticholinergic agent that, when used in combination with a<br />

~-adrenergic agonist, exhibits an additive effect by reducing bronchoconstriction and decreasing<br />

mucus production. In patients with severe exacerbations, ipratropium should be added to the first<br />

three ~ 2<br />

-agonist treatments; dose is 250-500 mcg via nebulizer or 4-8 puffs via a metered-dose<br />

inhaler.<br />

(a) lpratropium bromide 0.25 mg/dose in 2 ml normal saline every 20 minutes for three doses, then<br />

every 2-4 hours (agent of choice)<br />

(b) Atropine is associated with a high incidence of adverse effects and is rarely used.<br />

(6) Magnesium sulfate acts as a smooth muscle relaxant and improves FEV 1<br />

by 10%; dosage is 25-75<br />

mg/kg IV over 20 minutes.<br />

(7) leukotriene modifiers are potent inflammatory mediators that play an important role in outpatient<br />

therapy but have not yet been shown to have a role in the acute exacerbation of asthma. Their<br />

bronchodilating effects are additive to those of ~ 2<br />

-agonists, but further research is needed to investigate<br />

their role in acute asthma management.<br />

(8) Methylxanthines are no longer recommended for routine management of acute asthma.<br />

(9) Heliox (a mixture of helium and oxygen) should be considered for children with severe exacerbations<br />

and/or impending respiratory failure. By virtue of its lower density than that of air, it reduces airway<br />

resistance and, in turn, the work of breathing. It can provide additional time for first-line agents to take<br />

effect and so may avert intubation in these patients.<br />

(10) High flow humidified oxygen via nasal cannula may also be used with similar results.<br />

(11) Noninvasive positive-pressure ventilation (CPAP or Bi PAP) may successfully avert the need for intubation.<br />

(12) Ketamine is the agent of choice for patients in respiratory failure who require intubation because of<br />

its bronchodilatory effects.<br />

(13) Ventilator settings for intubated asthmatics should reflect a strategy of "permissive hypercapnia"<br />

(controlled hypoventilation) to minimize airway pressures and reduce the potential for barotrauma; use<br />

a tidal volume of 5-8 ml/kg in these patients.<br />

517


PEDIATRIC EMERGENCIES<br />

518<br />

A. The febrile child<br />

1. Pathophysiology of fever production: exogenous agents (bacterial endotoxins, antigen-antibody complexes) -;,<br />

formation and release of endogenous pyrogens (interleukins, tumor necrosis factor) - prostaglandin synthesis<br />

- raises the body's thermostatic set point - chills - shivering, peripheral vasoconstriction, behavioral<br />

activities (cover with blanket, put on a coat, etc) - increase in body temperature<br />

2. Heat loss occurs secondary to<br />

a. Radiation (60%)<br />

b. Evaporation (25%)<br />

c. Convection (1 0%)<br />

d. Conduction (5%)<br />

3. Temperature assessment<br />

a. A rectal temperature is the most accurate and is, therefore, the gold standard and preferred route in almost<br />

all infants


PEDIATRIC EMERGENCIES<br />

c. Aspirin is also effective, but is associated with Reye syndrome and should, therefore, be avoided.<br />

d. A tepid water (not alcohol) sponge bath may also be helpful.<br />

B. Bacteremia and sepsis<br />

1. Occult bacteremia = fever and positive blood culture in a well-appearing child without a major focus of<br />

infection<br />

a. Most common organisms<br />

(1) Streptococcus pneumoniae<br />

(2) Neisseria meningitidis<br />

(3) Haemophilus influenzae<br />

(4) Staphylococcus aureus<br />

(5) Group A streptococci<br />

(6) Salmonella spp<br />

b. Although children 3-36 months old are at risk of developing occult bacteremia, those 6-24 months old are<br />

most commonly affected; children 24<br />

months old are more immunocompetent.<br />

c. Since the introduction of the heptavalent pneumococcal conjugate vaccine in 2000 and the vaccine for<br />

H influenzae, the prevalence of bacteremia continues to decline (current risk ~ 1 %-2%). This is resulting in<br />

a reconsideration of occult bacteremia evaluation and management.<br />

d. Historically, the risk of occult bacteremia is increased in the presence of:<br />

(1) WBC count 2:l 5,000/mm 3<br />

(2) Temperature 2:102.2°F (39.4°C)<br />

(3) Absolute band count >1,500<br />

(4) Presence of toxic granulations or vacuolization of PMN leukocytes on peripheral smear<br />

e. Occult bacteremia is associated with a risk of progression to a major focus of infection (such as meningitis).<br />

(1) The risk of progression varies with the infecting organism.<br />

(2) It is much greater with H influenzae and N meningitidis than with S pneumoniae.<br />

2. Systemic inflammatory response system (SIRS)/sepsis/severe sepsis/septic shock<br />

a. Definitions<br />

(1) SIRS<br />

(a) Widespread inflammatory response secondary to infection or other physiologic stressor<br />

(b) SIRS criteria (must have two or more, at least one of which is temperature or WBC count)<br />

I. Temperature> 101 °F (38.5°C) or 95th percentile of normal for age; for children 10% immature neutrophils)<br />

(2) Sepsis: SIRS in the presence of or as a result of suspected or proven infection<br />

(3) Severe sepsis: sepsis plus one of the following:<br />

(a) Cardiovascular organ dysfunction<br />

(b) ARDS<br />

(c) Two or more organ dysfunctions<br />

(4) Septic shock: sepsis plus cardiovascular organ dysfunction<br />

b. Most common organisms<br />

(1) Neonatal period (0-4 weeks)<br />

(a) Group B streptococci<br />

(b) Escherichia coli<br />

(c) Listeria monocytogenes<br />

(2) Infancy and early childhood<br />

(a) 5 pneumoniae (declining due to vaccine)<br />

(b) N meningitidis<br />

(c) S aureus (methicillin susceptible and methicillin resistant)<br />

(d) H influenzae (almost eliminated because of vaccination)<br />

519


PEDIATRIC EMERGENCIES<br />

C. Meningitis<br />

c. Sickle cell disease/trait is associated with an increased risk of sepsis, particularly from encapsulated organisms<br />

such as 5 pneumoniae (400-fold increased risk), Neisseria, and H influenzae (5-fold increased risk). "-/<br />

d. Septic infants (


PEDIATRIC EMERGENCIES<br />

,,--.<br />

(3) Children >2 months<br />

(a)<br />

Ceftriaxone 1 00 mg/kg IV, or<br />

(b) Cefotaxime 50 mg/kg IV<br />

(c) If gram-positive cocci are identified on Gram stain of the CSF and there is concern regarding<br />

the presence of cephalosporin and penicillin-resistant strains of 5 pneumoniae in the area,<br />

vancomycin 15 mg/kg should be added to the above regimens in this age group.<br />

b. Corticosteroid therapy<br />

(1) Steroids neutralize the host response to bacterial cell lysis (because of their anti-inflammatory properties).<br />

(2) When dexamethasone is administered early, it is controversial whether it effectively decreases the<br />

incidence of neurologic sequelae (eg, sensorineural hearing loss) in children with bacterial meningitis<br />

due to H inf/uenzae or S pneumoniae.<br />

(3) Dexamethasone (0.15 mg/kg) can be administered before antibiotic therapy in children 2:6 weeks old<br />

with suspected bacterial meningitis due to H influenzae or S pneumoniae (although there is no clearcut<br />

benefit of steroids in pneumococcal meningitis).<br />

c. Antiviral therapy: acyclovir (10 mg/kg IV every 8 hours) should be administered if there is any clinical<br />

suspicion that the infant has herpes meningoencephalitis, eg, maternal herpes, bloody CSF, focal seizures.<br />

d. Indications for antimicrobial chemoprophylaxis for meningococcal disease<br />

(1) All intimate contacts of patients<br />

(2) Prehospital and hospital personnel with direct exposure to respiratory secretions<br />

(3) All daycare/nursery school contacts of these children<br />

e. Chemoprophylaxis regimens<br />

D. Pneumonia<br />

(1) Rifampin<br />

(a) Infants 1 month: 10 mg/kg orally every 12 hours x 2 days<br />

(c) Adults: 600 mg orally every 12 hours x 2 days<br />

(2) Ciprofloxacin 500 mg orally once<br />

(3) Azithromycin 10 mg/kg orally once (maximal dose 500 mg)<br />

1. Epidemiology and pathophysiology<br />

a. Incidence is greatest in children 6-12 months old and decreases with age.<br />

b. Principal route of acquisition is aspiration of infectio~s particles into the lower respiratory tract.<br />

2. Etiology<br />

a. The principal organisms responsible for producing pneumonia in children vary with:<br />

(1) Age<br />

(2) Immunization status<br />

(3) Comorbid conditions<br />

(4) Daycare attendance<br />

b. Most pneumonias in children (60%-90%) are caused by viruses; it is only in the newborn period that<br />

bacterial pathogens predominate.<br />

(1) Respiratory syncytial virus is the most common viral agent.<br />

(2) 5 pneumoniae remains the predominant bacterial pathogen in all age groups beyond the newborn<br />

period (although the 13-valent protein-polysaccharide pneumococcal conjugate vaccine is changing<br />

this).<br />

(3) Introduction of the H inf/uenzae B vaccine has resulted in a significant decline in the incidence of<br />

infection due to H influenzae.<br />

c. Common causes of pneumonia in children by age (listed in decreasing order of frequency) are the<br />

following:<br />

(1) Newborns 0-2 weeks old<br />

(a) Group B streptococci<br />

(b) E coli<br />

(c) Klebsiella pneumonia<br />

(d) Listeria monocytogenes<br />

521


PEDIATRIC EMERGENCIES<br />

(2) Infants 2 weeks to 3 months old<br />

(a) Viruses (respiratory syncytial virus, parainfluenza, human metapneumovirus)<br />

(b) Chlamydia trachomatis<br />

(c) 5 pneumoniae<br />

(d) H influenzae<br />

(e) 5 aureus<br />

(3) Children 3 months to 4 years old<br />

(a) Viruses<br />

(b) 5 pneumoniae<br />

(c) H influenzae (non-type B where vaccine not prevalent)<br />

(d) Mycoplasma pneumoniae<br />

(4) Children 5-18 years old<br />

(a) Mycoplasma pneumoniae<br />

(b) Viruses<br />

(c) 5 pneumoniae<br />

3. Clinical pictures associated with specific infecting organisms<br />

a. Viral pneumonia: a child presents with a history of preceding upper respiratory infection symptoms (often<br />

with an exanthem), followed by gradual onset of lower respiratory tract symptoms and a low-grade fever.<br />

b. Bordetella pertussis: an afebrile infant with a 1-2 week history of mild upper respiratory tract symptoms<br />

and cough now presents with a severe paroxysmal cough and inspiratory whoop that is frequently<br />

followed by post-tussive emesis.<br />

c. Mycoplasma pneumonia: familial spread of a respiratory tract infection over a long period of time in a<br />

school-aged child with a nonproductive hacking cough, inspiratory rales, and multiple-organ involvement<br />

(pharyngitis, bullous myringitis, otitis media).<br />

d. Chlamydia pneumonia: an infant 2-16 weeks old has a "staccato" cough, is tachypneic, and usually<br />

afebrile; there is often a history of conjunctivitis and nasal congestion.<br />

e. Pneumococcal pneumonia: an older child with abrupt onset of high fever, cough, and tachypnea; chest<br />

radiograph often has focal findings.<br />

f. Staphylococcal pneumonia: a child with radiographic findings of pneumothorax and empyema (or<br />

pyopneumothorax)<br />

4. Diagnostic evaluation<br />

a. Pulse oximetry (to check for hypoxia)<br />

b. Chest radiograph (mainstay of diagnosis)<br />

(1) Useful in confirming the diagnosis and identifying complications<br />

(2) May provide a clue as to the underlying cause of the pneumonia. Although there is considerable<br />

variability, a diffuse interstitial pattern suggests a viral or chlamydia! infection, while alveolar infiltrates<br />

in a lobar distribution are more consistent with a bacterial infection. Pyopneumothorax suggests<br />

staphylococcal pneumonia.<br />

c. CBC (nonspecific)<br />

(1) High WBC count with a left shift suggests bacterial pneumonia.<br />

(2) Lymphocytosis is common with viral, chlamydia!, and pertussis pneumonia; a WBC count of<br />

15,000-40,000/mm 3 with a marked lymphocytosis is characteristic of pertussis.<br />

(3) Normal WBC count and differential is typical of mycoplasmal pneumonia.<br />

(4) WBC count


PEDIATRIC EMERGENCIES<br />

f. Apnea in infants<br />

g. Underlying medical problems, immunodeficiency<br />

h. Presence of complications (pneumatocele, pleural effusion, empyema, pneumothorax)<br />

i. Unresponsive to outpatient therapy<br />

j. Unreliable caregiver<br />

6. Management<br />

a. Viral pneumonia will resolve without specific antibiotic therapy and may be treated with supportive<br />

measures alone when the diagnosis is clear.<br />

b. Bacterial pneumonia necessitates specific antimicrobial therapy that is based on the child's age and the<br />

likely pathogens seen in that age group. Because it is difficult to exclude bacterial disease with certainty on<br />

initial presentation, antibiotics are usually prescribed if the clinical picture suggests a treatable etiology.<br />

c. Antibiotic therapy for bacterial pneumonia<br />

(1) Newborns


PEDIATRIC EMERGENCIES<br />

E. Pertussis (whooping cough)<br />

1. Etiology<br />

a. Bordetella pertussis (a gram-negative coccobacillus) is a highly contagious infection caused by inhalation of<br />

contaminated droplets.<br />

b. Incubation period is 6-20 days with a mean of 7-10 days.<br />

2. Epidemiology<br />

a. Pertussis is seen primarily in nonimmunized or partially immunized children and adolescents but is also<br />

seen in adults because the immunization series does not guarantee life-time protection.<br />

b. Boosters are recommended in adults who have close contact with infants


PEDIATRIC EMERGENCIES<br />

infection, and when it occurs in the presence of bacterial colonization of the nasopharynx, the stage is set<br />

for the development of middle ear infection.<br />

b. Obstruction of the eustachian tube----;. negative pressure in the middle ear and development of a sterile<br />

effusion - aspiration of nasopharyngeal secretions in the middle ear - acute otitis media<br />

c. Infants have short, more horizontal (and more easily collapsible) eustachian tubes than older children.<br />

These anatomic differences predispose infants to eustachian tube obstruction and reflux of oropharyngeal<br />

secretions, thereby placing them at greater risk of development of acute otitis media.<br />

2. Etiology<br />

a. Streptococcus pneumoniae (30%-40%)<br />

b. Moraxella catarrhalis (25%)<br />

c. Nontypeable Haemophi/us influenzae (20%)<br />

d. Streptococcus pyogenes group A (3%)<br />

e. Staphylococcus aureus, group B streptococci, and gram-negative enteric bacilli (may be seen as pathogens<br />

in the neonatal period but are otherwise uncommon)<br />

f. Viral infection is probably the most common cause.<br />

3. Clinical presentation<br />

a. The most reliable sign of acute otitis media is decreased mobility of the tympanic membrane on pneumatic<br />

otoscopy.<br />

b. Other findings include abnormal erythema, distortion or bulging of the tympanic membrane, or loss of<br />

bony landmarks.<br />

4. Selection of antibiotics<br />

a. Some clinicians recommend withholding antibiotics in older children with mild signs and symptoms<br />

for 2-3 days when a virus is the suspected agent. Because most of these infections are self-limited, this<br />

reduces overtreatment and development of resistance.<br />

b. Amoxicillin 80-90 mg/kg/day (divided bid) is still the drug of choice.<br />

c. If J3-lactamase-producing H influenzae or M catarrhalis is suspected or documented, amoxicillinclavulanate<br />

or cefuroxime is preferred.<br />

d. Treatment failure with use of the above agents for 3 days necessitates the use of one of the following broadspectrum<br />

antibiotics:<br />

(1) Amoxicillin-clavulanate 40 mg/kg/day divided bid<br />

(2) Azithromycin suspension 10 mg/kg as a single dose on day 1, then 5 mg/kg on days 2-5<br />

(3) Cefuroxime axetil 30 mg/kg/day divided bid<br />

(4) Cefpodoxime proxetil 10 mg/kg/day divided bid<br />

(5) Cefdinir 14 mg/kg/day (can be divided bid) or clarithromycin 15 mg/kg/day divided bid<br />

(6) Ceftriaxone 50 mg/kg in a single IM dose is particularly useful if vomiting, compliance, or follow-up is<br />

a problem.<br />

Table 34: Recommended Antibiotics for Treatment of Otitis<br />

Initial Antibiotic Treatment<br />

Failure of Initial Antibiotic Treatment<br />

First-line<br />

Alternative<br />

(penicillin-allergy)<br />

First-line<br />

Alternative<br />

Amoxicillin<br />

Cefdinir or<br />

cefuroxime<br />

Amoxicillinclavulanate<br />

Ceftriaxone, cl i ndamyci n,<br />

with or without thirdgeneration<br />

cephalosporin<br />

Amoxicillinclavulanate<br />

Cefpodoxime or<br />

ceftriaxone<br />

Ceftriaxone<br />

Clindamycin plus thirdgeneration<br />

cephalospori n<br />

Tympanocentesis<br />

Consult specialist<br />

525


PEDIATRIC EMERGENCIES<br />

5. The possibility of systemic infection should be considered in infants with acute otitis media who have a fever<br />

and appear toxic (particularly those


PEDIATRIC EMERGENCIES<br />

7. Diagnostic evaluation<br />

a. Urinalysis<br />

(1) Specimen collection<br />

(a) In infants and young children, specimens should be obtained by bladder catheterization or<br />

suprapubic aspiration; bag specimens are often contaminated and are, therefore, unreliable.<br />

(b) In older children who are toilet-trained, an appropriately collected midstream clean-catch<br />

specimen is adequate.<br />

(2) Pyuria (> 10 WBCs per high-power field of spun urine) is suggestive of UT! but not diagnostic (also<br />

occurs in association with other inflammatory processes).<br />

(a) Renal causes<br />

i. Pyelonephritis (WBC casts on urinalysis)<br />

ii. Perinephric abscess<br />

iii. Acute glomerulonephritis (RBCs, RBC casts, and proteinuria)<br />

(b) Extrarenal causes<br />

i. Appendicitis<br />

ii. Vulvovaginitis<br />

iii. Vaginal foreign body<br />

iv. Gastroenteritis<br />

v. Dehydration<br />

vi. Sexually transmitted infections<br />

(3) Bacteriuria is a more reliable finding; there is a 95% probability of infection with 100,000 colonyforming<br />

units/ml. Presence of any bacteria on Gram stain of a noncentrifuged specimen is the best<br />

combination of high sensitivity and a low false-positive rate.<br />

(4) Nitrite test: detects nitrites produced by the reduction of dietary nitrates by urinary gram-negative<br />

bacteria (especially£ coli, Klebsiella spp, and Proteus spp)<br />

(a)<br />

Positive test is virtually diagnostic for UTI (high specificity).<br />

(b) False-negative results are common (low sensitivity).<br />

(5) Leukocyte esterase test<br />

(a)<br />

Detects esterases released from degraded leukocytes (an indirect test for WBCs)<br />

(b) Poor sensitivity and specificity for UTI<br />

b. Urine culture confirms the diagnosis and should be sent for:<br />

(1) Females


PEDIATRIC EMERGENCIES<br />

a. Infants and children are affected most commonly; 80% of cases occur in children 1.5 cm) that is usually unilateral occurs in 50%-75%<br />

of cases.<br />

c. Findings cannot be attributed to another disease process.<br />

4. Incomplete (atypical) Kawasaki disease<br />

a. Diagnostic criteria include fever for >5 days plus<br />

(1) Two or three of above characteristics<br />

(2) C-reactive protein >3 mg/dl, and erythrocyte sedimentation rate >40 mm/hr<br />

b. Management is determined by echocardiogram findings.<br />

5. Disease course is divided into three phases.<br />

a. Acute phase (days 1-11)<br />

(1) Characterized by fever and most of the other diagnostic features described above<br />

(2) Myocarditis is the most common cause of death during this time.<br />

b. Subacute phase (days 11-20)<br />

(1) Characterized by gradual resolution of fever, rash, and adenopathy, as well as the development of<br />

thrombocytosis and periungual desquamation<br />

(2) Risk of developing coronary artery thrombosis is greatest at this time.<br />

(3) Causes of death include myocardial infarction, aneurysm rupture, and myocarditis.<br />

c. Convalescent phase (days 21-60)<br />

(1) Begins when all signs and symptoms have disappeared and continues until the erythrocyte<br />

sedimentation rate and platelet count have normalized.<br />

(2) Death may occur from myocardial infarction secondary to thrombosis.<br />

6. Diagnostic evaluation<br />

a. ECG<br />

b. Echocardiography (to assess cardiac status)<br />

c. Chest radiograph<br />

d. CBC with platelet count<br />

(1) Moderate leukocytosis with left shift (acute phase)<br />

(2) Mild normochromic, normocytic anemia<br />

(3) Increased platelet count (subacute phase)<br />

e. Erythrocyte sedimentation rate increased<br />

f. C-reactive protein increased<br />

g. Serum transaminases mildly increased<br />

h. Urinalysis<br />

528


PEDIATRIC EMERGENCIES<br />

(1) Sterile pyuria (WBCs without bacteria)<br />

(2) Proteinuria<br />

1. Studies to exclude other diagnoses (streptococcal culture, blood culture, ASO titer, etc)<br />

7. Management<br />

a. Hospital admission (unit chosen depends on the extent of cardiac findings)<br />

b. Early consultation with a pediatric cardiologist<br />

c. IV immunoglobulin<br />

(1) Dosage is 2 g/kg over 1 0-12 hours.<br />

(2) When administered early in the illness (within 10 days of onset), it decreases systemic inflammation<br />

and the prevalence of coronary artery aneurysms.<br />

d. Oral aspirin<br />

(1) Anti-inflammatory dosage (80-100 mg/kg/day divided qid) during the acute phase<br />

(2) Antithrombotic dosage (3-5 mg/kg/day as a single dose) during the subacute and convalescent phases<br />

e. Steroids are not indicated and may increase the risk of coronary aneurysm development.<br />

8. Complications<br />

a. Cardiovascular (most important)<br />

(1) A child's prognosis and outcome are primarily determined by the cardiac complications of Kawasaki<br />

disease.<br />

(2) Coronary artery aneurysms and ectasia develop in 20%-30% of untreated children; those


PEDIATRIC EMERGENCIES<br />

g. Treatment is directed at lowering the fever and identifying any underlying infection.<br />

h. Seizures recur in 30%-40% of children; more likely when the first febrile seizure occurs before 1 year old.<br />

i. Prophylaxis with phenobarbital is not routinely recommended; although it is the drug of choice, it has not<br />

been shown to be of significant benefit and interferes with cognitive functioning.<br />

2. Generalized tonic-clonic seizures<br />

a. Clinical presentation<br />

(1) Both hemispheres are involved.<br />

(2) Motor involvement is bilateral.<br />

(3) Consciousness is impaired.<br />

(4) Patients are confused and lethargic during the postictal period.<br />

b. Diagnostic evaluation<br />

(1) Includes a directed history (description of the seizure and surrounding events, pertinent past medical<br />

history) and a complete physical examination<br />

(2) No role for routine laboratory or radiographic evaluation unless suggested by the history and physical<br />

examination.<br />

(3) Tests ordered should reflect the child's age and suspected cause of the seizure as suggested by the<br />

history and physical examination.<br />

(a) Electrolytes<br />

(b) Toxicology screening<br />

(c) Anticonvulsant medication serum levels (if applicable)<br />

(4) Indications for a CT scan of the head<br />

(5) MRI<br />

(a) Focal neurologic deficits<br />

(b) Signs of increased intracranial pressure<br />

(c) Infants 80% of seizures. Use one of the following:<br />

(a) Lorazepam 0.1 mg/kg IV is the preferred agent; it has a long half-life and the least effect on<br />

respiratory depression.<br />

(b) Diazepam 0.2-0.3 mg/kg IV may be used if lorazepam is unavailable; it has a much shorter halflife<br />

and produces a greater respiratory depression.<br />

(c) Midazolam 0.2 mg/kg IM or intranasal should be considered when IV access cannot be rapidly<br />

attained; it works as rapidly by this route as by the IV route but is very short acting.<br />

(d) Diazepam 0.5 mg/kg may also be given rectally if IV access cannot be attained and midazolam is<br />

unavailable; when administered via this route, its onset of action is ~5 minutes.<br />

(2) If seizure activity persists despite administration of a benzodiazepine, fosphenytoin (15-20 phenytoin<br />

equivalents [PE]lkg diluted in normal saline and infused at a rate ::'.50 mg/min) or phenobarbital (15-20<br />

mg/kg infused at a rate of 25 mg/min) should be administered.<br />

(3) Children presenting with their first afebrile, unprovoked seizure who make an uneventful recovery<br />

need not be treated with a long-term anticonvulsant unless they presented in status epilepticus. The<br />

rate of seizure recurrence in these children is quite variable; risk of recurrence within 1 year is ~25%;<br />

children with an underlying neurologic disease have a much higher recurrence rate (16%-62%), and<br />

the adverse effects associated with anticonvulsant therapy are significant. Therefore, it is recommended<br />

that anticonvulsant therapy be withheld until a second seizure occurs in these patients.<br />

(4) Most breakthrough seizures are a result of subtherapeutic drug levels or noncompliance.<br />

(5) Referral is generally indicated for ongoing monitoring.<br />

530


PEDIATRIC EMERGENCIES<br />

3. Infantile spasms<br />

a. Definition: convulsive disorders of infants and young children with onset between 3 and 9 months old and<br />

accompanied by developmental regression<br />

b. Clinical presentation<br />

(1) Spasms last only a split second and are associated with flexion or extension of the head and trunk.<br />

(2) Spasms may occur as a single event or in groups of 5-20 at a time.<br />

c. Electroencephalogram is almost always abnormal with hypsarrhythmia (a slow irregular rhythm) being<br />

present in 50% of cases.<br />

d. Management includes admission and a neurologic consult.<br />

B. Differential diagnosis<br />

1. Syncope<br />

2. Pseudoseizure<br />

3. Breath-holding spells<br />

4. Movement disorders (eg, tics)<br />

5. Night terrors<br />

6. Cardiac dysrhythmias<br />

7. Complicated migraines<br />

C. Etiology<br />

1. Infectious<br />

a. Meningitis/encephalitis<br />

b. Herpes simplex virus<br />

c. Bacterial meningitis<br />

d. Parasitic encephalitis<br />

2. Metabolic<br />

a. Hypoglycemia<br />

b. Hypo- or hypernatremia<br />

c. Hypocalcemia<br />

d. Hypoxia<br />

e. Pyridoxine deficiency<br />

3. Inborn errors of metabolism<br />

4. Traumatic<br />

5. Neoplastic<br />

6. Drug-related (intoxication or withdrawal)<br />

7. Child abuse (eg, shaken baby syndrome)<br />

8. Miscellaneous<br />

a. Congenital malformations<br />

b. Ventriculoperitoneal shunt malfunction<br />

c. idiopathic<br />

D. Adverse effects of medications used in management of pediatric seizures<br />

1. Phenytoin: gum hyperplasia, hirsutism, and drug rashes (Stevens-Johnson syndrome)<br />

2. Phenobarbital: lethargy, hyperactivity, and behavioral disorders<br />

3. Carbamazepine: neutropenia, rashes, hepatic dysfunction, and cardiac toxicity<br />

4. Valproic acid: thrombocytopenia, pancreatitis, hepatic failure, and a bleeding diathesis when used in<br />

conjunction with aspirin<br />

VIII. CEREBROSPINAL FLUID SHUNTS<br />

A. Shunt systems<br />

1. Generally consist of a ventricular catheter, a valve device, and distal tubing<br />

2. Ventriculoperitoneal shunts are the most common type, accounting for >90% of the shunts currently in use.<br />

3. Ventriculoperitoneal shunts allow CSF to be diverted around obstructed areas in the subarachnoid space or<br />

ventricles.<br />

531


PEDIATRIC EMERGENCIES<br />

4. Conditions that result in hydrocephalus and require CSF shunt placement include:<br />

a. Congenital stenosis of the aqueduct of Sylvius<br />

b. Acquired aqueductal stenosis or scarring secondary to infection<br />

c. Malignancy or hemorrhage<br />

d. Dandy-Walker malformations<br />

e. Arnold-Chiari syndrome<br />

5. Shunt failure is common; it can result from:<br />

a. lntracranial infection or blockage<br />

b. Valve malfunction<br />

c. Distal tubal obstruction or disruption<br />

d. Pseudocyst formation or disruption<br />

B. Complications<br />

1. Shunt malfunction<br />

a. Obstruction<br />

(1) Most common cause of shunt malfunction<br />

(2) Results from blockage to flow anywhere along the shunt system and may be caused by kinking,<br />

thrombosis, disconnection, migration, or infection of the catheter.<br />

(3) Patients present with signs of increased intracranial pressure.<br />

(a) Headache with mental status changes and associated nausea and vomiting are the classic<br />

symptoms. Other clues include:<br />

i. Seizures while on therapeutic medication doses<br />

ii. Poor sucking or feeding<br />

iii. Paradoxical crying<br />

(b) Findings on physical examination<br />

i. Upward gaze palsy ("sunsetting" or "sundowning")<br />

ii. Diplopia<br />

iii. Bulging fontanelle<br />

iv. Enlarging head<br />

v. Cushing triad of bradycardia, hypertension, and respiratory ataxia (preherniation syndrome)<br />

(4) If left untreated, it can result in herniation.<br />

b. Slit ventricle syndrome<br />

(1) Results from chronic overdrainage of CSF with collapse of the ventricles and transient obstruction of<br />

CSF flow; overdrainage __,. col lapse of the ventricles and catheter obstruction __,. increased intracranial<br />

pressure__,. reexpansion of the ventricular system and relief of catheter obstruction<br />

(2) Symptoms are those of an intermittently increased intracranial pressure.<br />

(a) Episodic minor headache associated with reduced performance and intervening asymptomatic<br />

periods are characteristic.<br />

(b) Severe headache, nausea, vomiting, and bradycardia progressing to obtundation may occur with<br />

more severe presentations.<br />

(3) CT reveals small ventricles.<br />

(4) Placement of the patient in Trendelenburg position frequently relieves these symptoms.<br />

(5) Definitive treatment involves placement of an antisiphon device.<br />

c. Evaluation of shunt malfunction<br />

(1) Locate the shunt and trace its course, looking for evidence of infection and palpating for<br />

disconnections.<br />

(2) Obtain a shunt survey (plain radiographs of the entire shunt system) to exclude kinks and breaks in<br />

the catheter.<br />

(3) Compress the subcutaneous shunt reservoir and pumping chamber to evaluate ease of pump<br />

compression/refill (may be helpful but should not be used to exclude shunt dysfunction).<br />

(a) When functioning normally, the chamber refills in


PEDIATRIC EMERGENCIES<br />

(4) Determine ventricular size via CT scanning, and compare the results with those of prior studies to see<br />

if there has been a change. In infants with open fontanelles, ventricle size can also be assessed by<br />

ultrasound. Again, comparative studies are necessary.<br />

(a) Enlarged ventricles indicate obstruction.<br />

(b) Smaller or slit ventricles are consistent with overdrainage.<br />

(5) In difficult cases, radionuclide shunt clearance scans or MRI may be helpful.<br />

(6) Management<br />

2. Shunt infections<br />

(a) Obtain urgent neurosurgical consult.<br />

(b) If seizures are secondary to increased intracranial pressure, manage as appropriate.<br />

a. 60% of the mortality that occurs in association with shunts is due to shunt infections.<br />

b. Most infections (75%) occur within the first 2 months after shunt placement and are caused by<br />

S epidermidis and 5 aureus.<br />

c. Resulting shunt obstruction that leads to increased intracranial pressure is common.<br />

d. Clinical presentation<br />

(1) Fever is the most common symptom of shunt infection.<br />

(2) Other findings<br />

(a) Nausea, vomiting, headache, and feeding problems<br />

(b) Meningismus<br />

(c) A subtle change in behavior as detected by the parents<br />

(3) Signs of increased intracranial pressure<br />

e. Diagnostic evaluation<br />

(1) Examine the course of the shunt looking for evidence of infection; although this is very specific for<br />

shunt infection, it is not sensitive.<br />

(2) Obtain laboratory studies.<br />

f. Management<br />

(a) CBC with differential (may be normal)<br />

(b) Blood culture<br />

(c) CSF from the shunt (consult with neurosurgeon) for evaluation of samples for cell count and<br />

differential, glucose, Gram stain, protein, and culture and sensitivity<br />

(1) Obtain urgent neurosurgical consult, and admit these patients to the hospital for antibiotic therapy (IV<br />

and/or intraventricular) and externalization of the shunt.<br />

(2) Shunt removal and replacement is required in 50% of patients.<br />

IX. PEDIATRIC GASTROINTESTINAL/GENITOURINARY<br />

EMERGENCIES<br />

A. Abdominal conditions associated with rectal bleeding<br />

1. Infectious colitis: infection with the organisms listed below can result in bloody diarrhea.<br />

a. Shigella<br />

b. Salmonella<br />

c. Enteroinvasive E coli<br />

d. Campylobacter jejuni<br />

e. Yersinia enterocolitica<br />

f. C!ostridium difficile<br />

g. Entamoeba histolytica<br />

h. Enterohemorrhagic E coli serotype O157:H7<br />

2. Painless rectal bleeding<br />

a. Anal fissure (most common cause in infants)<br />

b. Swallowed maternal blood<br />

(1) Occurs in infants


PEDIATRIC EMERGENCIES<br />

c. Meckel's diverticulum<br />

d. Infectious gastroenteritis<br />

e. Juvenile polyps<br />

3. Meckel's diverticulum can produce a variety of signs and symptoms.<br />

a. It is a true diverticulum (all layers of the bowel wall are present) and can contain a variety of heterotopic<br />

tissue, of which gastric mucosa is the most common.<br />

b. Painless rectal bleeding (which can be massive) is the most common presentation of this congenital<br />

anomaly in children


PEDIATRIC EMERGENCIES<br />

d. Diagnostic evaluation<br />

(1) Plain radiographs should be obtained first.<br />

(a) Findings<br />

i. An abdominal mass or filling defect in the right upper quadrant<br />

ii. Signs of bowel obstruction<br />

iii. Free intraperitoneal air<br />

(b) Normal plain radiographs do not exclude the diagnosis.<br />

(2) Ultrasound evaluation is a quick and sensitive modality to identify an intussusception. However, air<br />

insufflation and barium enema are the preferred diagnostic and treatment studies.<br />

(a) Air-contrast is as effective as barium in reducing the intussusception and has replaced the<br />

barium enema as the study of choice in most major medical centers; successful reduction occurs<br />

in 60%-80% of cases.<br />

(b) If barium enema is performed, the classic finding of a "coiled spring" appearance may be seen<br />

on the evacuation radiograph.<br />

e. Operative intervention is required for patients in whom the intussusception cannot be reduced by air<br />

insufflation or barium enema.<br />

f. Recurrence rate is 5%-10% after barium enema reduction and 2%-5% after surgical reduction, with most<br />

recurrences occurring in the first 24 hours after reduction.<br />

B. Abdominal/genitourinary conditions not associated with rectal bleeding (these are all surgical mergencies)<br />

1. Incarcerated hernia<br />

a. Clinical presentation<br />

(1) Common in children


PEDIATRIC EMERGENCIES<br />

c. Clinical presentation<br />

(1) Nonbilious projectile vomiting after feeding, after which the infant usually hungrily re-feeds<br />

(2) Evidence of dehydration and failure to thrive<br />

(3) Prominent peristaltic waves (sometimes)<br />

(4) Palpable olive-shaped mass (the hypertrophied pylorus) in the right upper quadrant (pathognomonic)<br />

(5) Jaundice (~5% of cases)<br />

d. Diagnostic evaluation<br />

(1) When the "olive" can be palpated, the diagnosis is established and further diagnostic studies are<br />

unnecessary.<br />

(2) When the "olive" cannot be palpated, ultrasound is the diagnostic study of choice; a hypoechogenic<br />

ring with a hyperdense center that resembles a "doughnut" or "bull's-eye" can be seen.<br />

(3) If the ultrasound is unavailable or negative, an upper GI series should be performed. Findings include<br />

the following:<br />

(a) Delayed gastric emptying<br />

(b) Indentation of the antrum by the "olive"<br />

(c) Elongation of the pyloric channel (the "string" or "beak" sign)<br />

(4) Hypochloremic, hypokalemic metabolic alkalosis<br />

e. Management<br />

4. Appendicitis<br />

(1) Correct dehydration and electrolyte imbalance.<br />

(2) Restrict oral feedings.<br />

(3) Obtain early surgical consult; definitive treatment (once fluid and electrolyte balance have seen<br />

restored) is pyloromyotomy.<br />

a. Epidemiology<br />

(1) Most common nontraumatic surgical emergency in children<br />

(2) Peak incidence 9-12 years old<br />

(3) Rare in children 10,000 cells/mm 3 ) is associated with the presence of appendicitis but<br />

has poor sensitivity and specificity and no clinical utility. Urinalysis may reveal sterile pyuria if the<br />

inflamed appendix is overlying the ureter.<br />

(2) Chest radiograph with supine and upright abdominal films are helpful in excluding pneumonia<br />

presenting as abdominal pain; abdominal radiograph may demonstrate a fecalith (10% of cases), loss<br />

of psoas shadow, localized ileus in right lower quadrant.<br />

(3) Ultrasound may be diagnostic but is highly operator dependent.<br />

(4) CT scanning is used to confirm the diagnosis.<br />

d. Findings suggestive of perforation<br />

(1) High temperature (>102°F [39°Cl)<br />

(2) High WBC count(> 15,000/mm 3 )<br />

(3) Symptoms >36 hours<br />

e. Management<br />

(1) IV fluid resuscitation<br />

(2) Antibiotics (ampicillin, gentamicin, and metronidazole or clindamycin; piperacillin/tazobactam or<br />

cefoxitin; meropenem if perforation)<br />

(3) Immediate surgical consult<br />

536


PEDIATRIC EMERGENCIES<br />

5. Testicular torsion<br />

a. General<br />

(1) Annual incidence 4.5 in 100,000 males 1-25 years old<br />

(2) Cannot be diagnosed by physical examination alone<br />

(3) Salvage rate: classically 80%-1 00% within 6 hours of onset and


PEDIATRIC EMERGENCIES<br />

4. Clues to the diagnosis<br />

a. Laboratory results do not "fit."<br />

b. Signs and symptoms do not respond to appropriate medical management.<br />

c. Signs and symptoms resolve when the child is separated from the perpetrator.<br />

5. Management<br />

a. Admit to hospital with continuous camera monitoring to assure safety.<br />

b. Institute appropriate therapy.<br />

c. Involve counseling and social services.<br />

C. Sexual abuse<br />

1. 20% of all girls and 9% of boys are sexually abused. The abuse has often occurred over a long period of time<br />

and, in 90% of cases, the perpetrator is well known to the child (eg, family member, relative).<br />

2. Physical examination<br />

a. Best performed with the child supine in the frog-leg position or prone in the knee-chest position; can be<br />

augmented by the use of a hand-held lens or colposcope<br />

b. Suggestive findings<br />

(1) Hymenal notches (concavities), scars, or tears<br />

(2) Vaginal discharge<br />

(3) Genital ulcers, warts, or vesicles<br />

(4) Anal fissures, hematomas, or tears; perinea! bruising<br />

(5) Immediate reflex anal dilatation (>20 mm) in the absence of feces in the ampulla<br />

c. The absence of physical findings does not exclude abuse; they are present in only 10% of cases of<br />

documented sexual abuse.<br />

3. Laboratory evaluation<br />

a. Cultures (not antigen assays) of:<br />

(1) Vagina or urethra-,, for Neisseria gonorrhoeae or Chlamydia trachomatis<br />

(2) Rectum -,, for N gonorrhoeae<br />

(3) Throat-,, for N gonorrhoeae<br />

b. Serologic testing for syphilis<br />

c. HIV testing (after counseling) if infection is suspected<br />

d. Pregnancy test if appropriate<br />

4. Management<br />

a. Gonoccoccal prophylaxis: indicated if abuse occurred


PEDIATRIC EMERGENCIES<br />

d. "Bucket handle" fractures<br />

e. Rib fractures<br />

f. Untreated healing fractures<br />

g. Shaken baby syndrome<br />

(1) Usually presents as unexplained seizures, abnormal behavior, unresponsiveness, or respiratory distress<br />

(2) Triad of severe intracranial bleeding (subdural, intraparenchymal, interhemispheric, epidurallt retinal<br />

hematomas (present in 70%-90% of casest and no or minimal external signs of trauma<br />

(3) Symptoms are the result of increased intracranial pressure.<br />

(4) Associated posterior rib fractures are common.<br />

h. Complex skul I fractures<br />

i. Duodenal hematomas from abdominal trauma<br />

j. Cigarette burns<br />

k. Immersion burns in a "glove-and-stocking" pattern<br />

5. Diagnostic evaluation<br />

a. Laboratory evaluation is guided by findings on examination and includes CBC platelet count and<br />

prothrombin time (INR)/partial thromboplastin time when the examination reveals multiple bruises and<br />

when there is any suggestion by the caregiver of easy bruisability.<br />

b. Radiographic evaluation<br />

(1) A skeletal survey when evidence of trauma is noted<br />

(2) CT of the head when retinal hemorrhages or an acute deterioration in neurologic status is present<br />

(3) Abdominal/pelvic CT for abdominal wall bruising, unexplained vomiting or abdominal pain, shock,<br />

anemia, or abnormal liver function tests<br />

6. Management<br />

a. Complete history and physical examination with verbatim quotations if possible<br />

b. Suspected cases must be reported to the police and/or child protection services.<br />

c. Hospitalization or placement in protective custody may be necessary to protect the child from further harm.<br />

Other children in the household must also be evaluated.<br />

d. Failure to report a suspected case may lead to misdemeanor charges and further harm or death to the child<br />

who is returned to the abusive environment.<br />

539


PEDIATRIC EMERGENCIES: PRACTICE CLINICAL SCENARIOS<br />

PEDIATRIC EMERGENCIES: PRACTICE CLINICAL SCENARIOS<br />

Answers immediately follow the practice clinical scenarios.<br />

Scenario A<br />

Presentation: A 4-year-old unimmunized child is sitting upright with chin thrust forward, mouth open, and<br />

neck slightly extended. He appears toxic and apprehensive and, on closer inspection, is drooling, with<br />

difficulty swallowing and stridor. The mother relates that the child was well until just a few hours ago,<br />

when he started to complain of a sore throat and developed a fever.<br />

What is the diagnosis?<br />

Scenario B<br />

Presentation: A 2-year-old child has a barky, "seal-like" cough that the mother says is usually worse at<br />

night and sometimes causes severe respiratory distress. The child had a recent upper respiratory infection,<br />

and the cough, hoarseness, and stridor appeared just a few days after. The child does not have a fever and<br />

is nontoxic in appearance.<br />

What is the diagnosis?<br />

Scenario C<br />

Presentation: A 2-year-old child has a high fever, stridor, and appears toxic. His mother reports that he has<br />

had nasal congestion, barking cough, and stridor for the past few days and then suddenly took a turn for<br />

the worse over the past few hours.<br />

What is the diagnosis?<br />

Scenario D<br />

Presentation: A young child has had a mild fever, rhinorrhea, and conjunctivitis for the last week or<br />

two. She is now having numerous episodes of unremitting paroxysmal coughing followed by a "whoop."<br />

Vomiting often occurs in association with these paroxysms.<br />

What is the diagnosis?<br />

540


PEDIATRIC EMERGENCIES: PRACTICE CLINICAL SCENARIOS<br />

ANSWERS TO PRACTICE CLINICAL SCENARIOS<br />

Scenario A<br />

Diagnosis: epiglottitis<br />

Diagnostic evaluation: If there is only slight respiratory distress, the diagnosis can be confirmed with<br />

a portable lateral soft-tissue radiograph of the neck. Findings include the "thumb print sign" and<br />

mild hypopharyngeal distention. Evaluation of the epiglottis by using a tongue blade (or a fiberoptic<br />

bronchoscope) should be avoided in children with any respiratory distress. Definitive diagnosis is ideally<br />

made in the operating room under controlled conditions by direct visualization of the "cherry-red"<br />

epiglottis. This is particularly true when severe respiratory distress is present.<br />

Management: Disturb the child as little as possible to reduce anxiety, because he is at high risk of<br />

complete airway obstruction. Allow him to assume a position of comfort (usually sitting upright) and<br />

remain with his parents. Do not force him to lie supine.<br />

Provide supplemental humidified oxygen, set up airway stabilization equipment at bedside, and obtain<br />

immediate ENT and anesthesia consults. Once the airway has been stabilized, draw blood for laboratory<br />

studies (including cultures), and administer parenteral antibiotics (cefuroxime, cefotaxime, ceftriaxone, or<br />

ampicillin-sulbactam). Admit to ICU.<br />

Scenario B<br />

Diagnosis: croup<br />

Diagnostic evaluation: The diagnosis can often be made on clinical grounds, but radiographs can be useful<br />

to exclude epiglottitis. The PA neck radiograph may reveal symmetric subglottic narrowing of the tracheal<br />

air column ("steeple sign"), and the lateral neck radiograph may show a distended hypopharynx and<br />

subglottic narrowing. The epiglottis and retropharyngeal space are normal.<br />

Management: Management is supportive, including cool mist, oxygen as needed, hydration (orally or<br />

IV), racemic epinephrine aerosol, steroids (one dose), and heliox if croup is severe or respiratory distress<br />

continues despite steroids and epinephrine. Consider admission if the child has persistent stridor at rest,<br />

cannot tolerate oral fluids, or has an incomplete response to epinephrine treatment.<br />

Scenario C<br />

Diagnosis: bacterial tracheitis<br />

Management: Provide supplemental humidified oxygen, and obtain immediate consult with ENT and<br />

anesthesiology for endoscopy. Secure the airway, and obtain appropriate laboratory studies (including a<br />

Gram stain and culture of tracheal secretions). Administer IV hydration and antibiotics effective against<br />

Staphylococcus aureus. Admit to ICU.<br />

Scenario D<br />

Diagnosis: pertussis (whooping cough)<br />

Diagnostic evaluation: A chest radiograph may show peribronchial thickening or a "shaggy" heart border.<br />

If the primary illness is complicated by pneumonia, infiltrates may be seen. The WBC count is 35,000/mm3<br />

with a marked absolute lymphocytosis. The organism can be cultured using nasopharyngeal or sputum<br />

specimens, or detected with fluorescent antibody staining (or PCR) of a nasopharyngeal swab specimen.<br />

Management: Begin erythromycin or azithromycin therapy and provide supplemental oxygen and<br />

postural drainage. Admit if the child has hypoxia and cyanosis during coughing spells or apnea. Monitor<br />

for pulmonary (eg, pneumonia, pneumothorax, hypoxia) and neurologic (eg, seizures, encephalitis)<br />

comp I ications.<br />

541


542<br />

NOTES


TOXICOLOGIC DISORDERS<br />

TOXICOLOGIC DISORDERS<br />

General Approach to the Poisoned Patient ................................................................................................................ 547<br />

History ............................................................................................................................................................... 547<br />

Physical Examination ......................................................................................................................................... 547<br />

Toxidromes ......................................................................................................................................................... 548<br />

Major Toxic Signs ............................................................................................................................................... 550<br />

Diagnostic Evaluation ......................................................................................................................................... 553<br />

General Treatment Measures .............................................................................................................................. 553<br />

Other Complications .......................................................................................................................................... 556<br />

Specific Overdoses and Poisonings ........................................................................................................................... 557<br />

Cyclic Antidepressants ....................................................................................................................................... 557<br />

Other Antidepressants ........................................................................................................................................ 558<br />

Lithium ............................................................................................................................................................... 559<br />

Sedative-Hypnotics ............................................................................................................................................ 559<br />

Phenothiazines ................................................................................................................................................... 561<br />

Phenytoin ........................................................................................................................................................... 562<br />

Opioids .............................................................................................................................................................. 562<br />

Cioni dine ........................................................................................................................................................... 563<br />

Alcohols ............................................................................................................................................................. 563<br />

Cocaine .............................................................................................................................................................. 566<br />

Amphetamines and Amphetamine-Like Drugs .................................................................................................... 567<br />

Hallucinogens .................................................................................................................................................... 568<br />

Salicylates .......................................................................................................................................................... 569<br />

Acetaminophen .................................................................................................................................................. 570<br />

Iron .................................................................................................................................................................... 571<br />

Hydrocarbons .................................................................................................................................................... 572<br />

Caustic Ingestions ............................................................................................................................................... 574<br />

Organophosphates ............................................................................................................................................. 576<br />

Mushroom Poisoning .......................................................................................................................................... 578<br />

Cyanide .............................................................................................................................................................. 579<br />

Digitalis .............................................................................................................................................................. 579<br />

f3-Blockers .......................................................................................................................................................... 580<br />

Calcium Channel Blockers ................................................................................................................................. 581<br />

Carbon Monoxide .............................................................................................................................................. 581<br />

Mercury ............................................................................................................................................................. 582<br />

Sulfonylureas ...................................................................................................................................................... 583<br />

543


TOXICOLOGIC DISORDERS: SELF-ASSESSMENT QUESTIONS<br />

TOXICOLOGIC DISORDERS: SELF-ASSESSMENT QUESTIONS<br />

1. All of the following are causes of a cholinergic toxidrome except:<br />

(a) Organophosphates<br />

(b) Sarin<br />

(c) Cyclic antidepressants<br />

(d) Carbamates<br />

2. Which is true regarding the end-organ damage associated with untreated methanol poisoning?<br />

(a)<br />

Renal failure can be permanent and develops soon after overdose.<br />

(b) Blindness can take over 24 hours to develop.<br />

(c) Renal damage associated with overdose is generally limited to acute tubular nephrosis and resolves after<br />

aggressive IV fluid rehydration.<br />

(d) Funduscopic examination may reveal petechial hemorrhages.<br />

3. GI hemorrhage is characteristic of which toxic ingestion?<br />

(a)<br />

Lithium<br />

(b) Iron<br />

(c) Lead<br />

(d) Household bleach<br />

4. The antidote for iron poisoning is:<br />

(a)<br />

Dimercaprol<br />

(b) Deferoxamine<br />

(c) N-acetylcysteine<br />

(d) Calcium disodium edetate (EDTA)<br />

5. Severe salicylate poisoning (serum level ~100 mg/dl) requires which of the following treatments immediately?<br />

(a)<br />

N-acetylcysteine<br />

(b) Acidification of the urine<br />

(c) Administration of chelator<br />

(d) Hemodialysis<br />

6. Clinical findings that indicate a serious overdose of a cyclic antidepressant include all of the following except:<br />

(a) Ventricular dysrhythmias<br />

(b) Wide QRS complex<br />

(c) AV block<br />

(d) Hypertension<br />

7. All of the following would be appropriate for treatment of symptomatic organophosphate poisoning except:<br />

(a)<br />

Physostigmine<br />

(b) Benzodiazepines<br />

(c) Pralidoxime<br />

(d) Atropine<br />

8. A homeless man arrives at the emergency department appearing intoxicated. As part of his diagnostic evaluation,<br />

the following laboratory studies are ordered. What is this patient's serum osmolarity?<br />

Sodium 125 mmol/L BUN 2.8 mg/dL<br />

Potassium 3.1 mmol/L Creatinine 1.8 mg/dl<br />

Chloride 100 mmol/L Glucose 360 mg/dL<br />

CO 2 22 mmol/L Ethanol 0.207 g/L<br />

(a) 294<br />

(b) 288<br />

(c) 261<br />

(d) 271<br />

544


TOXICOLOGIC DISORDERS: SELF-ASSESSMENT QUESTIONS<br />

9. The treatment of choice for ventricular dysrhythmias secondary to phenothiazine overdose is:<br />

(a) Acidification of the urine<br />

(b) Sodium bicarbonate<br />

(c) Phenytoin<br />

(d) Procainamide<br />

10. Seizures commonly occur after overdose of which of the following?<br />

(a) Morphine<br />

(b) Tramadol<br />

(c) Oxycodone<br />

(d) Fentanyl<br />

11. Early signs and symptoms of ethanol withdrawal include:<br />

(a)<br />

Hypertension, tachycardia, and irritability<br />

(b) Diaphoresis and dehydration<br />

(c) Hypotension, bradycardia, and fever<br />

(d) Visual hallucinations and paranoid ideation<br />

12. Cocaine-induced cardiac dysrhythmias should be treated with any of the following medications except:<br />

(a) Benzodiazepines<br />

(b) Sodium bicarbonate<br />

(c) Calcium channel blockers<br />

(d) P-blockers<br />

13. An elderly patient who presents with unexplained CNS dysfunction and arterial blood gases that reveal a mixed<br />

acid-base disturbance should be suspected of having:<br />

(a) Chronic lithium poisoning<br />

(b) Acetaminophen poisoning<br />

(c) Chronic salicylate poisoning<br />

(d) Iron poisoning<br />

14. Indications for hemodialysis in patients with acute salicylate toxicity include all of the following except:<br />

(a) Severe acid-base imbalance<br />

(b) Increasing serum ASA levels despite aggressive therapy<br />

(c) Urine pH


TOXICOLOGIC DISORDERS: SELF-ASSESSMENT QUESTIONS<br />

18. Seizures are an indication for hemodialysis in all of the following poisonings except:<br />

(a) Digoxin<br />

(b) Lithium<br />

(c) Theophylline<br />

(d) Methanol<br />

19. Which of the following is potentially harmful if given to a fire victim with suspected cyanide poisoning?<br />

(a) Hydroxocobalamin<br />

(b) Supplemental oxygen<br />

(c) Amyl nitrate<br />

(d) Sodium thiosulfate<br />

20. Which of the following is false regarding button battery ingestions?<br />

(a)<br />

Immediate surgical consult should be obtained if GI tract perforation is suspected.<br />

(b) If radiography demonstrates the battery is lodged in the esophagus, immediate endoscopic removal is<br />

indicated.<br />

(c) If radiography demonstrates the battery has entered the stomach, the patient should be monitored on an<br />

outpatient basis until the battery has passed.<br />

(d) If radiography demonstrates the battery has entered the stomach, immediate endoscopic removal is indicated.<br />

21. Which is least appropriate for the management of asymptomatic pediatric sulfonylurea ingestions?<br />

(a)<br />

Frequent blood glucose testing (every 15-20 minutes)<br />

(b) Empiric administration of glucose<br />

(c) Thorough physical examination, looking for signs of hypoglycemia<br />

(d) Hospital admission<br />

ANSWERS<br />

1. C<br />

2. b<br />

5.<br />

6.<br />

d<br />

d<br />

9.<br />

10.<br />

b<br />

b<br />

13. C 17. b<br />

14. C 18. a<br />

3. b 7. a 11. a 15. b 19. c<br />

4. b 8. d 12. d 16. c 20. d<br />

21. b<br />

Use the pre-chapter multiple choice question worksheet (page xvi) to record and determine the percentage of correct<br />

answers for this chapter.<br />

546


TOXICOLOGIC DISORDERS<br />

I. GENERAL APPROACH TO THE POISONED PATIENT<br />

A. History<br />

1. Determine the involved poison.<br />

2. Determine route of exposure: ingestion, inhalation/insufflation, injection, dermal, etc.<br />

3. Determine how much was taken and inquire about extended-release preparations.<br />

4. Determine when it was taken.<br />

5. Determine whether the exposure was accidental or intentional<br />

a. Consider child abuse in patients 5 years old.<br />

6. Determine what the patient was doing at the time he or she became ill. Examples:<br />

a. Fumigating a ship --,, cyanide<br />

b. Working in a garage--,, carbon monoxide, pesticides, toxic alcohols<br />

c. Applying chemicals to crops --,, organophosphates/carbamates<br />

B. Physical examination<br />

Table 35: Physiologic Effects Associated with Specific Poisons<br />

Poison<br />

Cyclic antidepressants<br />

Barbiturates<br />

Phenothiazines<br />

(eg, chlorpromazine,<br />

prochlorperazine)<br />

Digitalis<br />

Opioids<br />

Clonidine<br />

Ethanol, ethylene glycol,<br />

methanol, isopropanol<br />

Cocaine<br />

t pulse"<br />

t blood pressure<br />

t respiration<br />

t blood pressure<br />

t respiration<br />

t temperature 6<br />

t pulse<br />

t blood pressure<br />

t temperature<br />

t pulse"<br />

.j, blood pressure<br />

t pulse<br />

t blood pressure'<br />

.J, respiration<br />

.j, temperature<br />

Vital Signs<br />

.j, pulse'<br />

t blood pressure early (uncommon),<br />

then .j, blood pressure later<br />

.J, respiration (uncommon)<br />

.j, temperature<br />

t pulse<br />

t blood pressure<br />

t respirationa, c<br />

t temperature 6<br />

t pulse<br />

t blood pressure<br />

t or t res pi ration<br />

t temperature<br />

Level of Consciousness<br />

t (mostly in children)<br />

547


TOXICOLOGIC DISORDERS<br />

Table 35: Physiologic Effects Associated with Specific Poisons (continued)<br />

Poison<br />

Amphetamines<br />

Phencyclidine (PCP)<br />

Salicylates<br />

Iron<br />

Liquid petroleum distillates<br />

(gasoline, kerosene)<br />

Anticholinergics<br />

Organophosphates/carbamates<br />

Sedative-hypnotics<br />

a Earliest sign<br />

t pulse<br />

t blood pressure<br />

t temperature<br />

t pulse<br />

t blood pressure<br />

t temperature<br />

t res pi ration<br />

t temperature"<br />

t blood pressure<br />

t pulse<br />

t respirationa<br />

t temperature<br />

t pulse"<br />

t temperature<br />

t blood pressure<br />

Vital Signs<br />

t pulse early", then t pulse later<br />

t or t blood pressure<br />

t res pi ration<br />

t 0 2<br />

saturation<br />

t pulse<br />

t blood pressure<br />

t temperatureb<br />

Level of Consciousness<br />

b Poisoning can cause cutaneous vasodilation, resulting in poikilothermia. Because the ambient temperature is generally lower than<br />

body temperature, the more common clinical manifestation is hypothermia. Be aware that poisoning while exposed to high ambient<br />

temperatures (eg, unconscious victim in a hot car) can manifest hyperthermia by the same mechanism.<br />

c Depressed respiratory rate is an early sign associated with intoxication by various alcohols. In cases of ethylene glycol and<br />

methanol poisoning, when metabolic acidosis may occur, hyperventilation may occur as a compensatory mechanism and is<br />

expected to be a later finding.<br />

C. Toxidromes<br />

1. Cholinergic (wet manifestations)<br />

a. Clinical presentation<br />

(1) "DUMBBELS"<br />

_!2.iarrhea<br />

.!lrination<br />

Miosis<br />

_!!radycardia<br />

_!!ronchospasm/bronchorrhea<br />

fmesis<br />

.!,_acrimation<br />

Salivation<br />

(2) Seizures, coma, and respiratory failure<br />

(3) Muscle fasciculations and weakness<br />

b. Examples<br />

(1) Organophosphates<br />

(a) Insecticides (eg, parathion, diazinon)<br />

(b) Nerve agents (sarin, soman, VX)<br />

548


TOXICOLOGIC DISORDERS<br />

(2) Carbamates<br />

(a) Insecticides (eg, carbaryl)<br />

(b) Physostigmine, pyridostigmine (mostly peripheral symptoms)<br />

(3) Muscarine-containing mushrooms (Clitocybe, lnocybe)<br />

2. Anticholinergic (dry manifestations)<br />

a. Clinical presentation<br />

(1) Mad as a hatter (delirium), hot as a hare (hyperthermia), blind as a bat (loss of ability to accommodate<br />

near vision and mydriasis), dry as a bone (dry mucous membranes, full as a flask (urinary retention),<br />

bloated as a toad (decreased bowel sounds/ileus), red as a beet (flushing), tacky as a leisure suit<br />

(tachycardia)<br />

(2) Seizures<br />

b. Examples<br />

(1) Cyclic antidepressants<br />

(2) Atropine, scopolamine<br />

(3) Antihistamines<br />

(4) Jimsonweed<br />

3. Opioid drugs<br />

a. Clinical presentation<br />

(1) Clinical triad: coma, respiratory depression, and miosis<br />

(2) Miosis does not always occur (eg, meperidine, propoxyphene, diphenoxylate and atropine).<br />

(3) Clonidine and other central alpha 2<br />

-agonists (guanfacine, tizanidine) can cause a syndrome identical<br />

to the opioid triad.<br />

4. Sedative-hypnotics<br />

a. Clinical presentation: CNS and respiratory depression<br />

b. Examples: barbiturates, ethanol, benzodiazepines, gamma hydroxybutyric acid, others<br />

5. Sympathomimetics<br />

a. Clinical presentation<br />

(1) CNS stimulation<br />

(2) May lead to toxic psychosis; presence of abnormal vital signs (increased blood pressure, wide<br />

pulse pressure, hyperthermia), disorientation, and clouding of consciousness help differentiate from<br />

psychiatric conditions (schizophrenia).<br />

b. Examples: amphetamines, cocaine, PCP, bath salts<br />

6. Hallucinogens<br />

a. Clinical presentation<br />

(1) Hallucinations or cognitive disorders, although patients remain oriented to person, place, and time<br />

(2) Tachycardia, hypertension, and mydriasis<br />

b. Examples: LSD, certain species of mushrooms (Psilocybe, Conocybe, etc), parotid secretions of Bufo sp<br />

toads (hallucinogenic component of secretions formerly thought to be bufotenine but is probably actually<br />

5-Me-0-DMT), DMT, designer drugs of tryptamine class (foxy methoxy, AMT), ketamine<br />

7. Extrapyramidal<br />

a. Clinical presentation: a 11 parkinsonian' 1 picture: "TROD"<br />

Iremor, torticollis, trismus<br />

Rigidity<br />

Qpisthotonos, oculogyric crisis<br />

Qysphonia, dysphagia<br />

b. Examples: all of the "-zines" (chlorpromazine, promethazine, etc), haloperidol, metoclopramide<br />

8. Serotonin syndrome (mimics neuroleptic malignant syndrome)<br />

a. Clinical presentation<br />

(1) Altered mental status, seizures, coma<br />

(2) Neurologic findings: clonus (most important finding), tremor, hyperreflexia, muscle rigidity<br />

(3) Autonomic instability (hyperthermia, hypertension, tachycardia, tachypnea)<br />

b. Two sets of diagnostic criteria: Hunter's criteria and Sternbach's criteria<br />

c. Usually precipitated by overdose or drug-drug interaction between several serotonergic agents: SSRls,<br />

monoamine-oxidase inhibitors, lithium, meperidine, or dextromethorphan<br />

549


TOXICOLOGIC DISORDERS<br />

d. Treatment<br />

(1) Benzodiazepines<br />

(2) Cyproheptadine (first-generation histamine [H 1<br />

] blocker with nonspecific serotonin antagonism)<br />

9. Hemoglobinopathies<br />

a. Carboxyhemoglobinemia-clinical presentation<br />

(1) Headache<br />

(2) Nausea, vomiting, flu-like syndrome<br />

(3) Syncope, tachypnea, tachycardia, chest pain<br />

(4) Confusion, coma, convulsions<br />

(5) Cardiovascular collapse, respiratory failure<br />

b. Methemoglobinemia-clinical presentation<br />

(1) Cyanotic appearance (often more severe than would be expected based on degree of<br />

methemogl obi nem ia)<br />

(2) Chocolate-colored blood despite exposure to air<br />

(3) Asymptomatic (healthy patients with


TOXICOLOGIC DISORDERS<br />

(3) Differential diagnosis of anion gap metabolic acidosis:<br />

Methanol, metformin<br />

!J.remia<br />

Q_iabetic or alcoholic ketoacidosis<br />

faraldehyde, phenformin<br />

lsoniazid, iron, or inhalants (carbon monoxide, cyanide, hydrogen sulfide)<br />

lactic acid<br />

_Ethylene glycol (ethanol may produce a small gap)<br />

S.alicylates, solvents<br />

(4) Poisonings with a low anion gap (


TOXICOLOGIC DISORDERS<br />

6. Seizures<br />

a. Anticholinergics (cyclic antidepressants, antihistamines)<br />

b. Phencyclidine (PCP)<br />

c. LSD<br />

d. Stimulants (cocaine, amphetamines, theophylline)<br />

e. Sedative-hypnotic withdrawal<br />

f. Carbon monoxide<br />

g. Opioids (meperidine, tramadol)<br />

h. Disulfiram<br />

i. Organophosphates<br />

j. Camphor<br />

k. Phenothiazines<br />

I. lsoniazid (classic board scenario is status epilepticus)<br />

m. Monomethylhydrazine-containing mushrooms (Cyromitra) (classic board scenario is status epilepticus)<br />

n. Carbamazepine<br />

o. Monoamine-oxidase inhibitors (toxicity or drug/food interactions)<br />

p. Lindane<br />

q. Nicotine<br />

r. Gamma hydroxybutyrate<br />

7. Seizure mimics: strychnine, tetanus<br />

Note: These patients will have a normal level of consciousness despite abnormal, seizure-like movements;<br />

however, mental status may appear altered because of extreme pain.<br />

8. Pulmonary edema<br />

a. Opioids<br />

b. Salicylates<br />

c. Sedative-hypnotics<br />

d. Carbon monoxide<br />

e. Cardiac drugs: digoxin, ~-blockers, calcium channel blockers<br />

f. Toxic inhalations: chlorine, N 2<br />

O, phosgene, zinc chloride<br />

g. Cocaine<br />

h. Organophosphates<br />

9. Breath odor<br />

a. lsopropyl alcohol - fruit-like<br />

b. Phenols, creosol- disinfectants<br />

c. Cyanide - bitter almonds or silver polish<br />

d. Chloral hydrate-;, pear<br />

e. Organophosphates, arsine, phosphorus - garlic<br />

f. Nitrobenzene - shoe polish, aniline dye<br />

g. Turpentine-;, violets<br />

h. Carbon tetrachloride - cleaning fluid<br />

1. Ethchlorvynol -;, new vinyl shower curtain<br />

j. Hydrogen sulfide - rotten eggs<br />

k. Camphor, naphthalene, p-dichlorobenzene -- mothballs<br />

I. Phosgene - hay<br />

m. Methyl salicylate - wintergreen<br />

10. Skin findings<br />

a. Needle tracks: IV drug use<br />

b. Pressure sores and bullae: barbiturates, carbon monoxide, sedative-hypnotics (snake and spider bites also<br />

cause bullae)<br />

c. "Boiled lobster" skin: boric acid (roach tablets)<br />

d. Diaphoresis: salicylates, organophosphates, sympathomimetics, cholinergics, ethanol or sedative-hypnotic<br />

withdrawal<br />

552


TOXICOLOGIC DISORDERS<br />

e. Jaundice: acetaminophen, mushroom poisoning, arsine gas<br />

f. Alopecia: arsenic, thallium, cytotoxic agents<br />

g. Flushing: disulfiram, niacin, ethanol, monoamine-oxidase inhibitors, metronidazole, anticholinergics,<br />

scombroid fish poisoning<br />

h. Cyanosis (noncardiopulmonary, by way of methemoglobinemia): nitrates/nitrites, 11 -caine 11 anesthetics,<br />

aniline dyes, chlorates, dapsone, sulfonamides<br />

11. Alterations in motor tone<br />

a. Increased motor tone: lithium, levodopa, strychnine, phencyclidine (PCP), ketamine, extrapyramidals, black<br />

widow spider bite, serotonin syndrome, neuroleptic malignant syndrome<br />

b. Fasciculations: cholinergics, amphetamines, heavy metals, paralytic shellfish poisoning, scorpion, or<br />

pufferfish<br />

c. Flaccid: sedative-hypnotics, opioids, fasciculants listed above<br />

12. Bowel sounds<br />

a. Increased bowel sounds: cathartics (eg, sorbitol), cholinergics, heavy metals, mushrooms, withdrawal<br />

syndromes<br />

b. Decreased bowel sounds: opioids, anticholinergics, phenothiazines<br />

E. Diagnostic evaluation<br />

1. ECG -;, dysrhythmias and conduction blocks<br />

2. CBC - as indicated for specific poisons (eg, iron)<br />

3. Electrolytes-;, for calculating osmolar and anion gaps<br />

4. Glucose - hyper/hypoglycemia and osmolar gap determinations<br />

5. Serum osmolality - osmolar gap determination<br />

6. BUN/creatinine - osmolar gap determination (BUN), renal function<br />

7. Creatine kinase-;, as clinically indicated to evaluate for rhabdomyolysis<br />

8. Arterial blood gases - acid-base evaluation<br />

9. Specific drug levels: salicylate, acetaminophen, and ethanol, plus any other toxic agent that is suggested by the<br />

history and physical examination (findings uncovered in the history and physical examination frequently point<br />

to a specific toxic agent/class)<br />

10. Abdominal radiographs-;, look for radiopaque tablets ( 11 CHIPES 11 ); some poisons that can be radiopaque:<br />

.Chloral hydrate<br />

Heavy metals<br />

Iron<br />

fhenothiazines<br />

.[nteric-coated tablets<br />

.S.olvents<br />

11 . Urinalysis<br />

a. Urine pH as indicated for management of specific poisonings (eg, salicylates)<br />

b. Ketones-;, isopropanol, alcoholic/starvation ketoacidosis, salicylates<br />

c. Blood without RBCs-;, rhabdomyolysis (eg, cocaine)<br />

d. Crystal fluorescence-;, ethylene glycol (not reliable in practice)<br />

F. General treatment measures<br />

1. GI decontamination<br />

a. Activated charcoal<br />

(1) Action: binds (adsorbs) the toxic agent and prevents absorption<br />

(2) Dose: I g/kg or 10 g per gram of ingested toxin, whichever is greater<br />

(3) Not useful for the following ingestions (liquids and metals):<br />

(a) Lithium<br />

(b) Alkalis/acids (absolutely contraindicated)<br />

(c) Heavy metals<br />

(d) Iron<br />

(e) Alcohols<br />

(f)<br />

Hydrocarbons and solvents<br />

553


TOXICOLOGIC DISORDERS<br />

(4) Repeated doses of activated charcoal (0.5 g/kg every 4-6 hours) are used for poisoning due to<br />

salicylates, theophylline, barbiturates, valproic acid, carbamazepine, dapsone, and sustained-release<br />

preparations.<br />

(a) Should not be given to patients suspected of having a bowel obstruction or ileus (confirm<br />

presence of bowel sounds before administration)<br />

(b) Should not be accompanied by repeated doses of a cathartic (eg, sorbitol), which can cause<br />

severe fluid and electrolyte disturbances<br />

b. Whole-bowel irrigation (WBI)<br />

(1) Uses polyethylene glycol electrolyte lavage solution (PEG-ELS) to wash poisons from GI tract before<br />

they are absorbed<br />

(2) May be useful after ingestions of:<br />

(a) Lithium<br />

(b) Iron<br />

(c) Heavy metals<br />

(d) Sustained-release, highly toxic drugs (eg, verapamil, diltiazem, theophylline, buproprion)<br />

(e)<br />

Body packers<br />

(3) Must administer via nasogastric tube (volume required is too large for patient to drink independently):<br />

adults= 1,500-2,000 ml/hr; children= 25-40 ml/kg/hr<br />

(4) Activated charcoal can be used before and during WBI therapy.<br />

(5) End point is retrieval of all pills or packets from body packers/stuffers. Other commonly cited end<br />

points include presence of clear rectal effluent; however, WBI should continue despite clear rectal<br />

effluent if pills or packets are still present in GI tract.<br />

(6) Contraindications<br />

(a)<br />

Unprotected airway<br />

(b) Intestinal obstruction or ileus<br />

(c) Uncertainty regarding position of nasogastric tube<br />

(d) Intractable vomiting<br />

c. GI decontamination measures no longer recommended<br />

(1) Neutralization of acids or bases: produces exothermic reaction that can cause further tissue damage<br />

(2) Cathartics (to promote rapid transit of toxin through the GI tract): no proven benefit and can cause<br />

fluid and electrolyte imbalances<br />

2. Removal of unabsorbed poison assumes a secondary role and, in many cases, may not be performed.<br />

a. Induction of emesis<br />

(1) Not indicated for prehospital or in-hospital therapy<br />

(2) American Academy of Pediatrics no longer recommends ipecac be kept in the home.<br />

b. Gastric lavage<br />

(1) Indications<br />

(a)<br />

Known or suspected life-threatening ingestion and<br />

(b) Time of ingestion


TOXICOLOGIC DISORDERS<br />

3. Elimination of absorbed toxin<br />

a. Antidotes<br />

Table 36: Antidotes for Specific Poisons<br />

Antidotes<br />

N-Acetylcysteine<br />

Antivenin<br />

CroFab®<br />

Anascorp®<br />

Atropine sulfate<br />

Botulism antitoxin<br />

Calcium gluconate<br />

Calcium disodium EDTA<br />

Cyproheptadine<br />

Deferoxamine<br />

Dextrose<br />

Digoxin Fab<br />

Dimercaprol (BAL)<br />

Ethanol<br />

Flumazenil<br />

Folic acid<br />

Fomepizole (4-methylpyrazole)<br />

Glucagon<br />

Glucarpidase<br />

Hydroxocobalamin<br />

Insulin (high dose)<br />

ldarucizumab<br />

L-carnitine<br />

Leucovorin (folinic acid)<br />

Methylene blue<br />

Naloxone<br />

Nitrites (amyl nitrite, sodium<br />

nitrite)<br />

Octreotide<br />

Agents<br />

Acetaminophen<br />

Rattlesnakes, copperhead (rarely needed), water moccasin, Eastern<br />

coral snake, coral snake<br />

Black widow spider (Latrodectus) (antivenin rarely needed)<br />

Scorpion (Centuroides exilicauda)<br />

Organophosphates/carbamates<br />

C!ostridium botulinum<br />

Hydrofluoric acid (topical, intradermal, intravenous, intra-arterial),<br />

calcium channel blockers (intravenous)<br />

Lead<br />

Serotonin syndrome<br />

Iron<br />

Insulin/oral hypoglycemic agents<br />

Digoxin<br />

Cardiac glycoside plants (foxglove, oleander)<br />

Lead with encephalopathy (must be used with calcium disodium<br />

EDTA), mercuric salts and arsenic if GI tract compromised<br />

Ethylene glycol, methanol<br />

Benzodiazepines<br />

Methanol<br />

Ethylene glycol, methanol<br />

Calcium channel and ~-blockers<br />

Methotrexate<br />

Cyanide<br />

~-blockers, calcium channel blockers<br />

Dabigatran<br />

Valproic acid-induced hyperammonemia with mental status changes<br />

Methotrexate<br />

Nitrites, nitrates, aniline dyes (methemoglobinemia)<br />

Narcotics, diphenoxylate, propoxyphene<br />

Cyanide<br />

Su lfonyl ureas<br />

555


TOXICOLOGIC DISORDERS<br />

Table 36: Antidotes for Specific Poisons (continued)<br />

Antidotes<br />

Oxygen (normobaric and hyperbaric)<br />

Physostigmine<br />

Potassium iodide (Kl)<br />

Pralidoxime (2-PAM)<br />

Protamine sulfate<br />

Prussian blue<br />

Pyridoxine (vitamin B 6<br />

)<br />

Silibinin<br />

Sodium bicarbonate<br />

Sodium thiosulfate<br />

Succimer (2,3 dimercaptosuccinic<br />

acid, DMSA)<br />

Thiamine hydrochloride<br />

Vitamin K<br />

Agents<br />

Carbon monoxide<br />

Anticholinergic agents<br />

Radioactive iodine<br />

Organophosphates<br />

Heparin<br />

Thallium salts, cesium salts<br />

Ethylene glycol, INH, Gyrometra escu/enta (false morel)<br />

Amatoxin-induced mushroom poisoning (Amanita sp)<br />

Salicylates, tricyclic antidepressants<br />

Cyanide<br />

Lead, mercury, arsenic<br />

Ethylene glycol<br />

Warfarin, long-acting anticoagulant rodenticides<br />

b. Acidification of the urine is no longer recommended.<br />

c. Alkalinization therapy with IV sodium bicarbonate<br />

(1) Cyclic antidepressants and other sodium-channel blockers-;. alkalinization of the blood helps<br />

improve/prevent dysrhythmias/conduction blocks and improves hypotension<br />

(a)<br />

Use boluses of sodium bicarbonate<br />

(2) Alkalinization of the urine to a pH >7 (with a bicarbonate infusion) ionizes weak acids into ionized<br />

molecules, increasing excretion of the following toxic agents:<br />

(a) Salicylates<br />

(b) Phenobarbital<br />

(c) Chlorpropamide<br />

d. Charcoal hemoperfusion rarely used any longer<br />

e. Hemodialysis indications<br />

(1) Ethylene glycol, methanol<br />

(2) Methanol<br />

(3) Valproic acid<br />

(4) Lithium<br />

(5) Salicylates<br />

(6) Theophylline, caffeine<br />

G. Other complications<br />

1. Hepatotoxicity-;. acetaminophen, iron, carbon tetrachloride, cyclopeptide mushrooms (Amanita, Caterina,<br />

Lepiota), and pennyroyal oil<br />

2. GI hemorrhage<br />

a. ASA<br />

b. Iron<br />

c. Coumadin<br />

d. Mercuric salts<br />

e. Alkalis/acid caustics<br />

f. Long-acting anticoagulant rodenticides<br />

556


TOXICOLOGIC DISORDERS<br />

3. Renal failure - carbon tetrachloride, mercuric salts, acetaminophen, colchicine, ethylene glycol<br />

4. Dysrhythmias - cyclic antidepressants, cocaine, ~-blockers, calcium channel blockers, quinine,<br />

antidysrhythmics, chloroquine, propoxyphene, phenothiazine, theophylline, digoxin, chloral hydrate<br />

5. Hypertension - phencyclidine (PCP), monoamine-oxidase inhibitors, phenylpropanolamine, cocaine,<br />

amphetamines<br />

6. Hypotension - sustained-release calcium channel blockers<br />

7. Neurotoxicity - botulinum toxin, lithium, carbon monoxide, methanol, carbon disulfide<br />

8. Respiratory problems - nitrogen oxides, phosgene, paraquat<br />

II. SPECIFIC OVERDOSES AND POISONINGS<br />

A. Cyclic antidepressants<br />

1. Commonly prescribed agents<br />

a. lmipramine<br />

b. Desipramine<br />

C. Amitriptyline<br />

d. Nortriptyline<br />

e. Doxepin<br />

2. Pathophysiology (obtain cardiac monitoring and ECG early)<br />

a. Cardiac effects<br />

(1) Anticholinergic activity that can induce tachycardia<br />

(2) Quinidine-like activity (fast sodium channel blockade) that can induce intraventricular and AV blocks;<br />

avoid use of procainamide, quinidine, or other class IA and IC antiarrhythmics.<br />

(a) Bundle-branch and fascicular blocks are usually preceded by a widening QRS complex (wide<br />

QRS is a primary ECG finding).<br />

(b) AV blocks range from first degree to complete block.<br />

(c) Other ECG effects include tall R wave in aVR.<br />

(3) Hypotension due to peripheral a-adrenergic blockade, dysrhythmias, and norepinephrine depletion<br />

(4) Pulmonary edema<br />

b. CNS effects<br />

(1) Confusion, agitation, and hallucinations with rapid progression to coma<br />

(2) Seizures are common and usually single.<br />

c. Anticholinergic effects are common; absence does not exclude toxicity.<br />

d. Signs that indicate a serious overdose<br />

(1) Ventricular dysrhythmias<br />

(2) lntraventricular conduction defects with a QRS complex> 100 milliseconds<br />

(3) Bradycardia and AV blocks<br />

(4) Hypotension<br />

(5) Pulmonary edema<br />

(6) Seizures<br />

(7) Cardiac arrest<br />

3. Management<br />

a. Activated charcoal: use with caution because patients have the potential to rapidly decompensate.<br />

b. Consider orogastric lavage.<br />

c. IV boluses of sodium bicarbonate are the mainstay of therapy.<br />

(1) The most effective therapy for improving hypotension and abolishing dysrhythmias<br />

(2) Indications: if QRS complex :::100 milliseconds in the limb leads<br />

(3) Administration of sodium bicarbonate is usually via boluses: dosing 1-2 mEq/kg (1-2 ampules of<br />

NaHCOJ.<br />

(4) Avoid if arterial pH >7.55 or serum sodium> 150 mmol/L.<br />

557


TOXICOLOGIC DISORDERS<br />

d. Hypotension<br />

(1) Start with crystalloids and sodium bicarbonate.<br />

(2) If no or poor response, try vasopressors.<br />

e. Seizure control is important, because lactic acidosis can worsen cardiac toxicity. Therapeutic choices include:<br />

(1) Sodium bicarbonate<br />

(2) Benzodiazepines<br />

(3) Phenobarbital<br />

(4) Propofol<br />

f. Therapies to avoid<br />

(1) Class IA (eg, procainamide) and IC (eg, flecainide) antidysrhythmics - can worsen the "quinidine-like"<br />

toxicity on the myocardium<br />

(2) Flumazenil --,, risk of precipitating seizures<br />

(3) Physostigmine--,, risk of cardiac toxicity and asystole<br />

(4) Hemodialysis--,, ineffective because cyclic antidepressants are highly protein bound<br />

(5) Hemoperfusion<br />

B. Other antidepressants<br />

1. Differ from the cyclic antidepressants in that their mechanism of action (inhibition of biogenic amine reuptake<br />

in the CNS) occurs without sodium channel blockade; therefore, life-threatening toxicity is less common.<br />

2. Common agents<br />

a. SSRls: selective inhibition of serotonin reuptake<br />

(1) Fluoxetine<br />

(2) Sertraline<br />

(3) Paroxetine<br />

(4) Fluvoxamine<br />

(5) Citalopram<br />

b. Venlafaxine: serotonin-norepinephrine reuptake inhibitor<br />

c. Trazodone, nefazodone: inhibit 5-hydroxytryptamine (5-HT) reuptake and block postsynaptic 5-HT 2<br />

receptors; trazadone has peripheral alpha effects that can result in priapism when taken therapeutically or<br />

in overdose.<br />

d. Bupropion: inhibits dopamine reuptake<br />

e. Mirtazapine: nonselective reuptake of biogenic amines inhibited and blocks postsynaptic 5-HT 2<br />

and 5-HT 3<br />

receptors<br />

3. Clinical presentation<br />

a. SSRI overdose<br />

(1) Mild CNS depression, sinus tachycardia, GI symptoms (nausea, vomiting, diarrhea), and seizures (rare)<br />

(2) Serotonin syndrome<br />

b. Bupropion overdose: hallucinations, tachycardia, seizures (may be delayed up to 18 hours after overdose),<br />

status epilepticus, death<br />

c. Trazodone, nefazodone: prolonged QT interval<br />

4. Management<br />

a. Activated charcoal<br />

b. Monitor for 6 hours-longer for bupropion and extended-release preparations<br />

c. Benzodiazepines for seizures (rare)<br />

d. Serotonin syndrome<br />

(1) Critical care monitoring<br />

(2) IV crystalloid therapy<br />

(3) Active cooling therapy for hyperthermia<br />

(4) Cyproheptadine 4-8 mg orally every 8-12 hours<br />

(5) Muscle rigidity must be controlled quickly, because it contributes to the hyperthermia, rhabdomyolysis,<br />

and lactic acidosis.<br />

(6) Evaluate for underlying medical complications (including infections).<br />

(7) Watch for hyperkalemia and myoglobinuric-induced renal failure secondary to rhabdomyolysis.<br />

558


TOXICOLOGIC DISORDERS<br />

C. Lithium<br />

1. Primary therapeutic use is for bipolar disorder.<br />

2. Toxicity can develop after an acute overdose or in a patient taking the usual therapeutic dose who becomes<br />

dehydrated or develops renal insufficiency (chronic).<br />

3. Clinical presentation<br />

a. Toxicity is characterized primarily by neurologic manifestations<br />

(1) Mild: fatigue, tremor, mild confusion<br />

(2) Moderate: increased tremor, slurred speech, ataxia, increased confusion, drowsiness, hyperreflexia<br />

(3) Severe: tremor, clonus, choreoathetosis, rigidity, hyperthermia, irritability, stupor, coma, seizures<br />

(4) The normal therapeutic blood level of lithium is 0.6-1.2 mEq/L. Signs and symptoms of toxicity<br />

often do not correlate with serum levels (especially with chronic toxicity, because the lithium has<br />

shifted into the CNS [tissue] compartment); the neurologic status of the patient is the most important<br />

determinant of toxicity.<br />

b. Other<br />

(1) Nausea, vomiting, diarrhea<br />

(2) Nephrogenic diabetes insipidus (chronic)<br />

4. Diagnostic evaluation<br />

a. Lithium level<br />

b. ECG<br />

(1) ST-T wave abnormalities<br />

(2) Prolonged QT interval<br />

(3) Bradycardia<br />

5. Management<br />

a. General measures<br />

(1) Cardiac monitor<br />

(2) Whole-bowel irrigation (acute overdoses, especially with sustained-release preparations)<br />

b. Specific therapy<br />

(1) The key to the treatment of lithium overdose is awareness of its mechanism of excretion.<br />

(a) 97% is excreted unchanged in the urine.<br />

(b) 75%-80% is reabsorbed by the proximal tubule along with sodium; volume depletion enhances<br />

lithium retention.<br />

(2) Restoration of sodium and water balance with normal saline.<br />

(a) The choice of fluid and rate of administration should be based on the degree of dehydration and<br />

hypotension as wel I as the serum sodium level.<br />

(b) Normal saline should be used unless the patient is severely hypernatremic.<br />

(c) Goal: correct dehydration and maintain urinary output at 1-2 ml/kg/hr.<br />

(3) Because lithium is reabsorbed in the proximal renal tubule, diuretics that act on the ascending limb of<br />

the loop of Henle (furosemide) or distal tubule (thiazides) will not increase lithium excretion and may<br />

promote reabsorption if they lead to volume depletion.<br />

(4) Indications for hemodialysis<br />

(a) Clinical signs of moderate to severe poisoning (eg, seizure)<br />

(b) Worsening clinical condition despite falling lithium levels<br />

(c) Onset of dysrhythmias<br />

(d) Decreasing urine output or renal failure (increased BUN/creatinine)<br />

(e) Serum level ?:4.0 mEq/L at least 6 hours after an acute ingestion (controversial)<br />

(5) Watch for a rebound in serum lithium levels after hemodialysis, because tissue (CNS) levels<br />

reequilibrate with the serum. Repeat hemodialysis may be necessary in chronic toxicity.<br />

(6) Failure to recognize or provide appropriate treatment may result in permanent neurologic sequelae.<br />

D. Sedative-hypnotics<br />

1. Barbiturates<br />

a. Classification<br />

(1) Ultrashort acting<br />

(a) Methohexital<br />

(b) Thiopental<br />

559


TOXICOLOGIC DISORDERS<br />

(2) Short acting<br />

(a) Pentobarbital<br />

(b) Secobarbital<br />

(3) Intermediate acting<br />

(a) Amobarbital<br />

(b) Butabarbital<br />

(4) Long acting<br />

(a) Phenobarbital<br />

(b) Mephobarbital<br />

(c) Primidone<br />

b. Metabolism and major excretion mechanisms<br />

(1) Ultrashort-, short-, and intermediate-acting barbiturates are lipid-soluble and poorly ionized.<br />

(2) Long-acting barbiturates are not lipid-soluble, but they are ionized at an alkaline pH.<br />

(a) When ionized, drugs cannot enter neuronal cells or the resorptive cells that line the renal tubules.<br />

(b) This is why alkalinization of the urine is therapeutic in phenobarbital overdose.<br />

c. Clinical presentation<br />

(1) Coma<br />

(2) Hypothermia<br />

(3) Respiratory depression<br />

(4) Shock<br />

(5) Pulmonary edema<br />

(6) Absent corneal and deep tendon reflexes<br />

(7) Cutaneous bullae<br />

d. Management<br />

(1) General measures<br />

(a) Airway management<br />

(b) Fluid challenge if hypotension develops<br />

(c) Activated charcoal<br />

(2) Specific therapy of phenobarbital overdose<br />

2. Benzodiazepines<br />

(a) Alkalinization of the urine with sodium bicarbonate can increase the excretion of phenobarbital<br />

5- to 1 0-fold.<br />

1. Goal: urine pH 7.5-8<br />

ii. Use with caution, if at all, in patients dependent on phenobarbital (eg, those with alcohol<br />

dependence or seizure disorders). In these patients, supportive measures may be best.<br />

(b) One method of urinary alkalinization<br />

1. Begin therapy with a bolus of 1-2 mEq/kg (1-2 standard ampules; optional)<br />

ii. Follow with a continuous infusion of 150 mEq NaHCO 3<br />

(3 standard ampules) in 1,000 ml<br />

D5W and infuse at 1.5-2 times the maintenance fluid rate.<br />

iii. Adjust infusion rate as indicated to achieve goal urine pH.<br />

a. Agents include diazepam, lorazepam, midazolam, alprazolam, chlordiazepoxide, clonazepam,<br />

flurazepam, flunitrazepam ("roofies"), oxazepam, temazepam, and triazolam.<br />

b. Pathophysiology<br />

(1) Potency, duration of effects, and active metabolites vary widely.<br />

(2) Benzodiazepines enhance effects of gamma-aminobutyric acid (GABA) at central receptors.<br />

c. Clinical presentation<br />

(1) Progressive depression of level of consciousness to coma<br />

(2) Hypothermia, pulmonary aspiration, and respiratory arrest<br />

d. Management<br />

(1) General supportive measures: careful monitoring with attention to airway potency, oxygenation, and<br />

ventilation support<br />

(2) Flumazenil: a competitive antagonist for benzodiazepines at the receptor level<br />

560


TOXICOLOGIC DISORDERS<br />

(a) Generally not recommended for emergency department management and has no role in<br />

management of multiple substance or unknown overdose<br />

(b) Generally reserved for iatrogenic overdoses in non-benzodiazepine-dependent patients or<br />

pediatric exposures requiring active airway intervention (uncommon)<br />

(c) Contraindicated in benzodiazepine-dependent patients, because it can induce withdrawal (may<br />

be severe and life-threatening)<br />

(d) Contraindicated when coingestion with cyclic antidepressants is a possibility, because seizures<br />

may occur.<br />

(e) Re-sedation is common (short half-life of the drug)<br />

3. Gamma hydroxybutyrate<br />

a. Epidemiology<br />

(1) Youth rave culture<br />

(2) Drug-facilitated sexual assault<br />

(3) Bodybuilders<br />

b. Clinical presentation<br />

(1) Overdose<br />

(a) Bradycardia, respiratory depression<br />

(b) Lethargy, coma<br />

(c) Miosis, myoclonus, nystagmus, seizures<br />

(2) Withdrawal may resemble ethanol or sedative-hypnotic withdrawal (including delirium).<br />

c. Management<br />

(1) Overdose: supportive care, airway protection, intubation based on level of consciousness; typical<br />

course is abrupt awakening within a few hours.<br />

(2) Classic scenario: Comatose patient presents and is intubated; hours later awakes abruptly, pulls out<br />

endotracheal tube and flees the emergency department.<br />

(3) Withdrawal (seen after chronic use by body builders): benzodiazepines; pentobarbital for refractory cases<br />

E. Phenothiazines (eg, chlorpromazine, prochlorperazine)<br />

1 . Pathophysiology<br />

a. Erratic and unpredictable absorption after ingestion<br />

b. Half-life is 20-40 hours, and effects may persist for days.<br />

c. Metabolized in the liver<br />

2. Adverse effects seen with therapeutic doses and drug abuse<br />

a. Extrapyramidal reactions (most common)<br />

(1) Parkinsonian (rigidity, tremor)<br />

(2) Akathisia (incessant movement and restlessness)<br />

(3) Dystonic (grimacing, trismus, torticollis, dysphagia, dysarthria, oculogyric crisis); responds well to<br />

benztropine or diphenhydramine<br />

(4) Tardive dyskinesia (sucking, lip-smacking, perioral "rabbit" syndrome): appears late, after prolonged<br />

therapy.<br />

(5) Newer second-generation antipsychotics (eg, clozapine, risperidone, olanzapine) exhibit less<br />

dopamine2-receptor blockade and have a lower rate of extrapyramidal adverse effects.<br />

b. Orthostatic hypotension<br />

c. Hypo- or hyperthermia<br />

d. Neuroleptic malignant syndrome<br />

3. Acute overdose<br />

a. Clinical presentation<br />

(1) Anticholinergic<br />

(2) a-adrenergic blockade (miosis, vasodilation, hypotension, "warm shock")<br />

(3) Sodium/potassium channel blockade and cardiodepressant effects are most common with thioridazine<br />

and mesoridazine.<br />

(a) Prolonged PR, QRS, and QT intervals<br />

(b) ST and T wave changes<br />

(c) Ventricular dysrhythmias (including torsades de pointes) and AV dissociation<br />

(4) Coma, rigidity, and seizures<br />

561


TOXICOLOGIC DISORDERS<br />

F. Phenytoin<br />

b. Management (conservative)<br />

(1) Activated charcoal<br />

(2) Respiratory compromise - airway management<br />

(3) Hypotension - IV fluids<br />

(4) Seizures - diazepam, lorazepam, phenobarbital<br />

(5) Extrapyramidal reactions- benztropine or diphenhydramine<br />

(6) Ventricular dysrhythmias<br />

1 . Pathophysiology<br />

(a) NaHCO 3<br />

should be tried first (see cyclic antidepressants).<br />

(b) Magnesium is indicated for torsades de pointes.<br />

(c) Do not use procainamide, quinidine, or other class IA or IC antidysrhythmics.<br />

a. Correlation of blood levels with evolution of clinical effects<br />

(1) 10-20 mcg/ml - therapeutic level<br />

(2) 20 mcg/ml - lateral gaze nystagmus<br />

(3) 20-30 mcglml - nystagmus and cerebellar excitation<br />

(4) 30-40 mcglml - nausea, vomiting, ataxia, dysarthria<br />

(5) >40 mcg/ml - confusion and lethargy<br />

(6) 50 mcglml - choreoathetosis, opisthotonus (rarely)<br />

(7) > 100 mcglml - potentially lethal level<br />

b. Cardiac actions<br />

(1) Decreased automaticity<br />

(2) Cardiac toxicity, especially ventricular dysrhythmias<br />

(a) May occur with IV loading of phenytoin (commonly attributed to too rapid infusion of the<br />

propylene glycol diluent)<br />

(b) Fosphenytoin is a water-soluble phenytoin pro-drug for parenteral use and does not contain<br />

propylene glycol; cardiac toxicity rare with fosphenytoin but has been reported.<br />

c. Miscellaneous effects<br />

(1) Osteomalacia (chronic use)<br />

(2) Gingival hypertrophy (chronic use)<br />

(3) Megaloblastic anemia<br />

(4) Fetal hydantoin syndrome<br />

(5) Skin rashes and the anticonvulsant hypersensitivity syndrome (fever, rash, lymphadenopathy,<br />

eosinophilia), which will recur if phenobarbital or carbamazepine are substituted<br />

2. Management of acute ingestion<br />

G. Opioids<br />

a. Repeated doses of activated charcoal may be useful. Use with caution, if at all, in patients dependent<br />

on phenobarbital (eg, those with alcohol dependence or seizure disorders). In these patients, supportive<br />

measures may be best.<br />

b. Supportive care until confusion and ataxia clear.<br />

1. Classic clinical scenarios<br />

a. Opioid withdrawal<br />

(1) Clinical presentation: abdominal cramping/pain, chills, sweating, nausea, diarrhea, and arthralgias;<br />

distinctive physical findings include yawning, lacrimation, rhinorrhea, and piloerection.<br />

(2) Onset is usually within 12-24 hours of the last dose but is longer with sustained-release opioids and<br />

methadone.<br />

(3) Symptoms peak 1-2 days after onset and subside within a week.<br />

b. Opioid intoxication and overdose<br />

(1) Clinical presentation: euphoria, drowsiness, decreased respiratory rate, miosis<br />

(2) Severe overdoses can cause coma and respiratory acidosis.<br />

(3) Seizures may occur after tramadol, propoxyphene, or meperidine overdose.<br />

562


TOXICOLOGIC DISORDERS<br />

2. Specific treatment of opioid overdose__,. naloxone<br />

a. Initial dose is 0.1-2 mg in adults and children and should be titrated carefully. It may be given IV, IM,<br />

SC, endotracheally, intranasally. It is effective within minutes. Higher doses may be needed to reverse the<br />

following agents: buprenorphine, propoxyphene, pentazocine, meperidine, fentanyl, and diphenoxylate.<br />

b. It should be used to reverse respiratory depression and dosed until respiratory depression has resolved.<br />

Note: Naloxone will precipitate acute withdrawal in opioid-dependent patients, which is the major<br />

complication of its use.<br />

c. The effect of naloxone decreases 15-30 minutes after injection, but the action of the opioid may last for hours.<br />

(1) Buprenorphine, methadone, controlled-release opioids, and diphenoxylate/atropine in particular last<br />

> 12-24 hours.<br />

(2) Repeated doses and prolonged observation may be required.<br />

(3) Patients who require naloxone after buprenorphine, methadone, sustainedrelease opioid, or<br />

diphenoxylate/atropine overdose need admission.<br />

(4) Patients may need a continuous infusion of naloxone. Titrate based on respiratory depression versus<br />

signs of withdrawal.<br />

3. Acute complications of opioid abuse<br />

H. Clonidine<br />

a. Seizures (meperidine, propoxyphene, tramadol, pentazocine, dextromethorphan)<br />

b. Pulmonary edema<br />

c. Rhabdomyolysis<br />

d. Aspiration pneumonia<br />

1 . Pathophysiology<br />

a. A centrally acting a2-agonist, clonidine reduces sympathetic stimulation to the heart and peripheral blood<br />

vessels.<br />

b. The initial effect is hypertension followed by an anti hypertensive effect that occurs up to 10 hours later.<br />

c. Withdrawal is associated with severe headache, rebound hypertension, anxiety, and tremors.<br />

2. Overdose<br />

a. Clinical presentation<br />

(1) Mimics opioid overdose and includes apnea, altered level of consciousness (drowsiness__,. coma),<br />

seizures, and miosis; hypotension and bradycardia are prominent features.<br />

(2) Children are more severely affected than adults.<br />

b. Management<br />

(1) Supportive<br />

(2) Respiratory support, atropine, vasopressors, and GI decontamination<br />

(3) Naloxone has been used with variable success to reverse CNS and respiratory depression associated<br />

with clonidine poisoning. High doses may be needed (10-20 mg IV).<br />

I. Alcohols<br />

1. Methanol<br />

a. Sources and pathophysiology<br />

(1) Found in windshield washing fluids, solvents, paint thinners, and canned fuels. Suspect multiple<br />

victims if ingestion was recreational substitution for ethanol.<br />

(2) Converted by alcohol dehydrogenase to formaldehyde, which is then converted to formic acid.<br />

(a) The accumulation of formic acid correlates with the decrease in bicarbonate, the increase in<br />

anion gap, and the severity of metabolic acidosis.<br />

(b) Formic acid affects optic nerve function, producing optic papillitis and retinal edema __,. "blind<br />

drunk"<br />

b. Classic clinical scenario: A latent period of 8-30 hours is followed by the onset of abdominal pain,<br />

nausea and vomiting, blurred vision, and metabolic acidosis. The patient may have normal vision<br />

(and a normal eye examination) initially. As the formic acid accumulates, visual symptoms include<br />

photophobia, "snowstorm" vision, and blindness. The pupils dilate and react sluggishly, if at all, to light.<br />

The earliest funduscopic finding is hyperemia of the disc, followed by papilledema and optic atrophy. Late<br />

parkinsonism may result.<br />

c. Diagnostic evaluation<br />

(1) Anion gap metabolic acidosis associated with a low bicarbonate level<br />

563


TOXICOLOGIC DISORDERS<br />

(2) The osmol gap may be high, although a low or even "negative" gap does not exclude a toxic alcohol<br />

or glycol.<br />

(a) Osmolar gap: calculated osmolarity - measured osmolality<br />

(b) Calculated osmolarity<br />

(2 x Na) +<br />

BUN<br />

2.8<br />

+<br />

Glucose<br />

18<br />

+<br />

Ethanol<br />

4.6<br />

(c) Normal osmolar gap: 10-14<br />

(d) Osmotically active substances that may produce an osmol gap<br />

i. Methanol<br />

ii. Ethanol<br />

iii. lsopropyl alcohol<br />

iv. Glycerol<br />

v. Mannitol<br />

vi. Ethylene glycol<br />

(e) Osmol gaps can also be seen in lactic acidosis and alcoholic ketoacidosis, or with high serum<br />

ethanol concentrations (despite "correction" using the above formula).<br />

d. Management<br />

(1) General supportive measures, including airway management<br />

(2) Nasogastric aspiration of gastric contents if the patient is seen within the first 30-60 minutes after<br />

ingestion<br />

(3) Severe acidosis is treated with bicarbonate to reduce diffusion of formate into the CNS and protect<br />

the optic nerve.<br />

(4) Alcohol dehydrogenase inhibitor (ethanol or fomepizole) to block further metabolism of methanol.<br />

(a) Indications: methanol level >20 mg/dl (may start empirically while waiting for level)<br />

(b) Fomepizole (4-methylpyrazole) is the preferred agent.<br />

(c) Second-line agent: ethanol<br />

i. Difficult to find and to dose<br />

ii. Goal blood ethanol level: 100-150 mg/dl<br />

(5) Hemodialysis removes methanol and formic acid and is indicated for methanol level >50 mg/dl,<br />

metabolic acidosis (arterial pH :'c7.25), severe visual or CNS symptoms, or when ethanol is used<br />

for therapy.<br />

(6) Folic acid can be considered and may enhance metabolism of formic acid to nontoxic substances,<br />

which are then eliminated from the body.<br />

2. Ethylene glycol<br />

a. Source and pathophysiology<br />

(1) Found in antifreeze and brake fluids<br />

(2) Converted by alcohol dehydrogenase to glycolaldehyde, which is then metabolized to glycolic acid.<br />

(a) Glycolic acid is primarily responsible for the anion gap metabolic acidosis; it is ultimately<br />

metabolized to multiple metabolites, including oxalic acid.<br />

(b) Oxalic acid forms calcium oxalate crystals in the kidney, brain, and liver.<br />

b. Clinical presentation<br />

(1) CNS changes suggesting ethanol intoxication<br />

(2) Progressive toxicity over 9-12 hours can lead to seizures, stupor, and coma.<br />

(3) As toxicity progresses, pulmonary edema and myocardial dysfunction can develop; death may occur.<br />

(4) Late toxicity is characterized by renal failure.<br />

(5) Cranial nerve (frequently facial) dysfunction can develop (late finding).<br />

c. Diagnostic evaluation<br />

(1) Anion gap metabolic acidosis<br />

(2) Positive birefringent calcium oxalate crystals in the urine<br />

(a) Often absent initially<br />

(b) Commonly described as "envelope-shaped"<br />

564


TOXICOLOGIC DISORDERS<br />

(3) Urine that fluoresces under a Wood's lamp may be evident if the patient recently ingested a<br />

fluorescein-containing antifreeze; however, this has been shown to be very unreliable, and falsepositives<br />

and false-negatives are common.<br />

(4) Osmol gap (absence does not exclude ingestion)<br />

(5) Hypocalcemia (not always present)<br />

d. Management<br />

(1) Gastric decontamination, IV ethanol or fomepizole, and hemodialysis (same indications as for<br />

methanol toxicity)<br />

(2) Thiamine and pyridoxine can be administered to decrease production of oxalic acid.<br />

(3) Calcium administration as needed (long QT interval)<br />

3. Ethanol<br />

a. Intoxication<br />

(1) Degree of intoxication varies according to the patient's tolerance.<br />

(2) Clinical presentation<br />

(a) Mild intoxication (euphoria, expansiveness, loss of self-control)<br />

(b) Moderate intoxication<br />

i. Slurred speech, ataxia, and nystagmus<br />

ii. Altered sensory perceptions that are dull, distorted, or obviously psychotic<br />

iii. Mild tachycardia is common.<br />

(c) Severe intoxication (the four "H's")<br />

i. t!.ypotension<br />

ii. t!.ypoventilation<br />

iii. t!.ypothermia<br />

iv. t!.ypoglycemia (more common in small children)<br />

(3) Care must be taken to consider coexisting disease, hypoglycemia, trauma (especially closed head<br />

injury), or drug ingestion.<br />

(4) Management<br />

(a) Most patients with uncomplicated ethanol intoxication require only observation and basic<br />

supportive measures.<br />

(b) Gastric decontamination is indicated if polydrug ingestion is known or suspected.<br />

(c) Administer IV glucose, naloxone, and thiamine 100 mg, when necessary.<br />

(d) Hemodialysis is indicated only for life-threatening complications (refractory hypotension).<br />

b. Withdrawal<br />

(1) Clinical presentation<br />

(a) Early signs and symptoms: tremor, irritability, increased pulse, increased blood pressure<br />

(b) Delirium tremens (onset is 48-100 hours after cessation)<br />

i. Hypertension, tachycardia<br />

ii. Diaphoresis, dehydration<br />

iii. Visual hallucinations and paranoid ideation<br />

(c) Withdrawal seizures ("rum fits")<br />

i. Generally occur within 6-48 hours of last drink<br />

ii. Usually single or several brief generalized seizures with rapid recovery in between<br />

iii. lntracranial pathology must be considered in the presence of fever, prolonged postictal<br />

period, signs of trauma, or focality.<br />

(d) Wernicke-Korsakoff syndrome (thiamine deficiency)-two of the following signs:<br />

i. Dietary deficiencies<br />

ii. Cerebellar dysfunction<br />

iii. Oculomotor abnormalities<br />

iv. Altered mental state or mild memory impairment<br />

(2) Management<br />

(a) IV fluids, thiamine, and glucose<br />

(b) Replacement of potassium, magnesium, and phosphorus when indicated<br />

565


TOXICOLOGIC DISORDERS<br />

4. lsopropanol<br />

J. Cocaine<br />

(c) Benzodiazepines for control of the withdrawal state<br />

a. Pathophysiology<br />

1. Enough medication should be administered to calm or lightly sedate the patient. For<br />

example, diazepam can be started at 5-10 mg and then doubled until desired effect; very<br />

large doses may be needed.<br />

11. Phenytoin is not effective for alcohol-withdrawal seizures.<br />

iii. Physical restraints should be used to temporarily control patients only until the<br />

pharmacologic treatment becomes effective. A patient should never be allowed to thrash<br />

uncontrollably in physical restraints, unattended, for a prolonged time.<br />

iv. Avoid antipsychotics (haloperidol, ziprasidone): may lower seizure threshold.<br />

v. Sedation with propofol or barbiturates may be necessary in severe refractory cases.<br />

(1) lsopropyl (rubbing) alcohol causes more intoxication/CNS depression but generally less severe<br />

sequlae than methanol and ethylene glycol.<br />

(2) 80% is absorbed from the stomach within 30 minutes and is then metabolized (by alcohol<br />

dehydrogenase) to acetone.<br />

b. Clinical presentation<br />

(1) CNS depression<br />

(2) Hypotension (in severe cases)<br />

(3) Hemorrhagic gastritis/tracheobronchitis<br />

c. Diagnostic evaluation<br />

(1) Positive serum acetone and acetonuria<br />

(2) High osmolal gap<br />

(3) Anion gap metabolic acidosis is not a feature of isopropanol toxicity (unless hypotension with lactic<br />

acidosis is present). This finding should prompt a search for another cause (eg, sepsis, alcoholic<br />

ketoacidosis, ethylene glycol, or methanol coingestion).<br />

d. Management<br />

(1) Supportive care, including airway management<br />

(2) Nasogastric aspiration of gastric contents within initial 30-60 minutes<br />

(3) Alcohol dehydrogenase inhibitors (fomepizole, ethanol) are not indicated.<br />

(4) Search for any complication or concomitant condition (eg, trauma)<br />

(5) Indications for hemodialysis<br />

1 . Pathophysiology<br />

(a) Refractory hypotension<br />

(b) Serum levels >400-500 mg/dL<br />

a. Metabolized by the liver, plasma esterases, and spontaneous hydrolysis within a few hours, then excreted<br />

in the urine<br />

b. Causes euphoria with lack of fatigue, a sense of omnipotence, increased mental alertness, and sexual<br />

stimulation<br />

2. Clinical presentation<br />

a. CNS stimulation__,. apprehension, twitching, seizures; coma can develop later.<br />

b. Cardiac stimulation<br />

(1) Hypertension, tachycardia, ventricular dysrhythmias, cardiac conduction blocks<br />

(2) Effects on coronary arteries and increased platelet aggregation can lead to an acute Ml; this finding in<br />

a young person should raise the suspicion of cocaine use.<br />

c. Respiratory depression/failure<br />

(1) Pulmonary edema<br />

(2) Pneumothorax/mediastinal air<br />

(3) Crack lung (bronchospasm, infiltrates, eosinophilia hemorrhage)<br />

d. Hyperthermia (may occur suddenly and can be lethal)<br />

e. Rhabdomyolysis---,. hyperkalemia (can be life-threatening)<br />

f. Arterial vasospasm can lead to tissue infarction in various tissues, such as brain, lung, liver, and intestines.<br />

lntracerebral hemorrhages and aortic dissection can also occur.<br />

566


TOXICOLOGIC DISORDERS<br />

3. Cocaine-associated chest pain -s- acute Ml in 6% of patients<br />

a. Abnormal ECG is common (early repolarization, old and new nonspecific abnormalities)<br />

b. Management<br />

(1) Benzodiazepines are first-line treatment for hypertension and tachycardia.<br />

(2) Pharmacologic agents to control pulse rate and blood pressure (eg, phentolamine), along with<br />

benzodiazepines, should be started.<br />

(3) Angioplasty is preferred over systemic fibrinolysis for persistent ST-segment elevation Ml despite the<br />

above measures.<br />

4. Management<br />

(4) Patients without ischemic changes and negative cardiac markers over an appropriate observation<br />

period can usually be managed as outpatients.<br />

a. General supportive measures<br />

(1) Airway management and IV hydration to assure a urinary output of 1-2 ml/kg/hr.<br />

(2) Cardiac monitoring and repeat vital signs (including temperature) every few minutes.<br />

b. Seizures, agitation, and psychosis are best managed with benzodiazepines. Phenothiazines (eg,<br />

chlorpromazine) and haloperidol may lower the seizure threshold and prevent heat dissipation in the<br />

already hyperthermic patient; avoid when treating cocaine-induced psychosis.<br />

c. Cocaine-induced cardiac dysrhythmias should be treated with IV benzodiazepines. Sodium bicarbonate<br />

should be considered for dysrhythmias refractory to benzodiazepines (especially those with a wide QRS<br />

complex). Drugs that should be avoided include:<br />

(1) Lidocaine (may increase CNS toxicity -s- seizures)<br />

(2) ~-blockers (may precipitate cardiac ischemia or hypertension due to unopposed a-receptor stimulation)<br />

(3) a-blockers (eg, phentolamine) followed by use of a short-acting ~-blocker (or combination agents,<br />

such as labetalol) have been used in these patients to alleviate concerns for unopposed a-receptor<br />

stimulation and subsequent hypertensive crisis.<br />

d. Hyperthermia should be treated in the usual fashion. Aggressive measures may be needed. See page 649.<br />

e. Dialysis has no role in the management of cocaine poisoning.<br />

f. Body stuffers and body packers-specific management:<br />

(1) Activated charcoal for body stuffers<br />

(2) Whole-bowel irrigation for body packers<br />

(3) Observation<br />

(a) Body stuffers (nonprofessionally wrapped packets, eg, impromptu swallowing of baggies to avoid<br />

arrest) -s- until asymptomatic (6-12 hours)<br />

(b) Body packers (professionals) -s- until all packets have passed (may need to be confirmed with<br />

radiocontrast studies)<br />

K. Amphetamines/amphetamine-like drugs<br />

1. Commonly abused drugs<br />

a. Ephedrine (ma huang, herbal ecstasy)<br />

b. Methamphetamine (crank, ice)-can also be smoked<br />

c. MOMA (Ecstasy), MDEA (Eve), etc<br />

d. Cathinone (Khat)<br />

e. Synthetic cathinones ("bath salts")<br />

f. ADHD medications (eg, amphetamine, dextroamphetamine, lisdesamphetamine)<br />

2. Pathophysiology<br />

a. Rapidly absorbed from GI tract<br />

b. Eliminated via liver (30%-45%) and kidneys (60%-70%)<br />

3. Clinical presentation<br />

a. Prescription doses<br />

(1) Central effects (anorexia, hyperactivity, restlessness, and sleeplessness)<br />

(2) Peripheral effects (tachycardia, vasoconstriction, bronchodilation, and pupillary dilation)<br />

(3) Phenylpropanolamine was removed from OTC diet and decongestant preparations because of<br />

concerns with hypertension, seizures, intracerebral hemorrhage, and dysrhythmias.<br />

567


TOXICOLOGIC DISORDERS<br />

b. Withdrawal signs and symptoms<br />

(1) Headache<br />

(2) Increased appetite<br />

(3) Abdominal cramps and diarrhea<br />

c. Toxic signs and symptoms can be clinically indistinguishable from those of cocaine toxicity.<br />

4. Management<br />

(1) Cardiovascular: hypertension, tachycardia, dysrhythmias, Ml, intracerebral hemorrhage<br />

(2) Neurologic: diaphoresis, mydriasis, piloerection, extreme restlessness, psychotic behavior, seizures, coma<br />

(3) GI: nausea, vomiting, diarrhea<br />

(4) Other toxic signs<br />

(a) Hyperthermia<br />

(b) Rhabdomyolysis<br />

(c) Hallucinations (occur with designer amphetamines, eg, MOMA, MDEA)<br />

(d) Hyponatremia (due to syndrome of inappropriate antidiuretic hormone secretion with MOMA)<br />

a. General measures<br />

(1) Quiet area to reduce hyperactivity<br />

(2) Cardiac monitor<br />

(3) Aggressive cooling measures for hyperthermia<br />

(4) Activated charcoal for ingestions<br />

(5) Correction of fluid and electrolyte disturbances<br />

b. Specific therapy<br />

L. Hallucinogens<br />

(1) Benzodiazepines for seizures and agitated psychosis<br />

(2) Nitroprusside, phentolamine, or labetalol for hypertension unresponsive to benzodiazepine sedation<br />

(avoid ~-blockers)<br />

1. Phencyclidine (PCP)<br />

a. Clinical presentation<br />

(1) Most common findings are abnormal vital signs (hypertension, tachycardia, hyperthermia) and<br />

nystagmus (horizontal, vertical, or rotary) in a patient with bizarre behavior that can fluctuate between<br />

severe agitation and catatonic-like behavior.<br />

(2) Pupillary size is variable, but miosis is common.<br />

(3) Severe hyperthermia, seizures, coma, and rhabdomyolysis can occur with large ingestions.<br />

b. Management<br />

(1) Benzodiazepines for agitation and seizures<br />

(2) Aggressive cooling measures for severe hyperthermia<br />

(3) Hydrate with IV crystalloids to maintain a urinary output of 1-2 ml/kg/hr<br />

(4) Activated charcoal should be given; repeated doses may be efficacious because of the gastroenteric<br />

circulation of PCP.<br />

(5) Acidification of the urine is no longer recommended.<br />

2. Other hallucinogens<br />

a. LSD (psychedelic flashbacks are a prominent feature, especially in chronic users)<br />

b. Mescaline<br />

c. Psilocybin (see mushroom poisoning, page 578)<br />

d. Cannabinoids<br />

e. Jimsonweed (Datura) (anticholinergic toxicity)<br />

f. Morning glory plant (lpomoea) (anticholinergic toxicity)<br />

g. Nutmeg (large doses)<br />

h. Ketamine (vitamin K)<br />

i. Bufotenine (toad licking; 5-Me-0-DMT may be responsible chemical instead of bufotenine)<br />

j. Yohimbine<br />

k. Designer amphetamines (MOMA, MDEA, DOM/STP, "bath salts")<br />

568


TOXICOLOGIC DISORDERS<br />

3. General treatment of hallucinogens<br />

a. A quiet, semidarkened room<br />

b. Benzodiazepines for sedation (and seizures)<br />

c. Temperature monitoring and aggressive cooling for severe hyperthermia.<br />

M. Salicylates (includes ASA, oil of wintergreen, bismuth subsalicylate, liniments)<br />

1. Acid-base disorders occur in three phases<br />

a. First phase: primary respiratory alkalosis<br />

(1) Salicylate stimulates the respiratory center in cerebral medulla_,, respiratory alkalosis<br />

(2) May not be seen in children<br />

b. Second phase: primary metabolic acidosis, blood pH remains normal<br />

(1) Primary respiratory alkalosis also serves as compensation for this developing primary metabolic acidosis.<br />

(2) Salicylate uncouples oxidative phosphorylation, leading to anion gap metabolic acidosis.<br />

c. Third phase: overwhelming metabolic acidosis<br />

(1) Patient can no longer compensate for metabolic acidosis.<br />

(2) Acidemia and shock ensue.<br />

2. Acute versus chronic poisoning<br />

a. Acute ingestion<br />

(1) Mi Id toxicity (triad)<br />

(a) Ototoxicity (tinnitus, vertigo, and hearing distortion)<br />

(b) Tachypnea and hyperpnea<br />

(c) Nausea and vomiting, abdominal pain<br />

(2) Severe toxicity<br />

(a)<br />

CNS stimulation occurs initially, followed by CNS depression; can lead to stupor, convulsions, coma.<br />

(b) Definitive signs of severe toxicity<br />

i. Cardiac dysrhythmias<br />

ii. Noncardiogenic pulmonary edema<br />

iii. Acute renal failure<br />

iv. Hemorrhage<br />

v. Hypo- or hyperglycemia<br />

vi. Ketonemia, ketonuria<br />

(c) Other associated signs and symptoms<br />

i. Hyperthermia<br />

ii. Diaphoresis<br />

iii. Dehydration (especially in children)<br />

b. Chronic ingestion (most commonly seen in the elderly)<br />

(1) Suspect in any patient with unexplained CNS dysfunction, especially in the presence of a mixed acidbase<br />

disturbance<br />

(a) Change in mentation is the usual presenting complaint (confusion, disorientation, lethargy, or<br />

hal I ucinations).<br />

(b) Respiratory alkalosis with anion gap metabolic acidosis and a normal or increased pH is the most<br />

frequent presenting acid-base abnormality.<br />

(2) Noncardiogenic pulmonary edema<br />

3. Diagnostic evaluation<br />

a. Serum salicylate: a serum ASA level should be drawn at presentation and repeated every 4 hours until the<br />

serum level is clearly trending down and below 20 mg/dl. The Done nomogram has no use in assessing<br />

salicylate toxicity.<br />

b. Blood gases (venous or arterial) determine the type and degree of acid-base imbalance.<br />

c. Chemistries<br />

(1) Electrolytes_,, exclude 1 K+, 1 Na+, 1 HC0 3<br />

-<br />

(2) Glucose_,, exclude hypoglycemia, hyperglycemia; in salicylate-poisoned patients, CSF may be<br />

hypoglycemic despite a normal serum glucose.<br />

(3) BUN/creatinine _,, exclude renal failure<br />

569


TOXICOLOGIC DISORDERS<br />

570<br />

d. Urinary ferric chloride testing is littered with false-positives and false-negatives and cannot be reliably used<br />

to exclude salicylate poisoning.<br />

4. Management<br />

a. GI contamination: consider activated charcoal. Multiple doses of charcoal given every 2-4 hours can be<br />

given. Gastric lavage may be considered.<br />

b. Urine alkalinization<br />

(1) Renal excretion of salicylate (an acid) is enhanced by ionization; urinary excretion of salicylates<br />

increases 10- to 20-fold when urine pH is increased from 5 to 8.<br />

(2) Urinary alkalinization (see above)<br />

(3) Potassium replacement is necessary, because hypokalemia prevents urinary alkalinization due to<br />

exchange of H+ for K+ in the renal tubules.<br />

(4) Indications<br />

(a) Proven salicylate toxicity with salicylate >20 mg/dL<br />

(b) Caution: symptoms may be subtle.<br />

(c) Suspected toxicity based on history and/or physical examination (initiate before confirmatory<br />

laboratory results)<br />

(5) Contraindications<br />

(a) Urinary alkalinization with acetazolamide -- worsening of systemic acidosis -<br />

t ASA in CNS<br />

c. Hemodialysis is the most effective means of lowering the serum salicylate level and is indicated when one<br />

or more of the following conditions are present:<br />

(1) Serum ASA level >90-100 mg/dL in any patient regardless of symptoms. Some recommend<br />

hemodialysis in chronic salicylate poisoning when levels exceed 40-60 mg/dL.<br />

(2) Neurologic signs and symptoms (confusion, delirium, psychosis, stupor, coma, or seizures)<br />

(3) Renal or hepatic failure<br />

(4) Pulmonary edema<br />

(5) Severe cardiac toxicity<br />

(6) Severe acid-base imbalance<br />

(7) Rising serum ASA levels despite urinary alkalinization<br />

5. Complications of salicylate toxicity<br />

a. Gastric dilation and ileus<br />

b. GI hemorrhage<br />

c. Platelet dysfunction<br />

N. Acetaminophen<br />

1. Phases of acetaminophen poisoning<br />

a. Phase 1-30 minutes to 24 hours after ingestion: anorexia, nausea, and vomiting; pallor and diaphoresis<br />

may also be present<br />

b. Phase 11-24-72 hours after ingestion: increased hepatic enzymes, INR, and bilirubin; right upper quadrant<br />

pain may be present but generally asymptomatic<br />

c. Phase 111-72-96 hours after ingestion: hepatic encephalopathy, coagulation defects, jaundice and renal<br />

failure; myocardial dysfunction may be present; anuria and coma are ominous signs<br />

d. Phase IV-4 days to 2 weeks after ingestion: if the pathologic effects of Phase Ill are reversible, hepatic<br />

dysfunction will completely resolve.<br />

2. Evaluation of toxicity<br />

a. Toxicity may occur with ingestions >200 mg/kg.<br />

b. The Ru mack Matthew nomogram is used to determine the need for treatment after a single, acute ingestion.<br />

3. Management<br />

(1) The first serum acetaminophen level should be drawn no earlier than 4 hours after ingestion. Only one<br />

level is required to determine the need for treatment.<br />

(2) If the measured concentration fal Is above the lower line on the nomogram, treatment with<br />

N-acetylcysteine (NAC) should be started.<br />

(3) If the level falls below the lower line, no treatment is needed.<br />

(4) IV and oral NAC are equally effective.<br />

a. Activated charcoal should be considered for patients presenting within 1 hour of a potentially toxic<br />

ingestion (200 mg/kg). Simultaneous activated charcoal and NAC is unlikely to cause consequential<br />

decrease in NAC availability (as was previously thought).


TOXICOLOGIC DISORDERS<br />

0. Iron<br />

b. N-acetylcysteine therapy is most efficacious when started within 8 hours of ingestion; however, therapy<br />

begun after 8 hours retains a high margin of efficacy and should therefore still be given to late-presenting<br />

patients.<br />

(1) Mechanism of action<br />

(a) Acetaminophen is converted to an intermediate reactive metabolite by the cytochrome p-450<br />

system. This metabolite (NAPQI) is normally detoxified by glutathione in the liver.<br />

(b) Excess acetaminophen depletes glutathione stores and the toxic metabolite damages hepatic<br />

cells, causing centrilobular hepatic necrosis.<br />

(c) The exact mechanism by which NAC prevents hepatotoxicity is uncertain and likely<br />

multifactorial. Possible mechanisms include the following:<br />

i. May act by increasing synthesis of glutathione<br />

ii. Functions as a glutathione substitute, combining directly with the toxic metabolite<br />

iii. Enhances the sulfonation pathway of acetaminophen metabolism (nontoxic)<br />

iv. Serves as a free-radical scavenger<br />

v. Converts NAPQI back to acetaminophen<br />

(2) Dosage schedule<br />

(a) Oral (72-hour) regimen<br />

i. Adverse effects: diarrhea, rash<br />

(b) 21-hour IV NAC: sick patients, usually those who present late after an acetaminophen overdose,<br />

may require extension of the protocol beyond 21 hours.<br />

i. Adverse effects: anaphylactoid reaction (ranging from rash to respiratory failure)<br />

ii. If an anaphylactoid reaction occurs during the initial IV load, stop the infusion and<br />

administer diphenhydramine; then reassess the need for NAC therapy and, if indicated,<br />

resume at a slower rate. This will resolve most anaphylactoid reactions; however, should<br />

it recur, the patient can be switched to the oral regimen. Anaphylactoid reactions are not<br />

reported with the oral regimen.<br />

c. NAC is also indicated for late-presenting patients who have already developed hepatotoxicity. The<br />

mechanisms by which NAC may ameliorate hepatotoxicity in these patients include:<br />

(1) Free-radical scavenger<br />

(2) Improved hepatic oxygen delivery and consumption<br />

(3) NAC therapy should continue in all hepatotoxic patients until recovery (patient is awake, serum<br />

acetaminophen undetectable, AST or ALT falling, and INR


TOXICOLOGIC DISORDERS<br />

(2) A potentially toxic ingestion of elemental iron is 20 mg/kg. More than 5 tablets of any iron preparation<br />

may be a toxic ingestion for a child.<br />

2. Clinical stages of iron poisoning (in severe poisoning, stages may overlap)<br />

a. Stage I: 30 minutes to 6 hours after ingestion: emesis, diarrhea, abdominal pain, hematemesis, and<br />

hematochezia; conversely, the absence of GI symptoms for 6 hours after ingestion excludes severe<br />

toxicity.<br />

b. Stage II: 4-12 hours after ingestion: GI symptoms improve, although patient status is not normal;<br />

subclinical hypoperfusion and metabolic acidosis manifest. Assess carefully for signs of early<br />

hypoperfusion and monitor arterial blood gases.<br />

c. Stage Ill: 6-72 hours after ingestion: coma, metabolic acidosis, coagulation problems, shock, and seizures<br />

d. Stage IV: 12-96 hours after ingestion: hepatic failure with jaundice, hypoglycemia, and coagulopathy<br />

e. Stage V: 2-4 weeks after ingestion: vomiting, abdominal pain and pyloric scarring, gastric outlet and<br />

small-bowel obstruction<br />

3. Diagnostic evaluation<br />

a. Serum iron concentration<br />

(1) A level >350 mcg/dl 3-5 hours after ingestion may indicate the need for chelation therapy.<br />

(2) Repeat levels should be obtained to be sure they are not rising.<br />

(3) Total iron binding capacity is not reliable in this setting, as was previously thought.<br />

b. Other studies<br />

(1) CBC, coagulation profile, glucose, electrolytes, and BUN<br />

(2) Blood gases are indicated if the patient is symptomatic.<br />

(3) Blood glucose and a WBC count, formerly thought to be predictive of serum iron levels >300 mcg/dl,<br />

are now known to be nonspecific.<br />

c. An abdominal radiograph can identify undissolved tablets but is not helpful if negative.<br />

4. Management<br />

a. Decontamination<br />

(1) Gastric emptying<br />

(2) Whole-bowel irrigation is indicated for a large ingestion, especially if radiopaque tablets are seen on<br />

abdominal radiograph.<br />

b. IV fluids are indicated if the patient is hypotensive or has severe gastroenteritis.<br />

c. Deferoxamine therapy<br />

P. Hydrocarbons<br />

1. Definitions<br />

(1) Indications<br />

(a) Serum iron concentration >350 mcg/dl and the patient has symptoms (including protracted<br />

vomiting)<br />

(b) Shock<br />

(c) Coma or seizures<br />

(d) Acidosis<br />

(2) Administration: 15 mg/kg/hr continuous IV infusion to a maximum of 6 g; hypotension is the main<br />

adverse effect.<br />

(3) The iron-deferoxamine complex is excreted in the urine and turns it an orange-red color (classically<br />

referred to as "vin rose").<br />

(4) The deferoxamine challenge (IM deferoxamine) is no longer used diagnostically or therapeutically.<br />

a. Hydrocarbons are a broad group of organic compounds that contain carbon and hydrogen in various<br />

configurations. The halogenated hydrocarbons also contain halogens such as chlorine or fluorine.<br />

b. Petroleum distillates are hydrocarbons, but not all hydrocarbons are petroleum distillates. Petroleum<br />

distillates are the breakdown products remaining after processing crude oil. Hydrocarbons can also be<br />

derived from coal or plant sources.<br />

2. The aspirational hazard of a specific hydrocarbon depends on whether or not it has two physical properties:<br />

low viscosity and high volatility. Viscosity is by far the most important factor.<br />

3. Clinical presentation of hydrocarbon ingestion<br />

a. Signs and symptoms of aspiration include coughing, choking, and gagging. Cyanosis may be evident within<br />

minutes.<br />

572


TOXICOLOGIC DISORDERS<br />

b. Other signs<br />

(1) CNS (lethargy, seizures)_,. camphor, lindane, eucalyptus oil<br />

(2) Respiratory (wheezing, rales, rhonchi, decreased breath sounds, pulmonary edema); radiographic<br />

findings do not always correlate with physical signs.<br />

(3) Miscellaneous (tachycardia, fever, belching, vomiting, diarrhea)<br />

4. Management<br />

a. Asymptomatic patient<br />

(1) Indications for gastric aspiration (nasogastric tube)<br />

(a) Presence of a toxic, or "CHAMP," substance within the ingested agent:<br />

.Camphor<br />

.!::!.alogenated hydrocarbons<br />

Aromatics<br />

Metals<br />

festicides<br />

(b) Ingestion of a massive amount of a low-viscosity agent<br />

(2) Chest radiograph is controversial and generally not useful in asymptomatic, well-appearing patients.<br />

(3) Disposition<br />

(a) Children: observe and reassess for 6 hours in the emergency department.<br />

(b) Adults (responsible): discharge with specific instructions to return if symptoms develop.<br />

b. Symptomatic patient<br />

(1) Chest radiograph<br />

(2) Supportive care and airway management<br />

(3) Specific therapy<br />

(a) Bronchospasm---,, nebulized ~-agonists<br />

(b) Pulmonary edema---,, continuous positive-airway pressure/positive end-expiratory pressure<br />

(4) Precautions<br />

5. Aromatics (solvents)<br />

(a) Steroids are not useful.<br />

(b) Antibiotics should be used only for definite infection.<br />

a. Most exposures occur through inhalation, because aromatics evaporate readily at room temperature.<br />

b. Clinical presentation<br />

(1) Benzene<br />

(a) Acute exposure causes CNS depression.<br />

(b) Chronic exposure causes bone marrow aplasia, leukemia.<br />

(2) Toluene, xylene<br />

(a) Respiratory tract irritation<br />

(b) A state of inebriation that can progress to CNS depression<br />

(c) Renal tubular acidosis (toluene)<br />

c. Management is symptomatic and supportive. The use of epinephrine for bronchospasm should be avoided,<br />

because aromatics sensitize the myocardium.<br />

6. Halogenated hydrocarbons (toxicity is by inhalation, and treatment is supportive and symptomatic) sensitize<br />

the myocardium, thus predisposing the patient to dysrhythmias. Signs and symptoms that differentiate specific<br />

agents are listed below.<br />

a. Carbon tetrachloride (tetrachloromethane)<br />

(1) Causes marked hepatic and renal toxicity<br />

(2) Clinical presentation<br />

(a) Immediate effects_,. nausea, vomiting, diarrhea, and abdominal pain<br />

(b) Exposure to high concentrations_,. dizziness, confusion, coma, respiratory depression, and<br />

hypotension<br />

(c) Prolonged exposure _,. polyneuritis, visual disturbances, and anemia<br />

(3) In addition to supportive and symptomatic treatment, NAC and hyperbaric oxygen therapy are also<br />

used for serious poisoning.<br />

573


TOXICOLOGIC DISORDERS<br />

b. Methylene chloride<br />

(1) Found in paint remover, degreasing solvents, aerosol solvents/propellants, and refrigerants.<br />

(2) On inhalation, pulmonary and CNS signs and symptoms dominate the clinical picture.<br />

(3) Methylene chloride is partially metabolized to carbon monoxide, so carboxyhemoglobin levels should<br />

be monitored.<br />

(a)<br />

c. Essential oils<br />

Patients may develop toxic carboxyhemoglobin levels after inhalation or ingestion of methylene<br />

chloride.<br />

(b) Peak levels may not be seen for ~8 hours.<br />

(1) Found in popular home remedies, liniments, and "aromatherapy." All have CNS effects and cause oral<br />

numbness/irritation and GI distress when ingested.<br />

(2) Specific toxicities<br />

Q. Caustic ingestions<br />

(a) Camphor -<br />

(b) Nutmeg -<br />

1. Pathophysiology<br />

(c) Eucalyptus oil -<br />

(d) Pennyroyal oil -<br />

rapid onset of seizures<br />

hallucinations<br />

seizures, coma<br />

acetaminophen-like hepatotoxicity<br />

a. Alkali ingestion (eg, lye or sodium hydroxide)<br />

(1) The esophagus is injured more often than the stomach.<br />

(2) Injury results from liquefaction necrosis.<br />

b. Acid ingestion<br />

(1) The stomach is injured more often than the esophagus.<br />

(2) Injury results from coagulation necrosis.<br />

c. Degree of injury depends on:<br />

(1) Type of ingestion (alkali or acid)<br />

(2) Volume and concentration of ingested caustic<br />

(3) Presence or absence of food in the stomach<br />

(4) Tone of the pyloric sphincter<br />

2. Complications<br />

a. Immediate<br />

(1) Result from caustic injury to the larynx, epiglottis, and occasionally the vocal cords. Immediate airway<br />

management may be needed.<br />

(2) Esophageal perforation is the most frequent complication in the first few days after ingestion. An alkali<br />

burn is more likely to cause esophageal perforation than an acid burn.<br />

(3) Gastric perforation and death can occur within hours of a large acid ingestion; metabolic acidosis,<br />

disseminated intravascular coagulation, and hemolysis can also occur with acid ingestion.<br />

b. Delayed<br />

(1) Esophageal strictures (alkali ingestion)<br />

(2) Pyloric strictures (acid ingestion)<br />

3. Clinical presentation<br />

a. identify and treat life-threatening problems (airway obstruction and hemorrhage). Edema of the larynx,<br />

epiglottis, or vocal cords may necessitate cricothyrotomy. However, severe pain is the usual clinical<br />

presentation (not respiratory distress or shock).<br />

b. Assessment of GI injury<br />

(1) History of ingestion and any associated symptoms (dysphagia, pain in the mouth, chest, or abdomen)<br />

(2) Physical examination<br />

(a)<br />

Check the mouth for oral burns (not sensitive).<br />

(b) Check for signs and symptoms of overt or impending esophageal perforation.<br />

1. Pleuritic pain located in the chest, epigastrium or back is the single most reliable symptom of<br />

an esophageal injury and may be exacerbated by swallowing or neck flexion.<br />

574


TOXICOLOGIC DISORDERS<br />

11. Signs of mediastinal air<br />

• A nasal quality to the voice<br />

• A systolic, crunching heart sound (Hamman crunch)<br />

• Subcutaneous emphysema (although classic, may not be present)<br />

4. Diagnostic evaluation: indications for endoscopy<br />

a. Any adult patient with a history of alkali or acid ingestion<br />

b. Any pediatric patient with:<br />

(1) An acid ingestion<br />

(2) An alkali ingestion associated with stridor or two or more of the following:<br />

(a) Second- or third-degree oral burns<br />

(b) Drooling<br />

(c) Vomiting<br />

(3) Check for evidence of gastric perforation, which can occur within hours of an acid ingestion or<br />

massive alkali ingestion<br />

5. Management<br />

(a) Abdominal tenderness, rigidity<br />

(b) Decreased bowel sounds<br />

(c) Free air on abdominal radiograph (upright or lateral decubitus)<br />

a. Ensure airway is still patent, and treat hypovolemic shock if present.<br />

b. Treat pain.<br />

c. Use of diluents<br />

(1) The primary indication for the use of diluent (milk or water) is ingestion of solid alkali material (drain<br />

openers, automatic dishwashing detergents). Diluting a solid alkali may reduce the degree of tissue<br />

injury; in addition, solid alkali material that is adhering to the oropharynx and esophagus needs to be<br />

moved to the stomach, where it can be neutralized.<br />

(2) Diluents are of no value in the treatment of liquid alkali ingestion, because the tissue injury is immediate.<br />

(3) Diluents are of questionable value in the treatment of acid ingestions; although they have been<br />

recommended, no studies have demonstrated their benefit.<br />

(4) Contraindications<br />

(a) Liquid alkali ingestion<br />

(b) Vomiting<br />

(c) Signs of esophageal or gastric perforation<br />

(d) Shock<br />

(e)<br />

Upper airway obstruction<br />

(5) Neutralizers (eg, vinegar, sodium bicarbonate) should never be given, because the resulting<br />

exothermic reaction may worsen tissue damage.<br />

d. The usual postingestion removal measures are contraindicated in the treatment of caustic ingestions.<br />

(1) Lavage is contraindicated because of the risk of reexposure of the esophagus and possible perforation<br />

of the stomach. In addition, aspiration is a hazard.<br />

(2) Charcoal is contraindicated.<br />

(a) Alkali and acid are poorly absorbed by charcoal.<br />

(b) Charcoal does not prevent tissue injury, because the action of caustics is too rapid.<br />

(c) Charcoal interferes with endoscopic visualization of tissue injury.<br />

e. Steroid use is controversial and not currently recommended by major toxicology texts.<br />

f. Antibiotics should be used only for established infection or for perforation of the upper GI tract.<br />

g. Immediate surgical consult should be obtained if GI tract perforation is known/suspected or endoscopy<br />

reveals nonviable gastric tissue.<br />

6. Button battery ingestion<br />

a. If radiography demonstrates lodgement in the esophagus, immediate endoscopic removal is indicated.<br />

This is becoming more common with the shift to larger batteries (coin-sized or >20 mm in devices)<br />

b. If radiography demonstrates the battery has entered the stomach:<br />

(1) The patient can be followed as an outpatient to assure that the battery has passed within 4-7 days.<br />

575


TOXICOLOGIC DISORDERS<br />

(2) Endoscopic removal is indicated if the battery has not passed through the stomach after 48 hours or<br />

at any point that Cl symptoms develop.<br />

(3) Once the battery passes through the pylorus, the battery can take weeks to pass. The patient should<br />

be monitored on an outpatient basis until the battery has passed.<br />

R. Organophosphates<br />

1 . Pathophysiology<br />

a. Organophosphates inhibit acetylcholinesterase, which results in an excess of acetylcholine at the<br />

neuronal synapses and myoneural junctions. Excess acetylcholine initially excites-and then paralyzesneurotransmission<br />

at the motor endplate and stimulates CNS muscarinic and nicotinic sites.<br />

b. Carbamates have the same mechanism of action as organophosphates, except that they do not permanently<br />

bind to acetylcholinesterase; therefore, their mechanism of action is transient, and pralidoxime (2-PAM, the<br />

antidote) is not necessary.<br />

c. Clinical effects<br />

(1) Muscarinic effects<br />

(a)<br />

"DUMBBELS"<br />

_Qiarrhea<br />

Urination<br />

Miosis<br />

.B.radycardia<br />

.B.ronchorrhea/bronchospasm<br />

fmesis<br />

_Lacrimation<br />

.S.alivation<br />

(b) Hypotension and bradycardia<br />

(2) Nicotinic effects<br />

(a)<br />

Muscle fasciculations, cramps and weakness, paralysis<br />

(b) Hypoventilation and cyanosis<br />

(c) Diaphoresis<br />

(3) CNS effects<br />

(a)<br />

Restlessness, respiratory depression<br />

(b) Coma, convulsions<br />

d. Organophosphates bind with cholinesterase to form a diethylphosphate bond that can be broken with<br />

pralidoxime (2-PAM chloride).<br />

2. Clinical presentation<br />

a. The classic clinical scenario is a farmer with profuse vomiting and diarrhea, pinpoint pupils, and<br />

diaphoresis who has a breath odor of insecticide or garlic.<br />

b. The "DUMBBELS" syndrome develops first, although patients are usually tachycardic initially and later<br />

develop bradycardia. Respiratory insufficiency and sudden respiratory arrest may occur.<br />

3. Diagnostic evaluation<br />

a. Should include a plasma or RBC cholinesterase (results will not be available immediately in the emergency<br />

department)<br />

b. It will be lower than normal in the presence of organophosphate poisoning (


TOXICOLOGIC DISORDERS<br />

(b) Large doses are frequently needed and should be repeated until tracheobronchial secretions dry up.<br />

i. In symptomatic adults, the minimum dose is 2 mg IV every 10-15 minutes as needed; the<br />

dose may be doubled every 10 minutes until secretions are controlled.<br />

ii. In symptomatic children, the dosage is 0.05 mg/kg IV every 15 minutes as needed. The dose<br />

may be doubled every 10 minutes until secretions are controlled.<br />

(2) Pralidoxime (2-PAM chloride)<br />

(a) A biochemical antidote that reactivates the cholinesterase that has been phosphorylated (bound)<br />

by the organophosphate<br />

(b) If this antidote is not given within 24-36 hours, the cholinesterase molecule may be irrevocably<br />

bound ("aging," ie, the antidote function becomes limited), and new cholinesterase will take<br />

weeks to regenerate. However, pralidoxime given later may still be beneficial and should be<br />

administered.<br />

(c) Patients with signs of cholinergic toxicity should be given pralidoxime as soon as possible even if<br />

the precise toxin is not yet known.<br />

i. In adults, the initial dose of 1 g IV over 15-30 minutes may be repeated in 1-2 hours as needed.<br />

ii. In children, the initial dosage of 25-50 mg/kg IV over 15-30 minutes can be repeated in<br />

1-2 hours.<br />

(3) Benzodiazepines<br />

(a) Seizure prophylaxis (all severely poisoned patients)<br />

(b) Also help prevent rhabdomyolysis and discomfort associated with muscle fasciculations<br />

d. Drugs that should not be given to patients with organophosphate toxicity<br />

5. Nerve agents<br />

(1) Opioids (worsen respiratory depression)<br />

(2) Physostigmine/pyridostigmine (carbamates)<br />

(3) Succinylcholine - prolonged paralysis<br />

(4) Sympathomimetic agents (to treat bronchospasm, eg, aminophylline)<br />

a. Originally developed as insecticides prior to World War 11; they have been used by the military and, more<br />

recently, by terrorists.<br />

(1) Cyclohexyl methylphosphonofluoridate (CMPF or GF)<br />

(2) Soman (GD)<br />

(3) Tabun (GA)<br />

(4) Sarin (GB)<br />

(5) vx<br />

b. Nerve agents should be thought of as "potent organophosphates." Properties that distinguish nerve agents<br />

from conventional insecticides<br />

(1) High potential for multiple casualties presenting simultaneously, secondary to accidental release from<br />

military stockpiles, bioterrorist attacks, or chemical warfare<br />

(2) Highly potent (in severe exposures, death can occur within minutes)<br />

(3) More rapid "aging" than conventional insecticides, thus limiting the use of pralidoxime as an antidote.<br />

c. Management<br />

(1) Prehospital<br />

(a) Prepare for multiple casualties<br />

(b) Activate disaster plan<br />

(c) Assemble all appropriate available medications: atropine, glycopyrrolate, pralidoxime,<br />

benzodiazepines<br />

(2) Hospital decontamination area<br />

(a) Protect medical personnel and emergency department from secondary contamination.<br />

(b) Ensure surface decontamination of all affected patients before entering emergency department<br />

i. Remove clothing.<br />

ii. Surface washing as indicated.<br />

iii. Contain contaminated secretions.<br />

(3) Emergency department - pharmacologic therapy (same as above)<br />

577


TOXICOLOGIC DISORDERS<br />

S. Mushroom poisoning<br />

1. Distinct clinical classes based on mushrooms that contain:<br />

a. Cyclopeptides<br />

b. Monomethylhydrazine<br />

c. Muscarine<br />

d. Coprine<br />

e. Psilocybin<br />

2. Diagnostic information<br />

a. The most toxic classes have delayed onset of initial symptoms (nausea, vomiting, diarrhea).<br />

(1) Cyclopeptides - 6-10 hours after ingestion<br />

(2) Monomethylhydrazine - 6-10 hours after ingestion<br />

b. Identification of specific mushrooms is very difficult; assistance of the local poison center and a mycologist<br />

should be sought.<br />

3. Specific mushroom poisonings<br />

a. Cyclopeptide-containing mushrooms<br />

(1) Genera: Amanita phalloides, Calerina, Lepiota<br />

(2) Clinical effects are predominately renal, hepatic, and CNS.<br />

(a) Phase I: nausea, vomiting, and diarrhea, 6-10 hours after ingestion<br />

(b) Phase II: symptoms subside but liver function tests are increasing (24-48 hours)<br />

(c) Phase Ill: 1-6 days after ingestion: toxic hepatitis, renal failure, encephalopathy, seizures, coma<br />

(3) Management<br />

(a) Early recognition, supportive care<br />

(b) The following therapies have been used with varying success: multidose activated charcoal, highdose<br />

penicillin G, N-acetylcysteine, milk thistle<br />

b. Monomethylhydrazine-containing mushrooms<br />

(1) Genus: Cyromitra escu/enta (false morel)<br />

(2) Clinical presentation<br />

(a) Nausea and vomiting (6-10 hours after ingestion)<br />

(b) Hepatorenal failure<br />

(c) Seizures<br />

(3) Management<br />

(a) General supportive measures and activated charcoal<br />

(b) Benzodiazepines and pyridoxine (vitamin B6)<br />

c. Muscarine-containing mushrooms<br />

(1) Genera: Clitocybe and lnocybe (several varieties)<br />

(2) Pathophysiology and clinical effects: muscarinic ("DUMBBELS")<br />

(3) Management<br />

(a)<br />

Activated charcoal<br />

(b) Management geared toward cholinergic toxidrome.<br />

d. Coprine-containing mushrooms<br />

(1) Genus: Coprinus atramentarious ("inky cap")<br />

(2) Pathophysiology: disulfiram-like effect<br />

(3) Clinical presentation: tachycardia, flushing, nausea, and vomiting within 0.5-2 hours of ingestion<br />

(4) Management<br />

(a) Supportive care<br />

(b) Consider activated charcoal<br />

e. Psilocybin-containing mushrooms<br />

(1) Genera: Psilocybe, Cymnopilus, Conocybe, Panaeolus<br />

(2) Pathophysiology: act on the CNS in a manner similar to that of LSD<br />

(3) Clinical presentation: ataxia, hallucinations, hyperkinesis, seizures, nausea and vomiting, mydriasis,<br />

tachycardia (within 0.5-2 hours after ingestion)<br />

(4) Treatment: benzodiazepines for agitation and seizures<br />

578


TOXICOLOGIC DISORDERS<br />

T. Cyanide<br />

1. Sources of exposure<br />

a. Chemical laboratories--.. sudden collapse of a worker is suspicious<br />

b. Fires<br />

(1) Cyanide is a combustion product of wool, silk, polyurethane, synthetic rubber, and nitrocellulose.<br />

(2) Suspect in a fire victim/firefighter/paramedic with decreased level of consciousness, metabolic<br />

acidosis, and hypotension.<br />

c. Electroplating<br />

d. Precious metal refining<br />

e. Photography<br />

f. Ingestion of acetonitrile (artificial nail remover), which converts to cyanide<br />

g. Prolonged exposure to nitroprusside<br />

2. Pathophysiology: cyanide interacts with Fe+++ in cytochrome aa3 - blockage of electron transport chain-..<br />

1 ATP production, 1 H+ consumption (acidosis), and t lactic acid<br />

3. Clinical presentation<br />

a. Neurologic and cardiovascular signs and symptoms predominate the clinical picture.<br />

(1) Neurologic --.. agitation, seizures, 1 mental status, 1 respirations<br />

(2) Cardiovascular<br />

(a) Bradycardia<br />

(b) Hypotension<br />

(c) Pulmonary edema<br />

b. Odor of bitter almonds (only 50% of the population can detect this)<br />

c. Gastritis (with ingestions)<br />

d. Skin --.. cherry red color or cyanosis (late finding)<br />

e. Loss of color difference between retinal veins and arterioles on the funduscopic exam (arterialization due to<br />

high venous pO 2<br />

)<br />

4. Management<br />

U. Digitalis<br />

a. General measures<br />

(1) IV line, oxygen, cardiac monitor<br />

(2) Decontamination (unless inhaled)<br />

(a) Lavage, activated charcoal<br />

(b) Remove clothing and wash skin (hospital personnel should avoid self-contamination; vomitus may<br />

give off hydrogen cyanide gas).<br />

b. Specific therapy<br />

(1) Hydroxocobalamin 5 g IV and repeat once; causes red discoloration of skin and urine; may be used<br />

safely in smoke inhalation victims<br />

(2) Cyanide antidote kit: contains a 3-step regimen that should be started as soon as cyanide poisoning<br />

is suspected.<br />

(a) Amyl nitrite pearls: until IV access is obtained, crush into gauze and have patient intermittently<br />

inhale.<br />

i. Nitrites: administration of nitrates induces methemoglobinemia; cyanide attaches to the<br />

methemoglobin.<br />

ii. Do not administer amyl nitrite or sodium nitrite to fire victims, because their hemoglobinoxygen<br />

carrying capacity may already be compromised by coexistent carbon monoxide<br />

toxicity. Use sodium thiosulfate.<br />

(b) 3% sodium nitrite IV at 2.5 ml/min; in anemic patients, dosage may need to be decreased.<br />

Mechanism of action is induction of methemoglobinemia; do not use in fire victims.<br />

(c) 25% sodium thiosulfate IV bolus: sodium thiosulfate uses rhodanase (enzyme ubiquitous in body)<br />

to convert cyanide and thiosulfate to thiocyanate, which is excreted in the urine.<br />

1. Pharmacology and mechanism of toxicity<br />

a. Cardiac glycosides have a large volume of distribution and a long half-life; elimination is through the<br />

kidney; they are not dialyzable.<br />

b. Toxicity is via Na+-K+ ATPase inhibition--.. changes in serum electrolytes and cardiac conduction<br />

579


TOXICOLOGIC DISORDERS<br />

2. Classic clinical scenario<br />

a. Weakness, syncope<br />

b. Visual changes (yellow halos around lights, referred to as "xanthopsia," is classic)<br />

c. Nausea and vomiting (acute toxicity)<br />

d. Anorexia (chronic toxicity)<br />

e. Hyperkalemia (acute poisoning); hypokalemia (chronic poisoning)<br />

f. ECG changes<br />

(1) Bradycardia with AV block (most common finding)<br />

(2) Paroxysmal atrial tachycardia with block and bidirectional ventricular tachycardia (most characteristic<br />

findings)<br />

(3) Although digitalis does not precipitate atrial fibrillation, in patients with chronic poisoning, underlying<br />

atrial fibrillation can be regularized.<br />

3. Management<br />

V. ~-Blockers<br />

a. Gastric decontamination (charcoal)<br />

b. Indications for digoxin Immune Fab<br />

(1) Cardiovascular collapse<br />

(2) Conduction abnormalities<br />

(a) AV block (Mobitz II or third degree)<br />

(b) Ventricular dysrhythmias<br />

(3) Abnormal chemistries<br />

(a)<br />

K+ >5.5 mEq/L (some sources recommend use in patients with K+ >5.0 mEq/L)<br />

(b) Digitalis level >10 mg/ml<br />

c. Treatments to avoid<br />

(1) Routine supplementation with K+ or Mg+"<br />

(2) Transvenous pacing or cardioversion (may induce ventricular dysrhythmias)<br />

(3) Use of calcium is controversial.<br />

(4) ~-blockers and calcium channel blockers, as well as class IA and class IC drugs (specifically<br />

contraindicated; may worsen cardiac conduction)<br />

1. Produce toxicity through ~ 1<br />

- or ~ 2 -receptor blockade<br />

2. Classic clinical scenario<br />

a. Coma/seizures<br />

b. Hypotension/bradycardia<br />

c. Hypoglycemia<br />

d. Varying degrees of AV blockade<br />

e. ECG changes can reflect sodium- or potassium-channel blockade<br />

(1) Widened QRS<br />

(2) Prolonged QT interval<br />

3. Management<br />

a. Gastric decontamination<br />

b. Specific therapy for cardiovascular collapse<br />

(1) Vasopressors<br />

(2) Atropine can be given but may be ineffective.<br />

(3) Glucagon 2-5 mg IV bolus, then a 2-5 mg/hr infusion<br />

(4) Calcium chloride 20 mg/kg bolus, then a 20 mg/kg/hr infusion<br />

(5) High-dose insulin therapy (see calcium channel blockers, below)<br />

(6) IV lipid emulsion (see calcium channel blockers, below)<br />

(7) Pacing at a modest rate to allow channels to recycle<br />

(8) Intra-aortic balloon pump<br />

c. Contraindicated drugs<br />

(1) Calcium channel blockers<br />

(2) Class IA and class IC agents<br />

580


TOXICOLOGIC DISORDERS<br />

W. Calcium channel blockers<br />

1. Inhibit calcium channel conduction in both smooth and cardiac muscle, including the cardiac conduction<br />

system.<br />

2. Classic clinical scenario<br />

a. Hypotension and bradycardia, especially with diltiazem, verapamil; may have reflex tachycardia with the<br />

dihydropyridines<br />

b. Hyperglycemia<br />

c. Metabolic acidosis<br />

d. Pulmonary edema<br />

e. lleus<br />

f. ECG changes reflect conduction delay (AV dissociation is common)<br />

(1) Prolonged PR interval<br />

(2) Bradycardia, bradydysrhythmias<br />

3. Management<br />

a. Decontamination<br />

(1) Charcoal may be considered.<br />

(2) Whole-bowel irrigation (if sustained-release preparation)<br />

b. Specific therapies<br />

(1) Vasopressors (norepinephrine preferred)<br />

(2) Atropine (may be ineffective)<br />

(3) Clucagon 0.05-0.15 mg/kg bolus, then a 2-5 mg/hr infusion<br />

(4) Calcium<br />

(5) Pacing at a modest rate to allow channels to recycle<br />

(6) Invasive modalities: intra-aortic balloon pump, extracorporeal membrane oxygenation, cardiac bypass<br />

c. High-dose insulin (1-10 units/kg bolus, followed by 1-10 units/kg//hr) should be started in any patient<br />

not responding to IV fluids and calcium administration.<br />

d. Intravenous lipid emulsion (20% solution): 1.5 ml/kg bolus, followed by 0.25 ml/kg/min for 30-60<br />

minutes.<br />

e. Because of the prolonged absorption time of sustained-release preparations, 24-hour observation is<br />

mandatory.<br />

X. Carbon monoxide<br />

1. Primary mechanism of toxicity is carbon monoxide binding to cytochrome oxidase, disrupting oxidative<br />

phosphorylation, and interference with microvascular homeostasis. Of secondary importance is carbon<br />

monoxide binding to hemoglobin, which subsequently decreases the oxygen-carrying capacity of the blood.<br />

2. Clinical presentation<br />

a. Mild to moderate poisoning<br />

(1) Weakness<br />

(2) Headache, dizziness<br />

(3) Nausea, vomiting<br />

b. Severe poisoning<br />

(1) Syncope<br />

(2) Visual complaints<br />

(3) Chest pain<br />

(4) Dysrhythmias<br />

(5) Coma, seizures<br />

(6) Cardiovascular collapse<br />

c. Flu-like symptoms are common, accounting for frequent misdiagnosis. Diagnostic clues:<br />

(1) Flu without fever<br />

(2) Multiple victims from same dwelling<br />

(3) Small household pets ill or dead from unknown cause<br />

(4) Recent vehicle travel<br />

(5) Early winter when furnaces are being turned on<br />

(6) Power outages in winter<br />

581


TOXICOLOGIC DISORDERS<br />

d. Delayed neuropsychiatric sequelae are seen in 10%-43% of cases after apparent recovery<br />

(1) Headache, dizziness<br />

(2) Memory deficits<br />

(3) Personality alterations<br />

(4) Parkinsonism<br />

3. Diagnostic evaluation<br />

a. Serum carboxyhemoglobin levels<br />

(1) May not correlate well with toxicity, especially if drawn after empiric oxygen administration<br />

(2) Venous levels may be used.<br />

b. Carbon monoxide neuropsychiatric screening battery is a sensitive indicator of toxicity in adults.<br />

c. CT or MRI may show bilateral hypodensity in the basal ganglia, diffuse white matter hypodensities, or<br />

cerebral edema.<br />

4. Management: oxygen therapy<br />

Y. Mercury<br />

a. 100% oxygen by tight-fitting mask for 4 hours; longer therapy is indicated for infants and pregnant patients.<br />

b. Hyperbaric oxygen should be considered if any of the following conditions are present:<br />

(1) Carboxyhemoglobin level >25%<br />

(2) Carboxyhemoglobin level > 10% if pregnant<br />

(3) Any neurologic symptom other than mild headache, including brief loss of consciousness, focal<br />

neurologic deficit, seizure<br />

(4) Coma<br />

(5) Myocardial ischemia<br />

(6) Worsening symptoms despite oxygen therapy<br />

c. Half-life of carbon monoxide during treatment with various oxygen sources<br />

1. Forms<br />

(1) Room air: 240 minutes<br />

(2) Nonrebreather: 90 minutes<br />

(3) Hyperbaric oxygen: 20 minutes<br />

a. Elemental (liquid mercury, quicksilver-"' thermometers, barometers, sphygmomanometers)<br />

b. Inorganic (mercuric chloride)-"' chemistry laboratories<br />

c. Organic (methylmercury)-"' fish and shellfish<br />

2. Pathophysiology<br />

a. Elemental (CNS and lung)<br />

(1) Acute inhalation-"' pulmonary edema<br />

(2) Chronic exposure-"' tremor, acrodynia, neuropsychiatric changes<br />

(3) Contamination<br />

(a)<br />

Elemental mercury is most toxic when inhaled (especially if volatilized by heat).<br />

(b) Ingestions (eg, broken thermometer in mouth) are nontoxic unless extravasation into the<br />

peritoneum occurs.<br />

b. Inorganic (skin, GI tract, and kidney)<br />

(1) Corrosive to skin<br />

(2) Highly toxic when ingested<br />

(a)<br />

Hemorrhagic gastroenteritis -"' shock<br />

(b) Acute tubular necrosis<br />

c. Organic (chronic placental-fetal transfer-"' CNS): profound neurotoxicity to the developing brain<br />

3. Management<br />

a. Decontamination for ingestion of inorganic and organic mercury: despite the corrosiveness of inorganic<br />

mercury (and potential risk of perforation), the benefit of GI decontamination outweighs the risk.<br />

(1) If mercury has been ingested within the last 2 hours, lavage with a small orogastric (or nasogastric) tube.<br />

(2) When mercury is seen on plain radiograph of the abdomen, whole-bowel irrigation is indicated.<br />

582


TOXICOLOGIC DISORDERS<br />

b. Chelation for suspected ingestion (do not wait for blood levels)<br />

Z. Sulfonylureas<br />

(1) For all forms of mercury ingestion, may use dimercaptosuccinic acid (DMSA), administered orally<br />

(2) For significant, acute toxicity and/or inability to tolerate oral therapy, use BAL (dimercaprol) until the<br />

patient can tolerate DMSA.<br />

1. Examples (all are oral agents): glipizide, glimepiride, glyburide, chlorpropamide<br />

2. Pathophysiology: primary mechanism is binding to pancreatic f:\ cells and stimulating endogenous insulin secretion<br />

3. Clinical presentation<br />

a. Signs and symptoms of hypoglycemia (eg, tachycardia, tremor, diaphoresis, confusion, agitation, seizures,<br />

coma)<br />

b. Can range from mild to severe and may be delayed<br />

c. May not manifest in children, the elderly, those taking f:\-adrenergic receptor blockers; must check blood<br />

glucose levels!<br />

4. Diagnostic evaluation<br />

a. Rapid bedside glucose determination<br />

(1) Can be unreliable at extreme hypo- and hyperglycemia<br />

(2) Check frequently (less often as clinical scenario evolves and stabilizes)<br />

b. Electrolytes: potassium and magnesium levels can also be depressed.<br />

5. Management<br />

a. Activated charcoal in the appropriate clinical conditions.<br />

b. Supplemental glucose<br />

(1) Should be administered only to hypoglycemic patients. In healthy patients with sulfonylurea poisoning,<br />

administration of prophylactic glucose can precipitate severe hypoglycemia, because these drugs may<br />

facilitate exaggerated insulin secretion in response to a glucose load.<br />

(2) Repeated boluses and/or initiation of a glucose infusion (with D5 or D10) may be necessary<br />

c. Octreotide<br />

(1) Synthetic, long-acting analog of somatostatin that antagonizes pancreatic insulin secretion<br />

(2) Indications: hypoglycemic sulfonylurea-poisoned patients whose serum glucose concentrations cannot<br />

be maintained with a dextrose infusion. Some recommend its use as a first-line agent along with<br />

glucose administration.<br />

(3) Available for administration by IV or SC routes only<br />

d. Alkalinization of the urine has been used to enhance renal elimination of chlorpropamide but is of no<br />

value for treatment of poisoning by other sulfonylureas.<br />

e. Admission or 24-hour observation due to long-acting nature of these agents<br />

583


TOXICOLOGIC DISORDERS: PRACTICE CLINICAL SCENARIOS<br />

TOXICOLOGIC DISORDERS: PRACTICE CLINICAL SCENARIOS<br />

Answers immediately follow the practice clinical scenarios.<br />

Scenario A<br />

Presentation: A 36-year-old woman presents via EMS for evaluation of vomiting and lethargy. The<br />

paramedics report that the woman's family has been ill with flu-like symptoms. The patient has had no<br />

fever, diarrhea, or cough, and no significant past medical history.<br />

Physical examination: Vital signs are temperature 98.2°F (36.8°C), heart rate 85 beats per minute,<br />

respiratory rate 18 breaths per minute, blood pressure 122/60 mm Hg, and oxygen saturation of 94%<br />

on pulse oximetry. The patient appears sleepy and pale. Pupils are equal and reactive to light. Chest<br />

auscultation demonstrates clear lung fields and regular cardiac rate and rhythm. On neurologic<br />

examination, the patient is oriented to self and place but has some difficulty providing some details of the<br />

history. She has no focal deficits. An initial blood glucose level is 90 mg/dL.<br />

What is the diagnosis?<br />

Scenario 8<br />

Presentation: A 2-year-old boy presents with his parents for evaluation of abdominal pain, vomiting, and<br />

copious bloody diarrhea. The father reports that the boy was found playing under the bathroom sink, where<br />

his mother's prenatal vitamins are stored.<br />

Physical examination: Vital signs are temperature 100°F (38°C), heart rate 144 beats per minute, respiratory<br />

rate 30 breaths per minute, and blood pressure 60/palp. The child weighs 44 pounds (20 kg). He is pale,<br />

diaphoretic, and barely responsive to stimuli. Chest auscultation demonstrates clear lung fields and regular<br />

cardiac rate and rhythm with no murmur. Capillary refill time is 5 seconds. Examination of the abdomen<br />

reveals mild, diffuse tenderness without peritoneal signs. Rectal examination is positive for gross blood.<br />

The child is somnolent, with no focal deficits.<br />

Diagnostic evaluation<br />

Arterial blood gases: 7.12 I 90 I 22 I 9<br />

WBC = 18,300/mm 3 , hemoglobin= 8.9 mg/dL, hematocrit = 26<br />

Na+= 135 mEq/L, Cl-= 98 mEq/L, K+ = 4.5 mEq/L, HCO 1<br />

- = 8<br />

Glucose: 54 mg/dl<br />

BUN: 38 mg/dL<br />

What is the diagnosis?<br />

Scenario C<br />

Presentation: A 22-year-old woman presents after ingesting over 100 tablets of acetaminophen in a suicide<br />

attempt 4 hours before arrival. She denies coingestants and complains of nausea. She has no past medical<br />

history and denies pregnancy.<br />

Physical examination: Vital signs are temperature 98.7°F (37°C), heart rate 82 beats per minute, respiratory<br />

rate 14 breaths per minute, and blood pressure 123/71 mmHg. The patient is tearful but in no acute<br />

distress. Pupils are 3 mm and reactive to light, and mucus membranes are moist. Chest auscultation<br />

demonstrates clear lung fields and regular cardiac rate and rhythm with no murmurs. Bowel sounds are<br />

normal, and the abdomen is nontender on palpation. Neurologic examination is unremarkable.<br />

What is the diagnosis?<br />

584


TOXICOLOGIC DISORDERS: PRACTICE CLINICAL SCENARIOS<br />

Scenario D<br />

Presentation: A 20-year-old man presents with a friend 1 hour after drinking a quart of windshield wiper<br />

solution in a suicide attempt. The patient denies coingestants, vision changes, abdominal pain, nausea, and<br />

vomiting. He has no past medical history.<br />

Physical examination: Vital signs are temperature 97.8°F (37°C), heart rate 79 beats per minute, respiratory<br />

rate 32 breaths per minute, and blood pressure 124/77 mmHg. The patient is intoxicated but in no distress.<br />

Pupils are equal and reactive to light. Chest auscultation demonstrates clear lung fields and regular cardiac<br />

rate and rhythm. Bowel sounds are normal, and the abdomen is nontender on palpation. On neurologic<br />

examination, visual acuity is 20/20 bilaterally, the patient is oriented to person and place, cranial nerves<br />

II-XII are intact, and no focal deficits are seen.<br />

What is the diagnosis?<br />

Scenario E<br />

Presentation: An 82-year-old man presents for evaluation of "not feeling right." His wife states that he has<br />

become increasingly confused over the last 3 days. He has also been very irritable, unsteady on his feet,<br />

and seems to be breathing very rapidly and deeply at times. They deny a history of recent or past trauma.<br />

Past medical history includes hypertension, hyperlipidemia, and coronary artery disease, and medications<br />

include hydrochlorothiazide, simvastatin, aspirin, diltiazem, and nitroglycerin as needed.<br />

Physical examination: Vital signs are temperature 100.2°F (38°C), heart rate 99 beats per minute,<br />

respiratory rate 32 breaths per minute, and blood pressure 154/88 mmHg. The patient is alert, with no<br />

signs of trauma. His face is flushed, and mucous membranes are dry. His speech is slurred, but there is no<br />

facial drop. Pupils are equal and reactive to light. Chest auscultation demonstrates clear lung fields and<br />

regular cardiac rate and rhythm. Bowel sounds are normal, and the abdomen is nontender on palpation.<br />

On neurologic examination, motor strength is symmetric in upper and lower extremities. There is no<br />

clonus, and Babinski responses are down going bilaterally. Cranial nerves are intact. The patient appears<br />

to have ataxia with transfer into hospital bed; however, gait testing was not attempted because of his<br />

unsteadiness. Cardiac monitor shows normal sinus rhythm.<br />

What chronic poisoning can manifest in this fashion?<br />

Scenario F<br />

Presentation: A 42-year-old landscaper presents via EMS with profuse vomiting and diarrhea, lethargy, and<br />

diaphoresis. On their arrival at the scene, the paramedics note that the patient is very tachycardic and has<br />

pinpoint pupils. They report that the patient did not respond to naloxone before arrival in the emergency<br />

department.<br />

Physical examination: Vital signs are temperature 98.4°F, heart rate 55 beats per minute, respiratory rate 28<br />

breaths per minute, and blood pressure 105/65 mm Hg. The patient appears lethargic. Pupils are pinpoint,<br />

and eyes are tearing. The oropharynx is moist with excessive salivation, and the patient's breath has a<br />

strong odor of garlic. Chest auscultation demonstrates rhonchi bilaterally.<br />

What is the diagnosis?<br />

Scenario G<br />

Presentation: A 25-year-old woman is brought to the emergency department by her family who notes that<br />

she has had abdominal pain, diarrhea, increasing tremor, confusion, slurred speech, and difficulty walking/<br />

ataxia over the last 2 days. She has a history of bipolar disorder and has been hospitalized previously for<br />

suicide attempts.<br />

Physical examination: Vital signs are temperature 99.4°F, heart rate 95 beats per minute, respiratory rate<br />

22 breaths per minute, and blood pressure 118/70 mmHg. The patient appears very drowsy, and answers<br />

to your questions are slurred. Pupils are equal and reactive to light. Chest auscultation demonstrates clear<br />

585


TOXICOLOGIC DISORDERS: PRACTICE CLINICAL SCENARIOS<br />

lung fields and regular cardiac rate and rhythm. On neurologic examination, the patient is disoriented and<br />

has diffuse hyperreflexia and a fine tremor. She has no focal deficits. An initial blood glucose level is 85<br />

mg/dL, and ECG demonstrates QTc of 490 milliseconds.<br />

What is the diagnosis?<br />

Scenario H<br />

Presentation: A 76-year-old man diabetic man presents with acute confusion and agitation after his wife<br />

cal led EMS. The paramedics noted that the patient was tremulous, diaphoretic, drowsy, and agitated on<br />

their arrival. An initial blood glucose level at the scene was 25 mg/dL. The patient was given 25 grams of<br />

dextrose before transport, after which mental status improved.<br />

Physical examination: Vital signs are temperature 98.2°F (36.8°C), heart rate 115 beats per minute,<br />

respiratory rate 24 breaths per minute, and blood pressure 146/88 mm Hg. The patient appears drowsy and<br />

diaphoretic. Pupils are equal and reactive to light. Chest auscultation demonstrates clear lung fields and<br />

tachycardic rate and rhythm. On neurologic examination, the patient is oriented to self only and has some<br />

difficulty providing any history. He appears to have no focal deficits. A repeat blood glucose level is 55<br />

mgldl.<br />

What is the diagnosis?<br />

Scenario I<br />

Presentation: An 18-month-old, previously healthy baby boy is brought to the emergency department<br />

via EMS for evaluation of depressed mental status. His mother states that she found him playing in his<br />

older brother's pill holder 2 hours before the symptoms started, but she did not think any of the pills were<br />

missing. The brother takes medications for ADHD, but she is unsure of what they are called.<br />

Physical examination: Vital signs are temperature 96.8°F, heart rate 44 beats per minute, respiratory<br />

rate 10 breaths per minute, oxygen saturation 99% on room air, and blood pressure 72/48 mm Hg. The<br />

patient appears drowsy but responds to light touch. Pupils are pinpoint. Chest auscultation demonstrates<br />

bradycardia and regular rhythm, with no murmurs, rubs, or gallops; lung fields are clear bilaterally. On<br />

neurologic examination, the patient will follow simple commands after awakened with light touch but<br />

drifts back to sleep soon after.<br />

What is the diagnosis?<br />

ScenarioJ<br />

Presentation: A 64-year-old female with a past medical history of hypertension and depression with<br />

multiple suicide attempts presents to the emergency department via EMS after she was found unresponsive<br />

by her family.<br />

Physical examination: Vital signs are temperature 96.2°F, heart rate 42 beats per minute, respiratory rate 6<br />

breaths per minute, oxygen saturation 87% on 6 L nasal cannula, and blood pressure 62/34 mmHg. She<br />

appears ill, has pale skin, and responds minimally to painful stimuli. Pupils are 3 mm bilaterally. Heart rate<br />

is bradycardic with regular rhythm. Breath sounds clear bilaterally. Skin is cool and pale. On neurologic<br />

examination, the patient cannot follow any commands or provide any history.<br />

What is the diagnosis?<br />

586


TOXICOLOGIC DISORDERS: PRACTICE CLINICAL SCENARIOS<br />

ANSWERS TO PRACTICE CLINICAL SCENARIOS<br />

Scenario A<br />

Diagnosis: carbon monoxide poisoning<br />

Diagnostic evaluation: A carboxyhemoglobin level is essential to the diagnosis of carbon monoxide<br />

poisoning. Arterial or venous blood samples may be used. Note that the SaO 2<br />

will be normal, because it<br />

measures both oxygen and carbon monoxide-bound hemoglobin.<br />

Management: The patient should be given high-flow oxygen, and hyperbaric oxygen should be considered.<br />

Hyperbaric oxygen therapy decreases the rate of delayed neurologic sequelae in carbon monoxidepoisoned<br />

patients and reduces the half-life of carbon monoxide to 20 minutes, versus a half-life of 240<br />

minutes when a patient is breathing room air.<br />

Scenario B<br />

Diagnosis: iron poisoning<br />

Diagnostic evaluation: Iron poisoning is classically described as occurring in five sequential "stages."<br />

• First stage: first few hours after ingestion; direct irritative effect of iron on GI tract causes GI upset,<br />

including vomiting and diarrhea (possibly bloody). The absence of these symptoms within 6 hours of<br />

ingestion essentially excludes the diagnosis.<br />

• Second stage: may last for up to 24 hours; GI symptoms improve, but patient will not become<br />

asymptomatic and may have abnormal vital signs and evidence of poor perfusion.<br />

• Third stage: may appear early or develop hours after the second stage; shock and metabolic acidosis<br />

develop, bleeding and hypovolemia may worsen, and hepatic dysfunction, cardiomyopathy, and<br />

renal failure may occur.<br />

• Fourth stage: 2-5 days after ingestion; transaminases increase and may progress to hepatic failure.<br />

• Fifth stage: 4-6 weeks after ingestion; involves gastric outlet obstruction secondary to corrosive<br />

effects of iron on pyloric mucosa.<br />

Management: A serum iron concentration should be obtained. Serum iron concentrations of 300-500<br />

mcg!dL correlate with mild symptoms, 500-1,000 mcgldl with moderate symptoms, and > 1 000 mcgldl<br />

with severe symptoms. GI decontamination with orogastric lavage can be considered. Whole-bowel<br />

irrigation can be performed in standard fashion. Iron does not adsorb to activated charcoal. Deferoxamine<br />

(IV infusion) is the chelator of choice for severe iron poisoning and can be repeated every 4-6 hours as<br />

needed. Deferoxamine removes iron from tissues and free iron from plasma and is safe in children and in<br />

pregnancy. Give empirically for critically ill patients.<br />

Scenario C<br />

Diagnosis: acetaminophen toxicity<br />

Diagnostic evaluation: An acetaminophen level should be obtained and plotted on the Rumack-Matthew<br />

nomogram (can be used only after a single acute ingestion). A 4-hour level > 150 mcg/ml is considered<br />

toxic; the nomogram will allow prediction of potentially toxic acetaminophen levels between 4 and 24<br />

hours after acute ingestion.<br />

Management: GI decontamination with activated charcoal is recommended by some in the appropriate<br />

clinical conditions (patient protecting airway). Many forgo administration of activated charcoal given the<br />

high efficacy of N-acetylcysteine (NAC). Its administration may be more important when the presence of<br />

coingestants is of concern. Administration of activated charcoal is not thought to significantly hinder the<br />

efficacy of oral NAC, and no dosing adjustments to either need be made. If the patient's acetaminophen<br />

level is above the treatment line on the Rumack-Matthew nomogram, NAC should ideally be administered.<br />

NAC is most effective when initiated within 8 hours of ingestion but is still very effective at reducing<br />

toxicity in later presenting patients and should not be withheld.<br />

587


TOXICOLOGIC DISORDERS: PRACTICE CLINICAL SCENARIOS<br />

Scenario F<br />

Diagnosis: organophosphate poisoning<br />

Diagnostic evaluation: For patients with significant fasciculations, a CK should be obtained. An RSC<br />

cholinesterase should be obtained (results will not be available immediately in the emergency department)<br />

and will be lower than normal in the presence of organophosphate poisoning (


TOXICOLOGIC DISORDERS: PRACTICE CLINICAL SCENARIOS<br />

Scenario I<br />

Diagnosis: exploratory ingestion of clonidine<br />

Management: Supportive care, including intravenous access, cardiorespiratory monitoring. Respiratory<br />

failure is uncommon but can occur. Intravenous fluid boluses are first-line treatments for hypotension, and<br />

atropine can be used for bradycardia. Vasopressors are sometimes needed. Naloxone has been used with<br />

inconsistent success; large doses (10 mg) are needed.<br />

Scenario J<br />

Diagnosis: ~-blocker overdose; this scenario could also occur after an overdose of calcium channel<br />

blockers. Both types of ingestion are treated in a similar fashion; many patients are on both types of agents,<br />

and the history is unclear on presentation.<br />

Management: Supportive care included intravenous access and cardiorespiratory monitoring. Activated<br />

charcoal can be used in the correct clinical situation. Consider whole-bowel irrigation for extendedrelease<br />

preparations. Respiratory failure can occur, and this patient may require endotracheal intubation.<br />

Intravenous fluid boluses are first-line treatments for hypotension, and atropine can be used for bradycardia.<br />

Other therapies for persistent bradycardia and hypotension include glucagon (traditional antidote for<br />

~-blocker poisoning but may work for calcium channel blocker poisoning), calcium (boluses followed by<br />

an infusion; hypercalcemia is well tolerated in these patients, and generous doses of calcium are needed<br />

to see an effect), high-dose insulin therapy (1 unit/kg bolus followed by an infusion of 1 unit/kg/hr, titrating<br />

upward; doses of 10 units/kg/hr have been used without complications), vasopressors (norepinephrine is<br />

recommended as first choice), lipid emulsion therapy, transcutaneous/transvenous pacing (may not capture),<br />

intra-aortic balloon pump, extracorporeal membrane oxygenation, and cardiac bypass.<br />

590


ENDOCRINE, METABOLIC, AND NUTRITIONAL DISORDERS<br />

ENDOCRINE, METABOLIC, AND NUTRITIONAL<br />

DISORDERS<br />

ADULT FLUID VOLUMES ......................................................................................................................................... 596<br />

Normal Values ........................................................................................................................................................... 596<br />

Patient Assessment .................................................................................................................................................... 596<br />

FLUID AND ELECTROLYTE DISORDERS .................................................................................................................. 596<br />

Basic Science Review ................................................................................................................................................ 596<br />

Total Body Water/Fluid Compartments ............................................................................................................... 596<br />

Water Regulation ............................................................................................................................................... 596<br />

Sodium Regulation ............................................................................................................................................. 597<br />

Potassium Regulation ......................................................................................................................................... 598<br />

Calcium Regulation ............................................................................................................................................ 598<br />

Magnesium Regulation ....................................................................................................................................... 598<br />

Chloride Regulation ........................................................................................................................................... 598<br />

Hydrogen-Potassium Exchange to Maintain Electrolyte Balance ......................................................................... 599<br />

The Anion Gap ................................................................................................................................................... 599<br />

Pathophysiology of Acid-Base Disorders ............................................................................................................. 599<br />

Electrolyte Disorders ................................................................................................................................................. 601<br />

Hyponatremia .................................................................................................................................................... 601<br />

Hypernatremia ................................................................................................................................................... 603<br />

Dehydration ....................................................................................................................................................... 604<br />

Hypokalemia ...................................................................................................................................................... 605<br />

Hyperkalemia ..................................................................................................................................................... 606<br />

Hypocalcemia .................................................................................................................................................... 607<br />

Hypercalcemia ................................................................................................................................................... 608<br />

Hypomagnesemia .............................................................................................................................................. 609<br />

Hypermagnesemia ............................................................................................................................................. 610<br />

Hypochloremia .................................................................................................................................................. 610<br />

Hyperchloremia ................................................................................................................................................. 611<br />

Key Concepts in Electrolyte Disorders ................................................................................................................ 611<br />

Acid-Base Disorders .................................................................................................................................................. 611<br />

General Approach to the Patient with an Acid-Base Abnormality ........................................................................ 611<br />

Metabolic Acidosis ............................................................................................................................................. 613<br />

Metabolic Alkalosis ............................................................................................................................................ 614<br />

Respiratory Acidosis ........................................................................................................................................... 614<br />

Respiratory Alkalosis .......................................................................................................................................... 615<br />

Lactic Acidosis ................................................................................................................................................... 615<br />

Important Points to Remember in the Management of Acid-Base Disorders ........................................................ 616<br />

Clinical Situations and Associated Laboratory Findings ....................................................................................... 616<br />

METABOLIC DISORDERS ......................................................................................................................................... 617<br />

Hypoglycemia ........................................................................................................................................................... 617<br />

Diabetic Ketoacidosis ................................................................................................................................................ 618<br />

Hyperglycemic Hyperosmolar Nonketotic Coma ...................................................................................................... 621<br />

Alcoholic Ketoacidosis .............................................................................................................................................. 622<br />

Thyroid Storm ........................................................................................................................................................... 623<br />

Myxedema (Hypothyroid) Coma ............................................................................................................................... 624<br />

Adrenal Insufficiency (Addison Disease) and Crisis ................................................................................................... 626<br />

591


ENDOCRINE, METABOLIC, AND NUTRITIONAL DISORDERS: SELF-ASSESSMENT QUESTIONS<br />

ENDOCRINE, METABOLIC, AND NUTRITIONAL DISORDERS:<br />

SELF-ASSESSMENT QUESTIONS<br />

1. The major intracellular cation is:<br />

(a) Potassium<br />

(b) Calcium<br />

(c) Chloride<br />

(d) Sodium<br />

2. The weight of total body water in a 70-kg adult man is closest to:<br />

(a) 40 kg<br />

(b) 20 kg<br />

(c)<br />

60 kg<br />

(d) 30 kg<br />

3. All of the following are associated with an increased difference between the calculated and measured serum<br />

osmolality (osmolal gap) except:<br />

(a) An ethanol level of 200mg%<br />

(b) Administration of 100 grams of mannitol<br />

(c) Hyperglycemic nonketotic coma<br />

(d) Severe ketoacidosis<br />

4. All of the following are true about antidiuretic hormone (ADH) except:<br />

(a)<br />

It is released in response to decreases in serum osmolality.<br />

(b) It is released in response to decreases in intravascular volume.<br />

(c) It acts on the renal tubules to decrease free water excretion.<br />

(d) It is present in excessive amounts in syndrome of inappropriate ADH secretion (SIADH).<br />

5. All of the following may result in changes in the concentration of ionized calcium except:<br />

(a) Decreased serum albumin<br />

(b) Hyperventilation<br />

(c) Excessive parathyroid hormone<br />

(d) Vitamin D intoxication<br />

6. The regulation and function of magnesium are closely tied to:<br />

(a)<br />

Sodium<br />

(b) Chloride<br />

(c) Calcium and phosphate<br />

(d) Serum albumin<br />

7. Causes of an anion gap acidosis include all of the following except:<br />

(a) Salicylate poisoning<br />

(b) lsopropyl alcohol ingestion<br />

(c) Uremia<br />

(d) Methanol poisoning<br />

8. An increased anion gap(> 16 mEq/dL) and an increased osmolal gap(> 10 mOsm/dL) may be seen in all of the<br />

following except:<br />

(a)<br />

Uremia<br />

(b) Ethanol intoxication<br />

(c) Methanol poisoning<br />

(d) Diabetic ketoacidosis<br />

592


ENDOCRINE, METABOLIC, AND NUTRITIONAL DISORDERS: SELF-ASSESSMENT QUESTIONS<br />

9. The pulmonary excretion of CO 2<br />

(hyperventilation):<br />

(a) Increases the serum H+ concentration<br />

(b) Increases the serum pH<br />

(c) Decreases the renal excretion of bicarbonate<br />

(d) Increases the serum concentration of bicarbonate<br />

10. Physiologic compensation for metabolic acidosis occurs through all of the following mechanisms except:<br />

(a) Persistent vomiting<br />

(b) Pulmonary excretion of CO 2<br />

(c) Increased renal H+ excretion<br />

(d) Increased renal bicarbonate losses<br />

11. In human studies and experimental animal models, osmotic demyelination has been associated with all of the<br />

following except:<br />

(a) Rapid correction of symptomatic hyponatremia (2 days duration at a rate >0.6 mEq/L/hour<br />

(c) Correction of hyponatremia >2 days duration at a rate >25 mEq over 48 hours<br />

(d) Correction of hyponatremia >24 hours duration at a rate >2.5 mEq/hour<br />

12. An 80-year-old woman is found unconscious in her Houston apartment, which is not air-conditioned, in August. Her<br />

serum sodium is 185 mEq/L. Blood pressure is 60 mmHg by palpation, and pulse 130 beats per minute. The most<br />

appropriate fluid regimen for her initial resuscitation is:<br />

(a) D5W at 500 ml/hour<br />

(b) D5/0.45% normal saline at 250 ml/hr<br />

(c) Normal saline or lactated Ringer's 1 L in the first hour<br />

(d) D5/0.33% normal saline at 500 ml/hr<br />

13. A 60-year-old man with a history of congestive heart failure presents to the emergency department complaining of<br />

pedal edema. His mental status is clear. Blood pressure is 120/80 mmHg, and pulse is 80 beats per minute. Lungs<br />

are clear, and neck veins distended. Serum sodium is 1 05 mEq/L. Of the fol lowing therapies, which is the most<br />

appropriate?<br />

(a) Infusion of 3% normal saline at 50 ml/hour and concomitant administration of furosemide<br />

(b) Infusion of normal saline at 200 ml/hour with concomitant administration of furosemide<br />

(c) Water restriction<br />

(d) IV administration of 40 mg of furosemide every hour until serum sodium is normal<br />

14. A 2-year-old child has been vomiting for 6 days. In the emergency department she is listless, with a pulse of 180<br />

beats per minute and capillary refill >4 seconds. Her weight is 10 kg. The serum sodium is 120 mEq/L. The most<br />

appropriate initial therapy is:<br />

(a)<br />

Infusion of 3% normal saline at 1 ml/kg/hour and concomitant administration of furosemide<br />

(b) Infusion of 3% normal saline at 1 ml/kg/hr<br />

(c) A rapid infusion of normal saline at 20 ml/kg<br />

(d) D5/0.45% normal saline at 125 ml/hr<br />

15. Of the following clinical scenarios, which patient has the most urgent need for rapid potassium replacement?<br />

(a) A 4-year-old with persistent vomiting, metabolic alkalosis, and a serum potassium of 3.0 mEq/L<br />

(b) A 56-year-old woman taking a diuretic and digoxin, with a serum potassium of 3.0 mEq/L<br />

(c) A 22-year-old insulin-dependent diabetic patient with an arterial pH of 6.9 and a serum potassium of 3.0 mEq/L<br />

(d) A 45-year-old man with delirium tremens and a serum potassium of 3.0 mEq/L<br />

16. A 60-year-old man presents with confusion, polyuria, and a serum calcium of 16 mg/dl. Of the fol lowing therapies,<br />

which is the least appropriate?<br />

.~ (a) Normal saline IV at 500 ml/hour<br />

(b) Salmon calcitonin 4 U/kg IM<br />

(c) Methylprednisolone sodium succinate 100 mg IV<br />

(d) Vitamin D 25,000 units IM<br />

593


ENDOCRINE, METABOLIC, AND NUTRITIONAL DISORDERS: SELF-ASSESSMENT QUESTIONS<br />

17. A healthy 17-year-old woman suffers a cardiac arrest during an infusion of magnesium sulfate for treatment of<br />

eclampsia. In addition to starting CPR, the most appropriate initial treatment is:<br />

(a)<br />

Immediate hemodialysis<br />

(b) IV infusion of 1 g calcium chloride<br />

(c)<br />

IV infusion of 140 mEq of sodium bicarbonate<br />

(d) IV infusion of 10 mEq of potassium chloride<br />

18. A SO-year-old alcoholic patient presents with obtundation. Blood pressure is 150/100 mm Hg, and pulse is 120 beats<br />

per minute. He is treated empirically with 25 g of DS0W and 100 mg of thiamine. The pH is 7.2, pCO 2<br />

= 15, Na+=<br />

140 mEq/L, Cl-= 102 mEq/L, HCQ 3<br />

- = 6 mEq/L, K+ = 4 mEq/L, and glucose= 250 mg/dL. Urine ketones are negative.<br />

Ethanol is not detectable. The osmolal gap is 100 mOsm/L. The most urgent interventions are:<br />

(a) Administration of 1 mEq/kg of sodium bicarbonate, blood cultures, and broad-spectrum antibiotics, followed<br />

by observation in the ICU<br />

(b) Initiation of an ethanol infusion (or administration of 4-methylpyrazole) and arrangements for hemodialysis<br />

(c) Immediate head scan and neurosurgical consultation<br />

(d) Intubation and gastric lavage, followed by administration of 60 g of activated charcoal with sorbitol<br />

19. The mental obtundation seen in association with nonketotic hyperosmolar coma is most closely associated with:<br />

(a)<br />

Serum bicarbonate level<br />

(b) Serum osmolality<br />

(c) pH of the plasma<br />

(d) pH of the cerebrospinal fluid<br />

20. The nitroprusside test for ketones most accurately assesses the presence of:<br />

(a)<br />

Acetone<br />

(b) Acetoacetate<br />

(c) ~-hydroxybutyrate<br />

(d) It assesses all of the above equally we/ I.<br />

21. Which of the fol lowing drugs is not classically associated with producing drug-induced hypoglycemia?<br />

(a)<br />

Alcohol<br />

(b) Aspirin<br />

(c)<br />

NSAIDs<br />

(d) Disopyramide<br />

22. Expected findings in a patient with purely alcoholic ketoacidosis include all of the following except:<br />

(a)<br />

High anion gap acidosis<br />

(b) Markedly increased (>300 mg!dl) serum glucose<br />

(c) A nitroprusside test that may be only weakly positive<br />

(d) Nondetectable or low blood alcohol level<br />

23. The mainstay of therapy for patients in alcoholic ketoacidosis is IV hydration with:<br />

(a)<br />

A saline solution (normal saline or 0.45% normal saline)<br />

(b) A glucose solution (DSW)<br />

(c) A solution containing both glucose and saline (OS/normal saline or D5/0.45% normal saline)<br />

(d) The type of solution used is irrelevant.<br />

24. The acidosis seen in association with alcoholic ketoacidosis is primarily attributable to the presence of:<br />

(a) ~-hydroxybutyrate<br />

(b) Acetoacetate<br />

(c) Acetone<br />

(d) Lactate<br />

25. All of the following ingestions are associated with a high anion gap acidosis except:<br />

(a)<br />

Ethylene glycol<br />

(b) Paraldehyde<br />

(c) Aspirin<br />

(d) lsopropyl alcohol<br />

594


ENDOCRINE, METABOLIC, AND NUTRITIONAL DISORDERS: SELF-ASSESSMENT QUESTIONS<br />

26. A patient presents with hypertension, tachycardia, a fever, and slight confusion. You suspect thyroid pathology.<br />

Which of the following drugs should be avoided?<br />

(a) Aspirin<br />

(b) Propranolol<br />

(c) Ibuprofen<br />

(d) Acetaminophen<br />

27. The role of propylthiouracil in the treatment of thyroid storm is most accurately described as that of:<br />

(a) Inhibiting conversion ofT 3<br />

to T 4<br />

(b) Retarding release of stored thyroid hormone<br />

(c)<br />

Blocking the synthesis of thyroid hormone<br />

(d) None of the above<br />

28. Al I of the following statements regarding hypothyroidism are accurate except:<br />

(a)<br />

Primary hypothyroidism is more common than secondary hypothyroidism.<br />

(b) Patients with secondary hypothyroidism have increased levels of thyroid-stimulating hormone.<br />

(c) Treatment of Grave disease is the most common cause of primary hypothyroidism in adults.<br />

(d) Drugs that have been associated with development of primary hypothyroidism include lithium and<br />

phenylbutazone.<br />

29. After abrupt cessation of steroid therapy, a patient presents with vomiting, hypotension, and generalized malaise. All<br />

of the following laboratory findings would be suspected except:<br />

(a)<br />

Hyponatremia<br />

(b) Hypoglycemia<br />

(c) Hyperkalemia<br />

(d) Hypocalcemia<br />

ANSWERS<br />

I. a<br />

2. a<br />

3. C<br />

7.<br />

8.<br />

9.<br />

b<br />

b<br />

b<br />

4. a 10. d<br />

13.<br />

14.<br />

15.<br />

16.<br />

5. a 11. a 17. b<br />

C<br />

C<br />

C<br />

d<br />

19. b<br />

20. b<br />

21. C<br />

22. b<br />

23. C<br />

6. c 12. c 18. b 24. a<br />

25. d<br />

26. a<br />

27. C<br />

28. b<br />

29. d<br />

Use the pre-chapter multiple choice question worksheet (page xvi) to record and determine the percentage of correct<br />

answers for this chapter.<br />

595


ENDOCRINE, METABOLIC, AND NUTRITIONAL DISORDERS<br />

ADULT FLUID VOLUMES<br />

I. NORMAL VALUES<br />

A. Extracellular fluid= 200 ml/kg<br />

B. Estimated blood volume= 70 ml/kg<br />

C. Plasma volume= 35.5 ml/kg (approximately half the blood volume)<br />

II. PATIENT ASSESSMENT<br />

A. Normal capillary refill (nail beds) ::;;2 seconds<br />

1. If delayed >2 seconds, an approximated volume deficit :C::15% is present.<br />

2. Capillary refill can also be delayed from poor pump function.<br />

B. Acute onset tachycardia from fluid loss<br />

1. > 100 beats per minute= 15%-30% volume deficit<br />

2. >120 beats per minute= 30%-40% volume deficit<br />

3. >140 beats per minute= >40% volume deficit<br />

4. These values may be decreased in elderly patients and in patients on ~-blockers or other medications that may<br />

impair the tachycardic response. For example, an elderly patient on ~-blockers may have a heart rate


ENDOCRINE, METABOLIC, AND NUTRITIONAL DISORDERS<br />

c. At equilibrium, the concentration of molecules in the fluid of one compartment is equal to that in all<br />

the other compartments. When free water is lost from one compartment, its concentration of molecules<br />

increases; this causes water to diffuse into that compartment until the concentrations equalize.<br />

d. Normal serum osmolality is 275-295 mOsm/L.<br />

(1) The following equation may be used to calculate serum osmolality:<br />

(2 x Na) +<br />

BUN<br />

2.8<br />

+<br />

Glucose<br />

18<br />

+<br />

Ethanol<br />

4.6<br />

The calculated value should be within 10 mOsm/L of the measured value. The difference between<br />

the measured and calculated values is the osmolal gap. A high osmolal gap increases suspicion of an<br />

unidentified substance dissolved in the blood.<br />

(2) Causes of abnormal osmolar states:<br />

(a)<br />

Hyperosmolality = t measured serum osmolality<br />

i. t serum Na+ (no gap)<br />

ii. Alcohol ingestion: either ethanol or "toxic alcohols" (large gap)<br />

• Most common cause of coma+ t serum osmolality<br />

• t serum osmolality + t osmolal gap+ anion gap acidosis -<br />

• lsopropyl alcohol causes an osmolal gap but no acidosis.<br />

iii. Hyperosmolar hyperglycemic nonketotic coma (no gap)<br />

iv. Ketoacidosis (small gap)<br />

v. Uremia (small gap)<br />

(b) Hypo-osmolality - .j, serum Na+= .j, measured serum osmolality<br />

methanol or ethylene glycol<br />

(c) Remember that methanol or ethylene glycol causes anion gap acidosis and osmolal gap. lsopropyl<br />

alcohol does not cause an anion gap acidosis.<br />

3. Clinical pathophysiology<br />

a. The average adult requires about 2,000-3,000 ml of water per day (depending on losses).<br />

b. Water loss<br />

C. Sodium regulation<br />

1. Facts<br />

(1) The volume of water loss per day can be divided into two categories:<br />

(a) Insensible losses<br />

i. Respiratory tract (-600 ml)<br />

ii. Skin (-300 ml)<br />

iii. Feces(-100ml)<br />

(b) Urinary losses (-1,500 ml)<br />

i. Normal dietary intake produces -300 ml of solute/day.<br />

ii. A urine output of at least 0.5 ml/kg/hr indicates an adequate level of hydration and renal<br />

perfusion in adults with normal renal function.<br />

(2) Clinical manifestations of volume loss range from mild postural lightheadedness to resting tachycardia<br />

to a nondetectable blood pressure.<br />

(3) Flexibility in the body's handling of water intake is provided by two mechanisms.<br />

(a) Antidiuretic hormone (ADH)<br />

i. Increased body water - decreased serum osmolality - suppression of ADH secretion -<br />

diuresis of free water<br />

11. Decreased free water - increased serum osmolality - stimulation of ADH release -<br />

retention of free water<br />

(b) Aldosterone<br />

1. Released by the adrenal gland in response to renin release by the kidney in low-volume states<br />

ii. Stimulates retention of sodium by the renal tubules, which is followed by retention of water.<br />

a. Total body sodium= -40-50 mEq/kg.<br />

b. Distribution of major cations and anions<br />

597


ENDOCRINE, METABOLIC, AND NUTRITIONAL DISORDERS<br />

2. Concepts<br />

(1) Na+ is the major extracellular cation (140 mEq/L)<br />

(2) Cl- and HCO 3<br />

are the major extracellular anions.<br />

(3) K+ and Mg++ are the major intracellular cations.<br />

a. When physiologic changes cause intracellular-extracellular osmotic dysequilibrium, corrections are not<br />

made by movement of solute across the cell membrane but, rather, by movement of water.<br />

b. Disorders of sodium regulation actually represent a disorder of water regulation in which either too little or<br />

too much water is present in the body relative to solute.<br />

c. What this means is that hyponatremia and hypernatremia can occur with normal, low, or high total body<br />

water content.<br />

D. Potassium regulation<br />

1. The normal serum concentration is 3.5-4.5 mEq/L.<br />

a. Although the serum potassium is only 2% of total body potassium, in most settings it is a good indicator of<br />

total body stores.<br />

b. Potassium is the primary intracellular cation.<br />

c. An increased potassium concentration is a potent stimulus for aldosterone release.<br />

2. Physiologic actions of potassium<br />

a. Maintains resting membrane potential<br />

b. Changes in the intracellular/extracellular gradient facilitate propagation of electrical impulses.<br />

c. Filters freely through the glomerulus, is reabsorbed in the proximal and ascending tubules, and is then<br />

secreted into the distal tubule in exchange for sodium<br />

E. Calcium regulation<br />

1. Calcium is the most abundant cation in the body, but 99% is bound in bone; the rest is in the extracellular<br />

fluid, half of which is bound to plasma proteins and half as free (active) ions. The calcium gradient between<br />

extracellular and intracellular spaces is highly regulated at 10,000:1.<br />

2. Total plasma calcium levels (8.5-10.5 mg/dl) are maintained by the function of vitamin D and parathyroid<br />

hormone.<br />

a. Protein-bound = 4.0-4.5 mg/dL<br />

b. Ionized = 4.2-4.8 mg/dL<br />

c. A laboratory value reported in mEq/L =½the amount in mgldl, eg, 4.2 mgldl = 2.1 mEq/L= 1.05 mOsm/L<br />

3. Physiologic actions<br />

a. Changes in H+ concentration result in changes of ionized calcium because Ca++ binds to protein in place of<br />

H+. Decreased H+, such as in hyperventilation - respiratory alkalosis - t protein-bound calcium+<br />

t ionized serum Ca++ (relative hypocalcemia)<br />

b. The ionized fraction of calcium mediates physiologic effects. Clinical effects of hypocalcemia are seen<br />

when the ionized levels fall below 3 mg/dL. Clinical manifestations of hypercalcemia are noted at total<br />

serum calcium levels> 11 mg/dl.<br />

(1) Serum albumin level (a decrease of 1 gldl in albumin results in a decrease of 0.8 mg/dl in calcium) -<br />

no change in ionized fraction<br />

(2) Alkalosis (decreases the ionized fraction) - no change in total serum calcium<br />

(3) Acidosis (increases the ionized fraction) - no change in total serum calcium<br />

F. Magnesium regulation<br />

1. Magnesium is the second most abundant intracellular cation, and its regulation is closely related to calcium<br />

and phosphate.<br />

2. Total body content is 24 g (2,000 mEq), 60% of which is in bone; most of the remaining 40% is intracellular.<br />

Magnesium is also an essential element in the production of cellular energy, cardiac excitability, the clotting<br />

mechanism, and in neuromuscular activity.<br />

3. Dietary source is green vegetables; the usual daily requirement is ~25 mEq/day; normal serum level is<br />

1.4-2.2 mEq/L.<br />

G. Chloride regulation<br />

1. Chloride is an extracellular anion that increases or decreases in concentration whenever changes occur in the<br />

concentrations of other anions, eg, metabolic acidosis (due to bicarbonate loss) results in an increase in chloride.<br />

2. Functions of chloride<br />

598


ENDOCRINE, METABOLIC, AND NUTRITIONAL DISORDERS<br />

a. Acid-base balance<br />

b. Osmotic balance<br />

c. Water balance<br />

3. A change in serum chloride is seldom a primary problem.<br />

H. Hydrogen-potassium exchange to maintain electrolyte balance<br />

1. A hydrogen-potassium exchange may also occur. The classic teaching is that when metabolic acidosis is<br />

caused by organic acids that do not freely cross the cell membrane (eg, ketoacids), H+ moves inside the cell<br />

to be buffered and is exchanged for K+. However, while hyperkalemia is commonly associated with diabetic<br />

ketoacidosis, it is not commonly seen in alcoholic ketoacidosis, suggesting that insulin may have a significant<br />

effect on potassium regulation. Increases in serum K+ do not occur when the metabolic acidosis is caused by<br />

other organic ions (such as lactate). This concept is important to help remember key clinical pearls for patient<br />

management.<br />

I. The anion gap<br />

1. The anion gap is the difference between serum anions and cations. In a physiologically normal person,<br />

measured serum anions are Cl- and HCO 3<br />

and unmeasured anions are the negative charges on serum albumin,<br />

phosphate, and sulfate. Measured serum cations are Na+, K+, Mg++, and Ca++. In the clinical setting, the anion<br />

gap is calculated from the predominant measured cation (Na+) and anions (Cl- and HCO -). 3<br />

Anion gap = Na+ - (Cl- and HCO 3<br />

-)<br />

Normal range= 8-16 mEq (A good rule of thumb is that both osmolal gap and anion gap are around 10.)<br />

2. A significantly increased anion gap signifies the presence of excessive unmeasured anions (usually due to<br />

metabolic acidosis). A small or negative anion gap is seen when there is:<br />

a. An excess of unmeasured serum cations or<br />

b. An increase in unmeasured positively charged proteins or<br />

c. A decrease in serum albumin or<br />

d. An increase in negatively charged particles that are mistaken for chloride (such as bromide or iodide)<br />

e. Decreased anion gap: lithium toxicity, myeloma patients<br />

3. Causes of anion gap acidosis (MUDPILES)<br />

a. Methanol<br />

b. 1/.remia<br />

c. Qiabetic ketoacidosis<br />

d. faraldehyde<br />

e. lron, isoniazid, inhalants, inborn errors of metabolism<br />

f. lactic acidosis<br />

g. S.alicylates, solvents, starvation<br />

4. If an abnormally high anion gap occurs in the presence of an abnormally high osmolal gap (> 10 mOsm<br />

difference between the measured and calculated serum osmolarity), ingestion of methanol or ethylene glycol<br />

should be suspected.<br />

5. If an abnormally high osmolal gap is seen in the presence of a normal anion gap, suspect the presence of one<br />

of the following:<br />

a. Ethanol<br />

b. lsopropyl alcohol<br />

c. Glycerol<br />

d. Sorbitol<br />

e. Mannitol<br />

f. Acetone<br />

J. Pathophysiology of acid-base disorders<br />

1. The body normally maintains a critical acid-base balance primarily by handling three types of acids:<br />

a. Fixed acids (W), which are normally produced from dietary intake (sulfates and phosphates)<br />

b. Acids produced from pathologic processes and abnormal metabolic pathways<br />

c. Exogenous acids (toxins, medications, iatrogenic administration)<br />

2. Approximately 15,000 mg of volatile acid (CO 2<br />

) is excreted from the lungs each day.<br />

3. Approximately 70 mEq/L of nonvolatile acids are produced each day.<br />

599


ENDOCRINE, METABOLIC, AND NUTRITIONAL DISORDERS<br />

4. There are three mechanisms for renal excretion of these acids:<br />

a. Direct hydrogen excretion (~0.1 mEq/day)<br />

b. Excretion of urinary buffers (~20 mEq/day)<br />

c. Excretion with ammonia (~50 mEq/day)<br />

5. Hydrogen ion activity is clinically reported as pH. At normal body temperature, a system of buffers maintains<br />

the pH of blood at 7.4 (40 nMH+ concentration).<br />

[W] Halves for each 0.3 unit ~<br />

rise in pH<br />

t ;<br />

[H+] Doubles for each 0.3<br />

unit fall in pH<br />

t<br />

Alkalinization 36 44 Acidification<br />

(alkalemia)<br />

(acidemia)<br />

10 16 20 26 32 50 63 80 100<br />

125<br />

160<br />

pH units<br />

8.0 7.8 7.7 7.6 7.5 7.3 7.2 7.1 7.0<br />

6.9<br />

6.8<br />

7.44 - 7.36<br />

Limited range of linearity between pH and [H+]<br />

(Li 0.1 pH unit= Li [H+] of 10 nM)<br />

Relationship of pH to H+ Concentration<br />

6. Minute-to-minute regulation of hydrogen ion concentration in the blood and interstitium is maintained<br />

primarily by the bicarbonatecarbonic acid system:<br />

~ ~<br />

To maintain this equilibrium, reciprocal changes occur when a component of this equation is "out of balance."<br />

t HCO 3<br />

- """ t H+<br />

t pCO 2<br />

_,.<br />

tW<br />

t pCO 2<br />

_,.<br />

tH+<br />

;<br />

H+ = 24 x<br />

pCO 2<br />

HCO 3<br />

-<br />

7. When the balance between bicarbonate and carbonic acid is disrupted, compensatory mechanisms are<br />

stimulated. Metabolic compensation occurs in response to respiratory abnormalities, and respiratory<br />

compensation occurs in response to metabolic abnormalities. The following parameters can be used to predict<br />

the expected compensatory response with acute changes in the pCO 2<br />

:<br />

pH ! by 0.08 units for each 10 mmHg t in CO 2<br />

pH t by 0.08 units for each 10 mm Hg ! in CO 2<br />

a. Metabolic compensation for respiratory acidosis is facilitated by increased retention of bicarbonate by the<br />

kidney. In respiratory alkalosis, the kidney secretes bicarbonate.<br />

b. Respiratory compensation for metabolic alkalosis is limited by the development of hypoxemia that occurs<br />

during hypoventilation.<br />

c. Respiratory compensation for metabolic acidosis is rapid. Experimental observation of respiratory<br />

compensation for uncomplicated metabolic acidosis has determined the following:<br />

(1) The pH is never compensated to a normal level (>7.35).<br />

(2) The pCO 2<br />

almost never goes lower than 10.<br />

(3) The pCO 2<br />

approximates 1.5 x bicarb+ 8 ± 2.<br />

600


ENDOCRINE, METABOLIC, AND NUTRITIONAL DISORDERS<br />

II. ELECTROLYTE DISORDERS<br />

A. Hyponatremia<br />

1. Definition: serum Na+ 25 mEq/48<br />

hours in cases of hyponatremia that has been present >2 days.<br />

(b) Avoid an increase in sodium of >9 mEq/L in 24 hours.<br />

(c) The sodium deficit can be calculated, but generally a change of 4-6 mEq/L within the first few<br />

hours will be safe.<br />

(d) There is no evidence that correction of very acute (


ENDOCRINE, METABOLIC, AND NUTRITIONAL DISORDERS<br />

(2) Is the cause of hyponatremia evident on the basis of history and physical examination? If not,<br />

measurement of the serum osmolality will be very helpful.<br />

Electrolyte<br />

Normal<br />

nonelectrolyte<br />

Abnormal<br />

nonelectrolyte<br />

Serum<br />

osmolality<br />

2[Na+] mEq/L +<br />

I<br />

Effective osmoles<br />

(tonicity)<br />

mg!dL<br />

Ethanol<br />

lsopropanol<br />

/<br />

Ethylene glycol<br />

[glucose] "' BUN<br />

+ -- Methanol<br />

18 2.8 Mannitol<br />

I<br />

Serum Osmolality Calculated and Measured<br />

Normal:<br />

Osmolal gap:<br />

calculated Osm - 10 mOsm/L = measured Osm (Na+, glucose, BUN)<br />

calculated Osm < measured Osm (nonelectrolyte solute other than glucose or urea,<br />

or pseudohyponatrem ia)<br />

(3) Categorize the hyponatremic patient into one of two groups, which will guide therapy (normal<br />

or high osmolality, low osmolality). Low osmolality is true hyponatremia and requires treatment.<br />

Pseudohyponatremia does not require treatment.<br />

(a)<br />

Pseudohyponatremia<br />

i. If the measured osmolality is normal and the calculated osmolality is low:<br />

• Cause: low water volume with high solute volume; the sodium concentration is normal in<br />

the aqueous phase.<br />

• Differential diagnosis: multiple myeloma, hyperlipidemia<br />

• Treatment: none<br />

ii. If the measured osmolality is high and the calculated osmolality is normal to high:<br />

• Cause: free water moves from the intracellular space to the high solute concentration in the<br />

extracellular space.<br />

• Differential diagnosis: hyperglycemia, mannitol, glycerol excess<br />

• Increased glucose is the most common cause of pseudohyponatremia in clinical practice.<br />

• Treatment<br />

0 Hyperglycemia_,. insulin and saline<br />

o Mannitol _,. saline<br />

(b) Low osmolality (most common)<br />

I.<br />

[<br />

measured osmolality = low<br />

calculated osmolality = low<br />

> both equal<br />

J<br />

ii. The causes are classified and the treatments are based on evaluation of two parameters:<br />

• Volume status of the patient (hypovolemic, euvolemic, hypervolemic)<br />

• Measurement of urine sodium<br />

iii. Hypovolemic hypo-osmolar hyponatremia<br />

• Contracted effective intravascular volume_,. kidney avidly reabsorbs water_,. hyponatremia<br />

• Clinical examination reflects the presence of hypovolemia: postural hypotension, poor skin<br />

turgor, flat neck veins<br />

• Measurement of urinary sodium can help determine if the volume depletion is due to a<br />

renal or extrarenal source.<br />

602


ENDOCRINE, METABOLIC, AND NUTRITIONAL DISORDERS<br />

B. Hypernatremia<br />

o Renal sodium loss (urine Na+ >20 mEq/L)<br />

-Diuretics<br />

-Mineralocorticoid deficiency<br />

-Salt-wasting nephropathy<br />

-Osmotic diuresis (hyperglycemia, mannitol)<br />

0 Extrarenal sodium loss (urine Na+ 20 mEq/L).<br />

iv. The diagnosis of SIADH is one of exclusion.<br />

• Typical laboratory data: t BUN, t urine osmolality (> 150 mOsm), t urine Na+ (>20 mEq/L)<br />

• Treatment is based on the severity of the disorder. For acute, severe, symptomatic<br />

hyponatremia, hypertonic saline or furosemide plus saline may be indicated for rapid<br />

correction. For less severe cases, the restriction of free water is appropriate therapy.<br />

(d) Hypervolemic hypo-osmolar hyponatremia (most common form)<br />

1. Volume status: clinically edematous (the extracellular volume is expanded but "ineffective";<br />

the kidney perceives a low intravascular [extracellular] volume and resorbs excess salt and<br />

water; as a result, there is an excess of total body sodium and a greater excess of total body<br />

water)<br />

11. Measurement of urinary sodium<br />

• >20 mEq/L (renal): acute or chronic renal failure<br />

• 145 mEq/L (> 155 mEq/L = severe)<br />

2. Classic clinical scenario<br />

a. Clinical presentation: t level of consciousness, dehydrated, ± seizures<br />

b. There are no classic ECG findings.<br />

c. Etiology: sodium and water loss<br />

(1) t water loss (hyperpnea, excessive sweating, vomiting)<br />

(2) t diuresis (diabetes insipidus, acute tubular necrosis, diuretics, and postobstructive diuresis)<br />

d. Hypernatremic states can be categorized into one of two groups:<br />

(1) Water loss (t intake or t output)<br />

(a) Patients who have a low or normal total body sodium<br />

1. Reduced water intake<br />

ii. Abnormal thirst mechanism<br />

603


ENDOCRINE, METABOLIC, AND NUTRITIONAL DISORDERS<br />

iii. Inability to drink (physical, neurologic)<br />

iv. Osmotic diuresis<br />

• Diabetic ketoacidosis<br />

• Hyperosmolar nonketotic coma<br />

(b) Patients who have abnormalities in production of, or renal response to, ADH<br />

i. Diabetes insipidus (central, nephrogenic)<br />

ii. Drugs (lithium, aminoglycosides, phenytoin, fluoride)<br />

(2) Sodium gain (iatrogenic)-these patients have increased total body sodium.<br />

e. Management<br />

(a) Saline administration (including hypertonic saline)<br />

(b) Hypertonic dialysis<br />

(c) Hypertonic feedings<br />

(d) "Bicarb" administration<br />

(1) Guided by the rate of development of the hypernatremia, the presence of symptoms, and the absolute<br />

sodium level<br />

(2) Calculate the water deficit. (As a general rule of thumb, each liter of water deficit will cause an<br />

increase in serum sodium of 3-5 mEq/L.)<br />

water deficit (L) =<br />

(<br />

measured [Na+]<br />

normal [Na+]<br />

) -1<br />

C. Dehydration<br />

(3) The cornerstone of treatment is volume repletion, usually with 0.9% saline.<br />

(4) In patients with increased total body sodium due to sodium gain, it may be necessary to use a loop<br />

diuretic in conjunction with D5W. In severely ill patients, hemodialysis may be lifesaving.<br />

(5) Patients with prolonged hypernatremia have normal intracellular volume due to the acquisition of<br />

"idiogenic osmoles" that equalize intracellular and extracellular osmolarities. The rapid correction of<br />

hypernatremia may lead to excessive movement of water into these hyperosmolar cells, resulting in<br />

cerebral edema. Therefore, sodium levels should not be lowered faster than 10-15 mEq/L/day; only<br />

half of the water deficit should be corrected in the first 24 hours; in most clinical situations, the total<br />

deficit will require replacement over the next 24-48 hours so that the serum osmolality drops about 2<br />

mOsm/kg/hr.<br />

1. Three categories: hypotonic, isotonic, and hypertonic (Following is a summary of the information presented<br />

above on hypo- and-hypernatremia, which is important to understand the cause and type of dehydration in a<br />

particular patient.)<br />

2. Hypotonic dehydration<br />

a. Loss of sodium in excess of water<br />

b. Serum Na+ is low.<br />

c. Differential diagnoses<br />

(1) Diuretic excess<br />

(2) "Fluid space" losses<br />

(3) Vomiting<br />

(4) Adrenocortical insufficiency<br />

3. Isotonic dehydration<br />

a. Loss of sodium loss of water<br />

b. Serum Na+ is normal.<br />

c. Most commonly associated clinical condition is vomiting.<br />

4. Hypertonic dehydration<br />

a. Loss of water in excess of sodium<br />

b. Serum Na+ is increased.<br />

c. Differential diagnoses<br />

(1) Diarrhea<br />

(2) Lack of access to water<br />

604


ENDOCRINE, METABOLIC, AND NUTRITIONAL DISORDERS<br />

D. Hypokalemia<br />

(3) Nonketotic hyperosmolar coma<br />

(4) Diabetes insipidus<br />

1. Definition: serum K+


ENDOCRINE, METABOLIC, AND NUTRITIONAL DISORDERS<br />

E. Hyperkalemia<br />

1. Definition: serum K+ > 4.5 mEq/L (>6.5 mEq/L = severe)<br />

2. Pathophysiology<br />

a. Acidosis: renal failure, diabetic ketoacidosis<br />

b. Cell lysis: chemotherapy, hemolysis, burns, rhabdomyolysis<br />

c. Mineralocorticoids (Addison disease)<br />

d. Medications (triamterene, spironolactone, ACE inhibitors, NSAIDs, f3-blockers)<br />

3. Clinical presentation<br />

a. Signs and symptoms are multiple and often difficult to discern from those of the primary illness or condition<br />

that precipitated the hyperkalemia.<br />

b. Neuromuscular findings may begin with lethargy and weakness and progress to paralysis and areflexia.<br />

c. Cardiac findings<br />

4. Etiology<br />

(1) Hypotension<br />

(2) Dysrhythmias<br />

(3) ECG changes progress as severity of hyperkalemia progresses: tall peaked T waves ___,, wide QRS<br />

complexes ___,, sine waves; however, ECG findings may be minimal, especially in renal patients.<br />

a. Factitious<br />

(1) Thrombocytosis<br />

(2) Leukocytosis<br />

(3) Prolonged tourniquet time<br />

(4) In-vitro hemolysis<br />

b. Decreased renal excretion<br />

(1) Renal insufficiency<br />

(2) Adrenal or aldosterone insufficiency<br />

(3) Drugs (eg, potassium-sparing diuretics, ACE inhibitors)<br />

(4) Type IV renal tubular acidosis (diabetes)<br />

c. Increased potassium load<br />

(1) Cellular breakdown (trauma, tumor lysis, rhabdomyolysis)<br />

(2) Potassium-containing salt substitutes<br />

(3) Hemolysis<br />

(4) GI bleeding<br />

(5) High-dose penicillin VK<br />

d. Decreased cellular uptake of potassium<br />

(1) Diabetic ketoacidosis (serum K+ level increases 0.6 mEq/L for every 0.1 decrease in pH)<br />

(2) Drugs (f3-blocker or digoxin overdose, succinylcholine)<br />

5. Management<br />

a. If there are no ECG signs, treatment can be restricted to methods that increase potassium excretion through<br />

the bowel or kidneys or decrease potassium intake (diet).<br />

b. Because the kidney is the major determinant of the serum potassium concentration, emergent treatment of<br />

hyperkalemia requires a method that either blocks the site of action of the excess potassium or shifts the<br />

excess potassium from one compartment to another. Hyperkalemia due to familial periodic paralysis is<br />

caused by potassium shifts (not total body depletion); therefore, overcorrection may occur in this setting.<br />

(1) Calcium chloride or gluconate antagonizes the effects of potassium in the myocardium, thus<br />

decreasing membrane irritability, which reduces the risk of developing a ventricular dysrhythmia.<br />

(2) Glucose and insulin, bicarbonate, and f3-adrenergic agonists redistribute excess potassium from the<br />

extracellular to the intracellular compartment.<br />

(3) Therapy with dialysis or an exchange resin (polystyrene sulfonate) should be started shortly after using<br />

any emergent treatment.<br />

606


ENDOCRINE, METABOLIC, AND NUTRITIONAL DISORDERS<br />

Table 37: Treatments for Hyperkalemia<br />

Treatment<br />

Mechanism<br />

Dosage<br />

Onset<br />

Duration<br />

of Effect<br />

Calcium gluconate<br />

(10%)<br />

Membrane<br />

stabilization<br />

10-20 ml IV<br />

1-3 min<br />

20-50 min<br />

Sodium bicarbonate<br />

Redistribution of K+<br />

1 mEq/kg IV<br />

5-10 min<br />

1-2 hours<br />

Albuterol<br />

Redistribution of K+<br />

10-20 mg by inhaler<br />

30 min<br />

2-4 hours<br />

Insulin plus glucose<br />

Redistribution<br />

5-10 units of regular<br />

insulin with 1-2<br />

ampules 050 IV<br />

30 min<br />

4-6 hours<br />

Cation exchange resin<br />

(sodium polystyrene)<br />

Excretion<br />

25-50 g orally or per<br />

rectum with sorbitol<br />

1-2 hours<br />

4-6 hours<br />

Peritoneal or<br />

hemodialysis<br />

Excretion<br />

Within minutes<br />

after starting<br />

Until dialysis<br />

is completed<br />

Diuretics<br />

Excretion<br />

Furosemide<br />

40 mg IV<br />

With start of<br />

diuresis<br />

Until diuresis ends<br />

F. Hypocalcemia<br />

1. Definition: serum Ca++


ENDOCRINE, METABOLIC, AND NUTRITIONAL DISORDERS<br />

f. Alkalosis (0.1 t in pH, 0.05-0.08 mEq/L 1 in Ca++)<br />

(1) Respiratory_,. hyperventilation<br />

(2) Metabolic_,. vomiting, diarrhea, malabsorption<br />

g. Rapid and massive blood transfusions<br />

h. Rhabdomyolysis (increased calcium binding to injured tissue)<br />

i. Sepsis and shock states<br />

j. Renal failure<br />

5. Management<br />

a. The ultimate goal of therapy is treatment of the underlying cause.<br />

b. Emergent treatment with IV replacement is indicated if the patient is acutely symptomatic: 10 ml CaC1 2<br />

(10% solution) IV push over 15-20 minutes (repeat as needed).<br />

c. Transfusion of citrated blood may result in hypocalcemia (which is transient) and does not require routine<br />

treatment unless signs of hyperreflexia develop.<br />

G. Hypercalcemia<br />

1. Definition: serum Ca++> 10.5 mg/dl (> 12 mg/dl severe)<br />

2. Clinical presentation: "groans, stones, psych overtones"<br />

a. Neuromuscular<br />

b.<br />

C.<br />

d.<br />

(1) Weakness, hyporeflexia, hypotonia<br />

(2) Apathy, confusion, depression<br />

(3) Lethargy, stupor, coma (if the hypercalcemia is the cause of the coma, serum Ca++ is> 13.5 mg)<br />

Cardiovascu Jar<br />

(1) Hypertension<br />

(2) Dysrhyth mi as<br />

(3) ECG abnormalities<br />

(a) Short QT interval<br />

(b) Coving of the ST segment and T wave<br />

(c) Widening of the T wave<br />

(4) Digitalis sensitivity (hypercalcemia enhances digitalis toxicity)<br />

GI<br />

(1) Anorexia<br />

(2) Nausea, vomiting<br />

(3) Abdominal pain<br />

(4) Constipation<br />

Renal<br />

3. Etiology<br />

(1) Nephrolithiasis<br />

(2) Renal failure<br />

(3) Polyuria<br />

a. Malignancy<br />

(1) Hematologic (Hodgkin, lymphomas, multiple myeloma)<br />

(2) Metastasis to bone (thyroid, lung, breast, kidney); a serum Ca++> 13.5 mg/dl suggests this diagnosis.<br />

(3) Parathyroid hormone-producing lung/kidney cancers<br />

b. Excess intake<br />

(1) Milk-alkali syndrome<br />

(2) Vitamin D (or vitamin A) toxicity<br />

c. Acute osteoporosis<br />

(1) Immobilization of young patients<br />

(2) Paget disease (especially with immobilization)<br />

d. Hyperparathyroidism (a serum Ca++> 11 mg/dl suggests this diagnosis)<br />

e. Drugs<br />

(1) Lithium<br />

(2) Thiazide diuretics<br />

608


ENDOCRINE, METABOLIC, AND NUTRITIONAL DISORDERS<br />

4. Management<br />

a. Initiate for symptomatic patients or for Ca++> 14 mg/dl regardless of symptoms<br />

b. Saline hydration<br />

c. Furosemide 1 mg/kg as needed for fluid overload (thiazide diuretics contraindicated in patients with<br />

hypercal cem i a)<br />

d. Potassium replacement as needed<br />

e. Hydrocortisone<br />

f. Drugs that decrease bone absorption<br />

(1) Calcitonin<br />

(2) Glucocorticoids<br />

(3) B isphosphonates<br />

(4) Gallium nitrate<br />

g. Treatment of underlying neoplasm if present<br />

H. Hypomagnesemia<br />

1. Definition: serum Mg++


ENDOCRINE, METABOLIC, AND NUTRITIONAL DISORDERS<br />

e. Drugs<br />

(1) Aminoglycosides<br />

(2) Amphotericin B<br />

(3) Diuretics<br />

(4) ~-agonists<br />

(5) Cyclosporine<br />

4. Management<br />

a. Check for hypocalcemia, hypokalemia.<br />

b. Treat the underlying cause.<br />

c. Replacement therapy<br />

I. Hypermagnesemia<br />

(1) For life-threatening problems (dysrhythmias, seizures): 1-2 g 10% MgSO 4<br />

IV in 1-5 minutes, fol lowed<br />

by an infusion of 1-2 g/hr.<br />

(2) For less urgent presentations: 1-2 g 10% MgSO 4<br />

IV over 3 hours or 2 g of a 50% solution IM every<br />

4 hours x 5 doses<br />

1. Definition: serum Mg++ >2 .2 mEq/L (>3 mEq/L = severe)<br />

2. Clinical presentation (usually does not occur until the serum level is >3 mEq/L)<br />

a. Neuromuscular<br />

(1) Weakness<br />

(2) Drowsiness/lethargy<br />

(3) Slurred speech<br />

(4) Hyporeflexia<br />

(5) Coma and respiratory failure (serum Mg++ >5 mEq/L)<br />

b. Cardiovascular<br />

(1) Bradycardia - AV block - asystole<br />

(2) Prolonged PR and QT intervals with extreme elevations of ST segments and T waves<br />

(3) Vasodilation and hypotension (serum Mg++> 10-12 mEq/L)<br />

c. Because magnesium is used to treat some clinical situations (eg, preterm labor), recognizing these findings<br />

is important in case of iatrogenic hypermagnesemia.<br />

3. Etiology<br />

a. Renal failure (particularly in patients who were given magnesiumcontaining drugs)<br />

b. Iatrogenic: preeclamptic and eclamptic patients who were treated with MgSO 4<br />

to reduce blood pressure<br />

and/or control seizure activity<br />

c. Untreated diabetic ketoacidosis<br />

d. Adrenal insufficiency<br />

e. Rhabdomyolysis<br />

4. Management<br />

a. Discontinue exogenous magnesium<br />

b. Diuretics<br />

c. Calcium<br />

d. Dialysis<br />

J. Hypochloremia<br />

1. Definition: serum Cl-


ENDOCRINE, METABOLIC, AND NUTRITIONAL DISORDERS<br />

3. Management<br />

a. Treat underlying disorder<br />

b. Chloride replacement with NaCl is indicated if hypochloremia is severe or associated with hypokalemic<br />

alkalosis.<br />

K. Hyperchloremia<br />

1. Definition: serum Cl- >110 mEq/L (>120 mEq/L = severe)<br />

2. Etiology<br />

a. Dehydration<br />

b. Cardiac decompensation<br />

c. Bicarbonate loss (GI, renal)<br />

3. Management<br />

a. For GI bicarbonate loss - normal saline<br />

b. For renal bicarbonate loss - oral bicarbonate and renal consultation<br />

L. Key concepts in electrolyte disorders<br />

1. Potassium and pH are intrinsically linked in the body. As pH decreases, K+ increases. As pH increases, K+<br />

decreases.<br />

2. Many disorders mimic each other.<br />

a. Hypomagnesemia and hypocalcemia both result in neuromuscular excitability and can both present with<br />

Chvostek signs. Both can cause prolonged QT interval. Hypomagnesemia classically causes torsades.<br />

b. Hypermagnesemia and hypercalcemia both cause neuromuscular depression. If mild, both are treated<br />

with IV fluids.<br />

3. Severe hyperkalemia and hypermagnesemia are both treated with IV calcium.<br />

Ill. ACID-BASE DISORDERS<br />

A. General approach to the patient with an acid-base abnormality<br />

1. Basic definitions<br />

a. alkalosis: a process leading to alkalemia<br />

b. acidosis: a process leading to acidemia<br />

c. alkalemia: pH >7.45<br />

d. acidemia: pH


ENDOCRINE, METABOLIC, AND NUTRITIONAL DISORDERS<br />

(2) Respiratory acidosis (increased pC0 2<br />

)<br />

(a) Toxic: sedative hypnotics, opioids<br />

(b) Nontoxic: other causes of respiratory failure<br />

(3) Metabolic alkalosis (increased bicarbonate): volume depletion (any cause), Barter syndrome,<br />

hyper a I dosteron ism<br />

(4) Anion-gap metabolic acidosis (decreased bicarbonate)<br />

(a) Toxic: salicylates, phenformin, carbon monoxide, cyanide, isoniazid (seizures), iron, methanol,<br />

ethylene glycol<br />

(b) Nontoxic: seizures, shock, hypoxia, sepsis, ketoacidosis, uremia<br />

(5) Nonanion gap metabolic acidosis (decreased bicarbonate, increased chloride)<br />

(a) Toxic: chronic toluene exposure<br />

(b) Nontoxic: bicarbonate wasting conditions such as renal tubular acidosis or diarrhea, ketone-wasting<br />

(6) Mixed disorder<br />

(7) Remember: "Normal is not normal when it should be abnormal." For example, a person breathing 40<br />

times a minute should not have a pC0 2<br />

of 40. Even though this is a "normal" value, this is a sign of<br />

impending respiratory failure.<br />

c. If an anion gap is present, pinpoint the problem (see metabolic acidosis, page 613).<br />

(1) Consider causes (measurement of serum ketones, lactate, or glucose).<br />

(2) Consider an ethylene glycol, methanol, or salicylate level.<br />

(3) Consider causes of an anion gap other than metabolic acidosis.<br />

(a) Severe metabolic alkalosis<br />

(b) Anion infusion<br />

(c) Antibiotics<br />

(4) Causes of a decreased anion gap<br />

(a)<br />

Hypoalbuminemia<br />

(b) Bromide or iodide poisoning<br />

(c) Multiple myeloma<br />

(5) Measure the osmol gap (the difference between the measured and calculated osmolality).<br />

(a) The presence of an osmol gap suggests the laboratory is measuring an osmotically active<br />

substance that you are not using in your calculation.<br />

(b) Common causes of an osmol gap<br />

i. Ethanol<br />

ii. Methanol<br />

iii. Ethylene glycol<br />

iv. lsopropano 1<br />

v. Acetone<br />

vi. Glycerol<br />

vii. Mannitol<br />

viii.Uremia<br />

ix. Ketoacidosis<br />

(c) Of the alcohols, ethanol and isopropanol have an osmolal gap but a normal anion gap.<br />

(d) In uremia and ketoacidosis, the osmolal gap is usually 7.<br />

e. Questions to answer to determine if a mixed disorder is present:<br />

(1) Do the clinical rules for "pure metabolic acidosis" apply? Deviation implies the presence of a mixed<br />

disorder.<br />

(a) Respiratory compensation should not raise the pH to normal (>7.35).<br />

(b) The pC0 2<br />

approximates 1 .5 x bicarbonate+ 8 ± 2<br />

(c) For a pure anion gap acidosis, the increase in the anion gap should equal the decrease in the<br />

bicarbonate.<br />

612


ENDOCRINE, METABOLIC, AND NUTRITIONAL DISORDERS<br />

B. Metabolic acidosis<br />

(2) Is the degree of respiratory compensation for metabolic acidosis-<br />

(a) Too much (pH 2: normal)= superimposed respiratory alkalosis; common clinical settings include:<br />

i. Salicylate poisoning<br />

ii. Sepsis<br />

iii. Early cyanide poisoning<br />

iv. Increased intracranial pressure with shock or seizure<br />

v. Normal renal compensation for primary respiratory alkalosis<br />

(b) Too little (change in pC0 2<br />

< 1.5 x bicarbonate+ 8) = superimposed respiratory acidosis; common<br />

clinical settings include:<br />

i. Sedative-hypnotic overdose with shock or seizures<br />

ii. Patient with primary ventilatory impairment and metabolic acidosis<br />

iii. Partial renal compensation for primary respiratory acidosis<br />

(3) Is the magnitude of the increase in the anion gap equal to the magnitude of the decrease in serum<br />

bicarbonate? If not, is it:<br />

(a) Greater? If the bicarbonate is relatively increased, this indicates a superimposed metabolic<br />

alkalosis. Common clinical settings include:<br />

i. Vomiting patient with diabetic ketoacidosis or alcoholic ketoacidosis<br />

ii. Administration of NaH 2<br />

C0 3<br />

to an acidemic patient<br />

(b) Less? If the chloride is relatively increased, this indicates an associated hyperchloremic metabolic<br />

acidosis. Common clinical settings include:<br />

i. Renal excretion of ketones with retention of chloride (especially during initial treatment of<br />

diabetic ketoacidosis)<br />

11. Resuscitation with chloride-containing solution and dilution of serum bicarbonate<br />

iii. Severe diarrhea, renal tubular acidosis, and any primary cause of metabolic acidosis<br />

(eg, shock, sepsis)<br />

1. Definition: t pH (


ENDOCRINE, METABOLIC, AND NUTRITIONAL DISORDERS<br />

(4) faraldehyde<br />

(5) Iron, isoniazid, inhalants, inborn errors of metabolism<br />

(6) _lactic acidosis<br />

(7) Salicylates, solvents, starvation<br />

5. Management<br />

a. Controversial<br />

b. All agree that treatment of the underlying cause is paramount and clinically the most important.<br />

c. When the underlying cause is not readily reversed, some authors advocate administration of sodium<br />

bicarbonate to maintain a pH >7.10 or a serum bicarbonate >5 mEq/L.<br />

d. Bicarbonate can precipitate rapid electrolyte disturbances, resulting in symptomatic hypokalemia and<br />

hypocalcemia.<br />

e. Theoretically, paradoxical cerebral acidosis can occur because of the differential ability of CO 2<br />

and HCO 1<br />

-<br />

to cross the blood-brain barrier if bicarbonate is given too rapidly, ie, if CO 2<br />

crosses readily, HCO - 3<br />

crosses<br />

slowly.<br />

f. Specific therapy of the underlying cause<br />

(1) Insulin (diabetic ketoacidosis)<br />

(2) Pressors, antibiotics, and fluids (lactic acidosis)<br />

(3) Dialysis (renal failure, methanol, ethylene glycol, salicylates)<br />

C. Metabolic alkalosis<br />

(4) Ethanol or 4-methylpyrazole (methanol, ethylene glycol); 4-methylpyrazole (an alcohol dehydrogenase<br />

antagonist) is an alternative substitute for IV ethanol. 4-Methylpyrazole is the preferred current<br />

treatment if available.<br />

(5) Titrated HCO 3<br />

(renal tubular acidosis, according to type)<br />

(6) Benzodiazepines (status epilepticus)<br />

1. Definition: t pH (>7.45) and t HCO 3<br />

(>26 mEq/L)<br />

2. Most common reasons for a primary increase in blood bicarbonate concentration<br />

a. Vomiting and nasogastric suctioning<br />

b. Diuretic administration<br />

c. Adrenocortical hormone excess<br />

3. Metabolic alkalosis can be categorized as "chloride sensitive" or ''chloride resistant."<br />

a. Chloride-sensitive alkalosis (saline responsive): common<br />

(1) Diuretics - loss of K+ and Cl- - replace with saline and potassium<br />

(2) Vomiting and nasogastric suctioning - loss of H+, K+, and Cl- - (t renal excretion) - replace with<br />

saline. Vomiting is most likely the clinical scenario in a hypochloremic, hypokalemic metabolic<br />

acidosis.<br />

b. Chloride-resistant alkalosis (saline unresponsive): rare<br />

(1) Mineralocorticoid excess - t renal absorption of Na+ and HCO 3<br />

- with t renal excretion of K+, H+,<br />

and Cl- - replace with potassium<br />

(2) Large amounts of potassium are usually required to reduce renal excretion of H+ and treat the<br />

underlying cause.<br />

D. Respiratory acidosis<br />

1. Definition: t pH (45 mmHg)<br />

2. Primary cause is inadequate ventilation and/or increased dead space, both of which are associated with<br />

t excretion of pulmonary CO 2<br />

a. Etiology<br />

(1) Head or chest trauma<br />

(2) Oversedation<br />

(3) Metabolic coma<br />

(4) Neuromuscular disorders<br />

(a)<br />

Neuropathies<br />

(b) Myopathies<br />

(5) Chronic hypoventilation in obese patients<br />

(6) Obstruction<br />

614


ENDOCRINE, METABOLIC, AND NUTRITIONAL DISORDERS<br />

(a) Foreign body<br />

(b) Bronchospasm or laryngospasm<br />

b. Cause of increased dead space (with or without inadequate ventilation)-;, COPD<br />

3. Full renal compensation requires 48 hours of steady state alteration. Compensation is not expected to be<br />

complete; however, the pH may "normalize" in some patients.<br />

4. Proceed with caution when treating the underlying disorder.<br />

a. The pC0 2<br />

should not be lowered more than 5 mm Hg/hr (especially in those with chronic compensated<br />

respiratory acidosis, eg, patients with COPD).<br />

b. Ventilator assistance: continuous positive-airway pressure (CPAP) or bi-level positive-airway pressure<br />

(BiPAP) may be necessary in selected patients.<br />

c. Low-flow oxygen should be given to Pickwickian patients.<br />

E. Respiratory alkalosis<br />

1. Definition: 1' pH (>7.45) and t pC0 2<br />

(4-5 mEq/L)<br />

and there is an associated high anion gap acidosis. Other causes of high anion gap acidosis must be<br />

excluded to confirm this diagnosis:<br />

(1) Ingestions<br />

(a) Alcohols (methanol, ethylene glycol)<br />

(b) Salicylate<br />

(c) Paraldehyde<br />

(d) lsoniazid, iron<br />

(e) Cyanide<br />

(2) l


ENDOCRINE, METABOLIC, AND NUTRITIONAL DISORDERS<br />

(4) A mnemonic for causes of high anion gap acidosis is "A MUDPILES" and cyanide.<br />

Alcoholic ketoacidosis<br />

Methanol, Metformin<br />

.Uremia<br />

.!2iabetic (or alcoholic) ketoacidosis<br />

faraldehyde, Phenformin<br />

lsoniazid/lron/lnhalant (carbon monoxide, hydrogen sulfide) poisoning<br />

(mitochondrial injury caused by reverse transcriptase inhibitors [HIV treatment])<br />

lactic acidosis - shock, hypoxia, seizures, cyanide, metformin, phenformin<br />

fthylene glycol<br />

S.alicylates<br />

2. Management<br />

a. The presence of clinically significant lactic acidosis indicates a serious underlying disorder that must be<br />

identified and corrected.<br />

b. IV sodium bicarbonate has traditionally been the mainstay of therapy, but it remains a controversial issue.<br />

In general, sodium bicarbonate should be given only when the pH is 9.1. The approximate dose in<br />

milliequivalents can be calculated from the following formula:<br />

HCQ 3<br />

- deficit= 25 measured HCO 3<br />

- x 0.5 (body weight in kg)<br />

c. Most do not favor the use of insulin (or glucose and insulin infusions) in the treatment of lactic acidosis.<br />

The exception is diabetic ketoacidosis with concomitant lactic acidosis.<br />

d. Thiamine should be given to alcoholic patients with lactic acidosis, because it is needed in the oxidation<br />

of pyruvate.<br />

G. Important points to remember in the management of acid-base disorders<br />

1. An acid-base abnormality due to mixed disorders may exist in the presence of a normal pH or CO 2<br />

•<br />

2. If you suspect that respiratory acidosis is due to opioid use, administer naloxone early in the clinical<br />

evaluation.<br />

3. If you suspect a toxicant as the cause of an acid-base abnormality, you need specific quantitative serum levels<br />

of ASA, methanol, ethylene glycol and, if isoniazid can be detected by your laboratory, a qualitative level of<br />

this drug, which may be helpful. (Broad, routine urine and serum toxicology screens are not helpful.) Consider<br />

general interventions such as decontamination measures and specific interventions such as dialysis in methanol<br />

toxicity early in the treatment process.<br />

4. Consider isopropyl ingestion in ketotic patients with a normal glucose and no anion gap; these patients have an<br />

osmolar gap.<br />

5. Pearls and pitfalls of anion-gap acidosis<br />

a. Nitroprusside tests (urine reagent strips, reagent tablets) measure only acetoacetic acid (not ~-hydroxybutyric<br />

acid), which means that patients with diabetic ketoacidosis or alcoholic ketoacidosis may register falsely<br />

low ketones or no ketones at all.<br />

b. Patients who have ingested methanol (eg, "Sterno," windshield wiper fluid) may present 24-48 hours after<br />

ingestion with abdominal pain, an intoxicated appearance (but no breath odor of alcohol), and blurred<br />

vision with a hyperemic optic disc ("blind drunk"). All cases of methanol toxicity should be treated with IV<br />

ethanol (or 4-methylpyrazole) and most require dialysis.<br />

c. Patients who have ingested ethylene glycol (antifreeze) present within 12-24 hours with an intoxicated<br />

appearance (but no breath odor of alcohol). Hypocalcemia and calcium oxalate crystals in the urine may<br />

be seen, but their absence does not exclude the diagnosis.<br />

(1) Calcium oxalate crystals can also be seen in nontoxic ingestions (eg, tea, cola, spinach, rhubarb).<br />

(2) In addition to IV ethanol (or 4-methylpyrazole) and dialysis, these patients also need thiamine and<br />

pyridoxine. IV calcium may also be indicated for symptomatic hypocalcemia.<br />

d. Repeat laboratory values may be of help. Laboratories make mistakes, too!<br />

H. Clinical situations and associated laboratory findings<br />

1. Starvation/dehydration (decrease in total body water but not extracellular fluid volume), eg, the stroke patient<br />

left unattended at home for several days<br />

a. Hypernatremia (excess water loss)<br />

b. Hypokalemia (continuing urinary losses)<br />

c. Increased serum osmolality (increased sodium, metabolic acids)<br />

616


ENDOCRINE, METABOLIC, AND NUTRITIONAL DISORDERS<br />

d. Mild metabolic acidosis (ketones produced)<br />

e. Slightly increased anion gap<br />

2. Diarrhea/chronic laxative abuse<br />

a. Hyponatremia (fecal loss)<br />

b. Hypokalemia (fecal loss)<br />

c. Hypobicarbonatemia (fecal loss)<br />

d. Hyperchloremia (compensation for HCQ 3<br />

- loss)<br />

e. Hyperchloremic metabolic acidosis<br />

3. Vomiting<br />

a. Hypernatremia or hyponatremia or normal serum sodium<br />

b. Hypokalemia (potassium shifts intracellularly secondary to alkalosis; excess renal loss)<br />

c. HCQ 3<br />

- concentration >30 mEq/L<br />

d. Hypochloremia (Cl- loss)<br />

e. Metabolic' alkalosis (H+ loss)<br />

METABOLIC DISORDERS<br />

I. HYPOGLYCEMIA<br />

A. Definition<br />

1. Although there is significant individual variation, signs and symptoms of severe hypoglycemia do not usually<br />

occur until blood glucose falls below 50 mg/dL.<br />

2. Signs and symptoms<br />

B. Types<br />

a. Diaphoresis<br />

b. Tremulousness, nervousness<br />

c. Tachycardia<br />

d. Altered sensorium (confusion, agitation, unresponsiveness)<br />

e. Focal neurologic signs<br />

f. Seizures<br />

1. Although hypoglycemia can occur without diabetes (either insulin dependent or non-insulin dependent), it<br />

is most commonly seen as a result of treatment of diabetes. In the absence of diabetes, other diagnoses and<br />

medical conditions should be considered, but they are far less common than hypoglycemia seen as a diabetic<br />

emergency. These other illnesses are listed below:<br />

a. Medications such as ~-blockers<br />

b. Use of alcohol (usually excessive)<br />

c. Reactive hypoglycemia (delayed insulin release after a meal)<br />

d. Severe infection<br />

e. Cancers (eg, tumors such as hepatoma and mesothelioma can produce insulin-like factors) and insulinproducing<br />

tumors<br />

f. Renal insufficiency<br />

g. Liver failure<br />

h. Poor oral intake<br />

i. Congenital defects in metabolism<br />

j. Infants born to diabetic mothers, birth trauma, premature infants<br />

2. Hypoglycemia should be considered early in the management of any patient with altered mental status.<br />

Diagnosis is simple with bedside glucose testing.<br />

a. A blood glucose of


ENDOCRINE, METABOLIC, AND NUTRITIONAL DISORDERS<br />

3. Hypoglycemia management is based on severity of symptoms and patient's ability to tolerate oral intake.<br />

a. Mild symptoms and able to tolerate oral fluids: complex carbohydrate intake (meal tray)<br />

b. More severe symptoms or inability to tolerate oral fluids: IV administration of dextrose.<br />

(1) For adults: 1-3 ampules of 50% dextrose in water (D50W [1 ampule is 25 g dextrose in 50 ml])<br />

(2) For children: 0.5-1 g/kg IV of D25 or D10<br />

(3) Rule of 100: D25 dose is 4 ml/kg, D10 dose is 10 ml/kg.<br />

c. Additional treatment options<br />

4. Disposition<br />

(1) Glucagon can be given IM as an option when IV access cannot be obtained in an obtunded patient.<br />

This is not as effective in elderly or alcoholic patients (ineffective glycogen stores to make an effective<br />

increase in blood glucose).<br />

(2) Dose of glucagon in adults in 1 mg IM.<br />

a. Any patient with hypoglycemia from long-acting oral agents should be admitted to the hospital for<br />

observation because of continued risk of hypoglycemia.<br />

b. Patients with hypogylcemia requiring multiple doses of IV dextrose should be considered for observation in<br />

the hospital.<br />

c. Patients with straightforward hypoglycemia and no other significant factors such as infection, sepsis, etc,<br />

can be considered for discharge if only insulin is used for treatment of their diabetes.<br />

II. DIABETIC KETOACIDOSIS<br />

A. Etiology<br />

1. Metabolic derangements are due to a relative insufficiency of insulin and an excess of glucagon.<br />

a. Insulin insufficiency___,. cellular starvation ___,. release of stress hormones and glucagon ___,. increased<br />

gluconeogenesis, glycogenolysis, and lipolysis ___,. further hyperglycemia and increased free fatty acids___,.<br />

ketones (j3-hydroxybutyrate and acetoacetate)<br />

b. Glucagon has been implicated as the primary hormone responsible for hyperglycemia and ketonemia in<br />

diabetic ketoacidosis.<br />

2. Precipitating factors<br />

a. Lack of insulin<br />

b. Infection (a significant contributor to high mortality)<br />

(1) Pneumonia<br />

(2) Urinary tract infection<br />

c. Acute myocardial infarction (a significant contributor to high mortality)<br />

d. Cerebrovascular accident<br />

e. Trauma/surgery<br />

f. Pregnancy<br />

g. Alcohol/steroid use<br />

B. Clinical presentation<br />

1. Hyperglycemia results in decreased total body water:<br />

a. Hypotension, tachycardia, and dehydration<br />

b. Decreased serum electrolytes with depletion of:<br />

(1) Sodium: the dilutional effect of hyperglycemia depresses the serum sodium still further (1.6 mEq/L<br />

for each 100 mg/dl increase of serum glucose).<br />

(2) Potassium (total body): Because of acidosis, the potassium measurement may be high due to the<br />

shifting of potassium as the acidosis increases.<br />

2. Ketogenesis results in:<br />

a. Acidosis with a decrease in sodium bicarbonate<br />

b. A fruity breath odor (from acetone)<br />

c. Hyperventilation (Kussmaul respirations)<br />

618


ENDOCRINE, METABOLIC, AND NUTRITIONAL DISORDERS<br />

d. Hyperkalemia: potassium is initially increased, because the acidosis causes it to shift out of the cells in<br />

exchange for the hydrogen ion. However, correction of the acidosis may be associated with a profound<br />

hypokalemia.<br />

(1) As the acidosis improves, hydrogen ions will move out of the cells, allowing potassium to move back<br />

in and the serum potassium level to decrease.<br />

(2) Administration of insulin and glucose also allows potassium to move back into the cells, which<br />

further decreases serum potassium.<br />

(3) The average total body potassium deficit is -3-5 mEq/kg.<br />

C. Differential diagnosis of metabolic causes of coma in diabetic patients<br />

1. Diabetic ketoacidosis<br />

2. Hypoglycemia<br />

3. Hyperglycemic hyperosmolar nonketotic coma<br />

4. Alcoholic ketoacidosis<br />

5. Lactic acidosis<br />

6. Uremic acidosis<br />

D. Diagnostic evaluation<br />

1. Bedside testing (done immediately)<br />

a. Glucose reagent strip can rapidly differentiate hypoglycemia from hyperglycemia.<br />

b. Urine can be checked for:<br />

(1) Glucose: indicating hyperglycemia<br />

(2) Ketones with the nitroprusside test: a positive test is consistent with the diagnosis of diabetic<br />

ketoacidosis. However, this test primarily detects acetoacetate and will be falsely negative if most of<br />

the ketones are in the form of ~-hydroxybutyrate or acetone.<br />

2. Blood pH (arterial or venous blood gases) and serum CO 2<br />

content__,. confirms acidosis (1 pH and<br />

t bicarbonate)<br />

3. CBC (baseline)<br />

4. Glucose__,. hyperosmolarity secondary to hyperglycemia is the single most important determinant of the<br />

patient's mental status (serum osmolality >320 mOsm/kg = altered mental status)<br />

5. Serum ketones __,. the following are usually increased:<br />

a. Acetoacetate<br />

b. ~-Hydroxybutyrate<br />

c. Acetone (which is chemically neutral)<br />

6. Electrolytes__,. in addition to identifying abnormalities, you need to determine the anion gap.<br />

a. In the presence of a nondiagnostic ketone level, a wide gap suggests the presence of ~-hydroxybutyrate as<br />

the major ketone.<br />

b. In patients with mixed acid-base disturbances, the arterial pH may not accurately reflect the degree of<br />

acidosis; an anion gap does.<br />

7. Calcium, magnesium, phosphorus__,. if one is low, the other two generally are low as well.<br />

8. BUN/creatinine (baseline)<br />

9. Urinalysis, blood cultures, and chest radiograph __,. look for infection. Remember that any infection can cause<br />

diabetic ketoacidosis, including appendicitis, pyelonephritis, etc.<br />

10. ECG__,. look for evidence of acute myocardial infarction; estimate serum potassium level.<br />

11. Laboratory summary for diabetic ketoacidosis<br />

a. Blood glucose >250 mg/dL<br />

b. Serum acetone >2:1 dilution<br />

c. Bicarbonate


ENDOCRINE, METABOLIC, AND NUTRITIONAL DISORDERS<br />

b. If the patient is severely dehydrated, use normal saline for the initial 2 L of fluid resuscitation, then<br />

switch to 0.45% normal saline. In all other cases, alternating normal saline and 0.45% normal saline is<br />

recommended.<br />

c. When the serum glucose level falls to ~250 mgldL, add glucose to the IV to prevent iatrogenic<br />

hypoglycemia and cerebral edema.<br />

2. Insulin administration<br />

a. Large doses are not usually required to reverse diabetic ketoacidosis. In addition, complications<br />

(hypoglycemia, hypokalemia) are more likely to develop with large-dose insulin therapy.<br />

b. Continuous IV infusion of low doses of insulin (5-10 units/hr in adults and 0.1 unit/kg/hr in children) is<br />

simple, safe, and effective. If a patient does not respond within 1 hour (about a 50 mg/dL drop in blood<br />

glucose), double the infusion rate or administer a bolus.<br />

c. Onset of action is almost immediate; a priming IV bolus is not required, and current evidence suggests that<br />

children do not need an IV bolus. In adults, an IV bolus is optional.<br />

d. Insulin should initially be held if potassium level is


ENDOCRINE, METABOLIC, AND NUTRITIONAL DISORDERS<br />

d. Acute myocardial infarction<br />

e. Rhabdomyolysis<br />

2. Therapy-related comp I ications<br />

a. Hypoglycemia<br />

b. Hypokalemia, hypophosphatemia<br />

c. Paradoxical spinal fluid acidosis<br />

d. Cerebral edema<br />

e. Alkalosis (from too much bicarbonate replacement)<br />

f. Congestive heart failure (from overhydration)<br />

g. ARDS (from overhydration)<br />

G. Mortality from diabetic ketoacidosis<br />

1. Elderly: sepsis, cardiopulmonary complications<br />

2. Children and young adults: cerebral edema (onset usually


ENDOCRINE, METABOLIC, AND NUTRITIONAL DISORDERS<br />

C. Diagnostic evaluation/laboratory findings<br />

1. Blood glucose >400 mg/dL (may be> 1,000 mg!dl)<br />

2. Serum osmolality >315 mOsm/kg<br />

A serum osmolality should always be ordered, but results may not be readily available. Calculate osmolality<br />

using the following formula:<br />

[2 x (Na+)] + Gl~~ose +<br />

BUN<br />

2-"il<br />

3. Negative serum ketones<br />

4. Blood pH - no acidosis<br />

D. Management<br />

1. Fluid resuscitation: saline is the preferred fluid. Administer bolus fluids until the patient is hemodynamically<br />

stable.<br />

a. 0.9% normal saline at a rate of 15-20 ml/kg/hr can be used during the first hour and 4-14 ml/kg/hr after<br />

the first hour.<br />

b. Once hypotension, tachycardia, and urine output improve, 0.45% saline can be used to replace the<br />

remaining free water deficit.<br />

c. The average fluid deficit in hyperglycemic hyperosmolar nonketotic coma is 8-12 L.<br />

d. When the blood glucose reaches ~250 mg/dl, add glucose to the IV.<br />

2. Insulin administration is not the mainstay of treatment. Volume repletion should precede insulin therapy.<br />

a. Regular insulin given by continuous IV infusion is the customary method; dosage is 0.05-0.1 unit/kg/hr.<br />

There is no benefit to an IV bolus of insulin.<br />

b. No insulin should be given after the blood glucose reaches ~300 mg/dL.<br />

c. Patients who are insulin-naive may experience a rapid drop in blood glucose; in these cases, periodic<br />

serum glucose determinations are important.<br />

3. Potassium replacement<br />

a. KCI (10-20 mEq/hr) should be given during the acute phase of therapy (24-36 hours).<br />

b. It should be started within 2 hours of IV fluid and insulin therapy, or as soon as adequate renal function has<br />

been confirmed.<br />

c. Total body potassium depletion in nonketotic hyperosmolar coma is usually greater than that in diabetic<br />

ketoacidosis.<br />

4. Magnesium replacement: MgSO 4<br />

(1-2 g) should be given in the first 2 L of fluid if the serum magnesium is low.<br />

IV. ALCOHOLIC KETOACIDOSIS<br />

A. Clinical presentation<br />

Heavy ethanol consumption (or binge drinking) and decreased or no food intake for several days -<br />

} insulin and<br />

t glucagon and ethanol-induced inhibition of gluconeogenesis - lypolysis - t ketoacid production (acetoacetate<br />

and ~-hydroxybutyrate) by the liver - nausea, protracted vomiting, and abdominal pain that begins 24-72<br />

hours before presentation and usually terminates with consumption of any food or liquid (including alcohol) -<br />

tachypnea (ketoacidosis) and tachycardia (dehydration)<br />

B. Diagnostic evaluation<br />

1. A high anion gap acidosis is characteristic (Na+ - [Cl-+ HCQ 3<br />

-J = > 16) and is due primarily to high levels of<br />

~-hydroxybutyrate.<br />

2. Sometimes, ~-hydroxybutyrate cannot be detected with a urine test. If the patient has a high anion gap with a<br />

very low serum bicarbonate level, assume serum acidosis.<br />

3. The blood glucose level is not high (usually


ENDOCRINE, METABOLIC, AND NUTRITIONAL DISORDERS<br />

C. Management<br />

1. Administration of IV fluids and glucose is the mainstay of therapy.<br />

a. A solution containing both saline and glucose (OS/normal saline alternating with D5/0.45% normal saline)<br />

should be used, because recovery is faster when glucose-containing solutions are used.<br />

b. Approximately 3-6 L of fluid are required over 24-48 hours.<br />

2. Thiamine 50-100 mg IV should be given before the IV fluids are started, because glucose administration can<br />

precipitate Wernicke disease in the alcoholic patient.<br />

3. Correction of hypokalemia is with 30 mEq KCI IV or 30 mEq potassium orally.<br />

4. Insulin is not indicated and may be dangerous, because these patients usually have a near-normal or low<br />

blood glucose.<br />

5. Sodium bicarbonate therapy is generally unnecessary and should be avoided unless:<br />

a. The pH is


ENDOCRINE, METABOLIC, AND NUTRITIONAL DISORDERS<br />

c. Cooling blanket and acetaminophen to control fever<br />

d. Rate control of atrial fibrillation (may be refractory) and diuretics for CHF<br />

e. The order of treatment is very important to avoid iatrogenic injury.<br />

(1) Propranolol first<br />

(a) Decreases the life-threatening adrenergic activity<br />

(b) Can block the peripheral conversion ofT 4<br />

to T 3<br />

(c) Dosage: 1 mg/min IV with incremental increases of 1 mg every 10-15 minutes to a total of 10 mg<br />

(2) Propylthiouracil<br />

(a) Blocks the synthesis of thyroid hormone<br />

(b) Dosage: oral administration of a 600-1,000 mg load, followed by 250 mg every 6 hours<br />

(3) Iodide<br />

(a) Administer 1 hour after propylthiouracil because this can be used as a substrate in thyroid<br />

hormone production, worsening the disease process.<br />

(b) SSKI (potassium iodide) dosage: 3-5 drops orally every 8 hours<br />

(4) Dexamethasone<br />

(a) Prevents peripheral conversion ofT 4<br />

to T 3<br />

(b) Dosage: 2 mg IV every 6 hours<br />

2. Drugs to avoid in treating thyroid storm<br />

a. Aspirin: may t T 3<br />

and T 4<br />

levels<br />

b. Sedatives: interfere with CNS assessment during therapy<br />

c. Atropine: may accelerate heart rate and also counteract the effect of propranolol<br />

VI. MYXEDEMA (HYPOTHYROID) COMA<br />

A. An endocrinologic emergency with a 30%-40% mortality rate<br />

B. Pathophysiology of hypothyroidism<br />

1. Hypothyroidism may be primary (intrinsic failure of the thyroid gland to release T 3<br />

and T 4<br />

) or secondary to<br />

disease or destruction of the hypothalamus or pituitary gland. Primary hypothyroidism is far more common<br />

(95% of cases) than secondary hypothyroidism (5% of cases).<br />

a. Etiologies of primary hypothyroidism (TSH level is high)<br />

(1) Prior treatment of Grave disease with radioactive iodine or subtotal thyroidectomy (most common cause)<br />

(2) Autoimmune thyroid disorders (second most common cause)<br />

(a) Primary hypothyroidism secondary to autoimmune destruction of the thyroid - glandular atrophy<br />

- thyroid failure<br />

(b) Hashimoto thyroiditis<br />

i. The most common cause of goitrous hypothyroidism in areas with adequate iodine<br />

ii. Pathophysiologic mechanism may be defective hormone synthesis.<br />

(3) Rare causes of primary hypothyroidism<br />

(a)<br />

Iodine deficiency<br />

(b) Antithyroid drugs (lithium, phenylbutazone)<br />

(c) Spontaneous hypothyroidism from Grave disease<br />

(d) Congenital thyroid abnormalities<br />

b. Causes of secondary hypothyroidism (TSH level is normal or decreased)<br />

(1) Pituitary tumors<br />

(2) Postpartum hemorrhage - postpartum pituitary necrosis<br />

(3) Sarcoidosis<br />

(4) Dysfunction of the hypothalamus ("tertiary hypothyroidism")<br />

C. Evolution of hypothyroidism to myxedema and myxedema coma<br />

1. Myxedema is a rare and severe form of hypothyroidism seen in patients with undiagnosed or inadequately<br />

treated hypothyroidism. Like hypothyroidism, myxedema is most prevalent in older women and occurs most<br />

often in winter.<br />

2. The single most important factor in the evolution of myxedema to myxedema coma appears to be a physiologic<br />

stressor, including medications or another endocrine disorder.<br />

624


ENDOCRINE, METABOLIC, AND NUTRITIONAL DISORDERS<br />

D. Clinical presentation<br />

1. Overall, this is similar to a severe hypoadrenergic state and is classically characterized by hypotension,<br />

bradycardia, hypothermia, weakness, pseudomyotonic reflexes, alopecia, and a doughy, non-pitting edema.<br />

For the board exam, in the absence of an environmental insult: hypotension, bradycardia, and hypothermia is<br />

an endocrine problem (which can include sepsis).<br />

a. Hypothermia (80% of cases) without sweating or shivering is typical.<br />

b. Respiratory failure is also common and is characterized by hypercapnea, hypoventilation, and hypoxia.<br />

2. Key points to remember are that this is often associated with rhabdomyolysis and adrenal insufficiency. Key<br />

diagnostic findings include the following:<br />

a. Thyroid-stimulating hormone is usually increased.<br />

b. T 4<br />

is usually low.<br />

c. Glucose may be low.<br />

d. Serum cortisol may be low.<br />

e. Calcium level usually low.<br />

f. Creatine kinase is often increased.<br />

g. Look for effusions on chest radiograph and echocardiogram.<br />

h. ECG will often show bradycardia; may see prolonged QT interval or low voltage.<br />

1. Hyponatremia secondary to SIADH is a potentially grave finding that may lead to central edema.<br />

3. Evidence of hypothyroidism may be minimal or absent.<br />

a. A history of hypothyroidism should be sought in any comatose patient with hypothermia and respiratory<br />

failure. Historical clues:<br />

(1) Previous thyroid medication, radioactive iodine therapy, or subtotal thyroidectomy<br />

(2) Previous complaints of fatigue, weakness, cold intolerance, weight gain, menstrual irregularity, and<br />

muscle cramps<br />

(3) Previous evidence of neuropsychiatric abnormalities<br />

(a) Psychiatric disorders or personality changes<br />

(b) Seizures<br />

(c) Cerebellar signs (ataxia, intention tremors, nystagmus, difficulty with coordinated movements)<br />

(4) Previous history of paresthesias; a mononeuropathy is typical, with median nerve involvement (carpal<br />

tunnel syndrome) being the most common.<br />

b. Look for evidence of hypothyroidism on physical examination.<br />

(1) Bradycardia<br />

(2) Thyroidectomy scar<br />

(3) Thin eyebrows and scant body hair<br />

(4) Dry, scaly, yellow skin and puffy eyes<br />

(5) Abdominal distention due to fecal impaction, ascites, ileus, or urinary retention<br />

(6) Pseudomyotonic reflexes (brisk upstroke, slow relaxation), especially at the ankle<br />

(7) Nonpitting edema, including periorbital<br />

E. Diagnostic evaluation<br />

1. Although thyroid function tests (TSH, thyroxine-binding globulin, free T 4<br />

) should be ordered and can confirm<br />

the diagnosis of hypothyroidism, results are not always available in a timely fashion; initial therapy must be<br />

based on clinical impression.<br />

2. Characteristic laboratory findings<br />

a. Arterial blood gases: t pO 2<br />

and t pCO 2<br />

b. Electrolytes: t Na+ and t CJ- (and t Ca++ in thyroidectomized patients)<br />

c. Glucose: normal or low<br />

d. CBC: a left shift in the differential (with or without an increased WBC count) may be present if infection is<br />

the precipitating event.<br />

3. Nonspecific associated laboratory findings<br />

a. t creatine phosphokinase, lactate dehydrogenase, and aspartate aminotransferase (occasionally)<br />

b. t cerebrospinal fluid protein> 100 mg/dL (in most hypothyroid patients), although this may not be routinely<br />

discovered in the emergency department.<br />

625


ENDOCRINE, METABOLIC, AND NUTRITIONAL DISORDERS<br />

F. Management<br />

1. Supportive measures<br />

a. Levothyroxine<br />

(1) Dosage: 200-500 mcg IV<br />

(2) This is the most critical aspect of therapy, because clinical improvement will likely not be noticed<br />

with other measures until thyroid hormone is replaced.<br />

b. Slow rewarming of hypothermic patients<br />

c. IV fluids for hypotension<br />

d. Hydrocortisone 100 mg IV increments<br />

e<br />

If the patient is still hypotensive, consider pressors but usually a poor response.<br />

f. Correction of hyponatremia by fluid restriction (for dilutional hyponatremia) or hypertonic saline (for<br />

severe hyponatremia with altered mental state)<br />

g. Glucose infusion if there is hypoglycemia<br />

2. Identify and treat comorbid conditions (which are likely the inciting factor and are usually infectious). This may<br />

require empiric antibiotic therapy for infections.<br />

VII. ADRENAL INSUFFICIENCY (ADDISON DISEASE) AND CRISIS<br />

A. Pathophysiology<br />

1. Adrenal cortex produces glucocorticoids and mineralocorticoids.<br />

a. The major glucocorticoid is cortisol, which is produced by the following mechanism: hypothalamus<br />

releases corticotropin-releasing factor____.. pituitary secretes adrenocorticotropic hormone (ACTH)____.. adrenal<br />

cortex produces and secretes cortisol<br />

b. The major mineralocorticoid is aldosterone, which is regulated by the renin-angiotensin system and by<br />

plasma potassium concentrations.<br />

2. Hypofunction of the adrenal cortex<br />

a. When the physiologic demand for glucocorticoids and mineralocorticoids exceeds the capacity of the<br />

adrenal cortex to produce them, adrenal insufficiency occurs.<br />

(1) Primary adrenal insufficiency (Addison disease) is due to disease/destruction of the adrenal cortex or<br />

adrenalectomy.<br />

(a) Idiopathic atrophy, usually autoimmune mediated, is the leading cause of chronic adrenal<br />

insufficiency (70%-75% autoimmune, and 25%-30% truly idiopathic). Associated diseases<br />

include:<br />

i. Diabetes mellitus<br />

ii. Hashimoto thyroiditis<br />

iii. Grave disease<br />

iv. Hypoparathyroidism<br />

v. Pernicious anemia<br />

vi. Primary ovarian failure<br />

(b) Infectious/infiltrative causes<br />

i. Tuberculosis<br />

ii. Protozoal/fungal<br />

iii. Sarcoidosis/amyloidosis<br />

iv. AIDS/cytomegalovirus/herpes simplex virus<br />

v. Hemochromatosis<br />

vi. Metastatic cancer<br />

(c) Pharmacologic causes<br />

i. Methadone<br />

ii. Rifampin<br />

iii. Ketoconazole<br />

(d) Causes of bilateral adrenal hemorrhage (adrenal apoplexy) that can lead to insufficiency or crisis<br />

i. Anticoagulant therapy (especially after myocardial infarction)<br />

626


ENDOCRINE, METABOLIC, AND NUTRITIONAL DISORDERS<br />

ii. Fulminant neonatal sepsis (especially meningococcemia, ie, Waterhouse-Friderichsen syndrome)<br />

iii. Hemorrhage in the newborn<br />

(2) Secondary adrenal insufficiency due to dysfunction or destruction of the pituitary (and tertiary<br />

insufficiency due to hypothalamic dysfunction) leads to inability of the pituitary gland to secrete ACTH.<br />

(a) The most common cause of tertiary insufficiency (and crisis) is iatrogenic adrenal suppression<br />

from prolonged steroid use.<br />

(b) Causes of secondary insufficiency can include primary diseases of this pituitary axis, including<br />

tumors, hemorrhage, infarction, etc.<br />

b. When the adrenal reserve is exhausted (primarily of cortisol) in patients with chronic adrenal insufficiency<br />

who are subjected to stress or a concurrent illness, adrenal crisis occurs.<br />

(1) Most common cause is abrupt withdrawal of steroids in a patient whose adrenal function has been<br />

suppressed by prolonged steroid use.<br />

(2) Other causes include any physiologic stressor, which is a common theme in endocrine emergencies.<br />

B. Clinical presentation<br />

1. Primary adrenal insufficiency (Addison disease)<br />

a. Overal I, the patient appears weak and lethargic and exhibits fatigue on exertion. Postural hypotension and<br />

syncope are common. Heart sounds may be soft or inaudible.<br />

b. Signs and symptoms of cortisol deficiency<br />

(1) Lethargy<br />

(2) Anorexia, nausea, vomiting<br />

(3) Hypoglycemia with fasting<br />

(4) Inability to withstand even minor stress without shock<br />

c. Signs of aldosterone deficiency<br />

(1) 1 Heart size with cardiac output - postural hypotension, syncope, and azotemia (due to renal blood flow)<br />

(2) Dehydration with sodium depletion and hyperkalemia<br />

d. Prominent GI symptoms<br />

(1) Anorexia, nausea, vomiting, and occasionally diarrhea with weight loss<br />

(2) Abdominal pain<br />

e. Cutaneous manifestations: brownish pigmentation over exposed areas of the body and over friction or<br />

pressure points such as the elbows, fingers, knees, toes, and nipples<br />

2. Adrenal crisis<br />

a. Classic clinical scenario: The patient appears very ill, profoundly weak, and possibly confused.<br />

Hypotension (especially postural) is typical. Circulatory collapse, when present, may be profound (feeble,<br />

rapid pulse, and soft heart sounds). Fever is common. GI signs and symptoms are almost always present:<br />

anorexia, nausea, vomiting, and abdominal pain. Delirium and seizures may occur.<br />

b. Laboratory findings are variable, but the following is a classic triad:<br />

(1) Serum sodium is decreased (usual) or may be normal.<br />

(2) Serum potassium is normal or slightly increased.<br />

(3) Hypoglycemia is characteristic and may be profound.<br />

C. Diagnostic evaluation<br />

1. Laboratory findings<br />

a. Electrolyte abnormalities<br />

(1) Hyponatremia (88%)<br />

(2) Hyperkalemia (64%)<br />

(3) Hypercalcemia (6%-33%)<br />

b. Hypoglycemia<br />

c. Azotemia (t BUN/creatinine)<br />

2. ECG changes<br />

a. Signs of hyperkalemia<br />

(1) Tai I peaked T waves<br />

(2) Prolonged PR and QT intervals<br />

(3) Absent P waves<br />

627


ENDOCRINE, METABOLIC, AND NUTRITIONAL DISORDERS<br />

b. Nonspecific changes<br />

D. Management<br />

(1) Low voltage<br />

(2) Flat or inverted T waves (in the absence oft K+)<br />

(3) Depressed ST segment<br />

1. Adrenal crisis<br />

a. IV fluids: OS/normal saline with the first liter given over 1 hour. An additional 2-3 L may be needed during<br />

the first 8 hours of therapy.<br />

b. Glucocorticoid replacement<br />

(1) The drug of choice if adrenal insufficiency is not known and a corticotropin stimulation will be<br />

performed is dexamethasone (4 mg IV), because it does not interfere with measurement of steroids and<br />

it is long-acting (12-24 hours). Fludrocortisone should be added for mineralocorticoid effect.<br />

(2) If the diagnosis of adrenal insufficiency is known, hydrocortisone sodium succinate is preferred; the<br />

dose is 100-500 mg IV. This has mineralocorticoid activity, so additional medications are not needed.<br />

c. Hypoglycemia should be treated immediately.<br />

628


ENDOCRINE, METABOLIC, AND NUTRITIONAL DISORDERS: PRACTICE CLINICAL SCENARIOS<br />

ENDOCRINE, METABOLIC, AND NUTRITIONAL DISORDERS:<br />

PRACTICE CLINICAL SCENARIOS<br />

Answers immediately follow the clinical practice scenarios.<br />

Scenario A<br />

Presentation: A 60-year-old man presents with altered mental status and a blood glucose of 1,100 mg/dL.<br />

What is the diagnosis?<br />

Scenario B<br />

Presentation: A 45-year-old homeless person presents with an altered mental status.<br />

Diagnostic evaluation: Laboratory studies show an increased anion gap(> 16 mEq/dL) and an increased<br />

osmolar gap (> 10 mOsm/dL).<br />

What is the diagnosis?<br />

Scenario C<br />

Presentation: A 25-year-old injects heroin IV and becomes apneic.<br />

What is expected on the laboratory studies?<br />

Scenario D<br />

Presentation: A 20-year-old multi system trauma patient receiving a massive transfusion develops a wide<br />

QRS complex on the cardiac monitor.<br />

What is the diagnosis, and what treatment should be given?<br />

Scenario E<br />

Presentation: A 40-year-old man presents with weakness, lethargy, and nausea/vomiting. You notice he has<br />

bronzed skin despite living in a cloudy climate.<br />

Diagnostic evaluation: The ECG shows peaked T waves.<br />

What electrolyte abnormalities are expected, and what is the treatment?<br />

Scenario F<br />

Presentation: A 30-year-old woman is in septic shock.<br />

Diagnostic evaluation: Arterial blood gas results show a pH of 7.15 and a pCO 2<br />

of 14.<br />

Does she have a mixed acid-base disorder?<br />

629


ENDOCRINE, METABOLIC, AND NUTRITIONAL DISORDERS: PRACTICE CLINICAL SCENARIOS<br />

Scenario G<br />

Presentation: A 30-year-old, thin woman with no psychiatric history presents with tachycardia,<br />

diaphoresis, fever, agitation, and psychosis. Examination reveals hyperreflexia.<br />

What is the first medication that should be administered to treat the underlying disorder?<br />

Scenario H<br />

Presentation: A 20-year-old man with insulin-dependent diabetes presents 2 days after running out of<br />

insulin. He is vomiting, tachycardic, hypotensive, and lethargic.<br />

Why should insulin therapy not be initiated in this patient?<br />

Scenario I<br />

Presentation: A 60-year-old man with a chronic cough presents for weakness and confusion. While in the<br />

emergency department, he has a seizure.<br />

Diagnostic evaluation: The serum sodium is 110 mEq/L, and a chest radiograph shows a large mass in the<br />

right upper lobe.<br />

What findings are expected on a urinalysis? What treatment should be started?<br />

ScenarioJ<br />

Presentation: A 20-year-old woman with bipolar disorder presents for polydipsia and weakness.<br />

Diagnostic evaluation: The serum sodium is 150 mEq/L.<br />

What medication likely caused this patient's illness? What results are expected on a urinalysis?<br />

630


ENDOCRINE, METABOLIC, AND NUTRITIONAL DISORDERS: PRACTICE CLINICAL SCENARIOS<br />

ANSWERS TO PRACTICE CLINICAL SCENARIOS<br />

Scenario A<br />

Diagnosis: hyperosmolar nonketotic coma<br />

Diagnostic evaluation: Laboratory studies show hyperglycemia, ketoacidosis, and hypokalemia.<br />

Scenario B<br />

Diagnosis: isopropyl alcohol poisoning<br />

Diagnostic evaluation: Differential diagnoses include uremia, methanol poisoning, and ethylene glycol<br />

poisoning.<br />

Scenario C<br />

Diagnostic evaluation: Laboratory studies show respiratory acidosis, metabolic acidosis, and positive<br />

opiates on toxicologic screen.<br />

Scenario D<br />

Diagnosis: The diagnosis is hyperkalemia from massive transfusion, and potassium is leaking out of<br />

hemolyzed PRBCs. Treatment is calcium IV.<br />

Scenario E<br />

Diagnosis: The diagnosis is adrenal insufficiency/crisis. Hypoglycemia, hyponatremia, and hyperkalemia<br />

are expected. Treatment should include calcium for the ECG changes and a steroid such as hydrocortisone<br />

for the adrenal crisis.<br />

Scenario F<br />

A mixed acid-base disorder is unlikely given that the tenth and hundredth decimal points of the pH<br />

roughly estimate the pCO 2<br />

•<br />

ScenarioG<br />

The first medication to be administered should be a ~-blocker such as propranolol.<br />

Scenario H<br />

Insulin therapy should not be started until hypokalemia has not been excluded. If hypokalemia is<br />

discovered, it should be treated before starting insulin therapy.<br />

631


ENDOCRINE, METABOLIC, AND NUTRITIONAL DISORDERS: PRACTICE CLINICAL SCENARIOS<br />

Scenario I<br />

Expected findings include a concentrated urine despite a hypo-osmolar/hyponatremic serum, because this<br />

is likely SIADH. Hypertonic saline should be started to treat the severe hyponatremia.<br />

ScenarioJ<br />

Lithium can cause diabetes insipidus. A very dilute urine is expected despite a hyperosmolar/<br />

hypernatremic serum.<br />

632


ENVIRONMENTAL DISORDERS<br />

ENVIRONMENTAL DISORDERS<br />

Burns ......................................................................................................................................................................... 639<br />

Pathophysiology ................................................................................................................................................. 639<br />

Diagnostic Evaluation ......................................................................................................................................... 639<br />

Management ...................................................................................................................................................... 640<br />

Inhalation Injuries .............................................................................................................................................. 642<br />

Chemical Injuries ............................................................................................................................................... 643<br />

Electrical Injuries ................................................................................................................................................ 643<br />

Lightning Injuries ................................................................................................................................................ 645<br />

Heat Disorders .......................................................................................................................................................... 647<br />

Cold Disorders .......................................................................................................................................................... 650<br />

Frostbite ............................................................................................................................................................. 650<br />

Hypothermia ...................................................................................................................................................... 651<br />

Bites and Stings ......................................................................................................................................................... 655<br />

Human Bites ...................................................................................................................................................... 655<br />

Dog Bites ........................................................................................................................................................... 656<br />

Cat Bites ............................................................................................................................................................. 657<br />

Venomous Snake Bites ....................................................................................................................................... 658<br />

Bee Stings, Ant Bites, and Other Insect Bites and Stings ...................................................................................... 659<br />

Black Widow Spider ........................................................................................................................................... 659<br />

Brown Recluse Spider ........................................................................................................................................ 660<br />

Scorpion Stings ................................................................................................................................................... 660<br />

Jellyfish Stings .................................................................................................................................................... 661<br />

Altitude Disorders ..................................................................................................................................................... 661<br />

High-Altitude Disorders ...................................................................................................................................... 661<br />

Acute Mountain Sickness ............................................................................................................................ 661<br />

High-Altitude Pulmonary Edema (HAPE) ..................................................................................................... 661<br />

High-Altitude Cerebral Edema (HACE) ........................................................................................................ 662<br />

Dysbarism .......................................................................................................................................................... 662<br />

Submersion Injury ..................................................................................................................................................... 664<br />

Other Environmental Hazards .................................................................................................................................. 668<br />

Radiation Injuries ............................................................................................................................................... 668<br />

Crush Syndrome ................................................................................................................................................. 668<br />

Volcanic Eruptions ............................................................................................................................................. 669<br />

633


ENVIRONMENTAL DISORDERS: SELF-ASSESSMENT QUESTIONS<br />

ENVIRONMENTAL DISORDERS: SELF-ASSESSMENT QUESTIONS<br />

1. You are evaluating an adult burn victim of ideal body weight and find that he has partial-thickness burns of his<br />

entire right arm, anterior trunk, and one-half of his right leg. Using the "rule of nines," you estimate the total body<br />

surface area burned to be:<br />

(a) 27%<br />

(b) 31.5%<br />

(c) 36%<br />

(d) 54%<br />

2. The initial fluid of choice for the resuscitation of burn victims is:<br />

(a)<br />

D5/0.45% normal saline<br />

(b) OS/lactated Ringer's<br />

(c) Lactated Ringer's<br />

(d) Colloids<br />

3. Which of the following tissues has the least resistance?<br />

(a) Nerves, blood vessels<br />

(b) Bone and tendons<br />

(c) Fat<br />

(d) Skin<br />

4. Which of the following statements regarding electrical injuries is false?<br />

(a) Alternating current generally produces worse effects than direct current of the same voltage.<br />

(b) The dysrhythmia produced by direct current is usually ventricular fibrillation.<br />

(c) The exit wound produced by direct current is discrete.<br />

(d) Lightning behaves like direct current.<br />

5. All of the following statements regarding acclimatization are accurate except:<br />

(a)<br />

It is mediated by aldosterone.<br />

(b) It is a process through which an individual adapts to a warmer, more humid environment.<br />

(c) It results in a decreased concentration of sodium in the sweat and in the urine.<br />

(d) It results in decreased sweat volume.<br />

6. The mechanisms by which lightning produces injury include all of the following except:<br />

(a) Side flash ("splash")<br />

(b) Blunt trauma<br />

(c) Ground current<br />

(d) "Freezing"<br />

7. The appropriate method for rewarming a frostbitten extremity is to rewarm it rapidly in circulating water heated to:<br />

(a)<br />

100.4°-104°F (38°-40°C)<br />

(b) 104°-107.6°F (40°-42°C)<br />

(c) 107.6°-111.2°F (42°-44°C)<br />

(d) 111.2°-114.8°F (44°-46°C)<br />

8. All of the following ECG changes may be seen in association with hypothermia except:<br />

(a) Osborn waves<br />

(b) Prominent P and U waves<br />

(c) T wave inversion<br />

(d) Prolonged PR, QRS, and QT intervals<br />

9. The effect of hypothermia on the uncorrected arterial blood gas is to produce a falsely:<br />

(a) Low pH, high p0 2<br />

and pC0 2<br />

(b) High pH, low p0 2<br />

and pC0 2<br />

(c) Low pH, p0 2<br />

, and pC0 2<br />

(d) High pH, p0 2<br />

, and pC0 2<br />

634


ENVIRONMENTAL DISORDERS: SELF-ASSESSMENT QUESTIONS<br />

10. Cat-scratch disease is caused by:<br />

(a) A gram-positive coccus<br />

(b) A gram-negative bacillus<br />

(c) A virus<br />

(d) The causative organism is unknown.<br />

11. A patient presents with the complaint of a cat bite to her hand. You choose an antibiotic to cover the most common<br />

pathogen associated with cat bites, which is:<br />

(a) Staphylococcus aureus<br />

(b) Anaerobes<br />

(c) Eikenella corrodens<br />

(d) Pasteurella multocida<br />

12. What is the most venomous scorpion in North America?<br />

(a) Arizona bark scorpion (Centruroides genus)<br />

(b) Arizona hairy scorpion, aka desert hairy scorpion (Hadrurus arizonensis)<br />

(c) Stripe-tailed scorpion, aka "devil" scorpion (Hoffmannius genus)<br />

(d) Fat-tailed scorpion (Androctonus crassicauda)<br />

13. All of the following are appropriate antibiotic regimens for the treatment of a human bite wound except:<br />

(a)<br />

Dicloxacillin plus ampicillin or penicillin<br />

(b) Augmentin<br />

(c) Cefuroxime<br />

(d) Clindamycin<br />

14. The single leading cause of toxic death in the United States is:<br />

(a) Cyanide<br />

(b) Hydrogen sulfide<br />

(c) Carbon monoxide<br />

(d) Phosgene gas<br />

15. All of the following statements regarding "middle ear squeeze" are accurate except:<br />

(a) Symptoms (which include ear fullness and pain) develop on descent.<br />

(b) Divers usually begin to develop symptoms at 16-20 feet.<br />

(c) Symptoms result from failure to equalize the pressure between the middle ear and the water due to dysfunction<br />

of the eustachian tube.<br />

(d) Failure to abort the dive (or to return to the surface and then try a slower more controlled descent) will produce<br />

worsening pain and rupture of the tympanic membrane.<br />

16. All of the following statements regarding the treatment of black widow spider bites are accurate except:<br />

(a) The antivenin is equine-derived.<br />

(b) The mortality rate of these bites is rare.<br />

(c) Bites to the lower extremities or genitalia can produce pain simulating an "acute abdomen."<br />

(d) Antivenin should be administered to all patients.<br />

17. The organ system that is most resistant to the effects of ionizing radiation is:<br />

(a) GI system<br />

(b) CNS system<br />

(c) Hematopoietic system<br />

(d) Reproductive system<br />

18. The LD 50<br />

of whole body ionizing radiation is:<br />

(a) 250 rad<br />

(b) 350 rad<br />

(c) 650 rad<br />

(d) 850 rad<br />

635


ENVIRONMENTAL DISORDERS: SELF-ASSESSMENT QUESTIONS<br />

19. Which of the following measures is not considered to be useful in the treatment of patients with heat stroke?<br />

(a) Removing the patient from the heat source and completely undressing him or her<br />

(b) Applying atomized tepid water to the skin and fanning the patient<br />

(c) Administering an antipyretic agent (aspirin, ibuprofen, acetaminophen)<br />

(d) Controlling shivering with a benzodiazepine or chlorpromazine<br />

20. When treating heat stroke, active core cooling measures should be abandoned when the patient's core temperature<br />

reaches:<br />

(a) 96.8°F (36°C)<br />

(b) 98.6°F (37°C)<br />

(c) 100.4°F (38°C)<br />

(d) 102.2°F (39°C)<br />

21. The antibiotic of choice for treating wound infections caused by Pasteurella multocida is:<br />

(a)<br />

Penicillin<br />

(b) Cephalexin<br />

(c) Erythromycin<br />

(d) None of the above<br />

22. Although all of the following burns may occur in association with a lightning strike, ____ burns are<br />

pathognomon i c.<br />

(a)<br />

Linear<br />

(b) Punctate<br />

(c) Fern-like<br />

(d) Thermal<br />

23. A 38-year-old man presents 48 hours after being bitten by a neighbor's dog and is clearly septic. He had not sought<br />

prior medical attention, because the wound was relatively superficial and the dog was known to him and appeared<br />

healthy. Except for a splenectomy 6 years ago (due to trauma), his past medical history is unremarkable. Gram stain<br />

of the wound reveals a gram-negative bacillus. The most likely organism responsible is:<br />

(a)<br />

Eikenella corrodens<br />

(b) Pasteure/la multocida<br />

(c) Capnocytophaga canimorsus<br />

(d) Rochalimaea henselae<br />

24. All of the following antibiotics would be effective in treating the patient in the dog bite question (above) except:<br />

(a) Penicillin<br />

(b) A cephalosporin<br />

(c) Erythromycin<br />

(d) An aminoglycoside<br />

25. Permanent injury to the cochleovestibular system may occur in patients with:<br />

(a)<br />

Barotitis media<br />

(b) Barotitis interna<br />

(c) "Sinus squeeze"<br />

(d) Barotrauma of ascent<br />

26. The most common presentation of a pulmonary air embolism is a:<br />

(a) Neurologic event<br />

(b) Cardiovascular event<br />

(c) Pulmonary event<br />

(d) Psychological event<br />

636


ENVIRONMENTAL DISORDERS: SELF-ASSESSMENT QUESTIONS<br />

27. A patient with glucose-6-phosophate dehydrogenase deficiency has been bitten by a brown recluse spider. Which of<br />

the following medications is contraindicated?<br />

(a)<br />

Erythromycin<br />

(b) Antihistamine<br />

(c) Corticosteroids<br />

(d) Dapsone<br />

28. The best predictor of survival after radiation exposure is:<br />

(a) The whole body dose of ionizing radiation<br />

(b) Timing of the onset of symptoms<br />

(c) The absolute lymphocyte count 48 hours after exposure<br />

(d) The organ system involved<br />

29. On physical examination, the most sensitive indicator of a primary blast effect is:<br />

(a) Air emboli on funduscopic examination<br />

(b) Tympanic membrane rupture<br />

(c) Petechial hemorrhages in the nasopharynx<br />

(d) Tachypnea<br />

30. Frequent metabolic derangements in submersion-injury victims include all of the following except:<br />

(a) Hypoxemia<br />

(b) Acidosis<br />

(c) Serum electrolyte abnormalities<br />

(d) Hypercapnia<br />

31. All of the following statements regarding submersion injury are accurate except:<br />

(a) Saltwater drowning is more common than freshwater drowning.<br />

(b) Males are more commonly affected than females, regardless of age.<br />

(c) Children


ENVIRONMENTAL DISORDERS: SELF-ASSESSMENT QUESTIONS<br />

ANSWERS<br />

I. C 9. a 17. b 25. b 33. d<br />

2. C 10. b 18. b 26. a 34. d<br />

3. a 11. d 19. C 27. d<br />

4. b 12. a 20. d 28. C<br />

5. d 13. d 21. a 29. b<br />

6. d 14. C 22. C 30. C<br />

7. b 15. b 23. C 31. a<br />

8. b 16. d 24. d 32. C<br />

Use the pre-chapter multiple choice question worksheet (page xvi) to record and determine the percentage of correct<br />

answers for this chapter.<br />

638


ENVIRONMENTAL DISORDERS<br />

I. BURNS<br />

A. Pathophysiology<br />

1. Application of heat or chemicals causes "burning" of the skin and underlying tissues. Loss of the normal skin<br />

barrier allows fluids and electrolytes "out" and bacteria "in."<br />

2. Pulmonary damage occurs from direct heat, especially with steam exposure, as well as from inhalation of<br />

products of combustion and particulate matter.<br />

B. Diagnostic evaluation<br />

1. Obtain the following information:<br />

a. Was the victim burned in an enclosed or open space?<br />

b. Were there any toxic products of combustion present at the burn site?<br />

c. Does the patient have any respiratory symptoms?<br />

2. Estimate the body surface area (BSA) burned (and chart it). Only second- and third-degree burns should be<br />

included in the estimation.<br />

a. The "rule of nines" is used for adults and children >4 years old. It should not be used for children


ENVIRONMENTAL DISORDERS<br />

(2) Deep<br />

(a)<br />

Skin is white, with some erythematous areas, less blanching and moisture, tender<br />

(b) Deep reticular dermal injury<br />

(c) Causes: hot liquid or oil, steam, flame<br />

(d) Wound healing: 3-6 weeks (some scarring)<br />

c. Third degree (full thickness; dermal structures involved)<br />

(1) The skin is charred, pearly white, "leathery," and insensitive.<br />

(2) The entire skin layer and its nerve endings are destroyed and cannot grow back.<br />

(3) Causes: hot oil, steam, flame, high-voltage electrical burns<br />

(4) Wound healing: does not occur without surgical repair, skin grafting, or both (significant scarring)<br />

d. Fourth degree (musculoskeletal injury)<br />

(1) Extends below dermis to subcutaneous tissue (bone, muscle fascia)<br />

(2) Tissue is necrotic.<br />

(3) Causes: hot objects with high specific gravity (eg, molten metal), electrical burns<br />

4. Estimate the full extent of the burn (and chart it).<br />

a. Minor burns<br />

(1) Partial thickness 10% total BSA<br />

(3) Burns to the face, hands, major joints, feet, or perineum<br />

(4) Burns to an immunocompromised host<br />

(5) Burns and associated inhalation injury or major trauma<br />

(6) Electrical burns (high voltage)<br />

(7) Circumferential burns of the chest or extremities<br />

(8) Caustic chemical burns<br />

d. High-risk patients<br />

C. Management<br />

1. Prehospital<br />

(1) Younger than 10 or older than 50 years with minor to moderate burns<br />

(2) Concurrent medical illness or immunocompromised<br />

(3) Associated head injury, stroke, or psychiatric illness<br />

(4) Prolonged exposure in a confined space<br />

a. Stop the burning process.<br />

b. ABCs, including intubation if indicated, treatment with high-flow oxygen and IV fluids, followed by a<br />

head-to-toe survey looking for signs of trauma<br />

c. Cover the burn wounds with a clean dry sheet or sterile dressings.<br />

d. Severe pain may be lessened with frequent boluses of IV fentanyl or morphine as directed by medical<br />

protocol.<br />

2. Hospital (always use aseptic technique)<br />

a. ABCs: major pitfall is inadequate airway management<br />

(1) Make sure that the patient is receiving adequate supplemental oxygen; all patients with moderate<br />

to major burns should receive 100% oxygen via nonrebreather mask; place on pulse oximeter, but<br />

remember that the pulse oximeter may be inaccurate in the face of carbon monoxide and some other<br />

chemical inhalation injuries.<br />

(2) Early intubation is indicated if there are any signs of respiratory distress.<br />

(a) Unconscious or obtunded patients exposed to fire or smoke who were found in an enclosed<br />

space _,. consider possible cyanide poisoning (see page 579)<br />

640


ENVIRONMENTAL DISORDERS<br />

(b) Evidence of airway edema (stridor, dysphonia, tongue or oropharynx swelling) progresses over<br />

several hours; failure to intubate early may lead to failed airway.<br />

(c) Moderate to severe respiratory distress (including tachypnea, stridor, or labored breathing)<br />

(d) Mild respiratory distress and signs of inhalation injury<br />

i. Facial burns<br />

ii. Sooty sputum<br />

iii. Singed facial/nasal hairs<br />

iv. Oropharyngeal erythema, swelling, carbon deposits<br />

(e) Abnormal laboratory parameters<br />

i. Hypoxia despite 100% oxygen<br />

ii. High carboxyhemoglobin<br />

iii. Low peak flow<br />

b. Fluid resuscitation<br />

(1) Patients with burns >20% total BSA require fluid resuscitation; start two large-bore IV lines (above<br />

waist level) with lactated Ringer's (the fluid of choice for the first 24 hours after burn).<br />

(2) Use a common fluid resuscitation formula (eg, the Parkland formula). A general rule is to administer<br />

2-4 ml/kg per % total BSA burned/24 hours.<br />

(a) Give the first half of these fluids over the initial 8 hours (from time of burn).<br />

(b) Give the second half of these fluids over the next 16 hours.<br />

(c) Daily maintenance fluids should be administered in addition to the fluids above, particularly in<br />

infants and small children.<br />

(3) The above formula represents only an initial estimate of the fluid requirements and should be<br />

adjusted according to the patient's response (urinary output, heart rate, and mentation); administer<br />

fluids at a rate sufficient to maintain the pulse 1 ml/kg/hr in children), and a clear sensorium.<br />

c. Patients with extensive burns also need a Foley catheter (to monitor fluid resuscitation and check urine<br />

for myoglobinuria) and a nasogastric tube (prophylaxis for ileus).<br />

d. Avoid excessive cooling; do not apply ice to wounds or cool compresses to extensive burns.<br />

e. Medications<br />

(1) IV narcotics for pain relief<br />

(2) Tetanus prophylaxis (the only medication given IM)<br />

f. Baseline studies for patients with major burns<br />

(1) Arterial blood gases with a carboxyhemoglobin level (persistent metabolic acidosis +<br />

t carboxyhemoglobin level - possible cyanide toxicity)<br />

(2) ECG<br />

(3) Chest radiograph<br />

(4) Hematology panel<br />

(a) CBC with differential<br />

(b) Prothrombin time/lNR and partial thromboplastin time<br />

(c) Type and screen<br />

(5) Chemistries<br />

(a) Glucose<br />

(b) Electrolytes<br />

(c) BUN/creatinine<br />

(d) Creatine kinase<br />

(6) Urine studies<br />

(a) Urinalysis<br />

(b) Myoglobin<br />

(c) Human chorionic gonadotropin (hCG)<br />

g. Burn care<br />

(1) Circumferential burns<br />

(a) Assess the respiratory status in patients with a circumferentially burned thoracic cage; signs of<br />

respiratory insufficiency warrant consideration of escharotomy.<br />

641


ENVIRONMENTAL DISORDERS<br />

(b) Elevate circumferentially burned extremities, and monitor distal neurovascular status (sensation<br />

and pulses); loss of Doppler ultrasound signals mandates an escharotomy.<br />

(2) Major burns: cover burns with dry sterile sheets, and contact nearest burn center regarding any<br />

further management.<br />

(3) Minor burns<br />

h. Disposition<br />

(a) Apply cool compresses.<br />

(b) Administer pain medication.<br />

(c) Irrigate burn with sterile saline and gently cleanse with a mild soap.<br />

(d) Debride all loose tissue and broken blisters.<br />

(e) Large intact blisters (particularly tense ones) or blisters over very mobile joints can also be<br />

debrided or aspirated (controversial).<br />

(f) Apply a topical antibacterial agent (eg, bacitracin), and cover the wound with sterile dressing.<br />

Polymixin B is no longer recommended because of higher rates of contact allergic reaction.<br />

Silver sulfadiazine may be used but has fallen out of favor; it should be avoided in pregnant<br />

women in the late third trimester, because it may cause kernicterus and depigment skin (and<br />

because of cost).<br />

(1) Minor burns: discharge with pain medications and follow-up in 24 hours<br />

(2) Moderate burns: require hospital admission; patients with minor burns who are at high-risk or<br />

unreliable may also be appropriate for admission.<br />

(3) Major burns (see page 639): transfer to a burn center; patients with moderate burns who are highrisk<br />

or unreliable should also be considered for transfer.<br />

1. Contact the regional burn center early if transfer is contemplated, and tailor initial treatment to their<br />

standard regimen. In preparation for transfer, a protocol should be followed that includes:<br />

D. Inhalation injuries<br />

(1) Adequate resuscitation<br />

(2) Physician-to-physician contact<br />

(a) Do not delay initial airway management, fluid resuscitation, placement of catheters and lines, or<br />

diagnosis of associated life-threatening injuries.<br />

(b) Aggressive fluid resuscitation is especially important to maintain blood pressure and urine<br />

output to prevent renal failure; however, pulmonary edema and hypoxia can result from fluid<br />

overload (especially in elderly patients with inhalation injury).<br />

(3) A burn transfer checklist<br />

(4) A copy of the patient's completed chart<br />

1. Respiratory burns<br />

a. Intubate early, because subsequent edema may make this difficult or impossible later on.<br />

b. If at all possible, refer these patients to a regional burn center.<br />

2. Poisonous gas inhalation<br />

a. Phosgene gas or N 2<br />

O 2<br />

- immediate or delayed pulmonary edema<br />

b. Hydrogen sulfide (H 2 S) - rapid death from respiratory failure<br />

c. Carbon monoxide (CO)<br />

(1) The single leading cause of toxic deaths in the United States<br />

(2) A high index of suspicion is a must, because signs and symptoms are vague and multiple; any fire<br />

victim who was in an enclosed space should be evaluated for CO poisoning.<br />

(3) Treat all symptomatic patients because carboxyhemoglobin levels do not always correlate with severity<br />

of symptoms.<br />

(a) Minimum treatment is administration of 100% oxygen via a nonrebreather mask until the<br />

patient is asymptomatic and the carboxyhemoglobin level is 25%-40% at any time (level used varies with hospital policy and<br />

chamber accessibility)<br />

642


ENVIRONMENTAL DISORDERS<br />

E. Chemical injuries<br />

iii. Coma (or a history oft level of consciousness)<br />

iv. Pregnant patients who are symptomatic or have a carboxyhemoglobi n level > 15<br />

v. Neonates<br />

vi. Severe metabolic acidosis (pH 1,000 volts) or low voltage (


ENVIRONMENTAL DISORDERS<br />

(1) Current passing through the head and neck is more likely to induce cataract formation than current<br />

passing distal to these sites.<br />

(2) Current traversing the heart and thorax is associated with an increased likelihood of cardiac<br />

complications.<br />

5. Clinical presentation and physical examination<br />

a. Perform a head-to-toe survey looking for evidence of trauma.<br />

b. Look for entrance and exit wounds.<br />

(1) Electrical wounds are "bull's-eye" in appearance with a charred center; a grayish white area of<br />

coagulation necrosis is adjacent to the smrounding red, edematous tissue.<br />

(2) The true extent of underlying injury will not be apparent initially, so do not debride the wounds in<br />

the emergency department. Electrical injury may cause progressive intravascular thrombosis.<br />

6. Management<br />

a. Initial assessment and stabilization<br />

(1) ABCs<br />

(2) Monitor, IV access, and oxygen<br />

(a) Establish an IV line with a large-bore catheter; use lactated Ringer's or normal saline, and<br />

determine the rate based on the initial shock assessment (not the extent or degree of burn<br />

because most of the injury is internal); adjust the rate as needed to maintain a urine output of at<br />

least 0.5-1 ml/kg/hr.<br />

(b) Give oxygen to everyone (no exceptions).<br />

(c) Put the patient on a cardiac monitor.<br />

(3) Foley catheter (monitor urine output)<br />

(4) Nasogastric tube if injury is severe (increased risk of adynamic ileus)<br />

b. Diagnostic evaluation<br />

(1) ABGs<br />

(2) ECG<br />

(3) Creatine kinase (CK) + CK-MB<br />

(4) CBC<br />

(5) Prothrombin time/lNR and partial thromboplastin time<br />

(6) BUN/creatinine<br />

(7) Urinalysis (no RBCs but positive dipstick - presume myoglobinuria)<br />

(8) Serum electrolytes (including Ca++)<br />

(9) Urine myoglobin<br />

(10) Radiographic studies as indicated<br />

c. Treat myoglobinuria.<br />

(1) Administer fluids at a rate sufficient to maintain a urine output of 1.5-2 ml/kg/hr.<br />

(2) If this does not clear the pigment, administer mannitol as a bolus of 25 g, and add 12.5 g to each<br />

subsequent liter of fluids (at a rate of 1 L/hr) until the myoglobin is cleared. Furosemide (2 mg/kg IV)<br />

may also be administered to promote diuresis.<br />

(3) Alkalinization of the urine is suggested in some texts to be beneficial at CK concentrations >6,000<br />

IU/L; however, this has never been proved.<br />

d. Do not forget tetanus prophylaxis.<br />

e. Disposition<br />

(1) Hospitalization and cardiac monitoring are indicated for all patients with high-voltage burns and for<br />

those patients with low-voltage burns who are symptomatic (eg, chest pain, abnormal physical or<br />

laboratory findings). Those with severe electrical burns should be admitted to a burn center.<br />

(2) Asymptomatic patients with low-voltage injuries may be discharged to home after a period of<br />

observation and cardiac monitoring in the emergency department if their physical examination and<br />

diagnostic evaluation are unremarkable (no skin lesions, normal ECG, no urinary heme pigment).<br />

f. Delayed complications of major electrical burns<br />

(1) Myoglobinuric renal failure<br />

(2) Compartment syndromes<br />

(3) Gangrene (due to intravascular thrombosis)<br />

(4) Dysrhythmias<br />

644


ENVIRONMENTAL DISORDERS<br />

(5) Infection<br />

(6) Cataracts<br />

(7) Neurologic problems (peripheral nerve damage, trouble with memory/concentration, seizures,<br />

delayed spinal cord syndromes)<br />

7. Electrical lip burn<br />

a. Clinical presentation: usual history is a child bites on an electrical cord, and burns lips and commissures.<br />

b. Management<br />

(1) Do not debride these wounds. Cleanse and apply a petroleum-based antibiotic ointment.<br />

(2) These burns require close observation and referral to a plastic or oral surgeon for splinting and<br />

further care.<br />

c. Complications: Delayed hemorrhage from the labial artery may occur in 10%-15% of these patients<br />

3-14 days after injury, when the eschar separates.<br />

G. Lightning injuries<br />

1. Pathophysiology<br />

a. Lightning injuries are extremely high-intensity bursts of direct current (10 million to 2 billion volts) that<br />

are very brief (0.1-1 milliseconds). Although one might compare lightning injuries to a massive DC<br />

countershock, the injuries that occur from a lightning strike are different from those that occur with manmade<br />

electricity, as is the treatment; unlike victims of high-voltage electricity, lightning victims rarely have<br />

deep burns or underlying tissue damage, and they do not generally require fluid resuscitation.<br />

b. Because of the brief duration, lightning rarely breaks down the skin; instead, it passes over the body<br />

surface, and if the victim's body is wet or sweaty, can cause burns in one of two ways:<br />

(1) Moisture can be vaporized and produce "steam burns."<br />

(2) Sudden expansion of water converting to steam can cause the clothes to "explode" off the body,<br />

burning it in the process.<br />

2. Mechanisms of injury<br />

a. Direct strike: Lightning hits the victim directly and passes over or through him or her; morbidity and<br />

mortality are highest with this mechanism.<br />

b. Contact: lightning strikes an object that the victim is touching.<br />

c. Side flash or splash: lightning jumps from its initial strike site to a nearby person or object in its pathway.<br />

d. Ground current or step voltage: I ightning strikes the ground and spreads through it to the victim; if one of<br />

the victim's legs is nearer to the strike point than the other, a potential difference is created between them,<br />

resulting in the current passing through the legs and into the trunk.<br />

e. Thermal burns occur as a result of boiling sweat, burning clothes, or hot jewelry.<br />

f. Blunt trauma: lightning rapidly heating air followed by rapid cooling causes a blast effect that throws the<br />

victim; the organ affected most often by blast injury is the ear.<br />

3. Classic clinical scenario: An observed strike is obvious, but many are not observed. An important diagnostic<br />

clue in the unconscious patient is that clothes may be partially missing, especially the shoes.<br />

a. Clinical picture (minor lightning strike): patient appears "stunned," is somewhat confused, and has difficulty<br />

recalling events that transpired a few hours ago. He or she has no memory of the event described. Unless<br />

the lightning strike was witnessed, this diagnosis may be missed. Do not be fooled by the absence of<br />

entrance or exit wounds; they are rare. The patient has mild hypertension and tachycardia and is likely to<br />

complain of headache. There is frequently a history of temporary loss of vision or hearing.<br />

b. Possible physical findings (major lightning injuries)<br />

(1) Cardiopulmonary injuries<br />

(a) Cardiac arrest (asystole): most common cause of death<br />

(b) Respiratory arrest secondary to tetany of diaphragm muscles: may be more prolonged than<br />

cardiac arrest and can cause secondary ventricular fibrillation<br />

(c) Pulmonary edema<br />

(d) Transient hypertension<br />

(2) Neurologic injuries<br />

(a) Transient loss of consciousness (the most common neurologic event)<br />

(b) Confusion and amnesia (both anterograde and retrograde) are also common.<br />

(c) Peripheral nerve damage is another common finding.<br />

(d) Seizures result from hypoxia or concomitant head injury.<br />

645


ENVIRONMENTAL DISORDERS<br />

(e) Transient flaccid paralysis (keraunoparalysis) of the lower extremities (which may initially<br />

appear cold, pale, pulseless, and insensitive) is generally due to vascular spasm and sympathetic<br />

instability; it usually resolves spontaneously over several hours.<br />

(3) Eye and ear injuries<br />

(a) Immediate or delayed cataract formation<br />

(b) Corneal injuries, uveitis, hyphema, vitreous hemorrhage, retinal detachment, optic nerve atrophy<br />

(c) Unreactive dilated pupils secondary to autonomic instability (do not misinterpret as an indication<br />

of brain death)<br />

(d) Ruptured tympanic membranes (>50% of cases) are caused by:<br />

i. The blast effect or<br />

ii. Basilar skull fractures or<br />

iii. Direct burn<br />

(4) Skeletal injuries (fractures)<br />

(a) Much less common than with manmade high-voltage injuries.<br />

(b) Common sites are skull, spine, or long bones.<br />

(5) Effects on fetus<br />

(a) Intrauterine death (25%)<br />

(b) Neonatal death (25%)<br />

(c) Healthy babies (50%)<br />

(6) Burns (four types)<br />

(a) Linear: superficial burns that occur in areas where sweat and moisture accumulate.<br />

(b) Punctate: small, discrete, circular burns that occur in clusters<br />

(c) Lichtenberg figures: superficial fern-like burns that are pathognomonic for lightning strike<br />

(d) Thermal: result from the ignition of clothing or the heating up of objects worn by the victim<br />

4. Management<br />

a. "Resuscitate the dead" (even if there are multiple victims)<br />

(1) Those who are in cardiac or pulmonary arrest need immediate attention if they are to have any<br />

chance of survival.<br />

(2) Those who are moving are OK on their own initially.<br />

b. ABC assessment and initial stabilization followed by a head-to-toe survey for signs of trauma. Standard<br />

ACLS and ATLS treatment protocols should be followed. Massive fluid resuscitation is seldom indicated,<br />

because lightning injuries rarely produce significant tissue destruction.<br />

c. Burns, if present, should be treated in the usual manner. Do not forget tetanus prophylaxis.<br />

d. Baseline laboratory and diagnostic evaluation<br />

(1) CBC<br />

(2) Electrolytes, BUN/creatinine<br />

(3) ECG and cardiac enzymes<br />

(4) Urinalysis and urine for myoglobin<br />

(5) Chest radiograph and other radiographs as indicated<br />

(6) Brain CT (and cervical spine radiographs) should be ordered for patients with an altered level of<br />

consciousness.<br />

5. Disposition: In almost all instances, these patients should be admitted with continuous cardiac monitoring for<br />

24 hours.<br />

6. Late sequelae<br />

a. Posttraumatic stress disorder/keraunophobia (an abnormal fear of thunder and lightning)<br />

b. Paresis<br />

c. Impaired mental function<br />

d. Memory deficits<br />

e. Insomnia<br />

f. Cataracts<br />

g. Peripheral neuropathy<br />

646


ENVIRONMENTAL DISORDERS<br />

II. HEAT DISORDERS<br />

A. Normal body temperature regulation (heat gain= heat loss)<br />

1. Heat gain is the result of:<br />

a. Metabolic activity<br />

b. Environmental heat<br />

c. Strenuous exertion<br />

2. Heat loss results from:<br />

a. Radiation<br />

(1) Cutaneous vasodilation increases the amount of heat that can be dissipated through the skin.<br />

(2) As ambient temperature approaches and surpasses body temperature, heat loss via radiation ceases<br />

and radiation becomes a source of heat gain.<br />

b. Convection: heat loss due to air or water circulating across the body (wind chill)<br />

c. Conduction: direct physical contact with a cooler object or immersion in water<br />

d. Evaporation<br />

(1) The degree of sweating is controlled by cholinergic and sympathetic activity.<br />

(2) The effectiveness of sweating is decreased in the presence of high humidity.<br />

3. The wet-bulb globe thermometer index<br />

a. Accounts for the effects of humidity and radiant heat on the ambient temperature<br />

b. The most accurate measure of environmental heat stress and the risk of heat illness<br />

4. Acclimatization<br />

a. A process over a period of 1-2 weeks in which an individual adapts to a warmer, more humid climate via<br />

hypothalamic and thyroid adaptation<br />

b. Mediated by aldosterone, which reduces the sodium concentration in sweat and urine; this preserves fluid<br />

volume, thus preventing the development of a heat illness.<br />

c. Characterized by an earlier onset of sweating, greater sweat volume, and a decreased concentration of<br />

sodium in the sweat and in the urine<br />

B. Heat illness<br />

1. Imbalance between heat production and loss<br />

2. Predisposing factors<br />

a. Physical activity (especially in a hot, humid environment)<br />

b. Extremes of age, poor physical condition, fatigue<br />

c. Inadequate indoor cooling (lack of or malfunctioning air conditioning)<br />

d. Excessive clothing<br />

e. Dehydration<br />

f. Preexisting cardiovascular disease<br />

g. Skin disorders<br />

h. Obesity<br />

1. Drugs<br />

(1) Phenothiazines (act centrally on the hypothalamus)<br />

(2) Anticholinergics: reduce the ability to sweat<br />

(a) Atropine<br />

(b) Scopolamine<br />

(c) Cogentin<br />

(d) Antihistamines<br />

(e) Cyclic antidepressants<br />

(3) [:)-blockers/calcium channel blockers (1 heat loss)<br />

(4) Diuretics (t heat loss)<br />

(5) Amphetamines (t heat gain) and sympathomimetics, including OTC pseudoephedrine and bath salts<br />

(6) LSD ( t heat gain)<br />

(7) Cocaine (t heat gain)<br />

(8) Monoamine oxidase inhibitors (t heat gain)<br />

647


ENVIRONMENTAL DISORDERS<br />

(9) MDMNecstasy, "rave drugs"<br />

(1 0) Haloperidol (neuroleptic)<br />

(11) Alcohol withdrawal<br />

3. Pathophysiology<br />

a. Heat cramps: inadequate replacement of salt from loss through sweating -> hyponatremia -> muscle cramps<br />

b. Heat tetany: hyperventilation -> respiratory alkalosis _,. paresthesias and carpopedal spasm<br />

c. Heat exhauston: salt water depletion from sweat loss-> hypovolemia and hypoperfusion, and normal<br />

mental status and neurologic examinations<br />

d. Heat stroke<br />

(1) Heat stress (classic) or endogenous heat production (exertional) -> a breakdown of central<br />

thermoregulatory control -> hyperpyrexia and neurologic symptoms ± absence of sweating. Sweating<br />

is usually absent in classic heat stroke and present in exercise-induced or exertional heat stroke.<br />

Cerebral edema is common.<br />

(2) Loss of sweating ability aggravates (but does not cause) the problem.<br />

(a) The exact mechanism by which the ability to sweat is lost is unknown, but a direct thermal<br />

effect on sweat glands is a contributing factor.<br />

(b) Anticholinergic drugs are the most frequent cause of impaired sweating in classic heat stroke.<br />

(c) Specific disorders such as congenital anhydrosis, cystic fibrosis, and quadriplegia are rare<br />

contributing factors.<br />

(3) Volume depletion and electrolyte imbalance are not usually prominent features, but central venous<br />

pressure is usually increased.<br />

(4) Patients with preexisting cardiovascular disease have a reduced capacity to cope with the<br />

hemodynamic changes of peripheral vasodilatation.<br />

(5) End-organ and systemic injury may occur.<br />

(a) Cardiac (CHF, pulmonary edema)<br />

(b) ARDS<br />

(c) Liver (marked increases of AST and ALT)<br />

(d) Kidney (hematuria, proteinuria, acute tubular necrosis)<br />

(e) Muscular (mild increase in creatine kinase -> rhabdomyolysis and acute renal faiure)<br />

(f)<br />

Hematologic (altered coagulation -> bleeding, disseminated intravascular coagulation)<br />

C. Classic presentation<br />

1. Heat cramps: The patient complains of severe muscle cramping typically involving the calves, thighs, and<br />

shoulders. Questioning reveals that the cramps developed after a bout of intense physical activity and profuse<br />

sweating, during which the patient had been replacing fluid losses with a hypotonic solution. Body temperature<br />

is normal.<br />

2. Heat tetany: The patient is hyperventilating and complains of tingling and spasms of the hands and feet.<br />

3. Heat exhaustion: The patient presents with extreme fatigue and profuse sweating. If the onset was fairly recent,<br />

he or she will complain of light-headedness, nausea, vomiting, and a dull headache. If the patient started<br />

feeling ill several hours ago, concerned family or friends may have brought the patient in because he or she<br />

"didn't look right;" this patient is tachypneic, tachycardic, and may be hypotensive. Body temperature in<br />

patients with heat exhaustion is normal or slightly increased.<br />

4. Heat stroke: a true medical emergency characterized by an altered level of consciousness and any neurologic<br />

finding and an increased temperature<br />

a. Classic heat stroke occurs most often in middle-aged or elderly patients who live a sedentary lifestyle<br />

and are taking medications for chronic illnesses. It is caused by environmental heat exposure. Do not be<br />

fooled if the patient is sweating; some do initially and, if not treated, will stop sweating and develop hot,<br />

dry skin. It may be difficult to exclude a cerebrovascular or CNS infection on physical examination. An<br />

accurate history is critical to the diagnosis; ataxia may be the initial (or presenting) complaint. Laboratory<br />

findings include:<br />

(1) Respiratory alkalosis and mild metabolic acidosis<br />

(2) Mild coagulopathy and increased creatine kinase<br />

(3) Normal glucose and calcium levels<br />

b. Exertional heat stroke typically occurs in young, healthy patients who have been engaged in strenuous<br />

exercise. These patients are usually diaphoretic on presentation. Characteristic laboratory findings<br />

include:<br />

648


ENVIRONMENTAL DISORDERS<br />

D. Management<br />

(1) Respiratory alkalosis and marked lactic acidosis<br />

(2) Disseminated intravascular coagulation and rhabdomyolysis (urine specimen looks like "machine oil")<br />

(3) t BUN/creatinine (acute renal failure)<br />

(4) Hypoglycemia and hypocalcemia<br />

1. Heat cramps<br />

a. Oral or IV fluid and electrolyte replacement<br />

b. Rest in a cool environment with a gradual return to normal environment and activity<br />

2. Heat tetany<br />

a. Removal from heat<br />

b. Rebreathe expired air (bag-breathing)<br />

c. Exclude electrolyte abnormalities<br />

3. Heat exhaustion<br />

a. Bedrest in a cool environment<br />

b. Rapid IV fluid/electrolyte volume replacement, initially with normal saline<br />

c. Baseline studies: CBC, glucose, electrolytes, BUN/creatinine, liver profile, and urinalysis<br />

4. Heat stroke<br />

a. Management<br />

(1) Immediate, aggressive, rapid cooling down to 102.2°F (39°C). Morbidity is directly related to severity<br />

and duration of hyperthermia.<br />

(a) Remove patient from heat source, and undress him or her completely.<br />

(b) Apply atomized tepid water (not ice) to the skin, and fan the patient.<br />

(c) Apply ice packs to axillae and groin.<br />

(d) Iced gastric lavage is rarely used; if done, monitor ins and outs.<br />

(e) Immersion in ice water is also very effective but may precipitate seizures; monitoring and<br />

resuscitation are difficult with this technique. It is recommended, however, if above measures<br />

have failed to lower body temperature to 102.8°F (38.9°C) within 30 minutes.<br />

(2) Supportive measures<br />

(a) Administer high-flow oxygen to all patients, and monitor by pulse oximetry; intubate if there is<br />

obtundation, seizures, or a depressed gag reflex.<br />

(b) Establish IV access of normal saline. Young, healthy patients with exertional heat stroke are<br />

dehydrated and require aggressive fluid management. Those with classic heat stroke need IV<br />

fluids, but should not be rehydrated aggressively (fluid requirements are usually not large); rate<br />

of 250-300 ml/hr is generally adequate. Central venous pressure monitoring may be needed to<br />

guide fluid therapy in older patients and in those with cardiovascular disease.<br />

(c) Put the patient on a cardiac monitor. Tachydysrhythmias are common and respond to cooling<br />

(do not cardiovert); be careful not to insert the tip of the central venous pressure catheter into<br />

an irritable heart.<br />

(d) Establish continuous temperature monitoring via an esophageal or rectal probe.<br />

(e) Place a Foley catheter to monitor urine output.<br />

(f) Obtain baseline laboratory studies<br />

i. Arterial blood gases (corrected for temperature), lactate<br />

ii. CBC, glucose, electrolytes (including Ca++ and Mg++)<br />

iii. BUN/creatinine, prothrombin time/lNR and partial thromboplastin time, toxicology screen<br />

iv. Thyroid and liver function tests<br />

v. Urinalysis and urine myoglobin<br />

vi. ECG and chest radiograph (especially in older patients)<br />

b. Medications<br />

(1) Control shivering with a benzodiazepine or chlorpromazine as needed; induction of paralysis may<br />

be required.<br />

(2) Antipyretics (aspirin, ibuprofen, acetaminophen) are not useful.<br />

(3) Dantrolene has not been demonstrated to be effective.<br />

649


ENVIRONMENTAL DISORDERS<br />

Ill. COLD DISORDERS<br />

A. Frostbite (local cold injury can occur at temperatures above and below freezing)<br />

1. Pathophysiology<br />

a. Causes of tissue damage<br />

(1) Capillary stasis and thrombosis<br />

(a) Cooling to 59°F (15°C)-'> maximal vasoconstriction and a significant decrease in cutaneous<br />

blood flow<br />

(b) Continued cooling to 50°F (10°C) - cold-induced vasodilation (the "hunting response") in which<br />

vasoconstriction is interrupted in a cyclical fashion by periods of vasodilation in an effort to<br />

protect the extremity from the cold<br />

(c) When returning cold blood starts to decrease the core body temperature, blood flow to the<br />

extremity shuts down and the tissue temperature drops further-'> capillary endothelial damage-'><br />

increased platelet affinity and plasma leakage from the intravascular space-'> increased viscosity,<br />

RBC stasis, and vessel thrombosis; frostbite occurs when the tissue temperature drops below 32°F<br />

(0°C).<br />

(2) Ice crystal formation in tissues<br />

(a)<br />

Extracellular ice contributes to intracellular dehydration and destruction of protein.<br />

(b) Intracellular ice destroys cell architecture and function.<br />

(3) Reperfusion injury: rewarming - return of blood flow to the injured extremity and release of<br />

arachidonic acid metabolites (thromboxane, prostaglandins) from the damaged endothelial cells-'><br />

vasoconstriction, platelet clumping, and sludging of WBCs -'> progressive tissue loss<br />

b. Most likely sites of tissue damage because they are farthest from the body core: hands, feet, ears, face,<br />

and nose<br />

c. Factors that affect the severity of tissue injury<br />

(1) Temperature and duration of cold exposure: remember that wet skin and clothing freeze faster than dry<br />

skin and clothing.<br />

(2) Humidity: adds to correct evaporative heat loss.<br />

(3) Wind chill: moving air accelerates heat loss.<br />

(4) High altitude: hypoxia affects the CNS and one's judgment ability; dehydration results from the<br />

increased respiratory rate-'> decreased blood flow<br />

(5) Refreezing of a thawed extremity: greatly increases the severity of tissue damage and increases the<br />

likelihood of tissue loss<br />

2. Clinical presentation<br />

a. Nonfreezing, wet - trench foot (immersion foot)<br />

(1) Requires prolonged exposure (hours to days) to a wet environment<br />

(2) Early symptoms include numbness, painful paresthesias, and leg cramps.<br />

(3) Initial evaluation reveals a cold, pale, and anesthetic extremity.<br />

(4) A hyperemic phase follows and is accompanied by severe burning pain.<br />

(5) Tissue loss is uncommon.<br />

b. Nonfreezing, dry - chilblains (pernio)<br />

(1) Results from exposure to cold, damp air<br />

(2) Lesions develop on exposed areas after a delay of 12-14 hours and are characterized by erythema,<br />

pruritus, and burning paresthesias.<br />

(3) Women with Raynaud phenomenon are at highest risk.<br />

c. Freezing, dry or wet-'> frostnip and frostbite<br />

(1) Frostnip is an early response to cold exposure and is reversible.<br />

(a) The first sign is pale, painful, itchy skin.<br />

(b) If this progresses to frostbite, the skin remains cold to palpation and appears pale and gray.<br />

(2) Frostbite<br />

(a)<br />

Superficial: under the frozen surface, skin is soft.<br />

1. Once thawed: large, clear, fluid-filled vesicles develop within 24-48 hours; when these are<br />

reabsorbed, the skin turns black and hard (carapace).<br />

650


ENVIRONMENTAL DISORDERS<br />

3. Management<br />

ii. The skin takes weeks to demarcate, and the carapace sloughs slowly, leaving new pink<br />

hypersensitive skin.<br />

(b) Deep: under the frozen surface, skin is rock hard.<br />

i. Once thawed, the area is cold and has a purple or red discoloration; smal I hemorrhagic<br />

vesicles and edema develop within 1-3 weeks.<br />

ii. Weeks to months later, nonviable structures demarcate, become mummified, and slough off.<br />

a. Prevention is the most important aspect of treatment. Proper clothing and nutrition, good health, and<br />

avoidance of fatigue and alcohol seem to be the most crucial aspects of prevention.<br />

b. Frostnip<br />

(1) Rewarming may be initiated in the field (breath, body heat, other heat sources). No rubbing.<br />

(2) Return to work is possible after rewarming, but the patient should be cautioned about recurring symptoms.<br />

c. Frostbite<br />

(1) Rapid rewarming is key and should not be started until refreezing can be prevented.<br />

(2) Remove clothing and rewarm affected area in circulating water that is heated to 104°-107.6°F (40°-<br />

420C); rewarming with dry heat can be dangerous because the frozen (insensitive) skin cannot detect<br />

heat, and burns may result.<br />

(3) Initiate general rewarming measures (as appropriate) and rehydration to treat the systemic hypothermia<br />

and dehydration that frequently occur in association with frostbite.<br />

(4) Provide tetanus prophylaxis and analgesia as needed.<br />

(5) Local care after full thawing<br />

(a) Elevate the affected extremity.<br />

(b) Debride or aspirate white and clear blisters; they contain chemical mediators of ischemia (eg,<br />

prostaglandin F 2<br />

and thromboxane A2) that can further damage underlying tissue.<br />

(c) Leave hemorrhagic blisters intact, because debridement can result in extension of injury.<br />

(d) Apply a topical thromboxane inhibitor (eg, aloe vera).<br />

(e) Dress only open wounds and separate frostbitten digits with soft dressings; leave other frostbitten<br />

areas open to air.<br />

(f)<br />

(6) Administer<br />

Avoid early surgical intervention.<br />

(a) Ibuprofen 400 mg orally to inhibit the arachidonic acid cascade and promote fibrinolysis.<br />

(b) Penicillin G 500,000 units IV to prevent streptococcal infection.<br />

B. Hypothermia: core body temperature


ENVIRONMENTAL DISORDERS<br />

(a) Blood pressure, pulse, and cardiac output all increase during the metabolic excitation phase<br />

(90°-95°F [32°-35°Cl), but they all decrease once the temperature falls below 32°C (metabolic<br />

slowing phase).<br />

(b) ECG changes<br />

i. Osborn or "J" waves<br />

Courtesy of Dr. Michael McCrea<br />

ii. T-wave inversion<br />

iii. Prolonged PR, QRS, and QT intervals<br />

iv. Dysrhythmias: risk develops when core temperature is


ENVIRONMENTAL DISORDERS<br />

3. Predisposing factors<br />

a. Environmental exposure<br />

(1) Non immersion hypothermia: most common presentation<br />

(2) Immersion hypothermia: water temperature


ENVIRONMENTAL DISORDERS<br />

5. Staging and management<br />

Table 39: Staging and Management of Accidental Hypothermia*<br />

Stage<br />

HTI<br />

HT II<br />

HT Ill<br />

HT IV<br />

Clinical<br />

Symptoms<br />

Conscious,<br />

shivering<br />

Impaired<br />

consciousness,<br />

not shivering<br />

Unconscious,<br />

not shivering,<br />

vital signs<br />

present<br />

No vital signs<br />

Typical Core<br />

Temperature**<br />

35 to 32°C<br />


ENVIRONMENTAL DISORDERS<br />

(2) Active external rewarming<br />

(a) Apply heat to the body surface with one of the following:<br />

i. Forced-air rewarming (eg, temperature management blanket)<br />

ii. Warming blanket<br />

iii. Immersion in warm water<br />

iv. Radiant heat<br />

(b) This is effective but has potential problems.<br />

i. Cold blood returning from the periphery causes further cooling ("core temperature<br />

afterdrop"), brings lactic acid with it ("rewarming acidosis"), and causes a relative<br />

hypovolemia from peripheral vasodilation ("rewarming shock").<br />

ii. Metabolic demands in the periphery exceed perfusion capabilities. These complications can<br />

be minimized by applying the heat source to the trunk only, and by using this modality in<br />

combination with one or more of the active core rewarming methods described below.<br />

iii. Patients who are submerged in warm water are difficult to monitor.<br />

(3) Active core rewarming<br />

(a) Noninvasive methods<br />

,. Inhalation rewarming: warm humidified oxygen by mask or endotracheal tube at<br />

107 .6°-113°F (42°-45°C)<br />

ii. Heated IV fluids (DSNS is the preferred solution)<br />

(b) Invasive methods<br />

i. Gastric or colonic lavage with warm saline: rarely needed or used<br />

ii. Bladder lavage with warm saline using a Foley catheter: very poor heat transfer<br />

iii. Peritoneal lavage<br />

iv. Extracorporeal rewarming: usually accomplished by hemodialysis, cardiopulmonary bypass,<br />

or arteriovenous or venovenous rewarming<br />

v. Closed thoracic lavage (if extracorporeal membrane oxygenation unavailable): warm saline<br />

is infused through two thoracostomy tubes.<br />

(c) The advantage of active core rewarming is that preferential rewarming of the heart and internal<br />

organs results in improved function and avoidance of problems associated with peripheral<br />

vasodilatation.<br />

(d) Active core rewarming is the modality of choice for rewarming patients with HT Ill and IV and<br />

for those with cardiac instability.<br />

(4) Choice of rewarming method depends on the severity of the hypothermia, its cause and duration,<br />

and the patient's status on presentation to the emergency department. Guidelines follow:<br />

(a) Use multiple methods.<br />

(b) Patients who are still in the excitation phase probably do not need active rewarming.<br />

(c) Patients with life-threatening cardiac dysrhythmias need rapid rewarming; use both noninvasive<br />

and invasive active core rewarming techniques.<br />

(5) Outcome: the underlying or precipitating disease process and its severity are more important<br />

predictors of outcome than the degree of hypothermia on presentation.<br />

(6) Hypothermic patients should not be pronounced dead until after they have been rewarmed to<br />

95°F (35°C) and subsequent resuscitative efforts are still unsuccessful. "No one's dead until they're<br />

warm and dead." However, consider terminating resuscitation earlier for refractory cardiovascular<br />

unresponsiveness in patients with asphyxia (submersion or avalanche), lethal injuries, or serum K+<br />

> 12 mEq/L. (See also hypothermia in the Dysrhythmia section, where the treatment focus is entirely<br />

on the degree of hypothermia and the patient's cardiovascular status, pages 9-10.)<br />

IV. BITES AND STINGS<br />

A. Human bites<br />

1. The most common aerobic pathogens are Streptococcus spp and Staphylococcus aureus. Most infections are<br />

polymicrobial and often involve both aerobes and anaerobes, especially Eikenella corrodens.<br />

2. Intact skin surrounding these bites should be cleansed with a broad-spectrum antimicrobial agent such as<br />

povidine-iodine; if there is a history of sensitivity/allergy to iodine, chlorhexidine may be used (but it is less<br />

655


ENVIRONMENTAL DISORDERS<br />

effective and known to cause corneal ulcers if comes into contact with the eyes). The wound itself should be<br />

meticulously cleansed with copious amounts of sterile saline solution using a high-pressure irrigation technique.<br />

3. Inspect the wound for tooth fragments (from "fight bites") and sharply debride devitalized tissues. Hand<br />

wounds should be radiographed to exclude fractures, foreign bodies, and air in the joint spaces.<br />

4. Suturing<br />

a. Wounds of the head, face, and neck may be sutured (primary closure) if the patient presents within 24<br />

hours of injury and if there is no evidence of infection; if signs of infection are evident or the wound is >24<br />

hours old, it should be left open.<br />

b. All hand wounds should be left open initially; this includes closed fist/"fight bite" injuries (in which the<br />

extensor tendon and its sheath can be inoculated).<br />

c. Most wounds to other areas may be sutured as long as there is no evidence of infection or delay in treatment.<br />

5. Antibiotics are indicated for all human bites<br />

a. High-risk conditions<br />

(1) All hand wounds (strong evidence supporting infection prevention)<br />

(2) Any wound associated with comorbidity<br />

(a) Diabetes<br />

(b) Asplenia<br />

(c) Immune deficiency<br />

(d) Bite inflicted by a hospitalized or institutionalized person<br />

(3) Any wound with a poor blood supply, eg, anterior shin<br />

b. Wound type (laceration, puncture) does not influence the effectiveness of antibiotic prophylaxis (ie, same<br />

for cat, dog, and rabbit bites)<br />

c. Parenteral (IM or IV) antibiotics and possible admission<br />

(1) Wounds >24 hours old that are obviously infected<br />

(2) Wounds involving tendon, joint, or bone<br />

(3) Signs of systemic infection (tachycardia, fever, extensive cellulitis, lymphangitis, etc)<br />

d. Regimens for prophylaxis/treatment (3-5 days)<br />

(1) Provide coverage for 5 aureus, Streptococcus spp, Pasteure/la, Bacteroides, and Eikenella corrodens<br />

(2) E corrodens (a likely pathogen in hand wounds) is resistant to many agents including dicloxacillin,<br />

many firstgeneration cephalosporins, clindamycin, and the aminoglycosides.<br />

(3) Acceptable regimens (first two are best choices)<br />

(a) A second or third-generation cephalosporin (eg, cefuroxime, cefoxitin)<br />

(b) Dicloxacillin plus ampicillin or penicillin<br />

(c) Amoxicillin-clavulanate<br />

(d) Clarithromycin or clindamycin and TMP-SMX or doxycycline (in penicillin-allergic patients)<br />

(4) An aminoglycoside (eg, gentamicin) should be added to one of the above regimens in patients with<br />

diabetes, because these patients have a higher incidence of infection with gram-negative bacilli.<br />

6. Other appropriate measures<br />

B. Dog bites<br />

a. Provide tetanus prophylaxis as indicated.<br />

b. Consider administration of hepatitis B vaccine and immunoglobulin and testing for HIV when appropriate.<br />

1. Remember the mnemonic "RATS"<br />

Rabies<br />

Antibiotics<br />

Ietanus<br />

S.oap<br />

2. Common pathogens include a-hemolytic streptococci, 5 aureus, and (less often) Pasteurella multocida. A<br />

potentially lethal infection is caused by the organism Capnocytophaga canimorsus and occurs most often<br />

in immunocompromised patients (particularly the splenectomized and those with cirrhosis); mortality is<br />

secondary to sepsis, disseminated intravascular coagulation, or meningitis.<br />

3. Most lacerations (especially facial) that are seen within a few hours of injury should be closed primarily after<br />

meticulous irrigation and debridement.<br />

4. Wounds> 12 hours old and hand wounds should be left open initially with follow-up in 3-5 days for delayed<br />

primary closure; the exception is facial wounds, which should be closed primarily.<br />

656


ENVIRONMENTAL DISORDERS<br />

5. Management<br />

C. Cat bites<br />

a. Antibiotic regimens<br />

(1) Prophylaxis with cephalexin, dicloxacillin, or amoxicillin-clavulanate (erythromycin orTMP-SMX<br />

in the penicillinallergic patient) is indicated for all hand wounds and recommended for all puncture<br />

wounds, lacerations, and patients with comorbid conditions.<br />

(2) Treatment for infected wounds is determined by the probable pathogen involved, as follows:<br />

(a) Cephalexin, dicloxacillin, or amoxicillin-clavulanate (erythromycin orTMP-SMX in the penicillinallergic<br />

patient) is indicated for infections occurring >24 hours after injury (Streptococcus or<br />

Staphylococcus are most likely pathogens).<br />

(6) Penicillin VK (TMP-SMX, ciprofloxacin, or clarithromycin in the penicillin-allergic patient) is<br />

indicated for wound infections occurring within 24 hours of injury (Pasteure//a multocida most<br />

likely pathogen).<br />

(c) Penicillin or a cephalosporin (erythromycin in the penicillinallergic patient) is appropriate for<br />

suspected infection with C canimorsus.<br />

1. lmmunocompromised patient who presents with gangrene or severe systemic manifestations<br />

such as sepsis, endocarditis, or cardiopulmonary or acute renal failure<br />

ii. Because this is a potentially life-threatening infection, highrisk patients should be given<br />

prophylactic treatment.<br />

b. Indications for admission and empirical, parenteral antibiotic therapy with penicillin G and nafcillin<br />

(pending culture results)<br />

(1) Wound infection plus any of the following:<br />

(a) Lymphangitis/lymphaden itis<br />

(b) Tenosynovitis<br />

(c) Septic arthritis<br />

(d) Osteomyelitis<br />

(2) Systemic signs (fever, tachycardia, etc)<br />

(3) Injury to tendons, joints, or bone<br />

c. Tetanus prophylaxis should be administered as indicated.<br />

d. Rabies prophylaxis should be considered but is rarely indicated in the United States.<br />

1. Most common pathogen is Pasteure/la multocida; other, less commonly encountered organisms, are similar to<br />

those associated with dog bites (including C canimorsus).<br />

2. Wound closure<br />

a. Puncture wounds and lacerations


ENVIRONMENTAL DISORDERS<br />

4. Cat scratch disease<br />

a. Etiology and epidemiology<br />

(1) Most commonly affects children and adolescents<br />

(2) Causative oraganism is Bartone/la henselae, a small, pleomorphic, gram-negative bacillus<br />

(3) Incubation period is ~ 1 week (range 3 days to 6 weeks).<br />

b. Clinical presentation: characterized by persistent regional lymphadenitis that is often unilateral and<br />

typically involves the lymph nodes of the arms or legs<br />

c. Management<br />

(1) Most cases resolve spontaneously over a period of weeks to months and require only symptomatic<br />

treatment.<br />

(2) Antibiotics are usually reserved for patients with severe or persistent disease. Acceptable choices<br />

include TMP-SMX, gentamicin, ciprofloxacin, and erythromycin.<br />

D. Venomous snake bites<br />

1. Most venomous snake bites in the United States (>98%) are inflicted by pit vipers, and 95% of the bites<br />

involve the extremities.<br />

2. Management<br />

a. The mainstay of therapy is administration of antivenin IV (described below).<br />

b. Establish monitoring and begin aggressive supportive therapy as appropriate.<br />

c. Immobilize the affected extremity to delay systemic absorption of venom; application of a loosely<br />

constricting band proximal to the bite site within 30 minutes of envenomation may also be helpful. Local<br />

therapy (mechanical, suction, wound excision/incision, elastic wrap) is not recommended.<br />

d. Cleanse the wound, and administer tetanus prophylaxis (if indicated).<br />

3. Pit vipers (Crotalidae family)<br />

a. Crotalid venom produces both local and systemic effects; its major effects occur in local tissue, blood<br />

vessels, and blood components.<br />

b. The severity of envenomation and the severity of the bite (rattlesnake > cottonmouth > copperhead)<br />

determine both the need for and the amount of antivenin to be administered.<br />

Table 40: Clinical Parameters Used to Assess Need for and Amount of Antivenin<br />

Parameter Dry Bite Minimal Moderate Severe<br />

Signs Puncture wound Swelling, erythema, Evidence of Entire area or<br />

± swelling ecchymosis spreading limb involved<br />

Symptoms<br />

Focal Slight pain Minimal pain Progressive pain Severe pain<br />

Systemic None None or minimal Present Fulminant<br />

Coagulation Normal Normal Abnormal Abnormal<br />

tests but no bleeding and bleeding<br />

c. The antivenin Crotalidae Polyvalent Immune Fab Ovine not only demonstrates clinical efficacy but<br />

appears to be associated with fewer allergic symptoms and reduced incidence of serum sickness.<br />

However, because it has a short half-life, venom effects can recur, and a repeat dose regimen must<br />

be used.<br />

(1) Minimal to moderate signs/symptoms: 4-6 vials diluted in 100 ml normal saline administered IV<br />

over 1 hour<br />

(2) Moderate to severe signs/symptoms: 6 vials diluted in 100 ml normal saline administered IV over<br />

1 hour<br />

(3) Progression of signs/symptoms: 2 vials in 100 ml normal saline administered IV over 20-30 minutes<br />

(4) Recurrent signs/symptoms 2 vials in 100 ml normal saline every 6 hours x 3 (6 vials total)<br />

d. Patients with dry bites (no signs/symptoms of envenomation and normal laboratory studies) may be<br />

discharged after 8-12 hours of observation (possibly longer depending on the species); all others should<br />

be admitted. One of every five bites is a dry bite.<br />

658


ENVIRONMENTAL DISORDERS<br />

4. Coral snakes (Elapidae family)<br />

a. Coral snakes may be confused with king snakes and milk snakes. Coral snakes have wider bands of black<br />

and have red directly next to a yellow stripe. Remember "Red on yellow, kill a fellow. Red on black,<br />

venom lack."<br />

b. Elapid venom is largely neurotoxic; it has a curare-like effect on neuromuscular transmission and can<br />

produce total flaccid paralysis. It does not cause local edema.<br />

c. Clinical presentation<br />

(1) Signs and symptoms of envenomation may be delayed up to 12 hours.<br />

(2) A change in mental status and cranial nerve dysfunction (eg, diplopia) are the earliest findings to develop.<br />

(3) Respiratory paralysis is the major life threat.<br />

d. Antivenin is available and should be administered early (even in the absence of any signs or symptoms) in<br />

patients who clearly have been bitten.<br />

(1) Because the antivenin is equine-derived, precautions for a possible allergic reaction during<br />

administration must be observed. However, in the case of a positive skin test, antivenin should<br />

be withheld because these patients do well with aggressive supportive measures (eg, prolonged<br />

intubation and ventilation) alone.<br />

(2) Begin with 3 vials; administer 3-5 additional vials if signs and symptoms develop or worsen.<br />

e. All patients with suspected or known envenomation should be admitted for observation or definitive care.<br />

f. Delayed onset of serum sickness, 1-2 weeks after treatment, is the most frequent reaction to both types<br />

of antivenin.<br />

E. Bee stings, ant bites, and other insect bites and stings<br />

1. Immediate concerns: upper airway obstruction, anaphylaxis, and toxic reactions from multiple stings<br />

2. If stingers are present, remove them immediately by whatever means possible. While it has been traditionally<br />

recommended that stingers be removed by scraping rather than forceps, evidence refutes the benefit of this<br />

technique.<br />

F. Black widow spider (Latrodectus mactans)<br />

1. The female spider is more poisonous than the male. The female has a red hourglass shape on the thorax. The<br />

male is much smaller and has a blue, yellow, or white hourglass on the thorax.<br />

2. Clinical presentation (symptoms often wax and wane)<br />

a. Local - a dull, muscle cramping sensation<br />

(1) Upper extremity bite: chest pain<br />

(2) Lower extremity or genitalia bite<br />

(a) Abdominal pain that may simulate an "acute abdomen"; a differentiating feature is the presence<br />

of abdominal rigidity with little or no associated tenderness.<br />

(b) Rectal spasm with normal bowel sounds<br />

b. Systemic - neurotoxic effects (diffuse CNS and peripheral nervous excitation)<br />

(1) Diffuse muscle pain and stiffness<br />

(2) Dizziness and restlessness<br />

(3) Profuse sweating (which may be uniquely focal to one area) and weakness<br />

(4) Difficulty in speaking and ptosis<br />

3. Management<br />

(5) Hypertension and tachycardia<br />

a. First aid: ice pack to the bite area<br />

b. Treatment<br />

(1) Cleanse with soap and water (look for fang marks and halo lesions)<br />

(2) Tetanus toxoid<br />

(3) Disposition<br />

(a)<br />

1 hour if the spider not positively identified and the patient remains symptom-free during this time<br />

(b) 4 hours if the spider positively identified but the patient remains symptom-free during this time<br />

c. Specific therapeutic measures<br />

(1) Signs and symptoms may be treated with:<br />

(a)<br />

Narcotic analgesics (the mainstay of therapy and pain relief)<br />

(b) Benzodiazepines (relieve muscle spasms)<br />

659


ENVIRONMENTAL DISORDERS<br />

(2) Use of antivenin (one vial is usually sufficient)<br />

(a)<br />

Antivenin is not used in all cases because:<br />

1. The effects of black widow spider bites are self-limited with a low mortality rate; because use<br />

of antivenin has been associated with death, most patients are not given antivenin.<br />

ii. The antivenin is equine-derived (pretesting for horse serum sensitivity is advised) and can<br />

therefore produce anaphylaxis. Serum sickness can also occur but is uncommon, because so<br />

little antivenin is used.<br />

iii. The antivenin, when <strong>combined</strong> with -adrenergic blockers, can produce anaphylactic<br />

reactions that are refractory to treatment.<br />

(b) Indications for antivenin<br />

i. Patients 65 years old<br />

ii. Patients with severe pain (despite analgesics) or severe envenomations<br />

iii. Patients unable to stand the stress of the envenomation (due to concurrent illness)<br />

iv. Patients with dangerous hypertension<br />

v. Pregnant women<br />

G. Brown recluse spider (Loxosceles reclusa)<br />

1. The brown recluse is the size of a penny, and the head has a fiddle-shaped black marking.<br />

2. Clinical presentation<br />

a. Local pain develops at the site in 3-4 hours.<br />

(1) The lesion starts with a halo of vasoconstriction, which then develops a central bleb (the classic bull'seye<br />

lesion), which may continue to spread over several days.<br />

(2) Tissue necrosis is a major complication that may require surgical consultation.<br />

b. Systemic (\oxoscilism): thought to be an allergic reaction<br />

(1) Develops over 24-72 hours<br />

(2) Fever, chills, petechial rash, nausea, vomiting, weakness<br />

(3) May progress to hemolysis, shock, renal failure, disseminated intravascular coagulation<br />

(4) Most fatalities are in children.<br />

3. Local tissue necrosis is a major complication that may require surgical consultation.<br />

4. Management<br />

a. Wash wound with soap and water.<br />

b. Apply ice compresses locally to decrease pain and diminish the evolution of cutaneous inflammation.<br />

c. Administer tetanus toxoid.<br />

d. Observe the patient in the emergency department if elapsed time since the bite is


ENVIRONMENTAL DISORDERS<br />

I. Jellyfish stings<br />

(1) Symptomatic therapy (ice packs to area, analgesics, sedation) alone is adequate for grades I and II.<br />

(2) Antivenin may be required for grades Ill and IV; the antivenin is goat-serum derived, carries the risk<br />

of anaphylaxis and serum sickness, and is available only in Arizona (the Arizona bark scorpion is the<br />

most deadly scorpion in North America); skin testing is advised.<br />

1. Jellyfish envenomate their victims via stinging cells called nematocysts (cnidocysts).<br />

2. Victims of many severe toxic jellyfish stings (especially the Inda-Pacific box jellyfish) do not survive to shore;<br />

some succumb to the direct effects of the venom, but many drown due to the cardiopulmonary complications<br />

from the envenomation. Problems with survivors include removal of nematocysts, pain control, and prevention<br />

of infection.<br />

3. Treatment is controversial and unproven (therefore, less likely to be on the exam).<br />

a. Remove patient from the water.<br />

b. Wash the area with sea water; do not use fresh water, because it may cause the nematocysts to discharge,<br />

which may then cause severe pain.<br />

c. Do not rub the area with sand, because this worsens the condition.<br />

d. Remove tentacles with forceps or a thickly gloved hand.<br />

e. Any nematocysts that remain after washing should be fixed by pouring vinegar (or isopropyl alcohol)<br />

over the wound area for at least 30 minutes. Next, dust the area with talcum powder or cover with shaving<br />

cream. The nematocysts, now adherent to the powder or cream, can be scraped off with a knife or razor<br />

blade.<br />

f. Wash the area with sea water once again and then apply a topical anesthetic/antihistamine or low-potency<br />

steroid cream (which may be soothing).<br />

g. Administer tetanus toxoid.<br />

h. Allergic reactions are common and should be treated in the usual manner.<br />

4. Hospitalize patients who sustain severe envenomation or develop an allergic reaction.<br />

V. ALTITUDE DISORDERS<br />

A. High-altitude disorders (can be avoided by proper acclimatization or use of acetazolamide 1-2 days before<br />

ascent)<br />

1. Acute mountain sickness<br />

a. Clinical presentation<br />

(1) Onset of symptoms is within a few hours; duration is 3-4 days.<br />

(2) Symptoms include bifrontal headache, anorexia, nausea, and sleeping difficulties. The headache is<br />

usually worsened with the Valsalva maneuver and with stooping over; it is most severe during the<br />

night and in the morning on awakening.<br />

b. Management<br />

(1) Symptomatic treatment and a halting of the ascent for 12-36 hours until acclimatization has<br />

occurred is usually all that is needed in mild cases.<br />

(2) Effective agents include NSAIDs or acetaminophen for headache and prochlorperazine for nausea and<br />

vomiting. Prochlorperazine also has the advantage of increasing the hypoxic ventilatory response.<br />

(3) Supplemental low-flow oxygen is useful in relieving symptoms, especially at night, during sleep.<br />

(4) Acetazolamide 125-250 mg orally bid speeds acclimatization and is also effective for treatment of<br />

acute mountain sickness; it is contraindicated in patients allergic to sulfa.<br />

(5) Dexamethasone 4 mg orally, IM, or IV qid is also effective for prophylaxis as well as treatment of<br />

acute mountain sickness but, because of adverse effects and problems with rebound when the drug<br />

is stopped, it is recommended only as treatment (not prophylaxis) and is reserved for moderate to<br />

severe cases.<br />

(6) Descent is the definitive therapy and is rapidly effective but is not necessary unless symptoms<br />

progress or are prolonged and debilitating.<br />

2. High-altitude pulmonary edema (HAPE)<br />

a. This is noncardiogenic (wet lungs and normal heart size) in origin; it should not be confused with CHF.<br />

b. Clinical presentation<br />

661


ENVIRONMENTAL DISORDERS<br />

(1) Onset of signs and symptoms is 1-4 days after ascent and often preceded by exertion.<br />

(2) Symptoms include weakness, headache, cough, shortness of breath, cyanosis, and rales.<br />

c. Management<br />

(1) Bedrest, oxygen, increased positive-airway pressure (via CPAP, BiPAP, or endotracheal intubation)<br />

and descent<br />

(2) If descent is impossible:<br />

(a) The Gamow bag (portable hyperbaric chamber) simulates descent to a lower altitude.<br />

(b) Nifedipine may be helpful, because it reduces pulmonary artery pressure through its vasodilatory<br />

effect.<br />

3. High-altitude cerebral edema (HACE)<br />

B. Dysbarism<br />

a. Can result in permanent neurologic injury or even death<br />

b. Clinical presentation<br />

(1) Common signs/symptoms: confusion, ataxia, retinal hemorrhages, hallucinations, and headache<br />

(2) Can progress to coma if untreated<br />

c. Management<br />

(1) Oxygen, head elevation, descent, and dexamethasone; hyperbaric therapy is also effective if available.<br />

(2) Loop diuretics (eg, furosemide, bumetanide) may be helpful in the treatment of both HAPE and HACE,<br />

but hypoperfusion is a significant risk so they should be used cautiously, if at all.<br />

1. Barotrauma of descent ("squeeze") based on Boyle's Law: the volume of a gas varies inversely with the pressure.<br />

a. Symptoms develop because of compression of air that is trapped in various parts of the body during<br />

descent. Divers can develop symptoms in water as shallow as 4.5-6.5 feet.<br />

b. Barotitis externa ("external ear squeeze")<br />

(1) Occurs when the external auditory canal is occluded (eg, by cerumen, foreign bodies, exostoses, ear<br />

plugs, or a too-tight fitting wet suit hood)<br />

(2) Compression of the enclosed air with descent produces pain and/or bloody otorrhea (from bloodfilled<br />

cutaneous blebs along the canal or tympanic membrane rupture).<br />

(3) Management: keep canal dry; no diving (or swimming) until healed.<br />

c. Barotitis media ("middle ear squeeze")<br />

(1) Most common type of aural barotrauma<br />

(2) Results from failure to equalize the pressure between the middle ear and the water; this occurs<br />

because of occlusion or dysfunction of the eustachian tube.<br />

(3) Predisposing factors<br />

(a) Mucosal congestion secondary to an upper respiratory infection, allergies, or smoking<br />

(b) Mucosal polyps<br />

(c) Excessively vigorous autoinflation maneuvers<br />

(d) Previous maxillofacial trauma<br />

(4) As the pressure differential increases, the diver experiences ear fullness or pain (which will worsen<br />

until the tympanic membrane ruptures, unless the dive is aborted); if the tympanic membrane does<br />

rupture, the diver may become disoriented because of severe nausea and vertigo that result from the<br />

caloric stimulation of cold water entering the middle ear.<br />

(5) The severity of the injury can be assessed from the amount of hemorrhage associated with the<br />

eardrum; a grading system may be used, with "O" signifying no hemorrhage (only symptoms) to "5"<br />

indicating tympanic membrane rupture with gross hemorrhage.<br />

(6) Management: no diving and nasal decongestants until healed; if the tympanic membrane is ruptured,<br />

antibiotics are indicated; resolution occurs within 1 week.<br />

d. Barotitis interna ("internal ear squeeze")<br />

(1) Permanent injury to the cochleovestibular system may occur.<br />

(2) Results from sudden pressure differences between the internal and middle ear, which may occur<br />

after an overzealous Valsalva maneuver or extremely rapid descent.<br />

(3) Clinical presentation: classic symptomatology - tinnitus, deafness, and vertigo<br />

(4) Mechanisms of injury: inner ear hemorrhage, rupture in Reissner membrane - mixing of endolymph<br />

and perilymph, round or oval window fistulation, or a combination of these insults<br />

662


ENVIRONMENTAL DISORDERS<br />

(5) Management: bed rest with the head elevated, sedentary activities as tolerated, symptomatic therapy,<br />

and ENT referral; early surgical intervention is indicated for patients with total (or near total) hearing<br />

loss.<br />

e. "Sinus squeeze"<br />

(1) Hemorrhage or sensation of fullness, pressure, or pain in the affected sinuses<br />

(2) The maxillary and frontal sinuses are most often affected.<br />

(3) Antibiotics are usually indicated if the frontal sinuses are involved; otherwise, the treatment is the<br />

same as for barotitis media.<br />

2. Barotrauma of ascent (Boyle's Law)<br />

a. Symptoms develop because of expansion of air that is trapped in bodily spaces during ascent.<br />

Bare's (Scuba) law: Mr. Boyle Breaths out on ascent Before Bubbles in Bad places cause Barotrauma by<br />

Bursting Big organs and little eardrums.<br />

b. Examples<br />

(1) Middle ear and sinus barotrauma of ascent ("reverse squeeze")<br />

(2) Barodontalgia<br />

(3) Aerogastralgia (air in the gut)<br />

(4) Pulmonary barotrauma (Boyle's Law)<br />

(a) Mediastinal or subcutaneous emphysema (most common)<br />

(b) Pulmonary interstitial emphysema<br />

(c) Alveolar hemorrhage<br />

(d) Pneumothorax<br />

(e) Pulmonary air (arterial gas) embolism<br />

i. The most severe complication of pulmonary barotrauma<br />

ii. Immediately after ascent, air bubbles enter the systemic circulation through ruptured<br />

pulmonary veins.<br />

iii. Arterial occlusion from an air embolus can occur at any site, including the coronary arteries<br />

(Ml, cardiac arrest). However, the most common presentation is a neurologic event, eg,<br />

level of consciousness, blindness, deafness, vertigo, confusion, seizures, monoplegia, or<br />

asymptomatic multiplegia.<br />

iv. Symptoms develop within 10 minutes of surfacing from a dive, and often much sooner.<br />

v. Treatment of air embolism (pulmonary, coronary, or cerebral) is recompression in a<br />

hyperbaric chamber as soon as possible. During transport, the patient should be given<br />

oxygen and placed in the supine position. Placement in Trendelenburg is no longer<br />

recommended, because it may worsen cerebral edema and respiratory distress.<br />

3. Barotrauma of descent or ascent is the result of a direct effect of pressure changes on the body (Boyle's<br />

Law). Nitrogen narcosis and decompression sickness, on the other hand, are the result of an indirect effect of<br />

pressure changes on the body (Henry's and Dalton's Laws).<br />

a. Henry's Law: The amount of gas dissolved in a liquid is proportional to the partial pressure of the gas in<br />

contact with the liquid.<br />

b. Dalton's Law: The partial pressure of a gas increases with increasing pressure.<br />

c. Nitrogen narcosis and decompression sickness (DCS) occur secondary to breathing gases (usually<br />

nitrogen) at higher than normal atmospheric pressure.<br />

(1) Nitrogen narcosis occurs as the diver descends to depths >70 feet. The clinical picture resembles<br />

alcoholic intoxication and can impair judgment; the condition clears with resurfacing.<br />

(2) DCS occurs if the diver ascends too quickly. Symptoms result from dissolved inert gas (nitrogen)<br />

bubbles reentering tissues and blood vessels; it can involve any organ system. There are two main<br />

types of DCS:<br />

(a) DCS type 1<br />

1. Cutaneous - "skin bends"<br />

ii. Musculoskeletal, most frequently affected ----,► "the bends"<br />

• Delayed and insidious in onset<br />

• Often symptoms may not manifest for hours to days<br />

(b) DCS type 2: more serious and involves cardiopulmonary and nervous symptoms<br />

i. Pulmonary --;, "the chokes"<br />

663


ENVIRONMENTAL DISORDERS<br />

II, CNS<br />

• Spinal cord, most frequently affected -<br />

• Cerebellar -<br />

• Cerebral<br />

"the staggers"<br />

acute paraplegia<br />

(c) Treatment is recompression in a hyperbaric chamber as soon as possible. During the transport,<br />

100% oxygen should be administered to provide a favorable gradient for nitrogen washout and<br />

improved oxygenation of injured tissues. No flying for at least 3 days with DCS type 1, and no<br />

flying for 3-4 weeks after treatment of DCS type 2.<br />

VI. SUBMERSION INJURY<br />

A. Definitions<br />

1. Drowning is defined as a process resulting in primary respiratory impairment from submersion in a liquid<br />

medium. A liquid-air interface must be present in the victim's airway.<br />

2. The outcome of drowning may include:<br />

a. Delayed morbidity<br />

b. Delayed death<br />

c. Life without morbidity<br />

3. Immersion syndrome is sudden death after submersion in very cold water (presumed to be due to vagally<br />

mediated asystole or ventricular fibrillation).<br />

4. Postimmersion syndrome is delayed deterioration of a patient who is initially relatively asymptomatic;<br />

deterioration is generally due to respiratory insufficiency and may be delayed several hours to several days<br />

(ARDS). This is the major complication of submersion injury.<br />

B. Epidemiology<br />

1. Drowning is the second leading cause of death in children and the third most common cause of accidental<br />

death in the United States, claiming 8,000-9,000 lives/year, with brain injury being the major cause of death<br />

from drowning.<br />

2. Freshwater submersion injury (particularly in swimming pools) is more common than saltwater submersion<br />

injury, but the type of water has no bearing on morbidity and mortality.<br />

3. Age distribution is bimodal, two-thirds under the age of 30, with peaks occurring in children


ENVIRONMENTAL DISORDERS<br />

C. Pathophysiology<br />

1. Sequence of events (are essentially the same for freshwater and saltwater submersion)<br />

Panic and air hunger<br />

•<br />

Breath-holding<br />

•<br />

Air hunger<br />

+<br />

•<br />

Involuntary inspiratory effort forcing victim<br />

to inhale and swallow<br />

I<br />

+<br />

Aspiration ("wet drowning")<br />

85%-90%<br />

t<br />

•Direct pulmonary injury<br />

•Surfactant loss-+ atelectasis,<br />

V/Q mismatch, alveolar capillary<br />

membrane breakdown<br />

(Fresh water also changes surfactant<br />

surface tension)<br />

• Inflammatory contaminants<br />

(bacteria, emesis, chemical irritants)<br />

•Cerebral hypoxia<br />

+<br />

Noncardiogenic pulmonary edema<br />

Laryngospasm and glottis closure<br />

without aspiration<br />

("dry drowning")<br />

10% - 15%<br />

I<br />

i<br />

Profound hypoxemia<br />

(final common pathway)<br />

t<br />

Cardiac arrest<br />

Sequence of Events in Submersion Injury<br />

2. Other pathophysiologic characteristics of submersion injury<br />

a. Metabolic acidosis (lactic acidosis type A) develops in many patients and is due to hypoxemia (anaerobic<br />

metabolism) and poor perfusion.<br />

(1) It is usually well tolerated in children and may be reversed with adequate ventilation and oxygenation.<br />

(2) It is less well tolerated in adults.<br />

665


ENVIRONMENTAL DISORDERS<br />

(a)<br />

In addition to ventilation and oxygenation, small doses of NaHCO 3<br />

may be considered for<br />

refractory acidosis.<br />

(b) Lactic acidosis persists if severe hypoxemia is not corrected.<br />

b. Most victims do not aspirate enough fluid to cause life-threatening changes in blood volume or serum<br />

electrolyte concentrations.<br />

c. Renal failure occurs infrequently in submersion injury.<br />

(1) Acute tubular necrosis may develop secondary to hypoxemia.<br />

(2) In freshwater submersion injury, acute renal failure may result from hemolysis and hemoglobinuria.<br />

d. Cerebral edema may develop (usually within 6-12 hours) from generalized neuronal death. Cerebral<br />

edema-"' i intracranial pressure -"' t cerebral perfusion pressure and t cerebral blood flow -"' further<br />

ischemic neurologic injury<br />

D. Prehospital care<br />

1. Ineffective and potentially dangerous procedures<br />

a. In-water CPR (but can do airway management in water)<br />

b. Attempts to remove fluid from the lungs via postural drainage or the Heimlich maneuver<br />

2. Initial assessment and stabilization<br />

a. If patient is conscious, safer to throw flotation device to the victim than to jump in: "throw, tow, row, go."<br />

b. Start CPR in the pulseless, apneic victim even if there is only a remote possibility of success.<br />

c. Airway maintenance and supplemental oxygen are of primary importance.<br />

d. An IV line should be started if possible.<br />

e. Immobilize the cervical spine and look for signs of spinal injury:<br />

(1) Paradoxical respirations<br />

(2) Unexplained hypotension<br />

(3) Bradycardia<br />

(4) Flaccidity<br />

(5) Priapism<br />

f. Prevent onset or worsening of hypothermia: remove wet clothes, dry the patient, and wrap in dry blankets.<br />

3. Transport all submersion-injury victims to the hospital.<br />

E. Emergency department<br />

1. Remember that submersion injury is an airway problem first. Evaluate airway patency, adequacy of cervical<br />

spine immobilization, and mental status; naloxone and glucose (if hypoglycemic) are indicated if there is an<br />

altered level of consciousness.<br />

2. Assess respiratory status, looking for signs of pulmonary insufficiency:<br />

a. Tachypnea<br />

b. Dyspnea<br />

c. Use of accessory muscles<br />

d. Wheezing, rales, or rhonchi<br />

3. Provide supplemental oxygen.<br />

a. Always use 100% oxygen.<br />

b. Avoid prolonged suctioning, steroids, prophylactic antibiotics, and early extubation.<br />

c. Intubation is indicated if the pO 2<br />

is


ENVIRONMENTAL DISORDERS<br />

6. Correct shock (if present) with fluid boluses; if this is unsuccessful, use an inotrope (eg, norepinephrine).<br />

7. Maintain cervical spine immobilization until cervical and first thoracic vertebra have been cleared by imaging.<br />

8. Decompress the stomach with a nasogastric tube.<br />

9. Place Foley catheter to monitor urine output.<br />

10. Conduct a thorough search for associated injuries.<br />

11. Assess the patient's temperature to exclude hypothermia (which is not uncommon and changes the therapeutic<br />

approach).<br />

a. Use a rectal thermometer that is specially calibrated to record low temperatures; most clinical<br />

thermometers only read down to 95°F (35°C).<br />

(1) Patients who have been submerged in icy cold water (:S41 °-50°F [5°-10°C]) for >40 minutes have<br />

survived with a good neurologic outcome.<br />

(2) Cold water slows the metabolic rate and shunts blood to the brain, heart, and lungs (diving reflex).<br />

b. Institute appropriate rewarming measures for hypothermia (see page 654-655).<br />

c. Extracorporeal membrane oxygenation (if available) is indicated for patients with severe hypothermia and/<br />

or hypoxia.<br />

d. Continue resuscitation efforts until a near-normal core temperature (95°F [35°C]) is attained.<br />

12. Diagnostic evaluation<br />

a. Cervical spine imaging should be done if there is any suspicion of possible cervical injury.<br />

b. Arterial blood gases<br />

c. Chest radiograph (50% of patients with abnormal radiographs will require intubation). Typically, one of<br />

three patterns is seen:<br />

(1) Normal (although this does not necessarily mean there is no lung pathology)<br />

(2) Perihilar pulmonary edema<br />

(3) Generalized pulmonary edema<br />

d. CBC and urinalysis<br />

e. Glucose, electrolytes, prothrombin time (INR)/partial thromboplastin time<br />

f. Blood alcohol level and drug screen<br />

g. ECG<br />

13. Do not administer prophylactic antibiotics or steroids; neither has been shown to change the course of<br />

aspiration pneumonia in submersion-injury victims.<br />

F. Predicting outcome<br />

1. The most reliable predictors of outcome are the duration of both submersion and resuscitation.<br />

a. Prognostic indicators of a good outcome<br />

(1) Short submersion time<br />

(2) Basic and/or advanced life support at the scene<br />

(3) Favorable response to initial resuscitative efforts (return of spontaneous respiration)<br />

(4) Alert on admission<br />

(5) Older child (c::3 years old) or adult<br />

(6) Water temperature 41 °-50°F (5-1 0°C)<br />

b. Prognostic indicators of a poor outcome<br />

(1) Submersion duration >25 minutes<br />

(2) Cardiac arrest requiring>25 minutes advanced life support<br />

(3) Ongoing CPR in the emergency department<br />

(4) Fixed, dilated pupils in the emergency department<br />

(5) pH


ENVIRONMENTAL DISORDERS<br />

c. Patients who are comatose and have an abnormal response to pain and abnormal respirations have a more<br />

variable outcome; although some survive intact if appropriate care is provided, many die or survive with<br />

anoxic encephalopathy. These patients were generally submerged for a prolonged period of time.<br />

3. Pulmonary injury and hypoxia are the primary pathophysiologic determinants of outcome.<br />

G. Disposition<br />

1. Asymptomatic patients should be observed for 6 hours. If they remain asymptomatic, are oxygenating normally<br />

on room air, and have a normal chest radiograph, they may be discharged. Early follow-up should be arranged.<br />

2. Hypoxemic and/or symptomatic patients should be hospitalized.<br />

H. Postimmersion syndrome risk factors<br />

1. Severe transient hypoxia<br />

2. History of unconsciousness in the water<br />

3. Presence of symptoms such as dyspnea, coughing, and tachypnea<br />

4. Underlying cardiopulmonary disease<br />

VII. OTHER ENVIRONMENTAL HAZARDS<br />

A. Radiation injuries<br />

1. Pathophysiology: radiation - ionization - formation of free radicals from water - breakage of DNA and<br />

RNA strands<br />

2. The LD 50<br />

of whole-body ionizing radiation (the dose that will kill 50% of those exposed to it) is 350 rad (3.5 Gy).<br />

3. The whole-body dose of ionizing radiation determines the timing of the onset of symptoms, severity of the<br />

illness, and organ systems involved.<br />

a. The higher the exposure, the earlier symptoms develop: 400 rads), >6 hours (500 rads<br />

(3) The CNS is the most resistant to radiation effects; exposure >5,000 rads is required to damage this<br />

system.<br />

4. The best predictor of survival is the absolute lymphocyte count 48 hours after radiation exposure.<br />

a. > 1,200/mm 3 = good prognosis<br />

b. 300-1,200/mm 3 = fair prognosis<br />

c.


ENVIRONMENTAL DISORDERS<br />

C. Volcanic eruptions<br />

1. Mortality<br />

a. Immediate: ash in the upper airways - suffocation<br />

b. Delayed: ash (or other particulate matter) in the lungs - ARDS<br />

2. Morbidity<br />

a. Increased incidence of new-onset asthma<br />

b. Prolonged inhalation of volcanic ash - silica pneumoconiosis<br />

669


ENVIRONMENTAL DISORDERS: PRACTICE CLINICAL SCENARIOS<br />

ENVIRONMENTAL DISORDERS: PRACTICE CLINICAL SCENARIOS<br />

Answers immediately follow the practice clinical scenarios.<br />

Scenario A<br />

Presentation: A middle-aged antique dealer presents complaining of hand discoloration and weakness. His<br />

symptoms started after using a rust and glass cleaner on some newly purchased glass sculptures and metal<br />

etchings. Laboratory studies are normal except for hypocalcemia.<br />

Source: Dr. Watchorn (Wikimedia Commons: http://en.wikipedia.org/wiki/File:61569264_jamesheilman-224x2991.jpg)<br />

What is the diagnosis?<br />

Scenario B<br />

Presentation: A 4-year-old boy bites a household electrical cord and receives burn care at a local urgent<br />

care that day. Three days later, his parents bring him to the local emergency department with profuse lip<br />

bleeding with arterial pulsations. ABCs are started, and an emergent plastic surgery consult is obtained.<br />

What is the diagnosis?<br />

670


ENVIRONMENTAL DISORDERS: PRACTICE CLINICAL SCENARIOS<br />

Scenario C<br />

Presentation: A grandfather flys a kite with his grandson on a warm summer day when a storm rolls in.<br />

The grandfather attempts to rapidly bring down the kite when he is hit by lightning. He thinks he tripped<br />

while bringing the kite down but only recalls waking up on a grassy knoll. He is asymptomatic other than<br />

Lichtenberg figures.<br />

Source: United States Department of Health and Human Services (Wikimedia: http://commons.wikimedia.org/wiki/File:Rear_<br />

view _of _a_l ightn i ng-stri ke_su rvivor,_di spl ayi ng_Lichten berg_figu re_on_ski n .png)<br />

What is the diagnosis?<br />

Scenario D<br />

Presentation: A licensed California marijuana farmer brings in a dead snake. He states the snake bit him<br />

on the hand while he was trying to harvest his crop. The snake has wide red bands next to narrow black<br />

bands.<br />

What is the diagnosis?<br />

Scenario E<br />

Presentation: An emergency medicine physician from Ohio takes a morning flight into Denver after<br />

working all night. He and his wife drive immediately to a large ski resort a few hours outside of Denver. He<br />

begins to notice severe headache and nausea as he rides up the chairlift. His symptoms seem to improve as<br />

he descends the slope and recur as he rides back up the chairlift.<br />

What is the diagnosis?<br />

671


ENVIRONMENTAL DISORDERS: PRACTICE CLINICAL SCENARIOS<br />

ANSWERS TO PRACTICE CLINICAL SCENARIOS<br />

Scenario A<br />

Diagnosis: hydrofluoric acid burn<br />

Scenario B<br />

Diagnosis: electrical lip burn<br />

Management: Acutely, do not debride these wounds. Cleanse and apply a petroleum-based antibiotic<br />

ointment. Observe closely and refer to a plastic or oral surgeon for splinting and further care.<br />

Complications include delayed hemorrhage from the labial artery (10%-15% of patients) 3-14 days after<br />

injury, when the eschar separates, which is what occurred in this patient.<br />

Scenario C<br />

Diagnosis: lightning strike<br />

Scenario D<br />

Diagnosis: snake bite: "red on black, venom lack"<br />

LOUISIANA MILK<br />

TEXAS CORAL<br />

SNAKE<br />

Red and Black<br />

VENOM LACK<br />

'<br />

■<br />

■<br />

,<br />

red I black I yellow I black I red<br />

Red and Yellow<br />

KILL A FELLOW<br />

I<br />

yellow I red I black I red I yellow<br />

Scenario E<br />

Diagnosis: acute mountain sickness<br />

672


PSYCHOBEHAVIORAL DISORDERS<br />

PSYCHOBEHAVIORAL DISORDERS<br />

Clinical Approach to the Psychiatric Patient ............................................................................................................. 677<br />

Recognition ........................................................................................................................................................ 677<br />

Triage ................................................................................................................................................................. 677<br />

Safety ................................................................................................................................................................. 677<br />

Medical Evaluation ............................................................................................................................................. 678<br />

The Psychiatric Interview .................................................................................................................................... 680<br />

Treatment .................................................................................................................................................................. 681<br />

Management of the Agitated Patient ................................................................................................................... 681<br />

Crisis Intervention .............................................................................................................................................. 685<br />

Psychopharmacology ......................................................................................................................................... 686<br />

Specific Behavioral Disorders ................................................................................................................................... 692<br />

Psychosis ............................................................................................................................................................ 692<br />

Depression ......................................................................................................................................................... 693<br />

Suicide ............................................................................................................................................................... 6 94<br />

Mania and Hypomania ....................................................................................................................................... 696<br />

Catatonia ............................................................................................................................................................ 697<br />

Panic Attacks/Disorder ........................................................................................................................................ 698<br />

Somatoform Disorders ........................................................................................................................................ 699<br />

Insomnia and Sleep Disorders ............................................................................................................................ 700<br />

Eating Disorders ................................................................................................................................................. 701<br />

Intoxication and Withdrawal .............................................................................................................................. 703<br />

Delirium and Dementia ...................................................................................................................................... 705<br />

"Medical Mimickers" of Psychiatric 11 lness ......................................................................................................... 707<br />

673


PSYCHOBEHAVIORAL DISORDERS: SELF-ASSESSMENT QUESTIONS<br />

PSYCHOBEHAVIORAL DISORDERS: SELF-ASSESSMENT QUESTIONS<br />

1. Which of the following is not an adverse effect of anti psychotics?<br />

(a) Hypotension<br />

(b) Bradycardia<br />

(c) Dystonic reactions<br />

(d) Lowered seizure threshold<br />

2. A depressed 20-year-old woman arrives in the emergency department complaining of headache, chest tightness,<br />

and palpitations. Examination reveals tachycardia, increased blood pressure, and diaphoresis. She mentions that<br />

she is taking a medication for depression but cannot recall its name. She states that she started to feel ill shortly after<br />

taking an OTC decongestant/cough suppressant for her cold symptoms. These would be signs of a potentially lethal<br />

drug-drug interaction if the patient's antidepressant were a:<br />

(a)<br />

(b) SSRI<br />

Benzodiazepine<br />

(c) Monoamine oxidase inhibitor<br />

(d) Tricyclic antidepressant<br />

3. The patient most likely to benefit from crisis intervention is the patient with:<br />

(a) An acute grief reaction<br />

(b) Delirium<br />

(c) Agoraphobia<br />

(d) Seasonal affective disorder<br />

4. Which of the following statements regarding conversion reactions is inaccurate?<br />

(a) They have a sudden onset and are often triggered by an emotionally charged event.<br />

(b) The process is unconscious; the patient is not malingering.<br />

(c) Symptoms attributed to conversion reactions are often later found to be due to an occult general medical disorder.<br />

(d) The symptoms often involve involuntary muscle functions.<br />

5. Which of the following medications has depression as an adverse effect?<br />

(a)<br />

Propranolol<br />

(b) Furosemide<br />

(c) Sumatriptan<br />

(d) Atorvastatin<br />

6. A 55-year-old man presents to the emergency department with depression and suicidal threats. He recently had a<br />

cholecystectomy. He is a veteran and has a history of drug abuse, and has chronic low back pain due to an injury.<br />

His brother committed suicide 10 years ago. He lives at home with his wife. Which of the following is not a risk<br />

factor for suicide for this patient?<br />

(a)<br />

Recent surgery<br />

(b) Drug and alcohol abuse<br />

(c) Chronic pain<br />

(d) Family history of suicide<br />

7. A 28-year-old man presents to the emergency department by police after being picked up on the street where he<br />

was walking around on a crowded sidewalk threatening other pedestrians. He has been handcuffed and is carried<br />

in while he tries to kick and bite the police who are escorting him. The most appropriate medical treatment for his<br />

behavior is:<br />

(a) Chlorpromazine and diazepam<br />

(b) Haloperidol and lorazepam<br />

(c) Thioridazine and midazolam<br />

(d) Droperidol and clonazepam<br />

674


PSYCHOBEHAVIORAL DISORDERS: SELF-ASSESSMENT QUESTIONS<br />

8. Unlike the hallucinations that occur in patients with delirium due to an underlying medical condition, those<br />

occurring in patients with a psychosis (eg, schizophrenia) are usually:<br />

(a) Tactile and painful<br />

(b) Auditory and frightening<br />

(c) Visual and colorful<br />

(d) Olfactory and noxious<br />

9. Which of the following is not considered a symptom of depression?<br />

(a)<br />

Feelings of worthlessness or guilt<br />

(b) Pressured speech<br />

(c) Sleep disturbances<br />

(d) Weight loss, weight gain, or change in appetite<br />

10. Which of the following statements regarding antidepressants is accurate?<br />

(a)<br />

Routine prescription of these agents to patients evaluated in the emergency department for depression is<br />

appropriate.<br />

(b) SSRI antidepressants produce significant cardiotoxicity.<br />

(c) SSRls may influence other drug levels through cytochrome P450 pathways.<br />

(d) Second-generation antidepressants (venlafaxine, duloxetine, mirtazapine) exert their mechanism of action<br />

through dopamine and norepinephrine effects.<br />

11. An otherwise healthy 23-year-old woman presents to the emergency department with palpitations, shortness<br />

of breath, and feeling of panic associated with pain in her chest. Which of the following is false regarding her<br />

condition?<br />

(a) Symptoms include apprehension, shortness of breath, difficulty swallowing, chest tightness, and palpitations.<br />

(b) Patients often believe they have a medical problem.<br />

(c) Attacks are sudden in onset and last, on average,


PSYCHOBEHAVIORAL DISORDERS: SELF-ASSESSMENT QUESTIONS<br />

15. A 20-year-old woman presents with agitation, hallucinations, and abdominal pain. Her abdominal examination is<br />

benign, but she continues to be preoccupied with the pain. Her physician made a psychiatric referral and prescribed<br />

clonazepam for her anxiety, but her condition rapidly worsened and she became increasingly distressed by her<br />

abdominal pain before the psychiatric evaluation could be arranged. The most likely diagnosis is:<br />

(a) Manic psychosis<br />

(b) Acute intermittent porphyria<br />

(c) Somatoform pain disorder<br />

(d) Hypocalcemia<br />

ANSWERS<br />

1. b 4. d 7. b 10. c 13. a<br />

2. c 5. a 8. b 11. d 14. d<br />

3. a 6. a 9. b 12. d 15. b<br />

Use the pre-chapter multiple choice question worksheet (page xvi) to record and determine the percentage of correct<br />

answers for this chapter.<br />

676


PSYCHOBEHAVIORAL DISORDERS<br />

I. CLINICAL APPROACH TO THE PSYCHIATRIC PATIENT<br />

A. Recognition<br />

1. A psychiatric emergency arises when a patient poses a danger to others/self or is gravely disabled. The situation<br />

is complex and often characterized by intense symptoms, a sense of urgency, and perceived danger.<br />

2. The patient's primary problem (depression, psychosis) may be identified by himself/herself, family or friends, or<br />

others who are aware of the patient's abnormal behavior.<br />

3. Patients may present with physical symptoms that are manifestations of a psychiatric disorder, eg, a depressed<br />

patient may present with insomnia, weight loss, and somatic complaints.<br />

4. Patients may present with psychiatric complaints that are caused or exacerbated by general medical syndromes<br />

or medications/substances (eg, psychosis due to corticosteroids or phencyclidine [PCP] ingestion).<br />

5. Patients may present with a general medical problem or injury that was caused by an underlying psychiatric<br />

disorder (eg, dystonic reaction or a suicide attempt).<br />

6. Patients may also present with a general medical condition that has been exacerbated by neglect or with<br />

symptoms that are masked or distorted by their psychiatric disorder.<br />

B. Triage: determine the need for immediate treatment.<br />

1. Psychiatric patients should be seen in accordance with guidelines established for all patients (by severity of<br />

illness, then "first come, first served"). Patients should be categorized as emergent, urgent, or nonurgent and<br />

seen in order of priority.<br />

a. Emergent: ask yourself "Does this patient's condition pose a physical threat to himself or herself or others?"<br />

The answer is "yes" if:<br />

(1) Vital signs are abnormal.<br />

(2) A life-threatening illness or injury is present, even though the chief complaint is psychiatric (eg, the<br />

suicidal patient with a potentially lethal overdose or the combative patient who is hypoglycemic).<br />

(3) The patient is suicidal, violent (or potentially violent), or unable to care for himself or herself.<br />

b. Urgent: patient does not meet emergent criteria, and there is:<br />

(1) Agitation<br />

(2) Overwhelming anxiety<br />

(3) Suicidal or homicidal ideation<br />

(4) An inability on the part of the patient to explain the problem coherently<br />

(5) Extremely concerned family/collaterals<br />

(6) Intoxication<br />

c. Non urgent: patient requests psychiatric help but does not meet criteria for emergent or urgent treatment.<br />

2. All patients need a medical (as well as psychiatric) evaluation before leaving the emergency department,<br />

including a complete set of vital signs and a mental status examination. People with psychiatric illness may<br />

have comorbid medical/surgical conditions that may be overlooked because staff has labeled the patient and/or<br />

chief complaint as psychiatric ("anchoring" phenomenon).<br />

3. Patients with psychiatric problems must not be permitted to leave the emergency department without being<br />

evaluated as potentially harmful to self and/or others, incompetent to care for self, intoxicated, or psychotic<br />

and grossly disorganized.<br />

C. Safety: determine the need for seclusion/restraints.<br />

1. Attending to the safety of the patient, physician, and staff is a primary concern when managing a patient with<br />

agitation and/or thoughts of harm to self or others.<br />

2. Many emergency departments have rooms designed with this population in mind. The room should be devoid<br />

of medical equipment that could be used to harm oneself (eg, IV tubing that could be used to strangulate<br />

oneself) or others (eg, equipment that could be used as projectiles).<br />

3. Patients who are reporting active suicidal ideation should be placed on a direct-observation status (eg, use of a<br />

one-on-one sitter).<br />

4. Early warning signs indicating that a patient is a risk for violent behavior should be addressed immediately.<br />

5. Maintain a safe distance between you and the patient, and allow for you and the patient to both have an easy<br />

egress from the room.<br />

6. Keep in mind that the behavior of a patient with psychosis can be unpredictable, because the patient may be<br />

responding to thoughts (delusions) or perceptions (hallucinations) that are not based in reality.<br />

677


PSYCHOBEHAVIORAL DISORDERS<br />

D. Medical evaluation<br />

1. The goal is to recognize and distinguish general medical ("organic") from psychiatric ("functional") illness. If<br />

a general medical illness is identified or suspected, evaluate and treat appropriately before or while obtaining<br />

a psychiatric consult.<br />

a. Profile of a general medical disorder presenting as a behavioral emergency<br />

(1) Classic presentation: sudden or recent onset of a behavioral change in an older person without<br />

previously diagnosed psychiatric disorder<br />

(2) The patient may retain insight (but not always).<br />

(3) Specific findings<br />

(a) Sudden onset or change in symptoms<br />

(b) Recent diagnosis of a medical illness<br />

(c) Recent change in prescribed or OTC medications<br />

(d) Visual, tactile, or olfactory hallucinations are more common than auditory hallucinations.<br />

(4) Abnormal cognitive function including:<br />

(a) Altered level of consciousness (sedation or hyperarousal)<br />

(b) Disorientation (confusion is usually present)<br />

(c) Memory impairment<br />

(d) Poor attention<br />

b. Profile of a psychiatric disorder<br />

(1) Past history of behavioral problems in a younger patient<br />

(2) The patient lacks insight (but not always).<br />

(3) Specific findings<br />

(a) Auditory hallucinations are more common than other hallucinations.<br />

(b) Normal vital signs, physical examination, and laboratory studies<br />

(c) Disordered or illogical thinking, especially in the face of normal cognitive function. The patient<br />

should be oriented and can complete routine cognitive tasks (eg, spelling "world" backward).<br />

c. Specific medical illnesses presenting as psychiatric emergencies<br />

(1) Common "behavioral" presentations<br />

(a) Adverse effects of medications<br />

(b) Hyponatremia<br />

(c) Hypoglycemia or hyperglycemia<br />

(d) Alcohol intoxication or withdrawal<br />

(e) Illicit drug intoxication (eg, cocaine, bath salts, LSD, phencyclidine [PCP])<br />

(f) Drug withdrawal<br />

(g) Urinary tract infection leading to delirium<br />

(h) Pneumonia leading to delirium<br />

(i) COPD leading to hypoxia or hypercapnia<br />

(j) Liver disease, which may be accompanied by hyperammonemia<br />

(k) Chronic renal disease<br />

(I) Hypothyroidism or hyperthyroidism<br />

(2) Less common but potentially life-threatening "behavioral" presentations<br />

(a) Subarachnoid or intracranial hemorrhage<br />

(b) Encephalitis<br />

(c) Metastatic carcinoma or intracranial tumor<br />

(d) Malignant hypertension/hypertensive encephalopathy<br />

(e) Hypocalcemia/hypercalcemia<br />

(f) Steroid-induced psychosis<br />

(g) Autoimmune diseases<br />

2. Medical history<br />

a. All history must be corroborated.<br />

(1) Compare the patient's version with that of family, friends, and other sources (eg, police, paramedics).<br />

(2) Obtain old medical and psychiatric records.<br />

(3) Contact current or recent primary care physician.<br />

678


PSYCHOBEHAVIORAL DISORDERS<br />

b. Include medications, especially pain relievers, sedatives, and psychiatric medications. (Remember that<br />

many OTC and prescription drugs, as well as herbal supplements such as St. John's wort and ginseng,<br />

have psychiatric adverse effects.)<br />

c. Ask about drug and alcohol use (often overlooked in the professional, executive, "nice person," elderly,<br />

and children!)<br />

3. Physical examination<br />

a. If the patient is unable to cooperate, physical or chemical restraints may be required before examination.<br />

b. If the patient refuses to cooperate, procedures for involuntary commitment and/or emergency assessment<br />

should be followed. Do not permit the patient to leave the emergency department before adequate<br />

medical and psychiatric assessments have been completed.<br />

c. Patients must be disrobed and gowned, but first conduct search for possible weapons or concealed pills/<br />

other substances.<br />

(1) Proactively seek assistance from hospital security personnel and/or police.<br />

(2) If the patient has a weapon, security personnel are required; they must disarm him or her before any<br />

further evaluation or treatment can be safely conducted.<br />

d. Make sure vital signs have been taken and recorded (abnormalities suggest a possible medical problem).<br />

e. Neurologic examination: note particularly any signs of nonpsychiatric illness, eg, dysarthria, aphasia,<br />

dyspraxia, ataxia, dyskinesias, tremor, nystagmus, incontinence, paresis, paresthesias, and lateralizing<br />

neurologic signs.<br />

f. Documentation should include vital signs as well as signs of:<br />

(1) Trauma or exposure-related injuries<br />

(2) Previous suicide attempts<br />

(3) Thyroid, liver, or renal disease<br />

(4) Toxidromes (anticholinergics, cholinergics, sympathomimetics, opioids, etc)<br />

(5) Withdrawal syndromes (opioids, benzodiazepines, alcohol, etc)<br />

4. Mental status examination<br />

a. Level of consciousness: alert/somnolent/stuporous, fluctuating, or stable<br />

b. General appearance<br />

(1) Activity level: agitation, hyper-, normo-, hypomotor<br />

(2) Grooming and hygiene: appropriate, inappropriate, disheveled<br />

(3) Movements: tremor, tics, stereotypes, chorea, dyskinesias<br />

c. Orientation<br />

(1) Time, place, person, situation<br />

(2) Orientation to time is most sensitive: ask about day, date, month, season, and year.<br />

d. Memory<br />

(1) Immediate, short- and long-term<br />

(2) Give the patient three words and ask him or her to repeat them (immediate recall).<br />

(3) Ask the patient to repeat the words again after 3 minutes (short-term recall).<br />

(4) Long-term recall can be inferred in the course of history-taking.<br />

e. Mood and affect<br />

(1) Mood is self-reported ("sad," "angry"), whereas affect is observed (flat, irritable, tearful).<br />

(2) Affect ranges from flat to labile and can be described as mood-congruent or mood-incongruent (eg,<br />

inappropriate laughter).<br />

f. Speech: rate, rhythm, fluency<br />

g. Thought process: may be described as linear and logical, circumstantial, tangential, derailed, loose<br />

associations, thought blocking, illogical<br />

h. Thought content<br />

(1) Delusions ("fixed false beliefs")<br />

(a) Paranoid, persecutory, grandiose, self-deprecatory, erotic<br />

(b) Mood congruent/incongruent<br />

(c) Bizarre (impossible in real life) versus nonbizarre<br />

(2) Suicidal or homicidal ideation<br />

(3) Depressive cognitions: hopelessness, worthlessness, inappropriate guilt<br />

679


PSYCHOBEHAVIORAL DISORDERS<br />

1. Perceptual abnormalities<br />

(1) Hallucinations: visual, tactile, auditory, olfactory<br />

(2) Illusions: misperceptions of visual stimuli<br />

(3) Dissociative phenomena: flashbacks, deja vu, depersonalization<br />

j. Cognitive function: ask the patient to spell a word backward (such as "world" or "earth") and subtract 7<br />

from 100 serially (93, 86, 79, etc).<br />

k. Judgment and insight: can usually be inferred in the course of history-taking<br />

5. Screening diagnostic studies are not required to medically clear all patients with psychiatric complaints. The<br />

history, physical examination, and clinical judgment should direct appropriate diagnostic evaluation.<br />

a. "Medical clearance" is a controversial term used to communicate that the patient is stable for transfer to<br />

a psychiatric facility where medical resources may not be readily available.<br />

b. The history and medical examination may indicate the need for diagnostic evaluation, including<br />

electrolytes, BUN, creatinine, urinalysis, CBC, glucose, total CK, ECG, and chest radiograph.<br />

c. Tests such as an ethanol level or urine drug screen may aid psychiatric consultants in making an<br />

appropriate diagnosis and referral, particularly in a setting where dual diagnosis admission/treatment is<br />

available.<br />

d. Drug levels, including lithium, valproic acid, and carbamazepine, are useful in determining medication<br />

compliance. Trough levels are preferred for therapeutic monitoring.<br />

e. Be suspicious of suicidal patients who may not disclose an ingestion, and consider ordering<br />

acetaminophen, toxic alcohols, and salicylate levels; these ingestions are best discovered early.<br />

E. The psychiatric interview<br />

1. General approach<br />

a. Psychiatric conditions, like all medical problems, are biopsychosocial disorders. Evaluation should focus<br />

on signs and symptoms (onset, severity, and progression), the patient's physical and behavioral coping, and<br />

also the availability of therapeutic and/or supportive resources.<br />

b. Stages of the interview<br />

(1) Begin by asking open-ended questions and be silent afterward; the patient will fill in the silence.<br />

(2) Progress to structured, directive questioning to explore symptoms and circumstances.<br />

(3) Finish with a psychiatric "review of systems" to continue to consider or exclude covert, comorbid<br />

syndromes.<br />

(4) All history must be corroborated. In addition to the sources listed under the medical evaluation,<br />

contact with the patient's current treating psychiatrist and psychotherapist is essential to rapid, accurate<br />

diagnosis and treatment planning.<br />

(5) Collateral information is essential, particularly when planning to discharge a patient with safety<br />

concerns. It is considered standard of care.<br />

c. Pay attention to your emotional response to the patient.<br />

(1) The interviewer's emotional response will often mirror the patient's mood and thus is a valuable<br />

diagnostic instrument.<br />

(2) Maintain a calm demeanor and nonjudgmental approach with even difficult patients. Arguing with an<br />

agitated patient may exacerbate the situation; a better approach is to agree with the patient or "agree<br />

to disagree."<br />

2. The goals of the interview are to determine the:<br />

a. Nature of the problem: "What is wrong?" What is the patient upset or worried about, and how is he or she<br />

having difficulty functioning?<br />

b. Precipitant: "Why now?" What change in symptoms or circumstances has led to the current emergency<br />

department visit, and how did the patient come to the emergency department today?<br />

c. Progression, including course and duration: How long has the problem been going on, how has its nature<br />

and severity changed, has there been any prior treatment, and if so, to what effect?<br />

d. Patient's expectations: What is he or she (or the patient's family) hoping to have done at this time?<br />

e. Disposition: Has an appropriate plan, such as medical admission, psychiatric admission, outpatient<br />

psychiatric, and/or medical follow-up been decided?<br />

3. The interview: in addition to asking questions to determine the patient's mental/emotional stability, ask<br />

specifically about:<br />

a. Suicidal or homicidal ideation or threats<br />

b. Availability of firearms<br />

680


PSYCHOBEHAVIORAL DISORDERS<br />

c. Hallucinations, delusions<br />

d. Drug, alcohol, and prescription medication abuse<br />

e. Current (or past) physical or sexual abuse<br />

f. Any inconsistencies between the histories given by the patient and collaterals, or between the history given<br />

and your examination findings.<br />

4. Determining disposition<br />

a. Inpatient versus outpatient management is determined by whether the patient presents immediate danger to<br />

himself/herself or others, whether the patient is able to care for himself/herself, and what social supports are<br />

available at home.<br />

b. At the end of the interview, discuss your understanding of the psychiatric problem and any medical/surgical<br />

problems, and explain treatment recommendations.<br />

c. If immediate psychiatric hospitalization is indicated but unavailable (on site or by transfer), some<br />

hospitals will admit the patient to a medical service with appropriate supervision ("sitter") and psychiatric<br />

consultation, while other hospitals will have the patient "board" in the emergency department.<br />

d. There is a growing trend toward psychiatric observation units or psychiatric crisis units with the goal of


PSYCHOBEHAVIORAL DISORDERS<br />

emergency room. I would be upset, too, if I spent two hours waiting for a prescription and then was told<br />

that I would not get it. However, it is our policy not to give prescriptions for that medication."<br />

e. Do not argue with the patient or raise your voice in reaction to the patient. Yelling will make the patient<br />

more defensive, and the agitation will increase. Patients who remain agitated or become threatening<br />

should be informed that such behavior is not allowed, is scaring others (you, staff, other patients), and<br />

that restraints (either physical or chemical) may be required to control the behavior.<br />

f. The physician's body language should communicate confidence and calmness. Arms should be relaxed at<br />

the sides. Arms crossed in front of you can communicate that you feel afraid or closed off, whereas hands<br />

on the hips communicate that you are controlling and may be threatening.<br />

g. Always keep clear a lane of egress from the room when evaluating a psychiatric patient with the potential<br />

for violence and/or inappropriate behavior. Also consider having security nearby if there is concern the<br />

conversation may become violent.<br />

h. Verbal de-escalation is particularly likely to be successful in a patient who is cognitively intact.<br />

5. Pharmacologic management of agitation<br />

a. Use of medications to treat agitation is sometimes called "chemical restraint." Medications can be useful<br />

in obtaining behavioral control and should be considered once the initial evaluation has been completed<br />

or if the patient becomes agitated.<br />

(1) Indications<br />

(a) Agitation that would otherwise interfere with safe medical management<br />

(b) Treatment of the underlying cause of agitation (eg, giving an antipsychotic medication to a<br />

psychotic patient will decrease his or her agitation and also treat the psychosis)<br />

(c) If the patient requests a medication that was previously effective for safety<br />

(2) Goal: a calm patient in behavioral control (not a patient who is too sedated to interview, although<br />

sleep may result from these medications)<br />

(3) Relative contraindications<br />

(a) Incomplete assessment<br />

i. Medication may obscure diagnostic signs and symptoms.<br />

ii. Overmedication may delay completion of evaluation by several hours.<br />

(b) Unknown dosage of preexisting medication or overdose<br />

(c) For patients with dementia or other cognitive disorders, sedating medication (especially<br />

benzodiazepines) may cause disinhibition, potentially worsening agitation or confusion.<br />

(4) Protocols<br />

(a) Administration of the oral concentrate form is preferred if the patient is willing because:<br />

i. Consent is thereby implied.<br />

ii. Therapeutic alliance with the patient is furthered.<br />

iii. Absorption and efficacy are almost as rapid as with IM dosing.<br />

(b) "Rapid tranquilization" is well researched as safe and effective when using haloperidol,<br />

lorazepam, or both. This combination is considered by many to be the safest and most effective<br />

protocol for the severely agitated or violent patient.<br />

b. Classes of medication<br />

(1) Medications should be chosen to target the underlying cause of the agitation.<br />

682


PSYCHOBEHAVIORAL DISORDERS<br />

Based on response to interventions,<br />

medication is now required<br />

I<br />

I<br />

I<br />

I<br />

Ethanol or<br />

benzodiazepine<br />

withdrawal<br />

not suspected<br />

I<br />

Agitation associated<br />

with delirium<br />

Identify and correct any<br />

underlying condition<br />

Avoid benxodiazepines<br />

Oral second-generation<br />

anti psychotics<br />

(eg, risperidone,<br />

olanzapine)<br />

Oral first-generation<br />

anti psychotics<br />

(eg, haloperidol)#<br />

Parenteral secondgeneration<br />

antipsychotics<br />

(cg, olanzapine,<br />

ziprasidone)<br />

Parenteral first-generation<br />

anti psychotics<br />

(eg, haloperidol)#'<br />

#There is strong evidence<br />

that doses above 3 mg/day<br />

in patients with delirium<br />

are associated with<br />

significant risk of<br />

extrapyramidal side effects.<br />

'See FDA guidelines<br />

I<br />

Ethanol or<br />

benzodiazepine<br />

Oral benzodiazepines<br />

(eg, lorazepam,<br />

chlordiazepoxide,<br />

diazepam)<br />

Parenteral<br />

benzodiazepines<br />

(eg, lorazepam)<br />

I<br />

I<br />

CNS<br />

stimulant<br />

I<br />

Agitation due to<br />

intoxication<br />

I<br />

I<br />

CNS depressant<br />

(eg, ethanol)<br />

Avoid benzodiazepine<br />

if possible<br />

Oral first-generation<br />

anti psychotics<br />

(eg, haloperidol)<br />

Parenteral first-generation<br />

antipsychotics<br />

(eg, haloperidol)<br />

Agitation<br />

associated<br />

with psychosis<br />

in patient with<br />

known<br />

psychiatric<br />

disorder<br />

Oral second-generation<br />

antipsychotics<br />

(eg, risperidone', olanzapine')<br />

Oral first-generation antipsychotics<br />

(eg, haloperidol with<br />

benzodiazepine)<br />

Parenteral second-generation<br />

antipsychotics<br />

(eg, olanzapine', ziprasidone')<br />

Parenteral first-generation<br />

anti psychotics<br />

(eg, haloperidol with<br />

benzodiazepine')<br />

' If an antipsychotic alone does not<br />

work sufficiently, add lorazepam<br />

(oral or parenteral).<br />

I<br />

Undifferentiated<br />

agitation or<br />

complex<br />

presentation<br />

I<br />

No psychosis evident<br />

Same as agitaiton due<br />

to withdrawal<br />

Psychosis evident<br />

Same as for primary<br />

psychiatric disorder<br />

Protocol for Treatment of Agitation<br />

Source: Wilson MP, Pepper D, Currier GW, Holloman GH Jr, Feifel D. The psychopharmacology of agitation: consensus statement of the<br />

American Association for Emergency Psychiatry Project BETA Psychopharmacology Workgroup. Western j Emerg Med. Vol XIII, No. 1; Feb<br />

2012:31.<br />

(2) If a patient is violent or at imminent risk of violent behavior, then medications may be given IM<br />

against the patient's consent. Hospital security should be present to restrain the patient while the<br />

medication is administered to decrease risk of staff or patient injury.<br />

(3) Haloperidol and lorazepam are frequently considered agents of choice and may be used orally, IM,<br />

or IV (alone or in combination). They may be given together in the same syringe. In the absence of<br />

contraindications, <strong>combined</strong> dosing tends to reduce adverse effects as well as promote a synergistic<br />

drug effect. Lorazepam effectively prevents dystonia, while haloperidol permits the use of lower<br />

doses of lorazepam, thereby minimizing lorazepam's dose-dependent adverse effects (excessive<br />

sedation, respiratory depression).<br />

683


PSYCHOBEHAVIORAL DISORDERS<br />

(4) First-generation (typical) antipsychotics<br />

(a) Haloperidol 2-5 mg orally, IM, or IV every 30 minutes as needed up to 40 mg total for adults<br />

(0.5-1.0 mg every 30 minutes as needed in the elderly or children). Major complication is acute<br />

dystonias (may be prevented or treated with diphenhydramine 25-50 mg or benztropine I mg).<br />

Haloperidol is relatively contraindicated in patients with parkinsonism because of its potent<br />

dopamine D2 blockade.<br />

(b) Chlorpromazine 25-50 mg orally or IM every 30 minutes as needed. Hypotension may result<br />

with use of low-potency antipsychotics, particularly with concomitant use of a benzodiazepine.<br />

(5) Second-generation (atypical) antipsychotics<br />

(a) Medications in this class are being used increasingly in psychiatric emergency services, because<br />

they are less likely to cause extrapyramidal symptoms than older antipsychotics.<br />

(b) Olanzapine 5-10 mg orally or IM every 30 minutes, up to a total dose of 30 mg. It is also<br />

available in a rapidly dissolving wafer that can be useful in emergency situations in which the<br />

patient is somewhat cooperative. Do not give IM olanzapine along with IM lorazepam because<br />

of the risk of oversedation and hypotension.<br />

(c) Ziprasidone 10-20 mg orally or IM every hour, up to a total dose of 40 mg; may cause<br />

QT prolongation in some patients. Do a baseline ECG to measure QT interval before drug<br />

administration to avoid dangerous additive effect.<br />

(d) Risperidone does not have an immediate-acting IM formulation, but its oral preparation may still<br />

be helpful in treating agitation.<br />

(6) Benzodiazepines<br />

(a) Lorazepam 1-2 mg orally, IM, or IV every 30 minutes (0.25-0.5 mg every 30 minutes as needed<br />

in the elderly or children). Lorazepam is the only benzodiazepine with reliable and rapid IM<br />

efficacy.<br />

(b) Duration of action is intermediate (~6 hours).<br />

(c) Respiratory depression may occur, particularly in COPD, preexisting sedative-hypnotic<br />

ingestion, or in IV use. (IV administration requires availability of intubation and respiratory<br />

support.)<br />

(d) Disinhibition and agitation may occasionally occur, particularly in geriatric patients with/<br />

without underlying dementia.<br />

(e) In cases of substance abuse, particularly cocaine intoxication, benzodiazepines alone are<br />

preferable choices for chemical restraint.<br />

6. Seclusion<br />

a. The Centers for Medicare and Medicaid Services mandate detailed documentation of indications and<br />

procedures when restraints are used: "Seclusion or restraint can only be used in emergency situations ...<br />

only when less restrictive measures have been found to be ineffective to protect the patient or others<br />

from harm." Restraints may be ordered only for limited periods, and continuous face-to-face or audiovideo<br />

monitoring must be implemented. Transfers to other facilities are also subject to documentation of<br />

"least restrictive available" and "face-to-face monitoring" by trained staff.<br />

b. A seclusion room is designed and located to provide:<br />

(1) Safety: no potential weapons, minimal furnishings<br />

(2) Reduced stimulation: soundproofed, dimly lit, private<br />

(3) Continuous observation: by audio-video or in direct view of staff<br />

(4) Security staff and resuscitation area nearby<br />

(5) Clearly controlled entrance and exit<br />

c. Vital signs and triage should be done before leaving a patient in the seclusion room.<br />

7. Physical restraints (frequently required for violent or severely agitated psychotic, delirious/demented, or<br />

intoxicated patients)<br />

a. Advantages<br />

(1) Minimal adverse effects<br />

(2) Immediately reversible<br />

(3) Clearly visible reminders to staff and patient<br />

(4) Permits search for weapons and drugs (never, however, a primary indication)<br />

(5) Legally, it is appropriate medical treatment that is clearly established (Youngberg v. Romeo).<br />

684


PSYCHOBEHAVIORAL DISORDERS<br />

b. Indications<br />

(1) Patient is a danger to self or others.<br />

(2) Patient is unable to cooperate with assessment of a life-threatening condition.<br />

(3) Patient requests restraints for safety (rare).<br />

c. Contraindications<br />

(1) Unstable orthopedic injury<br />

(2) Unstable cardiovascular status<br />

(3) Use as punishment or solely for staff convenience<br />

d. Protocol should be clearly established, in written form, and institution-specific, with all emergency<br />

department personnel well trained.<br />

(1) Call for help immediately; the presence of a "show of force" is frequently very helpful in<br />

de-escalating such situations.<br />

(2) A team approach is required, with a leader and five helpers (one takes the head and each of the<br />

others an extremity).<br />

(3) Use leather (not cloth) restraints on all four extremities.<br />

(4) Restrain the patient so he or she can be examined and rolled onto his or her side quickly, if needed,<br />

to protect the airway. The patient should not be left restrained in the prone position because of the<br />

risk of positional asphyxia.<br />

(5) Tell the patient why restraints were applied.<br />

(6) Search for weapons and drugs.<br />

(7) Perform a physical examination.<br />

(8) Monitor continuously, face-to-face, or by audio-video.<br />

(9) Frequently (every 15 minutes or less) reassess the patient for safety and the need for continued<br />

physical restraints. Consider using medications if necessary to help the patient calm down and to<br />

allow for removal of physical restraints.<br />

(1 O) Document in the chart why restraints were required (to protect the patient and/or staff from harm)<br />

and the frequency and results of reassessments. Have an institutional policy and follow it.<br />

(11) Remove restraints only when the patient's behavior improves and he or she is no longer a threat. Don't<br />

bargain for removal. If a patient is calm and in behavioral control, do not keep him or her in restraints<br />

longer than needed as a form of punishment or prophylaxis against future episodes of agitation.<br />

B. Crisis intervention<br />

1. A patient with or without a psychiatric illness may present to the emergency department "in crisis," eg, after<br />

a trauma or in bereavement. The emergency physician can follow guidelines for crisis intervention to help the<br />

patient manage the crisis.<br />

2. Trauma may result in a severe psychological reaction. When the traumatic event is so severe or occurs so<br />

suddenly that patients are unable to master the resulting emotions, the patient experiences a crisis. Crisis<br />

intervention involves understanding the patient's view and assisting patients to change their perception of the<br />

problem in such a way that they are no longer overwhelmed by their own reaction.<br />

a. Goals<br />

(1) Stabilize the patient's emotional and behavioral reaction.<br />

(2) Prevent psychological sequelae and screen for pathologic reactions.<br />

(3) Assist the patient in developing mastery of the traumatic situation, thereby creating growth, maturation,<br />

and a long-term improvement in coping skills.<br />

b. Prerequisites<br />

(1) The traumatic event must be identified and must have ended.<br />

(2) The patient must have sufficient self-reliance and social support.<br />

c. Patients most likely to benefit are those with a prior history of adequate functioning.<br />

d. Technique<br />

(1) Develop rapport. Be supportive of the patient but avoid glib reassurance; mobilize hope by<br />

communicating an expectation for improvement. Reassure the patient that it is good that he or she is<br />

seeking help and that the catastrophe has indeed ended.<br />

(2) Let the patient talk about his or her feelings; be a sympathetic, nonjudgmental, active listener.<br />

(3) Help the patient to reassess the situation, survey critical issues, and develop a solution.<br />

685


PSYCHOBEHAVIORAL DISORDERS<br />

(4) Activate natural support systems in family and community.<br />

(a)<br />

If possible, access community resources (eg, government agencies, caseworker, minister,<br />

responsible family member, etc) before discharging the patient from your care.<br />

(b) Medical/surgical issues (eg, chronic pain) must be addressed.<br />

e. Warning signs of poor coping include dissociation, reexperiencing the traumatic event, hyperarousal<br />

(including hypervigilence or severe sleep disturbance), or avoidance of reminders of the trauma. These<br />

symptoms may suggest the need for referral to a counselor or psychiatrist.<br />

f. Debriefing (ie, discussion of the event in detail, often in a group of individuals who had witnessed the same<br />

trauma) was previously recommended as a method to help individuals process a traumatic event. However,<br />

recent evidence suggests that group debriefing may increase the risk of developing an acute stress reaction<br />

or post-traumatic stress disorder and should be avoided.<br />

3. Bereavement ("acute grief reaction")<br />

a. Etiology: awareness of a sudden loss, eg, sudden infant death syndrome, accidental death or suicide of a<br />

family member or close friend, spousal abandonment, job loss, etc<br />

b. Clinical stages of grief and mourning have been described, some or all of which the patient may evidence.<br />

(1) Shock and denial (may be accompanied by screaming/collapse)<br />

(2) Anger<br />

(3) Bargaining<br />

(4) Depression (feeling of powerlessness over the event)<br />

(5) Acceptance (in which loss and its consequences are constructively integrated into the patient's psyche<br />

and life)<br />

c. Exclude "pathological grief reaction" ie, depressive mood disorder.<br />

(1) Features characteristic of major depression (eg, pervasive sadness with insomnia, anorexia, agitation,<br />

impaired concentration, etc) may be found during the course of normal bereavement.<br />

(2) Some symptoms are abnormal (even in severe bereavement) and warrant a diagnosis of "depressive<br />

disorder" and psychiatric treatment.<br />

(a)<br />

Persistence of significant symptoms beyond 2 months<br />

(b) Suicidal ideation (other than wishing to be with the deceased)<br />

(c) Morbid preoccupation with worthlessness<br />

(d) Marked slowing of thought and movement ("psychomotor retardation")<br />

(e)<br />

(f)<br />

d. Treatment<br />

Prolonged and severe debilitation<br />

Hallucinations other than hearing the voice of, or transiently seeing the deceased (these<br />

experiences are common-and normal-in survivors)<br />

(g) Substance abuse or an increase in use of alcohol, pain medications, sedatives<br />

(1) Help patients identify the loss and encourage them to express their feelings about it; defuse blame,<br />

suspicion, and guilt.<br />

(2) The severity of the grief reaction is directly related to the suddenness and importance of the loss; signs<br />

of severe depression, psychosis, or other inappropriate response warrant psychiatric referral.<br />

(3) Most people will not require counseling; however, referral information should be provided, because<br />

some individuals may decide to use it later on.<br />

C. Psychopharmacology<br />

1. Anti anxiety agents<br />

a. Benzodiazepines<br />

(1) Useful for short-term management of anxious, agitated patients in crisis and for initial treatment of<br />

patients with panic disorder. They are also used to treat seizures, muscle tension, alcohol withdrawal,<br />

and catatonia.<br />

(2) When used in conjunction with antipsychotic agents, they have synergistic effects.<br />

(3) Sedation with low doses reduces anxiety; occasionally, disinhibition ("paradoxical excitation") occurs.<br />

(4) Lorazepam<br />

(a)<br />

0.5-2 mg may be repeated every 30-60 minutes as needed.<br />

(b) Onset of action varies with route of administration; 20-40 minutes when administered orally or<br />

IM and


PSYCHOBEHAVIORAL DISORDERS<br />

(5) Chlordiazepoxide 25-100 mg is sometimes given for alcohol withdrawal, particularly in a patient<br />

without liver dysfunction. Its long half-life provides a self-taper.<br />

(6) Diazepam 5-10 mg can be administered orally or IM. Diazepam is highly distributed in fat, and will<br />

(along with several active metabolites) accumulate after repeated administration.<br />

(7) Adverse effects<br />

(a) Dose-dependent sedation, impairment of motor coordination and reaction time<br />

(b) Impairment of consolidation of short-term memory (learning); at higher doses, ataxia/dysarthria<br />

and amnesia ("blackouts") may occur.<br />

(c) Respiratory depression is found at higher dosages but is essentially benign in oral and IM use for<br />

healthy individuals who are not taking other respiratory depressants. IV use, comorbid COPD<br />

(or other respiratory compromise), and interactions with other medications/substances can cause<br />

lethal apnea. Therapeutic use under these circumstances requires caution, less aggressive dosing,<br />

and the immediate availability of intubation and ventilatory support.<br />

(8) Tolerance may develop over time.<br />

(9) A withdrawal syndrome may occur with insomnia, agitation, increased vital signs, and withdrawal<br />

seizures.<br />

(10) These drugs have abuse potential and can be addictive. Diazepam and alprazolam are well-known<br />

offenders.<br />

(11) Benzodiazepines are very safe in oral and IM use.<br />

b. Buspirone<br />

(a) Lorazepam in particular has minimal cardiovascular effects, no active metabolites, and does not<br />

inhibit/induce cytochrome P450 isoenzyme catabolism of other substrates.<br />

(b) Although these drugs are frequently ingested in suicide attempts, a fatality is rare unless they are<br />

taken in combination with alcohol or other drugs.<br />

(1) A nonbenzodiazepine anxiolytic that is not subject to withdrawal or addiction<br />

(2) Requires 2-3 weeks for efficacy and therefore is not indicated in the emergency department<br />

c. Antihistamines<br />

(1) Diphenhydramine and hydroxyzine can be useful as mild oral antianxiety agents.<br />

(2) Dosage is 25-50 mg every 4 hours orally or IM (may also be given IV).<br />

(3) If a hypnotic is indicated for brief outpatient use after emergency department discharge, these agents<br />

are preferred because of their low incidence of adverse effects and safety.<br />

2. First-generation antipsychotic agents<br />

a. First-generation ("typical") antipsychotics have largely been replaced by second-generation ("atypical")<br />

antipsychotics for routine control of psychiatric symptoms. However, the first-generation agents continue<br />

to be used in some patients, including some who need long-acting injectable formulations. Antipsychotic<br />

medications are also called neuroleptics.<br />

Table 41: Characteristics of First-Generation (Typical) Neuroleptic Agents<br />

Potency Dystonia Drug<br />

Equivalent<br />

Dose (mg)<br />

Anticholinergic<br />

Sedation<br />

high high low low<br />

Haloperidol 2<br />

I I I I<br />

Droperidol 2<br />

Fl uphenazi ne 2<br />

Perphenazine 10<br />

low low Chlorpromazine 100 high high<br />

b. Mechanism of action is dopaminergic receptor blockade in the mesolimbic area of the CNS. This may result<br />

in reduced hallucinations, delusions, anxiety, impulsivity, and aggression.<br />

c. Al I anti psychotics cause some orthostatic hypotension and reflex tachycardia; these adverse effects<br />

(which are due to a-adrenergic blockade) are more common and severe with low-potency neuroleptics<br />

(chlorpromazine).<br />

d. The seizure threshold is lowered with use of antipsychotics.<br />

687


PSYCHOBEHAVIORAL DISORDERS<br />

e. Haloperidol is the neuroleptic agent of choice in many emergency departments; it reduces agitation, has<br />

benign cardiovascular (and minimal anticholinergic) effects, is not very sedating, and has a rapid onset.<br />

3. Second-generation antipsychotic agents<br />

a. Characteristics<br />

(1) Enhanced mechanism of action: postsynaptic antagonism of both serotonin and dopamine receptors<br />

(2) Less likely to cause extrapyramidal symptoms but more likely to produce metabolic syndrome than<br />

first-generation agents<br />

(3) Also used to treat bipolar disorder and to augment treatment of depression<br />

b. Clozapine<br />

(1) Very low risk of extrapyramidal symptoms, increased risk of seizures<br />

(2) Risk of agranulocytosis requires weekly CBC monitoring. Its use is limited to psychotic disorders<br />

unresponsive to standard agents or to patients with severe extrapyramidal symptoms. It should never<br />

be started in the emergency department.<br />

c. Risperidone, paliperidone, ziprasidone, olanzapine, quetiapine, and aripiprazole are other second-generation<br />

anti psychotics. These agents may cause sedation, weight gain, hyperlipidemia, and hyperglycemia.<br />

4. Adverse effects of anti psychotic agents<br />

a. Extrapyramidal symptoms: occur more often with high-potency neuroleptics but can also occur with<br />

second-generation agents<br />

(1) Dystonias<br />

(a) Painful clonus of voluntary muscles; they usually involve muscles of the face, neck, and tongue,<br />

but any muscle group can be affected.<br />

(b) Laryngospasm occurs rarely and can be life threatening.<br />

(c) Oculogyric crisis involves involuntary eye movements, usually in the upward direction.<br />

(d) Torticollis is a painful contraction of the sternocleidomastoid muscle.<br />

(e) These reactions generally occur within the first month of treatment (or after dose increases).<br />

(f) Treatment is with anticholinergic agents, followed by a maintenance prescription of<br />

anticholinergics prescribed twice daily to prevent recurrence.<br />

(2) Parkinsonism<br />

i. Benztropine 1-2 mg orally, IM, or IV<br />

ii. Diphenhydramine 25-50 mg orally, IM, or IV<br />

(a) One or more of the following: shuffling gait, bradykinesia or akinesia, resting tremor, cogwheel<br />

rigidity, masked fades, drooling<br />

(b) Dosage reduction and/or anticholinergic agents are usually effective.<br />

(3) Akathisia<br />

(a) An unpleasant and disturbing feeling of restlessness with an almost uncontrollable desire to<br />

move; easily mistaken for agitation<br />

(b) Reactions begin several days to weeks after start of therapy and are worsened by increases in dosage.<br />

(c) Treatment<br />

i. Propranolol 10-20 mg orally is the drug of choice for managing anti psychotic medicationinduced<br />

acute and chronic akathisia, and relief is usually seen within 20-40 minutes; if not,<br />

the dose may be repeated. Ongoing treatment is necessary to prevent recurrence.<br />

ii. For acute akathisia in the emergency department, treat with anticholinergics (eg,<br />

diphenhydramine) and/or lorazepam IV or IM.<br />

(4) Tardive dyskinesia<br />

(a) A disorder of abnormal, involuntary movements that develops after a period of antipsychotic use<br />

(b) Oral-buccal muscle involvement is common and characterized by lip smacking and tongue<br />

protrusion. Limb and truncal muscles can also be involved.<br />

(c) The patient can voluntarily suppress the movements temporarily.<br />

(d) There is no definitive treatment. The offending agent should be discontinued. This should be<br />

coordinated with the treating psychiatrist to prevent recurrence of psychosis. Discontinuation<br />

of the antipsychotic may also lead to a transient increase in the movement severity but a greater<br />

chance of improvement over time.<br />

b. Anticholinergic effects (occur more often with low-potency neuroleptics)<br />

(1) May be central (sedation, agitation, delirium/psychosis) and/or peripheral (eg, dry mucous<br />

membranes, flushed skin, mydriasis, tachycardia, urinary retention)<br />

688


PSYCHOBEHAVIORAL DISORDERS<br />

(2) Treatment is supportive, and the offending agent should be discontinued. Use of physostigmine<br />

should be avoided.<br />

c. Cardiovascular effects<br />

(1) The major cardiovascular adverse effects (seen more often with low-potency neuroleptics) are<br />

hypotension (usually managed with IV fluids) and tachycardia. The mechanism of action is through<br />

noradrenergic blockade.<br />

(2) Antipsychotics may result in QTc prolongation. Cases of torsades have been reported with use of IV<br />

haloperidol. Of the newer antipsychotics, ziprasidone is most associated with QTc prolongation.<br />

d. Hyperprolactinemia and its clinical consequences (galactorrhea, amenorrhea) may result from use of<br />

antipsychotics, especially risperidone.<br />

e. Neuroleptic malignant syndrome<br />

(1) This uncommon (0.5 %-1 %) idiosyncratic reaction (usually occurring in the first weeks of treatment<br />

or after a dose increase) constitutes a medical emergency. Mortality rates up to 30% can be<br />

improved to 10% with early recognition and aggressive supportive treatment.<br />

(2) More commonly associated with high-potency neuroleptics, it can also be associated with the lowpotency<br />

agents or withdrawal of anti-Parkinson agents (eg, discontinuation of L-dopa).<br />

(3) Clinical presentation<br />

(a) Generalized muscle "lead pipe" rigidity. Occasional patients may also have a superimposed<br />

tremor or a cogwheel phenomenon.<br />

(b) Mental status change, often presenting as confusion, agitation, or coma<br />

(c) Hyperthermia with temperatures often >104°F (40°C), often with diaphoresis<br />

(d) Autonomic instability with tachycardia and labile or high blood pressure<br />

(4) Laboratory abnormalities<br />

(a) Leukocytosis<br />

(b) Increased creatine phosphokinase: typically >1,000 IU/L, can be as high as 100,000 IU/L<br />

(5) Findings cannot be explained by a medical or neurologic disorder (eg, viral encephalitis), a mental<br />

disorder (eg, psychosis), or substance abuse (eg, phencyclidine).<br />

(6) Treatment<br />

(a) ICU admission is often required.<br />

(b) Supportive measures, including IV fluid hydration. Some patients may require mechanical<br />

ventilation.<br />

(c) The offending agent should be discontinued. Other psychotropic medications (eg, lithium or<br />

antidepressants) should also be held.<br />

(d) Anticholinergic agents should be avoided; they are not effective and may worsen the reaction by<br />

interfering with temperature regulation.<br />

(e) Benzodiazepines (eg, lorazepam 1-2 mg) can be used as needed for muscular rigidity or agitation.<br />

(f)<br />

5. Antidepressants<br />

Dantrolene may reduce rigidity and temperature. Its use is controversial and may cause<br />

hepatotoxicity and as monotherapy may increase mortality.<br />

a. Starting antidepressants in the emergency department is not recommended without a high certainty of<br />

reliable follow-up. These medications can be dangerous in overdose, and in the short-term may increase<br />

suicidal thoughts.<br />

b. SSRls (fluoxetine, sertraline, paroxetine, citalopram, escitalopram, fluvoxamine) are the current first-line<br />

agents for depressive disorders, premenstrual dysphoric disorder, and anxiety disorders (including panic,<br />

obsessive-compulsive, and post-traumatic stress disorders).<br />

(1) Adverse effects are generally mild and include nausea, headache, sleep disturbance, and sexual<br />

dysfunction.<br />

(2) Many of these agents inhibit cytochrome P450, causing drug-drug interactions.<br />

(a) Fluoxetine, paroxetine, and venlafaxine (a serotonin norepinephrine-reuptake inhibitor [SNRI])<br />

are P450 2D6 inhibitors and prolong/increase tricyclic antidepressant (TCA) levels, thereby<br />

potentiating their cardiotoxic effects. Risperidone, codeine, dextromethorphan, and class IC<br />

antidysrhythmics (eg, encainide, flecainide) levels are also increased via this mechanism.<br />

(b) Fluoxetine, fluvoxamine, and citalopram are P450 3A4 inhibitors (as is grapefruit juice) and<br />

prolong/increase levels of alprazolam, triazolam, clonazepam, and verapamil.<br />

(3) Toxicity develops only with large overdoses or when <strong>combined</strong> with other agents.<br />

689


PSYCHOBEHAVIORAL DISORDERS<br />

c. SNRls: inhibit reuptake of both serotonin and norepinephrine, a mechanism of action similar to that of the<br />

TCAs without the cardiotoxicity of the TCAs.<br />

(1) Venlafaxine: can cause hypertension or orthostatic hypotension<br />

(2) Duloxetine: often prescribed for patients with depression or anxiety and a comorbid pain disorder;<br />

should not be used in patients with hepatic dysfunction or active alcohol abuse<br />

d. Other newer antipsychotics<br />

(1) Trazodone: may increase serum digoxin and phenytoin levels; orthostasis and rarely priapism can occur.<br />

(2) Bupropion: increases norepinephrine and dopamine; causes seizures in overdose and in the<br />

predisposed (notably bulimic patients and those with seizure disorders)<br />

e. TCAs (eg, amitriptyline, nortriptyline, imipramine, and others) were the standard of care through the early<br />

1990s. They have been replaced by the SSRls because of their many adverse effects:<br />

(1) Anticholinergic effects (both peripheral and central)<br />

(2) a-adrenergic effects: TCAs act therapeutically by presynpatic norepinephrine reuptake inhibition;<br />

sympathetic adverse effects can include both hypertension and orthostatic hypotension with fal Is<br />

(particularly in older patients).<br />

(3) Cardiac effects: TCAs are type IA (quinidine-like) antidysrhythmics and delay conduction. In overdose,<br />

this causes nonspecific T-wave changes, AV block, t QT interval, and ventricular tachdysrhythmias,<br />

including torsades de pointes and fibrillation.<br />

(4) Increased seizure risk<br />

(5) Lethality in overdose<br />

f. Monoamine oxidase inhibitors (MAOls), eg, phenelzine, tranylcypromine, isocarboxazid, selegiline)<br />

(1) Use of MAOls has largely been replaced by newer antidepressants; however, emergency physicians<br />

should be aware of potential drug-food and drug-drug interactions with these agents.<br />

(2) Patients on an MAOI should be on a tyramine-free diet. Consumption of tyramine-containing foods<br />

(aged cheeses, cured meats, fava beans, red wine) can lead to a hypertensive crisis.<br />

(a)<br />

Clinical presentation of impending hypertensive crisis may include severe occipital headache,<br />

hypertension, palpitations, chest pain, tachycardia, dilated pupils, diaphoresis, nausea, and<br />

vomiting.<br />

(b) Treatment<br />

i. Phentolamine, an a-antagonist (2.5-5 mg slow IV push, which may be repeated every 10-15<br />

minutes as needed), or sublingual or IV nifedipine<br />

11. ~-blockers are contraindicated, because they may worsen hypertension by producing<br />

vasoconstriction.<br />

(3) Ingestion of an MAOI and other serotonergic agents can lead to life-threatening serotonin syndrome.<br />

Patients taking an MAOI should avoid SSRls, SNRls, tricyclics, meperidine, dextromethorphan, and<br />

MOMA.<br />

(4) The antibiotic linezolid is pharmacologically an MAOI. Patients should not be continued on<br />

serotonergic agents while taking linezolid.<br />

6. Serotonin syndrome<br />

a. May occur with drug-drug interactions between SSRls, MAOls, and other serotonergic agents<br />

(sumatriptan, meperidine, tramadol) or in overdose.<br />

b. By Hunter Criteria, serotonin syndrome is diagnosed if the patient took a serotonergic agent and has one<br />

of the following:<br />

(1) Spontaneous cl onus<br />

(2) Inducible clonus plus agitation or diaphoresis<br />

(3) Ocular clonus plus agitation or diaphoresis<br />

(4) Tremor and hyperreflexia<br />

(5) Hypertonia and temperature > 100.4°F (38°C) plus ocular or inducible clonus<br />

c. In addition to the Hunter Criteria, the patient may have additional symptoms including:<br />

(1) Altered mental status: confusion, hallucinations, delirium<br />

(2) Autonomic instability: diaphoresis, hyperthermia, orthostasis<br />

(3) GI signs: nausea, vomiting, diarrhea<br />

(4) Neuromuscular instability: tremor, myoclonus, hyperreflexia<br />

(5) Life-threatening seizures and cardiovascular collapse may occur.<br />

690


PSYCHOBEHAVIORAL DISORDERS<br />

d. Management<br />

(1) Discontinue all serotonergic drugs.<br />

(2) Supportive care, including IV fluid hydration and cardiac monitoring<br />

(3) Benzodiazepines (eg, lorazepam 1-2 mg) can be used to treat agitation.<br />

(4) Cyproheptadine is a serotonin and histamine antagonist that may be used if supportive care does not<br />

result in clinical improvement.<br />

(5) Patients with severe symptoms may require ICU admission. Patients with moderate symptoms may be<br />

admitted with cardiac monitoring until symptoms resolve.<br />

7. Mood stabilizers<br />

a. Mood stabilizers are not usually started in the emergency department given their complications and<br />

requirements for close monitoring.<br />

b. Lithium<br />

(1) First used in the late 1800s and a very effective mood stabilizer. It has been shown to reduce risk of<br />

suicide in patients with bipolar disorder.<br />

(2) Narrow therapeutic index: therapeutic level is 0.8-1.2 mEq/L; signs of toxicity can be evident at levels<br />

as low as 1 .5 mEq/L.<br />

(3) Acute toxicity<br />

(a) Can be precipitated by dehydration, diuretics, ACE inhibitors, and NSAIDs<br />

(b) Early signs of lithium toxicity include nausea, vomiting, and diarrhea. Patients then develop<br />

sluggishness, confusion, ataxia, and an irregular coarse tremor or myoclonic jerks, followed by<br />

seizures and nonconvulsive status epilepticus.<br />

(c) Hemodialysis is indicated for a lithium level >4 mEq/L regardless of clinical status, or for a level<br />

>2.5 mEg/L if severe symptoms are present. Otherwise, hydration with IV fluids will help facilitate<br />

excretion.<br />

(4) Chronic toxicity<br />

(a) Renal toxicity: chronic use of lithium can cause nephrogenic diabetes insipidus, nephrotic<br />

syndrome, and chronic interstitial nephritis. Occasionally, this progresses to end-stage renal<br />

disease. Creatinine should be monitored in patients taking lithium.<br />

(b) Hypothyroidism also develops as a result of chronic lithium use.<br />

c. Divalproex/valproic acid<br />

(1) Indicated for use in bipolar disorder, seizure disorders, and migraine prophylaxis<br />

(2) Contraindicated in pregnancy given high risk of neural tube defects<br />

(3) Targettrough level 50-125 mcg/ml<br />

(4) Valproic acid toxicity<br />

(a) Signs of toxicity are present at levels> 180 mcg/ml.<br />

(b) Symptoms include CNS suppression, respiratory depression, hyperammonemia, cerebral edema,<br />

tremors, and myoclonus.<br />

(c) May also result in a dose-related hepatotoxicity<br />

(5) Patients on divalproex may develop hyperammonemia and encephalopathy even with normal valproic<br />

acid and transaminase levels.<br />

(6) Rare but severe complications include hepatic failure and pancreatitis.<br />

d. Carbamazepine and oxcarbazepine<br />

(1) Indicated for maintenance treatment of bipolar disorder<br />

(2) Oxcarbazepine is used as an alternative to carbamazepine given its better tolerability but has an<br />

increased risk of hyponatremia.<br />

e. Lamotrigi ne<br />

(1) Indicated for treatment of bipolar depression<br />

(2) Risk of Stevens-Johnson syndrome, toxic epidermal necrolysis, and drug reaction with eosinophilia<br />

and symptoms (DRESS) syndrome, usually occurring in the first 2-8 weeks of therapy; therefore, slow<br />

titration of dose is required.<br />

691


PSYCHOBEHAVIORAL DISORDERS<br />

Ill. SPECIFIC BEHAVIORIAL DISORDERS<br />

A. Psychosis<br />

1. Definition: loss of contact with reality<br />

2. Clinical presentation<br />

a. Delusions: a "fixed false belief" that is not accepted by the patient's cultural group and that is firmly<br />

sustained despite what others believe, and in spite of incontrovertible evidence to the contrary (eg, "The<br />

FBI implanted radio transmitters in my heat ducts").<br />

b. Hallucinations: a false perception experienced in a sensory modality (usually auditory) occurring while<br />

the individual is fully conscious; if they are visual, olfactory, or tactile, suspect a medical- or substanceinduced<br />

("organic") disorder.<br />

c. Disorganized speech: the patient does not make sense, rapidly jumps from one topic to another, or takes<br />

long pauses before answering, which is suggestive of thought blocking.<br />

d. Crossly disorganized behavior: purposeless behavior that does not make sense in context, eg, spreading<br />

one's wardrobe across the lawn or masturbating in the emergency room<br />

e. Catatonia: a neuropsychiatric syndrome characterized by decreased responsiveness to one's environment<br />

and either hypo- or hyperactivity<br />

(1) A classic presentation includes "waxy flexibility," whereby the examiner can move the patient's limbs<br />

and they will stay fixed in the new position.<br />

(2) Catatonic excitation resembles phencyclidine (PCP) intoxication and can be extremely dangerous to<br />

others, including emergency department staff.<br />

f. Negative symptoms: flat affect, poverty of speech, loss of volition, interpersonal withdrawal, minor<br />

cognitive deficits (acutely psychotic patients are usually oriented)<br />

3. Treatment<br />

a. Determination of the underlying cause of an acute psychotic episode may not be possible in the emergency<br />

department, but initial treatment and evaluation are the same.<br />

b. Establish safety first.<br />

(1) Search for weapons (involve security personnel as indicated).<br />

(2) For patients who are violent or at imminent risk of becoming violent, manage their acute agitation.<br />

c. After behavioral stabilization, proceed immediately to exclude treatable general medical causes of the<br />

acute episode (including substance- or medication-induced disorders).<br />

4. Differential diagnosis<br />

a. Anatomic: head trauma, stroke, tumor, seizures<br />

b. Metabolic: hypoxia, hypoglycemia, ketoacidosis, hyper- or hypocalcemia<br />

c. Endocrinologic: hyper- or hypothyroid, hypoparathyroid<br />

d. Autoimmune: multiple sclerosis, systemic lupus erythematosus, Hashimoto encephalitis (rare)<br />

e. Infectious: HIV, syphilis, encephalitis/meningitis<br />

f. Toxic: anticholinergic delirium<br />

g. Medications: steroids<br />

h. Substances: alcohol intoxication/withdrawal, stimulants, hallucinogens, cocaine<br />

5. Major psychiatric disorders that can present as an acute psychotic episode<br />

a. Schizophrenia<br />

(1) A chronic disorder affecting~ 1 % of the population<br />

(2) Onset usually in late adolescence/early adulthood<br />

(3) Diagnosis requires:<br />

(a)<br />

At least two of the positive symptoms of psychosis for c::1 month ("positive symptoms" include<br />

hallucinations, delusions, disorganized speech, disorganized behavior, or catatonia)<br />

(b) Severe impairment in level of functioning<br />

(c) Duration of c::6 months of some signs of the disorder<br />

(d) Exclusion of substance abuse, medications, or a general medical condition as the cause of the<br />

signs and symptoms<br />

b. Major depression with psychotic features ("psychotic depression"): a psychosis occurring solely during the<br />

course of a depressive episode<br />

692


PSYCHOBEHAVIORAL DISORDERS<br />

c. Mania with psychotic features: a psychosis occurring solely during the course of a manic episode<br />

d. Schizoaffective disorder: a psychosis that is chronic and often associated with mood symptoms, but it also<br />

occurs for significant periods without mood symptoms<br />

e. Schizophreniform disorder: a psychosis lasting


PSYCHOBEHAVIORAL DISORDERS<br />

(c) Hypo- or hyperthyroidism<br />

(d) Diabetes (and its complications)<br />

(3) Other general medical conditions: HIV encephalitis, mononucleosis, renal failure, pancreatic cancer<br />

3. Focal points of the psychiatric history<br />

a. Previous psychiatric illness, including history of psychiatric hospitalizations, suicide attempts, and suicidal<br />

ideation<br />

b. Recent acute or chronic illnesses (especially life-threatening ones)<br />

b. Recent life changes<br />

d. Existing social supports or lack thereof<br />

e. Always ask about suicidal or homicidal ideation<br />

4. Treatment and disposition<br />

a. The primary goal in evaluating depression is to determine the patient's suicidal potential. An adaptation of<br />

the "SAD PERSONS" score correlates well with the need for hospitalization.<br />

Table 42: SAD PERSONS Score<br />

£ex (male)<br />

Age (45 years old)<br />

Points<br />

Q.epression (or signs and symptoms of)<br />

2<br />

frevious attempts or psychiatric care<br />

l;xcessive alcohol or drug use<br />

Rational thinking (loss of)<br />

2<br />

Separated/divorced/widowed/single<br />

Qrganized plan or serious attempt<br />

2<br />

~o social support<br />

Stated future intent, plan, or mechanism<br />

2<br />

Score: >8 = high risk, 6-8<br />

intermediate risk, 8: almost all patients require admission<br />

(2) 6-8: psychiatry consult with follow-up or possible admission<br />

(3)


PSYCHOBEHAVIORAL DISORDERS<br />

2. Risk assessment for suicide<br />

a. Anyone who attempts or threatens suicide must be taken seriously and evaluated for suicide risk potential.<br />

(1) Ask the patient how he or she feels about having survived an attempt and assess his or her reaction<br />

(eg, angry, doesn't care, relieved).<br />

(2) Ask directly about the suicidal intent; patients do not find this insulting and it will not "put ideas in<br />

their head."<br />

(3) Confirm all patient reports with collaterals.<br />

b. What is the content of the suicidal thoughts? Consider the following in your evaluation:<br />

(1) Expressed intent<br />

(2) Lethality of the method<br />

(3) Likelihood of rescue: was rescue a foregone conclusion, or purely fortuitous?<br />

(4) Suicidal thoughts can range from a passive death wish (eg, "I wish I would go to sleep and not wake<br />

up, but I would never hurt myself" to having a suicide plan with intent and means to act on this plan.<br />

The more detailed the plan, the greater the risk. For a patient who has had a self-injurious act, has<br />

anything changed as a result of the act, eg, improved ("I didn't realize people cared") or worsened<br />

("I'm so inadequate, I couldn't even commit suicide successfully").<br />

c. What are this patient's risk factors for suicide?<br />

(1) History of previous suicide attempts conveys the highest risk.<br />

(2) Age and ethnicity: white men >65 years old are the demographic group most likely to complete<br />

suicide.<br />

(3) Gender: men are 3-4 times more likely to complete suicide, whereas women are 3-4 times more<br />

likely to attempt suicide. Men are more likely to attempt suicide by a more lethal means (firearms,<br />

hanging) than women (ingestion, cutting).<br />

(4) Family history of completed suicide<br />

(5) Major psychiatric illness, including major depressive disorder, schizophrenia, bipolar disorder, and<br />

borderline personality disorder<br />

(6) Marital status: those who are separated, divorced, or single are at higher risk<br />

(7) Medical illness, particularly chronic illnesses (multiple sclerosis, HIV)<br />

(8) Hopelessness: an individual in a personal crisis who sees "no way out" or no recovery from mental<br />

illness is at high risk of acting on these thoughts.<br />

(9) Recent loss (of job, of relationship, recent death)<br />

(10) Alcohol abuse: one-third of completed suicides occur when the individual had consumed alcohol.<br />

(11) Additional risk factors include panic attack, agitation, insomnia, and physical pain. These can be<br />

addressed while the patient is in the emergency department.<br />

(12) In adolescents, the ratio of attempts to completions is 25:1 in girls and 3:1 in boys. Risk is increased<br />

with a history of running away, previous suicide threats or attempts, psychiatric disorder, alcohol or<br />

substance abuse, and being a lesbian/gay/bisexual/transexual teen. The presence of a firearm in the<br />

household markedly increases (by 5-10 times) the suicide risk.<br />

d. What factors mitigate suicide risk?<br />

(1) Caring for dependent children<br />

(2) Strong religious beliefs that deter suicide<br />

(3) Evidence of planning for the future (being "future-oriented")<br />

(4) The ability to put in place an action plan for what to do if the suicidal thoughts return<br />

e. Is there access to lethal means?<br />

(1) Firearms are used in 53% of all completed suicides in the United States. Their presence in the home<br />

constitutes an independent risk factor for completed suicide. All suicidal patients should be asked<br />

about access to firearms.<br />

(2) Inquire as to whether the family can safely remove sharp objects and lock up medications.<br />

f. Consider less obvious presentations.<br />

(1) Repeated medical emergencies due to noncompliance with treatment may indicate silent suicide, the<br />

act of killing oneself slowly by nonviolent means. This is often unrecognized and is most common in<br />

the elderly.<br />

(2) "Unintentional" overdoses, wrist lacerations, self-inflicted gunshot wounds, falls from heights, and<br />

single-automobile collisions may represent occult suicide, ie, self-destructive acts disguised<br />

as accidents.<br />

695


PSYCHOBEHAVIORAL DISORDERS<br />

3. Treatment<br />

a. Patients at high risk of committing suicide should be hospitalized, and safeguards must be put in place<br />

to limit a patient's ability for self-harm while in the hospital. In-hospital suicides are the second most<br />

commonly reported sentinel event and occur at a rate of 5-15 per 100,000 admissions.<br />

b. After a suicide attempt, medical stabilization of toxicity or trauma is required, followed by the identification<br />

and treatment of any comorbid medical condition. Once the patient is medically stabilized, placement in a<br />

psychiatric hospital can be considered.<br />

c. While in the emergency room:<br />

(1) Search for weapons and pills with assistance of security personnel.<br />

(2) Constant observation of the patient must be provided, eg, through use of 1 :1 sitters.<br />

(3) Patients must not be permitted to leave before their assessment has been completed, and if found to<br />

be at risk of suicide or to lack decision-making capacity, they must not be permitted to leave against<br />

medical advice.<br />

(4) Involuntary admission may be required.<br />

d. Disposition<br />

(1) If the patient's suicide intent is serious, or you are unsure how serious the potential suicide risk is after<br />

your evaluation, obtain an emergent psychiatric consultation.<br />

(2) While mental health professionals may assist in the evaluation, the emergency physician is required<br />

to make an independent judgment and retains final responsibility for medical decisions. In particular,<br />

"denial of medical necessity" by a managed care organization or third-party payer does not override<br />

the physician's duty to render appropriate and adequate medical care, including, if necessary,<br />

involuntary admission.<br />

(3) Admission may be required if:<br />

(a) The patient will not (or cannot) cooperate with assessment<br />

(b) The crisis is ongoing or unresolved<br />

(c) After your evaluation (or a psychiatrist's), the patient is still considered a suicide risk<br />

(d) When in doubt, err on the side of caution.<br />

(4) Discharge to outpatient treatment may be considered if:<br />

(a) The patient is perceived to be at low risk of acting on suicidal thoughts (eg, if the patient has only<br />

passive thoughts of suicide, has no history of attempts, and is able to agree to a "safety plan")<br />

(b) The patient has adequate social support, and<br />

(c)<br />

Referral for close outpatient follow up has been arranged.<br />

(5) Patients who present after an actual suicide gesture (not just thoughts) are almost always referred<br />

for inpatient psychiatric treatment. Those with a gesture that is "low lethality" (eg, ingesting a small<br />

number of OTC tablets or superficial cuts on the dorsal surface of the arm) and "high rescue" (eg,<br />

took the ingestion in front of a family member) might be considered for potential discharge only if<br />

the above criteria are already met and the patient's family and outpatient providers are prepared to<br />

manage outpatient care.<br />

(6) Patients deemed high risk may decline voluntary hospitalization and may require an involuntary<br />

hold to protect their safety. State laws determine the specific criteria for involuntary hospitalization,<br />

the duration of time a patient can be held, and the process for court proceedings for extending the<br />

hospitalization.<br />

D. Mania and hypomania<br />

1. A "manic episode" is defined as an excessive, persistently elevated, expansive, or irritable mood and 3 or more<br />

of the fol lowing symptoms for ::,:1 week duration:<br />

a. Grandiosity or inflated self-esteem<br />

b. Decreased need for sleep (without fatigue)<br />

c. Pressured speech or increased talkativeness<br />

d. "Flight of ideas" (repeatedly jumping from one topic to another loosely related topic in conversation, or a<br />

subjective feeling that the mind is "racing")<br />

e. Distractibility<br />

f. Agitation or increase in goal-directed activity<br />

g. lmpulsivity ("excessive involvement in pleasurable activities that have a high potential for painful<br />

consequences," eg, spending sprees, hypersexual behavior, fights, foolish behavior at work; psychosis is<br />

possible, often with paranoid, grandiose, or religiose features)<br />

696


PSYCHOBEHAVIORAL DISORDERS<br />

2. A hypomanic episode is of less intense severity, has less impact on the patient's ability to function, and may<br />

last


PSYCHOBEHAVIORAL DISORDERS<br />

f. Waxy flexibility: the examiner can move the patient's body position and the patient assumes the new position<br />

g. Negativism: the patient actively resists the examiner's attempts to move the patient<br />

3. Both medical and psychiatric etiologies<br />

a. Major depression<br />

b. Mania<br />

c. Chronic psychoses such as schizophrenia<br />

d. Stroke<br />

e. Cerebral tumors<br />

f. Encephalitis<br />

4. Malignant catatonia: catatonia accompanied by autonomic instability (hyperthermia, tachycardia, hypertension,<br />

or labile blood pressure); can be life threatening<br />

5. Treatment<br />

a. Benzodiazepines (particularly lorazepam) are the treatment of choice. An initial trial of lorazepam 1-2 mg<br />

can be administered orally, IM, or IV. Response should be seen within 1-2 hours. A second dose should be<br />

administered if there is no response after 3 hours.<br />

b. An initial positive response to lorazepam can confirm the diagnosis in many patients.<br />

c. If no response to benzodiazepine treatment, electroconvulsive therapy is the next recommended treatment<br />

of choice.<br />

d. If the catatonia is associated with an underlying psychiatric disorder, the patient responds to lorazepam,<br />

and his or her vital signs remain stable, psychiatric admission may be appropriate.<br />

e. Patients with malignant catatonia require medical hospitalization and may require treatment in the ICU.<br />

f. Antipsychotics should not be given while the patient is catatonic, because they may increase the likelihood<br />

of precipitating neuroleptic malignant syndrome. (Neuroleptic malignant syndrome and catatonia are<br />

believed to be related syndromes.)<br />

F. Panic attacks/disorder<br />

1. Clinical presentation<br />

a. Common: apprehension, tremor, sweats/flushing/chills, shortness of breath, chest tightness or pain,<br />

palpitations or tachycardia, difficulty swallowing, nausea, lightheadedness, paresthesias, and a sense of<br />

depersonalization, derealization, and feelings of impending doom<br />

b. Patients believe that they are suffering from an acute medical problem (and physicians are required to<br />

believe that these patients are suffering from an acute medical problem until proven otherwise).<br />

c. Attacks usually begin suddenly, sometimes without provocation, and last


PSYCHOBEHAVIORAL DISORDERS<br />

c. Immediate interventions aimed at reducing hyperventilation (such as having the patient lock his hands<br />

behind his neck) improve hypocapnia and respiratory alkalosis, and thus help to resolve paresthesias,<br />

lightheadedness, and derealization/depersonalization, as well as provide a reassuring sense of self-control.<br />

d. The most useful intervention is the communication to the patient that any potentially life-threatening<br />

medical disorder has indeed been excluded. During a panic attack, the patient feels that he or she is about<br />

to die (and, indeed, symptoms can be indistinguishable from an acute Ml). This is itself frightening, and the<br />

emotional reaction to the perceived life threat causes further sympathetic arousal, thus further exacerbating<br />

symptoms.<br />

G. Somatoform disorders<br />

1. Somatoform disorders include conversion disorder, somatization disorder, hypochondriasis, and pain disorder.<br />

These disorders are all characterized by psychological conflict that manifests either as physical symptoms or as<br />

concern about symptoms or illness.<br />

a. The link between psychological distress and physical symptoms may be apparent to the physician but is not<br />

realized by the patient.<br />

b. This is not malingering. Malingering is an intentional faking of symptoms for conscious personal gain such<br />

as obtaining drugs, avoiding legal responsibility, or gaining shelter in the hospital.<br />

c. This is not a factitious disorder in which symptoms are intentionally produced or feigned but with no<br />

motivation or incentive other than to assume the "sick role" (primary gain).<br />

d. Psychiatric consults may be of limited use for somatoform patients. The patient will not want (and may<br />

resent) this suggestion, because the somatoform symptom serves the purpose of masking a psychological<br />

conflict.<br />

2. Conversion disorder<br />

a. A psychological conflict "converts" into an acute loss of neurologic function that allows the patient to<br />

avoid or resolve the conflict; typical examples include loss of voluntary motor or sensory function (eg,<br />

paralysis of one or both legs), blindness, or nonepileptic seizures ("pseudoseizures").<br />

b. Diagnostic criteria<br />

(1) Symptom(s) or deficit(s) of voluntary sensory or motor function suggesting a neurologic (or other<br />

medical) condition, eg, stroke<br />

(2) These symptoms/deficits are judged to be due to psychological factors, because conflicts or stressors<br />

precede their onset/exacerbation.<br />

(3) They are not intentionally produced or "faked."<br />

(4) No demonstrable source from a general medical disorder, substance use/abuse, or culture-specific<br />

pattern (This is a diagnosis of exclusion and should not be made without an appropriate medical<br />

evaluation to exclude physiologic causes of the symptoms.)<br />

c. Clinical presentation<br />

(1) The patient appears healthy and may be less concerned about his or her symptoms than you would<br />

expect ("la belle indifference").<br />

(2) The physical symptoms typically developed suddenly and involve a voluntary muscle function (most<br />

common) or loss of sensation. They also tend to have a relationship to the conflict they "solve" (eg,<br />

a man who hates his manual labor job develops "paralyzed" legs at home Sunday night). Ask the<br />

patient what it is that he or she cannot do now and what it was that he or she was doing when the<br />

symptoms began.<br />

d. Treatment<br />

(1) Perform a detailed history and physical examination, and obtain diagnostic studies as indicated to<br />

exclude medical/surgical pathology.<br />

(2) Once the evaluation is completed, assure the patient that a serious medical problem has not been<br />

discovered and suggest that the symptom will resolve. Most cases of conversion disorder resolve<br />

without intervention.<br />

(3) About 25%-50% of patients initially diagnosed with conversion disorder are eventually found to<br />

be suffering from significant medical pathology (systemic lupus erythematosus, multiple sclerosis,<br />

polymyositis, thyroid disease, porphyria, hypo- or hyperparathyroidism); therefore, referral for<br />

thorough evaluation and ongoing follow up is a must.<br />

3. Somatization disorder<br />

a. Patients present repeatedly to medical practice settings, including the emergency department, with multiple<br />

physiologic complaints. Psychologic conflict is manifest as physical complaints that gain the focus of the<br />

patient's attention. The review of physical systems will have positive findings in multiple organ systems.<br />

699


PSYCHOBEHAVIORAL DISORDERS<br />

b. To meet diagnostic criteria, patients must have pain complaints on four body sites, two GI symptoms, one<br />

pseudoneurologic symptom, and one sexual symptom.<br />

c. Treatment is coordinated through an involved primary care physician. Appointments are scheduled at<br />

regular intervals, and discussions restricted to a limited number of complaints.<br />

d. Medical evaluation of physical symptoms should be thoughtfully selected and guided by objective findings.<br />

4. Hypochondriasis<br />

a. The patient intensely believes that he or she has a specific disease such as cancer or HIV. The patient<br />

presents with "evidence" of the disease and repeatedly requests testing.<br />

b. In hypochondriasis, the patient is concerned with disease, while in somatization disorder, the patient is<br />

concerned with symptoms.<br />

5. Somatoform pain disorder<br />

a. A patient presents with one or more pain symptoms that cause clinically significant distress or impairment.<br />

b. Psychological factors are believed to have an important role in the onset, severity, exacerbation, or<br />

maintenance of the pain. The patient may also have an underlying physiologic reason for the pain, but the<br />

pain is usually disproportionately more than what would be expected.<br />

c. The patient is experiencing real pain (ie, it is not faked) and usually believes it is due to a physical problem.<br />

d. This is difficult to detect in the emergency department. The physician may be concerned that the patient is<br />

"drug seeking" because of the disproportionate level of pain. Arranging continuity of care with a primary<br />

care provider may be of assistance to the patient.<br />

6. Psychophysiologic reactions<br />

a. A physical condition brought on by an event that is associated with a heavy emotional overlay, eg, nausea<br />

and diarrhea that occur before a performance. This is not a somatoform disorder but another example of a<br />

link between psychological factors and physical symptoms.<br />

b. Symptoms are real although engendered unconsciously; physiologic illness develops to express or resolve<br />

emotional conflict without the patient realizing it. These reactions can also be understood as "overflow"<br />

autonomic activation in response to a stimulus.<br />

c. Treat the medical condition without confronting the patient with the psychogenic cause.<br />

d. The reactive condition is not produced voluntarily or intentionally and is, in fact, realistically distressing/<br />

handicapping to the patient. Gently suggest a stress-related emotional-physical connection. It may be<br />

possible to illustrate this by having the patient reenact the situation.<br />

e. These patients may benefit from psychotherapy or biofeedback.<br />

7. Techniques that distinguish neurologic from psychogenic deficits<br />

a. Bowlus-Currier test (sensation): Have the patient extend his or her arms, crossing them at the wrists, so the<br />

palms are facing each other; then have him or her interlock the fingers and rotate the hands inward toward<br />

the chest. In this position, false responses to sensory testing are difficult.<br />

b. Hoover test (motor): Cup your hands under the heels of the patient, and ask him to raise his or her normal<br />

leg. With pseudoparalysis, the other leg will push downward.<br />

c. Gray test (pain): Patients with abdominal pain that is psychological in origin will close their eyes on<br />

palpation of the abdomen (most patients with organic pain watch your hand in anticipation of pain).<br />

d. Bell phenomenon (coma): In true coma, the eyes remain neutral when the lids are opened; in conversion,<br />

the eyes are diverted upward.<br />

e. Corneal reflex: intact with pseudoseizure<br />

f. Optokinetic drum: nystagmus in pseudoblindness<br />

H. Insomnia and sleep disorders<br />

1. Insomnia is one of the most common medical complaints. Patients report difficulty falling or staying asleep, or<br />

not feeling rested the following day.<br />

2. Myriad causes of sleep disturbance exist and include the following:<br />

a. Psychophysiologic insomnia (also called primary insomnia): the patient becomes increasingly concerned<br />

about the delay in falling sleep, which leads to an increase in anxiety and greater delay in sleep onset.<br />

b. Extrinsic factors<br />

(1) Poor sleep hygiene (eg, TV and lights remain on at night)<br />

(2) Shift-work sleep disorder<br />

(3) Jet lag<br />

c. Psychological contributors include mood disturbances (eg, mania or major depression) and anticipatory<br />

anxiety (eg, about an exam).<br />

700


PSYCHOBEHAVIORAL DISORDERS<br />

d. Alcohol and other substances directly disrupt the sleep-wake cycle.<br />

e. Medical problems can impair sleep, eg, obstructive sleep apnea, restless legs syndrome, orthopnea,<br />

physical pain.<br />

3. Treatment<br />

a. Treatment of insomnia focuses on identification and treatment of the underlying pathology (eg, obstructive<br />

sleep apnea, depression). However, this is not always possible in the emergency room setting.<br />

b. Sleep hygiene education should be provided. Patients should avoid caffeine, exercise, and alcohol in the<br />

evening, and they should go to bed and rise at the same time every day. The bedroom should be restricted<br />

to use for sleep and sex, and televisions should be removed from the room.<br />

c. Sleep aids can be useful for acute insomnia. They generally should not be used for >2 weeks, because<br />

tolerance and withdrawal can develop.<br />

(1) Zolpidem 5-10 mg is often used to treat insomnia. Adverse effects include increased risk of<br />

parasomnias (eg, sleep walking).<br />

(2) Long-acting benzodiazepines are used for those with middle-of-the-night insomnia.<br />

(3) Melatonin, a hormone produced by the pineal gland that triggers natural onset of sleep, is often<br />

recommended for jet lag. It is available OTC.<br />

(4) OTC sleep aids (eg, diphenhydramine) have limited effectiveness and make use of sedating,<br />

antihistamine effects.<br />

d. Patients presenting to the emergency department with insomnia should follow up with their primary care<br />

provider.<br />

I. Eating disorders<br />

1 . Anorexia nervosa<br />

a. Prevalence is approximately 1 % in Western societies; women are most commonly affected (95%),<br />

particularly those from middle and upper socioeconomic classes.<br />

b. Onset generally occurs between 12 and 18 years of age and usually follows an episode of severe dieting or<br />

other weight loss.<br />

c. The central characteristic is a preoccupying fear of being overweight that does not diminish with weight loss.<br />

It may or may not be associated with purging behaviors (self-induced vomiting, or laxative or diuretic abuse).<br />

d. Prognosis improves with earlier onset, diagnosis, and treatment. Chronicity increases morbidity.<br />

e. Mortality at 10-year follow-up is 7%; at 30-year follow-up, 18%.<br />

f. Diagnostic criteria<br />

(1) Weight 15% ideal body weight is sufficient to<br />

hospitalize)<br />

701


PSYCHOBEHAVIORAL DISORDERS<br />

(b) Hypokalemia<br />

(c)<br />

Hyponatremia<br />

(d) Bradycarclia with heart rate


PSYCHOBEHAVIORAL DISORDERS<br />

(5) Diuretic abuse can cause:<br />

(a)<br />

Dehydration<br />

(b) Hypokalemia, hypercalcemia, hyperuricemia, hypomagnesemia, and hyponatremia<br />

(6) Binge eating after a period of starvation has further complications.<br />

h. Treatment<br />

(a) Acute gastric distention (even rupture)<br />

(b) Acute pancreatitis (mortality 10%)<br />

(1) Normotensive, hypokalemic, hypochloremic metabolic alkalosis is a typical manifestation of chronic<br />

purging. The patient has usually adapted physiologically to this state. Overzealous efforts at treatment<br />

frequently lead to overcorrection and fluid overload (which can be dangerous).<br />

(2) Nutritional rehabilitation: normalization of the eating pattern is the primary consideration.<br />

(3) Medical admission is indicated for:<br />

(a) Vital sign abnormalities<br />

(b) Hypokalemia<br />

(c) Hyponatremia<br />

(d) Comorbid medical disorder (eg, diabetes, particularly in a patient who is withholding insulin as a<br />

means of losing weight)<br />

(4) Psychiatric admission may be indicated when:<br />

(a) Severe psychiatric illness or comorbid suicidality<br />

(b) Worsening ability to control self-induced vomiting, binge eating, or diuretic/laxative use, which<br />

may be life threatening<br />

(c) The patient's binging, purging, or obsessive thoughts leads him or her to be uncooperative with<br />

lower levels of treatment.<br />

(5) Definitive treatment after medical stabilization requires cognitive-behavioral psychotherapy and/or<br />

antidepressant medication (an SSRI); combining both psychotherapy and medication may enhance<br />

treatment efficacy.<br />

J. Intoxication and withdrawal<br />

1. Intoxication: impairment of judgment, perception, attention, emotional control, and psychomotor activity<br />

produced by recent ingestion of a drug<br />

a. Clinical presentation is predominately impaired judgment and motor coordination. With increasing<br />

severity, delirium, stupor, coma, and death may occur.<br />

b. Intoxication should be included in the differential diagnosis of any patient with an altered state of<br />

consciousness.<br />

2. Urine drug screens do not have perfect sensitivity. Watch for false-negatives (oxycodone and methylphenidate<br />

are often not detected) or false-positives (eg, pseudoephedrine showing positive for amphetamines).<br />

3. Treatment should be guided by the following principles:<br />

a. Provision of general supportive measures that include the ABC priorities<br />

b. Specific measures to remove toxic substances or substances that have been ingested in toxic amounts<br />

c. Exclusion of other serious medical, surgical, or psychiatric disorders<br />

d. Appropriate referral-substance abuse is a chronic and episodic disorder, and effective treatment is available.<br />

4. Alcohol or other sedative-hypnotics (benzodiazepines)<br />

a. Intoxication<br />

(1) Symptoms include an initial feeling of euphoria and disinhibition, followed by slurred speech and<br />

ataxia. New memories are not encoded into memory (anterograde amnesia). More severe intoxication<br />

may present with either agitation or sedation.<br />

(2) Degree of impairment depends on total alcohol consumption and also previous exposure (tolerance).<br />

"Legal limit" for driving in most states is a blood alcohol level of 0.08 gldl.<br />

(3) If tranquilization is required, use of additional sedativehypnotic medications (such as lorazepam)<br />

should be avoided. An antipsychotic (eg, haloperidol 5 mg or chlorpromazine 25-50 mg orally or IM<br />

every 30-60 minutes) is a safer choice because these contribute little to respiratory depression.<br />

(4) Treatment of alcohol intoxication requires placement in a secure setting and allowing time to pass<br />

for the patient to metabolize the alcohol. If large quantities of alcohol have been rapidly ingested,<br />

intubation may be required for airway protection.<br />

703


PSYCHOBEHAVIORAL DISORDERS<br />

b. Withdrawal<br />

(1) For a person with an average rate of alcohol metabolism, the blood alcohol level will drop by ~0.02<br />

g/dl per hour.<br />

(2) A seasoned alcoholic may begin to show alcohol withdrawal with a blood alcohol content well above<br />

the "legal limit."<br />

(3) Withdrawal symptoms: 6-24 hours from the last drink, patients experience a high-amplitude tremor<br />

and autonomic hyperactivity (nausea, tremor, hypertension, tachycardia, hyperreflexia, sleep<br />

disturbances, and anxiety)<br />

(4) Withdrawal seizures: generalized tonic-clonic seizures that usually occur 12-48 hours from last<br />

drink. Indications for CT of the head in patients who have seizures:<br />

(a) New-onset seizure<br />

(b) Focal seizure<br />

(c) Focal neurologic deficit<br />

(d) Status epilepticus<br />

(e) With significant head trauma<br />

(5) Alcohol withdrawal delirium (delirium tremens): delirium (characterized by an altered level of<br />

consciousness, disorientation, confusion, and often visual hallucinations) with onset 3-5 days after<br />

last drink. Autonomic instability with gross tremor, fever, incontinence, mydriasis, and frightening<br />

visual hallucinations may occur, leading to substantial mortality and often requiring ICU admission.<br />

(6) Alcohol withdrawal is treated with benzodiazepines. Lorazepam is metabolized by hepatic<br />

glucuronidation and does not require metabolism by hepatic oxidation and, therefore, is preferred<br />

for patients with hepatic dysfunction. Patients with liver disease and the elderly have decreased<br />

ability to oxidize benzodiazepines. Longer-acting agents (chlordiazepoxide or diazepam) are also<br />

used. Benzodiazepine dosages are based on the patient's Clinical Institute Withdrawal Assessment<br />

(CIWA) score.<br />

(a) These patients are generally volume-depleted, hypomagnesemic, malnourished, and<br />

hypoglycemic. Provide IV hydration (D5NS), thiamine 100 mg, and magnesium 2-4 g.<br />

(b) Do not administer glucose or other carbohydrate before replacing thiamine (IV); doing so can<br />

precipitate Wernicke encephalopathy and/or Korsakoff syndrome with brain damage that is<br />

frequently permanently disabling.<br />

(7) Disposition is determined by the patient's stage of withdrawal and history of complicated withdrawals.<br />

(a) Patients in early withdrawal (autonomic hyperactivity) who respond to emergency department<br />

treatment can often be managed as outpatients or in a dedicated detoxification/substance abuse<br />

center. These patients may require 1-2 days of therapy with a long-acting benzodiazepine. This is<br />

controversial, because these medications may mask symptoms of a medical problem or may be<br />

sold on the street. Close follow-up is required.<br />

(b) Patients may require medical admission for alcohol withdrawal if they have a history of<br />

complicated withdrawals (including delirium or seizures) or if their presenting symptoms cannot<br />

be controlled with oral benzodiazepines. The presence of seizures, hallucinations, delirium, or<br />

signs of Wernicke encephalopathy merits a medical admission.<br />

5. Opioids (including heroin, oxycodone, hydrocodone, hydromorphone, morphine, meperidine, opium,<br />

methadone)<br />

a. Intoxication<br />

(1) Toxidrome includes somnolence, decreased respiration rate, decreased tidal volume, decreased bowel<br />

sounds, and miotic pupils. The absence of miotic pupils does not exclude opioids as the cause of a<br />

change in mental status.<br />

(2) Usual presentation in the emergency department is of a person who has overdosed and may have<br />

received naloxone in the field. Accidental overdose of opioids is rising rapidly as a cause of death in<br />

the United States.<br />

(3) Naloxone is a competitive opioid antagonist that quickly reverses the effects of opioid intoxication. It<br />

can be given IV, IM, SC, or by nasal spray. Usual doses are 0.4-2 mg, and it can be repeated up to 10<br />

mg. Effects of naloxone last for 30-60 minutes, so repeated dosing may be needed.<br />

(4) Opioid-addicted individuals may also present to the emergency department with medical<br />

complications of their drug use, eg, cellulitis or abscesses from "skin popping" (injecting heroin<br />

subcutaneously).<br />

704


PSYCHOBEHAVIORAL DISORDERS<br />

b. Withdrawal<br />

(1) Unlike alcohol withdrawal, opioid withdrawal is unlikely to be life threatening.<br />

(2) Symptoms include muscle aches, abdominal cramps, malaise, nausea, yawning, rhinorrhea,<br />

piloerection, diarrhea, sweats, and chills.<br />

(3) Vital signs are often normal, although the patient may be tachycardic or hypertensive due to physical<br />

discomfort or pain.<br />

(4) Opioid withdrawal is treated symptomatically, for example:<br />

(a) Clonidine to decrease adrenergic tone, which results in many withdrawal symptoms<br />

(b) Ibuprofen for muscle aches<br />

(c) Dicyclomine for abdominal cramps<br />

(d) lmodium for diarrhea<br />

(e) Buprenorphine (sometimes)-Caution should be used in giving buprenorphine too soon after<br />

last opioid use, in particular with long-acting opioids such as methadone. Because it is a partial<br />

agonist that binds strongly to the mu receptor, buprenorphine may precipitate withdrawal if<br />

given too soon.<br />

(5) Medical admission is usually not warranted.<br />

6. Cocaine, bath salts, and other psychostimulants (amphetamine, methamphetamine, methylphenidate<br />

a. Intoxication<br />

(1) Cocaine can be snorted as powder, injected, or smoked as crack-cocaine. Effects usually last


PSYCHOBEHAVIORAL DISORDERS<br />

2. Patients with dementia are predisposed to bouts of delirium. For these patients, differentiating between delirium<br />

and dementia requires knowledge of the patient's baseline mental state.<br />

3. Delirium<br />

a. Diagnostic features<br />

(1) Disturbance of consciousness with reduced ability to focus, sustain, or shift attention.<br />

(2) Cognitive impairment (eg, memory deficit, disorientation, dysphasia) or perceptual disturbance<br />

(3) Acute onset (hours to days) with diurnal fluctuation of symptoms<br />

(4) Physiologic result of a general medical condition<br />

b. Clinical presentation<br />

(1) Fluctuating level of consciousness and alertness<br />

(2) Trouble paying attention; ask the patient to state the days of the week backward (a delirious patient<br />

will often state them forward)<br />

(3) May be characterized by hyperactive or hypoactive motor activity<br />

(4) Visual hallucinations (common)<br />

c. Search for underlying cause.<br />

(1) Metabolic disorders<br />

(2) CNS injury or mass lesion<br />

(3) Infection (CNS or elsewhere)<br />

(4) Medication effects/interactions<br />

(5) Drugs, heavy metals, toxins, poisons<br />

d. Treatment revolves around correction of underlying cause.<br />

(1) Patients with delirium are hospitalized in a medical facility because of their underlying medical illness.<br />

(2) If the cause of delirium is not determined in the emergency department, these patients are<br />

hospitalized to determine the underlying cause.<br />

4. Dementia: a global and progressive impairment of cognitive function without an alteration of consciousness<br />

a. Diagnostic features: multiple cognitive deficits with both<br />

(1) Memory impairment (either ability to learn or recall) and<br />

(2) At least one other cognitive disturbance<br />

(a) Aphasia (language deficits)<br />

(b) Apraxia (inability to perform known tasks)<br />

(c) Agnosia ("not knowing")<br />

(d) Impaired executive function (planning, organizing, sequencing, abstracting)<br />

b. Clinical presentation<br />

(1) Gradual onset (months or years)<br />

(2) Disturbance of recent memory may be the first sign.<br />

(3) Dementia may become acutely worse in response to another illness, change in living situation, or<br />

medications.<br />

(4) Common causes of reversible dementia<br />

c. Treatment<br />

(a) Metabolic and endocrine disorders<br />

i. Hypothyroidism<br />

ii. 8 12<br />

deficiency<br />

(b) Normal pressure hydrocephalus<br />

(c) Depression (depressive "pseudodementia") should be suspected if onset was relatively acute<br />

and accompanied by vegetative signs of depression (eg, loss of appetite and weight, sleep<br />

disturbances, etc). Reversible medical/surgical causes must be excluded.<br />

(d) Medication effects<br />

i. Anticholinergic medications<br />

ii. Polypharmacy<br />

iii. Incorrect medication usage (demented patients may take medications incorrectly)<br />

(1) Initial treatment focuses on correcting reversible causes of dementia and reviewing the drug regimen<br />

to identify medications that may exacerbate the cognitive symptoms.<br />

706


PSYCHOBEHAVIORAL DISORDERS<br />

(2) The progression of the dementia may be slowed by modulating risk factors (eg, treating hypertension in<br />

vascular dementia) or use of anticholinesterase inhibitors (eg, donepezil).<br />

(3) Discussions about advance directives, power of attorney, code status, and long-term care wishes<br />

should be started (if not done already) at the onset of the diagnosis, while patients still have capacity to<br />

make these decision for themselves.<br />

L. "Medical mimickers" of psychiatric illness (medical illnesses that manifest psychiatric symptoms)<br />

1. Acute intermittent porphyria<br />

a. Presents as episodic psychosis (with agitation and hallucinations) and acute abdominal pain<br />

b. Neurologic symptoms are due to accumulation of early heme precursors due to a deficiency of<br />

porphobilinogen deaminase.<br />

c. Attacks are precipitated by drugs, hormones, and diet that stress the hepatic aminolevulinic acid synthase<br />

and cytochrome P450 systems. Offenders include barbiturates and other anticonvulsants, sulfonamide<br />

antibiotics, low-calorie and low-carbohydrate diets, alcohol abuse, progesterone/progestins, and<br />

nonspecific severe stress.<br />

d. Diagnosis is made by laboratory quantification of urinary aminolevulinic acid and porphobilinogen,<br />

which are markedly increased during attacks and between recurring attacks. Urine dipstick urobilinogen<br />

tests are nonspecific, and urine exposed to sunlight turns dark or red in color.<br />

e. Treatment is primarily supportive. Patients may be admitted and observed for neurologic complications.<br />

2. Hyperthyroidism<br />

a. Symptoms include anxiety, emotional !ability, weakness, tremor, palpitations, heat intolerance, increased<br />

perspiration, and weight loss despite a normal or increased appetite.<br />

b. Physical examination may be notable for hyperactivity, rapid speech, and lid retraction, which may make<br />

the patient appear to be staring.<br />

c. Diagnosis is made on the basis of thyroid function tests. Thyroid-stimulating hormone is usually


PSYCHOBEHAVIORAL DISORDERS: PRACTICE CLINICAL SCENARIOS<br />

PSYCHOBEHAVIORAL DISORDERS: PRACTICE CLINICAL<br />

SCENARIOS<br />

Answers immediately follow the practice clinical scenarios.<br />

Scenario A<br />

Presentation: A 48-year-old man is brought in by EMS after he was found crawling in the hallway of<br />

his apartment building toward his door, and then laying against the wall. He admits to alcohol use and<br />

smoking marijuana. He is given IV fluids and thiamine. His alcohol level is 0.39 gldL. The patient is given<br />

IV fluids, a meal tray, and observed. He is on the monitor, and the nurse comes to you noting that the<br />

patient is becoming agitated and tachycardic. As you walk toward the room, the patient begins to seize.<br />

What is the diagnosis?<br />

Scenario B<br />

Presentation: A 21-year-old law student presents to the emergency department the night before his final<br />

exams with inability to move his fingers. On discussion with the patient, you find he will not be able to<br />

take his computer-based final exam tomorrow because of this paralysis.<br />

What is the diagnosis?<br />

Scenario C<br />

Presentation: A 24-year-old male patient with a history of depression and psychosis presents to the<br />

emergency department with his mother. She says that he has been having excited outbursts at home over<br />

the last few days. Today, he is stuporous and is not engaging in the history or physical examination. When<br />

you perform a neurologic examination, the patient resists any movement of his extremities.<br />

What is the diagnosis?<br />

Scenario D<br />

Presentation: A 66-year-old physician is brought to the emergency department with fatigue, sadness,<br />

difficulty concentrating, and suicidal ideation. His symptoms started about 6-9 months ago. He is an avid<br />

hunter. He has recently lost his practice because of questions from the medical board about his prescribing<br />

habits. His wife left him last year, because she couldn't handle the investigation by the medical board. He<br />

admits to alcohol use and is drinking at least a bottle of wine daily.<br />

What is the diagnosis, and what are the patient's risk factors for suicide?<br />

Scenario E<br />

Presentation: A 54-year-old woman is brought to the emergency department by her husband. She has a<br />

history of bipolar disorder and takes daily medications. Lately, she has been more tired, confused, and<br />

has had nausea and diarrhea. She was recently started on furosemide by her primary care provider for<br />

hypertension. On examination, she has a coarse tremor.<br />

What is the diagnosis?<br />

708


PSYCHOBEHAVIORAL DISORDERS: PRACTICE CLINICAL SCENARIOS<br />

ANSWERS TO PRACTICE CLINICAL SCENARIOS<br />

Scenario A<br />

Diagnosis: alcohol withdrawal seizures<br />

Management: Administer IV benzodiazepines (lorazepam is the preferred agent) and titrate to response,<br />

and consider scheduled dosing of IV benzodiazepines in addition to following the patient's symptoms<br />

of withdrawal for as-needed dosing. Also consider giving thiamine (100 mg), magnesium (2-4 g), and IV<br />

hydration with dextrose-containing fluids.<br />

Scenario B<br />

Diagnosis: conversion disorder<br />

Management: Provide reassurance that there is no serious medical problem, and suggest that the symptoms<br />

will resolve. Refer for medical evaluation, because 25%-50% of patients diagnosed with conversion<br />

disorder are found later to have medical illness that requires treatment, even if it does not require emergent<br />

care.<br />

Scenario C<br />

Diagnosis: ca ta ton ia<br />

Management: Administer IV benzodiazepines, in particular lorazepam 1-2 mg every 3 hours for 2 or 3<br />

doses and observe for response. If the patient responds to the benzodiazepines, he would still require<br />

admission for further treatment. If the patient does not respond to medical management, ECT is the secondline<br />

therapy.<br />

Scenario D<br />

Diagnosis: major depressive disorder<br />

Diagnostic evaluation: This patient's risk factors for suicide include access to guns, male sex, age, marital<br />

status, recent loss of job license, and alcohol abuse. Evaluation in the emergency department includes a<br />

drug screen in most cases, observation by a sitter, and search for weapons and/or pills with the assistance<br />

of security if needed. Remember that the second most common sentinel event resulting in in-hospital<br />

mortality is in-hospital suicide.<br />

Management: Admit for in-patient treatment of depression.<br />

Scenario E<br />

Diagnosis: lithium toxicity, likely caused by recent initiation of diuretic<br />

Diagnostic evaluation: Laboratory tests include a lithium level, serum biochemical screen, and urinalysis.<br />

A creatinine level is key, because lithium is renally excreted.<br />

Management: Administer IV fluids, and hold lithium and diuretic. If the lithium level is >4 mEq/L<br />

(regardless of symptoms) or >2.5 mEq/L with symptoms, the patient should have emergency dialysis.<br />

709


710<br />

NOTES


HEMATOLOGIC DISORDERS<br />

HEMATOLOGIC DISORDERS<br />

Recognition of Bleeding Disorders ............................................................................................................................ 714<br />

Pathophysiology of Hemostasis .......................................................................................................................... 714<br />

Laboratory Tests for Evaluation of Hemostasis .................................................................................................... 714<br />

Bleeding Disorders and Their Clinical Parameters ............................................................................................... 717<br />

Bleeding Disorders Associated with Platelet Function Problems and/or a Decreased Platelet Count ........................ 718<br />

Platelet Disorders .............................................................................................................................................. 718<br />

Heparin-Induced Thrombocytopenia .................................................................................................................. 721<br />

Disseminated lntravascular Coagulation ............................................................................................................. 723<br />

Bleeding (Coagulation) Disorders Associated with an Increased Partial Thromboplastin Time and Normal<br />

Prothrombin Time ..................................................................................................................................................... 724<br />

Hemophilia ........................................................................................................................................................ 724<br />

von Willebrand Disease ..................................................................................................................................... 725<br />

Sickle Cell Disease .................................................................................................................................................... 725<br />

Anemia ...................................................................................................................................................................... 728<br />

General Approach .............................................................................................................................................. 728<br />

Clinical Presentation .......................................................................................................................................... 728<br />

Diagnostic Evaluation ......................................................................................................................................... 728<br />

Nonhereditary Anemia: Hemolytic Anemia ........................................................................................................ 731<br />

Acquired Nonhemolytic Anemias ....................................................................................................................... 731<br />

Hereditary Anemia ............................................................................................................................................. 732<br />

Blood Transfusions .................................................................................................................................................... 733<br />

Transfusion of Specific Blood Products ............................................................................................................... 733<br />

Transfusion Methods .......................................................................................................................................... 734<br />

Complications .................................................................................................................................................... 734<br />

Reversal of Drug-Induced Anticoagulation ............................................................................................................... 735<br />

Reversal ofWarfarin-lnduced Coagulopathy ....................................................................................................... 735<br />

Reversal of Heparin and Low-Molecular-Weight Heparin ................................................................................... 737<br />

Reversal of Newer Anticoagulants ...................................................................................................................... 738<br />

Reversal ofThrombolysis .................................................................................................................................... 738<br />

711


HEMATOLOGIC DISORDERS: SELF-ASSESSMENT QUESTIONS<br />

HEMATOLOGIC DISORDERS: SELF-ASSESSMENT QUESTIONS<br />

1. All of the following factors are vitamin K-dependent except:<br />

(a)<br />

II<br />

(b) VII<br />

(c)<br />

IX<br />

(d) XI<br />

2. Isolated prolongation of the prothrombin time may be seen in each of the following patients except:<br />

(a) Patients taking warfarin<br />

(b) Patients with liver disease<br />

(c) Patients with vitamin K deficiency<br />

(d) Patients with hemophilia<br />

3. The test that is most appropriate for evaluating the tissue factor pathway (extrinsic) is:<br />

(a) Platelet aggregation studies<br />

(b) Prothrombin time<br />

(c) Thrombin time<br />

(d) Partial thromboplastin time<br />

4. All of the following laboratory findings are consistent with disseminated intravascular coagulation except:<br />

(a)<br />

Increased fibrinogen level<br />

(b) Decreased platelets<br />

(c) Fragmented RBCs and anemia<br />

(d) Increased fibrin split products<br />

5. The laboratory test that distinguishes hemophilia A and B from von Willebrand disease is:<br />

(a)<br />

Partial thromboplastin time<br />

(b) Platelet function studies<br />

(c) Prothrombin time<br />

(d) Platelet count<br />

6. Which of the following is effective in treating hemophilia A, hemophilia B, and von Willebrand disease (although<br />

not necessarily the therapy of choice)?<br />

(a)<br />

Cryoprecipitate<br />

(b) DDAVP<br />

(c) Prothrombin complex<br />

(d) Fresh frozen plasma<br />

7. Which of the following labile clotting factors are most depleted in a stored unit of blood?<br />

(a)<br />

Factors VIII and I<br />

(b) Factors VIII and V<br />

(c) Factors VII and V<br />

(d) Factors IX and XI<br />

8. Al I of the following statements regarding cryoprecipitate are true except:<br />

(a) It is effective in treating all types of von Willebrand disease.<br />

(b) It contains large quantities of factors I, VIII, and IX.<br />

(c) Use of this product is associated with risk of transmission of the AIDS and hepatitis viruses.<br />

(d) Although not the therapy of choice, it is effective in treatment of hemophilia A.<br />

9. Which of the following statements about aplastic crisis in sickle cell patients is inaccurate?<br />

(a) It is frequently precipitated by an infection.<br />

(b) Patients complain of abdominal pain, and physical examination reveals splenomegaly.<br />

(c) The diagnosis is suggested by the presence of a decreased reticulocyte count and a low hemoglobin.<br />

(d) Patients are usually shocky and require oxygen, folic acid, fluid resuscitation, and transfusion.<br />

712


HEMATOLOGIC DISORDERS: SELF-ASSESSMENT QUESTIONS<br />

10. The most accurate test for evaluating the adequacy of anticoagulation in a patient on chronic warfarin therapy is:<br />

(a) Prothrombin time<br />

(b) Partial thromboplastin time<br />

(c) INR<br />

(d) The prothrombin time and INR are of equal accuracy in such a patient and can be used interchangeably.<br />

11. A 7-year-old white boy is brought in from summer camp for evaluation of a head injury. The counselor<br />

accompanying him reports that the boy was accidentally struck in the head with a bat. The injury occurred about 30<br />

minutes ago and according to the counselor, the boy now seems somewhat more lethargic. The counselor also states<br />

that the child has hemophilia A. Examination reveals a frontal hematoma and a resolving hemarthrosis of the left<br />

knee. You decide to immediately administer factor VIII concentrate to this child in whom you suspect CNS bleeding.<br />

The most appropriate dosage of factor VI II concentrate is:<br />

(a)<br />

18 units/kg<br />

(b) 26 units/kg<br />

(c) 50 units/kg<br />

(d) Cannot be determined from the information given<br />

12. A 40-year-old woman presents with easy bruising, gingival bleeding when she brushes her teeth, and menorrhagia.<br />

Examination reveals multiple purpuric lesions that are particularly prevalent on the lower extremities, but there<br />

is no active bleeding. Laboratory findings include a platelet count of 25,000/mm3, a hemoglobin of 12.8 g!dL, a<br />

BUN of 15 mg/dL, a creatinine of 0.8, and a normal prothrombin time (INR)/partial thromboplastin time. The most<br />

appropriate initial therapy is:<br />

(a) Platelet transfusion<br />

(b) Plasmapheresis<br />

(c) Corticosteroids<br />

(d) Immediate splenectomy<br />

13. A 50-year-old-man presents with profuse rectal bleeding. He is currently on warfarin, and his INR is 5.2. His<br />

hemoglobin is 10.2 g/dl. The most appropriate initial therapy is:<br />

(a) Platelet transfusion<br />

(b) IV vitamin K<br />

(c) DDAVP<br />

(d) Cryoprecipitate<br />

14. Which of the following statements about anticoagulation reversal is false?<br />

(a) Owing to the risk of anaphylaxis, vitamin K should not be given IV.<br />

(b) Prothrombin complex concentrate (PCC) is indicated for the reversal of warfarin in patients with an increased<br />

INR who are bleeding.<br />

(c) Although not FDA approved, factor Vil inhibitor bypassing activity (FEIBA) has been shown in small studies to<br />

reverse the anticoagulant effects of dabigatran and rivaroxaban.<br />

(d) Protamine can partially reverse the effects of low-molecular-weight heparin (LMWH).<br />

15. A patient presents with progressive fatigue and shortness of breath. Laboratory studies show a hemoglobin of 8 g/dl,<br />

WBC count of 8.3 x10 9 /L, and a platelet count of 21 0/mm 3 . Her reticulocyte count is increased, and her serum<br />

haptoglobin is low. She has a total bilirubin of 4 and a direct bilirubin of 1.2. What is the most likely cause of the<br />

anemia?<br />

(a) Hemolysis<br />

(b) Iron deficiency<br />

(c) Thrombotic thrombocytopenia purpura<br />

(d) Bone marrow failure<br />

ANSWERS<br />

I. d 4. a 7. b 10. C 13. b<br />

2. d 5. b 8. b 11. c 14. a<br />

3. b 6. d 9. b 12. c 15. a<br />

Use the pre-chapter multiple choice question worksheet (page xvi) to record and determine the percentage of correct<br />

answers for this chapter.<br />

713


HEMATOLOGIC DISORDERS<br />

I. RECOGNITION OF BLEEDING DISORDERS<br />

A. Pathophysiology of hemostasis<br />

1. Hemostasis is a balance between excessive bleeding and thrombosis. It occurs when four components (listed<br />

below) interact synergistically with one another. If a component is not functioning normally, or it fails to<br />

interact appropriately with another component, excessive bleeding or clotting occurs.<br />

Contact Activation Pathway<br />

(Intrinsic)<br />

Deficiency =<br />

Hemophilia A<br />

•<br />

.<br />

.<br />

Deficiency =<br />

Hemophilia B<br />

Tissue Factor Pathway<br />

(Extrinsic)<br />

COMMON PATHWAY<br />

Thrombin (II)<br />

Thrombin<br />

Fibrin<br />

Ogen (I)<br />

Coagulation Very Simplified<br />

Courtesy of Colin Kaide, MD<br />

2. Components of hemostasis<br />

a. Vascular integrity: primary<br />

b. Platelet function: primary<br />

c. Coagulation factors: secondary<br />

d. Fibri nolysis<br />

3. Bleeding disorders result from a defect in one or more of the components of hemostasis.<br />

a. Inflammation of blood vessel walls, or a defect in the vessel supportive connective tissue, causes increased<br />

aggregation of platelets at the vessel site but does not affect the total number of circulating platelets.<br />

Bleeding, therefore, is localized to the vessel site.<br />

b. Platelet function depends on an adequate number of circulating platelets, as well as normally functioning<br />

platelets. Bleeding results when either the count is low or the function is abnormal.<br />

c. Coagulation factors are activated by substances in injured tissue that set off a series of steps that terminate<br />

in the formation of a fibrin clot. If there is an inadequate amount of one or more factors, or if a factor is<br />

abnormal, this series of steps is disrupted, thus preventing formation of a fibrin clot.<br />

d. Fibrinolysis normally results in dissolution of the fibrin clot as the vascular defect is repaired. Excessive<br />

fibrinolysis causes bleeding by functionally altering coagulation factors or by inducing excessive<br />

consumption of coagulation factors.<br />

B. Laboratory tests for evaluation of hemostasis<br />

1 . Platelet count<br />

a. Normal range: 150,000-400,000/mm 3<br />

b. Decreased in any condition that causes destruction, sequestration, or decreased production of platelets:<br />

714


HEMATOLOGIC DISORDERS<br />

(1) Idiopathic (autoimmune) thrombocytopenic purpura (ITP)<br />

(2) Thrombotic thrombocytopenic purpura (TTP)<br />

(3) Leukemia<br />

(4) Bone marrow suppression/replacement<br />

(5) Drug/toxic effect<br />

(6) Hypersplenism<br />

(7) Disseminated intravascular coagulation (DIC)<br />

(8) HIV infection<br />

(9) Sepsis (especially gram-negative sepsis)<br />

(10) Aplastic/hypoplastic anemia<br />

c. Post-traumatic bleeding occurs if the platelet count falls below 50,000/mm 3 ; spontaneous, lifethreatening<br />

hemorrhage can occur when the platelet count falls below 10,000-20,000/mm 3 •<br />

d. As many as 50% of patients with unexplained thrombocytosis (t platelet count) have an underlying<br />

malignancy.<br />

2. Testing of platelet function<br />

a. Bleeding time is no longer used clinically and is mentioned here only for historical purposes. It measures<br />

the time between skin incision and cessation of bleeding. Duration of this type of bleeding depends on<br />

the integrity of primary hemostasis (platelets, von Willebrand factor) and is independent of coagulation.<br />

b. Platelet function testing is not routinely used and has limited utility in the emergency department.<br />

(However, the emergency physician may come across these tests and their results and should at<br />

least be familiar with the concept.) It has replaced bleeding time as the test of choice for evaluating<br />

platelet activity. Platelet aggregation studies can confirm the inhibitory effects of ASA, thienopyridines<br />

(ticlopidine, clopidogrel), 13-lactam antibiotics, and other factors on platelet function.<br />

c. Platelet function tests measure platelet aggregation after exposure to a panel of agonists, which may<br />

include ADP, epinephrine, collagen, arachidonic acid, ristocetin, and thrombin. The pattern obtained<br />

allows for the diagnosis and classification of the platelet defect.<br />

3. Prothrombin time (PT)<br />

a. Tests the factors of the tissue factor (extrinsic) pathway (primarily factor VII) and the common pathway<br />

(X, V, II, and I).<br />

b. Normal control values are 10-12 seconds.<br />

c. Prolongation of PT c':2 seconds is considered significant.<br />

d. Increased PT (with a normal PTT) results from a deficiency of factor VI I. Causes of factor VII deficiency are:<br />

(1) Hereditary (rare)<br />

(2) Acquired secondary to:<br />

(a) Vitamin K deficiency<br />

(b) Warfarin use<br />

(c) Liver disease<br />

(3) Although the PT is very useful in evaluating patients with hemostatic abnormalities, it should not<br />

be used to monitor the adequacy of anticoagulation with warfarin. The response of the PT to a<br />

dose of warfarin is very dependent on the thromboplastin used in the assay, which varies from<br />

laboratory to laboratory. Thus, reliance on the PT alone in these patients can result in excessive or<br />

suboptimal anticoagulation and an increased risk of bleeding complications. For this reason, it is now<br />

recommended that the international normalized ratio (INR) be used to monitor these patients.<br />

4. INR<br />

a. The INR is the PT ratio that would be obtained if the World Health Organization's reference<br />

thromboplastin had been used.<br />

b. It is the test of choice for monitoring the adequacy of anticoagulation in patients taking warfarin.<br />

c. An INR of 1 is normal.<br />

d. An INR of 2-3 is considered to be therapeutic for most patients, while an INR of 2.5-3.5 is recommended<br />

for patients with mechanical prosthetic valves and recurrent embolization.<br />

5. Partial thromboplastin time (PTT)<br />

a. Essentially tests all of the factors except VII and XIII but is used primarily to evaluate the factors of the<br />

contact activation (intrinsic) pathway; it tests components of both the contact activation (intrinsic) and<br />

common pathways.<br />

b. Normal control values range from 25 to 35 seconds.<br />

715


HEMATOLOGIC DISORDERS<br />

c. A prolongation of 8-1 0 seconds above normal is considered significant.<br />

d. Increased PTT (with a normal PT) is seen with:<br />

(1) Factor VIII, IX, or XI deficiency (primarily)<br />

(2) Factor XII deficiency (asymptomatic)<br />

(3) Heparin therapy<br />

e. It is common to see the PTT reported as "aPTT," which stands for activated partial thromboplastin time,<br />

called this because an "activating" agent (kaolin) was added to the plasma before the test was run.<br />

f. When laboratory tests reveal an increased PT and increased PTT, this indicates an abnormality primarily<br />

in the common pathway (associated abnormalities of the intrinsic and extrinsic pathways may also be<br />

present).<br />

(1) Heparin overdose (factors X and 11 are primary sites of action)<br />

(2) Warfarin overdose (inhibits factors II, VII, IX, and X)<br />

(3) Vitamin K deficiency and moderate to severe liver disease (inhibit production of factors II, VII, IX,<br />

and X)<br />

(4) DIC (consumes factors I, V, VIII, and XIII)<br />

(5) Functional abnormality or deficiency of fibrinogen (factor I)<br />

(6) Massive transfusion of stored blood (deficient in factors V, VIII)<br />

(7) Thrombolytic therapy<br />

6. Thrombin time<br />

a. Tests the ability of fibrinogen (factor I) to convert into a fibrin clot. This is a useful screening test for both<br />

qualitative and quantitative abnormalities of fibrinogen.<br />

b. Increased thrombin time is seen with:<br />

(1) Hypofibrinogenemia<br />

(2) Myeloma, macroglobulinemia (interferes with fibrin polymerization)<br />

(3) Heparin therapy (inhibits fibrinogen formation)<br />

(4) DIC (consumes fibrinogen)<br />

(5) Liver disease<br />

(6) Uremia<br />

7. Fibrinogen degradation products test for specific degradation products of fibrin. Increased levels are seen<br />

with consumption coagulopathies (eg, DIC) and thrombolytic agent therapy.<br />

716


HEMATOLOGIC DISORDERS<br />

C. Bleeding disorders and their clinical parameters<br />

Table 43: Clinical Parameters of Hemostatic Disorders<br />

Hemostatic<br />

Platelet<br />

Function<br />

Disorder Clinical Findings Studies Platelets PT PTT Fibrinogen Clinical Disorder<br />

Primary hemostasis: platelet mediated<br />

Thrombocytopenia Purpura; petechiae; Normal Normal Normal Normal ITP, TTP, hemolytic<br />

epistaxis; GI, GU,<br />

uremic syndrome,<br />

and menstrual<br />

heparin-induced<br />

bleeding<br />

thrombocytopenia<br />

(abnormal PTT if<br />

still on heparin),<br />

others<br />

Platelet dysfunction Purpura; petechiae; Abnormal Normal Normal Normal Normal ASA, clopidogrel,<br />

epistaxis; GI, GU,<br />

ticlopidine,<br />

and menstrual<br />

inherited disorders,<br />

bleeding<br />

others<br />

von Willebrand Purpura; petechiae; Abnormal Normal Normal Normal Normal von Willebrand<br />

disease epistaxis; GI, GU, to t' disease<br />

and menstrual<br />

bleeding;<br />

hemarthrosis and<br />

muscular bleedinga<br />

Secondary hemostasis: coagulation<br />

Warfarin therapy Normal Normal Normal Normal Warfarin therapy,<br />

to fb<br />

rat poison<br />

(brodifacoum,<br />

others)<br />

Heparin Normal Normal Normal Normal Heparin therapy<br />

Low-molecularweight<br />

heparin<br />

(LMWH)<br />

Intramuscular,<br />

intracranial, GI,<br />

postoperative, and<br />

traumatic bleeding<br />

Normal Normal Normal Normal to Normal Enoxaparin,<br />

mildly t'<br />

dalteparin,<br />

fondaparinux,<br />

tinzaparin<br />

Hemophilia A Normal Normal Normal Normal Factor VIII levels<br />

decreased<br />

Hemophilia B Normal Normal Normal Normal Factor IX levels<br />

(Christmas disease)<br />

decreased<br />

Combined disorders of platelets and coagulation<br />

Disseminated Thrombosis, Normal td Sepsis, trauma,<br />

i ntravascu lar microangiopathic burns, acute<br />

coagulation hemolytic anemia, promyelocytic<br />

bleeding (purpura;<br />

leukemia, amniotic<br />

petechiae; epistaxis;<br />

fluid embolism,<br />

GI, GU, menstrual,<br />

placental<br />

intramuscular,<br />

abruption, snake<br />

i ntracran ial,<br />

bites (pit viper),<br />

postoperative, and<br />

liver disease<br />

traumatic bleeding)<br />

0<br />

In type 3 and type 2N (abnormal binding of von Willebrand factor to factor VIII, resembling hemophilia A)<br />

b<br />

'<br />

If INR/PT is sufficiently prolonged by warfarin, PTT will also be prolonged, owing to warfarin's effect on the common pathway.<br />

LMWH is monitored using antiXa levels.<br />

" Fibrin split products and d-dimers are increased as the result of concomitant thrombolysis during DIC.<br />

717


HEMATOLOGIC DISORDERS<br />

II. BLEEDING DISORDERS ASSOCIATED WITH PLATELET FUNCTION<br />

PROBLEMS AND/OR A DECREASED PLATELET COUNT<br />

718<br />

A. Platelet disorders<br />

1. Thrombocytopenia (the most common platelet abnormality)<br />

2. ITP<br />

a. Etiology<br />

(1) Decreased bone marrow production<br />

(a) Malignant infiltration of bone marrow<br />

(b) Myelofibrosis<br />

(c) Aplastic anemia<br />

(d) Drug suppression (ethanol, thiazide diuretics, chemotherapeutic agents)<br />

(e) Radiation therapy<br />

(f)<br />

Viral infections<br />

(g) Vitamin 8 1<br />

/folate deficiency<br />

(2) Hypersplenism (cirrhosis, uremia, sequestration)<br />

(3) Increased platelet destruction<br />

(a) ITP<br />

(b) TTP<br />

(c) Drug-related destruction (eg, penicillin, sulfonamides, quinine, furosemide, heparin, ASA, gold<br />

salts, cimetidine, digoxin)<br />

(d) Hemolysis, elevated liver enzymes, and low platelet count (HELLP) syndrome<br />

(e) Sepsis<br />

(f)<br />

Viral infection: HIV, mumps, varicella, Epstein-Barr virus<br />

(g) Collagen vascular diseases (especially lupus erythematosus)<br />

(h) Hemolytic uremic syndrome<br />

(i)<br />

Post-transfusion (middle-aged woman previously sensitized to the P1 Al antigen during pregnancy<br />

who received a transfusion 1 week ago)<br />

b. Classic clinical scenario<br />

(1) A patient (usually a woman) presents with easy bruising, epistaxis, gingival bleeding, hematuria,<br />

menorrhagia, and/or GI bleeding. The bleeding may be spontaneous or secondary to trauma and is<br />

usually controlled by local pressure. When secondary to trauma, the bleeding has immediate onset. In<br />

some cases, the patient may also relate that she was recently started on a new medication.<br />

(2) Examination reveals multiple petechiae and purpura on the mucous membranes and skin, particularly<br />

on the dependent parts of the body.<br />

(3) Laboratory findings include a low platelet count and a normal PT(INR)/PTT.<br />

c. Indications for platelet transfusion (6 units of random donor platelet concentrate increases the platelet<br />

count by 50,000/mm 3 )<br />

(1) Platelet count


HEMATOLOGIC DISORDERS<br />

(3) Emergency treatment for severe bleeding (platelets


HEMATOLOGIC DISORDERS<br />

von Willebrand Multimer<br />

Schistocytes<br />

Q Platelet<br />

von Willebrand factor<br />

Courtesy of Colin Kaide, MO<br />

f. Mortality is 80% if untreated.<br />

g. Early diagnosis is a key factor in survival.<br />

h. Clinical presentation<br />

(1) It is rare to see all of these findings in TTP, especially if the patient presents early in the course of the<br />

disease. Later in the course, more findings may be present.<br />

(a) Fever: least common finding in TTP<br />

(b) Anemia: microangiopathic hemolytic anemia<br />

(c) Thrombocytopenia<br />

(d) Renal abnormalities: less common in TTP and more common in hemolytic uremic syndrome<br />

(e) Neurologic abnormalities: found in two-thirds of patients with TTP<br />

(2) Most patients in this era do not have all five features, because it is now treated before progression to<br />

its natural end; untreated, the mortality is ~80%.<br />

(3) All that is needed for diagnosis is thrombocytopenia and microangiopathic hemolytic anemia<br />

(produces schistocytes). Schistocytes (> 1 %) are damaged and fragmented RBCs. They are never<br />

normal on a peripheral smear.<br />

(4) Neurologic symptoms in two-thirds of cases; global or focal via CNS thrombosis. The pattern of CNS<br />

symptoms can be confusing. Owing to microvascular plugging that can come and go in constantly<br />

varying locations, the patient may experience odd, transient atypical complaints. They can range<br />

from transient speech problems, to patchy areas of numbness, to focal weakness. It is important to<br />

ask about waxing and waning symptoms over the preceding few days.<br />

(5) Renal failure is more common in hemolytic uremic syndrome but can also happen in TTP.<br />

(6) Fevers are less common.<br />

(7) Mortality rate is high if untreated.<br />

(8) An increased lactate dehydrogenase and a decreased haptoglobin are usually seen in TTP and are<br />

markers of hemolysis.<br />

1. TTP versus DIC<br />

(1) DIC is a "consumptive coagulopathy."<br />

(a) Low platelets<br />

(b) Low coagulation factors (high PTT and INR)<br />

(c) Low fibrinogen<br />

(d) Microangiopathic hemolytic anemia and fibrin plugs<br />

(2) TTP is a platelet problem<br />

(a) Low platelets<br />

(b) Normal coagulation factors (normal PTT and INR)<br />

(c) Normal fibrin<br />

(d) Microangiopathic hemolytic anemia and platelet plugs<br />

720


HEMATOLOGIC DISORDERS<br />

j. Treatment<br />

(1) Begin steroids: methylprednisolone 125 mg<br />

(2) Dialysis may be needed if renal failure.<br />

(3) Plasma exchange transfusion is the key.<br />

(4) Give fresh frozen plasma (FFP) if plasma exchange will be delayed for any reason (extended transfer<br />

times, equipment issues, etc). This gives back the missing ADAMTS13 from someone else's blood and<br />

can transiently improve TTP symptoms until definitive care can be initiated.<br />

(5) Avoid platelet transfusions.<br />

B. Heparin-induced thrombocytopenia (HIT)<br />

1. Epidemiology<br />

a. Up to 5% of heparinized patients<br />

b. Develops in 5-10 days<br />

c. Can develop earlier if the patient has had prior HIT episodes<br />

d. Late presentation (>10 days after heparin) is also possible but less common.<br />

e. Occurs as the result of an lgG antibody that forms against heparin when it is bound to platelet factor 4<br />

f. HIT can develop after heparin doses as small as those used for repetitive heparin flushes.<br />

g. HIT can develop in patients who are heparinized repeatedly for dialysis.<br />

h. Regular heparin > LMWH<br />

1. Surgical > medical > obstetrics patients<br />

j. Female > male<br />

Heparin<br />

Platelet Removal<br />

Thrombocytopenia<br />

Thrombosis<br />

Heparin-induced thrombocytopenia<br />

Courtesy of Colin Kaide, MD<br />

2. Clinical presentation (4 T's)<br />

a. Ihrombocytopenia (


HEMATOLOGIC DISORDERS<br />

3. Treatment: The 3 Do's<br />

a. Stop heparin or LMWH!<br />

b. Start alternative anticoagulant. This should be a direct thrombin (factor Ila) inhibitor. Don't delay! These<br />

drugs include argatroban, lepirudin, and bivalirudin.<br />

c. Reverse warfarin. If INR is increased, warfarin reversal should be done with vitamin K 10 mg orally. There<br />

is no need to use other reversal agents such as FFP, prothrombin complex concentrates (eg, nonactivated<br />

factor IX complex), or recombinant coagulation factor Vila, because there is no active bleeding and<br />

reversal is not time dependent. (Warfarin reversal seems counterintuitive; why would you want to reverse<br />

anticoagulation in someone who is now hypercoagulable? Disturbance in procoagulant-anticoagulant<br />

balance is the reason for the need to reverse warfarin. Warfarin has two major effects. First, it causes<br />

an immediate decrease in protein C-mediated anticoagulation. This means that warfarin initially has<br />

a procoagulant effect. After a few days of use, warfarin also inhibits the production of active forms of<br />

factors II, VII, IX, and X. This is its anticoagulant effect. After a few days, the net balance of these two<br />

effects is to cause anticoagulation. When protein C levels are decreased by warfarin and HIT antibodies<br />

are present [procoagulants], the balance shifts toward the formation of clots.)<br />

4. Treatment: The 2 Don'ts<br />

a. Avoid warfarin: reverse with vitamin Kif INR increased.<br />

b. Avoid platelet transfusions: may worsen the hypercoagulability.<br />

5. Diagnostic evaluation<br />

a. Investigate for lower-limb deep-venous thrombosis (duplex ultrasonography).<br />

b. Test for HIT antibodies. This looks for lgG antibodies to the heparin-platelet factor 4 complex. This is not<br />

a useful test for the emergency physician. It is worth sending the test for the purpose of confirming the<br />

diagnosis for the inpatient physicians. The test will take a while to complete and will probably be a "send<br />

out" test for most emergency departments.<br />

Courtesy of Colin Kaide, MD<br />

Bleeding at LMWH injection site with HIT<br />

Courtesy of Colin Kaide, MD<br />

Clotting in extremity with HIT<br />

722


HEMATOLOGIC DISORDERS<br />

C. Disseminated intravascular coagulation (DIC)<br />

1. Definition: life-threatening bleeding disorder resulting from loss of platelets and coagulation factors,<br />

fibrinolysis, small vessel occlusion, and RBC destruction from fibrin deposition<br />

2. Clinical presentation<br />

a. Patients usually present with diffuse bleeding from multiple sites (skin, mucous membranes, and viscera)<br />

that is not control led by local measures.<br />

b. At times, acrocyanosis, thrombosis, and pregangrenous changes of fingers, toes, genitalia, and nose are<br />

presenting symptoms (purpura fulminans).<br />

3. Etiology<br />

a. Trauma (including burns, crush injuries, and brain trauma)<br />

b. Pregnancy complications (placental abruption, amniotic fluid emboli, fetal death in utero)<br />

c. Sepsis and a host of infectious diseases<br />

d. Transfusion and drug reactions<br />

e. Carcinoma and acute leukemias<br />

f. Liver disease<br />

g. Snake bites<br />

h. Prosthetic devices (especially peritoneovenous shunts)<br />

1. Heat stroke<br />

4. Diagnostic evaluation<br />

a. Clinical and laboratory triads<br />

(1) Purpura (petechiae) --.. decreased platelets<br />

(2) A bleeding tendency--.. increased PT(NR)/PTT<br />

(3) Signs of organ injury--.. decreased fibrinogen<br />

b. Additional laboratory findings<br />

5. Treatment<br />

(1) j Fibrin split products (the earliest sign) and j o-dimer (most specific sign)<br />

(2) i Thrombin time<br />

(3) Fragmented RBCs (schistocytes): evidence of microangiopathic hemolytic anemia<br />

a. Primary therapy is directed toward the underlying cause when it is easily identifiable (sepsis, pregnancy,<br />

drug or transfusion therapy) and correction of hemodynamic instability (IV fluids and packed RBCs).<br />

b. Specific therapy is based on the clinical presentation.<br />

(1) If hemorrhage predominates the clinical picture, then blood component therapy is indicated. (All three<br />

should be given.)<br />

(a)<br />

Platelets<br />

(b) FFP (coagulation factors and fibrinogen)<br />

(c) Cryoprecipitate (factors I and VIII)<br />

(2) If fibrin deposition and thrombosis predominate the clinical picture, then low-dose IV heparin is<br />

indicated. Disease states in which this occurs include:<br />

(a) Gram-negative sepsis in pregnancy<br />

(b) Retained dead fetus<br />

(c) Chronic DIC (eg, carcinoma)<br />

(d) Purpura fulminans<br />

(3) Antithrombin Ill concentrates may also be used to treat DIC, either alone or in conjunction with<br />

heparin. Antithrombin Ill is an endogenous inhibitor of coagulation that binds to and inactivates<br />

thrombin and several other factors.<br />

723


HEMATOLOGIC DISORDERS<br />

Ill. BLEEDING (COAGULATION) DISORDERS ASSOCIATED WITH<br />

AN INCREASED PARTIAL THROMBOPLASTIN TIME (PTT) AND<br />

NORMAL PROTHROMBIN TIME (PT)<br />

A. Hemophilia<br />

1. Characteristics of both hemophilia A and hemophilia B<br />

a. Bleeding history<br />

(1) Delayed and protracted bleeding after mild trauma or dental extractions<br />

(2) Spontaneous hematuria<br />

(3) Hemarthrosis and muscle hematomas (~90% of all bleeding)<br />

(4) lntracranial hemorrhages (a major cause of death in patients with hemophilia A)<br />

b. Diagnostic evaluation<br />

(1) Prolonged PTT<br />

(2) Normal PT and platelets<br />

2. Hemophilia A (classic hemophilia)<br />

a. A sex-linked recessive disorder due to a deficiency in the production of factor Vlll:C<br />

b. Although a sex-linked disorder with women being carriers and men being affected, spontaneous<br />

mutations can produce female hemophiliacs. Additionally, if a female carrier reproduces with a<br />

male hemophiliac, the predicted offspring can include a carrier daughter, a hemophiliac daughter, a<br />

hemophiliac son, and an unaffected son.<br />

c. In addition to a prolonged PTT, assay of factor Vlll:C is abnormal.<br />

d. Treatment<br />

(1) Factor VIII concentrate: the treatment of choice for those with moderate to severe disease. The<br />

products that are now available are all free of the risk of transmission of AIDS and hepatitis (types B<br />

and C) and include:<br />

(a) Heat-treated factor VIII concentrate (hepatitis A and parvovirus transmission is still possible)<br />

(b) Monoclonal antibody purified factor VIII concentrate<br />

(c) Recombinant factor VIII concentrate (an ultrapure product)<br />

(d) Dosing: primarily determined by the type/location of bleeding<br />

1. Mild bleeding (hematuria, early hemarthrosis, deep laceration) - 12.5 units/kg<br />

ii. Moderate bleeding (oral lacerations, dental extraction or minor surgery, late hemarthrosis)<br />

- 25 units/kg<br />

iii. Severe bleeding (CNS, GI, intra-abdominal or retroperitoneal bleeding; major trauma or<br />

surgery, documented or potentially serious head injury) - 50 units/kg<br />

iv. Each unit of factor VIII infused per kg of body weight increases the plasma factor VIII level<br />

by 2%. In a suspected bleeding emergency, a hemophiliac patient's present level of factor<br />

VIII is assumed to be zero unless previously known.<br />

(2) Cryoprecipitate carries a risk of hepatitis and AIDS viruses as well as of allergic reactions and, for<br />

this reason, its use has declined; it is no longer considered first-line therapy. (One bag equals 80-100<br />

units of factor VIII).<br />

(3) Desmopressin (DDAVP) is effective in increasing the levels of factor VIII in patients with mild<br />

hemophilia and is the treatment of choice for these patients; dosage is 0.3 mcg/kg. There is no risk of<br />

infectious disease transmission.<br />

(4) FFP: volume constraints generally limit its use to patients with mild hemophilia. Infectious disease<br />

transmission can occur. (1 unit of FFP raises the factor level only 3%-5%)<br />

3. Hemophilia B (Christmas disease)<br />

a. A sex-linked recessive disorder caused by a deficiency of factor IX<br />

b. In addition to a prolonged PTT, assay of factor IX is abnormal.<br />

c. Treatment<br />

(1) Factor IX concentrate is currently the treatment of choice for patients with hemophilia B. The<br />

monoclonal concentrates are free from the risk of transmission of hepatitis and AIDS.<br />

724


HEMATOLOGIC DISORDERS<br />

(2) Prothrombin complex concentrates contain factors II, VII, IX, and X, which are also effective but have<br />

thrombogenic adverse effects and may transmit hepatitis.<br />

(3) FFP is effective too, but carries the risk of infectious disease transmission.<br />

B. von Willebrand disease (vWD)<br />

1. This is an autosomal (usually dominant) disorder affecting both males and females.<br />

2. vWD is the most common genetic bleeding disorder. There are three major types:<br />

a. Type I (most common): decreased von Willebrand factor (vWF)<br />

b. Type II: abnormal or dysfunctional vWF<br />

c. Type Ill (rare): almost no vWF if present (serious disease)<br />

3. Depending on the specific type of vWD present, these patients have a deficiency or abnormality of one or<br />

more of the fol lowing:<br />

a. vWF: promotes platelet adhesion; this is the primary deficiency found in vWD.<br />

b. vWF: Ag<br />

c. FactorVlll:C: protected by vWF<br />

4. Clinical presentation<br />

a. Mucocutaneous bleeding (especially epistaxis, menorrhagia, and GI bleeding) predominates; hemarthroses,<br />

therefore, are rare.<br />

b. Bleeding episodes are usually milder than with hemophilia (particularly hemophilia A, which is typically<br />

associated with the most severe bleeding).<br />

5. Laboratory studies<br />

a. Increased PTT<br />

b. Abnormal platelet function studies<br />

c. Normal platelet count and PT<br />

d. Abnormal assay of one or more of the following: vWF, vWF:Ag, or factorVlll:C<br />

6. Management is determined by the type of vWD present.<br />

a. DDAVP: use is restricted to patients with mild to moderate type I vWD; it is the treatment of choice in these<br />

patients and has no risk of infectious disease transmission.<br />

b. Factor VIII concentrates containing large amounts of vWF (Humate-P and Koate-HS) are currently the<br />

treatment of choice for patients with type II and type Ill disease. Both products have been treated to<br />

eliminate the risk of transmission of infectious diseases.<br />

c. Cryoprecipitate (usual dose= 10 units/kg) is effective for all types of vWD but carries risk of transmission of<br />

hepatitis and HIV infection.<br />

d. FFP is also effective for all types of vWD but carries risk of transmission of infectious diseases. FFP may be<br />

used to treat all forms of hemophilia (hemophilia A and Band vWD), because it contains all factors. The<br />

major limitation to its use is volume overload; compared with factor VIII concentrate, factor IX concentrate,<br />

prothrombin complex, and cryoprecipitate, the concentration of factors in FFP (1 unit/ml) is low. Therefore,<br />

excessively large volumes would be required to treat patients with moderate or severe disease. FFP is most<br />

appropriate for patients in whom the exact nature of their hemophilia is unknown.<br />

IV. SICKLE CELL DISEASE<br />

A. A genetic disorder found predominantly in people of African ancestry that is due to an abnormal<br />

hemoglobin molecule (hemoglobin S). The normal hemoglobin molecule is hemoglobin A. Hemoglobin<br />

S induces red cell deformation (sickling). These sickled cells have two properties that account for their<br />

pathologic manifestations:<br />

1. Sickled RBCs are unable to pass freely through the systemic microvasculature. This leads to thrombosis<br />

(ischemia, infarction).<br />

2. Sickled RBCs hemolyze more easily than normal RBCs. This leads to chronic hemolytic anemia and increased<br />

susceptibility to infection.<br />

725


HEMATOLOGIC DISORDERS<br />

Table 44: Sickle Cell Hemoglobin (Hb)<br />

Phenotype Genotype Hemoglobin Abnormality<br />

Sickle trait HbNS a2 bl sl Valine for Glu at 6th position<br />

Sickle cell disease HbS a2 s2 Valine for Glu at 6th position<br />

Sickle thalassemia Hb S/A-thal* a2 s2/a2b2-thal thal=decreased production of Hb A<br />

Sickle eel I disease Hb S + Hb C** a2 s2/a2 c2 Lysine for Glu at 6th position<br />

* In thalassemias, Hb A is produced in decreased amounts.<br />

** In sickle cell disease, both forms of hemoglobin exist (Hb Sand Hb C).<br />

B. Sickle cell trait (heterozygous hemoglobin AS)<br />

1. When hemoglobin S is inherited from one parent and hemoglobin A from the other, RBCs will contain both<br />

hemoglobin A and hemoglobin S (


HEMATOLOGIC DISORDERS<br />

3. Sequestration crisis (most common precipitating cause of acute anemia)<br />

a. Seen in young children (6 months to 6 years old)<br />

b. Clinical presentation: patient presents with abdominal pain and distention, pallor, shock, and<br />

splenomegaly.<br />

c. Diagnostic evaluation<br />

(1) As above<br />

(2) Hemoglobin very low (may be as low as 1-2 g) and pancytopenia<br />

d. Treatment: as above and consultation with a pediatric surgeon for possible splenectomy<br />

4. Aplastic crisis<br />

a. Usually precipitated by an infection (most often human B19 parvovirus) that suppresses erythropoiesis;<br />

folate deficiency is rarely the underlying cause.<br />

b. Clinical presentation: patient is pale, lethargic, and in shock.<br />

c. Diagnostic evaluation<br />

(1) As above and appropriate cultures<br />

(2) Decreased reticulocyte count (secondary to severe bone marrow depression) is pathognomonic and<br />

associated with a very low hemoglobin.<br />

d. Treatment<br />

(1) Support shock and transfuse blood.<br />

(2) Administer oxygen, folic acid, and antibiotics as indicated.<br />

E. Other potential complications and sequelae of sickle cell disease<br />

1. Infections (most common cause of death)<br />

a. Sepsis, meningitis, pneumonia, and osteomyelitis occur with increased frequency due to functional<br />

asplenia, poorly migrating neutrophils, and decreased opsonin activity.<br />

b. Children (particularly those 103°F [39.4°C]) and a WBC count<br />

>20,000/mm 3 should be promptly treated with broad-spectrum IV antibiotics, and cultures should be done.<br />

2. Priapism<br />

a. Definition: a painful, sustained, pathologic erection<br />

b. Occurs in 10%-40% of men and can result in impotence<br />

c. Treatment<br />

(1) Obtain immediate urologic consult and administer terbutaline 0.25-0.5 mg SC.<br />

(2) Subsequent measures<br />

(a) Hydration<br />

(b) Transfusion of packed RBCs<br />

(c) Hyperbaric oxygen<br />

(d) Analgesia (appropriate for treatment of pain but ineffective in relieving priapism)<br />

3. Acute chest syndrome<br />

a. Most commonly affects older children, adolescents, and adults<br />

b. Patients present with fever, cough, chest pain, and shortness of breath.<br />

c. Cause is multifactorial and includes both infection and infarction of pulmonary tissue.<br />

d. Diagnostic evaluation: hypoxia and new chest radiograph abnormalities<br />

e. Treatment<br />

4. Strokes<br />

(1) Supplemental oxygen, hydration, and antibiotics<br />

(2) Exchange transfusion to reduce the proportion of hemoglobin S may also be needed.<br />

a. May be thrombotic or hemorrhagic (intracranial or subarachnoid)<br />

b. Clinical presentation: patients usually present with abrupt onset of aphasia, hemiparesis, change in vision,<br />

and/or seizures.<br />

c. Diagnostic evaluation: baseline laboratory studies and CT or MRI of the brain<br />

d. Treatment: immediate exchange transfusion to decrease the amount of hemoglobin S to


HEMATOLOGIC DISORDERS<br />

8. Potential sequelae<br />

a. CHF (chest radiograph - bibasilar congestion)<br />

b. Renal insufficiency/nephrotic syndrome<br />

c. Leg ulcers (particularly on the malleoli)<br />

d. Retinal infarction/detachment<br />

e. Avascular necrosis<br />

(1) Digits (children)<br />

(2) Femoral head (young adults)<br />

9. Caution with any eye trauma. There is a significant risk of hyphema, which tends to rebleed.<br />

V. ANEMIA<br />

A. General approach<br />

1. Decide how emergent and prioritize decision making.<br />

2. Profound nontraumatic anemia causing severe compromise to the patient's physiologic functions: attention<br />

should be focused first on resuscitation, then on diagnosis of underlying causes.<br />

B. Clinical presentation<br />

1. History<br />

a. Symptoms of anemia: chest pain, exertional dyspnea, syncope<br />

b. Bleeding diathesis<br />

(1) Bleeding after trauma, surgery, tooth extractions, or injections<br />

(2) Spontaneous bleeding: epistaxis, menorrhagia<br />

(3) Spontaneous purpura, petechiae<br />

c. Intermittent jaundice/dark urine<br />

d. Dietary history: vegetarian diet or poor nutrition<br />

e. Drug use/toxin exposure<br />

f. Family history and racial background: African-American, Middle Eastern<br />

g. Underlying disease<br />

(1) Renal or liver disease<br />

(2) Hypothyroidism<br />

(3) Chronic disease states: cancer, rheumatic or renal disease<br />

2. Physical examination<br />

a. Skin: pallor, purpura, petechiae, angiomas, ulcerations<br />

b. HEENT<br />

(1) Conjunctiva! jaundice/pallor/hemorrhage, funduscopic hemorrhage<br />

(2) Tongue atrophy, papillary soreness<br />

c. Cardiopulmonary<br />

(1) Heart size, murmurs, extra-cardiac sounds<br />

(2) Wheezing, rales, other signs of pulmonary edema<br />

d. Abdomen: hepatomegaly, ascites, splenomegaly, masses, rectal/pelvic<br />

e. Neurologic: altered position or vibratory sense, peripheral neuritis<br />

C. Diagnostic evaluation<br />

1. Basic laboratory studies should be obtained: CBC with differential, platelet count, mean corpuscular volume<br />

(MCV), reticulocyte count, peripheral smear<br />

2. Other tests can be ordered as indicated.<br />

a. Suspect hemolytic anemia: direct Coombs' test, serum free hemoglobin and haptoglobin<br />

b. Suspect microangiopathic hemolytic anemia (MAHA) or DIC: PT/PTT, o-dimer, fibrin degradation products,<br />

fibrinogen<br />

728


HEMATOLOGIC DISORDERS<br />

Table 45: Differential Diagnosis of Anemia<br />

Mechanism/Type<br />

Blood loss<br />

Examples<br />

Traumatic<br />

Nontraumatic<br />

GI/GU bleeding, menorrhagia, aneurysmal rupture, hemoptysis<br />

Decreased production<br />

Hypochromic/microcytic<br />

Macrocytic<br />

Normocytic<br />

Iron deficiency, thalassemia, sideroblastic anemia or lead poisoning,<br />

chronic disease (cancer, renal, inflammatory diseases)<br />

B 1<br />

/folate deficiency, liver disease, hypothyroidism<br />

Primary bone marrow involvement: aplastic anemia, myelofibrosis<br />

Underlying disease: hypoendocrine state (thyroid, adrenal, pituitary),<br />

uremia, liver disease, chronic inflammation<br />

Increased destruction (hemolytic anemias)<br />

Intrinsic<br />

Extrinsic<br />

Enzyme defects: pyruvate kinase deficiency, G6PD deficiency<br />

Membrane abnormality: spherocytosis/el I iptostomatocytosis,<br />

paroxysmal nocturnal hemoglobinuria, spur cell anemia<br />

Hemoglobin abnormality: hemoglobinopathies, thalassemias, unstable<br />

hemoglobin, hemoglobin M<br />

Immunologic: allo/autoantibodies<br />

Mechanical: microangiopathic hemolytic anemia, prosthetic heart<br />

valves<br />

Environmental<br />

Drugs/toxins<br />

Infections<br />

Thermal<br />

Abnormal sequestration: hypersplenism<br />

729


HEMATOLOGIC DISORDERS<br />

All cell lines affected<br />

or reticulocytes


HEMATOLOGIC DISORDERS<br />

D. Nonhereditary anemia: hemolytic anemia<br />

1. Acquired anemias<br />

a. Precipitating factor is extrinsic to the body.<br />

b. Result of exposure to some immunologic, toxic/infectious, or mechanical problem<br />

2. Autoimmune hemolytic anemia<br />

a. The direct Coombs' antiglobulin test is used to demonstrate lgG or complement (C3) attached to the<br />

surface of the RBCs: positive only in immune-mediated hemolytic anemias.<br />

b. Indirect Coombs' test looks for free-circulating antibodies and is used in pretransfusion screening.<br />

c. Warm and cold antibody hemolytic anemia exist.<br />

d. Drug-induced hemolytic anemias<br />

(1) Many drugs have been linked with induction of hemolytic anemias via various mechanisms.<br />

(2) The prototype drugs for each reaction are alpha-methyldopa, procainamide, penicillin, and quinidine.<br />

(3) Hemolysis generally resolves when the drug is stopped.<br />

3. Fragmentation hemolysis: MAHA<br />

a. Thrombotic thrombocytopenic purpura/hemolytic uremic syndrome<br />

b. Pregnancy-associated hemolysis<br />

(1) Preeclampsia, eclampsia, or placental abruption can result in MAHA.<br />

(2) Preeclampsia (even with minimal signs and symptoms) can be associated with the HELLP syndrome,<br />

which untreated can lead to liver failure, DIC, or CHF .<br />

.!::!.emolysis<br />

_!;levated liver enzymes<br />

low flatelets<br />

4. Toxic hemolysis<br />

a. Infectious agents<br />

(1) Malaria, babesiosis, and bartonel losis can cause hemolysis via parasitization of the RBCs.<br />

(2) Hemophilus influenzae type B can alter RBC membranes, which leads to their destruction by the<br />

immune system.<br />

(3) C/ostridium perfringens releases enzymes that break down RBC membranes.<br />

(4) Mycoplasma and mononucleosis induce cold agglutinins.<br />

b. Other toxins: insect and snake bites (especially cobras) can induce hemolysis; American pit vipers usually<br />

cause coagulopathies.<br />

5. Mechanical destruction: heat, repetitive trauma to hands or feet and shearing by passage of blood through<br />

oxygenators or artificial heart valves can induce breakage of RBC membranes.<br />

E. Acquired non hemolytic anemias<br />

1. General<br />

a. Problems with production of RBCs because of absent components (iron deficiency) or marrow disease<br />

b. Characterized by a decreased reticulocyte count<br />

c. The MCV and iron studies can help to further classify and differentiate anemias.<br />

2. Aplastic anemia<br />

a. Disease of stem cells in the bone marrow that results in a pancytopenia<br />

b. Most cases are idiopathic, but up to 20% are associated with drug or chemical exposure.<br />

(1) Benzene<br />

(2) Phenylbutazone, gold, o-penicillamine<br />

(3) Anticonvulsants<br />

(4) Sulfonamides, chloramphenicol<br />

(5) Clozapine (antipsychotic)<br />

(6) Chemotherapeutic agents<br />

c. 10% are due to viral infections, especially hepatitis, Epstein-Barr virus, cytomegalovirus<br />

d. Management<br />

(1) Best handled by a hematologist; is often supportive but can include bone marrow transplant,<br />

immunosuppression, and transfusions<br />

(2) Fever in a neutropenic patient (absolute neutrophil count


PSYCHOBEHAVIORAL DISORDERS<br />

3. Iron-deficiency anemia<br />

a. Results from blood loss (often GI or menstrual), increased iron requirements (pregnancy, lactation), or<br />

malabsorption of iron<br />

b. Typically a microcytic anemia; a normal MCV may be seen early on.<br />

c. Iron studies rarely help the emergency physician but are often requested by consultants.<br />

(1) Serum ferritin<br />

(a)


HEMATOLOGIC DISORDERS<br />

d. Treatment: expectant but the key is to avoid drugs that may exacerbate the condition, eg, sulfa,<br />

antimalarials, phenazopyridine, nitrofurantoin<br />

5. Hereditary spherocytosis<br />

a. The cytoskeleton of the RBCs is abnormal, causing them to have a spherical shape.<br />

b. The RBCs become trapped in the spleen and are destroyed.<br />

c. Most patients have mild, chronic anemia.<br />

d. The cure is splenectomy.<br />

VI. BLOOD TRANSFUSIONS<br />

A. Transfusion of specific blood products<br />

1 . Packed RBCs<br />

a. Indicated for red cell repletion (not volume repletion)<br />

b. Transfusion of packed RBCs coupled with crystalloid fluid resuscitation has become the standard of care for<br />

the treatment of the acutely hemorrhaging patient.<br />

c. Type O negative (universal donor) should be given for exsanguinating hemorrhage if type-specific blood is<br />

not immediately available, ie, if you ordered it, and you are losing the patient, don't wait for it; as soon as<br />

type-specific blood is ready, you can switch infusions if need be.<br />

d. One unit of packed cells is 250 ml volume; standard practice is to administer at least two units in adults,<br />

15 ml/kg in children.<br />

e. Packed RBCs can be leukocyte-reduced, washed, irradiated, or frozen for specific needs. Consider<br />

leukocyte-reduced packed cells for:<br />

(1) Protecting immunocompromised patients<br />

(2) Preventing sensitization<br />

(3) Preventing nonhemolytic febrile reactions<br />

2. Fresh frozen plasma (FFP)<br />

a. Contains all the coagulation factors except platelets<br />

b. Indications<br />

(1) Correction of clinically significant depletion of clotting factors, eg, patient on warfarin therapy who<br />

is actively bleeding or requires emergent surgery<br />

(2) Treatment of an acute undiagnosed bleeding disorder<br />

c. Requires ABO matching and carries risk of infectious disease transmission<br />

d. Typical dosage is 4 units (250 ml/unit) for adults, 15 ml/kg for children.<br />

3. Platelets<br />

a. Indicated for spontaneous bleeding resulting from thrombocytopenia or inadequate platelet function<br />

b. A platelet pack of 6 units (typical adult transfusion) raises the platelet count ~50,000/mm 3 •<br />

Ordering Platelets Made Simple<br />

• Platelets are ordered in packs, not individual units.<br />

• Some hospitals use 4 packs and some use 6 packs. This equals either 4 or 6 individual units of platelets,<br />

usually from random donors.<br />

• The 4 or 6 pack raises the platelet count by about 50,000.<br />

• Platelets can also be ordered as an apheresis unit. This is made from one single donor. A lot of blood is<br />

taken from this one donor, and a certain amount of platelets are removed from the blood; the rest of the<br />

blood and plasma is returned to the donor.<br />

• This should raise the platelet count by about 50,000 and expose the patient to fewer antigens and fewer<br />

possible transmissible infections.<br />

Courtesy of Colin Kaide, MO<br />

733


HEMATOLOGIC DISORDERS<br />

734<br />

c. ABO matching is preferred; platelets may also be leukocyte-reduced.<br />

d. Transfusion is associated with risk of infectious disease transmission.<br />

e. Contraindications: TTP and HIT<br />

4. Cryoprecipitate<br />

a. Contains large quantities of factor VIII:C, vWF, vWF:Ag, and fibrinogen (factor I)<br />

b. Effective in bleeding due to:<br />

(1) vWD: effective for all types of vWD but is not the treatment of first choice<br />

(2) Hemophilia A (not the therapy of first choice)<br />

(3) Hypofibrinogenemia (congenital or acquired)<br />

c. Does not require ABO matching<br />

d. Use is associated with risk of transmission of both hepatitis and AIDS.<br />

5. Factor concentrates/plasma coagulation factors (contain no fibrinogen) are used in management of coagulation<br />

factor deficiencies.<br />

a. Contain high concentrations of the deficient factor and therefore are particularly useful in patients with<br />

hemophilia<br />

(1) Hemophilia A_,. factor VIII concentrate<br />

(2) Hemophilia B _,. factor IX concentrate<br />

(3) vWD _,. factor VIII concentrate containing large amounts of vWF<br />

b. Almost completely free of the risk of transmission of both the hepatitis and AIDS viruses<br />

6. Albumin and plasma protein fraction are no longer used because of the lack of proven efficacy of volume<br />

resuscitation; crystalloid administration is preferable.<br />

B. Transfusion methods<br />

1. Blood products should be infused with warmed normal saline (hypotonic saline solutions; solutions containing<br />

calcium or glucose should not be used). Whole blood and packed RBCs should be typed and crossmatched. In<br />

certain situations, however, untyped or non-crossmatched blood must be given:<br />

a. When type-specific blood is not available in a patient with exsanguinating hemorrhage, type 0-negative<br />

blood should be given.<br />

b. Type-specific blood is indicated for:<br />

(1) Progressive hypovolemic shock in spite of aggressive shock management (lactated Ringer's or normal<br />

saline 2 Lover 10-15 minutes)<br />

(2) Profound hypovolemic shock unresponsive to aggressive shock management in the first 10-15 minutes<br />

of therapy<br />

2. Autotransfusion<br />

a. This is a patient-specific source of blood for the patient in urgent need who has a traumatic hemothorax.<br />

(Blood collected from the chest does not have normal levels of fibrinogen nor does it have normally<br />

functioning platelets.)<br />

b. Technique: blood is drained through a chest tube, screenfiltered to remove clots and debris, then reinfused<br />

through a micropore filter.<br />

C. Complications of transfusions<br />

1. Complications associated with massive transfusions: defined as the transfusion of a volume of blood equivalent<br />

to the patient's entire blood volume within a 24-hour period or the transfusion of the equivalent of one-half of<br />

the patient's blood volume at one time.<br />

a. Coagulopathies<br />

(1) May result from dilution of clotting factors, platelet destruction, or DIC<br />

(2) Administration of FFP should be reserved for the massively transfused and bleeding patient who is in<br />

shock. Routine prophylactic use of FFP is unwarranted.<br />

(3) Platelets should be administered to patients who receive a massive transfusion in


HEMATOLOGIC DISORDERS<br />

c. Microembolization<br />

(1) Occurs when microaggregates form in refrigerated blood from the degeneration products of platelets,<br />

leukocytes, and fibrin<br />

(2) Embolization of these aggregates is thought to be associated with development of ARDS.<br />

(3) Use of micropore filtration will decrease the incidence of this complication.<br />

d. Hypocalcemia (citrate toxicity)<br />

(1) Decreased Ca++ can occur, because citrate chelates ionized calcium until all the citrate is metabolized.<br />

(2) Routine prophylactic calcium supplementation, however, is not recommended.<br />

(3) Calcium supplementation should be reserved for those patients who develop evidence of myocardial<br />

impairment.<br />

2. Complications associated with any transfusion<br />

a. Immediate reactions<br />

(1) Hemolytic reaction (most worrisome): an antibody-mediated reaction characterized by the<br />

development of fever, chills, burning at the infusion site, joint and low back pain, chest tightness,<br />

bleeding, hematuria, and shock<br />

(2) Febrile reaction (most common and least worrisome): characterized by fever, chills, and malaise, but<br />

patients may occasionally go on to develop blood pressure and respiratory distress<br />

(3) Allergic reaction (rare): characterized by development of urticaria skin flushing, laryngeal edema,<br />

and/or shock<br />

(4) Altered hemoglobin function: storage of blood --;, t 2,3 diphosphoglycerate levels - increased<br />

affinity of hemoglobin for oxygen --;, decreased delivery of oxygen to tissues<br />

(5) Hyperkalemia and acidosis: may occur in association with massive transfusions<br />

b. Delayed reactions<br />

(1) Infections: many different blood-borne infections may be transmitted by transfusions, including<br />

hepatitis, AIDS, human T-cell lymphotropic viruses, cytomegalovirus, Epstein-Barr virus, and malaria.<br />

Hepatitis C (formerly non-A, non-B) is currently the most common infection acquired from blood<br />

transfusions.<br />

(2) Extravascular hemolysis: most commonly manifests as an unexplained decrease in the hemoglobin<br />

level 7-1 0 days after transfusion<br />

(3) Graft versus host disease<br />

VII. REVERSAL OF DRUG-INDUCED ANTICOAGULATION<br />

A. Reversal of warfarin-induced coagulopathy<br />

1. General<br />

a. Increased IN Rs


HEMATOLOGIC DISORDERS<br />

Contact Activation Pathway<br />

(Intrinsic)<br />

•<br />

.<br />

.<br />

Tissue Factor Pathway<br />

(Extrinsic)<br />

COMMON PATHWAY<br />

2 Thrombin<br />

Fibrin<br />

Ogen (I)<br />

Warfarin Effects (2, 7, 9, 10)<br />

Courtesy of Colin Kaide, MD<br />

2. Reversal agents<br />

a. Warfarin-induced anticoagulation is reversed by adding back into the system the missing factors (II, VII,<br />

IX, and X) and by providing additional vitamin K to allow the liver to produce new factors.<br />

b. Vitamin K is given orally or IV. SC or IM injections are not recommended, because absorption is erratic.<br />

The risk of serious or life-threatening reactions from IV vitamin K is very low. INR correction begins in 2<br />

hours and is usually completed within 24 hours (see Table 46 for dosing recommendations).<br />

c. HP is given to replace the missing factors II, VII, IX, and X. The minimum dose should be 4 units. With<br />

FFP, the lowest attainable INR is 1.5. This is because the IR of HP is 1.5.<br />

d. Prothrombin complex concentrates (PCC) are now recommended as a first-line reversal agent for<br />

warfarin-induced coagulopathy. PCCs have been used in Europe and other parts of the world for years<br />

as reversal agents. PCCs are highly purified concentrates of the vitamin K-dependent clotting factors<br />

II (prothrombin), VII, IX, and X. They are pooled from plasma and although purified, do carry a very<br />

small risk of viral transmission. Product formulations vary in concentrations of each factor. They can be<br />

3-factor preparations with minimal VII activity or 4-factor with good VII levels. In the United States,<br />

two PCC products are available: Profilnine SD® and Bebulin VH®. They both have very low factor VII<br />

activity and are 3-factor preparations. A new 4-factor preparation is now also available in the United<br />

States (Kcentra®). It contains protein C and S to limit the risk of inducing inappropriate clotting. It is FDA<br />

approved for use as a reversal agent in warfarin-associated bleeding.<br />

(1) All factors are referenced against 100 U of factor IX activity.<br />

(2) All preparations are dosed based on the amount of factor IX.<br />

(3) There are reports of thrombosis with PCC preparations; however, the risk appears low.<br />

(4) Onset is very fast (15 minutes), and IN Rs can return to normal in 10 minutes.<br />

e. Factor eight inhibitor bypassing activity (FEIBA)<br />

(1) FEIBA is used in patients with hemophilia who develop antibodies to factor VIII after receiving factor<br />

VIII replacement for a prolonged time.<br />

(2) It is an activated form of PCC. It contains mostly activated factor VII and mostly nonactivated factors<br />

II, IX, and X.<br />

(3) It has been shown to be an effective reversal agent for warfarin-induced anticoagulation. It may have<br />

a thromboembolic risk greater than that of PCC and is not routinely recommended as a reversal<br />

agent for warfarin.<br />

(4) There is no FDA-approved indication for FEIBA as a warfarin reversal agent.<br />

736


HEMATOLOGIC DISORDERS<br />

f. Activated factor VI I<br />

(1) A very large thrombin burst is theoretically possible to create by directly activating factor X using<br />

recombinant factor VII (rVlla).<br />

(2) The INR can be rapidly normalized, although redosing is needed every few hours.<br />

(3) Despite normalizing the INR, studies have called into question the effects of rVlla on actual clinical<br />

reversal of bleeding, and it is no longer recommended for warfarin bleeding.<br />

(4) There is no FDA-approved indication of rVlla as a warfarin reversal agent.<br />

Table 46: Recommendations for Managing Increased IN Rs or Bleeding in Patients Receiving Vitamin K<br />

Antagonists<br />

Condition<br />

INR above therapeutic range<br />

but


HEMATOLOGIC DISORDERS<br />

c. Protamine may have some effect on LMWH. Only 60%-75% of the anti-Xa activity of LMWH is<br />

neutralized by protamine. Effectiveness depends on which LMWH is used.<br />

d. There is a real concern when using protamine with LMWH-protamine when given by itself has<br />

anticoagulant effects. If there is reversal of the non-Xa activity and only partial (but not enough) reversal of<br />

the Xa activity, the net result will point to anticoagulation.<br />

C. Reversal of newer anticoagulants<br />

1. Data supporting the reversal of the newer anticoagulants rivaroxaban, apixaban, and dabigatran is largely<br />

limited to ex vivo and in vitro studies. There are very few randomized, controlled trials supporting the reversal<br />

of these agents.<br />

2. Rivaroxaban, apixaban, and edoxaban are direct factor Xa inhibitors, ie, they bind directly to factor Xa without<br />

usingATIII.<br />

3. Dabigatran is a direct factor II inhibitor, ie, it binds directly to factor II without using ATIII.<br />

4. These agents attack at the most critical part of the clotting cascade, making reversal difficult.<br />

5. Information on reversal with the following agents is not FDA approved, but it does appear on many written<br />

protocols. It represents the best information available at the time of this publication, until accepted protocols<br />

become standard. The exception to this is the newly released antibody to dabigatran, idarucizumab, which was<br />

FDA approved in fall 2015.<br />

6. Strategies that have been tried involve overwhelming the factor inhibition by introducing a large amount of<br />

replacement factors into the system. PCC and aPCC (FEIBA) have been tried for both agents. FEIBA thus far has<br />

shown promise in reversal of both agents. These studies have been conducted in volunteers and in animals and<br />

are not definitive. 4 factor PCCs are currently the preferred reversal agent for the Xa inhibitors. Activated PCC<br />

(FEIBA) is recommended as the reversal agent of choice for dabigatran, if idarucizumab is not available.<br />

7. In theory, rVlla may also work to reverse the effect of factor X inhibition by direct inhibitors. The possible<br />

reversal of rivaroxaban by rVlla has been suggested because of its ability to reverse the indirect Xa inhibitor<br />

fondaparinux. However, there is no convincing human clinical data as of yet to support the use of rVlla.<br />

D. Reversal of thrombolysis<br />

1. General<br />

a. Thrombolytics (rTPA and others) work by catalyzing the conversion of plasminogen to plasmin. Plasm in<br />

cleaves cross-linked fibrin into d-dimers and fibrin degradation products.<br />

b. Reversal is not an accurate term in that the best that can be done is to mitigate the effects as best as<br />

possible.<br />

2. Agents to "reverse" thrombolytic-induced bleeding<br />

a. E-Aminocaproic acid<br />

(1) Blocks fibrinolysis by reversibly blocking plasminogen, which then cannot be cleaved to plasm in;<br />

plasm in cleaves crosslinked fibrin and lyses clots.<br />

(2) Enhances hemostasis when fibrinolysis contributes to bleeding; used for bleeding in patients with<br />

hemophilia after dental procedures<br />

(3) There are no data on the use for rescue of rTPA (or other thrombolytics) for bleeding in the brain or<br />

other sites when rTPA is used for stroke, Ml, or pulmonary embolism.<br />

(4) Not FDA approved for this use<br />

b. Cryoprecipitate<br />

(1) Contains a significant amount of fibrinogen (see above for details)<br />

(2) Gives back the fibrinogen that is degraded by the action of tPA<br />

738


HEMATOLOGIC DISORDERS<br />

Table 47: Anticoagulant Reversal Agents<br />

Anticoagulant<br />

Warfarin<br />

Heparin<br />

LMWH<br />

Synthetic<br />

pentasacchari de<br />

(fondapari n ux)<br />

Direct thrombin<br />

inhibitors<br />

Direct Xa inhibitors<br />

Platelet inhibitors<br />

(eg, clopidogrel)<br />

Thrombolytics<br />

Mechanism of Anticoagulation<br />

Interferes with the production of II, VII, IX, X<br />

Potentiates the action of antithrombin,<br />

inactivating Xa and thrombin<br />

Acts via antithrombin to inactivate Xa and<br />

thrombin. Activity varies but mostly anti-Xa<br />

activity<br />

Indirectly inhibits Xa via antithrombin<br />

Inhibit thrombin without using antithrombin<br />

Inhibit Xa without using antithrombin<br />

Block various mechanisms by which<br />

platelets aggregate and degranulate<br />

Catalyze plasminogen to plasmin, causing<br />

clot lysis<br />

Reversal Agents<br />

Vitamin K, FFP, PCC, aPCC<br />

Protamine<br />

Protamine (partial), rVlla<br />

rVlla, PCC, aPCC may be<br />

effective but limited data<br />

aPCC may be effective but<br />

limited data; idarucizumab<br />

4-factor PCC<br />

Platelets(?), DDAVP (?), rVlla<br />

(?)<br />

Aminocaproic acid,<br />

cryoprecipitate<br />

739


HEMATOLOGIC DISORDERS: PRACTICE CLINICAL SCENARIOS<br />

HEMATOLOGIC DISORDERS: PRACTICE CLINICAL SCENARIOS<br />

Answers immediately follow the practice clinical scenarios.<br />

Scenario A<br />

Presentation: A patient presents to the emergency department with altered mental status. He is confused<br />

and lethargic.<br />

Laboratory studies: Platelet count is 75,000/mm 3 , and hemoglobin is 9.5 mg/dL. There has been no<br />

heparin exposure. The laboratory reports schistocytes on the peripheral blood smear.<br />

What is the diagnosis?<br />

Scenario B<br />

Presentation: A patient with sickle cell disease presents with complaints of chest pain. He complains of<br />

discomfort with breathing, shortness of breath, and fever.<br />

What is the diagnosis?<br />

Scenario C<br />

Presentation: A patient presents 3 days after being discharged from the hospital after a left knee<br />

replacement. The patient is on a LMWH 40 mg/day (prophylactic dosage).<br />

Physical examination: The patient has what appears to be bleeding at the sites of the LMWH injections.<br />

She also has a painful, pulseless lower extremity on the right side (the opposite side from the knee<br />

replacement).<br />

What is the diagnosis?<br />

Scenario D<br />

Presentation: A patient with hemophilia A comes in with swelling and pain in his knee after a minor injury.<br />

He took some of his factor at home but still complains of increased swelling and worsening pain.<br />

What is the diagnosis?<br />

Scenario E<br />

Presentation: A patient is on warfarin and was started on TMP-SMX. She presents with a massive GI bleed.<br />

Laboratory studies: The patient's INR is 9. Her INR before starting an antibiotic was 2.4. Her hemoglobin is<br />

9 mg/dL.<br />

What is the diagnosis?<br />

740


HEMATOLOGIC DISORDERS: PRACTICE CLINICAL SCENARIOS<br />

ANSWERS TO PRACTICE CLINICAL SCENARIOS<br />

Scenario A<br />

Diagnosis: thrombotic thrombocytopenic purpura (TTP)<br />

Diagnostic evaluation: Never ignore schistocytes on a peripheral smear; they are always bad! If> 1 % is<br />

reported, consider a microangiopathic hemolytic anemia. This patient meets the three most important<br />

criteria for TTP: thrombocytopenia, neurologic symptoms (due to microthrombi), and schistocytes on the<br />

peripheral blood smear (due to breakage of cells flowing through platelet plug debris). Fever and renal<br />

abnormalities are less common. Two-thirds of patients with TTP have some neurologic symptoms due to<br />

microthrombi in the CNS.<br />

Management: The main treatment is plasma exchange transfusion, which is done by a hematologist.<br />

Additional management includes steroids to decrease production of the pathologic antibody (against<br />

ADAMTS13) and FFP to replete ADAMTS13.<br />

Scenario B<br />

Diagnosis: sickle cell with chest pain and shortness of breath<br />

Diagnostic evaluation: The patient's oxygen saturation is 90%. Chest radiograph shows a patchy lower lobe<br />

infiltrate on the right side. Although this could be nothing more than a community acquired pneumonia,<br />

you must be concerned about acute chest syndrome, which is seen in SS, SC, sickle-thalassemia. The<br />

criteria for diagnosis are new pulmonary infiltrate involving at least one complete lung segment consistent<br />

with alveolar consolidation but excluding atelectasis, plus at least one or more clinical findings of cough,<br />

wheezing, tachypnea, temperature >38.5°C (101.3°F), dyspnea/hypoxia, or chest pain.<br />

Management: Testing should include a radiograph to look for infiltrate. Treatment is aimed at treating<br />

infection and maximizing symptom control and respiratory function. This can include bronchodilators,<br />

transfusions, pain medications, and respiratory support as needed.<br />

Scenario C<br />

Diagnosis: heparin-induced thrombocytopen ia<br />

Diagnostic evaluation: Although the patient has a platelet count that is seemingly normal (130,000/mm 3 ),<br />

postoperative platelet counts should be increased. Her platelet count on the last known laboratory study is<br />

600,000/mm3, which would be normal in the postoperative state. She has had a drop of >50%. Remember<br />

the T's: The patient has thrombocytopenia (relative), thrombosis, and no other explanation. The timing is<br />

within the appropriate range of 5-1 0 days.<br />

Management: Stop the LMWH (or regular heparin). An alternative form of anticoagulation is needed.<br />

Because the patient will be admitted, start a drug like argatroban. If the patient had been on warfarin,<br />

vitamin K therapy would be needed. There is no need for fresh frozen plasma.<br />

Scenario D<br />

Diagnosis: hemophilia<br />

Key facts:<br />

• Bleeding history in a hemophiliac<br />

• Bleeding after mild trauma or dental extractions<br />

• Spontaneous hematuria<br />

• Hemarthrosis and muscle hematomas (90% of all bleeding)<br />

• lntracranial hemorrhages (a major cause of death in patients with hemophilia A)<br />

741


HEMATOLOGIC DISORDERS: PRACTICE CLINICAL SCENARIOS<br />

Management: Treatment includes replacement of the missing factor. Factor VIII concentrate is the treatment<br />

of choice. Check the patient's record or ask the patient how much factor he would normally get in the<br />

situation. Patients with hemophilia very likely know a lot more about it than you do, because they have<br />

lived with it their entire life, whereas you may not see it very often. Dosing is determined by whether the<br />

bleeding is mild, moderate, or severe.<br />

• In mild bleeding (hematuria, early hemarthrosis, deep laceration), the goal is to increase factor levels by<br />

25% _,. 12.5 units/kg.<br />

• In moderate bleeding (oral lacerations, dental extraction or minor surgery, late hemarthrosis), the goal is<br />

to increase factor levels up to 50%. _,. 25 units/kg.<br />

• In severe bleeding (CNS, GI, intra-abdominal or retroperitoneal bleeding; major trauma or surgery;<br />

documented or potentially serious head injury), the goal is to increase factor levels up to 100% _,.<br />

50 units/kg.<br />

Scenario E<br />

Diagnosis: warfarin over-anticoagulation<br />

Alternative presentation: A patient on warfarin is sent over from his family doctor's office for an<br />

asymptomatic INR of 6.<br />

Diagnostic evaluation: Determine the need for reversal of anticoagulation. Severe bleeding with an<br />

increased INR almost always needs to be treated. Caution should be exercised in patients with a left<br />

ventricular assist device or mechanical artificial valve; these are not absolute contraindications to reversal,<br />

but consider discussing with the cardiovascular surgeon to establish a target final INR. In cases when the<br />

acute bleeding can be controlled (cauterized epistaxis, banded varices, etc), reversal of warfarin should be<br />

followed by cross-anticoagulation with heparin. Patients with increased IN Rs who are asymptomatic may<br />

be treated by holding a dose of warfarin ± smal I doses of vitamin K.<br />

Management: If needed, administer vitamin K orally or IV. Replace missing factors when indicated. Fresh<br />

frozen plasma or PCC can be used.<br />

742


ONCOLOGIC DISORDERS<br />

ONCOLOGIC DISORDERS<br />

Upper Airway Obstruction ........................................................................................................................................ 746<br />

Tamponade Secondary to Malignant Pericardia( Effusion .......................................................................................... 746<br />

Superior Vena Cava Obstruction ............................................................................................................................... 747<br />

Acute Spinal Cord Compression ................................................................................................................................ 748<br />

Hypercalcemia Secondary to Malignancy ................................................................................................................. 748<br />

Syndrome of Inappropriate ADH Secretion (SIADH) ................................................................................................ 749<br />

Acute Tumor Lysis Syndrome ..................................................................................................................................... 750<br />

Hyperviscosity Syndrome .......................................................................................................................................... 751<br />

Adrenal Insufficiency ................................................................................................................................................ 752<br />

Granulocytopenia, lmmunosuppression, and Overwhelming Infection in the Presence of Malignancy .................... 752<br />

Leptomeningeal Carcinomatosis ............................................................................................................................... 753<br />

743


ONCOLOGIC DISORDERS: SELF-ASSESSMENT QUESTIONS<br />

ONCOLOGIC DISORDERS: SELF-ASSESSMENT QUESTIONS<br />

1. All of the following statements regarding cardiac tamponade secondary to malignant pericardia I effusion are<br />

true except:<br />

(a)<br />

ECG findings of low QRS voltage and electrical alternans are consistent with this diagnosis.<br />

(b) Suggestive physical findings include hypotension with a narrow pulse pressure, jugular venous distention,<br />

and muffled heart tones.<br />

(c) Supportive measures include fluid restriction and diuretics.<br />

(d) The diagnosis may be confirmed by echocardiography or Swan-Ganz catheterization.<br />

2. All of the following statements regarding superior vena cava obstruction are accurate except:<br />

(a)<br />

Patients present with headache, swelling of the face and upper extremities, and shortness of breath.<br />

(b) Common causes of this syndrome are lung cancer and lymphoma.<br />

(c) Useful therapeutic measures include administration of oxygen, diuretics, and corticosteroids.<br />

(d) Initially, symptoms occur in the evening.<br />

3. The ECG finding most consistent with a diagnosis of hypercalcemia is:<br />

(a) Shortened QT interval<br />

(b) Prolonged PR interval<br />

(c)<br />

Prolonged QT interval<br />

(d) Hyperacute T waves<br />

4. Which of the fol lowing signs and symptoms is/are not consistent with the diagnosis of hypercalcemia?<br />

(a)<br />

Lethargy/decreased level of consciousness<br />

(b) Hypotension<br />

(c) Constipation<br />

(d) Anorexia/nausea/vomiting<br />

5. In a patient presenting with the triad of unexplained stupor or coma, anemia, and rouleaux formation on peripheral<br />

blood smear, the most likely diagnosis is:<br />

(a) Acute adrenal insufficiency<br />

(b) Hypercalcemia<br />

(c) Hyperviscosity syndrome<br />

(d) Superior vena cava syndrome<br />

6. Which of the fol lowing findings is inconsistent with the diagnosis of adrenal insufficiency?<br />

(a)<br />

Hyperkalemia<br />

(b) Hypotension<br />

(c) Hyponatremia<br />

(d) Hyperglycemia<br />

7. The most common presenting symptom in patients with leptomeningeal carcinomatosis is:<br />

(a)<br />

Headache<br />

(b) Incontinence<br />

(c)<br />

Back pain<br />

(d) Facial nerve palsy<br />

8. Common causes of the hyperviscosity syndrome include all of the following except:<br />

(a) Chronic myelocytic leukemia<br />

(b) Lymphomas<br />

(c) Multiple myeloma<br />

(d) Waldenstrom macroglobulinemia<br />

744


ONCOLOGIC DISORDERS: SELF-ASSESSMENT QUESTIONS<br />

9. The most appropriate initial therapy for malignancy-induced hypercalcemia consists of:<br />

(a) Administration of glucocorticoids<br />

(b) Administration of plicamycin (mithramycin)<br />

(c) Fluid hydration with IV normal saline followed by administration of furosemide<br />

(d) Administration of calcitonin<br />

10. Infection in the cancer patient is most commonly ___ in etiology.<br />

(a) Viral<br />

(b) Bacterial<br />

(c) Fungal<br />

(d) Protozoa!<br />

11. A reasonable antibiotic regimen for the cancer patient who presents with a history of fever and abdominal pain<br />

associated with peritonitis on physical examination is:<br />

(a) An antipseudomonal aminoglycoside plus an antipseudomonal penicillin<br />

(b) A fluoroquinolone and an antipseudomonal aminoglycoside plus an antipseudomonal third- or fourthgeneration<br />

cephalosporin<br />

(c) Metronidazole or clindamycin<br />

(d) A combination of A+ C or B + C<br />

ANSWERS<br />

I. c 4. b 7. a 10. b<br />

2. d 5. c 8. b 11. d<br />

3. a 6. d 9. c<br />

Use the pre-chapter multiple choice question worksheet (page xvi) to record and determine the percentage of correct<br />

answers for this chapter.<br />

745


ONCOLOGIC DISORDERS<br />

I. UPPER AIRWAY OBSTRUCTION (AT THE LEVEL OF THE<br />

MAINSTEM BRONCHI OR ABOVE)<br />

A. Signs and symptoms<br />

1. Voice change or hoarseness (indicates a slow process of local tumor compression or obstruction)<br />

2. Laryngeal stridor from a rapidly growing tumor<br />

3. Acute upper airway obstruction secondary to infection, tumor hemorrhage, or inspissated secretions;<br />

respiratory arrest is imminent.<br />

B. Etiology<br />

1. Carcinoma<br />

a. Laryngeal<br />

b. Thyroid<br />

c. Pulmonary metastases<br />

2. Lymphoma (especially Burkitt)<br />

C. Diagnostic evaluation<br />

1. Immediate assessment<br />

2. Palpation of mass<br />

3. Lateral soft-tissue radiograph of the neck<br />

4. Direct laryngoscopy<br />

5. Fiberoptic laryngoscopy<br />

6. CT if stable<br />

D. Management<br />

1. Establish an airway if there is airway compromise.<br />

a. Fiberoptic laryngoscope-assisted intubation can be very useful in securing the difficult airway.<br />

b. A surgical airway may be necessary.<br />

2. ENTconsult<br />

3. Treat the underlying neoplasm.<br />

II. TAMPONADE SECONDARY TO MALIGNANT PERICARDIAL<br />

EFFUSION<br />

A. Clinical presentation<br />

1. Related to the volume of fluid, its rate of accumulation, and the tempo of cardiac compressions<br />

2. Hypotension with narrow pulse pressure<br />

3. Shortness of breath and extreme anxiety and apprehension<br />

4. Jugular venous distention<br />

5. Pulsus paradoxus > 10 mm Hg (may be absent)<br />

6. Other signs that may by present<br />

B. Etiology<br />

a. Auscultation: diminished heart sounds ("muffled")<br />

b. ECG: low QRS voltage and electrical alternans<br />

c. Chest radiograph: enlargement of the cardiac silhouette in association with clear lung fields and a normal<br />

vascular pattern (no CHF)<br />

1 . Most common<br />

a. Carcinoma of the lung and breast<br />

b. Hodgkin and non-Hodgkin lymphoma<br />

c. Leukemia<br />

d. Malignant melanoma<br />

746


ONCOLOGIC DISORDERS<br />

2. Others<br />

a. Mesotheliomas and sarcomas of the pericardium and other metastatic tumors<br />

b. Postradiation pericarditis<br />

C. Differential diagnosis<br />

1. Massive pulmonary embolus<br />

2. Acute superior vena cava obstruction<br />

3. CHF<br />

4. Anxiety<br />

D. Diagnostic evaluation<br />

1. Echocardiography is the diagnostic study of choice; it is sensitive, specific, and noninvasive.<br />

2. Swan Ganz catheterization is performed if diagnostic confirmation is required; a positive finding is diastolic<br />

equalization of pressures (pulmonary artery= right ventricle= right atrial= intrapericardial)<br />

E. Management<br />

1. Supportive measures include IV fluids, inotropic support with dobutamine or isoproterenol, and<br />

supplemental oxygen<br />

2. Definitive therapeutic measures<br />

a. Pericardiocentesis (with ultrasound guidance if possible)<br />

b. Pericardia! stripping/window surgery<br />

c. Radiotherapy/ chemotherapy<br />

Ill. SUPERIOR VENA CAVA OBSTRUCTION<br />

A. Clinical presentation<br />

1. Reflect slow, progressive tumor development; initially, the symptoms occur in early morning.<br />

2. Most common complaints are edema and venous distention of the face and upper extremities plus shortness<br />

of breath.<br />

3. Headache or feeling of fullness/congestion in head<br />

4. Facial plethora and telangiectasia<br />

5. Papilledema (signifies a critical increase of intracranial pressure)<br />

6 Cough, difficulty swallowing<br />

B. Etiology<br />

1. Carcinoma of the lung (small-cell or squamous cell): bronchogenic carcinoma is the most common cause of<br />

superior vena cava obstruction (usually on the right side).<br />

2. Lymphoma<br />

C. Diagnostic evaluation<br />

1. Chest radiograph: wide mediastinum and/or mass<br />

2. Contrast-enhanced chest CT confirms the diagnosis.<br />

3. MRI is indicated if neck pain is present to exclude coexistent spinal cord compression (Rubin syndrome).<br />

4. Immediate consultation for tissue diagnosis, followed by radiation and/or chemotherapy<br />

5 Intravenous stents becoming more common<br />

D. Management<br />

1. Administer supplemental oxygen.<br />

2. Begin measures to decrease cerebral edema if symptoms are present.<br />

a. Elevate head to 30° midline position.<br />

b. Diuretics (eg, furosemide)<br />

c. Corticosteroids (eg, methylprednisolone)<br />

747


ONCOLOGIC DISORDERS<br />

IV. ACUTE SPINAL CORD COMPRESSION<br />

A. Clinical presentation<br />

1. Back pain (most commonly thoracic) is the initial symptom in 95 % of these patients.<br />

2. Pain occurs at the site of tumor metastases; it may be localized or radicular in nature and is usually worsened<br />

by percussion over the affected vertebral bodies.<br />

3. Other symptoms include lower extremity weakness, difficulty with ambulation, sensory deficits, paralysis,<br />

and urinary incontinence (acute urinary retention may also occur).<br />

4. Thoracic spine is most commonly affected, followed by lumbar spine. Can be life-threatening in the cervical<br />

spine because of potential respiratory depression.<br />

B. Etiology<br />

Multiple myeloma<br />

2. Lymphoma (Hodgkin or non-Hodgkin)<br />

3. Carcinoma<br />

a. Lung (most common cause)<br />

b. Breast<br />

c. Prostate<br />

C. Diagnostic evaluation<br />

1. Plain radiographs: identify the level of vertebral collapse<br />

2. MRI of spine (diagnostic modality of choice)<br />

3. Myelography<br />

a. Associated with significant morbidity resulting from lumbar puncture and dye insertion<br />

b. Should be done if MRI is unavailable or if the MR images were of poor quality<br />

4. CT of the spine, in conjunction with myelography, enhances detection of small areas of spinal destruction.<br />

D. Management<br />

1. Steroids (consider high-dose dexamethasone (100 mg); otherwise 4-6 mg every 6 hours) - t inflammation<br />

and edema<br />

2. Radiation (the definitive therapy) and surgery (decompression laminectomy) improve neurologic function.<br />

3. Early recognition is critical for preserving neurologic function.<br />

V. HYPERCALCEMIA SECONDARY TO MALIGNANCY<br />

(MOST COMMON LIFE-THREATENING METABOLIC DISORDER<br />

ASSOCIATED WITH CANCER)<br />

A. Clinical presentation<br />

1. Early presentation<br />

a. Anorexia, nausea, vomiting<br />

b. Fatigue, weakness<br />

c. Decreased level of consciousness<br />

d. Constipation<br />

e. Hypertension<br />

f. Back pain<br />

g. Polydipsia, polyuria<br />

2. Late presentation<br />

a. Oliguria<br />

b. Renal failure<br />

c. Stupor/coma<br />

d. lieus<br />

e. Heart blocks<br />

748


ONCOLOGIC DISORDERS<br />

3. Remember the symptoms as:<br />

Stones (kidney stones)<br />

Bones (osteitis fibrosa cystica: results in pain and pathologic fractures; also can see osteoporosis, osteomalacia,<br />

and arthritis)<br />

Groans (abdominal groans: GI symptoms of constipation, indigestion, nausea, and vomiting; hypercalcemia<br />

can lead to peptic ulcers and acute pancreatitis)<br />

Moans (psychic moans: lethargy, fatigue, depression, memory loss, psychosis, ataxia, delirium, and coma)<br />

B. Etiology<br />

1 . Bone destruction<br />

a. Carcinomas of lung, breast, and prostate<br />

b. Multiple myeloma<br />

2. Parathyroid hormone-like substance (squamous cell carcinoma of the lung)<br />

3. Osteoclast-activating factor (calcitriol overproduction)<br />

a. Non-Hodgkin lymphoma<br />

b. AdultT-cell lymphoma or leukemia<br />

4. New hormonal therapy in patients with breast cancer (that is not related to the underlying malignancy)<br />

C. Diagnostic evaluation<br />

1. Serum levels of the following:<br />

a. Calcium (t): check ionized Ca++ levels<br />

b. Phosphorus (normal or !)<br />

c. Alkaline phosphatase (normal or t)<br />

d. Chloride (


ONCOLOGIC DISORDERS<br />

4. Urinary sodium >40 mEq/L and urine osmolality > 100-150 mOsm/L associated with a urine specific gravity<br />

>1.002<br />

5. Normal renal, adrenal, and thyroid functions<br />

B. Clinical presentation<br />

1. Depends more on the rate of decrease in serum sodium than on the absolute serum sodium concentration<br />

2. Weakness, malaise<br />

3. Headache, dizziness<br />

4. Subtle changes in mental status<br />

5. Coma, seizures<br />

C. Etiology: malignancies (medications for malignancies, eg, cyclophosphamide, vincristine, can also cause<br />

SIADH)<br />

1. Lung: most common (small cell)<br />

2. Brain<br />

D. Diagnostic evaluation<br />

1. Blood<br />

a. EI ectro I ytes<br />

b. Serum osmolality<br />

c. Thyroid function studies<br />

d. Adrenal function studies (if myxedema or Addison disease is suspected)<br />

2. Urine<br />

a. Urinalysis<br />

b. Sodium<br />

c. Creatinine (1' in adrenal disease)<br />

d. Osmolality<br />

E. Management<br />

1 . Water restriction<br />

2. Furosemide 0.5-1 mg/kg with concomitant normal saline results in net free water clearance while<br />

maintaining euvolemia.<br />

3. 3% saline (for seizures/dysrhythmias) 100-250 ml over several hours to lower the risk of central<br />

demyelination: 0.5 mEq/L increase in sodium _,, 10-12 mEq in 12-24 hours<br />

4. Treat underlying tumor.<br />

5. Demeclocycline: longer-term benefit<br />

VII. ACUTE TUMOR LYSIS SYNDROME<br />

A. Etiology<br />

1. Most commonly occurs 1-5 days after chemotherapy for a hematologic malignancy such as leukemia and<br />

lymphoma (particularly Burkitt lymphoma) but can happen in any malignancy.<br />

2. Other factors that make acute tumor lysis syndrome more likely include small-cell lung cancer, a high tumor<br />

burden, and highly chemosensitive tumors.<br />

3. Results in the release of intracellular products into the circulation.<br />

4. Overwhelms the normal homeostatic mechanisms that control potassium, calcium, phosphorus, and uric acid<br />

5 Biochemical hallmarks include hyperuricemia (DNA breakdown), hyperkalemia (cytosol breakdown),<br />

hyperphosphatemia (protein breakdown), and hypocalcemia secondary to hyperphosphatemia.<br />

6. The integrity of renal function is a critical factor, because the kidney provides the primary mechanism for<br />

excretion of uric acid, potassium, and phosphate.<br />

a. Acute tumor lysis syndrome is more likely to be seen in patients with preexisting renal insufficiency, and<br />

the subsequent metabolic derangements are more likely to be severe than in those with normal renal<br />

function.<br />

b. Acute renal failure may also be precipitated by hyperuricemia and hyperphosphatemia (even in patients<br />

with preexisting normal renal function, especially in the presence of rapid tumor lysis).<br />

c. Hyperuricemia with resultant urate nephropathy is the most common metabolic cause of renal insufficiency.<br />

750


ONCOLOGIC DISORDERS<br />

B. Clinical presentation<br />

1. Paresthesia 1 altered mental status, seizures, nausea, vomiting, anorexia, flank pain, oliguria 1<br />

hematuria, edema<br />

2. Acute renal failure (t uric acid and/or t PO 4<br />

)<br />

3. Ventricular dysrhythmias, sudden death (t K+ or 1 Ca++)<br />

4. Neuromuscular instability (1 Ca++)<br />

C. Management<br />

1. Control preexisting hyperuricemia _,. hydration, allopurinol<br />

a. The primary means of reducing hyperuricemia is hydration and diuresis to maintain adequate urinary<br />

flow (at least 100 ml/hr).<br />

b. Alkalinization is no longer routinely recommended.<br />

2. Rasburicase is now widely used to treat tumor lysis syndrome. It is a recombinant version of a urate oxidase<br />

enzyme that is found in many mammals but not people. It catalyzes the conversion of uric acid to allantoin.<br />

Discuss with oncologist first, but an empiric dose should be considered in a patient with hyperuricemia and<br />

renal insufficiency.<br />

3. Hemodialysis should be considered a potentially life-saving measure.<br />

4. Frequent monitoring of electrolytes, calcium, and phosphorus.<br />

VIII. HYPERVISCOSITY SYNDROME<br />

A. The degree of viscosity reflects the flow-resisting characteristics of bodily fluids. This syndrome is<br />

characterized by marked increase in serum proteins, WBCs, or RBCs, which result in sludging and reduction<br />

in microcirculatory perfusion.<br />

B. Classic clinical scenario<br />

1. The patient presents with headache, fatigue, and somnolence or coma. Persistent bleeding from mucosa!<br />

surfaces is common (and the platelet count is normal). Focal signs of microthrombosis may be present and<br />

include deafness, blindness, and seizures.<br />

2. On funduscopic examination, 11 sausagelinked 11 retinal vessels, hemorrhages, and exudates may be seen.<br />

3. A clinical clue to this diagnosis is the patient with unexplained stupor or coma who is anemic and<br />

demonstrates "rouleaux cells" on the peripheral smear. The laboratory may tell you the chemical tests cannot<br />

be performed because of serum stasis in the analyzers, ie 1 the blood is too thick.<br />

C. Etiology<br />

1. Dysproteinemias<br />

a. Waldenstrom macroglobulinemia<br />

(1) Most common cause of hyperviscosity syndrome<br />

(2) Produces high-viscosity lgM<br />

b. Multiple myeloma (lgA and lgG myelomas): second most common cause<br />

2. Leukemias with blastic transformation, eg, chronic myelogenous leukemia<br />

3. Polycythemia vera, essential thrombocytosis<br />

D. Diagnostic evaluation<br />

1. Increased serum viscosity (>4-5 times the viscosity of water)<br />

2. Increased serum proteins on electrophoresis<br />

3. WBC count> 100,000/mm 3 in chronic myelogenous leukemia<br />

4. Urinalysis_,. Bence Jones protein in multiple myeloma<br />

E. Management<br />

1. Dysproteinemias<br />

a. Immediate rehydration followed by emergency plasmapheresis<br />

b. Phlebotomy of two units and replacement of the patient's RBCs with physiologic saline is appropriate for<br />

patients in coma due to hyperviscosity syndrome.<br />

c. Chemotherapy<br />

2. Leukemias with blastic transformation<br />

a. Immediate rehydration followed by leukapheresis<br />

b. Hydrea<br />

c. Chemotherapy<br />

751


ONCOLOGIC DISORDERS<br />

IX. ADRENAL INSUFFICIENCY<br />

A. Classic clinical scenario<br />

1. Sudden vasomotor collapse is classic.<br />

2. On physical examination, the patient is hypotensive with signs of dehydration.<br />

3. Laboratory findings are hypoglycemia, hyponatremia, hyperkalemia, and increased eosinophils.<br />

B. Etiology<br />

1. Carcinoma of the lung or breast<br />

2. Withdrawal of chronic steroid therapy<br />

3. Malignant melanoma<br />

4. Retroperitoneal malignancies<br />

C. Management (draw blood for a serum cortisol level first)<br />

1. Aggressive IV fluid therapy with D5 normal saline (1 Lover the first hour)<br />

2. Hydrocortisone 100 mg IV initially and every 6 hours; could use dexamethasone as an alternative if planning<br />

ACTH stimulation test.<br />

3. Correct precipitating factors (especially infection).<br />

X. GRANULOCYTOPENIA, IMMUNOSUPPRESSION, AND OVER­<br />

WHELMING INFECTION IN THE PRESENCE OF MALIGNANCY<br />

A. Significance<br />

1. Infection is the most common cause of death in the cancer patient and may be subtle in presentation.<br />

2. Fever is usually the initial clinical manifestation of the infectious process and may be the only sign present.<br />

3. The risk of infection increases as the number of granulocytes decreases and is particularly pronounced in the<br />

granulocytopenic patient (PMN count


ONCOLOGIC DISORDERS<br />

E. Diagnostic evaluation<br />

1. CBC with platelet count<br />

2. Urinalysis<br />

3. Chest radiograph<br />

4. Cultures from all appropriate sites<br />

5. Electrolytes<br />

6. Prothrombin time (INR)/partial thromboplastin time<br />

7. Lactate<br />

F. Management<br />

1. Emergency therapy with IV fluids and antibiotics is recommended.<br />

2. Because infection may be life-threatening, antibiotics should be started as soon as cultures are drawn.<br />

Possible regimens include:<br />

a. Monotherapy options: cefepime, ceftazidime, imipenem/cilastatin, meropenem, or piperacillin/<br />

tazobactam<br />

b. Dual therapy: monotherapy agent and gentamicin, tobramycin, or amikacin<br />

c. The above regimens should be modified as follows if specific infections are suspected.<br />

(1) Add vancomycin and ceftazidime or cefepime if a gram-positive infection is likely (a clinically<br />

obvious catheter related infection, severe mucositis, quinolone prophylaxis), if infection with<br />

methicillin-resistant S aureus (or S epidermidis) is suspected, or if the patient is hypotensive.<br />

(2) Add clindamycin or metronidazole if an anaerobic infection is probable (patients with abdominal or<br />

gynecologic complaints).<br />

d. Consider granulocyte macrophage colony-stimulating factor (CM-CSF) after oncology consult; generally<br />

will decrease length of chemotherapy-induced neutropenia by one day (may be important in critically ill<br />

patients).<br />

XI. LEPTOMENINGEAL CARCINOMATOSIS<br />

A. Etiology<br />

1. Solid tumors in adults (breast, lung, GI carcinomas/melanoma)<br />

2. Hematogenous malignancies in children<br />

3. Tumor cells reach the meninges by hematogenous spread or by direct extension from preexisting lesions.<br />

B. Clinical presentation<br />

1. Headache is the most common presenting complaint.<br />

2. Cranial nerve deficits are common.<br />

3. Spinal symptoms also occur (weakness, numbness, and pain that is frequently radicular in nature).<br />

C. Diagnostic evaluation<br />

1. Contrast CT or MRI<br />

2. Lumbar puncture: CSF t pressure, t protein, t glucose, positive cytology<br />

D. Management<br />

1. Emergency department treatment with steroids and osmotherapy (mannitol) is indicated only in the presence of<br />

impending herniation.<br />

2. Early diagnosis and therapy are critical to preserve neurologic function.<br />

753


ONCOLOGIC DISORDERS: PRACTICE CLINICAL SCENARIOS<br />

ONCOLOGIC DISORDERS: PRACTICE CLINICAL SCENARIOS<br />

Answers immediately follow the practice clinical scenarios.<br />

Scenario A<br />

Presentation: A 45-year-old man with colon cancer presents 10 days after starting chemotherapy, with<br />

the sole complaint of fever. He denies chest pain, cough, sputum, shortness of breath, upper respiratory<br />

symptoms, dysuria or urinary frequency, diarrhea, and vomiting. He has some nausea and decreased oral<br />

intake.<br />

Physical examination: Temperature is 102 .3°F (39°C), pulse is 110 beats per minute, respiration rate is 16<br />

breaths per minute, and saturation on room air is 98%. Physical examination is unremarkable except for<br />

dry mucous membranes.<br />

What is the diagnosis?<br />

Scenario B<br />

Presentation: A SO-year-old woman with breast cancer complains of being tired, weak, anorectic,<br />

nauseous, and constipated. Other than breast cancer, past medical history is negative, including for GI<br />

disorders or kidney stones.<br />

Physical examination: Temperature is 98.9°F (37.2°C), pulse is 96 beats per minute, respiration rate is<br />

18 breaths per minute, oxygen saturation on room air is 96%. Physical examination is unremarkable,<br />

including a nontender, nondistended abdomen with normal bowel sounds. Neurologic exam is nonfocal<br />

and within normal limits. Her family reports she is a little confused (slow to answer) but is oriented to<br />

person, place, and time.<br />

What is the diagnosis?<br />

Scenario C<br />

Presentation: A 65-year-old man with lymphoma presents with back pain that worsens when he lies flat.<br />

The pain is located in the mid upper back and is a radicular pain. He also complains of some difficulty<br />

with urination but no dysuria.<br />

Physical examination: Temperature is 98.3°F (36.8°C), pulse is 68 beats per minute, respiration rate is<br />

14 breaths per minute, and saturation on room air is 95%. Physical examination reveals tenderness on<br />

percussion over the thoracic spine in the area ofT9-T10. There is no swelling or skin abnormalities. He has<br />

decreased sensation on the left lower leg to light touch and some weakness of the left lower extremity.<br />

What is the diagnosis?<br />

Scenario D<br />

Presentation: EMS arrives with a SO-year-old man with a known history of lung cancer and reports that<br />

the patient complains of shortness of breath and seems very anxious. The patient says that he feels dizzy<br />

with standing. He has noticed that he has needed more oxygen at home over the past week. He denies<br />

associated chest pain or pressure.<br />

Physical examination: Temperature is 99.0°F, blood pressure is 90/69 mm Hg, and pulse is 110 beats per<br />

minute and regular. Respiratory rate is 18 breaths per minute, and oxygen saturation on 4 Lis 95%. On<br />

examination, the patient has a chronically ill appearance, seems anxious, and has diminished breath<br />

sounds at the bases bilaterally. Some venous distention is noted, and heart sounds are diminished.<br />

What is the diagnosis?<br />

754


ONCOLOGIC DISORDERS: PRACTICE CLINICAL SCENARIOS<br />

Scenario E<br />

Presentation: A 60-year-old woman with a history of lymphoma presents for progressive headache and<br />

shortness of breath. Headache onset has been gradual for 1 week. She reports that the shortness of breath<br />

has worsened over the past month. She has a history of right upper extremity edema following previous<br />

lymph node biopsy. The family reports they have noticed more swelling in her arm and face.<br />

Physical examination: Temperature is 98.0°F, blood pressure is 145/90 mm Hg, pulse is 100 beats<br />

per minute, respiratory rate is 15 breaths per minute, and oxygen saturation is 96%. Examination is<br />

remarkable for facial plethora, swelling of the eyelids, and telangiectasia. She is also noted to have a slight<br />

nonproductive cough. Her right upper extremity is nontender and edematous compared with the left.<br />

What is the diagnosis?<br />

Scenario F<br />

Presentation: A 30-year-old man with HIV and Burkitt lymphoma presents 5 days after chemotherapy for<br />

nausea, vomiting, and flank pain. He reports that he has been urinating less and that his urine has been<br />

dark despite drinking adequate water. His spouse reports that he has a history of mild renal insufficiency<br />

but has never required treatment beyond hydration.<br />

Physical examination: Temperature is 100.0°F, pulse is 105 beats per minute, respiratory rate is 14 breaths<br />

per minute, blood pressure is 110/70 mm Hg, and oxygen saturation is 97%. On physical examination, the<br />

patient is noted to be thin and in no distress. Mucous membranes are dry. He has mild tenderness in his<br />

lower abdomen and bilateral flank. Minimal edema is seen in both lower extremities. His urine sample is<br />

dark in color.<br />

What is the diagnosis?<br />

Scenario G<br />

Presentation: A 65-year-old woman presents via EMS for evaluation of witnessed generalized seizure and<br />

altered mental status. She has a history of chronic myeloid leukemia. No past history of seizure is noted.<br />

Laboratory studies from the nursing facility could not be performed because the "blood was too thick."<br />

Physical examination: Temperature is 99.0°F, pulse is 110 beats per minute, respiratory rate is 12 breaths<br />

per minute, blood pressure is 140/80 mmHg, and oxygen saturation is 98% on 2 L nasal cannula. On<br />

examination, the patient is confused and moving all extremities spontaneously. Oral examination shows<br />

evidence of bleeding from the tongue and mucosa! surface. Funduscopic examination reveals "sausagelinked"<br />

retinal vessels.<br />

What is the diagnosis?<br />

Scenario H<br />

Presentation: A 70-year-old man with a history of metastatic colon cancer presents for generalized<br />

weakness, vision changes, and headache. Symptoms have gradually progressed over the past month. He is<br />

scheduled to see his oncologist, and laboratory studies obtained yesterday were within normal limits. No<br />

trauma or injury is reported.<br />

Physical examination: Temperature is 98.9°F, pulse is 90 beats per minute, respiratory rate is 14 breaths per<br />

minute, blood pressure is 120/80 mmHg, and oxygen saturation is 97% on room air. On examination, the<br />

patient appears well and is in no acute distress. Neurologic examination is positive for double vision, mild<br />

lack of coordination, and equal generalized weakness of the extremities. The remainder of the examination<br />

is unremarkable.<br />

What is the diagnosis?<br />

755


ONCOLOGIC DISORDERS: PRACTICE CLINICAL SCENARIOS<br />

ANSWERS TO PRACTICE CLINICAL SCENARIOS<br />

Scenario A<br />

Diagnosis: fever and neutropenia<br />

Diagnostic evaluation: Chest radiograph and urinalysis are within normal limits. The WBC count is<br />

500/mm 3 with 10% neutrophils. Other laboratory studies include a metabolic panel (BUN, creatinine,<br />

electrolytes, etc), blood cultures, urine culture, and chest radiograph. The patient is examined for other<br />

sources of fever/abscess, including skin (eg, cellulitis), ears, nose, throat, rectal area, etc.<br />

Management: Normal saline is administered IV for mild dehydration. Because the patient has recently<br />

been on ciprofloxacin (a quinolone), imipenem and vancomycin are started for broad-spectrum antibiotic<br />

coverage, including for methicillin-resistant Staphylococcus aureus (MRSA). He is admitted to oncology.<br />

Scenario B<br />

Diagnosis: hypercalcemia<br />

Diagnostic evaluation: A CBC and urinalysis are within normal limits. The calcium concentration is<br />

14 mgldL. The ECG shows a short QT interval but no dysrhythmias. An abdominal radiograph shows a<br />

stone in the renal parenchyma.<br />

Management: The patient is placed on a cardiac monitor and 1 L of normal saline is administered IV<br />

immediately. After a second liter of normal saline is administered, she is given 40 mg furosemide and<br />

60 mg pamidronate (both IV) over 3-4 hours while waiting for a bed on the oncology floor.<br />

Scenario C<br />

Diagnosis: acute spinal cord compression<br />

Diagnostic evaluation: Laboratory studies, including a CBC, metabolic panel, and urinalysis, are<br />

unremarkable.<br />

Management: A consult with radiation oncology is obtained immediately. The patient is given<br />

dexamethasone 100 mg IV, and the diagnosis is confirmed by MRI. His oncologist is notified, and the<br />

patient is admitted. A consult with the spine surgeon is obtained.<br />

Scenario D<br />

Diagnosis: tamponade secondary to malignant pericardia! effusion<br />

Diagnostic evaluation: The WBC count is 1,400/mm 3 , and hemoglobin is normal. Mild renal insufficiency<br />

is noted on the serum chemistry panel. The ECG shows sinus tachycardia with low voltage QRS and<br />

electrical alternans. Chest radiograph shows a left lung mass, an enlarged heart, and bilateral pleural<br />

effusion. Bedside ultrasonography shows a large pericardia! effusion.<br />

Management: IV fluids are administered and inotropic support is provided. A consult with cardiology or<br />

cardiothoracic surgery is obtained for ultrasound-guided pericardiocentesis or pericardia! window.<br />

756


ONCOLOGIC DISORDERS: PRACTICE CLINICAL SCENARIOS<br />

Scenario E<br />

Diagnosis: superior vena cava obstruction<br />

Diagnostic evaluation: Laboratory studies show mild renal insufficiency but are otherwise normal. Chest<br />

radiograph shows a mass in the right upper lung and anterior mediastinum. CT scan of the chest with<br />

contrast shows a large obstructing mass compressing the superior vena cava.<br />

Management: Measures to control symptoms are instituted, and the head is elevated to 30°. Administration<br />

of diuretics and steroids is considered. A consult is obtained with radiation oncology and interventional<br />

radiology for possible radiation or intravenous stent placement.<br />

Scenario F<br />

Diagnosis: tumor lysis syndrome<br />

Diagnostic evaluation: The CBC shows mild anemia and an increased WBC count of 30,000/mm 3 • Serum<br />

creatinine is 5.0 mg/dl, and potassium is 5.4 mEq/L. The urine is positive for protein and is concentrated<br />

without evidence of infection. Uric acid is increased.<br />

Management: The patient is monitored for further hyperkalemia, and an ECG is obtained. IV fluids are<br />

administered for hydration with strict measurement of intake and output. Urinary flow should be at<br />

least 100 ml/hour, and diuresis may be necessary. Allopurinol or rasburicase can be considered after<br />

consultation with oncology and nephrology.<br />

ScenarioG<br />

Diagnosis: hyperviscosity syndrome<br />

Diagnostic evaluation: Emergent noncontrast CT scan of the brain shows no obvious infarct or bleed.<br />

Laboratory studies show a WBC count of 110,000/mm3, and pathology has been consulted to review a<br />

peripheral smear. Chest radiograph and ECG show no acute process.<br />

Management: Symptoms should be controlled and monitored. Immediate hydration, line placement,<br />

and consultation for emergent plasmapheresis. Hydroxyurea and chemotherapy can be considered after<br />

plasmapheresis has been arranged.<br />

Scenario H<br />

Diagnosis: leptomeningeal carcinomatosis<br />

Diagnostic evaluation: Laboratory studies are repeated and within normal limits. CT scan of the brain<br />

shows no evidence of mass or herniation. MRI of the brain and spine shows a cluster of cells concerning<br />

for cancerous lesion.<br />

Management: A consult is obtained with oncology for possible radiation therapy and to discuss the need<br />

for lumbar puncture for cytology.<br />

757


758<br />

NOTES


SYSTEMIC INFECTIOUS DISORDERS<br />

SYSTEMIC INFECTIOUS DISORDERS<br />

Sepsis and Septic Shock ............................................................................................................................................ 762<br />

HIV/AIDS .................................................................................................................................................................. 763<br />

Tetanus ...................................................................................................................................................................... 766<br />

Rabies ....................................................................................................................................................................... 768<br />

Vector-Borne Infections ............................................................................................................................................ 769<br />

Lyme Disease ..................................................................................................................................................... 769<br />

Rocky Mountain Spotted Fever ........................................................................................................................... 771<br />

West Nile Virus ................................................................................................................................................... 772<br />

Malaria ............................................................................................................................................................... 773<br />

Biological Warfare Agents ......................................................................................................................................... 774<br />

Smallpox ............................................................................................................................................................ 774<br />

Anthrax .............................................................................................................................................................. 775<br />

Other High-Priority Biologic Agents ................................................................................................................... 777<br />

Legionella pneumophila ........................................................................................................................................... 777<br />

Toxoplasmosis ........................................................................................................................................................... 778<br />

Ebola ......................................................................................................................................................................... 778<br />

759


SYSTEMIC INFECTIOUS DISORDERS: SELF-ASSESSMENT QUESTIONS<br />

SYSTEMIC INFECTIOUS DISORDERS: SELF-ASSESSMENT QUESTIONS<br />

1. Classically, the rash of Rocky Mountain spotted fever:<br />

(a) Begins on the wrists and ankles, spreads to the palms and soles, and then progresses centripetally<br />

(b) Is coincident with the onset of fever<br />

(c) Appears initially at the site of the bite<br />

(d) Begins on the chest, face, and neck and spreads to the extremities<br />

2. The vector for spread of Rocky Mountain spotted fever is<br />

(a) The male lxodes tick<br />

(b) The female lxodes tick<br />

(c) The male Dermacentor tick<br />

(d) The female Dermacentor tick<br />

3. All of the following statements regarding erythema migrans are true except:<br />

(a) It is tender and painful.<br />

(b) It is seen in Stage I.<br />

(c) It is warm to the touch.<br />

(d) It is nonpruritic.<br />

4. Antibiotics that may be used in treating children with Lyme carditis include all of the following except:<br />

(a) Penicillin<br />

(b) Tetracycline<br />

(c) Amoxicillin<br />

(d) Ceftriaxone<br />

5. The animals most commonly associated with the transmission of rabies in the United States are:<br />

(a) Rodents, lagomorphs, and cats<br />

(b) Skunks, bats, and raccoons<br />

(c) Cats, dogs, and foxes<br />

(d) Cattle, foxes, and rabbits<br />

6. Characteristic features of anthrax on chest radiograph include all of the following except:<br />

(a) Widespread edema<br />

(b) Diffuse hemorrhagic lymphadenitis<br />

(c) Lobar pneumonia<br />

(d) Mediastinal widening<br />

7. Initial antibiotic therapy for inhalational anthrax is:<br />

(a) Ciprofloxacin or doxycycline<br />

(b) Levofloxacin or a third-generation cephalosporin<br />

(c) Ofloxacin or vancomycin<br />

(d) Streptomycin or clindamycin<br />

8. Al I of the following are characteristic of primary HIV except:<br />

(a) Positive HIV ELISA<br />

(b) Fever<br />

( c) Sore throat<br />

(d) Increased HIV viral load<br />

9. A minor criteria required for the diagnosis of smallpox is:<br />

(a) A febrile prodrome<br />

(b) "Belly-button" lesions<br />

(c) Lesions at the same stage of evolution<br />

(d) Centrifugal distribution of the rash with the greatest concentration of lesions on the face and distal extremities<br />

760


SYSTEMIC INFECTIOUS DISORDERS: SELF-ASSESSMENT QUESTIONS<br />

10. A suspected case of smallpox is one that:<br />

(a) Is epidemiologically linked to a laboratory-confirmed case<br />

(b) Does not meet the clinical criteria but is clinically consistent with smallpox<br />

(c) Is characterized by fever that precedes development of the rash<br />

(d) Meets the clinical criteria of smallpox<br />

11. CSF findings in patients with West Nile virus include:<br />

(a) PMNs + t protein+ pleocytosis<br />

(b) Lymphocytes+ t protein+ pleocytosis<br />

(c) Lymphocytes + t protein + pleocytosis<br />

(d) PMNs + t protein + pleocytosis<br />

12. Which of the following statements is true regarding the role of steroids in the treatment of Pneumocystis jerovici<br />

(PJP) pneumonia?<br />

(a) They have no role.<br />

(b) They are beneficial as adjunctive therapy in patients with moderate to severe PJP pneumonia.<br />

(c) They are beneficial as adjunctive therapy in patients with mild PJP pneumonia.<br />

(d) They should be used as a primary therapeutic modality in all patients with PJP pneumonia.<br />

13. Severe malaria is defined by:<br />

(a) Parasitemia > 1 %<br />

(b) Pulmonary edema<br />

(c) Thrombocytosis<br />

(d) High fever<br />

14. All of the following features define systemic inflammatory response syndrome (SIRS) except:<br />

(a) Mean arterial pressure 90 beats per minute<br />

(c) Respiratory rate >20 breaths per minute or PaCO 2 12,000/mm 3 , 10% bands<br />

ANSWERS<br />

I. a<br />

2. d<br />

5.<br />

6.<br />

b<br />

C<br />

9.<br />

10.<br />

3. a 7. a 11. b<br />

4. b 8. a 12. b<br />

d<br />

C<br />

13. b<br />

14. a<br />

Use the pre-chapter multiple choice question worksheet (page xvi) to record and determine the percentage of correct<br />

answers for this chapter.<br />

761


SYSTEMIC INFECTIOUS DISORDERS<br />

I. SEPSIS AND SEPTIC SHOCK<br />

A. Definitions<br />

1. Sepsis is defined as a life-threatening organ dysfunction caused by dysregulated host response to infection<br />

2. For ICU patients, a sequential organ failure assessment (SOFA) score of 2 or more points meets the criteria<br />

for sepsis (associated with in-hospital mortality> 10%). Because the SOFA score requires multiple laboratory<br />

tests, the following bedside clinical score, termed quickSOFA, has been proposed for use in the emergency<br />

department (2 or more of the following): respiratory rate c':22 breaths per minute, systolic blood pressure<br />

::;100 mm Hg, or altered mental status<br />

3. Septic shock is defined as requirement of a vasopressor to maintain a mean arterial pressure c':65 mmHg<br />

despite adequate volume resuscitation and a serum lactate level >2 mmol/L.<br />

4. The term "severe sepsis" is no longer used.<br />

5. These new sepsis definitions abandon the concept of systemic inflammatory response syndrome (SIRS) because<br />

those symptoms occur in a large majority of hospitalized patients and are often associated with benign<br />

conditions. SIRS is defined by two or more of the following:<br />

B. Risk factors<br />

a. Temperature > 101 °F (38.3°C) or 90 beats per minute<br />

c. Respiratory rate >20 breaths per minute or PaCO, 12,000 cells/mm 3 , 10% bands<br />

1. Age >65 years old<br />

2. lmmunosuppression (AIDS, chronic steroid therapy, chemotherapy, transplant medicines, diabetes)<br />

3. Bacteremia<br />

4. Chronic indwelling lines and tubes (Foley catheters, central lines)<br />

C. Pathogens<br />

1. Gram-positive cocci are most common, followed by gram-negative rods (in the United States).<br />

2. Hospital-acquired pathogens are more likely to be resistant to multiple drugs and result in higher mortality rates.<br />

3. Pathogens from a urinary source result in a lower mortality rate than those from pulmonary, GI, or unknown<br />

sources.<br />

D. Diagnostic evaluation<br />

1. History and physical examination for source, such as intra-abdominal infections (cholecystitis, appendicitis,<br />

or peritonitis), skin examination (cellulitis, evidence of IV drug abuse), indwelling catheters (dialysis catheter,<br />

urinary catheters, central Ii nes), endocarditis<br />

2. Complementary diagnostic studies: chest radiograph, urinalysis, urine culture, blood culture, CBC, serum<br />

chemistries, lactate<br />

E. Management<br />

1. Early aggressive resuscitation<br />

a. Focused on reversing end-organ hypoperfusion/hypoxia by aggressive fluid administration, early broad<br />

antibiotic administration, and vasopressor support<br />

b. Administration of broad-spectrum antibiotics within 1 hour of recognition of septic shock<br />

c. An initial fluid bolus of 30 ml/kg of normal saline<br />

2. Supplemental oxygen therapy and intubation/mechanical ventilation as indicated by patient's clinical status<br />

(increased work of breathing, hypoxia, altered mental status, inability to protect airway)<br />

3. Norepinephrine is vasopressor of first-choice. Epinephrine or vasopressin can be added as needed to maintain<br />

mean arterial pressure >65 mmHg.<br />

4. Dopamine is not recommended except in highly select circumstances (eg, patients with low risk of<br />

tachyarrhythmias and absolute or relative bradycardia).<br />

5. Corticosteroids are only used to treat adult septic patients with shock if adequate fluid resuscitation and<br />

vasopressor therapy are unable to restore hemodynamic stability.<br />

6. Antibiotic therapy<br />

a. Community-acquired urinary tract infection: fluoroquinolone or third-generation cephalosporin<br />

b. Community-acquired pneumonia (non-ICU): ceftriaxone + macrolide<br />

c. Pneumonia, recently hospitalized: fourth-generation cephalosporin or piperacillin/tazobactam + vancomycin<br />

762


SYSTEMIC INFECTIOUS DISORDERS<br />

d. Unknown source: broad spectrum covering gram-positive, gram-negative and anaerobic organisms with<br />

special consideration for methicillin-resistant Staphylococcus aureus (MRSA), eg, piperacillin/tazobactam<br />

+ vancomycin<br />

II. HIV/ AIDS<br />

A. Etiology and pathogenesis<br />

1. Disease complex resulting from an incompetent immune system. It begins when a patient becomes infected<br />

with HIV, a retrovirus.<br />

2. CDC estimates that in the United States, over 1.2 million people are HIV positive, and 50,000 people are<br />

newly infected yearly.<br />

3. HIV reduces the number of normally immunocompetent cells (especially T-helper lymphocytes).<br />

4. AIDS is defined by CD4 count 95% after 6 months).<br />

6. HIV antibody screening with ELISA confirmed by Western blot assay.<br />

E. AIDS-related diseases<br />

1. Pneumocystis jiroveci pneumonia (PJP)<br />

a. Epidemiology<br />

(1) The causative organism is P jiroveci (formerly P carinii), a genus of unicellular fungi found in the<br />

respiratory tract.<br />

(2) Seen almost exclusively in immunosuppressed patients<br />

(a) Patients with AIDS<br />

(b) Patients receiving immunosuppressive therapy for cancer (especially corticosteroids) or organ<br />

transplantation<br />

(c) Premature and malnourished infants<br />

(d) Children with primary immunodeficiency disease<br />

(3) Most common opportunistic infection seen in HIV patients; the leading cause of death in these patients<br />

(a) 80% of HIV patients acquire PJP at some time during their illness.<br />

(b) It is the initial opportunistic infection in 2:60% of those who are not receiving prophylactic therapy.<br />

(c) In adults, infection generally does not occur until the CD4 lymphocyte count is


SYSTEMIC INFECTIOUS DISORDERS<br />

b. Clinical presentation<br />

(1) Signs and symptoms develop in a slow and insidious fashion in AIDS patients; most have been<br />

symptomatic for 2-3 weeks at the time of diagnosis. Abrupt onset of signs and symptoms with rapid<br />

progression occurs more commonly with oncology patients.<br />

(2) Patients usually present with dyspnea, nonproductive cough, and fever. Decreased exercise tolerance<br />

is also common. Other more variable complaints include weight loss, night sweats, chest pain, fatigue,<br />

and chills.<br />

(3) Typical physical findings are cyanosis with tachypnea, tachycardia, and a moderately increased<br />

temperature.<br />

c. Diagnostic evaluation<br />

(1) Arterial blood gases (or pulse oximetry as substitute) are frequently abnormal.<br />

(a) Decreased pO 2<br />

(b) Increased alveolar-arterial (A-a) oxygen gradient<br />

(c)<br />

Low oxygen saturation or desaturation with 10 minutes of exercise<br />

(2) Chest radiograph<br />

(a) May be normal in up to 20%-30% of patients; this is more common early in the disease process.<br />

(b) Classically demonstrates bilateral diffuse interstitial or alveolar infiltrates beginning in the perihilar<br />

region and extending in a "bat-wing" pattern.<br />

(c) PJP is the most common cause of pneumothorax in patients with AIDS.<br />

(3) Lactate dehydrogenase: increased levels in the setting of clinical suspicion and normal chest<br />

radiograph are helpful in making the diagnosis. Degree of increase correlates with prognosis.<br />

(4) A high-resolution CT scan that reveals patchy nodular densities suggests the diagnosis.<br />

(5) The diagnosis in the emergency department is a clinical one. Inpatient evaluation may include<br />

bronchoscopy/lavage and sputum monoclonal antibodies.<br />

d. Treatment<br />

(1) Oxygen<br />

(2) Antibiotics<br />

(a) TMP-SMX<br />

1. Initial drug of choice in patients who can tolerate sulfa drugs. It has the advantage of providing<br />

coverage for some bacterial pneumonias and is well tolerated in non-AIDS patients.<br />

ii. Dosage is trimethoprim at 15-20 mg/kg/day and sulfamethoxazole at 75-100 mg/kg/day IV or<br />

orally divided in four doses x 14-21 days.<br />

iii. Adverse reactions include nausea and vomiting, fever, rash, increased liver enzymes, and<br />

neutropenia.<br />

(b) IV Pentamidine<br />

1. May be used as an alternative drug for patients with a history of severe allergy or adverse<br />

reactions to sulfonamides.<br />

ii. Dosage is 4 mg/kg/day IV over 1 hour x 14-21 days. Blood pressure must be carefully<br />

monitored during infusion, because hypotension is a common adverse effect.<br />

iii. Adverse reactions include hypotension, syncope, tachycardia, fever, facial flushing, pruritus,<br />

renal toxicity, increased liver enzymes, hypoglycemia, rash, thrombocytopenia, neutropenia,<br />

pancreatitis, and hallucinations.<br />

(3) Steroids<br />

(a) Beneficial as adjunctive therapy in patients with moderate to severe PJP. They limit oxygen<br />

deterioration, decrease mortality and respiratory failure, and accelerate recovery.<br />

(b) Administer them to all children and to adult patients with a p0 2<br />

35 mmHg.<br />

(c) Start before the antibiotic is given, because hypoxemia may worsen.<br />

(d) Dosage<br />

i. Prednisone is administered at a starting dosage of 40 mg bid x 5 days, followed by 40 mg/<br />

day x 5 days, followed by 20 mg/day x 11 days.<br />

ii. Methylprednisolone may be substituted for prednisone at 75% of the above dosages if IV<br />

therapy is preferred.<br />

764


SYSTEMIC INFECTIOUS DISORDERS<br />

e. Disposition<br />

(1) Hospitalization is indicated for most patients, especially children and those with a prior history of PJP,<br />

because the mortality rate increases with subsequent episodes.<br />

(2) Patients with mild disease and favorable respiratory parameters can be treated on an outpatient basis if<br />

close follow-up can be assured.<br />

f. Prophylaxis<br />

(1) Recommended for the following patients:<br />

(a) Those with a prior episode of PJP pneumonia (the recurrence rate is 60% in AIDS patients)<br />

(b) HIV-infected patients with a CD4 count 3 weeks for esophageal<br />

candidiasis<br />

765


SYSTEMIC INFECTIOUS DISORDERS<br />

6. Herpes zoster (shingles)<br />

a. Can be a severe and recurrent problem at any stage of HIV infection<br />

b. Acyclovir orally (in large doses) or parenterally is the treatment of choice.<br />

7. Cytomegalovirus retinitis<br />

a. Most common ocular complication of AIDS and, if untreated, leads to blindness<br />

b. Funduscopy: small, white perivascular infiltrates (early); hemorrhagic necrotizing retinitis (late)<br />

c. Treatment with foscarnet or ganciclovir is indicated.<br />

8. A host of other AIDS-related diseases can affect one or more organ systems, including tuberculosis (see<br />

Thoracic and Respiratory Disorders, pages 231-291 ). The management of AIDS and associated opportunistic<br />

infections is rapidly evolving; early consult with an infectious disease specialist regarding therapeutic options is<br />

highly recommended.<br />

F. Postexposure prophylaxis<br />

1. Risk of HIV transmission: 0.3% for percutaneous inoculation<br />

a. Risk greatest if hollow-bore needle in contact with infected blood<br />

b. Splashes on mucous membranes or broken skin 0.09% risk of transmission<br />

2. Fluids that can transmit HIV: blood, seminal/vaginal, breast milk (feces, nasal, saliva, sweat, tears, urine, and<br />

vomit are not infectious unless bloody)<br />

3. Nonoccupational exposure<br />

a. Risk greatest with receptive anal intercourse (1 %-30%), insertive anal or receptive vaginal (0.1%-10%),<br />

and insertive vaginal (0.1 %-1 %).<br />

b. Risk of needle sharing for IV drug abuse is 0.67% per contact.<br />

4. Management<br />

a. Start as soon as possible.<br />

b. Three-drug regimen: emtricitabine (FTC), tenofovir DF, and raltegravir<br />

c. 28-day duration unless the source patient is found to be HIV-negative<br />

d. Adverse effects: nausea/vomiting, diarrhea, headache<br />

e. Renal toxicity possible with tenofovir DF<br />

f. 24-hour postexposure prophylaxis hotline (PEPline): (888) 448-4911<br />

Ill. TETANUS<br />

A. Etiology<br />

1. Tetanus is a potentially fatal neuroparalytic disease caused by tetanospasmin, which is an exotoxin produced<br />

by Clostridium tetani, a gram-positive anaerobic rod found chiefly in soil and the feces of many animals<br />

(including people).<br />

2. The organism cannot invade healthy tissue, because it requires an anaerobic environment to convert the spores<br />

into toxin-producing vegetative forms. The toxin enters peripheral nerve endings and ascends to the spinal<br />

cord and brain. The major effect occurs in the spinal cord where the toxin prevents release of the inhibitory<br />

neurotransmitters, thereby producing neuromuscular irritability and generalized spasms.<br />

3. High-risk patients<br />

a. The elderly (inadequate immunization)<br />

b. IV drug abusers<br />

c. Patients with decubiti or diabetic ulcers<br />

4. In 10%-20% of cases, no causative wound or injury is ever determined.<br />

B. Clinical presentation<br />

1. The toxin has no effect on the sensorium, and patients are alert and oriented.<br />

2. Trismus ("lockjaw") is the presenting symptom in >50% of cases.<br />

3. The longer the incubation period (range 1 month), the less severe the disease.<br />

4. Four forms of clinical tetanus<br />

a. Local tetanus<br />

(1) Muscle rigidity close to the site of injury<br />

(2) Resolves in weeks to months without sequelae<br />

(3) May progress to the generalized form<br />

766


SYSTEMIC INFECTIOUS DISORDERS<br />

b. Generalized tetanus (most common form)<br />

(1) Pain and stiffness in the jaw and trunk muscles - facial rigidity (trismus and risus sardonicus) -<br />

spasms and tonic contractions - dysphagia, opisthotonos - glottal spasm (with respiratory distress<br />

that may lead to arrest) and painful convulsive spasms (arms flexed, fists clenched, legs extended). This<br />

may be confused with seizures.<br />

(2) Autonomic nervous system dysfunction (occurs during the second week of clinical tetanus) - signs<br />

of catecholamine excess (hyperpyrexia, sweating, tachycardia, and labile hypertension) - t blood<br />

pressure, pulse, and temperature - high morbidity and mortality rate<br />

c. Cephalic tetanus (may occur in fully immunized patients)<br />

(1) Head injury or otitis media - cranial nerve dysfunction (cranial nerve VII most common) and trismus<br />

(2) Poor prognosis<br />

d. Neonatal tetanus<br />

(1) Secondary to inadequate maternal immunization<br />

(2) Usually results from an infected umbilical stump<br />

(3) Extremely high mortality rate<br />

C. Differential diagnosis<br />

1. Strychnine poisoning (strychnine is detectable in the urine)<br />

2. Dystonic reactions to phenothiazines (improves with diphenhydramine)<br />

3. Hypocalcemic tetany<br />

4. Early rabies (unlike tetanus, trismus is uncommon)<br />

D. Management<br />

1. Maintain an adequate airway to prevent asphyxia.<br />

2. Use a benzodiazepine to control muscular spasms.<br />

3. If the patient needs aggressive airway management, neuromuscular blockade with vecuronium. Concomitant<br />

sedation is mandatory; use a benzodiazepine, barbiturate, or propofol.<br />

4. Autonomic dysfunction, manifested by sympathetic overactivity, can be treated with labetalol.<br />

5. Meticulous surgical debridement of the causative wound is critical. Theoretically, this decreases continued<br />

toxin production.<br />

6. Give human tetanus immune globulin 500 IU IM to neutralize any unbound toxin.<br />

7. The antibiotic of choice is IV metronidazole.<br />

8. Clinical disease does not produce immunity, so give the first dose of tetanus toxoid in the emergency<br />

department.<br />

9. A quiet room is essential to prevent precipitation of generalized spasms.<br />

E. Tetanus prophylaxis protocol<br />

Table 48: Tetanus Immunization Protocol<br />

Previous Immunization<br />

Uncertain or 5 years ago<br />

c::3 doses with last dose > 10 years ago<br />

Minor Wound<br />

Vaccinate<br />

Vaccinate<br />

All Other Wounds<br />

Vaccinate + tetanus<br />

immune globulin<br />

Vaccinate<br />

Vaccinate<br />

1. Complete primary tetanus immunization for adults consists of three IM injections ofTd 0.5 ml; the second<br />

dose is given 6 weeks after the first, and the third is given 6 months later.<br />

2. Contaminated (tetanus-prone) wounds<br />

a. Those contaminated with feces, soil, or saliva<br />

b. Punctures, crush wounds, avulsions<br />

c. Burns and frostbite<br />

d. Those from missiles<br />

e. Those >6 hours old or infected on initial presentation<br />

767


SYSTEMIC INFECTIOUS DISORDERS<br />

3. Patients >60 years old and immigrants are most likely to have inadequate prior tetanus immunization. If a<br />

patient's vaccination status is unknown or uncertain, he or she should receive the complete vaccination series.<br />

4. Give human tetanus immune globulin to individuals with a tetanus-prone wound who have not completed a<br />

primary immunization series or who have a contraindication to receiving Td.<br />

5. The CDC recommends that emergency physicians offer tetanus prophylaxis routinely to all patients<br />

(regardless of the presenting complaint) who have not been immunized within the last 10 years.<br />

IV. RABIES<br />

A. Etiology<br />

1. A rhabdoviral infection of the CNS with an incubation period that usually ranges from 30 days to years<br />

2. Extremely rare in the United States (22 cases from 1980 to 1997)<br />

3. Transmitted primarily by the saliva of an infected animal through a break in the victim's skin or mucous membrane.<br />

4. In the United States, ~90% of animal rabies is found only in wildlife (raccoons, skunks, bats, foxes, coyotes,<br />

and bobcats). Almost all cases of rabid domestic animals have been reported in rural areas; rabies is prevalent<br />

in local wildlife species.<br />

a. Major wildlife reservoir is the raccoon, with> 60% of all reported cases (most prevalent in the eastern states).<br />

b. Skunk rabies is predominant in the central and western states.<br />

c. Since 1980, 78% of documented rabies cases have had no history of exposure.<br />

B. Clinical presentation<br />

1. The prodromal period may resemble "viral illness." Symptoms are variable and include fever, headache,<br />

malaise, sore throat, dry cough, anorexia, and nausea and vomiting; back pain is common. The patient may not<br />

tell you about the bite or the associated tingling, pain, or numbness if other symptoms are more distressing.<br />

2. The excitement phase of clinical presentation is variable and includes thought disturbances with lucid intervals,<br />

hypersensitivity to light/sound/touch, agitation, confusion, hallucinations, restlessness, or even seizures.<br />

3. Finally, evidence of brainstem dysfunction becomes apparent. If the patient is lucid, check for diplopia, facial<br />

palsies, and dysphagia (which leads to the characteristic hydrophobia). From this point, the patient lapses into a<br />

coma. Involvement of the respiratory center follows, and an apneic death ensues.<br />

C. Postexposure prophylactic management<br />

1. Thoroughly irrigate the wound; this can decrease the amount of virus at the wound site and is an important<br />

step in rabies prevention.<br />

2. Tetanus prophylaxis as indicated.<br />

3. Vaccine selection<br />

a. Active immunization with human diploid cell vaccine (HDCV) is indicated after a bite from an animal in<br />

the suspected group; 1 ml is given IM on day 0 and repeated on days 3, 7, and 14.<br />

b. Passive immunization with human rabies immune globulin (HRIG) is advised if the animal is (or may<br />

be) rabid; 20 IU/kg, with as much as possible infiltrated at the site of the bite, and the remainder given<br />

IM at a site near to the wound (avoid gluteal administration because vaccine deposited in fat is poorly<br />

immunogenic).<br />

D. Management<br />

1. Report the incident to the public health department.<br />

2. Domestic animals (cats and dogs)<br />

a. If the animal is secured/captured, its behavior is normal, and it remains normal for a 10-day observation<br />

period, no treatment is necessary.<br />

b. If the animal is secured/captured and exhibits abnormal behavior or becomes ill during the 10-day<br />

observation period, the animal should be euthanized, the head/brain placed in a plastic bag, and<br />

sent refrigerated (not frozen or fixed in formalin) to the state public health department for direct<br />

immunofluorescent antibody studies.<br />

(1) If laboratory analysis is negative for rabies, no treatment is needed.<br />

(2) If laboratory analysis is positive for rabies, treatment with HRIG and the HDVC series is indicated and<br />

should be started as soon as possible.<br />

c. If the animal cannot be found and captured, pub I ic health officials should be contacted regarding the<br />

prevalence of rabies in the involved species in your locality and the need for treatment.<br />

768


SYSTEMIC INFECTIOUS DISORDERS<br />

3. Wild animals (skunks, bats, etc): assume they are rabid unless proved negative by direct immunofluorescent<br />

antibody testing.<br />

a. If the wild animal cannot be found and captured, begin treatment with HRIC and the HDCV series.<br />

b. Wild animals that are captured should be tested immediately.<br />

(1) If laboratory analysis is negative for rabies, no treatment is needed.<br />

(2) If laboratory analysis is positive for rabies, administer HRIC and the HDCV series.<br />

c. Consider postexposure prophylaxis for those who were in the same room as a bat and might be unaware of<br />

bite or contact.<br />

4. Bites from rodents, squirrels, and rabbits almost never require postexposure prophylaxis.<br />

E. Diagnostic evaluation<br />

1. Performed on a specimen of brain tissue obtained postmortem or by biopsy<br />

2. Fluorescent antibody testing is the procedure of choice (both sensitive and specific).<br />

3. Staining for Negri bodies: although these intracytoplasmic inclusion bodies are pathognomonic for rabies, they<br />

are absent in up to 25% of cases (unacceptably high falsenegative rate).<br />

V. VECTOR-BORNE INFECTIONS (SPREAD BY MOSQUITOES,<br />

TICKS, AND FLEAS)<br />

A. Lyme disease<br />

1. Etiology and epidemiology<br />

a. This tick-borne illness has been reported on every continent in the world except Antarctica; in the United<br />

States, the highest frequency is in New England and the middle Atlantic and upper Midwestern states.<br />

b. The lxodes tick (deer tick) harbors the spirochete Borrelia burgdorferi.<br />

c. The most common tick- and vector-borne disease in the United States; most cases occur between May and<br />

August (when tick and human outdoor activity are greatest) and where deer populations are increasing.<br />

2. Clinical presentation<br />

a. Early localized disease (within 1 month)<br />

(1) History from patients who have been in or near wooded areas<br />

(a) Fever and fatigue<br />

(b) Malaise and myalgia<br />

(c) Headache<br />

(2) Physical examination: erythema migrans is the hallmark of early Lyme disease---;,. circular skin lesions<br />

that begin as a macule or papule at the site of the tick bite and gradually enlarge, at times attaining<br />

diameters of c::25 cm. Classically, erythema migrans has a bright-red to blue-red border with a pale<br />

interior; it is warm to the touch and is neither tender nor pruritic. Erythema migrans is absent in 20%<br />

of cases. Note: For pictures of erythema migrans and further specifics on treatment guidelines, see the<br />

Centers for Disease Control website (http://www.cdc.gov/lyme).<br />

Source: http://phil.cec.gov/phil ID #9875<br />

Content provider: CDC/James Cathany<br />

769


SYSTEMIC INFECTIOUS DISORDERS<br />

b. Early disseminated disease (weeks to months)<br />

(1) Neurologic abnormalities predominate the clinical picture.<br />

(a)<br />

Fluctuating meningoencephalitis (most common)<br />

(b) Cranial neuropathy (especially Bell's palsy)<br />

(c) Peripheral neuropathy (mononeuritis, radiculitis, brachia! plexitis)<br />

(2) Myocarditis is the most common cardiac abnormality; it occurs transiently and is associated with<br />

varying degrees of AV block.<br />

c. Late disease (months to years later-,, chronic infection)<br />

(1) Migratory oligoarthritis of the large joints is characteristic.<br />

(a) The knee is most often affected and may be associated with an effusion.<br />

(b) A clue to the diagnosis is the intermittent nature of the joint pain and swelling that occurs for<br />

weeks or months; then it completely disappears and recurs sometime later.<br />

(2) Neurologic symptoms include a wide range of manifestations that may mimic other neurologic<br />

conditions and, at this stage of disease, may be the only presenting complaint. Be especially suspicious<br />

of any of the following:<br />

3. Diagnostic evaluation<br />

(a) Subtle encephalopathy (disturbances in mood, memory, sleep) or polyneuropathy (spinal/radicular<br />

pain or numbness/tingling in the hands and feet)<br />

(b) Cognitive dysfunction (confusion -,, dementia)<br />

(c) Incapacitating fatigue<br />

a. Clinical suspicion is of utmost importance, because laboratory testing may be inaccurate or misinterpreted.<br />

A history of tick bite (or exposure in an endemic area) followed by a flu-like syndrome and a rash<br />

(erythema migrans) suggests the possibility of Lyme disease. Unfortunately, 66% of patients do not recall a<br />

tick bite, and 20%-40% never develop or notice the rash.<br />

b. Cerebrospinal fluid (CSF) is almost always abnormal but is frequently confused with viral meningitis,<br />

because lymphocytes and monocytes are the dominant cell types; however, when associated with an t CSF<br />

protein, the diagnosis should be suspected.<br />

c. The two-test approach is recommended.<br />

(1) Initial positive ELISA screen is followed by Western blot. ELISA has a high false-positive rate but good<br />

sensitivity. PCR is not recommended because of poor sensitivity and specificity. There is no value in<br />

testing in early localized disease.<br />

(2) lgM antibodies develop within 1-2 weeks; lgG antibodies develop within 2-6 weeks. Antibodies may<br />

be present for years after Lyme disease treatment.<br />

(3) Antibiotics in early Lyme disease may prevent seroconversion.<br />

d. ECG for evaluation of AV block<br />

4. Post-tick exposure prophylaxis<br />

a. Deer tick exposure with tick engorgement for >36 hours is treated with one dose of doxycycline (200 mg)<br />

given orally.<br />

b. Ticks that are embedded in the skin must be removed ensuring that there is no compression of the gut by<br />

using fine tweezers. Grasp the tick as close as possible to the head and pull straight out.<br />

5. Management<br />

a. Early localized disease: erythema migrans treated with doxycycline or amoxicillin for 10-14 days<br />

b. Early disseminated disease<br />

(1) Neurologic<br />

(a) Isolated facial palsy: oral doxycycline or amoxicillin for 14 days<br />

(b) Other neurologic: IV therapy for 14-21 days<br />

(2) Cardiac<br />

c. Late disease<br />

(a) Cardiac monitor and temporary pacing if needed<br />

(b) IV therapy until cardiac symptoms resolve up to 21 days<br />

(1) Arthritis: 28-day course of oral antibiotics<br />

(2) Neurologic: 28-day course of IV antibiotics<br />

770


SYSTEMIC INFECTIOUS DISORDERS<br />

d. Medication<br />

(1) Oral<br />

(2) IV<br />

(a) Doxycycline 100 mg twice daily (contraindicated in children 102.2°F [39°C]).<br />

b. Although rash is the hallmark of RMSF, it is absent in 5 %-15 % of cases.<br />

(1) An erythematous blanching rash (with 2-3 mm macules) appears initially on the flexor surfaces of<br />

the wrists and ankles, spreads to the palms and soles, and then moves rapidly (centrally) to cover<br />

most of the body.<br />

(2) In the next 2-3 days, the rash becomes maculopapular, darker red, fixed, and finally petechial.<br />

c. A diagnostic clue on physical examination is extreme tenderness of the gastrocnemius muscle.<br />

4. Diagnostic evaluation (usually useful only retrospectively)<br />

a. Because serologic tests are often negative in the early phase of illness (results are not usually available<br />

during emergency department assessment and availability of specific tests vary from hospital to hospital),<br />

the diagnosis of RMSF is a clinical one. The triad of fever, headache, and rash, occurring late spring to early<br />

fall, is presumptive evidence for treatment and should not await positive serologic testing. The mortality rate<br />

remains high because of delay in starting appropriate antibiotic therapy.<br />

(1) Laboratory findings of neutropenia, thrombocytopenia, increased liver function studies, and<br />

hypernatremia are suggestive.<br />

(2) Indirect fluorescent antibody assay<br />

(a) The most sensitive and specific test<br />

(b) A titer> 1 :64 is diagnostic<br />

(3) Indirect hemagglutination (second most sensitive and specific)<br />

(4) The Weil-Felix, complement fixation, and latex agglutination tests are much less sensitive.<br />

771


SYSTEMIC INFECTIOUS DISORDERS<br />

b. Skin biopsy: immunofluorescent antibody staining of a rash specimen<br />

(1) 70% sensitivity, 1 00% specificity<br />

(2) The best rapid diagnostic test; if available, it may provide rapid diagnosis (as early as the third day<br />

of illness).<br />

5. Management<br />

a. Early antibiotic therapy significantly reduces the mortality rate so, if you suspect RMSF, start doxycycline<br />

therapy.<br />

(1) Adults: 100 mg orally or IV bid<br />

(2) Children 102.2°F [39°(], headache, and myalgias).<br />

(1) Signs of meningitis: nuchal rigidity<br />

(2) Symptoms of early encephalitis: mental status changes (confusion, disorientation, t level of<br />

consciousness)<br />

(3) Signs of severe encephalitis: stupor or coma, some degree of paresis (including flaccid paralysis) or<br />

proximal muscle weakness<br />

(4) These are typically older and/or debilitated patients. Patients with neuroinvasive illness have a<br />

mortality rate of approximately 10%.<br />

772


SYSTEMIC INFECTIOUS DISORDERS<br />

4. Diagnostic evaluation<br />

a. CSF: lymphocytes+ t protein+ pleocytosis (30-100 cells/mm 3 ) and no bacterial pathogens<br />

b. Radiologic studies<br />

(1) CT rarely shows signs of CNS inflammation.<br />

(2) MRI demonstrates enhancement of leptomeninges and/or periventriular areas in 30% of patients.<br />

c. Differential diagnosis<br />

5. Treatment<br />

(1) Herpes simplex virus 1 meningoencephalitis<br />

(a) Predominance of PMNs (not lymphocytes), RBCs, and possibly glucose<br />

(b) Electroencephalography: temporal lobe focus<br />

(2) Bacterial meningitis: positive Gram stain and culture<br />

a. Empiric treatment for bacterial meningitis and/or herpes simplex viral encephalitis is appropriate until<br />

results of testing for West Nile virus-lgM are clarified.<br />

b. Symptomatic and supportive measures are the mainstay of therapy; there is no specific therapy for the<br />

treatment of West Nile virus.<br />

6. Prevention<br />

a. No human vaccine is currently available.<br />

b. Reduce mosquito breeding.<br />

(1) Drainage of standing water<br />

(2) Chemical spraying (to reduce vector transmission)<br />

c. Avoid mosquito exposure.<br />

D. Malaria<br />

1. Etiology<br />

a. Plasmodium fa/ciparum (only one that can cause severe disease)<br />

b. P malariae<br />

c. P vivax and P ovale can develop dormant stage for years.<br />

d. Vector: Anopheles mosquito<br />

2. Epidemiology<br />

a. Consider malaria in all febrile travelers from areas where malaria is endemic.<br />

b. Incubation 7-30 days (month with inadequate prophylaxis)<br />

c. 1-2.7 million related deaths per year (75% African children)<br />

d. Leading cause of death in children worldwide<br />

e. Leading reason for blood transfusion worldwide<br />

3. Clinical presentation<br />

a. Classic malaria<br />

(1) Cyclic stages of rigors, fever, headache, vomiting, myalgias, seizures (infants), sweats, fatigue<br />

(2) Every 2 days (P falciparum, P vivax, P ovale)<br />

(3) Every 3 days (P malariae)<br />

(4) Splenomegaly<br />

(5) Anemia<br />

b. Severe malaria (P fa/ciparum)<br />

(1) Severe anemia (parasitemia >5%)<br />

(2) Cerebral malaria: confusion, coma, seizures, permanent deficits (deaf, blind, palsies)<br />

(3) Pulmonary edema, ARDS<br />

(4) Thrombocytopenia<br />

(5) Shock<br />

(6) Acute renal failure<br />

4. Diagnostic evaluation: thin and thick smears<br />

5. Management<br />

a. Infectious disease consult (for latest chloroquine sensitivities)<br />

b. Chloroquine sensitive (Middle East, Central America west of Panama Canal)<br />

c. Chloroquine-resistant strains: quinine plus doxycycline or atovaquone/proguanil<br />

d. Severe anemia requires parenteral therapy: quinidine + doxycycline, or artesunate<br />

773


SYSTEMIC INFECTIOUS DISORDERS<br />

VI. BIOLOGICAL WARFARE AGENTS<br />

A. Smallpox<br />

1. Overview<br />

a. Variola (smal I pox) is an orthopoxvirus. Formerly used as a weapon in the biological warfare program of<br />

the Soviet Union, smallpox is one of the most feared potential bioterrorist agents because of the high<br />

mortality rate in unvaccinated patients. A single case, anywhere in the world, would constitute a global<br />

health emergency.<br />

b. Flu-like symptoms (mild to severe) precede the onset of rash by 1-4 days.<br />

c. Characteristic clinical features<br />

(1) Febrile prodrome (> 101 °F [38.3°C]) and at least one of the following:<br />

(a) Headache<br />

(b) Backache<br />

(c) Chills<br />

(d) Vomiting<br />

(e) Severe abdominal pain<br />

(f)<br />

Prostration<br />

(2) Smallpox lesions: distinct well-circumscribed vesicles or pustules that may become umbilicated or<br />

confluent as they evolve<br />

(3) High mortality rates (30%) in unvaccinated patients primarily because of lack of specific therapy<br />

2. Epidemiology<br />

a. There are no known animal or insect vectors. The virus is spread person to person by droplet nuclei or<br />

aerosols expelled from the oropharynx; it can also be spread by direct contact (skin lesions or contaminated<br />

clothing or bedding).<br />

b. Risk of transmission occurs during the prodrome but maximizes with onset of the rash, which usually<br />

begins in the mouth and throat and lasts 7-10 days. The risk of infectivity is significantly decreased when<br />

the lesions scab over. When the scabs fall off the skin, there is no longer any risk of transmission from the<br />

individual, but the scabs remain infectious.<br />

3. Clinical presentation (depends on the evolutionary stage of the illness)<br />

a. Incubation period (7-17 days): not contagious<br />

b. Prodrome (2-4 days): sometimes contagious<br />

Initial symptoms include fever (usually high [101 °-1 04°F; 38.3 °-40°C]), head/body aches, malaise,<br />

± vomiting; patients usually feel too sick to carry on normal activities.<br />

c. Early rash (-4 days): most contagious<br />

Fever and red spots in the mouth - spread to face - arms and legs - hands and feet. Rash develops to<br />

bumps with a thick, opaque fluid and that look like a bellybutton (a major distinguishing characteristic of<br />

smallpox).<br />

d. Pustular rash (-5 days): contagious<br />

The bumps become pustules (sharply raised, round, and firm to the touch) often described by patients as a<br />

feeling of BB pellets embedded under the skin.<br />

e. Pustules and scabs (-5 days): contagious<br />

The pustules form a crust and then scab; by the end of the second week (after the appearance of the rash),<br />

most of the sores have scabbed over.<br />

f. Resolving scabs (-6 days): contagious<br />

As the scabs fall off, they leave marks on the skin that eventually become pitted scars; most of the scabs are<br />

gone by 3 weeks after the rash appeared.<br />

g. Scabs resolved: not contagious<br />

Smallpox remains contagious until al I of the scabs have fallen off. The scabs continue to remain infectious.<br />

4. Diagnostic evaluation<br />

a. Suspicious setting: unexplained fever and a rash in a patient with small red spots or sores on the tongue<br />

and buccal mucosa<br />

774


SYSTEMIC INFECTIOUS DISORDERS<br />

b. Clinical diagnosis<br />

(1) Major criteria<br />

(a) Febrile prodrome with temperature > 101 °F (38.3°C)<br />

(b) Classic "belly-button" lesions<br />

(c) All lesions in the same stage of evolution<br />

(2) Minor criteria<br />

5. Differential diagnosis<br />

(a) Centrifugal distribution of the rash on the face and distal extremities<br />

(b) Lesions appeared first on the oral mucosa and/or palate, face, or arms<br />

(c) Patient appears toxic or moribund<br />

(d) Slow evolution of lesions (macules-;, papules-;, pustules), each type over a period of 1-2 days<br />

(e) Lesions on the palms and soles<br />

a. Chickenpox (varicella) is the most likely infectious disease to be confused with smallpox; however,<br />

chickenpox has distinctive features that should make the diagnosis easier.<br />

(1) Varicella lesions tend to be more superficial, appear in crops, and are present in different stages of<br />

development at any given time, ie, papules, vesicles, and scabs all appear together.<br />

(2) Varicella lesions tend to be more on the abdomen and back and less on the extremities.<br />

b. Disseminated herpes zoster is more likely in an immunocompromised host.<br />

6. Specific treatment<br />

B. Anthrax<br />

a. Smallpox vaccination: if administered within 4 days of the first exposure, attenuates the course of the illness<br />

and protects against a fatal outcome.<br />

b. Vaccinia immunoglobulin: antibodies against a similar virus may shorten duration of disease.<br />

c. Postexposure isolation and infection control practices: critical to limiting transmission of smallpox to<br />

populations outside the target source.<br />

1. Overview<br />

a. lnhalational anthrax is a rare and rapidly fatal disease. A potential biological warfare agent, it is often<br />

difficult to diagnose early; a high index of suspicion is required.<br />

b. Clinical presentation: mild flu-like symptoms progress rapidly to respiratory distress-;, septic shock-;,<br />

multiple organ failure<br />

c. Characteristic features on chest radiograph<br />

(1) Diffuse hemorrhagic lymphadenitis<br />

(2) Widespread edema<br />

(3) Mediastinal widening<br />

d. Management<br />

(1) Fluoroquinolones<br />

(2) Adequate management of respiratory distress and septic/hemorrhagic shock in an ICU setting<br />

(3) Vaccination programs<br />

e. Morbidity/mortality rate is high despite above measures.<br />

2. Epidemiology<br />

a. The distribution of Bacillus anthracis is worldwide and most prevalent among herbivores (cattle, sheep,<br />

horses, and goats) that contaminate soil and water holes with 8 anthracis (which may subsequently<br />

sporulate and persist in the environment).<br />

b. While aggressive animal vaccination has lowered the incidence of anthrax among livestock in the United<br />

States, the microorganism remains endemic in the soil ofTexas, Oklahoma, and the lower Mississippi valley.<br />

c. Most human cases of anthrax are due to agricultural or industrial exposure. Shepherds, farmers, and<br />

workers in manufacturing plants using infected animal products (particularly contaminated hide, goat hair,<br />

wool, or bone) are at highest risk.<br />

d. There are three predominant clinical forms of anthrax:<br />

(1) Cutaneous anthrax (>95% of cases) from entry of spores through skin abrasions<br />

(2) lnhalational anthrax<br />

(3) GI anthrax (contaminated meat)<br />

775


SYSTEMIC INFECTIOUS DISORDERS<br />

3. Microbiology<br />

a. 8 anthracis is a large, square-ended, nonmotile, aerobic, gram-positive rod with a centrally located spore.<br />

b. On gram stain, the spore appears as an "unstained" area.<br />

c. Spores are highly resistant to drying, boiling for 10 minutes, and most disinfectants.<br />

4. Clinical presentation (depends on route of inoculation)<br />

a. Cutaneous anthrax (most common form; recognition is important because its presence could possibly<br />

herald the more serious inhalational anthrax)<br />

(1) Through cuts, abrasions, or biting flies, spores of B anthracis are introduced into the skin; within hours,<br />

they germinate, and vegetative cells multiply and produce anthrax toxin, which causes edema and<br />

necrosis.<br />

(2) Within 5 days of exposure, small painless (but pruritic) papules appear; 24-48 hours later, the papules<br />

enlarge and become vesicular. Edema out of proportion to the size of the vesicle surrounds the lesion<br />

and is often associated with fever, malaise, and regional adenopathy.<br />

(3) Near the end of the first week, the vesicle usually ruptures, and the remaining ulcer progresses to a<br />

black eschar. If recognized and treated promptly, cutaneous anthrax is rarely fatal.<br />

b. Pharyngeal and GI anthrax<br />

(1) After ingestion of contaminated undercooked meat, pharyngeal ulcers and edema of the neck develop<br />

as the anthrax bacilli multiply.<br />

(2) After intestinal absorption, bacteria are transported to mesenteric and regional lymph nodes---,,<br />

multiplication and dissemination---,, hemorrhagic adenitis, ascites, and bacteremia<br />

(3) Within 5 days of ingestion of contaminated meat, the patient experiences severe abdominal pain<br />

associated with hematemesis and hematochezia. Early diagnosis is difficult, resulting in high mortality.<br />

c. lnhalational anthrax<br />

(1) Aerosolized anthrax spores >5 µm in size are usually trapped in the upper airway (pharynx, larynx,<br />

trachea) or cleared by the mucociliary system.<br />

(2) Spores between 2 and 5 µm in size are able to reach the alveolar ducts and alveoli, where they are<br />

engulfed by pulmonary macrophages and transported to mediastinal and hilar lymph nodes.<br />

(3) After a period of germination, a large amount of anthrax toxin is produced. Regional lymph nodes are<br />

quickly overwhelmed, and the toxin finds its way into the systemic circulation.<br />

(a)<br />

Because the organisms are initially transported to mediastinal lymph nodes, a major site of<br />

involvement is the mediastinum, which leads to massive hemorrhagic mediastinitis (a typical<br />

outcome).<br />

(b) Widespread dissemination of the toxin, leading to edema, hemorrhage, necrosis, and septic shock<br />

is an ominous sign; death soon follows.<br />

d. Disease progression<br />

(1) Incubation period (up to 6 days): insidious onset of malaise, myalgia, fatigue, nonproductive cough,<br />

fever, and an occasional sensation of retrosternal pressure (continues for ~4 days)<br />

(2) Sudden onset acute respiratory distress, cyanosis, and hypoxemia. Stridor suggests partial tracheal<br />

compression from enlarged mediastinal lymph nodes. The patient may be diaphoretic, with crackles<br />

heard on auscultation of the lungs. This stage lasts 24-36 hours and often culminates in death.<br />

(a) Chest radiograph: widened mediastinum and pleural effusions are characteristic; lung parenchyma<br />

may appear normal.<br />

(b) Meningeal involvement: seen in up to 50% of cases; usually bloody and may be associated with<br />

subarachnoid hemorrhage<br />

5. Diagnosis and treatment of inhalational anthrax<br />

a. "Suspicion" is the key to diagnosing and treating inhalational anthrax in the first stage. Guidelines:<br />

(1) Do an epidemiologic profile that includes a history of exposure, occupation, and where the victims<br />

have been (to identify a possible "hot zone" where anthrax has been found).<br />

(2) Has the patient been exposed to someone who has the flu? (lnhalational anthrax is not spread person<br />

to person.)<br />

(3) Clinical findings suggesting anthrax rather than flu<br />

(a)<br />

Nausea and vomiting without sore throat, nasal congestion, or rhinorrhea<br />

(b) Chest discomfort or pleuritic pain<br />

(c) Stuttering, onset of dyspnea, and nonproductive cough<br />

(d) Abnormal chest radiograph; consider chest CT if pleural effusion or mediastinal abnormalities<br />

are present.<br />

776


SYSTEMIC INFECTIOUS DISORDERS<br />

(4) Gram stains and cultures should be obtained on blood samples of patients in whom anthrax is<br />

suspected; suspicious results should be reported to the CDC for further evaluation.<br />

b. Antibiotics and supportive care in an ICU setting are the mainstays of therapy.<br />

(1) Consensus recommendations include fluoroquinolones, irrespective of age. Nontoxic victims with<br />

cutaneous anthrax can be treated as outpatients with oral ciprofloxacin or doxycycline for 7-10 days.<br />

Victims with inhalational, cutaneous, or GI disease and toxicity require IV therapy with ciprofloxacin<br />

or doxycycline plus at least two other antibiotics (eg, rifampin, clindamycin, imipenem, or an<br />

aminoglycoside).<br />

(2) The ICU setting is especially useful for hemodynamic monitoring and management of septic/<br />

hemorrhagic shock; also, progressive respiratory insufficiency may necessitate use of ventilatory support.<br />

c. People with a credible history of exposure (based on results of forensic investigation by law enforcement<br />

agencies) or contact with a known or suspected environment contaminated with B anthracis (regardless of<br />

laboratory results) should be offered antimicrobial prophylaxis. Oral ciprofloxacin (500 mg) or doxycycline<br />

(1 00 mg) twice a day is recommended.<br />

d. For images of cutaneous anthrax and chest radiographs, see http://en.wikipedia.org/wiki/Anthrax.<br />

C. Other high-priority biologic agents (can be easily disseminated or transmitted person to person)<br />

1. Pneumonic plague (Yersinia pestis)<br />

2. Botulism (Clostridium botulinum)<br />

3. Tularemia (Francisella tularensis)<br />

4. Viral hemorrhagic fevers (Ebola, Marburg, Lassa, and Argentine viruses)<br />

VII. LEG/ONELLA PNEUMOPHILA<br />

A. Pathophysiology<br />

1. L pneumophila is a gram-negative facultative intracellular bacillus that lives in natural and manmade water<br />

systems. It is implicated in as many as 6% of community-acquired pneumonia cases.<br />

2. Transmission occurs via inhalation of contaminated aqueous aerosols from equipment such as cooling towers,<br />

evaporative condensers, and shower heads. Person-to-person spread has not been documented.<br />

3. The illness occurs seasonally (summer and fall) and has an incubation period of 2-10 days.<br />

4. Populations at risk<br />

a. Patients (particularly men) >50 years old<br />

b. Cigarette smokers<br />

c. Patients with a significant underlying disease (alcoholism, diabetes mellitus, COPD)<br />

d. lmmunosuppressed patients (especially those with transplants)<br />

e. Patients who live or work near construction or excavation sites<br />

f. Recent travel (especially to spas) or changes in plumbing<br />

B. Clinical presentation<br />

1. General systemic manifestations (rigors, high fever, headache, malaise, myalgias, and weakness)<br />

2. Pulmonary symptoms<br />

a. An initially dry cough often becomes productive of purulent sputum and is occasionally accompanied by<br />

hemoptysis.<br />

b. Dyspnea<br />

c. Pleuritic chest pain (33%)<br />

d. Pontiac fever refers to a milder form of the infection resembling influenza.<br />

3. GI symptoms<br />

a. Watery diarrhea (50%)<br />

b. Nausea, vomiting, abdominal pain<br />

4. Neurologic signs<br />

a. Altered level of consciousness<br />

b. Gait disturbance<br />

c. Seizures<br />

777


SYSTEMIC INFECTIOUS DISORDERS<br />

5. Clinical clues within the past history<br />

a. No response to ~-lactamase drugs (penicillin, cephalosporins) or aminoglycosides for a recent infection<br />

b. Onset of symptoms within 10 days of hospital discharge<br />

6. Physical examination<br />

a. Toxic appearance<br />

b. Disorientation and confusion<br />

c. Diffuse inspiratory rales, progressing to signs of consolidation<br />

d. Relative bradycardia (50%)<br />

C. Diagnostic evaluation<br />

1. WBC count 10,000-20,000/mm 3 with a left shift<br />

2. Increased sedimentation rate<br />

3. Abnormal serum chemistries<br />

a. Increased liver function tests<br />

b. Hyponatremia (


SYSTEMIC INFECTIOUS DISORDERS<br />

3. Human-to-human transmission can occur by direct contact with infectious body fluids, including:<br />

a. Saliva<br />

b. Mucus<br />

C. Vomit<br />

d. Feces<br />

e. Tears<br />

f. Breast milk<br />

g. Urine<br />

h. Semen<br />

i. Respiratory droplets<br />

4. Transmission can occur when infected body fluids contact the eyes, nose, mouth, open wound, or abrasion of<br />

an uninfected person.<br />

5. First cases of hemorrhagic fever caused by Ebola occurred in Zaire in 1976.<br />

6. Sporadic isolated outbreaks have been documented between 1976 and 2014, usually in remote forested areas<br />

of central Africa.<br />

7. In 2014, a larger scale epidemic spread to urban areas of central Africa, including cities in Guinea, Sierra<br />

Leone, Liberia, and Nigeria.<br />

8. Several cases were imported from these areas to western nations, including the United States.<br />

B. Clinical presentation<br />

1. Classic presenting symptoms<br />

a. Fever<br />

b. Excessive diarrhea<br />

C. Vomiting<br />

d. Abdominal pain<br />

e. Weakness<br />

f. Headache<br />

g. Malaise<br />

2. Early symptoms can be difficult to distinguish from influenza, malaria, and other more common infections.<br />

3. Distinguishing physical examination findings<br />

a. Conjunctiva! injection (early)<br />

b. Diffuse erythematous maculopapular rash (early)<br />

c. Bruising<br />

d. Petechiae<br />

e. Mucosal bleeding<br />

f. GI bleeding/hemorrhage<br />

g. Vaginal bleeding<br />

C. Diagnostic evaluation<br />

1. Laboratory abnormalities<br />

a. Increased BUN and creatinine<br />

b. Transaminitis<br />

c. Hypocalcemia<br />

d. Lymphopenia<br />

e. Leukocytosis<br />

f. Thrombocytopenia<br />

g. Disseminated intravascular coagulation<br />

2. Reverse transcriptase PCR is the fastest, most reliable test.<br />

3. False-negatives are possible within 3 days of symptom onset, so retesting may be required if high suspicion and<br />

early clinical presentation.<br />

4. Recommend concurrent testing for alternative diagnoses such as malaria.<br />

779


SYSTEMIC INFECTIOUS DISORDERS<br />

D. Infection control measures<br />

1. Standard, contact, and droplet precautions are advised with all suspected cases.<br />

2. Providers must don and doff personal protective equipment with a trained observer for optimal infection<br />

control.<br />

3. Suspected cases should be treated in single private treatment rooms, with a private bathroom or covered<br />

bedside commode and closable room door.<br />

E. Treatment<br />

1. Respiratory and intravascular supportive measures<br />

2. Early aggressive oral and/or IV hydration<br />

3. Correction of coagulopathies<br />

4. Electrolyte repletion<br />

5. Empiric antibiotics and antimalarial drugs should be considered initially to ensure coverage of sepsis and<br />

malaria for undifferentiated patients.<br />

780


SYSTEMIC INFECTIOUS DISORDERS: PRACTICE CLINICAL SCENARIOS<br />

SYSTEMIC INFECTIOUS DISORDERS: PRACTICE CLINICAL<br />

SCENARIOS<br />

Answers immediately follow the practice clinical scenarios.<br />

Scenario A<br />

Presentation: An elderly, demented nursing-home patient presents with altered mental status from baseline<br />

(less communicative). The patient is febrile and tachycardic, with a borderline low blood pressure.<br />

What is the diagnosis?<br />

Scenario B<br />

Presentation: An avid hiker presents to the emergency department unable to close one eye or drink water<br />

because of a unilateral facial paralysis. She recalls having a rash on her thigh about a month ago that<br />

resembled a bull's eye and was not painful or pruritic.<br />

What is the diagnosis?<br />

Scenario C<br />

Presentation: A thin patient with temporal wasting and thrush with a history of IV drug abuse is brought in<br />

by concerned family for bizarre behavior. He has had a headache for weeks, and bright light bothers his<br />

eyes. Results of laboratory studies show a low absolute lymphocyte count and positive HIV serology. A CT<br />

of the brain with and without contrast excludes any intracranial ring-enhancing lesions. A lumbar puncture<br />

reveals a high opening pressure and is positive for cryptococcal antigen.<br />

What is the diagnosis?<br />

Scenario D<br />

Presentation: A patient presents with a history of HIV and medication noncompliance. She recalls that she<br />

is supposed to take TMP-SMX once per day. She complains of shortness of breath and a nonproductive<br />

cough. Her pulse oximetry is in the mid 80s.<br />

What is the diagnosis?<br />

781


SYSTEMIC INFECTIOUS DISORDERS: PRACTICE CLINICAL SCENARIOS<br />

ANSWERS TO PRACTICE CLINICAL SCENARIOS<br />

Scenario A<br />

Diagnosis: severe sepsis<br />

Diagnostic evaluation: Chest radiographs are done, as well as blood and urine cultures, urinalysis, CBC,<br />

and lactate.<br />

Management: Treatment includes early and aggressive IV fluid resuscitation, broad-spectrum antibiotics,<br />

and admission.<br />

Scenario B<br />

Diagnosis: Lyme disease<br />

Diagnostic evaluation: Diagnostic evaluation involves confirming that there is Lyme disease in the area<br />

where the patient has been hiking. Serology should be sent at this stage of the disease. An ECG should be<br />

done to screen for cardiac involvement (AV nodal block).<br />

Management: Treatment for isolated facial palsy is doxycycline for 2 weeks. If there is cardiac<br />

involvement, IV ceftriaxone is given for up to 3 weeks.<br />

Scenario C<br />

Diagnosis: cryptococcal meningitis<br />

Management: The patient is admitted for IV antifungal therapy and infectious disease consult.<br />

Scenario D<br />

Diagnosis: Pneumocystis jiroveci pneumonia<br />

Diagnostic evaluation: Chest radiograph is unremarkable; results of laboratory studies include low WBC<br />

and absolute lymphocyte counts, and an increased lactate dehydrogenase level.<br />

Management: The patient is admitted to the hospital, and supplemental oxygen is administered. Steroids<br />

are started before TMP-SMX therapy in the emergency department.<br />

782


IMMUNE SYSTEM DISORDERS<br />

IMMUNE SYSTEM DISORDERS<br />

RHEUMATOLOGIC DISORDERS .............................................................................................................................. 786<br />

Raynaud Disease ....................................................................................................................................................... 786<br />

Reiter Syndrome ....................................................................................................................................................... 786<br />

Scleroderma .............................................................................................................................................................. 787<br />

Systemic Lupus Erythematosus .................................................................................................................................. 788<br />

ALLERGIC EMERGENCIES (REACTIONS AND ANAPHYLAXIS) ................................................................................ 789<br />

Types of Allergic (Hypersensitivity) Reactions ........................................................................................................... 789<br />

Immediate Reactions (Type 1) .............................................................................................................................. 789<br />

Cytotoxic Reactions (Type II) .............................................................................................................................. 789<br />

Immune Complex-Mediated Reactions (Type 111) ................................................................................................ 789<br />

Delayed Cell-Mediated Reactions (Type IV) ........................................................................................................ 789<br />

Clinical Manifestations of Allergic (Hypersensitivity) Reactions ................................................................................ 790<br />

Urticaria ............................................................................................................................................................. 790<br />

Angioedema ....................................................................................................................................................... 791<br />

Erythema Multiforme .......................................................................................................................................... 791<br />

Stevens-Johnson Syndrome ................................................................................................................................. 792<br />

Allergic Drug Reactions ...................................................................................................................................... 792<br />

Adverse Food Reactions ..................................................................................................................................... 794<br />

Anaphylaxis ....................................................................................................................................................... 795<br />

Anaphylactoid Reactions .................................................................................................................................... 796<br />

783


IMMUNE SYSTEM DISORDERS: SELF-ASSESSMENT QUESTIONS<br />

IMMUNE DISORDERS: SELF-ASSESSMENT QUESTIONS<br />

1. The type of reaction that occurs with blood transfusions is most accurately classified as a:<br />

(a) Type I allergic reaction<br />

(b) Type II allergic reaction<br />

(c) Type Ill allergic reaction<br />

(d) Type IV allergic reaction<br />

2. Type Ill allergic reactions are mediated by:<br />

(a)<br />

lgE complexes and platelet aggregates<br />

(b) lgG or lgM<br />

(c) lgM<br />

(d) None of the above<br />

3. Which of the fol lowing statements regarding the skin lesions of erythema multiforme are true?<br />

(a) They are typically red, raised, purpura.<br />

(b) They may not associated with Stevens-Johnson syndrome.<br />

(c) They are generally very pruritic.<br />

(d) They are classically located on the palms and soles, dorsum of the hands and feet, and on extensor surfaces of<br />

the extremities.<br />

4. Which of the fol lowing statements regarding the Jarisch-Herxheimer reaction are true?<br />

(a)<br />

It is a febrile reaction to parasitic or bacterial antigens that are liberated when the organisms are destroyed.<br />

(b) It is most commonly seen in association with the treatment of encapsulated organism infections.<br />

(c) It generally starts several days after the start of antibiotic therapy.<br />

(d) It is frequently life threatening.<br />

5. Which of the following reactions does not require prior exposure?<br />

(a)<br />

Phototoxic reactions<br />

(b) Photoallergic reactions<br />

(c) Anaphylactic reactions<br />

(d) None of the above<br />

6. All of the following agents have been associated with the development of photoallergic reactions except:<br />

(a) Musk ambrette<br />

(b) Thiazide diuretics<br />

(c) Para-aminobenzoic acid<br />

(d) Phenothiazines<br />

7. A patient who was stung by a bee approximately 30 minutes before arrival presents with acute respiratory distress<br />

and hypotension. On examination, her blood pressure is 70/30 mmHg, and she is wheezing and has perioral and<br />

periorbital swelling. Her skin is cold and clammy. The nurses have already placed her on a monitor, given her<br />

supplemental oxygen, and started a large-bore IV line. The initial drug therapy of choice is:<br />

(a) Epinephrine 0.01 ml/kg (up to 0.5 ml of a 1 :1,000 solution) SC<br />

(b) Benadryl 1-2 mg/kg IV push<br />

(c) Inhaled ~-adrenergic agent such as albuterol<br />

(d) Epinephrine 0.1 ml/kg (up to 5 ml of a 1:10,000 solution) slow IV push<br />

8. Which of the following agents have been associated with anaphylactoid reactions?<br />

(a)<br />

Radiographic contrast media<br />

(b) Thiazide diuretics<br />

(c) Acetaminophen<br />

(d) Penicillin<br />

784


IMMUNE SYSTEM DISORDERS: SELF-ASSESSMENT QUESTIONS<br />

9. A 40-year-old woman presents with a 5-day history of fever, arthralgias, red and irritated eyes, and rash that has<br />

a target appearance on her palms and soles of her feet. Over the last 24 hours, she has had increasing pain in her<br />

mouth and has been unable to eat or drink. Physical examination shows bullous lesions on the skin, target lesions<br />

on the palms and soles, purulent conjunctivitis, and oral mucosa lesions and desquamation. The most appropriate<br />

management is:<br />

(a) Antipyretics, anti-inflammatories, 7-day course of acyclovir, and outpatient ophthalmology consult<br />

(b) IV fluids, steroids, analgesics, and admission for observation<br />

(c) Antipyretics, analgesics, 10-day course of doxycycline, and outpatient follow up<br />

(d) IV Vancomycin, analgesics, gentamicin ophthalmic drops, and admission for observation<br />

10. Erythema multiforme is commonly induced by which of the following?<br />

(a) ACE inhibitors<br />

(b) Lyme disease<br />

(c) Oral hypoglycemics<br />

(d) Barbiturates<br />

11. Which of the fol lowing is part of the classic triad of systemic lupus erythematosus?<br />

(a) Fever<br />

(b) Joint pain<br />

(c) Rash in a female of childbearing age<br />

(d) All of the above<br />

12. Which of the following is true about scleroderma?<br />

(a) It has no association with Raynaud phenomenon.<br />

(b) It has no relationship with Sjogren syndrome.<br />

(c) It is predominately a disease affecting the skin and rarely involves other organ systems.<br />

(d) Classic presentation includes skin tightness, induration, and pruritus.<br />

13. Which of the following is not true about Reiter syndrome?<br />

(a) Classic triad is conjunctivitis, urethritis, and arthritis.<br />

(b) Enteral involvement should not be treated with antibiotics.<br />

(c) It is associated with E coli and Clostridium difficile infections.<br />

(d) Fever, myalgias, and arthralgias can be safely treated with NSAIDs.<br />

14. An 18-year-old previously healthy woman presents with a complaint of pale and numb finger tips on her left hand<br />

after snow skiing for several hours. On physical examination, there is a well-demarcated area of pallor on the distal<br />

portion of the second, third, and fourth digits on her left hand. They are numb to the touch. The remainder of the<br />

hand is normal. The most appropriate management includes all of the following except:<br />

(a) Warm the local body part.<br />

(b) Evaluate the patient for arterial occlusion.<br />

(c) Educate on cessation of vasoconstricting agents such as nicotine.<br />

(d) Educate on avoiding precipitating factors.<br />

ANSWERS<br />

1. b 4. a 7. d 10. C 13.<br />

2. a 5. a 8. a 11 . d 14.<br />

3. d 6. b 9. b 12. d<br />

Use the pre-chapter multiple choice question worksheet (page xvi) to record and determine the percentage of correct<br />

answers for this chapter.<br />

C<br />

b<br />

785


IMMUNE SYSTEM DISORDERS<br />

RHEUMATOLOGIC DISORDERS<br />

I. RAYNAUD DISEASE<br />

A. Recurrent artery or arteriole vasospasm most commonly in the fingers and toes, usually in response to<br />

stress or cold exposure<br />

B. Clinical presentation<br />

1. Numbness and parethesias<br />

2. Pain in the affected area<br />

3. Affected areas show at least two color changes<br />

a. Pallor<br />

b. Cyanosis<br />

c. Hyperemia<br />

4. Transient lasting minutes to hours<br />

C. Management<br />

1. Warm local body part<br />

2. Discontinue vasoconstricting agents such as nicotine<br />

3. Avoid precipitating factors<br />

4. Calcium channel blockers<br />

II. REITER SYNDROME<br />

A. Reactive arthritis following enteric or venereal infections associated with human leukocyte antigen (HLA)-B27<br />

B. Associated bacteria<br />

1. Chlamydia (most common sexually transmitted infection)<br />

2. Campylobacter (most common enteric infection)<br />

3. Salmonella<br />

4. Shigella<br />

5. Yersinia<br />

6. Cyc!ospora<br />

7. Group A streptococci<br />

C. Clinical presentation-classic triad: "can't see, can't pee, can't dance with me"<br />

1. Conjunctivitis-"can't see"<br />

a. Erythema<br />

b. Burning<br />

C. Tearing<br />

d. Pain<br />

e. Photophobia<br />

2. Nongonococcal urethritis-"can't pee"<br />

a. Frequency<br />

b. Dysuria<br />

c. Urgency<br />

d. Urethral discharge<br />

3. Arthritis-"can't dance with me"<br />

a. Asymmetric arthralgia<br />

b. Joint stiffness<br />

c. Primarily involving the knees, ankles, and feet; worse with rest or inactivity<br />

4. May also have fever, myalgias, low back pain, mouth ulcers, and keratoderma blennorrhagica (patches of scaly<br />

skin on palms, soles, and scalp)<br />

786


IMMUNE SYSTEM DISORDERS<br />

D. Management<br />

1. NSAIDs<br />

2. Tetracycline for cases caused by Chlamydia<br />

3. Antibiotics are no benefit in cases with enteral causes.<br />

Ill. SCLERODERMA<br />

A. Systemic autoimmune disease characterized by skin induration and thickening accompanied by<br />

tissue fibrosis, chronic inflammatory infiltration of visceral organs, and cellular immune alterations<br />

B. Clinical presentation<br />

1. Skin<br />

a. Tightness<br />

b. lnduration<br />

c. Pruritus<br />

d. CREST is the limited cutaneous sclerosis: .C.alcinosis, Raynaud, fsophageal dysmotility, .Sclerodactyly,<br />

Ielangiectasias<br />

2. Vascular<br />

a. 95% have associated Raynaud phenomenon<br />

3. GI<br />

b. Ulcers on fingertips<br />

c. Telangiectasis<br />

a. Gastroesophageal reflux disease<br />

b. Dyspepsia<br />

c. Constipation alternating with diarrhea<br />

4. Pulmonary<br />

a. Dry cough<br />

b. Dyspnea<br />

c. Pulmonary hypertension<br />

5. Musculoskeletal<br />

a. Arthralgia<br />

b. Myalgia<br />

c. Loss of joint range of motion and joint flexion contractures<br />

d. Muscle weakness<br />

6. Cardiovascular<br />

a. Pericardia! effusion<br />

b. CHF<br />

c. Myocardial fibrosis<br />

7. ENT<br />

a. Sicca/Sjogren syndrome with dry eyes and dry mouth (due to destruction of salivary and lacrimal glands)<br />

b. Poor dentition<br />

c. Hoarseness<br />

8. Renal<br />

a. Hypertension<br />

b. Renal insufficiency<br />

9. Neurologic<br />

a. Trigeminal neuralgia<br />

b. Paresthesias and weakness<br />

c. Headache<br />

787


IMMUNE SYSTEM DISORDERS<br />

C. Management<br />

1. Most treatment focused on complications of the disease<br />

2. Skin symptoms: phototherapy<br />

3. GI symptoms: antacids, H 2<br />

-blockers and proton-pump inhibitors<br />

4. Pulmonary symptoms: cyclophosphamides<br />

5. Renal symptoms: ACE inhibitors<br />

6. Muscle symptoms: low-dose steroids<br />

IV. SYSTEMIC LUPUS ERYTHEMATOSUS<br />

A. Chronic autoimmune disease that can affect almost any organ system, making its presentation and<br />

course highly variable<br />

B. Clinical presentation<br />

1. Classic triad: fever, joint pain, and rash in a woman of childbearing age<br />

2. Constitutional: fatigue, fever, weight changes<br />

3. Musculoskeletal: arthralgia, arthropathy, myalgia, avascular necrosis<br />

4. Dermatologic: malar rash, photosensitivity, discoid lupus<br />

5. Renal: acute or chronic renal failure, acute nephritic disease<br />

6. Neuropsychiatric: seizure, psychosis, stroke<br />

7. Pulmonary: pleurisy, pleural effusion, pneumonitis, pulmonary hypertension, interstitial lung disease,<br />

pulmonary embolism<br />

8. GI: nausea, dyspepsia, abdominal pain<br />

9. Cardiac: pericarditis, myocarditis, acute Ml, tamponade<br />

10. Hematologic: leukopenia, lymphopenia, anemia, or thrombocytopenia<br />

C. Management<br />

1. Emergency department management often focuses on the complications of the disease (acute Ml, pulmonary<br />

embolism, etc).<br />

2. Antimalarials: hydroxychloroquine<br />

3. Corticosteroids<br />

4. NSAIDs<br />

5. Rheumatology consult<br />

788


IMMUNE SYSTEM DISORDERS<br />

ALLERGIC EMERGENCIES (REACTIONS AND<br />

ANAPHYLAXIS)<br />

I. TYPES OF ALLERGIC (HYPERSENSITIVITY} REACTIONS<br />

A. Immediate reactions (Type I)<br />

1. Sequence of events: initial exposure to antigen - activates T-helper lymphocyte itself or binding with protein<br />

(hapten-protein complex) - induces plasma cells to produce specific lgE antibody that binds to mast cells<br />

and basophils - subsequent reexposure to the antigen - antigen binding with lgE on cell surfaces - release<br />

of preformed chemical mediators (histamine, eosinophil chemotactic factor of anaphylaxis [ECF-A], high<br />

molecular weight neutrophil chemotactic factor [HMW-NCF], kallikreins) and spontaneously generated<br />

mediators (leukotrienes, prostaglandins, platelet-activating factor, thromboxane)<br />

2. These reactions are lgE- or lgG 4<br />

-mediated and are responsible for most anaphylactic reactions.<br />

3. Examples<br />

a. Drug-induced reactions: penicillin is the most common and the leading cause of fatal anaphylaxis (others<br />

include ASA, vancomycin, TMP-SMX, and NSAIDs)<br />

b. Hymenoptera venoms<br />

c. Foods (shellfish, nuts, egg white, soybeans, wheat, and peanuts [the most common cause of fatal food<br />

anaphylaxis])<br />

d. Environmental reactions (dust, ragweed, etc)<br />

e. Radiologic contrast material<br />

f. Food additives (MSG, nitrates/nitrites, tartrazine dyes)<br />

B. Cytotoxic reactions (Type II)<br />

1. Sequence of events: target antigen (a cellular component or a substance that cross-reacts with a cellular<br />

component) - complement activation or direct injury to lymphocytes - release of cell mediators<br />

2. These reactions involve binding of lgG or lgM to a cell-bound antigen.<br />

3. Examples<br />

a. Blood transfusion reactions<br />

b. Immune hemolytic anemias<br />

c. Thrombocytopenia (idiopathic thrombocytopenic purpura)<br />

C. Immune complex-mediated reactions (Type Ill)<br />

1. Sequence of events: soluble antigen-antibody complex - activation of complement system and platelets -<br />

lgE complexes and platelet aggregates - release of cell mediators<br />

2. Examples<br />

a. Arthus reactions<br />

(1) lntradermal injection of an antigen to which the patient has been previously sensitized - edema and<br />

erythema at the injection site<br />

(2) Can occur after administration of tetanus toxoid in the patient who has received too-frequent doses<br />

of tetanus toxoid<br />

b. Serum sickness (fever, rash, arthralgia, and lymphadenopathy)<br />

D. Delayed cell-mediated reactions (Type IV)<br />

1. Sequence of events: antigen-specific T lymphocytes migrate to the site of the antigen injection - release of<br />

cell mediators<br />

2. These reactions do not involve antibodies or complement.<br />

3. Signs/symptoms do not develop for 24-72 hours after exposure.<br />

4. Examples: tuberculin skin test, contact dermatitis<br />

5. This classification is somewhat oversimplified in that many allergic reactions consist of more than one type<br />

of immune mechanism. Because Types 1-111 involve complement activation (which can result in varying<br />

degrees of anaphylaxis), some degree of overlap does occur. However, the most severe anaphylactic reactions<br />

are primarily Type I; Type II is generally less severe, and Type Ill is the least severe. Type IV does not involve<br />

complement activation and therefore does not cause any form of anaphylaxis.<br />

789


IMMUNE SYSTEM DISORDERS<br />

II. CLINICAL MANIFESTATIONS OF ALLERGIC (HYPERSENSITIVITY)<br />

REACTIONS<br />

A. Urticaria<br />

1. A vascular reaction involving the skin with transient, pruritic wheals and welts<br />

2. Clinical presentation<br />

a. Edematous papules coalescing into plaques that can affect any area of the skin<br />

b. White, pink, or erythematous nonscaling lesions that range in size from 10 cm<br />

c. Wax and wane over 24-48 hours<br />

3. Causes<br />

a. Drug reaction (most common)<br />

(1) Penicillin (most commonly implicated) and sulfa<br />

(2) Local anesthetics<br />

(3) ASA (most commonly implicated)<br />

(4) Diuretics<br />

(5) Laxatives<br />

(6) NSAIDs<br />

(7) Morphine and codeine<br />

(8) Progesterone<br />

b. Infection<br />

(1) Infectious mononucleosis<br />

(2) Hepatitis B<br />

(3) Coxsackie virus<br />

(4) Parasitic infestations<br />

c. Environmental and physical factors<br />

(1) Heat or cold<br />

(2) Sun exposure<br />

(3) Exercise<br />

(4) Latex<br />

(5) Raw fish<br />

(6) Metals (eg, nickel)<br />

(7) Animal saliva<br />

d. Malignancies<br />

e. Hyperthyroidism<br />

f. Pregnancy<br />

g. Foods (fish, eggs, nuts, peanuts, strawberries, lobster)<br />

h. Mastocytosis (overproliferation of mast cells in tissues)<br />

4. Management<br />

a. Symptomatic in most cases; however, urticaria alone has been known to progress to full-blown anaphylaxis.<br />

b. Diphenhydramine (an H 1<br />

-blocker) is usually the drug of choice for these reactions. Other excellent choices<br />

include:<br />

(1) Hydroxyzine<br />

(2) Chlorpheniramine (pregnancy)<br />

(3) Cyproheptadine (cold urticaria)<br />

(4) Long-acting nonsedating antihistamine: fexofenadine, loratadine, cetirizine<br />

c. The addition of an H 2<br />

-blocker (eg, cimetidine) can provide additional benefit in 10%-15% patients who<br />

have an incomplete response to diphenhydramine alone.<br />

d. Epinephrine and/or steroid may be required in resistant cases.<br />

(1) Moderate to severe cases - methylprednisolone 125 mg IV<br />

(2) Severe, life-threatening cases - add IV epinephrine 0.1 mg/kg IV in a 1 :10,000 solution<br />

790


IMMUNE SYSTEM DISORDERS<br />

e. Patients who are discharged should be:<br />

B. Angioedema<br />

(1) Continued on oral antihistamines for 3 days<br />

(2) Continued on steroids if these drugs were required during emergency department treatment<br />

(3) Advised to avoid offending agent if known; if unknown, advised to follow up with a dermatologist to<br />

identify the offending agent<br />

1. Edema due to capillary dilation of the deeper layers of skin caused by a C1 esterase inhibitor deficiency or<br />

defect that results in decreased levels of C4<br />

2. Clinical presentation<br />

a. Painless, nonpruritic, nonpitting edema of skin<br />

b. Most commonly affects tongue, lips, and face<br />

c. Hands and feet<br />

d. Scrotum and foreskin<br />

e. Can affect abdominal organs and upper airway in more severe cases<br />

3. Drug-induced angioedema<br />

a. ACE inhibitors (a common cause and can occur any time in course of use, even a year later); NSAIDs,<br />

aspirin, contrast media<br />

b. Management<br />

(1) Be ready to establish a definitive airway (fiberoptic intubation procedure of choice).<br />

(2) Symptomatic (similar to that for urticaria) in most cases, but progression to anaphylaxis can occur.<br />

(Angioedema is not lgE associated; antihistamines and steroids have not clearly been shown to be of<br />

benefit.)<br />

(3) Administration of fresh frozen plasma to replace C1 esterase inhibitor in severe cases<br />

(4) Administration of racemic epinephrine should be considered if there is evidence of impending upper<br />

airway obstruction.<br />

4. Hereditary angioedema<br />

a. Edema of the face, airway, or extremities and abdominal pain with nausea, vomiting, and diarrhea<br />

is characteristic; hereditary angioedema is caused by C1 esterase inhibitor deficiency and is usually<br />

precipitated by stress or trauma.<br />

b. Management<br />

(1) Frozen plasma (containing C1 esterase inhibitor)<br />

(2) High-dose epinephrine may also be effective. However, life-threatening acute attacks do not respond<br />

to standard doses of epinephrine or to antihistamines or steroids.<br />

(3) The attenuated androgens (danazol and stanozolol) may help for both acute and chronic hereditary<br />

angioedema.<br />

(4) There are also several new FDA-approved drugs for treatment of hereditary angioedema, including<br />

ecallantide, Cinryze®, and Berinert®, which have shown good efficacy in treating acute episodes.<br />

C. Erythema multiforme<br />

1. A more severe variant of an urticaria! reaction causing a limited hypersensitivity reaction in the skin<br />

2. Clinical presentation<br />

a. Typical lesions are red, raised, and multishaped with clear centers (iris or target lesions) that may become<br />

bullous. These nonpruritic lesions have a symmetric distribution and are typically located on the:<br />

(1) Palms and soles (characteristic)<br />

(2) Dorsum of the hands/wrist (usual location of target lesions) and feet<br />

(3) Extensor surfaces of the extremities<br />

b. Often following a course offever, myalgias, headache, and malaise<br />

3. Etiology<br />

a. Drug induced is most common: mnemonic "SOAPS" for Sulfa, Oral hypoglycemic, Anticonvulsants,<br />

Penicillin, NSAIDs<br />

b. Infectious agents: herpes simplex virus, Mycoplasma, tuberculosis<br />

c. Malignancy: lymphoma<br />

791


IMMUNE SYSTEM DISORDERS<br />

4. Management<br />

a. Self-limited<br />

b. Treat the underlying condition and remove the offending agent.<br />

c. Acyclovir for cases involving herpes simplex virus<br />

D. Stevens-Johnson syndrome<br />

1. A severe, potentially fatal form of erythema multiforme characterized by bullae, mucous membrane lesions,<br />

and multisystem involvement<br />

2. Clinical presentation<br />

a. Often preceded by a prodrome of malaise, fever, arthralgias, and myalgias<br />

b. Purulent conjunctivitis may be so severe that the eyes are swollen shut.<br />

c. Mucous membrane breakdown and systemic involvement<br />

3. Precipitating factors<br />

a. Long-acting sulfonamides (eg, TMP-SMX)<br />

b. Herpes simplex virus and Mycoplasma infection<br />

c. Drugs: typically antibiotics and anticonvulsants<br />

d. Malignancy<br />

4. Management<br />

a. Without systemic manifestation and mucous membrane involvement, it can be outpatient with steroid bursts.<br />

b. Patients with extensive disease need ICU management, fluids and electrolyte replacement, and<br />

management of infectious and thermoregulatory issues.<br />

E. Allergic drug reactions<br />

1. Cutaneous manifestations<br />

a. Urticaria with or without angioedema<br />

b. Erythema multiforme (or nodosum)<br />

C. Anaphylaxis<br />

d. Maculopapular eruptions<br />

e. Pruritus with or without a rash<br />

f. Contact dermatitis<br />

g. Photodermatitis<br />

h. Purpura<br />

I. Fixed drug eruption<br />

j. Exfoliative dermatitis<br />

k. Stevens-Johnson syndrome<br />

I. Toxic epidermal necrolysis<br />

2. Systemic manifestations<br />

a. Serum sickness<br />

b. Interstitial nephritis and glomerulonephritis<br />

C. Vasculitis<br />

d. Arthralgias and arthritis<br />

3. Drug reactions associated with cutaneous and systemic manifestations<br />

a. Serum sickness<br />

(1) Clinical presentation<br />

(a)<br />

Urticaria<br />

(b) Fever and lymphadenopathy<br />

(c) Arthralgia and arthritis<br />

(d) Glomerulonephritis<br />

(2) Drug etiology<br />

(a)<br />

Penicillin and sulfa<br />

(b) Barbiturates<br />

(c) Thiazide diuretics<br />

(3) Treatment<br />

(a)<br />

Remove offending agent.<br />

792


IMMUNE SYSTEM DISORDERS<br />

(b) Anti-inflammatories<br />

(c) Anithistamines<br />

b. Acute interstitial nephritis<br />

(1) Clinical presentation<br />

(a) Fever and skin rash<br />

(b) Proteinuria, hematuria, and leukocyturia<br />

(c) Eosinophilia and eosinophiluria<br />

(d) Azotemia and oliguria<br />

(2) Drug etiology<br />

(a)<br />

Penicillins (especially methicillin)<br />

(b) Cephalosporins<br />

(c) NSAIDs (especially fenoprofen)<br />

(d) Cimetidine<br />

(e) Sulfonamides<br />

(f)<br />

Phenytoin<br />

(g) Thiazide diuretics<br />

(h) Vancomycin<br />

(i)<br />

(j)<br />

Diltiazem<br />

Omeprazole<br />

(k) Snake antivenin<br />

(I)<br />

Streptomycin<br />

(3) Management<br />

(a) Supportive care measures with fluid and electrolyte maintenance<br />

(b) Symptomatic relief of fever and systemic symptoms<br />

(c) Avoid nephrotoxic drugs<br />

(d) Corticosteroid therapy<br />

c. Hypersensitivity vasculitis<br />

(1) Clinical presentation<br />

(a) Palpable purpura on lower extremities<br />

(b) Vasculitis involving the CNS, GI tract, lungs, and kidneys<br />

(2) Drug etiology<br />

(a) Penicillin and sulfa<br />

(b) Phenytoin<br />

(c) Cephalosporins<br />

(d) Thiazide diuretics<br />

(e) Allopurinol<br />

(3) Associated infections (occur in some patients)<br />

(a) Chronic bacteremia<br />

(b) Hepatitis B and C<br />

(c) HIV<br />

(4) Management<br />

(a) Remove offending agent.<br />

(b) Systemic corticosteroid therapy<br />

4. Other allergic drug reactions<br />

a. Jarisch-Herxheimer reaction<br />

(1) A febrile reaction to parasitic or bacterial antigens that are liberated when the organisms are<br />

destroyed by antibiotic therapy<br />

(2) Most commonly occurs in association with treatment of spirochete infections (syphilis, Lyme disease,<br />

relapsing tick-borne fever, and leptospirosis)<br />

(3) Generally occurs 2-12 hours after the start of antibiotic therapy (especially penicillin or a<br />

tetracycline) and lasts


IMMUNE SYSTEM DISORDERS<br />

b. Maculopapular eruptions<br />

(1) Can be caused by any drug, but ampicillin is an especially common offender<br />

(2) If the causative agent is not discontinued, erythroderma or exfoliative dermatitis may develop.<br />

c. Contact dermatitis<br />

(1) Application of any topical agent (including cosmetic creams and lotions) can cause a cell-mediated<br />

reaction (Type IV) characterized by redness, itching, maculopapular, or vesicular eruptions.<br />

(2) Treatment consists of topical steroids, antihistamines, and cold wet dressings soaked in Burrow solution.<br />

d. Fixed drug eruption<br />

(1) A rash that recurs at the same site each time the offending drug is used<br />

(2) Lesions are often pigmented (violaceous or dusky red), round or oval in shape, and can be quite pruritic.<br />

(3) Most common cause is phenolphthalein in OTC laxatives. Other offenders include:<br />

(a) Aspirin<br />

(b) Sulfa<br />

(c) Tetracycline<br />

(d) Barbiturates<br />

(e)<br />

NSAIDs (especially naproxen and tolmetin)<br />

(4) Management consists of discontinuing the offending agent, steroids, and antihistamines.<br />

e. Photosensitive reactions<br />

(1) Adverse reactions to a combination of a drug (systemic or topically applied) and ultraviolet light<br />

(2) Lesions are restricted to sun-exposed areas of the skin (face, nuchal area of the neck, dorsal aspects of<br />

the upper extremities).<br />

(3) Two types of photosensitive reactions: phototoxic (most common) and photoallergic<br />

(a) A photoallergic reaction is a delayed hypersensitivity reaction. Therefore, it occurs only in<br />

sensitized individuals. The eruption is generally intensely pruritic and resembles that seen with<br />

contact dermatitis. Topical agents are the most common sensitizers, especially those containing<br />

one of the following:<br />

i. Para-aminobenzoic acid<br />

ii. Musk ambrette (colognes and perfumes)<br />

iii. Halogenated salicylanilides (in soaps, cosmetics)<br />

iv. Phenothiazines<br />

(b) A phototoxic reaction does not involve immunologic mechanisms and can, therefore, occur on<br />

first exposure (drug and adequate sunlight). These reactions appear as exaggerated sunburns;<br />

associated pruritus is mild or absent. Commonly implicated drugs include:<br />

i. Tetracyclines (especially doxycycline)<br />

ii. Phenothiazines (especially chlorpromazine)<br />

iii. Sulfonamides and sulfonylureas<br />

iv. Thiazide diuretics<br />

v. Chlordiazepoxide<br />

vi. NSAIDs (especially piroxicam and naproxen)<br />

(4) Management consists of discontinuing the offending agent, avoiding sun exposure, and steroids.<br />

F. Adverse food reactions - direct histamine release<br />

1. "Chinese restaurant syndrome": the most common because the food contains most, if not all, of the potential<br />

causes (allergens, preservatives, color additives, flavor enhancers, and toxins)<br />

2. Su lfites (preservatives)<br />

a. Can cause flushing, bronchospasm, urticaria, and hypotension within minutes of ingestion; nausea,<br />

vomiting, diarrhea, and abdominal cramps can also occur.<br />

b. Foods containing sulfite: packaged salads, shrimp, dried fruit, gelatin, pickles, sausages, cheeses, wine, and<br />

fruit juices<br />

c. 5%-10% of the asthmatic population is sensitive to sulfites, which may produce anaphylactic reactions in<br />

these patients.<br />

3. Tartrazine dye (yellow dye #5, a food and drug color additive) and parabeus (a pharmaceutical additive) can<br />

cause urticaria, angioedema, acute bronchospasm, or anaphylaxis.<br />

794


IMMUNE SYSTEM DISORDERS<br />

4. Monosodium glutamate (flavor enhancer) can cause flushing, chest pain, wheezing, facial pressure and<br />

burning, a burning sensation at the back of the neck, dizziness, paresthesias, headache, palpitations, weakness,<br />

nausea, and vomiting.<br />

5. Scombroidosis (food toxins) is due to spoiled salt water fish and can cause flushing (resembles a sunburn),<br />

headache, urticaria, itching, abdominal cramping, and diarrhea; sometimes a metallic, bitter, or "peppery" taste<br />

is noted at the time of ingestion.<br />

G. Anaphylaxis<br />

1. Pathophysiology<br />

a. A severe systemic allergic reaction to an antigen that is precipitated by the abrupt release of chemical<br />

mediators in a previously sensitized patient and involving two or more organ systems. Prior exposure to<br />

the antigen is necessary for anaphylactic shock to occur.<br />

b. Antigens that can cause anaphylaxis include foods, drugs, vaccines, blood products, pollens, and insect<br />

venoms; penicillin remains the leading cause of fatal anaphylaxis.<br />

c. Patients on ~-blockers may have a higher risk of anaphylaxis and of a more severe reaction due to<br />

interference with traditional therapies.<br />

2. Clinical presentation<br />

a. Early signs of impending anaphylaxis<br />

(1) Nasal itching or stuffiness<br />

(2) A "lump" in the throat (laryngeal edema) or hoarseness<br />

(3) lightheadedness and syncope<br />

(4) Chest pain, shortness of breath, and tachypnea<br />

(5) Skin complaints: warmth and tingling of the face (especially the mouth), upper chest, and palms or<br />

soles are usually the first clinical manifestations of anaphylaxis.<br />

(6) GI complaints: nausea, vomiting, diarrhea with tenesmus or crampy abdominal pain<br />

(7) An "aura" of impending disaster (may occur in patients with a prior history of anaphylaxis)<br />

b. Ful I-blown anaphylaxis<br />

(1) Angioedema of the tongue, pharynx, and larynx that can lead quickly to upper airway obstruction<br />

(2) Hypotension, tachycardia (or other dysrhythmias), altered sensorium, dizziness, wheezing, and<br />

cyanosis that can lead quickly to cardiopulmonary failure. Coughing is an ominous sign that frequently<br />

heralds the onset of pulmonary edema.<br />

(3) The skin may or may not show the classic wheal-and-flare reaction. If the patient is severely<br />

hypotensive, a skin reaction may be difficult to see because of poor perfusion.<br />

3. Management<br />

a. Supportive measures<br />

(1) Supplemental high-flow oxygen and cardiac monitoring<br />

(2) Normal saline or lactated Ringer's IV (at least 2 l) for circulatory support<br />

(3) laryngospasm may necessitate oral intubation (the procedure of choice) or cricothyrotomy; racemic<br />

epinephrine (2.25% solution 5 ml in 2.5 ml normal saline) may be used as a temporizing measure<br />

while setting up for definitive airway management. Sedation and paralysis should be used with<br />

caution, because a distorted airway may preclude intubation after paralysis.<br />

b. Parenteral drug therapy<br />

(1) Epinephrine (drug of choice)<br />

(a) Mild anaphylaxis (urticaria, rhinitis, conjunctivitis, mild bronchospasm)-"" 0.3-0.5 ml 1 :1,000<br />

solution IM<br />

(b) Moderate anaphylaxis (generalized urticaria, angioedema, early laryngeal edema, hypotension<br />

[blood pressure >80 mmHg])-"" 0.3-0.5 ml 1 :1,000 solution IM. May repeat every 5-20 minutes<br />

to a total of 3 doses as needed.<br />

Studies have shown that IM administration in the lateral thigh achieves a rapid rise of serum<br />

levels of epinephrine, much faster than administration IM in the deltoid or SC in the arm.<br />

(c) Severe anaphylaxis (laryngeal edema, respiratory failure, shock)-"" 1-5 ml 1:10,000 solution IV<br />

over 10 minutes. If no improvement, start epinephrine drip as follows: mix 1 ml 1 :1,000 solution<br />

in 250 ml D5W (4 mcg/ml); run at 1-4 mcg/min. Patients on ~-blockers may be refractory to<br />

standard dosages of epinephrine; administration of IV glucagon may allow the epinephrine to<br />

work.<br />

795


IMMUNE SYSTEM DISORDERS<br />

(2) Diphenhydramine<br />

(a) Mild anaphylaxis: 25-50 mg orally or IM<br />

(b) Moderate anaphylaxis: 50-100 mg IM or IV<br />

(c) Severe anaphylaxis: 50-100 mg IV over 3 minutes, may be repeated every 4-6 hours as needed<br />

(3) Cimetidine or other H 2<br />

-blockers should be administered IV to those patients whose symptoms persist<br />

despite the above therapy. However, if the patient is on a r-blocker, cimetidine should be avoided<br />

because it may prolong the activity of the r-blocker.<br />

(4) Aerosolized albuterol 0.5 ml (of a 0.5% solution) in 2.5 ml normal saline every 20 minutes as needed<br />

should be used to treat persistent bronchospasm. Aerosolized ipratropium with albuterol is also<br />

effective; dosage is 0.5 mg in 2.5 ml normal saline every 20 minutes.<br />

(5) Corticosteroids<br />

(a)<br />

Potentiate the effects of epinephrine and decrease capillary permeability, but these effects are not<br />

immediate.<br />

(b) Initial dose of methylprednisolone is 125-250 mg IV; may be repeated every 6 hours.<br />

(6) Glucagon (I mg IV every 6 minutes as needed) may be beneficial for patients on a r-blocker who<br />

have persistent hypotension, as well as for other patients in whom IV epinephrine is relatively<br />

contraindicated (eg, pregnant patients, those >35 years old, and those with coronary artery disease) or<br />

who are epinephrine-resistant.<br />

(7) Heliox (helium oxygen) may be administered to patients with resistant upper airway obstruction or<br />

angioedema.<br />

c. Disposition<br />

(1) These patients should generally be admitted for observation and repeated doses of antihistamines and<br />

steroids; this includes all patients on r-blockers in whom "rebound" can occur once the therapeutic<br />

medication effects have dissipated, as well as all other patients whose symptoms were resistant to<br />

epinephrine and diphenhydramine therapy.<br />

(2) If discharged, these patients should be continued on oral antihistamines and steroids for 3 days and<br />

given a prescription for an epinephrine injection kit for initial self-treatment in the event of reexposure.<br />

(3) Recommended criteria for discharge<br />

H. Anaphylactoid reactions<br />

(a) Mild anaphylaxis (no hypotension or signs of upper airway obstruction)<br />

(b) Rapid and complete response to emergency department therapy<br />

(c) 6-hour observation without symptom recurrence<br />

(d) Safe discharge to the care of a responsible adult with instructions to return immediately if<br />

symptoms recur<br />

1. Anaphylactoid reactions resemble anaphylactic reactions but do not require prior exposure; they are not<br />

immunologically mediated. The final pathway is identical to that of anaphylaxis but does not require initial<br />

sensitizing to the antigen.<br />

2. Commonly implicated agents<br />

a. Radiographic contrast media (most common)<br />

b. Aspirin<br />

c. NSAIDs (especially the pyrazole derivatives)<br />

d. Codeine<br />

3. Treatment is the same as that for anaphylaxis.<br />

796


IMMUNE SYSTEM DISORDERS: PRACTICE CLINICAL SCENARIOS<br />

IMMUNE DISORDERS: PRACTICE CLINICAL SCENARIOS<br />

Answers immediately follow the practice clinical scenarios.<br />

Scenario A<br />

Presentation: A patient presents several days after an uncomplicated illness on TMP-SMX. She has fever,<br />

arthralgias, red and irritated eyes, a rash that has a target appearance on her palms and soles of her feet,<br />

and oral pain and lesions.<br />

What is the diagnosis?<br />

Scenario B<br />

Presentation: A patient presents with an abrupt onset of swelling of the upper lip and tongue. He denies<br />

any difficulty swallowing and shortness of breath. He has no fever, pain, or itching and no history of this in<br />

the past. He has a history of hypertension for which he takes a "blood pressure medication" but no other<br />

medical problems or allergies.<br />

Physical examination: Heart rate is 86 beats per minute, blood pressure is 124/84 mmHg, respiratory<br />

rate is 16 breaths per minute, oxygen saturation is 99%, and temperature is 98.6°F (3 7°(). Physical<br />

examination shows edema and swelling in the upper lip and tongue, which seems to be more notable on<br />

the left half of the tongue. The patient is swallowing without difficulties and has a normal neck and breath<br />

sounds. The remainder of the dermatologic examination is normal.<br />

What is the diagnosis?<br />

Scenario C<br />

Presentation: A patient presents with a gradual onset of redness, burning, and pain in both eyes. On<br />

questioning, patient also notes that he has been suffering from a significant bout of diarrhea and significant<br />

pain on urination. He denies any chemical exposure to the eyes and does not recall any injuries. He does<br />

not wear contacts or glasses and denies any changes in his ability to see or read.<br />

Physical examination: Heart rate is 104 beats per minute, blood pressure is 118/78 mm Hg, respiratory<br />

rate is 16 breaths per minute, oxygen saturation is 99%, and temperature is 101 °F (38.3 °C). Both eyes are<br />

erythematous, tearing, and photophobic. Heart and lungs are normal, as is the abdominal examination.<br />

The patient seems to have diffuse joint discomfort with range of motion. The remainder of examination<br />

is normal.<br />

What is the diagnosis?<br />

Scenario D<br />

Presentation: A patient presents after an abrupt onset of facial flushing, hives, itching, and wheezing while<br />

having dinner in "Chinatown." While in the emergency department, he also develops nausea, vomiting,<br />

and diffuse abdominal pain.<br />

Physical examination: Heart rate is 110 beats per minute, blood pressure is 90/50 mm Hg, respiratory rate<br />

is 18 breaths per minute, oxygen saturation is 99%, and temperature is 98.6°F (37°(). The patient's cheeks<br />

are flushed, and there is a diffuse urticaria! rash on his neck and trunk. Faint end expiratory wheezes are<br />

noted on chest auscultation, and the abdomen is soft without peritoneal signs.<br />

What is the diagnosis?<br />

797


IMMUNE SYSTEM DISORDERS: PRACTICE CLINICAL SCENARIOS<br />

Scenario E<br />

Presentation: A 28-year-old woman presents with fatigue, dyspnea, chest pain, fever, and diffuse arthralgias<br />

that have become progressively worse over the last 2 months.<br />

Physical examination: The patient is febrile and has a malar rash. On chest auscultation, there is a cardiac<br />

friction rub and decreased breath sounds on the left side. Heart rate is 115 beats per minute, blood<br />

pressure is 110/65 mm Hg, respiratory rate is 22 breaths per minute, and oxygen saturation is 93%.<br />

What is the diagnosis?<br />

Scenario F<br />

Presentation: The patient complains of numbness and parethesias in several of her fingertips. The affected<br />

areas become quite painful and seem to be associated with cold exposure.<br />

Physical examination: Several of her fingertips are blanched at the tips and well demarcated. The rest of<br />

the examination is normal<br />

What is the diagnosis?<br />

Scenario G<br />

Presentation: The patient presents with dry mouth, irritated eyes, and skin feeling tight. He has<br />

occasionally had difficulties swallowing his food and reflux symptoms. He also notes that he had a recent<br />

diagnosis of trigeminal neuralgia.<br />

Physical examination: Vital signs are normal. Some ulcerations are noted on several fingertips. The oral<br />

mucosa is dry, and the patient's voice is hoarse. Eyes are injected and dry. Some diffuse arthralgias with<br />

joint movement are also noted.<br />

What is the diagnosis?<br />

Scenario H<br />

Presentation: The patient presents with a rash several days after starting glipizide for diabetes. He notes no<br />

associated pain or pruritus with the rash. He has had associated fever, myalgias, and headache.<br />

Physical examination: Vital signs are normal. The skin shows red, raised patches with central clearing on<br />

his arms, hands, palms, and soles. The remainder of the examination is normal.<br />

What is the diagnosis?<br />

Scenario I<br />

Presentation: The patient presents with a fever. He was seen at an urgent care center earlier today and was<br />

diagnosed with Lyme disease and started on tetracycline. He denies any other symptoms beyond the joint<br />

pain and rash that prompted him to go to the urgent care. He has not has any fever before this point.<br />

Physical examination: Heart rate is 95 beats per minute, blood pressure is 120/76 mmHg, oxygen<br />

saturation is 98%, respiratory rate is 16 breaths per minute, temperature is 101 °F. He has a classic<br />

erythema migrans rash and diffuse arthralgias consistent with his diagnosis of Lyme disease. The remainder<br />

of the examination is normal.<br />

What is the diagnosis?<br />

798


IMMUNE SYSTEM DISORDERS: PRACTICE CLINICAL SCENARIOS<br />

ScenarioJ<br />

Presentation: An 18-year-old woman presents after being stung by a wasp at a family picnic. Very soon<br />

after the sting, she felt her skin itching and a lump in her throat, closely followed by facial flushing,<br />

shortness of breath, nausea, and vomiting. Her family brought her to the emergency department<br />

immediately and shortly after arriving at the triage area she has a syncopal episode. The family notes that<br />

she has been stung before last summer but never had any symptoms like this.<br />

Physical examination: Heart rate is 115 beats per minute, blood pressure is 80/42 mmHg, oxygen<br />

saturation is 98%, respiratory rate is 26 breaths per minute, and temperature is 98.6°F. The patient is in<br />

obvious respiratory distress with wheezing noted in all lung fields. She is diaphoretic and has a generalized<br />

swelling of the tongue and face.<br />

What is the diagnosis?<br />

799


IMMUNE SYSTEM DISORDERS: PRACTICE CLINICAL SCENARIOS<br />

ANSWERS TO PRACTICE CLINICAL SCENARIOS<br />

Scenario A<br />

Diagnosis: Stevens-Johnson syndrome<br />

Diagnostic evaluation: Physical examination findings are consistent with erythema multiforme (target<br />

lesions) and associated bullae, mucous membrane lesions and breakdown, and multisystem involvement.<br />

The patient may also have a history of herpes simplex virus or Mycoplasma infection. Diagnosis is based<br />

on physical examination with the consistent clinical presentation.<br />

Management: These patients should be admitted to the ICU for fluids and electrolyte therapy, and<br />

management of infectious and thermoregulatory issues.<br />

Scenario B<br />

Diagnosis: angioedema<br />

Key facts: The patient may also have a family history of such presentation, which would differentiate the<br />

drug-induced from the hereditary form of angioedema. The patient would not respond well to treatment<br />

with steroids and antihistamines and may develop significant airway issues. In severe cases, epinephrine<br />

and fresh frozen plasma may be required.<br />

Scenario C<br />

Diagnosis: Reiter syndrome<br />

Diagnostic evaluation: The patient may also have a history of urethral discharge, a recently treated<br />

sexually transmitted infection, or bloody diarrhea representing possible pathogens that may be causing this<br />

syndrome. This patient is presenting with the triad of conjunctivitis, urethritis, and arthritis.<br />

Management: In this particular case, an enteral bacterial cause is likely, and the patient would not benefit<br />

from antibiotic therapy. Treatment includes NSAIDs and other antipyretics.<br />

Scenario D<br />

Diagnosis: adverse food reaction<br />

Diagnostic evaluation: The patient may also have a history of recently consuming wine, cheese, or other<br />

foods that may contain high levels of sulfites. Other possibilities include heavy doses of food dyes, MSG,<br />

or fish that potentially could be spoiled.<br />

Management: This direct histamine release reaction due to ingestion of a particular food should be treated<br />

with antihistamines and other supportive measures.<br />

Scenario E<br />

Diagnosis: systemic lupus erythematosus<br />

Key facts: The patient could have numerous other presentations suggesting acute emergent complications<br />

of the disease involving almost any organ system. This scenario suggests potential pericarditis, pleural<br />

effusion, cardiac tamponade, pulmonary embolism, pneumonia, or sepsis. Diagnosis would likely not be<br />

formally made in the emergency department but focus on the acute management of the complications<br />

of the disease, followed by managing the disease itself with hydroxychloroquine, steroids, NSAIDs, and<br />

rheumatology consult.<br />

800


IMMUNE SYSTEM DISORDERS: PRACTICE CLINICAL SCENARIOS<br />

Scenario F<br />

Diagnosis: Raynaud disease<br />

Diagnostic evaluation: This patient is experiencing the recurrent artery or arteriole vasospasm of Raynaud<br />

disease, which occurs most commonly in the fingers and toes and usually in response to stress or cold<br />

exposure. The affected areas show at least two color changes (pallor, cyanosis, or hyperemia), and spasm is<br />

transient lasting minutes to hours.<br />

Management: Local body parts should be warmed and vasoconstricting agents (such as nicotine)<br />

discontinued. Precipitating factors should be avoided. Calcium channel blockers may be used in extreme<br />

cases.<br />

Scenario G<br />

Diagnosis: scleroderma<br />

Diagnostic evaluation: This patient is expressing some of the many symptoms associated with a systemic<br />

autoimmune disease characterized by skin induration and thickening accompanied by tissue fibrosis,<br />

chronic inflammatory infiltration of visceral organs and the salivary and lacrimal glands, and cellular<br />

immune alterations.<br />

Management: Treatment is focused on managing complications of the disease.<br />

Scenario H<br />

Diagnosis: erythema multiforme<br />

Diagnostic evaluation: This patient is experiencing a classic rash of erythema multiforme that is related<br />

to the glipizide he just started. Remember the mnemonic "SOAPS" (for Sulfa, Oral hypoglycemic,<br />

Anticonvulsants, Penicillin, NSAIDs). This is a more severe variant of an urticaria! reaction causing a<br />

limited hypersensitivity reaction in the skin. Attention should be given to examining the patient for signs of<br />

Stevens-Johnson syndrome, which this reaction can progress to.<br />

Management: This process is self-limiting, and management should focus on treating the underlying<br />

condition and removing the offending agent. Acyclovir is used in cases involving herpes simplex virus.<br />

Scenario I<br />

Diagnosis: Jarisch-Herxheimer reaction<br />

Diagnostic evaluation: This patient has been treated appropriately for a Borrelia infection. He is now<br />

experiencing a febrile reaction to the parasitic or bacterial antigens that are liberated when the organisms<br />

are destroyed by antibiotic therapy; this typically occurs 2-12 hours after antibiotic therapy has started.<br />

Management: Management consists of supportive care, antipyretics, and occasionally corticosteroids.<br />

ScenarioJ<br />

Diagnosis: anaphylaxis<br />

Diagnostic evaluation: This patient has a history of a previous exposure to insect venom, which is<br />

necessary for anaphylactic shock to occur. The severe systemic allergic reaction to an antigen is<br />

precipitated by the abrupt release of chemical mediators. In this patient, anaphylaxis is categorized as<br />

severe because of the presence of laryngeal edema, respiratory failure, and shock.<br />

Management: This patient is in severe anaphylaxis and needs epinephrine. She would likely also benefit<br />

from concurrent administration of diphenhydramine, cimetidine, and corticosteroids. Aerosolized albuterol<br />

would be beneficial because of the respiratory symptoms.<br />

801


802<br />

NOTES


RENAL AND UROLOGIC DISORDERS<br />

RENAL AND UROLOGIC DISORDERS<br />

Acute Kidney Injury/Failure ...................................................................................................................................... 806<br />

Chronic Kidney Disease ............................................................................................................................................ 811<br />

Hemodialysis-Related Problems ................................................................................................................................ 812<br />

Complications Associated with Kidney Transplantation ............................................................................................ 813<br />

Urinary Tract Infections ............................................................................................................................................ 814<br />

Nephrolithiasis .......................................................................................................................................................... 816<br />

Male Genital Problems .............................................................................................................................................. 818<br />

Urethritis ............................................................................................................................................................ 818<br />

Orchitis .............................................................................................................................................................. 819<br />

Acute Bacterial Prostatitis ................................................................................................................................... 819<br />

Genital Ulcers .................................................................................................................................................... 819<br />

Fournier Gangrene ............................................................................................................................................. 821<br />

Balanoposthitis ................................................................................................................................................... 821<br />

Phimosis ............................................................................................................................................................. 821<br />

Paraphimosis ...................................................................................................................................................... 822<br />

Constriction Injuries to the Penis/Tourniquet Syndrome ...................................................................................... 822<br />

Fracture of the Penis ........................................................................................................................................... 822<br />

Peyronie Disease ................................................................................................................................................ 822<br />

Carcinoma ......................................................................................................................................................... 822<br />

Priapism ............................................................................................................................................................. 822<br />

Testicular Torsion ................................................................................................................................................ 823<br />

Torsion of Appendices Epididymis and Testis ...................................................................................................... 824<br />

Epididymitis ....................................................................................................................................................... 824<br />

Urethral Stricture ................................................................................................................................................ 825<br />

Urethral Foreign Bodies ...................................................................................................................................... 825<br />

Urinary Retention ............................................................................................................................................... 825<br />

803


RENAL AND UROLOGIC DISORDERS: SELF-ASSESSMENT QUESTIONS<br />

RENAL AND UROLOGIC DISORDERS: SELF-ASSESSMENT QUESTIONS<br />

1. A patient presents with acute kidney injury. Urinalysis reveals a specific gravity of 1.025 and occasional hyaline<br />

casts but is otherwise normal. The urine sodium is


RENAL AND UROLOGIC DISORDERS: SELF-ASSESSMENT QUESTIONS<br />

9. A healthy 28-year-old man presents with rectal itching and a discharge. When specifically asked, he states that his<br />

sexual preference is male and that he practices anal receptive intercourse. Examination reveals a purulent discharge<br />

and inflammation. The most appropriate initial therapy is:<br />

(a) Ceftriaxone 250 mg IM<br />

(b) Doxycycline 100 mg orally bid x 10 days<br />

(c) Ceftriaxone 250 mg IM plus doxycycline 100 mg orally bid x 7 days<br />

(d) Erythromycin base 500 mg orally qid x 7 days<br />

10. All of the following drug classes are typically associated with urinary retention except:<br />

(a) Sympatholytics<br />

(b) a-Adrenergic stimulants<br />

(c) Cyclic antidepressants<br />

(d) Antihistamines<br />

11. An elderly man from a nursing home is sent to the emergency department for evaluation of abdominal distention<br />

and discomfort. Examination reveals a markedly distended bladder. A Foley catheter is passed and returns 2,000 ml<br />

initially and 900 ml/hr over the next 3 hours. The most appropriate management is:<br />

(a)<br />

Discharge to the nursing home with the catheter in place<br />

(b) Removal of the catheter and discharge to the nursing home<br />

(c) Admission to the hospital for IV fluid resuscitation and correction of electrolyte imbalance<br />

(d) None of the above<br />

ANSWERS<br />

1 . a 7. C<br />

2. C 8. d<br />

3. d 9. C<br />

4. C 10. a<br />

5. C 11. C<br />

6. b<br />

Use the pre-chapter multiple choice question worksheet (page xvi) to record and determine the percentage of correct<br />

answers for this chapter.<br />

805


RENAL AND UROLOGIC DISORDERS<br />

I. ACUTE KIDNEY INJURY /FAILURE<br />

A. Causes of acute kidney injury, defined as an abrupt decline in glomerular filtration rate (GFR) are primarily<br />

prerenal causes and acute tubular necrosis, accounting for 65%-75% of cases. All causes include:<br />

1. Prerenal<br />

a. Occurs when normal kidneys are hypoperfused<br />

(1) These patients are oliguric (urine volume


RENAL AND UROLOGIC DISORDERS<br />

(3) Urethral obstruction<br />

3. Renal intrinsic<br />

(a) Strictures<br />

(b) Phimosis (uncommon cause of azotemia)<br />

(c) Meatal stenosis (uncommon cause of azotemia)<br />

(d) Posterior urethral valves (a common cause in children)<br />

a. Occurs when there is pathology of the kidney or renal tubule<br />

(1) Signs and symptoms (including urine flow) depend on the site and nature of the disorder. The urine<br />

sediment is abnormal; exact findings vary with the underlying cause of the renal failure.<br />

(a) Acute tubular necrosis: renal tubular casts and muddy brown granular casts<br />

(b) Acute interstitial nephritis: eosinophilia, granular and white cell casts<br />

(c) Acute glomerulonephritis: RBCs, RBC casts, and proteinuria<br />

(2) In these patients, the renal tubules lose their ability to concentrate and reabsorb sodium; therefore, the<br />

urine is dilute (specific gravity 1.010), and the urine sodium concentration is increased (>40 mEq/L).<br />

b. Etiology<br />

(1) Acute tubular necrosis: responsible for 90% of intrinsic cases and in general is a reversible injury to<br />

the renal tubule<br />

(a)<br />

lschemia (most common cause of acute tubular necrosis)<br />

i. Prolonged hypoperfusion<br />

ii. Hemorrhage<br />

iii. Sepsis<br />

(b) Nephrotoxins<br />

i. Aminoglycosides<br />

ii. Heavy metals<br />

iii. Contrast<br />

iv. IV immunoglobulins<br />

(c) Heme pigments<br />

(2) Vascular disease<br />

(a) Vasoconstrictive disease (malignant hypertension, thrombocytopenic purpura)<br />

(b) Vasculitis<br />

(3) Thrombosis (renal artery/vein)<br />

(4) Acute interstitial nephritis: interstitial inflammation most commonly due to medication<br />

(a)<br />

Drug-related (penicillin, cephalosporins, sulfonamides, NSAIDs, diuretics, allopurinol, cimetidine,<br />

anticoagulants); methicillin is the most commonly implicated drug.<br />

(b) Infection-related (bacterial, protozoan, fungal)<br />

(c) Immune-related (lupus, leukemia, lymphoma, sarcoidosis)<br />

(5) Glomerular disease: characterized by inflammation of the glomeruli; seen most commonly in children<br />

(a) Postinfectious: poststreptococcal (primary process)<br />

(b) Noninfectious: lupus, Goodpasture syndrome (secondary processes)<br />

(c) lgA nephropathy: most common cause of primary glomerulonephritis<br />

B. Diagnosis of acute kidney injury according to type<br />

1. The history and physical examination should suggest etiology.<br />

a. Hypoperfusion - prerenal<br />

b. Signs and symptoms of obstruction - postrenal<br />

c. History of a primary disorder that can cause renal disease or a history of exposure to a contrast material,<br />

drug, or heavy metal - intrinsic<br />

2. Laboratory studies, procedures, and other diagnostic measures help to identify the cause and site of obstruction<br />

due to postrenal causes.<br />

a. Postvoid residual urine measurement (by catheter or ultrasound bladder scan)<br />

b. Urinalysis<br />

c. Renal ultrasound (98% sensitivity in identifying upper-tract obstruction)<br />

807


RENAL AND UROLOGIC DISORDERS<br />

3. Helpful laboratory studies in defining specific abnormalities related to intrinsic renal disease<br />

a. Microscopic examination of urine sediment (eosinophilia suggests a drug-related cause)<br />

b. Urine for myoglobin<br />

c. Urine and serum sodium and creatinine<br />

d. Urine osmolality (measures concentrating ability)<br />

4. In cases in which you are not sure of the (acute) type of failure (prerenal, postrenal, or renal), learn to calculate<br />

the renal indices.<br />

a. These tests are not useful in patients with chronic renal failure, on diuretics, or with a history of obstruction.<br />

There can be an overlap between categories, which emphasizes the importance of a good history and<br />

physical examination in determining etiology.<br />

b. Calculate the indices.<br />

FENa = fractional excretion of sodium<br />

FENa (%) =<br />

urine sodium/serum sodium<br />

urine creatinine/serum creatinine<br />

X 100<br />

RFI = renal failure index<br />

RFI (%) =<br />

urine sodium<br />

urine creatinine/serum creatinine<br />

X 100<br />

c. Interpret the indices.<br />

Table 49: Laboratory Findings in Acute Kidney Injury<br />

Prerenal<br />

Acute tubular Acute glomerular Acute interstitial<br />

necrosis nephritis nephritis<br />

Postrenal<br />

Serum BUN/ >20:1 20:1 20:1<br />

creatinine ratio<br />

UNa (mEq/L) 20


RENAL AND UROLOGIC DISORDERS<br />

D. Complications of acute kidney injury<br />

1. Hyperkalemia (most immediate life-threatening complication; see also pages 606-607)<br />

a. A serum potassium and ECG are probably two of the most important laboratory studies to obtain in patients<br />

with acute renal failure.<br />

b. Hyperkalemia primarily manifests as cardiotoxicity (cardiac dysrhythmias and sudden death).<br />

c. Management: determined by the potassium level and the ECG findings and can involve a combination of<br />

the following modalities:<br />

(1) IV calcium<br />

(2) IV glucose and insulin<br />

(3) IV sodium bicarbonate<br />

(4) IV sodium chloride 3% (in severe hyperkalemia only)<br />

(5) Inhaled f:3-adrenergic agents<br />

(6) IV diuretics<br />

(7) Cation exchange resins: sodium polystyrene sulfonate and patiromer sorbitex calcium (Note: not for<br />

initial treatment of life-threatening hyperkalemia)<br />

(8) Dialysis<br />

2. Hypocalcemia (see also pages 607-608)<br />

a. Common in chronic renal failure but can occur in acute renal failure in which it is usually asymptomatic<br />

b. Treatment: IV calcium gluconate or calcium chloride in symptomatic patients<br />

3. Hypermagnesemia is common but rarely significant; magnesiumcontaining antacids and laxatives should be<br />

avoided in these patients.<br />

4. High anion gap metabolic acidosis is almost always present but is generally mild and rarely requires correction.<br />

5. Volume overload<br />

a. Common in oliguric patients<br />

b. Can lead to CHF and pulmonary edema<br />

c. IV diuretics and nitroglycerin as well as Bi PAP can be used as temporizing measures while arrangements<br />

for dialysis are being made.<br />

E. Antibiotics to avoid in the presence of kidney injury<br />

1. Tetracycline<br />

2. Nitrofurantoin<br />

3. Penicillin (most common offender)<br />

4. Cephalosporins and aminoglycosides<br />

5. Bacitracin IM (uncommonly used)<br />

F. Indications for dialysis in acute kidney injury<br />

1. Hyperkalemia (ECG or clinical manifestations are resistant to therapy)<br />

2. Fluid overload<br />

3. Significant acidosis<br />

4. Presence of certain toxins (ethylene glycol, methanol)<br />

5. Uremic symptoms (pericarditis, encephalopathy, bleeding complications)<br />

6. BUN >100 mg/dl, creatinine >10 mg/dl (relative indication)<br />

G. Heme pigment-induced kidney injury<br />

1. Pathophysiology: injury occurs three ways:<br />

a. Tubular obstruction<br />

b. Direct proximal tubular cell injury<br />

c. Vasoconstriction<br />

2. Rhabdomyolysis (see pages 331 and 404-405, crush injury)<br />

a. Trauma (especially crush injuries, burns, seizures, prolonged strenuous muscle contraction) -a><br />

rhabdomyolysis (destruction of skeletal muscle cells) - release of myoglobin into the serum, which passes<br />

through the kidneys and has a direct toxic effect on the renal tubules -a> acute tubular necrosis -a> renal<br />

failure (t serum creatinine)<br />

b. Also can be from hemoglobin in hemolytic anemia<br />

809


RENAL AND UROLOGIC DISORDERS<br />

3. Epidemiology: suspect in the clinical settings listed below:<br />

a. Major trauma<br />

b. Major burns<br />

c. Electrical injuries<br />

d. Intoxicated patients (especially alcohol and phencyclidine [PCP])<br />

(1) In alcoholic patients, rhabdomyolysis is due to the direct toxic effect of ethanol on the skeletal muscle<br />

cell membrane in addition to pressure necrosis of muscle as a result of prolonged immobilization.<br />

(2) In phencyclidine (PCP) overdose, rhabdomyolysis is due to the excessive muscle activity and agitation<br />

that this drug induces.<br />

e. Voluntary/involuntary muscle activity (exertion/seizure)<br />

f. Infections (influenza A and B, coxsackie virus, Ebstein-Barr virus, HIV, Legione//a spp)<br />

4. Diagnostic evaluation<br />

a. Urinary findings<br />

(1) Color may be smoky (reddish brown) because of the presence of myoglobin (but do not wait for a<br />

color change before pursuing the diagnosis).<br />

(2) Positive for blood on dipstick examination, but no RBCs are seen on microscopic examination<br />

(indicates myoglobinuria).<br />

(3) Renal tubular casts and muddy brown granular casts may be present on microscopic examination.<br />

(4) Myoglobinuria may be present; however, it is not a reliable marker of rhabdomyolysis, because it is<br />

rapidly cleared from the urine and can be absent (despite the presence of significant rhabdomyolysis).<br />

b. Serum myoglobin may be positive but, like urine myoglobin, is rapidly cleared and is, therefore, not a<br />

reliable indicator of rhabdomyolysis.<br />

c. Serum creatine kinase<br />

(1) The most sensitive test for detecting rhabdomyolysis<br />

(2) Released immediately after muscle injury; clearance from serum is slow.<br />

d. Hypocalcemia (63%), hyperkalemia (


RENAL AND UROLOGIC DISORDERS<br />

U. CHRONIC KIDNEY DISEASE<br />

A. Chronic kidney disease is a heterogenous group of disorders characterized by alterations in kidney structure<br />

and function. Defined as a presence of kidney damage or decreased kidney function for


RENAL AND UROLOGIC DISORDERS<br />

Ill. HEMODIALYSIS-RELATED PROBLEMS<br />

A. Complications during hemodialysis<br />

1 . Hypotension<br />

2. Cramps<br />

3. Nausea and vomiting<br />

4. Headache<br />

5. Dyspnea<br />

6. Chest pain/arrhythmias<br />

7. Back pain<br />

8. Air embolism: rare because of detection filters<br />

9. Dysequilibrium syndrome<br />

a. Occurs during or immediately after dialysis (most frequently during a patient's first dialysis treatment)<br />

b. Caused by cerebral edema hypothesized to be due to reverse osmotic shift (reduction in urea causes<br />

transient osmotic gradient) or intracerebral acidosis<br />

c. Clinical presentation: symptoms range from headache, nausea, vomiting, and muscle cramps to altered<br />

mental status, grand mal seizures, and coma.<br />

d. Management: symptomatic treatment (rest, analgesics, antiemetics) is all that is needed for most patients;<br />

however, other causes of neurologic dysfunction (hypoglycemia, hypocalcemia, intracranial hemorrhage,<br />

etc) should be considered and excluded, particularly in patient whose symptoms persist or worsen over a<br />

period of observation.<br />

B. Complications associated with vascular access<br />

1 . Hemorrhage<br />

a. Minor bleeding associated with dialysis puncture or mild trauma<br />

(1) Apply nonocclusive pressure to the site. If this is unsuccessful, apply topical thrombin or neutralize<br />

excessive anticoagulation with vitamin Kor protamine sulfate.<br />

(2) Recheck access for presence of a thrill.<br />

(3) Evaluate extent of blood loss.<br />

(4) Observe in emergency department for rebleeding.<br />

b. Extensive bleeding from an aneurysm or pseudoaneurysm<br />

(1) Control bleeding with direct pressure.<br />

(2) Stabilize patient with IV fluids (normal saline or lactated Ringer's), oxygen, and cardiac monitoring.<br />

(3) Draw blood for laboratory studies (CBC, INR, type and cross).<br />

(4) Obtain stat vascular surgery consult.<br />

2. Thrombosis (signaled by loss of a thrill in the access)<br />

a. Obtain immediate vascular surgery consult to determine whether to remove the clot surgically or use a<br />

thrombolytic agent.<br />

b. Do not attempt to irrigate the access device, because this can result in clot embolization.<br />

3. Infection (usually due to Staphylococcus)<br />

a. More common in artificial than native grafts<br />

b. Can produce recurrent bacteremia and loss of access site<br />

c. Local signs (redness, warmth, tenderness, and induration of access site) may be absent; some patients<br />

present with only a fever or recurrent episodes of bacteremia.<br />

d. Management<br />

(1) Obtain blood cultures.<br />

(2) Administer IV antibiotics (usually vancomycin).<br />

812


RENAL AND UROLOGIC DISORDERS<br />

IV. COMPLICATIONS ASSOCIATED WITH KIDNEY<br />

TRANSPLANTATION<br />

A. Kidney allograft dysfunction<br />

1. Principal causes are categorized<br />

a. Hyperacute rejection (rarest) occurs within a few minutes to a few hours after surgery and results in<br />

irreversible allograft destruction.<br />

b. Acute rejection (most common) generally occurs within 1-12 weeks after surgery; 90% may be successfully<br />

reversed if recognized and treated promptly.<br />

c. Chronic rejection is indolent and difficult to arrest.<br />

2. Clinical presentation<br />

a. Tenderness over the allograft (located in the iliac fossa)<br />

b. Decreased urine output<br />

c. Fever (low-grade)<br />

d. Generalized discomfort<br />

e. Worsening hypertension<br />

f. Precipitous weight gain<br />

g. Peripheral edema<br />

3. Diagnostic evaluation should be directed toward excluding other causes of decreased renal function that occur<br />

in renal transplant patients (eg, volume contraction, drug-induced nephrotoxicity, infection, etc) and should<br />

include:<br />

a. Urinalysis<br />

b. BUN/creatinine<br />

c. Renal ultrasonography and bladder scan for residual urine<br />

d. Trough level of cyclosporine<br />

4. Management<br />

B. Infections<br />

a. High-dose steroids (IV or orally) for 3-4 days followed by:<br />

b. Allograft biopsy (to document the presence of ongoing rejection) in those patients who show little or no<br />

improvement, and<br />

c. Administration of an antibody preparation directed toward lymphocytes (such as OKT3, a monoclonal antihuman<br />

T cell antibody [lgG2]) to those patients with documented unremitting allograft rejection.<br />

1 . Pathophysiology<br />

a. Because of ongoing immunosuppression, infections (particularly opportunistic ones) are common in these<br />

patients.<br />

b. May present with a paucity of findings (fever and typical physical findings may be absent)<br />

2. Prevention<br />

a. In an attempt to prevent these infections, patients are given multiple vaccines and prophylactic<br />

medications.<br />

b. The regimen generally includes the following:<br />

(1) Pneumococcal vaccine<br />

(2) Hepatitis B vaccine<br />

(3) Nystatin<br />

(4) TMP-SMX (for prevention of Pneumocystis jiroveci pneumonia)<br />

3. Cytomegalovirus infection<br />

a. The most common life-threatening infection in these patients (as well as recipients of other solid organ<br />

transplants and bone marrow recipients)<br />

b. Generally occurs 1-6 months after transplantation surgery<br />

c. Manifestations range from daily fever and malaise to leukopenia, lymphadenopathy, t liver function tests,<br />

epigastric pain, diarrhea, and pneumonia.<br />

d. Evaluation should include a CBC, liver function tests, viral PCR, and chest radiograph.<br />

813


RENAL AND UROLOGIC DISORDERS<br />

e. Management<br />

(1) Reducing immunosuppressive therapy and<br />

(2) IV ganciclovir (with or without immune globulin) if lung, GI, or liver involvement is present<br />

4. Urinary tract infections develop in 20% of patients in the first 4 months after kidney transplantation.<br />

V. URINARY TRACT INFECTIONS<br />

A. Definition: significant bacteriuria in a symptomatic patient<br />

B. Incidence<br />

1. Females have a steadily increasing incidence with age; peaks occur in infancy, preschool, and childbearing years.<br />

2. Males have approximately the same incidence as females during infancy, but then this rapidly declines and<br />

remains low until middle age when prostatism occurs.<br />

C. Etiology<br />

1. Uncomplicated urinary tract infections (UTls)<br />

a. Represents most UTls<br />

b. Occur in normal individuals in whom no anatomic defect in the urinary system can be found<br />

c. Usually caused by gram-negative aerobic bacilli from the gut; Escherichia coli is the predominant pathogen.<br />

Occasional other species of Enterobacteriaceae (such as Proteus mirabilis and Klebsiella pneumoniae) or<br />

other bacteria such as Staphylococcus saprophyticus.<br />

2. Complicated UTls<br />

a. Occur in patients with underlying structural, neurologic, or immunologic disease<br />

b. May be caused by unusual organisms (Pseudomonas, Serratia marcescens, etc); however, typical pathogens<br />

predominate.<br />

D. Normal host-defense mechanisms<br />

1. Acidity: pH


RENAL AND UROLOGIC DISORDERS<br />

(a) 2-5 WBCs/high-power field on a centrifuged specimen is considered to be significant by most<br />

authors.<br />

(b) 1-2 WBCs/high-power field in men if bacteria are also present<br />

(2) Significant bacteriuria<br />

(a)<br />

Any bacteria seen on unspun urine is considered significant when viewed under high-powered field.<br />

(b) > 15/high-powered field (high dry) on spun sediment<br />

c. Diagnosis of a UTI based on a urinalysis is presumptive.<br />

(1) False-positives occur secondary to contamination with bacteria from the skin or from other causes of<br />

pyuria.<br />

(2) False-negatives occur secondary to diuresis or an infection that is due to a more virulent bacterium (in<br />

which case, few organisms are required to cause infection), and they may not be seen in significant<br />

numbers on microscopic examination.<br />

2. Definitive diagnosis is made by urine culture.<br />

a. A colony count> 10 2 /ml is considered significant in the symptomatic patient (much literature still suggests<br />

10 5 /ml).<br />

b. Cultures should be ordered on:<br />

(1) Extremes of age (infants, children, and the elderly)<br />

(2) Adult men<br />

(3) Pregnant women<br />

(4) Patients with any of the following:<br />

(a)<br />

Pyelonephritis<br />

(b) Prolonged symptoms (>6 days)<br />

(c) Underlying medical disease/immunocompromised (diabetes mellitus, sickle cell anemia, cancer)<br />

or history of kidney stones<br />

(d) Urologic abnormalities (structural or neurologic)<br />

(e) History of recent urologic instrumentation<br />

(f)<br />

G. Management<br />

Those who relapse, require hospitalization, or have a chronic indwelling catheter<br />

1. In females, look for and exclude cervicitis 1<br />

pelvic inflammatory disease, and vulvovaginitis.<br />

2. In males, look for and exclude urethritis.<br />

3. Outpatient antibiotic therapy for UTls<br />

a. Patients with uncomplicated UTls should be treated with an agent effective against community-acquired<br />

E coli. A 3-day course of antibiotics is optimal if there are no complicating factors; this regimen is<br />

inexpensive, associated with good compliance, has fewer adverse effects than a traditional 10-14 day<br />

regimen, and has a cure rate similar to that of 7-day therapy.<br />

(1) TMP-SMX or amoxicillin-clavulanate (first-line drugs only if local resistance patterns to f coli are 1 0% or<br />

(b) If the patient is male with urethritis not caused by a sexually transmitted disease (3-day therapy) or<br />

cystitis (7-10 day regimen)<br />

b. Pregnant patients: despite increasing resistance rates, ampicillin, amoxicillin, and cephalosporins remain<br />

first-line agents.<br />

(1) Optimal duration of treatment is not known (usually 3-7 days).<br />

(2) Nitrofurantoin is becoming a first-line agent because it is efficacious, inexpensive, and well-tolerated;<br />

because of its short half-life, it must be taken for at least 7 days. Do not give to patients with glucose<br />

6-phosphate dehydrogenase deficiency. Can be used in pregnancy but not at term because of concern<br />

for hemolytic anemia in infant related to glutathione instability.<br />

c. If Chlamydia trachomatis is the suspected pathogen (history of new sexual partner or the presence of pyuria<br />

without bacteriuria) 1<br />

culture for Chlamydia trachomatis and begin therapy with doxycycline, TMP-SMX, or<br />

a quinolone (levofloxacin or ofloxacin; ciprofloxacin does not have activity against Chlamydia). Therapy<br />

should be for 10-14 days.<br />

d. Complicated, resistant, or recurrent infections (but no pyelonephritis): start therapy with a quinolone (eg,<br />

ciprofloxacin), and adjust as needed based on culture and sensitivity; may require 1 0-14 days of therapy.<br />

815


RENAL AND UROLOGIC DISORDERS<br />

e. Single-dose therapy is not appropriate for typical emergency department patients, many of whom have<br />

subclinical pyelonephritis and no identified medical doctor.<br />

(1) Single-dose therapy has not been uniformly accepted in spite of reported cure rates (80%-100%) with<br />

TMP-SMX.<br />

(2) Single-dose therapy should be reserved for the reliable patient with an uncomplicated UTI and an<br />

identified personal physician who can reassess the patient within 1 week.<br />

(3) Single-dose therapy should be avoided in infants and adult men.<br />

f. Consider offering a 2-day course of a bladder analgesic such as phenazopyridine to patients experiencing<br />

painful urination, and encourage frequent voiding.<br />

4. Outpatient antibiotic therapy for patients with mild to moderate uncomplicated pyelonephritis who tolerate<br />

oral intake<br />

a. Consider "sequential" treatment with a parenteral dose of gentamicin, ceftriaxone, or a fluoroquinolone<br />

before starting oral therapy.<br />

b. Discharge on a fluoroquinolone for 7-14 days with instructions for follow-up in 1 week.<br />

5. Indications for admission in patients with pyelonephritis<br />

a. Severely ill/uroseptic<br />

b. The very young or elderly; men >60 years old<br />

c. Diagnostic uncertainty<br />

d. Underlying anatomic urinary tract abnormality or medical problems (including renal failure)<br />

e. History of obstruction, stones, or instrumentation<br />

f. Progression of uncomplicated UTl/failed outpatient management<br />

g. Persistent vomiting or inability to tolerate oral antibiotics<br />

h. lmmunocompromised patient (diabetes, cancer, sickle cell disease, transplant recipient)<br />

1. Poor social situation/inadequate access to follow up<br />

j. Pregnancy<br />

6. Indications for urologic referral to search for an underlying anatomic abnormality<br />

a. All children and adult men with UTls<br />

b. Any adult with multiple recurrences of infection (>3/year)<br />

7. Patients with obstruction and a UTI (stones are the most common cause) have a potentially life-threatening<br />

condition; a genitourinary consult should be obtained immediately.<br />

VI. NEPHROLITHIASIS<br />

A. Etiology<br />

1. Most kidney stones (80%) contain calcium (oxalate or phosphate) and are radiopaque.<br />

2. The theory of calcium stone formation is that stones form in the medullary portion of the kidney or by<br />

hypersaturation of the urine in chronic hypercalcemic states such as:<br />

a. Hyperthyroidism<br />

b. Hyperparathyroidism<br />

c. Neoplasm<br />

d. Sarcoidosis<br />

e. Multiple myeloma<br />

f. Distal renal tubular acidosis<br />

2. Struvite stones (magnesium-ammonium-phosphate) are radiopaque and are caused by a chronic urinary tract<br />

infection associated with urea-splitting organisms (usually Proteus or Klebsiel/a).<br />

3. Uric acid stones are radiolucent and are seen in patients with:<br />

a. Gout<br />

b. Myeloproliferative disease or leukemia<br />

c. High-protein diet<br />

4. Cystine stones (1 %-3%) are radiopaque and associated with a rare, inherited defect of amino acid transport<br />

within the kidney.<br />

816


RENAL AND UROLOGIC DISORDERS<br />

B. Risk factors<br />

1. Increased in patients with enhanced enteric oxalate absorption (eg, gastric bypass, bariatric surgery, short<br />

bowel syndrome).<br />

2. Stone disease is 2-fold higher in patients with hypertension. This accounts for change in male to female ratio<br />

from 3:1 to 1.3-1.6:1.<br />

3. Other diseases, such as diabetes, gout, and obesity, increase the risk. Commonly have increased uric acid states<br />

or acidic urine.<br />

4. There is a familial and hereditary predisposition.<br />

5. Excessive physical exercise<br />

6. Prior history of nephrolithiasis. Recurrence can be as high as 50% at 10 years, and the risk increases with each<br />

additional stone episode.<br />

C. Clinical presentation<br />

1. Unilateral colicky pain in the flank, back, or lower quadrant with radiation to the groin, labia, or testicles;<br />

distinguishing from an abdominal aortic aneurysm is essential.<br />

2. The pain is usually severe and often accompanied by dysuria, frequency, and hematuria.<br />

3. Nausea and vomiting occur as the pain increases in severity.<br />

4. If the stone passes into the bladder, the severity of pain diminishes markedly but may persist as a dull ache due<br />

to ureteral spasm.<br />

D. Diagnostic evaluation<br />

1. Definitive diagnosis of the type of stone requires stone analysis and is indicated for patients in whom stones<br />

recur. A 24-hour urine collection and analysis can provide definitive recommendations for prevention of<br />

stones; studies include urine volume and pH as well as stone identification (calcium, oxalate, citrate, or<br />

magnesium).<br />

2. Laboratory studies<br />

a. CBC is not indicated but should be considered in the presentation that suggests anemia, infection, or<br />

abdominal aortic aneurysm.<br />

b. Urinalysis/urine human chorionic gonadotropin (in women of reproductive age)<br />

(1) Hematuria is typical of urolithiasis, but its absence does not exclude the diagnosis; 20% of patients<br />

will not have microscopic hematuria. Also, there does not appear to be any correlation between the<br />

degree of obstruction and the absence of hematuria.<br />

(2) If the urinary pH is >7.6, suspect the presence of urea-splitting organisms (Proteus); a pH


RENAL AND UROLOGIC DISORDERS<br />

3. Facilitation of stone passage: alpha blocker therapy (eg, tamsulosin) and calcium channel blockers (eg,<br />

nifedipine) have been used; tamsulosin 0.4 mg/day is considered mainstay therapy for larger stones (5-10 mm).<br />

4. Patients who are discharged should:<br />

a. Be encouraged to drink two glasses of water every 2 hours (no studies prove that hydration facilitates<br />

stone passage)<br />

b. Strain their urine to collect the stone for analysis<br />

c. Be referred to urology for follow-up<br />

d. Be advised to return if problems develop (persistent vomiting, fever, chills, unremitting pain)<br />

5. Indications for admission<br />

a. Presence of an associated urinary tract infection (pyuria, bacteriuria, leukocytosis, fever)<br />

b. Uncontrolled pain requiring parenteral narcotics<br />

c. Intractable nausea and vomiting<br />

d. Hypercalcemic crisis<br />

e. Acute kidney injury or anuria<br />

f. Solitary kidney (relative)<br />

g. Intrinsic renal disease (relative)<br />

h. High-grade obstruction (relative)<br />

1. Stone >5 mm (relative); stone size is a determinant of spontaneous passage but not used solely to determine<br />

treatment.<br />

VII. MALE GENITAL PROBLEMS<br />

A. Urethritis<br />

1. Etiology<br />

a. Neisseria gonorrhoeae<br />

b. Chlamydia trachomatis is most common cause of non-gonococcal urethritis, followed by Mycoplasma<br />

genitalium.<br />

c. In almost half of cases of non-gonococcal urethritis, no cause is identified.<br />

d. Simultaneous infection (which may be asymptomatic) with both gonorrhea and Chlamydia occurs in<br />

30%-50% of cases; antibiotic therapy should target both of these organisms.<br />

2. Classic clinical scenario: the patient usually complains of dysuria and a urethral discharge but may be<br />

asymptomatic.<br />

3. Diagnostic criteria<br />

a. Mucopurulent or purulent discharge<br />

b. Gram stain of the discharge reveals c::2 WBCs/high-power field (oil) and may demonstrate gram-negative<br />

intracellular diplococci with gonorrhea.<br />

c. Positive leukocyte esterase test on dipstick.<br />

4. Management<br />

a. Recommended regimens for gonorrhea and Chlamydia must also be prescribed.<br />

(1) Ceftriaxone 250 mg IM with azithromycin 1 g; doxycycline can be used as a substitute for<br />

azithromycin at 100 mg twice daily for 7 days.<br />

(2) Alternative regimens can be used if injectable cephalosporin is not available. Cefixime 400 mg po<br />

used in combination with the above other drugs can be used if microbiologic test of cure can be done<br />

1 week later.<br />

(3) For severe cephalosporin allergies, azithromycin 2 g with either gentamicin (240 mg IM, once) or<br />

gemifloxacin (320 mg) may be considered.<br />

b. If an associated epididymitis is suspected, 7-10 days of therapy are required.<br />

c. Infection with Trichomonas vagina/is, if present, should be treated with metronidazole 2 g orally.<br />

Trichomonal infection should be considered in those patients who present with persistent symptoms (in the<br />

absence of noncompliance and reexposure) despite adequate treatment for gonorrhea and Chlamydia.<br />

818


RENAL AND UROLOGIC DISORDERS<br />

B. Orchitis (inflammation of the testicle)<br />

1. Isolated orchitis is rare.<br />

2. Most cases of orchitis are the result of direct extension of an epididymal infection and occur as a complication<br />

of a systemic illness, either bacterial or viral.<br />

3. Viral orchitis is most often caused by mumps.<br />

4. The patient presents with testicular pain and swelling (urinary and urethral symptoms are usually absent). In<br />

the case of mumps orchitis, symptoms usually evolve several days after the onset of parotitis, whereas bacterial<br />

orchitis can have a more acute presentation.<br />

5. An immediate urologic consult should be obtained to confirm the diagnosis, because orchitis is extremely<br />

uncommon; testicular torsion and tumor are far more common.<br />

C. Acute bacterial prostatitis<br />

1. Etiology<br />

a. Usually bacterial, primarily gram-negative organisms (80% are E coli, 20% are Klebsiella, Enterobacter,<br />

Proteus, or Pseudomonas)<br />

b. Mixed bacterial infections are uncommon.<br />

c. Tuberculosis is a possibility when renal tuberculosis is present.<br />

2. Clinical presentation<br />

a. The patient presents acutely ill with dysuria, frequency, urgency, and occasionally urinary retention<br />

accompanied by pain in the perineum, pelvis, and low back; systemic signs of infection may also occur<br />

(chills, fever, myalgias, malaise).<br />

b. Examination reveals a tender, swollen prostate that is warm and firm to the touch.<br />

c. Prostatic massage should not be done in patients with acute infection, because it can precipitate<br />

bacteremia; urine culture is sufficient for diagnosis, because cystitis is also present in most cases.<br />

3. Management<br />

a. Antibiotics<br />

(1) Outpatient<br />

(a) TMP-SMX one double-strength tablet every 12 hours<br />

(b) Fluoroquinolones (eg, levofloxacin 500 mg/day orally or ciprofloxacin 500 mg bid)<br />

(c) Treatment is for 6 weeks.<br />

(2) Inpatient (sepsis, urinary retention): IV fluoroquinolone plus gentamicin 3-5 mg/kg/day<br />

b. Prompt urologic consultation is indicated for patients who present with acute urinary retention.<br />

c. Supportive measures<br />

D. Genital ulcers<br />

1. Etiology<br />

(1) Hydration<br />

(2) Bed rest<br />

(3) Analgesics (NSAIDs or narcotics as needed)<br />

(4) Fecal softeners/laxatives (especially if narcotics are prescribed)<br />

a. Herpes simplex virus (types I and II)<br />

b. Syphilis<br />

c. Chancroid (Haemophilus ducreyi)<br />

d. Granuloma inguinale (Klebsiella granulomatis [formerly Calymmatobacterium granulomatis])<br />

e. Lymphogranuloma venereum (Chlamydia trachomatis)<br />

f. Behcet syndrome<br />

(1) Vasculitis of uncertain etiology but thought to be an autoimmune disorder<br />

(2) Clinical triad<br />

(a) Chronic oral ulcerations<br />

(b) Relapsing iridocyclitis<br />

(c) Genital ulcers<br />

(3) Can be comp I icated by polyarthritis and erythema nodosum<br />

(4) Lasts for many years with relapses and remissions but can be suppressed by corticosteroids.<br />

819


RENAL AND UROLOGIC DISORDERS<br />

2. Diagnostic evaluation<br />

a. Serologic testing for syphilis: a VDRL or rapid plasma reagent (nontreponemal tests) should be done initially<br />

and, if positive, should be confirmed with an FTA-ABS or other treponemal test. A VDRL should also be<br />

repeated at 3, 6, and 12 months after treatment to monitor response; with successful treatment, VDRL titers<br />

should gradually decrease.<br />

b. Bacterial and viral cultures (especially for herpes simplex virus) of lesions<br />

c. Examination of material from base of ulcer can be done.<br />

(1) Tzanck smear (done with Wright or Giemsa stain) - multi nucleated giant cells or immunofluorescent<br />

staining (herpes)<br />

(2) Darkfield examination - spirochetes (syphilis)<br />

(3) Gram stain<br />

(a) Gram-negative rods in a linear or parallel arrangement (chancroid)<br />

(b) "Donovan bodies" (granuloma inguinale)<br />

(4) Lymphogranuloma venereum complement fixation test is positive or microimmunofluorescence.<br />

3. Management: empiric therapy recommended<br />

a. Herpes<br />

(1) Acyclovir<br />

(a)<br />

Primary infections are treated with 400 mg orally every 8 hours x 7-10 days or until clinical<br />

resolution.<br />

(b) Patients with recurrent episodes may benefit from acyclovir if therapy is started within 24 hours of<br />

the appearance of lesions. Dosage regimens are:<br />

i. 400 mg orally every 8 hours x 5 days or<br />

ii. 800 mg orally every 12 hours x 5 days<br />

(c) Daily suppressive therapy is useful for patients with >6 recurrences per year. The dosage is 400 mg<br />

orally bid. Alternative regimens include famciclovir 250 mg orally every 12 hours, valacyclovir<br />

500 mg/day orally, or valacyclovir 1 g!day orally.<br />

(d) Patients with severe disease or complications requiring hospitalization are treated with IV<br />

acyclovir. The dosage is 5-10 mg/kg IV every 8 hours x 5-7 days or until clinical resolution.<br />

(2) Other regimens that are equally efficacious<br />

b. Syphilis<br />

(a) Famciclovir 250 mg orally every 8 hours x 7-10 days or<br />

(b) Valacyclovir 1 g orally every 12 hours x 7-10 days<br />

(1) Primary, secondary, and early latent (1 year duration) and late syphilis (except neurosyphilis)<br />

(a) Benzathine penicillin G 2.4 million units IM weekly for 3 doses or<br />

(b) Doxycycline 100 mg orally every 12 hours x 4 weeks or<br />

(c) Tetracycline 500 mg orally every 6 hours x 4 weeks<br />

(3) Neurosyphilis<br />

(a) Aqueous crystalline penicillin G 2-4 million units IV every 4 hours x 10-14 days or<br />

(b) Procaine penicillin G 2.4 million units IM every day plus probenecid 500 mg orally every 6 hours<br />

for 1 0-14 days<br />

(c) Penicillin-allergic patients require desensitization followed by one of the above regimens.<br />

Pregnant patients allergic to penicillin should be desensitized and treated with the appropriate<br />

penicillin regimen; all patients with syphilis should be tested for HIV.<br />

c. Chancroid (high rate of HIV co-infection)<br />

(1) Azithromycin 1 g orally in a single dose or<br />

(2) Ceftriaxone 250 mg IM in a single dose or<br />

(3) Erythromycin base 500 mg orally three times a day x 7 days or<br />

(4) Ciprofloxacin 500 mg orally every 12 hours x 3 days<br />

820


RENAL AND UROLOGIC DISORDERS<br />

d. Granuloma inguinale (all medications must be given for at least 3 weeks and until all lesions have<br />

completely healed)<br />

(1) Doxycycline 100 mg orally every 12 hours or<br />

(2) Alternative regimens<br />

(a) Azithromycin 1 g orally once per week<br />

(b) Ciprofloxacin 500 mg orally every 12 hours x 3 days<br />

(c) Erythromycin base 500 mg orally 3 times a day x 7 days<br />

(d) TMP-SMX one double-strength tablet (160 mg/800 mg) orally every 12 hours<br />

e. Lymphogranuloma venereum<br />

(1) Doxycycline 100 mg orally every 12 hours x 21 days or<br />

(2) Erythromycin 500 mg orally every 6 hours x 21 days<br />

E. Fournier gangrene (idiopathic scrotal gangrene)<br />

1. Clinical presentation<br />

a. Painful, erythematous or necrotic scrotum, penis, or perineum<br />

b. Febrile and appears toxic<br />

c. Acute onset of pain and swelling<br />

d. Urinary tract symptoms<br />

e. The patient is usually older and immunocompromised in some way (diabetes, chronic steroids, chronic<br />

alcoholism).<br />

2. This is a potentially life-threatening disease.<br />

3. Etiology<br />

a. Usually due to infection originating from the perianal area<br />

b. Several organisms, both anaerobic and aerobic, are generally involved. Bacteroides fragilis is the<br />

predominant anaerobe and f coli is usually the predominant aerobe. Other agents include hemolytic<br />

Streptococcus, Staphylococcus, and C!ostridium spp.<br />

4. Management<br />

a. Early and aggressive surgical debridement<br />

b. Broad-spectrum parenteral antibiotics against anaerobes and gram-negative enteric organisms<br />

F. Balanoposthitis<br />

1. An inflammation of the glans penis (balanitis) and foreskin (posthitis) that is often caused by Candida,<br />

Cardnerella, anaerobes, herpes simplex virus, human papillomavirus.<br />

2. Noninfectious causes should be considered, eg, psoriasis, eczema, scabies.<br />

3. Recurrent balanoposthitis can be the sole presenting sign of diabetes.<br />

4. Clinical presentation<br />

a. Retraction of the foreskin reveals foul, purulent material.<br />

b. The glans is red, swollen, and tender to palpation.<br />

5. Management<br />

G. Phimosis<br />

a. Good hygiene<br />

b. Consider circumcision<br />

c. Antifungal creams (nystatin or clotrimazole)<br />

d. Broad-spectrum antibiotics if secondary infection present (Streptococcus infection may be indistinguishable<br />

from nonspecific balanitis; cultures or empiric therapy should be considered.)<br />

e. Evaluation for diabetes (if infection recurrent)<br />

1. Definition<br />

a. Inability to retract the foreskin behind the glans<br />

b. Usually secondary to chronic infection of the foreskin associated with progressive scarring<br />

2. Management<br />

a. Emergency treatment with a dorsal slit of the foreskin is occasionally required in severe cases.<br />

b. Definitive therapy has traditionally been circumcision (after the infection has been controlled with broadspectrum<br />

antibiotics). Topical steroids (triamcinolone 0.025% every 12 hours or betamethasone 0.05%<br />

every day) have been shown to be 70%-90% effective in treating phimosis to allow for some degree of<br />

foreskin retraction and avert circumcision.<br />

821


RENAL AND UROLOGIC DISORDERS<br />

H. Paraphimosis<br />

1. Definition: inability to pull retracted foreskin back over the glans<br />

2. Management<br />

a. Emergency treatment is indicated; vascular compromise can occur.<br />

b. Apply continuous, firm pressure to the glans penis for 5-10 minutes to reduce edema, and then pull the<br />

foreskin over the glans.<br />

c. If manual reduction is unsuccessful, infiltrate the constricting ring with a local anesthetic and make a<br />

superficial vertical incision dorsally in the midline (dorsal slit).<br />

d. Definitive treatment is circumcision (after inflammation subsides).<br />

I. Constriction injuries to the penis/tourniquet syndrome<br />

1. Must be treated immediately<br />

2. Rings, rubber bands, wire, hair, and other objects can transect the urethra and cause neurovascular<br />

compromise.<br />

J. Fracture of the penis<br />

1. Acute tear of the tunica albuginea (usually during erection) causes a swollen (or bent), painful penis. Occurs<br />

suddenly and "snap" may be heard. If Buck's fascia remains intact, swelling confined to penile shaft. If not,<br />

blood may dissect into scrotum, perineum, and suprapubic spaces.<br />

2. Diagnostic evaluation<br />

a. Retrograde urethrogram to exclude associated urethral injury<br />

b. Immediate urologic consult<br />

3. Treatment: surgical repair<br />

K. Peyronie disease<br />

1. Clinical presentation<br />

a. Patients present with a history of dorsal penile curvature with painful erections (and may be associated with<br />

Dupuytren contracture of the hand).<br />

b. Examination reveals a thickened plaque involving the tunica albuginea of the corporal bodies.<br />

2. Management<br />

L. Carcinoma<br />

a. This is not an emergency.<br />

b. If the patient is bothered by it, urologic referral is in order.<br />

1. Clinical presentation<br />

a. A nontender or warty growth is usually found on the glans of uncircumcised men >50-60 years old.<br />

b. On physical examination, always retract the foreskin to look for this.<br />

2. Management: immediate referral for this aggressive tumor<br />

M. Priapism<br />

1. Definition: a sustained erection unrelated to sexual stimulation; there are two forms:<br />

a. Low-flow (ischemic) priapism: painful<br />

(1) Pathophysiology: venous outflow-,, venous stasis and ischemia that involves the corpus cavernosae<br />

but spares the glans and corpus spongiosum-,, rigid, painful penile shaft and soft glans<br />

(2) Because the spongiosum is spared, the patient should be able to void.<br />

(3) A surgical emergency: irreversible damage occurs in 24-48 hours.<br />

(4) Etiology<br />

(a) Reversible (may respond to medical therapy)<br />

i. Sickle cell disease or trait (most common cause in children)<br />

ii. lntracavernosal injections of papaverine, phentolamine, alprostadil, and prostaglandin E1 for<br />

erectile dysfunction (most common cause in adults)<br />

iii. Leukemic infiltration<br />

(b) Irreversible (generally unresponsive to medical therapy)<br />

i. Medications (antihypertensives, anticoagulants, and antidepressents)<br />

ii. illegal substances (alcohol, marijuana, cocaine)<br />

iii. Idiopathic<br />

822


RENAL AND UROLOGIC DISORDERS<br />

b. High-flow (nonischemic) priapism: rare and usually painless<br />

(1) Not a true emergent condition; long-term sequelae are rare.<br />

(2) Pathophysiology: t arterial blood flow to the corpus cavernosae - t venous blood flow - partially<br />

rigid, painless penile shaft and hard glans<br />

(3) Etiology<br />

2. Management<br />

(a) Groin or straddle injury -<br />

(b) High spinal cord injury/lesion<br />

a. Obtain immediate urologic consult.<br />

arterial-cavernosal shunt formation (most common cause)<br />

b. Manage ischemic priapism in a stepwise fashion to achieve resolution as quickly as possible: dry<br />

aspiration - aspiration with irrigation - intracavernosus injections of phenylephrine (dilute with normal<br />

saline to a concentration of 100-500 mcg/ml)<br />

(1) Repeated injections (1 ml every 3-5 minutes as needed up to 1 hour) should be performed before<br />

deciding that this treatment is unsuccessful and a surgical shunt is required.<br />

(2) Monitor for adverse effects during and after intracavernosus injection(s):<br />

(a) Abnormal vital signs (hypertension, tachycardia/bradycardia)<br />

(b) Cardiac dysrhythmias<br />

(c) Headache, palpitations<br />

(3) In patients with an underlying disorder (sickle cell disease, hematologic malignancy), systemic<br />

treatment of the disorder should be administered concurrently with the intracavernosus injection(s).<br />

c. Initial management of nonischemic priapism should be observation. Corporal aspiration should be<br />

performed only if the diagnosis is in question. Selective arterial embolization is recommended for patients<br />

who request treatment.<br />

N. Testicular torsion<br />

1. Incidence: may occur at any age but has bimodal peaks at the first few days of life and then between ages 12<br />

and 18.<br />

2. Classic clinical scenario<br />

a. Acute onset of severe, unilateral testicular pain or lower abdominal pain is typical; there may be a recent<br />

history of strenuous physical activity or a past history of testicular pain with spontaneous relief.<br />

b. Swelling occurs within hours; examination (conducted with the patient in the standing position) reveals<br />

a swollen, firm, "high-riding" testicle with a transverse lie; the contralateral testicle may also have a<br />

transverse lie, because the underlying anatomic abnormality ("bell-clapper deformity"), which predisposes<br />

the patient to torsion, may be bilateral.<br />

c. An associated reactive hydrocele may be present; loss of the cremasteric reflex is an associated finding with<br />

high specificity, but it is not diagnostic and is not always present.<br />

d. Urinary symptoms, pyuria, leukocytosis, and fever are typically absent.<br />

e. The most common misdiagnosis is "epididymitis," because there is tenderness posteriorly where the<br />

epididymis is normally located. However, what you are actually feeling is a tender testicle, because when<br />

the testicle is "torsed" (twisted around), the epididymis is then located anteriorly.<br />

3. Diagnostic evaluation<br />

a. Never exclude the diagnosis of testicular torsion based on a single element of the history or physical<br />

examination finding.<br />

(1) A prior history of torsion and orchiopexy does not exclude the diagnosis (especially if absorbable<br />

sutures were used).<br />

(2) Up to 80% of patients report anorexia, nausea, and vomiting.<br />

(3) Of all the examination findings, presence of the cremasteric reflex is the most helpful in excluding the<br />

diagnosis of torsion.<br />

b. Ultrasound examination and technetium testicular scanning detect the amount of blood flow to the testicle<br />

(with the normal testicle serving as control); flow is decreased or absent on the affected side with torsion.<br />

Color Doppler ultrasound is more accurate than the traditional Doppler examination and is now the test of<br />

choice in most hospitals. Sensitivity ranges from 83% to 100%; however, sensitivity is decreased in younger<br />

age groups, especially neonates.<br />

c. Do not delay urologic consult to wait for a scan.<br />

d. Emergency surgical exploration of the scrotum is the definitive diagnostic test and is indicated if testicular<br />

torsion cannot be excluded with certainty.<br />

823


RENAL AND UROLOGIC DISORDERS<br />

4. Management<br />

a. Obtain immediate urologic consult.<br />

b. Order a CBC and urinalysis (usually normal in torsion) and prepare patient for surgery.<br />

c. Radionuclide scanning or Doppler examination may be performed if either is immediately available and<br />

desired by the urologic consultant, but they should not delay definitive treatment.<br />

d. Surgery is indicated as soon as the diagnosis is made, because testicular viability decreases rapidly; salvage<br />

rate is 80%-100% up to 6 hours of ischemia, 20% after 1 0 hours, and approaches zero after 24 hours.<br />

e. Symptomatic therapy for patients awaiting surgery is most appropriate.<br />

0. Torsion of the appendices epididymis and testis<br />

1. These appendages are small pedunculated structures (without any known function) attached to the epididymis<br />

and testis that can torse and produce pain.<br />

2. Clinical presentation<br />

a. Early on, pain is localized and a tender nodule is palpated. The "blue dot sign" on transillumination of the<br />

testis is pathognomonic.<br />

b. If seen late, pain is diffuse and swelling is present, making diagnosis more difficult.<br />

3. Management<br />

a. If the diagnosis is certain, surgery is not required because these structures calcify.<br />

b. If the diagnosis is uncertain, surgery is indicated to exclude testicular torsion.<br />

P. Epididymitis<br />

1. Clinical presentation<br />

a. Inflammation of the epididymis is usually caused by bacterial infection and is more common in young<br />

adults than in preadolescent boys or older men.<br />

b. There is a gradual onset of unilateral pain and swelling (sudden onset is more common with torsion).<br />

c. Associated fever and dysuria are common.<br />

d. On examination, the epididymis is tender, swollen, and located in its normal posterior position.<br />

e. Elevation of the testicle is associated with relief of pain (Prehn sign), but this sign is unreliable; it cannot be<br />

used to distinguish epididymitis from torsion.<br />

2. Etiology is age dependent.<br />

a. Chlamydia trachomatis (usually sexually transmitted) is the most common pathogen (followed by Neisseria<br />

gonorrhoeae) in patients 35 years old.<br />

c. Fungal infection must be considered in homosexual men with epididymitis or epididymo-orchitis.<br />

3. Diagnostic evaluation<br />

a. Urinalysis: a few WBCs are common (but may also be seen with torsion).<br />

b. Urine culture<br />

c. CBC: a leukocytosis of 10,000-30,000/mm 3 is common.<br />

d. Urethral Gram stain<br />

e. Urethral culture for gonorrhea and Chlamydia<br />

4. Management<br />

a<br />

Supportive measures<br />

(1) Bed rest initially, followed by scrotal support<br />

(2) Ice packs<br />

(3) Analgesics<br />

(4) Fecal softeners<br />

b. Start antibiotics: antibiotic selection for acute epididymitis should be based on the patient's age, likelihood<br />

of gonorrhea and Chlamydia, and other presumed pathogens.<br />

(1) Homosexual (men who practice insertive anal sex) or sexually active men (coliforms)<br />

(a)<br />

Ceftriaxone 250 mg IM plus levofloxacin 500 mg orally every 12 hours or ofloxacin 200 mg orally<br />

every 12 hours x 10-14 days is the usual regimen.<br />

(b) If


RENAL AND UROLOGIC DISORDERS<br />

(2) Heterosexual men 35 years old or bisexual men (usually coliforms, but gonorrhea and Chlamydia<br />

may contribute): levofloxacin 500 mg/day orally or ofloxacin 300 mg orally bid x 10 days<br />

c. Refer to urologist for follow-up in 5-7 days.<br />

d. Admission for IV antibiotics is indicated for:<br />

Q. Urethral stricture<br />

(1) Toxic-appearing patients (particularly those who are older and in whom a scrotal abscess is suspected)<br />

(2) lmmunosuppressed patients<br />

(3) Patients with severe bilateral epididymitis<br />

1. Etiology: trauma, urethral instrumentation, or a complication of chlamydia! or gonorrheal infection<br />

2. Diagnostic evaluation<br />

a. History of partial or complete retention<br />

b. If unable to pass a catheter, a retrograde urethrogram can determine the extent and location of the stricture.<br />

3. Management<br />

a. Catheterization<br />

b. If unable to pass a regular or Coude catheter after 2 or 3 careful attempts, obtain a urologic consult for<br />

catheterization using fi I iforms.<br />

R. Urethral foreign bodies<br />

1. Occur at any age<br />

2. Clinical presentation: hematuria or signs and symptoms of obstruction or infection<br />

3. Diagnosis: made by palpation and confirmed on plain radiograph, retrograde urethrogram, or cystoscopy<br />

4. Management: urologic consult for removal<br />

S. Urinary retention<br />

1. Etiology<br />

a. Benign prostatic hypertrophy with bladder neck obstruction (most common cause in men >50 years old)<br />

b. Strictures (history of a sexually transmitted infection or pelvic trauma)<br />

c. Drugs<br />

(1) Antihistamines<br />

(2) Anticholinergic agents<br />

(3) Antispasmodic agents<br />

(4) Cyclic antidepressants<br />

(5) a-adrenergic stimulants<br />

(6) Antipsychotic agents<br />

d. Meatal stenosis<br />

e. Bladder neck contracture<br />

f. Bladder cancer<br />

g. Cancer of the prostate<br />

h. Neurogenic<br />

2. Clinical presentation<br />

a. The patient is in distress (unless he has a neurogenic bladder) and complains of hesitancy or poor stream<br />

followed by low abdominal pain and inability to void for 6-8 hours.<br />

b. On physical examination, the bladder is often visibly distended and easily palpable.<br />

3. Diagnostic evaluation<br />

a. Urinalysis to exclude co-infection<br />

b. BUN/creatinine to assess renal function<br />

825


RENAL AND UROLOGIC DISORDERS<br />

4. Management<br />

a<br />

Pass a Foley catheter; this is both diagnostic and therapeutic.<br />

b. Observe patient with chronic urinary retention for 2-4 hours after relief of urinary retention for<br />

development of postobstructive diuresis.<br />

(1) Syndrome of massive urine output that can produce volume depletion, electrolyte imbalance, and<br />

hypotension<br />

(2) These patients require hospitalization and additional laboratory studies (serum and urine electrolytes).<br />

c. Most other patients can be discharged to home with the catheter in place and referred to a urologist<br />

for evaluation.<br />

d. Antibiotics (trimethoprim or TMP-SMX) should be prescribed if a concomitant urinary tract infection<br />

is present.<br />

e. Belladonna and opium suppositories should not be prescribed for these patients. Continued use will<br />

prevent a successful voiding trial when the catheter is removed.<br />

826


RENAL AND UROLOGIC DISORDERS: PRACTICE CLINICAL SCENARIOS<br />

RENAL AND UROLOGIC DISORDERS: PRACTICE CLINICAL SCENARIOS<br />

Answers immediately follow the practice clinical scenarios.<br />

Scenario A<br />

Presentation: A 7-year-old child presents with facial swelling and "dark urine." The child had a sore throat<br />

last week.<br />

What is the diagnosis?<br />

Scenario B<br />

Presentation: A 60-year-old man on peritoneal dialysis presents with fever and abdominal pain. He states<br />

the dialysate fluid on return is "cloudy."<br />

What is the diagnosis?<br />

Scenario C<br />

Presentation: A 57-year-old diabetic man has pain in his scrotum.<br />

Physical examination: There is evidence of ecchymosis with surrounding erythema.<br />

What is the diagnosis?<br />

Scenario D<br />

Presentation: A 15-year-old male has pain on the right side of his flank and testicle.<br />

Physical examination: The testicle is swollen, firm, and high-riding with a transverse lie; the cremasteric<br />

reflex is absent.<br />

What is the diagnosis?<br />

Scenario E<br />

Presentation: A pregnant woman presents with urinary tract symptoms of dysuria. Urinalysis shows 50<br />

WBC/high-power field on a centrifuged specimen.<br />

What is the diagnosis?<br />

827


RENAL AND UROLOGIC DISORDERS: PRACTICE CLINICAL SCENARIOS<br />

ANSWERS TO PRACTICE CLINICAL SCENARIOS<br />

Scenario A<br />

Diagnosis: acute glomerulonephritis<br />

Diagnostic evaluation: Urine sediment shows RBCs, RBC casts, and proteinuria.<br />

Scenario B<br />

Diagnosis: spontaneous bacterial peritonitis<br />

Scenario C<br />

Diagnosis: Fournier gangrene<br />

Management: Management includes general supportive measures, cultures, and broad-spectrum parenteral<br />

antibiotics against anaerobes and gram-negative enteric organisms. Prompt urology consult should be<br />

obtained for surgical debridement. Hyperbaric oxygen has been used, but its efficacy is unknown and it is<br />

rarely necessary.<br />

Scenario D<br />

Diagnosis: testicular torsion<br />

Diagnostic evaluation: Never exclude the diagnosis of testicular torsion based on a single element of the<br />

history or physical examination finding. A prior history of torsion and orchiopexy does not exclude the<br />

diagnosis (especially if absorbable sutures were used). Up to 80% of patients report anorexia, nausea,<br />

and vomiting. Of all the examination findings, presence of the cremasteric reflex is the most helpful in<br />

excluding the diagnosis of torsion.<br />

Ultrasound examination and technetium testicular scanning detect the amount of blood flow to the testicle<br />

(with the normal testicle serving as control); flow is decreased or absent on the affected side with torsion.<br />

Color Doppler ultrasound is more accurate than the traditional Doppler examination and is now the test<br />

of choice in most hospitals. Sensitivity ranges from 83% to 100%; however, sensitivity is decreased in<br />

younger age groups, especially neonates. Do not delay urofogic consult to wait for a scan.<br />

Emergency surgical exploration of the scrotum is the definitive diagnostic test and is indicated if testicular<br />

torsion cannot be excluded with certainty.<br />

Scenario E<br />

Diagnosis: UT!<br />

Management: In pregnant patients, ampicillin, amoxicillin, and cephalosporins remain first-line agents<br />

despite increasing resistance rates. Duration of therapy of 3-7 days is recommended. Nitrofurantoin<br />

is becoming a first-line agent because it is efficacious, inexpensive, and well-tolerated; because of its<br />

short half-life, it must be taken for at least 7 days. Do not give to patients with glucose-6-phosphate<br />

dehydrogenase deficiency or who are at 38 weeks because of concern for hemolytic anemia in the infant.<br />

828


CUTANEOUS DISORDERS<br />

CUTANEOUS DISORDERS<br />

Introduction .............................................................................................................................................................. 832<br />

General Approach to the Patient Presenting with a Rash .......................................................................................... 832<br />

Toxicodendrons (Poison Ivy, Poison Oak, Poison Sumac) .......................................................................................... 832<br />

Diaper Dermatitis ..................................................................................................................................................... 833<br />

Eczema ...................................................................................................................................................................... 834<br />

Psoriasis .................................................................................................................................................................... 835<br />

Exfoliative Dermatitis/Exfoliative Erythroderma Syndrome ....................................................................................... 835<br />

Erythema Multiforme ................................................................................................................................................ 836<br />

Steven-Johnsons Syndrome and Toxic Epidermal Necrosis ........................................................................................ 838<br />

Erythema Nodosum ................................................................................................................................................... 839<br />

Pityriasis Rosea ......................................................................................................................................................... 840<br />

Cellulitis/Erysipelas ................................................................................................................................................... 840<br />

Impetigo .................................................................................................................................................................... 842<br />

Necrotizing Fasciitis .................................................................................................................................................. 844<br />

Cutaneous Abscess .................................................................................................................................................... 844<br />

Staphylococcal Scalded Skin Syndrome .................................................................................................................... 845<br />

Pemphigus Vulgaris ................................................................................................................................................... 846<br />

Bullous Pemphigoid ................................................................................................................................................... 846<br />

Herpes Simplex ......................................................................................................................................................... 847<br />

Herpes Zoster (Shingles) ........................................................................................................................................... 848<br />

Aphthous Ulcers ....................................................................................................................................................... 850<br />

Mulloscum Contagiosum ........................................................................................................................................... 850<br />

Fungal Disorders ....................................................................................................................................................... 850<br />

Pressure Ulcers (Decubitus Ulcers) ........................................................................................................................... 853<br />

Dermatologic Cancers .............................................................................................................................................. 853<br />

829


CUTANEOUS DISORDERS: SELF-ASSESSMENT QUESTIONS<br />

CUTANEOUS DISORDERS: SELF-ASSESSMENT QUESTIONS<br />

1. Which of the following is true regarding erythema multiforme?<br />

(a)<br />

Both erythema multiforme minor and major involve the mucous membranes.<br />

(b) The rash may be precipitated by trauma.<br />

(c) It is most commonly precipitated by a bacterial infection.<br />

(d) The rash will generally fade within 2 weeks.<br />

2. The presence of "target lesions" (erythematous plaques with dusky centers and bright red borders) on the palms and<br />

soles is classically associated with:<br />

(a)<br />

Erythema multiforme<br />

(b) Secondary syphilis<br />

(c) Disseminated gonococcal disease<br />

(d) Pityriasis rosea<br />

3. A positive Nikolsky sign is typical of skin lesions associated with:<br />

(a) Toxic shock syndrome<br />

(b) Herpes zoster<br />

(c) Toxic epidermal necrolysis<br />

(d) Contact dermatitis<br />

4. A Tzanck smear can be useful in making the diagnosis of all of the following except:<br />

(a) Herpes simplex (types 1 and 2)<br />

(b) Varicella (chickenpox)<br />

(c) Erythema multiforme<br />

(d) Herpes zoster (shingles)<br />

5. Which of the following statements regarding erysipelas is true?<br />

(a) It is caused by Staphylococcus aureus.<br />

(b) The rash is typically a tender, red plaque with sharply demarcated borders.<br />

(c) It is seen most commonly on the chest and upper back.<br />

(d) Supportive care is the mainstay of treatment.<br />

6. A pink to red maculopapular rash that starts on the face and rapidly spreads to the trunk and extremities and is associated<br />

with lymphadenopathy (especially postauricular, suboccipital, and posterior cervical) is most characteristic of:<br />

(a)<br />

Rubella<br />

(b) Roseola<br />

(c) Rubeola<br />

(d) Varicella<br />

7. All of the following statements about Koplik spots are true except:<br />

(a) They are found on the buccal mucosa.<br />

(b) They are pruritic.<br />

(c) They are described as white papules on an erythematous base.<br />

(d) They appear 1-3 days before the onset of the rash associated with measles.<br />

8. A herald patch is most closely associated with:<br />

(a)<br />

Psoriasis<br />

(b) Exfoliative dermatitis<br />

(c) Toxic shock syndrome<br />

(d) Pityriasis rosea<br />

9. In which of the fol lowing diseases are mucous membranes spared?<br />

(a)<br />

Stevens-Johnson syndrome<br />

(b) Erythema multiforme<br />

(c) Toxic epidermal necrolysis<br />

(d) Staphylococcal scalded skin syndrome<br />

830


CUTANEOUS DISORDERS: SELF-ASSESSMENT QUESTIONS<br />

10. Which of the fol lowing statements regarding oral acyclovir or valcyclovir in the treatment of herpes simplex is true?<br />

(a) It is recommended for the treatment of primary infections, especially urogenital infections.<br />

(b) It decreases viral shedding, accelerates healing, and shortens the duration of symptoms.<br />

(c) It may be used prophylactically in patients who have very frequent recurrences.<br />

(d) All of the above<br />

11. Which of the fol lowing is a characteristic of toxic epidermal necrolysis but not of staphylococcal-induced scalded<br />

skin syndrome?<br />

(a) Mucous membrane involvement<br />

(b) A positive Nikolsky sign<br />

(c) Often follows an upper respiratory infection<br />

(d) The rash results from a toxin-mediated infection.<br />

12. All of the following conditions classically produce lesions involving the soles and palms except:<br />

(a) Erythema multiforme<br />

(b) Varicella<br />

(c) Secondary syphilis<br />

(d) Hand-foot-and-mouth disease<br />

13. An immunocompromised patient presents with a painful vesicular rash that has a dermatomal distribution. The most<br />

appropriate management for this patient is:<br />

(a) Oral acyclovir<br />

(b) Analgesics and drying compresses<br />

(c) Systemic corticosteroids<br />

(d) IV acyclovir and hospital admission<br />

14. Which of the fol lowing statements regarding disseminated gonococcal infection is true?<br />

(a)<br />

Rash is typically distributed over the periarticular regions of the distal extremities.<br />

(b) It occurs more commonly in men than women.<br />

(c) Sending urethral or cervical gonorrhea cultures is contraindicated.<br />

(d) The diagnosis is unlikely in the absence of genitourinary symptoms.<br />

15. Potential complications of impetigo contagiosa include:<br />

(a) Rheumatic fever<br />

(b) Myocarditis<br />

(c) Scarlet fever<br />

(d) Acute poststreptococcal glomerulonephritis<br />

16. Which of the following statements regarding herpes zoster is accurate?<br />

(a) The cervical dermatome is the most commonly involved.<br />

(b) It is not communicable.<br />

(c) Lesions at the tip of the nose indicate possible ocular involvement.<br />

(d) Post-herpetic neuralgia, a complication of this infection, is most commonly seen in patients >30 years old.<br />

17. The lesions of erythema nodosum are most commonly found on the:<br />

(a) Arms<br />

(b) Face<br />

(c) Legs<br />

(d) Trunk<br />

ANSWERS<br />

I. d 4. C 7. b 10. d 13. d 16. C<br />

2. a 5. b 8. d 11. a 14. a 17. c<br />

3. c 6. a 9. d 12. b 15. d<br />

Use the pre-chapter multiple choice question worksheet (page xvi) to record and determine the percentage of correct<br />

answers for this chapter.<br />

831


CUTANEOUS DISORDERS<br />

I. INTRODUCTION<br />

Cutaneous disorders comprise a small portion (1 %) of the examination content. Pictorial identification is especially<br />

important. The lesions and rashes you are likely to be asked to identify are described in this chapter. You may also<br />

wish to consult a color dermatology atlas, such as Fitzpatrick's Color Atlas and Synopsis of Clinical Dermatology,<br />

Weston and Lane's Color Textbook of Pediatric Dermatology, or Knoop's The Atlas of Emergency Medicine. You<br />

should be familiar with all the color plates in the most recent edition ofTintinalli's textbook.<br />

II. GENERAL APPROACH TO THE PATIENT PRESENTING WITH<br />

A RASH<br />

A. Inquire about prodromal symptoms, time course, and antecedent events (eg, new medications).<br />

B. Note patient's age, immune status, past medical history, sexual history, medications, allergies, and<br />

presence/absence of toxicity.<br />

C. Examine the rash and determine its characteristics.<br />

1 . Appearance<br />

a. Macular - flat and ::cl cm<br />

b. Patchy - flat and > 1 cm<br />

c. Papular - raised and ::cl cm<br />

d. Plaque - raised and > 1 cm<br />

e. Maculopapular, nodular - dermal or subcutaneous solid lesion 1-2 cm<br />

f. Tumor - dermal or subcutaneous solid lesion >2 cm<br />

g. Vesicular - blister ::cl cm<br />

h. Bullous - blister >1 cm<br />

i. Pustules - small blister containing purulent material<br />

j. Scales or keratoses - built up epidermis<br />

k. Crusts, erosions - loss of part or all of epidermis<br />

I. Ulceration - loss of dermis or deeper<br />

2. Evolution: determine where it started and how it has spread.<br />

3. Distribution: note location of the rash, including involvement of mucous membranes, palms, and soles.<br />

4. Symptoms: determine if pruritic or painful; note any systemic symptoms (fever, odynophagia, malaise).<br />

5. Treatments: determine what, if anything, the patient has done to treat the rash (eg, applied topical steroids,<br />

zinc, or a neomycin-containing antibacterial ointment), because this might have changed the appearance of the<br />

rash or caused a secondary contact dermatitis.<br />

Ill. TOXICODENDRONS (POISON IVY, POISON OAK, POISON<br />

SUMAC}<br />

A. Overview<br />

1. Toxicodendron reactions are the most common cause of allergic contact dermatitis in North America.<br />

2. The development of a reaction requires prior sensitization (days or years earlier); these are type IV cellmediated,<br />

delayed hypersensitivity reactions.<br />

3. The antigen responsible for producing the skin reaction is urushiol, which is found in poison ivy, poison oak,<br />

poison sumac, and mangoes.<br />

B. Clinical presentation<br />

1. The rash is characterized by erythema (frequently in a linear configuration), papules, and intense pruritus,<br />

typically in exposed areas. The rash can progress to vesicles, or bullae with edema. It might weep fluid.<br />

832


CUTANEOUS DISORDERS<br />

Courtesy of David Effron, MO, FACEP<br />

2. The rash usually appears 5 hours to 15 days after an exposure (most often between 24 and 48 hours).<br />

3. A widely held misconception is that rupture of the vesicles spreads the rash, but the blister fluid does not<br />

contain antigen and therefore cannot spread the rash. Of note, different areas of the skin can react at<br />

different times, furthering this misconception.<br />

C. Management<br />

1. Mild dermatitis: topical calamine lotion, colloidal oatmeal or baking soda baths, topical steroids (0.1 %<br />

triamcinolone or 0.1 % betamethasone applied bid or tid), cool compresses, and an oral antihistamine<br />

2. Moderate to severe dermatitis<br />

a. Wet-to-dry compresses with water or aluminum acetate, eg, Burow solution<br />

b. Oral antihistamines (eg, hydroxyzine, diphenhydramine)<br />

c. Systemic corticosteroids are indicated for severe reactions. They should be continued for 2-3 weeks, with<br />

a gradual taper to avoid rebound. Too short a duration can result in rebound flare. Typical starting dose is<br />

prednisone 1-2 mg/kg/day x 1 week.<br />

3. Avoid topical antihistamines and antibiotics.<br />

4. Without treatment, symptoms resolve in 3-6 weeks.<br />

D. Prophylaxis<br />

1. After an exposure, the area should be washed immediately with cold soapy water. Urushiol is absorbed in<br />

10-30 minutes, so the allergen must be removed quickly.<br />

2. This practice can prevent dermatitis from developing, or at least lessen its severity.<br />

IV. DIAPER DERMATITIS<br />

A. Overview<br />

1. This is not a specific entity but rather a nonspecific term referring to several dermatologic conditions that occur<br />

in the diaper area. Be able to distinguish irritant, contact dermatitis, and Candida diaper rashes.<br />

2. The most important inciting factor is constant moisture. Other factors are diaper detergents, disinfectants, an<br />

alkaline pH, fecal material, intestinal enzymes, and friction caused by the diaper.<br />

3. The effects of local irritants are amplified by infrequent diaper changes, inadequate skin cleansing, and<br />

occlusive diapers.<br />

4. Incidence is highest in babies 9-12 months old, rarely seen at age


CUTANEOUS DISORDERS<br />

Courtesy of David Effron, MD, FACEP<br />

2. The involved skin may become hyperpigmented after resolution.<br />

3. Diaper dermatitis can be a manifestation of the following conditions:<br />

a. Atopic or seborrheic dermatitis: a clue is the presence of concurrent facial lesions (atopic dermatitis) or<br />

scalp lesions (seborrheic dermatitis).<br />

b. Primary irritant contact dermatitis: from ammonia and bacterially produced putrefactive enzymes (the<br />

odors are characteristic and the skin folds are notably spared) or from a new soap, shampoo, or laundry<br />

detergent.<br />

c. Secondary infection with Candida a/bicans: a clue is the classic appearance of the rash (moist, beefyred<br />

plaques with wellmarked edges in association with satellite lesions and skin fold involvement) and a<br />

duration >3 days.<br />

C. Management<br />

1 . General measures<br />

a. Discontinue use of plastic or rubber occlusive diaper pants.<br />

b. Use super-absorbent disposable diapers or cloth diapers, and change them frequently.<br />

c. Avoid the use of harsh cleansing agents, even "baby wipes" and soaps (which remove protective skin oils);<br />

use tepid water, and pat dry.<br />

d. Leave diapers off for extended periods of time or dry diaper area with cool hairdryer.<br />

e. Protect the skin from maceration with a barrier cream such as commercial zinc oxide diaper rash creams or<br />

petrolatum ointment. Do not use baby powder with cornstarch, which is metabolized by bacteria.<br />

2. Additional specific measures<br />

a. For mild inflammation secondary to atopic, seborrheic, or contact dermatitis, use 1 % hydrocortisone cream<br />

with a barrier, such as zinc oxide ointment or petrolatum gel (to prevent it from washing off).<br />

b. If the rash is more severe or persists >3-4 days, or if the patient has overt signs of Candida infection, add<br />

nystatin, clotrimazole, or miconazole cream for 2-3 weeks.<br />

V. ECZEMA<br />

A. Overview<br />

1. A pruritic, chronic, relapsing atopic dermatitis: "the itch that rashes"<br />

2. 85% of cases appear by age 5<br />

3. Familial predominance<br />

4. Associated with food allergies and asthma<br />

B. Clinical presentation<br />

1. Children: face, scalp, and torso are most commonly affected<br />

2. Adults: flexure aspects of the extremities<br />

3. Rash is papules and plaques that are intensely pruritic, scaly, and erythematous.<br />

4. Dark-skinned patients could have hypo- or hyperpigmented areas.<br />

834


CUTANEOUS DISORDERS<br />

C. Management<br />

1. Topical corticosteroids should be applied to the affected areas.<br />

2. Minimize showers; avoid bathing to help maintain skin hydration.<br />

3. Use antihistamines for pruritus.<br />

4. Topical emollients or petroleum jelly can be applied to damp skin to maintain moisture.<br />

VI. PSORIASIS<br />

A. Overview<br />

1. An intermittent, chronic inflammatory disease that can develop at any age<br />

2. Affects approximately 2% of the world's population<br />

3. Chronic plaque psoriasis can affect 1 %-90% of the body surface area.<br />

4. Inverse psoriasis is a variant that affects intertriginous or thin-skinned areas (eg, genitals, eyelids).<br />

5. Pustular psoriasis is a rare variant characterized by diffuse erythematous papules.<br />

6. Guttate psoriasis is seen in children/young adults with sores that are shaped like a water drop and are<br />

commonly triggered by a bacterial infection like a streptococcal pharyngitis. Scales tend to be less thick, and<br />

resolution is expectant but relapses can occur.<br />

B. Clinical presentation<br />

1. Sharply demarcated thick, silver, scaly, mildly pruritic plaques generally affecting the scalp, elbow, and knees<br />

2. Can also affect the hands, feet, trunk, or nails<br />

3. Removing plaques causes pinpoint bleeding (Auspitz sign).<br />

C. Management<br />

1. High-potency topical corticosteroids (eg, fluocinonide applied every 12 hours) if disease is localized. Do not<br />

use on areas of thin skin or near the eyes.<br />

2. Tar-based shampoos for scalp lesions<br />

3. The use of systemic steroids can lead to severe exacerbations of the disease when therapy is discontinued and<br />

should therefore be avoided.<br />

4. Requires long-term management by a dermatologist or primary care physician.<br />

VII. EXFOLIATIVE DERMATITIS/EXFOLIATIVE ERYTHRODERMA<br />

SYNDROME<br />

A. Overview<br />

1. Widespread erythematous pruritic dermatitis<br />

2. Etiology<br />

a. Idiopathic is most common form.<br />

b. Drug-induced (>50 drugs have been implicated)<br />

c. Underlying malignancy (cutaneous lymphoma, leukemia, or other lymphoreticular malignancy) or<br />

immunosuppression (HIV)<br />

d. Preexisting dermatoses (eg, psoriasis, eczema, seborrhea)<br />

e. Allergic contact dermatitis<br />

B. Clinical presentation<br />

1. Shiny, erythematous, pruritic rash with scaling; begins localized then spreads and generalizes; spares the palms<br />

and soles<br />

2. Classic clinical scenario: A 57-year-old man who has a past medical history of seizures and who is taking a<br />

new medication appears ill and complains of itching, a chilly sensation, and "tightness" of the skin. He has<br />

a low-grade fever and is hypotensive with tachycardia. On examination, a scaly, warm, erythematous rash is<br />

found to be covering >50% of his body surface area. There is no oral involvement, and the rash is not tender to<br />

the touch. Nikolsky sign is negative.<br />

835


CUTANEOUS DISORDERS<br />

Courtesy of David Effron, MD, FACEP<br />

3. Other findings can include fever or hypothermia, dehydration, lymphadenopathy, hepatosplenomegaly, lower<br />

extremity edema, or gynecomastia.<br />

4. Scratching can result in lichenification and erosions.<br />

5. Because of increased blood flow to the skin, the patient might have high output heart failure.<br />

C. Diagnostic evaluation<br />

1. Consider CBC, serum chemistries, liver function tests, erythrocyte sedimentation rate, urinalysis, and HIV<br />

testing in the search for systemic causes.<br />

2. Because this disorder is usually the result of an underlying cutaneous disease, a systemic disease, or a response<br />

to a drug or chemical, patients should be admitted for a diagnostic evaluation. The mortality rate is as high as<br />

30%.<br />

3. Obtain skin biopsy (lymph node biopsy if significant lymphadenopathy is present)<br />

D. Differential diagnosis<br />

1. Erythema multiforme<br />

2. Toxic epidermal necrolysis<br />

3. Toxic shock syndrome<br />

4. Staphylococcal scalded skin syndrome<br />

5. Kawasaki disease (children)<br />

E. Management<br />

1. Goal is to correct/eliminate the underlying cause while providing symptomatic relief and maintaining skin<br />

moisture.<br />

2. Stop new medications, if at all possible.<br />

3. Antihistamines<br />

4. Topical steroids covered with an occlusive dressing and continued for weeks or months.<br />

5. Warm water baths with bath oils and skin emollients are also helpful.<br />

6. Severe or resistant cases are treated with systemic corticosteroids.<br />

VIII. ERYTHEMA MULTIFORME (EM}<br />

A. Overview<br />

1. There has been controversy regarding the nomenclature of EM through the years, but it is now generally<br />

agreed that there are two forms: erythema multiforme minor and erythema multiforme major (or bullous EM)<br />

2. Occurs as a response to infection<br />

a. Most commonly HSV-1 or HSV-2<br />

b. Varicella, parapoxvirus, adenovirus, coxsackievirus, influenza, HIV, hepatitis, M pneumoniae, Salmonella,<br />

streptococci<br />

B. Clinical presentation<br />

1. EM minor: target lesions or raised papules with acral distribution; no mucous membrane involvement<br />

836


CUTANEOUS DISORDERS<br />

Courtesy of David Effron, MO, FACEP<br />

2. EM major (or bullous EM):


CUTANEOUS DISORDERS<br />

IX. STEVEN-JOHNSONS SYNDROME AND TOXIC EPIDERMAL<br />

NECROLYSIS<br />

A. Overview<br />

1. A drug reaction that causes sloughing of skin and mucous membranes<br />

2. Previously thought to be a variant of erythema mu ltiforme<br />

3. Now the spectrum of Stevens-Johnson syndrome (SJS) __,. SJS/toxic epidermal necrolysis (SJS/TEN) overlap__,.<br />

TEN is recognized as its own entity.<br />

4. Death rate: SJS, 1 %-5%; SJS/TEN, 6%; TEN, 25%-35%<br />

5. Worse outcome with increased BSA involvement, advanced age, malignancy, tachycardia, BUN > 10 mEq/L,<br />

acidosis, and hyperglycemia.<br />

6. Common inciting agents are NSAIDs, antibiotics (sulfonamides, penicillins, cephalosporins), anticonvulsants,<br />

and allopurinol.<br />

B. Clinical presentation<br />

1. SJS:


CUTANEOUS DISORDERS<br />

Courtesy of David Effron, MD, FACEP<br />

X. ERYTHEMA NODOSUM<br />

A. Overview<br />

1. An inflammatory/immunologic reaction of the panniculus (fat) that most commonly affects women 15-30 years old<br />

2. Streptococcal infection is the most common precipitant in children.<br />

3. Sarcoidosis, inflammatory bowel disease, malignancy, and drug reaction (particularly to birth control pills and<br />

sulfonamides) are the most common triggers in adults.<br />

B. Clinical presentation<br />

1. Characterized by painful, deep-seated nodules (2-5 cm) on the lower extremities (bilateral but not symmetrical)<br />

2. Nodules are initially bright red and slightly elevated; then, over 1-2 weeks, they become violet or maroon and<br />

less raised.<br />

3. Nodules are typically located in the pre-tibial region but can also be seen on the arms and (rarely) the face.<br />

4. Arthralgias (particularly of the ankles) often precede the cutaneous eruption.<br />

5. Single lesions generally last about 2 weeks, but new lesions may continue to recur up to 3-6 weeks.<br />

Courtesy of David Effron, MO, FACEP<br />

839


CUTANEOUS DISORDERS<br />

C. Management<br />

1. Identify and treat the underlying cause; this condition is self-limited (3-6 weeks) if the cause can be eliminated.<br />

2. Symptomatic measures include bed rest and leg elevation, compressive dressings, NSAIDs, and colchicine.<br />

XI. PITYRIASIS ROSEA<br />

A. Overview<br />

1. A self-limited rash, typically lasting 4-8 weeks<br />

2. Unclear cause, presumed to be viral<br />

3. May be preceded by a flu-like prodrome<br />

4. Risk of premature birth if it occurs during pregnancy<br />

B. Clinical presentation<br />

1. A single salmon-colored oval patch or plaque (the herald patch) appears first.<br />

2. Days to weeks later, salmon-colored circular or oval patches and plaque with scaling appear in a Christmas<br />

tree pattern.<br />

3. Rash may be asymptomatic or pruritic.<br />

Courtesy of Laura J. Bontempo, MD, MEd, FACEP<br />

C. Management<br />

1. Self-limited; no treatment required.<br />

2. Antihistamines, if needed, for pruritus.<br />

XII. CELLULITIS/ERYSIPELAS<br />

A. Overview<br />

1. Cellulitis is a local soft-tissue inflammatory reaction secondary to bacterial invasion of the skin.<br />

2. Although cellulitis is classically associated with comorbidities such as diabetes or peripheral vascular disease,<br />


CUTANEOUS DISORDERS<br />

Courtesy of David Effron, MO, FACEP<br />

2. Systemic involvement such as fever and bacteremia can occur and is most common in immunosuppressed<br />

patients.<br />

3. Erysipelas<br />

a. A specific type of cellulitis caused by group A streptococci, usually Streptococcus pyogenes<br />

b. Characterized by a bright red color, sharp margins, and tender plaques<br />

c. Commonly affects the face and lower extremities (especially in patients with venous insufficiency)<br />

d. Generally affects patients >60 years old and is associated with fever, chills, and malaise<br />

Courtesy of David Effron, MO, FACEP<br />

C. Diagnostic evaluation<br />

1. Blood cultures, tissue biopsies, and swabs are not routinely recommended.<br />

2. Blood cultures are recommended for patients with cellulitis who are immunocompromised, who suffered an<br />

immersion injury, and who were bitten by an animal.<br />

D. Management<br />

1. Elevate the affected area as much as possible.<br />

2. Cellulitis associated with penetrating trauma, MRSA infection, a history of MRSA, IV drug abuse, or systemic<br />

infection should be treated with vancomycin.<br />

3. Cellulitis in an immunocompromised host, in a patient with neutropenia, after an animal bite, or after an<br />

immersion injury should be treated with vancomycin plus either piperacillin/tazobactam, imipenem, or<br />

meropenem.<br />

4. Duration of treatment is usually 5 days but may be extended if infection is not improving.<br />

5. Outpatient treatment is acceptable for immunocompetent patients without systemic infection.<br />

841


CUTANEOUS DISORDERS<br />

6. Systemic corticosteroids can be considered as adjunctive treatment for erysipelas in nondiabetic adult<br />

patients.<br />

7. Erysipelas: treat with penicillin G or VK, dicloxacillin, or clindamycin (if the patient is allergic to penicillin.<br />

8. In cases of exposure to fresh, salt, or brackish water, treat with doxycycline, fluoroquinolones, or<br />

ceftazidime.<br />

9. Treat patients with cellulitis after a mammalian bite with amoxicillinclavulanate. If penicillin-allergic, treat<br />

with fluoroquinolone + clindamycin or TMP-SMX + metronidazole.<br />

XIII. IMPETIGO<br />

A. Overview<br />

1. Superficial gram-positive bacterial skin infection that can occur in any group but is most prevalent in children<br />


CUTANEOUS DISORDERS<br />

Courtesy of David Effron, MD, FACEP<br />

b. Lesions<br />

(1) The lesions are flaccid, thin-walled bullae (1-3 cm in diameter) that contain a purulent material. The<br />

bullae rupture, leaving shiny, rounded, erythematous erosions with peeling edges and crusting ( 11 coin 11<br />

lesions).<br />

(2) There is no surrounding erythema.<br />

(3) Associated regional lymphadenopathy is rare.<br />

(4) Nikolsky sign is negative.<br />

3. Ecthyma is a condition very close to impetigo that is usually caused by streptococci. The lesions are typically<br />

on the legs and begin as pustules that erode and then ulcerate. The infection is usually deeper than impetigo, is<br />

painful, and has a high rate of scarring.<br />

C. Diagnostics: not usually needed<br />

D. Management<br />

1. Wound care: remove crust with warm water or hydrogen peroxide.<br />

2. Antibiotics (oral or topical)<br />

a. Usually treated with a 7-day course of an oral antibiotic effective against both staphylococci and streptococci<br />

(1) 13-lactamase-resistant antibiotics are first-line treatment (dicloxacillin, cephalexin).<br />

(2) Doxycycline, clindamycin, or TMP-SMX for suspected MRSA<br />

(3) Clindamycin or azithromycin if patient is allergic to penicillin<br />

b. A topical antibiotic that can be used alone for localized infections.<br />

(1) Mupirocin 2% bid x 5 days<br />

(2) Retapamulin ointment bid<br />

3. Good hand washing and personal hygiene, and not sharing linens (eg, towels) limit spread of the infection<br />

to others.<br />

E. Complications (focal and systemic)<br />

1. Spread (both locally and to others); children should not return to school until after 24 hours of antibiotic<br />

treatment.<br />

2. Acute poststreptococcal glomerulonephritis (incidence


CUTANEOUS DISORDERS<br />

XIV. NECROTIZING FASCIITIS<br />

A. Overview<br />

1. A bacterial infection that tracks along the fascial planes; surface erythema often underrepresents the extent of<br />

infection.<br />

2. Classically caused by group A ~-hemolytic streptococci but may also be caused by 5 aureus, Vibrio vulnificus,<br />

Enterobacteriaceae, and Bacteroides.<br />

3. V vulnificus infections are related to exposure to seawater.<br />

4. Aeromonas hydrophila infections are associated with exposure to brackish water, soil, wood, and ditches.<br />

5. Risk factors include skin trauma, diabetes mellitus, malignancy, and alcoholism.<br />

6. The mortality rate is high.<br />

7. Fournier gangrene is necrotizing fasciitis of the groin, scrotum, and lower abdomen.<br />

B. Clinical presentation<br />

1. Patients are ill appearing with systemic symptoms, including fever, tachycardia, lethargy, and possibly<br />

hypotension.<br />

2. Pain and tenderness extend beyond the area of erythema and can be severe.<br />

3. Skin may have erythema resembling cellulitis, edema, hemorrhagic bullae, or necrosis depending on the stage<br />

of the disease.<br />

4. Skin findings progress quickly.<br />

C. Diagnostic evaluation<br />

1. CT or MRI may show edema along fascial planes or gas in the tissue; however, a negative imaging study does<br />

not exclude the diagnosis.<br />

2. Blood cultures can be helpful for long-term management.<br />

3. Definitive diagnosis is made through direct tissue sampling.<br />

D. Management<br />

1. Surgical debridement of the necrotic tissue is the mainstay of treatment.<br />

2. Broad-spectrum antibiotics are needed.<br />

a. Piperacillin-tazobactam plus vancomycin<br />

b. Doxycycline plus ceftazidime if V fulnificus is suspected<br />

c. Doxycycline plus ciprofloxacin if A hydrophila is suspected<br />

3. Admit to ICU.<br />

XV. CUTANEOUS ABSCESS<br />

A. Overview<br />

1. A cutaneous abscess is a localized collection of pus with associated pain, fluctuant mass, and erythema.<br />

2. Most abscesses (95%) contain bacteria, most commonly S aureus and streptococci.<br />

3. While abscesses are usually localized, the presence of fever or other systemic symptoms suggests possible<br />

bacteremia.<br />

4. A furuncle is a localized abscess associated with hair follicles. A carbuncle results if several furuncles<br />

coalesce and interconnect by sinus tracts; it may develop in areas of thick skin such as the back of the neck.<br />

Courtesy of David Effron, MD, FACEP<br />

844


CUTANEOUS DISORDERS<br />

8. Clinical presentation<br />

1. The development of cutaneous abscess depends on location. On extremities, the cause is usually minor<br />

trauma that damages the integrity of the epithelium. In intertriginous regions, abscesses are associated with<br />

obstructed apocrine sweat glands.<br />

2. Community-associated MRSA has emerged as the prevalent pathogen of skin and soft-tissue infections in<br />

urban areas. Furunculosis appears to be the strongest predictor of community-associated MRSA.<br />

C. Management<br />

1. Abscess size and depth can be evaluated by ultrasound.<br />

2. Incision and drainage are the mainstay of therapy.<br />

3. Consider antibiotics only if the patient is systemically ill, is immunosuppressed, has recurrent or multiple<br />

abscesses, or on follow-up evaluation is found to have not responded to incision and drainage alone.<br />

4. Packing of the abscess is not necessary.<br />

XVI. STAPHYLOCOCCAL SCALDED SKIN SYNDROME<br />

A. Overview<br />

1. A skin infection by exotoxin-producing S aureus of phage group 2<br />

2. Mainly affects children 60% and potentially higher if immunocompromised.<br />

B. Clinical presentation<br />

1. Often follows an upper respiratory infection or purulent conjunctivitis<br />

2. Tender erythema of the face (perioral area is classic), neck, or axillae that generalizes over the body within<br />

48 hours. Flaccid bullae develop and, within 48 hours, skin sloughs. Desquamation duration is 5 days.<br />

3. Crusting around the mouth and eyes and lip fissuring are also frequently present.<br />

4. In newborns, the entire skin surface may be involved (Ritter disease).<br />

Courtesy of David Effron, MD, FACEP<br />

5. Mucous membranes are not involved, which helps differentiate this syndrome from toxic epidermal necrolysis.<br />

6. Tense pressure applied to the bullae can result in extension of the bullae because of an unstable epidermis;<br />

this is a positive Nikolsky sign.<br />

7. The cleavage plane is intraepidermal; therefore, only the superficial layers of the epidermis are shed, which<br />

also differentiates it from toxic epidermal necrolysis.<br />

8. Lesions usually resolve in 2 weeks without scarring.<br />

C. Management<br />

1. Antibiotics to treat 5 aureus: ~-lactamase/penicillinase-resistant penicillin such as IV oxacillin/nafcillin is the<br />

treatment of choice. Azithromycin IV is an alternative therapy. Vancomycin may be considered if MRSA is a<br />

concern.<br />

2. Steroids are contraindicated (they may exacerbate the illness).<br />

3. Hospitalization for hydration and skin care is indicated for most patients (especially infants).<br />

4. Treatment is similar to that for thermal burn patients; therefore, consider burn unit admission.<br />

845


CUTANEOUS DISORDERS<br />

XVII. PEMPHIGUS VULGARIS<br />

A. Overview<br />

1. An autoimmune bullous disease of skin and mucous membranes<br />

2. A pathologic lgG attacks the intercellular binding substance of epidermal keratinocytes, disrupting cell-cell<br />

adhesion and resulting in the formation of bullae<br />

3. Most commonly affects patients 40-60 years old; equal gender incidence<br />

4. Associated with the presence of other autoimmune diseases<br />

5. Mortality rate is currently 5%; however, before the use of corticosteroid therapy, this disease was almost<br />

uniformly fatal.<br />

B. Clinical presentation<br />

1. Classic clinical scenario: A 55-year-old patient with a past medical history of myasthenia gravis presents with a<br />

multiple fluid-filled bullae and painful, crusted ulcers. She has a recent history of several months of oral lesions<br />

of unknown etiology. Physical examination is significant for a positive Nikolsky sign.<br />

2. Mucous membrane involvement (most commonly of the mouth) typically precedes development of cutaneous<br />

lesions by several months.<br />

3. The cutaneous lesions are flaccid vesicles and bullae that rupture easily, leaving behind superficial erosions<br />

and crusted ulcerations. They are painful and can be seen anywhere.<br />

4. Nikolsky sign is positive.<br />

5. Large areas of denuded skin increase the risk of secondary infection.<br />

C. Diagnostic evaluation<br />

1. Skin biopsy with Tzanck smear: a positive test (the presence of acantholytic epidermal cells with large nuclei in<br />

condensed cytoplasm) is suggestive but not specific.<br />

D. Management<br />

1. Dermatology consult<br />

2. High-dose oral steroids (prednisone 1 mg/kg daily)<br />

3. Concomitant immunosuppressive therapy (eg, azathioprine, mycophenolate)<br />

4. Antibiotics to treat secondary bacterial infection<br />

5. Local therapy (eg, topical steroids)<br />

6. Hospitalization for patients with extensive bullae and erosions<br />

7. Other treatments include dapsone, rituximab, IV immunoglobulin, and plasmaphoresis.<br />

XVIII. BULLOUS PEMPHIGOID<br />

A. Overview<br />

1. An lgG autoimmune, chronic disease of the elderly<br />

2. Associated with other autoimmune diseases<br />

3. Duration may be months to years<br />

B. Clinical presentation<br />

1. Pruritic tense bullae on normal skin or an erythematous base<br />

2. Bullae filled with clear or blood-tinged fluid<br />

3. Most often seen on lower abdomen and flexure surfaces of the thighs and forearms<br />

4. Typically no mucosal involvement<br />

5. No Nikolsky sign (lesions do not erode or spread with light pressure)<br />

C. Management<br />

1. Localized disease: high-potency topical corticosteroids (eg, clobetasol)<br />

2. Extensive disease: prednisone, 1 mg/kg daily<br />

3. Dermatology might consider IVIG for resistant cases.<br />

846


CUTANEOUS DISORDERS<br />

XIX. HERPES SIMPLEX<br />

A. Overview<br />

See also herpetic whitlow, page 402; genital herpes, pages 448-449; and herpetic keratitis, pages 117-118.<br />

1. Two variants: type 1 (HSV-1) and type 2 (HSV-2)<br />

2. Most HSV-1 infections involve nongenital areas: the lips (herpes labialis), the eyes (herpes keratitis), and<br />

the fingers (herpetic whitlow). However, HSY-1 is responsible for 10%-30% of infections involving the<br />

urogenital area.<br />

Courtesy of David Effron, MD, FACEP<br />

3. Most HSV-2 infections involve the urogenital area: the vulva, vagina, cervix, and perineum in females and the<br />

penile shaft, glans penis, and anal area in males.<br />

Courtesy of David Effron, MD, FACEP<br />

4. Spread of infection occurs through direct contact of the vesicular fluid with mucous membranes or abraded<br />

skin, or through contact with body fluids during viral shedding.<br />

5. Incubation period for primary HSY infection is 2-20 days (average 6 days); resolution occurs within 2-6 weeks.<br />

6. Once a person is infected, the virus remains in the dorsal root ganglia. Symptoms occur with viral<br />

reactivation. Recurrent disease occurs in 30%-50% of oral and 95% of genital HSY infections.<br />

B. Clinical presentation<br />

1. The skin lesions begin as groups of vesicles (uniform in size) on an erythematous base that later rupture and<br />

form a crusted ulcer. The lesions develop at the site of inoculation, have a nondermatomal distribution, and<br />

are exquisitely tender.<br />

2. Symptomatic primary HSY infections are usually more severe than recurrent infections and are often<br />

accompanied by systemic signs and symptoms such as fever, malaise, and regional adenopathy. Systemic<br />

symptoms are usually absent in recurrent episodes.<br />

3. Many primary HSY-1 infections are asymptomatic (but they can be severe). Herpetic gingivostomatitis<br />

presents with fever, malaise, painful gums, pain with eating, and possibly pharyngitis.<br />

4. Primary HSY-2 genital infection presents with headache, fever, malaise, pain, and dysuria. Genital herpes<br />

often has urethral involvement and may present with severe dysuria and urinary retention.<br />

5. In recurrent herpes (HSY-1 or HSY-2), a prodrome of local pain, burning, itching, and hyperesthesia usually<br />

precedes development of visible cutaneous lesions.<br />

847


CUTANEOUS DISORDERS<br />

C. Diagnostic evaluation<br />

1. Tzanck smear (look for multinucleated giant cells), viral culture (highest yield when lesions are vesicular),<br />

PCR, or a direct fluorescent antibody test can be used to confirm the diagnosis.<br />

D. Management<br />

1. Antiviral medications decrease viral shedding, accelerate healing, and shorten the duration of symptoms but<br />

do not eliminate the herpes infection or affect the frequency of recurrence. Options are listed below.<br />

a. Oral acyclovir 400 mg tid for 7-10 days<br />

b. Famciclovir 250 mg tid for 7-10 days<br />

c. Valacyclovir 1 g bid 7-10 days<br />

2. IV acyclovir is reserved for patients with severe disease or complications requiring hospitalization; dosage is<br />

5-10 mg/kg IV every 8 hours for 5-7 days or until clinical resolution.<br />

3. Episodic treatment (treatment with each outbreak) should be started at the onset of the initial prodromal<br />

symptoms or within 1-2 days after lesions appear.<br />

4. Continuous daily suppressive therapy with oral antivirals is occasionally recommended for patients with<br />

frequent recurrences (>6 per year); it decreases the recurrence rate by up to 80%.<br />

5. Analgesia is often needed, particularly for primary infections. Topical viscous lidocaine is very effective, but<br />

oral agents may also be required.<br />

6. Prompt ophthalmologic consult should be obtained for patients with ocular lesions.<br />

7. Advise patients to avoid skin-to-skin contact when lesions or prodromal symptoms are present.<br />

E. Complications<br />

1. Disseminated HSV can occur in neonates and adults with AIDS, malignancy, immunosuppression, or<br />

dermatitis and may result in multisystem involvement. Disseminated infection can lead to pneumonitis.<br />

2. Patients with HSV are at risk of bacterial superinfection, radiculoneuropathy, Bell's palsy, encephalitis, and<br />

hepatitis.<br />

3. Eczema herpeticum refers to an acute infection of herpes simplex involving the skin lesions of atopic<br />

dermatitis. Lesions are typically vesicular but may umbilicate and ulcerate. Dissemination may occur. Most<br />

severe cases are in the very young or immunosuppressed and require hospitalization for IV antiviral therapy.<br />

XX. HERPES ZOSTER {SHINGLES)<br />

A. Overview<br />

1. Herpes zoster is caused by reactivation of latent varicella-zoster virus (present in the dorsal ganglia since the<br />

initial infection with chickenpox).<br />

2. The elderly and immunocompromised are most commonly affected.<br />

B. Clinical presentation<br />

1. Classic clinical scenario: A 77-year-old woman has a history of 3 days of constant right-sided chest pain in a<br />

band-like distribution. She reports recent increased stress from the loss of a pet. Her past medical history is<br />

significant for having had varicella as a child. On examination, a vesicular rash in a dermatomal distribution<br />

is seen.<br />

2. Burning pain or paresthesia develops in a dermatomal distribution 3-5 days (range 1-21 days) before the<br />

skin eruption.<br />

3. Rash appears as herpetiform clusters of vesicles (which vary in size) on an erythematous, edematous base,<br />

with most of the vesicles concentrated on the proximal end of the dermatome.<br />

4. The lesions are almost always unilateral in distribution and limited to one or two contiguous dermatomes.<br />

The most commonly involved dermatomes are the thoracic (>50%), lumbosacral (10%-20%), and trigeminal<br />

(10%-20%).<br />

848


CUTANEOUS DISORDERS<br />

Courtesy of David Effron, MD, FACEP<br />

5. Disseminated disease is defined as >20 lesions outside the affected or immediately adjacent dermatome. It is<br />

primarily a disease of immunosuppressed patients.<br />

6. Hutchinson sign<br />

a. Lesions at the tip of the nose signal possible eye involvement (the virus travels along the nasociliary<br />

branch of the trigeminal nerve, which is the ophthalmic division).<br />

b. Antiviral therapy is necessary to prevent blindness.<br />

7. Ramsay Hunt syndrome<br />

a. Facial palsy (similar to Bell palsy) associated with vesicles in the ear canal and on the pinna, tympanic<br />

membrane, and/or the pharynx (often associated with involvement of cranial nerve VIII)<br />

b. Presents with vertigo, ipsilateral facial weakness, and ipsilateral deafness<br />

C. Diagnostic evaluation<br />

1. Tzanck smear (looking for multinucleated giant cells), PCR, or direct fluorescent antibody can confirm the<br />

diagnosis.<br />

D. Management<br />

1. Cover the affected area with drying compresses (aluminum acetate, potassium permanganate).<br />

2. Analgesics<br />

3. Oral antiviral therapy for immunocompetent patients if it can be administered within 72 hours after the<br />

eruption.<br />

a. Shortens the length and duration of symptoms and could decrease the incidence of post-herpetic<br />

neuralgia<br />

b. Acyclovir 800 mg 5 times a day for 7-10 days<br />

c. Valacyclovir 1,000 mg tid for 7 days<br />

d. Famciclovir 500 mg tid for 7 days<br />

4. IV antivirals for immunocompromised patients (including patients receiving steroids or chemotherapy), those<br />

with disseminated zoster, and those with eye involvement.<br />

a. Decreases the rate of dissemination as well as morbidity in these groups<br />

b. Acyclovir 1 0 mg/kg tid for 5-7 days<br />

c. Foscarnet 40 mg/kg IV tid for 10 days; used if patient is resistant to acyclovir<br />

5. Systemic corticosteroids may acute pain but have not been proved to prevent post-herpetic neuralgia.<br />

6. Immediate ophthalmologic consult is indicated if the ophthalmic branch of the trigeminal nerve is involved.<br />

(See also herpes zoster ophthalmicus and its management, page 118.)<br />

7. Herpes zoster is a communicable disease; nonimmune contacts can develop chickenpox (varicella).<br />

a. Patients with herpes zoster should be advised to avoid contact with people who have not had chickenpox<br />

(the nonimmune), pregnant women, and immunocompromised patients.<br />

b. Susceptible immunocompromised patients who are exposed should be given varicella-zoster immune<br />

globulin within 72 hours after exposure.<br />

849


CUTANEOUS DISORDERS<br />

E. Complications<br />

1. Post-herpetic neuralgia (persistent pain in the affected dermatome): more common in the elderly, women,<br />

and those with greater rash severity; risk is >40% in patients >60 years old.<br />

2. Encephalitis due to herpes zoster: appears within 2 weeks after lesion onset.<br />

3. Bacterial superinfection<br />

4. Reactivation or recurrence (immunocompromised host)<br />

5. Cranial and peripheral nerve palsies<br />

6. Acute retinal necrosis (may cause blindness in HIV patients)<br />

XXI. APTHOUS ULCERS<br />

A. Overview<br />

1. An acute, recurrent, painful ulcerative disease of the mouth<br />

2. Commonly referred to as canker sores<br />

3. Highest incidence during second through fourth decades<br />

4. The cause is unclear but is believed to be noninfectious.<br />

B. Clinical presentation<br />

1. Painful ulcers with a surrounding erythematous halo and a yellow pseudomembrane at their base<br />

2. Present on the inner lips and buccal surfaces of the mouth; not on the hard palate or gingiva<br />

3. Ulcers can vary in size from 0.1 to >1 cm; may present in batches<br />

C. Management<br />

1. Topical corticosteroid gel (eg, clobetasol, triamcinolone)<br />

2. Symptomatic treatments include topical anesthetics (lidocaine gel), anesthetic mouthwash, sucralfate, and pain<br />

medicines.<br />

XXII. MOLLUSCUM CONTAGIOSUM<br />

A. Overview<br />

1. A contagious rash caused by a DNA poxvirus<br />

2. Spread by sexual transmission or direct contact<br />

3. Host may autoinoculate, causing the rash to spread<br />

4. Usually self-limited, with duration of months to 2 years<br />

B. Clinical presentation<br />

1. Smooth, firm, white or skin-colored papules with central umbilication<br />

2. Adults: lesions typically appear on the lower abdomen and perineum<br />

C. Management<br />

1. Self-limited disease<br />

2. Cryotherapy<br />

XXIII. FUNGAL DISORDERS<br />

A. Overview<br />

1. A group of fungal (including yeast) disorders that affect various parts of the body; disorders are generally<br />

named by the body area affected.<br />

2. Dermatophyte is the term used for three types of fungi that cause skin diseases.<br />

3. Fungal infections generally create an annular, erythematous, scaling plaque rash.<br />

4. Candida albicans is responsible for 80% of yeast infections.<br />

5. Infections are more prevalent in warmer environments.<br />

850


CUTANEOUS DISORDERS<br />

B. Clinical presentation<br />

1. Tinea capitis<br />

a. A dermatophyte infection of the scalp characterized by areas of alopecia, broken hairs ("black dots"),<br />

and peripheral scaling<br />

Courtesy of David Effron, MD, FACEP<br />

b. Seen most commonly in children, with increased frequency among black Americans<br />

c. Uncommonly, an intense inflammatory response results in a boggy, tender, indurated plaque with<br />

pustules and alopecia referred to as a kerion. This lesion may result in permanent scarring and hair loss.<br />

2. Tinea corporis<br />

a. A dermatophyte of the skin, commonly referred to as ringworm<br />

b. Characterized by annular, erythematous, scaling plaques with central clearing; erythematous border may<br />

demonstrate papules and vesicles<br />

c. Can spread on its host through self-inoculation<br />

3. Tinea barbae<br />

a. A dermatophyte infection of the beard area characterized by areas of plaques and patches<br />

b. Can resemble bacterial folliculitis<br />

c. Most common in adult men<br />

4. Tinea pedis<br />

a. A dermatophyte infection of the feet seen most often in young adult men<br />

b. Three main types of infection: interdigital (most common), moccasin, and vesicular<br />

5. Tinea manuum<br />

a. A dermatophyte infection of the palmar surface of the hands<br />

b. Characterized by dry scales and minimal inflammation<br />

6. Tinea cruris<br />

a. A dermatophyte infection of the groin common in adult men<br />

b. Characterized by erythema with scaling borders of the inner thighs and buttocks, avoiding the scrotum<br />

and penis<br />

7. Tinea versicolor<br />

a. A superficial yeast infection caused by Malassezia globosa<br />

b. Characterized by superficial scaling patches of various colors<br />

Courtesy of David Effron, MD, FACEP<br />

851


CUTANEOUS DISORDERS<br />

8. Candida! intertrigo<br />

a. Infections that usually favor moist occluded areas such as skin folds and diaper areas<br />

b. Characterized by erythema and maceration as well as surrounding erythematous papules (satellites)<br />

Courtesy of David Effron, MD, FACEP<br />

c. Predisposing factors include previous antibiotic or corticosteroid use, as well as immunocompromised<br />

states and obesity.<br />

9. Oral or esophageal candidiasis (thrush)<br />

a. Can affect immunocompetent patients but is more common in immunocompromised patients<br />

b. Patients complain of burning soreness in their mouth.<br />

c. Curd-like white papules and plaques that wipe off are seen on the mucous membranes of the mouth.<br />

10. Sporotrichosis<br />

a. A mycotic infection, also known as Rose Gardener disease, caused by the fungus Sporothrix schenckii,<br />

which is common in soil and plants<br />

b. Result of traumatic inoculation that results in a local inflammatory response involving lymphatics;<br />

however, dissemination may occur and result in systemic illness.<br />

c. Characterized by painless nodules in lymphatic distribution, especially on upper extremities, which<br />

appear over weeks.<br />

B. Diagnostic evaluation<br />

1. Rash scrapings may be prepared with potassium hydroxide and viewed under a microscope; a positive<br />

examination reveals hyphae. Yeast such as Candida demonstrate pseudohyphae and spores.<br />

2. Wood's light examination is of less help, because many dermatophytes do not fluoresce.<br />

C. Management<br />

1. Topical imidazole antifungal agents (eg, clotrimazole 1 %) and allylamine antifungal agents (eg, naftifine 1 %)<br />

are indicated for most cutaneous fungal infections. Treatment should continue for an additional week after<br />

lesions have cleared.<br />

2. Tinea capitis<br />

a. Oral griseofulvin for 8 weeks<br />

b. A topical shampoo containing ketoconazole is used to reduce contagion.<br />

c. Kerion: oral prednisone is indicated as well as oral cephalexin for bacterial superinfection.<br />

3. Sporotrichosis<br />

a. Oral itraconazole for 3-6 months<br />

b. IV amphotericin B is reserved for disseminated cases.<br />

4. Tinea versicolor<br />

a. 2.5% selenium shampoo<br />

b. Recurrence rates are as high as 50%.<br />

5. Candida<br />

a. Clotrimazole troches 5 times a day or nystatin rinse 5 ml 4 times a day for thrush<br />

b. lntertrigo: nystatin or miconazole cream or powder<br />

6. All lesions should be kept clean and dry. Astringents such as 5% aluminum acetate aid in drying.<br />

7. Do not treat with topical steroids.<br />

852


CUTANEOUS DISORDERS<br />

XXIV. PRESSURE ULCERS (DECUBITUS ULCERS}<br />

A. Overview<br />

1. The pressure of the individual's weight against a hard surface causes tissue ischemia.<br />

2. The normal instinct to shift positions is impaired by mobility issues, sedation, contraction/spasticity, or sensory<br />

loss.<br />

3. Other contributors include malnutrition, anemia, comorbidities, and improper skin care.<br />

4. Patients at risk include the elderly, those who are hospitalized or in an extended-care facility, and those with<br />

neurologic impairments.<br />

5. The Braden scale is used to assess patients at risk of developing pressure ulcers.<br />

B. Clinical presentation<br />

1. Classic clinical presentation: A 67-year-old woman from an extended-care facility has severe multiple sclerosis<br />

and a fever. On examination, a large, malodorous, necrotic decubitus ulcer is noted over her sacrum. On<br />

probing, the tract is found to extend to the sacrum. Results of laboratory studies include increased WBC count,<br />

erythrocyte sedimentation rate, and (-reactive protein. Radiographs reveal osteomyelitis.<br />

Courtesy of David Effron, MO, FACEP<br />

2. Occurs primarily over bony prominences on the hip/buttocks (70%) and lower extremities (25%)<br />

3. Muscle necrosis occurs before skin breakdown; a small breakdown of skin can herald a large underlying cavity.<br />

4. Stages of pressure ulcers<br />

a. Stage I: skin intact with nonblanching erythema<br />

b. Stage II: partial-thickness loss of skin; no necrotic tissue<br />

c. Stage Ill: full-thickness loss of skin extending into subcutaneous tissue with a crater-like appearance<br />

d. Stage IV: full-thickness loss of skin with extension to muscle, bone, tendon, joint, or other deep structure;<br />

may be associated with osteomyelitis<br />

C. Management<br />

1. Position patient to remove pressure from the affected area.<br />

2. Cleanse the wound with saline, and debride it.<br />

3. If the wound appears infected (odor, drainage, necrosis), then treat with broad-spectrum antibiotics.<br />

4. Wet-to-dry or occlusive dressings should be applied.<br />

5. Consult a wound care team.<br />

XXV. DERMATOLOGIC CANCERS<br />

A. Basal cell carcinoma<br />

1. Overview<br />

a. Basal cell carcinoma is the most common skin cancer.<br />

b. Most commonly found on the head, neck, face, and nose but can occur anywhere<br />

c. Risk factors include fair skin, chronic sun exposure, immunosuppression, and therapeutic radiation exposure.<br />

853


CUTANEOUS DISORDERS<br />

2. Clinical presentation<br />

a. Dry, scaly, pink to red, flat papules or plaques with distinct borders; the border might be raised.<br />

b. Enlarge slowly<br />

3. Management<br />

B. Melanoma<br />

a. No emergent treatment<br />

b. Recognition and referral are key.<br />

1. Overview<br />

a. A malignancy of melanocytes (pigment producing cells)<br />

b. Metastases go to the lungs, liver, and brain.<br />

c. Risk factors include family history, prior severe sunburns, fair skin, older age, and multiple atypical nevi.<br />

2. Clinical presentation<br />

a. Asymmetric macule with variegated pigmentation and ragged borders<br />

b. ABCDE assessment: Asymmetry, _Harders that are irregular and ragged, .Color that is variegated, _!2iameter<br />

>6 mm, and .Evolving (changing) appearance<br />

3. Management<br />

a. No emergent treatment<br />

b. Recognition and early referral are key.<br />

C. Squamous cell carcinoma<br />

1. Overview<br />

a. A malignancy that occurs in sun-exposed areas (commonly the face and dorsal surface of the hands)<br />

b. May occur in the oral mucosa and lips<br />

c. Risk factors include family history, fair skin, blue eyes, chronic sun exposure, immunosuppression, and<br />

radiation exposure.<br />

2. Clinical presentation<br />

a. An erythematous, hyperkeratotic papule or nodule on sun-exposed areas of skin<br />

b. Lesion may ulcerate and become fixed to deeper structures.<br />

3. Management<br />

a. No emergency treatment<br />

b. Recognition and early referral are key.<br />

854


CUTANEOUS DISORDERS<br />

CUTANEOUS DISORDERS: QUICK REVIEW TIPS<br />

Differential diagnosis of genital ulcers<br />

• Chancre (primary syphilis) - Treponema pallidum<br />

• Genital herpes - herpes simplex<br />

• Chancroid _,, Haemophi/us ducreyi<br />

• Lymphogranuloma venereum - Chlamydia trachomatis<br />

• Granuloma inguinale (donovanosis) - Klebsiella granulomatis<br />

Wood's light<br />

The Wood's light is useful in detecting certain organisms and conditions (listed below) because they fluoresce. It<br />

can give false-positive results when certain substances that fluoresce (eg, exudates, tetracycline in sweat, make-up,<br />

deodorant, and soap) are also detected. Trichophyton tonsurans (the most common cause of tinea capitis) does not<br />

fluoresce.<br />

• Golden yellow (Tinea versicolor)<br />

• Pale green (Trichophyton schoenleim)<br />

• Bright yellow-green (Microsporum audouinii or M canis)<br />

• Aquagreen to blue (Pseudomonas aeruginosa)<br />

• Pink to pink-orange (porphyria cutanea tarda)<br />

• Ash-leaf-shaped spot (tuberous sclerosis)<br />

• Blue-white (leprosy)<br />

• Pale white (hypopigmentation)<br />

• Bright white or blue white (depigmentation, vitiligo)<br />

• Bright white (albinism)<br />

855


CUTANEOUS DISORDERS: PRACTICE CLINICAL SCENARIOS<br />

CUTANEOUS DISORDERS: PRACTICE CLINICAL SCENARIOS<br />

Answers immediately follow the practice clinical scenarios.<br />

Scenario A<br />

Presentation: A patient presents with salmon-colored papules and plaques (sharply marginated) covered<br />

with silvery white scales. Lesions are also found on the scalp, presacral area, knees, elbows, gluteal cleft,<br />

palms, soles, and extensor surfaces of the arms and legs. There is nail involvement with "pitting" and "oil<br />

spots."<br />

Courtesy of David Effron, MD, FACEP<br />

What is the diagnosis?<br />

Scenario B<br />

Presentation: A patient presents with erythematous papules and whitish "burrows" that are intensely<br />

pruritic and excoriated. Lesions are seen on the web spaces of the fingers, the flexion creases of the wrist<br />

and elbow, the penis, the buttocks, and the nipples. Pruritus intensifies at night.<br />

Courtesy of David Effron, MD, FACEP<br />

What is the diagnosis?<br />

856


CUTANEOUS DISORDERS: PRACTICE CLINICAL SCENARIOS<br />

Scenario C<br />

Presentation: A patient has a painless, indurated ulcer with a smooth base and raised border on the<br />

genitals and another on the mucous membranes of the mouth (the site of inoculation). He also has a<br />

painless regional adenopathy.<br />

Courtesy of David Effron, MO, FACEP<br />

What is the diagnosis?<br />

Scenario D<br />

Presentation: A patient presents with an erythematous or pink maculopapular or papulosquamous eruption<br />

with a symmetric distribution over the entire trunk and the extremities, including the palms and soles.<br />

Lesions are nonpruritic and accompanied by generalized lymphadenopathy and malaise.<br />

Courtesy of David Effron, MD, FACEP<br />

What is the diagnosis?<br />

857


CUTANEOUS DISORDERS: PRACTICE CLINICAL SCENARIOS<br />

Scenario E<br />

Presentation: A patient presents with pedunculated, pale, cauliflower-like warts. The lesions are soft, moist,<br />

and painless.<br />

Courtesy of David Effron, MO, FACEP<br />

What is the diagnosis?<br />

Scenario F<br />

Presentation: A young, sexually active woman presents with tender, erythematous (or hemorrhagic)<br />

macules or papules that are evolving into pustules and vesicles with a "red halo" and a gray necrotic<br />

center. Lesions are few in number and have a predilection for the periarticular regions of the distal<br />

extremities. The patient also has a low-grade fever, chills, and migratory polyarthralgias.<br />

Courtesy of David Effron, MO, FACEP<br />

What is the diagnosis?<br />

858


CUTANEOUS DISORDERS: PRACTICE CLINICAL SCENARIOS<br />

Scenario G<br />

Presentation: A teenager presents with oval-shaped, salmon-colored papules or plaques that have erupted<br />

on the trunk and proximal extremities. The lesions have a marginal collarette of scales, are mildly pruritic,<br />

and are distributed along the lines of skin cleavage, with the long axis of the lesions parallel to the ribs,<br />

forming a "Christmas tree" pattern on the trunk. On further questioning, the patient describes a different<br />

larger patch-like lesion that occurred about a week ago.<br />

Courtesy of David Effron, MO, FACEP<br />

What is the diagnosis?<br />

Scenario H<br />

Presentation: A patient presents with a pink/red maculopapular rash that first appeared on the face, but<br />

then rapidly spread to the neck, trunk, and extremities, and now on the third day is fading.<br />

Courtesy of David Effron, MD, FACEP<br />

What is the diagnosis?<br />

859


CUTANEOUS DISORDERS: PRACTICE CLINICAL SCENARIOS<br />

Scenario I<br />

Presentation: An active well-appearing child had a high fever for 3-4 days followed by eruption of an<br />

evanescent, blanching, rose-colored macular to maculopapular rash that started on the trunk and spread<br />

rapidly to the neck and proximal extremities. Mucous membrane involvement is absent. His fever broke as<br />

the rash erupted.<br />

Courtesy of David Effron, MD, FACEP<br />

What is the diagnosis?<br />

ScenarioJ<br />

Presentation: A patient presents with a vesicular skin eruption that started on the trunk and then spread<br />

to the face (including the mucous membranes of the mouth) and extremities. The palms and soles are<br />

spared. The vesicles (described as "dew drops on a rose petal") rapidly evolve into pustules that umbilicate<br />

and crust. Successive crops of lesions continue to erupt over several days, so that lesions in all stages of<br />

development are present.<br />

Courtesy of David Effron, MD, FACEP<br />

What is the diagnosis?<br />

860


CUTANEOUS DISORDERS: PRACTICE CLINICAL SCENARIOS<br />

Scenario K<br />

Presentation: A patient presents with a tender vesicular rash that started on the oral mucosa and then<br />

spread a day later to involve the buttocks, hands, and feet (including the palms and soles). The vesicles are<br />

flat-topped and have an erythematous base.<br />

Courtesy of David Effron, lv!D, FACEP<br />

What is the diagnosis?<br />

Scenario L<br />

Presentation: A patient presents after having had a bright-red malar rash ("slapped cheek" appearance) for<br />

2 days. It has now developed into an erythematous, maculopapular rash that has spread to the trunk and<br />

limbs (the palms and soles are spared). The rash is fading with central clearing, creating a reticulated or<br />

lacy pattern.<br />

Courtesy of David Effron, MD, FACEP<br />

What is the diagnosis?<br />

861


CUTANEOUS DISORDERS: PRACTICE CLINICAL SCENARIOS<br />

Scenario M<br />

Presentation: A patient presents with red to purple skin and mucous membrane lesions that do not blanch<br />

with pressure.<br />

Courtesy of David Effron, MO, FACEP<br />

What is the diagnosis?<br />

Scenario N<br />

Presentation: A patient reports mucous membrane hyperemia and a diffuse blanching, macular<br />

erythroderma (looked like a first-degree sunburn) that faded within 72 hours of its appearance. After<br />

another 1-2 weeks, there is now desquamation, particularly on the palms and soles. The patient has a fever<br />

(c':38.9°(), hypotension, and involvement of three or more organ systems.<br />

Courtesy of David Effron, MO, FACEP<br />

What is the diagnosis?<br />

862


CUTANEOUS DISORDERS: PRACTICE CLINICAL SCENARIOS<br />

Scenario 0<br />

Presentation: A patient presents with a tender, shiny, erythematous plaque with raised and sharply<br />

demarcated borders on the face.<br />

Courtesy of David Effron, MD, FACEP<br />

What is the diagnosis?<br />

Scenario P<br />

Presentation: A patient presents with erythematous macules that evolved quickly to bullae or pustules. The<br />

epidermis is sloughing, revealing an indurated, gunmetal gray, painless ulcer.<br />

Courtesy of David Effron, MD, FACEP<br />

What is the diagnosis?<br />

863


CUTANEOUS DISORDERS: PRACTICE CLINICAL SCENARIOS<br />

ANSWERS TO CLINICAL PRACTICE SCENARIOS<br />

Scenario A<br />

Diagnosis: psoriasis<br />

Management: Treatment options include topical steroids, UV light, coal tar-based shampoos, and<br />

immunosuppressants for severe cases; refer to dermatology.<br />

Scenario B<br />

Diagnosis: scabies (Be able to recognize a picture of the mite that causes scabies.)<br />

Management: Treat with a scabicide (lindane or permethrin) and an antihistamine or topical steroid for<br />

pruritus. Treat household contacts and counsel the patient regarding the washing of linens.<br />

Scenario C<br />

Diagnosis: chancre of primary syphilis<br />

Management: 2.4 million units penicillin G (1 dose) IM. Note that the average incubation period is 21 days<br />

but ranges from 1 0 to 90 days.<br />

Scenario D<br />

Diagnosis: secondary syphilis<br />

Management: 2.4 million units penicillin G (1 dose) IM. Note that the rash usually erupts 2 weeks to 6<br />

months (average 9 weeks) after the primary chancre appears.<br />

Scenario E<br />

Diagnosis: condyloma lata (a highly contagious manifestation of secondary syphilis)<br />

Scenario F<br />

Diagnosis: disseminated gonococcal disease<br />

Management: Admit for treatment with IV ceftriaxone. Watch for complications, which include septic<br />

joint, meningitis, and endocarditis.<br />

Scenario G<br />

Diagnosis: pityriasis rosea<br />

Management: Treatment is not necessary in most cases. Children can return to school with this rash.<br />

864


CUTANEOUS DISORDERS: PRACTICE CLINICAL SCENARIOS<br />

Scenario H<br />

Diagnosis: German or three-day measles (rubella)<br />

Key facts:<br />

• Caused by a genus of the rubivirus<br />

• Incubation period of 14-21 days<br />

• Period of infectivity extends from 7 days before until 7 days after onset of the rash.<br />

• A prodrome of headache, malaise, sore throat, coryza, and low-grade fever is common in adults and<br />

adolescents 1-5 days before onset of the rash but is often absent or minimal in children.<br />

• Forchheimer sign (pinpoint petechiae on the soft palate) may be present during the prodrome.<br />

• Lymphadenopathy (suboccipital, postauricular, and posterior cervical) is characteristic.<br />

• Complications include arthritis in adults (especially women), encephalitis, thrombocytopenia, and<br />

the congenital rubella syndrome when exposure occurs during the first trimester of pregnancy.<br />

Scenario I<br />

Diagnosis: roseola infantum (exanthem subitum)<br />

Key facts:<br />

• Caused by human herpesvirus types 6 and 7<br />

• Incubation period of 10-14 days<br />

• Most commonly affects children 6 months to 3 years old<br />

• Febrile seizures are a common complication.<br />

ScenarioJ<br />

Diagnosis: chickenpox (varicella)<br />

Key facts:<br />

• Caused by the varicella-zoster virus<br />

• Incubation period of 14-21 days<br />

• Period of infectivity is from several days before onset of the rash until all the lesions have crusted over.<br />

• Prodrome of low-grade fever, malaise, and headache occurs in adults and adolescents 1 or 2 days<br />

before the rash appears.<br />

• Diagnosis is usually made clinically but can be confirmed by Tzanck smear (multi nucleated giant<br />

cells) or culture.<br />

• Common complications include bacterial superinfection of the skin lesions, pneumonia, and<br />

encephalitis. Maternal infection during the first trimester can result in the congenital varicella<br />

syndrome. Perinatal maternal infection (5 days before to 2 days after delivery) can result in<br />

disseminated herpes in the neonate.<br />

Scenario K<br />

Diagnosis: hand-foot-and-mouth disease<br />

Key facts:<br />

• Caused by an enterovirus (usually coxsackie A 16)<br />

• Incubation period of 3-6 days<br />

• Brief prodrome (if any) of low-grade fever, malaise, and sore mouth<br />

• Infection during the first trimester of pregnancy may result in spontaneous abortion.<br />

865


CUTANEOUS DISORDERS: PRACTICE CLINICAL SCENARIOS<br />

Scenario L<br />

Diagnosis: fifth disease (erythema infectiosum)<br />

Key facts:<br />

• Caused by human parvovirus B 19<br />

• Incubation period of 4-14 days<br />

• May be accompanied by fever, headache, malaise, and myalgia; in adults (particularly women),<br />

arthralgia and arthritis are also common.<br />

• May recur with various stimuli over several months<br />

• Once the rash appears, children are generally no longer contagious.<br />

• Complications are rare unless the infection occurs during pregnancy (which may cause hydrops<br />

fetal is or fetal death), or they may occur in patients with chronic hemolytic anemias (which may<br />

cause aplastic crisis).<br />

Scenario M<br />

Diagnosis: purpura<br />

Key facts:<br />

• Purpuric lesions 90% of menstrual-related and -60%<br />

of nonmenstrual-related) are caused by strains of S aureus that produce toxic shock syndrome toxin 1.<br />

• Although prevalent in the 1980s, the number of cases of menstrual-related toxic shock syndrome<br />

has declined sharply and currently represents only 50% of all toxic shock syndrome cases.<br />

• Treatment includes early antibiotics (clindamycin is preferred), removal of source, and aggressive<br />

resuscitation.<br />

866


CUTANEOUS DISORDERS: PRACTICE CLINICAL SCENARIOS<br />

Scenario 0<br />

Diagnosis: erysipelas<br />

Key facts:<br />

• This variety of cellulitis (caused by group A streptococci) involves only the upper dermis. Traditional<br />

cellulitis has a much less distinct border and involves the deep dermis and subcutaneous fat.<br />

• Incubation period of several days<br />

• Infants, young children, and the elderly are most commonly affected.<br />

• Associated signs and symptoms (high fever, chills, and anorexia) develop rapidly and may precede<br />

the appearance of the cellulitis by 1-2 days.<br />

• Appropriate antibiotics include penicillins, cephalosporins, and macrolides.<br />

Scenario P<br />

Diagnosis: ecthyma gangrenosum<br />

Management: Admit for IV antibiotics; choices include broad-spectrum penicillins such as piperacillin,<br />

third-generation cephalosporins such as cefepime, aminoglycosides, and fluoroquinolones. The most<br />

common cause is Pseudomonas aeruginosa. The mortality rate is >50% mortality when the condition is<br />

associated with bacteremia.<br />

867


868<br />

NOTES


EMERGENCY MEDICAL SERVICES<br />

EMERGENCY MEDICAL SERVICES<br />

Background Information ........................................................................................................................................... 872<br />

Definition ........................................................................................................................................................... 872<br />

Origin of the Current EMS System ...................................................................................................................... 872<br />

Models of EMS Service/System Design ............................................................................................................... 873<br />

Planning for EMS System Demands .................................................................................................................... 874<br />

Components of an Emergency Medical Services System ........................................................................................... 874<br />

Prevention .......................................................................................................................................................... 87 4<br />

Manpower .......................................................................................................................................................... 874<br />

Education and Training of EMS Providers ............................................................................................................ 874<br />

Communications ................................................................................................................................................ 875<br />

Hospital Emergency Department and Specialty Service Categorization Program ................................................. 875<br />

Medical Control ................................................................................................................................................. 875<br />

Transport ............................................................................................................................................................ 876<br />

Medical-Legal Issues .......................................................................................................................................... 876<br />

Occupational Issues for EMS Providers ............................................................................................................... 877<br />

Aeromedical Transport. ....................................................................................................................................... 877<br />

Disaster Planning and Operation .............................................................................................................................. 878<br />

869


EMERGENCY MEDICAL SERVICES: SELF-ASSESSMENT QUESTIONS<br />

EMERGENCY MEDICAL SYSTEMS: SELF-ASSESSMENT QUESTIONS<br />

1. You are a physician at a remotely located community hospital, and you determine that your patient is in need of<br />

specialized cardiac care. Your patient requires the services of a cardiac catheterization lab almost 60 miles away.<br />

Helicopter transport is not an option because of weather, and your local EMS crew is a basic life support (BLS)-only<br />

service. Your patient is hemodynamically stable, on a heparin drip, and still reports active chest pain. What is the<br />

best course of action for your patient?<br />

(a)<br />

Send the patient BLS and suspend the heparin infusion.<br />

(b) Stabilize the patient locally and wait until the helicopter is back in service.<br />

(c) Utilize the services of a regional ground critical care interfacility transport team.<br />

(d) Call 911 and request an advanced life support (ALS) ambulance.<br />

2. The major role of the hospital during a disaster operation is:<br />

(a) Triage, stabilization, and transportation<br />

(b) Providing scene-response medical teams<br />

(c) Assuring the safety of emergency personnel, victims, and bystanders<br />

(d) Medical care of disaster victims and the ongoing routine medical needs of the community<br />

3. The primary goal of triage during a disaster operation is to:<br />

(a) Treat victims as quickly as possible<br />

(b) Do everything medically possible for each disaster victim<br />

(c) Treat as many patients as possible with the given resources<br />

(d) Classify victims according to treatment priorities to do the most good for the greatest number of potential<br />

survivors<br />

4. During a disaster operation, patients who are unlikely to survive are classified by the triage color:<br />

(a)<br />

Red<br />

(b) Yellow<br />

(c) Green<br />

(d) Black<br />

5. In order to be in compliance with The Joint Commission, all hospitals must have a written disaster plan and conduct<br />

disaster drills:<br />

(a) One time per year<br />

(b) Two times per year<br />

(c) Three times per year<br />

(d) Disaster drills are recommended but are not required, because they disrupt normal hospital operations.<br />

6. An on-scene physician wants to assist advanced life support providers at the scene of an emergency call. The<br />

physician is recommending treatment that may differ from local EMS operational protocols. The EMS providers<br />

should take the following action:<br />

(a) Permit the on-scene physician to carry out interventions outside the scope of the provider's practice<br />

(b) Defer al I decisions about patient care to the on-scene physician<br />

(c)<br />

Refuse to relinquish care unless the physician provides verification of current licensure<br />

(d) Request that the physician accompany the patient and providers to the hospital and document all physicianperformed<br />

interventions<br />

7. Which of the following is true regarding aeromedical transport?<br />

(a) There is research to support use of rotary wing transport for all critically ill patients.<br />

(b) Ground transportation is a better option for travel up to 30 miles or 30 minutes.<br />

(c) Cost should not be considered in making the decision to use a specific transport method.<br />

(d) When used for medical transport, fixed-wing aircraft fly at lower than standard altitudes to avoid problems such<br />

as air embolism and hypoxemia.<br />

870


EMERGENCY MEDICAL SERVICES: SELF-ASSESSMENT QUESTIONS<br />

8. Which of the following EMS providers can give oxygen, transport a patient, and assist with administration of a<br />

patient's own prescribed medications?<br />

(a)<br />

Emergency Medical Technician<br />

(b) Advanced Emergency Medical Technician<br />

(c)<br />

Emergency Medical Responder<br />

(d) Emergency Medical Technician-Intermediate<br />

9. An ambulance transporting a critically ill trauma patient arrives unexpectedly at your emergency department<br />

without prior consultation with medical control. The patient requires rapid administration of blood products and<br />

placement of a chest tube. What are the most appropriate next steps?<br />

(a)<br />

Refuse to accept responsibility for the patient, because the providers failed to consult and your emergency<br />

department does not have sufficient resources.<br />

(b) Stabilize the patient to the best of your ability, and instruct the ambulance to proceed to the nearest trauma<br />

center.<br />

(c) Perform the necessary lifesaving and stabilizing interventions, and ensure that the ambulance that will<br />

transfer the patient to the accepting trauma center has personnel trained to assess and monitor for emergency<br />

interventions.<br />

(d) Have an emergency department nurse accompany the patient to the nearest trauma center.<br />

ANSWERS<br />

I. c 4. d 7. b<br />

2. d 5. b 8. a<br />

3. d 6. d 9. c<br />

Use the pre-chapter multiple choice question worksheet (page xvi) to record and determine the percentage of correct<br />

answers for this chapter.<br />

871


EMERGENCY MEDICAL SERVICES<br />

I. BACKGROUND INFORMATION<br />

A. Definition<br />

1. The modern emergency medical services (EMS) system encompasses the delivery of comprehensive patient<br />

care outside of the hospital. EMS personnel interface with public health agencies, medical systems, and<br />

emergency departments to deliver time-sensitive and condition-appropriate care.<br />

2. The EMS system began in the 1960s as an extension of emergency medical care into the community. Today,<br />

there are three distinct phases within the spectrum of the EMS system.<br />

a. Prehospital care<br />

b. Emergency department care<br />

c. In-hospital care<br />

3. In 2010, EMS was recognized as a distinct medical subspecialty, and a number of post-residency fellowships<br />

accredited by the ACGME (Accreditation Council for Graduate Medical Education) are available for physicians<br />

seeking additional recognition or board certification.<br />

B. Origin of the current EMS system<br />

1. Highway Safety Act of 1966<br />

a. Established the Department ofTransportation and gave it legislative and financial authority to improve EMS,<br />

with each state required to develop regional EMS systems to handle their specific prehospital needs<br />

b. Emphasis was placed on developing highway safety programs, EMS standards, improvement of ambulance<br />

service, and provider training.<br />

c. Matching funds from state and local government targeted millions of dollars toward EMS research and<br />

development in the 1970s and 1980s.<br />

2. Emergency Medical Services Act: initiated in 1973 and provided funds for the development of local, state, and<br />

regional EMS systems; included development of guidelines for emergency medical care<br />

a. State EMS system<br />

(1) The central EMS authority with ultimate responsibility for planning, implementing, and operating the<br />

state EMS system.<br />

(2) May delegate authority to regional, county, and local authorities<br />

b. Established 15 essential components of an EMS system<br />

(1) Manpower<br />

(2) Training<br />

(3) Mutual aid<br />

(4) Disaster plan<br />

(5) Facilities<br />

(6) Transportation<br />

(7) Access to care<br />

(8) Communications<br />

(9) Critical care units<br />

(10) Public safety agencies<br />

(11) Consumer participation<br />

(12) Patient transfer<br />

(13) Public info/education<br />

(14) Review and evaluation<br />

(15) Coordinated patient record keeping<br />

3. Establishment of communication and ambulance standards<br />

a. The Department ofTransportation and White House Office ofTelecommunications advocated for national<br />

standard emergency access (911 ).<br />

b. The Federal Communications Commissions established rules and regulations for EMS communications and<br />

dedicated specific radio frequencies for emergency systems.<br />

c. The Department of Health, Education and Welfare delineated optimal ambulance design as well as<br />

essential equipment recommendations.<br />

d. The General Services Administration issued federal specifications for ambulance safety and design.<br />

872


EMERGENCY MEDICAL SERVICES<br />

4. Establishment of industrial and EMS-related standards developed by the National Highway Transportation<br />

Services Administration's Emergency Services Bureau include the following:<br />

a. EMS curriculum<br />

b. Medical aircraft (rotary and fixed-wing)<br />

c. EMS system organization<br />

5. Trauma Care Systems Planning and Development Act of 1990 encouraged further EMS development and set a<br />

standard for directing patients to an appropriate designated facility for time-sensitive treatment.<br />

6. The EMS Agenda for the Future, first published in 1996, developed by a multidisciplinary group, outlined<br />

the current state of EMS and posted future goals with emphasis on improving system efficiency, utilizing<br />

technology, developing healthcare networks that include EMS, and engaging in public health activities.<br />

a. An "Implementation Guide" set forth clear goals and objectives, along with interim steps to ensure their<br />

attainment.<br />

b. The EMS Education Agenda for the Future Uune 2000) revised the National Standard Curriculum.<br />

c. The most recent Agenda cal Is for an updated scope of practice model. The scope of practice model is<br />

consistent with training paradigms found in other allied health professions. Nationally recognized provider<br />

levels include:<br />

(1) Emergency Medical Responder (formerly "First Responder")<br />

(2) Emergency Medical Technician (formerly Emergency Medical Technician-Basic)<br />

(3) Advanced Emergency Medical Technician (formerly EMT-Intermediate)<br />

(4) Paramedic (formerly "EMT-Paramedic")<br />

d. The EMS functions at the federal level are overseen in the Department ofTransportation and National<br />

Highway Traffic Safety Administration office of EMS.<br />

Table 50: Comparison of Nationally Recognized Emergency Medical Services Providers<br />

Emergency<br />

Medical Emergency Medical Advanced Emergency<br />

Responder Technician Medical Technician Paramedic<br />

Focus Initiates life-saving Provides emergency Provides basic and Provides advanced level<br />

care care and limited advanced life care to critically i 11 and<br />

transportation support care injured; provides link<br />

from the scene to the<br />

emergency health care<br />

system<br />

Skill set Noninvasive Basic life support Limited medications, Full complement of<br />

airway skills, oxygen airway management, advanced cardiac life<br />

management, administration, patient IV fluids support medications,<br />

hemorrhage transport advanced airway<br />

control, use<br />

management, advanced<br />

of automated<br />

patient assessment<br />

defibri I la tor<br />

C. Models of EMS service/system design<br />

1. Fire service-based EMS model: Under this design, the fire department provides all EMS services.<br />

2. "Third service" model: In this model, the EMS delivery system is housed in a specially formed municipal<br />

department (separate from police and fire services) and provides basic life support (BLS) and advanced life<br />

support (ALS) services using municipal owned, operated and staffed ambulances.<br />

3. "Public utility" model: In this model, a community contracts with a private ambulance company to provide BLS<br />

and ALS ambulance services for all emergency calls in their community.<br />

4. Volunteer model: BLS, and in some systems, ALS ambulance service is provided by unpaid public service<br />

personnel.<br />

5. Combination models: This service model combines different first-response and transport agencies.<br />

873


EMERGENCY MEDICAL SERVICES<br />

D. Planning for EMS system demands<br />

1. EMS providers should anticipate 100 service requests for EMS annually per 1,000 population in the area they<br />

are serving.<br />

2. In service areas with a high volume of elderly or medically underserved, the call volume will be greater.<br />

3. The EMS Agenda for the Future encourages collaboration with emergency health care services and public<br />

health agencies to participate in disease surveillance and injury prevention.<br />

4. Currently, the Centers for Medicare and Medicaid Services (CMS) sets levels of reimbursement for patients<br />

transported by emergency medical services. Different rates apply for basic, advanced, and "specialty" care<br />

transportation.<br />

II. COMPONENTS OF AN EMERGENCY MEDICAL SERVICES SYSTEM<br />

A. Prevention (public education)<br />

1 . Safety and accident prevention<br />

2. Vehicle safety and protective equipment<br />

3. Poisoning prevention (including "childproof medications")<br />

4. Education in schools<br />

5. CPR, first aid, and automated defibrillator training<br />

B. Manpower<br />

1. 70%-80% of emergency medical technicians (EMTs) in the United States are volunteers.<br />

2. The percentage of volunteers is highest in rural areas.<br />

C. Education and training of EMS providers<br />

1. In previous years, an emergency medical responder's scope of practice was defined by the National Standard<br />

Curriculum. Future EMS certification examinations and provider designations will be based on the National<br />

Scope of Practice EMS provider level. Although classifications are similar, the Scope of Practice model attempts<br />

to standardize training requirements.<br />

2. Emergency Medical Responder (formerly "First Responder")<br />

a. Basic life support skills<br />

b. Hemorrhage control<br />

c. Scene safety and vehicle extrication techniques<br />

3. Emergency Medical Technician (formerly "Emergency Medical Technician-Basic")<br />

a. Includes skill set and knowledge base of the emergency medical responder<br />

b. Automated external defibrillation<br />

c. Patient-assisted medication delivery (nitroglycerin, albuterol)<br />

4. Advanced Emergency Medical Technician (formerly "Emergency Medical Technician-Intermediate")<br />

a. Includes EMT skills<br />

b. Supraglottic airway management<br />

c. Limited medications (epinephrine for anaphylaxis, narcotic antagonist)<br />

d. Patient-assisted medication administration<br />

5. Paramedic (formerly "Emergency Medical Technician-Paramedic")<br />

a. Includes Advanced EMT skills<br />

b. Training in differential diagnosis, medical decision making<br />

c. Invasive airway management<br />

d. Advanced patient assessment skills<br />

e. Licensure requirement includes graduation from nationally accredited certificate or associate's degree program<br />

6. States may designate specialty levels of emergency medical services providers according to regional needs.<br />

Other commonly encountered "specialty" providers include:<br />

a. Critical care paramedic: EMS professional trained to the "paramedic" level and credentialed to provide<br />

interventions outside the typical scope of practice. Critical care providers may be trained to manage<br />

ventilators, administer blood, or perform advanced skills<br />

b. Advanced practice or community paramedic: In some regions, paramedics function as physician extenders.<br />

Advanced practice paramedics may collaborate with local health departments to conduct follow-up visits<br />

and monitor medications.<br />

874


EMERGENCY MEDICAL SERVICES<br />

D. Communications<br />

1. The purpose of a communications system is to provide a means of accessing and coordinating the EMS system.<br />

2. Basic elements of a communications system<br />

a. Discovery and response<br />

(1) Recognition of the need for emergency notification<br />

(2) Best resuscitation outcome is possible when BLS is started within 4 minutes of arrest and ALS is started<br />

within 8 minutes of arrest.<br />

b. Access (the ability to activate the EMS system)<br />

(1) A single telephone number (911) to access police, fire, and EMS is preferred over multiple seven-digit<br />

phone numbers, which may be difficult to locate/recall in a crisis.<br />

(2) An enhanced/augmented 911 system (E-911) is a computerized system that automatically displays the<br />

address and phone number of the 911 cal l's origin to the emergency medical dispatcher.<br />

(3) New system elements are present for automatic location of cell phone calls, which now account for a<br />

large percentage of 911 calls.<br />

c. Receiving and dispatch<br />

(1) Some EMS systems have separate 911 access/receiving and dispatch functions.<br />

(2) The goal is to dispatch the most appropriate personnel and equipment to the location; initiating a<br />

maximal response (lights and sirens) for all requests is ineffective in terms of manpower (and financial<br />

resources) and can result in unnecessary harm to EMS providers and the public because of EMS<br />

vehicle accidents.<br />

(3) The emergency medical dispatcher is responsible for:<br />

(a) Taking and triaging the call<br />

1. Relevant information about the emergency is ascertained first (including verification of<br />

location, nature of the call, and number of people involved).<br />

ii. The dispatcher prioritizes the call as requiring a maximal or prompt response.<br />

(b) Alerting and dispatching the appropriate unit<br />

(c) Ensuring that prehospital personnel can find the address<br />

(d) Providing prearrival instructions on how to assist the victim until the ambulance arrives<br />

(e) Some systems have developed alternative call-management systems for 911. This allows lowpriority<br />

calls to be managed over the phone or by using nonemergency resources. Alternative callmanagement<br />

systems may include the use of registered nurses or other professionals to provide<br />

medical-related advice via telephone.<br />

d. Hospital notification and participation<br />

(1) EMT crews may communicate with medical control (on-line medical direction), which may or may not<br />

be the receiving hospital.<br />

(2) An advantage of hospital notification is that it allows time for emergency department personnel to<br />

mobilize the appropriate resources for patients' needs (particularly when an incident involves multiple<br />

casualties).<br />

E. Hospital emergency department and specialty service categorization program (developed by CMS)<br />

1. Type A: fully equipped and staffed emergency department with equipment and personnel available 24/7, either<br />

with a license or accepted by the public as able to treat patients emergently<br />

2. Type B: licensed by the state as an emergency department, held out to the public by posted signs, seeing at<br />

least one-third of visits on an urgent basis without appointment (Type B emergency departments are paid at a<br />

lower rate than Type A.)<br />

F. Medical control<br />

1. Physician input and surveillance assures medical competence of an EMS system. Medical control can be the<br />

responsibility of one physician, a group of physicians, or a hospital (referred to as the "resource hospital").<br />

2. Components<br />

a. Off-line (indirect) medical control<br />

(1) Development of procedural/treatment protocols<br />

(2) Provision of standing orders when contact with medical control is not feasible in a timely fashion<br />

(3) Training and testing<br />

(4) Ongoing education and surveillance<br />

(5) Quality reviews and improvement<br />

875


EMERGENCY MEDICAL SERVICES<br />

G. Transport<br />

b. On-line (direct) medical control<br />

(1) Provision of direct medical orders to EMTs in the field<br />

(2) Direct in-field observation: the most effective method for assessing the quality of care provided by<br />

EMS personnel; provides the most complete data gathering as well as prompt feedback<br />

1. Vehicle standards (physical and medical) and standards for essential equipment are established jointly by<br />

government agencies and medical specialty groups.<br />

2. The cost of an EMS ambulance varies according to its function and design. For example, ambulances providing<br />

rescue services (extrication) in addition to patient care may cost in excess of $200,000. A basic life-support<br />

interfacility transport service may meet all patient care objectives through utilization of a smaller, van-type<br />

(Type II) chassis.<br />

3. The type of transport used is determined by the distance and time needed to reach the hospital, the patient's<br />

condition, the weather, and terrain. The following guidelines are helpful in determining the optimal use of<br />

each mode of transportation. The type of transport is best understood as a medical decision. Physicians should<br />

have a basic understanding of the capabilities and limitations of various ambulance crew configurations and<br />

transport modalities.<br />

a. Ground transportation: best choice for distance ranges up to 30 miles or a transport time 100 miles; use may be limited by weather<br />

conditions, runways of inappropriate length/condition, refueling requirements, and altitude problems<br />

(hypoxemia, air embolism, expansion of air within catheter or endotracheal balloons).<br />

H. Medical-legal issues<br />

1. Consent to care is same as described for the emergency department<br />

2. Refusal of care<br />

a. A potentially significant legal problem (>50% of litigation against EMS providers is related to the issue of<br />

failure or refusal to treat or transport a patient who requests treatment or transportation).<br />

b. A competent, conscious adult may make an informed decision to refuse treatment or transportation.<br />

(1) The decision whether the patient is competent may be made in conjunction with the on-line medical<br />

control physician.<br />

(2) The patient is informed of the risks involved in refusing care.<br />

(3) The refusal should be well documented, and a release of liability waiver completed.<br />

c. An incompetent patient (as determined by the EMS on-scene providers with or without EMS physician<br />

involvement) should not be allowed to refuse care. (Appropriate treatment and transportation should be<br />

provided, even if the patient physically resists it. Police may need to be called to the scene to help manage<br />

such patients.)<br />

3. On-scene physician<br />

a. An on-scene physician who is unknown to EMS providers must be able to provide proof of identity and<br />

medical licensure before being allowed to provide patient care guidance.<br />

b. An on-scene physician may assist with treatment that is in line with EMS protocols without assuming<br />

responsibility and being required to accompany the patient to the hospital.<br />

c. No physician, including the on-scene physician, can direct EMS providers to administer emergency care<br />

and treatment that is beyond the EMS providers' training and state regulations.<br />

d. The on-line medical control physician may relinquish responsibility for guiding medical care of a patient to<br />

the on-scene physician; if the on-scene physician wishes to assume medical control, he or she must accept<br />

full medicolegal responsibility for all patient care and should accompany the patient to the hospital.<br />

4. lnterfacility transport pearls<br />

a. A "sending" emergency physician is responsible for ensuring that a patient in need of transfer receives<br />

appropriate care en route. Simply stated, the training and capabilities of the interfacility transport crew<br />

must be matched with patient needs. For example, a registered nurse might be needed for patients<br />

receiving certain vasoactive medications or specialized medication infusions.<br />

876


EMERGENCY MEDICAL SERVICES<br />

b. In specific circumstances, it may be appropriate to consider the transfer of "unstable" patients. Patients<br />

in need of definitive care (such as a cardiac catheterization or a specialized trauma intensive care unit)<br />

may be better served by transport to a specialized care facility. The sending physician is responsible for<br />

assuring that the transfer is made with appropriate equipment and qualified medical personnel.<br />

I. Occupational issues for EMS providers<br />

1 Infectious disease risk<br />

a. EMS personnel should observe universal precautions with every patient encounter because of risk of<br />

exposure to blood-borne pathogens.<br />

b. EMS personnel should also observe appropriate precautions for disease-specific risks (eg, meningitis, HIV,<br />

tuberculosis, etc)<br />

2. Occupational risks<br />

a. Slips, falls<br />

b. Lifting injuries<br />

c. Assault<br />

3. Legal risk<br />

a. Malpractice risk: 1 claim/24,000 EMS calls<br />

b. Operational risk: areas of liability include vehicle accidents, patient injury, medication errors, etc.<br />

J. Aeromedical transport<br />

1. lnterfacility transports account for a larger percentage of all hospital-based transport flights versus scene<br />

responses.<br />

2. Aeromedical transport of severely injured victims from a trauma scene has been shown to reduce mortality. Its<br />

effectiveness in reducing the morbidity and mortality of nontrauma patients, however, has not yet been clearly<br />

established.<br />

3. The Federal Aviation Administration (FAA) regulates the aviation component (aircraft, flight crews,<br />

maintenance) of air medical services; specific FAA regulations apply only to EMS aircraft (not to in-flight patient<br />

care). Because of multiple aircraft crashes and fatalities, the FAA is increasing industry regulation.<br />

4. The Commission on Accreditation of Air Medical Services, established in 1990, was developed to create<br />

a national voluntary accreditation process for air medical services. In 1997, it changed its name to the<br />

Commission on Accreditation of Medical Transport Systems (CAMTS). Accreditation standards were extended to<br />

address critical-care ground transport teams. While there are no regulatory implications, several managed care<br />

companies, state EMS agencies, and federal organizations are using CAMTS standards for benchmarking quality<br />

services as a prerequisite for contractural relationships and a requirement for state licensure.<br />

5. Helicopter (rotary-wing) transport programs<br />

a. Many rotary-wing programs permit flight only under visual flight rule conditions (clear weather). Many<br />

programs mandate that all aeromedical aircraft be equipped for instrument flight rule conditions to allow<br />

safe landing should poor weather develop en route.<br />

b. Recent increases in the rate of fatal helicopter accidents have motivated discussions about rotary-wing<br />

safety. A decision to use helicopter transport should be predicated on:<br />

(1) A thorough understanding of the patient's condition<br />

(2) Associated hazards, including weather and risk to personnel<br />

c. Helicopter emergency medical services have significant costs associated with operation and transport.<br />

Costs to the patient for an aeromedical evaluation can exceed $6,000.<br />

6. Airplane (fixed-wing) transport<br />

a. Fixed-wing aircraft have a better safety record than helicopters; they are able to fly under a wider range of<br />

weather conditions and are generally less expensive to operate.<br />

b. The cost for transport by fixed-wing aircraft ranges in the thousands, depending on the aircraft used and the<br />

distance flown.<br />

c. Airplanes also provide an increased mile range; greater speed; and increased patient, crew, and equipment<br />

capacity. However, they are limited to areas with airports.<br />

877


EMERGENCY MEDICAL SERVICES<br />

Ill. DISASTER PLANNING AND OPERATION<br />

A. Definition of a medical disaster<br />

1. An incident that results in multiple casualties, overwhelming numbers, or occurring at a rate that cannot be<br />

handled by the community resources<br />

a. Natural disasters (hurricanes, floods, earthquakes, etc)<br />

b. Human disasters (crashes lair, train, water], explosions, fires, environmental contamination, riots, terrorism, etc)<br />

2. The key issue is not the absolute number of victims but the relationship between the needs of the victims and<br />

the ability of the health care system to meet those needs using normal operating procedures.<br />

B. Disaster classification: disasters are most often classified according to the ability of the community, region,<br />

or state to meet the needs of the disaster response. Federally recognized disasters are usually those that<br />

cross jurisdictional boundaries and overwhelm the capabilities of local government.<br />

1. Level I disaster: local medical resources are adequate and quickly mobilized; declared by local officials.<br />

2. Level II disaster: requires medical resources from adjacent communities and regions (mutual aid); mobilization<br />

of such additional resources may take several hours to a day.<br />

3. Level Ill disaster: requires state or federal resources; requested by a local official, but actually declared by the<br />

governor or president; mobilization of these resources may take 2-3 days.<br />

C. Creating a disaster plan<br />

1. Characteristics of a disaster plan<br />

a. Must comply with the National Response Framework (NRF); the NRF document, distributed by the Federal<br />

Emergency Management Agency, outlines guiding principles to fol low in the event of a disaster. The goal<br />

of the NRF is for agencies involved in emergency preparedness to follow uniform disaster management<br />

strategies.<br />

b. Must be simple but flexible enough to accommodate the type and size of the disaster<br />

c. Must be as closely aligned with normal daily operating procedures as possible<br />

d. Must be coordinated with adjacent geopolitical areas (mutual aid agreements)<br />

e. Must include input and roles of local emergency management agencies, law enforcement, fire, public<br />

health departments, ambulance services, EMS council, hospitals, and the local medical community<br />

f. Must be organized with a structure including the Incident Command System (JCS) and the National<br />

Incident Management System (NIMS)<br />

(1) Incident command functions are unified under a designated "Incident Commander."<br />

(2) The ICS is a standardized, yet flexible template for local disaster operations.<br />

(3) The ICS is further subdivided into four major "sections." Each section has specific responsibilities.<br />

Table 51: Key Components of the Incident Command System (ICS)<br />

Operations Planning logistics Finance<br />

Search and rescue Collect and manage Provide necessary Manage financial<br />

activities incident-related data support for incident aspects of operation<br />

Tactical operations Formulate incident operations Track responder time<br />

Treat and transport action plans Facilities and pay<br />

injured Conduct required Transport of supplies Ensure ongoing financial<br />

Triage of injured meetings Equipment maintenance support and distribution<br />

Ensure adequate and and fueling of funding to branches/<br />

transparent flow of<br />

sections<br />

Food and medical<br />

information between services for responders<br />

sections<br />

g. Courses on incident management free of charge to health care providers are available on the Federal<br />

Emergency Management Agency website (http://www.fema.gov/incident-command-system#item7).<br />

h. Must include a joint public information operation<br />

2. Essential phases of a disaster plan<br />

a. Activation<br />

878


EMERGENCY MEDICAL SERVICES<br />

(1) Notification and initial response: The initial EMS personnel arriving on the scene report on the nature<br />

of the incident, extent of damage, the estimated numbers and types of injuries, hazards for the victims<br />

and rescuers, and the best access to the scene or routes that are known to be blocked. Activation is<br />

rapid (within minutes).<br />

(2) Organization of an incident command post by senior fire/emergency response officials. A more indepth<br />

assessment of the scene and associated hazards takes place after command post set-up.<br />

b. Implementation<br />

(1) Search and rescue (fire/rescue operation of identifying victims)<br />

(2) Triage, stabilization, and transport (medical providers)<br />

(3) Definitive management of scene hazards (firefighters) and victims (medical providers)<br />

(4) Hospital activities during field operations<br />

c. Recovery<br />

(a) Alert all area hospitals<br />

(b) Define levels of care needed<br />

(c) Monitor the status of the scene<br />

(1) Scene withdrawal (includes a systematic recheck for any missed victims)<br />

(2) Return to normal operations (includes restocking)<br />

(3) Debriefing<br />

D. The disaster operation<br />

(a) Evaluation of the disaster response<br />

(b) Identification of psychological difficulties experienced by the rescuers<br />

1. The most important prerequisite for good disaster management is that the EMS system must be functioning well<br />

on a routine basis. The most important first step is to assess the extent of the disaster and mobilize resources.<br />

2. Prehospital phase<br />

a. First responders<br />

(1) The major role of law enforcement is to secure the area and assure the safety of emergency workers,<br />

victims, and bystanders; in addition, they often establish initial communications.<br />

(2) The fire department plays an important role in life safety, rescue and hazard containment, and<br />

removing victims from hazardous areas.<br />

b. Triage<br />

(1) Definition: an ongoing process of sorting and classifying victims according to treatment priorities;<br />

field triage is usually performed by EMS personnel, because physicians and nurses are most useful<br />

in hospital settings. Triage should be incorporated into daily practice to facilitate its use during<br />

more chaotic, mass casualty situations. Most hospital systems adopt a codified system of triage that<br />

incorporates the use of pre-printed tags for victim identification and classification.<br />

(a) Primary triage: initial field triage in which classification (usually with a color-coding system) is<br />

begun.<br />

(b) Secondary triage: reexamination, re-triage, and further stabilization occurs at the casualty<br />

collection area.<br />

(c) Tertiary triage: reexamination, re-triage, and definitive care occurs at the appropriate receiving<br />

hospital.<br />

(2) Categories<br />

(a) Hopelessly injured (unsalvageable or dead): in the disaster setting, resuscitative efforts and<br />

resources should not be expended on any patient in cardiorespiratory arrest or those with<br />

injuries deemed unsalvageable.<br />

(b) Severe (first priority): seriously injured but salvageable patients that require immediate<br />

treatment<br />

(c) Moderate (second priority): seriously injured patients in whom treatment can be delayed<br />

without loss of life or limb<br />

(d) Mild (walking wounded): patients with minor injuries<br />

(3) Marking triage categories using four colors is the most commonly used system and is based on injury<br />

severity and prognosis:<br />

(a) Black: unsalvageable or dead<br />

(b) Red: first priority<br />

879


EMERGENCY MEDICAL SERVICES<br />

(c) Yellow: second priority<br />

(d) Green: walking wounded<br />

(4) Triage team: usually consists of experienced EMS personnel. Staff may be drawn from local hospitals,<br />

but they must be trained (or experienced) in disaster operations. Triage personnel are under the<br />

command of the medical triage officer.<br />

c. Scene control<br />

(1) Overall scene control by fire and rescue personnel to receive and collate information regarding<br />

efforts at the scene<br />

(2) Incident command system established. A command post should be established by the fire personnel<br />

at the disaster site (usually uphill and upwind of any hazard).<br />

d. Communications (the most common problem in a disaster)<br />

(1) The disaster plan should provide for a variety of communication systems capable of reaching all<br />

responding agencies.<br />

(2) Preplanning is required to determine the modes of communication that will be used (landline, cell<br />

phone, radio) and to develop protocols for their use.<br />

(3) Use of standard language that avoids jargon and "10-codes" facilitates ease of understanding and is<br />

mandated under the National Incident Management System.<br />

(4) Hospitals should be surveyed to update the availability of critical care and surgical beds.<br />

e. Miscellaneous considerations<br />

(1) A support system should be provided for disaster victims and workers to deal with the psychological<br />

effects of the disaster.<br />

(2) A mechanism for storing the dead must be established.<br />

3. Hospital phase<br />

a. The Joint Commission requires all hospitals to have a written disaster plan. Disaster drills are required<br />

twice yearly.<br />

b. Hospitals develop preparedness plans based on a hazard vulnerability analysis. Each hospital has elements<br />

of the items below, arranged according to specific risks and needs.<br />

c. Emergency Operations Center or hospital command center: An in-house control center that is near but not<br />

in the emergency department should be established to monitor the activities of participating hospitals.<br />

d. Plan activation: A system of rapid activation must be established that includes a clear delineation of each<br />

participant's responsibilities. Additional requirements are the following:<br />

(1) A system for calling in necessary personnel<br />

(2) Rapid clearing of patients that are currently in the emergency department (and in-house) to make room<br />

for the incoming casualties; these patients should be admitted or discharged as appropriate.<br />

(3) Establishing a temporary morgue may be necessary.<br />

e. Treatment areas<br />

(1) Receiving and triage area: Established near the emergency entrance, tertiary triage is conducted here<br />

(primary and secondary triage are generally conducted in the prehospital phase).<br />

(2) Critical treatment area: for patients who require the most intensive care; patients stabilized in this area<br />

should be placed in an intermediate care area to await surgery or admission.<br />

(3) Intermediate treatment area: for patients who are seriously injured but stable<br />

(4) Delayed treatment area: for patients whose treatment can be delayed without adverse effect (eg, the<br />

walking wounded)<br />

(5) Palliative care area: for unsalvageable patients<br />

f. Documentation<br />

(1) Patient charts<br />

(a)<br />

Information should be limited to critical findings and treatments; ideally, the chart should be kept<br />

with the patient at all times.<br />

(b) Prelabeled kits are preferable to the standard chart system when dealing with unidentified<br />

critically ill or injured patients, because the registration process is cumbersome in this situation.<br />

All of the contents are prelabeled with a disaster number. These numbers are acceptable<br />

to medical records and laboratory computers and become the patient's "identity" until full<br />

registration is possible. Contents of the kit include:<br />

880


EMERGENCY MEDICAL SERVICES<br />

1. The emergency department record<br />

11. Radiology requests<br />

iii. Laboratory slips and tubes<br />

iv. Wrist bands<br />

(2) A separate casualty list should be kept to account for and identify all deceased victims.<br />

g. Security: Additional security is needed early during the mitigation of any incident. Security personnel are<br />

essential to the integrity of entrance points and the maintenance of order.<br />

h. Waiting areas<br />

(1) There should be designated waiting areas away from the treatment areas; media will need an<br />

appropriate area, but a separate area must be available for family members only.<br />

(2) A hospital public relations officer should act as the public information officer with emergency<br />

department personnel, law enforcement, media, and families to provide accurate information to those<br />

in the waiting areas.<br />

(3) Each waiting area should have adequate telephone access.<br />

4. The National Disaster Medical System (NDMS), created in 1984, is an organization of civilian resources that<br />

handles large-scale military or civilian disasters.<br />

a. Components of the NDMS<br />

(1) Organization of the participating civilian hospitals and health care providers of designated NDMS<br />

metropolitan areas.<br />

(2) Development of disaster medical assistance teams<br />

b. The NDMS is not designated to replace local, state, or regional disaster plans; it is a federal resource<br />

available after about 72 hours and is used only in the event of a massive disaster.<br />

881


EMERGENCY MEDICAL SERVICES: PRACTICE CLINICAL SCENARIOS<br />

EMERGENCY MEDICAL SERVICES: QUICK REVIEW TIPS<br />

Basic Disaster Medical Operations: Triage<br />

Triage is the process of sorting patients according to the severity of injury and the availability of medical resources. In a<br />

disaster of mass casualty incident, it is imperative to distinguish between the critically ill and the walking wounded. The<br />

first round of triage is usually dedicated to sorting patients into predetermined categories.<br />

Triage tips<br />

• Moribund patients or those in respiratory/cardiac arrest: Patients without spontaneous respirations or without a<br />

pulse are generally categorized as "black" or unsalvageable. Triaging patients into this category does not preclude<br />

palliative treatments such as oxygen or morphine for pain.<br />

• Ambulatory patients: Patients able to follow commands and ambulate without assistance may be tagged as<br />

"green" or minor. Patients in this category have sustained minor injuries such as lacerations, abrasions, or sprains.<br />

Reverse triage: Multiple injuries resulting from a lightning strike may benefit from reverse triage. Patients in<br />

cardiopulmonary arrest from a lightning strike may benefit from cardiac compressions and resuscitation. This is a reversal<br />

of the usual triage process in that a significant amount of resources are dedicated to patients without initial vital signs.<br />

Compliance with the Emergency Medical Treatment and Active Labor Act (EMTALA)<br />

Patients presenting to the emergency department via EMS are essentially requesting a medical screening examination,<br />

performed to identify life-threatening conditions and prompt initial stabilizing interventions. Patients transported<br />

by EMS have "arrived at the hospital" as soon as the ambulance arrives on hospital property. The lack of prior radio<br />

contact or a delay in an orderly transfer of care from EMS to hospital providers does not release the hospital from<br />

responsibility under EMTALA. If an emergency department cannot provide services required to stabilize or treat an<br />

identified emergency medical condition, then the emergency department is obligated to transfer patients for definitive<br />

care. The transfer must take place with resources and personnel sufficient enough to prevent deterioration of the<br />

patient's condition en route. If a hospital emergency department is experiencing an internal disaster (eg, flooding,<br />

loss of power) or other catastrophic operational condition, it should officially declare "diversion" or "reroute"<br />

status so that incoming ambulances are aware of the emergency department's inability to perform medical screening<br />

examinations.<br />

882


PROCEDURES AND SKILLS<br />

PROCEDURES AND SKILLS INTEGRAL TO THE<br />

PRACTICE OF EMERGENCY MEDICINE<br />

Adult Cardiopulmonary Resuscitation ....................................................................................................................... 886<br />

Universal Algorithm ........................................................................................................................................... 886<br />

Bystander CPR .................................................................................................................................................... 886<br />

Healthcare Provider CPR .................................................................................................................................... 886<br />

Synchronized Cardioversion ............................................................................................................................... 887<br />

Pacing ................................................................................................................................................................ 887<br />

Postresuscitation Care ......................................................................................................................................... 887<br />

Resuscitation Procedures .......................................................................................................................................... 889<br />

Noninvasive Positive-Pressure Ventilation ........................................................................................................... 889<br />

Tracheal Intubation ............................................................................................................................................. 890<br />

Rapid Sequence Intubation ................................................................................................................................. 890<br />

Surgical Airway Management ............................................................................................................................. 894<br />

Venous and lntraosseous Access in Adults .......................................................................................................... 896<br />

lntraosseous and Central Venous Access in Children and Neonates .................................................................... 901<br />

Arterial Catheter Insertion ................................................................................................................................... 902<br />

Tube Thoracostomy ............................................................................................................................................ 903<br />

Emergency Department Thoracotomy ................................................................................................................. 904<br />

Pericardiocentesis ............................................................................................................................................... 907<br />

Peri mortem Cesarean Section ............................................................................................................................. 908<br />

Diagnostic Procedures .............................................................................................................................................. 908<br />

Lumbar Puncture ................................................................................................................................................ 908<br />

Paracentesis ........................................................................................................................................................ 909<br />

Thoracentesis ..................................................................................................................................................... 910<br />

Arthrocentesis .................................................................................................................................................... 911<br />

Pain Management ...................................................................................................................................................... 911<br />

Acute Pain Management in Adults ...................................................................................................................... 911<br />

Pain Management in Children and Neonates ...................................................................................................... 913<br />

Procedural Sedation and Analgesia ..................................................................................................................... 915<br />

Cutaneous Diagnostic and Therapeutic Procedures .................................................................................................. 917<br />

Wound Management .......................................................................................................................................... 917<br />

Wound Closure Techniques ................................................................................................................................ 920<br />

Incision and Drainage ........................................................................................................................................ 923<br />

Nail Trephination ............................................................................................................................................... 924<br />

Escharotomy ....................................................................................................................................................... 925<br />

Ultrasound ................................................................................................................................................................ 926<br />

Bedside Ultrasound ............................................................................................................................................ 926<br />

Trauma eFAST .................................................................................................................................................... 92 7<br />

Aortic Ultrasound ............................................................................................................................................... 930<br />

Pelvic Ultrasound ............................................................................................................................................... 931<br />

Cardiac Ultrasound ............................................................................................................................................ 932<br />

Biliary Ultrasound .............................................................................................................................................. 934<br />

Renal Ultrasound ............................................................................................................................................... 936<br />

Deep-Vein Thrombosis Ultrasound ..................................................................................................................... 937<br />

Procedural Ultrasound Uses ............................................................................................................................... 938<br />

883


PROCEDURES AND SKILLS: SELF-ASSSESSMENT QUESTIONS<br />

PROCEDURES AND SKILLS: SELF-ASSESSMENT QUESTIONS<br />

1. A 60-year-old man collapses at work. Optimal citizen's CPR for this patient is:<br />

(a)<br />

No CPR, just defibrillation with an automatic external defibrillator<br />

(b) CPR with a compression/ventilation ratio of 30:2<br />

(c) Chest compression only CPR<br />

(d) CPR with a compression/ventilation ration of 15:1<br />

2. Which of the following is true regarding targeted temperature management for the post-arrest patient?<br />

(a)<br />

It can be used for patients who experience traumatic arrest.<br />

(b) It decreases ischemic-reperfusion injury.<br />

(c) The goal is to maintain a temperature


PROCEDURES AND SKILLS: SELF-ASSSESSMENT QUESTIONS<br />

10. Nonabsorbable sutures retain tensile strength for:<br />

(a) At least 60 days<br />

(b) At least 30 days<br />

(c) At least 120 days<br />

(d) At least 7 days<br />

11. Which of the following is not true regarding hair removal before suturing?<br />

(a) It reduces the risk of infection.<br />

(b) It can hinder suture removal.<br />

(c) Shaving increases the risk of infection 3-9 times compared with clipping.<br />

(d) Hair near hairlines and in eyebrows should not be removed.<br />

12. Which of the following is true regarding OB ultrasound?<br />

(a) The patient definitely does not have an ectopic pregnancy with a serum HCC


PROCEDURES AND SKILLS<br />

I. ADULT CARDIOPULMONARY RESUSCITATION<br />

A. Universal algorithm<br />

1 . Check for responsiveness.<br />

2. Activate emergency response system (911, code team, call for help, etc).<br />

3. Cal I for defibri I la tor.<br />

B. Bystander CPR<br />

1. Assess responsiveness, no check for breathing, no pulse check.<br />

2. Compression-only CPR at a rate of 100-120/minute has been shown to improve survival when compared with<br />

conventional CPR by lay rescuers.<br />

3. For lay rescuers who are appropriately trained, consider administration of naloxone for known or suspected<br />

opiate overdose.<br />

C. Healthcare provider CPR<br />

1. CABD: if no response, start CPR<br />

a . .Circulation: check pulse, start CPR within 10 sec<br />

b. Airway: open airway<br />

c. .B.reathing: look, listen, feel<br />

d. Qefibrillator: attach monitor/defibrillator (consider earlier defibrillation when available)<br />

2. Analyze rhythm: shock if ventricular fibrillation/pulseless ventricular tachycardia<br />

3. CPR for 2 minutes at a rate of at least 100-120/minute; 30 compressions/2 breaths until definitive airway<br />

established, minimize interruptions<br />

4. Secondary ABCD<br />

a. Airway: secure airway<br />

b. Breathing: confirm placement and adequacy of ventilation<br />

c. Circulation: intravenous/intraosseous access, medications, monitor and determine rhythm<br />

d. Differential diagnosis and treat reversible causes (Table 52)<br />

Table 52: Potentially Reversible Causes and Treatment of Cardiac Arrest<br />

Hypovolemia/hemorrhage (IV fluids, blood)<br />

Hypoxia (0 2<br />

, ventilate)<br />

Hydrogen/acidosis (ventilate, possibly HCO 3<br />

)<br />

Hyperkalemia (Ca 2 +, insulin/D50 vs KCI)<br />

Toxins/Tablets/Trauma - overdose (naloxone,<br />

possibly HCO 3<br />

)<br />

Tamponade (pericardiocentesis)<br />

Tension pneumothorax (needle decompress)<br />

Thrombosis - acute coronary syndrome or<br />

pulmonary embolism (PCI vs tPA)<br />

Hypothermia (warm)<br />

Note: tPA = tissue plasminogen activator; PCI = percutaneous coronary intervention<br />

5. Continue CPR, check pulse every 2 minutes.<br />

6. If ventricular fibrillation/ventricular tachycardia<br />

a. Pads preferred over paddles (improved contact and safety)<br />

b. Biphasic (preferred): follow manufacturer's recommendation; usually 150 joules for first shock, then 200 joules<br />

c. Monophasic defibrillator: 360 joules for all shocks<br />

d. 2 minutes of CPR between shocks<br />

7. Medications: introduce after first 2 minutes of CPR<br />

a. Vasopressors<br />

(1) Epinephrine 1 mg every 3-5 minutes<br />

b. Antiarrhythmics<br />

(1) Amiodarone 300 mg once, may repeat 150 mg<br />

(2) Lidocaine 1-1.5 mg/kg, then 0.5 mg/kg every 5 minutes (maximum 3 mg/kg)<br />

(3) Magnesium 1-2 g IV push (consider if concern for torsades de pointes)<br />

886


PROCEDURES AND SKILLS<br />

c. Calcium chloride 1 ampule (consider if concern for hyperkalemia or hypermagnesemia)<br />

d. Bicarbonate (consider in toxic overdose, prolonged arrest)<br />

e. Atropine 1 mg every 5 minutes up to 3 mg for symptomatic bradycardia (routine use in asystole/pulseless<br />

electrical activity unlikely to provide benefit)<br />

8. Return of spontaneous circulation (ROSC)<br />

a. Avoid postarrest hypotension (mean arterial pressure


PROCEDURES AND SKILLS<br />

(4) Electrolyte imbalance<br />

(5) Disseminated intravascular coagulation<br />

(6) Bacteria translocation from intestines to blood<br />

2. Postresuscitation care goals<br />

a. Goal-directed restoration of circulation, oxygenation, and ventilation<br />

(1) Secure airway<br />

(a) Maintain SpO 2<br />

>94%, avoid PaO 2<br />

>300 mmHg<br />

(b) Maintain PaCO 2<br />

40-45 mmHg (prevents hypocapnia-induced cerebral vasoconstriction)<br />

(2) Establish central venous access<br />

(3) Volume resuscitation<br />

(a) Crystal loid infusion to maintain central venous pressure 10-15 mm Hg<br />

(b) Blood transfusion to maintain hemoglobin> 10 mg/dL<br />

(4) Hemodynamic support<br />

(a) Vasopressors to maintain mean arterial pressure >65 mmHg<br />

i. Norepinephrine is preferred agent 0.01-1 mcg!kg/min.<br />

ii. Consider dobutamine 2-15 mcg/kg/min in cardiogenic shock.<br />

(b) Establish arterial line to monitor blood pressure.<br />

(c) Antiarrhythmic medications for recurrent or unstable arrhythmias<br />

i. Treat the underlying etiology of arrhythmia.<br />

ii. There are no data to recommend prophylactic use of antiarrhythmic medications.<br />

(d) Consider intra-aortic balloon pump for persistent hypotension.<br />

b. Identify and treat cause of arrest.<br />

(1) ECG: evaluate for STEM!<br />

Some authors advocate PCI for all patients with ROSC and suspected cardiac etiology of arrest.<br />

(2) Bedside ultrasound: evaluate for pericardia! effusion, right ventricular strain, global or focal wall<br />

motion abnormalities, inferior vena cava diameter<br />

(3) Chest radiography<br />

(4) Laboratory assessment<br />

(a) Hematocrit/hemoglobin<br />

(b) Electrolytes<br />

(c) Lactate: monitor for persistent ischemia<br />

(d) Serial troponin: consider PCI if rising troponin<br />

(e) Blood gas analysis<br />

c. Minimize ischemic brain injury: targeted temperature management (therapeutic hypothermia)<br />

(1) Evidence<br />

(a) Two studies demonstrated improved outcomes in patients after ventricular fibrillation arrest who<br />

were cooled to 32°-34°C and maintained for 12-24 hours after ROSC.<br />

(b) Further studies demonstrated benefit in other arrest situations, providing ROSC was attained in<br />


PROCEDURES AND SKILLS<br />

(4) Contraindications<br />

(a) Advance directives, indicating do-not-resuscitate<br />

(b) Traumatic arrest<br />

(c) Pregnancy<br />

(d) Active bleeding<br />

(e) Sepsis<br />

(5) Cooling technique: goal is 33°-36°C (based on institutional preference; evidence is insufficient to<br />

suggest particular temperature goal)<br />

(a) External cooling: ice packs in axillae and groin<br />

(b) Cold IV fluids (lactated Ringer's) x 2 L<br />

(c) Cutaneous commercial cooling device<br />

(d) lntravascular cooling device<br />

(6) Monitoring<br />

(a) Core temperature monitor (eg, bladder or esophageal temperature probe)<br />

(b) Cardiac monitor: monitor for dysrhythmias<br />

(7) Medications<br />

(a) Sedation/seizure prevention: fentanyl or midazolam<br />

(b) Prevention of shivering: paralysis with vecuronium<br />

d. Manage cardiovascular dysfunction<br />

(1) Hemodynamic support (see above)<br />

(2) ASA per rectum if no contraindications for myocardial infarction<br />

e. Manage complications of global ischemia and reperfusion injury<br />

(1) EEG<br />

(2) Goal-directed resuscitation<br />

(3) Treat blood glucose> 150 mg/dL with regular insulin IV<br />

II. RESUSCITATION PROCEDURES<br />

A. Noninvasive positive-pressure ventilation<br />

1. Indicated for respiratory distress due to COPD and cardiogenic pulmonary edema.<br />

2. Both continuous positive-airway pressure (CPAP) and bi-level positive-airway pressure (BPAP) are effective in<br />

reducing rates of intubation/intubation complications.<br />

3. Contraindications: cardiopulmonary arrest, altered mental status, vomiting or excessive secretions, recent upper<br />

airway or upper GI surgery, inability to wear/tolerate mask<br />

4. Cautions: abdominal distention, oral secretions, autopositive end-expiratory pressure (PEEP) in COPD, reassess<br />

frequently, intubate if not improved<br />

5. Prehospital CPAP: decreases emergency department intubations and overall mortality<br />

6. CHF in emergency department<br />

a. No advantage of BPAP over CPAP<br />

b. Improves heart rate, blood pressure, 0 2<br />

saturation, Pa0 2<br />

, A-a gradient<br />

c. Reduces intubation rate, mortality<br />

7. COPD in emergency department<br />

a. Only advantageous in severe distress<br />

b. Decreases hypoxia, acidosis, hypercapnia within 1 hour<br />

c. Reduces length of stay, intubation rate, mortality<br />

8. Settings<br />

a. May use full mask or nasal pillows<br />

b. CPAP: 5 cm H 2<br />

0 1 titrate up by 1-2 cm H 2<br />

0<br />

c. BPAP inspiratory positive-airway pressure (IPAP) 8-10, expiratory positive-airway pressure (EPAP) 4-5<br />

(1) Most studies show effective range IPAP 14-20 1<br />

EPAP 5-8<br />

(2) Increase EPAP/IPAP by 1-2, maintain IPAP:EPAP ratio of about 2.5:1<br />

889


PROCEDURES AND SKILLS<br />

(3) Consider increasing IPAP:EPAP ratio for hypercarbia<br />

(4) Avoid !PAP >20 cm H 2<br />

0 to prevent gastric insufflation<br />

d. Oxygen bleed-in/Fi0 2<br />

to maintain 0 2<br />

saturation 86%-92%<br />

B. Tracheal intubation<br />

1. Indications<br />

a. Inability to protect airway<br />

b. Hypoxia not corrected with oxygen or noninvasive positive-pressure ventilation<br />

c. Hypercarbia<br />

d. Hypoventi lation<br />

e. Respiratory failure or impending failure<br />

2. Routes<br />

a. Orotracheal<br />

(1) Standard direct laryngoscopy versus video-assisted<br />

(2) Tube size: 7.5-9.0 mm for men, 7.0-8.0 mm for women, (age/4) + 4 mm uncuffed for children,<br />

decrease size by 0.5 for cuffed tubes<br />

b. Nasotracheal<br />

(1) Infrequently used because more morbidity and technically more difficult than endotracheal intubation<br />

(2) Benefit for spontaneously breathing patients with angioedema of tongue, clenched teeth, oral trauma,<br />

inability to move neck<br />

(3) Contraindications: nasal/midface trauma, basilar skull fracture, caution with patients receiving tPA or<br />

anticoagu I ants<br />

(4) Special nasal intubation that allows flexion of the tip highly recommended.<br />

(5) Insert lubricated tube (no stylet) through largest nare that has been prepared with topical<br />

vasoconstrictor.<br />

(6) Orient bevel toward septum, pass tube toward occiput, gently rotate tube to facilitate passage, listen for<br />

breath through tube, advance tube during initiation of inspiration using breath sounds to guide tube.<br />

(7) Advance to 28 cm at nare for men, 26 cm for women.<br />

(8) May be used in conjunction with fiberoptic scope for patients at high risk of failed airway<br />

C. Rapid sequence intubation (RSI)<br />

1. Overview<br />

a. Simultaneous administration of induction agent and paralytic agent<br />

b. Method of choice in emergency department intubations: highest success rates, fewest complications<br />

c. Exceptions<br />

(1) Cardiac/respiratory arrest, patient with no gag reflex<br />

(2) Spontaneously breathing patient with high risk of inability to ventilate or intubate (see below)<br />

2. Airway evaluation<br />

a. Predict difficulty with bag-valve-mask ventilation and/or intubation (Table 53)<br />

b. Indicated except in cases of crash airway (respiratory arrest)<br />

c. Predictors of difficulty with ventilation: presence of c::2 factors predictive of inability to ventilate effectively<br />

(1) Obesity (also pregnancy): difficult to achieve adequate tidal volume<br />

(2) Beard: difficult seal<br />

(3) Elderly (>55 years old)<br />

(4) Snorers: airway obstruction<br />

(5) Edentulous: difficult seal<br />

890


PROCEDURES AND SKILLS<br />

Table 53: LEMON Rule for Prediction of Difficult Intubation<br />

Predictor of Difficult Intubation<br />

look externally<br />

_!;valuate 3-3-2 rule<br />

Mallampati Score<br />

Qbstruction/obesity<br />

l's.eek mobility<br />

Sunken cheeks, edentulous, protruding incisors, micrognathia, facial/<br />

cervical trauma<br />

Mouth opening


PROCEDURES AND SKILLS<br />

Table 54: Sedative Agents<br />

Agent<br />

Dosage<br />

Onset<br />

(sec)<br />

Duration of<br />

Action (min)<br />

Benefits<br />

Cautions<br />

Etomidate 0.3 mg/kg IV 10-15 4-10 Little hemodynamic effect Causes myoclonus<br />

(total body Decreases ICP No analgesia<br />

weight)<br />

Decreased cortisol<br />

release*<br />

Propofol 1.5-3 mg/kg 15-45 5-10 Bronchodilation Can cause hypotension<br />

IV (total body Decreases ICP Avoid in patients with<br />

weight)<br />

Anticonvu lsant<br />

low ejection fraction<br />

Avoid with egg allergy<br />

No analgesia<br />

Ketamine 1-2 mg/kg IV 60-90 10-20 Increases blood pressure Use in increased ICP<br />

(ideal body Bronchodilation controversial**<br />

weight)<br />

Provides analgesia<br />

Can cause hypotension<br />

in low ejection fraction<br />

or catecholaminedepleted<br />

patients<br />

May experience<br />

emergence reaction<br />

Midazolam 0.2-0.3 mg/kg 60-90 15-30 Anticonvu lsant Hypotension<br />

IV (total body Potent amnestic Slow onset<br />

weight)<br />

Frequently underdosed<br />

* Etomidate is associated with transient suppression of adrenal function but no clear change in mortality; no study to date powered to detect<br />

effects on hospital or ICU length of stay, ventilator days, or mortality.<br />

**<br />

Historic teaching about ketamine has recommended this agent be avoided in patients with suspected increased ICP. However, recent data<br />

has shown that use of ketamine is safe in patients with increased ICP.<br />

(3) Neuromuscular blocking agents decrease complications and improve intubating conditions<br />

(a) Muscle paralysis through noncompetitive (Table 55) or competitive (Table 55) binding to<br />

acetylcholine receptors<br />

892


PROCEDURES AND SKILLS<br />

Table 55: Neuromuscular Blocking Agents<br />

Agent<br />

Dosage<br />

Noncompetitive depolarizing agent<br />

Succinylcholine<br />

1.5 mg/kg IV<br />

(total body<br />

weight)<br />

Competitive nondepolarizing agents<br />

Rocuronium<br />

Vecuronium<br />

0.6-1 .2 mg/kg<br />

IV (ideal body<br />

weight)<br />

0.1 mg/kg IV<br />

(ideal body<br />

weight)<br />

Onset<br />

(sec)<br />

45-60 sec<br />

1-4 min<br />

2-4 min<br />

Duration of<br />

Action (min)<br />

5-9<br />

30-45<br />

45-60<br />

Benefits<br />

Drug of choice for<br />

RSI due to onset and<br />

duration of action<br />

First choice alternative to<br />

succinylcholine<br />

Least ideal agent for RSI<br />

Cautions<br />

Hyperkalemia*<br />

Fasciculations<br />

Masseter spasm<br />

Malignant hyperthermia<br />

Bradycardia<br />

Increased intraocular<br />

pressure**<br />

Paralysis time longer<br />

than induction agent<br />

Slow onset<br />

Prolonged paralysis<br />

(duration may<br />

be prolonged in<br />

hepatorenal failure)<br />

* Patients at risk include those with known hyperkalemia, burns >5 days old, denervation/severe crush injury >5 days old, severe infection<br />

>5 days old, preexisting myopathy, or neuromuscular disease.<br />

** May increase intraocular pressure and ICP, although this is refuted by recent studies.<br />

f. Placement with confirmation<br />

(1) Placement: goal is to identify vocal cords and pass tube<br />

(a)<br />

Direct laryngoscopy<br />

i. Place blade in right side of mouth, sweep tongue to the left.<br />

ii. Identify epiglottis.<br />

iii. Straight (Miller) blade: lift epiglottis.<br />

iv. Curved (Macintosh) blade: insert into vallecula and lift up and away.<br />

v. Lift up on jaw to expose vocal cords; do not rock laryngoscope back on patient's teeth.<br />

(b) Video-guided laryngoscopy<br />

i. Insert blade in midline of tongue, advance along midline until vocal cords are visualized.<br />

ii. Special stylet may be required.<br />

iii. Limited by secretions or blood in airway<br />

iv. Improves visualization of cords with minimal neck movement<br />

(c) Visualization improved with tracheal manipulation as needed by intubator with position, then<br />

maintained by assistant.<br />

(d) Routine cricoid pressure is no longer recommended.<br />

(e) If unable to visualize larynx, withdraw and try secondary method.<br />

(f)<br />

Insert endotracheal tube on right side of mouth, rotate through cords.<br />

(g) Difficulties with passing tube usually due to too large tube size, cricoid pressure, improper stylet<br />

bend (35° optimal)<br />

(h) Advance tube 3-4 cm after cuff is beyond vocal cords.<br />

(i)<br />

(2) Confirmation<br />

Withdraw blade, inflate cuff


PROCEDURES AND SKILLS<br />

(d) Colorimetric end-tidal CO 2<br />

i. Give six breaths to ensure not false positive<br />

ii. May produce false negative in low-flow states<br />

(e) Air aspiration<br />

(f)<br />

g. Postintubation care<br />

i. 30-ml syringe attached to endotracheal tube, withdraw.<br />

ii. If in trachea, air is easily withdrawn.<br />

Chest radiograph to confirm absence of right main-stem bronchus intubation but not a reliable<br />

means to confirm endotracheal placement<br />

(1) Secure tube in place without decreasing venous return.<br />

(2) Place orogastric or nasogastric tube.<br />

(3) Maintain adequate sedation/analgesia to prevent agitation/extubation.<br />

(4) Usual initial ventilator settings<br />

(a) Mode: assist control (NC)<br />

(b) Fi 0 2<br />

: 100%, decrease to maintain PO 2<br />

60-90 mm Hg<br />

(c) Tidal volume 6-8 ml/kg (use ideal body weight in obese patients)<br />

(d) Respiratory rate: 10-14 breaths per minute, increase in acidosis<br />

(e)<br />

(f)<br />

4. Troubleshooting<br />

lnspiration:expiration ratio 1 :2; increase expiratory duration for obstructive lung disease<br />

PEEP: start 5 cm H 2<br />

0, gradually increase in hypoxia<br />

a. Disconnect from ventilator and use bag-valve-mask.<br />

b. Evaluate for malfunctions ("DOPE" mnemonic).<br />

(1) Q.isplacement: evaluate for tube displacement<br />

(2) Qbstruction: suction tube and deep airways<br />

(3) fneumothorax/autoPEEP: evaluate for pneumothorax or stacked breaths<br />

(4) J;quipment: reevaluate ventilator settings<br />

5. Difficult airway tools<br />

a. Gum elastic bougie<br />

(1) Flexible, plastic rod with angled tip<br />

(2) Used with direct laryngoscopy when only arytenoid cartilage visible<br />

(a)<br />

Insert bougie into trachea using direct laryngoscopy.<br />

(b) Confirm placement by advancing bougie to carina and feeling firm tracheal rings when advancing,<br />

retracting bougie.<br />

(c) Thread endotracheal tube over bougie.<br />

b. Video-assisted intubation (see above)<br />

c. Fiberoptic<br />

(1) Requires training/practice, set up time, and compliant, spontaneously breathing patient<br />

(2) Nasal preferred to oral route: topical anesthetic and vasoconstrictor keys to success<br />

(3) Requires :::7.5 mm endotracheal tube<br />

(4) Thread tube over fiberoptic scope (adapter removed)<br />

(5) Pass fiberoptic scope through cords and advance tube<br />

D. Surgical airway management<br />

1. Surgical cricothyrotomy<br />

a. Indication: failed airway (ie, can't intubate, can't ventilate)<br />

b. Contraindications<br />

(1) Age :S:8 years: cricothyroid membrane too smal I and risk of injuring surrounding structures<br />

(2) Partial or complete tracheal transection<br />

(3) Obstruction deep to the cricothyroid membrane<br />

c. Minimum equipment required<br />

(1) Scalpel (#10 or 11 blade)<br />

(2) 6.0 or smaller endotracheal or tracheostomy tube<br />

894


OTHER COMPONENTS<br />

5. Watch endorsements, and make sure your identity is clear to avoid violation of the Federal Trade Commission's<br />

Guides Concerning the Use ofTestimonials and Endorsements.<br />

6. Make sure you are familiar with your employer's and hospital system's social media and electronic<br />

communication policies.<br />

V. LEADERSHIP AND MANAGEMENT PRINCIPLES<br />

A. Many guides on how to be a good leader involve switching focus from "I" to "we" or "team:'<br />

B. Leadership roles<br />

1. Traditional leadership roles focused on a top-down approach.<br />

2. Modern leadership roles focus on a collaborative approach with a partnership model. A leader seeks to<br />

empower and motivate others rather than demand performance.<br />

VI. WELL-BEING<br />

A. Rights of emergency physicians: ACEP Policy<br />

1. Autonomy in clinical decision making shall be respected.<br />

2. Physicians have the right to expect adequate staffing and equipment.<br />

3. Physicians will be compensated for services and not be required to pay for privileges or referrals, nor shall they<br />

receive anything of value for referrals to others.<br />

4. Autonomy should not be restricted by cost-saving protocols.<br />

B. Fatigue and impairment<br />

1. Fatigue can be an impairment (see also ethical principles, page 967). Every physician should be mindful of his<br />

or her limits.<br />

2. Residents have work hour restrictions that teaching attending should be aware of.<br />

3. Schedules for emergency physicians should be created keeping in mind patient volume and the need for sleep<br />

between busy shifts. Strategies may vary by group (eg, 10 hours minimum between busy shifts, 24 hours off<br />

between 24-hour shifts, etc).<br />

C. Time management, life balance, and organizational skills<br />

1. Physicians are often busy and overstretched. Learning how to prioritize and organize can help with job and<br />

personal life satisfaction. Each person needs to decide which clinical, professional, and social activities are<br />

worth pursuing.<br />

2. Learning how to say "no" without feeling guilty may have to be learned and practiced. Seminars, lectures, and<br />

books are available and can help. A mentor or life coach might also be beneficial.<br />

D. Work dysphoria (burnout)<br />

1. In the United States, 30%-40% of physicians experience some degree of burnout.<br />

2. Work-related stress causes burnout.<br />

3. Increasing paperwork burden, decreasing autonomy, decreasing reimbursement, lack of tort reform, problems<br />

with work/life balance, and physician shortages may all contribute to burnout.<br />

4. Medical schools, medical societies, hospitals, and residency programs should have policies to promote<br />

physician well-being and patient safety.<br />

969


OTHER COMPONENTS: PRACTICE CLINICAL SCENARIOS<br />

OTHER COMPONENTS OF THE PRACTICE OF EMERGENCY<br />

MEDICINE: PRACTICE CLINICAL SCENARIOS<br />

Answers immediately follow the practice clinical scenarios.<br />

Scenario A<br />

Presentation: A patient presents to your hospital in labor, but her insurance does not cover admissions.<br />

What law requires that your hospital handle the delivery of the fetus and placenta before she and the baby<br />

can be transferred?<br />

Scenario B<br />

Presentation: A patient is brought in by police for calling 911 to express suicidal ideation. The patient has<br />

ongoing suicidal thoughts and auditory hallucinations. The patient states he is going to kill his brother and<br />

gives his name. The patient requests discharge.<br />

What is the management?<br />

Scenario C<br />

Presentation: An elderly patient presents from home via ambulance with a chief complaint of fever.<br />

Examination reveals infected bedsores, soiled clothes, and unkempt appearance. The patient tells you she<br />

lives with her son, who feeds her once a day and uses her social security check to buy his cigarettes. She is<br />

bedbound from a previous stroke.<br />

What is the diagnosis?<br />

ScenarioD<br />

Presentation: A patient is brought in by EMS for a fall. She had fallen the day before and neighbor found<br />

her on the floor when he came to check on her. She lives alone with her cat. EMS noted little human<br />

food in the house, cat food tins piled up around the living room and multiple medications from different<br />

providers with duplications. Patient appears frail and unkempt. Her nails are long and dirty and her clothes<br />

are unwashed. The patient is delirious and has multiple bruises in various states of healing<br />

What is the diagnosis?<br />

Scenario E<br />

Presentation: Dr. Smith notices that her coworker, Dr. Jones, seems anxious and annoyed over the busy<br />

pace in the emergency department. He is easily frustrated and expresses anger at nurses. A nurse comes<br />

to Dr. Smith and expresses concern that Dr. Jones did not examine a patient with hypotension and put the<br />

chart up for discharge. Dr. Smith asks Dr. Jones about it, and he states he knows what he is doing and to<br />

leave him alone. Dr. Smith notices that Dr. Jones is sweating and seems shaky.<br />

What is the diagnosis?<br />

970


OTHER COMPONENTS: PRACTICE CLINICAL SCENARIOS<br />

Scenario F<br />

Presentation: A 16-year-old arrives by herself for evaluation. She refuses to have her parents called for<br />

consent to treat. When she is interviewed by the emergency physician, she indicates that she is concerned<br />

she is pregnant. A medical screening examination is performed, and a urine pregnancy test is positive.<br />

Ultrasound shows a 6-week intrauterine pregnancy. She has no signs of illness and denies pain or<br />

bleeding. She is given a prescription for prenatal vitamins, and a follow-up appointment is scheduled with<br />

the hospital obstetrics clinic. She is urged to tell her family about the pregnancy, because she will need<br />

support. She is discharged without her parents having been contacted.<br />

What type of situation is being described?<br />

Scenario G<br />

Presentation: A 70-year-old male retired factory worker presents with hypotension and a lower GI bleed.<br />

He refuses a blood transfusion on religious grounds. He is urged to change his mind given his hypotension<br />

despite treatment with IV fluids. Laboratory tests show severe anemia. A surgeon has been called in to try<br />

to stop the bleeding but has not arrived yet. The patient states he understands that he might die without a<br />

blood transfusion, but he still refuses to undergo the procedure.<br />

What type of situation is being described?<br />

Scenario H<br />

Presentation: A celebrity presents to the emergency department with a gunshot wound to the leg. He asks<br />

that this not be reported, because he does not want the publicity and it was an accident. He would like to<br />

be treated and discharged without having to talk to the police.<br />

What is the management?<br />

971


OTHER COMPONENTS: PRACTICE CLINICAL SCENARIOS<br />

ANSWERS TO PRACTICE CLINICAL SCENARIOS<br />

Scenario A<br />

EMTALA<br />

Key facts: Under EMT ALA, the patient and her infant can be transferred once medically stable. If the infant<br />

is unstable, all attempts to stabilize him or her must be made. If there is no inpatient neonatal unit, the<br />

infant can be transported even if he or she remains unstable if:<br />

• You as the treating physician certify that the medical benefits outweigh the risks.<br />

• You send copies of the medical records.<br />

• The receiving hospital has the space and personnel to treat.<br />

• The receiving hospital has agreed to accept the transfer, and<br />

• The transfer will be performed by qualified personnel with appropriate equipment.<br />

Scenario B<br />

Diagnosis: mental health<br />

Management: The physician has the duty to treat the patient and to warn the patient's brother. The<br />

physician cannot discharge the patient despite the patient's request because of the potential harm to<br />

the patient and the brother. The patient can be committed against his will for a fixed period of time. The<br />

physician should attempt to contact the brother and the police about the patient's homicidal thoughts.<br />

Scenario C<br />

Diagnosis: elder maltreatment<br />

Management: You check your state laws and you have mandatory reporting of elder abuse. You contact the<br />

appropriate agency that will investigate. You admit the patient for treatment of her infected bedsores and<br />

initiate a social work consult.<br />

Scenario D<br />

Diagnosis: elder self-neglect<br />

Laboratory evaluation: Laboratory tests show dehydration, infection, and low albumin.<br />

Management: The patient should be admitted for treatment and social work evaluation.<br />

Scenario E<br />

Diagnosis: physician impairment<br />

Management: Dr. Smith, concerned that Dr. Jones is impaired, needs to take immediate action to prevent<br />

harm to patients. Fatigue, drug and alcohol abuse or withdrawal, or mental illness may cause a physician<br />

to become impaired. Hospital and state policies guide the removal of a physician from duty, but no delay<br />

should occur when a patient is at risk.<br />

972


OTHER COMPONENTS: PRACTICE CLINICAL SCENARIOS<br />

Scenario F<br />

Diagnosis: minor seeking treatment without consent of parent or guardian<br />

Management: Minors are allowed to be treated without parental consent for mental illness, pregnancy,<br />

drug abuse, and in some cases pregnancy prevention. If the pregnancy is the result of sexual abuse or<br />

statutory rape, then the physician is obligated to report to children's services and should detain the patient<br />

for the investigation.<br />

ScenarioG<br />

Diagnosis: refusal of care<br />

Management: Adult patients who are conscious have the right to refuse all medical care, including that<br />

for life-threatening conditions. The treating physician should assess the patient's decision-making capacity<br />

and should have the treatment discussion witnessed by another health care provider and if possible the<br />

patient's family.<br />

Scenario H<br />

Diagnosis: mandatory reporting<br />

Management: In all states, gunshot wounds or suspected gunshot wounds must be reported, even if<br />

accidental. The reporting should be done per the local law and hospital policy without any unnecessary<br />

release of protected health information. Physicians should defer all questions from the media, should they<br />

occur, to the hospital public relations personnel.<br />

973


974<br />

NOTES


MECHANICS OF THE WRITTEN BOARD EXAM<br />

MECHANICS OF THE WRITTEN BOARD EXAM<br />

The written board exam in emergency medicine tests your knowledge base relative to predetermined criteria of clinical<br />

competency. Consequently, there are two critical aspects of this test:<br />

1. What is the written exam actually testing?<br />

2. How are you being evaluated?<br />

In addition, multiple-choice exams usually assess the goals of learning, which include:<br />

• Knowledge of terminology or specific facts<br />

• Knowledge and application of methods and procedures<br />

• Ability to apply facts and principles in a given situation<br />

• Ability to interpret cause and effect relationships<br />

The written board exam tests both the outcomes of learning as well as the goals of learning. This text was designed and<br />

written with this in mind, which is why it is a teaching text (not just a reference text). In addition to providing facts,<br />

special attention has been given to the reader's need to understand concepts and to think critically, thus improving the<br />

ability to analyze, apply, synthesize, and interpret information. This is essential to effective learning, which is what this<br />

exam is really testing.<br />

The second critical aspect of the written board exam is understanding how you are being evaluated. Some written exams<br />

discriminate among individuals, ranking them on relative levels of learning. These exams are "norm referenced" and<br />

are designed to discriminate in a way so that a certain percentage of candidates will not pass. The written board exam<br />

in emergency medicine does not discriminate; it is "criterion referenced," which means that there is no ranking, ie,<br />

100% of candidates can pass. It describes what an individual can or cannot perform, with the difficulty of the test item<br />

matching the difficulty of the task.<br />

Examination Format<br />

The Written Board Exam in emergency medicine may be administered by the American Board of Emergency Medicine<br />

(ABEM), the American Osteopathic Board of Emergency Medicine (AOBEM), or the American Board of Physician<br />

Specialties (ASPS). For the most recent information on the examination format for ABEM, visit www.abem.org. For the<br />

most recent information on the examination format for AOBEM, visit www.aobem.org. For the most recent information<br />

on the examination format for ASPS, visit www.abpsus.org. At each of these individual websites, candidates may be able<br />

to access exam specifics, including logistical and procedural information as well as mechanics of each individual exam.<br />

Helpful resources such as sample questions, content specifications, candidate booklets, and scoring information may<br />

also be posted on these websites.<br />

ABEM Written Examination<br />

The American Board of Emergency Medicine (ABEM) is recognized by the American Board of Medical Specialties.<br />

ABEM certifies emergency physicians who meet its educational, professional standing, and examination standards. The<br />

Model of the Clinical Practice of Emergency Medicine (EM Model) forms the basis of each of ABEM's multiple-choice<br />

examination questions and describes the relative weight on the ABEM exam. The content of the ABEM's written board<br />

exams is based on a national (not regional) standard of care. Regardless of where you practice or the patient volume you<br />

encounter, you will be tested on a "national norm," which we have adhered to in this text. The initial ABEM written exam<br />

is called the Qualifying Exam. ABEM-certified physicians must then pass the written ConCertTM Examination during years<br />

6-10 of their certification. A score of 75% or greater is required to pass the ABEM examination. Additional information<br />

can be found at www.abem.org.<br />

AOBEM Written Examination<br />

The American Osteopathic Board of Emergency Medicine (AOBEM) is the certifying body of the American Osteopathic<br />

Association. The AOBEM Table of Specificity forms the basis of each of AOBEM's multiple-choice examination questions<br />

and ranks the Core Content Categories into high, medium, and low relative weights. The initial written AOBEM exam<br />

is called the Certification Examination. The Formal Recertification Examination includes a computer-based exam. It<br />

is offered once a year, and AOBEM has established a predetermined pass rate. The scores are not curved in any way.<br />

Additional information can be found at www.aobem.org.<br />

ABPS Written Examination<br />

The American Board of Physician Specialties (ASPS), the official certifying body of the American Association of Physician<br />

Specialists, Inc. (AAPS), administers a written certification examination for the Board of Certification in Emergency<br />

Medicine (BCEM). The Domain Table forms the basis of each of ASPS' multiple-choice examination questions and lists<br />

the number of questions in each of the domain categories. The exam is offered twice a year. Currently the minimum<br />

score for passing, depending on the form of the examination, ranges from approximately 75% to 82%. Additional<br />

information can be found at www.abpsus.org.<br />

975


MECHANICS OF THE WRITTEN BOARD EXAM<br />

How To Take a Multiple-Choice Exam<br />

Your skill in taking a multiple-choice exam is as important as adequate preparation. A common misconception is that<br />

all candidates who fail this type of exam do so because they have failed to master the subject matter. In truth, some<br />

test-takers have not learned how to apply their knowledge to a multiple-choice exam. Did you know that changing the<br />

sentence structure, either in the question or the answers, can dramatically affect performance (whether one gets that<br />

question right or wrong) on that question? A good test taker will get it right with either structure, while the less well able<br />

(trained) test taker will not. For those of you who have previously encountered some difficulty with this type of exam,<br />

included are some general recommendations as well as specific test-taking strategies.<br />

Remember<br />

1. The exam is testing basic knowledge, not sophisticated, new age issues, not nuances, not areas of controversy (eg,<br />

if two sources disagree, this will not be on the exam). It is based on national standards, not what is going on in your<br />

hospital. Know the basics. Know the standard of care.<br />

2. Field questions are part of the exam. Field questions do not count either for you or against you. So if you do not<br />

know the answer to any particular question, tell yourself, convince yourself, "this was a field question and will not<br />

count anyway."<br />

3. This is an exam for emergency physicians. It is an action-oriented exam; you are expected to do something.<br />

4. Absolute contraindications are more important to know than relative contraindications.<br />

5. This is a time-sensitive exam. You have approximately 1 minute per question.<br />

6. Read the question and all the answers carefully. If part of the answer is wrong, and part of the answer is right,<br />

it is the wrong answer.<br />

7. Associations are important on board exams. Learn them! You cannot ask for any additional information when<br />

taking the exam-no new laboratory studies, no new imaging studies, etc. The exam question writer must give you<br />

information that can help you associate what is in the question to the right answer. Here are some examples of<br />

associations:<br />

a. The most common cause of osteomyelitis caused by a rusty nail through a running shoe is Pseudomonas<br />

aeruginosa. However, the most common cause of soft-tissue infection caused by that same rusty nail through a<br />

running shoe is Staphylococcus or Streptococcus.<br />

b. In a patient with a recent past history of Guillain-Barre syndrome who now has diarrhea, the cause of the<br />

diarrhea is most likely Campylobacter.<br />

c. Humeral shaft fracture is associated with wrist drop due to radial nerve injury.<br />

d. Ring-enhancing lesions on CT brain are associated with toxoplasmosis, neurocysticercosis, or brain abscess.<br />

These are just a few examples of many. But be assured that ABEM loves associations.<br />

8. Acuity frames in terms of percentage on the ABEM exam<br />

a. Critical 30%<br />

b. Emergent 40%<br />

c. Lower acuity 21 %<br />

d. None 9%<br />

9. Modifying factors for age on the ABEM exam<br />

a. Pediatrics 8%<br />

b. Geriatrics 6%<br />

10. Spend your time on what is important.<br />

a. Most common<br />

b. Triads, pentads<br />

c. Diagnosis<br />

d. Deadly causes<br />

e. Indications, contraindications<br />

f. Complications<br />

g. Associations<br />

h. Worst-case scenario for a given presentation<br />

i. Things we do that may kill<br />

j. Things that if we do not do, may kill<br />

k. Things we get sued for<br />

I. ACLS, ATLS, PALS<br />

976


MECHANICS OF THE WRITTEN BOARD EXAM<br />

11. Red flags: abnormal vital signs, missing vital signs<br />

12. Missing information is common on the exam. If you were given al I the information in a particular question,<br />

the answer would jump out at you and the question would be too easy.<br />

13. Be careful of similarities, which can fool you.<br />

a. Erythema migrans (Lyme disease) vs erythema marginatum (Jones' criteria/rheumatic fever)<br />

b. Erlichiosis (tickborne bacterial infection) vs erysipelas (Streptococcus/St Anthony's fire)<br />

c. Condylomata acuminate (human papillomavirus) vs condylomata lata (syphilis)<br />

d. Using nimodipine for spontaneous subarachnoid hemorrhage, not nifedipine<br />

14. Keycepts to types of questions<br />

a. Knowledge questions<br />

(1) If in your knowledge base, do notfal I for the sucker punch, which is the answer most people will<br />

be drawn to at first glance and it usually placed before the right answer.<br />

(2) If not in your knowledge base, guess smart.<br />

b. Concept questions: time awareness (one minute per question overall exam)<br />

c. Math questions: math will be simple (this is not a mathematics exam).<br />

d. Dosages of medications: know the ones used every day in the emergency department. You will be given<br />

the dosage of medications used less frequently.<br />

General Recommendations<br />

1. Learn how to relax during the written exam. If you feel tense before the exam actually begins, take a few seconds to<br />

induce a sense of positive self-expectancy. Say to yourself, "I will do well because I expect to do well." Believe it or<br />

not, psychological studies have shown that if you expect to do well, the odds are better that you will.<br />

After you have induced a sense of positive self-expectancy, close your eyes, take a deep breath, and command<br />

yourself to relax. As you begin taking the exam, some of the tension will dissipate spontaneously simply because the<br />

conscious mind cannot do two things at once; you cannot think about being tense while thinking about the answer<br />

to a question. To stay relaxed during the exam, work rapidly but carefully. If you are not sure of an answer, skip<br />

that one and come back to it later. This is very important psychologically. Answer all the questions you know first<br />

and then go back later to work on those you don't know. You can tag questions you want to go back to later on the<br />

computer during the test.<br />

2. Avoid fatigue. Have a good night's sleep before your exam day. Because the exam is a long one, give yourself a rest<br />

period every hour or so during the exam, even if it is a very short one. Relax in your seat, and close your eyes for a<br />

minute or two. These rest periods will help maintain your mental capacity/efficiency at a high level.<br />

Specific Test-Taking Strategies<br />

Guessing is part of any multiple-choice exam. Although guessing will probably not significantly improve your score, it<br />

will not lower it either. Guessing smart is the key to guessing. Here are some guidelines that can help you select the best<br />

answer when the answer is not coming to you.<br />

The first thing you have to determine is whether you understand the question but don't know the answer or you not sure<br />

what the question is asking.<br />

Let's begin with questions you understand but are unsure of the correct (best) answer:<br />

1. Eliminate obviously incorrect answers. Then concentrate on the remaining answers.<br />

a. An answer that "doesn't fit" with the other answers is probably wrong.<br />

b. A wordy answer that is unclear or does not read well is probably wrong.<br />

c. An answer with the words "always" or "never" is probably wrong (but these are rarely found in tests today<br />

because they were recognized as "give aways").<br />

d. An answer that does not correlate grammatically with the question is probably wrong.<br />

2. If you have eliminated obvious and probable incorrect answers and you still have more than one answer remaining,<br />

try looking at the question and completing it mentally before looking at the answers.<br />

a. A spontaneous completion that corresponds to one of the answers is usually correct.<br />

b. A spontaneous completion that is similar to two answers suggests that one of them is correct. Determine which<br />

one best completes the statement.<br />

c. A spontaneous completion that is opposite to one of the remaining answers eliminates that choice.<br />

3. Look at adjectives and adverbs in the question. Descriptive (key) words are frequently clues to the correct answer.<br />

Words like "acute" or "classic" or "routinely" are examples. This is a particularly valuable strategy if you are dealing<br />

with a question that seems to have two possible answers (the key word will fit one answer better than the other).<br />

977


MECHANICS OF THE WRITTEN BOARD EXAM<br />

Another example is "definitive" treatment versus "initial" (the ABCs) treatment, and classic (original description) vs<br />

most common. Exact terms in the answer, such as all, always, never, none, must necessarily, are usually part of a<br />

wrong answer.<br />

4. If you encounter a question containing a clinical presentation and you are asked to make a diagnosis based on an<br />

image (ECG, radiograph, ultrasound, photo of a skin lesion, etc), answer the question before looking at the picture.<br />

Then, look at the picture to confirm your answer was right. If the picture is required to answer the question and<br />

you are not sure what the image shows, try to answer the question again, now considering the picture. The correct<br />

answer should "match" the clinical presentation.<br />

5. If a question covers a subject you know little or nothing about, see if there are other questions within the<br />

examination that include the same material. You can sometimes find useful information contained in other questions<br />

that can help you answer this question.<br />

Points to Consider About Answers<br />

1. If there are numbers contained in the answers and two answers have similar numbers, one of them is usually the<br />

correct one.<br />

2. If one of the answers is "all of the above," look for two correct answers. If you find them, "all of the above" is the<br />

correct answer.<br />

3. If one of the answers is "none of the above," look for one correct answer. This excludes "none of the above."<br />

All of these techniques can be used to answer questions you understand.<br />

Now let's deal with questions you don't understand.<br />

Some questions are designed to test your ability to think critically or analytically. The answers to these questions are not<br />

obvious-they are inferred. Here's an example:<br />

"What is the best pharmacologic choice for the patient with supraventricular tachycardia and hypertension?"<br />

(a) Adenosine<br />

(b) Digoxin<br />

(c) Methoxamine<br />

(d) Edrophonium<br />

If you look in standard emergency medicine textbooks, you will not find a drug listed for the treatment of<br />

supraventricular tachycardia with hypertension. You will find, however, several drugs listed that should not be given<br />

to patients with hypertension: all the vasopressors (methoxamine, norepinephrine, metaraminol, phenylephrine, etc),<br />

digitalis preparations, and edrophonium chloride. Therefore, the question does not ask you to directly identify the<br />

treatment of choice. Instead, the question asks you to infer the treatment of choice by eliminating the agents you should<br />

not use; the agent remaining will be the correct answer (adenosine).<br />

Here is another example of an "inferred" answer:<br />

"What drug should be used in the patient with CHF who also has signs of digitalis toxicity?"<br />

(a) Furosemide (Remember, the question is about digitalis toxicity, not CHF.)<br />

(b) Quinidine<br />

(c) Nitroprusside<br />

(d) Propranolol<br />

Again, there is no "drug of choice" for patients with CHF who are digitalis toxic. There are, however, drugs that should<br />

not be used in the presence of digitalis toxicity. Quinidine and calcium channel blockers (nifedipine, verapamil, and<br />

diltiazem) decrease digitalis clearance, so they should not be used. Furosemide tends to induce hypokalemia, which<br />

increases the risk of digitalis toxicity, so this is not a good choice. ~-blockers (propranolol, nadolol, metaprolol, atenolol,<br />

timolol) are contraindicated in patients with severe CHF and should not be used. On the other hand, vasodilators<br />

(nitroglycerin, nitroprusside, prazosin, and hydralazine) lower systemic vascular resistance and improve cardiac output.<br />

The correct answer, therefore, is nitroprusside.<br />

Practice, Practice, Practice Makes Perfect<br />

In developing any skill, practice is important. Practice answering multiple-choice questions on a regular basis. It is not<br />

only a good way to assess your knowledge base, but it is also a training mechanism for developing test-taking skills. As<br />

you go through the current PEER products, apply the techniques you have learned here when confronted with problem<br />

questions. With practice, this skill will improve, and you will feel more confident about your ability to use it effectively.<br />

On any given exam, test-taking strategies can be successfully applied to 5%-15% of the questions. Some candidates fail<br />

this exam by only one or two points, so using these techniques might be just enough to help you pass.<br />

978


ADDITIONAL TIPS FOR GOOD PERFORMANCE<br />

ADDITIONAL TIPS FOR GOOD PERFORMANCE<br />

Preparing for the written board exam is an arduous task, one that requires intense review and study. In some respects,<br />

the final phase of preparation is more difficult than the earlier ones. For one thing, you are tired. In addition to studying,<br />

most of you are working full time. Another problem is that your anxiety level is likely to peak as the exam date draws<br />

near. Finally, your self-study plan/preparation may leave you with a feeling of being overwhelmed by the amount of<br />

academic information you have been learning and relearning.<br />

In this section are some guidelines to follow in the time remaining before the exam. It is recommended that you map out<br />

a program that will reduce your fatigue, lessen your anxiety, and allow you to focus your attention on the most important<br />

academic information you need to know.<br />

Self-Care<br />

Good health habits are important. If you have unhealthy habits, it will become increasingly harder to study and you<br />

will retain less.<br />

1. Get plenty of sleep.<br />

2. Eat more fiber-containing foods and less sugar and fat. You are more likely to be alert when you study as well as<br />

have greater endurance.<br />

3. Engage in some form of mild exercise. Walking is excellent if you are not a runner; aerobics and swimming<br />

are also helpful.<br />

Positive Self-Expectancy<br />

A positive attitude is also important. Your attitude, both before and during the exam, can affect your exam score. A<br />

negative attitude will foster procrastination before the exam and will interfere with your performance during the exam.<br />

On the other hand, a positive attitude will facilitate your preparatory efforts and enhance your performance during the<br />

exam.<br />

A technique that can help you build and maintain a positive attitude for the written board exam is called "mental<br />

rehearsal," and it can be very effective in reaching a goal and achieving success.<br />

Mental rehearsal consists of previewing an upcoming event in a positive frame of mind. This should be done during the<br />

weeks before the exam. The purpose is twofold: to allow you to prepare for the exam as completely and thoroughly as<br />

you can in the allotted time, and to maximize the probability of a good performance on the exam day.<br />

The art of positive self-expectancy is one that is practiced by many professionals before a major event. Essentially it<br />

consists of mentally rehearsing a perfect performance during the days, and even weeks, before the actual event. To use<br />

this technique, set aside time each day when you are completely alone and relaxed and imagine yourself taking the<br />

exam in a calm, confident manner with the expectation of doing well.<br />

When you use the mental rehearsal technique, concentrate on what you know rather than on what you don't know.<br />

Do not worry about subjects or areas that you didn't have time to study. Rather, focus on what you have studied and<br />

then make some affirmations about your ability to do well on exam day: "I will do well on the exam. I have studied<br />

and worked hard to get where I am today and now, I'm going to go further still. I can do it." Positive affirmations have a<br />

powerful effect on the subconscious mind because they foster our belief in ourselves, which translates into confidence in<br />

our ability.<br />

Don't put yourself into a position to be discouraged when you start rehearsing. Just do your best. Within a short time, the<br />

result of this technique will speak for itself, providing you with all the justification you need.<br />

Written Board Courses<br />

Many state ACEP chapters (and other organizations) conduct courses for emergency physicians taking the written board<br />

exam. These are intensive programs (4-7 days) that provide a comprehensive review of emergency medicine. They<br />

are usually given about a month before the exam. The utility of these courses varies from one course to another and<br />

from one physician to another. Not everyone needs or benefits from a written board review course. Many emergency<br />

physicians have passed the exam without it.<br />

Ohio ACEP offers a 5-day comprehensive academic review of emergency medicine, as well as a 3-day course that<br />

focuses on key facts and pearls of each emergency medicine topic.<br />

The ACEP master calendar (www.acep.org) can provide information on a course that suits your schedule and location<br />

needs. Also, EMRA puts out a periodic review of board review courses; refer to them as a reliable, independent resource<br />

to select a course.<br />

979


ADDITIONAL TIPS FOR GOOD PERFORMANCE<br />

How to Use an Emergency Medicine Written Board Course<br />

If you attend a written board course without any prior preparation, the program can be overwhelming. A lot of material is<br />

delivered very fast. If you have been reading this text before the course, your task will be a lot easier.<br />

Read as much of this text as possible before the course. Remember to highlight key concepts and important facts. Take<br />

this book with you to the course; this is your base on which to build. Use the book as your guide during the course.<br />

If you are attending an ACEP course, take the daily pretests and grade yourself; participate in evening review sessions,<br />

which cover hundreds of board-type questions. If you missed any questions, determine whether you didn't know the<br />

material or you did know the material but misread (misinterpreted) the question. Pay attention to pretest scores that are<br />

lower than 75% or several questions you did not know on certain topics. These are weak areas.<br />

For those of you who have been studying for more than 3 months, listen to your speakers very selectively. Listen for<br />

information that is new to you. If you have been studying with some regularity the last few months or so, your database<br />

is fairly strong. The course will not only be a review for you but will also fill gaps in your knowledge base. Use this text<br />

to write in new information that seems important.<br />

For those of you who have been studying less than 3 months, a different approach may be helpful. If at all possible,<br />

review the syllabus material before each lecture. While you are listening to the lecture, take notes and mark your<br />

syllabus (underline) important information. Later, insert the syllabus material in this text.<br />

For all of you: From the time you finish the course until a couple of days before the exam, review only the highlights of<br />

the study materials you have been using; it should all be in this text now.<br />

1. Review any material that is likely to be on the exam, ie, any information that your speakers pointed out as "must<br />

know for the exam. 11 You don't want to miss questions that you expect to see on the exam.<br />

2. Review any information you learned in this text or during the course that clarifies your understanding of a particular<br />

subject. This is important because if you encounter questions on this topic and you are not sure of the answer, you<br />

will be able to figure it out.<br />

3. Review any area where you are weak. Do not pick more than three because you do not want to 11 cram 11 at this stage<br />

of your preparation. Select the most important topics (those that are most heavily covered on the exam), and let the<br />

rest go.<br />

Practice the Exam Process<br />

Set aside some time shortly before the exam to run through multiple-choice questions and answers (as many as you can).<br />

If you haven't taken this type of exam for a while, you need to retrain your mind in this type of test-taking.<br />

In addition to answering the multiple-choice questions throughout this text, go through a self-assessment or mock exam<br />

product. Most written board courses have pretests (and post-tests) that amount to a couple of hundred or more multiplechoice<br />

questions.<br />

Go through the questions as if you were actually taking the exam. Answer all the questions you know first. Go back later<br />

to work on the questions you didn't know or were unsure of. There is a reason for this based on test-taking psychology:<br />

answering questions you are sure of first increases your confidence in answering questions you are unsure of later. If you<br />

are still unsure of some questions when you return to them, try using the techniques described in the section "How to<br />

Take a Multiple-Choice Exam. 11<br />

At the Exam<br />

1. Set your watch.<br />

2. Concentrate.<br />

3. Read the question and all answers carefully.<br />

4. Recognize key terms.<br />

5. Psych yourself up for a good game.<br />

A Few Points about Studying<br />

First remember this reality: You must read things five times to store them in your long-term memory.<br />

1. When do you study best? Early morning, late evening, other? Use that time to study.<br />

2. How do you learn best? Reading, listening, watching, computer-generated learning? Use that technique the most.<br />

3. No matter how or when you learn best, practice multiple choice questions over and over and over again.<br />

4. Identify areas of weakness and focus on these areas. Focus first on the topics with the most questions on the exam<br />

(eg, trauma, cardiovascular, etc).<br />

5. When you are freshest, do the things you find difficult.<br />

6. When you are less fresh, do pictures, ECGs, radiographs, ultrasound, CT scans, MRls, VQ scan, etc.<br />

980


ADDITIONAL TIPS FOR GOOD PERFORMANCE<br />

7. When you are least fresh, do some multiple choice questions.<br />

8. Improve retention by doing the following:<br />

a. Study in cycles<br />

(1) Study for 30 minutes; concentrate hard.<br />

(2) Take a 5-minute break.<br />

(3) Go back and do another 30 minutes study time.<br />

(4) Take a 5-minute break.<br />

(5) Go back and do another 30 minutes study time.<br />

(6) This is the end of the cycle.<br />

b. You can do 3 of these cycles per day (no more).<br />

9. Foil analysis: When you answer questions, analyze the foils, looking at which ones were wrong and why were they<br />

wrong. And if you answered the question wrong, ask yourself, "why did I miss that question?"<br />

10. Use review cards for everything except for those topics in which you are strong.<br />

11. Do:<br />

a. Get plenty of rest<br />

b. Exercise<br />

c. Meditate<br />

12. Do not:<br />

a. Use sleeping aids before the exam.<br />

b. Use alcohol.<br />

c. Drink too much coffee.<br />

Do not study intently the last day or two before the exam. Your ability to answer questions correctly will be greater<br />

because your analytical ability will be high and not overridden by specific recall from the last day's cramming.<br />

981

Hooray! Your file is uploaded and ready to be published.

Saved successfully!

Ooh no, something went wrong!